You are on page 1of 433

‫اﻟﻜﺎﻓﻲ ﻓﻲ اﻟﺮﻳﺎﺿﻴﺎت‬

‫‪Ŕž Ŧ Ľ‬‬
‫‪BACCALAURÉAT‬‬
‫ﺩﺭﻭﺱ ﻣﻔﺼﻠﺔ ﻭ ﻣﺴﺎﺋﻞ ﻣﺤﻠﻮﻟﺔ‬
‫‪.‬‬

‫‪œǃ‬‬
‫ﺍﻷﻋﺪﺍﺩ ﺍﳌﺮﻛﺒﺔ‬

‫‪Ŀ‬‬
‫‪I‬‬‫‪.‬‬

‫‪.‬‬
‫و ﺗﻄﺒﯿﻘﺎﺗﻬﺎ‬
‫تايضا‪.‬يرلا‬
‫‪.‬‬ ‫‪.‬‬
‫جامعة‬

‫من‬
‫الثا‬
‫ٕالى ال‬

‫نويّة‬
‫‪ņőƱŀ Ţ‬‬
‫‪ƕ‬‬

‫ّ‬
‫‪.‬‬
‫‪.‬‬

‫ﻞ‬ ‫ﻛ‬
‫‪.‬‬

‫ﻠ‬ ‫ﻳ‬ ‫ﺎ‬‫ﻣ‬


‫ّﺰﻣﻚ‬
‫‪ũŏ‬‬

‫ﻟﻠﺘﻔﻮﻕ‬
‫ﺗﺄﻟﻴﻒ ‪ :‬ﻋﺒﺪﺍﻟﺤﻖ ﻓﺮﻗﻮﺱ‬
‫‪http ://tinyurl.com/Malki1718‬‬ ‫‪0‬‬
Ŕž Ŧ Ľ
œǃ
Ŀ
ņőƱŀ Ţ
ũŏ ƕ

http ://tinyurl.com/Malki1718 0
‫‪Ŕž Ŧ Ľ‬‬
‫‪œǃ‬‬
‫‪otttttttttttttttttttttttttttttttttttttttttttp‬‬
‫‪s‬‬ ‫‪u‬‬
‫‪s‬‬
‫‪s‬‬
‫ﺗﺮﻣﻴﺰات‬ ‫‪u‬‬
‫‪u‬‬
‫‪qvvvvvvvvvvvvvvvvvvvvvvvvvvvvvvvvvvvvvvvvvvvr‬‬

‫‪Ŀ‬‬
‫ﳎﻤﻮﻋﺔ اﻷﻋﺪاد اﻟﻄﺒﻴﻌﻴﺔ‬ ‫‪ℕ‬‬

‫ﳎﻤﻮﻋﺔ اﻷﻋﺪاد اﻟﺼﺤﻴﺤﺔ‬ ‫‪ℤ‬‬

‫ﳎﻤﻮﻋﺔ اﻷﻋﺪاد اﻟﻨﺎﻃﻘﺔ‬ ‫‪ℚ‬‬

‫ﳎﻤﻮﻋﺔ اﻷﻋﺪاد اﳊﻘﻴﻘﻴﺔ‬ ‫‪ℝ‬‬

‫∗‬
‫ﳎﻤﻮﻋﺔ اﻷﻋﺪاد اﳊﻘﻴﻘﻴﺔ ﻏﲑ اﳌﻌﺪوﻣﺔ‬ ‫‪ℝ‬‬

‫ﳎﻤﻮﻋﺔ اﻷزواج اﳌﺮﺗﺒﺔ ﻣﻦ اﻷﻋﺪاد اﳊﻘﻴﻘﻴﺔ‬ ‫‪ℝ‬‬

‫ﳎﻤﻮﻋﺔ اﻷﻋﺪاد اﳌﺮﻛﺒﺔ‬ ‫‪ℂ‬‬

‫∗‬
‫ﳎﻤﻮﻋﺔ اﻷﻋﺪاد اﳌﺮﻛﺒﺔ ﻏﲑ اﳌﻌﺪوﻣﺔ‬ ‫‪ℂ‬‬
‫‪ņőƱŀ Ţ‬‬
‫ﳎﻤﻮﻋﺔ اﻷﻋﺪاد اﳊﻘﻴﻘﻴﺔ ‪ x‬ﺑﺤﻴﺚ ‪) a ≤ x ≤ b‬ﳎﺎل ﻣﻐﻠﻖ(‬ ‫]‪[a, b‬‬

‫ﳎﻤﻮﻋﺔ اﻷﻋﺪاد اﳊﻘﻴﻘﻴﺔ ‪ x‬ﺑﺤﻴﺚ ‪) a < x < b‬ﳎﺎل ﻣﻔﺘﻮح(‬ ‫[‪]a, b‬‬

‫ﻣﺮاﻓﻖ اﻟﻌﺪد اﳌﺮﻛﺐ ‪z‬‬ ‫‪z‬‬

‫ﻃﻮﻳﻠﺔ اﻟﻌﺪد اﳌﺮﻛﺐ ‪z‬‬ ‫|‪|z‬‬


‫‪ƕ‬‬

‫اﻟﺸﻌﺎع اﻟﺬي ﺑﺪاﻳﺘﻪ اﻟﻨﻘﻄﺔ ‪ A‬و ﳖﺎﻳﺘﻪ اﻟﻨﻘﻄﺔ ‪B‬‬ ‫‪AB‬‬

‫اﳌﺴﺘﻘﻴﻢ اﳌﺎر ﺑﺎﻟﻨﻘﻄﺘﲔ ‪ A‬و ‪B‬‬ ‫)‪(AB‬‬


‫‪ũŏ‬‬

‫ﻗﻄﻌﺔ اﳌﺴﺘﻘﻴﻢ اﻟﺘﻲ ﺑﺪاﻳﺘﻬﺎ اﻟﻨﻘﻄﺔ ‪ A‬و ﳖﺎﻳﺘﻬﺎ اﻟﻨﻘﻄﺔ ‪B‬‬ ‫]‪[AB‬‬

‫ﳎﻤﻮﻋﺔ اﳉﺬور اﻟﻨﻮﻧﻴﺔ ﻟﻠﻮﺣﺪة‬ ‫‪U‬‬

‫اﻟﺪاﺋﺮة اﻟﺘﻲ ﻣﺮﻛﺰﻫﺎ اﻟﻨﻘﻄﺔ ‪ P‬و ﻧﺼﻒ ﻗﻄﺮﻫﺎ ‪r‬‬ ‫)‪𝒞 (P, r‬‬

‫𝗶𝗶𝘃‬

‫‪http ://tinyurl.com/Malki1718‬‬ ‫‪0‬‬


Ŕž Ŧ Ľ
œǃ
Ŀ
ņőƱŀ Ţ
ũŏ ƕ

http ://tinyurl.com/Malki1718 0
‫‪Ŕž Ŧ Ľ‬‬
‫‪œǃ‬‬
‫‪otttttttttttttttttttttttttttttttttttttttttttp‬‬
‫‪s‬‬ ‫‪u‬‬
‫‪s‬‬
‫‪s‬‬ ‫اﻟﻤﺤﺘﻮﻳﺎت )ﺑﺎﺧﺘﺼﺎر(‬ ‫‪u‬‬
‫‪u‬‬
‫‪s‬‬ ‫‪u‬‬
‫‪qvvvvvvvvvvvvvvvvvvvvvvvvvvvvvvvvvvvvvvvvvvvr‬‬

‫‪1‬‬
‫‪3‬‬

‫‪15‬‬

‫‪41‬‬

‫‪51‬‬
‫‪Ŀ‬‬ ‫اﻟﺪرس‬
‫ﳎﻤﻮﻋﺔ اﻷﻋﺪاد اﳌﺮﻛﺒﺔ‬

‫‪ II‬اﻟﺸﻜﻞ اﳌﺜﻠﺜﻲ ﻟﻌﺪد ﻣﺮﻛﺐ‬

‫‪ III‬ﺗﻄﺒﻴﻘﺎت اﻷﻋﺪاد اﳌﺮﻛﺒﺔ‬

‫‪ IV‬ﲤﺜﻴﻞ اﻟﺘﺤﻮﻳﻼت اﻟﻨﻘﻄﻴﺔ ﰲ اﳌﺴﺘﻮي اﳌﺮﻛﺐ‬


‫‪I‬‬

‫‪63‬‬ ‫ﺗﻤﺎرﻳﻦ ﻣﺤﻠﻮﻟﺔ‬


‫‪ V‬اﺧﺘﱪ اﺳﺘﻴﻌﺎﺑﻚ ﻟﻠﺪرس‬
‫‪ņőƱŀ Ţ‬‬
‫‪65‬‬

‫‪69‬‬ ‫‪ VI‬ﲤﺎرﻳﻦ ﺗﻄﺒﻴﻘﻴﺔ‬

‫‪259‬‬ ‫‪ VII‬ﲤﺎرﻳﻦ ﻟﻠﺘﻌﻤﻖ‬

‫‪371‬‬ ‫ﻣﻼﺣﻖ‬
‫‪373‬‬ ‫ﻛﺜﲑات اﳊﺪود‬ ‫ا‬
‫‪ƕ‬‬

‫‪387‬‬ ‫ب اﳌﻌﺎدﻻت ﻣﻦ اﻟﺪرﺟﺔ اﻟﺜﺎﻟﺜﺔ و اﻟﺮاﺑﻌﺔ‬

‫‪403‬‬ ‫ج اﳊﺮوف اﻟﻴﻮﻧﺎﻧﻴﺔ و اﻟﻼﺗﻨﻴﺔ‬


‫‪ũŏ‬‬

‫‪407‬‬ ‫ﻣﻠﺨﺺ اﻟﺪرس‬ ‫د‬

‫‪415‬‬ ‫ﺣﺴﺎب اﳌﺜﻠﺜﺎت‬ ‫ﻫ‬

‫𝘅𝗶‬

‫‪http ://tinyurl.com/Malki1718‬‬ ‫‪0‬‬


Ŕž Ŧ Ľ
œǃ
Ŀ
ņőƱŀ Ţ
ũŏ ƕ

http ://tinyurl.com/Malki1718 0
‫‪Ŕž Ŧ Ľ‬‬
‫‪œǃ‬‬
‫‪otttttttttttttttttttttttttttttttttttttttttttp‬‬
‫‪s‬‬ ‫‪u‬‬
‫‪s‬‬
‫‪s‬‬
‫اﻟﻤﺤﺘﻮﻳﺎت‬
‫‪u‬‬
‫‪u‬‬
‫‪qvvvvvvvvvvvvvvvvvvvvvvvvvvvvvvvvvvvvvvvvvvvr‬‬

‫‪1‬‬
‫‪3‬‬
‫‪3‬‬
‫‪3‬‬
‫‪8‬‬
‫‪8‬‬
‫‪8‬‬
‫‪11‬‬
‫‪11‬‬
‫‪12‬‬
‫‪Ŀ‬‬
‫ﻣﻘﺪﻣﺔ ‪. . . . . . . . . . . . . . . . . . . . . . . . . . . . . . . . . . . . . .‬‬

‫ﺗﻌﺮﻳﻒ ‪. . . . . . . . . . . . . . . . . . . . . . . . . . . . . . . . .‬‬
‫ﺧﻮاص ‪. . . . . . . . . . . . . . . . . . . . . . . . . . . . . . . . .‬‬

‫اﻟﺘﻤﺜﻴﻞ اﻟﻨﻘﻄﻲ ‪. . . . . . . . . . . . . . . . . . . . . . . . . . . . . .‬‬


‫اﻟﺘﻤﺜﻴﻞ اﻟﺸﻌﺎﻋﻲ ‪. . . . . . . . . . . . . . . . . . . . . . . . . . . . .‬‬
‫اﻟﺪرس‬
‫ﳎﻤﻮﻋﺔ اﻷﻋﺪاد اﳌﺮﻛﺒﺔ‬

‫‪ 1.I‬ﲢﻠﻴﻞ اﳌﺴﺄﻟﺔ اﳌﻄﺮوﺣﺔ ‪. . . . . . . . . . . . . . . . . . . . . . . . . . . . . . . .‬‬


‫‪ 2.I‬ﻣﺮاﻓﻖ ﻋﺪد ﻣﺮﻛﺐ ‪. . . . . . . . . . . . . . . . . . . . . . . . . . . . . . . . .‬‬
‫‪1.2.I‬‬
‫‪2.2.I‬‬
‫‪ 3.I‬اﻟﺘﻤﺜﻴﻞ اﳍﻨﺪﳼ ﻟﻌﺪد ﻣﺮﻛﺐ ‪. . . . . . . . . . . . . . . . . . . . . . . . . . . . .‬‬
‫‪1.3.I‬‬
‫‪2.3.I‬‬
‫‪I‬‬

‫‪15‬‬ ‫‪ II‬اﻟﺸﻜﻞ اﳌﺜﻠﺜﻲ ﻟﻌﺪد ﻣﺮﻛﺐ‬


‫‪15‬‬ ‫‪ 1.II‬ﻃﻮﻳﻠﺔ ﻋﺪد ﻣﺮﻛﺐ ‪. . . . . . . . . . . . . . . . . . . . . . . . . . . . . . . . .‬‬
‫‪ 1.1.II‬ﺗﻌﺮﻳﻒ ‪. . . . . . . . . . . . . . . . . . . . . . . . . . . . . . . . .‬‬
‫‪ņőƱŀ Ţ‬‬
‫‪15‬‬
‫‪16‬‬ ‫‪ 2.1.II‬ﺧﻮاص ‪. . . . . . . . . . . . . . . . . . . . . . . . . . . . . . . . .‬‬
‫‪18‬‬ ‫‪ 2.II‬ﻋﻤﺪة ﻋﺪد ﻣﺮﻛﺐ ﻏﲑ ﻣﻌﺪوم ‪. . . . . . . . . . . . . . . . . . . . . . . . . . . . .‬‬
‫‪18‬‬ ‫‪ 1.2.II‬ﺗﻌﺮﻳﻒ ‪. . . . . . . . . . . . . . . . . . . . . . . . . . . . . . . . .‬‬
‫‪19‬‬ ‫‪ 2.2.II‬اﻟﺸﻜﻞ اﳌﺜﻠﺜﻲ ﻟﻌﺪد ﻣﺮﻛﺐ ﻏﲑ ﻣﻌﺪوم ‪. . . . . . . . . . . . . . . . . . . .‬‬
‫‪20‬‬ ‫‪ 3.2.II‬اﻹﻧﺘﻘﺎل ﻣﻦ اﻟﺸﻜﻞ اﳉﱪي إﱃ اﻟﺸﻜﻞ اﳌﺜﻠﺜﻲ و اﻟﻌﻜﺲ ‪. . . . . . . . . . . . . .‬‬
‫‪21‬‬ ‫‪ 3.II‬اﻷﻋﺪاد اﳌﺮﻛﺒﺔ اﳌﻜﺘﻮﺑﺔ ﻋﲆ اﻟﺸﻜﻞ اﳌﺜﻠﺜﻲ ‪. . . . . . . . . . . . . . . . . . . . . . . .‬‬
‫‪21‬‬ ‫‪ 1.3.II‬ﺣﺎﺻﻞ ﴐب ﻋﺪدﻳﻦ ﻣﺮﻛﺒﲔ ‪. . . . . . . . . . . . . . . . . . . . . . . .‬‬
‫‪23‬‬ ‫‪ 2.3.II‬ﺣﺎﺻﻞ ﻗﺴﻤﺔ ﻋﺪدﻳﻦ ﻣﺮﻛﺒﲔ ﻏﲑ ﻣﻌﺪوﻣﲔ ‪. . . . . . . . . . . . . . . . . .‬‬
‫‪ 4.II‬دﺳﺘﻮر ﻣﻮاﻓﺮ ‪. . . . . . . . . . . . . . . . . . . . . . . . . . . . . . . . . . . .‬‬
‫‪ƕ‬‬

‫‪25‬‬
‫‪26‬‬ ‫‪ 5.II‬اﳉﺬور اﻟﻨﻮﻧﻴﺔ ﻟﻌﺪد ﻣﺮﻛﺐ ‪. . . . . . . . . . . . . . . . . . . . . . . . . . . . . .‬‬
‫‪30‬‬ ‫‪ 6.II‬اﳉﺬور اﻟﻨﻮﻧﻴﺔ ﻟﻠﻮﺣﺪة ‪. . . . . . . . . . . . . . . . . . . . . . . . . . . . . . . .‬‬
‫‪34‬‬ ‫‪ 7.II‬اﳌﻌﺎدﻟﺔ ‪. . . . . . . . . . . . . . . . . . . . . . . . . . . . . . . . . . Z = a‬‬
‫‪36‬‬ ‫‪ 8.II‬اﻟﺪاﻟﺔ اﻷﺳﻴﺔ اﳌﺮﻛﺒﺔ ‪. . . . . . . . . . . . . . . . . . . . . . . . . . . . . . . . .‬‬
‫‪ũŏ‬‬

‫‪41‬‬ ‫‪ III‬ﺗﻄﺒﻴﻘﺎت اﻷﻋﺪاد اﳌﺮﻛﺒﺔ‬


‫‪ 1.III‬اﳉﺬور اﻟﱰﺑﻴﻌﻴﺔ ﻟﻌﺪد ﻣﺮﻛﺐ ‪41 . . . . . . . . . . . . . . . . . . . . . . . . . . . . .‬‬
‫‪ 2.III‬ﺗﻌﻴﲔ اﳉﺬور اﻟﱰﺑﻴﻌﻴﺔ ﻟﻌﺪد ﻣﺮﻛﺐ ‪41 . . . . . . . . . . . . . . . . . . . . . . . . . . .‬‬
‫‪ 1.2.III‬اﻟﻄﺮﻳﻘﺔ اﻷوﱃ ‪ :‬ﺑﺎﺳﺘﻌﲈل اﻟﺸﻜﻞ اﳌﺜﻠﺜﻲ ‪41 . . . . . . . . . . . . . . . . . . . .‬‬
‫‪ 2.2.III‬اﻟﻄﺮﻳﻘﺔ اﻟﺜﺎﻧﻴﺔ ‪ :‬ﺑﺎﺳﺘﻌﲈل اﻟﺸﻜﻞ اﳉﱪي ‪42 . . . . . . . . . . . . . . . . . . .‬‬

‫𝗶𝘅‬

‫‪http ://tinyurl.com/Malki1718‬‬ ‫‪0‬‬


‫اﶈﺘﻮايت‬

‫اﳌﻌﺎدﻻت ﻣﻦ اﻟﺪرﺟﺔ اﻟﺜﺎﻧﻴﺔ ‪43 . . . . . . . . . . . . . . . . . . . . . . . . . . . . . .‬‬ ‫‪3.III‬‬


‫‪ 1.3.III‬اﳊﻞ ‪ ،‬ﰲ ‪ ، ℂ‬ﳌﻌﺎدﻟﺔ ﻣﻦ اﻟﺪرﺟﺔ اﻟﺜﺎﻧﻴﺔ ‪43 . . . . . . . . . . . . . . . . . . . .‬‬

‫‪Ŕž Ŧ Ľ‬‬
‫‪ 2.3.III‬اﳊﻞ ‪ ،‬ﰲ ‪ ، ℂ‬ﳌﻌﺎدﻟﺔ ﻣﻦ اﻟﺪرﺟﺔ اﻟﺜﺎﻧﻴﺔ ﺑﻤﻌﺎﻣﻼت ﺣﻘﻴﻘﻴﺔ ‪45 . . . . . . . . . . . . .‬‬
‫ﺣﺴﺎب 𝜃‪ cos n‬و 𝜃‪ sin n‬ﺑﺪﻻﻟﺔ 𝜃 ‪ cos‬و 𝜃 ‪45 . . . . . . . . . . . . . . . . . . . . . . sin‬‬ ‫‪4.III‬‬

‫‪œǃ‬‬
‫اﻟﻌﺒﺎرة اﳋﻄﻴﺔ ﻟﻜﻞ ﻣﻦ 𝜃 ‪ cos‬و 𝜃 ‪47 . . . . . . . . . . . . . . . . . . . . . . . . sin‬‬ ‫‪5.III‬‬

‫‪51‬‬ ‫‪ IV‬ﲤﺜﻴﻞ اﻟﺘﺤﻮﻳﻼت اﻟﻨﻘﻄﻴﺔ ﰲ اﳌﺴﺘﻮي اﳌﺮﻛﺐ‬


‫‪ 1.IV‬اﻷﻋﺪاد اﳌﺮﻛﺒﺔ و اﳍﻨﺪﺳﺔ اﳌﺴﺘﻮﻳﺔ ‪51 . . . . . . . . . . . . . . . . . . . . . . . . . . .‬‬

‫‪Ŀ‬‬
‫‪ 1.1.IV‬اﳌﺴﺎﻓﺔ ‪51 . . . . . . . . . . . . . . . . . . . . . . . . . . . . . . . . .‬‬
‫‪ 2.1.IV‬ﻣﺮﻛﺰ اﳌﺴﺎﻓﺎت اﳌﺘﻨﺎﺳﺒﺔ ‪51 . . . . . . . . . . . . . . . . . . . . . . . . . .‬‬
‫‪ 3.1.IV‬اﻟﺰواﻳﺎ ‪52 . . . . . . . . . . . . . . . . . . . . . . . . . . . . . . . . .‬‬
‫‪ 4.1.IV‬اﳉﺪاء اﻟﺴﻠﻤﻲ ‪53 . . . . . . . . . . . . . . . . . . . . . . . . . . . . . .‬‬
‫‪ 2.IV‬ﲤﺜﻴﻞ اﻟﺘﺤﻮﻳﻼت اﻟﻨﻘﻄﻴﺔ ﰲ اﳌﺴﺘﻮي اﳌﺮﻛﺐ ‪53 . . . . . . . . . . . . . . . . . . . . . . .‬‬
‫‪ 1.2.IV‬اﻹﻧﺴﺤﺎب و اﻟﺘﺤﺎﻛﻲ ‪54 . . . . . . . . . . . . . . . . . . . . . . . . . . .‬‬
‫‪ 2.2.IV‬اﻟﺪوران ‪56 . . . . . . . . . . . . . . . . . . . . . . . . . . . . . . . . .‬‬
‫‪ 3.IV‬اﻟﺘﺸﺎﺑﻪ اﳌﺴﺘﻮي اﳌﺒﺎﴍ ‪57 . . . . . . . . . . . . . . . . . . . . . . . . . . . . . . . .‬‬
‫‪ 1.3.IV‬ﲢﻮﻳﻞ ﻧﻘﻄﻲ آﺧﺮ ‪ :‬اﻹﻧﺰﻻق ‪61 . . . . . . . . . . . . . . . . . . . . . . . .‬‬

‫‪63‬‬ ‫ﺗﻤﺎرﻳﻦ ﻣﺤﻠﻮﻟﺔ‬


‫‪65‬‬ ‫‪ V‬اﺧﺘﱪ اﺳﺘﻴﻌﺎﺑﻚ ﻟﻠﺪرس‬

‫‪69‬‬ ‫‪ VI‬ﲤﺎرﻳﻦ ﺗﻄﺒﻴﻘﻴﺔ‬


‫‪69‬‬ ‫‪.‬‬ ‫‪.‬‬ ‫‪.‬‬ ‫‪.‬‬ ‫‪.‬‬ ‫‪.‬‬ ‫‪.‬‬ ‫‪.‬‬ ‫‪.‬‬ ‫‪.‬‬ ‫‪.‬‬ ‫‪.‬‬ ‫‪.‬‬ ‫‪.‬‬ ‫‪.‬‬ ‫‪.‬‬ ‫‪.‬‬ ‫‪.‬‬ ‫‪.‬‬ ‫‪.‬‬ ‫‪.‬‬ ‫‪.‬‬ ‫‪.‬‬ ‫‪.‬‬ ‫‪.‬‬ ‫‪.‬‬ ‫‪.‬‬ ‫‪.‬‬ ‫‪.‬‬ ‫‪.‬‬ ‫‪ 1.VI‬ﳎﻤﻮﻋﺔ اﻷﻋﺪاد اﳌﺮﻛﺒﺔ ‪. .‬‬
‫‪115‬‬ ‫‪.‬‬ ‫‪.‬‬ ‫‪.‬‬ ‫‪.‬‬ ‫‪.‬‬ ‫‪.‬‬ ‫‪.‬‬ ‫‪.‬‬ ‫‪.‬‬ ‫‪.‬‬ ‫‪.‬‬ ‫‪.‬‬ ‫‪.‬‬ ‫‪.‬‬ ‫‪.‬‬ ‫‪.‬‬ ‫‪.‬‬ ‫‪.‬‬ ‫‪.‬‬ ‫‪.‬‬ ‫‪.‬‬ ‫‪.‬‬ ‫‪.‬‬ ‫‪.‬‬ ‫‪.‬‬ ‫‪.‬‬ ‫‪.‬‬ ‫‪.‬‬ ‫‪.‬‬ ‫‪.‬‬ ‫‪ 2.VI‬اﻟﺸﻜﻞ اﳌﺜﻠﺜﻲ ﻟﻌﺪد ﻣﺮﻛﺐ‬
‫‪149‬‬ ‫‪.‬‬ ‫‪.‬‬ ‫‪.‬‬ ‫‪.‬‬ ‫‪.‬‬ ‫‪.‬‬ ‫‪.‬‬ ‫‪.‬‬ ‫‪.‬‬ ‫‪.‬‬ ‫‪.‬‬ ‫‪.‬‬ ‫‪.‬‬ ‫‪.‬‬ ‫‪.‬‬ ‫‪.‬‬ ‫‪.‬‬ ‫‪.‬‬ ‫‪.‬‬ ‫‪.‬‬ ‫‪.‬‬ ‫‪.‬‬ ‫‪.‬‬ ‫‪.‬‬ ‫‪.‬‬ ‫‪.‬‬ ‫‪.‬‬ ‫‪.‬‬ ‫‪.‬‬ ‫‪.‬‬ ‫‪ 3.VI‬ﺗﻄﺒﻴﻘﺎت اﻷﻋﺪاد اﳌﺮﻛﺒﺔ ‪.‬‬
‫‪217‬‬ ‫‪.‬‬ ‫‪.‬‬ ‫‪.‬‬ ‫‪.‬‬ ‫‪.‬‬ ‫‪.‬‬ ‫‪.‬‬ ‫‪.‬‬ ‫‪.‬‬ ‫‪.‬‬ ‫‪.‬‬ ‫‪.‬‬ ‫‪.‬‬ ‫‪.‬‬ ‫‪.‬‬ ‫‪.‬‬ ‫‪.‬‬ ‫‪.‬‬ ‫‪.‬‬ ‫‪.‬‬ ‫‪.‬‬ ‫‪.‬‬ ‫‪.‬‬ ‫‪.‬‬ ‫‪.‬‬ ‫‪.‬‬ ‫‪.‬‬ ‫‪.‬‬ ‫‪.‬‬ ‫‪.‬‬ ‫‪ 4.VI‬ﲤﺎرﻳﻦ ﻣﺘﻨﻮﻋﺔ ‪. . . . .‬‬
‫‪ņőƱŀ Ţ‬‬
‫‪259‬‬ ‫‪ VII‬ﲤﺎرﻳﻦ ﻟﻠﺘﻌﻤﻖ‬

‫‪371‬‬ ‫ﻣﻼﺣﻖ‬
‫‪373‬‬ ‫ﻛﺜﲑات اﳊﺪود‬ ‫ا‬
‫‪373‬‬ ‫ا‪ 1.‬ﳎﻤﻮﻋﺔ ﻛﺜﲑات اﳊﺪود ﺑﻤﻌﺎﻣﻼت ﰲ 𝕂 ‪. . . . . . . . . . . . . . . . . . . . . . . .‬‬
‫‪375‬‬ ‫ا‪ 2.‬اﻟﻘﺴﻤﺔ ﰲ ]‪. . . . . . . . . . . . . . . . . . . . . . . . . . . . . . . . . . 𝕂[X‬‬
‫‪ƕ‬‬

‫‪375‬‬ ‫ﻋﻼﻗﺔ اﻟﻘﺴﻤﺔ ‪. . . . . . . . . . . . . . . . . . . . . . . . . . . . . .‬‬ ‫ا‪1.2.‬‬


‫‪375‬‬ ‫اﻟﻘﺴﻤﺔ اﻹﻗﻠﻴﺪﻳﺔ ‪. . . . . . . . . . . . . . . . . . . . . . . . . . . . .‬‬ ‫ا‪2.2.‬‬
‫‪376‬‬ ‫ا‪ 3.‬اﻟﻘﻮاﺳﻢ و اﳌﻀﺎﻋﻔﺎت اﳌﺸﱰﻛﺔ ‪. . . . . . . . . . . . . . . . . . . . . . . . . . . .‬‬
‫‪376‬‬ ‫اﻟﻘﺎﺳﻢ اﳌﺸﱰك اﻷﻛﱪ ‪. . . . . . . . . . . . . . . . . . . . . . . . . . .‬‬ ‫ا‪1.3.‬‬
‫‪ũŏ‬‬

‫‪378‬‬ ‫ﻛﺜﲑات اﳊﺪود اﻷوﻟﻴﺔ ﻓﻴﲈ ﺑﻴﻨﻬﺎ ‪. . . . . . . . . . . . . . . . . . . . . . .‬‬ ‫ا‪2.3.‬‬


‫‪379‬‬ ‫اﳌﻀﺎﻋﻒ اﳌﺸﱰك اﻷﺻﻐﺮ ‪. . . . . . . . . . . . . . . . . . . . . . . . .‬‬ ‫ا‪3.3.‬‬
‫‪379‬‬ ‫ا‪ 4.‬ﻛﺜﲑات اﳊﺪود ﻏﲑ اﻟﻘﺎﺑﻠﺔ ﻟﻠﺘﺤﻠﻴﻞ ‪. . . . . . . . . . . . . . . . . . . . . . . . . . .‬‬
‫‪380‬‬ ‫ا‪ 5.‬اﻟﺪوال ﻛﺜﲑات اﳊﺪود و اﳉﺬور ‪. . . . . . . . . . . . . . . . . . . . . . . . . . . .‬‬
‫‪380‬‬ ‫اﻟﺪوال ﻛﺜﲑات اﳊﺪود ‪. . . . . . . . . . . . . . . . . . . . . . . . . . .‬‬ ‫ا‪1.5.‬‬
‫‪380‬‬ ‫ﺟﺬور ﻛﺜﲑات اﳊﺪود ‪. . . . . . . . . . . . . . . . . . . . . . . . . . .‬‬ ‫ا‪2.5.‬‬

‫𝗶𝗶𝘅‬

‫‪http ://tinyurl.com/Malki1718‬‬ ‫‪0‬‬


‫اﶈﺘﻮايت‬

‫إﺷﺘﻘﺎق ﻛﺜﲑات اﳊﺪود ‪381 . . . . . . . . . . . . . . . . . . . . . . . . . .‬‬ ‫ا‪3.5.‬‬


‫اﻟﻌﻼﻗﺎت ﺑﲔ اﳌﻌﺎﻣﻼت و اﳉﺬور ‪382 . . . . . . . . . . . . . . . . . . . . . .‬‬ ‫ا‪4.5.‬‬

‫‪Ŕž Ŧ Ľ‬‬
‫اﻟﻨﻈﺮﻳﺔ اﻷﺳﺎﺳﻴﺔ ﰲ اﳉﱪ ‪385 . . . . . . . . . . . . . . . . . . . . . . . . . .‬‬ ‫ا‪5.5.‬‬

‫‪387‬‬ ‫ب اﳌﻌﺎدﻻت ﻣﻦ اﻟﺪرﺟﺔ اﻟﺜﺎﻟﺜﺔ و اﻟﺮاﺑﻌﺔ‬

‫‪œǃ‬‬
‫‪387‬‬ ‫ب‪ 1.‬اﳌﻌﺎدﻻت اﻟﺘﻜﻌﻴﺒﻴﺔ ‪. . . . . . . . . . . . . . . . . . . . . . . . . . . . . . . . .‬‬
‫‪387‬‬ ‫ب‪ 1.1.‬ﻣﻘﺪﻣﺔ ﺗﺎرﳜﻴﺔ ‪. . . . . . . . . . . . . . . . . . . . . . . . . . . . . .‬‬
‫‪387‬‬ ‫ب‪ 2.1.‬اﻟﻄﺮﻳﻘﺔ اﻷوﱃ ‪ :‬ﻃﺮﻳﻘﺔ دال ﻓﲑو ‪. . . . . . . . . . . . . . . . . . . . . . .‬‬
‫‪393‬‬ ‫ب‪ 3.1.‬اﻟﻄﺮﻳﻘﺔ اﻟﺜﺎﻧﻴﺔ ‪ :‬ﻃﺮﻳﻘﺔ ﻻﭬﺮاﻧﺞ ‪. . . . . . . . . . . . . . . . . . . . . . .‬‬

‫‪Ŀ‬‬
‫‪394‬‬ ‫ب‪ 4.1.‬اﻟﻄﺮﻳﻘﺔ اﻟﺜﺎﻟﺜﺔ ‪ :‬ﻃﺮﻳﻘﺔ ﻓﻴﻴﺖ ‪. . . . . . . . . . . . . . . . . . . . . . . .‬‬
‫‪396‬‬ ‫ب‪ 2.‬اﳌﻌﺎدﻻت ﻣﻦ اﻟﺪرﺟﺔ اﻟﺮاﺑﻌﺔ ‪. . . . . . . . . . . . . . . . . . . . . . . . . . . . .‬‬
‫‪396‬‬ ‫ب‪ 1.2.‬اﻟﻄﺮﻳﻘﺔ اﻷوﱃ ‪ :‬ﻃﺮﻳﻘﺔ دﻳﻜﺎرت ‪. . . . . . . . . . . . . . . . . . . . . . .‬‬
‫‪398‬‬ ‫ب‪ 2.2.‬اﻟﻄﺮﻳﻘﺔ اﻟﺜﺎﻧﻴﺔ ‪ :‬ﻃﺮﻳﻘﺔ ﻓﲑاري ‪. . . . . . . . . . . . . . . . . . . . . . . .‬‬
‫‪399‬‬ ‫ب‪ 3.2.‬اﻟﻄﺮﻳﻘﺔ اﻟﺜﺎﻟﺜﺔ ‪ :‬ﻃﺮﻳﻘﺔ ﻻﭬﺮاﻧﺞ ‪. . . . . . . . . . . . . . . . . . . . . . .‬‬
‫‪400‬‬ ‫ب‪ 3.‬و اﳌﻌﺎدﻻت ﻣﻦ اﻟﺪرﺟﺔ اﳋﺎﻣﺴﺔ ؟ ‪. . . . . . . . . . . . . . . . . . . . . . . . . . .‬‬

‫‪403‬‬ ‫ج اﳊﺮوف اﻟﻴﻮﻧﺎﻧﻴﺔ و اﻟﻼﺗﻨﻴﺔ‬


‫‪403 . . . . . . . . . . . . . . . . . . . . . . . . . . . . . . . . .‬‬ ‫ج‪ 1.‬اﻷرﻗﺎم ‪. . . . .‬‬
‫‪404 . . . . . . . . . . . . . . . . . . . . . . . . . . . . . . . . .‬‬ ‫ج‪ 2.‬اﳊﺮوف اﻟﻴﻮﻧﺎﻧﻴﺔ ‪.‬‬
‫‪405 . . . . . . . . . . . . . . . . . . . . . . . . . . . . . . . . .‬‬ ‫ج‪ 3.‬اﳊﺮوف اﻟﻼﺗﻨﻴﺔ ‪.‬‬

‫‪407‬‬ ‫ﻣﻠﺨﺺ اﻟﺪرس‬ ‫د‬


‫‪407‬‬ ‫‪.‬‬ ‫‪.‬‬ ‫‪.‬‬ ‫‪.‬‬ ‫‪.‬‬ ‫‪.‬‬ ‫‪.‬‬ ‫‪.‬‬ ‫‪.‬‬ ‫‪.‬‬ ‫‪.‬‬ ‫‪.‬‬ ‫‪.‬‬ ‫‪.‬‬ ‫‪.‬‬ ‫‪.‬‬ ‫‪.‬‬ ‫‪.‬‬ ‫‪.‬‬ ‫‪.‬‬ ‫‪.‬‬ ‫‪.‬‬ ‫‪.‬‬ ‫‪.‬‬ ‫‪.‬‬ ‫د‪ 1.‬اﻟﺸﻜﻞ اﳉﱪي ﻟﻸﻋﺪاد اﳌﺮﻛﺒﺔ ‪. . . .‬‬
‫‪408‬‬ ‫‪.‬‬ ‫‪.‬‬ ‫‪.‬‬ ‫‪.‬‬ ‫‪.‬‬ ‫‪.‬‬ ‫‪.‬‬ ‫‪.‬‬ ‫‪.‬‬ ‫‪.‬‬ ‫‪.‬‬ ‫‪.‬‬ ‫‪.‬‬ ‫‪.‬‬ ‫‪.‬‬ ‫‪.‬‬ ‫‪.‬‬ ‫‪.‬‬ ‫‪.‬‬ ‫‪.‬‬ ‫‪.‬‬ ‫‪.‬‬ ‫‪.‬‬ ‫‪.‬‬ ‫‪.‬‬ ‫د‪ 2.‬اﻟﺘﻔﺴﲑ اﳍﻨﺪﳼ ﻟﻸﻋﺪاد اﳌﺮﻛﺒﺔ ‪. . .‬‬
‫‪410‬‬ ‫‪.‬‬ ‫‪.‬‬ ‫‪.‬‬ ‫‪.‬‬ ‫‪.‬‬ ‫‪.‬‬ ‫‪.‬‬ ‫‪.‬‬ ‫‪.‬‬ ‫‪.‬‬ ‫‪.‬‬ ‫‪.‬‬ ‫‪.‬‬ ‫‪.‬‬ ‫‪.‬‬ ‫‪.‬‬ ‫‪.‬‬ ‫‪.‬‬ ‫‪.‬‬ ‫‪.‬‬ ‫‪.‬‬ ‫‪.‬‬ ‫‪.‬‬ ‫‪.‬‬ ‫‪.‬‬ ‫د‪ 3.‬اﻟﺸﻜﻞ اﳌﺜﻠﺜﻲ )اﻷﳼ( ﻟﻸﻋﺪاد اﳌﺮﻛﺒﺔ‬
‫‪411‬‬ ‫‪.‬‬ ‫‪.‬‬ ‫‪.‬‬ ‫‪.‬‬ ‫‪.‬‬ ‫‪.‬‬ ‫‪.‬‬ ‫‪.‬‬ ‫‪.‬‬ ‫‪.‬‬ ‫‪.‬‬ ‫‪.‬‬ ‫‪.‬‬ ‫‪.‬‬ ‫‪.‬‬ ‫‪.‬‬ ‫‪.‬‬ ‫‪.‬‬ ‫‪.‬‬ ‫‪.‬‬ ‫‪.‬‬ ‫‪.‬‬ ‫‪.‬‬ ‫‪.‬‬ ‫‪.‬‬ ‫د‪ 4.‬ﲢﻮﻳﻼت اﳌﺴﺘﻮي اﳌﺮﻛﺐ ‪. . . . . .‬‬

‫‪415‬‬ ‫ﺣﺴﺎب اﳌﺜﻠﺜﺎت‬ ‫ﻫ‬


‫‪ņőƱŀ Ţ‬‬
‫‪ũŏ‬‬ ‫‪ƕ‬‬

‫𝗶𝗶𝗶𝘅‬

‫‪http ://tinyurl.com/Malki1718‬‬ ‫‪0‬‬


Ŕž Ŧ Ľ
œǃ
Ŀ
ņőƱŀ Ţ
ũŏ ƕ

http ://tinyurl.com/Malki1718 0
Ŕž Ŧ Ľ
œǃ
Ŀ ‫ﺍﻟﺪﺭﺱ‬
ņőƱŀ Ţ
ũŏ ƕ

http ://tinyurl.com/Malki1718 0
Ŕž Ŧ Ľ
œǃ
Ŀ
ņőƱŀ Ţ
ũŏ ƕ

http ://tinyurl.com/Malki1718 0
‫‪.‬‬

‫‪1‬‬
‫‪Ŕž Ŧ Ľ‬‬
‫‪I‬‬

‫‪œǃ‬‬
‫‪otttttttttttttttttttttttttttttttttttttttttttp‬‬
‫‪s‬‬ ‫‪u‬‬
‫‪s‬‬ ‫ﻣﺠﻤﻮﻋﺔ اﻷﻋﺪاد اﻟﻤﺮﻛﺒﺔ‬
‫‪u‬‬

‫‪Ŀ‬‬
‫‪s‬‬ ‫‪u‬‬
‫‪qvvvvvvvvvvvvvvvvvvvvvvvvvvvvvvvvvvvvvvvvvvvr‬‬
‫‪.‬‬

‫ﻣﻘﺪﻣﺔ‬
‫ﰲ أواﺋﻞ اﻟﻘﺮن اﻟﺴﺎدس ﻋﴩ ﻣﻴﻼدي‪ ،‬ﲤﻜّﻦ اﻟﻌﺎﱂ اﻹﻳﻄﺎﱄ ﺩﺍﻝ ﻓﻴﺮﻭ )‪ (D F‬ﻣﻦ إﳚﺎد ﺻﻴﻐﺔ ﻋﺎﻣﺔ ﳊﻠﻮل اﳌﻌﺎدﻻت‬
‫اﻟﺘﻜﻌﻴﺒﻴﺔ ‪) 1‬اﳌﻌﺎدﻻت ﻣﻦ اﻟﺪرﺟﺔ اﻟﺜﺎﻟﺜﺔ ‪ (2‬اﻟﺘﻲ ﻋﲆ اﻟﺸﻜﻞ ‪: x + ax = b‬‬
‫‪b−‬‬ ‫‪b + 4a /27‬‬ ‫‪b+‬‬ ‫‪b + 4a /27‬‬
‫=‪x‬‬ ‫‪+‬‬
‫‪2‬‬ ‫‪2‬‬
‫و ﺑﻌﺪ ﺳﻨﲔ ﻣﻦ ﻫﺬا اﻹﻛﺘﺸﺎف )ﰲ أواﺧﺮ اﻟﻘﺮن اﻟﺴﺎدس ﻋﴩ(‪ ،‬ﻗﺎم اﻟﻌﺎﱂ ﺑﻮﻣﺒﻴﻠﻲ )‪ (B‬ﺑﺘﻄﺒﻴﻖ ﻫﺬه اﻟﺼﻴﻐﺔ ﻋﲆ‬
‫اﳌﻌﺎدﻟﺔ ‪ x − 15x = 4‬ﻟﻴﺠﺪ ‪:‬‬
‫=‪x‬‬ ‫‪2 − 11√−1 +‬‬ ‫‪2 + 11√−1‬‬

‫و ﻫﺬه اﻟﻜﺘﺎﺑﺔ ﻻ ﻣﻌﻨﻰ ﳍﺎ ﻷﻧّﻨﺎ ﻻ ﻧﻌﻠﻢ ﻣﺎذا ﻳﻤ ّﺜﻞ اﻟﺮﻣﺰ ‪) √−1‬ﺟﺬور اﻷﻋﺪاد اﻟﺴﺎﻟﺒﺔ ﻏﲑ ّ‬
‫ﻣﻌﺮﻓﺔ(‪.‬‬
‫ﻣﻦ ﺟﻬﺔ أﺧﺮى‪ ،‬ﻻﺣﻆ ﺑﻮﻣﺒﻴﻠﻲ )ﺑﺎﺳﺘﺨﺪام اﻟﻘﻮاﻋﺪ اﻹﻋﺘﻴﺎدﻳﺔ ﻟﻠﺤﺴﺎب ‪ّ (3‬‬
‫‪ņőƱŀ Ţ‬‬
‫أن ‪:‬‬

‫‪2 − √−1‬‬ ‫‪= 2 − 11√−1‬‬ ‫و‬ ‫‪2 + √−1‬‬ ‫‪= 2 + 11√−1‬‬

‫=‪x‬‬ ‫‪2 − √−1‬‬ ‫‪+‬‬ ‫‪2 + √−1‬‬ ‫ﻟﻴﺠﺪ ﰲ اﻷﺧﲑ ‪= 2 − √−1 + 2 + √−1 = 4‬‬
‫أن ‪ّ x = 4‬‬
‫ﺣﻞ ﳍﺎ!‬ ‫و ﺑﺎﻟﺘﻌﻮﻳﺾ ﰲ اﳌﻌﺎدﻟﺔ ‪ x − 15x = 4‬وﺟﺪ ّ‬
‫و ﺑﺪراﺳﺔ أﻋﻤﻖ‪ ،‬اﺳﺘﺨﻠﺺ ّ‬
‫أن ﳎﻤﻮﻋﺔ ﺣﻠﻮل ﻫﺬه اﳌﻌﺎدﻟﺔ ﻫﻲ ‪. 4, −2 − √3, −2 + √3‬‬
‫ﻓﻮﺟﺪ ﻧﻔﺴﻪ أﻣﺎم ﺗﺴﺎءل »ﻃﺒﻴﻌﻲ و ﻣﴩوع«‪:‬‬
‫‪ƕ‬‬

‫ﻛﻴﻒ ﳝﻜﻦ ﺗﱪﻳﺮ ﺍﺳﺘﻌﻤﺎﻝ ﺭﻣﻮﺯ ”ﲣﻴ‪‬ﻠﻴ‪‬ﺔ“ ﻟﻠﺘﻌﺒﲑ ﻋﻦ ﺣﻠﻮﻝ ﻛﻠّﻬﺎ ﺣﻘﻴﻘﻴﺔ ﳍﺬﻩ ﺍﳌﻌﺎﺩﻟﺔ ﺍﻟﺒﺴﻴﻄﺔ ؟‬

‫ﲢﻠﻴﻞ ﺍﳌﺴﺄﻟﺔ ﺍﳌﻄﺮﻭﺣﺔ‬ ‫‪1.I‬‬


‫‪ũŏ‬‬

‫إذا ﲣﻴ‪‬ﻠﻨﺎ أﻧّﻪ ﻳﻮﺟﺪ ﻋﺪد ‪ ı‬ﻣﺮ ّﺑﻌﻪ ‪ −1‬و إذا ﺗﻌﺎﻣﻠﻨﺎ ﻣﻌﻪ ﰲ اﳊﺴﺎب و ﻛﺄﻧّﻪ ﻋﺪد ﺣﻘﻴﻘﻲ‪ ،‬ﻓﺈﻧّﻪ ّ‬
‫ﻟﻜﻞ ﻋﺪد ﺣﻘﻴﻘﻲ ﺳﺎﻟﺐ ‪،𝛼 < 0‬‬
‫ﻳﻜﻮن ‪:‬‬
‫‪1‬ﻇﻬﺮت اﻷﻋﺪاد اﳌﺮﻛﺒﺔ ﻷول ﻣﺮة ﻋﻨﺪ اﻟﺒﺤﺚ ﻋﻦ ﺣﻠﻮل اﳌﻌﺎدﻻت ﻣﻦ اﻟﺪرﺟﺔ اﻟﺜﺎﻟﺜﺔ و ﻟﻴﺲ اﻟﺜﺎﻧﻴﺔ ﻋﲆ ﺧﻼف ﻣﺎ ﻳﻤﻜﻦ ﺗﻮﻗﻌﻪ‪.‬‬
‫‪2‬ﰲ اﳌﻠﺤﻖ ب ﺻﻔﺤﺔ ‪ 387‬دراﺳﺔ ﻋﺎﻣﺔ ﻟﻠﻤﻌﺎدﻻت ﻣﻦ اﻟﺪرﺟﺔ اﻟﺜﺎﻟﺜﺔ و اﻟﺮاﺑﻌﺔ‪.‬‬
‫‪. +‬‬ ‫=‬ ‫‪+‬‬ ‫‪+‬‬ ‫‪+‬‬ ‫‪3‬ﻧﺬﻛّﺮ ﺑﺎﳌﺘﻄﺎﺑﻘﺔ اﻟﺸﻬﲑة ‪:‬‬

‫𝟯‬

‫‪http ://tinyurl.com/Malki1718‬‬ ‫‪0‬‬


‫‪ .1.I‬ﲢﻠﻴﻞ اﳌﺴﺄةل اﳌﻄﺮوﺣﺔ‬
‫‪.‬‬

‫𝛼‪ı√−‬‬ ‫‪=ı‬‬ ‫𝛼‪√−‬‬ ‫𝛼 = )𝛼‪= − (−‬‬

‫‪Ŕž Ŧ Ľ‬‬
‫و ﻫﻜﺬا ﻧﻜﻮن ﻗﺪ ﺣﻠﻠﻨﺎ ﻣﺸﻜﻠﺔ ﺟﺬور اﻷﻋﺪاد اﻟﺴﺎﻟﺒﺔ‪ .‬و ﻟﻜﻦ إذا ﺳﻤﺤﻨﺎ ﻷﻧﻔﺴﻨﺎ ﺑﺘﻄﺒﻴﻖ ﻗﻮاﻋﺪ اﳊﺴﺎب اﻹﻋﺘﻴﺎدﻳﺔ ﻋﲆ ﻫﺬا‬

‫‪I‬‬
‫ﱪا ﻣﻦ اﻷﻋﺪاد‪ ،‬ﻣﺜﻞ ‪ −2ı‬أو ‪ − √3ı‬أو أﻋﺪاد ِﺟﺪّ ﻣﻌ ّﻘﺪة ﻣﺜﻞ ‪:‬‬
‫اﻟﻌﺪد ﺍﻟﺘﺨﻴ‪‬ﻠﻲ ‪ ،ı‬ﻓﺈﻧّﻨﺎ »ﻧﺨﱰع«ﻛﲈ ُﻣﻌﺘ ً‬

‫‪œǃ‬‬
‫‪−‬‬
‫‪1 − 2ı‬‬ ‫‪1−ı‬‬
‫‪⎛ 2 − 5ı‬‬ ‫‪−‬‬
‫⎜‬ ‫⎞ ‪3 + 2ı‬‬
‫⎟‬
‫⎜=𝜆‬ ‫⎟‬
‫‪⎜ 2 − ı5‬‬ ‫‪−ı‬‬ ‫⎟‬

‫‪Ŀ‬‬
‫‪+‬‬
‫⎝‬ ‫‪2ı‬‬ ‫‪3 + 2ı‬‬ ‫⎠‬
‫ﻟﻠﻮﻫﻠﺔ اﻷوﱃ‪ ،‬ﺗﺒﺪو اﻷﻣﻮر ﰲ ﻏﺎﻳﺔ اﻟﺘّﻌﻘﻴﺪ‪ ،‬ﻟﻜﻦ إذا ﲤﻌﻨّﺎ اﻟﻨﻈﺮ ﻋﻦ ﻛﺜﺐ‪ ،‬ﻧﺠﺪﻫﺎ ﺧﻼف ذﻟﻚ ‪...‬‬

‫ﻧﺴﻤﻲ ﻋﺪدا ﻣﺮﻛّﺒﺎ ‪ّ 4‬‬


‫ﻛﻞ ﻋﺪد ﻣﻦ اﻟﺸﻜﻞ ‪ a + ıb‬ﺑﺤﻴﺚ ‪ a‬و ‪ b‬ﻋﺪدان ﺣﻘﻴﻘﻴﺎن و ‪ ı‬اﻟﻌﺪد اﻟﺘﺨﻴﲇ )أو اﻟﻮﺣﺪة‬ ‫ّ‬ ‫ﺗﻌﺮﻳﻒ ‪: 1‬‬
‫اﻟﺘﺨﻴﻠﻴﺔ( ‪ 5‬اﻟﺬي ﳛﻘﻖ ‪.ı = −1‬‬
‫‪ℂ = a + ıb ; a, b ∈ ℝ‬‬ ‫ﻧﺮﻣﺰ ﺑـِ ‪ ℂ‬إﱃ ﳎﻤﻮﻋﺔ اﻷﻋﺪاد اﳌﺮﻛّﺒﺔ ‪:‬‬

‫‪1‬‬
‫اﻷﻋﺪاد ‪ 1 + ı ،e + ı √7 ،𝜋 ،√2 ، ،−1 ،0‬و ‪ −2ı‬أﻋﺪاد ﻣﺮﻛﺒﺔ )أي ﺗﻨﺘﻤﻲ إﱃ اﳌﺠﻤﻮﻋﺔ ‪.(ℂ‬‬ ‫ﺃﻣﺜﻠﺔ ‪: 1‬‬
‫‪2‬‬

‫ﳊﺪّ اﻵن‪ ،‬ﻻ ﻳﻮﺟﺪ ﺗﻌﻘﻴﺪ ﻟﻜﻦ ﻛﻴﻒ ُﻳﻤﻜﻦ إﺟﺮاء اﻟﻌﻤﻠﻴﺎت اﻷﺳﺎﺳﻴﺔ ﻋﲆ ﻫﺬه اﻷﻋﺪاد ‪ :‬اﳉﻤﻊ‪ ،‬اﻟﻄﺮح‪ ،‬اﻟﴬب و اﻟﻘﺴﻤﺔ؟‬
‫ﺑﺎﻟﻨﺴﺒﺔ ﻟﻠﺠﻤﻊ و اﻟﻄﺮح‪ ،‬اﳉﻮاب ﺑﺴﻴﻂ ‪ :‬ﳎﻤﻮع أو ﻓﺮق ﻋﺪدﻳﻦ ﻣﺮﻛﺒﲔ ﻫﻮ ﻋﺪد ﻣﺮﻛﺐ ‪: 6‬‬
‫‪(a + ıb) + a + ıb‬‬ ‫‪= a+a +ı b+b‬‬ ‫)𝟏(‬

‫‪(a + ıb) − a + ıb‬‬ ‫‪= a−a +ı b−b‬‬ ‫)𝟐(‬


‫و ﻛﺬﻟﻚ اﻷﻣﺮ ﺑﺎﻟﻨﺴﺒﺔ ﻟﻠﴬب إذا أﺧﺬﻧﺎ ﺑﻌﲔ اﻹﻋﺘﺒﺎر اﳋﺎﺻﻴﺔ ‪:ı = −1‬‬
‫‪ņőƱŀ Ţ‬‬
‫‪(a + ıb) a + ıb‬‬ ‫‪= aa + aıb + ıba + ı bb = aa + ı ab + ba − bb‬‬

‫أي ‪:‬‬
‫‪(a + ıb) a + ıb‬‬ ‫‪= aa − bb + ı ab + a b‬‬ ‫)𝟑(‬
‫‪a + ıb‬‬
‫؟ ﻟﻜﺘﺎﺑﺔ اﻟﻜﴪ اﻟﺴﺎﺑﻖ ﻋﲆ ﺷﻜﻞ ﻋﺪد ﻣﺮﻛّﺐ ‪ ،‬ﻳﻜﻔﻲ أن ﻧﺠﻌﻞ ﻣﻘﺎﻣﻪ ﻋﺪ ًدا ﺣﻘﻴﻘ ًﻴﺎ‪ .‬ﻟﻜﻦ‬ ‫و ﻣﺎذا ﻋﻦ اﻟﻘﺴﻤﺔ ‪:‬‬
‫‪a + ıb‬‬
‫ﻛﻴﻒ ذﻟﻚ؟‬
‫أن ‪ (a + ıb) (a − ıb) = a + b‬و ﻫﻮ ﻋﺪد ﺣﻘﻴﻘﻲ‪ ،‬إ ًذا ‪:‬‬ ‫ﻧﻼﺣﻆ ّ‬
‫‪ƕ‬‬

‫‪a + ıb‬‬ ‫)‪a + ıb (a − ıb‬‬ ‫‪aa + bb + ı ab − a b‬‬


‫=‬ ‫=‬
‫‪a + ıb‬‬ ‫)‪(a + ıb) (a − ıb‬‬ ‫‪a +b‬‬
‫أي‬
‫‪ũŏ‬‬

‫‪a + ıb‬‬ ‫‪aa + bb‬‬ ‫‪ab − a b‬‬


‫=‬ ‫‪+ı‬‬ ‫)𝟒(‬
‫‪a + ıb‬‬ ‫‪a +b‬‬ ‫‪a +b‬‬

‫و ﻫﻮ ﻣﻦ اﻟﺸﻜﻞ 𝛽‪ 𝛼 + ı‬ﻣﻊ ‪.𝛼, 𝛽 ∈ ℝ‬‬

‫‪.(3 − ı) + (5 + 2ı) = (3 + 5) + ı (−1 + 2) = 8 + 1 × ı = 8 + ı‬‬ ‫•‬ ‫ﺃﻣﺜﻠﺔ ‪: 2‬‬

‫ﻛﺄي ﻋﺪد ﺣﻘﻴﻘﻲ‪.‬‬


‫ّ‬ ‫‪6‬ﻓﺮﺿﻨﺎ ﻣﺴﺒ ًﻘﺎ أﻧّﻪ ﺑﺈﻣﻜﺎﻧﻨﺎ اﻟﺘﻌﺎﻣﻞ ﻣﻊ‬

‫𝟰‬

‫‪http ://tinyurl.com/Malki1718‬‬ ‫‪0‬‬


‫‪ .I‬ﶍﻮﻋﺔ اﻷﻋﺪاد اﳌﺮﻛﺒﺔ‬
‫‪.‬‬
‫‪.(3 − ı) = 3 − 2 × 3ı + ı = 9 − 6ı − 1 = 8 − 6ı‬‬ ‫•‬

‫‪3 − ı (3 − ı) (1 − ı) 3 − 3ı − ı + ı‬‬ ‫‪2 − 4ı‬‬

‫‪Ŕž Ŧ Ľ‬‬
‫‪.‬‬ ‫=‬ ‫=‬ ‫=‬ ‫‪= 1 − 2ı‬‬ ‫•‬
‫)‪1 + ı (1 + ı) (1 − ı‬‬ ‫‪1 −ı‬‬ ‫‪2‬‬
‫‪I‬‬

‫‪œǃ‬‬
‫ﻛﺄي ﻋﺪد ﺣﻘﻴﻘﻲ‪ ،‬ﻧﻜﻮن ﻗﺪ أﻧﺸﺄﻧﺎ ﳎﻤﻮﻋﺔ اﻷﻋﺪاد‬‫ﺧﻼﺻﺔ ﺍﻟﻘﻮﻝ‪ :‬ﺑﺎﺧﱰاع ﻋﺪد ‪ ı‬ﺑﺤﻴﺚ ‪ ı = −1‬و ﺑﺎﺳﺘﻌﲈﻟﻪ ﰲ اﳊﺴﺎب ّ‬
‫ﻣﻦ اﻟﺸﻜﻞ ‪ a + ıb‬ﻣﻊ ‪ ،a, b ∈ ℝ‬و اﻟﻌﻤﻠﻴﺎت اﻷﺳﺎﺳﻴﺔ )اﳉﻤﻊ‪ ،‬اﻟﻄﺮح‪ ،‬اﻟﴬب و اﻟﻘﺴﻤﺔ( ﻋﻠﻴﻬﺎ ﺗُﻨﺘﺞ أﻋﺪا ًدا ﻣﻦ ﻧﻔﺲ‬

‫‪Ŀ‬‬
‫اﻟﺸﻜﻞ‪.‬‬
‫ْ‬
‫ﺗﻢ إﻧﺸﺎء ﳎﻤﻮﻋﺔ اﻷﻋﺪاد اﳌﺮﻛّﺒﺔ ‪...‬‬
‫ﻫﻜﺬا ّ‬

‫ﻛﻞ ﻋﺪد ﺣﻘﻴﻘﻲ ‪ a‬ﻫﻮ ﻣﻦ اﻟﺸﻜﻞ ‪ a + 0ı‬أي ّ‬


‫أن‬ ‫ﻓﺈن ّ‬
‫أن ‪ ، 0 × ı = 0‬و ﺑﺄﺧﺬ ‪ b = 0‬و ‪ a ∈ ℝ‬ﻛﻴﻔﻲ ّ‬ ‫ﰲ اﳌﺠﻤﻮﻋﺔ ﻧﻼﺣﻆ ّ‬
‫‪. ℝ⊂ℂ‬‬
‫ﻟﻜﻦ ﻗﺎﻋﺪة اﻹﺷﺎرات )ﺟﺪاء ﻋﺪدﻳﻦ ﻣﻦ ﻧﻔﺲ اﻹﺷﺎرة ﻫﻮ ﻋﺪد ﻣﻮﺟﺐ و ﺟﺪاء ﻋﺪدﻳﻦ ﳐﺘﻠﻔﲔ ﰲ اﻹﺷﺎرة ﻫﻮ ﻋﺪد ﺳﺎﻟﺐ(‬
‫أن ﻣﺮ ّﺑﻊ أي ﻋﺪد ﳚﺐ أن ﻳﻜﻮن ﻣﻮﺟﺒ ًﺎ ﻟﻜﻦ ﰲ اﳌﺠﻤﻮﻋﺔ‬‫ﺗﻔﻘﺪ ﺻﺤﺘﻬﺎ ﰲ ﳎﻤﻮﻋﺔ اﻷﻋﺪاد اﳌﺮﻛﺒﺔ ‪ :‬ﻓﻘﺎﻋﺪة اﻹﺷﺎرات ﺗﺴﺘﻠﺰم ّ‬
‫‪ّ ℂ‬‬
‫ﻛﻞ ﻣﻦ ‪ 1‬و ‪ −1‬ﻫﻮ ﻣﺮ ّﺑﻊ ) ‪ 1 = 1‬و ‪ (−1 = ı‬و ﺑﺎﻟﺘﺎﱄ ﻻ ﻳﻤﻜﻦ أن ﻳﻜﻮﻧﺎ ﻣﻮﺟﺒﲔ ﻣﻌ ًﺎ )إذا ﻛﺎن ‪ّ 1 ≥ 0‬‬
‫ﻓﺈن ‪ −1 ≤ 0‬أي‬
‫ﻓﺈن ‪ 1 ≤ 0‬أي ‪ 1 ≤ 0‬و ﻫﺬا ﺗﻨﺎﻗﺾ أﻳﻀ ًﺎ(‪.‬‬
‫‪ ı ≤ 0‬و ﻫﺬا ﺗﻨﺎﻗﺾ ؛ و إذا ﻛﺎن ‪ّ −1 ≥ 0‬‬

‫ﰲ ﳎﻤﻮﻋﺔ اﻷﻋﺪاد اﳌﺮﻛﺒﺔ‪ ،‬اﻟﺮﻣﻮز ≥ ‪ < ، ≤ ، > ،‬ﻟﻴﺲ ﳍﺎ ﻣﻌﻨﻰ!‬ ‫‪u‬‬

‫و ﺑﺎﻟﺘﺎﱄ ﻛﺘﺎﺑﺔ ﻣﻦ اﻟﺸﻜﻞ ‪ z ≥ 3 − ı‬ﰲ ورﻗﺔ إﻣﺘﺤﺎن ﺗُﻌﺪّ ﺟﺮﻳﻤﺔ ﻻ ﺗُﻐﺘﻔﺮ !‬

‫ُﺴﻮل ﻟﻨﺎ ﻧﻔﻮﺳﻨﺎ ﻛﺘﺎﺑﺔ ﺟﺬور أﻋﺪاد ﺳﺎﻟﺒﺔ و ﻫﺬا ﻻ ُﻳﻨﺼﺢ ﺑﻪ ﻷﻧّﻪ ﻳﺆدي ﺑﻨﺎ إﱃ ﺗﻨﺎﻗﻀﺎت‪ ،‬ﻣﺜ ً‬
‫ﻼ‪:‬‬ ‫ﻗﺪ ﺗ ﱢ‬ ‫ﻣﻼﺣﻈﺔ ‪⧏ : 1‬‬
‫= ‪1 = √1‬‬ ‫‪−1 (−1) = ı√−1 = i = −1‬‬

‫⧐‬
‫‪ņőƱŀ Ţ‬‬
‫أن اﳌﺴﺎواة ‪ a + ıb = a + ıb‬ﻻ‬ ‫أدق ﻟﻸﻋﺪاد اﳌﺮﻛﺒﺔ ‪ :‬ﻧﻼﺣﻆ ّ‬ ‫ﻟﻠﺘﺨ ّﻠﺺ ﻣﻦ ﻫﺬه اﻹﻟﺘﺒﺎﺳﺎت‪ ،‬ﺳﻨﺤﺎول إﻋﻄﺎء ﺗﻌﺮﻳﻒ ّ‬
‫‪a−a‬‬ ‫ﻓﺈن ‪ a − a = ı b − b‬ﻣﻨﻪ =‬ ‫ﺗﺘﺤ ّﻘﻖ إﻻّ إذا ﻛﺎن ‪ a = a‬و ‪ b = b‬ﻷﻧّﻪ إذا ﻛﺎن ‪ّ a + ıb = a + ıb‬‬
‫‪ − b − b‬و ﻫﺬا ﻻ ﻳﺘﺤ ّﻘﻖ ﰲ ﳎﻤﻮﻋﺔ اﻷﻋﺪاد اﳊﻘﻴﻘﻴﺔ إﻻّ إذا ﻛﺎن ‪ a = a‬و ‪.b = b‬‬
‫أن أي ﻋﺪد ﻣﺮﻛّﺐ ‪ a + ıb‬ﻫﻮ ﰲ‬ ‫أن اﻟﻌﺪد اﳌﺮﻛّﺐ ‪ a + ıb‬ﻳﻘﺎﺑﻠﻪ و ﺑﺼﻔﺔ وﺣﻴﺪة اﻟﺰوج اﳌﺮﺗّﺐ )‪ ،(a, b‬ﻳﻤﻜﻦ إﻋﺘﺒﺎر ّ‬
‫و ﺑﲈ ّ‬
‫أي ﻋﺪد ﺣﻘﻴﻘﻲ ‪ a‬ﻣﺎ ﻫﻮ‬ ‫أن اﻟﻮﺣﺪة اﻟﺘﺨﻴﻠﻴﺔ ‪ ı‬ﻳﻤ ّﺜﻞ اﻟﺰوج )‪ (0, 1‬و ّ‬
‫أن ّ‬ ‫اﳊﻘﻴﻘﺔ زوج ﻣﺮﺗّﺐ )‪ (a, b‬ﻣﻦ اﻷﻋﺪاد اﳊﻘﻴﻘﻴﺔ‪ ،‬و ّ‬
‫إﻻّ اﻟﺰوج )‪.(a, 0‬‬
‫ﻣﻦ ﻫﺬا ا ُﳌﻨﻄ َﻠﻖ‪ ،‬ﻳﻤﻜﻦ ﺗﻌﺮﻳﻒ ﳎﻤﻮﻋﺔ اﻷﻋﺪاد اﳌﺮﻛﺒﺔ ﺑﻜﻮﳖﺎ اﳌﺠﻤﻮﻋﺔ ‪ ℝ‬ﻟﻸزواج اﳊﻘﻴﻘﻴﺔ اﳌﺮﺗﺒﺔ )‪ (a, b‬و اﻟﺘﻲ ﱢ‬
‫ﻧﻌﺮف‬
‫‪ƕ‬‬

‫ﻓﻴﻬﺎ ﻋﻤﻠﻴﺘﻲ اﳉﻤﻊ و اﻟﴬب ﻛﲈ ﻳﲇ ‪:‬‬


‫‪(a, b) + a , b‬‬ ‫‪= a + a ,b + b‬‬
‫‪(a, b) × a , b‬‬ ‫‪= aa − bb , ab + a b‬‬
‫‪ũŏ‬‬

‫ﻳﻤﻜﻦ اﻟﺘﺤ ّﻘﻖ و ﺑﻜﻞ ﺳﻬﻮﻟﺔ ّ‬


‫أن ﻫﺎﺗﲔ اﻟﻌﻤﻠﻴﺘﲔ ﺗﺘﻤﺘﻌﺎن ﺑﺎﳋﻮاص اﻹﻋﺘﻴﺎدﻳﺔ ﻟﻠﺠﻤﻊ و اﻟﴬب ‪ :‬ﻓﻬﲈ ﺗﺒﺪﻳﻠﻴﺘﺎن‪ ،‬ﲡﻤﻴﻌﻴﺘﺎن‬
‫و اﻟﴬب ﺗﻮزﻳﻌﻲ ﻋﲆ اﳉﻤﻊ‪ ،‬و اﻟﺰوج )‪ (0, 0‬ﻫﻮ اﻟﻮﺣﻴﺪ اﻟﺬي ﻟﻴﺲ ﻟﻪ ﻣﻘﻠﻮب )ﻧﻈﲑ ﺑﺎﻟﻨﺴﺒﺔ ﻟﻠﴬب(‪.‬‬
‫ﺣﻞ اﳌﻌﺎدﻟﺔ ‪ (a, b) × (x, y) = a , b‬و ّ‬
‫ﺑﻜﻞ ﺑﺴﺎﻃﺔ ﻧﺠﺪ ‪:‬‬ ‫ﻹﳚﺎد ﺣﺎﺻﻞ ﻗﺴﻤﺔ زوﺟﲔ‪ ،‬ﻳﻜﻔﻲ ّ‬
‫‪a ,b‬‬ ‫‪aa + bb ab − a b‬‬
‫=‬ ‫‪,‬‬
‫)‪(a, b‬‬ ‫‪a +b‬‬ ‫‪a +b‬‬

‫𝟱‬

‫‪http ://tinyurl.com/Malki1718‬‬ ‫‪0‬‬


‫‪ .1.I‬ﲢﻠﻴﻞ اﳌﺴﺄةل اﳌﻄﺮوﺣﺔ‬
‫‪.‬‬
‫ﺑﺎﻹﺿﺎﻓﺔ إﱃ ذﻟﻚ‪ ،‬ﻣﻬﲈ ﻳﻜﻦ اﻟﻌﺪدان اﳊﻘﻴﻘﻴﺎن ‪ a‬و ‪ b‬ﻟﺪﻳﻨﺎ ‪:‬‬
‫)‪(a, b) = (a, 0) + (0, 1) × (b, 0‬‬

‫‪Ŕž Ŧ Ľ‬‬
‫أن اﻟﺰوﺟﲔ )‪ (a, 0‬و )‪ (b, 0‬ﻳﻤﺜﻼن اﻟﻌﺪدﻳﻦ اﳊﻘﻴﻘﻴﲔ ‪ a‬و ‪ ،b‬و ّ‬
‫أن اﻟﺰوج )‪ (0, 1‬ﻫﻮ اﻟﻌﺪد اﻟﺘﺨﻴﲇ ‪ ،ı‬ﻓﺎﳌﺴﺎواة اﻟﺴﺎﺑﻘﺔ‬ ‫و ﺑﲈ ّ‬

‫‪I‬‬
‫ﻛﻞ ﻋﺪد ﻣﺮﻛّﺐ )‪ُ (a, b‬ﻳﻜﺘﺐ ﻋﲆ اﻟﺸﻜﻞ ‪.a + ıb‬‬‫أن ّ‬
‫ﺗٌﺜﺒﺖ ّ‬

‫‪œǃ‬‬
‫ُﺴﻤﻰ اﻟﺸﻜﻞ اﳉﱪي ﻟﻠﻌﺪد اﳌﺮﻛﺐ ‪.z‬‬ ‫ﺗﻌﺮﻳﻒ ‪ : 2‬اﻟﻜﺘﺎﺑﺔ ‪ z = a + ıb‬ﺗ ّ‬
‫‪ a‬ﻫﻮ اﳉﺰء اﳊﻘﻴﻘﻲ ﻟﻠﻌﺪد ‪ z‬و ﻧﻜﺘﺐ )‪.a = Re(z‬‬

‫‪Ŀ‬‬
‫و ‪ b‬ﻫﻮ ﺟﺰؤه اﻟﺘﺨ ّﻴﲇ ‪ 7‬و ﻧﻜﺘﺐ )‪.b = Im(z‬‬
‫إذا ﻛﺎن اﳉﺰء اﻟﺘﺨ ّﻴﲇ ﻟﻠﻌﺪد ‪ z‬ﻣﻌﺪوﻣ ًﺎ‪ ،‬ﻧﻘﻮل ّ‬
‫أن ‪ z‬ﺣﻘﻴﻘﻲ‪.‬‬
‫ِ‬
‫أن ‪ z‬ﲣ ّﻴﲇ ﴏف‪.‬‬‫إذا ﻛﺎن اﳉﺰء اﳊﻘﻴﻘﻲ ﻟﻠﻌﺪد ‪ z‬ﻣﻌﺪوﻣ ًﺎ‪ ،‬ﻧﻘﻮل ّ‬

‫أن ﻫﺬه اﻟﻜﺘﺎﺑﺔ ﻫﻲ اﻟﺸﻜﻞ اﳉﱪي‬‫ﻣﻼﺣﻈﺔ ‪ ⧏ : 2‬ﻋﻨﺪﻣﺎ ﻧﺘﺤﺪّ ث ﻋﻦ »اﻟﻌﺪد اﳌﺮﻛﺐ«‪ z = a + ıb‬ﻓﺈﻧّﻨﺎ ﻧﻔﺮض ِﺿﻤﻨﻴ ًﺎ ّ‬
‫أن ‪ a‬و ‪ b‬ﻋﺪدان ﺣﻘﻴﻘﻴﺎن‪ .‬ﰲ ﻏﻴﺎب ﻫﺬه اﻟﻔﺮﺿﻴﺔ‪ ،‬أي إذا ﱂ ﻳﻜﻦ أﺣﺪ اﻟﻌﺪدﻳﻦ ‪ a‬و ‪) b‬أو ﻛﻼﳘﺎ( ﺣﻘﻴﻘ ًﻴﺎ‪ّ ،‬‬
‫ﻓﺈن‬ ‫ﻟﻠﻌﺪد ‪ z‬أي ّ‬
‫ﻼ اﻟﻌﻼﻗﺔ ‪ Re (x + ıy) = x :‬ﺧﺎﻃﺌﺔ ﻷﺟﻞ ‪ x = 1‬و ‪. y = ı‬‬ ‫اﻟﻜﺜﲑ ﻣﻦ اﻟﻨﺘﺎﺋﺞ اﳌﺘﻌﻠﻘﺔ ﺑﺎﻷﻋﺪاد اﳌﺮﻛﺒﺔ ﺗُﺼﺒﺢ ﺧﺎﻃﺌﺔ‪ ،‬ﻣﺜ ً‬
‫⧐‬
‫اﻟﺴﻬﻞ إﺛﺒﺎت ّ‬
‫أن ‪:‬‬ ‫ﻣﻦ ّ‬
‫) ‪Re (z + z ) = Re (z ) + Re (z‬‬ ‫و‬ ‫) ‪Im (z + z ) = Im (z ) + Im (z‬‬
‫و ﺑﺼﻔﺔ ﻋﺎﻣﺔ ‪:‬‬

‫ﻟﻴﻜﻦ ‪ n ≥ 2‬ﻋﺪد ًا ﻃﺒﻴﻌﻴﺎ و ﻟﺘﻜﻦ ‪ z ، ⋯ ، z ، z‬أﻋﺪا ًدا ﻣﺮﻛﺒﺔ‪ .‬ﻟﺪﻳﻨﺎ ‪:‬‬ ‫ﻣﺒﺮﻫﻨﺔ ‪: 1‬‬

‫) ‪Re (z + z + ⋯ + z ) = Re (z ) + Re (z ) + ⋯ + Re (z‬‬ ‫•‬ ‫‪1‬‬


‫) ‪Im (z + z + ⋯ + z ) = Im (z ) + Im (z ) + ⋯ + Im (z‬‬ ‫•‬ ‫‪2‬‬
‫‪ņőƱŀ Ţ‬‬
‫‪ •1‬ﺑﺎﻟﱰاﺟﻊ ﻋﲆ اﻟﻌﺪد اﻟﻄﺒﻴﻌﻲ ‪: n ≥ 2‬‬ ‫ﺍﻟﱪﻫﺎﻥ‪.‬‬
‫‪ : n = 2‬ﻟﻴﻜﻦ ‪ z = a + ıb‬و ‪ z = a + ıb‬ﻋﺪدﻳﻦ ﻣﺮﻛﺒﲔ‪.‬‬ ‫•‬
‫ﻟﺪﻳﻨﺎ ) ‪ z + z = (a + a ) + ı (b + b‬ﻣﻨﻪ ‪Re (z + z ) = b + b‬‬
‫أي ) ‪.Re (z + z ) = Re (z ) + Re (z‬‬
‫ﻧﻔﺮض اﳋﺎﺻﻴﺔ ﺻﺤﻴﺤﺔ ّ‬
‫ﻟﻜﻞ ﻋﺪد ﻃﺒﻴﻌﻲ أﺻﻐﺮ ﻣﻦ أو ﻳﺴﺎوي ‪ n‬و ﻟﻨﺜﺒﺖ ّأﳖﺎ ﺻﺤﻴﺤﺔ ﻷﺟﻞ ‪: n + 1‬‬ ‫•‬
‫‪ƕ‬‬

‫‪Re z + z + ⋯ + z + z‬‬ ‫‪+‬‬ ‫‪= Re (z + z + ⋯ + z ) + z +‬‬


‫‪= Re (z + z + ⋯ + z ) + Re z‬‬ ‫‪+‬‬

‫ﺣﺴﺐ ﻓﺮﺿﻴﺔ اﻟﱰاﺟﻊ‪ ،‬اﳋﺎﺻﻴﺔ ﺻﺤﻴﺤﺔ ﺣﺘﻰ اﻟﺮﺗﺒﺔ ‪ n‬ﻣﻨﻪ‬


‫‪ũŏ‬‬

‫) ‪Re (z + z + ⋯ + z ) = Re (z ) + Re (z ) + ⋯ + Re (z‬‬
‫أي‬
‫‪Re z + z + ⋯ + z + z‬‬ ‫‪+‬‬ ‫‪= Re (z ) + Re (z ) + ⋯ + Re (z ) + Re z‬‬ ‫‪+‬‬
‫‪= Re (z ) + Re (z ) + ⋯ + Re (z ) + Re z +‬‬

‫ﺣﺴﺐ ﻣﺒﺪأ اﻟﱪﻫﺎن ﺑﺎﻟﱰاﺟﻊ‪ ،‬ﻓﺎﳋﺎﺻﻴﺔ ﺻﺤﻴﺤﺔ ﻟﻜﻞ ﻋﺪد ﻃﺒﻴﻌﻲ ‪.n ≥ 2‬‬

‫𝟲‬

‫‪http ://tinyurl.com/Malki1718‬‬ ‫‪0‬‬


‫‪ .I‬ﶍﻮﻋﺔ اﻷﻋﺪاد اﳌﺮﻛﺒﺔ‬
‫‪.‬‬
‫‪ •2‬ﺑﺎﻟﱰاﺟﻊ ﻋﲆ اﻟﻌﺪد اﻟﻄﺒﻴﻌﻲ ‪:n‬‬

‫‪Ŕž Ŧ Ľ‬‬
‫‪ z‬و ‪ z = a + ıb‬ﻋﺪدﻳﻦ ﻣﺮﻛﺒﲔ‪.‬‬ ‫‪ : n = 2‬ﻟﻴﻜﻦ ‪= a + ıb‬‬ ‫•‬
‫‪ z + z = (a‬ﻣﻨﻪ ‪Im (z + z ) = b + b‬‬ ‫ﻟﺪﻳﻨﺎ ) ‪+ a ) + ı (b + b‬‬
‫‪I‬‬

‫‪.Im (z + z‬‬ ‫أي ) ‪) = Im (z ) + Im (z‬‬

‫‪œǃ‬‬
‫ﻧﻔﺮض اﳋﺎﺻﻴﺔ ﺻﺤﻴﺤﺔ ّ‬
‫ﻟﻜﻞ ﻋﺪد ﻃﺒﻴﻌﻲ أﺻﻐﺮ ﻣﻦ أو ﻳﺴﺎوي ‪ n‬و ﻟﻨﺜﺒﺖ ّأﳖﺎ ﺻﺤﻴﺤﺔ ﻷﺟﻞ ‪: n + 1‬‬ ‫•‬

‫‪Im z + z + ⋯ + z + z‬‬ ‫‪+‬‬ ‫‪= Im (z + z + ⋯ + z ) + z +‬‬

‫‪Ŀ‬‬
‫‪= Im (z + z + ⋯ + z ) + Im z‬‬ ‫‪+‬‬

‫ﺣﺴﺐ ﻓﺮﺿﻴﺔ اﻟﱰاﺟﻊ‪ ،‬اﳋﺎﺻﻴﺔ ﺻﺤﻴﺤﺔ ﺣﺘﻰ اﻟﺮﺗﺒﺔ ‪ n‬ﻣﻨﻪ‬


‫) ‪Im (z + z + ⋯ + z ) = Im (z ) + Im (z ) + ⋯ + Im (z‬‬
‫أي‬
‫‪Im z + z + ⋯ + z + z‬‬ ‫‪+‬‬ ‫‪= Im (z ) + Im (z ) + ⋯ + Im (z ) + Im z‬‬ ‫‪+‬‬
‫‪= Im (z ) + Im (z ) + ⋯ + Im (z ) + Im z +‬‬

‫ﺣﺴﺐ ﻣﺒﺪأ اﻟﱪﻫﺎن ﺑﺎﻟﱰاﺟﻊ‪ ،‬ﻓﺎﳋﺎﺻﻴﺔ ﺻﺤﻴﺤﺔ ﻟﻜﻞ ﻋﺪد ﻃﺒﻴﻌﻲ ‪.n ≥ 2‬‬
‫ﻣﻼﺣﻈﺔ ‪ ⧏ : 3‬ﻳﻤﻜﻦ إﺛﺒﺎت ﻫﺬه اﻟﻨﺘﻴﺠﺔ ﺑﻄﺮﻳﻘﺔ أﺧﺮى ﺗﻌﺘﻤﺪ ﻋﲆ اﻟﻨﺘﻴﺠﺔ اﻟﺴﺎﺑﻘﺔ ‪ :‬إذا ﻛﺎن ‪ّ z = x + ıy‬‬
‫ﻓﺈن‬
‫‪ ız = ı (x + ıy) = −y + ıx‬ﻣﻨﻪ )‪ Re (z) = Im (ız‬و )‪ Im (z) = − Re (ız‬ﻣﻨﻪ ‪:‬‬
‫)) ‪Im (z + z + ⋯ + z ) = − Re (ı (z + z + ⋯ + z‬‬
‫) ‪= − Re (ız + ız + ⋯ + ız‬‬
‫) ‪= − Re (ız ) − Re (ız ) − ⋯ − Re (ız‬‬ ‫)ﺣﺴﺐ اﻟﻨﺘﻴﺠﺔ اﻟﺴﺎﺑﻘﺔ(‬
‫) ‪= Im (z ) + Im (z ) + ⋯ + Im (z‬‬

‫⧐‬
‫و ﻫﻮ اﳌﻄﻠﻮب‬
‫■‬
‫‪ņőƱŀ Ţ‬‬
‫‪ z‬ﻋﺪدﻳﻦ ﻣﺮﻛﺒﲔ ﻓﺈﻧﻪ ﰲ اﳊﺎﻟﺔ اﻟﻌﺎﻣﺔ ) ‪ Re (z ⋅ z ) ≠ Re (z )⋅Re (z‬و ≠ ) ‪Im (z ⋅ z‬‬ ‫ﻣﻼﺣﻈﺔ ‪ ⧏ : 4‬إذا ﻛﺎن ‪ z‬و‬
‫) ‪ Im (z ) ⋅ Im (z‬و ﻳﻤﻜﻦ اﻟﺘﺤﻘﻖ ﻣﻦ ذﻟﻚ ﺑﺄﺧﺬ ‪ z = ı‬و ‪ . z = ı‬ﻟﻜﻦ‪ ،‬إذا ﻛﺎن ‪ 𝜆 ∈ ℝ‬ﻋﺪد ًا ﺣﻘﻴﻘ ًﻴﺎ و ‪ z ∈ ℂ‬ﻋﺪد ًا‬
‫ﻓﺈن )‪ Re (𝜆z) = 𝜆 Re (z‬و )‪ Im (𝜆z) = 𝜆 Im (z‬و ﻫﻮ ﺣﺎﻟﺔ ﺧﺎﺻﺔ ﻣﻦ ﺗﻌﺮﻳﻒ ﻋﻤﻠﻴﺔ اﻟﴬب ﰲ ‪.ℂ‬‬ ‫ﻣﺮﻛﺒﺎ ّ‬
‫⧐‬

‫ﻣﺒﺮﻫﻨﺔ ‪ : 2‬ﻳﺘﺴﺎوى ﻋﺪدان ﻣﺮﻛﺒﺎن إذا و ﻓﻘﻂ إذا ﺗﺴﺎوى ﺟﺰءاﳘﺎ اﳊﻘﻴﻘﻴﺎن و ﺗﺴﺎوى ﺟﺰءاﳘﺎ اﻟﺘﺨ ّﻴﻠﻴﺎن‪ ،‬ﺑﻤﻌﻨﻰ ‪:‬‬
‫إذا ﻛﺎن ‪ z = a + ıb‬و ‪ z = a + ıb‬ﻋﺪدﻳﻦ ﻣﺮﻛﺒﲔ‪ّ ،‬‬
‫ﻓﺈن ‪:‬‬
‫‪b = b‬و ‪z = z ⟺ a = a‬‬
‫‪ƕ‬‬

‫ﺍﻟﱪﻫﺎﻥ‪ .‬ﻟﻴﻜﻦ ‪ z = a + ıb‬و ‪ z = a + ıb‬ﻋﺪدﻳﻦ ﻣﺮﻛﺒﲔ ﺑﺤﻴﺚ ‪ .z = z‬ﻧﻔﺮض ّ‬


‫أن ‪ .b ≠ b‬ﻟﺪﻳﻨﺎ ‪:‬‬

‫‪z =z‬‬ ‫‪⟺ a + ıb = a + ıb‬‬ ‫) ‪⟺ a − a = ı (b − b‬‬


‫‪ũŏ‬‬

‫‪a −a‬‬
‫= ‪ ı‬أي ‪ ı ∈ ℝ‬و ﻫﺬا ﺗﻨﺎﻗﺾ!‬ ‫أن ‪ّ b − b ≠ 0‬‬
‫ﻓﺈن‬ ‫و ﺑﲈ ّ‬
‫‪b −b‬‬
‫ﻫﺬا ﻳﻌﻨﻲ ّ‬
‫أن ‪ b = b‬ﺑﺎﻟﴬورة‪ .‬ﻣﻨﻪ ‪.a = a‬‬
‫) ‪a − a = ı (b − b ) ⟹ (a − a ) = − (b − b‬‬ ‫)ﻳﻤﻜﻦ أﻳﻀ ًﺎ ﻣﻼﺣﻈﺔ ّ‬
‫أن ‪:‬‬
‫■‬ ‫و ﻫﺬا ﻻ ﻳﺘﺤﻘﻖ ﰲ ﳎﻤﻮﻋﺔ اﻷﻋﺪاد اﳊﻘﻴﻘﻴﺔ إﻻ إذا ﻛﺎن ‪ a = a‬و ‪.(b = b‬‬

‫𝟳‬

‫‪http ://tinyurl.com/Malki1718‬‬ ‫‪0‬‬


‫‪ .2.I‬ﻣﺮاﻓﻖ ﻋﺪد ﻣﺮﻛﺐ‬
‫‪.‬‬
‫ﻣﺮﺍﻓﻖ ﻋﺪﺩ ﻣﺮﻛﺐ‬ ‫‪2. I‬‬

‫‪Ŕž Ŧ Ľ‬‬
‫ﺗﻌﺮﻳﻒ‬ ‫‪1.2.I‬‬

‫‪I‬‬
‫‪œǃ‬‬
‫‪1‬‬
‫)ﻣﻘﻠﻮب اﻟﻌﺪد اﳌﺮﻛﺐ ‪ ، (a + ıb‬رأﻳﻨﺎ اﻟﺪور اﻟﺬي ﻟﻌﺒﻪ اﻟﻌﺪد اﳌﺮﻛﺐ ‪ a − ıb‬و ﻫﻮ‬ ‫ﰲ اﻟﻔﻘﺮة اﻟﺴﺎﺑﻘﺔ‪ ،‬و ﻹﳚﺎد‬
‫‪a + ıb‬‬
‫ﻋﺪد ﻳﺘﻤﺘﻊ ﺑﻜﺜﲑ ﻣﻦ اﳋﻮاص اﳍﺎﻣﺔ ﺟﱪﻳﺔ ﻛﺎﻧﺖ أو ﻫﻨﺪﺳﻴﺔ‪.‬‬

‫‪Ŀ‬‬
‫ﻧﺴﻤﻲ ﻣﺮاﻓﻖ اﻟﻌﺪد اﳌﺮﻛﺐ ‪ z‬ﺣﻴﺚ ‪ z = a + ıb‬اﻟﻌﺪد اﳌﺮﻛﺐ ‪ z‬ﺣﻴﺚ ‪ z = a − ıb‬و ﻧﻘﺮأ »‪.«z barre‬‬ ‫ﺗﻌﺮﻳﻒ ‪: 3‬‬

‫إﺳﺘﻨﺎ ًدا إﱃ اﻟﺘﻌﺮﻳﻒ اﻟﺴﺎﺑﻖ‪ ،‬ﻟﺪﻳﻨﺎ اﻟﻨﺘﺎﺋﺞ اﻟﺘﺎﻟﻴﺔ ‪:‬‬

‫‪ z = z •1‬و ﻧﻘﻮل أن اﻟﻌﺪدﻳﻦ ‪ z‬و ‪ z‬ﻣﱰاﻓﻘﺎن‪.‬‬


‫‪z+z‬‬
‫= )‪.Re (z‬‬ ‫‪ z + z = 2a •2‬أي‬
‫‪2‬‬
‫‪z−z‬‬
‫= )‪.Im (z‬‬ ‫‪ z − z = 2ıb •3‬أي‬
‫‪2ı‬‬
‫‪.z ⋅ z = a + b‬‬ ‫•‬ ‫‪4‬‬
‫‪ z •5‬ﺣﻘﻴﻘﻲ ⟺ ‪.z = z‬‬
‫‪ z •6‬ﲣﻴﲇ ﴏف ⟺ ‪.z + z = 0‬‬

‫ﺧﻮﺍﺹ‬ ‫‪2.2.I‬‬
‫‪ņőƱŀ Ţ‬‬
‫‪ z‬و ‪ z‬ﻋﺪدان ﻣﺮﻛﺒﺎن ﺣﻴﺚ ‪ z = a + ıb‬و ‪ .z = a + ıb‬ﻟﺪﻳﻨﺎ ‪:‬‬

‫‪.z − z = z − z‬‬ ‫و‬ ‫‪z+z =z+z‬‬ ‫•‬ ‫‪1‬‬ ‫ﻣﺒﺮﻫﻨﺔ ‪: 3‬‬

‫‪.z × z = z × z •2‬‬
‫‪1‬‬ ‫‪1‬‬
‫)ﻣﻊ ‪.(z ≠ 0‬‬ ‫=‬ ‫•‬ ‫‪3‬‬
‫‪z‬‬ ‫‪z‬‬
‫‪ƕ‬‬

‫‪z‬‬ ‫‪z‬‬
‫)ﻣﻊ ‪.(z ≠ 0‬‬ ‫=‬ ‫•‬ ‫‪4‬‬
‫‪z‬‬ ‫‪z‬‬
‫)‪ (z ) = (z‬ﻟﻜﻞ ﻋﺪد ﺻﺤﻴﺢ ‪.n ∈ ℤ‬‬ ‫•‬ ‫‪5‬‬
‫‪ũŏ‬‬

‫‪ •1‬ﻟﺪﻳﻨﺎ ‪:‬‬ ‫ﺍﻟﱪﻫﺎﻥ‪.‬‬


‫‪z + z = (a + ıb) + a + ıb‬‬ ‫‪= (a + a ) + ı b + b‬‬
‫‪= a+a −ı b+b‬‬ ‫‪= (a − ıb) + a − ıb‬‬
‫‪=z+z‬‬

‫𝟴‬

‫‪http ://tinyurl.com/Malki1718‬‬ ‫‪0‬‬


‫‪ .I‬ﶍﻮﻋﺔ اﻷﻋﺪاد اﳌﺮﻛﺒﺔ‬
‫‪.‬‬
‫و‬

‫‪Ŕž Ŧ Ľ‬‬
‫‪z − z = (a + ıb) − a + ıb‬‬ ‫‪= (a − a ) + ı b − b‬‬
‫‪= a−a −ı b−b‬‬ ‫‪= (a − ıb) − a − ıb‬‬
‫‪I‬‬

‫‪œǃ‬‬
‫‪=z−z‬‬

‫‪ •2‬ﻟﺪﻳﻨﺎ ﻣﻦ ﺟﻬﺔ ‪:‬‬

‫‪Ŀ‬‬
‫‪z ⋅ z = (a + ıb) a + ıb‬‬
‫‪= aa − bb + ı ab + a b‬‬
‫‪= aa − bb − ı ab + a b‬‬

‫و ﻣﻦ ﺟﻬﺔ أﺧﺮى ‪:‬‬


‫‪z ⋅ z = (a − ıb) a − ıb‬‬
‫‪= aa − bb − ı ab + a b‬‬

‫‪1‬‬ ‫‪1‬‬ ‫‪1‬‬ ‫‪1‬‬


‫‪.‬‬ ‫=‬ ‫× ‪ z‬و ﻣﻨﻪ‬ ‫× ‪ z‬أي ‪= 1‬‬ ‫ﻟﺪﻳﻨﺎ ‪= 1 :‬‬ ‫‪3‬‬
‫•‬
‫‪z‬‬ ‫‪z‬‬ ‫‪z‬‬ ‫‪z‬‬

‫‪z‬‬ ‫‪1‬‬ ‫‪1‬‬ ‫‪1‬‬ ‫‪z‬‬


‫‪.‬‬ ‫=‬ ‫×‪z‬‬ ‫×‪=z‬‬ ‫×‪=z‬‬ ‫=‬ ‫‪ •4‬ﻟﺪﻳﻨﺎ ‪:‬‬
‫‪z‬‬ ‫‪z‬‬ ‫‪z‬‬ ‫‪z‬‬ ‫‪z‬‬
‫‪ •5‬ﰲ اﻟﺒﺪاﻳﺔ ﻧﺜﺒﺖ ﺻﺤﺘﻬﺎ ﻷﺟﻞ ‪ n ∈ ℕ‬ﺑﺎﻟﱰاﺟﻊ ‪:‬‬

‫إﺻﻄﻼﺣﺎ ‪ 0 = 1‬و ﻧﺬﻛّﺮ ّ‬


‫أن ‪ u = 1 :‬إذا ﻛﺎن ‪.(u ≠ 0‬‬ ‫ً‬ ‫اﳋﺎﺻﻴﺔ ﺻﺤﻴﺤﺔ ﻷﺟﻞ ‪ n = 0‬و ‪) n = 1‬ﻧﻀﻊ‬ ‫•‬

‫)‪ z = z × z = z × z = (z‬ﻓﺎﳋﺎﺻﻴﺔ ﺻﺤﻴﺤﺔ ﻷﺟﻞ ‪.n = 2‬‬ ‫•‬

‫ﻧﻔﺮض اﳋﺎﺻﻴﺔ ﺻﺤﻴﺤﺔ ﻟﻜﻞ اﻷﻋﺪاد اﻟﻄﺒﻴﻌﻴﺔ اﻷﺻﻐﺮ ﻣﻦ ‪ .n‬ﻟﺪﻳﻨﺎ )ﺑﺎﺳﺘﺨﺪام ﺧﺎﺻﻴﺔ ﻣﺮاﻓﻖ ﺟﺪاء(‪:‬‬
‫‪ņőƱŀ Ţ‬‬
‫•‬

‫‪(z ) = z‬‬ ‫‪−‬‬ ‫‪×z= z‬‬ ‫‪−‬‬ ‫‪×z‬‬


‫‪−‬‬ ‫‪−‬‬
‫‪ ، z‬ﻣﻨﻪ‬ ‫)‪= (z‬‬ ‫ﻟﻜﻦ ﺣﺴﺐ ﻓﺮﺿﻴﺔ اﻟﱰاﺟﻊ‬
‫‪−‬‬ ‫‪−‬‬
‫‪(z ) = z‬‬ ‫)‪× z = (z‬‬ ‫)‪× z = (z‬‬

‫و اﻵن‪ ،‬ﻧﺜﺒﺖ ﺻﺤﺘﻬﺎ ﻷﺟﻞ اﻷﻋﺪاد اﻟﺼﺤﻴﺤﺔ اﻟﺴﺎﻟﺒﺔ ‪:‬‬


‫ﻟﻴﻜﻦ ‪ .n ∈ ℤ∗−‬ﻧﻀﻊ ‪ .m = −n‬ﻟﺪﻳﻨﺎ ‪ m > 0‬و ‪:‬‬
‫‪1‬‬ ‫‪1‬‬ ‫‪1‬‬ ‫‪1‬‬ ‫‪−‬‬
‫= ) ‪(z‬‬ ‫=‬ ‫=‬ ‫=‬ ‫)‪= (z‬‬ ‫)‪= (z‬‬
‫‪ƕ‬‬

‫‪z−‬‬ ‫‪z‬‬ ‫) ‪(z‬‬ ‫)‪(z‬‬


‫■‬

‫‪.(3 − 2ı) = 3 + 2ı‬‬ ‫•‬ ‫ﺃﻣﺜﻠﺔ ‪: 3‬‬


‫‪ũŏ‬‬

‫‪.(3ı) = −3ı‬‬ ‫•‬

‫‪1‬‬ ‫‪1‬‬ ‫‪1‬‬ ‫‪1+ı‬‬ ‫‪1 1‬‬


‫‪.‬‬ ‫=‬ ‫=‬ ‫=‬ ‫‪= + ı‬‬ ‫•‬
‫‪1+ı‬‬ ‫‪1+ı 1−ı 1 −ı‬‬ ‫‪2 2‬‬

‫‪1 + ı√3‬‬ ‫‪1 + ı√3‬‬ ‫‪1 − ı√3‬‬ ‫)‪1 − ı√3 (2 − 3ı‬‬ ‫‪2 − 3√3‬‬ ‫‪3 + 2√3‬‬
‫‪.‬‬ ‫=‬ ‫=‬ ‫=‬ ‫=⋯=‬ ‫‪−ı‬‬ ‫•‬
‫‪2 − 3ı‬‬ ‫)‪(2 − 3ı‬‬ ‫‪2 + 3ı‬‬ ‫)‪(2 + 3ı) (2 − 3ı‬‬ ‫‪13‬‬ ‫‪13‬‬

‫𝟵‬

‫‪http ://tinyurl.com/Malki1718‬‬ ‫‪0‬‬


‫‪ .2.I‬ﻣﺮاﻓﻖ ﻋﺪد ﻣﺮﻛﺐ‬
‫‪.‬‬
‫أن اﻟﻌﺪد اﳌﺮﻛﺐ ﳚﺐ أن ﻳﻜﻮن ﻋﲆ اﻟﺸﻜﻞ ‪ .z = a + ıb‬ﰲ‬ ‫ﻣﺎ ﻳﻤﻜﻦ ﻣﻼﺣﻈﺘﻪ ﻓﻴﲈ ﳜﺺ اﻟﺘﻌﺮﻳﻒ اﻟﺴﺎﺑﻖ ﻟﻠﻤﺮاﻓﻖ‪ ،‬ﻫﻮ ّ‬
‫ﺣﺎﻟﺔ اﻟﻌﺪد اﳌﺮﻛﺐ 𝜆 اﻟﺬي أﺧﺬﻧﺎه ﻛﻤﺜﺎل ﻟﻌﺒﺎرة ﻣﻌﻘﺪة ﰲ اﻟﺼﻔﺤﺔ ‪ ،4‬ﻳﻤﻜﻦ أن ﻧﻈ ّﻦ ّ‬
‫أن أﻣﺎﻣﻨﺎ إﻣﻜﺎﻧﻴﺘﲔ ﻓﻘﻂ ﻹﳚﺎد ﻣﺮاﻓﻘﻪ‪:‬‬

‫‪Ŕž Ŧ Ľ‬‬
‫إ ّﻣﺎ أن ﻧﻜﺘﻔﻲ ﺑﻜﺘﺎﺑﺔ‬

‫‪I‬‬
‫‪œǃ‬‬
‫‪−‬‬
‫‪1 − 2ı‬‬ ‫‪1−ı‬‬
‫‪⎛ 2 − 5ı‬‬ ‫‪−‬‬
‫⎜‬ ‫⎞ ‪3 + 2ı‬‬
‫⎟‬
‫⎜=𝜆‬ ‫⎟‬
‫‪⎜ 2 − ı5‬‬ ‫‪−ı‬‬ ‫⎟‬
‫‪+‬‬

‫‪Ŀ‬‬
‫⎝‬ ‫‪2ı‬‬ ‫‪3 + 2ı‬‬ ‫⎠‬
‫أو أن ﻧﺒﺤﺚ ﻋﻦ ﺟﺰﺋﻪ اﳊﻘﻴﻘﻲ و ﺟﺰﺋﻪ اﻟﺘﺨﻴﲇ‪ ،‬ﺣﻴﺚ ﻧﺠﺪ ﺑﺎﺳﺘﺨﺪام ﻧﻈﺎم ﺟﱪ اﳊﺎﺳﻮب ‪: 9 wxMaxima 8‬‬

‫)‪(%i1‬‬ ‫)))‪((((1-2*%i)/(2-5*%i))^(-2)-((1-%i)/(3+2*%i‬‬
‫;‪/(((2-%i*5)/(2*%i^8))^2+((-%i)/(3+2*%i))^3))^4‬‬
‫‬ ‫‪4‬‬
‫‪(2−5 i)2‬‬ ‫‪1−i‬‬
‫‪2 − 2 i+3‬‬
‫)‪(1−2 i‬‬
‫ )‪(%o1‬‬ ‫‪4‬‬
‫‪i)2‬‬
‫‪i‬‬
‫‪(2 i+3)3‬‬
‫‪+ (2−5‬‬‫‪4‬‬

‫)‪(%i2‬‬ ‫;)‪rectform(%‬‬
‫‪848987277819393118952749056 i 2574612635075629849043048192‬‬
‫)‪(%o2‬‬ ‫‪−‬‬
‫‪7185835807781959059187890625 7185835807781959059187890625‬‬

‫ﺷﻜﻞ ‪ :1.I‬ﺣﺴﺎب 𝜆 ﺑﺎﺳﺘﻌﲈل ‪.wxMaxima‬‬

‫‪2574612635075629849043048192‬‬ ‫‪848987277819393118952749056‬‬
‫‪𝜆=−‬‬ ‫‪+‬‬ ‫‪ı‬‬
‫‪7185835807781959059187890625 7185835807781959059187890625‬‬
‫ﻣﻨﻪ‬
‫‪2574612635075629849043048192‬‬ ‫‪848987277819393118952749056‬‬
‫‪𝜆=−‬‬ ‫‪−‬‬ ‫‪ı‬‬
‫‪7185835807781959059187890625 7185835807781959059187890625‬‬
‫ﻟﻜﻦ ﻫﻨﺎك إﻣﻜﺎﻧﻴﺔ أﺧﺮى ‪...‬‬
‫‪ņőƱŀ Ţ‬‬
‫ﻧﻈﺮﻳﺔ ‪ : 1‬ﻹﳚﺎد ﻣﺮاﻓﻖ ﻋﺪد ﻣﺮﻛﺐ ﻣﻜﺘﻮب ﻋﲆ ﺻﻴﻐﺔ ﺗﺴﻠﺴﻞ ﻋﻤﻠﻴﺎت اﳉﻤﻊ‪ ،‬اﻟﻄﺮح‪ ،‬اﻟﴬب و اﻟﻘﺴﻤﺔ‪ ،‬ﻳﻜﻔﻲ‬
‫ﻣﺮة‪.‬‬
‫اﻹﺣﺘﻔﺎظ ﺑﻨﻔﺲ اﻟﺼﻴﻐﺔ ﻣﻊ ﺗﻌﻮﻳﺾ اﻟﻌﺪد ‪ ı‬ﺑﺎﻟﻌﺪد ‪ −ı‬ﰲ ﻛﻞ ّ‬

‫ﺍﻟﱪﻫﺎﻥ‪ .‬ﺑﲈ أﻧﻨﺎ ﻧُﺠﺮي اﻟﻌﻤﻠﻴﺎت ﰲ ﺗﺴﻠﺴﻞ‪ ،‬ﻳﻜﻔﻲ أن ﻧﺜﺒﺖ أﻧﻨﺎ ﻧﺤﺼﻞ ﻋﲆ ﻣﺮاﻓﻖ ﳎﻤﻮع‪ ،‬ﻓﺮق‪ ،‬ﺣﺎﺻﻞ ﴐب و ﺣﺎﺻﻞ‬
‫ﻗﺴﻤﺔ ﻋﺪدﻳﻦ ﻣﺮﻛﺒﲔ ﺑﺎﻟﺘﻌﻮﻳﺾ ﻋﻦ ‪ ı‬ﺑﺎﻟﻌﺪد ‪.−ı‬‬
‫■‬ ‫ﻟﻜﻦ ﺑﺎﻟﺘﻤ ّﻌﻦ ﰲ إﺛﺒﺎت اﳌﱪﻫﻨﺔ ‪ ،3‬ﻧﺠﺪ ّ‬
‫أن ﻫﺬا ﻣﺎ ﻧﻘﻮم ﺑﻪ ﺑﺎﻟﺘﺤﺪﻳﺪ!‬
‫‪ƕ‬‬

‫أﺻﺒﺢ اﻵن ﺑﺈﻣﻜﺎﻧﻨﺎ إﻋﻄﺎء ﻋﺒﺎرة أﺧﺮى ﳌﺮاﻓﻖ اﻟﻌﺪد 𝜆 ‪:‬‬


‫‪−‬‬
‫⎛‬ ‫‪1 + 2ı‬‬ ‫‪1+ı‬‬ ‫⎞‬
‫⎜‬ ‫‪−‬‬ ‫⎟‬
‫‪ũŏ‬‬

‫⎜‬ ‫‪2 + 5ı‬‬ ‫‪3 − 2ı‬‬ ‫⎟‬


‫⎜=𝜆‬ ‫⎟‬
‫⎜‬ ‫⎟‬
‫⎜‬ ‫‪2 + ı5‬‬ ‫‪ı‬‬ ‫⎟‬
‫‪+‬‬
‫⎝‬ ‫)‪2(−ı‬‬ ‫‪3 − 2ı‬‬ ‫⎠‬

‫‪ CAS8‬أو ‪) Computer Algebra System‬ﺑﺎﻹﻧﺠﻠﻴﺰﻳﺔ( ؛ ‪) Logiciel de Calcul Formel‬ﺑﺎﻟﻔﺮﻧﺴﻴﺔ(‪.‬‬


‫‪http://andrejv.github.com/wxmaxima/9‬‬

‫𝟬𝟭‬

‫‪http ://tinyurl.com/Malki1718‬‬ ‫‪0‬‬


‫‪ .I‬ﶍﻮﻋﺔ اﻷﻋﺪاد اﳌﺮﻛﺒﺔ‬
‫‪.‬‬
‫ﺍﻟﺘﻤﺜﻴﻞ ﺍﳍﻨﺪﺳﻲ ﻟﻌﺪﺩ ﻣﺮﻛﺐ‬ ‫‪3. I‬‬

‫‪Ŕž Ŧ Ľ‬‬
‫اﳌﺴﺘﻮي ﻣﻨﺴﻮب إﱃ ﻣﻌﻠﻢ ﻣﺘﻌﺎﻣﺪ و ﻣﺘﺠﺎﻧﺲ ⃗ ‪. O, i,⃗ j‬‬
‫‪I‬‬

‫‪œǃ‬‬
‫ﺍﻟﺘﻤﺜﻴﻞ ﺍﻟﻨﻘﻄﻲ‬ ‫‪1 . 3. I‬‬

‫ﻣﺪرج ﺑﺤﻴﺚ ّ‬
‫ﻛﻞ ﻧﻘﻄﺔ ﻣﻦ ﻫﺬا اﳌﺴﺘﻘﻴﻢ ﺗﻘﺎﺑﻞ ﻋﺪ ًدا ﺣﻘﻴﻘ ًﻴﺎ‪.‬‬ ‫ﻋﻨﺪ دراﺳﺔ ﳎﻤﻮﻋﺔ اﻷﻋﺪاد اﳊﻘﻴﻘﻴﺔ‪ ،‬ﻗﻤﻨﺎ ﺑﺘﻤﺜﻴﻠﻬﺎ ﺑﻤﺴﺘﻘﻴﻢ ّ‬

‫‪Ŀ‬‬
‫ﻛﻞ زوج ﻣﻦ اﻷﻋﺪاد اﳊﻘﻴﻘﻴﺔ ﻳﻤ ّﺜﻞ إﺣﺪاﺛ َﻴ ْﻲ ﻧﻘﻄﺔ ﰲ‬ ‫ﻛﻞ ﻋﺪد ﻣﺮﻛﺐ ﻣﺎ ﻫﻮ إﻻّ زوج ﻣﻦ اﻷﻋﺪاد اﳊﻘﻴﻘﻴﺔ‪ ،‬و ّ‬
‫أن ّ‬ ‫أن ّ‬‫و ﺑﲈ ّ‬
‫اﳌﺴﺘﻮي ﻓﺒﺈﻣﻜﺎﻧﻨﺎ أن ﻧﺮﻓﻖ ﺑﻜﻞ ﻋﺪد ﻣﺮﻛﺐ ‪ z‬ﺣﻴﺚ ‪ z = a + ıb‬اﻟﻨﻘﻄﺔ ‪ M‬ذات اﻹﺣﺪاﺛﻴﲔ )‪.(a, b‬‬
‫ﻧﻘﻮل أﻧﻨﺎ ﻣﺜﻠﻨﺎ اﻟﻌﺪد اﳌﺮﻛﺐ ‪ z‬ﺑﺎﻟﻨﻘﻄﺔ ‪.M‬‬
‫أدق‪ ،‬ﰲ اﳌﺴﺘﻮي اﳌﻨﺴﻮب إﱃ ﻣﻌﻠﻢ ﻣﺘﻌﺎﻣﺪ و ﻣﺘﺠﺎﻧﺲ ⃗ ‪ ، O, i,⃗ j‬اﻟﻨﻘﻄﺔ ‪ M‬ذات اﻹﺣﺪاﺛﻴﲔ )‪ (a, b‬ﲤ ّﺜﻞ اﻟﻌﺪد‬ ‫ﺑﺘﻌﺒﲑ ّ‬
‫اﳌﺮﻛﺐ ‪) z = a + ıb‬ﺷﻜﻞ ‪.(2.I‬‬

‫‪z = a + ıb‬‬
‫‪b‬‬
‫‪M‬‬
‫‪−‬‬
‫→‬
‫‪j‬‬

‫‪O‬‬ ‫→‬
‫‪−‬‬ ‫‪a‬‬
‫‪i‬‬

‫ﺷﻜﻞ ‪ :2.I‬اﳌﺴﺘﻮي اﳌﺮﻛﺐ‪.‬‬

‫ﺗﻌﺮﻳﻒ ‪ : 4‬ﺗﺴﻤﻰ اﻟﻨﻘﻄﺔ ‪ M‬ﺻﻮرة اﻟﻌﺪد اﳌﺮﻛﺐ ‪ z‬و ﻧﻜﺘﺐ )‪ M (z‬؛ و ﻳﺴﻤﻰ اﻟﻌﺪد اﳌﺮﻛﺐ ‪ z‬ﻻﺣﻘﺔ اﻟﻨﻘﻄﺔ ‪.M‬‬
‫‪ņőƱŀ Ţ‬‬
‫ﺻﻮرة ﻛﻞ ﻋﺪد ﺣﻘﻴﻘﻲ ﻫﻲ ﻧﻘﻄﺔ ذات اﻹﺣﺪاﺛﻴﲔ )‪ (x, 0‬ﻓﻬﻲ ﻧﻘﻄﺔ ﻣﻦ ﺣﺎﻣﻞ ﳏﻮر اﻟﻔﻮاﺻﻞ‪ .‬ﻳﺴﻤﻰ ﻫﺬا اﳌﺤﻮر ﳏﻮر‬
‫اﻷﻋﺪاد اﳊﻘﻴﻘﻴﺔ‪.‬‬
‫ﺻﻮرة ﻛﻞ ﻋﺪد ﲣﻴﲇ ﴏف ﻫﻲ ﻧﻘﻄﺔ ذات اﻹﺣﺪاﺛﻴﲔ )‪ (0, y‬ﻓﻬﻲ ﻧﻘﻄﺔ ﻣﻦ ﺣﺎﻣﻞ ﳏﻮر اﻟﱰاﺗﻴﺐ‪ .‬ﻳﺴﻤﻰ ﻫﺬا اﳌﺤﻮر ﳏﻮر‬
‫اﻷﻋﺪاد اﻟﺘﺨﻴﻠﻴﺔ‪.‬‬

‫ﻓﺈن ﻓﺎﺻﻠﺔ ‪ M‬ﻫﻲ )‪ Re (z‬و ﺗﺮﺗﻴﺒﺔ ‪ M‬ﻫﻲ )‪. Im (z‬‬‫و ﺑﺎﻟﺘﺎﱄ‪ ،‬إذا ﻛﺎن اﻟﻌﺪد ‪ z‬ﻻﺣﻘﺔ اﻟﻨﻘﻄﺔ ‪ّ M‬‬
‫• ﺻﻮرة اﻟﻌﺪد اﳊﻘﻴﻘﻲ ‪ 1‬ﻫﻲ اﻟﻨﻘﻄﺔ )‪.A (1, 0‬‬ ‫ﺃﻣﺜﻠﺔ ‪: 4‬‬
‫‪ƕ‬‬

‫اﻟﻮﺣﺪة اﻟﺘﺨﻴﻠﻴﺔ ‪ ı‬ﻫﻲ ﻻﺣﻘﺔ اﻟﻨﻘﻄﺔ )‪.B (0, 1‬‬ ‫•‬

‫)ﲡﺎوزا( »اﻟﻨﻘﻄﺔ ‪«z = a + ıb‬ﺑﺪﻻً ﻣﻦ »اﻟﻨﻘﻄﺔ ذات اﻟﻼﺣﻘﺔ ‪«z = a + ıb‬أي أﻧﻨﺎ ﻧُﻄﺎﺑﻖ ﻣﺎ‬
‫ً‬ ‫ﻣﻼﺣﻈﺔ ‪ : 5‬ﻏﺎﻟﺒﺎ ﻣﺎ ﻧﻘﻮل‬
‫ﺑﲔ ﳎﻤﻮﻋﺔ اﻷﻋﺪاد اﳌﺮﻛﺒﺔ ‪ ℂ‬و اﳌﺴﺘﻮي و ﻧُﻄﻠﻖ ﻋﲆ ﻫﺬا اﻷﺧﲑ إﺳﻢ اﳌﺴﺘﻮي اﳌﺮﻛﺐ )ﺷﻜﻞ ‪.(2.I‬‬
‫‪ũŏ‬‬

‫ﲠﺬا اﻟﺘﻤﺜﻴﻞ‪ ،‬ﺗﻜﻮن اﻟﻨﻘﻄﺔ ‪ z‬ﻧﻈﲑة اﻟﻨﻘﻄﺔ ‪ z‬ﺑﺎﻟﻨﺴﺒﺔ ﳌﺤﻮر اﻷﻋﺪاد اﳊﻘﻴﻘﻴﺔ‪ ،‬و اﻟﻨﻘﻄﺔ ‪ −z‬ﻧﻈﲑة اﻟﻨﻘﻄﺔ ‪ z‬ﺑﺎﻟﻨﺴﺒﺔ ﻟﻠﻤﺒﺪأ ‪O‬‬
‫)ﺷﻜﻞ ‪.(3.I‬‬

‫𝟭𝟭‬

‫‪http ://tinyurl.com/Malki1718‬‬ ‫‪0‬‬


‫‪ .3.I‬اﻟﳣﺜﻴﻞ اﻟﻬﻨﺪﳼ ﻟﻌﺪد ﻣﺮﻛﺐ‬
‫‪.‬‬

‫‪Ŕž Ŧ Ľ‬‬
‫‪−z‬‬ ‫‪z‬‬

‫‪I‬‬
‫‪œǃ‬‬
‫‪Ŀ‬‬
‫‪−z‬‬ ‫‪z‬‬

‫ﺷﻜﻞ ‪ :3.I‬اﻟﺘﻨﺎﻇﺮات ﺑﺎﻟﻨﺴﺒﺔ ﻟﻠﻤﺤﺎور و اﳌﺒﺪأ‪.‬‬

‫ﺍﻟﺘﻤﺜﻴﻞ ﺍﻟﺸﻌﺎﻋﻲ‬ ‫‪2.3.I‬‬


‫‪a‬‬
‫‪.‬‬ ‫ﺑﺎﳌﺜﻞ‪ ،‬ﻳﻤﻜﻦ أن ﻧﺮﻓﻖ ﺑﻜﻞ ﻋﺪد ﻣﺮﻛﺐ ‪ z = a + ıb‬اﻟﺸﻌﺎع ⃗‪ V‬اﻟﺬي ﻣﺮﻛﺒﺘﺎه‬
‫‪b‬‬
‫ﻧﻘﻮل أﻧﻨﺎ ﻣﺜﻠﻨﺎ اﻟﻌﺪد اﳌﺮﻛﺐ ‪ z‬ﺑﺎﻟﺸﻌﺎع ⃗‪.V‬‬

‫ﻳﺴﻤﻰ اﻟﺸﻌﺎع ⃗‪ V‬ﺻﻮرة اﻟﻌﺪد اﳌﺮﻛﺐ ‪ z‬و ﻳﺴﻤﻰ اﻟﻌﺪد اﳌﺮﻛﺐ ‪ z‬ﻻﺣﻘﺔ اﻟﺸﻌﺎع ⃗‪.V‬‬ ‫ﺗﻌﺮﻳﻒ ‪: 5‬‬

‫إذا ﻛﺎن ‪ z‬و ‪ z‬ﻋﺪدﻳﻦ ﻣﺮﻛﺒﲔ ﺣﻴﺚ ‪ z = a + ıb‬و ‪ ، z = a + ıb‬و ﻛﺎن ⃗‪ V‬و ‪ V‬ﺻﻮرﺗﻴﻬﲈ ﻋﲆ اﻟﱰﺗﻴﺐ ّ‬
‫ﻓﺈن ‪:‬‬
‫‪z+z = a+a +ı b+b‬‬
‫و‬
‫‪z−z = a−a +ı b−b‬‬
‫‪a+a‬‬
‫‪V⃗ + V‬‬ ‫أي اﻟﺸﻌﺎع‬ ‫أن ﺻﻮرة اﻟﻌﺪد اﳌﺮﻛﺐ ‪ z + z‬ﻫﻲ اﻟﺸﻌﺎع اﻟﺬي ﻣﺮﻛﺒﺘﺎه‬‫أي ّ‬
‫‪ņőƱŀ Ţ‬‬
‫‪b+b‬‬
‫‪a−a‬‬
‫أي اﻟﺸﻌﺎع ‪.V⃗ − V‬‬ ‫و ﺻﻮرة اﻟﻌﺪد اﳌﺮﻛﺐ ‪ z − z‬ﻫﻲ اﻟﺸﻌﺎع اﻟﺬي ﻣﺮﻛﺒﺘﺎه‬
‫‪b−b‬‬

‫‪z+z‬‬ ‫إذا ﻛﺎن اﻟﺸﻌﺎﻋﺎن ⃗‪ V‬و ‪ V‬ﺻﻮرﰐ اﻟﻌﺪدﻳﻦ اﳌﺮﻛﺒﲔ ‪ z‬و ‪ ،z‬ﻳﻜﻮن اﻟﺸﻌﺎع ‪ V⃗ + V‬ﺻﻮرة اﻟﻌﺪد اﳌﺮﻛﺐ‬ ‫ﻣﺒﺮﻫﻨﺔ ‪: 4‬‬

‫و اﻟﺸﻌﺎع ‪ V⃗ − V‬ﺻﻮرة اﻟﻌﺪد اﳌﺮﻛﺐ ‪.z − z‬‬


‫‪ƕ‬‬

‫ﻣﺒﺮﻫﻨﺔ ‪ : 5‬ﻟﻴﻜﻦ )‪ V⃗ (v‬و )‪ W (w‬ﺷﻌﺎﻋﲔ ﻏﲑ ﻣﻌﺪوﻣﲔ‪.‬‬


‫ﻳﻜﻮن اﻟﺸﻌﺎﻋﺎن ⃗‪ V‬و ‪ W‬ﻣﺮﺗﺒﻄﲔ ﺧﻄ ًﻴﺎ )ﻣﺘﻮازﻳﲔ( إذا و ﻓﻘﻂ إذا ﻛﺎن ‪. Im (v ⋅ w) = 0‬‬
‫‪ũŏ‬‬

‫ﺍﻟﱪﻫﺎﻥ‪ .‬اﻟﺸﻌﺎﻋﺎن ⃗‪ V‬و ‪ W‬ﻣﺘﻮازﻳﺎن إذا ﻛﺎن ⃗‪ W = 𝜆V‬ﻣﻊ ‪ 𝜆 ∈ ℝ‬أي إذا ﻛﺎن ‪ w = 𝜆v‬ﻣﻊ ‪ ، 𝜆 ∈ ℝ‬و ﺑﲈ ّ‬
‫أن ‪v ≠ 0‬‬
‫‪w‬‬ ‫‪w‬‬ ‫‪w‬‬ ‫‪w‬‬
‫⋅ ‪ Im vv‬أي‬ ‫‪) vv ⋅ Im‬ﻣﻼﺣﻈﺔ ‪ 4‬ﺻﻔﺤﺔ ‪ (7‬أي ‪= 0‬‬ ‫‪ Im‬أي ‪= 0‬‬ ‫أي ‪= 0‬‬ ‫ﻓﻬﺬا ﻳﻌﻨﻲ ّ‬
‫أن ‪∈ ℝ‬‬
‫‪v‬‬ ‫‪v‬‬ ‫‪v‬‬ ‫‪v‬‬
‫■‬ ‫‪. Im (vw) = 0‬‬

‫𝟮𝟭‬

‫‪http ://tinyurl.com/Malki1718‬‬ ‫‪0‬‬


‫‪ .I‬ﶍﻮﻋﺔ اﻷﻋﺪاد اﳌﺮﻛﺒﺔ‬
‫‪.‬‬
‫ﻣﺜﺎﻝ ‪ : 5‬اﳌﺴﺘﻘﻴﻢ اﳌﺎر ﺑﺎﻟﻨﻘﻄﺘﲔ ) ‪ M (z‬و ) ‪ ، M (z‬ﻣﻊ ‪) M ≠ M‬أي ‪ ، (z ≠ z‬ﻫﻮ ﳎﻤﻮﻋﺔ اﻟﻨﻘﻂ ‪ M‬ﺑﺤﻴﺚ‬
‫‪ M M = tM M‬ﻣﻊ ‪ t‬ﻋﺪد ﺣﻘﻴﻘﻲ ؛ أي ﻫﻲ ﳎﻤﻮﻋﺔ اﻟﻨﻘﻂ ‪ M‬ذات اﻟﻼﺣﻘﺔ ‪ z‬ﺑﺤﻴﺚ ‪ z = tz + (1 − t) z‬ﻣﻊ‬

‫‪Ŕž Ŧ Ľ‬‬
‫‪.t ∈ ℝ‬‬
‫و ﻛﺤﺎﻟﺔ ﺧﺎﺻﺔ‪ ،‬ﳎﻤﻮﻋﺔ ﻧﻘﻂ ﻗﻄﻌﺔ اﳌﺴﺘﻘﻴﻢ اﻟﺮاﺑﻄﺔ ﺑﲔ ‪ M‬و ‪ M‬ﻫﻲ اﻟﺘﻲ ُﲢﻘﻖ ‪ 0 ≤ t ≤ 1‬أي ﻫﻲ اﻟﻨﻘﻂ ‪ M‬ذات‬
‫‪I‬‬

‫‪œǃ‬‬
‫اﻟﻼﺣﻘﺔ ‪ z‬ﺑﺤﻴﺚ ‪ z = tz + (1 − t) z‬ﻣﻊ ]‪. t ∈ [0, 1‬‬

‫ﻛﺘﻄﺒﻴﻘﺎت أوﻟﻴﺔ‪ ،‬ﻧﺪرج ﻓﻴﲈ ﻳﲇ اﻟﻌﻼﻗﺔ ﺑﲔ ﺑﻌﺾ اﻟﻌﻤﻠﻴﺎت ﻋﲆ اﻷﻋﺪاد اﳌﺮﻛﺒﺔ و ﺑﻌﺾ اﻟﺘﺤﻮﻳﻼت اﻟﺒﺴﻴﻄﺔ ﻟﻠﻤﺴﺘﻮي‪.‬‬
‫‪a‬‬
‫‪ .‬اﻹﻧﺴﺤﺎب اﻟﺬي ﺷﻌﺎﻋﻪ ⃗‪ُ V‬ﳛ ّﻮل اﻟﻨﻘﻄﺔ )‪ M (x, y‬إﱃ اﻟﻨﻘﻄﺔ‬ ‫‪ •1‬ﻟﻴﻜﻦ ⃗‪ V‬اﻟﺸﻌﺎع اﻟﺬي ﻣﺮﻛﺒﺘﺎه‬ ‫ﺃﻣﺜﻠﺔ ‪: 6‬‬

‫‪Ŀ‬‬
‫‪b‬‬
‫)‪ M (x + a, y + b‬و ﺑﺎﻟﺘﺎﱄ ‪:‬‬
‫ﻻﺣﻘﺔ ‪ = M‬ﻻﺣﻘﺔ ‪ + M‬ﻻﺣﻘﺔ ⃗‪V‬‬

‫‪ •2‬ﻟﻴﻜﻦ 𝜆 ﻋﺪ ًدا ﺣﻘﻴﻘ ًﻴﺎ‪ .‬اﻟﺘﺤﺎﻛﻲ اﻟﺬي ﻧﺴﺒﺘﻪ 𝜆 و ﻣﺮﻛﺰه ‪ُ O‬ﳛ ّﻮل اﻟﻨﻘﻄﺔ )‪ M (x, y‬إﱃ اﻟﻨﻘﻄﺔ )‪ M (𝜆x, 𝜆y‬ﻣﻨﻪ ‪:‬‬
‫ﻻﺣﻘﺔ ‪ × 𝜆 = M‬ﻻﺣﻘﺔ )‪(M‬‬

‫‪ •3‬ﻛﺤﺎﻟﺔ ﺧﺎﺻﺔ‪ ،‬اﻟﺘﻨﺎﻇﺮ اﳌﺮﻛﺰي ﺑﺎﻟﻨﺴﺒﺔ إﱃ اﻟﻨﻘﻄﺔ ‪ O‬؛ و اﻟﺬي ﻫﻮ ﰲ اﳊﻘﻴﻘﺔ ﻋﺒﺎرة ﻋﻦ اﻟﺘﺤﺎﻛﻲ اﻟﺬي ﻧﺴﺒﺘﻪ ‪ −1‬؛‬
‫ُﳛ ّﻮل اﻟﻨﻘﻄﺔ )‪ M (x, y‬إﱃ اﻟﻨﻘﻄﺔ )‪ M (−x, −y‬و ﺑﺎﻟﺘﺎﱄ ‪:‬‬
‫ﻻﺣﻘﺔ ‪ − = M‬ﻻﺣﻘﺔ )‪(M‬‬

‫‪ •4‬اﻟﺘﺤﺎﻛﻲ اﻟﺬي ﻧﺴﺒﺘﻪ 𝜆 و ﻣﺮﻛﺰه ‪ُ A‬ﳛ ّﻮل اﻟﻨﻘﻄﺔ ‪ M‬إﱃ اﻟﻨﻘﻄﺔ ‪ M‬ﺑﺤﻴﺚ ‪ AM = 𝜆AM :‬و ﺑﺎﻟﺘﺎﱄ ‪:‬‬
‫ﻻﺣﻘﺔ ‪ − M‬ﻻﺣﻘﺔ )‪ × 𝜆 = (A‬ﻻﺣﻘﺔ )‪ − (M‬ﻻﺣﻘﺔ )‪(A‬‬
‫𝜋‬
‫ُﳛ ّﻮل اﻟﺸﻌﺎع ⃗ ‪ i‬إﱃ اﻟﺸﻌﺎع ⃗ ‪ j‬و ُﳛ ّﻮل اﻟﺸﻌﺎع ⃗ ‪ j‬إﱃ اﻟﺸﻌﺎع ⃗ ‪ −i‬؛ و ﺑﺎﻟﺘﺎﱄ ُﳛ ّﻮل‬
‫اﻟﺪوران اﻟﺬي ﻣﺮﻛﺰه ‪ O‬و زاوﻳﺘﻪ‬ ‫‪5‬‬
‫•‬
‫‪2‬‬
‫‪ x , y‬ﻣﻨﻪ‬ ‫اﻟﻨﻘﻄﺔ )‪ M (x, y‬إﱃ اﻟﻨﻘﻄﺔ ‪ M x , y‬ﺑﺤﻴﺚ ⃗ ‪ x i ⃗ + y j ⃗ = xj ⃗ + y −i‬أي )‪= (−y, x‬‬
‫)‪ x + ıy = ı (x + ıy‬و ﺑﺎﻟﺘﺎﱄ ‪:‬‬
‫‪ × ı = M‬ﻻﺣﻘﺔ )‪(M‬‬ ‫ﻻﺣﻘﺔ‬
‫‪ņőƱŀ Ţ‬‬
‫‪ •6‬اﻟﺘﻨﺎﻇﺮ ﺑﺎﻟﻨﺴﺒﺔ إﱃ ﳏﻮر اﻟﻔﻮاﺻﻞ ُﳛ ّﻮل اﻟﻨﻘﻄﺔ )‪ M (x, y‬إﱃ اﻟﻨﻘﻄﺔ )‪ . M (x, −y‬إذا ﻛﺎن ‪ z‬ﻻﺣﻘﺔ ‪ M‬و ‪ z‬ﻻﺣﻘﺔ‬
‫‪ّ M‬‬
‫ﻓﺈن ‪. z = z‬‬
‫اﻟﺘﻨﺎﻇﺮ ﺑﺎﻟﻨﺴﺒﺔ إﱃ ﳏﻮر اﻟﱰاﺗﻴﺐ ُﳛ ّﻮل اﻟﻨﻘﻄﺔ )‪ M (x, y‬إﱃ اﻟﻨﻘﻄﺔ )‪ M (−x, y‬أي ‪) z = −z‬أﻧﻈﺮ اﻟﺸﻜﻞ ‪.(3.I‬‬
‫‪a‬‬
‫⃗‪ . V‬ﻟﺘﻜﻦ ‪ v‬ﻻﺣﻘﺔ‬ ‫‪ M‬و ﻳﻮازي اﻟﺸﻌﺎع ⃗‪≠ 0‬‬ ‫‪ •7‬ﻟﻨﺒﺤﺚ ﻋﻦ ﻣﻌﺎدﻟﺔ اﳌﺴﺘﻘﻴﻢ )‪ (D‬اﻟﺬي ّ‬
‫ﻳﻤﺮ ﺑﺎﻟﻨﻘﻄﺔ‬
‫‪x ,y‬‬
‫‪b‬‬
‫⃗‪ z ، V‬ﻻﺣﻘﺔ ‪ M‬و ‪ z‬ﻻﺣﻘﺔ ‪. M‬‬
‫)‪ M ∈ (D‬إذا و ﻓﻘﻂ إذا ﻛﺎن اﻟﺸﻌﺎﻋﺎن ‪ M M‬و ⃗‪ V‬ﻣﺘﻮازﻳﲔ أي إذا و ﻓﻘﻂ إذا ُوﺟﺪ ‪ 𝜆 ∈ ℝ‬ﺑﺤﻴﺚ ⃗‪M M = 𝜆V‬‬
‫‪.‬‬
‫‪ƕ‬‬

‫ﺣﺴﺐ اﳌﱪﻫﻨﺔ ‪ 5‬ﺻﻔﺤﺔ ‪ 12‬ﻫﺬا ﻳﻜﺎﻓﺊ ‪ Im (v (z − z )) = 0‬أي )ﺑﻌﺪ اﻟﻨﴩ( ‪b (x − x ) − a y − y = 0 :‬‬


‫‪.‬‬
‫‪a‬‬
‫⃗‪. V‬‬ ‫‪ M‬و ﻳﻌﺎﻣﺪ اﻟﺸﻌﺎع ⃗‪≠ 0‬‬ ‫‪x ,y‬‬ ‫‪ •8‬ﻟﻨﺒﺤﺚ ﻋﻦ ﻣﻌﺎدﻟﺔ اﳌﺴﺘﻘﻴﻢ ‪ D‬اﻟﺬي ّ‬
‫ﻳﻤﺮ ﺑﺎﻟﻨﻘﻄﺔ‬
‫‪b‬‬
‫‪ũŏ‬‬

‫اﻟﺪوران اﻟﺬي زاوﻳﺘﻪ 𝜋 ﻳﴬب ﻻﺣﻘﺔ اﻟﺸﻌﺎع ⃗‪ V‬ﺑﺎﻟﻌﺪد ‪ ، ı‬و ﺑﺎﺳﺘﺨﺪام ﻧﻔﺲ ﺗﺮﻣﻴﺰات اﳌﺜﺎل اﻟﺴﺎﺑﻖ‪ ،‬ﻧﺠﺪ ّ‬
‫أن‬
‫‪2‬‬
‫‪ M ∈ D‬إذا و ﻓﻘﻂ إذا ﻛﺎن ‪ Im (ıv (z − z )) = 0‬أي إذا و ﻓﻘﻂ إذا ﻛﺎن أي ‪ . Re (v (z − z )) = 0‬ﺑﻌﺪ‬
‫اﻟﻨﴩ‪ ،‬ﻧﺠﺪ اﳌﻌﺎدﻟﺔ ‪. a (x − x ) + y − y = 0 :‬‬
‫ﻧﻠﻔﺖ اﻹﻧﺘﺒﺎه إﱃ اﻟﻨﺘﻴﺠﺔ اﻟﺘﺎﻟﻴﺔ ‪ :‬ﻧﻌﺘﱪ اﻟﺸﻌﺎﻋﲔ )‪ V⃗ (z‬و ‪ . V z‬ﻟﺪﻳﻨﺎ ‪:‬‬
‫‪V⃗ ⟂ V ⟺ Re zz‬‬ ‫‪=0‬‬

‫𝟯𝟭‬

‫‪http ://tinyurl.com/Malki1718‬‬ ‫‪0‬‬


‫‪ .3.I‬اﻟﳣﺜﻴﻞ اﻟﻬﻨﺪﳼ ﻟﻌﺪد ﻣﺮﻛﺐ‬
‫‪.‬‬
‫ﰲ اﻟﺒﺎب ‪ IV‬ﺻﻔﺤﺔ ‪ 51‬دراﺳﺔ أﺷﻤﻞ و أﻋﻤﻖ ﻟﻜﻞ اﳌﻔﺎﻫﻴﻢ اﳍﻨﺪﺳﻴﺔ اﳌﺄﻟﻮﻓﺔ و اﻟﺘﻌﺒﲑ ﻋﻨﻬﺎ ﺑﺎﻷﻋﺪاد اﳌﺮﻛﺒﺔ‪.‬‬

‫‪Ŕž Ŧ Ľ‬‬
‫‪I‬‬
‫‪œǃ‬‬
‫‪Ŀ‬‬
‫‪ņőƱŀ Ţ‬‬
‫‪ũŏ‬‬ ‫‪ƕ‬‬

‫𝟰𝟭‬

‫‪http ://tinyurl.com/Malki1718‬‬ ‫‪0‬‬


‫‪.‬‬

‫‪Ŕž Ŧ Ľ‬‬
‫‪2‬‬
‫‪œǃ‬‬
‫‪otttttttttttttttttttttttttttttttttttttttttttp‬‬
‫‪s‬‬ ‫‪u‬‬
‫‪s‬‬ ‫اﻟﺸﻜﻞ اﻟﻤﺜﻠﺜﻲ ﻟﻌﺪد ﻣﺮﻛﺐ‬
‫‪u‬‬

‫‪Ŀ‬‬
‫‪s‬‬ ‫‪u‬‬
‫‪II‬‬

‫‪s‬‬ ‫‪u‬‬
‫‪qvvvvvvvvvvvvvvvvvvvvvvvvvvvvvvvvvvvvvvvvvvvr‬‬
‫‪.‬‬

‫ﻓﻴﲈ ﻳﲇ‪ ،‬اﳌﺴﺘﻮي ﻣﻨﺴﻮب إﱃ ﻣﻌﻠﻢ ﻣﺘﻌﺎﻣﺪ و ﻣﺘﺠﺎﻧﺲ ⃗ ‪. O, i,⃗ j‬‬

‫ﻃﻮﻳﻠﺔ ﻋﺪﺩ ﻣﺮﻛﺐ‬ ‫‪1.II‬‬

‫ﺗﻌﺮﻳﻒ‬ ‫‪1.1.II‬‬
‫ﻟﺘﻌﻠﻴﻢ اﻟﻨﻘﻂ ﰲ اﳌﺴﺘﻮي اﳌﺮﻛﺐ‪ ،‬ﻧﺴﺘﻌﻤﻞ اﻹﺣﺪاﺛﻴﺎت اﻟﺪﻳﻜﺎرﺗﻴﺔ أي اﳉﺰء اﳊﻘﻴﻘﻲ و اﳉﺰء اﻟﺘﺨﻴﲇ‪ .‬ﻟﻜﻦ ﻫﻨﺎك ﻃﺮﻳﻘﺔ‬
‫أﺧﺮى ﻟﻠﻘﻴﺎم ﺑﺬﻟﻚ و ﻫﺬا ﺑﺎﺳﺘﺨﺪام ﻋﺪدﻳﻦ ‪ 𝜌 ،‬و 𝜃 ‪ ،‬و اﻟﺬﻳﻦ ﻳﻤﺜﻼن اﻹﺣﺪاﺛﻴﺎت اﻟﻘﻄﺒﻴﺔ )ﺷﻜﻞ ‪.(1.II‬‬

‫‪b‬‬ ‫‪z‬‬
‫‪ņőƱŀ Ţ‬‬
‫‪ρ‬‬

‫‪θ‬‬
‫‪a‬‬
‫‪O‬‬
‫‪ƕ‬‬

‫ﺷﻜﻞ ‪ :1.II‬اﻹﺣﺪاﺛﻴﺎت اﻟﻘﻄﺒﻴﺔ‪.‬‬


‫‪ũŏ‬‬

‫اﻟﻌﺪد 𝜌 ﻳﻤﺜﻞ اﳌﺴﺎﻓﺔ ﺑﲔ ﻣﺒﺪأ اﳌﻌﻠﻢ ‪ 0‬و اﻟﻨﻘﻄﺔ ‪ z‬و ﻫﻮ ﻋﺪد ﺣﻘﻴﻘﻲ ﻣﻮﺟﺐ‪.‬‬
‫)‪.𝜌 = Re (z) + Im (z‬‬ ‫‪ 𝜌 = a + b‬أي‬ ‫ﺣﺴﺐ ﻧﻈﺮﻳﺔ ﻓﻴﺜﺎﻏﻮرث ‪:‬‬
‫اﻟﻌﺪد 𝜃 ﻫﻮ اﻟﺰاوﻳﺔ اﳌﻮﺟﻬﺔ ﺑﲔ ﳏﻮر اﻷﻋﺪاد اﳊﻘﻴﻘﻴﺔ و اﳌﺴﺘﻘﻴﻢ اﻟﺮاﺑﻂ ﺑﲔ اﻟﻨﻘﻄﺘﲔ ‪ 0‬و ‪.z‬‬

‫𝟱𝟭‬

‫‪http ://tinyurl.com/Malki1718‬‬ ‫‪0‬‬


‫‪ .1.II‬ﻃﻮﯾةل ﻋﺪد ﻣﺮﻛﺐ‬
‫‪.‬‬
‫ﺗﻌﺮﻳﻒ ‪ z : 6‬ﻋﺪد ﻣﺮﻛﺐ ﺣﻴﺚ ‪.z = a + ıb‬‬

‫‪Ŕž Ŧ Ľ‬‬
‫ﻧﺮﻣﺰ إﱃ ﻃﻮﻳﻠﺔ ‪ z‬ﺑﺎﻟﺮﻣﺰ |‪.|z‬‬ ‫‪.‬‬ ‫‪a +b‬‬ ‫ﻧﺴﻤﻲ ﻃﻮﻳﻠﺔ ‪ z‬اﻟﻌﺪد اﳊﻘﻴﻘﻲ اﳌﻮﺟﺐ‬

‫‪œǃ‬‬
‫= ‖‖‪.𝜌 = OM = ‖‖OM‬‬ ‫|‪a + b = |z‬‬ ‫إذا ﻛﺎﻧﺖ اﻟﻨﻘﻄﺔ ‪ M‬ﺻﻮرة اﻟﻌﺪد ‪ّ z‬‬
‫ﻓﺈن‬
‫و ﺑﺎﻟﺘﺎﱄ‪ ،‬ﻳﻤﻜﻦ أن ﻧﻜﺘﺐ ‪:‬‬
‫‪z‬‬ ‫‪zz‬‬ ‫‪zz‬‬ ‫‪aa + bb‬‬ ‫‪ab − ba‬‬
‫=‬ ‫=‬ ‫=‬ ‫‪+ı‬‬

‫‪Ŀ‬‬
‫‪z‬‬ ‫‪zz‬‬ ‫|‪|z‬‬ ‫‪a +b‬‬ ‫‪a +b‬‬

‫‪II‬‬
‫ﻣﺜﺎﻝ ‪.||1 + ı|| = 1 + 1 = √2 : 7‬‬
‫ﻣﻦ اﻟﺘﻌﺮﻳﻒ‪ ،‬ﻧﺴﺘﻨﺘﺞ ﻣﺒﺎﴍة اﳋﻮاص اﻟﺘﺎﻟﻴﺔ ‪:‬‬

‫|‪|−z| = |z‬‬ ‫و‬ ‫|‪||z|| = |z‬‬ ‫•‬ ‫‪1‬‬


‫‪ •2‬إذا ﻛﺎن ‪ z‬ﺣﻘﻴﻘﻴﺎ ّ‬
‫ﻓﺈن ﻃﻮﻳﻠﺘﻪ ﺗﺴﺎوي ﻗﻴﻤﺘﻪ اﳌﻄﻠﻘﺔ‪.‬‬

‫ﺧﻮﺍﺹ‬ ‫‪2.1.II‬‬
‫ﻟﻴﻜﻦ ‪ z‬و ‪ z‬ﻋﺪدﻳﻦ ﻣﺮﻛﺒﲔ ﺣﻴﺚ ‪ z = a + ıb‬و ‪ . z = a + ıb‬ﻟﺪﻳﻨﺎ ‪:‬‬

‫‪. |z| = 0 ⟺ z = 0 •1‬‬ ‫ﻣﺒﺮﻫﻨﺔ ‪: 6‬‬

‫‪.|z| = √z z •2‬‬
‫‪.||z ⋅ z || = |z| ⋅ ||z || •3‬‬
‫‪ņőƱŀ Ţ‬‬
‫|‪|1‬‬ ‫‪1‬‬
‫)ﻣﻊ ‪.(z ≠ 0‬‬ ‫=| |‬ ‫•‬ ‫‪4‬‬
‫|‪|| z || |z‬‬

‫|‪| z | |z‬‬
‫)ﻣﻊ ‪.(z ≠ 0‬‬ ‫| ‪|| z || = |z‬‬ ‫•‬ ‫‪5‬‬
‫| |‬
‫‪. Re (z) ≤ |z| •6‬‬
‫‪. Re zz ≤ |z| ⋅ ||z || •7‬‬
‫‪ƕ‬‬

‫‪ ||z + z || ≤ |z| + ||z || •8‬و ﻧﺴﻤﻲ ﻫﺬه اﳌﺘﺒﺎﻳﻨﺔ )اﳌﱰاﺟﺤﺔ( اﳌﺘﺒﺎﻳﻨﺔ اﳌﺜﻠﺜﻴﺔ أو ﻣﺘﺒﺎﻳﻨﺔ اﳌﺜﻠﺚ‪.‬‬

‫⟺ ‪. |z| = 0‬‬ ‫‪a +b =0 ⟺ a +b =0 ⟺ a=b=0 ⟺ z=0‬‬ ‫•‬ ‫‪1‬‬ ‫ﺍﻟﱪﻫﺎﻥ‪.‬‬


‫‪ũŏ‬‬

‫‪.|z| = √z z‬‬ ‫أي‬ ‫‪z z = (a + ıb) ⋅ (a − ıb) = a − (ıb) = a + b‬‬ ‫•‬ ‫‪2‬‬
‫= || ‪.||z ⋅ z‬‬ ‫‪(z ⋅ z ) ⋅ z ⋅ z‬‬ ‫=‬ ‫‪(z ⋅ z ) ⋅ z ⋅ z‬‬ ‫=‬ ‫‪(z z) ⋅ z z‬‬ ‫⋅ ‪= √z z‬‬ ‫|| ‪z z = |z| ⋅ ||z‬‬ ‫•‬ ‫‪3‬‬
‫‪1‬‬ ‫|‬ ‫|‪1‬‬ ‫|‪|1‬‬ ‫|‪|1‬‬ ‫‪1‬‬
‫× ‪.z‬‬ ‫× ‪= 1 ⟹ |z‬‬ ‫= | | ⟹ ‪| = ||1|| = 1 ⟹ |z| × | | = 1‬‬ ‫•‬ ‫‪4‬‬
‫‪z‬‬ ‫||‬ ‫|| ‪z‬‬ ‫|| ‪|| z‬‬ ‫|‪|| z || |z‬‬

‫𝟲𝟭‬

‫‪http ://tinyurl.com/Malki1718‬‬ ‫‪0‬‬


‫‪ .II‬اﻟﺸﲁ اﳌﺜﻠﱻ ﻟﻌﺪد ﻣﺮﻛﺐ‬
‫‪.‬‬
‫‪z‬‬ ‫|‬ ‫|‪1‬‬ ‫|‪|1‬‬ ‫‪1‬‬ ‫|‪|z‬‬
‫‪.||| ||| = |z × | = |z| × | | = |z| × | | = | | •5‬‬
‫‪z‬‬ ‫||‬ ‫|| ‪z‬‬ ‫|| ‪|| z‬‬ ‫| ‪|z | |z‬‬

‫‪Ŕž Ŧ Ľ‬‬
‫≤ ‪.a‬‬ ‫‪a +b‬‬ ‫‪ •6‬ﻣﺎ ﳚﺐ إﺛﺒﺎﺗﻪ ﻫﻮ ‪:‬‬

‫‪œǃ‬‬
‫ﻓﺈن ‪ a ≤ 0 ≤ a + b‬و اﳌﱰاﺟﺤﺔ ﳏﻘﻘﺔ ؛‬ ‫إذا ﻛﺎن ‪ّ a ≤ 0‬‬ ‫•‬

‫أ ّﻣﺎ إذا ﻛﺎن ‪ a > 0‬ﻓﱰﺑﻴﻊ ﻃﺮﰲ اﳌﱰاﺟﺤﺔ ﻧﺠﺪ أﻧﻪ ﻋﻠﻴﻨﺎ أن ﻧﺜﺒﺖ ّ‬
‫أن ‪ a ≤ a + b‬أي ‪ b ≥ 0‬و ﻫﺬا ﳏﻘﻖ‬ ‫•‬

‫دو ًﻣﺎ‪.‬‬

‫‪Ŀ‬‬
‫‪ •7‬إذا ﻛﺎن ‪ z = 0‬ﻓﺎﳌﱰاﺟﺤﺔ ﳏﻘﻘﺔ‪ .‬ﻧﻔﺮض إذن ّ‬
‫أن ‪ . z ≠ 0‬ﺑﻘﺴﻤﺔ ﻃﺮﰲ اﳌﱰاﺟﺤﺔ ﻋﲆ اﻟﻌﺪد اﳊﻘﻴﻘﻲ اﳌﻮﺟﺐ ﲤﺎ ًﻣﺎ‬
‫‪II‬‬

‫‪1‬‬ ‫|| ‪|z| ⋅ ||z‬‬


‫‪.‬‬ ‫‪Re zz‬‬ ‫≤‬ ‫‪ zz‬ﻳﻨﺘﺞ‬
‫‪zz‬‬ ‫‪zz‬‬
‫‪1‬‬ ‫‪zz‬‬ ‫‪z‬‬ ‫‪1‬‬
‫)ﻣﻼﺣﻈﺔ ‪ 4‬ﺻﻔﺤﺔ ‪ (7‬أي ﳚﺐ أن‬ ‫‪Re zz‬‬ ‫‪= Re‬‬ ‫‪= Re‬‬ ‫ﻋﺪد ﺣﻘﻴﻘﻲ و ﺑﺎﻟﺘﺎﱄ‬ ‫ﻟﻜﻦ‬
‫‪zz‬‬ ‫‪zz‬‬ ‫‪z‬‬ ‫‪zz‬‬
‫‪z‬‬ ‫‪z‬‬ ‫| ‪|z‬‬
‫= ‪ w‬و ﻫﻮ ﻣﺎ أﺛﺒﺘﻨﺎه ﺳﺎﺑﻘ ًﺎ‪.‬‬ ‫‪ Re‬أي |‪ Re (w) ≤ |w‬ﺑﻮﺿﻊ‬ ‫أن | | ≤‬ ‫ﻧﱪﻫﻦ ّ‬
‫‪z‬‬ ‫‪z‬‬ ‫|| ‪|| z‬‬

‫‪ •8‬ﻣﺎ ﳚﺐ إﺛﺒﺎﺗﻪ ﻫﻮ ‪:‬‬


‫‪(a + a ) + b + b‬‬ ‫≤‬ ‫‪a +b +‬‬ ‫‪a‬‬ ‫‪+b‬‬ ‫)𝟏(‬
‫أن داﻟﺔ اﳉﺬر اﻟﱰﺑﻴﻌﻲ ⋯√ ﻣﺘﺰاﻳﺪة‪ ،‬ﻓﺎﳌﱰاﺟﺤﺔ ‪ x ≤ y‬ﺗﻜﺎﻓﺊ ‪ x ≤ y‬ﻟﻜﻞ ﻋﺪدﻳﻦ ﺣﻘﻴﻘﻴﲔ ﻣﻮﺟﺒﲔ ‪ x‬و ‪.y‬‬
‫و ﺑﲈ ّ‬
‫ﺑﱰﺑﻴﻊ ﻃﺮﰲ اﳌﱰاﺟﺤﺔ )𝟏( و ﺑﻌﺪ اﻟﺘﺒﺴﻴﻂ ‪ ،‬ﻧﺠﺪ أﻧﻪ ﻋﻠﻴﻨﺎ إﺛﺒﺎت ّ‬
‫أن اﳌﱰاﺟﺤﺔ ‪:‬‬

‫≤ ‪aa + bb‬‬ ‫‪a +b‬‬ ‫‪a‬‬ ‫‪+b‬‬

‫ﺻﺤﻴﺤﺔ ﻣﻬﲈ ﺗﻜﻦ اﻷﻋﺪاد اﳊﻘﻴﻘﻴﺔ ‪ a ، b ، a‬و ‪.b‬‬

‫≤ ‪ aa + bb < 0‬أي اﳌﱰاﺟﺤﺔ ﳏﻘﻘﺔ !‬ ‫‪a +b‬‬ ‫‪a‬‬ ‫‪+b‬‬ ‫إذا ﻛﺎن ‪ّ aa + bb < 0‬‬
‫ﻓﺈن‬ ‫•‬
‫‪ņőƱŀ Ţ‬‬
‫أ ّﻣﺎ إذا ﻛﺎن ‪ ،aa + bb ≥ 0‬ﻓﺒﱰﺑﻴﻊ ﻃﺮﰲ اﳌﱰاﺟﺤﺔ و اﻟﺘﺒﺴﻴﻂ‪ ،‬ﺗﻜﻮن ﻫﺬه اﻷﺧﲑة ﻣﻜﺎﻓﺌﺔ ﻟﻠﻤﱰاﺟﺤﺔ‬ ‫•‬

‫‪ 0 ≤ ab − ba‬و ﻫﺬا ﺻﺤﻴﺢ دو ًﻣﺎ‪.‬‬ ‫‪ 2aa bb ≤ a b + b a‬أي‬


‫■‬
‫ﻣﻼﺣﻈﺔ ‪ ⧏ : 6‬ﻣﻦ اﳌﻌﺮوف ّ‬
‫أن اﳋﻂ اﳌﺴﺘﻘﻴﻢ ﻫﻮ أﻗﴫ ﻣﺴﺎﻓﺔ ﺑﲔ ﻧﻘﻄﺘﲔ‪ ،‬و اﳌﺘﺒﺎﻳﻨﺔ اﳌﺜﻠﺜﻴﺔ ﺗﻨﺺ ﻋﲆ أﻧّﻪ ﰲ اﳌﺜﻠﺚ‬
‫اﻟﺬي رؤوﺳﻪ ‪ z ،0‬و ‪ ، z + z‬اﻟﻄﺮﻳﻖ اﳌﺴﺘﻘﻴﻢ ﻣﻦ ‪ 0‬إﱃ ‪ z + z‬أﻗﴫ ﻣﻦ اﻟﺬي ﻳﻤﺮ ﺑﺎﻟﻨﻘﻄﺔ ‪) z‬ﺷﻜﻞ ‪ (2.II‬و ﻫﺬا ﻣﺎ ُﻳ ﱢ‬
‫ﱪر‬
‫ﺗﺴﻤﻴﺘﻬﺎ ﺑﺎﳌﺘﺒﺎﻳﻨﺔ ﺍﳌﺜﻠﺜﻴﺔ‪.‬‬
‫‪ƕ‬‬

‫⧐‬
‫أن || ‪ّ |z| ⋅ ||z || = ||z z‬‬
‫ﻓﺈن‬ ‫ﻟﺪﻳﻨﺎ ‪ z ⋅ z = aa − bb + ı ab + ba :‬و ﺑﲈ ّ‬ ‫ﻣﻼﺣﻈﺔ ‪⧏ : 7‬‬
‫× ‪a +b‬‬ ‫‪a +b‬‬ ‫=‬ ‫‪aa − bb‬‬ ‫‪+ ab + ba‬‬
‫‪ũŏ‬‬

‫و ﺑﱰﺑﻴﻊ اﻟﻄﺮﻓﲔ‬
‫‪a +b‬‬ ‫‪a +b‬‬ ‫‪= aa − bb‬‬ ‫‪+ ab + ba‬‬

‫ﻓﺈن ﻫﺬه اﳌﺴﺎواة ﺗﻨﺺ ﻋﲆ ّ‬


‫أن‬ ‫و ﻫﻲ ﻣﺘﻄﺎﺑﻘﺔ ﳍﺎ أﳘﻴﺔ ﻛﺒﲑة ﰲ اﻟﺮﻳﺎﺿﻴﺎت )ﻣﺜﻼً‪ ،‬إذا ﻛﺎﻧﺖ اﻷﻋﺪاد ‪ a ، b ، a‬و ‪ b‬ﻃﺒﻴﻌﻴﺔ‪ّ ،‬‬
‫ﺟﺪاء ﻋﺪدﻳﻦ ُﻳﻜﺘﺒﺎن ﻋﲆ ﺷﻜﻞ ﳎﻤﻮع ُﻣﺮ ّﺑﻌﲔ ﺗﺎ ﱠﻣﲔ ﻫﻮ ﻋﺪد ُﻳﻜﺘﺐ ﻋﲆ ﺷﻜﻞ ﳎﻤﻮع ُﻣﺮ ّﺑﻌﲔ ﺗﺎ ﱠﻣﲔ(‪.‬‬
‫أﻧﻈﺮ اﻟﺘﻤﺮﻳﻦ ‪ 137‬ﺻﻔﺤﺔ ‪206‬‬ ‫‬
‫𝟳𝟭‬

‫‪http ://tinyurl.com/Malki1718‬‬ ‫‪0‬‬


‫‪ .2.II‬ﲻﺪة ﻋﺪد ﻣﺮﻛﺐ ﻏﲑ ﻣﻌﺪوم‬
‫‪.‬‬

‫‪z + z′‬‬

‫‪Ŕž Ŧ Ľ‬‬
‫‪œǃ‬‬
‫‪z‬‬

‫‪Ŀ‬‬
‫‪z′‬‬

‫‪II‬‬
‫‪O‬‬

‫ﺷﻜﻞ ‪ :2.II‬اﳌﺘﺒﺎﻳﻨﺔ اﳌﺜﻠﺜﻴﺔ‪.‬‬

‫⧐‬

‫ﻳﻤﻜﻦ ﺗﻌﻤﻴﻢ اﳌﺘﺒﺎﻳﻨﺔ اﳌﺜﻠﺜﻴﺔ ‪:‬‬

‫ﻣﺒﺮﻫﻨﺔ ‪ : 7‬ﻣﻬﲈ ﺗﻜﻦ اﻷﻋﺪاد اﳌﺮﻛﺒﺔ ‪ّ z ، ⋯ ، z ، z‬‬


‫ﻓﺈن ‪:‬‬
‫|| ‪||z + z + ⋯ + z || ≤ ||z || + ||z || + ⋯ + ||z‬‬

‫■‬ ‫ﺍﻟﱪﻫﺎﻥ‪ .‬ﺑﺎﻟﱰاﺟﻊ ﻋﲆ اﻟﻌﺪد اﻟﻄﺒﻴﻌﻲ ‪ n ≥ 2‬و ﻧﱰك ذﻟﻚ ﻟﻠﻘﺎرئ اﻟﻜﺮﻳﻢ‪.‬‬
‫ﺑﺘﻄﺒﻴﻖ اﳌﺘﺒﺎﻳﻨﺔ اﳌﺜﻠﺜﻴﺔ ﻋﲆ اﻟﻌﺪدﻳﻦ ‪ z‬و ‪ z − z‬ﻳﻨﺘﺞ || ‪ ||z + (z − z )|| ≤ ||z || + ||z − z‬أي || ‪||z || ≤ ||z || + ||z − z‬‬
‫‪ņőƱŀ Ţ‬‬
‫ﻣﻨﻪ || ‪ . ||z ||−||z || ≤ ||z − z‬ﺑﺎﳌﺜﻞ ﻣﻊ اﻟﻌﺪدﻳﻦ ‪ z‬و ‪ z −z‬ﻟﻨﺠﺪ || ‪ . ||z ||−||z || ≤ ||z − z‬ﻟﻜﻦ || ‪||z − z || = ||z − z‬‬
‫إذن || ‪. ||z || − ||z || ≤ ||z − z‬‬
‫أن || ‪ ||| ||z || − ||z || ||| ≤ ||z − z‬و‬
‫ﰲ اﻷﺧﲑ‪ ،‬ﻟﺪﻳﻨﺎ || ‪ ||z || − ||z || ≤ ||z − z‬و || ‪ − ||z || − ||z || ≤ ||z − z‬و ﻫﺬا ﻳﻌﻨﻲ ّ‬
‫ﻫﻲ اﳌﺘﺒﺎﻳﻨﺔ اﳌﺜﻠﺜﻴﺔ اﻟﺜﺎﻧﻴﺔ ‪.‬‬

‫ﻋﻤﺪﺓ ﻋﺪﺩ ﻣﺮﻛﺐ ﻏﲑ ﻣﻌﺪﻭﻡ‬ ‫‪2.II‬‬


‫‪ƕ‬‬

‫ﺗﻌﺮﻳﻒ‬ ‫‪1.2.II‬‬
‫‪ũŏ‬‬

‫ﺗﻌﺮﻳﻒ ‪ : 7‬ﻟﻴﻜﻦ ‪ z‬ﻋﺪ ًدا ﻣﺮﻛ ًﺒﺎ ﻏﲑ ﻣﻌﺪوم و ‪ M‬ﺻﻮرﺗﻪ‪.‬‬


‫و ﻧﻜﺘﺐ )‪.𝜃 = arg (z‬‬ ‫‪𝜃 = i,⃗ OM‬‬ ‫ﻛﻞ ﻗﻴﺲ ﻟﻠﺰاوﻳﺔ‬‫ﻧﺴﻤﻲ ُﻋﻤﺪ ًة ﻟﻠﻌﺪد اﳌﺮﻛﺐ ‪ّ z‬‬

‫ﻣﻦ اﻟﺸﻜﻞ ‪ 1.II‬ﻧﺴﺘﻨﺘﺞ ّ‬


‫أن ﻋﻤﺪة اﻟﻌﺪد اﳌﺮﻛﺐ ‪ z‬ﻫﻮ ﻛﻞ ﻋﺪد ُﳛﻘﻖ‬
‫‪a‬‬ ‫‪b‬‬
‫= 𝜃 ‪cos‬‬ ‫و‬ ‫= 𝜃 ‪sin‬‬
‫‪a +b‬‬ ‫‪a +b‬‬

‫𝟴𝟭‬

‫‪http ://tinyurl.com/Malki1718‬‬ ‫‪0‬‬


‫‪ .II‬اﻟﺸﲁ اﳌﺜﻠﱻ ﻟﻌﺪد ﻣﺮﻛﺐ‬
‫‪.‬‬
‫ﻓﺈن ﻛﻞ ﻋﺪد ﻣﻦ اﻟﺸﻜﻞ 𝜋‪ 𝜃 + 2k‬ﺣﻴﺚ ‪ k ∈ ℤ‬ﻫﻮ أﻳﻀﺎ ﻋﻤﺪة ﻟﻪ و‬‫ﰲ آن واﺣﺪ ؛ و ﻫﺬا ﻳﻌﻨﻲ أﻧﻪ إذا ﻛﺎن 𝜃 ﻋﻤﺪ ًة ﻟﻠﻌﺪد ‪ّ z‬‬
‫أن اﻷﻋﺪاد 𝜋‪ 𝜃 + 2k‬ﻫﻲ اﻟﻮﺣﻴﺪة اﻟﺘﻲ ﺗﺘﻤﺘﻊ ﲠﺬه اﳋﺎﺻﻴﺔ‪ ،‬و ﻟﺘﺄﻛﻴﺪ ذﻟﻚ ﻧﻜﺘﺐ‬ ‫ّ‬

‫‪Ŕž Ŧ Ľ‬‬
‫)𝜋‪arg (z) = 𝜃 (mod 2‬‬
‫»ﻋﻤﺪة ‪ z‬ﺗﺴﺎوي 𝜃 ﺑﱰدﻳﺪ 𝜋‪.«2‬‬ ‫و ﻧﻘﺮأ ‪:‬‬

‫‪œǃ‬‬
‫ﻟﻴﻜﻦ 𝜃 ﻋﻤﺪة ﻟﻠﻌﺪد اﳌﺮﻛﺐ ‪ . ı‬ﻟﺪﻳﻨﺎ ‪:‬‬ ‫ﻣﺜﺎﻝ ‪: 8‬‬
‫‪0‬‬ ‫‪1‬‬
‫= 𝜃 ‪cos‬‬ ‫‪=0‬‬ ‫و‬ ‫= 𝜃 ‪sin‬‬ ‫‪=1‬‬
‫‪0 +1‬‬ ‫‪0 +1‬‬

‫‪Ŀ‬‬
‫‪II‬‬

‫أي‬
‫𝜋‬
‫= )‪arg (ı‬‬ ‫)𝜋‪(mod 2‬‬
‫‪2‬‬
‫و ﻫﺬه اﻟﻜﺘﺎﺑﺔ اﻷﺧﲑة ﺗﻌﻨﻲ ّ‬
‫أن ﻛﻞ ﻋﻨﴫ ﻣﻦ اﳌﺠﻤﻮﻋﺔ )ﻏﲑ اﳌﻨﺘﻬﻴﺔ(‬
‫𝜋‪7𝜋 3𝜋 𝜋 5𝜋 9‬‬
‫‪⋯,−‬‬ ‫⋯‪,− , , , ,‬‬
‫‪2‬‬ ‫‪2 2 2 2‬‬
‫ﻫﻮ ﻋﻤﺪة ﻟﻠﻮﺣﺪة اﻟﺘﺨﻴﻠﻴﺔ ‪.ı‬‬
‫ﻣﻼﺣﻈﺔ ‪⧏ : 8‬‬

‫اﻟﻌﺪد 𝜃 ﻫﻮ زاوﻳﺔ اﻟﺪوران اﻟﺬي ُﳛ ﱢﻮل ﳏﻮر اﻷﻋﺪاد اﳊﻘﻴﻘﻴﺔ إﱃ اﳌﺴﺘﻘﻴﻢ )‪ (OM‬ا ُﳌ َﻮ ّﺟﻪ ﻣﻦ ‪ O‬إﱃ ‪.M‬‬ ‫•‬

‫أن اﻟﺰاوﻳﺔ ‪ i,⃗ OO‬ﻏﲑ ﻣﻌﺮﻓﺔ ّ‬


‫ﻓﺈن اﻟﻌﺪد اﳌﺮﻛﺐ اﳌﻌﺪوم ﻻ ﻋﻤﺪة ﻟﻪ‪.‬‬ ‫ﺻﻮرة اﻟﻌﺪد اﳌﺮﻛﺐ ‪ 0‬ﻫﻲ اﳌﺒﺪأ ‪ O‬و ﺑﲈ ّ‬ ‫•‬

‫أن ﺻﻮرﰐ اﻟﻌﺪدﻳﻦ ‪ z‬و ‪ z‬ﻣﺘﻨﺎﻇﺮﺗﺎن ﺑﺎﻟﻨﺴﺒﺔ إﱃ ﳏﻮر اﻟﻔﻮاﺻﻞ ّ‬


‫ﻓﺈن ‪:‬‬ ‫ﺑﲈ ّ‬ ‫•‬

‫‪arg (z) = − arg (z) + 2k𝜋 ,‬‬ ‫‪k∈ℤ‬‬

‫⧐‬

‫ﺑﻌﺾ اﳊﺎﻻت اﳋﺎﺻﺔ اﳌﻬﻤﺔ ‪ :‬اﻷﻋﺪاد اﳌﺮﻛﺒﺔ اﻟﺘﻲ ))𝜋‪ 0 ( (mod 2‬ﻋﻤﺪة ﳍﺎ ﻫﻲ اﻷﻋﺪاد اﳊﻘﻴﻘﻴﺔ اﳌﻮﺟﺒﺔ ﲤﺎ ًﻣﺎ‪.‬‬
‫اﻷﻋﺪاد اﳌﺮﻛﺒﺔ اﻟﺘﻲ ))𝜋‪ 𝜋 ( (mod 2‬ﻋﻤﺪة ﳍﺎ ﻫﻲ اﻷﻋﺪاد اﳊﻘﻴﻘﻴﺔ اﻟﺴﺎﻟﺒﺔ ﲤﺎ ًﻣﺎ‪.‬‬
‫‪ņőƱŀ Ţ‬‬
‫𝜋‬
‫اﻷﻋﺪاد اﳌﺮﻛﺒﺔ اﻟﺘﻲ ))𝜋‪ ( (mod 2‬ﻋﻤﺪة ﳍﺎ ﻫﻲ ﻋﻨﺎﴏ اﳌﺠﻤﻮﻋﺔ ‪. ıℝ∗+‬‬
‫‪2‬‬
‫∗‬ ‫𝜋‬
‫‪.‬‬ ‫‪ıℝ−‬‬ ‫اﻷﻋﺪاد اﳌﺮﻛﺒﺔ اﻟﺘﻲ ))𝜋‪ − ( (mod 2‬ﻋﻤﺪة ﳍﺎ ﻫﻲ ﻋﻨﺎﴏ اﳌﺠﻤﻮﻋﺔ‬
‫‪2‬‬

‫ﺍﻟﺸﻜﻞ ﺍﳌﺜﻠﺜﻲ ﻟﻌﺪﺩ ﻣﺮﻛﺐ ﻏﲑ ﻣﻌﺪﻭﻡ‬ ‫‪2.2.II‬‬

‫ﻟﻴﻜﻦ ‪ z‬ﻋﺪ ًدا ﻣﺮﻛ ًﺒﺎ ﻏﲑ ﻣﻌﺪوم ﺷﻜﻠﻪ اﳉﱪي ‪.z = a + ıb‬ﻟﺘﻜﻦ ‪ M‬ﺻﻮرة ‪ z‬و 𝜃 ﻋﻤﺪة ﻟﻪ‪.‬‬
‫ﻟﺪﻳﻨﺎ )أﻧﻈﺮ اﻟﺸﻜﻞ ‪: (1.II‬‬
‫‪ƕ‬‬

‫‪a‬‬ ‫‪a‬‬ ‫‪b‬‬ ‫‪b‬‬


‫= 𝜃 ‪cos‬‬ ‫=‬ ‫و‬ ‫= 𝜃 ‪sin‬‬ ‫=‬
‫‪a +b‬‬ ‫𝜌‬ ‫‪a +b‬‬ ‫𝜌‬

‫ﻣﻊ |‪ 𝜌 = |z‬؛ أي‬


‫‪ũŏ‬‬

‫𝜃 ‪a = 𝜌 cos‬‬ ‫و‬ ‫𝜃 ‪b = 𝜌 sin‬‬


‫ُﻌﲔ ﻫﺘﺎن اﳌﺴﺎوﺗﺎن اﻟﻌﺪدﻳﻦ ‪ a‬و ‪ b‬ﺑﺪﻻﻟﺔ 𝜌 و 𝜃 ﺑﺼﻔﺔ وﺣﻴﺪة أي‬
‫ﺗ ﱢ‬

‫ﻳﻜﻮن ﻋﺪد ﻣﺮﻛﺐ ‪ُ z‬ﻣﻌ ﱠﻴﻨًﺎ إذا ُأﻋﻄﻴﺖ ﻃﻮﻳﻠﺘﻪ 𝜌 و ﻋﻤﺪة ﻟﻪ 𝜃‪.‬‬

‫)𝜃 ‪z = 𝜌 (cos 𝜃 + ı sin‬‬ ‫ﻣﻦ اﻟﻌﺒﺎرﺗﲔ اﻟﺴﺎﺑﻘﺘﲔ ﻟﻠﻌﺪدﻳﻦ ‪ a‬و ‪ ، b‬ﻧﺴﺘﻨﺘﺞ اﻟﻜﺘﺎﺑﺔ اﻟﺘﺎﻟﻴﺔ ﻟﻠﻌﺪد ‪: z‬‬

‫𝟵𝟭‬

‫‪http ://tinyurl.com/Malki1718‬‬ ‫‪0‬‬


‫‪ .2.II‬ﲻﺪة ﻋﺪد ﻣﺮﻛﺐ ﻏﲑ ﻣﻌﺪوم‬
‫‪.‬‬
‫ﺗﺴﻤﻰ اﻟﻜﺘﺎﺑﺔ‬ ‫ﺗﻌﺮﻳﻒ ‪: 8‬‬

‫‪Ŕž Ŧ Ľ‬‬
‫)𝜃 ‪z = 𝜌 (cos 𝜃 + ı sin‬‬
‫]𝜃 ‪.z = [𝜌,‬‬ ‫اﻟﺸﻜﻞ اﳌﺜﻠﺜﻲ ﻟﻠﻌﺪد اﳌﺮﻛﺐ )ﻏﲑ اﳌﻌﺪوم( ‪ z‬و ﻧﻜﺘﺐ‬

‫‪œǃ‬‬
‫ﻟﻴﻜﻦ ‪ z‬اﻟﻌﺪد اﳌﺮﻛﺐ ‪ . z = 1 + ı‬ﻟﺪﻳﻨﺎ ‪ |z| = √2‬و إذا ﻛﺎن 𝜃 ﻋﻤﺪة ﻟﻪ ّ‬
‫ﻓﺈن‬ ‫ﻣﺜﺎﻝ ‪: 9‬‬
‫‪1‬‬ ‫‪√2‬‬ ‫‪1‬‬ ‫‪√2‬‬
‫= 𝜃 ‪cos‬‬ ‫=‬ ‫و‬ ‫= 𝜃 ‪sin‬‬ ‫=‬

‫‪Ŀ‬‬
‫‪√2‬‬ ‫‪2‬‬ ‫‪√2‬‬ ‫‪2‬‬

‫‪II‬‬
‫𝜋‬
‫أي ّ‬
‫أن اﻟﺸﻜﻞ اﳌﺜﻠﺜﻲ ﻟﻠﻌﺪد ‪ 1 + ı‬ﻫﻮ‬ ‫= 𝜃‬ ‫)𝜋‪(mod 2‬‬ ‫ﻣﻨﻪ‬
‫‪4‬‬
‫𝜋‬ ‫𝜋‬ ‫𝜋‬
‫‪1 + ı = √2 cos‬‬ ‫‪+ ı sin‬‬ ‫‪= √2 ,‬‬
‫‪4‬‬ ‫‪4‬‬ ‫‪4‬‬
‫• ‪ u‬إذا ﻛُﺘﺐ اﻟﻌﺪد اﳌﺮﻛﺐ ‪ z‬ﻋﲆ اﻟﺸﻜﻞ )𝜃 ‪ 𝜌 (cos 𝜃 + ı sin‬ﻓﻬﺬا ﻻ ﻳﻌﻨﻲ ﺑﺎﻟﴬورة أﻧﻪ ﰲ ﺷﻜﻠﻪ‬ ‫ﻣﻼﺣﻈﺔ ‪: 9‬‬
‫اﳌﺜﻠﺜﻲ ‪ ،‬ﻓﺎﻟﻌﺪد اﳊﻘﻴﻘﻲ 𝜌 ﳚﺐ أن ﻳﻜﻮن ﻣﻮﺟﺒﺎ )أﻧﻈﺮ اﳌﱪﻫﻨﺔ ‪.(8‬‬

‫إذا ﻛﺎن ‪ z‬و ‪ z‬ﻋﺪدﻳﻦ ﻣﺮﻛﺒﲔ ﺣﻴﺚ ]𝜃 ‪ z = [𝜌,‬و 𝜃 ‪ّ z = 𝜌 ,‬‬


‫ﻓﺈن‬ ‫•‬

‫‪ 𝜃 = 𝜃 + 2k𝜋 , k ∈ ℤ‬و 𝜌 = 𝜌 ⟺ ‪z = z‬‬

‫ﻟﻴﻜﻦ اﻟﻌﺪد اﳌﺮﻛﺐ )‪ z = r (cos t + ı sin t‬ﻣﻊ ‪ r‬و ‪ t‬ﻋﺪدان ﺣﻘﻴﻘﻴﺎن‪ ،‬و ‪ . r ≠ 0‬ﻟﺪﻳﻨﺎ ‪:‬‬ ‫ﻣﺒﺮﻫﻨﺔ ‪: 8‬‬

‫إذا ﻛﺎن ‪ّ r > 0‬‬


‫ﻓﺈن ‪ |z| = r‬و )𝜋‪. arg (z) = t (mod 2‬‬ ‫•‬

‫إذا ﻛﺎن ‪ّ r < 0‬‬


‫ﻓﺈن ‪ |z| = −r‬و )𝜋‪. arg (z) = t + 𝜋 (mod 2‬‬ ‫•‬

‫‪ |z| = (r cos t) + (r sin t) = r‬ﻣﻨﻪ |‪) |z| = |r‬اﻟﻘﻴﻤﺔ اﳌﻄﻠﻘﺔ ﻟﻠﻌﺪد‬ ‫‪cos t + sin t‬‬ ‫ﺍﻟﱪﻫﺎﻥ‪ .‬ﻟﺪﻳﻨﺎ‬
‫‪= r‬‬
‫اﳊﻘﻴﻘﻲ ‪.(r‬‬
‫‪ņőƱŀ Ţ‬‬
‫إذا ﻛﺎن 𝜃 ﻋﻤﺪة ﻟﻠﻌﺪد اﳌﺮﻛﺐ ‪ّ z‬‬
‫ﻓﺈن )‪: z = |r| (cos 𝜃 + ı sin 𝜃) = r (cos t + ı sin t‬‬
‫إذا ﻛﺎن ‪ّ r > 0‬‬
‫ﻓﺈن ‪ cos 𝜃 = cos t‬و ‪ sin 𝜃 = sin t‬أي )𝜋‪. 𝜃 = t (mod 2‬‬ ‫•‬

‫إذا ﻛﺎن ‪ّ r < 0‬‬


‫ﻓﺈن )‪ z = −r (− cos t − ı sin t‬ﻣﻊ ‪ −r > 0‬ﻣﻨﻪ ‪ cos 𝜃 = − cos t‬و ‪ sin 𝜃 = − sin t‬أي‬ ‫•‬
‫)𝜋‪. 𝜃 = t + 𝜋 (mod 2‬‬
‫■‬
‫‪ƕ‬‬

‫ﺍﻹﻧﺘﻘﺎﻝ ﻣﻦ ﺍﻟﺸﻜﻞ ﺍﳉﱪﻱ ﺇﱃ ﺍﻟﺸﻜﻞ ﺍﳌﺜﻠﺜﻲ ﻭ ﺍﻟﻌﻜﺲ‬ ‫‪3.2.II‬‬

‫ﻋﻨﺪ اﻟﻨﻈﺮ إﱃ اﻟﺼﻴﻐﺔ اﳌﺜﻠﺜﻴﺔ ﻟﻌﺪد ﻣﺮﻛﺐ ﻧﺠﺪ ّأﳖﺎ ﻣﻌﻘﺪة ﻣﻘﺎرﻧ ًﺔ ﺑﺎﻟﺼﻴﻐﺔ اﳉﱪﻳﺔ ﻷﳖﺎ ﲢﺘﻮي ﻋﲆ اﳉﺬر اﻟﱰﺑﻴﻌﻲ )ﰲ اﻟﻄﻮﻳﻠﺔ(‬
‫‪ũŏ‬‬

‫و اﻟﺪوال اﳌﺜﻠﺜﻴﺔ )ﰲ اﻟﻌﻤﺪة(‪ ،‬ﻟﻜﻦ ﰲ اﳊﻘﻴﻘﻴﺔ اﻷﻣﺮ اﻟﻮﺣﻴﺪ اﳌﻌ ّﻘﺪ ﻧﻮ ًﻋﺎ ﻣﺎ ﻫﻮ اﻹﻧﺘﻘﺎل ﻣﻦ إﺣﺪى اﻟﺼﻴﻐﺘﲔ إﱃ اﻷﺧﺮى و‬
‫ﻛﻼ ﻣﻨﻬﲈ ﳍﺎ ﻓﻮاﺋﺪﻫﺎ ‪ :‬ﻓﺎﻟﺼﻴﻐﺔ اﳉﱪﻳﺔ أﻛﺜﺮ ﻣﻼءﻣﺔ ﻟﻠﺠﻤﻊ و اﻟﻄﺮح ) أﻧﻈﺮ اﻟﺼﻴﻐﺘﲔ )𝟏( و )𝟐( ﺻﻔﺤﺔ ‪ ،( 4‬أ ّﻣﺎ اﻟﺼﻴﻐﺔ‬
‫اﳌﺜﻠﺜﻴﺔ ﻓﻬﻲ ﻣﻼﺋﻤﺔ ﻟﻠﴬب و اﻟﻘﺴﻤﺔ ﻛﲈ ﺳﻴﺄﰐ ) ﻓﻘﺮة ‪.( 3.II‬‬

‫𝟬𝟮‬

‫‪http ://tinyurl.com/Malki1718‬‬ ‫‪0‬‬


‫‪ .II‬اﻟﺸﲁ اﳌﺜﻠﱻ ﻟﻌﺪد ﻣﺮﻛﺐ‬
‫‪.‬‬
‫إذا ُأﻋﻄﻲ ﻋﺪد ﻣﺮﻛﺐ ‪ z‬ﻋﲆ ﺷﻜﻠﻪ اﳌﺜﻠﺜﻲ ّ‬
‫ﻓﺈن اﳌﺴﺎوﺗﲔ‬ ‫•‬

‫𝜃 ‪a = 𝜌 cos‬‬ ‫و‬ ‫𝜃 ‪b = 𝜌 sin‬‬

‫‪Ŕž Ŧ Ľ‬‬
‫ﺗُﻌ ﱢﻴﻨﺎن ‪ a‬و ‪ b‬و ﺑﺎﻟﺘﺎﱄ اﻟﺸﻜﻞ اﳉﱪي ﻟﻠﻌﺪد ‪.z‬‬

‫‪œǃ‬‬
‫ُﻌﲔ اﻟﻌﺪد 𝜌 ‪،‬‬
‫ﺗ ﱢ‬ ‫=𝜌‬ ‫‪a +b‬‬ ‫إذا ُأﻋﻄﻲ ﻋﺪد ﻣﺮﻛﺐ ‪ z‬ﻋﲆ ﺷﻜﻠﻪ اﳉﱪي ّ‬
‫ﻓﺈن اﳌﺴﺎواة‬ ‫•‬

‫‪b‬‬ ‫‪a‬‬
‫= 𝜃 ‪ sin‬ﺗُﻌ ﱢﻴﻨﺎن اﻟﻌﺪد 𝜃 ‪،‬‬ ‫= 𝜃 ‪ cos‬و‬ ‫و اﳌﺴﺎوﺗﲔ‬

‫‪Ŀ‬‬
‫‪a +b‬‬ ‫‪a +b‬‬
‫‪II‬‬

‫و ﺑﺎﻟﺘﺎﱄ ﻳﺘﻢ ﺗﻌﻴﲔ اﻟﺸﻜﻞ اﳌﺜﻠﺜﻲ ﻟﻠﻌﺪد ‪. z‬‬


‫‪1‬‬ ‫‪3‬‬
‫ﻟﻴﻜﻦ ‪ j‬اﻟﻌﺪد اﳌﺮﻛﺐ √ ‪. j = − + ı‬‬ ‫ﻣﺜﺎﻝ ‪: 10‬‬
‫‪2‬‬ ‫‪2‬‬
‫ﻟﺪﻳﻨﺎ‬
‫‪1‬‬ ‫‪√3‬‬ ‫‪1 3‬‬
‫= ||‪||j‬‬ ‫‪−‬‬ ‫‪+‬‬ ‫=‬ ‫‪+ =1‬‬
‫‪2‬‬ ‫‪2‬‬ ‫‪4 4‬‬

‫‪√3‬‬ ‫‪−1‬‬
‫𝜋‪2‬‬ ‫‪√3‬‬ ‫‪−1‬‬
‫= 𝜃‪.‬‬ ‫= ‪ sin 𝜃 = 2‬أي )𝜋‪(mod 2‬‬ ‫= ‪ cos 𝜃 = 2‬و‬ ‫و إذا ﻛﺎن 𝜃 ﻋﻤﺪ ًة ﻟﻪ ّ‬
‫ﻓﺈن‬
‫‪3‬‬ ‫‪1‬‬ ‫‪2‬‬ ‫‪1‬‬ ‫‪2‬‬
‫و ﻣﻨﻪ اﻟﺸﻜﻞ اﳌﺜﻠﺜﻲ ﻟﻠﻌﺪد ‪: j‬‬
‫𝜋‪2‬‬ ‫𝜋‪2‬‬ ‫𝜋‪2‬‬
‫‪j = cos‬‬ ‫‪+ ı sin‬‬ ‫‪= 1,‬‬
‫‪3‬‬ ‫‪3‬‬ ‫‪3‬‬

‫ﺍﻷﻋﺪﺍﺩ ﺍﳌﺮﻛﺒﺔ ﺍﳌﻜﺘﻮﺑﺔ ﻋﻠﻰ ﺍﻟﺸﻜﻞ ﺍﳌﺜﻠﺜﻲ‬ ‫‪3.II‬‬

‫ﺣﺎﺻﻞ ﺿﺮﺏ ﻋﺪﺩﻳﻦ ﻣﺮﻛﺒﲔ‬ ‫‪1.3.II‬‬


‫‪ņőƱŀ Ţ‬‬
‫‪.z z = 0‬‬ ‫إذا ﻛﺎن أﺣﺪ اﻟﻌﺪدﻳﻦ اﳌﺮﻛﺒﲔ ‪ z‬و ‪ z‬ﻣﻌﺪو ًﻣﺎ ّ‬
‫ﻓﺈن ‪:‬‬
‫أن اﻟﻌﺪدﻳﻦ ‪ z‬و ‪ z‬ﻏﲑ ﻣﻌﺪوﻣﲔ‪.‬‬‫ﻓﻴﲈ ﻳﲇ ﻧﻔﺮض ّ‬

‫ﻟﻴﻜﻦ ‪ z‬و ‪ z‬ﻋﺪدﻳﻦ ﻣﺮﻛﺒﲔ ﻏﲑ ﻣﻌﺪوﻣﲔ ﺣﻴﺚ 𝜃 ‪ z = 𝜌 ,‬و 𝜃 ‪ . z = 𝜌 ,‬ﻟﺪﻳﻨﺎ ‪:‬‬ ‫ﻣﺒﺮﻫﻨﺔ ‪: 9‬‬
‫𝜃 ‪z ⋅ z = z z ,𝜃 +‬‬
‫و ﺑﺘﻌﺒﲑ آﺧﺮ‬
‫|| ‪||z z || = ||z || ⋅ ||z‬‬ ‫و‬ ‫)𝜋‪arg (z z ) = arg (z ) + arg (z ) (mod 2‬‬
‫‪ƕ‬‬

‫ﺍﻟﱪﻫﺎﻥ‪ .‬ﻧﻀﻊ )𝜃 ‪. z z = [𝜌, 𝜃] = 𝜌 (cos 𝜃 + ı sin‬‬


‫‪ũŏ‬‬

‫ﻟﺪﻳﻨﺎ ) 𝜃 ‪ z = 𝜌 (cos 𝜃 + ı sin‬و ) 𝜃 ‪ z = 𝜌 (cos 𝜃 + ı sin‬ﻣﻨﻪ‬


‫) 𝜃 ‪z ⋅ z = 𝜌 (cos 𝜃 + ı sin 𝜃 ) ⋅ 𝜌 (cos 𝜃 + ı sin‬‬
‫) 𝜃 ‪= 𝜌 𝜌 (cos 𝜃 cos 𝜃 − sin 𝜃 sin 𝜃 ) + ı (sin 𝜃 cos 𝜃 + cos 𝜃 sin‬‬
‫)) 𝜃 ‪= 𝜌 𝜌 (cos (𝜃 + 𝜃 ) + ı sin (𝜃 +‬‬

‫■‬ ‫و ﺑﲈ أن ‪ّ 𝜌 > 0 , 𝜌 > 0‬‬


‫ﻓﺈن ‪ 𝜌 𝜌 > 0‬و ﺑﺘﻄﺒﻴﻖ اﳌﱪﻫﻨﺔ ‪ 8‬ﻳﻨﺘﺞ 𝜌 𝜌 = 𝜌 و )𝜋‪. 𝜃 = 𝜃 + 𝜃 (mod 2‬‬

‫𝟭𝟮‬

‫‪http ://tinyurl.com/Malki1718‬‬ ‫‪0‬‬


‫‪ .3.II‬اﻷﻋﺪاد اﳌﺮﻛﺒﺔ اﳌﻜﺘﻮﺑﺔ ﻋﲆ اﻟﺸﲁ اﳌﺜﻠﱻ‬
‫‪.‬‬
‫𝜋‪2‬‬
‫‪ ، j = 1,‬ﻣﻨﻪ‬ ‫رأﻳﻨﺎ ﰲ اﳌﺜﺎل ‪ 9‬أن ّ 𝜋 ‪ 1 + ı = √2 ,‬و ﰲ اﳌﺜﺎل ‪ّ 10‬‬
‫أن‬ ‫ﻣﺜﺎﻝ ‪: 11‬‬
‫‪3‬‬ ‫‪4‬‬

‫‪Ŕž Ŧ Ľ‬‬
‫𝜋‪𝜋 2‬‬ ‫𝜋‪11‬‬
‫‪(1 + ı) × j = √2 × 1 ,‬‬ ‫‪+‬‬ ‫‪= √2 ,‬‬
‫‪4‬‬ ‫‪3‬‬ ‫‪12‬‬

‫‪œǃ‬‬
‫ﻟﻜﻦ ﻣﻦ ﺟﻬﺔ‬
‫‪1‬‬ ‫‪√3‬‬ ‫‪1 + √3 −1 + √3‬‬
‫‪(1 + ı) × j = (1 + ı) − + ı‬‬ ‫‪=−‬‬ ‫‪+‬‬ ‫‪ı‬‬
‫‪2‬‬ ‫‪2‬‬ ‫‪2‬‬ ‫‪2‬‬

‫‪Ŀ‬‬
‫و ﻣﻦ ﺟﻬﺔ أﺧﺮى ) ﻟﻠﺘﺄﻛﺪ (‬

‫‪II‬‬
‫‪1 + √3‬‬ ‫‪−1 + √3‬‬
‫= ||‪||(1 + ı) × j‬‬ ‫‪−‬‬ ‫‪+‬‬ ‫‪= √2‬‬
‫‪2‬‬ ‫‪2‬‬
‫ﻣﻨﻪ‬
‫‪1 + √3‬‬ ‫‪−1 + √3‬‬
‫𝜋‪11‬‬ ‫‪−‬‬ ‫‪−√2 − √6‬‬ ‫𝜋‪11‬‬ ‫‪−√2 + √6‬‬
‫‪cos‬‬ ‫=‬ ‫‪2‬‬ ‫=‬ ‫و‬ ‫‪sin‬‬ ‫=‬ ‫‪2‬‬ ‫=‬
‫‪12‬‬ ‫‪√2‬‬ ‫‪4‬‬ ‫‪12‬‬ ‫‪√2‬‬ ‫‪4‬‬

‫𝜋‬ ‫𝜋‪11‬‬
‫ﻓﺈن ‪: 1‬‬
‫ّ‬ ‫‪=𝜋−‬‬ ‫⧏ ﺑﲈ ّ‬
‫أن‬ ‫ﻣﻼﺣﻈﺔ ‪: 10‬‬
‫‪12‬‬ ‫‪12‬‬
‫𝜋‬ ‫‪√6 + √2‬‬ ‫𝜋‬ ‫‪√6 − √2‬‬
‫‪cos‬‬ ‫=‬ ‫و‬ ‫‪sin‬‬ ‫=‬
‫‪12‬‬ ‫‪4‬‬ ‫‪12‬‬ ‫‪4‬‬
‫⧐‬
‫𝛼 ‪1 + cos‬‬ ‫𝜋‬ ‫ﻟﻨ ِ‬
‫= 𝛼 ‪: cos‬‬ ‫ﻟﻠﻌﺪد 𝛼 ﰲ اﳌﺘﻄﺎﺑﻘﺔ‬ ‫ُﻌﻂ اﻟﻘﻴﻤﺔ‬ ‫ﻣﻼﺣﻈﺔ ‪: 11‬‬
‫‪2‬‬ ‫‪12‬‬
‫𝜋‬ ‫‪√3‬‬
‫𝜋‬ ‫‪1 + cos‬‬ ‫‪1+‬‬
‫‪cos‬‬ ‫=‬ ‫= ‪6‬‬ ‫‪2 = 2 + √3‬‬
‫‪12‬‬ ‫‪2‬‬ ‫‪2‬‬ ‫‪4‬‬
‫𝜋‬ ‫‪2 + √3‬‬ ‫‪2 + √3‬‬
‫)ﻷن 𝜋 < 𝜋 < ‪ّ ( 0‬‬ ‫𝜋‬
‫‪ņőƱŀ Ţ‬‬
‫‪. cos‬‬ ‫=‬ ‫=‬ ‫ﻓﺈن‬ ‫ّ‬ ‫‪cos‬‬ ‫أن ‪> 0‬‬‫و ﺑﲈ ّ‬
‫‪12‬‬ ‫‪4‬‬ ‫‪2‬‬ ‫‪12 2‬‬ ‫‪12‬‬
‫ﻫﻞ ﻫﺬا ﺗﻨﺎﻗﺾ؟ اﳉﻮاب ‪ :‬ﻻ ‪ّ ...‬‬
‫ﻷن‬

‫‪2 + √3 2 2 + √3‬‬ ‫‪8 + 4 √3‬‬ ‫‪2 + 6 + 2√12‬‬ ‫‪√2 + √6‬‬ ‫‪√2 + √6‬‬
‫=‬ ‫=‬ ‫=‬ ‫=‬ ‫=‬
‫‪2‬‬ ‫‪4‬‬ ‫‪4‬‬ ‫‪4‬‬ ‫‪4‬‬ ‫‪4‬‬

‫𝜋‬ ‫‪2 − √3 √6 − √2‬‬


‫‪. sin‬‬ ‫=‬ ‫=‬ ‫و ﺑﻨﻔﺲ اﻟﻄﺮﻳﻘﺔ ﻧﺜﺒﺖ ّ‬
‫أن‬
‫‪12‬‬ ‫‪2‬‬ ‫‪4‬‬
‫ﻳﻤﻜﻦ ﺗﻌﻤﻴﻢ ﻧﺘﻴﺠﺔ اﳌﱪﻫﻨﺔ ‪ 9‬إﱃ ‪ n ≥ 2‬ﻋﺪد ًا ﻣﻦ اﻷﻋﺪاد اﳌﺮﻛﺒﺔ ‪:‬‬
‫‪ƕ‬‬

‫ﻣﺒﺮﻫﻨﺔ ‪ : 10‬ﻟﺘﻜﻦ ‪ z ، ⋯ ، z ، z‬أﻋﺪا ًدا ﻣﺮﻛﺒﺔ ﻏﲑ ﻣﻌﺪوﻣﺔ ﺑﺤﻴﺚ 𝜃 ‪، ⋯ ، z = 𝜌 , 𝜃 ، z = 𝜌 ,‬‬


‫𝜃 ‪ . z = 𝜌 ,‬ﻟﺪﻳﻨﺎ ‪:‬‬
‫‪ũŏ‬‬

‫𝜃 ‪z ⋅ z ⋅ ⋯ ⋅ z = z z ⋯z ,𝜃 + 𝜃 + ⋯ +‬‬
‫و ﺑﺘﻌﺒﲑ آﺧﺮ‬
‫|| ‪||z z ⋯ z || = ||z || ⋅ ||z || ⋅ ⋯ ⋅ ||z‬‬ ‫)𝜋‪ arg (z z ⋯ z ) = arg (z ) + arg (z ) + ⋯ + arg (z ) (mod 2‬و‬

‫‪. sin‬‬ ‫‪−‬‬ ‫‪= sin‬‬ ‫‪ cos‬و‬ ‫‪−‬‬ ‫‪= − cos‬‬ ‫‪1‬ﻧُﺬﻛﱢﺮ ّ‬
‫أن‬

‫𝟮𝟮‬

‫‪http ://tinyurl.com/Malki1718‬‬ ‫‪0‬‬


‫‪ .II‬اﻟﺸﲁ اﳌﺜﻠﱻ ﻟﻌﺪد ﻣﺮﻛﺐ‬
‫‪.‬‬
‫ﺍﻟﱪﻫﺎﻥ‪ .‬ﺑﺎﻟﱰاﺟﻊ ﻋﲆ اﻟﻌﺪد اﻟﻄﺒﻴﻌﻲ ‪: n‬‬

‫‪Ŕž Ŧ Ľ‬‬
‫‪ : n = 2‬و ﻫﻮ ﻧﺺ اﳌﱪﻫﻨﺔ ‪. 9‬‬ ‫•‬

‫إذا ﻛﺎﻧﺖ اﻟﻨﺘﻴﺠﺔ ﺻﺤﻴﺤﺔ ﻟﻜﻞ ﻋﺪد ﻃﺒﻴﻌﻲ أﺻﻐﺮ ﻣﻦ ‪ّ n‬‬


‫ﻓﺈن ‪:‬‬ ‫•‬

‫‪œǃ‬‬
‫‪||z z ⋯ z‬‬ ‫‪−‬‬ ‫‪z || = ||(z‬‬ ‫‪z ⋯z‬‬ ‫‪−‬‬ ‫|| ‪) ⋅ z‬‬
‫‪= ||(z‬‬ ‫‪z ⋯z −‬‬ ‫|| ‪)|| ⋅ ||z‬‬ ‫)ﺣﺴﺐ اﳌﱪﻫﻨﺔ ‪(9‬‬
‫‪= ||z‬‬ ‫‪|| ⋅ ||z || ⋅ ⋯ ⋅ ||z‬‬ ‫|| ‪|| ⋅ ||z‬‬ ‫)ﺑﺘﻄﺒﻴﻖ ﻓﺮﺿﻴﺔ اﻟﱰاﺟﻊ(‬

‫‪Ŀ‬‬
‫‪−‬‬
‫‪II‬‬

‫‪= ||z || ⋅ ||z || ⋅ ⋯ ⋅ ||z‬‬ ‫‪−‬‬


‫|| ‪|| ⋅ ||z‬‬

‫ﺑﺎﳌِﺜﻞ ‪:‬‬

‫‪arg (z z ⋯ z‬‬ ‫‪−‬‬ ‫‪z ) = arg‬‬ ‫‪z z ⋯z‬‬ ‫‪−‬‬ ‫‪⋅z‬‬


‫‪= arg (z z ⋯ z‬‬ ‫‪−‬‬ ‫)𝜋‪) + arg (z ) (mod 2‬‬
‫)ﺣﺴﺐ اﳌﱪﻫﻨﺔ ‪(9‬‬
‫=‬ ‫‪arg (z ) + arg (z ) + ⋯ ⋅ arg (z‬‬ ‫‪−‬‬ ‫)𝜋‪) (mod 2𝜋) + arg (z ) (mod 2‬‬
‫)ﺑﺘﻄﺒﻴﻖ ﻓﺮﺿﻴﺔ اﻟﱰاﺟﻊ(‬
‫‪= arg (z ) + arg (z ) + ⋯ + arg (z‬‬ ‫‪−‬‬ ‫)𝜋‪) + arg (z ) (mod 2‬‬

‫■‬
‫ّ‬
‫ﻓﺈن‬ ‫]𝜃 ‪z = z = ⋯ = z = z = [𝜌 ,‬‬ ‫ﺣﺎﻟﺔ ﺧﺎﺻﺔ ‪ :‬إذا ﻛﺎن‬
‫]𝜃‪z = [𝜌 , n‬‬
‫أي‬
‫|‪||z || = |z‬‬ ‫و‬ ‫)𝜋‪arg (z ) = n arg (z) (mod 2‬‬
‫‪ņőƱŀ Ţ‬‬
‫ﺣﺎﺻﻞ ﻗﺴﻤﺔ ﻋﺪﺩﻳﻦ ﻣﺮﻛﺒﲔ ﻏﲑ ﻣﻌﺪﻭﻣﲔ‬ ‫‪2.3.II‬‬
‫‪z‬‬
‫‪.‬‬ ‫‪=0‬‬ ‫إذا ﻛﺎن اﻟﻌﺪد اﳌﺮﻛﺐ ‪ z‬ﻣﻌﺪو ًﻣﺎ و ﻛﺎن ‪ّ z ≠ 0‬‬
‫ﻓﺈن ‪:‬‬
‫‪z‬‬
‫ﻓﻴﲈ ﻳﲇ ﻧﻔﺮض ّ‬
‫أن اﻟﻌﺪدﻳﻦ ‪ z‬و ‪ z‬ﻏﲑ ﻣﻌﺪوﻣﲔ‪.‬‬

‫ﻟﻴﻜﻦ ‪ z‬و ‪ z‬ﻋﺪدﻳﻦ ﻣﺮﻛﺒﲔ ﻏﲑ ﻣﻌﺪوﻣﲔ ﺣﻴﺚ 𝜃 ‪ z = 𝜌 ,‬و 𝜃 ‪ . z = 𝜌 ,‬ﻟﺪﻳﻨﺎ ‪:‬‬ ‫ﻣﺒﺮﻫﻨﺔ ‪: 11‬‬

‫‪z‬‬ ‫𝜌‬
‫‪ƕ‬‬

‫=‬ ‫𝜃 ‪,𝜃 −‬‬


‫‪z‬‬ ‫𝜌‬
‫و ﺑﺘﻌﺒﲑ آﺧﺮ‬
‫|| ‪| z | ||z‬‬ ‫‪z‬‬
‫‪ũŏ‬‬

‫=| |‬ ‫و‬ ‫‪arg‬‬ ‫)𝜋‪= arg (z ) − arg (z ) (mod 2‬‬


‫|| ‪|| z || ||z‬‬ ‫‪z‬‬

‫‪z‬‬
‫ﺍﻟﱪﻫﺎﻥ‪ .‬ﻧﻀﻊ )𝜃 ‪= [𝜌, 𝜃] = 𝜌 (cos 𝜃 + ı sin‬‬
‫‪.‬‬
‫‪z‬‬
‫ﻟﺪﻳﻨﺎ ) 𝜃 ‪ z = 𝜌 (cos 𝜃 + ı sin‬و ) 𝜃 ‪ z = 𝜌 (cos 𝜃 + ı sin‬ﻣﻨﻪ‬

‫𝟯𝟮‬

‫‪http ://tinyurl.com/Malki1718‬‬ ‫‪0‬‬


‫ اﻷﻋﺪاد اﳌﺮﻛﺒﺔ اﳌﻜﺘﻮﺑﺔ ﻋﲆ اﻟﺸﲁ اﳌﺜﻠﱻ‬.3.II
.

z 𝜌 (cos 𝜃 + ı sin 𝜃 )

Ŕž Ŧ Ľ
=
z 𝜌 (cos 𝜃 + ı sin 𝜃 )
𝜌 (cos 𝜃 + ı sin 𝜃 ) (cos 𝜃 − ı sin 𝜃 )

œǃ
= ⋅
𝜌 (cos 𝜃 + ı sin 𝜃 ) (cos 𝜃 − ı sin 𝜃 )
𝜌 (cos 𝜃 + ı sin 𝜃 ) (cos (−𝜃 ) + ı sin (−𝜃 ))
= ⋅
𝜌 cos 𝜃 + sin 𝜃
𝜌

Ŀ
= (cos (𝜃 − 𝜃 ) + ı sin (𝜃 − 𝜃 )) (9 ‫)ﺣﺴﺐ اﳌﱪﻫﻨﺔ‬
𝜌

II
𝜌
: 20 ‫ ﺻﻔﺤﺔ‬8 ‫ﻓﺤﺴﺐ اﳌﱪﻫﻨﺔ‬ >0 ّ ‫و ﺑﲈ‬
‫أن‬
𝜌
𝜌
𝜌= ‫و‬ 𝜃 = 𝜃 − 𝜃 (mod 2𝜋)
𝜌

ّ z = z = [𝜌 , 𝜃] ‫ و‬z = 1 = [1 , 0] ‫ إذا ﻛﺎن‬: ‫ﺣﺎﻟﺔ ﺧﺎﺻﺔ‬
: ‫ﻓﺈن‬
1 1
= , −𝜃
z 𝜌

‫و ﺑﺘﻌﺒﲑ آﺧﺮ‬
|1| 1 1
| |= ‫و‬ arg = − arg (z) (mod 2𝜋)
|| z || |z| z

ّ z = [𝜌 , 𝜃] ‫ ﻧُﱪﻫﻦ ﺑﺴﻬﻮﻟﺔ ) ﺑﺎﻟﱰاﺟﻊ ( أﻧّﻪ إذا ﻛﺎن‬، 10 ‫و ﺑﺎﺗﱢﺒﺎع ﻧﻔﺲ ﻃﺮﻳﻘﺔ إﺛﺒﺎت اﳌﱪﻫﻨﺔ‬
: ‫ﻓﺈن‬
1 1
= , −n𝜃
z 𝜌

. z = −√3 − ı ‫ و‬z = ı ‫ﻟﻴﻜﻦ‬ : 12 ‫ﻣﺜﺎﻝ‬


ņőƱŀ Ţ
√3 1 −5𝜋 −5𝜋 𝜋 𝜋
z =2⋅ − − ı = 2 ⋅ cos + ı sin ‫ و‬z = 1 ⋅ cos + ı sin ‫ﻟﺪﻳﻨﺎ‬
2 2 6 6 2 2
‫ﻣﻨﻪ‬
| z | ||z || 1
| |= =
|| z || ||z || 2
‫و‬
z 𝜋 5𝜋 4𝜋
arg = arg (z ) − arg (z ) (mod 2𝜋) = − − (mod 2𝜋) = (mod 2𝜋)
z 2 6 3
‫أي‬
ƕ

z 1 4𝜋 4𝜋 1 1 √3 1 √3
= cos + ı sin = − −ı =− −ı
z 2 3 3 2 2 2 4 4
ũŏ

𝟮𝟰

http ://tinyurl.com/Malki1718 0
‫‪ .II‬اﻟﺸﲁ اﳌﺜﻠﱻ ﻟﻌﺪد ﻣﺮﻛﺐ‬
‫‪.‬‬
‫ﺩﺳﺘﻮﺭ ﻣﻮﺍﻓﺮ‬ ‫‪4.II‬‬

‫‪Ŕž Ŧ Ľ‬‬
‫ﻟﻴﻜﻦ ‪ z‬ﻋﺪ ًدا ﻣﺮﻛ ًﺒﺎ ﻃﻮﻳﻠﺘﻪ ‪ 1‬و 𝜃 ﻋﻤﺪة ﻟﻪ ‪. z = [1 , 𝜃] :‬‬
‫ﻟﻴﻜﻦ ‪ n‬ﻋﺪ ًدا ﻃﺒﻴﻌ ًﻴﺎ ‪ .‬ﺣﺴﺐ اﳌﱪﻫﻨﺔ ‪) 10‬اﳊﺎﻟﺔ اﳋﺎﺻﺔ( ‪:‬‬

‫‪œǃ‬‬
‫]𝜃‪z = 1 , n𝜃 = [1 , n‬‬
‫ﺑﻤﻌﻨﻰ‬
‫𝜃 ‪cos 𝜃 + ı sin‬‬ ‫)𝜃‪= cos (n𝜃) + ı sin (n‬‬

‫‪Ŀ‬‬
‫‪−‬‬
‫‪1‬‬
‫‪II‬‬

‫= ‪ z‬و ﺑﻮﺿﻊ ‪ n = −n‬ﻳﻜﻮن ‪ n > 0‬و‬ ‫ﺻﺤﻴﺤﺎ ﺳﺎﻟ ًﺒﺎ ﲤﺎ ًﻣﺎ ّ‬


‫ﻓﺈن‬ ‫ً‬ ‫و إذا ﻛﺎن ‪ n‬ﻋﺪ ًدا‬
‫‪z‬‬
‫‪′‬‬ ‫‪′‬‬
‫‪1‬‬ ‫‪1‬‬
‫= ‪z‬‬ ‫=‬ ‫]𝜃‪, −n 𝜃 = [1 , n‬‬
‫‪z‬‬ ‫‪1‬‬

‫ﻣﻨﻪ‬ ‫)إﺳﺘﻌﻤﻠﻨﺎ اﳊﺎﻟﺔ اﳋﺎﺻﺔ ﻟﻠﻤﱪﻫﻨﺔ ‪.( 11‬‬


‫𝜃 ‪cos 𝜃 + ı sin‬‬ ‫)𝜃‪= cos (n𝜃) + ı sin (n‬‬
‫إذن ‪:‬‬

‫ﻣﺒﺮﻫﻨﺔ ‪: 12‬‬

‫‪∀n ∈ ℤ :‬‬ ‫𝜃 ‪cos 𝜃 + ı sin‬‬ ‫)𝜃‪= cos (n𝜃) + ı sin (n‬‬

‫ﻳﻤﻜﻦ إﻋﻄﺎء إﺛﺒﺎت آﺧﺮ ﳍﺬه اﳌﱪﻫﻨﺔ و ذﻟﻚ ﺑﺎﺳﺘﻌﲈل اﻟﱪﻫﺎن ﺑﺎﻟﱰاﺟﻊ ‪:‬‬

‫)𝜃 × ‪z = 1 = cos 0 + ı sin 0 = cos (0 × 𝜃) + ı sin (0‬‬


‫)𝜃 × ‪z = z = cos 𝜃 + ı sin 𝜃 = cos (1 × 𝜃) + ı sin (1‬‬

‫= ‪z‬‬ ‫𝜃 ‪cos 𝜃 + ı sin‬‬ ‫)𝜃‪= cos 𝜃 − sin 𝜃 + 2ı cos 𝜃 sin 𝜃 = cos (2𝜃) + ı sin (2‬‬
‫‪ņőƱŀ Ţ‬‬
‫ﻓﺎﳋﺎﺻﻴﺔ ﺻﺤﻴﺤﺔ ﻣﻦ أﺟﻞ ‪ n = 1 ، n = 0‬و ‪. n = 2‬‬ ‫•‬

‫ﻧﻔﺮض ّ‬
‫أن اﳋﺎﺻﻴﺔ ﺻﺤﻴﺤﺔ ﻟﻜﻞ ﻋﺪد ﻃﺒﻴﻌﻲ أﺻﻐﺮ ﻣﻦ أو ﻳﺴﺎوي ‪ . n‬ﻟﺪﻳﻨﺎ ‪:‬‬ ‫•‬

‫‪z‬‬ ‫‪+‬‬ ‫)𝜃 ‪= z ⋅ z = (cos (n𝜃) + ı sin (n𝜃)) (cos 𝜃 + ı sin‬‬


‫)𝜃 ‪= (cos (n𝜃) cos 𝜃 − sin (n𝜃) sin 𝜃) + ı (cos (n𝜃) sin 𝜃 + sin (n𝜃) cos‬‬
‫)𝜃 )‪= cos ((n + 1) 𝜃) + ı sin ((n + 1‬‬

‫أي ّ‬
‫أن اﳋﺎﺻﻴﺔ ﺻﺤﻴﺤﺔ ﻟﻜﻞ ﻋﺪد ﻃﺒﻴﻌﻲ ‪. n‬‬
‫‪ƕ‬‬

‫ﻟﻨﱪﻫﻦ ّ‬
‫أن ﻫﺬه اﻟﻨﺘﻴﺠﺔ ﺗﺒﻘﻰ ﺻﺤﻴﺤﺔ إذا ﻛﺎن ‪ n‬ﺳﺎﻟ ًﺒﺎ ‪:‬‬ ‫•‬

‫ﻧﻀﻊ ‪ n = −n‬و ﻣﻦ أﺟﻞ ﻛﻞ ﻋﺪد ﻃﺒﻴﻌﻲ ‪: n > 0‬‬


‫‪ũŏ‬‬

‫‪−‬‬ ‫‪′‬‬
‫‪1‬‬ ‫‪1‬‬
‫= ‪z‬‬ ‫𝜃 ‪cos 𝜃 + ı sin‬‬ ‫=‬ ‫=‬
‫𝜃 ‪cos 𝜃 + ı sin‬‬ ‫𝜃 ‪cos 𝜃 + ı sin‬‬
‫‪′‬‬
‫))𝜃‪= (cos (−𝜃) + ı sin (−‬‬
‫𝜃 ‪= cos −n 𝜃 + ı sin −n‬‬
‫)𝜃‪= cos (n𝜃) + ı sin (n‬‬

‫𝟱𝟮‬

‫‪http ://tinyurl.com/Malki1718‬‬ ‫‪0‬‬


‫‪ .5.II‬اﳉﺬور اﻟﻨﻮﻧﻴﺔ ﻟﻌﺪد ﻣﺮﻛﺐ‬
‫‪.‬‬
‫■‬
‫‪1‬‬ ‫‪3‬‬ ‫𝜋‬
‫ﻣﻦ أﺟﻞ = 𝜃 ‪ cos 𝜃 = √ ،‬و = 𝜃 ‪ sin‬ﻣﻨﻪ ‪:‬‬ ‫ﻣﺜﺎﻝ ‪: 13‬‬

‫‪Ŕž Ŧ Ľ‬‬
‫‪2‬‬ ‫‪2‬‬ ‫‪6‬‬
‫‪√3 1‬‬ ‫𝜋‬ ‫𝜋‬ ‫𝜋‬ ‫𝜋‬
‫‪+ ı‬‬ ‫⋅ ‪= cos (3𝜃) + ı sin (3𝜃) = cos 3‬‬ ‫⋅ ‪+ ı sin 3‬‬ ‫‪= cos + ı sin = ı‬‬
‫‪2‬‬ ‫‪2‬‬ ‫‪6‬‬ ‫‪6‬‬ ‫‪2‬‬ ‫‪2‬‬

‫‪œǃ‬‬
‫ﺍﳉﺬﻭﺭ ﺍﻟﻨﻮﻧﻴﺔ ﻟﻌﺪﺩ ﻣﺮﻛﺐ‬ ‫‪5.II‬‬

‫‪Ŀ‬‬
‫‪II‬‬
‫ﻟﻴﻜﻦ ‪ w‬ﻋﺪ ًدا ﻣﺮﻛ ًﺒﺎ و ‪ n‬ﻋﺪ ًدا ﻃﺒﻴﻌ ًﻴﺎ ﺣﻴﺚ ‪. n > 1‬‬

‫‪.z =w‬‬ ‫ﺬرا ﻧُﻮﻧِ ًﻴﺎ ‪ 2‬ﻟﻠﻌﺪد ‪ w‬ﻛﻞ ﻋﺪد ﻣﺮﻛﺐ ‪ z‬ﳛﻘﻖ‬
‫ﻧﺴﻤﻲ ِﺟ ً‬ ‫ﺗﻌﺮﻳﻒ ‪: 9‬‬

‫ﰲ اﳌﺜﺎل ‪ 13‬اﻟﺴﺎﺑﻖ وﺟﺪﻧﺎ ّ‬


‫أن‬ ‫ﻣﺜﺎﻝ ‪: 14‬‬

‫‪√3 1‬‬
‫‪+ ı‬‬ ‫‪=ı‬‬
‫‪2‬‬ ‫‪2‬‬

‫‪3 1‬‬
‫أي ‪ z = √ + ı‬ﺟﺬر ﺗﻜﻌﻴﺒﻲ ﻟﻠﻮﺣﺪة اﻟﺘﺨﻴﻠﻴﺔ ‪.w = ı‬‬
‫‪2‬‬ ‫‪2‬‬

‫ﺟﺬرا ﻧﻮﻧ ًﻴﺎ وﺣﻴﺪً ا ﻫﻮ ﻧﻔﺴﻪ‪.‬‬ ‫ﻧﻌﻠﻢ ّ‬


‫أن اﳌﻌﺎدﻟﺔ ‪ z = 0‬ﺗﻘﺒﻞ ﺣﻼ وﺣﻴﺪً ا ‪ ، z = 0‬إذن اﻟﻌﺪد ‪ 0‬ﻳﻘﺒﻞ ً‬ ‫•‬

‫ﻟﻴﻜﻦ ‪ w‬ﻋﺪ ًدا ﻣﺮﻛ ًﺒﺎ ﻏﲑ ﻣﻌﺪوم ﻃﻮﻳﻠﺘﻪ ‪ r‬و 𝛼 ﻋﻤﺪة ﻟﻪ‪.‬‬ ‫•‬

‫ﻟﻨﺒﺤﺚ ﻋﻦ اﻷﻋﺪاد اﳌﺮﻛﺒﺔ ‪ z‬اﻟﺘﻲ ﲢﻘﻖ ‪ . z = w‬ﻧﻀﻊ )𝜃 ‪ z = 𝜌 (cos 𝜃 + ı sin‬ﺣﻴﺚ ‪ . 𝜌 > 0‬ﻟﺪﻳﻨﺎ ‪:‬‬

‫‪z = w ⟺ 𝜌 (cos 𝜃 + ı sin 𝜃) = w‬‬


‫‪ņőƱŀ Ţ‬‬
‫)𝛼 ‪⟺ 𝜌 (cos (n𝜃) + ı sin (n𝜃)) = w = r (cos 𝛼 + ı sin‬‬
‫‪𝜌 =r‬‬
‫⟺‬
‫‪n𝜃 = 𝛼 + 2k𝜋 ,‬‬ ‫‪k∈ℤ‬‬
‫‪⎧ 𝜌 = √r‬‬
‫⟺‬
‫‪⎨ 𝜃 = 𝛼 + 2k𝜋 ,‬‬ ‫‪k∈ℤ‬‬
‫⎩‬ ‫‪n‬‬ ‫‪n‬‬

‫ﻧﻌﻠﻢ أﻧﻪ إذا ﻛﺎن ‪ k‬ﻋﺪدا ﺻﺤﻴﺤﺎ ﻓﺈﻧﻪ ُﻳﻜﺘﺐ ﻋﲆ اﻟﺸﻜﻞ ‪ k = nq + s‬ﺣﻴﺚ ‪ 0 ≤ s < n‬و ‪) q ∈ ℤ‬اﻟﻘﺴﻤﺔ‬
‫‪ƕ‬‬

‫اﻹﻗﻠﻴﺪﻳﺔ(‪ ،‬أي أﻧﻪ ُﻳﻜﺘﺐ ﻋﲆ أﺣﺪاﻷﺷﻜﺎل اﻟﺘﺎﻟﻴﺔ ‪.nq , nq + 1 , nq + 2 , ⋯ , nq + (n − 1) :‬‬


‫𝛼‬
‫=𝜃 ‪.‬‬ ‫𝜋‪+ 2q‬‬ ‫ﻣﻦ أﺟﻞ ‪ k = nq‬ﻳﻜﻮن‬ ‫•‬
‫‪n‬‬
‫‪ũŏ‬‬

‫𝜋‪𝛼 2‬‬
‫=𝜃 ‪.‬‬ ‫‪+‬‬ ‫𝜋‪+ 2q‬‬ ‫ﻣﻦ أﺟﻞ ‪ k = nq + 1‬ﻳﻜﻮن‬ ‫•‬
‫‪n‬‬ ‫‪n‬‬
‫⋮‬
‫• ⋮‬
‫⋮‬
‫𝜋)‪𝛼 2(n − 1‬‬
‫=𝜃 ‪.‬‬ ‫‪+‬‬ ‫𝜋‪+ 2q‬‬ ‫ﻣﻦ أﺟﻞ )‪ k = nq + (n − 1‬ﻳﻜﻮن‬ ‫•‬
‫‪n‬‬ ‫‪n‬‬

‫𝟲𝟮‬

‫‪http ://tinyurl.com/Malki1718‬‬ ‫‪0‬‬


‫‪ .II‬اﻟﺸﲁ اﳌﺜﻠﱻ ﻟﻌﺪد ﻣﺮﻛﺐ‬
‫‪.‬‬

‫‪Ŕž Ŧ Ľ‬‬
‫‪n=3‬‬ ‫‪n=4‬‬ ‫‪n=5‬‬ ‫‪n=6‬‬ ‫‪n=7‬‬

‫‪œǃ‬‬
‫‪n=8‬‬ ‫‪n=9‬‬ ‫‪n = 10‬‬ ‫‪n = 11‬‬ ‫‪n = 12‬‬

‫‪Ŀ‬‬
‫‪II‬‬

‫‪n = 13‬‬ ‫‪n = 16‬‬ ‫‪n = 19‬‬ ‫‪n = 22‬‬ ‫‪n = 25‬‬

‫ﺷﻜﻞ ‪3.II‬‬

‫إذن اﻟﻌﺪد ‪ w‬ﻳﻘﺒﻞ ‪ n‬ﺟﺬرا ﻧﻮﻧﻴﺎ و ﻫﻲ اﻷﻋﺪاد ‪ z‬ﺣﻴﺚ }‪ s ∈ {0, 1, ⋅, n − 1‬و ‪:‬‬

‫𝜋‪𝛼 2s‬‬ ‫𝜋‪𝛼 2s‬‬


‫‪z = √r cos‬‬ ‫‪+‬‬ ‫‪+ ı sin‬‬ ‫‪+‬‬
‫‪n‬‬ ‫‪n‬‬ ‫‪n‬‬ ‫‪n‬‬

‫ﻛﻞ ﻋﺪد ﻣﺮﻛﺐ ﻏﲑ ﻣﻌﺪوم ﻳﻘﺒﻞ ‪ n‬ﺟﺬرا ﻧﻮﻧﻴﺎ‬

‫أن اﳉﺬور اﻟﻨﻮﻧﻴﺔ ﳍﺎ ﻧﻔﺲ اﻟﻄﻮﻳﻠﺔ ‪ √r‬ﻓﻬﻲ ﺗﻘﻊ ﻋﲆ داﺋﺮة ‪ 3‬ﻧﺼﻒ ﻗﻄﺮﻫﺎ ‪ √r‬و ﻣﺮﻛﺰﻫﺎ ‪ ، O‬و ﺑﲈ ّ‬
‫أن اﻟﻔﺮق‬ ‫ﺑﲈ ّ‬ ‫•‬

‫ﺑﲔ ﻋﻤﺪﰐ ﺟﺬرﻳﻦ ﻣﺘﺘﺎﺑﻌﲔ ‪ z‬و ‪ z +‬ﻳﺴﺎوي 𝜋‪ّ 2‬‬


‫ﻓﺈن ﻫﺬه اﳉﺬور ﻣﻮزﻋﺔ ﺑﺼﻔﺔ ﻣﻨﺘﻈﻤﺔ ﻋﲆ ﻫﺬه اﻟﺪاﺋﺮة أي ّ‬
‫أن‬
‫‪n‬‬
‫‪ņőƱŀ Ţ‬‬
‫اﳌﺴﺎﻓﺔ ﺑﲔ ﺟﺬرﻳﻦ ﻣﺘﺘﺎﻟﻴﲔ ﻳﺴﺎوي ﻋﺪ ًدا ﺛﺎﺑﺘﺎ ‪:‬‬

‫ﻓﺈن ﺻﻮرﻫﺎ ﺗُﺸﻜﱢﻞ ﻣﻀﻠّﻌ‪‬ﺎ ﻣﻨﺘﻈﻤ‪‬ﺎ‬


‫ّ‬ ‫إذا ﻣﺜﻠﻨﺎ ﻫﺬه اﳉﺬور ﰲ اﳌﺴﺘﻮي اﳌﻨﺴﻮب إﱃ ﻣﻌﻠﻢ ﻣﺘﻌﺎﻣﺪ و ﻣﺘﺠﺎﻧﺲ ⃗ ‪O, i,⃗ j‬‬
‫ﻣﺮﺳﻮ ًﻣﺎ ﻋﲆ داﺋﺮة ﻣﺮﻛﺰﻫﺎ اﳌﺒﺪأ ‪ O‬و ﻧﺼﻒ ﻗﻄﺮﻫﺎ ‪. √r‬‬

‫𝜋‪2k‬‬
‫‪ z = 1,‬و ﻧﻌﺘﱪ اﻟﻨﻘﻂ ‪ M‬اﻟﺘﻲ ﻟﻮاﺣﻘﻬﺎ اﻷﻋﺪاد ‪ z‬ﻷﺟﻞ‬ ‫ﺗﻄﺒﻴﻖ ‪ : 1‬ﻟﻴﻜﻦ ‪ n ≥ 3‬ﻋﺪد ًا ﻃﺒﻴﻌ ًﻴﺎ‪ .‬ﻧﻀﻊ‬
‫‪n‬‬
‫‪ƕ‬‬

‫‪ M‬ﻫﻲ رؤوس ﻣﻀ ّﻠﻊ ﻣﻨﺘﻈﻢ ﻣﺮﺳﻮم داﺧﻞ اﻟﺪاﺋﺮة اﳌﺜﻠﺜﻴﺔ‪.‬‬ ‫‪−‬‬ ‫أن اﻟﻨﻘﻂ ‪، ⋅ ، M ، M‬‬ ‫‪ . 0 ≤ k ≤ n − 1‬ﻧﻌﻠﻢ ّ‬
‫ﺛﻢ أوﺟﺪ ﳖﺎﻳﺔ ﻛﻼ ﻣﻦ ﻫﺬا اﳌﺤﻴﻂ و ﻫﺬه اﳌﺴﺎﺣﺔ ﻋﻨﺪﻣﺎ‬
‫أوﺟﺪ ﳏﻴﻂ ﻫﺬا اﳌﻀﻠﻊ اﳌﻨﺘﻈﻢ و ﻣﺴﺎﺣﺔ اﳊ ّﻴﺰ اﳌﺴﺘﻮي اﻟﺬي ﳛﺪده ّ‬
‫∞‪. n → +‬‬
‫‪ũŏ‬‬

‫و ﻧﺼﻒ ﻗﻄﺮﻫﺎ‬ ‫ﺗﻘﻊ ﻋﲆ اﻟﺪاﺋﺮة اﻟﺘﻲ ﻣﺮﻛﺰﻫﺎ‬ ‫أن ﻫﺬا ﻳﻌﻨﻲ ّ‬


‫أن اﻟﻨﻘﻂ اﻟﺘﻲ ﻟﻮاﺣﻘﻬﺎ‬ ‫|| و ﺳﻨﺮى ﰲ اﻟﺒﺎب اﻟﺮاﺑﻊ ّ‬ ‫= || ‪−‬‬ ‫‪ّ 3‬‬
‫ﻷن‬
‫‪.‬‬

‫𝟳𝟮‬

‫‪http ://tinyurl.com/Malki1718‬‬ ‫‪0‬‬


‫‪ .5.II‬اﳉﺬور اﻟﻨﻮﻧﻴﺔ ﻟﻌﺪد ﻣﺮﻛﺐ‬
‫‪.‬‬
‫‪M1‬‬
‫‪sin 2π‬‬

‫‪Ŕž Ŧ Ľ‬‬
‫‪n‬‬
‫‪M2‬‬

‫‪œǃ‬‬
‫‪M0‬‬

‫‪O‬‬ ‫‪cos 2π‬‬ ‫‪1‬‬


‫‪n‬‬

‫‪Ŀ‬‬
‫‪II‬‬
‫‪M3‬‬

‫‪M4‬‬

‫ﺷﻜﻞ ‪4.II‬‬

‫ﺍﳊﻞّ‪ .‬ﻟﺪﻳﻨﺎ ‪ z = z :‬و ﺑﺎﻟﺘﺎﱄ ﻃﻮل ﺿﻠﻊ ﻫﺬا اﳌﻀﻠﻊ ﻫﻮ ‪:‬‬

‫‪+‬‬
‫‪M M‬‬ ‫‪+‬‬ ‫‪= ||z‬‬ ‫‪+‬‬ ‫‪− z || = ||z‬‬ ‫||‪− z || = ||z (z − 1)|| = ||z || ⋅ ||z − 1|| = ||z || ⋅ ||z − 1‬‬
‫=‬

‫|‬ ‫𝜋‪2‬‬ ‫𝜋‪2‬‬ ‫|‬ ‫𝜋‪2‬‬ ‫𝜋‪2‬‬


‫‪= |cos‬‬ ‫‪+ ı sin‬‬ ‫= |‪− 1‬‬ ‫‪cos‬‬ ‫‪−1‬‬ ‫‪+ sin‬‬
‫||‬ ‫‪n‬‬ ‫‪n‬‬ ‫||‬ ‫‪n‬‬ ‫‪n‬‬
‫𝜋‪2‬‬ ‫| 𝜋‬ ‫|𝜋‬
‫=‬ ‫‪2 − 2 cos‬‬ ‫=‬ ‫‪4 sin‬‬ ‫| ‪= |2 sin‬‬
‫‪n‬‬ ‫| ‪n‬‬ ‫|‪n‬‬
‫𝜋‬ ‫𝜋‬ ‫𝜋‬
‫ﻓﺈن‬ ‫أن ّ‬
‫ﻛﻞ اﻷﺿﻼع ﻣﺘﻘﺎﻳﺴﺔ )اﳌﻀﻠﻊ ﻣﻨﺘﻈﻢ !( ّ‬ ‫‪ M M‬و ﺑﲈ ّ‬ ‫‪+‬‬ ‫ﻟﻜﻦ 𝜋 < < ‪ 0‬إذن ‪≥ 0‬‬
‫‪= 2 sin‬‬ ‫‪ sin‬و ﺑﺎﻟﺘﺎﱄ‬
‫‪n‬‬ ‫‪n‬‬ ‫‪n‬‬
‫𝜋‬
‫أن ﻋﺪد اﻷﺿﻼع ﻫﻮ ‪ّ n‬‬
‫ﻓﺈن ﳏﻴﻂ ﻫﺬا اﳌﻀﻠﻊ ﻫﻮ ‪ . 2n sin‬و ﻟﺪﻳﻨﺎ اﻟﻨﻬﺎﻳﺔ اﻟﺘﺎﻟﻴﺔ ‪:‬‬ ‫ﻃﻮل ّ‬
‫ﻛﻞ ﻣﻨﻬﺎ ﻫﻮ ‪ ، M M +‬و ﺑﲈ ّ‬
‫‪ņőƱŀ Ţ‬‬
‫‪n‬‬
‫𝜋‬ ‫‪sin‬‬
‫‪lim 2n sin = 2𝜋 × lim‬‬ ‫𝜋‪= 2𝜋 × 1 = 2‬‬
‫∞‪→+‬‬ ‫‪n‬‬ ‫∞‪→+‬‬

‫و ﻫﻮ ﳏﻴﻂ اﻟﺪاﺋﺮة اﳌﺜﻠﺜﻴﺔ و ﻫﻲ ﻧﺘﻴﺠﺔ ﻳﻤﻜﻦ ﺗﻮﻗﻌﻬﺎ ﻛﲈ ﻳﺒﲔ ذﻟﻚ اﻟﺸﻜﻞ ‪. 3.II‬‬
‫ﻣﻦ ﺟﻬﺔ أﺧﺮى‪ ،‬ﻛﻞ اﳌﺜﻠﺜﺎت ‪ OM M +‬ﻣﺘﻘﺎﻳﺴﺔ ّ‬
‫)ﻷن ﳍﺎ ﺛﻼﺛﺔ أﺿﻼع ﻣﺘﻘﺎﻳﺴﺔ( و ﺑﺎﻟﺘﺎﱄ ﻟﺪﳞﺎ ﻧﻔﺲ اﳌﺴﺎﺣﺔ‪.‬‬
‫ﻧﺤﺴﺐ ﻣﺴﺎﺣﺔ اﳌﺜﻠﺚ ‪ OM M‬ﺑﺄﺧﺬ اﻟﻘﺎﻋﺪة ‪) OM‬اﻟﺘﻲ ﻃﻮﳍﺎ ‪ (1‬و اﻹرﺗﻔﺎع اﳌﺘﻌﻠﻖ ﺑﺎﻟﺮأس ‪) M‬اﻟﺬي ﻃﻮﻟﻪ ﻫﻮ‬
‫‪1‬‬ ‫𝜋‪2‬‬ ‫𝜋‪2‬‬
‫‪) sin‬ﺷﻜﻞ ‪ (4.II‬و ﺑﺎﻟﺘﺎﱄ ّ‬
‫ﻓﺈن ﻣﺴﺎﺣﺔ اﳊ ّﻴﺰ اﳌﺴﺘﻮي‬ ‫‪ sin‬أي ﺗﺮﺗﻴﺒﺔ ‪ (M‬ﻓﺘﻜﻮن ﻣﺴﺎﺣﺔ ﻫﺬا اﳌﺜﻠﺚ ﺗﺴﺎوي‬
‫‪2‬‬ ‫‪n‬‬ ‫‪n‬‬
‫‪n‬‬ ‫𝜋‪2‬‬
‫‪ƕ‬‬

‫‪ .‬و ﻟﺪﻳﻨﺎ اﻟﻨﻬﺎﻳﺔ اﻟﺘﺎﻟﻴﺔ ‪:‬‬ ‫‪sin‬‬ ‫اﻟﻮاﻗﻊ داﺧﻞ ﻫﺬا اﳌﻀﻠﻊ ﺗﺴﺎوي‬
‫‪2‬‬ ‫‪n‬‬
‫‪n‬‬ ‫𝜋‪2‬‬ ‫‪sin‬‬
‫‪lim‬‬ ‫‪sin‬‬ ‫‪= 𝜋 × lim‬‬ ‫𝜋=‪=𝜋×1‬‬
‫∞‪→+‬‬ ‫‪2‬‬ ‫‪n‬‬ ‫∞‪→+‬‬

‫و ﻫﻲ ﻣﺴﺎﺣﺔ ﻗﺮص اﻟﻮﺣﺪة ؛ ﻧﺘﻴﺠﺔ ُﻣﺘ ََﻮ ّﻗﻌﺔ أﻳﻀﺎ )ﺷﻜﻞ ‪. (3.II‬‬
‫‪ũŏ‬‬

‫■‬

‫ﻣﺜﺎﻝ ‪ : 15‬اﳉﺬور اﻟﺘﻜﻌﻴﺒﻴﺔ ﻟﻠﻌﺪد ‪ ı‬ﻫﻲ اﻷﻋﺪاد اﳌﺮﻛﺒﺔ‬


‫𝜋‬ ‫𝜋‬
‫‪2‬‬ ‫𝜋‪2s‬‬ ‫⎤⎞ 𝜋‪⎞ + ı sin ⎛ 2 + 2s‬‬
‫‪z = √1 ⎡cos ⎛ +‬‬
‫‪3‬‬ ‫‪3‬‬ ‫‪3‬‬ ‫‪3‬‬
‫⎝ ⎣‬ ‫⎠‬ ‫⎝‬ ‫⎦⎠‬

‫𝟴𝟮‬

‫‪http ://tinyurl.com/Malki1718‬‬ ‫‪0‬‬


‫‪ .II‬اﻟﺸﲁ اﳌﺜﻠﱻ ﻟﻌﺪد ﻣﺮﻛﺐ‬
‫‪.‬‬
‫ﺣﻴﺚ ‪ 0 ≤ s < 3‬أي ﻫﻲ اﻷﻋﺪاد اﻟﺜﻼﺛﺔ ‪:‬‬

‫‪Ŕž Ŧ Ľ‬‬
‫𝜋‬ ‫𝜋‬ ‫‪√3‬‬ ‫‪1‬‬
‫‪z = 1 ⋅ cos‬‬ ‫‪+ ı sin‬‬ ‫=‬ ‫‪+ ı‬‬
‫‪6‬‬ ‫‪6‬‬ ‫‪2‬‬ ‫‪2‬‬
‫𝜋‪𝜋 2‬‬ ‫𝜋‬ ‫𝜋‪2‬‬ ‫𝜋‪5‬‬ ‫𝜋‪5‬‬ ‫‪√3 1‬‬

‫‪œǃ‬‬
‫‪z = 1 ⋅ cos‬‬ ‫‪+‬‬ ‫‪+ ı sin‬‬ ‫‪+‬‬ ‫‪= cos‬‬ ‫‪+ ı sin‬‬ ‫‪=−‬‬ ‫‪+ ı‬‬
‫‪6‬‬ ‫‪3‬‬ ‫‪6‬‬ ‫‪3‬‬ ‫‪6‬‬ ‫‪6‬‬ ‫‪2‬‬ ‫‪2‬‬
‫𝜋‪𝜋 4‬‬ ‫𝜋‬ ‫𝜋‪4‬‬ ‫𝜋‪3‬‬ ‫𝜋‪3‬‬
‫‪z = 1 ⋅ cos‬‬ ‫‪+‬‬ ‫‪+ ı sin‬‬ ‫‪+‬‬ ‫‪= cos‬‬ ‫‪+ ı sin‬‬ ‫‪= −ı‬‬
‫‪6‬‬ ‫‪3‬‬ ‫‪6‬‬ ‫‪3‬‬ ‫‪2‬‬ ‫‪2‬‬

‫‪Ŀ‬‬
‫ﺻﻮر ﻫﺬه اﳉﺬور ﻫﻲ رؤوس ﻣﺜﻠﺚ ﻣﺘﻘﺎﻳﺲ اﻷﺿﻼع ﻣﺮﺳﻮم داﺧﻞ اﻟﺪاﺋﺮة ذات اﳌﺮﻛﺰ ‪ O‬و ﻧﺼﻒ اﻟﻘﻄﺮ ‪) 1‬ﺷﻜﻞ ‪.(5.II‬‬
‫‪II‬‬

‫‪1‬‬

‫) ‪M1 (z1‬‬ ‫) ‪M0 (z0‬‬


‫‪2π‬‬
‫‪3‬‬

‫‪π‬‬
‫‪6‬‬

‫‪2π‬‬ ‫‪O‬‬ ‫‪1‬‬


‫‪3‬‬

‫) ‪M2 (z2‬‬

‫ﺷﻜﻞ ‪ :5.II‬اﳉﺬور اﻟﺘﻜﻌﻴﺒﻴﺔ ﻟﻠﻮﺣﺪة اﻟﺘﺨﻴﻠﻴﺔ ‪.ı‬‬


‫‪ņőƱŀ Ţ‬‬
‫)‪ (a + ıb‬؟‬ ‫ﺗﻄﺒﻴﻖ ‪ : 2‬ﻣﺎ ﻫﻮ ﻋﺪد اﻷزواج ا ُﳌﺮﺗّﺒﺔ )‪ (a, b‬ﻣﻦ اﻷﻋﺪاد اﳊﻘﻴﻘﻴﺔ اﻟﺘﻲ ﲢﻘﻖ ‪= a − ıb‬‬

‫‪ ، z‬و ﺑﲈ ّ‬
‫أن‬ ‫= |‪ . |z‬اﳌﺴﺎواة اﳌﻌﻄﺎة ﺗُﺼﺒﺢ ‪= z‬‬ ‫‪a +b‬‬ ‫ﺍﳊﻞّ‪ .‬ﻟﻴﻜﻦ ‪ . z = a + ıb‬ﻟﺪﻳﻨﺎ ‪ z = a − ıb‬و‬
‫|‪|z‬‬ ‫‪= ||z‬‬ ‫‪|| = ||z|| = |z| ,‬‬

‫ّ‬
‫ﻓﺈن‬
‫‪ƕ‬‬

‫|‪|z| |z‬‬ ‫‪−1 =0‬‬

‫|‪ |z‬أي ‪ z = 0‬أو ‪ |z| ) |z| = 1‬ﻋﺪد ﺣﻘﻴﻘﻲ‪ ،‬و اﻟﻌﺪد اﳊﻘﻴﻘﻲ اﻟﻮﺣﻴﺪ اﻟﺬي ُﳛ ﱢﻘﻖ‬ ‫ﻣﻨﻪ ﻧﺴﺘﻨﺘﺞ ّ‬
‫أن ‪ |z| = 0‬أو ‪= 1‬‬
‫‪ x‬ﻫﻮ ‪.( x = 1‬‬ ‫‪=1‬‬
‫‪ũŏ‬‬

‫‪ z = 0‬أي )‪.(a, b) = (0, 0‬‬ ‫•‬

‫ﰲ اﳊﺎﻟﺔ ‪ |z| = 1‬ﻟﺪﻳﻨﺎ ‪:‬‬ ‫•‬

‫‪z‬‬ ‫‪=z ⟺ z×z‬‬ ‫‪=z×z ⟺ z‬‬ ‫‪= |z| = 1‬‬

‫‪ z‬ﺗﻘﺒﻞ ‪ 2013‬ﺣﻼ )ﰲ ‪ ( ℂ‬ﻓﺈﻧّﻪ ﻳﻮﺟﺪ ‪ 2013‬زوج ﻣﺮﺗّﺐ )‪ (a, b‬ﺑﺤﻴﺚ ‪ a + b = 1‬و‬ ‫و ﺑﲈ ّ‬


‫أن اﳌﻌﺎدﻟﺔ ‪= 1‬‬
‫)‪.(a + ıb‬‬ ‫‪= a − ıb‬‬

‫𝟵𝟮‬

‫‪http ://tinyurl.com/Malki1718‬‬ ‫‪0‬‬


‫‪ .6.II‬اﳉﺬور اﻟﻨﻮﻧﻴﺔ ﻟﻠﻮﺣﺪة‬
‫‪.‬‬
‫)‪ (a + ıb‬ﻫﻮ‬ ‫ﰲ اﻷﺧﲑ‪ ،‬ﻋﺪد اﻷزواج اﳌﺮﺗﺒﺔ )‪ (a, b‬ﻣﻦ اﻷﻋﺪاد اﳊﻘﻴﻘﻴﺔ و اﻟﺘﻲ ﲢ ّﻘﻖ ‪= a − ıb‬‬
‫‪2013 + 1 = 2014‬‬

‫‪Ŕž Ŧ Ľ‬‬
‫■‬

‫‪œǃ‬‬
‫ﺍﳉﺬﻭﺭ ﺍﻟﻨﻮﻧﻴﺔ ﻟﻠﻮﺣﺪﺓ‬ ‫‪6.II‬‬
‫إذا ﻛﺎن ‪ w = 1‬ﻓﺈﻧّﻨﺎ ﻧﺤﺼﻞ ﻋﲆ اﳉﺬور اﻟﻨﻮﻧﻴﺔ ﻟﻠﻌﺪد ‪.1‬‬

‫‪Ŀ‬‬
‫‪II‬‬
‫ﺣﻠﻮل اﳌﻌﺎدﻟﺔ ‪ z = 1‬ﺗﺴﻤﻰ اﳉﺬور اﻟﻨﻮﻧﻴﺔ ﻟﻠﻮﺣﺪة ‪) 4‬أو اﻟﻌﺪد ‪.(1‬‬ ‫ﺗﻌﺮﻳﻒ ‪: 10‬‬

‫ﺟﺬرا ﻧﻮﻧ ًﻴﺎ ﺗُﻌﻄﻰ ﺑﺎﻟﻌﺒﺎرة‬ ‫ﳑﺎ ﺳﺒﻖ ﻧﺴﺘﺨﻠﺺ ّ‬


‫أن اﻟﻌﺪد ‪ 1‬ﻳﻘﺒﻞ ‪ً n‬‬
‫𝜋‪2k‬‬ ‫𝜋‪2k‬‬
‫‪𝜔 = cos‬‬ ‫‪+ ı sin‬‬ ‫‪,‬‬ ‫}‪k ∈ {0, 1, 2, ⋯ , n − 1‬‬
‫‪n‬‬ ‫‪n‬‬
‫ﺑﻤﻌﻨﻰ‬

‫;‪𝜔 = cos 0 + ı sin 0 = 1‬‬


‫𝜋‪2‬‬ ‫𝜋‪2‬‬
‫‪𝜔 = cos‬‬ ‫‪+ ı sin‬‬ ‫;𝜔 =‬
‫‪n‬‬ ‫‪n‬‬
‫𝜋‪4‬‬ ‫𝜋‪4‬‬
‫‪𝜔 = cos‬‬ ‫‪+ ı sin‬‬ ‫; 𝜔=‬
‫‪n‬‬ ‫‪n‬‬
‫⋮‬
‫𝜋)‪2(n − 1‬‬ ‫𝜋)‪2(n − 1‬‬ ‫‪−‬‬
‫‪𝜔 − = cos‬‬ ‫‪+ ı sin‬‬ ‫𝜔=‬ ‫;‬
‫‪n‬‬ ‫‪n‬‬
‫‪ņőƱŀ Ţ‬‬
‫‪. U = 1, 𝜔, 𝜔 , ⋯ , 𝜔 −‬‬ ‫و ﻧﻜﺘﺐ‬
‫أن اﻟﻌﺪد 𝜔 ُﻣ َﻮ ﱢﻟﺪ )أو ُﻳﻮ ّﻟﺪ( اﳌﺠﻤﻮﻋﺔ ‪. U‬‬ ‫أن ﻋﻨﺎﴏ ‪ U‬ﻫﻲ ﻗﻮى ﻟﻠﻌﺪد 𝜔 ) 𝜔 = ‪ : ( 1‬ﻧﻘﻮل ّ‬ ‫ﻧﻼﺣﻆ ّ‬
‫ﻣﻨﺘﻈﲈ ﺗﻘﻊ رؤوﺳﻪ ﻋﲆ اﻟﺪاﺋﺮة اﻟﺘﻲ ﻣﺮﻛﺰﻫﺎ اﳌﺒﺪأ ‪ O‬و‬ ‫ً‬ ‫ﻓﺈن ﺻﻮر اﳉﺬور اﻟﻨﻮﻧﻴﺔ ﻟﻠﻮﺣﺪة ﺗُﺸﻜّﻞ ﻣﻀ ّﻠ ًﻌﺎ‬
‫و ﺣﺴﺐ ﻣﺎ ﺳﺒﻖ‪ّ ،‬‬
‫ﻧﺼﻒ ﻗﻄﺮﻫﺎ ‪. 1‬‬
‫ﻓﻴﲈ ﻳﲇ دراﺳﺔ ﻟﺒﻌﺾ اﳊﺎﻻت اﳋﺎﺻﺔ ‪:‬‬

‫ﻣﻦ أﺟﻞ ‪ ، n = 2‬ﻟﻠﻤﻌﺎدﻟﺔ ‪ z = 1‬ﺟﺬران ‪ −1 :‬و ‪ ، 1‬و ﳘﺎ اﳉﺬران ﺍﻟﱰﺑﻴﻌﻴﺎﻥ ﻟﻠﻮﺣﺪة‪.‬‬ ‫•‬

‫ﻣﻦ أﺟﻞ ‪ ، n = 3‬اﳉﺬور ﺍﻟﺘﻜﻌﻴﺒﻴﺔ ﻟﻠﻮﺣﺪة )أي ﺣﻠﻮل اﳌﻌﺎدﻟﺔ ‪ ( z = 1‬ﺗُﻌﻄﻰ ﺑﺎﻟﻌﺒﺎرة‬
‫‪ƕ‬‬

‫•‬
‫𝜋‪2k‬‬ ‫𝜋‪2k‬‬
‫‪𝜔 = cos‬‬ ‫‪+ ı sin‬‬ ‫‪,‬‬ ‫}‪k ∈ {0, 1, 2‬‬
‫‪3‬‬ ‫‪3‬‬
‫أي‬
‫‪ũŏ‬‬

‫𝜋‪2‬‬ ‫𝜋‪2‬‬ ‫‪1‬‬ ‫‪√3‬‬ ‫𝜋‪4‬‬ ‫𝜋‪4‬‬ ‫‪1‬‬ ‫‪√3‬‬


‫‪𝜔 = 1 , 𝜔 = cos‬‬ ‫‪+ ı sin‬‬ ‫‪=− +ı‬‬ ‫‪= j , 𝜔 = cos‬‬ ‫‪+ ı sin‬‬ ‫‪=− −ı‬‬ ‫‪=j‬‬
‫‪3‬‬ ‫‪3‬‬ ‫‪2‬‬ ‫‪2‬‬ ‫‪3‬‬ ‫‪3‬‬ ‫‪2‬‬ ‫‪2‬‬
‫و ﻫﻲ ﺗُﺸﻜّﻞ ﻣﺜﻠﺜﺎ ﻣﺘﻘﺎﻳﺲ اﻷﺿﻼع ﺗﻘﻊ رؤوﺳﻪ ﻋﲆ اﻟﺪاﺋﺮة )‪ ) 𝒞 (0, 1‬ﺷﻜﻞ ‪. ( 6.II‬‬
‫ﻣﻦ أﺟﻞ ‪ ، n = 4‬اﳉﺬور ﺍﻟﺮﺍﺑﻌﺔ ﻟﻠﻌﺪد ‪ 1‬ﻫﻲ‬ ‫•‬
‫𝜋‪2k‬‬ ‫𝜋‪2k‬‬
‫‪𝜔 = cos‬‬ ‫‪+ ı sin‬‬ ‫‪,‬‬ ‫}‪k ∈ {0, 1, 2, 3‬‬
‫‪4‬‬ ‫‪4‬‬

‫𝟬𝟯‬

‫‪http ://tinyurl.com/Malki1718‬‬ ‫‪0‬‬


‫‪ .II‬اﻟﺸﲁ اﳌﺜﻠﱻ ﻟﻌﺪد ﻣﺮﻛﺐ‬
‫‪.‬‬
‫‪y‬‬

‫‪Ŕž Ŧ Ľ‬‬
‫‪j‬‬

‫‪œǃ‬‬
‫‪1‬‬

‫‪Ŀ‬‬
‫‪O‬‬ ‫‪x‬‬
‫‪II‬‬

‫‪j2‬‬

‫ﺷﻜﻞ ‪ :6.II‬اﳉﺬور اﻟﺘﻜﻌﻴﺒﻴﺔ ﻟﻠﻮﺣﺪة‪.‬‬

‫أو‬
‫𝜋‪2‬‬ ‫𝜋‪2‬‬
‫‪𝜔 = cos 0 + ı sin 0 = 1 , 𝜔 = cos‬‬ ‫‪+ ı sin‬‬ ‫‪=ı‬‬
‫‪4‬‬ ‫‪4‬‬
‫𝜋‪4‬‬ ‫𝜋‪4‬‬ ‫𝜋‪6‬‬ ‫𝜋‪6‬‬
‫‪𝜔 = cos‬‬ ‫‪+ ı sin‬‬ ‫‪= −1 , 𝜔 = cos‬‬ ‫‪+ ı sin‬‬ ‫‪= −ı‬‬
‫‪4‬‬ ‫‪4‬‬ ‫‪4‬‬ ‫‪4‬‬
‫‪ U = 1, ı, ı , ı‬و ﺻﻮر ﻫﺬه اﳉﺬور ﻫﻲ رؤوس ﻣﺮﺑﻊ ﻣﺮﺳﻮم ﻋﲆ اﻟﺪاﺋﺮة )‪𝒞 (0, 1‬‬ ‫أي ّ‬
‫أن }‪= {1, ı, −1, −ı‬‬
‫)ﺷﻜﻞ ‪.( 7.II‬‬

‫‪. U = {−1, 1, −ı, ı} ، U = 1, j, j‬‬ ‫ﻟﺪﻳﻨﺎ إذن ‪، U = {−1, 1} ، U = {1} :‬‬


‫‪ņőƱŀ Ţ‬‬
‫ﻟﻴﻜﻦ ‪ z‬ﻋﺪ ًدا ﻣﺮﻛ ًﺒﺎ و ﻟﺘﻜﻦ ‪ n ، m ، ℓ‬و ‪ d‬أﻋﺪا ًدا ﻃﺒﻴﻌﻴ ًﺔ أﻛﱪ ﻣﻦ أو ﺗﺴﺎوي ‪ . 2‬ﻟﺪﻳﻨﺎ ‪:‬‬ ‫ﻣﺒﺮﻫﻨﺔ ‪: 13‬‬

‫ﺣﻞ ﻟﻠﻤﻌﺎدﻟﺔ ‪ z = 1‬ﻫﻮ ّ‬


‫ﺣﻞ ﻟﻠﻤﻌﺎدﻟﺔ ‪ z = 1‬أي ‪. U ⊂ U‬‬ ‫ﻓﺈن أي ّ‬
‫‪ •1‬إذا ﻛﺎن ‪ّ n|ℓ‬‬

‫‪ •2‬اﳊﻠﻮل اﳌﺸﱰﻛﺔ ﻟﻠﻤﻌﺎدﻟﺘﲔ ‪ z = 1‬و ‪ z = 1‬ﻫﻲ ﺣﻠﻮل اﳌﻌﺎدﻟﺔ ‪ z = 1‬ﺣﻴﺚ‬


‫‪ ، d = pgcd (m, n) = m ∧ n‬أي ‪. U ∩ U = U‬‬
‫‪ƕ‬‬

‫‪ •1‬ﻧﻀﻊ ‪ ℓ = qn‬ﻣﻨﻪ ‪: 5‬‬ ‫ﺍﻟﱪﻫﺎﻥ‪.‬‬


‫‪z −1=z‬‬ ‫‪= (z ) − 1 = (z − 1) z‬‬ ‫‪−‬‬ ‫‪+z‬‬ ‫‪−‬‬ ‫‪+⋯+z +1‬‬
‫‪ũŏ‬‬

‫ﻣﻨﻪ‬
‫‪z =1 ⟹ z −1=0× z‬‬ ‫‪−‬‬ ‫‪+z‬‬ ‫‪−‬‬ ‫‪+⋯+z +1 =0‬‬

‫ﺣﻞ ﻟﻠﻤﻌﺎدﻟﺔ ‪ z = 1‬ﻫﻮ ّ‬


‫ﺣﻞ ﻟﻠﻤﻌﺎدﻟﺘﲔ ‪ z = 1‬و ‪. z = 1‬‬ ‫ﻛﻞ ّ‬
‫أن ‪ d|m‬و ‪ d|n‬ﻓﺤﺴﺐ ﻣﺎ ﺳﺒﻖ ّ‬
‫‪ •2‬ﺑﲈ ّ‬

‫‪.‬‬ ‫‪−‬‬ ‫=‬ ‫‪−‬‬ ‫‪−‬‬


‫‪+‬‬ ‫‪−‬‬
‫‪+⋯+‬‬ ‫‪+‬‬ ‫‪+‬‬ ‫‪5‬ﻧُﺬﻛّﺮ ﺑﺎﳌﺘﻄﺎﺑﻘﺔ‬

‫𝟭𝟯‬

‫‪http ://tinyurl.com/Malki1718‬‬ ‫‪0‬‬


‫‪ .6.II‬اﳉﺬور اﻟﻨﻮﻧﻴﺔ ﻟﻠﻮﺣﺪة‬
‫‪.‬‬
‫‪y‬‬

‫‪Ŕž Ŧ Ľ‬‬
‫‪ı‬‬

‫‪œǃ‬‬
‫‪1‬‬

‫‪Ŀ‬‬
‫‪−1‬‬
‫‪O‬‬ ‫‪x‬‬

‫‪II‬‬
‫‪−ı‬‬

‫ﺷﻜﻞ ‪ :7.II‬اﳉﺬور اﻟﺮاﺑﻌﺔ ﻟﻠﻌﺪد ‪.1‬‬

‫𝜋‪2q‬‬ ‫𝜋‪2q‬‬ ‫𝜋‪2p‬‬ ‫𝜋‪2p‬‬


‫‪𝜔 = cos‬‬ ‫‪+ ı sin‬‬ ‫‪ 𝜔 = cos‬ﺣﻼ ﻟﻠﻤﻌﺎدﻟﺔ ‪ z = 1‬و‬ ‫‪+ ı sin‬‬ ‫ﻣﻦ ﺟﻬﺔ أﺧﺮى‪ ،‬ﻟﻴﻜﻦ‬
‫‪n‬‬ ‫‪n‬‬ ‫‪m‬‬ ‫‪m‬‬
‫ّ‬
‫ﻓﺈن‬ ‫أن || 𝜔|| = || 𝜔||‬
‫ﺣﻼ ﻟﻠﻤﻌﺎدﻟﺔ ‪ . z = 1‬ﺑﲈ ّ‬

‫𝜋‪2p𝜋 2q‬‬
‫𝜔= 𝜔‬ ‫𝜔 ‪⟺ arg‬‬ ‫𝜔 ‪= arg‬‬ ‫⟺ )𝜋‪(mod 2‬‬ ‫=‬ ‫‪+ 2r𝜋 ,‬‬ ‫‪r∈ℤ‬‬
‫‪m‬‬ ‫‪n‬‬
‫‪p‬‬ ‫‪q‬‬
‫⟺‬ ‫‪− = r ⟺ pn − qm = rmn‬‬
‫‪m n‬‬
‫‪ņőƱŀ Ţ‬‬
‫ﻟﻜﻦ ‪ m = m d‬و ‪ n = n d‬ﻣﻊ ‪ّ pgcd m , n = 1‬‬
‫)ﻷن )‪ ( d = pgcd (m, n‬ﻣﻨﻪ‬
‫‪pn − qm = rmn ⟺ n p − m q = rm n d ⟺ m rn d + q = n p‬‬

‫ﻓﺈن ‪ m |p‬أي ‪ p = p m‬ﻣﻊ ∗‪ p ∈ ℕ‬ﻣﻨﻪ‬


‫أن ‪ّ pgcd m , n = 1‬‬
‫ﻣﻨﻪ ‪ m |n p‬و ﺑﲈ ّ‬
‫𝜋 ‪2p𝜋 2p m 𝜋 2p‬‬
‫𝜔 ‪arg‬‬ ‫=‬ ‫=‬ ‫=‬
‫‪m‬‬ ‫‪md‬‬ ‫‪d‬‬
‫أن ) دﺳﺘﻮر ﻣﻮﺍﻓﺮ ( ‪:‬‬
‫و ﻫﺬا ﻳﻌﻨﻲ ّ‬
‫𝜋 ‪2p‬‬ ‫𝜋 ‪2p‬‬ ‫𝜋 ‪2dp‬‬ ‫𝜋 ‪2dp‬‬
‫𝜔‬ ‫=‬ ‫‪cos‬‬ ‫‪+ ı sin‬‬ ‫‪= cos‬‬ ‫‪+ ı sin‬‬ ‫‪= cos 2p 𝜋 + ı sin 2p 𝜋 = 1‬‬
‫‪ƕ‬‬

‫‪d‬‬ ‫‪d‬‬ ‫‪d‬‬ ‫‪d‬‬


‫أي أﻧّﻨﺎ أﺛﺒﺘﻨﺎ ‪:‬‬
‫𝜔= 𝜔‬ ‫⟹‬ ‫𝜔‬ ‫‪=1‬‬
‫‪ũŏ‬‬

‫■‬

‫رأﺳﺎ و ﻋﺪد‬ ‫ﺗﻄﺒﻴﻖ ‪ : 3‬ﻧﻌﺘﱪ ﻣﻀ ّﻠﻌﲔ ﻣﻨﺘﻈﻤﲔ ﻣﺮﺳﻮﻣﲔ ﻋﲆ ﻧﻔﺲ اﻟﺪاﺋﺮة‪ .‬إذا ﻛﺎن ﻋﺪد رؤوس اﳌﻀﻠﻊ ّ‬
‫اﻷول ﻫﻮ ‪ً 1966‬‬
‫رأﺳﺎ ﻣﺸﱰﻛًﺎ ﻓﻜﻢ ﻫﻮ )ﺑﺎﻟﺘﺤﺪﻳﺪ( ﻋﺪد اﻟﺮؤوس اﳌﺸﱰﻛﺔ ﺑﲔ ﻫﺬﻳﻦ اﳌﻀﻠﻌﲔ ؟‬ ‫رأﺳﺎ و إذا ﻋﻠﻤﻨﺎ ّ‬
‫أن ﳍﲈ ً‬ ‫رؤوس اﻟﺜﺎﲏ ‪ً 2949‬‬

‫𝟮𝟯‬

‫‪http ://tinyurl.com/Malki1718‬‬ ‫‪0‬‬


‫‪ .II‬اﻟﺸﲁ اﳌﺜﻠﱻ ﻟﻌﺪد ﻣﺮﻛﺐ‬
‫‪.‬‬
‫‪ z‬و ﺣﺴﺐ اﳌﱪﻫﻨﺔ‬ ‫=‬ ‫‪z‬و‬ ‫‪=1‬‬ ‫ﺍﳊﻞّ‪ .‬ﻋﺪد اﻟﺮؤوس اﳌﺸﱰﻛﺔ ﻟﻠﻤﻀﻠﻌﲔ ﻫﻮ ﻋﺪد اﳊﻠﻮل اﳌﺸﱰﻛﺔ ﻟﻠﻤﻌﺎدﻟﺘﲔ‬

‫‪Ŕž Ŧ Ľ‬‬
‫‪ ) 13‬اﻟﺸﻄﺮ ‪ّ ، ( 2‬‬
‫ﻓﺈن ﻫﺬا اﻟﻌﺪد ﻳﺴﺎوي‬
‫‪d = pgcd (1966, 2949) = 983‬‬
‫■‬

‫‪œǃ‬‬
‫ﻟﺘﻜﻦ 𝜔 ‪ 𝜔 − ، ⋯ ، 𝜔 ،‬اﳉﺬور اﻟﻨﻮﻧﻴﺔ ﻟﻠﻮﺣﺪة‪ .‬ﻣﻦ أﺟﻞ ّ‬
‫ﻛﻞ ﻋﺪد ﻃﺒﻴﻌﻲ ‪ k‬ﻟﺪﻳﻨﺎ ‪:‬‬ ‫ﻣﺒﺮﻫﻨﺔ ‪: 14‬‬
‫‪−‬‬ ‫‪n,‬‬ ‫إذا ﻛﺎن ‪n|k‬‬

‫‪Ŀ‬‬
‫= 𝜔‬
‫‪II‬‬

‫=‬ ‫ﰲ اﳊﺎﻻت اﻷﺧﺮى ‪0 ,‬‬

‫𝜋‪2‬‬ ‫𝜋‪2‬‬
‫‪.∀𝛼 ∈ {0, 1, 2, ⋯ , n − 1} , 𝜔 ≠ 1‬‬ ‫‪ . 𝜔 = cos‬ﻟﺪﻳﻨﺎ ‪ 𝜔 ∈ U‬و‬ ‫‪+ ı sin‬‬ ‫ﺍﻟﱪﻫﺎﻥ‪ .‬ﻧﻌﺘﱪ اﻟﻌﺪد اﳌﺮﻛﺐ‬
‫‪n‬‬ ‫‪n‬‬
‫إذن ‪. 𝜔 = 1 ⟺ n|m‬‬

‫إذا ﻛﺎن ‪ n‬ﻻ ﻳﻘﺴﻢ ‪ّ ( n ∤ k ) k‬‬


‫ﻓﺈن ‪ 𝜔 ≠ 1‬ﻣﻨﻪ‬ ‫•‬
‫‪−‬‬ ‫‪−‬‬ ‫‪−‬‬
‫𝜔 ‪1−‬‬ ‫‪1−1‬‬
‫= 𝜔‬ ‫𝜔‬ ‫=‬ ‫𝜔‬ ‫=‬ ‫=‬ ‫‪=0‬‬
‫=‬ ‫=‬ ‫=‬
‫𝜔‪1−‬‬ ‫𝜔‪1−‬‬

‫إذا ﻛﺎن ‪ k = qn‬ﻣﻊ ‪ّ q ∈ ℕ‬‬


‫ﻓﺈن‬ ‫•‬
‫‪−‬‬ ‫‪−‬‬ ‫‪−‬‬ ‫‪−‬‬ ‫‪−‬‬
‫= 𝜔‬ ‫𝜔‬ ‫=‬ ‫𝜔‬ ‫=‬ ‫= ‪1‬‬ ‫‪1=n‬‬
‫=‬ ‫=‬ ‫=‬ ‫=‬ ‫=‬

‫■‬

‫𝜋‪2‬‬ ‫𝜋‪2‬‬
‫‪ . 𝜔 = cos‬ﻟﻴﻜﻦ ‪ z‬ﻋﺪ ًدا ﻣﺮﻛ ًﺒﺎ ُﳛ ّﻘﻖ‬ ‫‪+ ı sin‬‬ ‫ﻧﻀﻊ‬ ‫ﺗﻄﺒﻴﻖ ‪: 4‬‬
‫‪ņőƱŀ Ţ‬‬
‫‪n‬‬ ‫‪n‬‬
‫‪∀j ∈ {0, 1, ⋯ , n − 1} ,‬‬ ‫‪||z − 𝜔 || ≤ 1‬‬

‫‪.z=0‬‬ ‫ﺑﺮﻫﻦ ّ‬
‫أن‬

‫ﺍﳊﻞّ‪ .‬ﻟﺪﻳﻨﺎ‬
‫⟺ ‪||z − 𝜔 || ≤ 1‬‬ ‫𝜔‪z−‬‬ ‫𝜔‪z−‬‬ ‫‪≤1‬‬
‫و ﺑﻌﺪ اﻟﻨﴩ و اﻟﱰﺗﻴﺐ ﻧﺠﺪ ) ﻧُﺬﻛّﺮ ّ‬
‫أن |‪ zz = |z‬و ‪( 𝜔𝜔 = |𝜔| = 1‬‬
‫‪ƕ‬‬

‫𝜔‪z−‬‬ ‫𝜔‪z−‬‬ ‫⟺ ‪≤1‬‬ ‫𝜔‪z−‬‬ ‫𝜔‪z−‬‬ ‫‪≤1‬‬


‫‪⟺ zz − z𝜔 − z𝜔 + 𝜔 𝜔 ≤ 1‬‬
‫‪ũŏ‬‬

‫|𝜔| ‪⟺ |z| − z𝜔 − z𝜔 +‬‬ ‫‪≤1‬‬

‫‪⟺ |z| − z𝜔 − z𝜔 + 1 ≤ 1‬‬


‫𝜔‪⟺ |z| ≤ z𝜔 + z‬‬

‫إذن‬
‫‪∀j ∈ {0, 1, ⋯ , n − 1} ,‬‬ ‫𝜔‪|z| ≤ z𝜔 + z‬‬

‫𝟯𝟯‬

‫‪http ://tinyurl.com/Malki1718‬‬ ‫‪0‬‬


‫‪ .7.II‬اﳌﻌﺎدةل ‪Z = a‬‬
‫‪.‬‬
‫ﺑﺠﻤﻊ ﻫﺬه اﳌﱰاﺟﺤﺎت ﻃﺮ ًﻓﺎ إﱃ ﻃﺮف ﻳﻨﺘﺞ‬
‫‪−‬‬ ‫‪−‬‬
‫𝜔 ‪|z| + |z| + ⋯ + |z| ≤ z ⋅ 𝜔 + 𝜔 + ⋯ +‬‬ ‫𝜔‪+z⋅ 𝜔 +𝜔 +⋯+‬‬

‫‪Ŕž Ŧ Ľ‬‬
‫⎭‬
‫⎪‬
‫⎪‬
‫⎪‬
‫⎪‬
‫⎪‬
‫⎪‬
‫⎪‬
‫⎪‬
‫⎬‬
‫⎪‬
‫⎪‬
‫⎪‬
‫⎪‬
‫⎪‬
‫⎪‬
‫⎪‬
‫⎪‬
‫⎫‬
‫ﻣﺮة‬
‫ّ‬

‫‪œǃ‬‬
‫ﻣﻨﻪ‬
‫‪−‬‬ ‫‪−‬‬
‫⎛ ⋅ ‪n |z| ≤ z‬‬ ‫⎛⋅‪𝜔 ⎞+z‬‬ ‫⎞𝜔‬
‫⎝‬ ‫=‬ ‫⎠‬ ‫⎝‬ ‫=‬ ‫⎠‬

‫‪Ŀ‬‬
‫و ﺑﲈ أﻧّﻪ ﻣﻦ أﺟﻞ ّ‬
‫ﻛﻞ ‪ j‬ﻣﻦ }‪ j ، {0, 1, 2, ⋯ , n − 1‬ﻻ ﻳﻘﺴﻢ ‪ّ ) n‬‬
‫ﻷن ‪ ( j < n‬ﻓﺤﺴﺐ اﳌﱪﻫﻨﺔ ‪) 14‬ﻣﻊ ‪ k = 1‬؛ ﻣﻨﻪ ‪n ∤ k‬‬

‫‪II‬‬
‫ّ‬
‫ﻷن ‪: ( k < n‬‬
‫‪−‬‬ ‫‪−‬‬
‫= 𝜔‬ ‫‪𝜔 =0‬‬
‫=‬ ‫=‬

‫■‬ ‫‪ 0 ≤ n |z| ≤ z ⋅ 0 + z ⋅ 0 = 0‬أي ‪ . n |z| = 0‬ﻟﻜﻦ ‪ n ≥ 2‬ﻣﻨﻪ ‪ |z| = 0‬أي ‪. z = 0‬‬ ‫ﻣﻨﻪ‬

‫ﺍﳌﻌﺎﺩﻟﺔ ‪Z = a‬‬ ‫‪7.II‬‬


‫‪n‬‬

‫ﻟﻴﻜﻦ ‪ a‬ﻋﺪ ًدا ﻣﺮﻛ ًﺒﺎ و ‪ n‬ﻋﺪ ًدا ﻃﺒﻴﻌ ًﻴﺎ ﺣﻴﺚ ‪. n ≥ 2‬‬
‫ﺣﻞ اﳌﻌﺎدﻟﺔ ‪ Z = a‬ﻳﺆول إﱃ إﳚﺎد اﳉﺬور اﻟﻨﻮﻧﻴﺔ ﻟﻠﻌﺪد اﳌﺮﻛﺐ ‪ . a‬و ﺣﺴﺐ ﻣﺎ ﺳﺒﻖ ّ‬
‫ﻓﺈن ﻫﺬه اﳌﻌﺎدﻟﺔ ﺗﻘﺒﻞ ‪ n‬ﺣﻼ‬ ‫ّ‬
‫ﳐﺘﻠ ًﻔﺎ‪ .‬ﻧﻀﻊ ]𝜃 ‪. Z = [𝜌,‬‬

‫‪ •1‬ﻧﻔﺮض ّ‬
‫أن ‪. a ∈ ℝ‬‬
‫إذا ﻛﺎن ‪ a > 0‬أي ]‪ّ a = [a, 0‬‬
‫ﻓﺈن‬ ‫•‬

‫]‪[𝜌, 𝜃] = [a, 0] ⟺ [𝜌 , n𝜃] = [a, 0‬‬


‫‪ņőƱŀ Ţ‬‬
‫‪𝜌 =a‬‬
‫⟺‬
‫‪n𝜃 = 0 + 2k𝜋 ,‬‬ ‫‪k∈ℤ‬‬
‫‪⎧ 𝜌 = √a‬‬
‫⟺‬
‫‪⎨ 𝜃 = 2k𝜋 ,‬‬ ‫‪k∈ℤ‬‬
‫⎩‬ ‫‪n‬‬

‫أي اﳊﻠﻮل ﻫﻲ‬


‫𝜋‪2k‬‬ ‫𝜋‪2k‬‬
‫‪Z = √a cos‬‬ ‫‪+ ı sin‬‬ ‫‪,‬‬ ‫}‪k ∈ {0, 1, 2, ⋯ , n − 1‬‬ ‫)𝟐(‬
‫‪ƕ‬‬

‫‪n‬‬ ‫‪n‬‬
‫•‬

‫إذا ﻛﺎن ‪ a < 0‬أي ]𝜋 ‪ّ a = [−a,‬‬


‫ﻓﺈن‬ ‫•‬
‫‪ũŏ‬‬

‫]𝜋 ‪[𝜌, 𝜃] = [−a, 𝜋] ⟺ [𝜌 , n𝜃] = [−a,‬‬


‫‪𝜌 = −a‬‬
‫⟺‬
‫‪n𝜃 = 𝜋 + 2k𝜋 ,‬‬ ‫‪k∈ℤ‬‬
‫‪⎧ 𝜌 = √−a‬‬
‫⟺‬
‫‪⎨ 𝜃 = (2k + 1)𝜋 ,‬‬ ‫‪k∈ℤ‬‬
‫⎩‬ ‫‪n‬‬

‫𝟰𝟯‬

‫‪http ://tinyurl.com/Malki1718‬‬ ‫‪0‬‬


‫‪ .II‬اﻟﺸﲁ اﳌﺜﻠﱻ ﻟﻌﺪد ﻣﺮﻛﺐ‬
‫‪.‬‬
‫أي اﳊﻠﻮل ﻫﻲ‬
‫𝜋)‪(2k + 1‬‬ ‫𝜋)‪(2k + 1‬‬
‫‪Z = √−a cos‬‬ ‫‪+ ı sin‬‬ ‫‪,‬‬ ‫}‪k ∈ {0, 1, 2, ⋯ , n − 1‬‬ ‫)𝟑(‬

‫‪Ŕž Ŧ Ľ‬‬
‫‪n‬‬ ‫‪n‬‬
‫)‪arg (a‬‬

‫‪œǃ‬‬
‫‪ [r, t] = √|a|,‬و ‪:‬‬
‫‪n‬‬ ‫‪ •2‬و اﻵن ﻧﺪرس اﳊﺎﻟﺔ اﻟﻌﺎﻣﺔ ‪ . a ∈ ℂ‬ﻟﻴﻜﻦ ]‪ً 𝛼 = [r, t‬‬
‫ﺟﺬرا ﻧﻮﻧ ًﻴﺎ ﻟﻠﻌﺪد ‪ . a‬ﻟﺪﻳﻨﺎ‬

‫]‪Z = a ⟺ [𝜌, 𝜃] = [r, t‬‬ ‫]‪⟺ [𝜌 , n𝜃] = [r , nt‬‬


‫‪𝜌 =r‬‬

‫‪Ŀ‬‬
‫⟺‬
‫‪n𝜃 = nt + 2k𝜋 ,‬‬ ‫‪k∈ℤ‬‬
‫‪II‬‬

‫‪⎧𝜌 = r‬‬
‫⟺‬
‫‪⎨ 𝜃 = t + 2k𝜋 , k ∈ ℤ‬‬
‫⎩‬ ‫‪n‬‬
‫= 𝜌⎧‬ ‫|‪|a‬‬
‫⟺‬
‫‪⎨ 𝜃 = arg (a) + 2k𝜋 , k ∈ ℤ‬‬
‫⎩‬ ‫‪n‬‬ ‫‪n‬‬

‫أي اﳊﻠﻮل ﻫﻲ‬


‫𝜋‪arg (a) 2k‬‬ ‫𝜋‪arg (a) 2k‬‬
‫= ‪Z‬‬ ‫‪|a| cos‬‬ ‫‪+‬‬ ‫‪+ ı sin‬‬ ‫‪+‬‬ ‫‪,‬‬ ‫}‪k ∈ {0, 1, 2, ⋯ , n − 1‬‬
‫‪n‬‬ ‫‪n‬‬ ‫‪n‬‬ ‫‪n‬‬
‫)𝟒(‬

‫أن اﻟﻌﺒﺎرة )𝟒( ‪ ،‬ﺗﺸﻤﻞ اﻟﻌﺒﺎرﺗﲔ )𝟐( و )𝟑( ‪ .‬ﻣﻦ ﻫﻨﺎ ﻧﺴﺘﺨﻠﺺ ّ‬
‫أن ‪:‬‬ ‫ﻧﻼﺣﻆ ّ‬

‫ﺣﻠﻮل اﳌﻌﺎدﻟﺔ ‪ ، Z = a‬ﺣﻴﺚ ‪ ، a ≠ 0‬ﺗُﻌﻄﻰ ﺑﺎﻟﻌﺒﺎرة ‪:‬‬


‫𝜋‪arg (a) 2k‬‬ ‫𝜋‪arg (a) 2k‬‬
‫= ‪Z‬‬ ‫‪|a| cos‬‬ ‫‪+‬‬ ‫‪+ ı sin‬‬ ‫‪+‬‬ ‫‪,‬‬ ‫}‪k ∈ {0, 1, 2, ⋯ , n − 1‬‬
‫‪n‬‬ ‫‪n‬‬ ‫‪n‬‬ ‫‪n‬‬
‫‪ņőƱŀ Ţ‬‬
‫آﺧﺮا‪ ،‬ﻓﺈﻧّﻨﺎ ﻧﺤﺼﻞ‬
‫ﺟﺬرا ﻧﻮﻧ ًﻴﺎ ً‬ ‫ﻣﻼﺣﻈﺔ ‪ : 12‬اﳉﺬر اﻟﻨﻮﲏ اﻟﺬي ّ‬
‫ﺗﻢ اﺧﺘﻴﺎره أﻋﻼه ﻻ ﻳﺆ ّﺛﺮ ﻋﲆ اﻟﻨﺘﻴﺠﺔ‪ ،‬ﺑﻤﻌﻨﻰ إذا اﺧﱰﻧﺎ ً‬
‫ﻋﲆ ﻧﻔﺲ اﳊﻠﻮل ﻟﻜﻦ ﻗﺪ ﳜﺘﻠﻒ ﺗﺮﺗﻴﺒﻬﺎ ﻋﻦ اﳊﻠﻮل اﻟﺴﺎﺑﻘﺔ‪.‬‬

‫ﺣﻞ ﰲ اﳌﺠﻤﻮﻋﺔ ‪ ℂ‬اﳌﻌﺎدﻟﺔ ‪. Z + 8 = 0 :‬‬ ‫ّ‬ ‫ﻣﺜﺎﻝ ‪: 16‬‬


‫اﳊﻞ ‪ :‬ﻧﻀﻊ )𝜃 ‪. Z = 𝜌 (cos 𝜃 + ı sin‬‬

‫)𝜋 ‪Z + 8 = 0 ⟺ Z = −8 ⟺ 𝜌 (cos 𝜃 + ı sin 𝜃) = 8 (cos 𝜋 + ı sin‬‬


‫‪ƕ‬‬

‫)𝜋 ‪⟺ 𝜌 (cos 3𝜃 + ı sin 3𝜃) = 8 (cos 𝜋 + ı sin‬‬


‫‪𝜌 =8‬‬
‫⟺‬
‫‪3𝜃 = 𝜋 + 2k𝜋 , k ∈ ℤ‬‬
‫‪ũŏ‬‬

‫‪⎧𝜌 = 2‬‬
‫⟺‬
‫‪⎨ 𝜃 = 𝜋 + 2k𝜋 , k ∈ ℤ‬‬
‫⎩‬ ‫‪3‬‬ ‫‪3‬‬

‫أي اﳊﻠﻮل ﻫﻲ‬


‫𝜋‪𝜋 2k‬‬ ‫𝜋‪𝜋 2k‬‬
‫‪Z = 2 cos‬‬ ‫‪+‬‬ ‫‪+ ı sin‬‬ ‫‪+‬‬ ‫‪,‬‬ ‫}‪k ∈ {0, 1, 2‬‬
‫‪3‬‬ ‫‪3‬‬ ‫‪3‬‬ ‫‪3‬‬

‫𝟱𝟯‬

‫‪http ://tinyurl.com/Malki1718‬‬ ‫‪0‬‬


‫‪ .8.II‬ادلاةل اﻷﺳـﻴﺔ اﳌﺮﻛﺒﺔ‬
‫‪.‬‬
‫ﻧﻌﺘﱪ اﳌﻌﺎدﻟﺔ‬ ‫ﻣﺜﺎﻝ ‪: 17‬‬
‫‪Z − (1 + ı) Z + ı = 0‬‬ ‫)𝟓(‬

‫‪Ŕž Ŧ Ľ‬‬
‫‪. Z −1‬‬ ‫‪Z −i‬‬ ‫‪ •1‬أﻧﴩ ّ‬
‫ﺛﻢ رﺗﱢﺐ اﻟﻌﺒﺎرة‬

‫‪œǃ‬‬
‫‪ •2‬إﺳﺘﻨﺘﺞ ﺣﻠﻮل اﳌﻌﺎدﻟﺔ )𝟓( ‪.‬‬
‫اﳊﻞ ‪:‬‬

‫‪Ŀ‬‬
‫•‬ ‫‪1‬‬

‫‪II‬‬
‫‪Z −1‬‬ ‫‪Z − i = Z − ıZ − Z + ı = Z − (1 + ı) Z + ı‬‬

‫‪ •2‬ﺣﺴﺐ ﻣﺎ ﺳﺒﻖ‬
‫‪(𝟓) ⟺ Z − 1‬‬ ‫‪Z −i =0 ⟺ Z −1=0‬‬ ‫‪ Z − ı = 0‬أو‬
‫ﺍﻟﺤﺎﻟﺔ ﺍﻷﻭﻟﻰ ‪:‬‬

‫‪Z − 1 = 0 ⟺ Z = 1 ⟺ 𝜌 (cos 𝜃 + ı sin 𝜃) = cos 0 + ı sin 0‬‬


‫‪⟺ 𝜌 (cos 3𝜃 + ı sin 3𝜃) = cos 0 + ı sin 0‬‬
‫‪𝜌 =1‬‬
‫⟺‬
‫‪3𝜃 = 0 + 2k𝜋 , k ∈ ℤ‬‬

‫أي‬
‫𝜋‪2k‬‬ ‫𝜋‪2k‬‬
‫‪Z = 𝜔 = cos‬‬ ‫‪+ ı sin‬‬ ‫‪,‬‬ ‫}‪k ∈ {0, 1, 2‬‬
‫‪3‬‬ ‫‪3‬‬
‫ﺍﻟﺤﺎﻟﺔ ﺍﻟﺜﺎﻧﻴﺔ ‪:‬‬
‫𝜋‬ ‫𝜋‬
‫‪Z − i = 0 ⟺ Z = i ⟺ 𝜌 (cos 𝜃 + ı sin 𝜃) = cos + ı sin‬‬
‫‪2‬‬ ‫‪2‬‬
‫𝜋‬ ‫𝜋‬
‫‪⟺ 𝜌 (cos 3𝜃 + ı sin 3𝜃) = cos + ı sin‬‬
‫‪2‬‬ ‫‪2‬‬
‫‪ņőƱŀ Ţ‬‬
‫⎧‬ ‫𝜌‬ ‫=‬ ‫‪1‬‬
‫⟺‬ ‫𝜋‬
‫‪⎨ 3𝜃 = + 2k𝜋 , k ∈ ℤ‬‬
‫⎩‬ ‫‪2‬‬
‫𝜋‪𝜋 2k‬‬ ‫𝜋‪𝜋 2k‬‬
‫‪Z = cos‬‬ ‫‪+‬‬ ‫‪+ ı sin‬‬ ‫‪+‬‬ ‫‪,‬‬ ‫}‪k ∈ {0, 1, 2‬‬
‫‪6‬‬ ‫‪3‬‬ ‫‪6‬‬ ‫‪3‬‬

‫ﰲ اﻷﺧﲑ‪ ،‬ﳎﻤﻮﻋﺔ اﳊﻠﻮل ﻫﻲ‬


‫‪ƕ‬‬

‫𝜋‪2k‬‬ ‫𝜋‪2k‬‬ ‫𝜋‪𝜋 2k‬‬ ‫𝜋‪𝜋 2k‬‬


‫=‪S‬‬ ‫‪cos‬‬ ‫‪+ ı sin‬‬ ‫‪; cos‬‬ ‫‪+‬‬ ‫‪+ ı sin‬‬ ‫‪+‬‬ ‫‪,‬‬ ‫}‪k ∈ {0, 1, 2‬‬
‫‪3‬‬ ‫‪3‬‬ ‫‪6‬‬ ‫‪3‬‬ ‫‪6‬‬ ‫‪3‬‬

‫ﺍﻟﺪﺍﻟﺔ ﺍﻷﺳﻴﺔ ﺍﳌﺮﻛﺒﺔ‬ ‫‪8.II‬‬


‫‪ũŏ‬‬

‫ﳌﺘﻐﲑ ﺣﻘﻴﻘﻲ ﻛﻞ داﻟﺔ ﺗﺮﻓﻖ ﺑﻌﺪد ﺣﻘﻴﻘﻲ ‪ t‬ﻋﺪ ًدا ﻣﺮﻛﺒﺎ )‪. z = z (t‬‬
‫ﻧﺴﻤﻲ داﻟﺔ ﻣﺮﻛﺒﺔ ﱢ‬
‫إذا ﻛﺎﻧﺖ ‪ f‬داﻟﺔ ﻣﺮﻛﺒﺔ ﳌﺘﻐﲑ ﺣﻘﻴﻘﻲ ‪ t‬ﻓﺈﻧﻪ ﻳﻤﻜﻦ أن ﻧﺮﻓﻖ ﲠﺎ اﻟﺪاﻟﺘﲔ اﻟﻌﺪدﻳﺘﲔ )‪ a(t‬و )‪ b(t‬اﳌﻌﺮﻓﺘﲔ ﺑﺎﻟﻌﺒﺎرﺗﲔ = )‪a(t‬‬
‫أن )‪ّ ، f(t) = a(t) + ıb(t‬‬
‫ﻓﺈن اﻟﺪاﻟﺔ ‪ُ f‬ﻣﻌ ّﻴﻨﺔ إذا ُﻋﺮﻓﺖ اﻟﺪاﻟﺘﺎن )‪ a(t‬و )‪ ، b(t‬و‬ ‫))‪ Re (f(t‬و ))‪ . b(t) = Im (f(t‬و ﺑﲈ ّ‬
‫ﺑﺎﻟﻌﻜﺲ‪ ،‬إذا ُﻋﺮﻓﺖ اﻟﺪاﻟﺔ ‪ f‬ﻓﺒﺎﻹﻣﻜﺎن إﳚﺎد )‪ a(t‬و )‪. b(t‬‬

‫𝟲𝟯‬

‫‪http ://tinyurl.com/Malki1718‬‬ ‫‪0‬‬


‫‪ .II‬اﻟﺸﲁ اﳌﺜﻠﱻ ﻟﻌﺪد ﻣﺮﻛﺐ‬
‫‪.‬‬
‫‪1‬‬
‫ُﻌﺮف داﻟﺔ ﻣﺮﻛﺒﺔ ﳌﺘﻐﲑ ﺣﻘﻴﻘﻲ ‪.‬‬
‫= )‪ . f(t‬ﻫﺬه اﻟﻌﺒﺎرة ﺗ ﱢ‬ ‫ﻧﻀﻊ‬ ‫ﻣﺜﺎﻝ ‪: 18‬‬
‫‪t−ı‬‬
‫ﻟﺪﻳﻨﺎ ‪:‬‬

‫‪Ŕž Ŧ Ľ‬‬
‫‪1‬‬ ‫‪t+ı‬‬ ‫‪t‬‬ ‫‪1‬‬
‫= )‪f(t‬‬ ‫=‬ ‫=‬ ‫‪+‬‬ ‫‪ı‬‬
‫‪t − ı (t − ı) (t + ı) t + 1 t + 1‬‬

‫‪œǃ‬‬
‫‪1‬‬ ‫‪t‬‬
‫■‬ ‫= )‪b(t‬‬ ‫= )‪ a(t‬و‬ ‫ﻣﻨﻪ‬
‫‪t +1‬‬ ‫‪t +1‬‬

‫ﻳﻤﻜﻦ ﺗﻌﺮﻳﻒ ﻛﻞ ﻣﻦ اﻹﺳﺘﻤﺮارﻳﺔ )اﻹﺗﺼﺎل( و ﻗﺎﺑﻠﻴﺔ اﻹﺷﺘﻘﺎق ﺑﺎﻟﻨﺴﺒﺔ ﻟﻠﺪوال اﳌﺮﻛﺒﺔ ﳌﺘﻐﲑ ﺣﻘﻴﻘﻲ ‪:‬‬

‫‪Ŀ‬‬
‫ﻣﺴﺘﻤﺮﺗﲔ و ﻗﺎﺑﻠﺔ ﻟﻺﺷﺘﻘﺎق إذا ﻛﺎﻧﺖ ‪ a‬و ‪ b‬ﻗﺎﺑﻠﺘﲔ ﻟﻺﺷﺘﻘﺎق ﻣﻊ ‪f (t) = :‬‬
‫ّ‬ ‫ﻓﺎﻟﺪاﻟﺔ ‪ f‬ﻣﺴﺘﻤﺮة )ﻣﺘّﺼﻠﺔ( إذا ﻛﺎﻧﺖ ‪ a‬و ‪b‬‬
‫‪II‬‬

‫)‪. a (t) + ıb (t‬‬

‫ﻟﻴﻜﻦ 𝛼 و 𝛽 ﻋﺪدﻳﻦ ﺣﻘﻴﻘﻴﲔ ‪ .‬ﻧﻌﺘﱪ اﻟﺪاﻟﺔ ‪ f‬اﳌﻌﺮﻓﺔ ﻛﲈ ﻳﲇ ‪:‬‬


‫‪f(t) = e‬‬ ‫‪(cos 𝛽t + ı sin 𝛽t) = e‬‬ ‫‪cos 𝛽t + ıe‬‬ ‫)‪sin 𝛽t = a(t) + ıb(t‬‬
‫ﻫﺬه اﻟﺪاﻟﺔ ﻣﺴﺘﻤﺮة و ﻗﺎﺑﻠﺔ ﻟﻺﺷﺘﻘﺎق ﻋﲆ ‪) ℝ‬ﻷن اﻟﺪاﻟﺘﲔ اﻟﻌﺪدﻳﺘﲔ ‪ a‬و ‪ b‬ﻣﺴﺘﻤﺮﺗﺎن و ﻗﺎﺑﻠﺘﺎن ﻟﻺﺷﺘﻘﺎق ﻋﲆ ‪ ( ℝ‬و ﻟﺪﻳﻨﺎ ‪:‬‬
‫‪f (t) = a (t) + ıb (t) = 𝛼e‬‬ ‫‪(cos 𝛽t + ı sin 𝛽t) + e‬‬ ‫)‪(−𝛽 sin 𝛽t + ı𝛽 cos 𝛽t) = 𝛼f(t) + ı𝛽f(t‬‬

‫أي ‪. f (t) = (𝛼 + ı𝛽) f(t) :‬‬


‫ﺑﺎﻹﺿﺎﻓﺔ إﱃ ذﻟﻚ ‪:‬‬
‫‪f (t + t ) = e‬‬ ‫‪+‬‬ ‫) ‪cos 𝛽 (t + t ) + ı sin 𝛽 (t + t‬‬
‫‪=e‬‬ ‫‪e‬‬ ‫) ‪(cos 𝛽t cos 𝛽t − sin 𝛽t sin 𝛽t ) + ı (sin 𝛽t cos 𝛽t + cos 𝛽t sin 𝛽t‬‬
‫‪=e‬‬ ‫‪e‬‬ ‫‪(cos 𝛽t + ı sin 𝛽t ) (cos 𝛽t + ı sin 𝛽t ) .‬‬

‫ﻣﻨﻪ ‪. f (t + t ) = f (t ) f (t ) :‬‬
‫اﻟﺪاﻟﺔ اﻷﺳ ّﻴﺔ )اﳊﻘﻴﻘﻴﺔ( ‪ g(t) = e‬ﲢﻘﻖ ‪ g (t) = 𝛼g(t) :‬و ) ‪. g((t + t ) = g (t ) g (t‬‬
‫‪. f(t) = e +‬‬ ‫ﻫﺬا اﻟﺘﺸﺎﺑﻪ ﺑﲔ ﺧﻮاص ‪ f‬و ‪ g‬دﻓﻊ ﺑﺎﻟﻌﺎﱂ اﻟﺮﻳﺎﴈ ﺃﻭﻟﺮ )‪ (E‬إﱃ وﺿﻊ ‪:‬‬
‫‪ņőƱŀ Ţ‬‬
‫إذن ‪:‬‬

‫إذا ﻛﺎن 𝛽‪ u = 𝛼 + ı‬ﻋﺪ ًدا ﻣﺮﻛﺒﺎ‪ ،‬ﻧﻀﻊ ‪:‬‬ ‫ﺗﻌﺮﻳﻒ ‪: 11‬‬


‫‪e =e‬‬ ‫‪+‬‬ ‫= )𝛽 ‪= e (cos 𝛽 + ı sin‬‬ ‫‪eRe‬‬ ‫)))‪(cos (Im (u)) + ı sin (Im (u‬‬
‫ﺗﺴﻤﻰ ﻫﺬه اﻟﻌﺒﺎرة اﻷﺳﻴﺔ اﳌﺮﻛﺒﺔ ﻟﻠﻌﺪد ‪ u‬و ﻟﺪﻳﻨﺎ ‪:‬‬
‫‪||e || = eRe‬‬ ‫‪,‬‬ ‫)𝜋‪arg (e ) = Im (u) (mod 2‬‬
‫‪ƕ‬‬

‫‪. z = |z| (cos (arg (z)) + ı sin (arg (z))) = |z| e arg‬‬ ‫إذا ﻛﺎن ‪ z‬ﻋﺪد ًاﻣﺮﻛﺒﺎ ﻏﲑ ﻣﻌﺪوم ّ‬
‫ﻓﺈن ‪:‬‬

‫ﺗﻌﺮﻳﻒ ‪: 12‬‬
‫‪ũŏ‬‬

‫ﻟﻴﻜﻦ ‪ z‬ﻋﺪد ًا ﻣﺮﻛﺒﺎ ﻏﲑ ﻣﻌﺪوم‪ .‬اﻟﻜﺘﺎﺑﺔ‬


‫‪z = |z| e‬‬ ‫‪arg‬‬

‫ﺗﺴﻤﻰ اﻟﺸﻜﻞ اﻷُﳼ ﻟﻠﻌﺪد اﳌﺮﻛﺐ ‪. z‬‬


‫ﺑﺸﻜﻞ ﺧﺎص‪ ،‬ﻛﻞ ﻋﺪد ﻣﺮﻛﺐ ﻃﻮﻳﻠﺘﻪ ‪ُ 1‬ﻳﻜﺘﺐ ﻋﲆ اﻟﺸﻜﻞ ‪ e‬ﺣﻴﺚ ‪ t‬ﻋﺪد ﺣﻘﻴﻘﻲ‪.‬‬

‫𝟳𝟯‬

‫‪http ://tinyurl.com/Malki1718‬‬ ‫‪0‬‬


‫‪ .8.II‬ادلاةل اﻷﺳـﻴﺔ اﳌﺮﻛﺒﺔ‬
‫‪.‬‬
‫ﺃﻣﺜﻠﺔ ‪: 19‬‬

‫‪Ŕž Ŧ Ľ‬‬
‫‪1+ı‬‬
‫‪−1 = e ,‬‬ ‫‪ı=e‬‬ ‫‪,‬‬ ‫‪=e‬‬ ‫‪,‬‬ ‫‪√3 + ı = 2e‬‬ ‫‪,‬‬ ‫‪1 + ı√3 = 2e‬‬
‫‪√2‬‬

‫‪œǃ‬‬
‫ﻳﻤﻜﻦ أن ﻧﺜﺒﺖ ﺑﺴﻬﻮﻟﺔ اﳋﻮاص اﻟﺘﺎﻟﻴﺔ ) ‪ 𝜃 ∈ ℝ ، 𝜌 > 0‬و ‪: ( n ∈ ℤ‬‬
‫‪𝜌 e‬‬ ‫‪𝜌 e‬‬ ‫𝜌 𝜌 =‬ ‫‪e‬‬ ‫‪+‬‬

‫‪1‬‬ ‫‪1‬‬
‫‪= e−‬‬

‫‪Ŀ‬‬
‫‪𝜌e‬‬ ‫𝜌‬

‫‪II‬‬
‫‪𝜌e‬‬ ‫‪=𝜌 e‬‬
‫‪−𝜌e‬‬ ‫‪= 𝜌e‬‬ ‫‪+‬‬

‫ﻷﺟﻞ ﻛﻞ ﻋﺪدﻳﻦ ﻣﺮﻛﺒﲔ ‪ z‬و ‪ z‬ﻟﺪﻳﻨﺎ اﻟﺘﻜﺎﻓﺆ اﻵﰐ ‪:‬‬ ‫ﻣﺒﺮﻫﻨﺔ ‪: 15‬‬
‫‪′‬‬
‫‪e =e‬‬ ‫‪⟺ z = z + 2k𝜋ı ,‬‬ ‫‪k∈ℤ‬‬

‫ﺍﻟﱪﻫﺎﻥ‪ .‬ﺑﺪاﻳ ًﺔ‪ ،‬ﻟﻨﺒﺤﺚ ﻋﻦ ﺣﻠﻮل اﳌﻌﺎدﻟﺔ ‪ . e = 1 :‬ﻧﻀﻊ ‪ u = a + ıb‬ﺣﻴﺚ ‪ a‬و ‪ b‬ﻋﺪدان ﺣﻘﻴﻘﻴﺎن‪.‬‬
‫)ﻷن ‪ a‬ﻋﺪد ﺣﻘﻴﻘﻲ و اﻟﺪاﻟﺔ اﻷﺳﻴﺔ اﳊﻘﻴﻘﻴﺔ َﺗ َﻘﺎ ُﺑﻞ ﻣﻦ ‪ ℝ‬ﻧﺤﻮ [∞‪ . ( ]0, +‬ﺑﺎﻹﺿﺎﻓﺔ‬
‫ﻟﺪﻳﻨﺎ ‪ 1 = ||e || = e :‬ﻣﻨﻪ ‪ّ a = 0‬‬
‫إﱃ ذﻟﻚ ‪ arg (1) = 0 :‬و )𝜋‪ arg (e ) = b (mod 2‬ﻣﻨﻪ )𝜋‪ b = 0 (mod 2‬أي ‪ b‬ﻣﻦ اﻟﺸﻜﻞ 𝜋‪ 2k‬ﺣﻴﺚ ‪. k ∈ ℤ‬‬
‫أن 𝜋‪. u = 0 + bı = 2ık‬‬‫أي ّ‬
‫‪.e = e‬‬ ‫و ﺑﺎﻟﻌﻜﺲ‪ ،‬إذا ﻛﺎن 𝜋‪ّ u = 2ık‬‬
‫ﻓﺈن ‪= cos (2k𝜋) + ı sin (2k𝜋) = 1‬‬
‫أن اﻷﻋﺪاد اﳌﺮﻛﺒﺔ اﻟﺘﻲ ﲢﻘﻖ ‪ e = 1‬ﻫﻲ اﻷﻋﺪاد اﻟﺘﻲ ﻣﻦ اﻟﺸﻜﻞ ‪. u = 2ık𝜋 , k ∈ ℤ‬‬ ‫ﻫﺬا ﻳﻌﻨﻲ ّ‬
‫اﻵن ‪:‬‬
‫‪′‬‬ ‫‪′‬‬ ‫‪′−‬‬
‫‪e =e‬‬ ‫‪⟺ e e− = e e−‬‬ ‫‪⟺ e‬‬ ‫‪=e‬‬ ‫‪−‬‬ ‫‪= e = 1 ⟺ z − z = 2k𝜋ı ,‬‬ ‫‪k∈ℤ‬‬

‫■‬
‫‪ņőƱŀ Ţ‬‬
‫‪. (e ) = e‬‬ ‫ّ‬
‫ﻓﺈن ‪:‬‬ ‫‪e +‬‬ ‫‪= e (cos 𝛽 − sin 𝛽) = e −‬‬ ‫و ﺑﲈ ّ‬
‫أن‬
‫ﻧﺺ اﻟﻨﻈﺮﻳﺔ ‪ 1‬ﺻﻔﺤﺔ ‪ 10‬ﻟﻴﺸﻤﻞ اﻷﺳﻴﺔ اﳌﺮﻛﺒﺔ ﻛﲈ ﻳﲇ ‪:‬‬
‫إذن ﻳﻤﻜﻦ ﺗﻌﻤﻴﻢ ّ‬

‫ﻧﻈﺮﻳﺔ ‪ : 2‬ﻹﳚﺎد ﻣﺮاﻓﻖ ﻋﺪد ﻣﺮﻛﺐ ﻣﻜﺘﻮب ﻋﲆ ﺻﻴﻐﺔ ﺗﺴﻠﺴﻞ ﻋﻤﻠﻴﺎت اﳉﻤﻊ‪ ،‬اﻟﻄﺮح‪ ،‬اﻟﴬب‪ ،‬اﻟﻘﺴﻤﺔ و اﻷُس ‪،‬‬
‫ﻣﺮة‪.‬‬
‫ﻳﻜﻔﻲ اﻹﺣﺘﻔﺎظ ﺑﻨﻔﺲ اﻟﺼﻴﻐﺔ ﻣﻊ ﺗﻌﻮﻳﺾ اﻟﻌﺪد ‪ ı‬ﺑﺎﻟﻌﺪد ‪ −ı‬ﰲ ﻛﻞ ّ‬
‫‪ƕ‬‬

‫ﰲ اﳊﺎﻟﺔ اﳋﺎﺻﺔ ‪ ، u = ıt‬ﻣﻊ ‪ t‬ﻋﺪد ﺣﻘﻴﻘﻲ‪ ،‬ﻳﻜﻮن ‪:‬‬


‫‪1‬‬
‫‪||e || = 1‬‬ ‫‪,‬‬ ‫‪arg e‬‬ ‫‪=t‬‬ ‫‪,‬‬ ‫‪e−‬‬ ‫= ) ‪= (e‬‬
‫‪e‬‬
‫‪ũŏ‬‬

‫و اﻵن ﻧﺪرج ﻧﺺ اﳌﱪﻫﻨﺔ اﻷﺳﺎﺳﻴﺔ ﰲ ﻫﺬه اﻟﻔﻘﺮة و اﻟﺘﻲ ُﻳﻄﻠﻖ ﻋﻠﻴﻬﺎ إﺳﻢ ﻧﻈﺮﻳﺔ ﺃﻭﻟﺮ أو ِﺻ َﻴﻎ ﺃﻭﻟﺮ ‪:‬‬

‫ﻟﻴﻜﻦ ‪ t‬ﻋﺪد ًا ﺣﻘﻴﻘﻴﺎ ‪ .‬ﻟﺪﻳﻨﺎ ‪:‬‬ ‫ﻣﺒﺮﻫﻨﺔ ‪: 16‬‬


‫‪e +‬‬ ‫‪e−‬‬ ‫‪e −‬‬ ‫‪e−‬‬
‫= ‪cos t‬‬ ‫‪,‬‬ ‫= ‪sin t‬‬
‫‪2‬‬ ‫‪2ı‬‬

‫𝟴𝟯‬

‫‪http ://tinyurl.com/Malki1718‬‬ ‫‪0‬‬


‫‪ .II‬اﻟﺸﲁ اﳌﺜﻠﱻ ﻟﻌﺪد ﻣﺮﻛﺐ‬
‫‪.‬‬
‫ﺍﻟﱪﻫﺎﻥ‪ .‬ﻟﺪﻳﻨﺎ ‪:‬‬
‫‪e‬‬ ‫‪= cos t + ı sin t‬‬ ‫)𝟔(‬

‫‪Ŕž Ŧ Ľ‬‬
‫‪e−‬‬ ‫‪= cos t − ı sin t‬‬ ‫)𝟕(‬
‫ﺑﺠﻤﻊ ﻃﺮﰲ اﳌﻌﺎدﻟﺘﲔ )𝟔( و )𝟕( ﻳﻨﺘﺞ ‪. e + e− = 2 cos t :‬‬

‫‪œǃ‬‬
‫■‬ ‫و ﺑﻄﺮﺣﻬﲈ ﻃﺮﻓ ًﺎ إﱃ ﻃﺮف ﻧﺠﺪ ‪. e − e− = 2ı sin t :‬‬
‫‪ e‬ﻹﳚﺎد ﲨﻴﻊ‬ ‫‪= ıe‬‬ ‫‪e‬و‬ ‫‪= 1،e‬‬ ‫‪+‬‬ ‫‪=e e‬‬ ‫أن‬ ‫ِ‬
‫ﻟﺼ َﻴﻎ ﺃﻭﻟﺮ أﳘﻴﺔ ﻛﺒﲑة ﻓﻤﺜﻼً‪ ،‬ﻳﻜﻔﻲ أن ﻧﻌﻠﻢ ّ‬

‫‪Ŀ‬‬
‫اﳌﺘﻄﺎﺑﻘﺎت اﳌﺜﻠﺜﻴﺔ ‪ ،‬ﻣﺜ ً‬
‫ﻼ ﻹﳚﺎد ‪ cos x cos y‬ﻧﻜﺘﺐ ‪:‬‬
‫‪II‬‬

‫‪e‬‬ ‫‪+ e−‬‬ ‫‪e‬‬ ‫‪+ e−‬‬ ‫‪e‬‬ ‫‪+‬‬ ‫‪+e‬‬ ‫‪−‬‬ ‫‪+e‬‬ ‫‪− +‬‬ ‫‪+e‬‬ ‫‪− −‬‬
‫×‬ ‫=‬
‫‪2‬‬ ‫‪2‬‬ ‫‪4‬‬
‫‪1‬‬ ‫‪e‬‬ ‫‪+‬‬ ‫‪+ e−‬‬ ‫‪+‬‬ ‫‪e‬‬ ‫‪−‬‬ ‫‪+ e−‬‬ ‫‪−‬‬
‫=‬ ‫‪+‬‬
‫‪2‬‬ ‫‪2‬‬ ‫‪2‬‬
‫‪1‬‬
‫= ‪. cos x cos y‬‬ ‫)‪cos (x + y) + cos (x − y‬‬ ‫ﻣﻨﻪ‬
‫‪2‬‬
‫ﺑﺎﳌﺜﻞ‪ ،‬ﻹﳚﺎد ‪ cos x sin y‬ﻧﻜﺘﺐ ‪:‬‬
‫‪e‬‬ ‫‪+ e−‬‬ ‫‪e‬‬ ‫‪− e−‬‬ ‫‪e‬‬ ‫‪+‬‬ ‫‪−e‬‬ ‫‪−‬‬ ‫‪+e‬‬ ‫‪− +‬‬ ‫‪−e‬‬ ‫‪− −‬‬
‫×‬ ‫=‬
‫‪2‬‬ ‫‪2ı‬‬ ‫‪4ı‬‬
‫‪1‬‬ ‫‪e‬‬ ‫‪+‬‬ ‫‪− e−‬‬ ‫‪+‬‬ ‫‪e‬‬ ‫‪−‬‬ ‫‪− e−‬‬ ‫‪−‬‬
‫=‬ ‫‪−‬‬
‫‪2‬‬ ‫‪2ı‬‬ ‫‪2ı‬‬
‫‪1‬‬
‫= ‪. cos x sin y‬‬ ‫)‪sin (x + y) − sin (x − y‬‬ ‫ﻣﻨﻪ‬
‫‪2‬‬
‫و أﻳﻀﺎ ﺑﺎﻟﻨﺴﺒﺔ ﻟـِ ‪: sin x sin y‬‬
‫‪e‬‬ ‫‪− e−‬‬ ‫‪e‬‬ ‫‪− e−‬‬ ‫‪e‬‬ ‫‪+‬‬ ‫‪−e‬‬ ‫‪−‬‬ ‫‪−e‬‬ ‫‪− +‬‬ ‫‪+e‬‬ ‫‪− −‬‬
‫×‬ ‫=‬
‫‪2ı‬‬ ‫‪2ı‬‬ ‫‪−4‬‬
‫‪1‬‬ ‫‪e‬‬ ‫‪−‬‬ ‫‪+ e−‬‬ ‫‪−‬‬ ‫‪e‬‬ ‫‪+‬‬ ‫‪+ e−‬‬ ‫‪+‬‬
‫=‬ ‫‪−‬‬
‫‪2‬‬ ‫‪2‬‬ ‫‪2‬‬
‫‪ņőƱŀ Ţ‬‬
‫‪1‬‬
‫)‪. sin x sin y = cos (x − y) − cos (x + y‬‬ ‫ﻣﻨﻪ‬
‫‪2‬‬
‫ﻛﲈ ﻳﻤﻜﻦ إﺛﺒﺎت دﺳﺘﻮر ﻣﻮﺍﻓﺮ ﺑﺎﻹﻋﺘﲈد ﻋﻠﻴﻬﺎ ‪:‬‬
‫)‪. cos nt + ı sin nt = (cos t + ı sin t‬‬ ‫‪ e = e‬ﻣﻨﻪ‬

‫|‪x − y‬‬
‫‪. ||e − e || = 2 |||sin‬‬ ‫إذا ﻛﺎن ‪ x‬و ‪ y‬ﻋﺪدﻳﻦ ﺣﻘﻴﻘﻴﲔ ّ‬
‫ﻓﺈن‬ ‫ﻣﺒﺮﻫﻨﺔ ‪: 17‬‬
‫|| ‪2‬‬

‫‪||e‬‬ ‫‪− e || = ||e || ⋅ ||e‬‬ ‫‪−‬‬ ‫‪ e − e = e‬ﻣﻨﻪ ||‪− 1‬‬ ‫‪e‬‬ ‫ﺍﻟﱪﻫﺎﻥ‪ .‬ﻟﺪﻳﻨﺎ ‪− 1‬‬
‫‪−‬‬
‫‪ƕ‬‬

‫‪. ||e − e || = ||e −‬‬ ‫ﻟﻜﻦ ‪ ||e || = 1‬ﻣﻨﻪ ||‪− 1‬‬


‫ﻧﻀﻊ ‪ . 𝜃 = x − y‬ﺣﺴﺐ ﺗﻌﺮﻳﻒ اﻟﻄﻮﻳﻠﺔ ‪:‬‬
‫‪||e − 1|| = e − 1‬‬ ‫‪e −1 = e −1‬‬ ‫‪e− − 1‬‬
‫‪ũŏ‬‬

‫‪= e e− − e + e−‬‬ ‫‪+1‬‬


‫‪= 1 − 2 Re e‬‬ ‫𝜃 ‪+ 1 = 2 − 2 cos‬‬
‫𝜃‬ ‫𝜃‬
‫ﻓﺈن ‪. ||e − 1|| = 4 sin‬‬ ‫أن ‪ّ 1 − cos 𝜃 = 2 sin‬‬ ‫و ﺑﲈ ّ‬
‫‪2‬‬ ‫‪2‬‬
‫|‪x − y‬‬
‫■‬ ‫‪. ||e − e || = 2 |||sin‬‬ ‫و ﺑﺄﺧﺬ اﳉﺬر اﻟﱰﺑﻴﻌﻲ ﻟﻠﻄﺮﻓﲔ ﻳﻨﺘﺞ ‪|| :‬‬
‫‪2‬‬

‫𝟵𝟯‬

‫‪http ://tinyurl.com/Malki1718‬‬ ‫‪0‬‬


.

Ŕž Ŧ Ľ
œǃ
Ŀ
II
ņőƱŀ Ţ
ũŏ ƕ

http ://tinyurl.com/Malki1718 0
‫‪.‬‬

‫‪3‬‬
‫‪Ŕž Ŧ Ľ‬‬
‫‪œǃ‬‬
‫‪otttttttttttttttttttttttttttttttttttttttttttp‬‬
‫‪s‬‬ ‫‪u‬‬
‫‪s‬‬ ‫ﺗﻄﺒﻴﻘﺎت اﻷﻋﺪاد اﻟﻤﺮﻛﺒﺔ‬
‫‪u‬‬

‫‪Ŀ‬‬
‫‪s‬‬ ‫‪u‬‬
‫‪qvvvvvvvvvvvvvvvvvvvvvvvvvvvvvvvvvvvvvvvvvvvr‬‬
‫‪.‬‬
‫‪III‬‬

‫ﺍﳉﺬﻭﺭ ﺍﻟﺘﺮﺑﻴﻌﻴﺔ ﻟﻌﺪﺩ ﻣﺮﻛﺐ‬ ‫‪1.III‬‬


‫ﰲ ﻫﺬه اﻟﻔﻘﺮة‪ ،‬ﻧﺪرس اﳊﺎﻟﺔ اﳋﺎﺻﺔ ‪ n = 2‬ﻟﻠﺠﺬور اﻟﻨﻮﻧﻴﺔ ﻟﻌﺪد ﻣﺮﻛﺐ و اﻟﺘﻲ ﲤّﺖ دراﺳﺘﻬﺎ ﰲ اﻟﻔﻘﺮة ‪ 5.II‬ﺻﻔﺤﺔ ‪. 26‬‬

‫‪.z = L‬‬ ‫ﺟﺬرا ﺗﺮﺑﻴﻌ ًﻴﺎ ﻟﻠﻌﺪد اﳌﺮﻛﺐ ‪ L‬ﻛﻞ ﻋﺪد ﻣﺮﻛﺐ ‪ z‬ﳛﻘﻖ ‪:‬‬
‫ﻧﺴﻤﻲ ً‬ ‫ﺗﻌﺮﻳﻒ ‪: 13‬‬

‫ﺟﺬرا ﻧﻮﻧ ًﻴﺎ‪ .‬إذن ﻛﻞ ﻋﺪد ﻣﺮﻛﺐ ﻏﲑ ﻣﻌﺪوم ‪ L‬ﻳﻘﺒﻞ ﺟﺬرﻳﻦ ﺗﺮﺑﻴﻌﻴﲔ‪ ،‬و إذا ﻛﺎن‬
‫أن ﻛﻞ ﻋﺪد ﻣﺮﻛﺐ ﻏﲑ ﻣﻌﺪوم ﻳﻘﺒﻞ ‪ً n‬‬ ‫ﻧﻌﻠﻢ ّ‬
‫ﻓﺈن ) ‪ (−z‬ﻫﻮ أﻳﻀﺎ ﺟﺬر ﺗﺮﺑﻴﻌﻲ ﻟﻪ ّ‬
‫ﻷن ‪ (−z ) = L‬و ﻣﻨﻪ اﳌﱪﻫﻨﺔ اﻟﺘﺎﻟﻴﺔ ‪:‬‬ ‫ﺟﺬرا ﺗﺮﺑﻴﻌ ًﻴﺎ ﻟﻠﻌﺪد ‪ّ L‬‬
‫‪ً z‬‬

‫ﻛﻞ ﻋﺪد ﻣﺮﻛﺐ ﻏﲑ ﻣﻌﺪوم ﻳﻘﺒﻞ ﺟﺬرﻳﻦ ﺗﺮﺑﻴﻌﻴﲔ ﻣﺘﻨﺎﻇﺮﻳﻦ‪.‬‬ ‫ﻣﺒﺮﻫﻨﺔ ‪: 18‬‬
‫‪ņőƱŀ Ţ‬‬
‫ﺗﻌﻴﲔ ﺍﳉﺬﻭﺭ ﺍﻟﺘﺮﺑﻴﻌﻴﺔ ﻟﻌﺪﺩ ﻣﺮﻛﺐ‬ ‫‪2.III‬‬
‫ﻟﻴﻜﻦ ‪ L‬ﻋﺪ ًدا ﻣﺮﻛ ًﺒﺎ ﺣﻴﺚ ‪. L = a + ıb = 𝜌 (cos 𝜃 + ı sin 𝜃) :‬‬
‫ﻟﻠﺒﺤﺚ ﻋﻦ اﳉﺬرﻳﻦ اﻟﱰﺑﻴﻌﻴﲔ ﻟﻠﻌﺪد اﳌﺮﻛﺐ ‪ ، L‬ﻳﻤﻜﻦ أن ﻧﺴﺘﻌﻤﻞ إ ّﻣﺎ ﺷﻜﻠﻪ اﳌﺜﻠﺜﻲ ]𝜃 ‪ L = [𝜌,‬أو ﺷﻜﻠﻪ اﳉﱪي = ‪L‬‬
‫‪. a + ıb‬‬

‫ﺍﻟﻄﺮﻳﻘﺔ ﺍﻷﻭﱃ ‪ :‬ﺑﺎﺳﺘﻌﻤﺎﻝ ﺍﻟﺸﻜﻞ ﺍﳌﺜﻠﺜﻲ‬ ‫‪1.2.III‬‬


‫‪ƕ‬‬

‫ﻧﺘّﺒﻊ ﻓﻴﲈ ﻳﲇ ﻧﻔﺲ ﻃﺮﻳﻘﺔ اﻟﻔﻘﺮة ‪ 5.II‬ﺻﻔﺤﺔ ‪ 26‬و ﻳﻤﻜﻦ ﺗﻄﺒﻴﻖ اﻟﻨﺘﻴﺠﺔ اﻟﺘﻲ وﺟﺪﻧﺎﻫﺎ ﻓﻴﻬﺎ‪ ،‬ﻟﻜﻦ ﻟﱰﺳﻴﺦ ﻫﺬه اﻟﻄﺮﻳﻘﺔ ﰲ‬
‫اﻷذﻫﺎن ﻧﻘﻮم ﺑﺈﻋﺎدﲥﺎ ﻫﻨﺎ ‪:‬‬
‫‪ũŏ‬‬

‫‪z = [r, t] = r , 2t‬‬ ‫ﻟﻨﺒﺤﺚ ﻋﻦ اﻷﻋﺪاد اﳌﺮﻛﺒﺔ ‪ z‬اﻟﺘﻲ ﲢﻘﻖ ‪ . z = L‬ﻧﻀﻊ ]‪ z = [r, t‬و ﻣﻨﻪ ‪:‬‬
‫ﻟﺪﻳﻨﺎ ‪:‬‬
‫)𝜃 ‪z = L ⟺ r (cos (2t) + ı sin (2t)) = 𝜌 (cos 𝜃 + ı sin‬‬

‫𝜌= ‪r‬‬ ‫𝜌√ = ‪⎧ r‬‬


‫⟺‬ ‫⟺‬
‫‪2t = 𝜃 + 2k𝜋 ,‬‬ ‫‪k∈ℤ‬‬ ‫‪⎨ t = 𝜃 + k𝜋 ,‬‬ ‫‪k∈ℤ‬‬
‫⎩‬ ‫‪2‬‬

‫𝟭𝟰‬

‫‪http ://tinyurl.com/Malki1718‬‬ ‫‪0‬‬


‫‪ .2.III‬ﺗﻌﻴﲔ اﳉﺬور اﻟﱰﺑﻴﻌﻴﺔ ﻟﻌﺪد ﻣﺮﻛﺐ‬
‫‪.‬‬
‫ﻓﺎﳉﺬران اﻟﱰﺑﻴﻌﻴﺎن ﻟﻠﻌﺪد ‪ L‬ﳘﺎ ‪:‬‬
‫𝜃‬ ‫𝜃‬ ‫𝜃‬ ‫𝜃‬

‫‪Ŕž Ŧ Ľ‬‬
‫‪z = √r cos‬‬ ‫‪+ ı sin‬‬ ‫و‬ ‫‪z = √r cos‬‬ ‫‪+ 𝜋 + ı sin‬‬ ‫𝜋‪+‬‬ ‫‪= −z‬‬
‫‪2‬‬ ‫‪2‬‬ ‫‪2‬‬ ‫‪2‬‬
‫𝜋‬
‫‪ z = 1 + ı√3 = 2,‬ﳘﺎ ‪:‬‬ ‫ﻣﺜﺎﻝ ‪ : 20‬اﳉﺬران اﻟﱰﺑﻴﻌﻴﺎن ﻟﻠﻌﺪد‬

‫‪œǃ‬‬
‫‪3‬‬
‫𝜋‬ ‫𝜋‬ ‫𝜋‬ ‫‪√3 1‬‬ ‫‪√2‬‬
‫‪z = √2,‬‬ ‫‪= √2 cos + ı sin‬‬ ‫‪= √2‬‬ ‫= ‪+ ı‬‬ ‫‪√3 + ı‬‬
‫‪6‬‬ ‫‪6‬‬ ‫‪6‬‬ ‫‪2‬‬ ‫‪2‬‬ ‫‪2‬‬
‫و‬

‫‪Ŀ‬‬
‫‪√2‬‬
‫‪z = −z = −‬‬ ‫‪√3 + ı‬‬
‫‪2‬‬

‫ﺍﻟﻄﺮﻳﻘﺔ ﺍﻟﺜﺎﻧﻴﺔ ‪ :‬ﺑﺎﺳﺘﻌﻤﺎﻝ ﺍﻟﺸﻜﻞ ﺍﳉﱪﻱ‬ ‫‪2.2.III‬‬

‫ﻧﻀﻊ ‪ z = x + ıy‬ﺣﻴﺚ ‪ . (x, y) ∈ ℝ‬ﻟﺪﻳﻨﺎ ‪:‬‬

‫‪III‬‬
‫‪z = L ⟺ (x + ıy) = a + ıb‬‬
‫⟺‬ ‫‪x −y‬‬ ‫‪+ (2xy) ı = a + ıb‬‬
‫‪x −y =a‬‬ ‫)𝟏(‬
‫⟺‬
‫‪2xy = b‬‬ ‫)𝟐(‬

‫و ﻫﻲ ﲨﻠﺔ ﻣﻌﺎدﻟﺘﲔ ﺑﻤﺠﻬﻮﻟﲔ ﻓﺒﺎﻹﻣﻜﺎن ﺣ ﱡﻠﻬﺎ‪.‬‬


‫اﻟﻨﺎﲡﺔ ﻣﻦ‬ ‫= ‪x +y‬‬ ‫‪a +b‬‬ ‫ﻟﻜﻦ ﻟﺘﺴﻬﻴﻞ ّ‬
‫ﺣﻞ ﻫﺬه اﳉﻤﻠﺔ‪ ،‬ﺗُﻀﺎف ﻋﺎد ًة إﱃ اﳌﻌﺎدﻟﺘﲔ )𝟏( و )𝟐( اﳌﻌﺎدﻟﺔ‬
‫||‪ ||z || = ||L‬و ﺗﺼﺒﺢ ﻋﻨﺪﺋﺬ اﳉﻤﻠﺔ ﻛﲈ ﻳﲇ ‪:‬‬

‫‪⎧x − y = a‬‬
‫⎪‬
‫)𝟏(‬
‫⎪‬
‫‪2xy = b‬‬ ‫)𝟐(‬
‫⎨‬
‫‪ņőƱŀ Ţ‬‬
‫⎪‬
‫⎪‬
‫‪⎩x + y = a + b‬‬ ‫)𝟑(‬

‫‪a +b +a‬‬
‫= ‪ ، x‬و ﺑﻄﺮح اﳌﻌﺎدﻟﺔ اﻷوﱃ ﻣﻦ اﻟﺜﺎﻟﺜﺔ ﻧﺠﺪ‬ ‫ﺑﺠﻤﻊ اﳌﻌﺎدﻟﺘﲔ اﻷوﱃ و اﻟﺜﺎﻟﺜﺔ ﻃﺮﻓﺎ إﱃ ﻃﺮف ﻳﻨﺘﺞ‬
‫‪2‬‬
‫‪a +b −a‬‬
‫= ‪.y‬‬
‫‪2‬‬
‫‪a +b −a‬‬ ‫‪a +b +a‬‬
‫و ﺑﺎﻟﺘﺎﱄ ‪:‬‬ ‫و‪≥ 0‬‬ ‫ﻣﻦ اﻟﻮاﺿﺢ ّ‬
‫أن ‪≥ 0‬‬
‫‪2‬‬ ‫‪2‬‬
‫‪ƕ‬‬

‫⎧‬
‫⎪‬ ‫‪a +b +a‬‬
‫⎪‬
‫⎪‬
‫⎪‬
‫⎪‬ ‫‪x=±‬‬
‫⎪‬
‫⎪‬ ‫‪2‬‬
‫‪2xy = b‬‬
‫⎨‬
‫⎪‬
‫‪ũŏ‬‬

‫⎪‬
‫⎪‬
‫⎪‬
‫⎪‬ ‫‪a +b −a‬‬
‫⎪‬
‫⎪‬ ‫‪y=±‬‬
‫⎩‬ ‫‪2‬‬

‫ﻧﺨﺘﺎر ‪ x‬و ‪ y‬ﺑﺤﻴﺚ ﺗﻜﻮن إﺷﺎرة اﳉﺪاء ‪ xy‬ﻣﻦ ﻧﻔﺲ إﺷﺎرة ‪ b‬أي ‪:‬‬

‫‪a +b −a‬‬ ‫‪a +b +a‬‬


‫= ‪y‬؛‬ ‫= ‪x‬و‬ ‫‪ •1‬إذا ﻛﺎن ‪ ، b > 0‬ﻧﺨﺘﺎر‬
‫‪2‬‬ ‫‪2‬‬

‫𝟮𝟰‬

‫‪http ://tinyurl.com/Malki1718‬‬ ‫‪0‬‬


‫‪ .III‬ﺗﻄﺒﻴﻘﺎت اﻷﻋﺪاد اﳌﺮﻛﺒﺔ‬
‫‪.‬‬
‫‪a +b −a‬‬ ‫‪a +b +a‬‬
‫‪.y = −‬‬ ‫= ‪x‬و‬ ‫‪ •2‬و إذا ﻛﺎن ‪ ، b < 0‬ﻧﺨﺘﺎر‬
‫‪2‬‬ ‫‪2‬‬

‫‪Ŕž Ŧ Ľ‬‬
‫ﰲ ﻛﻼ اﳊﺎﻟﺘﲔ‪ ،‬اﳉﺬران اﻟﱰﺑﻴﻌﻴﺎن ﻟﻠﻌﺪد ‪ L‬ﳘﺎ )‪. ± (x + ıy‬‬

‫‪œǃ‬‬
‫‪ •1‬ﻹﳚﺎد اﳉﺬرﻳﻦ اﻟﱰﺑﻴﻌﻴﲔ ﻟﻠﻌﺪد ‪ ، L = 5 − 12ı‬ﻧﻀﻊ ‪ z = x + ıy‬و ّ‬
‫ﻧﺤﻞ ‪:‬‬ ‫ﺃﻣﺜﻠﺔ ‪: 21‬‬

‫⎧‬ ‫‪x −y =5‬‬


‫⎪‬
‫⎪‬

‫‪Ŀ‬‬
‫⟺ ‪(x + ıy) = 5 − 12ı‬‬ ‫‪2xy = −12‬‬
‫⎨‬
‫⎪‬
‫⎪‬
‫‪⎩ x + y = 5 + (−12) = 13‬‬

‫ﺑﺠﻤﻊ ﺣﺪود اﳌﻌﺎدﻟﺘﲔ اﻷوﱃ و اﻟﺜﺎﻟﺜﺔ ﻃﺮﻓﺎ إﱃ ﻃﺮف ﻧﺤﺼﻞ ﻋﲆ ‪ 2x = 18 :‬أي ‪. x = ±3‬‬

‫ﻣﻦ أﺟﻞ ‪ x = −3‬و ﺑﺎﻟﺘﻌﻮﻳﺾ ﰲ اﳌﻌﺎدﻟﺔ اﻟﺜﺎﻧﻴﺔ ﻧﺤﺼﻞ ﻋﲆ ‪. y = 2‬‬ ‫•‬


‫‪III‬‬

‫ﻣﻦ أﺟﻞ ‪ x = 3‬و ﺑﺎﻟﺘﻌﻮﻳﺾ ﰲ اﳌﻌﺎدﻟﺔ اﻟﺜﺎﻧﻴﺔ ﻧﺤﺼﻞ ﻋﲆ ‪. y = −2‬‬ ‫•‬

‫إذن اﳉﺬران اﻟﱰﺑﻴﻌﻴﺎن ﻟﻠﻌﺪد ‪ L = 5 − 12ı‬ﳘﺎ ‪ z = −3 + 2ı‬و ‪. z = 3 − 2ı‬‬


‫‪ •2‬ﻟﻴﻜﻦ ‪ً z = x + ıy‬‬
‫ﺟﺬرا ﺗﺮﺑﻴﻌ ًﻴﺎ ﻟﻠﻌﺪد ‪ . L = ı‬ﻟﺪﻳﻨﺎ ‪ 2xy = 1 ، x − y = 0‬و ‪ x + y = 1‬و ﺑﺎﻟﺘﺎﱄ‬
‫‪2‬‬ ‫‪2‬‬ ‫‪2‬‬ ‫‪2‬‬ ‫‪1‬‬
‫= ‪ x = y‬؛ و ‪ x‬و ‪ y‬ﻣﻦ ﻧﻔﺲ اﻹﺷﺎرة‪ ،‬إذن ﻓﺎﳉﺬران اﻟﱰﺑﻴﻌﻴﺎن ﻟﻠﻌﺪد ‪ ı‬ﳘﺎ √ ‪ √ + ı‬و √ ‪. − √ − ı‬‬
‫‪2‬‬ ‫‪2‬‬ ‫‪2‬‬ ‫‪2‬‬ ‫‪2‬‬
‫‪ •3‬ﻹﳚﺎد اﳉﺬرﻳﻦ اﻟﱰﺑﻴﻌﻴﲔ ﻟﻠﻌﺪد ‪ −ı‬ﻧﻼﺣﻆ ّ‬
‫أن ‪:‬‬

‫‪√2‬‬ ‫‪√2‬‬ ‫‪√2‬‬ ‫‪√2‬‬ ‫‪√2‬‬ ‫‪√2‬‬


‫‪−ı = −‬‬ ‫‪+ı‬‬ ‫‪=ı‬‬ ‫‪+ı‬‬ ‫=‬ ‫‪ı‬‬ ‫‪+ı‬‬
‫‪2‬‬ ‫‪2‬‬ ‫‪2‬‬ ‫‪2‬‬ ‫‪2‬‬ ‫‪2‬‬

‫‪√2 √2‬‬ ‫‪√2‬‬ ‫‪√2‬‬


‫=‬ ‫‪ı‬‬ ‫‪−‬‬ ‫=‬ ‫‪−‬‬ ‫‪+ı‬‬
‫‪ņőƱŀ Ţ‬‬
‫‪2‬‬ ‫‪2‬‬ ‫‪2‬‬ ‫‪2‬‬

‫‪2‬‬ ‫‪2‬‬
‫√‪. ± −√ + ı‬‬ ‫و ﺑﺎﻟﺘﺎﱄ ﻓﺎﳉﺬران اﻟﱰﺑﻴﻌﻴﺎن ﻟﻠﻌﺪد ‪ −ı‬ﳘﺎ‬
‫‪2‬‬ ‫‪2‬‬

‫ﺍﳌﻌﺎﺩﻻﺕ ﻣﻦ ﺍﻟﺪﺭﺟﺔ ﺍﻟﺜﺎﻧﻴﺔ‬ ‫‪3.III‬‬

‫ﺍﳊﻞ ‪ ،‬ﰲ ‪ ، ℂ‬ﳌﻌﺎﺩﻟﺔ ﻣﻦ ﺍﻟﺪﺭﺟﺔ ﺍﻟﺜﺎﻧﻴﺔ‬ ‫‪1.3.III‬‬


‫‪ƕ‬‬

‫ﻧﻌﺘﱪ ﰲ ‪ ℂ‬اﳌﻌﺎدﻟﺔ ‪:‬‬


‫‪az + bz + c = 0‬‬ ‫)𝟒(‬
‫‪ũŏ‬‬

‫ﺣﻴﺚ ‪ c ، b ، a‬أﻋﺪاد ﻣﺮﻛﺒﺔ و ‪. a ≠ 0‬‬


‫ﻣﻦ أﺟﻞ ﻛﻞ ﻋﺪد ﻣﺮﻛﺐ ‪ z‬ﻟﺪﻳﻨﺎ ‪:‬‬
‫‪b‬‬ ‫‪c‬‬
‫‪az + bz + c = a z +‬‬ ‫‪z+‬‬
‫‪a‬‬ ‫‪a‬‬
‫‪b‬‬ ‫‪b‬‬ ‫‪c‬‬
‫‪=a‬‬ ‫‪z+‬‬ ‫‪−‬‬ ‫‪+‬‬
‫‪2a‬‬ ‫‪4a‬‬ ‫‪a‬‬

‫𝟯𝟰‬

‫‪http ://tinyurl.com/Malki1718‬‬ ‫‪0‬‬


‫‪ .3.III‬اﳌﻌﺎدﻻت ﻣﻦ ادلرﺟﺔ اﻟﺜﺎﻧﻴﺔ‬
‫‪.‬‬
‫أي‬
‫‪b‬‬ ‫‪b − 4ac‬‬

‫‪Ŕž Ŧ Ľ‬‬
‫‪az + bz + c = a‬‬ ‫‪z+‬‬ ‫‪−‬‬
‫‪2a‬‬ ‫‪4a‬‬

‫ﻟﻴﻜﻦ 𝛿 أﺣﺪ اﳉﺬرﻳﻦ اﻟﱰﺑﻴﻌﻴﲔ ﻟﻠﻌﺪد ‪ . b − 4ac‬ﻟﺪﻳﻨﺎ ‪:‬‬

‫‪œǃ‬‬
‫‪b‬‬ ‫‪b − 4ac‬‬
‫‪az + bz + c = 0 ⟺ a‬‬ ‫‪z+‬‬ ‫‪−‬‬ ‫‪=0‬‬
‫‪2a‬‬ ‫‪4a‬‬

‫‪Ŀ‬‬
‫‪b‬‬ ‫𝛿‬
‫‪⟺ a‬‬ ‫‪z+‬‬ ‫‪−‬‬ ‫‪=0‬‬
‫‪2a‬‬ ‫‪4a‬‬

‫‪b‬‬ ‫𝛿‬
‫‪⟺ a‬‬ ‫‪z+‬‬ ‫‪−‬‬ ‫‪=0‬‬
‫‪2a‬‬ ‫‪2a‬‬
‫‪b‬‬ ‫𝛿‬ ‫‪b‬‬ ‫𝛿‬
‫⟺‬ ‫‪z+‬‬
‫‪−‬‬ ‫‪z+‬‬ ‫‪+‬‬ ‫‪=0‬‬
‫‪2a 2a‬‬ ‫‪2a 2a‬‬

‫‪III‬‬
‫𝛿 ‪−b −‬‬ ‫𝛿 ‪−b +‬‬
‫=‪⟺ z‬‬ ‫أو‬
‫‪2a‬‬ ‫‪2a‬‬
‫إذن‬

‫𝛿 ‪−b +‬‬ ‫𝛿 ‪−b −‬‬


‫ﺣﻴﺚ 𝛿 ﻫﻮ‬ ‫و‬ ‫اﳌﻌﺎدﻟﺔ ‪ ، az + bz + c = 0‬ﺣﻴﺚ ‪ ، a ≠ 0‬ﺗﻘﺒﻞ ﰲ ‪ ℂ‬ﺣ ّﻠﲔ ﳘﺎ‬ ‫ﻣﺒﺮﻫﻨﺔ ‪: 19‬‬
‫‪2a‬‬ ‫‪2a‬‬
‫أﺣﺪ اﳉﺬرﻳﻦ اﻟﱰﺑﻴﻌﻴﲔ ﻟﻠﻌﺪد ‪. b − 4ac‬‬

‫ﺴﻤﻰ اﻟﻌﺪد ‪ُ b − 4ac‬ﳑ َ ﱢﻴﺰ اﳌﻌﺎدﻟﺔ و ﻧﺮﻣﺰ إﻟﻴﻪ ﺑﺎﻟﺮﻣﺰ ‪ Δ‬ﻛﲈ ﻫﻮ اﳊﺎل ﰲ ﳎﻤﻮﻋﺔ اﻷﻋﺪاد اﳊﻘﻴﻘﻴﺔ ‪. ℝ‬‬
‫ُﻳ ّ‬
‫‪. (1 + ı) z + (3 − 4ı) z + ı − 5 = 0‬‬ ‫ﻣﺜﺎﻝ ‪ : 22‬ﻧﻌﺘﱪ ﰲ ‪ ℂ‬اﳌﻌﺎدﻟﺔ ‪:‬‬
‫ﻟﺪﻳﻨﺎ ‪. Δ = (3 − 4ı) − 4 (1 + ı) (ı − 5) = −3 − 4ı :‬‬
‫ﻟﻴﻜﻦ ‪ 𝛿 = x + ıy‬ﻋﺪ ًدا ﻣﺮﻛﺒﺎ ﺣﻴﺚ ‪ . 𝛿 = Δ‬ﻟﺪﻳﻨﺎ ‪:‬‬
‫‪ņőƱŀ Ţ‬‬
‫‪(x + ıy) = −3 − 4ı ⟺ x − y + 2ıxy = −3 − 4ı‬‬

‫‪⎪ x − y = −3‬‬
‫⎧‬
‫⟺‬ ‫‪2xy = −4‬‬
‫⎨‬
‫⎪‬
‫‪⎩x + y = 5‬‬
‫و ‪(x = 1‬‬ ‫)‪y = −2‬‬
‫⟺‬ ‫أو‬
‫و ‪(x = −1‬‬ ‫)‪y = 2‬‬

‫و ﻣﻨﻪ ‪) 𝛿 = −1 + 2ı‬أو ‪.( 𝛿 = 1 − 2ı‬‬


‫إذن اﳊﻼن ‪ z‬و ‪ z‬ﻟﻠﻤﻌﺎدﻟﺔ اﳌﻘﱰﺣﺔ ﳘﺎ ‪:‬‬
‫‪ƕ‬‬

‫)‪− (3 − 4ı) − (−1 + 2ı‬‬


‫= ‪z‬‬ ‫‪=ı‬‬
‫)‪2 (1 + ı‬‬
‫‪ũŏ‬‬

‫‪− (3 − 4ı) + (−1 + 2ı) 1 5‬‬


‫= ‪z‬‬ ‫‪= + ı‬‬
‫)‪2 (1 + ı‬‬ ‫‪2 2‬‬

‫ﻣﻼﺣﻈﺔ ‪ ⧏ : 13‬إذا ﻛﺎن ‪ b = 2b‬أي إذا ﻛﺎﻧﺖ اﳌﻌﺎدﻟﺔ ‪ az + 2b z + c = 0 :‬ﻓﺈﻧﻨﺎ ﻧ ّ‬


‫ُﻌﺮف ا ُﳌﻤ ﱢﻴﺰ ا ُﳌﺨﺘﴫ‬
‫ﺟﺬرا ﺗﺮﺑﻴﻌ ًﻴﺎ ﻟﻠﻌﺪد ‪ّ Δ‬‬
‫ﻓﺈن ﺣﻠﻮل اﳌﻌﺎدﻟﺔ اﻟﺴﺎﺑﻘﺔ ﺗُﻌﻄﻰ ﺑﺎﻟﻌﺒﺎرة ‪:‬‬ ‫‪ Δ = b − ac‬؛ و إذا ﻛﺎن 𝛿 ً‬
‫𝛿 ‪−b −‬‬ ‫𝛿 ‪−b +‬‬
‫= ‪z‬‬ ‫و‬ ‫= ‪z‬‬
‫‪a‬‬ ‫‪a‬‬
‫𝟰𝟰‬

‫‪http ://tinyurl.com/Malki1718‬‬ ‫‪0‬‬


‫‪ .III‬ﺗﻄﺒﻴﻘﺎت اﻷﻋﺪاد اﳌﺮﻛﺒﺔ‬
‫‪.‬‬
‫⧐‬
‫‪c‬‬ ‫‪b‬‬
‫ﻧﻼﺣﻆ ّ‬

‫‪Ŕž Ŧ Ľ‬‬
‫‪.‬‬ ‫أن ﳎﻤﻮع اﳉﺬرﻳﻦ ﻫﻮ ‪ −‬و ﺟﺪاؤﳘﺎ ﻫﻮ‬
‫‪a‬‬ ‫‪a‬‬
‫و ﺑﺎﻟﻌﻜﺲ‪ ،‬ﻹﳚﺎد ﻋﺪدﻳﻦ ﻣﺮﻛﺒﲔ ﳎﻤﻮﻋﻬﲈ ‪ s‬و ﺟﺪاؤﳘﺎ ‪ p‬ﻳﻜﻔﻲ ّ‬
‫ﺣﻞ اﳌﻌﺎدﻟﺔ ‪. z − sz + p = 0 :‬‬

‫‪œǃ‬‬
‫ﺍﳊﻞ ‪ ،‬ﰲ ‪ ، ℂ‬ﳌﻌﺎﺩﻟﺔ ﻣﻦ ﺍﻟﺪﺭﺟﺔ ﺍﻟﺜﺎﻧﻴﺔ ﲟﻌﺎﻣﻼﺕ ﺣﻘﻴﻘﻴﺔ‬ ‫‪2.3.III‬‬

‫إذا ﻛﺎﻧﺖ ‪ c ، b ، a‬أﻋﺪا ًدا ﺣﻘﻴﻘﻴ ًﺔ ﻳﻜﻮن اﻟﻌﺪد ‪ Δ‬ﺣﻘﻴﻘﻴﺎ‪.‬‬

‫‪Ŀ‬‬
‫ﻓﺈن اﳌﻌﺎدﻟﺔ )𝟒( ﺗﻘﺒﻞ ﺣ ّﻠﲔ ﺣﻘﻴﻘﻴﲔ ﻣﺘﲈﻳﺰﻳﻦ ﳘﺎ ‪:‬‬
‫إذا ﻛﺎن ‪ّ Δ > 0‬‬ ‫•‬
‫‪−b − √Δ‬‬ ‫‪−b + √Δ‬‬
‫= ‪z‬‬ ‫‪,‬‬ ‫= ‪z‬‬
‫‪2a‬‬ ‫‪2a‬‬
‫‪−b‬‬
‫‪.‬‬ ‫إذا ﻛﺎن ‪ّ Δ = 0‬‬
‫ﻓﺈن اﳌﻌﺎدﻟﺔ )𝟒( ﺗﻘﺒﻞ ﺣﻼ ﺣﻘﻴﻘﻴﺎ ﻣﻀﺎﻋﻔﺎ ﻫﻮ ‪:‬‬ ‫•‬
‫‪2a‬‬
‫‪III‬‬

‫‪ Δ = ı√−Δ‬و اﳌﻌﺎدﻟﺔ )𝟒( ﺗﻘﺒﻞ ﺣﻠﲔ ﻣﺮﻛﺒﲔ ﻣﱰاﻓﻘﲔ ﳘﺎ ‪:‬‬ ‫إذا ﻛﺎن ‪ Δ < 0‬ﻓﺈﻧﻪ ﻳﻤﻜﻦ ﻛﺘﺎﺑﺔ ‪ Δ‬ﻋﲆ اﻟﺸﻜﻞ‬ ‫•‬

‫‪−b − ı√−Δ‬‬ ‫‪−b + ı√−Δ‬‬


‫= ‪z‬‬ ‫‪,‬‬ ‫= ‪z‬‬
‫‪2a‬‬ ‫‪2a‬‬
‫إذن‬

‫ﻛﻞ ﻣﻌﺎدﻟﺔ ﻣﻦ اﻟﺪرﺟﺔ اﻟﺜﺎﻧﻴﺔ ﺑﻤﻌﺎﻣﻼت ﺣﻘﻴﻘﻴﺔ ﺗﻘﺒﻞ‪ ،‬ﰲ ‪ ، ℂ‬ﺣ ّﻠﲔ ﺣﻘﻴﻘﻴﲔ أو ﻣﺮﻛﺒﲔ ﻣﱰاﻓﻘﲔ‪.‬‬ ‫ﻣﺒﺮﻫﻨﺔ ‪: 20‬‬

‫ﻣﻼﺣﻈﺔ ‪ : 14‬اﻟﻔﺮﺿﻴﺔ )اﻟﴩط( »اﳌﻌﺎﻣﻼت ‪ c ، b ، a‬ﺣﻘﻴﻘﻴﺔ«ﻣﻬﻤﺔ ﺟﺪّ ا ﻷﻧﻪ إذا ﱂ ﻳﺘﺤﻘﻖ ﻫﺬا اﻟﴩط‪ ،‬ﻓﻨﺘﻴﺠﺔ اﳌﱪﻫﻨﺔ‬
‫‪ 20‬ﺧﺎﻃﺌﺔ ﻛﲈ ُﻳ ﱢﺒﲔ ذﻟﻚ اﳌﺜﺎل ‪. 22‬‬

‫𝜃 ‪sin‬‬ ‫ﺣﺴﺎﺏ 𝜃‪ cos n‬ﻭ 𝜃‪ sin n‬ﺑﺪﻻﻟﺔ 𝜃 ‪ cos‬ﻭ‬ ‫‪4.III‬‬


‫‪ņőƱŀ Ţ‬‬
‫ﻳﻤﻜﻦ ﺣﺴﺎب 𝜃‪ cos n‬و𝜃‪ sin n‬ﺑﺪﻻﻟﺔ 𝜃 ‪ cos‬و 𝜃 ‪ sin‬و ذﻟﻚ ﺑﺎﺗﺒﺎع اﻟﻄﺮﻳﻘﺔ اﻟﺘﺎﻟﻴﺔ ‪:‬‬
‫)𝜃 ‪. cos n𝜃 + ı sin n𝜃 = (cos 𝜃 + ı sin‬‬ ‫ﻟﺪﻳﻨﺎ ﺣﺴﺐ دﺳﺘﻮر ﻣﻮﺍﻓﺮ ‪:‬‬
‫ﻋﻨﺪ ﻧﴩ اﻟﻄﺮف اﻟﺜﺎﲏ ﻣﻦ ﻫﺬه اﳌﺴﺎواة‪ ،‬ﺣﺴﺐ دﺳﺘﻮر ﺛﻨﺎﺋﻲ اﳊﺪ‪ ،‬ﻧﺤﺼﻞ ﻋﲆ ﻋﺪد ﻣﺮﻛﺐ ﺟﺰؤه اﳊﻘﻴﻘﻲ ﻳﺴﺎوي 𝜃‪cos n‬‬
‫و ﺟﺰؤه اﻟﺘﺨﻴﲇ ﻳﺴﺎوي 𝜃‪ sin n‬ﻛﲈ ﰲ اﻷﻣﺜﻠﺔ اﻟﺘﺎﻟﻴﺔ ‪:‬‬
‫‪ •1‬ﻧﺄﺧﺬ ‪ . n = 2‬ﻟﺪﻳﻨﺎ ‪:‬‬
‫)𝜃 ‪cos 2𝜃 + ı sin 2𝜃 = (cos 𝜃 + ı sin‬‬
‫‪ƕ‬‬

‫𝜃 ‪= cos 𝜃 − sin 𝜃 + 2ı cos 𝜃 sin‬‬

‫و ﻣﻨﻪ اﻟﻨﺘﻴﺠﺘﺎن اﳌﻌﺮوﻓﺘﺎن ‪:‬‬


‫𝜃 ‪cos 2𝜃 = cos 𝜃 − sin‬‬ ‫‪,‬‬ ‫𝜃 ‪sin 2𝜃 = 2 sin 𝜃 ⋅ cos‬‬
‫‪ũŏ‬‬

‫‪ •2‬ﻧﺄﺧﺬ ‪ . n = 3‬ﻟﺪﻳﻨﺎ ‪:‬‬


‫)𝜃 ‪cos 3𝜃 + ı sin 3𝜃 = (cos 𝜃 + ı sin‬‬
‫)𝜃 ‪= cos 𝜃 + 3 (cos 𝜃) (ı sin 𝜃) + 3 (cos 𝜃) (ı sin 𝜃) + (ı sin‬‬
‫𝜃 ‪= cos 𝜃 − 3 cos 𝜃 ⋅ sin 𝜃 + ı 3 cos 𝜃 ⋅ sin 𝜃 − sin‬‬

‫𝟱𝟰‬

‫‪http ://tinyurl.com/Malki1718‬‬ ‫‪0‬‬


sin 𝜃 ‫ و‬cos 𝜃 ‫ ﺑﺪﻻةل‬sin n𝜃 ‫ و‬cos n𝜃 ‫ ﺣﺴﺎب‬.4.III
.
: ‫و ﻣﻨﻪ اﻟﻨﺘﻴﺠﺘﺎن اﻟﺘﺎﻟﻴﺘﺎن‬
cos 3𝜃 = cos 𝜃 − 3 cos 𝜃 ⋅ sin 𝜃 , sin 3𝜃 = 3 cos 𝜃 ⋅ sin 𝜃 − sin 𝜃

Ŕž Ŧ Ľ
: ‫ ﻟﺪﻳﻨﺎ‬. n = 4 ‫• ﻧﺄﺧﺬ‬3

œǃ
cos 4𝜃 + ı sin 4𝜃 = (cos 𝜃 + ı sin 𝜃)
= cos 𝜃 + 4 (cos 𝜃) (ı sin 𝜃) + 6 (cos 𝜃) (ı sin 𝜃) + 4 (cos 𝜃) (ı sin 𝜃) + (ı sin 𝜃)
= cos 𝜃 − 6 cos 𝜃 ⋅ sin 𝜃 + sin 𝜃 + ı 4 cos 𝜃 ⋅ sin 𝜃 − 4 cos 𝜃 ⋅ sin 𝜃

Ŀ
: ‫و ﻣﻨﻪ اﻟﻨﺘﻴﺠﺘﺎن اﻟﺘﺎﻟﻴﺘﺎن‬
cos 4𝜃 = cos 𝜃 − 6 cos 𝜃 ⋅ sin 𝜃 + sin 𝜃 , sin 4𝜃 = 4 cos 𝜃 ⋅ sin 𝜃 − 4 cos 𝜃 ⋅ sin 𝜃

: ‫ ﻟﺪﻳﻨﺎ‬. n = 5 ‫• ﻧﺄﺧﺬ‬4

cos 5𝜃 + ı sin 5𝜃 = (cos 𝜃 + ı sin 𝜃)

III
= cos 𝜃 + 5 (cos 𝜃) (ı sin 𝜃) + 10 (cos 𝜃) (ı sin 𝜃)
+10 (cos 𝜃) (ı sin 𝜃) + 5 (cos 𝜃) (ı sin 𝜃) + (ı sin 𝜃)
= cos 𝜃 − 10 cos 𝜃 ⋅ sin 𝜃 + 5 cos 𝜃 ⋅ sin 𝜃
+ı 5 cos 𝜃 ⋅ sin 𝜃 − 10 cos 𝜃 ⋅ sin 𝜃 + sin 𝜃

: ‫و ﻣﻨﻪ اﻟﻨﺘﻴﺠﺘﺎن اﻟﺘﺎﻟﻴﺘﺎن‬

cos 5𝜃 = cos 𝜃 − 10 cos 𝜃 ⋅ sin 𝜃 + 5 cos 𝜃 ⋅ sin 𝜃


sin 5𝜃 = 5 cos 𝜃 ⋅ sin 𝜃 − 10 cos 𝜃 ⋅ sin 𝜃 + sin 𝜃

ّ ‫• ﻧﺠﺪ ﺑﻨﻔﺲ اﻟﻄﺮﻳﻘﺔ‬5


: ‫أن‬
ņőƱŀ Ţ
cos 6𝜃 = cos 𝜃 − 15 cos 𝜃 ⋅ sin 𝜃 + 15 cos 𝜃 ⋅ sin 𝜃 − sin 𝜃
sin 6𝜃 = 6 cos 𝜃 ⋅ sin 𝜃 − 20 cos 𝜃 ⋅ sin 𝜃 + 6 cos 𝜃 ⋅ sin 𝜃

cos 7𝜃 = cos 𝜃 − 21 cos 𝜃 ⋅ sin 𝜃 + 35 cos 𝜃 ⋅ sin 𝜃 − 7 cos 𝜃 ⋅ sin 𝜃


sin 7𝜃 = 7 cos 𝜃 ⋅ sin 𝜃 − 35 cos 𝜃 ⋅ sin 𝜃 + 21 cos 𝜃 ⋅ sin 𝜃 − sin 𝜃
ƕ

ّ ‫ ﺑﺎﺳﺘﻌﲈل دﺳﺘﻮر ﻣﻮﺍﻓﺮ‬،‫ﻳﻤﻜﻦ أن ﻧﱪﻫﻦ‬


: ‫أن‬
/
n −
cos nx = (−1) (cos x) 1 − cos x
=
2ℓ
ũŏ

− /
n − −
sin nx = sin x (−1) (cos x) 1 − cos x
=
2ℓ + 1

. t ‫ ﻟﻠﻌﺪد اﳊﻘﻴﻘﻲ‬1 ‫[ ﻳﺮﻣﺰ إﱃ اﳉﺰء اﻟﺼﺤﻴﺢ‬t] ‫ﺣﻴﺚ‬


، . = ً ‫ ﻣﺜ‬.
:‫ﻼ‬ ≤ < + ‫ ﻫﻮ أﻛﱪ ﻋﺪد ﺻﺤﻴﺢ ُﳛﻘﻖ‬، ‫ و اﻟﺬي ﻧﺮﻣﺰ إﻟﻴﻪ ﺑﺎﻟﺮﻣﺰ‬، ‫اﳉﺰء اﻟﺼﺤﻴﺢ ﻟﻠﻌﺪد اﳊﻘﻴﻘﻲ‬1
.(− ‫ )و ﻟﻴﺲ‬− . =− ، =

𝟰𝟲

http ://tinyurl.com/Malki1718 0
‫‪ .III‬ﺗﻄﺒﻴﻘﺎت اﻷﻋﺪاد اﳌﺮﻛﺒﺔ‬
‫‪.‬‬
‫‪sin nx‬‬
‫ﻋﺒﺎرة ﻋﻦ ﻛﺜﲑ ﺣﺪود ﰲ ‪ . cos x‬ﺑﺘﻌﺒﲑ ّ‬
‫أدق‪ ،‬ﻟﺪﻳﻨﺎ ‪:‬‬ ‫و ﺑﺎﻟﺘﺎﱄ‪ّ ،‬‬
‫ﻓﻜﻞ ﻣﻦ ‪ cos nx‬و‬
‫‪sin x‬‬
‫‪sin nx‬‬

‫‪Ŕž Ŧ Ľ‬‬
‫)‪cos nx = T (cos x‬‬ ‫‪,‬‬ ‫)‪= U (cos x‬‬
‫‪sin x‬‬
‫ﺣﻴﺚ ‪:‬‬

‫‪œǃ‬‬
‫‪/‬‬ ‫‪−‬‬ ‫‪/‬‬
‫‪n‬‬ ‫‪−‬‬ ‫‪n‬‬ ‫‪−‬‬ ‫‪−‬‬
‫= )‪T (X‬‬ ‫‪(−1) X‬‬ ‫‪1−X‬‬ ‫= )‪U (X‬‬ ‫‪(−1) X‬‬ ‫‪1−X‬‬
‫=‬
‫‪2ℓ‬‬ ‫=‬
‫‪2ℓ + 1‬‬

‫ُﻳﺪﻋﻰ ﻛﺜﲑ اﳊﺪود ‪ T‬ﺑﻜﺜﲑ ﺣﺪود ﺗﺸﻴﺒﻴﺘﺸﻴﻒ )‪ (T‬ﻣﻦ اﻟﺼﻨﻒ ّ‬

‫‪Ŀ‬‬
‫اﻷول و ُﻳﺪﻋﻰ ﻛﺜﲑ اﳊﺪود ‪ U‬ﺑﻜﺜﲑ ﺣﺪود‬
‫ﺗﺸﻴﺒﻴﺘﺸﻴﻒ ﻣﻦ اﻟﺼﻨﻒ اﻟﺜﺎﲏ ‪.2‬‬

‫𝜃 ‪sin‬‬
‫‪n‬‬
‫ﺍﻟﻌﺒﺎﺭﺓ ﺍﳋﻄﻴﺔ ﻟﻜﻞ ﻣﻦ 𝜃 ‪ cosn‬ﻭ‬ ‫‪5.III‬‬
‫اﳍﺪف‪ ،‬ﰲ ﻫﺬه اﻟﻔﻘﺮة‪ ،‬ﻫﻮ اﻟﺘﻌﺒﲑ ﻋﻦ ﻛﻞ ﻣﻦ 𝜃 ‪ cos‬و 𝜃 ‪ sin‬ﻛﻤﺠﻤﻮع ﺣﺪود ﻣﻦ اﻟﺸﻜﻞ 𝜃‪ 𝛼 cos k‬أو ﻣﻦ اﻟﺸﻜﻞ‬
‫‪III‬‬

‫𝜃‪ 𝛼 sin k‬ﺣﻴﺚ ‪ 𝛼 ∈ ℝ‬و ‪. k ∈ ℕ‬‬


‫ﻟﺬﻟﻚ ﻧﺘّﺒﻊ اﻟﻄﺮﻳﻘﺔ اﻟﺘﺎﻟﻴﺔ ‪:‬‬
‫‪1‬‬
‫𝜃 ‪= cos 𝜃 − ı sin‬‬ ‫ﻧﻀﻊ 𝜃 ‪ z = cos 𝜃 + ı sin‬و ﻣﻨﻪ‬
‫‪z‬‬
‫‪1‬‬
‫‪.‬‬ ‫و ﻳﻜﻮن ﻋﻨﺪﺋﺬ ‪ z = cos n𝜃 + ı sin n𝜃 :‬و 𝜃‪= cos n𝜃 − ı sin n‬‬
‫‪z‬‬
‫و ﺑﺎﻟﺘﺎﱄ ‪:‬‬

‫‪1‬‬ ‫‪1‬‬ ‫‪1‬‬ ‫‪1‬‬


‫= 𝜃 ‪cos‬‬ ‫‪z+‬‬ ‫‪,‬‬ ‫= 𝜃 ‪sin‬‬ ‫‪z−‬‬
‫‪2‬‬ ‫‪z‬‬ ‫‪2ı‬‬ ‫‪z‬‬
‫‪1‬‬ ‫‪1‬‬ ‫‪1‬‬ ‫‪1‬‬
‫= 𝜃‪cos n‬‬ ‫‪z +‬‬ ‫‪,‬‬ ‫= 𝜃‪sin n‬‬ ‫‪z −‬‬
‫‪2‬‬ ‫‪z‬‬ ‫‪2ı‬‬ ‫‪z‬‬
‫‪ņőƱŀ Ţ‬‬
‫‪1‬‬ ‫‪1‬‬ ‫‪1‬‬ ‫‪1‬‬
‫ﻣﺴﺘﻌﻤﻠﲔ ﰲ اﳊﺎﻟﺘﲔ‬ ‫‪z−‬‬ ‫و ﻟﻠﺤﺼﻮل ﻋﲆ 𝜃 ‪ sin‬ﻧﻨﴩ‬ ‫‪z+‬‬ ‫ﻟﻠﺤﺼﻮل ﻋﲆ 𝜃 ‪ cos‬ﻧﻨﴩ‬
‫‪2ı‬‬ ‫‪z‬‬ ‫‪2‬‬ ‫‪z‬‬
‫دﺳﺘﻮر ﺛﻨﺎﺋﻲ اﳊﺪ‪.‬‬
‫ﺃﻣﺜﻠﺔ ‪ : 23‬اﻟﻌﺒﺎرة اﳋﻄﻴﺔ ﻟﻜﻞ ﻣﻦ 𝜃 ‪ cos 𝜃 ، sin 𝜃 ، cos‬و 𝜃 ‪: sin‬‬
‫•‬ ‫‪1‬‬
‫‪1‬‬ ‫‪1‬‬
‫= 𝜃 ‪cos‬‬ ‫‪z+‬‬
‫‪2‬‬ ‫‪z‬‬
‫‪ƕ‬‬

‫‪1‬‬ ‫‪1‬‬ ‫‪1‬‬ ‫‪1‬‬


‫=‬ ‫‪z + 3z × + 3z × +‬‬
‫‪8‬‬ ‫‪z‬‬ ‫‪z z‬‬
‫‪1‬‬ ‫‪1‬‬ ‫‪1‬‬
‫=‬ ‫‪z +‬‬ ‫‪+3 z+‬‬
‫‪8‬‬ ‫‪z‬‬ ‫‪z‬‬
‫‪ũŏ‬‬

‫‪1‬‬
‫]𝜃 ‪= [2 cos 3𝜃 + 3 × 2 cos‬‬
‫‪8‬‬

‫ّ‬
‫ﻓﺈن ‪:‬‬ ‫≥‬ ‫‪2‬ﻳﻤﻜﻦ أن ﻧﱪﻫﻦ أﻧﻪ ﻟﻜﻞ ﻋﺪد ﻃﺒﻴﻌﻲ‬
‫‪−‬‬ ‫‪−‬‬
‫=‬ ‫‪− cos‬‬ ‫=‬ ‫‪− cos‬‬
‫=‬ ‫=‬
‫‪+‬‬

‫𝟳𝟰‬

‫‪http ://tinyurl.com/Malki1718‬‬ ‫‪0‬‬


sin 𝜃 ‫ و‬cos 𝜃 ‫ اﻟﻌﺒﺎرة اﳋﻄﻴﺔ ﻟﲁ ﻣﻦ‬.5.III
.
‫و ﻣﻨﻪ‬
1 3
cos 𝜃 = cos 3𝜃 + cos 𝜃

Ŕž Ŧ Ľ
4 4
• 2

œǃ
1 1
sin 𝜃 = z−
2ı z
1 1 1 1
=− z − 3z × + 3z × −

Ŀ
8ı z z z
1 1 1
=− z − +3 z−
8ı z z
1
= − [2ı sin 3𝜃 − 3 × 2ı sin 𝜃]

‫و ﻣﻨﻪ‬
1 3

III
sin 𝜃 = − sin 3𝜃 + sin 𝜃
4 4
• 3
1 1
cos 𝜃 = z+
2 z
1 1 1 1 1
= z + 4z × + 6z × + 4z × +
16 z z z z
1 1 1
= z + +4 z + +6
16 z z
1
= [2 cos 4𝜃 + 4 × 2 cos 2𝜃 + 6]
16
‫و ﻣﻨﻪ‬
1 1 3
cos 𝜃 = cos 4𝜃 + cos 2𝜃 +
8 2 8
ņőƱŀ Ţ
• 4
1 1
sin 𝜃 = z−
2ı z
1 1 1 1 1
= z − 4z × + 6z × − 4z × +
16 z z z z
1 1 1
= z + −4 z + +6
16 z z
1
ƕ

= [2 cos 4𝜃 − 4 × 2 cos 2𝜃 + 6]
16
‫و ﻣﻨﻪ‬
1 1 3
sin 𝜃 = cos 4𝜃 − cos 2𝜃 +
ũŏ

8 2 8

ّ ‫• ﻧﺠﺪ ﺑﻨﻔﺲ اﻟﻄﺮﻳﻘﺔ‬5


: ‫أن‬
1 5 5 1 3 15 5
cos 𝜃 = cos 5𝜃 + cos 3𝜃 + cos 𝜃 cos 𝜃 = cos 6𝜃 + cos 4𝜃 + cos 2𝜃 +
16 16 8 32 16 32 16
1 5 5 1 3 15 5
sin 𝜃 = sin 5𝜃 − sin 3𝜃 + sin 𝜃 sin 𝜃 = − cos 6𝜃 + cos 4𝜃 − cos 2𝜃 +
16 16 8 32 16 32 16

𝟰𝟴

http ://tinyurl.com/Malki1718 0
‫ ﺗﻄﺒﻴﻘﺎت اﻷﻋﺪاد اﳌﺮﻛﺒﺔ‬.III
.
1 7 21 35
cos 𝜃 = cos 7𝜃 + cos 5𝜃 + cos 3𝜃 + cos 𝜃
64 64 64 64

Ŕž Ŧ Ľ
1 7 21 35
sin 𝜃 = − sin 7𝜃 + sin 5𝜃 − sin 3𝜃 + sin 𝜃
64 64 64 64

œǃ
Ŀ
III

ņőƱŀ Ţ
ũŏ ƕ

𝟰𝟵

http ://tinyurl.com/Malki1718 0
.

Ŕž Ŧ Ľ
œǃ
Ŀ
III
ņőƱŀ Ţ
ũŏ ƕ

http ://tinyurl.com/Malki1718 0
‫‪.‬‬

‫‪Ŕž Ŧ Ľ‬‬
‫‪4‬‬
‫‪œǃ‬‬
‫‪otttttttttttttttttttttttttttttttttttttttttttp‬‬
‫‪s‬‬ ‫‪u‬‬
‫‪s‬‬ ‫ﺗﻤﺜﻴﻞ اﻟﺘﺤﻮﻳﻼت اﻟﻨﻘﻄﻴﺔ ﻓﻲ اﻟﻤﺴﺘﻮي اﻟﻤﺮﻛﺐ‬
‫‪u‬‬

‫‪Ŀ‬‬
‫‪s‬‬ ‫‪u‬‬
‫‪s‬‬ ‫‪u‬‬
‫‪qvvvvvvvvvvvvvvvvvvvvvvvvvvvvvvvvvvvvvvvvvvvr‬‬
‫‪.‬‬

‫ﻓﻴﲈ ﻳﲇ‪ ،‬اﳌﺴﺘﻮي ﻣﻨﺴﻮب إﱃ ﻣﻌﻠﻢ ﻣﺘﻌﺎﻣﺪ و ﻣﺘﺠﺎﻧﺲ ⃗ ‪. O, i,⃗ j‬‬

‫ﺍﻷﻋﺪﺍﺩ ﺍﳌﺮﻛﺒﺔ ﻭ ﺍﳍﻨﺪﺳﺔ ﺍﳌﺴﺘﻮﻳﺔ‬ ‫‪1.IV‬‬


‫‪IV‬‬

‫ﺍﳌﺴﺎﻓﺔ‬ ‫‪1.1.IV‬‬

‫ﻣﺒﺮﻫﻨﺔ ‪ : 21‬ﻟﺘﻜﻦ ‪ M‬و ‪ M‬ﻧﻘﻄﺘﲔ ﻣﻦ اﳌﺴﺘﻮي و ﻟﻴﻜﻦ ‪ z‬و ‪ z‬ﺻﻮرﺗﻴﻬﲈ ﻋﲆ اﻟﱰﺗﻴﺐ‪.‬‬


‫اﳌﺴﺎﻓﺔ ﺑﲔ اﻟﻨﻘﻄﺘﲔ ‪ M‬و ‪ M‬ﻫﻲ || ‪. M M = ||z − z‬‬
‫‪ņőƱŀ Ţ‬‬
‫‪M‬‬ ‫ﻓﺈن اﳌﺴﺎﻓﺔ ﺑﲔ ‪ M‬و‬ ‫ﺍﻟﱪﻫﺎﻥ‪ .‬ﺻﻮرة اﻟﺸﻌﺎع ‪ M M‬ﻫﻲ ‪ . z − z‬و ﺑﲈ ّ‬
‫أن || ‪ّ M M = ‖M M ‖ = ||z − z‬‬
‫■‬ ‫ﻫﻲ || ‪. ||z − z‬‬

‫ﻣﺮﻛﺰ ﺍﳌﺴﺎﻓﺎﺕ ﺍﳌﺘﻨﺎﺳﺒﺔ‬ ‫‪2.1.IV‬‬

‫ﻣﺒﺮﻫﻨﺔ ‪ : 22‬ﻟﺘﻜﻦ ‪ M ، M‬و ‪ B‬ﺛﻼث ﻧﻘﻂ ﺻﻮرﻫﺎ ‪ z ، z‬و ‪ b‬ﻋﲆ اﻟﱰﺗﻴﺐ‪ .‬ﻟﻴﻜﻦ 𝛼 و 𝛼 ﻋﺪدﻳﻦ ﺣﻘﻴﻘﻴﲔ ﺣﻴﺚ‬
‫‪.𝛼 +𝛼 ≠ 0‬‬
‫اﻟﻨﻘﻄﺔ ‪ B‬ﻫﻲ ﻣﺮﻛﺰ اﳌﺴﺎﻓﺎت اﳌﺘﻨﺎﺳﺒﺔ ﻟﻠﻨﻘﻄﺘﲔ ‪ M‬و ‪ M‬اﳌﺮﻓﻮﻗﺘﲔ ﺑﺎﳌﻌﺎﻣﻠﲔ 𝛼 و 𝛼 إذا و ﻓﻘﻂ إذا ﻛﺎن ‪:‬‬
‫‪ƕ‬‬

‫‪𝛼 (b − z ) + 𝛼 (b − z ) = 0‬‬

‫ﺍﻟﱪﻫﺎﻥ‪ .‬ﺣﺴﺐ ﺗﻌﺮﻳﻒ ﻣﺮﻛﺰ اﳌﺴﺎﻓﺎت اﳌﺘﻨﺎﺳﺒﺔ ‪ :‬اﻟﻨﻘﻄﺔ ‪ B‬ﻫﻲ ﻣﺮﻛﺰ اﳌﺴﺎﻓﺎت اﳌﺘﻨﺎﺳﺒﺔ ﻟﻠﻨﻘﻄﺘﲔ ‪ M‬و ‪ M‬اﳌﺮﻓﻮﻗﺘﲔ‬
‫‪ũŏ‬‬

‫ﺑﺎﳌﻌﺎﻣﻠﲔ 𝛼 و 𝛼 إذا و ﻓﻘﻂ إذا ﻛﺎن ⃗‪ 𝛼 BM + 𝛼 BM = 0‬أي إذا و ﻓﻘﻔﻂ إذا ﻛﺎن ‪:‬‬

‫‪𝛼 (b − z ) + 𝛼 (b − z ) = 0‬‬
‫■‬

‫‪ M M‬و ﻧﺠﺪ اﳌﺴﺎواة‬ ‫ﰲ اﳊﺎﻟﺔ اﳋﺎﺻﺔ ‪ ، 𝛼 = 𝛼 = 1‬اﻟﻨﻘﻄﺔ ‪ B‬ﻫﻲ ﻣﻨﺘﺼﻒ اﻟﻘﻄﻌﺔ‬ ‫ﻣﻼﺣﻈﺔ ‪⧏ : 15‬‬
‫‪z +z‬‬
‫= ‪.b‬‬ ‫)اﳌﻌﺮوﻓﺔ( ‪:‬‬
‫‪2‬‬
‫𝟭𝟱‬

‫‪http ://tinyurl.com/Malki1718‬‬ ‫‪0‬‬


‫‪ .1.IV‬اﻷﻋﺪاد اﳌﺮﻛﺒﺔ و اﻟﻬﻨﺪﺳﺔ اﳌﺴـﺘﻮﯾﺔ‬
‫‪.‬‬

‫)‪C (c‬‬

‫‪Ŕž Ŧ Ľ‬‬
‫)‪B (b‬‬

‫‪œǃ‬‬
‫)‪(c − a‬‬

‫‪−‬‬
‫→‬ ‫)‪A (a‬‬
‫‪j‬‬ ‫)‪(b − a‬‬

‫‪Ŀ‬‬
‫‪−‬‬
‫→‬
‫‪O‬‬ ‫‪i‬‬

‫ﺷﻜﻞ ‪ :1.IV‬زاوﻳﺔ ﺷﻌﺎﻋﲔ‪.‬‬

‫⧐‬

‫ُﻌﻤﻢ ﻫﺬه اﻟﻨﺘﻴﺠﺔ ﻛﲈ ﻳﲇ ‪:‬‬


‫وﺗ ﱠ‬

‫‪IV‬‬
‫ﻣﺒﺮﻫﻨﺔ ‪ : 23‬ﻟﻴﻜﻦ ‪ n ≥ 2‬ﻋﺪ ًدا ﻃﺒﻴﻌﻴﺎ و ﻟﺘﻜﻦ ‪ A ، ⋯ ، A ، A‬ﻧﻘﻄ ًﺎ ﻣﻦ اﳌﺴﺘﻮي ﻟﻮاﺣﻘﻬﺎ ‪ z ، ⋯ ، z ، z‬ﻋﲆ‬
‫اﻟﱰﺗﻴﺐ‪ .‬ﻟﺘﻜﻦ 𝛼 ‪ 𝛼 ، ⋯ ، 𝛼 ،‬أﻋﺪا ًدا ﺣﻘﻴﻘﻴﺔ ﺑﺤﻴﺚ ‪. 𝛼 + 𝛼 + ⋯ + 𝛼 ≠ 0‬‬
‫اﻟﻨﻘﻄﺔ ‪ G‬ﻫﻲ ﻣﺮﻛﺰ اﳌﺴﺎﻓﺎت اﳌﺘﻨﺎﺳﺒﺔ ﻟﻠﻨﻘﻂ ‪ A‬اﳌﺮﻓﻮﻗﺔ ﺑﺎﳌﻌﺎﻣﻼت 𝛼 ‪ ، i = 1, 2, ⋯ , n ،‬إذا و ﻓﻘﻂ إذا ﺣﻘﻘﺖ ﻻﺣﻘﺘﻬﺎ‬
‫‪ z‬اﳌﻌﺎدﻟﺔ ‪:‬‬
‫‪𝛼 (z − z) + 𝛼 (z − z) + ⋯ + 𝛼 (z − z) = 0‬‬

‫ﻳﻜﻔﻲ إﻋﺎدة ﻛﺘﺎﺑﺔ اﳌﻌﺎدﻟﺔ )ﻣﻦ ﺗﻌﺮﻳﻒ ﻣﺮﻛﺰ اﳌﺴﺎﻓﺎت اﳌﺘﻨﺎﺳﺒﺔ( ⃗‪= 0‬‬ ‫ﺍﻟﱪﻫﺎﻥ‪.‬‬
‫‪ņőƱŀ Ţ‬‬
‫‪𝛼 GA + 𝛼 GA + ⋯ + 𝛼 GA‬‬
‫■‬ ‫ﺑﺎﺳﺘﻌﲈل اﻟﻼﺣﻘﺔ‪.‬‬

‫ﺍﻟﺰﻭﺍﻳﺎ‬ ‫‪3.1.IV‬‬

‫ﻟﺘﻜﻦ ‪ B ، A‬و ‪ C‬ﺛﻼث ﻧﻘﻂ ﻣﻦ اﳌﺴﺘﻮي ﺑﺤﻴﺚ اﻟﻨﻘﻄﺔ ‪ C‬ﲣﺘﻠﻒ ﻋﻦ ‪ A‬و ‪ ، B‬و ﻟﺘﻜﻦ ‪ b ، a‬و ‪ c‬ﻟﻮاﺣﻘﻬﺎ‬ ‫ﻣﺒﺮﻫﻨﺔ ‪: 24‬‬
‫ﻋﲆ اﻟﱰﺗﻴﺐ‪.‬‬
‫‪b−c‬‬
‫‪. CA, CB = arg‬‬ ‫)𝜋‪(mod 2‬‬ ‫إﺣﺪى ﻗﻴﺎﺳﺎت اﻟﺰاوﻳﺔ ‪ CA, CB‬ﻫﻲ ‪:‬‬
‫‪ƕ‬‬

‫‪a−c‬‬

‫‪b−c‬‬
‫‪. arg‬‬ ‫ﻧﻌﻠﻢ ّ‬
‫أن )𝜋‪= arg (b − c) − arg (a − c) (mod 2‬‬ ‫ﺍﻟﱪﻫﺎﻥ‪.‬‬
‫‪ũŏ‬‬

‫‪a−c‬‬
‫ﻟﻜﻦ ‪ b − c‬ﻫﻲ ﻻﺣﻘﺔ ‪ CB‬و ‪ a − c‬ﻫﻲ ﻻﺣﻘﺔ ‪ CA‬ﻣﻨﻪ ‪ arg (b − c) = i,⃗ CB‬و ‪. arg (a − c) = i,⃗ CA‬‬
‫‪b−c‬‬
‫‪CA, CB‬‬ ‫‪= arg‬‬ ‫)𝜋‪(mod 2‬‬ ‫ّ‬
‫ﻓﺈن ‪:‬‬ ‫أن )𝜋‪CA, CB = i,⃗ CB − i,⃗ CA (mod 2‬‬
‫و ﺑﲈ ّ‬
‫‪a−c‬‬
‫■‬ ‫‪.‬‬
‫ﻣﻦ ﻫﻨﺎ ﻧﺴﺘﺨﻠﺺ ﻣﺎ ﻳﲇ ‪:‬‬

‫𝟮𝟱‬

‫‪http ://tinyurl.com/Malki1718‬‬ ‫‪0‬‬


‫‪ .IV‬ﲤﺜﻴﻞ اﻟﺘﺤﻮﯾﻼت اﻟﻨﻘﻄﻴﺔ ﰲ اﳌﺴـﺘﻮي اﳌﺮﻛﺐ‬
‫‪.‬‬
‫ﻟﺘﻜﻦ ‪ B ، A‬و ‪ C‬ﺛﻼث ﻧﻘﻂ ﻣﻦ اﳌﺴﺘﻮي ﺑﺤﻴﺚ اﻟﻨﻘﻄﺔ ‪ C‬ﲣﺘﻠﻒ ﻋﻦ ‪ ، A‬و ﻟﺘﻜﻦ ‪ b ، a‬و ‪ c‬ﻟﻮاﺣﻘﻬﺎ ﻋﲆ‬ ‫ﻻﺯﻣﺔ ‪: 1‬‬

‫‪Ŕž Ŧ Ľ‬‬
‫اﻟﱰﺗﻴﺐ‪.‬‬
‫‪c−b‬‬
‫ﺣﻘﻴﻘﻴﺎ‪.‬‬ ‫اﻟﻨﻘﻂ ‪ B ، A‬و ‪ C‬ﻋﲆ اﺳﺘﻘﺎﻣﺔ واﺣﺪة إذا و ﻓﻘﻂ إذا ﻛﺎن‬

‫‪œǃ‬‬
‫•‬
‫‪c−a‬‬
‫‪c−b‬‬
‫ﲣﻴﻠﻴﺎ ﴏﻓﺎ‪.‬‬ ‫اﳌﺴﺘﻘﻴﲈن )‪ (CA‬و )‪ (CB‬ﻣﺘﻌﺎﻣﺪان إذا و ﻓﻘﻂ إذا ﻛﺎن‬ ‫•‬
‫‪c−a‬‬

‫‪Ŀ‬‬
‫اﻟﻨﻘﻂ ‪ B ، A‬و ‪ C‬ﻋﲆ اﺳﺘﻘﺎﻣﺔ واﺣﺪة إذا ﻛﺎﻧﺖ ‪:‬‬ ‫•‬ ‫ﺍﻟﱪﻫﺎﻥ‪.‬‬

‫)𝜋‪. CA, CB = 𝜋 (mod 2‬‬ ‫أو‬ ‫‪CA, CB‬‬ ‫)𝜋‪= 0 (mod 2‬‬
‫‪b−c‬‬
‫‪arg‬‬ ‫‪ CA, CB‬ﻣﻌﺪوﻣﺔ إذا و ﻓﻘﻂ إذا ﻛﺎن )𝜋‪= 0 (mod 2‬‬ ‫ﺣﺴﺐ اﳌﱪﻫﻨﺔ اﻟﺴﺎﺑﻘﺔ‪ ،‬اﻟﺰاوﻳﺔ‬
‫‪a−c‬‬
‫‪b−c‬‬
‫‪. arg‬‬ ‫و ﻣﺴﺘﻘﻴﻤﺔ إذا و ﻓﻘﻂ إذا ﻛﺎن )𝜋‪= 𝜋 (mod 2‬‬
‫‪a−c‬‬

‫أﻧﻈﺮ اﻟﺘﻤﺮﻳﻦ ‪ 25‬ﺻﻔﺤﺔ ‪91‬‬ ‫‬ ‫ﺣﻘﻴﻘﻴﺎ‪.‬‬


‫‪c−b‬‬
‫‪c−a‬‬
‫أي ) ﰲ اﳊﺎﻟﺘﲔ ( إذا و ﻓﻘﻂ إذا ﻛﺎن‬
‫𝜋‬
‫‪IV‬‬

‫‪CA, CB‬‬ ‫اﳌﺴﺘﻘﻴﲈن )‪ (CA‬و )‪ (CB‬ﻣﺘﻌﺎﻣﺪان إذا و ﻓﻘﻂ إذا ﻛﺎن )𝜋‪(mod 2‬‬
‫‪= ±‬‬ ‫•‬
‫‪2‬‬
‫‪c−b‬‬ ‫‪b−c‬‬ ‫𝜋‬
‫ﲣﻴﻠﻴﺎ‬ ‫‪ arg‬أي إذا و ﻓﻘﻂ إذا ﻛﺎن‬ ‫أي ) ﺣﺴﺐ اﳌﱪﻫﻨﺔ اﻟﺴﺎﺑﻘﺔ ( إذا و ﻓﻘﻂ إذا ﻛﺎن )𝜋‪= ± (mod 2‬‬
‫‪c−a‬‬ ‫‪a−c‬‬ ‫‪2‬‬
‫ﴏﻓﺎ‪.‬‬
‫■‬

‫ﺍﳉﺪﺍء ﺍﻟﺴﻠﻤﻲ‬ ‫‪4.1.IV‬‬


‫‪ņőƱŀ Ţ‬‬
‫‪x‬‬ ‫‪x‬‬
‫‪ V‬ﺷﻌﺎﻋﲔ ﻣﻦ اﳌﺴﺘﻮي 𝒫 ﻻﺣﻘﺘﺎﳘﺎ ‪ z = x + ıy‬و ‪ z = x + ıy‬ﻋﲆ‬ ‫‪V‬و‬ ‫ﻟﻴﻜﻦ‬ ‫ﻣﺒﺮﻫﻨﺔ ‪: 25‬‬
‫‪y‬‬ ‫‪y‬‬
‫اﻟﱰﺗﻴﺐ‪.‬‬
‫اﻟﻌﺒﺎرة اﳌﺮﻛﺒﺔ ﻟﻠﺠﺪاء اﻟﺴﻠﻤﻲ ‪ V ⋅ V‬ﻟﻠﺸﻌﺎﻋﲔ ‪ V‬و ‪ V‬ﻫﻲ ) ‪. V ⋅ V = Re (z ⋅ z‬‬

‫ﺍﻟﱪﻫﺎﻥ‪ .‬ﻟﺪﻳﻨﺎ ﻣﻦ ﺟﻬﺔ‬


‫‪V ⋅V =x x +y y‬‬
‫‪ƕ‬‬

‫و ﻣﻦ ﺟﻬﺔ أﺧﺮى‬
‫‪z ⋅ z = x − ıy‬‬ ‫‪x + ıy‬‬ ‫‪=x x +y y + x y −y x‬‬ ‫‪ı‬‬
‫■‬ ‫ﻣﻨﻪ اﻟﻨﺘﻴﺠﺔ‪.‬‬
‫‪ũŏ‬‬

‫ﲤﺜﻴﻞ ﺍﻟﺘﺤﻮﻳﻼﺕ ﺍﻟﻨﻘﻄﻴﺔ ﰲ ﺍﳌﺴﺘﻮﻱ ﺍﳌﺮﻛﺐ‬ ‫‪2.IV‬‬


‫ُد ِرﺳﺖ اﻟﺘﺤﻮﻳﻼت اﻟﻨﻘﻄﻴﺔ ﰲ ﻣﺎ ﺳﺒﻖ دراﺳﺔ ﲢﻠﻴﻠﻴﺔ )أي ﺑﺎﺳﺘﺨﺪام اﳍﻨﺪﺳﺔ اﻟﺘﺤﻠﻴﻠﻴﺔ(‪ ،‬و ﺳﻨﺤﺎول ﻓﻴﲈ ﻳﲇ إﻋﻄﺎء ِﺻﻴﻎ‬
‫ﺴﻬﻞ اﻟﻌﻤﻞ و اﳊﺴﺎﺑﺎت اﳌﻌﻘﺪة‪.‬‬‫ﺟﺪﻳﺪة ﳍﺎ ﺑﺎﺳﺘﻌﲈل اﻷﻋﺪاد اﳌﺮﻛﺒﺔ‪ ،‬اﻟﺸﺊ اﻟﺬي ُﻳ ﱢ‬

‫𝟯𝟱‬

‫‪http ://tinyurl.com/Malki1718‬‬ ‫‪0‬‬


‫‪ .2.IV‬ﲤﺜﻴﻞ اﻟﺘﺤﻮﯾﻼت اﻟﻨﻘﻄﻴﺔ ﰲ اﳌﺴـﺘﻮي اﳌﺮﻛﺐ‬
‫‪.‬‬
‫ﻟﻴﻜﻦ 𝒫 اﳌﺴﺘﻮي و 𝒱 ﳎﻤﻮﻋﺔ أﺷﻌﺔ اﳌﺴﺘﻮي‪.‬‬
‫ﻧﺴﻤﻲ ﲢﻮﻳﻼ ﻧﻘﻄﻴﺎ ﻛﻞ ﺗﻄﺒﻴﻖ ﺗﻘﺎﺑﲇ ﻟﻠﻤﺴﺘﻮي 𝒫 ﰲ ﻧﻔﺴﻪ‪.‬‬
‫ّ‬

‫‪Ŕž Ŧ Ľ‬‬
‫𝒫⟶ 𝒫‬
‫‪f:‬‬
‫)‪M ⟼ M = f(M‬‬

‫‪œǃ‬‬
‫ﻳﻤﻜﻦ أن ﻧﺮﻓﻖ ﺑﻜﻞ ﲢﻮﻳﻞ ﻧﻘﻄﻲ ‪ f‬اﻟﺘﻄﺒﻴﻖ ‪ g‬ﻟﻠﻤﺴﺘﻮي اﳌﺮﻛﺐ ﰲ ﻧﻔﺴﻪ و اﻟﺬي ﻳﺮﻓﻖ ﺑﺎﻟﻌﺪد اﳌﺮﻛﺐ ‪ z‬ذي اﻟﺼﻮرة ‪، M‬‬
‫اﻟﻌﺪد اﳌﺮﻛﺐ ‪ z‬ﻻﺣﻘﺔ اﻟﻨﻘﻄﺔ )‪. M = f (M‬‬
‫‪ℂ⟶ℂ‬‬
‫‪g:‬‬

‫‪Ŀ‬‬
‫‪z⟼z‬‬

‫أن اﻟﺘﻄﺒﻴﻖ ‪ُ g‬ﻳﻤ ﱢﺜﻞ اﻟﺘﺤﻮﻳﻞ اﻟﻨﻘﻄﻲ ‪ f‬ﰲ اﳌﺴﺘﻮي اﳌﺮﻛﺐ و ﻧﻘﻮل ﲡﺎﻭ‪‬ﺯًﺍ أﻧّﻪ ﲢﻮﻳﻞ ﻧﻘﻄﻲ ُﻣ ﱠ‬
‫ﻌﺮف ﺑﺎﻷﻋﺪاد اﳌﺮﻛﺒﺔ‪.‬‬ ‫ﻧﻘﻮل ﻋﻨﺪﺋﺬ ّ‬
‫ﻣﺜﺎﻝ ‪ : 24‬ﻟﻴﻜﻦ ‪ f‬اﻟﺘﺤﻮﻳﻞ اﻟﻨﻘﻄﻲ اﳌﻌﺮف ﺑﺎﻟﻌﺒﺎرة ‪:‬‬
‫⎧‬ ‫𝒫⟶ 𝒫‬
‫⎪‬
‫‪f:‬‬ ‫‪x =x −y‬‬
‫⎨‬
‫) ‪⎪ M(x, y) ⟼ M (x , y‬‬ ‫ﺣﻴﺚ‬
‫⎩‬ ‫‪y = 2xy‬‬
‫إذا ﻛﺎن ‪ z = x + ıy‬و ‪ z = x + ıy‬ﺣﻴﺚ ‪ّ ı = −1‬‬
‫ﻓﺈن اﻟﺘﻄﺒﻴﻖ ‪ g‬اﳌﺮﻓﻖ ﺑﺎﻟﺘﺤﻮﻳﻞ ‪ f‬ﳛﻘﻖ‬
‫‪z = g(z) = x + ıy = x − y + 2ıxy = (x + ıy) = z‬‬
‫أي‬
‫‪ℂ⟶ℂ‬‬
‫‪g:‬‬

‫‪IV‬‬
‫‪z ⟼ g(z) = z‬‬
‫⧏ ﻳﻤﻜﻦ ﺗﻌﻴﲔ اﻟﻌﺒﺎرة اﳌﺮﻛﺒﺔ ﻟﺘﺤﻮﻳﻞ ﻧﻘﻄﻲ اﻧﻄﻼ ًﻗﺎ ﻣﻦ ﻋﺒﺎرﺗﻪ اﻟﺘﺤﻠﻴﻠﻴﺔ و اﻟﻌﻜﺲ ﺻﺤﻴﺢ‪.‬‬ ‫ﻣﻼﺣﻈﺔ ‪: 16‬‬
‫⧐‬
‫ﻟﻴﻜﻦ ‪ g‬اﻟﺘﻄﺒﻴﻖ اﻟﺬي ﻋﺒﺎرﺗﻪ اﳌﺮﻛﺒﺔ ‪:‬‬ ‫ﻣﺜﺎﻝ ‪: 25‬‬
‫‪ℂ⟶ℂ‬‬
‫‪g:‬‬
‫‪z ⟼ g(z) = ız + 1 − ı‬‬
‫ﻟﺪﻳﻨﺎ ‪:‬‬
‫‪ņőƱŀ Ţ‬‬
‫‪z = g(z) = ız + 1 − ı = ı (x + ıy) + 1 − ı = (−y + 1) + (x − 1) ı‬‬
‫أي ّ‬
‫أن ‪ ، x = −y + 1 , y = x − 1 :‬إذن اﻟﻌﺒﺎرة اﻟﺘﺤﻠﻴﻠﻴﺔ ﻟﻠﺘﺤﻮﻳﻞ ‪ f‬اﳌﻮاﻓﻖ ﻟﻠﺘﻄﺒﻴﻖ ‪ g‬ﻫﻲ ‪:‬‬
‫⎧‬ ‫𝒫⟶ 𝒫‬
‫⎪‬
‫‪f:‬‬ ‫‪x = −y + 1‬‬
‫⎨‬
‫) ‪⎪ M(x, y) ⟼ M (x , y‬‬ ‫ﺣﻴﺚ‬
‫⎩‬ ‫‪y =x−1‬‬

‫ﺍﻹﻧﺴﺤﺎﺏ ﻭ ﺍﻟﺘﺤﺎﻛﻲ‬ ‫‪1.2.IV‬‬


‫‪ƕ‬‬

‫ﺗﻌﺮﻳﻒ ‪: 14‬‬
‫ﻟﻴﻜﻦ ⃗‪ V‬ﺷﻌﺎ ًﻋﺎ ﻣﻦ اﳌﺴﺘﻮي 𝒫‪.‬‬ ‫•‬

‫ﻧﺴﻤﻲ اﻧﺴﺤﺎﺑﺎ ﺷﻌﺎﻋﻪ ⃗‪ ، V‬و ﻧﺮﻣﺰ إﻟﻴﻪ ﺑﺎﻟﺮﻣﺰ ⃗ 𝒯 ‪ ،‬اﻟﺘﺤﻮﻳﻞ اﻟﻨﻘﻄﻲ ﻟﻠﻤﺴﺘﻮي 𝒫 ﰲ ﻧﻔﺴﻪ و اﻟﺬي ﻳﺮﻓﻖ ﺑﻜﻞ ﻧﻘﻄﺔ‬
‫‪V‬‬
‫‪ũŏ‬‬

‫‪ M‬ﻣﻦ 𝒫 اﻟﻨﻘﻄﺔ ‪ M‬ﺑﺤﻴﺚ ﻳﻜﻮن ‪. MM = V⃗ :‬‬


‫ﻟﺘﻜﻦ ‪ Ω‬ﻧﻘﻄﺔ ﻣﻦ اﳌﺴﺘﻮي و ﻟﻴﻜﻦ ‪ k‬ﻋﺪد ًا ﺣﻘﻴﻘﻴﺎ ﻏﲑ ﻣﻌﺪوم‪.‬‬ ‫•‬

‫اﻟﺘﺤﺎﻛﻲ اﻟﺬي ﻣﺮﻛﺰه ‪ Ω‬و ﻧﺴﺒﺘﻪ ‪ ، k‬و اﻟﺬي ﻧﺮﻣﺰ إﻟﻴﻪ ﺑﺎﻟﺮﻣﺰ )‪ ، ℋ (Ω, k‬ﻫﻮ اﻟﺘﺤﻮﻳﻞ اﻟﻨﻘﻄﻲ ﻟﻠﻤﺴﺘﻮي 𝒫 ﰲ ﻧﻔﺴﻪ‬
‫و اﻟﺬي ﻳﺮﻓﻖ ﺑﻜﻞ ﻧﻘﻄﺔ ‪ M‬ﻣﻦ 𝒫 اﻟﻨﻘﻄﺔ ‪ M‬ﺑﺤﻴﺚ ﻳﻜﻮن ‪. ΩM = k ⋅ ΩM :‬‬

‫𝟰𝟱‬

‫‪http ://tinyurl.com/Malki1718‬‬ ‫‪0‬‬


‫‪ .IV‬ﲤﺜﻴﻞ اﻟﺘﺤﻮﯾﻼت اﻟﻨﻘﻄﻴﺔ ﰲ اﳌﺴـﺘﻮي اﳌﺮﻛﺐ‬
‫‪.‬‬
‫‪M′‬‬
‫‪A′‬‬

‫‪Ŕž Ŧ Ľ‬‬
‫‪M‬‬ ‫→‬
‫‪−‬‬
‫‪V‬‬
‫‪M‬‬
‫‪Ω‬‬
‫‪k>0‬‬

‫‪œǃ‬‬
‫‪Ω‬‬ ‫‪A‬‬ ‫‪M′‬‬

‫‪k<0‬‬

‫‪M‬‬

‫‪Ŀ‬‬
‫‪M′‬‬

‫ﺷﻜﻞ ‪ :3.IV‬اﻟﺘﺤﺎﻛﻲ‬ ‫ﺷﻜﻞ ‪ :2.IV‬اﻹﻧﺴﺤﺎب‬

‫ﺧﻮاص )ﻣﺮاﺟﻌﺔ( ‪:‬‬


‫ﻧُﺬﻛّﺮ ﻓﻴﲈ ﻳﲇ )ﺑﺪون ﺑﺮﻫﺎن( ﺑﺎﳋﻮاص اﻷﺳﺎﺳﻴﺔ ﻟﻜﻞ ﻣﻦ اﻹﻧﺴﺤﺎب و اﻟﺘﺤﺎﻛﻲ ‪:‬‬

‫‪ •1‬اﻹﻧﺴﺤﺎب ⃗ 𝒯 ﺗﻄﺒﻴﻖ ﺗﻘﺎﺑﲇ ﻟﻠﻤﺴﺘﻮي 𝒫 ﰲ ﻧﻔﺴﻪ‪ ،‬و اﻟﺘﺤﻮﻳﻞ اﻟﻌﻜﴘ ﻟﻪ ﻫﻮ اﻹﻧﺴﺤﺎب ⃗ 𝒯 ‪.‬‬
‫‪−V‬‬ ‫‪V‬‬
‫إذا ﻛﺎن ⃗‪ V⃗ ≠ 0‬ﻓﻼ ﺗﻮﺟﺪ ﻧﻘﻄﺔ ﻣﻀﺎﻋﻔﺔ )ﺻﺎﻣﺪة( ‪ ،‬و إذا ﻛﺎن ‪ّ V⃗ = 0‬‬
‫ﻓﺈن 𝒫𝟙 = ⃗ 𝒯 ﺣﻴﺚ 𝒫𝟙 ﻫﻮ اﻟﺘﺤﻮﻳﻞ‬ ‫⃗‬
‫‪0‬‬
‫اﳊﻴﺎدي )اﳌﻄﺎﺑﻖ( ﻟﻠﻤﺴﺘﻮي )اﻟﺘﺤﻮﻳﻞ اﻟﺬي ﻳﺮﻓﻖ ﺑﻜﻞ ﻧﻘﻄﺔ 𝒫 ∈ ‪ M‬اﻟﻨﻘﻄﺔ ‪ M‬ﻧﻔﺴﻬﺎ(‪.‬‬
‫‪IV‬‬

‫𝒯‪.‬‬ ‫ﺗﺮﻛﻴﺐ اﻹﻧﺴﺤﺎﺑﲔ ⃗ 𝒯 و 𝒯 ﻫﻮ اﻹﻧﺴﺤﺎب‬


‫‪V⃗ + V‬‬ ‫‪V‬‬ ‫‪V‬‬
‫اﻹﻧﺴﺤﺎب ﳛﻔﻆ اﳌﺴﺎﻓﺎت )ﻧﻘﻮل أﻧﻪ ﺗﺴﺎوي ﻗﻴﺎس( و اﻟﺘﺤﺎﻛﻲ اﻟﺬي ﻧﺴﺒﺘﻪ ‪ k‬ﻳﴬﲠﺎ ﺑﺎﻟﻌﺪد |‪. |k‬‬
‫‪1‬‬
‫‪) ℋ Ω,‬ﻟﻪ ﻧﻔﺲ‬ ‫‪ •2‬اﻟﺘﺤﺎﻛﻲ )‪ ℋ (Ω, k‬ﺗﻄﺒﻴﻖ ﺗﻘﺎﺑﲇ ﻟﻠﻤﺴﺘﻮي 𝒫 ﰲ ﻧﻔﺴﻪ ‪ ،‬و اﻟﺘﺤﻮﻳﻞ اﻟﻌﻜﴘ ﻟﻪ ﻫﻮ اﻟﺘﺤﺎﻛﻲ‬
‫‪k‬‬
‫اﳌﺮﻛﺰ( ‪.‬‬
‫إذن ﻳﻜﻮن اﻟﺘﺤﺎﻛﻲ ﺗﻀﺎﻣﻨﻴﺎ إذا و ﻓﻘﻂ إذا ﻛﺎن ‪. k = ±1‬‬
‫‪ņőƱŀ Ţ‬‬
‫إذا ﻛﺎن ‪ k ≠ 1‬ﺗﻜﻮن اﻟﻨﻘﻄﺔ ‪ Ω‬ﻫﻲ اﻟﻨﻘﻄﺔ اﳌﻀﺎﻋﻔﺔ )اﻟﺼﺎﻣﺪة( اﻟﻮﺣﻴﺪة ‪ ،‬و إذا ﻛﺎن ‪ّ k = 1‬‬
‫ﻓﺈن ﲨﻴﻊ اﻟﻨﻘﻂ ﺻﺎﻣﺪة‬
‫)ﻷن ⃗‪.(ΩM = kΩM ⟺ (1 − k) ΩM = 0‬‬ ‫ّ‬
‫ﺗﺮﻛﻴﺐ اﻟﺘﺤﺎﻛﻴﲔ ) ‪ ℋ (Ω, k‬و ) ‪) ℋ (Ω, k‬ﻧﻔﺲ اﳌﺮﻛﺰ( ﻫﻮ اﻟﺘﺤﺎﻛﻲ ) ‪) ℋ (Ω, k k‬ﻧﻔﺲ اﳌﺮﻛﺰ( ‪.‬‬
‫‪ •3‬ﺗﺮﻛﻴﺐ اﻧﺴﺤﺎب و ٍ‬
‫ﲢﺎك ﻧﺴﺒﺘﻪ ‪) k‬ﺑﺄي ﺗﺮﺗﻴﺐ( ﻫﻮ ﲢﺎك ﻧﺴﺒﺘﻪ ‪. k‬‬
‫‪ƕ‬‬

‫‪ •1‬ﻟﻴﻜﻦ ⃗‪ V‬ﺷﻌﺎﻋ ًﺎ ﻻﺣﻘﺘﻪ ‪. z‬‬ ‫ﻣﺒﺮﻫﻨﺔ ‪: 26‬‬

‫‪.z = z+z‬‬ ‫اﻟﻌﺒﺎرة اﳌﺮﻛﺒﺔ ﻟﻺﻧﺴﺤﺎب ⃗ 𝒯 اﻟﺬي ﺷﻌﺎﻋﻪ ⃗‪ V‬ﻫﻲ ‪:‬‬


‫‪V‬‬
‫‪ũŏ‬‬

‫‪ •2‬ﻟﺘﻜﻦ ‪ Ω‬ﻧﻘﻄﺔ ﻣﻦ اﳌﺴﺘﻮي ﻻﺣﻘﺘﻬﺎ 𝜔 و ﻟﻴﻜﻦ ‪ k‬ﻋﺪد ًا ﺣﻘﻴﻘﻴﺎ ﻏﲑ ﻣﻌﺪوم‪.‬‬


‫اﻟﻌﺒﺎرة اﳌﺮﻛﺒﺔ ﻟﻠﺘﺤﺎﻛﻲ )‪ ℋ (Ω, k‬اﻟﺬي ﻣﺮﻛﺰه ‪ Ω‬و ﻧﺴﺒﺘﻪ ‪ k‬ﻫﻲ ‪:‬‬
‫𝜔 )‪.( z = kz + (1 − k‬‬ ‫)𝜔 ‪) z − 𝜔 = k (z −‬أو‬

‫‪a‬‬
‫⃗‪) V‬أي ‪ z = x + ıy ،(z = a + ıb‬و ‪. z = x + ıy‬‬ ‫‪ •1‬ﻧﻀﻊ‬ ‫ﺍﻟﱪﻫﺎﻥ‪.‬‬
‫‪b‬‬

‫𝟱𝟱‬

‫‪http ://tinyurl.com/Malki1718‬‬ ‫‪0‬‬


‫‪ .2.IV‬ﲤﺜﻴﻞ اﻟﺘﺤﻮﯾﻼت اﻟﻨﻘﻄﻴﺔ ﰲ اﳌﺴـﺘﻮي اﳌﺮﻛﺐ‬
‫‪.‬‬
‫‪M‬‬ ‫‪M‬‬

‫‪Ŕž Ŧ Ľ‬‬
‫‪M′‬‬ ‫‪M′‬‬

‫‪œǃ‬‬
‫‪A‬‬ ‫‪A‬‬
‫‪A‬‬ ‫‪′‬‬ ‫‪A′‬‬
‫‪θ‬‬ ‫‪θ‬‬

‫‪Ŀ‬‬
‫‪Ω‬‬ ‫‪Ω‬‬

‫‪θ>0‬‬ ‫‪θ<0‬‬
‫ﺷﻜﻞ ‪ :4.IV‬اﻟﺪوران‬

‫‪x =x+a‬‬
‫ﻧﻌﻠﻢ ّ‬
‫أن اﻟﻌﺒﺎرة اﻟﺘﺤﻠﻴﻠﻴﺔ ﻟﻺﻧﺴﺤﺎب ⃗ 𝒯 ﻫﻲ ‪:‬‬
‫‪y =y+b‬‬ ‫‪V‬‬
‫و ﻣﻨﻪ ‪:‬‬
‫‪z = x + ıy = (x + a) + ı (y + b) = (x + ıy) + (a + ıb) = z + z‬‬

‫‪ •2‬ﻻﺣﻘﺔ اﻟﺸﻌﺎع ‪ ΩM‬ﻫﻲ 𝜔 ‪ z −‬و ﻻﺣﻘﺔ اﻟﺸﻌﺎع ‪ ΩM‬ﻫﻲ 𝜔 ‪ z −‬إذن اﳌﺴﺎواة ‪ ΩM = k ⋅ ΩM‬ﺗُﻜﺘﺐ ‪:‬‬

‫‪IV‬‬
‫)𝜔 ‪. z − 𝜔 = k (z −‬‬
‫■‬

‫ﺍﻟﺪﻭﺭﺍﻥ‬ ‫‪2.2.IV‬‬

‫ﺗﻌﺮﻳﻒ ‪ : 15‬ﻟﺘﻜﻦ ‪ Ω‬ﻧﻘﻄﺔ ﺛﺎﺑﺘﺔ ﰲ اﳌﺴﺘﻮي 𝒫 و 𝜃 زاوﻳﺔ ﻣﻮﺟﻬﺔ‪.‬‬


‫‪ņőƱŀ Ţ‬‬
‫ﻧﺴﻤﻲ دوراﻧ ًﺎ ﻣﺮﻛﺰه ‪ Ω‬و زاوﻳﺘﻪ 𝜃‪ ،‬و اﻟﺬي ﻧﺮﻣﺰ إﻟﻴﻪ ﺑﺎﻟﺮﻣﺰ )𝜃 ‪ ، ℛ (Ω,‬اﻟﺘﺤﻮﻳﻞ اﻟﻨﻘﻄﻲ اﻟﺬي ﻳﺮﻓﻖ ﺑﺎﻟﻨﻘﻄﺔ ‪ Ω‬اﻟﻨﻘﻄﺔ ‪Ω‬‬
‫⎧‬ ‫‪ΩM = ΩM‬‬
‫‪.‬‬ ‫ﻧﻔﺴﻬﺎ و ﻳﺮﻓﻖ ﺑﻜﻞ ﻧﻘﻄﺔ ‪ M‬ﲣﺘﻠﻒ ﻋﻦ ‪ Ω‬اﻟﻨﻘﻄﺔ ‪ M‬ﺑﺤﻴﺚ ‪:‬‬
‫‪⎨ ΩM, ΩM‬‬ ‫)𝜋‪= 𝜃 (mod 2‬‬
‫⎩‬

‫ﳏﻮﻟﺔ ﻧﻘﻄﺔ ‪ M‬وﻓﻖ اﻟﺪوران )𝜃 ‪ ، ℛ (Ω,‬ﻧﺮﺳﻢ ﻗﻮﺳ ًﺎ ﻗﻴﺎﺳﻪ 𝜃 ﻣﻦ داﺋﺮة ﻣﺮﻛﺰﻫﺎ ‪ Ω‬و ﻧﺼﻒ ﻗﻄﺮﻫﺎ ‪ ، ΩM‬ﻓﺘﻜﻮن‬
‫ﻹﳚﺎد ّ‬
‫ﳖﺎﻳﺘﻪ ﻫﻲ اﻟﻨﻘﻄﺔ ‪. M‬‬
‫‪ũŏ‬‬ ‫‪ƕ‬‬

‫𝟲𝟱‬

‫‪http ://tinyurl.com/Malki1718‬‬ ‫‪0‬‬


‫‪ .IV‬ﲤﺜﻴﻞ اﻟﺘﺤﻮﯾﻼت اﻟﻨﻘﻄﻴﺔ ﰲ اﳌﺴـﺘﻮي اﳌﺮﻛﺐ‬
‫‪.‬‬
‫ﺧﻮاص )ﻣﺮاﺟﻌﺔ( ‪:‬‬

‫‪Ŕž Ŧ Ľ‬‬
‫ﻧُﺬﻛّﺮ ﻓﻴﲈ ﻳﲇ )ﺑﺪون ﺑﺮﻫﺎن( ﺑﺎﳋﻮاص اﻷﺳﺎﺳﻴﺔ ﻟﻠﺪوران ‪:‬‬

‫‪œǃ‬‬
‫ﻳﻤﻜﻦ اﻟﺘﻌﺒﲑ ﻋﻦ اﻟﺪوران )𝜃 ‪ ℛ (Ω,‬ﻛﱰﻛﻴﺐ ﺗﻨﺎﻇﺮﻳﻦ ﻋﻤﻮدﻳﲔ ﺣﻮل ﻣﺴﺘﻘﻴﻤﲔ إﺧﺘﻴﺎرﻳﲔ ﻣﺘﻘﺎﻃﻌﲔ ﰲ اﻟﻨﻘﻄﺔ ‪Ω‬‬ ‫•‬‫‪1‬‬
‫و ﻗﻴﺲ اﻟﺰاوﻳﺔ ﺑﻴﻨﻬﲈ ﻳﺴﺎوي 𝜃 ‪ ،‬و ذﻟﻚ ﺑﻌﺪد ﻏﲑ ٍ‬
‫ﻣﻨﺘﻪ ﻣﻦ اﳊﺎﻻت )ﺑﻤﻌﻨﻰ ّ‬
‫أن أﺣﺪ اﳌﺴﺘﻘﻴﻤﲔ إﺧﺘﻴﺎري و اﻵﺧﺮ‬
‫‪2‬‬
‫𝜃‬

‫‪Ŀ‬‬
‫ﻳﺼﻨﻊ ﻣﻌﻪ اﻟﺰاوﻳﺔ اﳌﻌﻄﺎة (‪.‬‬
‫‪2‬‬
‫‪ •2‬اﻟﺪوران )𝜃 ‪ ℛ (Ω,‬ﺗﻄﺒﻴﻖ ﺗﻘﺎﺑﲇ ﻟﻠﻤﺴﺘﻮي 𝒫 ﰲ ﻧﻔﺴﻪ‪ ،‬و ﺗﻄﺒﻴﻘﻪ اﻟﻌﻜﴘ ﻫﻮ اﻟﺪوران )𝜃‪. ℛ − (Ω, −‬‬

‫‪ •3‬ﻳﻜﻮن اﻟﺪوران ﺗﻀﺎﻣﻨﻴﺎ إذا و ﻓﻘﻂ إذا ﻛﺎﻧﺖ زاوﻳﺘﻪ ﻣﻌﺪوﻣﺔ )اﻟﺘﻄﺒﻴﻖ اﳊﻴﺎدي( أو ﻛﺎﻧﺖ زاوﻳﺘﻪ ﻣﺴﺘﻘﻴﻤﺔ )ﺗﻨﺎﻇﺮ ﺑﺎﻟﻨﺴﺒﺔ‬
‫إﱃ ﻧﻘﻄﺔ(‪.‬‬
‫‪ •4‬ﻣﺮﻛﺰ اﻟﺪوران ﻫﻮ اﻟﻨﻘﻄﺔ اﻟﺼﺎﻣﺪة اﻟﻮﺣﻴﺪة إذا ﻛﺎن )𝜋‪. 𝜃 ≠ 0 (mod 2‬‬
‫‪ •5‬ﺗﺮﻛﻴﺐ اﻟﺪوراﻧﲔ ) 𝜃 ‪ ℛ (Ω,‬و ) 𝜃 ‪) ℛ (Ω,‬ﻧﻔﺲ اﳌﺮﻛﺰ( ﻫﻮ اﻟﺪوران ) 𝜃 ‪) ℛ (Ω, 𝜃 +‬ﻧﻔﺲ اﳌﺮﻛﺰ(‪.‬‬
‫‪ •6‬ﺗﺮﻛﻴﺐ اﻧﺴﺤﺎب ﻣﻊ دوران )أو اﻟﻌﻜﺲ( ﻫﻮ دوران ﻟﻪ ﻧﻔﺲ اﻟﺰاوﻳﺔ‪.‬‬
‫‪IV‬‬

‫ﻣﺒﺮﻫﻨﺔ ‪ : 27‬ﻟﺘﻜﻦ 𝒫 ∈ ‪ Ω‬ﻧﻘﻄﺔ ﻣﻦ اﳌﺴﺘﻮي ﻻﺣﻘﺘﻬﺎ 𝜔 و ﻟﻴﻜﻦ 𝜃 ﻋﺪد ًا ﺣﻘﻴﻘﻴﺎ‪.‬‬


‫اﻟﻌﺒﺎرة اﳌﺮﻛﺒﺔ ﻟﻠﺪوران )𝜃 ‪ ℛ (Ω,‬ﻫﻲ )𝜔 ‪) z − 𝜔 = e (z −‬أو 𝜔 ‪.( z = e z + 1 − e‬‬

‫ﺍﻟﱪﻫﺎﻥ‪ .‬ﻟﺘﻜﻦ ‪ z‬و ‪ z‬ﻻﺣﻘﺘ َْﻲ ‪ M‬و ‪ M‬ﻋﲆ اﻟﱰﺗﻴﺐ‪ .‬إذا ﻛﺎن ‪ّ M ≠ Ω‬‬
‫ﻓﺈن ‪:‬‬

‫‪M = ℛ (Ω, 𝜃) (M) ⟺ ΩM = ΩM‬‬ ‫‪ ΩM, ΩM‬و‬ ‫)𝜋‪= 𝜃 (mod 2‬‬


‫‪ņőƱŀ Ţ‬‬
‫𝜔‪z −‬‬
‫||𝜔 ‪⟺ |z − 𝜔| = ||z −‬‬ ‫‪ arg‬و‬ ‫)𝜋‪= 𝜃 (mod 2‬‬
‫𝜔‪z−‬‬
‫|𝜔 ‪|z −‬‬ ‫𝜔‪z −‬‬
‫| ⟺‬ ‫‪ arg‬و ‪| = 1‬‬ ‫)𝜋‪= 𝜃 (mod 2‬‬
‫|| 𝜔 ‪|| z −‬‬ ‫𝜔‪z−‬‬
‫𝜔‪z −‬‬
‫⟺‬ ‫‪= [1, 𝜃] = e‬‬
‫𝜔‪z−‬‬
‫)𝜔 ‪⟺ z − 𝜔 = e (z −‬‬
‫‪ƕ‬‬

‫■‬ ‫ﰲ اﳊﺎﻟﺔ اﳋﺎﺻﺔ ‪ّ ، M = Ω‬‬


‫ﻓﺈن ‪ M = Ω‬أي 𝜔 = ‪ z = z‬و اﳌﺴﺎواة )𝜔 ‪ z − 𝜔 = e (z −‬ﳏﻘﻘﺔ‪.‬‬

‫ﺍﻟﺘﺸﺎﺑﻪ ﺍﳌﺴﺘﻮﻱ ﺍﳌﺒﺎﺷﺮ‬ ‫‪3.IV‬‬


‫‪ũŏ‬‬

‫ﻌﺮف اﻟﺘﺸﺎﺑﻪ اﳌﺴﺘﻮي اﳌﺒﺎﴍ )𝜃 ‪ 𝒮 (k,‬اﻟﺬي ﻧﺴﺒﺘﻪ ‪ k > 0‬و زاوﻳﺘﻪ 𝜃 ﺑﺎﻟﻌﺒﺎرة ‪:‬‬
‫ُﻳ ّ‬
‫⎧‬ ‫‪M N = k ⋅ MN‬‬
‫𝒫 ∈ ‪∀M, N‬‬ ‫‪:‬‬
‫‪⎨ MN, M N‬‬ ‫)𝜋‪= 𝜃 (mod 2‬‬
‫⎩‬
‫ﺴﻬﻞ ﺑﻌﺾ اﻟﺸﺊ دراﺳﺘﻪ‪.‬‬
‫ﻟﻜﻦ ﰲ ﺑﺤﺜﻨﺎ ﻫﺬا‪ ،‬ﺳﻨﻌﻄﻲ ﺗﻌﺮﻳﻔﺎ آﺧﺮ ﻟﻪ ُﻳ ّ‬
‫𝟳𝟱‬

‫‪http ://tinyurl.com/Malki1718‬‬ ‫‪0‬‬


‫‪ .3.IV‬اﻟﺘﺸﺎﺑﻪ اﳌﺴـﺘﻮي اﳌﺒﺎﴍ‬
‫‪.‬‬
‫‪:‬‬ ‫ﻫﻲ‬ ‫اﳌﺮﻛﺒﻪ‬ ‫ﻋﺒﺎرﺗﻪ‬ ‫اﻟﺬي‬ ‫اﳌﺴﺘﻮي‬ ‫اﻟﺘﺤﻮﻳﻞ‬ ‫ﻫﻮ‬ ‫اﳌﺒﺎﴍ‬ ‫اﻟﺘﺸﺎﺑﻪ‬ ‫‪: 16‬‬ ‫ﺗﻌﺮﻳﻒ‬

‫‪Ŕž Ŧ Ľ‬‬
‫‪ℂ⟶ℂ‬‬
‫‪.‬‬ ‫∗‬
‫‪z ⟼ az + b‬‬ ‫;‬ ‫‪a∈ℂ , b∈ℂ‬‬

‫‪œǃ‬‬
‫ﺑﺪاﻳ ًﺔ‪ ،‬ﺑﺈﻣﻜﺎﻧﻨﺎ إﺛﺒﺎت ﻣﺎ ﻳﲇ ‪:‬‬

‫‪Ŀ‬‬
‫ﻣﺒﺮﻫﻨﺔ ‪ : 28‬اﻟﺘﺸﺎﺑﻪ اﳌﺒﺎﴍ ﳛﺎﻓﻆ ﻋﲆ اﻟﺰواﻳﺎ اﳌﻮﺟﻬﺔ و اﻟﻨﺴﺒﺔ ﺑﲔ اﳌﺴﺎﻓﺎت ﺑﻤﻌﻨﻰ ‪ :‬إذا ﻛﺎﻧﺖ ‪ A ، A ، A‬و ‪A‬‬
‫أرﺑﻊ ﻧﻘﻂ ﻣﻦ اﳌﺴﺘﻮي 𝒫 ﺑﺤﻴﺚ ‪ A ≠ A‬و ‪ A ≠ A‬و ﻛﺎﻧﺖ ‪ A ، A ، A‬و ‪ A‬ﺻﻮرﻫﺎ ﺑﺎﻟﺘﺸﺎﺑﻪ اﳌﺒﺎﴍ )𝜃 ‪𝒮 (k,‬‬
‫ّ‬
‫ﻓﺈن‪:‬‬
‫‪A A‬‬ ‫‪A A‬‬
‫‪A A ,A A‬‬ ‫‪= A A ,A A‬‬ ‫)𝜋‪(mod 2‬‬ ‫و‬ ‫=‬
‫‪A A‬‬ ‫‪A A‬‬

‫ﺍﻟﱪﻫﺎﻥ‪ .‬ﻟﻴﻜﻦ ‪ f : z ↦ az + b‬اﻟﺘﻤﺜﻴﻞ اﳌﺮﻛﺐ ﻟﻠﺘﺸﺎﺑﻪ )𝜃 ‪ . 𝒮 (k,‬ﻟﺘﻜﻦ ‪ z ، z ، z ، z‬ﻟﻮاﺣﻖ اﻟﻨﻘﻂ ‪، A ، A‬‬


‫‪ A ، A‬ﻋﲆ اﻟﱰﺗﻴﺐ و ﻟﺘﻜﻦ ‪ z ، z ، z ، z‬ﻟﻮاﺣﻖ اﻟﻨﻘﻂ ‪ A ، A ، A ، A‬ﻋﲆ اﻟﱰﺗﻴﺐ‪ .‬ﻟﺪﻳﻨﺎ ‪z = az + b :‬‬
‫ﻷﺟﻞ }‪ . i ∈ {1, 2, 3, 4‬ﻣﻨﻪ ‪:‬‬
‫) ‪z − z = a (z − z‬‬

‫‪IV‬‬
‫) ‪z − z = a (z − z‬‬

‫‪z −z‬‬ ‫‪z −z‬‬


‫=‬ ‫أن ‪ّ a ≠ 0‬‬
‫ﻓﺈن ‪:‬‬ ‫و ﺑﲈ ّ‬
‫‪z −z‬‬ ‫‪z −z‬‬
‫ﻣﻨﻪ ‪:‬‬
‫‪z −z‬‬ ‫‪z −z‬‬
‫‪A A ,A A‬‬ ‫‪= arg‬‬ ‫‪= arg‬‬ ‫‪= A A ,A A‬‬ ‫)𝜋‪(mod 2‬‬
‫‪z −z‬‬ ‫‪z −z‬‬
‫و‬
‫‪ņőƱŀ Ţ‬‬
‫‪A A‬‬ ‫‪|z − z | |z − z | A A‬‬
‫|=‬ ‫|=|‬ ‫=|‬
‫‪A A‬‬ ‫‪|| z − z || || z − z || A A‬‬
‫■‬

‫‪ •1‬اﻟﺘﺸﺎﺑﻪ اﳌﺒﺎﴍ ‪ f : z ↦ az + b‬ﲢﻮﻳﻞ ﺗﻘﺎﺑﲇ ﻟﻠﻤﺴﺘﻮي ﰲ ﻧﻔﺴﻪ و ﲢﻮﻳﻠﻪ اﻟﻌﻜﴘ ﻫﻮ اﻟﺘﺸﺎﺑﻪ اﳌﺒﺎﴍ‬ ‫ﻣﺒﺮﻫﻨﺔ ‪: 29‬‬
‫‪1‬‬ ‫‪b‬‬
‫‪. f−‬‬ ‫‪: z↦ z−‬‬
‫‪a‬‬ ‫‪a‬‬
‫‪ •2‬ﺗﺮﻛﻴﺐ ﺗﺸﺎﲠﲔ ﻣﺒﺎﴍﻳﻦ ﻫﻮ ﺗﺸﺎﺑﻪ ﻣﺒﺎﴍ‪.‬‬
‫‪ƕ‬‬

‫أن ‪ّ a ≠ 0‬‬
‫ﻓﺈن ‪:‬‬ ‫‪ •1‬ﺑﲈ ّ‬ ‫ﺍﻟﱪﻫﺎﻥ‪.‬‬
‫‪z −b‬‬
‫‪ũŏ‬‬

‫= ‪z = az + b ⟺ az = z − b ⟺ z‬‬
‫‪a‬‬
‫‪ f : z ↦ a z + b‬و ‪ f : z ↦ a z + b‬ﺗﺸﺎﲠﲔ ﻣﺒﺎﴍﻳﻦ‪ .‬ﻟﺪﻳﻨﺎ ‪:‬‬ ‫‪ •2‬ﻟﻴﻜﻦ‬
‫‪f (f (z)) = f (a z + b ) = a (a z + b ) + b = a a z + a b + b‬‬
‫و ﺑﻮﺿﻊ ‪ c = a a‬و ‪ d = a b + b‬ﻧﺠﺪ ‪ f ∘ f : z ↦ cz + d‬ﺣﻴﺚ ‪ c ≠ 0‬و ﻫﻲ اﻟﻌﺒﺎرة اﳌﺮﻛﺒﺔ ﻟﺘﺸﺎﺑﻪ‬
‫ﻣﺒﺎﴍ‪.‬‬
‫■‬

‫𝟴𝟱‬

‫‪http ://tinyurl.com/Malki1718‬‬ ‫‪0‬‬


‫‪ .IV‬ﲤﺜﻴﻞ اﻟﺘﺤﻮﯾﻼت اﻟﻨﻘﻄﻴﺔ ﰲ اﳌﺴـﺘﻮي اﳌﺮﻛﺐ‬
‫‪.‬‬

‫‪Ŕž Ŧ Ľ‬‬
‫‪M ′′‬‬

‫‪œǃ‬‬
‫‪M′‬‬

‫‪Ŀ‬‬
‫‪−‬‬
‫→‬ ‫‪M‬‬
‫‪j‬‬
‫‪α‬‬
‫‪Ω‬‬

‫‪O‬‬ ‫‪−‬‬
‫→‬
‫‪i‬‬
‫ﺷﻜﻞ ‪ :5.IV‬اﻟﺘﺸﺎﺑﻪ اﳌﺒﺎﴍ‬

‫ﻟﻴﻜﻦ ∗‪ a ∈ ℂ‬و ‪ . b ∈ ℂ‬ﻟﻴﻜﻦ ‪ f‬اﻟﺘﺸﺎﺑﻪ اﳌﺒﺎﴍ اﻟﺬي ﻋﺒﺎرﺗﻪ اﳌﺮﻛﺒﺔ ﻫﻲ ‪. f : z ↦ az + b‬‬ ‫ﻧﻈﺮﻳﺔ ‪: 3‬‬
‫‪IV‬‬

‫‪ •1‬إذا ﻛﺎن ‪ّ a = 1‬‬


‫ﻓﺈن ‪ f‬ﻫﻮ اﻹﻧﺴﺤﺎب اﻟﺬي ﻻﺣﻘﺔ ﺷﻌﺎﻋﻪ ‪. b‬‬
‫‪ •2‬إذا ﻛﺎن ‪ a ≠ 1‬ﻓﺈﻧّﻪ ﻟﻠﺘﺸﺎﺑﻪ اﳌﺒﺎﴍ ‪ f‬ﻧﻘﻄﺔ ﺻﺎﻣﺪة )ﻣﻀﺎﻋﻔﺔ( ﻭﺣﻴﺪﺓ ))‪ Ω . (Ω = f (Ω‬ﺗُﺪﻋﻰ ﻣﺮﻛﺰ اﻟﺘﺸﺎﺑﻪ‪.‬‬
‫ﺑﺎﻹﺿﺎﻓﺔ إﱃ ذﻟﻚ‪ ،‬إذا ﻛﺎن ‪:‬‬
‫𝛼 ﻋﻤﺪة ﻟﻠﻌﺪد اﳌﺮﻛﺐ ‪.a‬‬ ‫•‬

‫)𝛼 ‪ ℛ = ℛ (Ω,‬ﻫﻮ اﻟﺪوران اﻟﺬي ﻣﺮﻛﺰه ‪ Ω‬و زاوﻳﺘﻪ 𝛼 ‪.‬‬ ‫•‬

‫)|‪ ℋ = ℋ (Ω, |a‬ﻫﻮ اﻟﺘﺤﺎﻛﻲ اﻟﺬي ﻣﺮﻛﺰه ‪ Ω‬و ﻧﺴﺒﺘﻪ |‪. |a‬‬ ‫•‬
‫‪ņőƱŀ Ţ‬‬
‫ّ‬
‫ﻓﺈن ‪ُ f‬ﻳﻜﺘﺐ ﻋﲆ ﺷﻜﻞ ﺗﺮﻛﻴﺐ ‪ ℛ‬و ‪. f = ℛ ∘ ℋ = ℋ ∘ ℛ : ℋ‬‬
‫اﺧﺘﺼﺎرا‪ ،‬ﻗﻴﺎس( زاوﻳﺔ اﻟﺘﺸﺎﺑﻪ‪.‬‬
‫ً‬ ‫ُﻳﺴﻤﻰ اﻟﻌﺪد اﳊﻘﻴﻘﻲ |‪ |a‬ﻧﺴﺒﺔ اﻟﺘﺸﺎﺑﻪ و اﻟﻌﺪد 𝛼 إﺣﺪى ﻗﻴﺎﺳﺎت )أو‪،‬‬
‫‪ •3‬إذا ﻛﺎن ∗‪ّ a ∈ ℝ‬‬
‫ﻓﺈن ‪ f‬ﻫﻮ اﻟﺘﺤﺎﻛﻲ اﻟﺬي ﻣﺮﻛﺰه ‪ Ω‬و ﻧﺴﺒﺘﻪ |‪. |a‬‬
‫‪ •4‬إذا ﻛﺎن ‪ّ |a| = 1‬‬
‫ﻓﺈن ‪ f‬ﻫﻮ اﻟﺪوران اﻟﺬي ﻣﺮﻛﺰه ‪ Ω‬و زاوﻳﺘﻪ 𝛼 ‪.‬‬
‫‪ƕ‬‬

‫‪ •1‬إذا ﻛﺎن ‪ّ a = 1‬‬


‫ﻓﺈن ‪ f(z) = z + b‬و ﺣﺴﺐ اﳌﱪﻫﻨﺔ ‪ f ، 26‬ﻫﻮ اﻹﻧﺴﺤﺎب اﻟﺬي ﺷﻌﺎﻋﻪ ‪. b‬‬ ‫ﺍﻟﱪﻫﺎﻥ‪.‬‬
‫‪ •2‬ﻧﻔﺮض ‪ a ≠ 1‬و ﻟﻨﺒﺤﺚ ﻋﻦ اﻟﻨﻘﻂ اﻟﺼﺎﻣﺪه ﺑﺎﻟﺘﺤﻮﻳﻞ ‪: f‬‬
‫‪b‬‬
‫‪ũŏ‬‬

‫= ‪ ) z‬ﻓﺮﺿﻨﺎ ‪ . (! a ≠ 1‬ﻟﺘﻜﻦ ‪ Ω‬اﻟﻨﻘﻄﺔ ذات‬ ‫ﻟﺘﻜﻦ ‪ z‬ﻻﺣﻘﺔ ﻧﻘﻄﺔ ﺻﺎﻣﺪة‪ .‬ﻟﺪﻳﻨﺎ ‪ z = az + b‬ﻣﻨﻪ‬
‫‪1−a‬‬
‫اﻟﻼﺣﻘﺔ ‪ . z‬ﺣﺴﺐ ﻣﺎ ﺳﺒﻖ‪ ،‬اﻟﻨﻘﻄﺔ ‪ Ω‬ﻫﻲ اﻟﻨﻘﻄﺔ اﻟﺼﺎﻣﺪة اﻟﻮﺣﻴﺪة ﺑﺎﻟﺘﺤﻮﻳﻞ ‪ . f‬ﻟﺘﻜﻦ ‪ M‬اﻟﻨﻘﻄﺔ ذات اﻟﻼﺣﻘﺔ ‪z‬‬
‫و ﻟﺘﻜﻦ ‪ M‬اﻟﻨﻘﻄﺔ ذات اﻟﻼﺣﻘﺔ ‪ . z = f(z) = az + b‬ﻟﺪﻳﻨﺎ ‪ . z − z = a (z − z ) :‬ﻟﺘﻜﻦ 𝛼 ﻋﻤﺪة ﻟﻠﻌﺪد ‪a‬‬
‫‪ ℋ ،‬اﻟﺘﺤﺎﻛﻲ )|‪ ℋ (Ω, |a‬و ‪ ℛ‬اﻟﺪوران )𝛼 ‪. ℛ (Ω,‬‬
‫ﻟﻨُﺜﺒﺖ ّ‬
‫أن ‪ُ f‬ﻳﻜﺘﺐ ﻋﲆ ﺷﻜﻞ ﺗﺮﻛﻴﺐ ‪ ℋ‬و ‪ . ℛ‬ﻟﺘﻜﻦ ‪ z‬ﻻﺣﻘﺔ )‪ ℛ (M‬و ‪ z‬ﻻﺣﻘﺔ ))‪ . ℋ (ℛ (M‬ﺣﺴﺐ‬

‫𝟵𝟱‬

‫‪http ://tinyurl.com/Malki1718‬‬ ‫‪0‬‬


‫‪ .3.IV‬اﻟﺘﺸﺎﺑﻪ اﳌﺴـﺘﻮي اﳌﺒﺎﴍ‬
‫‪.‬‬
‫اﳌﱪﻫﻨﺔ ‪ 26‬ﺻﻔﺤﺔ ‪ 55‬ﻟﺪﻳﻨﺎ ‪:‬‬

‫‪Ŕž Ŧ Ľ‬‬
‫‪z −z =e‬‬ ‫) ‪(z − z‬‬
‫) ‪z − z = |a| (z − z‬‬

‫‪œǃ‬‬
‫إذن‬
‫‪z − z = |a| e‬‬ ‫) ‪(z − z ) = a (z − z‬‬
‫أن ‪ z = z‬أي ّ‬
‫أن ‪ z‬ﻫﻲ ﻻﺣﻘﺔ اﻟﻨﻘﻄﺔ )‪ f (M‬أو ﺑﺘﻌﺒﲑ آﺧﺮ ‪. f = ℋ ∘ ℛ‬‬ ‫و ﻫﺬا ﻳﻌﻨﻲ ّ‬

‫‪Ŀ‬‬
‫ﻧﺜﺒﺖ ﺑﻨﻔﺲ اﻟﻄﺮﻳﻘﺔ ّ‬
‫أن ‪. f = ℛ ∘ ℋ‬‬
‫■‬

‫اﳌﻌﺮف ﺑﺎﻟﺼﻴﻐﺔ اﳌﺮﻛﺒﺔ ‪:‬‬


‫ﻋﲔ ﰲ ﻛﻞ ﺣﺎﻟﺔ ﻃﺒﻴﻌﺔ و ﻋﻨﺎﴏ اﻟﺘﺤﻮﻳﻞ ّ‬
‫ﱢ‬ ‫ﺗﻄﺒﻴﻖ ‪: 5‬‬

‫‪f : z ↦ (1 − ı) z‬‬ ‫•‬ ‫‪f : z ↦ ız + ı − 1‬‬ ‫•‬

‫‪f : z ↦ −2z + 1 − ı‬‬ ‫•‬ ‫‪f : z ↦ (1 + ı) z + 2 − 4ı‬‬ ‫•‬

‫ﺍﳊﻞّ‪.‬‬
‫𝜋‬
‫‪.a = ı = 1,‬‬ ‫‪ f‬دوران ّ‬
‫ﻷن‬ ‫•‬
‫‪2‬‬
‫‪ı−1‬‬

‫‪IV‬‬
‫= ‪ z‬أي )‪. Ω (−1, 0‬‬ ‫ﻣﺮﻛﺰه اﻟﻨﻘﻄﺔ ‪ Ω‬اﻟﺘﻲ ﻻﺣﻘﺘﻬﺎ ‪= −1‬‬
‫‪1−ı‬‬
‫𝜋‬
‫إذن ‪ f‬ﻫﻮ اﻟﺪوران ‪. ℛ (−1, 0) ,‬‬
‫‪2‬‬
‫𝜋‬
‫ﻷن ‪. a = 1 + ı = √2,‬‬ ‫‪ f‬ﺗﺸﺎﺑﻪ ّ‬ ‫•‬
‫‪4‬‬
‫‪2 − 4ı‬‬
‫= ‪ z‬أي ّ‬
‫أن )‪. Ω (4, 2‬‬ ‫ﻻﺣﻘﺔ اﻟﻨﻘﻄﺔ اﻟﺼﺎﻣﺪة اﻟﻮﺣﻴﺪة ‪ Ω‬ﻫﻲ ‪= 4 + 2ı‬‬
‫)‪1 − (1 + ı‬‬
‫𝜋‬
‫‪ņőƱŀ Ţ‬‬
‫إذن ‪ f‬ﻫﻮ اﻟﺘﺸﺎﺑﻪ ‪. 𝒮 (4, 2) , √2,‬‬
‫‪4‬‬
‫𝜋‬
‫ﻷن ‪. a = 1 − ı = √2, −‬‬ ‫‪ f‬ﺗﺸﺎﺑﻪ ّ‬ ‫•‬
‫‪4‬‬
‫‪0‬‬
‫= ‪ z‬أي ّ‬
‫أن )‪. Ω (0, 0‬‬ ‫ﻻﺣﻘﺔ اﻟﻨﻘﻄﺔ اﻟﺼﺎﻣﺪة اﻟﻮﺣﻴﺪة ‪ Ω‬ﻫﻲ ‪= 0‬‬
‫)‪1 − (1 − ı‬‬
‫𝜋‬
‫إذن ‪ f‬ﻫﻮ اﻟﺘﺸﺎﺑﻪ ‪. 𝒮 O, √2, −‬‬
‫‪4‬‬
‫‪ f‬ﲢﺎﻛﻲ ﻷن ّ ∗‪. a = −2 ∈ ℝ‬‬ ‫•‬
‫‪ƕ‬‬

‫‪1 1‬‬ ‫‪1−ı‬‬ ‫‪1 1‬‬


‫‪.Ω‬‬ ‫‪,−‬‬ ‫= ‪ z‬أي ّ‬
‫أن‬ ‫ﻻﺣﻘﺔ اﻟﻨﻘﻄﺔ اﻟﺼﺎﻣﺪة اﻟﻮﺣﻴﺪة ‪ Ω‬ﻫﻲ ‪= − ı‬‬
‫‪3 3‬‬ ‫‪1 − (−2) 3 3‬‬
‫‪1 1‬‬
‫‪.ℋ‬‬ ‫‪,−‬‬ ‫‪, −2‬‬ ‫إذن ‪ f‬ﻫﻮ اﻟﺘﺤﺎﻛﻲ‬
‫‪3 3‬‬
‫‪ũŏ‬‬

‫■‬

‫ﺗﻄﺒﻴﻖ ‪ M ، A : 6‬و ‪ P‬ﺛﻼث ﻧﻘﻂ ﻣﻦ اﳌﺴﺘﻮي ﻟﻮاﺣﻘﻬﺎ ﻫﻲ ‪ z ، 1‬و ‪ z‬ﻋﲆ اﻟﱰﺗﻴﺐ‪.‬‬


‫أوﺟﺪ اﻷﻋﺪاد اﳌﺮﻛﺒﺔ ‪ z‬ﻟﻜﻲ ﻳﻜﻮن اﳌﺜﻠﺚ ‪ AMP‬ﻗﺎﺋﲈ ﰲ ‪ A‬و ﻣﺘﺴﺎوي اﻟﺴﺎﻗﲔ‪.‬‬

‫𝟬𝟲‬

‫‪http ://tinyurl.com/Malki1718‬‬ ‫‪0‬‬


‫‪ .IV‬ﲤﺜﻴﻞ اﻟﺘﺤﻮﯾﻼت اﻟﻨﻘﻄﻴﺔ ﰲ اﳌﺴـﺘﻮي اﳌﺮﻛﺐ‬
‫‪.‬‬
‫‪M‬‬

‫‪Ŕž Ŧ Ľ‬‬
‫‪œǃ‬‬
‫‪S‬‬
‫‪O‬‬

‫‪Ŀ‬‬
‫ﺷﻜﻞ ‪7.IV‬‬ ‫‪M′‬‬ ‫‪T‬‬ ‫‪M ′′‬‬

‫ﺷﻜﻞ ‪6.IV‬‬

‫ﺍﳊﻞّ‪ .‬ﻟﻜﻲ ُﳛﻘﻖ اﻟﻌﺪد ‪ z‬اﳌﻄﻠﻮب‪ ،‬ﳚﺐ و ﻳﻜﻔﻲ أن ﻧﻨﺘﻘﻞ ﻣﻦ ‪ M‬إﱃ ‪ P‬ﺑﺎﻟﺪوران اﻟﺬي ﻣﺮﻛﺰه ‪ A‬و زاوﻳﺘﻪ ‪. ±‬‬
‫𝜋‬
‫‪2‬‬
‫𝜋‬ ‫𝜋‬
‫اﻟﻌﺪد اﳌﺮﻛﺐ اﻟﺬي ﻃﻮﻳﻠﺘﻪ ‪ 1‬و ‪ −‬ﻋﻤﺪة ﻟﻪ ﻫﻮ ‪ ، −ı‬و اﻟﻌﺪد اﳌﺮﻛﺐ اﻟﺬي ﻃﻮﻳﻠﺘﻪ ‪ 1‬و ﻋﻤﺪة ﻟﻪ ﻫﻮ ‪. ı‬‬
‫‪2‬‬ ‫‪2‬‬
‫ﻓﺎﻷﻋﺪاد ‪ z‬اﻟﺘﻲ ﻧﺒﺤﺚ ﻋﻨﻬﺎ ﻫﻲ اﻷﻋﺪاد اﻟﺘﻲ ﲢﻘﻖ )‪ z − 1 = −ı (z − 1‬أو )‪z − 1 = ı (z − 1‬‬
‫أي ‪ (z − 1) (z + 1 + ı) = 0‬أو ‪. (z − 1) (z + 1 − ı) = 0‬‬
‫■‬ ‫ﻧﺠﺪ ‪ P ، M ، A ) z = 1 :‬ﻣﺘﻄﺎﺑﻘﺔ( و ‪ z = −1 + ı‬و ‪z = −1 − ı‬‬
‫‪IV‬‬

‫ﲢﻮﻳﻞ ﻧﻘﻄﻲ ﺁﺧﺮ ‪ :‬ﺍﻹﻧﺰﻻﻕ‬ ‫‪1.3.IV‬‬

‫‪ℂ⟶ℂ‬‬
‫‪f :‬؛‬ ‫ﻟﻨﺪرس اﻟﺘﺤﻮﻳﻞ اﻟﻨﻘﻄﻲ اﳌﻌﺮف ﺑﺎﻟﻌﺒﺎرة اﳌﺮﻛﺒﺔ ‪:‬‬
‫‪z ⟼ f(z) = z + u‬‬
‫ﺣﻴﺚ ‪ u‬ﻋﺪد ﺣﻘﻴﻘﻲ ﻏﲑ ﻣﻌﺪوم‪. u ∈ ℝ∗ :‬‬
‫ﻟﺘﻜﻦ ‪ M‬ﻧﻘﻄﺔ ﻣﻦ اﳌﺴﺘﻮي ﻻﺣﻘﺘﻬﺎ ‪ ، z = x + ıy‬ﺣﻴﺚ ‪ . x, y ∈ ℝ‬اﻟﻨﻘﻄﺔ ‪ M‬اﻟﺘﻲ ﻻﺣﻘﺘﻬﺎ ‪ z = x − ıy‬ﻫﻲ ﻧﻈﲑة اﻟﻨﻘﻄﺔ‬
‫ﺳﻤﻴﻨﺎه ﳏﻮر اﻷﻋﺪاد اﳊﻘﻴﻘﻴﺔ(‪ .‬ﻟﺘﻜﻦ ‪ M‬اﻟﻨﻘﻄﺔ اﻟﺘﻲ ﻻﺣﻘﺘﻬﺎ ‪f(z) = z + u‬‬ ‫‪ M‬ﺑﺎﻟﻨﺴﺒﺔ ﳌﺤﻮر اﻟﻔﻮاﺻﻞ )‪) (Ox‬اﻟﺬي ّ‬
‫‪ņőƱŀ Ţ‬‬
‫∗‬
‫ﻓﺈن ⃗ ‪. M M = ui‬‬‫أن ‪ّ u ∈ ℝ‬‬ ‫)أي ﺻﻮرة اﻟﻨﻘﻄﺔ ‪ M‬ﺑﺎﻟﺘﺤﻮﻳﻞ ‪ .( f‬ﻻﺣﻘﺔ اﻟﺸﻌﺎع ‪ M M‬ﻫﻲ ‪ ، f(z) − z = u‬و ﺑﲈ ّ‬
‫اﳌﻌﺮف ﺑﺎﻟﻌﺒﺎرة اﳌﺮﻛﺒﺔ ‪ f‬ﻋﺒﺎرة ﻋﻦ ﺗﺮﻛﻴﺐ اﻹﻧﺴﺤﺎب 𝒯 اﻟﺬي ﺷﻌﺎﻋﻪ ⃗ ‪ ui‬ﻣﻊ اﻟﺘﻨﺎﻇﺮ اﻟﻌﻤﻮدي ‪S‬‬
‫اﻟﺘﺤﻮﻳﻞ ‪ّ M ⟼ M‬‬
‫ﺑﺎﻟﻨﺴﺒﺔ ﳌﺤﻮر اﻟﻔﻮاﺻﻞ )‪ . 𝒯 ∘ S : (Ox‬ﻫﺬا اﻟﺘﺤﻮﻳﻞ ُﻳﺪﻋﻰ اﻹﻧﺰﻻق ذو اﳌﺤﻮر )‪. (Ox‬‬
‫ﻟﺪﻳﻨﺎ ‪:‬‬
‫‪f ∘ f (z) = f (z + u) = z + u + u‬‬
‫‪= z + u + u = z + 2u ,‬‬ ‫)‪ّ u = u‬‬
‫ﻷن ‪ u‬ﺣﻘﻴﻘﻲ(‬
‫‪ƕ‬‬

‫أن اﻟﻨﻘﻄﺔ )‪ f ∘ f (z‬ﻫﻲ ﺻﻮرة اﻟﻨﻘﻄﺔ ‪ z‬ﺑﺎﻹﻧﺴﺤﺎب اﻟﺬي ﺷﻌﺎﻋﻪ ⃗ ‪. 2ui‬‬


‫ﻫﺬا ﻳﻌﻨﻲ ّ‬
‫اﻟﺸﻜﻞ ‪ 6.IV‬ﻳﻤﺜﻞ ﺑﻌﺾ اﻟﺼﻮر اﳌﺘﺘﺎﺑﻌﺔ ﻟﻠﻤﻀﻠﻊ اﻷﺳﻮد ﺑﺎﻹﻧﺰﻻق ‪ f‬و اﻟﺸﻜﻞ ‪ّ 7.IV‬‬
‫ﻳﺒﲔ ﺑﻌﺾ اﻟﺰﺧﺎرف اﻟﺘﻲ ﻳﻤﻜﻦ‬
‫اﳊﺼﻮل ﻋﻠﻴﻬﺎ ﺑﺘﻄﺒﻴﻖ إﻧﺰﻻق ﺑﺼﻔﺔ ﻣﺘﺘﺎﺑﻌﺔ )ﺗﻜﺮارﻳﺔ(‪.‬‬
‫‪ũŏ‬‬

‫𝟭𝟲‬

‫‪http ://tinyurl.com/Malki1718‬‬ ‫‪0‬‬


.

Ŕž Ŧ Ľ
œǃ
Ŀ
IV
ņőƱŀ Ţ
ũŏ ƕ

http ://tinyurl.com/Malki1718 0
.

Ŕž Ŧ Ľ
œǃ
Ŀ ‫ﲤﺎﺭﻳﻦ ﳏﻠﻮﻟﺔ‬
IV

ņőƱŀ Ţ
ũŏ ƕ

𝟲𝟯

http ://tinyurl.com/Malki1718 0
.

Ŕž Ŧ Ľ
œǃ
Ŀ
IV
ņőƱŀ Ţ
ũŏ ƕ

http ://tinyurl.com/Malki1718 0
‫‪.‬‬

‫‪5‬‬
‫‪Ŕž Ŧ Ľ‬‬
‫‪œǃ‬‬
‫‪otttttttttttttttttttttttttttttttttttttttttttp‬‬
‫‪s‬‬ ‫‪u‬‬
‫‪s‬‬ ‫اﺧﺘﺒﺮ اﺳﺘﻴﻌﺎﺑﻚ ﻟﻠﺪرس‬
‫‪u‬‬

‫‪Ŀ‬‬
‫‪s‬‬ ‫‪u‬‬
‫‪qvvvvvvvvvvvvvvvvvvvvvvvvvvvvvvvvvvvvvvvvvvvr‬‬
‫‪.‬‬

‫اﳍﺪف ﻣﻦ اﻷﺳﺌﻠﺔ اﻵﺗﻴﺔ ﻫﻮ اﺧﺘﺒﺎر ﻣﺪى اﺳﺘﻴﻌﺎب اﻟﻄﺎﻟﺐ ﻟﻠﺪرس و ﻓﻬﻤﻪ ﻟﻠﻄﺮق اﳌﺘﺒﻌﺔ ﰲ إﺛﺒﺎت ﺧﺎﺻﻴﺔ أو إﳚﺎد ﺻﻴﻐﺔ‬
‫أو ﻋﻼﻗﺔ ﻣﺎ‪.‬‬
‫ﺗﻘﻞ ﻋﻦ ‪ 10‬دﻗﺎﺋﻖ‪ ،‬ﻓﻼ ﻳﻨﺒﻐﻲ ﻋﻠﻴﻪ اﻟﻨﻈﺮ‬‫إذا أﺧﻔﻖ اﻟﻄﺎﻟﺐ ﰲ اﳉﻮاب أو ﻋﺠﺰ ﻋﻦ اﻟﺮ ّد‪ ،‬ﺑﻌﺪ ﻣﺪة ﺗﻔﻜﲑ و ﺑﺤﺚ ﻻ ﳚﺐ أن ّ‬
‫إﱃ اﳊﻞ ﻣﺒﺎﴍة ﺑﻞ ﺑﺈﻣﻜﺎﻧﻪ ﺑﺪاﻳ ًﺔ اﻟﻨﻈﺮ ﰲ اﳌﻠﺨﺺ اﳌﻮﺟﻮد ﺻﻔﺤﺔ د ‪ ،‬و إذا ﱂ ﳚﺪ ﺿﺎﻟﺘﻪ ﻓﻴﻪ‪ ،‬ﻳﺮاﺟﻊ اﻟﺪرس ؛ ﻓﺈذا ﱂ ﻳﺴﺎﻋﺪه‬
‫ﺑﻜﻞ ﺗﺮﻛﻴﺰ و ﻻ ﻳﻨﺴﻰ أن ُﻳﻌﺎود اﳌﺤﺎوﻟﺔ ﺑﻌﺪ أﻳﺎم أو أﺳﺎﺑﻴﻊ و ﻻ ﻳﺘﻮﻗﻒ ﺣﺘﻰ ﻳﺘﻤﻜﻦ ﻣﻦ اﻟﺴﺆال …‬ ‫ذﻟﻚ ﻛﻠﻪ‪ ،‬ﻳﺘﺄ ّﻣﻞ اﳊﻞ ّ‬
‫ﻓﺎﳌﺜﺎﺑﺮة أﺳﺎس اﻟﻨﺠﺎح‪.‬‬

‫ﻋﻠ ًﲈ أﻧﻪ ‪) ∀z, z ∈ ℂ , ||z + z || ≤ |z| + ||z || :‬اﳌﺘﺒﺎﻳﻨﺔ اﳌﺜﻠﺜﻴﺔ( ‪ ،‬أﺛﺒﺖ ّ‬


‫أن ‪:‬‬ ‫‪١ .‬‬
‫‪.‬‬

‫‪∀z, z ∈ ℂ ,‬‬ ‫|| ‪|||z| − ||z |||| ≤ ||z + z‬‬


‫‪V‬‬

‫ﺍﳉﻮﺍﺏ‪ .‬ﻟﻴﻜﻦ ‪ z‬و ‪ z‬ﻋﺪدﻳﻦ ﻣﺮﻛﺒﲔ‪ .‬اﳌﺘﺒﺎﻳﻨﺔ اﳌﺜﻠﺜﻴﺔ ﺗﺴﻤﺢ ﻟﻨﺎ ﺑﻜﺘﺎﺑﺔ ‪:‬‬
‫‪ņőƱŀ Ţ‬‬
‫|| ‪. ||z || − |z| ≤ ||z + z‬‬ ‫أي‬ ‫|‪|| z + z + (−z)|| ≤ ||z + z || + |−z‬‬
‫أن || ‪ |z| − ||z || ≤ ||z + z‬أي || ‪. − ||z || − |z| ≤ ||z + z‬‬‫و ﺑﻨﻔﺲ اﻟﻄﺮﻳﻘﺔ‪ ،‬ﻧﺜﺒﺖ ّ‬
‫أن |||| ‪ |||z| − ||z‬ﻳﺴﺎوي إ ّﻣﺎ || ‪ |z| − ||z‬أو |‪ّ ||z || − |z‬‬
‫ﻓﺈن ‪:‬‬ ‫و ﺑﲈ ّ‬
‫‪∀z, z ∈ ℂ ,‬‬ ‫|| ‪|||z| − ||z |||| ≤ ||z + z‬‬
‫■‬

‫إﺑﺤﺚ ﻋﻦ اﳋﻄﺄ ﰲ اﻹﺛﺒﺎت اﻵﰐ ‪:‬‬ ‫‪٢ .‬‬ ‫‪.‬‬


‫‪ƕ‬‬

‫أن ‪ a ≥ 0‬و ‪ّ b ≥ 0‬‬


‫ﻓﺈن اﳌﺠﻤﻮع ‪ a + b‬ﻻ‬ ‫» ﻟﻴﻜﻦ ‪ a‬و ‪ b‬ﻋﺪدﻳﻦ ﻣﺮﻛﺒﲔ ﺑﺤﻴﺚ ‪ . a + b = 0‬ﺑﲈ ّ‬
‫ﻳﻜﻮن ﻣﻌﺪوﻣ ًﺎ إﻻّ إذا ﻛﺎن ‪ a = 0‬و ‪ b = 0‬أي إذا ﻛﺎن ‪. a = b = 0‬‬
‫ﻓﺈن ‪ a + b = 0‬ﻟﻜﻦ ‪ a ≠ 0‬و ‪« .( b ≠ 0‬‬‫ﻼ إذا ﻛﺎن ‪ a = 1‬و ‪ّ b = ı‬‬
‫)ﻫﺬا اﻹﺛﺒﺎت ﺧﺎﻃﺊ ﻷﻧﻪ ﻣﺜ ً‬
‫‪ũŏ‬‬

‫ﺍﳉﻮﺍﺏ‪ .‬ﻣﺮﺑﻊ ﻋﺪد ﻣﺮﻛﺐ ﻟﻴﺲ‪ ،‬ﰲ اﳊﺎﻟﺔ اﻟﻌﺎﻣﺔ‪ ،‬ﻋﺪد ًا ﺣﻘﻴﻘﻴ ًﺎ إذن ﻻ ﻳﻤﻜﻦ اﻟﻘﻮل أﻧﻪ ﻣﻮﺟﺐ أو ﻣﻌﺪوم‪ .‬ﻣﺜ ً‬
‫ﻼ ‪. ı = −1‬‬
‫■‬ ‫أن ﻋﻼﻗﺔ اﻟﱰﺗﻴﺐ ≤ )أو ≥( ﻟﻴﺲ ﳍﺎ ﻣﻌﻨﻰ ﰲ ‪ ℂ‬ﻛﲈ ﺗﻢ ﴍﺣﻪ ﰲ اﻟﺪرس ﺻﻔﺤﺔ ‪. 5‬‬ ‫اﻟﺴﺒﺐ ﻳﺮﺟﻊ إﱃ ّ‬

‫ﻛﻴﻒ ﻳﺘﻢ ﲢﺪﻳﺪ اﻟﺸﻜﻞ اﳌﺜﻠﺜﻲ )اﻷﳼ( ﻟﻌﺪد ﻣﺮﻛﺐ ؟ أﻛﺘﺐ ‪ 1 + ı‬و ‪ −3‬ﻋﲆ اﻟﺸﻜﻞ اﻷﳼ‪.‬‬ ‫‪٣ .‬‬
‫‪.‬‬

‫𝟱𝟲‬

‫‪http ://tinyurl.com/Malki1718‬‬ ‫‪0‬‬


‫‪.‬‬
‫ﺍﳉﻮﺍﺏ‪ .‬اﻟﻄﺮﻳﻘﺔ ﺑﺴﻴﻄﺔ ‪ :‬ﻧﻀﻊ ﻃﻮﻳﻠﺔ اﻟﻌﺪد ﻛﻌﺎﻣﻞ ﻣﺸﱰك و ﻳﺒﻘﻰ اﳉﺰء » 𝜃 ‪.« e = cos 𝜃 + ı sin‬‬
‫أن ‪ّ ||1 + ı|| = √2‬‬
‫ﻓﺈن ‪:‬‬ ‫ﻣﺜﻼً‪ ،‬ﺑﲈ ّ‬

‫‪Ŕž Ŧ Ľ‬‬
‫‪√2 √2‬‬ ‫𝜋‬ ‫𝜋‬ ‫‪/‬‬
‫‪1 + ı = √2‬‬ ‫‪+‬‬ ‫‪ı‬‬ ‫‪= √2 cos‬‬ ‫‪+ ı sin‬‬ ‫‪= √2e‬‬
‫‪2‬‬ ‫‪2‬‬ ‫‪4‬‬ ‫‪4‬‬

‫‪œǃ‬‬
‫ﺑﺎﳌﺜﻞ‪ ،‬اﻟﺸﻜﻞ اﻷﳼ ﻟﻠﻌﺪد ‪ −3‬ﻫﻮ ‪. −3 = 3 × (−1) = 3e :‬‬
‫■‬ ‫ﻷن اﻟﻄﻮﻳﻠﺔ ﳚﺐ أن ﺗﻜﻮن ﻋﺪد ًا ﺣﻘﻴﻘﻴ ًﺎ ﻣﻮﺟﺒ ًﺎ‪.‬‬
‫ﳚﺐ اﳊﺬر ﻣﻦ اﻟﻜﺘﺎﺑﺔ ‪ −3e‬اﻟﺘﻲ ﻟﻴﺴﺖ اﻟﺸﻜﻞ اﳌﺜﻠﺜﻲ ﻟﻠﻌﺪد ‪ّ −3‬‬

‫‪Ŀ‬‬
‫أوﺟﺪ‪ ،‬ﻣﻊ اﻟﴩح‪ ،‬اﳉﺬور اﻟﺮاﺑﻌﺔ ﻟﻠﻌﺪد ‪. −16‬‬ ‫‪٤ .‬‬ ‫‪.‬‬

‫ﺍﳉﻮﺍﺏ‪ .‬ﻟﻴﻜﻦ ‪ z‬ﺟﺬر ًا راﺑﻌ ًﺎ ﻟﻠﻌﺪد ‪ . −16‬ﻟﺪﻳﻨﺎ ‪:‬‬


‫‪z = −16 ⟺ z = 16e‬‬ ‫ﺑﺪاﻳ ًﺔ‪ ،‬ﻧﻜﺘﺐ ‪ −16‬ﻋﲆ اﻟﺸﻜﻞ اﻷﳼ ‪:‬‬
‫‪⟺ z = 2e‬‬ ‫‪/‬‬ ‫ﺛﻢ ﻧﻜﺘﺐ اﻟﻄﺮف اﻷﻳﻤﻦ ﻟﻠﻤﻌﺎدﻟﺔ ﻋﲆ ﺷﻜﻞ ﺟﺬر راﺑﻊ ‪:‬‬
‫ّ‬
‫‪z‬‬
‫⟺‬ ‫‪/‬‬
‫‪=1‬‬ ‫ُﺤﻮل اﳌﻌﺎدﻟﺔ إﱃ اﻟﺸﻜﻞ ‪: 𝜔 = 1‬‬
‫ﺑﻌﺪﻫﺎ‪ ،‬ﻧ ّ‬
‫‪2e‬‬
‫‪z‬‬ ‫‪/‬‬
‫∣ }‪⟺ ∃k ∈ {0, 1, 2, 3‬‬ ‫‪=e‬‬ ‫ﰲ اﻷﺧﲑ‪ ،‬ﻧُﻄ ّﺒﻖ ﻧﺘﻴﺠﺔ اﻟﺪرس ﺣﻮل ﺟﺬور اﻟﻮﺣﺪة ‪:‬‬
‫‪2e /‬‬
‫‪⟺ ∃k ∈ {0, 1, 2, 3} ∣ z = 2e‬‬ ‫‪+‬‬ ‫‪/‬‬

‫■‬ ‫‪. 2e±‬‬ ‫‪/‬‬ ‫‪ 2e±‬و‬ ‫‪/‬‬ ‫ﻹﲤﺎم اﳊﻞ‪ ،‬ﻧُﻌﻮض ﻋﻦ ‪ِ k‬‬
‫ﺑﺎﻟﻘ َﻴﻢ ‪ 2 ، 1 ، 0‬و ‪ . 3‬ﻧﺠﺪ ّ‬
‫أن اﳊﻠﻮل ﻫﻲ ‪:‬‬ ‫ّ‬

‫‪٥ .‬‬ ‫‪.‬‬

‫‪ c ، b ، a‬أﻋﺪاد ﻣﺮﻛﺒﺔ ﺑﺤﻴﺚ ‪ . a ≠ 0‬ﻟﻴﻜﻦ ‪ z‬و ‪ z‬ﱠ ْ‬


‫ﺣﲇ اﳌﻌﺎدﻟﺔ ‪ az + bz + c = 0‬ذات اﳌﺠﻬﻮل‬
‫ﻋﱪ ﻋﻦ ‪ s = z + z‬و ‪ p = z z‬ﺑﺪﻻﻟﺔ اﳌﻌﺎﻣﻼت ‪ b ، a‬و ‪. c‬‬ ‫‪ .z ∈ ℂ‬ﱢ‬
‫ِ‬
‫إذا ﱂ ﻧﻜﻦ ﻧﺤﻔﻆ ﻫﺬه اﻟﺼ َﻴﻎ‪ ،‬ﻓﻜﻴﻒ ﻳﻤﻜﻦ إﳚﺎدﻫﺎ ﺑﴪﻋﺔ ؟‬

‫‪V‬‬
‫‪ņőƱŀ Ţ‬‬
‫= ‪ p‬و ﻳﻤﻜﻦ إﺛﺒﺎت ذﻟﻚ ﻛﲈ ﻳﲇ ‪:‬‬
‫‪c‬‬ ‫‪b‬‬
‫ﺍﳉﻮﺍﺏ‪ .‬ﻧﻌﻠﻢ ّ‬
‫أن ‪ s = −‬و‬
‫‪a‬‬ ‫‪a‬‬
‫‪az + bz + c = a (z − z ) (z − z ) = az − a (z + z ) z + az z = az − asz + ap‬‬
‫■‬ ‫ﻫﺬه اﳌﺴﺎواة ﺻﺤﻴﺤﺔ ﻟﻜﻞ ﻋﺪد ﻣﺮﻛﺐ ‪ ، z ∈ ℂ‬إذن ﺑﺎﳌﻄﺎﺑﻘﺔ ﻳﻨﺘﺞ ‪ b = −as :‬و ‪. c = ap‬‬

‫‪ c ، b ، a‬أﻋﺪاد ﻣﺮﻛﺒﺔ ﺑﺤﻴﺚ ‪ . a ≠ 0‬أﻛﺘﺐ ﻋﲆ اﻟﺸﻜﻞ اﻟﻨﻤﻮذﺟﻲ اﻟﻌﺒﺎرة ‪. az + bz + c‬‬ ‫‪٦ .‬‬ ‫‪.‬‬

‫ﻃﺒﻖ ﻫﺬه اﻟﻨﺘﻴﺠﺔ ﻋﲆ ‪ z + 8z − 6‬و ‪. 5z − 2z + 1‬‬


‫‪ƕ‬‬

‫ﺍﳉﻮﺍﺏ‪ .‬ﺑﺪاﻳ ًﺔ‪ ،‬ﻧﻀﻊ ‪ a‬ﻛﻌﺎﻣﻞ ﻣﺸﱰك‬


‫‪b‬‬ ‫‪c‬‬
‫ﺛﻢ ﻧﺘﺒﻊ ﻃﺮﻳﻘﺔ إﲤﺎم اﳌﺮﺑﻊ ‪:‬‬
‫‪ّ .a z +‬‬ ‫‪z+‬‬
‫‪2a‬‬ ‫‪a‬‬

‫‪b‬‬ ‫‪c‬‬ ‫‪b‬‬ ‫‪b‬‬ ‫‪c‬‬ ‫‪b‬‬ ‫‪b − 4ac‬‬


‫‪az + bz + c = a z +‬‬ ‫‪a+‬‬ ‫‪=a‬‬ ‫‪z+‬‬ ‫‪−‬‬ ‫‪+‬‬ ‫‪=a‬‬ ‫‪z+‬‬ ‫‪−‬‬
‫‪ũŏ‬‬

‫‪2a‬‬ ‫‪a‬‬ ‫‪2a‬‬ ‫‪2a‬‬ ‫‪a‬‬ ‫‪2a‬‬ ‫‪4a‬‬

‫‪z + 8z − 6 = (z + 4) − 4 − 6 = (z + 4) − 22‬‬ ‫إذن ‪:‬‬


‫‪2‬‬ ‫‪1‬‬ ‫‪1‬‬ ‫‪1‬‬ ‫‪1‬‬ ‫‪1‬‬ ‫‪4‬‬
‫‪5z − 2z + 1 = 5 z − z +‬‬ ‫‪=5‬‬ ‫‪z−‬‬ ‫‪−‬‬ ‫‪+‬‬ ‫‪=5‬‬ ‫‪z−‬‬ ‫‪+‬‬ ‫و‬
‫‪5‬‬ ‫‪5‬‬ ‫‪5‬‬ ‫‪5‬‬ ‫‪5‬‬ ‫‪5‬‬ ‫‪25‬‬

‫𝟲𝟲‬

‫‪http ://tinyurl.com/Malki1718‬‬ ‫‪0‬‬


‫‪ .V‬اﺧﺘﱪ اﺳﺘﻴﻌﺎﺑﻚ ﻟدلرس‬
‫‪.‬‬
‫■‬

‫‪Ŕž Ŧ Ľ‬‬
‫إﴍح ﻃﺮﻳﻘﺔ ﻛﺘﺎﺑﺔ اﳉﺪاء ‪ ، sin x cos x‬ﻣﻊ ‪ m, n ∈ ℕ‬و ‪ ، x ∈ ℝ‬ﻋﲆ ﺷﻜﻞ ﻋﺒﺎرة ﺧﻄﻴﺔ ‪.‬‬ ‫‪٧ .‬‬ ‫‪.‬‬

‫‪œǃ‬‬
‫)ﻳﻤﻜﻦ ﴍﺣﻬﺎ ﰲ اﳊﺎﻟﺔ ‪ m = 4‬و ‪. (n = 3‬‬

‫ﺍﳉﻮﺍﺏ‪ .‬ﻧﴩح اﻟﻄﺮﻳﻘﺔ ﰲ اﳊﺎﻟﺔ ‪ m = 4‬و ‪ . n = 3‬ﻟﻴﻜﻦ ‪ . x ∈ ℝ‬ﻟﺪﻳﻨﺎ ‪:‬‬

‫‪Ŀ‬‬
‫‪e‬‬ ‫‪− e−‬‬ ‫‪e‬‬ ‫‪+ e−‬‬ ‫‪1‬‬
‫= ‪sin x cos x‬‬ ‫=‬ ‫‪(e‬‬ ‫‪− e− ) (e‬‬ ‫) ‪+ e−‬‬ ‫ﺑﺪاﻳ ًﺔ‪ ،‬ﻧﻄﺒﻖ ِﺻﻴﻎ ﺃﻭﻟﺮ ‪:‬‬
‫‪2ı‬‬ ‫‪2‬‬ ‫‪2‬‬
‫‪1‬‬
‫=‬ ‫‪e‬‬ ‫‪− 4e‬‬ ‫‪+ 6 − 4e−‬‬ ‫‪+ e−‬‬ ‫‪e‬‬ ‫‪+ 3e‬‬ ‫‪+ 3e−‬‬ ‫‪+ e−‬‬ ‫ﺑﻌﺪﻫﺎ‪ ،‬دﺳﺘﻮر ﺛﻨﺎﺋﻲ اﳊﺪ ‪:‬‬
‫‪2‬‬
‫‪1‬‬
‫=‬ ‫‪e‬‬ ‫‪−e‬‬ ‫‪− 3e‬‬ ‫‪+ 3e‬‬ ‫‪+ 3e−‬‬ ‫‪− 3e−‬‬ ‫‪− e−‬‬ ‫‪+e‬‬ ‫ﺛﻢ‪ ،‬اﻟﻨﴩ و اﻟﱰﺗﻴﺐ ‪:‬‬
‫ّ‬
‫‪2‬‬
‫‪1‬‬
‫=‬ ‫)‪(cos (7x) − cos (5x) − 3 cos (3x) + 3 cos x‬‬ ‫ﰲ اﻷﺧﲑ‪ِ ،‬ﺻﻴﻎ ﺃﻭﻟﺮ ‪:‬‬
‫‪2‬‬
‫■‬

‫‪ x‬و ‪ y‬ﻋﺪدان ﺣﻘﻴﻘﻴﺎن‪ .‬أﻛﺘﺐ اﻟﻌﺪد ‪ e + e‬ﻋﲆ اﻟﺸﻜﻞ اﻷﳼ ﻣﻊ اﻟﴩح‪.‬‬ ‫‪٨ .‬‬ ‫‪.‬‬

‫اﺳﺘﻨﺘﺞ اﻟﺸﻜﻞ اﻷﳼ ﻟﻠﻌﺪد ‪. e − e‬‬

‫‪ e‬ﻛﻌﺎﻣﻞ ﻣﺸﱰك ﻓﻴﻜﻮن ‪:‬‬ ‫‪+‬‬ ‫‪/‬‬ ‫ﺍﳉﻮﺍﺏ‪ .‬ﻧﺘﺒﻊ ﻃﺮﻳﻘﺔ »اﻟﺰاوﻳﺔ اﻟﻨﺼﻒ« و ﻧﻀﻊ‬
‫‪+‬‬ ‫‪/‬‬ ‫‪−‬‬ ‫‪/‬‬ ‫‪x−y‬‬
‫‪e‬‬ ‫‪+e‬‬ ‫‪=e‬‬ ‫‪e‬‬ ‫‪+‬‬ ‫‪e−‬‬ ‫‪−‬‬ ‫‪= 2 cos‬‬ ‫‪e‬‬ ‫‪+‬‬ ‫‪/‬‬
‫‪2‬‬
‫و ﺑﺎﻟﺘﺎﱄ ‪:‬‬
‫‪−‬‬
‫ﻓﺈن ‪) e + e = 0‬اﻟﺸﻜﻞ اﻷﳼ ﻏﲑ ﻣﻌﺮف( ؛‬ ‫‪ّ cos‬‬ ‫• إذا ﻛﺎن ‪= 0‬‬
‫‪V‬‬

‫ﻓﺈن اﻟﺸﻜﻞ اﻷﳼ ﻟﻠﻌﺪد ‪ e + e‬ﻫﻮ ‪ 2 cos − e + /‬؛‬ ‫• إذا ﻛﺎن ‪ّ cos − > 0‬‬
‫‪ņőƱŀ Ţ‬‬
‫ﻓﺈن اﻟﺸﻜﻞ اﻷﳼ ﻟﻠﻌﺪد ‪ e + e‬ﻫﻮ ‪. −2 cos − e + + /‬‬ ‫• و إذا ﻛﺎن ‪ّ cos − < 0‬‬
‫أن ‪ ، e − e = e + e +‬ﻓﺤﺴﺐ ﻣﺎ ﺳﺒﻖ ﻧﺴﺘﻨﺘﺞ ّ‬
‫أن ‪:‬‬ ‫و ﺑﲈ ّ‬
‫‪− −‬‬
‫)اﻟﺸﻜﻞ اﻷﳼ ﻏﲑ ﻣﻌﺮف( ؛‬ ‫• ‪ e − e = 0‬إذا ﻛﺎن ‪= 0‬‬
‫‪−‬‬ ‫‪+‬‬ ‫‪+‬‬ ‫‪/‬‬ ‫‪− −‬‬ ‫‪+‬‬ ‫‪+‬‬ ‫‪/‬‬
‫‪2 sin‬‬ ‫‪e‬‬ ‫‪ 2 cos‬أي‬ ‫‪e‬‬ ‫• اﻟﺸﻜﻞ اﻷﳼ ﻟﻠﻌﺪد ‪ e + e‬ﻫﻮ‬
‫‪− −‬‬
‫‪ cos‬؛‬ ‫‪ (cos t −‬إذا ﻛﺎن ‪> 0‬‬ ‫ّ‬
‫)ﻷن ‪= sin t‬‬
‫‪−‬‬ ‫‪+‬‬ ‫‪+‬‬ ‫‪+‬‬ ‫‪/‬‬ ‫‪− −‬‬ ‫‪+‬‬ ‫‪+‬‬ ‫‪+‬‬ ‫‪/‬‬
‫‪−2 sin‬‬ ‫‪e‬‬ ‫‪ −2 cos‬أي‬ ‫‪e‬‬ ‫• و اﻟﺸﻜﻞ اﻷﳼ ﻟﻠﻌﺪد ‪ e + e‬ﻫﻮ‬
‫‪− −‬‬
‫‪. cos‬‬ ‫‪ (cos t −‬إذا ﻛﺎن ‪< 0‬‬ ‫ّ‬
‫)ﻷن ‪= sin t‬‬
‫■‬
‫‪ƕ‬‬

‫ﻋﱪ ﻋﻦ ‪ z‬ﺑﺪﻻﻟﺔ ‪. z‬‬


‫𝜋‬ ‫‪٩ .‬‬
‫‪.‬‬

‫‪ z‬ﻋﺪد ﻣﺮﻛﺐ و ‪ z‬ﺻﻮرﺗﻪ ﺑﺎﻟﺪوران اﻟﺬي ﻻﺣﻘﺔ ﻣﺮﻛﺰه ‪ ı‬و زاوﻳﺘﻪ ‪ .‬ﱢ‬
‫‪4‬‬
‫‪ũŏ‬‬

‫‪ّ e‬‬
‫ﻓﺈن ‪:‬‬‫ّ ‪√2 √2‬‬
‫‪+‬‬ ‫‪/‬‬ ‫أن )‪ z − ı = e / (z − ı‬و ﺑﲈ أن ‪ı‬‬
‫=‬ ‫ﻧﻌﻠﻢ ّ‬ ‫ﺍﳉﻮﺍﺏ‪.‬‬
‫‪2‬‬ ‫‪2‬‬
‫‪√2 √2‬‬ ‫‪√2 + ı√2‬‬ ‫‪√2 + ı 2 − √2‬‬
‫‪z =ı+‬‬ ‫‪+‬‬ ‫= )‪ı (z − ı‬‬ ‫‪z+‬‬
‫‪2‬‬ ‫‪2‬‬ ‫‪2‬‬ ‫‪2‬‬
‫■‬

‫𝟳𝟲‬

‫‪http ://tinyurl.com/Malki1718‬‬ ‫‪0‬‬


.

Ŕž Ŧ Ľ
œǃ
Ŀ
V
ņőƱŀ Ţ
ũŏ ƕ

http ://tinyurl.com/Malki1718 0
.

6
Ŕž Ŧ Ľ
œǃ
otttttttttttttttttttttttttttttttttttttttttttp
s u
s ‫ﺗﻤﺎرﻳﻦ ﺗﻄﺒﻴﻘﻴﺔ‬
u

Ŀ
s u
qvvvvvvvvvvvvvvvvvvvvvvvvvvvvvvvvvvvvvvvvvvvr
.

‫ﳎﻤﻮﻋﺔ ﺍﻷﻋﺪﺍﺩ ﺍﳌﺮﻛﺒﺔ‬ 1.VI

: ‫أﻧﺠﺰ اﳊﺴﺎﺑﺎت اﻟﺘﺎﻟﻴﺔ‬


 𝟏. .

(1 + ı) (4 + 3ı) 1+ı (1 + ı) •1
• 7 • 4
(5 − ı) (2 + ı) 1−ı
(4 − 5ı) ⋅ (6 + 3ı) • 5 (3 − ı) •2
2+ı 3−ı
+ • 8 (2 + 3ı) ⋅ (2 − 3ı) • 6 (3 − 2ı) •3
3−ı 2+ı
ņőƱŀ Ţ
.ّ‫ﺍﳊﻞ‬

(1 + ı) = 1 + 2 × 1 × ı + ı = 1 + 2ı − 1 = 2ı •1
VI

(3 − ı) = 3 − 2 × 3 × ı + ı = 9 − 6ı − 1 = 8 − 6ı •2

(3 − 2ı) = 3 − 3 × 3 × 2ı + 3 × 3 (2ı) − (2ı) •3

= 27 − 54ı − 36 + 8ı = −9 − 46ı
1+ı (1 + ı) 2ı
ƕ

= = =ı : ‫ﻟﺪﻳﻨﺎ‬ • 4
1 − ı 1 + (−1) 2
1+ı
=ı = ı =1 = 1 : ‫ﻣﻨﻪ‬
1−ı
ũŏ

(4 − 5ı) ⋅ (6 + 3ı) = 4 × 6 + 4 × 3ı − 5ı × 6 − 5ı × 3ı • 5
= 24 + 12ı − 30ı − 15ı = 24 − 18ı + 15 = 39 − 18ı

: ‫ و ﺑﺎﻟﺘﺎﱄ‬a = 2 − 3ı ‫ ﻓﻴﻜﻮن‬a = 2 + 3ı ‫ﻧﻀﻊ‬ • 6

(2 + 3ı) ⋅ (2 − 3ı) = a ⋅ a = (aa) = |a|

𝟲𝟵

http ://tinyurl.com/Malki1718 0
‫ ﶍﻮﻋﺔ اﻷﻋﺪاد اﳌﺮﻛﺒﺔ‬.1.VI
.
= 2 +3 = 13 = 2197

Ŕž Ŧ Ľ
(1 + ı) (4 + 3ı) 1 × 4 + 1 × 3ı + ı × 4 + ı × 3ı
= • 7
(5 − ı) (2 + ı) 5×2+5×ı−ı×2−ı
4 + 3ı + 4ı + 3ı 4 + 7ı − 3 1 + 7ı

œǃ
= = =
10 + 5ı − 2ı + 1 11 + 3ı 11 + 3ı
(1 + 7ı) ⋅ (11 − 3ı) 1 × 11 + 1 × (−3ı) + 7ı × 11 + 7ı × (−3ı)
= =
11 + 3 130

Ŀ
11 − 3ı + 77ı − 21ı 11 + 74ı + 21 32 + 74ı 16 37
= = = = + ı
130 130 130 65 65
2 + ı 3 − ı (2 + ı) (3 + ı) (3 − ı) (2 − ı)
+ = + • 8
3−ı 2+ı 3 + (−1) 2 +1
2×3+2×ı+ı×3+ı 3 × 2 + 3 × (−ı) − ı × 2 + ı
= +
10 5
6 + 5ı − 1 6 − 5ı − 1 5 + 5ı 5 − 5ı
= + = +
10 5 10 5
1+ı 1 + ı + 2 − 2ı 3 − ı 3 1
= +1−ı= = = − ı
2 2 2 2 2

: ‫أﻛﺘﺐ ﻋﲆ اﻟﺸﻜﻞ اﳉﱪي اﻷﻋﺪاد‬


 𝟐. .

2 + 5ı 2 − 5ı 1+ı 3 + 6ı 3 + 6ı
+ • 3 + • 2 • 1
1−ı 1+ı 2−ı 3 − 4ı 3 − 4ı

: ‫• ﻟﺪﻳﻨﺎ‬1 .ّ‫ﺍﳊﻞ‬
ņőƱŀ Ţ
3 + 6ı (3 + 6ı) (3 + 4ı) 9 + 12ı + 18ı − 24 −15 + 30ı 3 6
= = = =− + ı
3 − 4ı (3 − 4ı) (3 + 4ı) 3 +4 25 5 5
• 2
1 + ı (1 + ı) (2 + ı) 2 + ı + 2ı − 1 1 + 3ı 1 3

VI
= = = = + ı : ‫ﻟﺪﻳﻨﺎ‬
2 − ı (2 − ı) (2 + ı) 2 +1 5 5 5
1+ı 1 + 3ı (1 + 3ı) 1 + 6ı − 9 −8 + 6ı 8 6
= = = = =− + ı : ‫ﻣﻨﻪ‬
2−ı 5 5 25 25 25 25
3 + 6ı 1+ı 3 6 8 6 −15 + 30ı −8 + 6ı −23 + 36ı
+ =− + ı− + ı= + = : ‫ﻣﻨﻪ‬
ƕ

3 − 4ı 2−ı 5 5 25 25 25 25 25
23 36
=− + ı
25 25
2 + 5ı 2 − 5ı 2 − 5ı 2 + 5ı
ũŏ

: ‫ ﻟﻜﻦ‬. + = a + a = 2 Re (a) : ‫و ﺑﺎﻟﺘﺎﱄ‬ = a ‫ ﻓﻴﻜﻮن‬a = ‫• ﻧﻀﻊ‬3


1−ı 1+ı 1+ı 1−ı
2 + 5ı (2 + 5ı) (1 + ı) 2 + 2ı + 5ı − 5 −3 + 7ı 3 7
a= = = = =− + ı
1−ı (1 − ı) (1 + ı) 1 +1 2 2 2
: ‫إذن‬
2 + 5ı 2 − 5ı 3 7 3
+ = a + a = 2 Re (a) = 2 Re − + ı =2 − = −3
1−ı 1+ı 2 2 2

𝟳𝟬

http ://tinyurl.com/Malki1718 0
‫ ﲤﺎرﻳﻦ ﺗﻄﺒﻴﻘﻴﺔ‬.VI
.

 𝟑. .

Ŕž Ŧ Ľ
‫أﻛﺘﺐ اﻷﻋﺪاد اﳌﺮﻛﺒﺔ اﻟﺘﺎﻟﻴﺔ ﻋﲆ ﺷﻜﻠﻬﺎ اﳉﱪي‬
ı+7 1−ı 3 + 2ı 2+ı

œǃ
+ • 5 • 3 • 1
3 + 7ı 1 + ı 2 − 5ı ı

(1 + ı) (1 − ı) (1 + 2ı) (−2 + 5ı) 1


+ • 6 • 4 2
1−ı (1 + ı) (7 + 3ı) (−12 + ı) 1 − 4ı

Ŀ
.ّ‫ﺍﳊﻞ‬

2 + ı (2 + ı) (−ı) −2ı − ı
= = = 1 − 2ı • 1
ı −ı 1
1 1 + 4ı 1 + 4ı 1 4
= = = + ı • 2
1 − 4ı 1 + 4 17 17 17
3 + 2ı (3 + 2ı) (2 + 5ı) 3 ⋅ 2 + 3 ⋅ 5ı + 2 ⋅ 2ı + 2ı ⋅ 5ı
= = • 3
2 − 5ı 2 +5 29
6 + 15ı + 4ı + 10ı 6 + 19ı − 10 −4 + 19ı 4 19
= = = = − + ı
29 29 29 29 29
(1 + 2ı) (−2 + 5ı) −2 + 5ı − 4ı + 10ı −2 + ı − 10
= = • 4
(7 + 3ı) (−12 + ı) −84 + 7ı − 36ı + 3ı −84 − 29ı − 3
−12 + ı (−12 + ı) (−87 + 29ı)
= =
−87 − 29ı (−87) + 29
1044 − 348ı − 87ı + 29ı 1044 − 435ı − 29
= =
8410 8410
1015 − 435ı 7 3
ņőƱŀ Ţ
= = − ı
8410 58 58
ı+7 1 − ı (7 + ı) (3 − 7ı) (1 − ı)
+ = + • 5
3 + 7ı 1 + ı 3 +7 1 +1
VI

21 − 49ı + 3ı − 7ı 1 − 2ı − 1 21 − 46ı + 7
= + = −ı
58 2 58
28 − 46ı − 58ı 28 − 104ı 14 52
= = = − ı
58 58 29 29
(1 + ı) (1 − ı) (1 + ı) (1 − ı) (1 + ı)
ƕ

+ = + • 6
1−ı (1 + ı) 1 +1 1 +1

(1 + ı)(1 − ı) (1 + 2ı − 1) (1 − ı)
ũŏ

= + = +
2 4 2 4
(2ı) (−2ı) −4 8ı
= + = + = −2 + 2ı
2 4 2 4

𝟳𝟭

http ://tinyurl.com/Malki1718 0
‫ ﶍﻮﻋﺔ اﻷﻋﺪاد اﳌﺮﻛﺒﺔ‬.1.VI
.

 𝟒. .

Ŕž Ŧ Ľ
1 3
. j ‫ أﺣﺴﺐ‬. j = − + √ ı ‫• ﻧﻀﻊ‬1

œǃ
2 2
1
. ( j ‫ ﻫﻮ ﻣﺮاﻓﻖ‬j ) =j =j ّ ‫• إﺳﺘﻨﺘﺞ‬2
‫ و‬j = 1 ‫ ؛‬1 + j + j = 0 : ‫أن‬
j

Ŀ
: ‫• أﺣﺴﺐ اﻷﻋﺪاد اﻵﺗﻴﺔ‬3
1−j 1−ı j (j + 1) (‫)ا‬
(‫)ﻫ‬ (‫)ج‬
(1 + ı) 1−j
j+1 1+j j
(‫)و‬ (‫)د‬ (‫)ب‬
j−1 (1 − ı) j +1

: ‫• ﻟﺪﻳﻨﺎ‬1 .ّ‫ﺍﳊﻞ‬
1 √3 1 √3 3 1 √3
j = − +ı = − 2ı − =− −ı
2 2 4 4 4 2 2

: ‫• ﺣﺴﺐ ﻣﺎ ﺳﺒﻖ‬2
1 √3 1 √3
1+j+j =1− +ı − −ı =0
2 2 2 2
. j = 1 ‫ أي‬1 − j = 0 ‫( أي‬1 − j) 1 + j + j = 0 : ‫ﻣﻨﻪ‬
1
. = j ‫ أي‬j × j = 1 :‫ﺑﺘﻌﺒﲑ آﺧﺮ‬
j
1
ّ ||j|| = 1 ‫أن‬
ّ ‫و ﺑﲈ‬
ņőƱŀ Ţ
.j = =j ‫ﻓﺈن‬
j
• 3
j (j + 1) = j + j = −1 (‫)ا‬

VI
j j
= = −1 (‫)ب‬
j + 1 −j

1−ı 1−ı 2 − 2ı (2 − 2ı) 3 − ı√3


= = = (‫)ج‬
1−j 1+ +ı 3 + ı√3 3 + √3
ƕ

6 − 2ı√3 − 6ı + 2√3ı 6 − 2ı 3 + √3 − 2√3 3 − √3 3 + √3


= = = −ı
12 12 6 6
1+j −j j 1 1 1 √3 √3 1
ũŏ

= = = − ıj = − ı − − ı = − ı (‫)د‬
(1 − ı) 1 − 2ı − 1 2ı 2 2 2 2 4 4

1−j 1+ −ı 1 3 √3 √3 3
= =− ı −ı = − − ı (‫)ﻫ‬
(1 + ı) 1 + 2ı − 1 2 2 2 4 4

j + 1 (j + 1) (j − 1) (j + 1) j − 1
= = (‫)و‬
j − 1 (j − 1) (j − 1) (j − 1) j − 1

𝟳𝟮

http ://tinyurl.com/Malki1718 0
‫ ﲤﺎرﻳﻦ ﺗﻄﺒﻴﻘﻴﺔ‬.VI
.
j +j −j−1 1 + (−j − 1) − j − 1 −1 − 2j
= = =
j − (j + j) + 1 1 − (−1) + 1 3

Ŕž Ŧ Ľ

œǃ
√3 + ı √3 − ı
 𝟓. .
z= + +ı−1 ‫ ﻋﺪد ﻣﺮﻛﺐ ﺣﻴﺚ‬z
√3 − ı √3 + ı

Ŀ
1
.z + ‫ ﻋﲆ ﺷﻜﻠﻪ اﳉﱪي ﺛﻢ ُاﺣﺴﺐ‬z ‫أﻛﺘﺐ‬
z

‫ ﻧﻀﻊ‬.ّ‫ﺍﳊﻞ‬
1 √3 + ı
.z = a+ + ı − 1 ‫ ﻓﻴﻜﻮن‬a =
a √3 − ı

√3 + ı √3 + ı 3 + 2ı√3 − 1 1 √3
a= = = = +ı : ‫ﻟﺪﻳﻨﺎ‬
√3 − ı 4 2 2
√3 + 1
1 1 −ı 1 √3
= = = −ı : ‫ﻣﻨﻪ‬
a +ı 2 2
+
1 √3 1 √3

z= + −ı +ı−1=ı : ‫و ﺑﺎﻟﺘﺎﱄ‬
2 2 2 2
1 1
z+ =ı+ =ı−ı=0 ‫و‬
z ı

z +z+1  𝟔. .
ņőƱŀ Ţ
.‫ﻋﲆ ﺷﻜﻠﻪ اﳉﱪي‬ ‫ ﺿﻊ اﻟﻌﺪد اﳌﺮﻛﺐ‬. z = 3 − 2ı ‫ ﻋﺪد ﻣﺮﻛﺐ ﺣﻴﺚ‬z
z −1

.ّ‫ﺍﳊﻞ‬
VI

z = (3 − 2ı) = 3 − 2 × 3 × 2ı + (2ı) = 9 − 12ı − 4 = 5 − 12ı : ‫ﻟﺪﻳﻨﺎ‬


z = z = (5 − 12ı) = 5 − 2 × 5 × 12ı + (12ı) : ‫ﻣﻨﻪ‬
= 25 − 120ı − 144 = −119 − 120ı
ƕ

z + z + 1 5 − 12ı + 3 − 2ı + 1 9 − 14ı 1 9 − 14ı


= = =− ⋅ : ‫و ﺑﺎﻟﺘﺎﱄ‬
z −1 −119 − 120ı − 1 −120 − 120ı 120 1 + ı
1 (9 − 14ı) (1 − ı) 1 9 − 9ı − 14ı + 14ı
=− ⋅ =− ⋅
120 1 +1 120 2
ũŏ

1 1 23
=− (−5 − 23ı) = + ı
240 48 240

𝟳𝟯

http ://tinyurl.com/Malki1718 0
‫ ﶍﻮﻋﺔ اﻷﻋﺪاد اﳌﺮﻛﺒﺔ‬.1.VI
.

 𝟕. .

Ŕž Ŧ Ľ
ı 1 + ı√3
.z = ‫و‬ z = ‫ ﻋﺪدان ﻣﺮﻛﺒﺎن ﺣﻴﺚ‬z ‫ و‬z
2ı√3 − 2 1 − ı√3
: ‫أﻛﺘﺐ اﻷﻋﺪاد اﻵﺗﻴﺔ ﻋﲆ ﺷﻜﻠﻬﺎ اﳉﱪي‬

œǃ
z
• 5 z ⋅z • 4 z +z • 3 z 2
• z • 1
z

Ŀ
.ّ‫ﺍﳊﻞ‬

1 + ı√3 1 + ı√3 1 + 2ı√3 − 3 −2 + 2ı√3 1 √3


z = = = = = − +ı • 1
1 − ı√3 1 + √3 4 4 2 2

ı 1 ı 1 ı −1 − ı√3
z = = ⋅ = ⋅ • 2
−2 + 2ı√3 2 −1 + ı√3 2 1 + √3
1 √3 − ı √3 1
= ⋅ = − ı
2 4 8 8
1 √3 √3 1 √3 − 4 4√3 − 1
z +z =− +ı + − ı= +ı • 3
2 2 8 8 8 8
1 √3 √3 1 1
z ⋅z = − +ı − ı = −1 + ı√3 √3 − ı • 4
2 2 8 8 16
1 1
= −√3 + ı + 3ı + √3 = ı
16 4

z − +ı −1 + ı√3 −1 + ı√3 √3 + ı
= =4⋅ =4⋅ 5
ņőƱŀ Ţ

z − ı √3 − ı √3 + 1
−√3 − ı + 3ı − √3
=4⋅ = −2√3 + 2ı
4

VI

1 1 + ı√2
 𝟖. .
.b = ‫و‬a = : ‫ ﻋﺪدﻳﻦ ﻣﺮﻛﺒﲔ ﺣﻴﺚ‬b ‫ و‬a ‫ﻟﻴﻜﻦ‬
√2 − ı 1 − ı√2
ƕ

a
‫ و‬ab ، a − b ، a + b ، b ، a : ‫أﻛﺘﺐ ﻋﲆ اﻟﺸﻜﻞ اﳉﱪي ﻛﻼ ﻣﻦ‬
b

: ‫ ﻟﺪﻳﻨﺎ‬.ّ‫ﺍﳊﻞ‬
ũŏ

1 + ı√2 1 + ı√2 1 + ı√2 1 + 2ı√2 − 2 1 2√2


a= = = = − +ı
1 − ı√2 1 − ı√2 1 + ı√2 1+2 3 3

1 √2 + ı √2 + ı √2 1
b= = = = + ı
√2 − ı √2 − ı √2 + ı 2+1 3 3

𝟳𝟰

http ://tinyurl.com/Malki1718 0
‫‪ .VI‬ﲤﺎرﻳﻦ ﺗﻄﺒﻴﻘﻴﺔ‬
‫‪.‬‬
‫‪1‬‬ ‫‪2√2 √2 1‬‬ ‫‪√2 − 1‬‬ ‫‪1 + 2√2‬‬
‫‪a+b=− +ı‬‬ ‫‪+‬‬ ‫=‪+ ı‬‬ ‫‪+ı‬‬ ‫ﻣﻨﻪ ‪:‬‬
‫‪3‬‬ ‫‪3‬‬ ‫‪3‬‬ ‫‪3‬‬ ‫‪3‬‬ ‫‪3‬‬

‫‪Ŕž Ŧ Ľ‬‬
‫‪1‬‬ ‫‪2√2 √2 1‬‬ ‫‪√2 + 1‬‬ ‫‪2√2 − 1‬‬
‫‪a−b=− +ı‬‬ ‫‪−‬‬ ‫‪− ı= −‬‬ ‫‪+ı‬‬
‫‪3‬‬ ‫‪3‬‬ ‫‪3‬‬ ‫‪3‬‬ ‫‪3‬‬ ‫‪3‬‬

‫‪œǃ‬‬
‫‪1‬‬ ‫‪2√2‬‬ ‫‪√2 1‬‬ ‫‪√2 1‬‬ ‫‪4‬‬ ‫‪2√2‬‬ ‫‪√2 1‬‬
‫= ‪ab‬‬ ‫‪− +ı‬‬ ‫‪+ ı‬‬ ‫‪=−‬‬ ‫‪− ı+ ı−‬‬ ‫‪= −‬‬ ‫‪+ ı‬‬
‫‪3‬‬ ‫‪3‬‬ ‫‪3‬‬ ‫‪3‬‬ ‫‪9‬‬ ‫‪9‬‬ ‫‪9‬‬ ‫‪9‬‬ ‫‪3‬‬ ‫‪3‬‬
‫‪1‬‬ ‫‪2√2‬‬

‫‪Ŀ‬‬
‫‪a −3 + ı 3‬‬ ‫‪1‬‬ ‫‪2√2‬‬ ‫‪√2 1‬‬ ‫‪4‬‬ ‫‪2√2‬‬ ‫‪√2 5‬‬
‫=‬ ‫=‬ ‫‪− +ı‬‬ ‫‪√2 − ı = −‬‬ ‫‪+ ı+ ı+‬‬ ‫=‬ ‫‪+ ı‬‬
‫‪b‬‬ ‫‪1‬‬ ‫‪3‬‬ ‫‪3‬‬ ‫‪3‬‬ ‫‪3‬‬ ‫‪3‬‬ ‫‪3‬‬ ‫‪3‬‬ ‫‪3‬‬
‫‪√2 − ı‬‬
‫■‬

‫اﳌﺴﺘﻮي ﻣﻨﺴﻮب إﱃ ﻣﻌﻠﻢ ﻣﺘﻌﺎﻣﺪ و ﻣﺘﺠﺎﻧﺲ ⃗ ‪ . O, i,⃗ j‬ﻧﻌﺘﱪ اﻟﻨﻘﻂ ‪ B ، A‬و ‪ C‬اﻟﺘﻲ ﻟﻮاﺣﻘﻬﺎ ‪، a = 2ı‬‬
‫‬
‫‪𝟗. .‬‬

‫‪ b = −√3 + ı‬و ‪ c = −√3 − ı‬ﻋﲆ اﻟﱰﺗﻴﺐ‪.‬‬


‫‪ •1‬أﻛﺘﺐ اﻷﻋﺪاد ‪ b ، a‬و ‪ c‬ﻋﲆ اﻟﺸﻜﻞ اﻷﳼ ّ‬
‫ﺛﻢ ﻋﻠﻢ اﻟﻨﻘﻂ ‪ B ، A‬و ‪ C‬ﰲ اﳌﺴﺘﻮي‪.‬‬
‫‪a−b‬‬
‫= ‪.Z‬‬ ‫‪ •2‬ﻟﻴﻜﻦ‬
‫‪c−b‬‬
‫ﺛﻢ اﻷﳼ‪.‬‬
‫)ا( أﻛﺘﺐ ‪ Z‬ﻋﲆ اﻟﺸﻜﻞ اﳉﱪي ّ‬
‫)ب( اﺳﺘﻨﺘﺞ ﻃﺒﻴﻌﺔ اﳌﺜﻠﺚ ‪. ABC‬‬

‫‪ •1‬ﻟﺪﻳﻨﺎ ‪:‬‬ ‫ﺍﳊﻞّ‪.‬‬


‫‪ņőƱŀ Ţ‬‬
‫𝜋‬ ‫𝜋‬
‫‪a = 2ı = 2 cos‬‬ ‫‪+ ı sin‬‬ ‫‪= 2e /‬‬
‫‪2‬‬ ‫‪2‬‬
‫𝜋‪5‬‬ ‫𝜋‪5‬‬ ‫‪/‬‬
‫‪b = −√3 + ı = 2 cos‬‬ ‫‪+ ı sin‬‬ ‫‪= 2e‬‬
‫‪6‬‬ ‫‪6‬‬
‫‪VI‬‬

‫𝜋‪7‬‬ ‫𝜋‪7‬‬ ‫‪/‬‬


‫‪c = −√3 − ı = 2 cos‬‬ ‫‪+ ı sin‬‬ ‫‪= 2e‬‬ ‫‪= 2e−‬‬ ‫‪/‬‬
‫‪6‬‬ ‫‪6‬‬

‫أﻧﻈﺮ اﻟﺸﻜﻞ ‪. 1.VI‬‬


‫‪ƕ‬‬

‫)ا( ﻟﺪﻳﻨﺎ ‪:‬‬ ‫•‬‫‪2‬‬


‫‪a−b‬‬ ‫‪2ı + √3 − ı‬‬ ‫‪√3 + ı −1 + ı√3‬‬
‫=‪Z‬‬ ‫=‬ ‫=‬ ‫=‬ ‫)اﻟﺸﻜﻞ اﳉﱪي(‬
‫‪c−b‬‬ ‫‪−√3 − ı + √3 − ı‬‬ ‫‪−2ı‬‬ ‫‪2‬‬
‫‪ũŏ‬‬

‫𝜋‪2‬‬ ‫𝜋‪2‬‬
‫‪= cos‬‬ ‫‪+ ı sin‬‬ ‫‪=e /‬‬ ‫)اﻟﺸﻜﻞ اﻷﳼ(‬
‫‪3‬‬ ‫‪3‬‬

‫‪ّ ||e‬‬
‫ﻓﺈن ||‪ ||a − b|| = ||c − b‬أي ‪ BA = BC‬و ﻫﺬا ﻳﻌﻨﻲ ّ‬
‫أن اﳌﺜﻠﺚ ‪ ABC‬ﻣﺘﺴﺎوي اﻟﺴﺎﻗﲔ‬ ‫‪/‬‬ ‫أن ‪|| = 1‬‬
‫ﺑﲈ ّ‬ ‫)ب(‬
‫ﻋﻨﺪ اﻟﺮأس ‪. B‬‬
‫■‬

‫𝟱𝟳‬

‫‪http ://tinyurl.com/Malki1718‬‬ ‫‪0‬‬


‫‪ .1.VI‬ﶍﻮﻋﺔ اﻷﻋﺪاد اﳌﺮﻛﺒﺔ‬
‫‪.‬‬

‫‪A‬‬

‫‪Ŕž Ŧ Ľ‬‬
‫‪2‬‬

‫‪œǃ‬‬
‫‪B‬‬ ‫‪1‬‬

‫‪Ŀ‬‬
‫‪−2‬‬ ‫‪−1‬‬

‫‪C‬‬ ‫‪−1‬‬

‫ﺷﻜﻞ ‪1.VI‬‬

‫أﺛﺒﺖ اﻟﺘﻜﺎﻓﺆات اﻟﺘﺎﻟﻴﺔ ‪:‬‬


‫‬ ‫‪𝟏𝟎.‬‬ ‫‪.‬‬

‫⟺ ‪Z=Z‬‬ ‫‪ Z‬ﺣﻘﻴﻘﻲ‬
‫⟺ ‪Z∈ℝ‬‬ ‫)𝜋 ‪ arg (Z) = 0 (mod‬أو‬
‫‪Z=0‬‬
‫‪ Z‬ﲣﻴﲇ ﴏف ⟺ ‪Z + Z = 0‬‬
‫𝜋‬
‫⟺ ‪Z ∈ ıℝ‬‬ ‫‪Z=0‬‬ ‫= )‪ arg (Z‬أو‬ ‫)𝜋 ‪(mod‬‬
‫‪2‬‬
‫‪ņőƱŀ Ţ‬‬
‫ﺗﻄﺒﻴﻘﺎت ‪:‬‬

‫‪ •1‬ﻛﻴﻒ ﳚﺐ إﺧﺘﻴﺎر اﻟﻌﺪد ‪ z‬ﺣﺘﻰ ﻳﻜﻮن ‪ Z = z + 2z − 3‬ﻋﺪد ًا ﺣﻘﻴﻘﻴﺎ ؟ ﺣﺪّ د اﳌﺠﻤﻮﻋﺔ ‪ E‬ﻟﻸﻋﺪاد اﳌﺮﻛﺒﺔ ‪ z‬اﻟﺘﻲ‬
‫ﻳﻜﻮن ﻣﻦ أﺟﻠﻬﺎ ‪ Z‬ﻋﺪد ًا ﺣﻘﻴﻘﻴﺎ‪.‬‬

‫‪VI‬‬
‫‪1−z‬‬
‫= ‪.Z‬‬ ‫‪ •2‬ﻟﺘﻜﻦ ‪ A‬ﺻﻮرة اﻟﻌﺪد ‪ ı‬و ‪ B‬ﺻﻮرة اﻟﻌﺪد ‪ .1‬ﻟﺘﻜﻦ ‪ M‬ﻧﻘﻄﺔ ﻣﻦ اﳌﺴﺘﻮي ﲣﺘﻠﻒ ﻋﻦ ‪ .A‬ﻧﻀﻊ‬
‫‪ı−z‬‬
‫ﺣﺪّ د اﳌﺠﻤﻮﻋﺔ ‪ E‬ﻟﻠﻨﻘﻂ ‪ M‬اﻟﺘﻲ ﻳﻜﻮن ﻣﻦ أﺟﻠﻬﺎ ‪ Z‬ﻋﺪد ًا ﺣﻘﻴﻘﻴﺎ‪.‬‬ ‫•‬

‫ﺣﺪّ د اﳌﺠﻤﻮﻋﺔ ‪ F‬ﻟﻠﻨﻘﻂ ‪ M‬اﻟﺘﻲ ﻳﻜﻮن ﻣﻦ أﺟﻠﻬﺎ ‪ Z‬ﲣﻴﻠﻴﺎ ﴏﻓﺎ‪.‬‬


‫‪ƕ‬‬

‫•‬

‫ﺍﳊﻞّ‪ .‬ﻟﺪﻳﻨﺎ ﻣﻦ ﺟﻬﺔ ‪:‬‬


‫‪ũŏ‬‬

‫‪1‬‬
‫⟺ ‪ Z ⟺ Im (Z) = 0‬ﺣﻘﻴﻘﻲ‬‫‪Z−Z =0 ⟺ Z=Z‬‬
‫‪2ı‬‬
‫‪1‬‬
‫⟺ ‪ Z ⟺ Re (Z) = 0‬ﲣﻴﲇ ﴏف‬ ‫‪Z+Z =0 ⟺ Z+Z=0‬‬
‫‪2‬‬
‫و ﻣﻦ ﺟﻬﺔ أﺧﺮى ‪:‬‬
‫⟺ ‪Z∈ℝ‬‬ ‫)𝜋‪ arg (z) = 0 (mod 2‬أو ‪Z = 0‬‬ ‫)𝜋‪ arg (z) = 𝜋 (mod 2‬أو‬

‫𝟲𝟳‬

‫‪http ://tinyurl.com/Malki1718‬‬ ‫‪0‬‬


‫‪ .VI‬ﲤﺎرﻳﻦ ﺗﻄﺒﻴﻘﻴﺔ‬
‫‪.‬‬
‫⟺‬ ‫)𝜋 ‪ arg (z) = 0 (mod‬أو ‪Z = 0‬‬
‫𝜋‬ ‫𝜋‬
‫⟺ ‪Z ∈ ıℝ‬‬ ‫)𝜋‪ arg (z) = (mod 2‬أو ‪Z = 0‬‬ ‫)𝜋‪ arg (z) = − (mod 2‬أو‬

‫‪Ŕž Ŧ Ľ‬‬
‫‪2‬‬ ‫‪2‬‬
‫𝜋‬
‫⟺‬ ‫)𝜋 ‪ arg (z) = (mod‬أو ‪Z = 0‬‬
‫‪2‬‬

‫‪œǃ‬‬
‫ﺗﻄﺒﻴﻘﺎت ‪:‬‬
‫‪ •1‬ﺣﺴﺐ ﻣﺎ ﺳﺒﻖ و ﺣﺴﺐ ﺧﻮاص اﳌﺮاﻓﻖ ‪:‬‬

‫‪Ŀ‬‬
‫‪ Z ⟺ Z = Z ⟺ z + 2z − 3 = z + 2z − 3 ⟺ (z − z) [(z + z) + 2] = 0‬ﺣﻘﻴﻘﻲ‬
‫⟺‬ ‫‪z=z‬‬ ‫⟺ ‪ 2 Re (z) = −2‬أو‬ ‫‪ Re (z) = −1‬أو ‪ z‬ﺣﻘﻴﻘﻲ‬

‫ﻓﺎﳌﺠﻤﻮﻋﺔ ‪ E‬اﻟﺘﻲ ﻧﺒﺤﺚ ﻋﻨﻬﺎ ﻫﻲ ّاﲢﺎد اﳌﺴﺘﻘﻴﻤﲔ ‪ y = 0‬و ‪. x = −1‬‬


‫ﲢﺪﻳﺪ اﳌﺠﻤﻮﻋﺔ ‪ : E‬ﻧُﺬﻛّﺮ ّ‬
‫أن ‪ z ≠ ı‬أي ‪ M‬ﲣﺘﻠﻒ ﻋﻦ ‪: A‬‬ ‫•‬ ‫‪2‬‬
‫•‬

‫⟺ ‪Z∈ℝ‬‬ ‫)𝜋 ‪ arg (Z) = 0 (mod‬أو ‪Z = 0‬‬


‫‪z−z‬‬
‫⟺‬ ‫‪ arg‬أو ‪z = 1‬‬ ‫)𝜋 ‪= 0 (mod‬‬
‫‪z−z‬‬
‫⟺‬ ‫)𝜋 ‪ AM, BM = 0 (mod‬أو ‪M = B‬‬
‫اﻟﻨﻘﻂ ‪ M ، A‬و ‪ B‬ﻋﲆ اﺳﺘﻘﺎﻣﺔ واﺣﺪة ﻣﻊ ‪⟺ M ≠ A‬‬

‫ﻧﺴﺘﻨﺘﺞ ّ‬
‫أن اﳌﺠﻤﻮﻋﺔ ‪ E‬ﻫﻲ اﳌﺴﺘﻘﻴﻢ )‪ (AB‬ﺑﺎﺳﺘﺜﻨﺎء اﻟﻨﻘﻄﺔ ‪.A‬‬
‫ﲢﺪﻳﺪ اﳌﺠﻤﻮﻋﺔ ‪ : F‬ﻧُﺬﻛّﺮ ّ‬
‫أن ‪ z ≠ ı‬أي ‪ M‬ﲣﺘﻠﻒ ﻋﻦ ‪: A‬‬ ‫•‬

‫𝜋‬
‫⟺ ‪Z ∈ ıℝ‬‬ ‫= )‪ arg (Z‬أو ‪Z = 0‬‬‫)𝜋 ‪(mod‬‬
‫‪2‬‬
‫‪z−z‬‬ ‫𝜋‬
‫‪ņőƱŀ Ţ‬‬
‫⟺‬ ‫‪ arg‬أو ‪z = 1‬‬ ‫)𝜋 ‪= (mod‬‬
‫‪z−z‬‬ ‫‪2‬‬
‫𝜋‬
‫⟺‬ ‫)𝜋 ‪ AM, BM = (mod‬أو ‪M = B‬‬
‫‪2‬‬
‫‪VI‬‬

‫ﻧﺴﺘﻨﺘﺞ ّ‬
‫أن اﳌﺠﻤﻮﻋﺔ ‪ F‬ﻫﻲ اﻟﺪاﺋﺮة اﻟﺘﻲ ﻗﻄﺮﻫﺎ ﻫﻮ ]‪ [AB‬ﺑﺎﺳﺘﺜﻨﺎء اﻟﻨﻘﻄﺔ ‪.A‬‬
‫■‬

‫‬ ‫‪.‬‬
‫‪ƕ‬‬

‫‪.‬‬
‫𝟏𝟏‬
‫ّ‬
‫ﺣﻞ ﰲ اﳌﺠﻤﻮﻋﺔ ‪ ℂ‬ﻛﻼ ﻣﻦ اﳌﻌﺎدﻻت اﻵﺗﻴﺔ ذات اﳌﺠﻬﻮل ‪ z‬و اﻛﺘﺐ اﳊﻠﻮل ﻋﲆ اﻟﺸﻜﻞ اﳉﱪي ‪:‬‬
‫‪z + 2ız = 1 + ı‬‬ ‫•‬ ‫‪5‬‬ ‫‪(1 + 4ı) z + 3 + 2ı√2 = 0‬‬ ‫•‬‫‪1‬‬
‫‪•2‬‬
‫‪ũŏ‬‬

‫‪2ız − 3ı + (1 + ı) z = 0‬‬
‫‪z + 2 (1 + ı) z = ı‬‬ ‫•‬ ‫‪6‬‬
‫‪2z − 2 + ız = 8 − 6ı + (1 + 2ı) z‬‬ ‫‪•3‬‬

‫‪ı (z + z) + (z + 2z) = 3‬‬ ‫•‬ ‫‪7‬‬ ‫‪(ız + 2) (3z − 1 + ı) = 0‬‬ ‫‪•4‬‬

‫ﺍﳊﻞّ‪ .‬ﻟﻴﻜﻦ ‪ z = x + ıy‬ﺣﻴﺚ ‪ x‬و ‪ y‬ﻋﺪدان ﺣﻘﻴﻘﻴﺎن‪.‬‬

‫𝟳𝟳‬

‫‪http ://tinyurl.com/Malki1718‬‬ ‫‪0‬‬


‫ ﶍﻮﻋﺔ اﻷﻋﺪاد اﳌﺮﻛﺒﺔ‬.1.VI
.
: ‫• ﻟﺪﻳﻨﺎ‬1

Ŕž Ŧ Ľ
3 + 2ı√2
(1 + 4ı) z + 3 + 2ı√2 = 0 ⟺ (1 + 4ı) z = −3 − 2ı√2 ⟺ z = −
1 + 4ı
3 + 2ı√2 (1 − 4ı)

œǃ
⟺ z=−
1 +4
3 − 12ı + 2ı√2 + 8√2 3 + 8√2 12 − 2√2
⟺ z=− =− +ı
17 17 17

Ŀ
3 + 8√2 12 − 2√2 3 + 8√2 12 − 2√2
.𝒮 = − +ı ‫ أي ﳎﻤﻮﻋﺔ اﳊﻠﻮل ﻫﻲ‬z = − +ı ‫إذن‬
17 17 17 17

: ‫• ﻟﺪﻳﻨﺎ‬2
2ız − 3ı + (1 + ı) z = 0 ⟺ (2ı + 1 + ı) z = 3ı ⟺ (1 + 3ı) z = 3ı
3ı 3ı (1 − 3ı) 3ı + 9 9 3
⟺ z= = = = + ı
1 + 3ı 1 +3 10 10 10

9 3 9 3
.𝒮 = + ı ‫ أي ﳎﻤﻮﻋﺔ اﳊﻠﻮل ﻫﻲ‬z = + ı ‫إذن‬
10 10 10 10

: ‫• ﻟﺪﻳﻨﺎ‬3
2z − 2 + ız = 8 − 6ı + (1 + 2ı) z ⟺ (2 + ı) z − (1 + 2ı) z = 8 − 6ı + 2
⟺ (2 + ı − 1 − 2ı) z = 10 − 6ı ⟺ (1 − ı) z = 10 − 6ı
10 − 6ı (10 − 6ı) (1 + ı)
⟺ z= =
1−ı 1 +1
10 + 10ı − 6ı + 6
⟺ z= = 8 + 2ı
2
ņőƱŀ Ţ
. 𝒮 = {8 + 2ı} ‫ أي ﳎﻤﻮﻋﺔ اﳊﻠﻮل ﻫﻲ‬z = 8 + 2ı ‫إذن‬
: ‫• ﻟﺪﻳﻨﺎ‬4
(ız + 2) (3z − 1 + ı) = 0 ⟺ ız + 2 = 0 ‫أو‬ 3z − 1 + ı = 0

VI
⟺ ız = −2 ‫أو‬ 3z = 1 − ı
2 1 1
⟺ z = − = 2ı ‫أو‬ z= − ı
ı 3 3
1 1 1 1
ƕ

. 𝒮 = 2ı, − ı ‫ أي ﳎﻤﻮﻋﺔ اﳊﻠﻮل ﻫﻲ‬z = − ı ‫ أو‬z = 2ı ‫إذن‬


3 3 3 3

: ‫• ﻟﺪﻳﻨﺎ‬5
z + 2ız = 1 + ı ⟺ x − ıy + 2ıx − 2y = 1 + ı ⟺ (x − 2y) + (2x − y) ı = 1 + ı
ũŏ

⎧ 1
x − 2y = 1 x − 2(2x − 1) = 1 ⎪x=
⟺ ⟺ ⟺ 3
2x − y = 1 y = 2x − 1 ⎪ y = −1

⎩ 3
1 1 1 1
.𝒮 = − ı ‫ أي ﳎﻤﻮﻋﺔ اﳊﻠﻮل ﻫﻲ‬z = − ı ‫إذن‬
3 3 3 3

𝟳𝟴

http ://tinyurl.com/Malki1718 0
‫ ﲤﺎرﻳﻦ ﺗﻄﺒﻴﻘﻴﺔ‬.VI
.
: ‫• ﻟﺪﻳﻨﺎ‬6

Ŕž Ŧ Ľ
z + 2 (1 + ı) z = ı ⟺ x + ıy + 2 (1 + ı) (x − ıy) = ı ⟺ (3x + 2y) + (2x − y) ı = ı
⎧ 2
3x + 2y = 0 (3x + 2y) + 2(2x − y) = 0 + 2 × 1 ⎪x=
7

œǃ
⟺ ⟺ ⟺
2x − y = 1 y = 2x − 1 ⎪ y = −3

⎩ 7
2 3 2 3
.𝒮 = − ı ‫ أي ﳎﻤﻮﻋﺔ اﳊﻠﻮل ﻫﻲ‬z = − ı ‫إذن‬

Ŀ
7 7 7 7
: ‫• ﻟﺪﻳﻨﺎ‬7
ı (z + z) + (z + 2z) = 3 ⟺ 2ıx + 3x − ıy = 3 ⟺ 3x + (2x + y) ı = 3
3x = 3 x=1
⟺ ⟺
2x + y = 1 y = −2x = −2

. 𝒮 = {1 − 2ı} ‫ أي ﳎﻤﻮﻋﺔ اﳊﻠﻮل ﻫﻲ‬z = 1 − 2ı ‫إذن‬


ız − 2z = −4 + 3ı
 .
𝟏𝟐 .
ّ
: ‫ﺣﻞ ﰲ ﳎﻤﻮﻋﺔ اﻷﻋﺪاد اﳌﺮﻛﺒﺔ اﳉﻤﻠﺔ‬
z + 2z = 3

.ّ‫ﺍﳊﻞ‬

ız − 2z = −4 + 3ı ız − 2z = −4 + 3ı ız − 2z = −4 + 3ı
⟺ ⟺
z + 2z = 3 z + 2z = 3 z + 2z = 3
ņőƱŀ Ţ
⎧ ız − 2z + z + 2z = (−4 + 3ı) + 3 ⎧ (1 + ı) z = −1 + 3ı
⟺ 1 ⟺ 1
⎨ z = (3 − z) ⎨ z = (3 − z)
⎩ 2 ⎩ 2
−1 + 3ı (−1 + 3ı) (1 − ı) 1
VI



⎪ z= 1+ı = = (2 + 2ı) = 1 + 2ı
1 +1 2


⎪ z = 1 (3 − (1 + 2ı)) = 1 − ı

⎩ 2
■ . z, z = (1 + 2ı, 1 − ı) : ‫إذن ﻟﻠﺠﻤﻠﺔ ﺣﻞ وﺣﻴﺪ و ﻫﻮ‬
ƕ

: z ‫ و‬z ‫ﳉﻤﻞ اﻟﺘﺎﻟﻴﺔ ذات اﳌﺠﻬﻮﻟﲔ‬ ّ


ُ ‫ ا‬ℂ ‫ﺣﻞ ﰲ اﳌﺠﻤﻮﻋﺔ‬
 .
𝟏𝟑 .
ũŏ

(1 + ı)z + ız = 2 − ı 2ız + z = 2ı
• 3 •1
(2 − ı)z + (3 − ı)z = 5 + 3ı 3z − ız = 1

ız − 2z = −3(1 − ı) 2z + 2(1 + ı)z = 4 − 6ı


• 4 •2
(1 − ı)z − (1 − ı)z = 0 z + 3ız = −3 − 2ı

𝟳𝟵

http ://tinyurl.com/Malki1718 0
‫‪ .1.VI‬ﶍﻮﻋﺔ اﻷﻋﺪاد اﳌﺮﻛﺒﺔ‬
‫‪.‬‬
‫ﺍﳊﻞّ‪ •1 .‬ﻣﻦ اﳌﻌﺎدﻟﺔ اﻷوﱃ ﻧﺠﺪ ‪ . z = 2ı − 2ız‬ﺑﺎﻟﺘﻌﻮﻳﺾ ﰲ اﳌﻌﺎدﻟﺔ اﻟﺜﺎﻧﻴﺔ ﻳﻨﺘﺞ ‪ 3z − ı (2ı − 2ız) = 1 :‬أي‬
‫‪ 3z + 2 − 2z = 1‬ﻣﻨﻪ ‪ z = −1‬و ﺑﺎﻟﺘﺎﱄ ‪. z = 2ı − 2ı (−1) = 4ı‬‬

‫‪Ŕž Ŧ Ľ‬‬
‫)‪. (−1, 4ı‬‬ ‫إذن ﻟﻠﺠﻤﻠﺔ ّ‬
‫ﺣﻞ وﺣﻴﺪ ﻫﻮ ‪:‬‬

‫‪œǃ‬‬
‫‪ •2‬ﻣﻦ اﳌﻌﺎدﻟﺔ اﻟﺜﺎﻧﻴﺔ ﻧﺠﺪ ‪ . z = −3ız − 3 − 2ı‬ﺑﺎﻟﺘﻌﻮﻳﺾ ﰲ اﳌﻌﺎدﻟﺔ اﻷوﱃ ﻳﻨﺘﺞ ‪:‬‬
‫‪ 2 −3ız − 3 − 2ı +2 (1 + ı) z = 4−6ı‬أي ‪2 (1 − 2ı) z = 10−2ı (−6ı + 2 + 2ı) z = 4−6ı+6+4ı‬‬
‫ﻣﻨﻪ ‪:‬‬
‫)‪2 (5 − ı‬‬ ‫‪(5 − ı) (1 + 2ı) 5 + 10ı − ı + 2 7 9‬‬

‫‪Ŀ‬‬
‫= ‪z‬‬ ‫=‬ ‫=‬ ‫‪= + ı‬‬
‫)‪2 (1 − 2ı‬‬ ‫‪1 +2‬‬ ‫‪5‬‬ ‫‪5 5‬‬
‫و ﺑﺎﻟﺘﺎﱄ ‪:‬‬
‫‪7 9‬‬ ‫‪27 21‬‬ ‫‪12 31‬‬
‫‪z = −3ı‬‬ ‫= ‪+ ı − 3 − 2ı‬‬ ‫= ‪− ı − 3 − 2ı‬‬ ‫‪− ı‬‬
‫‪5 5‬‬ ‫‪5‬‬ ‫‪5‬‬ ‫‪5‬‬ ‫‪5‬‬
‫‪12 31 7 9‬‬
‫‪.‬‬ ‫‪− ı, + ı‬‬ ‫إذن ﻟﻠﺠﻤﻠﺔ ّ‬
‫ﺣﻞ وﺣﻴﺪ ﻫﻮ ‪:‬‬
‫‪5‬‬ ‫‪5 5 5‬‬

‫‪ •3‬ﻣﻦ اﳌﻌﺎدﻟﺔ اﻷوﱃ ﻧﺠﺪ ‪:‬‬


‫‪1‬‬
‫‪z = (2 − ı − (1 + ı) z) = −ı (2 − ı − (1 + ı) z) = −1 − 2ı + (−1 + ı) z‬‬
‫‪ı‬‬
‫ﺑﺎﻟﺘﻌﻮﻳﺾ ﰲ اﳌﻌﺎدﻟﺔ اﻟﺜﺎﻧﻴﺔ ﻳﻨﺘﺞ ‪ (2 − ı) z + (3 − ı) (−1 − 2ı + (−1 + ı) z) = 5 + 3ı :‬أي‬
‫‪10 + 8ı‬‬
‫= ‪ z‬أي‬ ‫)‪ (2 − ı + (3 − ı) (−1 + ı)) z = 5 + 3ı + (3 − ı) (1 + 2ı‬ﻣﻨﻪ ‪ 3ız = 10 + 8ı‬ﻣﻨﻪ‬
‫‪3ı‬‬
‫‪8 10‬‬ ‫‪10 + 8ı‬‬
‫‪ z = −‬أي ‪− ı‬‬
‫= ‪ . z‬و ﺑﺎﻟﺘﺎﱄ ‪:‬‬ ‫‪⋅ı‬‬
‫‪3‬‬ ‫‪3‬‬ ‫‪3‬‬
‫‪8 10‬‬ ‫‪2‬‬ ‫‪1‬‬
‫)‪z = −1 − 2ı + (−1 + ı‬‬ ‫‪− ı‬‬ ‫‪= −1 − 2ı + + 6ı = − + 4ı‬‬
‫‪3‬‬ ‫‪3‬‬ ‫‪3‬‬ ‫‪3‬‬
‫‪8 10‬‬ ‫‪1‬‬
‫‪.‬‬ ‫‪− ı, − + 4ı‬‬ ‫إذن ﻟﻠﺠﻤﻠﺔ ّ‬
‫ﺣﻞ وﺣﻴﺪ ﻫﻮ ‪:‬‬
‫‪3‬‬ ‫‪3‬‬ ‫‪3‬‬

‫‪ •4‬اﳌﻌﺎدﻟﺔ اﻟﺜﺎﻧﻴﺔ ﺗُﻜﺎﻓِﺊ ‪ (1 − ı) z = (1 − ı) z :‬أي‬


‫‪ņőƱŀ Ţ‬‬
‫‪ . z = z‬ﺑﺎﻟﺘﻌﻮﻳﺾ ﰲ اﳌﻌﺎدﻟﺔ اﻷوﱃ ﻳﻨﺘﺞ ‪ız − 2z = −3 + 3ı‬‬
‫أي‬
‫‪−3 + 3ı (−3 + 3ı) (−2 − ı) 6 + 3ı − 6ı + 3 9 3‬‬
‫=‪z =z‬‬ ‫=‬ ‫=‬ ‫‪= − ı‬‬
‫‪ı−2‬‬ ‫‪2 +1‬‬ ‫‪5‬‬ ‫‪5 5‬‬

‫‪VI‬‬
‫‪9 3 9 3‬‬
‫‪.‬‬ ‫‪− ı, − ı‬‬ ‫إذن ﻟﻠﺠﻤﻠﺔ ّ‬
‫ﺣﻞ وﺣﻴﺪ ﻫﻮ ‪:‬‬
‫‪5 5 5 5‬‬
‫■‬

‫‬
‫‪ƕ‬‬

‫‪𝟏𝟒.‬‬ ‫‪.‬‬
‫⎧‬
‫⎪‬ ‫‪x+y+z=a‬‬
‫⎪‬
‫ﻟﺘﻜﻦ ‪ b ، a‬و ‪ c‬أﻋﺪادا ﻣﺮﻛﺒﺔ‪ّ .‬‬
‫ﺣﻞ ﰲ ‪ ℂ‬اﳉﻤﻠﺔ ‪. x + jy + j z = b :‬‬
‫⎨‬
‫⎪‬
‫⎪‬
‫‪⎩ x + j y + jz = c‬‬
‫‪ũŏ‬‬

‫ﻛﻴﻒ ﳚﺐ إﺧﺘﻴﺎر اﻷﻋﺪاد ‪ c ، b ، a‬ﺣﺘﻰ ﺗﻜﻮن اﳊﻠﻮل ﺣﻘﻴﻘﻴﺔ ؟‬

‫‪ •1‬ﻧﺴﻤﻲ )‪ (2) ، (1‬و )‪ (3‬ﻣﻌﺎدﻻت اﳉﻤﻠﺔ ﻋﲆ اﻟﱰﺗﻴﺐ‪ .‬ﻧُﺬﻛﱢﺮ ّ‬


‫أن ‪ 1 + j + j = 0 ، j = 1‬و ‪ . j = j‬ﻟﺪﻳﻨﺎ ‪:‬‬ ‫ﺍﳊﻞّ‪.‬‬

‫⬫ ﺑﺠﻤﻊ )‪ (1) + (2) + (3‬ﻳﻨﺘﺞ ‪ . 3x + (1 + j + j )y + (1 + j + j)z = a + b + c‬و ﺑﲈ ّ‬


‫أن ‪1 + j + j = 0‬‬
‫ّ‬
‫ﻓﺈن ‪. 3x = a + b + c‬‬

‫𝟬𝟴‬

‫‪http ://tinyurl.com/Malki1718‬‬ ‫‪0‬‬


‫ ﲤﺎرﻳﻦ ﺗﻄﺒﻴﻘﻴﺔ‬.VI
.
.(1 + j + j)x + (1 + j + j )y + (1 + j + j )z = a + bj + cj ‫( ﻳﻨﺘﺞ‬1) + j × (2) + j × (3) ‫⬫ ﺑﺠﻤﻊ‬
ّ 1 + j + j = 0 ‫ و‬j = j ، j = 1 ‫أن‬
. 3y = a + bj + cj ‫ﻓﺈن‬ ّ ‫و ﺑﲈ‬

Ŕž Ŧ Ľ
.(1 + j + j )x + (1 + j + j )y + (1 + j + j )z = a + bj + cj ‫( ﻳﻨﺘﺞ‬1) + j × (2) + j × (3) ‫⬫ ﺑﺠﻤﻊ‬
ّ 1 + j + j = 0 ‫ و‬j = j ، j = 1 ‫أن‬
. 3z = a + bj + cj ‫ﻓﺈن‬ ّ ‫و ﺑﲈ‬

œǃ
a + bj + cj a + bj + cj a+b+c
.z = ‫و‬y = ،x = ‫إذن‬
3 3 3
a + bj + cj a + bj + cj a+b+c

Ŀ
.‫ ﺣﻠﻮل ﻟﻠﺠﻤﻠﺔ‬z = ‫و‬y = ،x = ّ ‫ ﻧﺘﺤﻘﻖ ﺑﺴﻬﻮﻟﺔ ﻣﻦ‬،‫و ﺑﺎﻟﻌﻜﺲ‬
‫أن‬
3 3 3
ّ ‫ ﺣﻘﻴﻘﻴ ًﺔ‬z ‫ و‬y ، x ‫• إذا ﻛﺎﻧﺖ اﳊﻠﻮل‬2
: ‫ ﺣﻘﻴﻘﻲ و‬a = x + y + z ‫ﻓﺈن‬
b = x + j y + jz = x + j y + jz = x + jy + j z = c
. c = b ‫ و‬a ∈ ℝ ‫إذن‬
ّ c = b ‫ و‬a ∈ ℝ ‫ إذا ﻛﺎن‬،‫ﺑﺎﻟﻌﻜﺲ‬
: ‫ﻓﺈن‬
a+b+c a+b+b a+b+b a+c+b
x= = = = =x
3 3 3 3
a + bj + cj a + bj + bj a + bj + bj a + cj + bj
y= = = = =y
3 3 3 3
a + bj + cj a + bj + bj a + bj + bj a + cj + bj
z= = = = =z
3 3 3 3
ّ ‫أي‬
.‫ ﺣﻘﻴﻘﻴﺔ‬z ‫ و‬y ، x ‫أن اﳊﻠﻮل‬
. c = b ‫ و‬a ∈ ℝ ‫ ﺗﻜﻮن ﺣﻠﻮل اﳉﻤﻠﺔ ﺣﻘﻴﻘﻴ ًﺔ إذا و ﻓﻘﻂ إذا ﻛﺎن‬،‫ﰲ اﻷﺧﲑ‬


ņőƱŀ Ţ
.
𝟏𝟓 .
ّ
: ‫ ﻣﺮاﻓﻘﻪ‬z ‫ ﻫﻮ اﳌﺠﻬﻮل و‬z ‫ اﳌﻌﺎدﻻت اﻟﺘﺎﻟﻴﺔ ﺣﻴﺚ‬، ℂ ‫ ﰲ اﳌﺠﻤﻮﻋﺔ‬،‫ﺣﻞ‬
zz + z − z − 5 − 2ı = 0 • 4 z −z+2=0 1

VI

(3 + ı) z + (1 − 2ı) z + 2 − 3ı = 0 • 5 z = (1 + ı) z + 3 − 2ı 2

(3 + ı) z + (1 − 3ı) z + 12 − 6ı = 0 • 6 z + zz − 4 − 6ı = 0 3

. x, y ∈ ℝ ‫ ﻣﻊ‬z = x + ıy ‫ ﻧﻀﻊ‬.ّ‫ﺍﳊﻞ‬
ƕ

: ‫• ﻟﺪﻳﻨﺎ‬1
z − z + 2 = 0 ⟺ (x + ıy) − (x − ıy) + 2 = 0
ũŏ

⟺ x − y + 2ıxy − x + ıy + 2 = 0
⟺ x − y − x + 2 + ıy (2x + 1) = 0
x −y −x+2=0

y (2x + 1) = 0

1
ّ ‫ﻣﻦ اﳌﻌﺎدﻟﺔ اﻟﺜﺎﻧﻴﺔ ﻧﺴﺘﻨﺘﺞ‬
. x = − ‫ أو‬y = 0 ‫أن‬
2

𝟴𝟭

http ://tinyurl.com/Malki1718 0
‫‪ .1.VI‬ﶍﻮﻋﺔ اﻷﻋﺪاد اﳌﺮﻛﺒﺔ‬
‫‪.‬‬
‫إذا ﻛﺎن ‪ّ y = 0‬‬
‫ﻓﺈن اﳌﻌﺎدﻟﺔ اﻷوﱃ ﺗُﺼﺒﺢ ‪ُ . x − x + 2 = 0 :‬ﳑ ﱢﻴﺰﻫﺎ ﻫﻮ ‪Δ = (−1) − 4 ⋅ 2 = −7 < 0‬‬ ‫•‬

‫إذن ﻟﻴﺲ ﳍﺎ ﺣﻠﻮل ﺣﻘﻴﻘﻴﺔ )ﻧُﺬﻛﱢﺮ ّ‬


‫أن ‪ x‬و ‪ y‬ﻋﺪدان ﺣﻘﻴﻘﻴﺎن( ‪.‬‬

‫‪Ŕž Ŧ Ľ‬‬
‫‪11‬‬ ‫‪11‬‬ ‫‪1‬‬ ‫‪1‬‬ ‫‪1‬‬
‫√‪. y = ±‬‬ ‫= ‪ y‬ﻣﻨﻪ‬ ‫إذا ﻛﺎن ‪ّ x = −‬‬
‫ﻓﺈن اﳌﻌﺎدﻟﺔ اﻷوﱃ ﺗُﺼﺒﺢ ‪ − y + + 2 = 0 :‬أي‬ ‫•‬
‫‪2‬‬ ‫‪4‬‬ ‫‪4‬‬ ‫‪2‬‬ ‫‪2‬‬

‫‪œǃ‬‬
‫‪1‬‬ ‫‪11‬‬ ‫‪1‬‬ ‫‪11‬‬
‫√‪. − + ı‬‬ ‫√‪ − − ı‬و‬ ‫إذن ﻟﻠﻤﻌﺎدﻟﺔ ﺣﻼّن ﳘﺎ‬
‫‪2‬‬ ‫‪2‬‬ ‫‪2‬‬ ‫‪2‬‬
‫‪ •2‬ﻟﺪﻳﻨﺎ ‪:‬‬

‫‪Ŀ‬‬
‫‪z = (1 + ı) z + 3 − 2ı ⟺ x + ıy − (1 + ı) (x − ıy) = 3 − 2ı‬‬
‫‪⟺ x + ıy − x − y − ıx + ıy = 3 − 2ı‬‬
‫‪⟺ −y + (2y − x) ı = 3 − 2ı‬‬
‫‪−y = 3‬‬ ‫‪y = −3‬‬
‫⟺‬ ‫⟺‬
‫‪2y − x = −2‬‬ ‫‪x = 2y + 2 = −6 + 2 = −4‬‬

‫إذن ﻟﻠﻤﻌﺎدﻟﺔ ّ‬
‫ﺣﻞ وﺣﻴﺪ ﻫﻮ ‪. −4 − 3ı‬‬
‫‪ •3‬ﻟﺪﻳﻨﺎ ‪:‬‬
‫‪z + zz − 4 − 6ı = 0 ⟺ x − y + 2ıxy + x + y − 4 − 6ı = 0 ⟺ 2x + 2ıxy = 4 + 6ı‬‬
‫‪2x = 4‬‬ ‫‪x =2‬‬
‫⟺‬ ‫⟺‬
‫‪2xy = 6‬‬ ‫‪xy = 3‬‬

‫ﻣﻦ اﳌﻌﺎدﻟﺔ اﻷوﱃ ﻧﺴﺘﻨﺘﺞ ّ‬


‫أن ‪. x = ±√2‬‬
‫‪3 2‬‬ ‫‪3‬‬ ‫‪3√2‬‬
‫= ‪ y‬أي ‪ z = −√2 − √ ı‬؛‬ ‫‪=−‬‬ ‫إذا ﻛﺎن ‪ x = −√2‬ﻓﻤﻦ اﳌﻌﺎدﻟﺔ اﻟﺜﺎﻧﻴﺔ ﻧﺠﺪ ّ‬
‫أن‬ ‫•‬
‫‪2‬‬ ‫‪−√2‬‬ ‫‪2‬‬
‫‪3 2‬‬ ‫‪3‬‬ ‫‪3 √2‬‬
‫= ‪ y‬أي ‪. z = √2 + √ ı‬‬ ‫=‬ ‫و إذا ﻛﺎن ‪ّ x = √2‬‬
‫ﻓﺈن‬ ‫•‬
‫‪2‬‬ ‫‪√2‬‬ ‫‪2‬‬
‫‪ņőƱŀ Ţ‬‬
‫‪3 2‬‬ ‫‪3 2‬‬
‫إذن ﻟﻠﻤﻌﺎدﻟﺔ ﺣﻼّن ﳘﺎ ‪ −√2 − √ ı :‬و ‪. √2 + √ ı‬‬
‫‪2‬‬ ‫‪2‬‬
‫‪ •4‬ﻟﺪﻳﻨﺎ ‪:‬‬
‫‪x +y =5‬‬

‫‪VI‬‬
‫⟺ ‪zz + z − z − 5 − 2ı = 0 ⟺ x + y + 2ıy = 5 + 2ı‬‬
‫‪2y = 2‬‬
‫‪x =4‬‬ ‫‪x = ±2‬‬
‫⟺‬ ‫⟺‬
‫‪y=1‬‬ ‫‪y=1‬‬
‫‪ƕ‬‬

‫إذن ﻟﻠﻤﻌﺎدﻟﺔ ﺣﻼّن ﳘﺎ ‪ −2 + ı :‬و ‪. 2 + ı‬‬


‫‪ •5‬ﻟﺪﻳﻨﺎ ‪:‬‬
‫‪(3 + ı) z + (1 − 2ı) z + 2 − 3ı = 0 ⟺ (3 + ı) (x + ıy) + (1 − 2ı) (x − ıy) = −2 + 3ı‬‬
‫‪ũŏ‬‬

‫‪⟺ 3x + 3ıy + ıx − y + x − ıy − 2ıx − 2y = −2 + 3ı‬‬


‫‪⟺ 4x − 3y + ı (−x + 2y) = −2 + 3ı‬‬
‫‪4x − 3y = −2‬‬ ‫‪4 (2y − 3) − 3y = −2‬‬
‫⟺‬ ‫⟺‬
‫‪−x + 2y = 3‬‬ ‫‪x = 2y − 3‬‬
‫‪5y = 10‬‬ ‫‪y=2‬‬
‫⟺‬ ‫⟺‬
‫‪x = 2y − 3‬‬ ‫‪x = 2y − 3 = 1‬‬

‫𝟮𝟴‬

‫‪http ://tinyurl.com/Malki1718‬‬ ‫‪0‬‬


‫‪ .VI‬ﲤﺎرﻳﻦ ﺗﻄﺒﻴﻘﻴﺔ‬
‫‪.‬‬
‫إذن ﻟﻠﻤﻌﺎدﻟﺔ ّ‬
‫ﺣﻞ وﺣﻴﺪ ﻫﻮ ‪. 1 + 2ı :‬‬
‫‪ •6‬ﻟﺪﻳﻨﺎ ‪:‬‬

‫‪Ŕž Ŧ Ľ‬‬
‫‪(3 + ı) z + (1 − 3ı) z + 12 − 6ı = 0 ⟺ (3 + ı) (x + ıy) + (1 − 3ı) (x − ıy) = −12 + 6ı‬‬

‫‪œǃ‬‬
‫‪⟺ 4x − 4y + ı (−2x + 2y) = −12 + 6ı‬‬
‫‪4x − 4y = −12‬‬ ‫‪x − y = −3‬‬
‫⟺‬ ‫⟺‬
‫‪−2x + 2y = 6‬‬ ‫‪x − y = −3‬‬

‫‪Ŀ‬‬
‫‪y=t‬‬
‫⟺‬
‫‪x=t+3,‬‬ ‫‪t∈ℝ‬‬

‫إذن ﻟﻠﻤﻌﺎدﻟﺔ ﻋﺪد ﻻﳖﺎﺋﻲ ﻣﻦ اﳊﻠﻮل و ﻫﻲ اﻷﻋﺪاد )‪ t + ı (t + 3‬ﻣﻊ ‪. t ∈ ℝ‬‬


‫■‬

‫‪z +z+1‬‬ ‫‬ ‫‪.‬‬


‫𝟔𝟏‬ ‫‪.‬‬
‫= ‪.Z‬‬ ‫ﻟﻴﻜﻦ ‪ z = x + ıy‬ﻋﺪد ًا ﻣﺮﻛﺒﺎ ﺣﻴﺚ ‪ . z ≠ 1‬ﻧﻀﻊ‬
‫‪z−1‬‬
‫‪c‬‬
‫‪. ∀z ≠ 1 ,‬‬ ‫‪Z = az + b +‬‬ ‫‪ •1‬أوﺟﺪ اﻷﻋﺪاد اﳌﺮﻛﺒﺔ ‪ b ، a‬و ‪ c‬ﺑﺤﻴﺚ ﻳﻜﻮن ‪:‬‬
‫‪z−1‬‬

‫‪ •2‬ﱢ‬
‫ﻋﱪ ﻋﻦ )‪ Re (Z‬و )‪ Im (Z‬ﺑﺪﻻﻟﺔ ‪ x‬و ‪. y‬‬

‫‪ •3‬ﻟﺘﻜﻦ ‪ M‬ﺻﻮرة ‪ z‬ﰲ اﳌﺴﺘﻮي اﳌﻨﺴﻮب إﱃ ﻣﻌﻠﻢ ﻣﺘﻌﺎﻣﺪ و ﻣﺘﺠﺎﻧﺲ ⃗ ‪. O, i,⃗ j‬‬
‫أوﺟﺪ ‪ ℰ‬ﳎﻤﻮﻋﺔ اﻟﻨﻘﻂ ‪ M‬ﺑﺤﻴﺚ ﻳﻜﻮن ‪ Z ∈ ℝ‬و ‪. z ∉ ℝ‬‬

‫‪ •1‬ﻟﺪﻳﻨﺎ ‪:‬‬ ‫ﺍﳊﻞّ‪.‬‬


‫‪ņőƱŀ Ţ‬‬
‫‪c‬‬ ‫‪z +z+1‬‬ ‫‪c‬‬
‫‪Z = az + b +‬‬ ‫⟺‬ ‫‪= az + b +‬‬
‫‪z−1‬‬ ‫‪z−1‬‬ ‫‪z−1‬‬
‫‪z +z+1‬‬ ‫‪az (z − 1) + b (z − 1) + c‬‬
‫⟺‬ ‫=‬
‫‪z−1‬‬ ‫‪z−1‬‬
‫‪VI‬‬

‫‪z +z+1‬‬ ‫‪az + (b − a) z + c‬‬


‫⟺‬ ‫=‬
‫‪z−1‬‬ ‫‪z−1‬‬
‫⎧‬ ‫=‪a‬‬ ‫‪1‬‬ ‫‪⎪a = 1‬‬
‫⎧‬
‫⟺‬ ‫‪b‬‬ ‫‪−‬‬ ‫=‪a‬‬ ‫⟺ ‪1‬‬ ‫‪b= 2‬‬
‫⎨‬ ‫⎨‬
‫⎪‬
‫=‪⎩c−b‬‬ ‫‪1‬‬ ‫‪⎩c= 3‬‬
‫‪ƕ‬‬

‫‪3‬‬
‫‪. ∀z ≠ 1 ,‬‬ ‫‪Z=z+2+‬‬ ‫إذن ‪:‬‬
‫‪z−1‬‬
‫‪ •2‬ﻟﺪﻳﻨﺎ ‪:‬‬
‫‪ũŏ‬‬

‫‪3‬‬ ‫‪3‬‬ ‫)‪3 (x − 1 − ıy‬‬


‫‪Z=z+2+‬‬ ‫‪= x + ıy + 2 +‬‬ ‫‪= x + 2 + ıy +‬‬
‫‪z−1‬‬ ‫‪x − 1 + ıy‬‬ ‫‪(x − 1) + y‬‬
‫)‪3 (x − 1‬‬ ‫‪3y‬‬
‫‪=x+2+‬‬ ‫‪+‬‬ ‫‪y−‬‬ ‫‪ı‬‬
‫‪(x − 1) + y‬‬ ‫‪(x − 1) + y‬‬
‫‪3y‬‬ ‫)‪3 (x − 1‬‬
‫‪. Im (Z) = y −‬‬ ‫‪ Re (Z) = x + 2 +‬و‬ ‫إذن ‪:‬‬
‫‪(x − 1) + y‬‬ ‫‪(x − 1) + y‬‬

‫𝟯𝟴‬

‫‪http ://tinyurl.com/Malki1718‬‬ ‫‪0‬‬


‫‪ .1.VI‬ﶍﻮﻋﺔ اﻷﻋﺪاد اﳌﺮﻛﺒﺔ‬
‫‪.‬‬
‫‪ •3‬ﻟﺘﻜﻦ )‪ M (x, y‬ﻧﻘﻄﺔ ﻣﻦ ‪ . ℰ‬ﻟﺪﻳﻨﺎ ‪:‬‬

‫‪Ŕž Ŧ Ľ‬‬
‫⎧‬ ‫‪3y‬‬ ‫⎧‬
‫‪⎪y 1 −‬‬ ‫‪3‬‬
‫‪Z∈ℝ‬‬ ‫‪Im (Z) = 0‬‬ ‫‪⎪y −‬‬ ‫‪=0‬‬ ‫‪=0‬‬
‫⟺‬ ‫⟺‬ ‫‪(x − 1) + y‬‬ ‫⟺‬ ‫‪(x − 1) + y‬‬
‫‪z∉ℝ‬‬ ‫‪Im (z) ≠ 0‬‬ ‫⎨‬
‫⎪‬ ‫⎨‬
‫⎪‬

‫‪œǃ‬‬
‫⎩‬ ‫‪y≠0‬‬ ‫⎩‬ ‫‪y≠0‬‬
‫⎧‬ ‫‪3‬‬
‫‪⎪1 −‬‬ ‫‪=0‬‬ ‫‪(x − 1) + y = 3‬‬
‫⟺‬ ‫‪(x − 1) + y‬‬ ‫⟺‬
‫⎨‬
‫⎪‬ ‫‪y≠0‬‬
‫‪y≠0‬‬

‫‪Ŀ‬‬
‫⎩‬

‫ﰲ اﳌﻌﺎدﻟﺔ اﻷﺧﲑة‪ ،‬إذا ﻛﺎن ‪ّ y = 0‬‬


‫ﻓﺈن ‪ (x − 1) = 3‬ﻣﻨﻪ ‪ . x = 1 ± √3‬إذن‬
‫‪ ℰ‬ﻫﻲ اﻟﺪاﺋﺮة اﻟﺘﻲ ﻣﺮﻛﺰﻫﺎ )‪Ω (1, 0‬‬
‫و ﻧﺼﻒ ﻗﻄﺮﻫﺎ ‪ √3‬ﺑﺎﺳﺘﺜﻨﺎء اﻟﻨﻘﻄﺘﲔ ‪ M 1 − √3, 0‬و ‪. M 1 + √3, 0‬‬
‫■‬

‫اﳌﺴﺘﻮي ﻣﻨﺴﻮب إﱃ ﻣﻌﻠﻢ ﻣﺘﻌﺎﻣﺪ و ﻣﺘﺠﺎﻧﺲ‪.‬‬


‫‬ ‫‪𝟏𝟕.‬‬ ‫‪.‬‬

‫‪4‬‬
‫‪.L = z+‬‬ ‫‪ z‬ﻋﺪد ﻣﺮﻛﺐ ﺻﻮرﺗﻪ ‪ . M‬ﻧﻀﻊ‬
‫‪z‬‬
‫ﻋﲔ اﳌﺠﻤﻮﻋﺔ ‪ ℰ‬ﻟﻠﻨﻘﻂ ‪ M‬ﻣﻦ اﳌﺴﺘﻮي ﺑﺤﻴﺚ ﻳﻜﻮن ‪ L‬ﻋﺪ ًدا ﺣﻘﻴﻘﻴﺎ‪.‬‬ ‫ﱢ‬ ‫•‬

‫ﻋﲔ اﳌﺠﻤﻮﻋﺔ ‪ ℰ‬ﻟﻠﻨﻘﻂ ‪ M‬ﻣﻦ اﳌﺴﺘﻮي ﺑﺤﻴﺚ ﻳﻜﻮن ‪ L‬ﲣﻴﻠﻴﺎ ِﴏ ًﻓﺎ‪.‬‬‫ﱢ‬ ‫•‬

‫ﺍﳊﻞّ‪ .‬ﻧﻀﻊ ‪ z = x + ıy‬ﻣﻊ ‪ x, y ∈ ℝ‬و ‪ . z ≠ 0‬ﻟﺪﻳﻨﺎ ‪:‬‬

‫‪4 z + 4 x − y + 2ıxy + 4‬‬ ‫)‪x − y + 4 + 2ıxy (x − ıy‬‬


‫‪L=z+‬‬ ‫=‬ ‫=‬ ‫=‬
‫‪z‬‬ ‫‪z‬‬ ‫‪x + ıy‬‬ ‫‪x +y‬‬
‫‪x − ıx y − xy + ıy + 4x − 4ıy + 2ıx y + 2xy‬‬
‫‪ņőƱŀ Ţ‬‬
‫=‬
‫‪x +y‬‬
‫‪x + 4x + xy‬‬ ‫‪y − 4y + x y‬‬
‫=‬ ‫‪+ı‬‬
‫‪x +y‬‬ ‫‪x +y‬‬

‫‪VI‬‬
‫و ﺑﺎﻟﺘﺎﱄ ‪:‬‬
‫•‬

‫‪y − 4y + x y‬‬
‫⟺ ‪M ∈ ℰ ⟺ Im (L) = 0‬‬ ‫‪= 0 ⟺ y − 4y + x y = 0‬‬
‫‪x +y‬‬
‫‪ƕ‬‬

‫‪⟺ y x +y −4 =0 ⟺ y=0‬‬ ‫أو‬ ‫‪x +y =2‬‬

‫إذن ﳎﻤﻮﻋﺔ اﻟﻨﻘﻂ اﻟﺘﻲ ﻧﺒﺤﺚ ﻋﻨﻬﺎ ﻫﻲ اﳌﺴﺘﻘﻴﻢ ‪ Δ‬اﻟﺬي ﻣﻌﺎدﻟﺘﻪ ‪) y = 0‬ﳏﻮر اﻟﻔﻮاﺻﻞ( ﺑﺎﺳﺘﺜﻨﺎء اﻟﻨﻘﻄﺔ )‪O (0, 0‬‬
‫و اﻟﺪاﺋﺮة )‪ 𝒞 (O, 2‬اﻟﺘﻲ ﻣﺮﻛﺰﻫﺎ )‪ O (0, 0‬و ﻧﺼﻒ ﻗﻄﺮﻫﺎ ‪. 2‬‬
‫‪ũŏ‬‬

‫أي ‪. ℰ = 𝒞 ∪ (Δ ⧵ {O}) :‬‬


‫•‬

‫‪x + 4x + xy‬‬
‫‪M∈ℰ‬‬ ‫⟺ ‪⟺ Re (L) = 0‬‬ ‫‪= 0 ⟺ x + 4x + xy = 0‬‬
‫‪x +y‬‬
‫‪⟺ x x +y +4 =0 ⟺ x=0‬‬ ‫ّ‬
‫)ﻷن ‪(x + y + 4 > 0‬‬

‫𝟰𝟴‬

‫‪http ://tinyurl.com/Malki1718‬‬ ‫‪0‬‬


‫‪ .VI‬ﲤﺎرﻳﻦ ﺗﻄﺒﻴﻘﻴﺔ‬
‫‪.‬‬
‫و ﺑﺎﻟﺘﺎﱄ ‪ ℰ‬ﻫﻲ اﳌﺴﺘﻘﻴﻢ ‪ Δ‬اﻟﺬي ﻣﻌﺎدﻟﺘﻪ ‪) x = 0‬ﳏﻮر اﻟﱰاﺗﻴﺐ( ﺑﺎﺳﺘﺜﻨﺎء اﻟﻨﻘﻄﺔ )‪. O (0, 0‬‬
‫إذن ‪. ℰ = Δ ⧵ {O} :‬‬

‫‪Ŕž Ŧ Ľ‬‬
‫■‬

‫‪œǃ‬‬
‫اﳌﺴﺘﻮي ﻣﻨﺴﻮب إﱃ ﻣﻌﻠﻢ ﻣﺘﻌﺎﻣﺪ و ﻣﺘﺠﺎﻧﺲ‪.‬‬
‫‬ ‫‪.‬‬
‫𝟖𝟏‬ ‫‪.‬‬

‫‪ z‬ﻋﺪد ﻣﺮﻛﺐ ﺻﻮرﺗﻪ ‪ M‬و ‪ z‬ﻣﺮاﻓﻘﻪ‪.‬‬

‫‪Ŀ‬‬
‫ﻋﲔ ﳎﻤﻮﻋﺔ اﻟﻨﻘﻂ ‪ M‬ﻣﻦ اﳌﺴﺘﻮي ﺑﺤﻴﺚ ﻳﻜﻮن ‪. zz + 3 (z + z) = 7 :‬‬ ‫ّ‬

‫ﺍﳊﻞّ‪ .‬ﻧﻀﻊ ‪ z = x + ıy‬ﻣﻊ ‪ . x, y ∈ ℝ‬ﻟﺪﻳﻨﺎ ‪:‬‬

‫‪zz + 3 (z + z) = (x + ıy) (x − ıy) + 3 (x + ıy + x − ıy) = x − (ıy) + 6x = x + y + 6x‬‬

‫و ﺑﺎﻟﺘﺎﱄ ‪:‬‬

‫‪zz + 3 (z + z) = 7 ⟺ x + y + 6x = 7 ⟺ (x + 3) − 9 + y = 7‬‬
‫‪⟺ (x + 3) + (y − 0) = 16 = 4‬‬

‫و ﻫﻲ ﻣﻌﺎدﻟﺔ اﻟﺪاﺋﺮة )‪ 𝒞 (Ω, 4‬اﻟﺘﻲ ﻣﺮﻛﺰﻫﺎ )‪ Ω (−3, 0‬و ﻧﺼﻒ ﻗﻄﺮﻫﺎ ‪. 4‬‬
‫■‬ ‫إذن ﳎﻤﻮﻋﺔ اﻟﻨﻘﻂ )‪ M (z‬ﻣﻦ اﳌﺴﺘﻮي ﺑﺤﻴﺚ ﻳﻜﻮن ‪ zz + 3 (z + z) = 7 :‬ﻫﻲ اﻟﺪاﺋﺮة 𝒞 ‪.‬‬

‫‪.‬‬
‫𝟗𝟏‬ ‫‪.‬‬

‫‪ M ، M ، A •1‬ﻧﻘﻂ ﻣﻦ اﳌﺴﺘﻮي ﻟﻮاﺣﻘﻬﺎ ﻋﲆ اﻟﱰﺗﻴﺐ ‪. 1 + z ، z ، 1 :‬‬


‫ﻋﲔ ﳎﻤﻮﻋﺔ اﻟﻨﻘﻂ ‪ M‬ﺑﺤﻴﺚ ﺗﻜﻮن ‪ M ، M ، A‬ﻋﲆ اﺳﺘﻘﺎﻣﺔ واﺣﺪة‪.‬‬
‫‪ņőƱŀ Ţ‬‬
‫ّ‬
‫‪ M ، M ، B •2‬ﻧﻘﻂ ﻣﻦ اﳌﺴﺘﻮي ﻟﻮاﺣﻘﻬﺎ ﻋﲆ اﻟﱰﺗﻴﺐ ‪. ız ، z ، ı :‬‬
‫ﻋﲔ ﳎﻤﻮﻋﺔ اﻟﻨﻘﻂ ‪ M‬ﺑﺤﻴﺚ ﺗﻜﻮن ‪ M ، M ، B‬ﻋﲆ اﺳﺘﻘﺎﻣﺔ واﺣﺪة‪.‬‬
‫ّ‬
‫‪VI‬‬

‫ﺣﺪّ د ﰲ ﻫﺬه اﳊﺎﻟﺔ ﳎﻤﻮﻋﺔ اﻟﻨﻘﻂ ‪. M‬‬

‫ﺍﳊﻞّ‪ .‬ﻧﻀﻊ ‪ z = x + ıy‬ﻣﻊ ‪. x, y ∈ ℝ‬‬


‫‪ƕ‬‬

‫أن اﻟﻨﻘﻂ )‪ M (2) ، M (1) ، A (1‬ﻋﲆ اﺳﺘﻘﺎﻣﺔ واﺣﺪة ّ‬


‫)ﻷن ‪ M = A‬و ﻣﻦ اﻟﺒﺪﳞﻲ‬ ‫‪ •1‬إذا ﻛﺎن ‪ z = 1‬ﻓﻤﻦ اﻟﻮاﺿﺢ ّ‬
‫أي ﻧﻘﻄﺘﲔ ﻣﻦ اﳌﺴﺘﻮي ﻋﲆ اﺳﺘﻘﺎﻣﺔ واﺣﺪة( ‪ .‬ﻧﻔﺮض إذن ّ‬
‫أن ‪. z ≠ 1‬‬ ‫أن ّ‬‫ّ‬
‫ﺗﻜﻮن اﻟﻨﻘﻂ )‪ M 1 + z ، M (z) ، A (1‬ﻋﲆ اﺳﺘﻘﺎﻣﺔ واﺣﺪة إذا و ﻓﻘﻂ إذا ﻛﺎن )𝜋 ‪ MAM = 0 (mod‬أي‬
‫‪ũŏ‬‬

‫‪z +1−1‬‬
‫‪ AM, AM = arg‬و‬ ‫‪ . AM, AM‬ﻟﻜﻦ )𝜋‪(mod 2‬‬ ‫إذا و ﻓﻘﻂ إذا ﻛﺎن )𝜋 ‪= 0 (mod‬‬
‫‪z−1‬‬
‫‪z‬‬
‫‪arg‬‬ ‫‪ M 1 + z‬ﻋﲆ اﺳﺘﻘﺎﻣﺔ واﺣﺪة إذا و ﻓﻘﻂ إذا ﻛﺎن )𝜋 ‪= 0 (mod‬‬ ‫ﺑﺎﻟﺘﺎﱄ اﻟﻨﻘﻂ )‪، M (z) ، A (1‬‬
‫‪z−1‬‬

‫𝟱𝟴‬

‫‪http ://tinyurl.com/Malki1718‬‬ ‫‪0‬‬


‫‪ .1.VI‬ﶍﻮﻋﺔ اﻷﻋﺪاد اﳌﺮﻛﺒﺔ‬
‫‪.‬‬
‫‪z‬‬
‫‪ . Im‬ﻟﻜﻦ ‪:‬‬ ‫أي إذا و ﻓﻘﻂ إذا ﻛﺎن ‪= 0‬‬
‫‪z−1‬‬

‫‪Ŕž Ŧ Ľ‬‬
‫‪z‬‬ ‫‪x − y + 2ıxy‬‬ ‫)‪x − y + 2ıxy (x − 1 − ıy‬‬
‫=‬ ‫=‬
‫‪z−1‬‬ ‫‪x − 1 + ıy‬‬ ‫‪(x − 1) + y‬‬

‫‪œǃ‬‬
‫‪x − x − ıx y − xy + y + ıy + 2ıx y − 2ıxy + 2xy‬‬
‫=‬
‫‪(x − 1) + y‬‬
‫‪x − x + xy + y‬‬ ‫‪y + x y − 2xy‬‬
‫=‬ ‫‪+ı‬‬

‫‪Ŀ‬‬
‫‪(x − 1) + y‬‬ ‫‪(x − 1) + y‬‬
‫‪z‬‬ ‫‪y + x y − 2xy‬‬
‫‪Im‬‬ ‫⟺ ‪=0‬‬ ‫‪= 0 ⟺ y + x y − 2xy = 0‬‬ ‫ﻣﻨﻪ ‪:‬‬
‫‪z−1‬‬ ‫‪(x − 1) + y‬‬
‫‪⟺ y x + y − 2x = 0 ⟺ y = 0‬‬ ‫أو‬ ‫‪(x − 1) − 1 + y = 0‬‬
‫‪⟺ y=0‬‬ ‫أو‬ ‫‪(x − 1) + (y − 0) = 1‬‬

‫ّ‬
‫)ﻷن ‪ (z ≠ 1‬ﻣﻊ اﻟﺪاﺋﺮة‬ ‫و ﻫﻲ اﲢﺎد اﳌﺴﺘﻘﻴﻢ ‪ Δ‬اﻟﺬي ﻣﻌﺎدﻟﺘﻪ ‪) y = 0‬ﳏﻮر اﻟﻔﻮاﺻﻞ( ﺑﺎﺳﺘﺜﻨﺎء اﻟﻨﻘﻄﺔ )‪A (1‬‬
‫)‪ 𝒞 (Ω, 1‬اﻟﺘﻲ ﻣﺮﻛﺰﻫﺎ )‪) Ω (1, 0‬أي ‪ (Ω = A‬و ﻧﺼﻒ ﻗﻄﺮﻫﺎ ‪.1‬‬
‫ﰲ اﻷﺧﲑ‪ ،‬ﳎﻤﻮﻋﺔ اﻟﻨﻘﻂ اﻟﺘﻲ ﲢﻘﻖ اﳌﻄﻠﻮب ﻫﻲ اﲢﺎد اﳌﺴﺘﻘﻴﻢ ‪ Δ‬ﻣﻊ اﻟﺪاﺋﺮة 𝒞 )ﻻ ﻧﺴﺘﺜﻨﻲ اﻟﻨﻘﻄﺔ )‪ A (1‬ﻷﳖﺎ ﲢﻘﻖ‬
‫اﳌﻄﻠﻮب أﻳﻀ ًﺎ و درﺳﻨﺎ اﳊﺎﻟﺔ اﳋﺎﺻﺔ ‪ M = A‬ﰲ اﻟﺒﺪاﻳﺔ( ‪ .‬أﻧﻈﺮ اﻟﺸﻜﻞ ‪.2.VI‬‬
‫ﻓﺈن ‪ M = B‬و ﺑﺎﻟﺘﺎﱄ اﻟﻨﻘﻂ ‪ M ، M ، B‬ﻋﲆ اﺳﺘﻘﺎﻣﺔ واﺣﺪة‪ .‬ﻧﻔﺮض إذن ّ‬
‫أن ‪.z ≠ ı‬‬ ‫‪ •2‬إذا ﻛﺎن ‪ّ z = ı‬‬
‫‪ız − ı‬‬
‫‪) Im‬ﺑﻨﻔﺲ اﻟﻄﺮﻳﻘﺔ‬ ‫ﺗﻜﻮن اﻟﻨﻘﻂ )‪ M (ız) ، M (z) ، B (ı‬ﻋﲆ اﺳﺘﻘﺎﻣﺔ واﺣﺪة إذا و ﻓﻘﻂ إذا ﻛﺎن ‪= 0‬‬
‫‪z−ı‬‬
‫اﻟﺴﺎﺑﻘﺔ( ‪ .‬ﻟﻜﻦ ‪:‬‬
‫‪ız − ı‬‬ ‫‪z−1‬‬ ‫‪x − 1 + ıy‬‬ ‫))‪(x − 1 + ıy) (x − ı (y − 1‬‬
‫‪=ı‬‬ ‫‪=ı‬‬ ‫‪=ı‬‬
‫‪z−ı‬‬ ‫‪z−ı‬‬ ‫)‪x + ı (y − 1‬‬ ‫)‪x + (y − 1‬‬
‫)‪x − ıx (y − 1) − x + ı (y − 1) + ıxy + y (y − 1‬‬
‫‪ņőƱŀ Ţ‬‬
‫‪=ı‬‬
‫)‪x + (y − 1‬‬
‫‪−x − y + 1‬‬ ‫‪x +y −x−y‬‬
‫=‬ ‫‪+ı‬‬
‫)‪x + (y − 1‬‬ ‫)‪x + (y − 1‬‬

‫‪VI‬‬
‫‪ız − ı‬‬ ‫‪x +y −x−y‬‬
‫‪Im‬‬ ‫⟺ ‪=0‬‬ ‫‪=0 ⟺ x +y −x−y=0‬‬ ‫ﻣﻨﻪ ‪:‬‬
‫‪z−ı‬‬ ‫)‪x + (y − 1‬‬

‫‪1‬‬ ‫‪1‬‬ ‫‪1‬‬ ‫‪√2‬‬


‫⟺‬ ‫‪x−‬‬ ‫‪+ y−‬‬ ‫=‬ ‫=‬
‫‪2‬‬ ‫‪2‬‬ ‫‪2‬‬ ‫‪2‬‬
‫‪ƕ‬‬

‫‪ Ω‬و ﻧﺼﻒ ﻗﻄﺮﻫﺎ ‪ √2‬ﺑﺎﺳﺘﺜﻨﺎء اﻟﻨﻘﻄﺔ )‪ّ B (ı‬‬


‫)ﻷن‬
‫‪1 1‬‬
‫‪,‬‬ ‫𝒞 اﻟﺘﻲ ﻣﺮﻛﺰﻫﺎ‬ ‫‪Ω,‬‬
‫‪√2‬‬
‫و ﻫﻲ ﻣﻌﺎدﻟﺔ اﻟﺪاﺋﺮة‬
‫‪2‬‬ ‫‪2 2‬‬ ‫‪2‬‬
‫‪. (z ≠ ı‬‬
‫‪ũŏ‬‬

‫ﰲ اﻷﺧﲑ‪ ،‬ﳎﻤﻮﻋﺔ اﻟﻨﻘﻂ اﻟﺘﻲ ﲢﻘﻖ اﳌﻄﻠﻮب ﻫﻲ اﻟﺪاﺋﺮة 𝒞 )أﻧﻈﺮ اﻟﺸﻜﻞ ‪. (2.VI‬‬
‫‪/‬‬ ‫𝜋‬
‫‪. (ı = e‬‬ ‫ّ‬
‫)ﻷن‬ ‫ﳎﻤﻮﻋﺔ اﻟﻨﻘﻂ ‪ : M‬اﻟﻨﻘﻄﺔ ‪ M‬ﻫﻲ ﺻﻮرة ‪ M‬ﺑﺎﻟﺪوران اﻟﺬي ﻣﺮﻛﺰه ‪ O‬و زاوﻳﺘﻪ‬
‫‪2‬‬
‫‪1 1‬‬ ‫‪1 1‬‬ ‫‪1 1‬‬ ‫‪1 1‬‬
‫‪ (ı‬إذن ﺻﻮرة اﻟﺪاﺋﺮة‬ ‫‪ّ Ω‬‬
‫)ﻷن ‪+ ı = − + ı‬‬ ‫‪− ,‬‬ ‫‪ Ω‬ﲠﺬا اﻟﺪوران ﻫﻲ اﻟﻨﻘﻄﺔ‬ ‫‪,‬‬ ‫ﺻﻮرة‬
‫‪2 2‬‬ ‫‪2 2‬‬ ‫‪2 2‬‬ ‫‪2 2‬‬
‫‪√2‬‬ ‫‪√2‬‬
‫𝒞‪.‬‬ ‫‪Ω ,‬‬ ‫𝒞 ﻫﻲ اﻟﺪاﺋﺮة‬ ‫‪Ω,‬‬
‫‪2‬‬ ‫‪2‬‬

‫𝟲𝟴‬

‫‪http ://tinyurl.com/Malki1718‬‬ ‫‪0‬‬


‫‪ .VI‬ﲤﺎرﻳﻦ ﺗﻄﺒﻴﻘﻴﺔ‬
‫‪.‬‬
‫‪C′‬‬
‫‪1‬‬ ‫‪C‬‬

‫‪Ŕž Ŧ Ľ‬‬
‫‪B‬‬

‫‪Ω′‬‬

‫‪œǃ‬‬
‫‪A‬‬
‫‪1‬‬ ‫∆‬ ‫‪2‬‬

‫‪Ŀ‬‬
‫‪−1‬‬

‫ﺷﻜﻞ ‪2.VI‬‬

‫‪2‬‬ ‫‪1 1‬‬


‫‪ Ω‬و ﻧﺼﻒ ﻗﻄﺮﻫﺎ √ ‪.‬‬ ‫‪− ,‬‬ ‫إذن ﳎﻤﻮﻋﺔ اﻟﻨﻘﻂ ‪ M‬ﻫﻲ اﻟﺪاﺋﺮة اﻟﺘﻲ ﻣﺮﻛﺰﻫﺎ‬
‫‪2‬‬ ‫‪2 2‬‬
‫■‬

‫‪.‬‬
‫𝟎𝟐‬ ‫‪.‬‬
‫‪1‬‬
‫و ‪ z − 1‬ﻧﻔﺲ اﻟﻄﻮﻳﻠﺔ‪.‬‬ ‫ﻋﲔ اﻷﻋﺪاد اﳌﺮﻛﺒﺔ ‪ z‬اﻟﺘﻲ ﻣﻦ أﺟﻠﻬﺎ ﻳﻜﻮن ﻟﻸﻋﺪاد ‪، z‬‬
‫ّ‬
‫‪z‬‬

‫ﺍﳊﻞّ‪ .‬ﻟﺪﻳﻨﺎ ‪:‬‬

‫|‪|1‬‬ ‫⎧‬ ‫‪1‬‬


‫|‪= |z‬‬ ‫‪|z| = 1‬‬
‫= |‪|z‬‬ ‫|‬ ‫|‬ ‫|‬ ‫|‬
‫⟺ |‪= |1 − z‬‬ ‫|‪|z‬‬ ‫⟺‬
‫|| ‪|| z‬‬ ‫⎨‬ ‫|‪||1 − z|| = |z‬‬
‫|⎩‬‫|‬‫‪1‬‬ ‫‪−‬‬ ‫‪z‬‬ ‫|‬
‫|‬ ‫=‬ ‫|‪|z‬‬
‫⎧‬
‫⎪‬ ‫‪x +y =1‬‬ ‫‪x +y =1‬‬
‫‪ņőƱŀ Ţ‬‬
‫⟺‬ ‫⟺‬
‫⎨‬
‫⎪‬ ‫‪(x − 1) + y = x + y‬‬
‫⎩‬ ‫= ‪(x − 1) + y‬‬ ‫‪x +y‬‬
‫‪y =1−x‬‬ ‫‪y =1−x‬‬
‫⟺‬ ‫⟺‬
‫‪VI‬‬

‫‪x − 2x + 1 + y = x + y‬‬ ‫‪2x + 1 = 0‬‬


‫⎧‬
‫⎪‬ ‫‪1‬‬ ‫‪3‬‬ ‫⎧‬
‫⎪‬ ‫‪√3‬‬
‫‪⎪y = 1 −‬‬ ‫=‬ ‫‪⎪y = ±‬‬
‫⟺‬ ‫‪2‬‬ ‫⟺ ‪4‬‬ ‫‪2‬‬
‫⎨‬
‫⎪‬ ‫⎨‬
‫⎪‬ ‫‪1‬‬
‫‪⎪ x= 1‬‬ ‫=‪⎪x‬‬
‫⎩‬ ‫‪2‬‬ ‫⎩‬ ‫‪2‬‬
‫‪ƕ‬‬

‫‪1 √3‬‬ ‫‪1 √3‬‬


‫=‪z‬‬ ‫‪+‬‬ ‫‪ z = −‬و ‪ı = e / = −j‬‬ ‫إذن ﻳﻮﺟﺪ ﻋﺪدان ﻣﺮﻛﺒﺎن ﳛﻘﻘﺎن اﳌﻄﻠﻮب و ﳘﺎ ‪ı = e− / = −j :‬‬
‫‪2‬‬ ‫‪2‬‬ ‫‪2‬‬ ‫‪2‬‬
‫‪ũŏ‬‬

‫‪ .‬ﻫﺬان اﻟﻌﺪدان ُﻳﻤﺜﻼن ﻧﻘﻄﺘ َْﻲ ﺗﻘﺎﻃﻊ اﻟﺪاﺋﺮة اﻟﺘﻲ ﻣﺮﻛﺰﻫﺎ )‪ O (0‬و ﻧﺼﻒ ﻗﻄﺮﻫﺎ ‪) 1‬أي اﻟﺘﻲ ﻣﻌﺎدﻟﺘﻬﺎ ‪ (|z| = 1‬ﻣﻊ اﻟﺪاﺋﺮة‬
‫اﻟﺘﻲ ﻣﺮﻛﺰﻫﺎ )‪ A (1‬و ﻧﺼﻒ ﻗﻄﺮﻫﺎ ‪) 1‬و اﻟﺘﻲ ﻣﻌﺎدﻟﺘﻬﺎ ‪. (||z − 1|| = 1‬‬
‫■‬ ‫‪. 𝒮 = −j, −j‬‬ ‫ﰲ اﻷﺧﲑ‪ ،‬ﳎﻤﻮﻋﺔ اﳊﻠﻮل ﻫﻲ ‪:‬‬

‫𝟳𝟴‬

‫‪http ://tinyurl.com/Malki1718‬‬ ‫‪0‬‬


‫‪ .1.VI‬ﶍﻮﻋﺔ اﻷﻋﺪاد اﳌﺮﻛﺒﺔ‬
‫‪.‬‬

‫‪𝟐𝟏.‬‬ ‫‪.‬‬

‫‪Ŕž Ŧ Ľ‬‬
‫ﻣﺎ ﻫﻮ اﻟﴩط اﻟﻼزم و اﻟﻜﺎﰲ اﻟﺬي ﳚﺐ أن ﳛﻘﻘﻪ اﻟﻌﺪد اﳌﺮﻛﺐ ‪ z‬ﺣﺘﻰ ﺗﻜﻮن اﻟﻨﻘﻂ ‪ C ، B ، A‬اﻟﺘﻲ‬
‫ﻟﻮاﺣﻘﻬﺎ ‪ z ، z ، z‬ﻋﲆ اﺳﺘﻘﺎﻣﺔ واﺣﺪة ؟‬

‫‪œǃ‬‬
‫ﺍﳊﻞّ‪ .‬ﻧُﻤ ّﻴﺰ ﺣﺎﻟﺘﲔ ‪ :‬إ ّﻣﺎ أن ﺗﻜﻮن اﻟﻨﻘﻂ ‪ B ، A‬و ‪ C‬ﳐﺘﻠﻔﺔ ﻣﺜﻨﻰ ﻣﺜﻨﻰ أو أن ﺗﻜﻮن ﻧﻘﻄﺘﺎن )ﻋﲆ اﻷﻗﻞ( ﻣﺘﻄﺎﺑﻘﺘﲔ‪.‬‬
‫ﺗﻜﻮن ﻧﻘﻄﺘﺎن )ﻋﲆ اﻷﻗﻞ( ﻣﺘﻄﺎﺑﻘﺘﲔ إذا ‪:‬‬

‫‪Ŀ‬‬
‫•‬

‫➛ ﻛﺎن ‪ A = B‬أي ‪ z = z‬أي ‪ z (z − 1) = 0‬أي }‪ z ∈ {0, 1‬؛‬


‫➛ أو ‪ A = C‬أي ‪ z = z‬أي ‪ z z − 1 = 0‬أي ‪ z ∈ 0, 1, j, j‬؛‬
‫➛ أو ‪ B = C‬أي ‪ z = z‬أي ‪ z z − 1 = 0‬أي }‪ z ∈ {0, 1, −1‬؛‬
‫➛ أو ‪ A = B = C‬أي ‪ z = z = z‬أي }‪ z ∈ {0, 1} ∩ {0, 1, −1‬أي }‪ z ∈ {0, 1‬؛‬

‫‪ z ∈ 0, 1, −1, j, j‬و ﰲ ﻫﺬه اﳊﺎﻟﺔ ﻓﻤﻦ اﻟﺒﺪﳞﻲ أﳖﺎ ﻋﲆ اﺳﺘﻘﺎﻣﺔ واﺣﺪة‪.‬‬ ‫أي إذا ﻛﺎن‬

‫‪ z ∉ 0, 1, −1, j, j‬و ﺣﺴﺐ اﳌﱪﻫﻨﺔ ‪ 1‬ﺻﻔﺤﺔ ‪ّ 53‬‬


‫ﻓﺈن ‪:‬‬ ‫إذا ﻛﺎﻧﺖ اﻟﻨﻘﻂ ‪ B ، A‬و ‪ C‬ﳐﺘﻠﻔﺔ ﻣﺜﻨﻰ ﻣﺜﻨﻰ ّ‬
‫ﻓﺈن‬ ‫•‬

‫‪z −z‬‬
‫⟺ اﻟﻨﻘﻂ ‪ B ، A‬و ‪ C‬ﻋﲆ اﺳﺘﻘﺎﻣﺔ واﺣﺪة‬ ‫‪∈ℝ ⟺ z +z∈ℝ‬‬
‫‪z −z‬‬
‫‪1‬‬ ‫‪1‬‬ ‫‪1‬‬
‫⟺‬ ‫‪z+‬‬ ‫‪− ∈ℝ ⟺ z+‬‬ ‫‪∈ℝ‬‬
‫‪2‬‬ ‫‪4‬‬ ‫‪2‬‬
‫‪1‬‬ ‫‪1‬‬
‫⟺‬ ‫‪ z + ∈ ıℝ‬أو ‪z + ∈ ℝ‬‬
‫‪2‬‬ ‫‪2‬‬
‫‪1‬‬
‫⟺‬ ‫أو ‪z ∈ ℝ‬‬ ‫‪Re z +‬‬ ‫‪=0‬‬
‫‪2‬‬
‫‪1‬‬
‫⟺‬ ‫أو ‪z ∈ ℝ‬‬ ‫‪Re (z) = −‬‬
‫‪ņőƱŀ Ţ‬‬
‫‪2‬‬

‫‪1‬‬
‫‪Re (z) = −‬‬ ‫ﰲ اﻷﺧﲑ‪ ،‬ﺗﻜﻮن اﻟﻨﻘﻂ ‪ B ، A‬و ‪ C‬ﻋﲆ اﺳﺘﻘﺎﻣﺔ واﺣﺪة إذا و ﻓﻘﻂ إذا ﻛﺎن ‪) z ∈ ℝ‬و ﻫﺬا ﻳﺸﻤﻞ ‪ 1 ، 0‬و ‪ (−1‬أو‬
‫‪2‬‬
‫■‬ ‫)و ﻫﺬا ﻳﺸﻤﻞ ‪ j‬و ‪. ( j‬‬

‫‪VI‬‬
‫ﻟﻴﻜﻦ ‪ p‬و ‪ q‬ﻋﺪدﻳﻦ ﻣﺮﻛﺒﲔ ‪ .‬ﻧﻌﺘﱪ اﳌﻌﺎدﻟﺔ اﻟﺘﺎﻟﻴﺔ ذات اﳌﺠﻬﻮل ‪: z ∈ ℂ‬‬
‫!‬ ‫‪𝟐𝟐.‬‬ ‫‪.‬‬

‫‪z + pz + q = 0‬‬ ‫)𝟏(‬


‫‪ƕ‬‬

‫ﻟﺘﻜﻦ 𝛼 ‪ 𝛾 ، 𝛽 ،‬ﺣﻠﻮل ﻫﺬه اﳌﻌﺎدﻟﺔ‪.‬‬

‫‪ •2‬ﻣﺎ ﻫﻮ اﻟﴩط اﻟﻼزم و اﻟﻜﺎﰲ اﻟﺬي ﳚﺐ أن ُﲢﻘﻘﻪ‬ ‫⎧‬


‫⎪‬ ‫‪𝛼+𝛽+𝛾=0‬‬
‫اﻷﻋﺪاد ‪ p‬و ‪ q‬ﺣﺘﻰ ﺗﻜﻮن ُﺻ َﻮر ﺣﻠﻮل اﳌﻌﺎدﻟﺔ )𝟏(‬ ‫‪. 𝛼𝛽 + 𝛼𝛾 + 𝛽𝛾 = p‬‬ ‫‪ •1‬أﺛﺒﺖ ّ‬
‫أن ‪:‬‬
‫⎨‬
‫‪ũŏ‬‬

‫⎪‬
‫رؤوس ﻣﺜﻠﺚ ﻗﺎﺋﻢ و ﻣﺘﺴﺎوي اﻟﺴﺎﻗﲔ ؟‬ ‫⎩‬ ‫‪𝛼𝛽𝛾 = −q‬‬

‫‪ •1‬ﺣﻠﻮل اﳌﻌﺎدﻟﺔ )𝟏( ﻫﻲ ﺟﺬور ﻛﺜﲑ اﳊﺪود ‪ z + pz + q‬إذن ‪:‬‬ ‫ﺍﳊﻞّ‪.‬‬


‫𝛾𝛽𝛼 ‪z + pz + q = (z − 𝛼) (z − 𝛽) (z − 𝛾) = z + (𝛼 + 𝛽 + 𝛾) + (𝛼𝛽 + 𝛼𝛾 + 𝛽𝛾) z −‬‬
‫ﺑﺎﳌﻄﺎﺑﻘﺔ ﻧﺴﺘﻨﺘﺞ ّ‬
‫أن ‪ 𝛼𝛽 + 𝛼𝛾 + 𝛽𝛾 = p ، 𝛼 + 𝛽 + 𝛾 = 0‬و ‪. 𝛼𝛽𝛾 = −q‬‬

‫𝟴𝟴‬

‫‪http ://tinyurl.com/Malki1718‬‬ ‫‪0‬‬


‫‪ .VI‬ﲤﺎرﻳﻦ ﺗﻄﺒﻴﻘﻴﺔ‬
‫‪.‬‬
‫‪ •2‬ﺣﺘﻰ ﺗﻜﻮن اﻟﻨﻘﻂ ‪ B ، A‬و ‪ C‬رؤوس ﻣﺜﻠﺚ ﻗﺎﺋﻢ ﰲ ‪ B‬و ﻣﺘﺴﺎوي اﻟﺴﺎﻗﲔ ﻳﻜﻔﻲ أن ﻳﻜﻮن ‪ BA‬ﺻﻮرة ‪ BC‬ﺑﺎﻟﺪوران‬
‫𝜋‬
‫اﻟﺬي ﻣﺮﻛﺰه ‪ B‬و زاوﻳﺘﻪ ‪.‬‬

‫‪Ŕž Ŧ Ľ‬‬
‫‪2‬‬
‫‪ 𝛾 − 𝛼 = e‬أي )𝛼 ‪ 𝛾 − 𝛼 = ı (𝛽 −‬أي 𝛽‪ 𝛾 = (1 − ı) 𝛼 + ı‬ﻣﻨﻪ ‪:‬‬ ‫‪/‬‬
‫إذن ﻓﺎﳊﻠﻮل 𝛼 ‪ُ 𝛾 ، 𝛽 ،‬ﲢﻘﻖ )𝛼 ‪(𝛽 −‬‬

‫‪œǃ‬‬
‫⎧‬ ‫𝛽‪𝛾 = (1 − ı) 𝛼 + ı‬‬
‫⎧‬ ‫𝛽‪𝛾 = (1 − ı) 𝛼 + ı‬‬ ‫⎧‬ ‫𝛽‪𝛾 = (1 − ı) 𝛼 + ı‬‬ ‫⎪‬
‫⎪‬
‫⎪‬
‫⎪‬ ‫⎪‬
‫⎪‬ ‫⎪‬
‫⎪‬
‫⎪‬ ‫‪𝛼+𝛽+𝛾=0‬‬ ‫‪⎪ (2 − ı) 𝛼 + (1 + ı) 𝛽 = 0‬‬ ‫𝛼 ‪⎪ 𝛽 = − 2 − ı 𝛼 = −1 + 3ı‬‬
‫⟺‬ ‫⟺‬ ‫‪1+ı‬‬ ‫‪2‬‬
‫⎨‬
‫⎪‬ ‫‪𝛼𝛽 + 𝛼𝛾 + 𝛽𝛾 = p‬‬ ‫⎨‬
‫⎪‬ ‫‪𝛼 (𝛽 + 𝛾) + 𝛽𝛾 = p‬‬ ‫⎨‬
‫⎪‬
‫⎪‬ ‫⎪‬ ‫⎪‬ ‫𝛼‪−‬‬ ‫‪+‬‬ ‫𝛾𝛽‬ ‫‪=p‬‬

‫‪Ŀ‬‬
‫⎪‬ ‫⎪‬ ‫⎪‬
‫⎪‬
‫⎩‬ ‫‪𝛼𝛽𝛾 = −q‬‬ ‫⎩‬ ‫‪𝛼𝛽𝛾 = −q‬‬ ‫⎪‬
‫⎩‬ ‫‪𝛼𝛽𝛾 = −q‬‬
‫⎧‬ ‫‪−1 + 3ı‬‬ ‫‪−1 − 3ı‬‬
‫⎪‬
‫⎪‬ ‫‪𝛾 = (1 − ı) 𝛼 + ı‬‬ ‫𝛼‬ ‫=‬ ‫𝛼‬
‫⎪‬
‫⎪‬ ‫‪2‬‬ ‫‪2‬‬
‫⎪‬
‫⎪‬
‫⎪‬ ‫‪−1 + 3ı‬‬
‫⟺‬ ‫=𝛽‬ ‫𝛼‬
‫⎨‬
‫⎪‬ ‫‪2‬‬
‫⎪‬
‫⎪‬
‫⎪‬
‫⎪‬ ‫‪−𝛼 + 𝛽𝛾 = p‬‬
‫⎪‬
‫⎪‬
‫⎩‬ ‫‪𝛼𝛽𝛾 = −q‬‬
‫⎧‬ ‫‪−1 − 3ı‬‬ ‫‪−1 − 3ı‬‬
‫⎪‬
‫⎪‬ ‫=𝛾‬ ‫𝛼‬ ‫⎧‬
‫⎪‬ ‫=𝛾‬ ‫𝛼‬
‫⎪‬
‫⎪‬ ‫‪2‬‬ ‫⎪‬
‫⎪‬ ‫‪2‬‬
‫⎪‬
‫⎪‬ ‫⎪‬
‫⎪‬
‫⎪‬
‫⎪‬ ‫‪−1 + 3ı‬‬ ‫⎪‬
‫⎪‬ ‫‪−1 + 3ı‬‬
‫⎪‬
‫⎪‬ ‫=𝛽‬ ‫𝛼‬ ‫⎪‬
‫= 𝛽⎪‬ ‫𝛼‬
‫‪2‬‬ ‫‪2‬‬
‫⟺‬ ‫‪−1 − 3ı‬‬ ‫‪−1 + 3ı‬‬ ‫⟺‬
‫⎨‬
‫⎪‬ ‫‪−𝛼 +‬‬ ‫𝛼‬ ‫‪𝛼 =p‬‬ ‫⎨‬
‫⎪‬ ‫‪3‬‬
‫⎪‬
‫⎪‬ ‫‪2‬‬ ‫‪2‬‬ ‫⎪‬
‫⎪‬ ‫‪𝛼 =p‬‬
‫⎪‬
‫⎪‬ ‫⎪‬
‫⎪‬ ‫‪2‬‬
‫⎪‬
‫⎪‬ ‫⎪‬
‫⎪‬
‫⎪‬
‫⎪‬ ‫‪−1 − 3ı‬‬ ‫‪−1 + 3ı‬‬ ‫⎪‬
‫⎪‬ ‫‪5‬‬
‫⎪‬ ‫𝛼‬ ‫𝛼‬ ‫‪𝛼 = −q‬‬ ‫‪𝛼 = −q‬‬
‫⎩‬ ‫‪2‬‬ ‫‪2‬‬ ‫⎩‬ ‫‪2‬‬
‫‪−1 − 3ı‬‬ ‫⎧‬ ‫‪−1 − 3ı‬‬
‫⎧‬
‫⎪‬ ‫=𝛾‬ ‫𝛼‬ ‫⎪‬
‫⎪‬ ‫=𝛾‬ ‫𝛼‬
‫⎪‬
‫⎪‬ ‫‪2‬‬ ‫⎪‬
‫⎪‬ ‫‪2‬‬
‫⎪‬
‫⎪‬ ‫⎪‬
‫⎪‬
‫⎪‬
‫⎪‬ ‫‪−1 + 3ı‬‬ ‫⎪‬
‫⎪‬ ‫‪−1 + 3ı‬‬
‫⎪‬ ‫⎪‬
‫⎪‬ ‫=𝛽‬ ‫𝛼‬
‫= 𝛽⎪‬ ‫𝛼‬ ‫⎪‬ ‫‪2‬‬
‫⟺‬ ‫‪2‬‬ ‫⟺‬
‫⎨‬ ‫‪2‬‬ ‫⎨‬ ‫‪2‬‬
‫‪ņőƱŀ Ţ‬‬
‫⎪‬
‫⎪‬ ‫‪𝛼 = p‬‬ ‫⎪‬
‫⎪‬ ‫‪𝛼 =− q‬‬
‫⎪‬
‫⎪‬ ‫⎪‬
‫⎪‬ ‫‪5‬‬
‫⎪‬
‫⎪‬ ‫‪3‬‬ ‫⎪‬
‫⎪‬
‫⎪‬
‫⎪‬ ‫‪2‬‬ ‫⎪‬
‫⎪‬
‫⎪‬ ‫⎪‬
‫⎪‬ ‫‪2‬‬ ‫‪2‬‬
‫‪𝛼 =− q‬‬ ‫‪⎪𝛼 = − q‬‬ ‫=‬ ‫‪p‬‬
‫⎩‬ ‫‪5‬‬ ‫⎩‬ ‫‪5‬‬ ‫‪3‬‬
‫‪VI‬‬

‫إذن اﻟﴩط اﻟﻼزم و اﻟﻜﺎﰲ اﻟﺬي ﳚﺐ أن ُﲢﻘﻘﻪ اﻷﻋﺪاد ‪ p‬و ‪ q‬ﺣﺘﻰ ﺗﻜﻮن ﺻﻮر 𝛼 ‪ 𝛽 ،‬و 𝛾 رؤوس ﻣﺜﻠﺚ ﻗﺎﺋﻢ و‬
‫‪2‬‬ ‫‪2‬‬
‫‪ − q‬أي ‪. 50p = 27q‬‬ ‫=‬ ‫‪p‬‬ ‫ﻣﺘﺴﺎوي اﻟﺴﺎﻗﲔ ﻫﻮ‬
‫‪5‬‬ ‫‪3‬‬
‫■‬
‫‪ƕ‬‬

‫ﻟﺘﻜﻦ ‪ b ، a‬و ‪ c‬ﺛﻼﺛﺔ أﻋﺪاد ﻣﺮﻛﺒﺔ ‪ .‬ﻧﻌﺘﱪ اﳌﻌﺎدﻟﺔ اﻟﺘﺎﻟﻴﺔ ذات اﳌﺠﻬﻮل ‪: z ∈ ℂ‬‬
‫‬ ‫‪.‬‬
‫𝟑𝟐‬ ‫‪.‬‬

‫‪z + az + bz + c = 0‬‬ ‫)𝟐(‬


‫‪ũŏ‬‬

‫ﻟﺘﻜﻦ 𝛼 ‪ 𝛿 ، 𝛾 ، 𝛽 ،‬ﺣﻠﻮل ﻫﺬه اﳌﻌﺎدﻟﺔ‪.‬‬


‫‪ •1‬أﺛﺒﺖ ّ‬
‫أن ‪.𝛼 + 𝛽 + 𝛾 + 𝛿 = 0 :‬‬
‫‪ •2‬ﻣﺎ ﻫﻮ اﻟﴩط اﻟﻼزم و اﻟﻜﺎﰲ اﻟﺬي ﳚﺐ أن ُﲢﻘﻘﻪ اﻷﻋﺪاد ‪ b ، a‬و ‪ c‬ﺣﺘﻰ ﺗﻜﻮن ُﺻ َﻮر ﺣﻠﻮل اﳌﻌﺎدﻟﺔ )𝟐( رؤوس‬
‫ﻣﺮﺑﻊ ؟‬

‫𝟵𝟴‬

‫‪http ://tinyurl.com/Malki1718‬‬ ‫‪0‬‬


‫‪ .1.VI‬ﶍﻮﻋﺔ اﻷﻋﺪاد اﳌﺮﻛﺒﺔ‬
‫‪.‬‬
‫‪ •1‬ﺣﻠﻮل اﳌﻌﺎدﻟﺔ )𝟐( ﻫﻲ ﺟﺬور ﻛﺜﲑ اﳊﺪود ‪ P (z) = z + az + bz + c‬إذن ‪:‬‬ ‫ﺍﳊﻞّ‪.‬‬

‫‪Ŕž Ŧ Ľ‬‬
‫)𝛿 ‪z + az + bz + c = (z − 𝛼) (z − 𝛽) (z − 𝛾) (z −‬‬
‫‪= z − (𝛼 + 𝛽 + 𝛾 + 𝛿) z + (𝛼𝛽 + 𝛼𝛾 + 𝛼𝛿 + 𝛽𝛾 + 𝛽𝛿 + 𝛾𝛿) z‬‬

‫‪œǃ‬‬
‫𝛿𝛾𝛽𝛼 ‪− (𝛼𝛽𝛾 + 𝛼𝛽𝛿 + 𝛼𝛾𝛿 + 𝛽𝛾𝛿) z +‬‬

‫ﺑﺎﳌﻄﺎﺑﻘﺔ ﺑﲔ ﻣﻌﺎﻣﻞ ‪ z‬ﰲ اﻟﻄﺮﻓﲔ‪ ،‬ﻧﺴﺘﻨﺘﺞ ّ‬


‫أن ‪ − (𝛼 + 𝛽 + 𝛾 + 𝛿) = 0‬أي ‪. 𝛼 + 𝛽 + 𝛾 + 𝛿 = 0‬‬

‫‪Ŀ‬‬
‫‪ •2‬إذا ﻛﺎﻧﺖ ﺻﻮر اﳊﻠﻮل 𝛼 ‪ 𝛿 ، 𝛾 ، 𝛽 ،‬رؤوس ﻣﺮﺑﻊ ّ‬
‫ﻓﺈن ﻻﺣﻘﺔ ﻣﺮﻛﺰ ﻫﺬا اﳌﺮﺑﻊ )اﻟﺬي ﻫﻮ ﻧﻘﻄﺔ ﺗﻘﺎﻃﻊ ﻗﻄﺮﻳﻪ( ﻫﻲ‬
‫‪1‬‬
‫أي رأس ﳍﺬا اﳌﺮﺑﻊ ﻫﻮ ﺻﻮرة اﻟﺮأس‬ ‫‪ (𝛼 + 𝛽 + 𝛾 + 𝛿) = 0‬إذن ﻣﺮﻛﺰ ﻫﺬا اﳌﺮﺑﻊ ﻫﻮ ‪ O‬ﻣﺒﺪأ اﳌﻌﻠﻢ‪ .‬و ﺑﺎﻟﺘﺎﱄ ّ‬
‫ﻓﺈن ّ‬ ‫‪4‬‬
‫𝜋‬
‫اﳌﺠﺎور )ﰲ اﻹﲡﺎه اﳌﺜﻠﺜﻲ أي ﻋﻜﺲ ﻋﻘﺎرب اﻟﺴﺎﻋﺔ( ﺑﺎﻟﺪوران اﻟﺬي ﻣﺮﻛﺰه ‪ O‬و زاوﻳﺘﻪ ‪.‬‬
‫‪2‬‬
‫‪ 𝛽 − 0 = e‬أي 𝛼‪ . 𝛽 = ı‬ﺑﺎﳌﺜﻞ‪ 𝛾 = ı𝛽 = −𝛼 ،‬و 𝛼‪ 𝛿 = ı𝛾 = −ı‬إذن اﳉﺬور ﻫﻲ 𝛼 ‪−𝛼 ، ı𝛼 ،‬‬ ‫‪/‬‬
‫إذن )‪(𝛼 − 0‬‬
‫و 𝛼‪ −ı‬و ﺑﺎﻟﺘﺎﱄ ّ‬
‫ﻓﺈن ‪:‬‬
‫𝛼 ‪z + az + bz + c = (z − 𝛼) (z − ı𝛼) (z + 𝛼) (z + ı𝛼) = z −‬‬ ‫𝛼‪z +‬‬ ‫𝛼‪=z −‬‬
‫ﺑﺎﳌﻄﺎﺑﻘﺔ‪ ،‬ﻧﺴﺘﻨﺘﺞ ّ‬
‫أن ‪ b = 0 ، a = 0‬و 𝛼‪. c = −‬‬
‫ﻓﺈن ‪ P (z) = z + c‬إذن ﺟﺬور ‪ P‬ﻫﻲ اﳉﺬور اﻟﺮاﺑﻌﺔ ﻟﻠﻌﺪد ‪ −c‬و ﻧﻌﻠﻢ ّ‬
‫أن ﺻﻮر‬ ‫ﺑﺎﻟﻌﻜﺲ إذا ﻛﺎن ‪ a = 0‬و ‪ّ b = 0‬‬
‫اﳉﺬور اﻟﺮاﺑﻌﺔ ﻟﻌﺪد ﻣﺮﻛﺐ ﻏﲑ ﻣﻌﺪوم ﻫﻲ رؤوس ﻣﺮﺑﻊ ﻣﺮﻛﺰه ‪. O‬‬
‫ﰲ اﻷﺧﲑ‪ ،‬اﻟﴩط اﻟﺬي ﻧﺒﺤﺚ ﻋﻨﻪ ﻫﻮ ‪. a = b = 0 :‬‬
‫■‬

‫‪𝟐𝟒.‬‬ ‫‪.‬‬
‫ﻟﻴﻜﻦ ‪ z‬ﻋﺪد ًا ﻣﺮﻛﺒﺎ و ‪ C ، B ، A‬ﺛﻼث ﻧﻘﻂ ﻣﻦ اﳌﺴﺘﻮي ﳐﺘﻠﻔﺔ ﻣﺜﻨﻰ ﻣﺜﻨﻰ ﻟﻮاﺣﻘﻬﺎ ‪ z ، z ، z‬ﻋﲆ‬
‫اﻟﱰﺗﻴﺐ‪.‬‬
‫ﻣﺎ ﻫﻮ اﻟﴩط اﻟﻼزم و اﻟﻜﺎﰲ ﺣﺘﻰ ﻳﻜﻮن اﳌﺒﺪأ ‪ O‬ﻫﻮ ﻧﻘﻄﺔ ﺗﻘﺎﻃﻊ أﻋﻤﺪة اﳌﺜﻠﺚ ‪ ABC‬؟‬
‫‪ņőƱŀ Ţ‬‬
‫ﺍﳊﻞّ‪ .‬ﺑﲈ ّ‬
‫أن اﻟﻨﻘﻂ ‪ B ، A‬و ‪ C‬ﳐﺘﻠﻔﺔ ﻣﺜﻨﻰ ﻣﺜﻨﻰ ّ‬
‫ﻓﺈن ‪ z ≠ z‬و ‪ z ≠ z‬و ‪ z ≠ z‬أي }‪. z ∉ {0, 1, −1‬‬
‫ﻧﻀﻊ ‪ z = x + ıy‬ﻣﻊ ‪ . x, y ∈ ℝ‬ﺣﺴﺐ اﻟﻼزﻣﺔ ‪ 1‬ﺻﻔﺤﺔ ‪: 53‬‬

‫)‪ O ⟺ (OA) ⟂ (BC‬ﻧﻘﻄﺔ ﺗﻘﺎﻃﻊ أﻋﻤﺪة ‪ABC‬‬ ‫و‬ ‫)‪(OB) ⟂ (AC‬‬ ‫‪VI‬‬
‫‪z −z‬‬ ‫‪z −z‬‬
‫⟺‬ ‫‪∈ ıℝ‬‬ ‫و‬ ‫‪∈ ıℝ‬‬
‫‪z−0‬‬ ‫‪z‬‬
‫‪ƕ‬‬

‫‪z −1‬‬ ‫‪z −1‬‬


‫‪⟺ Re z − z = 0‬‬ ‫و‬ ‫‪=−‬‬
‫‪z‬‬ ‫‪z‬‬
‫‪z −1‬‬ ‫‪z −1‬‬
‫‪⟺ Re‬‬ ‫‪x −x−y‬‬ ‫‪+ ıy (2x − 1) = 0‬‬ ‫و‬ ‫‪=−‬‬
‫‪z‬‬ ‫‪z‬‬
‫‪ũŏ‬‬

‫‪⟺ x −x−y =0‬‬ ‫و‬ ‫‪z z −1 +z z −1 =0‬‬

‫‪1‬‬ ‫‪1‬‬
‫⟺‬ ‫‪x−‬‬ ‫= ‪−y‬‬ ‫و‬ ‫‪z |z| − z + z |z| − z = 0‬‬
‫‪2‬‬ ‫‪4‬‬
‫‪1‬‬ ‫‪1‬‬
‫⟺‬ ‫‪x−‬‬ ‫= ‪−y‬‬ ‫و‬ ‫‪(z + z) |z| − 1 = 0‬‬
‫‪2‬‬ ‫‪4‬‬

‫𝟬𝟵‬

‫‪http ://tinyurl.com/Malki1718‬‬ ‫‪0‬‬


‫‪ .VI‬ﲤﺎرﻳﻦ ﺗﻄﺒﻴﻘﻴﺔ‬
‫‪.‬‬
‫‪1‬‬ ‫‪1‬‬
‫⟺‬ ‫‪x−‬‬ ‫= ‪−y‬‬ ‫و‬ ‫‪2 Re (z) × |z| − 1 = 0‬‬
‫‪2‬‬ ‫‪4‬‬

‫‪Ŕž Ŧ Ľ‬‬
‫‪1‬‬ ‫‪1‬‬
‫⟺‬ ‫‪x−‬‬ ‫= ‪−y‬‬ ‫و‬ ‫‪Re (z) = 0‬‬ ‫أو‬ ‫‪|z| = 1‬‬
‫‪2‬‬ ‫‪4‬‬

‫‪œǃ‬‬
‫‪1‬‬ ‫‪1‬‬
‫⟺‬ ‫‪x−‬‬ ‫= ‪−y‬‬ ‫و‬ ‫‪x=0‬‬ ‫أو‬ ‫‪x +y =1‬‬
‫‪2‬‬ ‫‪4‬‬
‫‪⎪ x− 1‬‬
‫⎧‬
‫= ‪−y‬‬
‫‪1‬‬ ‫‪⎪ x− 1‬‬
‫⎧‬
‫= ‪−y‬‬
‫‪1‬‬

‫‪Ŀ‬‬
‫⟺‬ ‫‪2‬‬ ‫‪4‬‬ ‫أو‬ ‫‪2‬‬ ‫‪4‬‬
‫⎨‬
‫⎪‬ ‫⎨‬
‫⎪‬
‫⎩‬ ‫‪x=0‬‬ ‫⎩‬ ‫‪x +y =1‬‬
‫‪y=0‬‬ ‫‪y =x −x‬‬
‫⟺‬ ‫أو‬
‫‪x=0‬‬ ‫‪x + x −x =1‬‬

‫‪y=0‬‬ ‫‪⎧y = x − x‬‬


‫⟺‬ ‫أو‬ ‫‪1‬‬
‫‪x=0‬‬ ‫‪⎨ x=1‬‬ ‫أو‬ ‫‪x=−‬‬
‫⎩‬ ‫‪2‬‬
‫⎧‬
‫⎪‬ ‫‪x=1‬‬ ‫و‬ ‫‪y =0‬‬
‫‪y=0‬‬
‫⟺‬ ‫أو‬
‫‪x=0‬‬ ‫‪⎪ x = −1‬‬
‫⎨‬
‫و‬ ‫= ‪y‬‬
‫‪3‬‬
‫أو‬
‫⎩‬ ‫‪2‬‬ ‫‪4‬‬
‫‪⟺ z=0‬‬ ‫أو‬ ‫‪z=1‬‬ ‫أو‬ ‫‪z=j‬‬ ‫أو‬ ‫‪z=j‬‬

‫‪ z, z , z‬إذن ﻳﻮﺟﺪ ﻣﺜﻠﺚ ‪ABC‬‬ ‫‪= 1, j, j‬‬ ‫ﻓﺈن ‪ z = j‬أو ‪ z = j‬و ﰲ ﻛﻠﺘﺎ اﳊﺎﻟﺘﲔ ّ‬
‫ﻓﺈن‬ ‫أن }‪ّ z ∉ {0, 1, −1‬‬
‫و ﺑﲈ ّ‬
‫وﺣﻴﺪ و ﻫﻮ اﳌﺜﻠﺚ اﻟﺬي ﻟﻮاﺣﻖ رؤوﺳﻪ ﻫﻲ ‪ j ، 1‬و ‪. j‬‬
‫ﻣﻼﺣﻈﺔ ‪ ⧏ : 17‬ﻛﺎن ﺑﺎﻹﻣﻜﺎن إﺿﺎﻓﺔ اﻟﴩط )‪ (OC) ⟂ (AB‬ﻟﻜﻦ ذﻟﻚ ﻻ ُﻳ ﱢ‬
‫ﻐﲑ اﻟﻨﺘﻴﺠﺔ ّ‬
‫ﻷن اﻷﻋﻤﺪة اﻟﺜﻼﺛﺔ ﺗﻠﺘﻘﻲ‬
‫ﰲ ﻧﻘﻄﺔ واﺣﺪة و ﻫﻲ ﻧﻘﻄﺔ ﺗﻘﺎﻃﻊ ﻋﻤﻮدﻳﻦ ﻛﻴﻔ َﻴ ْﲔ‪.‬‬
‫ِ‬
‫ﻛﺎن ﺑﺎﻹﻣﻜﺎن أﻳﻀ ًﺎ اﺗﺒﺎع ﻧﻔﺲ ﻃﺮﻳﻘﺔ اﻟﺸﻄﺮ ّ‬
‫اﻷول ﻣﻦ اﻟﺘﻜﺎﻓﺆ ‪:‬‬
‫‪ņőƱŀ Ţ‬‬
‫‪z −z‬‬ ‫‪z −1‬‬
‫⟺ )‪(OB) ⟂ (AC‬‬ ‫‪∈ ıℝ ⟺ Re‬‬ ‫‪=0‬‬
‫‪z‬‬ ‫‪z‬‬
‫‪x x +y −1‬‬ ‫‪y x +y +1‬‬
‫‪⟺ Re‬‬ ‫‪+ı‬‬ ‫‪=0‬‬
‫‪x +y‬‬ ‫‪x +y‬‬
‫‪VI‬‬

‫‪⟺ x x +y −1 =0‬‬
‫⧐‬

‫■‬
‫‪ƕ‬‬

‫ﻣﺎ ﻫﻮ اﻟﴩط اﻟﻼزم و اﻟﻜﺎﰲ اﻟﺬي ﳚﺐ أن ﲢﻘﻘﻪ اﻷﻋﺪاد اﳌﺮﻛﺒﺔ ‪ c ، b ، a‬ﺣﺘﻰ ﺗﻜﻮن اﻟﻨﻘﻂ ‪C ، B ، A‬‬
‫!‬
‫𝟓𝟐‬‫‪.‬‬ ‫‪.‬‬

‫اﻟﺘﻲ ﻟﻮاﺣﻘﻬﺎ ‪ c ، b ، a‬ﻋﲆ اﺳﺘﻘﺎﻣﺔ واﺣﺪة ؟‬


‫‪ũŏ‬‬

‫ﺑﺪاﻳ ًﺔ‪ ،‬ﻧﻔﺮض ّ‬


‫أن اﻟﻨﻘﻂ ‪ C ، B ، A‬ﳐﺘﻠﻔﺔ ﻣﺜﻨﻰ ﻣﺜﻨﻰ أي ‪ a ≠ b‬و ‪ a ≠ c‬و ‪ . b ≠ c‬ﺣﺴﺐ اﻟﻼزﻣﺔ ‪ 1‬ﺻﻔﺤﺔ‬ ‫•‬ ‫ﺍﳊﻞّ‪.‬‬

‫𝟭𝟵‬

‫‪http ://tinyurl.com/Malki1718‬‬ ‫‪0‬‬


‫‪ .1.VI‬ﶍﻮﻋﺔ اﻷﻋﺪاد اﳌﺮﻛﺒﺔ‬
‫‪.‬‬
‫‪: 53‬‬
‫‪c−b‬‬ ‫‪c−b‬‬ ‫‪c−b‬‬

‫‪Ŕž Ŧ Ľ‬‬
‫⟺ ‪ B ، A‬و ‪ C‬ﻋﲆ اﺳﺘﻘﺎﻣﺔ واﺣﺪة‬ ‫⟺ ‪∈ℝ‬‬ ‫=‬
‫‪c−a‬‬ ‫‪c−a‬‬ ‫‪c−a‬‬
‫‪c−b c−b‬‬

‫‪œǃ‬‬
‫⟺‬ ‫=‬ ‫)‪⟺ c − b (c − a) = (c − a) (c − b‬‬
‫‪c−a c−a‬‬
‫‪⟺ cc − ca − bc + ba = cc − cb − ac + ab‬‬
‫‪⟺ ab + bc + ca = ab + bc + ca‬‬

‫‪Ŀ‬‬
‫ﻟﻜﻦ ‪ ab + bc + ca = ab + bc + ca‬إذن ‪:‬‬

‫‪ B ، A ⟺ ab + bc + ca = ab + bc + ca‬و ‪ C‬ﻋﲆ اﺳﺘﻘﺎﻣﺔ واﺣﺪة‬


‫‪⟺ ab + bc + ca ∈ ℝ‬‬

‫و ﻫﻮ اﻟﴩط اﻟﻼزم و اﻟﻜﺎﰲ اﻟﺬي ﳚﺐ أن ﲢﻘﻘﻪ اﻷﻋﺪاد اﳌﺮﻛﺒﺔ ‪ c ، b ، a‬ﺣﺘﻰ ﺗﻜﻮن اﻟﻨﻘﻂ ‪ C ، B ، A‬اﻟﺘﻲ ﻟﻮاﺣﻘﻬﺎ‬
‫‪ c ، b ، a‬ﻋﲆ اﺳﺘﻘﺎﻣﺔ واﺣﺪة‪.‬‬
‫إذا ﱂ ﺗﻜﻦ اﻟﻨﻘﻂ ‪ C ، B ، A‬ﳐﺘﻠﻔﺔ ﻣﺜﻨﻰ ﻣﺜﻨﻰ أي إذا ﻛﺎن ‪ a = b‬أو ‪ a = c‬أو ‪ b = c‬أو ‪ّ a = b = c‬‬
‫ﻓﺈن اﻟﻨﻘﻂ ‪، A‬‬ ‫•‬

‫‪ C ، B‬ﻋﲆ اﺳﺘﻘﺎﻣﺔ واﺣﺪة و ﰲ ﻫﺬه اﳊﺎﻟﺔ أﻳﻀ ًﺎ اﻟﻌﺪد ‪ ab + bc + ca‬ﺣﻘﻴﻘﻲ‪ .‬ﻣﺜﻼً‪ ،‬إذا ﻛﺎن ‪ّ a = b‬‬
‫ﻓﺈن ‪:‬‬
‫‪ab + bc + ca = aa + ac + ac = |a| + ac + ac = |a| + 2 Re (ac) ∈ ℝ‬‬

‫إذن‪ ،‬اﻟﴩط اﻟﻼزم و اﻟﻜﺎﰲ اﻟﺬي ﳚﺐ أن ﲢﻘﻘﻪ اﻷﻋﺪاد اﳌﺮﻛﺒﺔ ‪ c ، b ، a‬ﺣﺘﻰ ﺗﻜﻮن اﻟﻨﻘﻂ ‪ C ، B ، A‬اﻟﺘﻲ ﻟﻮاﺣﻘﻬﺎ ‪b ، a‬‬
‫‪ c ،‬ﻋﲆ اﺳﺘﻘﺎﻣﺔ واﺣﺪة ﻫﻮ أن ﻳﻜﻮن اﻟﻌﺪد ‪ ab + bc + ca‬ﺣﻘﻴﻘﻴ ًﺎ‪.‬‬
‫أﻋﻢ ﻣﻦ ﴍط اﻟﻼزﻣﺔ ‪ 1‬ﺻﻔﺤﺔ ‪ 53‬ﻷﻧﻪ ﻻ ﻳﺸﱰط أن ﺗﻜﻮن اﻟﻨﻘﻂ ‪ C ، B ، A‬ﳐﺘﻠﻔﺔ‬ ‫ﻣﻼﺣﻈﺔ ‪ ⧏ : 18‬ﻫﺬا اﻟﴩط ّ‬
‫⧐‬
‫ﻣﺜﻨﻰ ﻣﺜﻨﻰ‪.‬‬
‫■‬
‫‪ņőƱŀ Ţ‬‬
‫‪ B ، A‬و ‪ C‬ﺛﻼث ﻧﻘﻂ ﻣﻦ اﳌﺴﺘﻮي ﻟﻮاﺣﻘﻬﺎ ﻫﻲ ‪ z ، z‬و ‪ z‬ﻋﲆ اﻟﱰﺗﻴﺐ‪.‬‬
‫‬
‫‪𝟐𝟔. .‬‬

‫أوﺟﺪ اﻷﻋﺪاد اﳌﺮﻛﺒﺔ ‪ z‬اﻟﺘﻲ ﻣﻦ أﺟﻠﻬﺎ ﻳﻜﻮن اﳌﺜﻠﺚ ‪ ABC‬ﻣﺘﺴﺎوي اﻟﺴﺎﻗﲔ‪.‬‬

‫ﺍﳊﻞّ‪ .‬ﻧﻀﻊ ‪ z = x + ıy‬ﻣﻊ ‪. x, y ∈ ℝ‬‬ ‫‪VI‬‬


‫اﳌﺜﻠﺚ ‪ ABC‬ﻣﺘﺴﺎوي اﻟﺴﺎﻗﲔ إذا ﻛﺎن ‪ AB = AC‬أو ‪ BA = BC‬أو ‪ . CA = CB‬إذن ﻧُﻤ ﱢﻴﺰ ‪ 3‬ﺣﺎﻻت ‪:‬‬
‫اﳊﺎﻟﺔ اﻷوﱃ ‪:‬‬ ‫•‬
‫‪ƕ‬‬

‫||‪AB = AC ⟺ ||z − z|| = ||z − z|| ⟺ |z| ⋅ ||z − 1|| = |z| ⋅ ||z − 1|| ⋅ ||z + 1‬‬
‫‪⟺ |z| ⋅ ||z − 1|| ||z + 1|| − 1 = 0‬‬
‫‪⟺ |z| = 0‬‬ ‫‪ ||z + 1|| − 1 = 0‬أو ‪ ||z − 1|| = 0‬أو‬
‫‪ũŏ‬‬

‫‪⟺ z=0‬‬ ‫‪ ||z + 1|| = 1‬أو ‪ z − 1 = 0‬أو‬


‫‪⟺ z=0‬‬ ‫‪ ||z + 1|| = 1‬أو ‪ z = 1‬أو‬
‫‪⟺ z=0‬‬ ‫‪ (x + 1) + y = 1‬أو ‪ z = 1‬أو‬

‫𝒞‬ ‫ﻧﻼﺣﻆ ّ‬
‫أن ‪ُ z = 0‬ﳛﻘﻖ ‪ . ||z + 1|| = 1‬إذن اﳌﺠﻤﻮﻋﺔ اﻟﺘﻲ ﻧﺒﺤﺚ ﻋﻨﻬﺎ ﻫﻲ اﲢﺎد اﻟﻨﻘﻄﺔ )‪ M (−1, 0‬و اﻟﺪاﺋﺮة‬
‫اﻟﺘﻲ ﻣﺮﻛﺰﻫﺎ )‪ Ω (1, 0‬و ﻧﺼﻒ ﻗﻄﺮﻫﺎ ‪. 1‬‬

‫𝟮𝟵‬

‫‪http ://tinyurl.com/Malki1718‬‬ ‫‪0‬‬


‫‪ .VI‬ﲤﺎرﻳﻦ ﺗﻄﺒﻴﻘﻴﺔ‬
‫‪.‬‬
‫اﳊﺎﻟﺔ اﻟﺜﺎﻧﻴﺔ ‪:‬‬ ‫•‬

‫‪Ŕž Ŧ Ľ‬‬
‫||‪BA = BC ⟺ ||z − z || = ||z − z || ⟺ |z| ⋅ ||z − 1|| = |z| ⋅ ||z − 1‬‬
‫‪⟺ |z| ⋅ ||z − 1|| (|z| − 1) = 0‬‬

‫‪œǃ‬‬
‫‪⟺ |z| = 0‬‬ ‫‪ |z| − 1 = 0‬أو ‪ ||z − 1|| = 0‬أو‬
‫‪⟺ z=0‬‬ ‫‪ |z| = 1‬أو ‪ z − 1 = 0‬أو‬
‫‪⟺ z=0‬‬ ‫‪ |z| = 1‬أو ‪ z = 1‬أو‬

‫‪Ŀ‬‬
‫‪⟺ z=0‬‬ ‫‪ x + y = 1‬أو ‪ z = 1‬أو‬

‫ﻧﻼﺣﻆ ّ‬
‫أن ‪ُ z = 1‬ﳛﻘﻖ ‪ . |z| = 1‬إذن اﳌﺠﻤﻮﻋﺔ اﻟﺘﻲ ﻧﺒﺤﺚ ﻋﻨﻬﺎ ﻫﻲ اﲢﺎد اﻟﻨﻘﻄﺔ )‪ O (0, 0‬و اﻟﺪاﺋﺮة 𝒞 اﻟﺘﻲ‬
‫ﻣﺮﻛﺰﻫﺎ )‪ O (0, 0‬و ﻧﺼﻒ ﻗﻄﺮﻫﺎ ‪. 1‬‬
‫اﳊﺎﻟﺔ اﻟﺜﺎﻟﺜﺔ ‪:‬‬ ‫•‬

‫||‪CA = CB ⟺ ||z − z || = ||z − z || ⟺ |z| ⋅ ||z − 1|| ⋅ ||z + 1|| = |z| ⋅ ||z − 1‬‬
‫‪⟺ |z| ⋅ ||z − 1|| ||z + 1|| − |z| = 0‬‬
‫‪⟺ |z| = 0‬‬ ‫‪ ||z + 1|| − |z| = 0‬أو ‪ ||z − 1|| = 0‬أو‬
‫‪⟺ z=0‬‬ ‫|‪ ||z + 1|| = |z‬أو ‪ z − 1 = 0‬أو‬
‫‪⟺ z=0‬‬ ‫‪ (x + 1) + y |z| = x + y‬أو ‪ z = 1‬أو‬
‫‪1‬‬
‫‪⟺ z=0‬‬ ‫أو‬ ‫‪z=1‬‬ ‫أو‬ ‫‪x=−‬‬
‫‪2‬‬
‫‪1‬‬
‫‪x=−‬‬ ‫إذن اﳌﺠﻤﻮﻋﺔ اﻟﺘﻲ ﻧﺒﺤﺚ ﻋﻨﻬﺎ ﻫﻲ اﲢﺎد اﻟﻨﻘﻄﺘﲔ )‪ O (0, 0‬و )‪ M (1, 0‬و اﳌﺴﺘﻘﻴﻢ )‪ (Δ‬اﻟﺬي ﻣﻌﺎدﻟﺘﻪ‬
‫‪2‬‬
‫‪1‬‬
‫)أي ‪. (Re (z) = −‬‬
‫‪2‬‬
‫𝒞∈‪O‬‬ ‫ﰲ اﻷﺧﲑ‪ ،‬ﳎﻤﻮﻋﺔ اﻷﻋﺪاد اﳌﺮﻛﺒﺔ اﻟﺘﻲ ﲢﻘﻖ اﳌﻄﻠﻮب ﻫﻲ اﲢﺎد اﻟﺪاﺋﺮﺗﲔ 𝒞 و 𝒞 و اﳌﺴﺘﻘﻴﻢ )‪) (Δ‬ﻻﺣﻆ ّ‬
‫أن‬
‫■‬ ‫و 𝒞 ∈ ‪.(M‬‬
‫‪ņőƱŀ Ţ‬‬
‫اﳌﺴﺘﻮي ﻣﻨﺴﻮب إﱃ ﻣﻌﻠﻢ ﻣﺘﻌﺎﻣﺪ و ﻣﺘﺠﺎﻧﺲ ⃗ ‪. O, i,⃗ j‬‬
‫‬ ‫‪.‬‬
‫𝟕𝟐‬ ‫‪.‬‬
‫‪VI‬‬

‫‪ z‬ﻋﺪد ﻣﺮﻛﺐ ﺻﻮرﺗﻪ ‪ .M‬ﻧﻀﻊ ‪:‬‬


‫‪z+1‬‬
‫=‪L‬‬
‫‪z−1‬‬
‫ﻋﲔ اﳌﺠﻤﻮﻋﺔ ‪ ℰ‬ﻟﻠﻨﻘﻂ ‪ M‬ﻣﻦ اﳌﺴﺘﻮي ﺑﺤﻴﺚ ﻳﻜﻮن ‪ L‬ﻋﺪ ًدا ﺣﻘﻴﻘﻴﺎ‪.‬‬‫‪ •1‬ﱢ‬
‫‪ƕ‬‬

‫ﻋﲔ اﳌﺠﻤﻮﻋﺔ ‪ ℰ‬ﻟﻠﻨﻘﻂ ‪ M‬ﻣﻦ اﳌﺴﺘﻮي ﺑﺤﻴﺚ ﻳﻜﻮن ‪ L‬ﲣﻴﻠﻴﺎ ِﴏ ًﻓﺎ‪.‬‬‫‪ •2‬ﱢ‬
‫‪ •3‬ﻧﺴﻤﻲ ‪ M‬ﺻﻮرة ‪ّ . L‬‬
‫ﻋﲔ ﳎﻤﻮﻋﺔ اﻟﻨﻘﻂ ‪ M‬ﺑﺤﻴﺚ ﺗﻜﻮن اﻟﻨﻘﻂ ‪ M ، M ، O‬ﻋﲆ اﺳﺘﻘﺎﻣﺔ واﺣﺪة‪.‬‬
‫‪ũŏ‬‬

‫𝟯𝟵‬

‫‪http ://tinyurl.com/Malki1718‬‬ ‫‪0‬‬


‫‪ .1.VI‬ﶍﻮﻋﺔ اﻷﻋﺪاد اﳌﺮﻛﺒﺔ‬
‫‪.‬‬
‫ﺍﳊﻞّ‪ .‬ﻧﻀﻊ )‪ A (1‬و ‪ z = x + ıy‬ﻣﻊ ‪ . x, y ∈ ℝ‬ﺣﺘﻰ ﻳﻜﻮن ‪ُ L‬ﻣ َ‬
‫ﻌﺮﻓ ًﺎ ﳚﺐ أن ﻳﻜﻮن ‪ z ≠ 1‬أي ‪ . M ≠ A‬ﻟﺪﻳﻨﺎ ‪:‬‬

‫‪Ŕž Ŧ Ľ‬‬
‫)‪z + 1 x + 1 + ıy (x + 1 + ıy) (x − 1 − ıy‬‬
‫=‪L‬‬ ‫=‬ ‫=‬
‫‪z − 1 x − 1 + ıy‬‬ ‫‪(x − 1) + y‬‬

‫‪œǃ‬‬
‫‪x − x − ıxy + x − 1 − ıy + ıxy − ıy + y‬‬
‫=‬
‫‪(x − 1) + y‬‬
‫‪x +y −1‬‬ ‫‪2y‬‬
‫=‬ ‫‪−ı‬‬
‫‪(x − 1) + y‬‬ ‫‪(x − 1) + y‬‬

‫‪Ŀ‬‬
‫‪ •1‬ﻟﺘﻜﻦ ‪ . M ≠ A‬ﻟﺪﻳﻨﺎ ‪:‬‬
‫‪2y‬‬
‫‪M ∈ ℰ ⟺ Im (L) = 0 ⟺ −‬‬ ‫‪=0 ⟺ y=0‬‬
‫‪(x − 1) + y‬‬
‫و ﻫﻲ اﳌﺴﺘﻘﻴﻢ ‪ Δ‬اﻟﺬي ﻣﻌﺎدﻟﺘﻪ ‪) y = 0‬ﳏﻮر اﻟﻔﻮاﺻﻞ( ﺑﺎﺳﺘﺜﻨﺎء اﻟﻨﻘﻄﺔ )‪. A (1‬‬
‫إذن ‪. ℰ = Δ ⧵ {A} :‬‬
‫‪ •2‬ﻟﺘﻜﻦ ‪ . M ≠ A‬ﻟﺪﻳﻨﺎ ‪:‬‬
‫‪x +y −1‬‬
‫‪M∈ℰ‬‬ ‫⟺ ‪⟺ Re (L) = 0‬‬ ‫‪=0 ⟺ x +y =1‬‬
‫‪(x − 1) + y‬‬
‫و ﻫﻲ اﻟﺪاﺋﺮة )‪ 𝒞 (O, 1‬اﻟﺘﻲ ﻣﺮﻛﺰﻫﺎ )‪ O (0, 0‬و ﻧﺼﻒ ﻗﻄﺮﻫﺎ ‪ 1‬ﺑﺎﺳﺘﺜﻨﺎء اﻟﻨﻘﻄﺔ ‪. A‬‬
‫إذن ‪. ℰ = 𝒞 ⧵ {A} :‬‬
‫‪ •3‬ﻟﻴﻜﻦ ‪. z ≠ 1‬‬
‫ﻓﺈن ‪ M = O‬و ﺑﺎﻟﺘﺎﱄ اﻟﻨﻘﻂ )‪ M (L) ، M (z) ، O (0‬ﻋﲆ اﺳﺘﻘﺎﻣﺔ واﺣﺪة‪ .‬ﻧﻔﺮض إذن ّ‬
‫أن ‪. z ≠ 0‬‬ ‫إذا ﻛﺎن ‪ّ z = 0‬‬
‫ﺗﻜﻮن اﻟﻨﻘﻂ )‪ M (L) ، M (z) ، O (0‬ﻋﲆ اﺳﺘﻘﺎﻣﺔ واﺣﺪة إذا و ﻓﻘﻂ إذا ﻛﺎن )𝜋 ‪ MOM = 0 (mod‬أي إذا و‬
‫‪L−0‬‬
‫‪ OM, OM‬و ﺑﺎﻟﺘﺎﱄ‬ ‫‪= arg‬‬ ‫ﻟﻜﻦ )𝜋‪(mod 2‬‬ ‫‪. OM, OM‬‬ ‫ﻓﻘﻂ إذا ﻛﺎن )𝜋 ‪= 0 (mod‬‬
‫‪z−0‬‬
‫‪L‬‬
‫‪ņőƱŀ Ţ‬‬
‫‪ arg‬أي إذا و ﻓﻘﻂ‬ ‫اﻟﻨﻘﻂ )‪ M (L) ، M (z) ، O (0‬ﻋﲆ اﺳﺘﻘﺎﻣﺔ واﺣﺪة إذا و ﻓﻘﻂ إذا ﻛﺎن )𝜋 ‪= 0 (mod‬‬
‫‪z‬‬
‫‪L‬‬
‫‪ . Im‬ﻟﻜﻦ ‪:‬‬ ‫إذا ﻛﺎن ‪= 0‬‬
‫‪z‬‬

‫‪VI‬‬
‫‪L‬‬ ‫‪z+1‬‬ ‫‪z+1‬‬ ‫‪x + 1 + ıy‬‬ ‫‪x + 1 + ıy‬‬
‫=‬ ‫=‬ ‫=‬ ‫=‬
‫‪z‬‬ ‫)‪z (z − 1) z − z x − y + 2ıxy − x − ıy x − y − x + ıy (2x − 1‬‬
‫)‪(x + 1 + ıy) x − y − x − ıy (2x − 1‬‬
‫=‬
‫)‪x − y − x + y (2x − 1‬‬
‫‪ƕ‬‬

‫)‪x − xy − x − ıxy(2x − 1) + x − y − x − ıy(2x − 1) + ıx y − ıy − ıxy + y (2x − 1‬‬


‫=‬
‫)‪(x − y − x) + y (2x − 1‬‬
‫‪x + xy − x − 2y‬‬ ‫‪−y − x y − 2xy + y‬‬
‫=‬ ‫‪+ı‬‬
‫‪x −y −x‬‬ ‫)‪+ y (2x − 1‬‬ ‫‪x −y −x‬‬ ‫)‪+ y (2x − 1‬‬
‫‪ũŏ‬‬

‫‪L‬‬ ‫‪−y − x y − 2xy + y‬‬


‫‪Im‬‬ ‫⟺ ‪=0‬‬ ‫ﻣﻨﻪ ‪:‬‬
‫‪z‬‬ ‫‪x −y −x‬‬ ‫)‪+ y (2x − 1‬‬
‫‪⟺ y x + y + 2x − 1 = 0‬‬
‫‪⟺ y=0‬‬ ‫أو‬ ‫‪x + y + 2x − 1 = 0‬‬

‫‪⟺ y=0‬‬ ‫أو‬ ‫‪(x + 1) + y = 2 = √2‬‬

‫𝟰𝟵‬

‫‪http ://tinyurl.com/Malki1718‬‬ ‫‪0‬‬


‫‪ .VI‬ﲤﺎرﻳﻦ ﺗﻄﺒﻴﻘﻴﺔ‬
‫‪.‬‬
‫ّ‬
‫)ﻷن ‪ z ≠ 0‬و‬ ‫و ﻫﻲ اﲢﺎد اﳌﺴﺘﻘﻴﻢ ‪ Δ‬اﻟﺬي ﻣﻌﺎدﻟﺘﻪ ‪) y = 0‬ﳏﻮر اﻟﻔﻮاﺻﻞ( ﺑﺎﺳﺘﺜﻨﺎء اﻟﻨﻘﻄﺘﲔ )‪ O (0‬و )‪A (1‬‬
‫‪ (z ≠ 1‬ﻣﻊ اﻟﺪاﺋﺮة ‪ 𝒞 Ω, √2‬اﻟﺘﻲ ﻣﺮﻛﺰﻫﺎ )‪ Ω (−1, 0‬و ﻧﺼﻒ ﻗﻄﺮﻫﺎ ‪.√2‬‬

‫‪Ŕž Ŧ Ľ‬‬
‫ﰲ اﻷﺧﲑ‪ ،‬ﳎﻤﻮﻋﺔ اﻟﻨﻘﻂ اﻟﺘﻲ ﲢﻘﻖ اﳌﻄﻠﻮب ﻫﻲ اﲢﺎد اﳌﺴﺘﻘﻴﻢ ‪ Δ‬ﺑﺎﺳﺘﺜﻨﺎء اﻟﻨﻘﻄﺔ )‪ A (1‬ﻣﻊ اﻟﺪاﺋﺮة 𝒞 )ﻻ ﻧﺴﺘﺜﻨﻲ‬

‫‪œǃ‬‬
‫اﻟﻨﻘﻄﺔ )‪ O (0‬ﻷﳖﺎ ﲢﻘﻖ اﳌﻄﻠﻮب أﻳﻀ ًﺎ و درﺳﻨﺎ اﳊﺎﻟﺔ اﳋﺎﺻﺔ ‪ M = O‬ﰲ اﻟﺒﺪاﻳﺔ( ‪.‬‬
‫■‬

‫‪Ŀ‬‬
‫= ‪ . Z‬ﺣﺪﱢ د ﺛﻢ أﻧﺸﺊ ﳎﻤﻮﻋﺔ اﻟﻨﻘﻂ ‪ M‬ذات اﻟﻼﺣﻘﺔ ‪ z‬ﺑﺤﻴﺚ ‪:‬‬
‫‪1+z‬‬
‫ﻟﻴﻜﻦ }‪ . z ∈ ℂ ⧵ {1‬ﻧﻀﻊ‬
‫‬ ‫‪.‬‬
‫𝟖𝟐‬ ‫‪.‬‬
‫‪1−z‬‬
‫‪ Z •3‬ﺣﻘﻴﻘﻲ )أي ‪(Z ∈ ℝ‬‬ ‫‪||Z|| = 1‬‬ ‫‪1‬‬
‫•‬

‫‪ Z •4‬ﲣﻴﲇ ﴏف )أي ‪(Z ∈ ıℝ‬‬ ‫‪||Z|| = 2‬‬ ‫‪2‬‬


‫•‬

‫ﺍﳊﻞّ‪ .‬ﻧﺴﺘﻌﺮض ﻓﻴﲈ ﻳﲇ ﻃﺮﻳﻘﺘﲔ ‪ :‬اﻷوﱃ ﺑﺎﳊﺴﺎب و اﻟﺜﺎﻧﻴﺔ ﻫﻨﺪﺳﻴﺔ ‪:‬‬


‫ﻟﻴﻜﻦ ‪ z ∈ ℂ‬ﺣﻴﺚ ‪ . z ≠ 1‬ﻧﻀﻊ ‪ z = x + ıy‬ﻣﻊ ‪ x, y ∈ ℝ‬و )‪. (x, y) ≠ (1, 0‬‬
‫ﻟﺘﻜﻦ ‪ A‬اﻟﻨﻘﻄﺔ اﻟﺘﻲ ﻻﺣﻘﺘﻬﺎ ‪ −1‬؛ ‪ B‬اﻟﻨﻘﻄﺔ ذات اﻟﻼﺣﻘﺔ ‪ 1‬و ‪ ℰ‬اﳌﺠﻤﻮﻋﺔ اﻟﺘﻲ ﻧﺒﺤﺚ ﻋﻨﻬﺎ‪.‬‬
‫ﻟﺘﻜﻦ ‪ M‬ﻧﻘﻄﺔ ﻣﻦ اﳌﺴﺘﻮي ﲣﺘﻠﻒ ﻋﻦ ‪ ، B‬ﻻﺣﻘﺘﻬﺎ ‪. z‬‬
‫‪ •1‬ﺑﺎﳊﺴﺎب ‪ :‬ﻟﺪﻳﻨﺎ‬
‫||‪|1 + z‬‬
‫| ⟺ ‪||Z|| = 1‬‬ ‫‪= 1 ⟺ (1 + x) + y = (1 − x) + y‬‬ ‫‪⟺ 4x = 0 ⟺ x = 0‬‬
‫||‪||1 − z‬‬

‫إذن ﳎﻤﻮﻋﺔ اﻟﻨﻘﻂ اﻟﺘﻲ ﲢﻘﻖ اﳌﻄﻠﻮب ﻫﻲ اﳌﺴﺘﻘﻴﻢ اﻟﺬي ﻣﻌﺎدﻟﺘﻪ ‪ x = 0‬أي )‪) (Oy‬ﳏﻮر اﻟﱰاﺗﻴﺐ( ‪.‬‬
‫ﻫﻨﺪﺳﻴ ًﺎ ‪ :‬ﻟﺪﻳﻨﺎ‬
‫‪ņőƱŀ Ţ‬‬
‫‪M ∈ ℰ ⟺ ||z + 1|| = ||z − 1|| ⟺ AM = BM‬‬
‫إذن اﳌﺠﻤﻮﻋﺔ ‪ ℰ‬ﻫﻲ ﳏﻮر اﻟﻘﻄﻌﺔ اﳌﺴﺘﻘﻴﻤﺔ ]‪ . [AB‬ﻧﺘﺤﻘﻖ ﺑﺴﻬﻮﻟﺔ ّ‬
‫أن ﻫﺬا اﳌﺤﻮر ﻫﻮ اﳌﺴﺘﻘﻴﻢ )‪) (Oy‬ﻋﻤﻮدي ﻋﲆ‬
‫]‪ [AB‬أي ﻋﲆ )‪ (Ox‬و ﻳﻤﺮ ﻣﻦ ﻣﻨﺘﺼﻒ ]‪ [AB‬أي ﻣﻦ ‪.(O‬‬
‫‪VI‬‬

‫‪ •2‬ﺑﺎﳊﺴﺎب ‪ :‬ﻟﺪﻳﻨﺎ‬

‫||‪|1 + z‬‬
‫| ⟺ ‪||Z|| = 2‬‬ ‫‪= 4 ⟺ (1 + x) + y = 4 (1 − x) + 4y‬‬
‫||‪||1 − z‬‬
‫‪ƕ‬‬

‫‪⟺ x + y + 2x + 1 = 4x + 4y − 8x + 4 ⟺ 3x + 3y − 10x + 3 = 0‬‬


‫‪10‬‬ ‫‪5‬‬ ‫‪16‬‬ ‫‪4‬‬
‫‪⟺ x +y −‬‬ ‫⟺ ‪x+1=0‬‬ ‫‪x−‬‬ ‫= )‪+ (y − 0‬‬ ‫=‬
‫‪3‬‬ ‫‪3‬‬ ‫‪9‬‬ ‫‪3‬‬
‫‪ũŏ‬‬

‫‪4‬‬ ‫‪5‬‬
‫)اﻟﻨﻘﻄﺔ )‪ (1, 0‬ﻻ‬ ‫‪ Ω‬و ﻧﺼﻒ ﻗﻄﺮﻫﺎ‬ ‫‪,0‬‬ ‫إذن ﳎﻤﻮﻋﺔ اﻟﻨﻘﻂ اﻟﺘﻲ ﲢﻘﻖ اﳌﻄﻠﻮب ﻫﻲ اﻟﺪاﺋﺮة اﻟﺘﻲ ﻣﺮﻛﺰﻫﺎ‬
‫‪3‬‬ ‫‪3‬‬
‫ﺗﻨﺘﻤﻲ إﻟﻴﻬﺎ( ‪.‬‬
‫ﻫﻨﺪﺳﻴ ًﺎ ‪ :‬ﻟﺘﻜﻦ )𝜔( ‪ Ω‬ﻣﺮﻛﺰ اﳌﺴﺎﻓﺎت اﳌﺘﻨﺎﺳﺒﺔ ﻟﻠﺠﻤﻠﺔ })‪. {A (1) , B (−4‬‬

‫𝟱𝟵‬

‫‪http ://tinyurl.com/Malki1718‬‬ ‫‪0‬‬


‫‪ .1.VI‬ﶍﻮﻋﺔ اﻷﻋﺪاد اﳌﺮﻛﺒﺔ‬
‫‪.‬‬
‫‪5‬‬ ‫‪5‬‬
‫= ‪ Ω‬و ﺑﺎﻟﺘﺎﱄ ‪:‬‬ ‫‪,0‬‬ ‫= 𝜔 إذن‬ ‫ﻟﺪﻳﻨﺎ ‪ 1 ⋅ (𝜔 − (−1)) + (−4) (𝜔 − 1) = 0 :‬ﻣﻨﻪ ‪+ 0ı‬‬
‫‪3‬‬ ‫‪3‬‬

‫‪Ŕž Ŧ Ľ‬‬
‫||‪M ∈ ℰ ⟺ ||z + 1|| = 4 ||z − 1‬‬ ‫‪⟺ AM = 4BM‬‬ ‫‪⟺ AM − 4BM = 0‬‬

‫‪œǃ‬‬
‫⟺‬ ‫‪AΩ + ΩM‬‬ ‫‪− 4 BΩ + ΩM‬‬ ‫‪=0‬‬

‫‪⟺ −3ΩM + 2 AΩ − 4BΩ ⋅ΩM + AΩ − 4BΩ = 0‬‬

‫⎦‬
‫⎥‬
‫⎥‬
‫⎥‬
‫⎥‬
‫⎤‬

‫‪Ŀ‬‬
‫=‬ ‫⃗‪0‬‬
‫‪1‬‬
‫‪⟺ −3ΩM = ΩA − 4ΩB‬‬ ‫= ‪⟺ ΩM‬‬ ‫‪ΩA − 4ΩB‬‬
‫‪3‬‬

‫‪5‬‬ ‫‪4‬‬ ‫‪5‬‬ ‫‪64‬‬


‫= ‪ ΩB‬إذن ‪:‬‬ ‫‪−1‬‬ ‫= ‪+0‬‬ ‫= ‪ ΩA‬و‬ ‫‪+1‬‬ ‫= ‪+0‬‬ ‫ﻟﻜﻦ‬
‫‪3‬‬ ‫‪9‬‬ ‫‪3‬‬ ‫‪9‬‬

‫‪1‬‬ ‫‪1‬‬ ‫‪64 16‬‬ ‫‪16‬‬


‫‪ΩA − 4ΩB‬‬ ‫=‬ ‫‪−‬‬ ‫=‬
‫‪3‬‬ ‫‪3‬‬ ‫‪9‬‬ ‫‪9‬‬ ‫‪9‬‬
‫‪16‬‬ ‫‪4‬‬
‫= ‪M ∈ ℰ ⟺ ΩM‬‬ ‫= ‪⟺ ΩM‬‬ ‫ﻣﻨﻪ ‪:‬‬
‫‪9‬‬ ‫‪3‬‬
‫‪4‬‬ ‫‪5‬‬
‫‪ Ω‬و ﻧﺼﻒ ﻗﻄﺮﻫﺎ ‪.‬‬ ‫‪,0‬‬ ‫و ﻧﺠﺪ اﻟﺪاﺋﺮة اﻟﺘﻲ ﻣﺮﻛﺰﻫﺎ‬
‫‪3‬‬ ‫‪3‬‬
‫ﻃﺮﻳﻘﺔ ﻫﻨﺪﺳﻴﺔ أﺧﺮى ﰲ اﻟﺘﻤﺮﻳﻦ ‪ 169‬ﺻﻔﺤﺔ ‪.253‬‬ ‫‬
‫‪ •3‬ﺑﺎﳊﺴﺎب ‪ :‬ﻟﺪﻳﻨﺎ‬
‫‪1+z 1+z‬‬
‫⟺ ‪Z∈ℝ ⟺ Z=Z‬‬ ‫=‬ ‫)‪⟺ (1 + z) (1 − z) = (1 − z) (1 + z‬‬
‫‪1−z 1−z‬‬
‫‪⟺ z−z=z−z ⟺ z=z ⟺ z∈ℝ‬‬
‫‪ņőƱŀ Ţ‬‬
‫أن ‪ّ z = 1 ∈ ℝ‬‬
‫ﻓﺈن اﳌﺠﻤﻮﻋﺔ اﻟﺘﻲ ﻧﺒﺤﺚ ﻋﻨﻬﺎ ﻫﻲ }‪ ℝ ⧵ {1‬أي اﳌﺴﺘﻘﻴﻢ )‪ (Ox‬ﺑﺎﺳﺘﺜﻨﺎء اﻟﻨﻘﻄﺔ )‪. B (1, 0‬‬ ‫و ﺑﲈ ّ‬
‫ﻫﻨﺪﺳﻴ ًﺎ ‪ :‬ﻟﺪﻳﻨﺎ‬
‫‪1+z‬‬

‫‪VI‬‬
‫‪M∈ℰ ⟺ Z=0‬‬ ‫أو‬ ‫‪arg (Z) = 0 (mod 𝜋) ⟺ z = −1‬‬ ‫أو‬ ‫‪arg‬‬ ‫)𝜋 ‪= 0 (mod‬‬
‫‪1−z‬‬
‫‪⟺ M=A‬‬ ‫أو‬ ‫}‪BM, AM = 0 (mod 𝜋) ⟺ M ∈ (AB) ⧵ {B‬‬

‫و ﻧﺠﺪ اﳌﺴﺘﻘﻴﻢ )‪ (Ox‬ﺑﺎﺳﺘﺜﻨﺎء اﻟﻨﻘﻄﺔ )‪. B (1, 0‬‬


‫‪ƕ‬‬

‫‪ •4‬ﺑﺎﳊﺴﺎب ‪ :‬ﻟﺪﻳﻨﺎ‬
‫‪1+z‬‬ ‫‪1+z‬‬
‫⟺ ‪Z ∈ ıℝ ⟺ Z = −Z‬‬ ‫‪=−‬‬ ‫)‪⟺ (1 + z) (1 − z) = − (1 − z) (1 + z‬‬
‫‪ũŏ‬‬

‫‪1−z‬‬ ‫‪1−z‬‬
‫‪⟺ 1 − zz = −1 + zz ⟺ |z| = 1 ⟺ |z| = 1‬‬

‫و ﻫﻲ اﻟﺪاﺋﺮة اﻟﺘﻲ ﻣﺮﻛﺰﻫﺎ ‪ O‬و ﻧﺼﻒ ﻗﻄﺮﻫﺎ ‪ 1‬ﺑﺎﺳﺘﺜﻨﺎء اﻟﻨﻘﻄﺔ )‪. (1, 0‬‬

‫𝟲𝟵‬

‫‪http ://tinyurl.com/Malki1718‬‬ ‫‪0‬‬


‫‪ .VI‬ﲤﺎرﻳﻦ ﺗﻄﺒﻴﻘﻴﺔ‬
‫‪.‬‬
‫ﻫﻨﺪﺳﻴ ًﺎ ‪ :‬ﻟﺪﻳﻨﺎ‬
‫𝜋‬ ‫‪1+z‬‬

‫‪Ŕž Ŧ Ľ‬‬
‫‪M∈ℰ ⟺ Z=0‬‬ ‫أو‬ ‫‪(mod 𝜋) ⟺ z = −1‬‬
‫= )‪arg (Z‬‬ ‫أو‬ ‫‪arg‬‬ ‫)𝜋 ‪= 0 (mod‬‬
‫‪2‬‬ ‫‪1−z‬‬
‫𝜋‬
‫أو ‪⟺ M = A‬‬ ‫)𝜋 ‪BM, AM = (mod‬‬

‫‪œǃ‬‬
‫‪2‬‬
‫اﻟﻨﻘﻄﺔ ‪ M‬ﺗﻨﺘﻤﻲ إﱃ اﻟﺪاﺋﺮة اﻟﺘﻲ ﻗﻄﺮﻫﺎ ‪ AB‬ﺑﺎﺳﺘﺜﻨﺎء اﻟﻨﻘﻄﺔ ‪⟺ B‬‬
‫‪1‬‬
‫𝒞∈‪⟺ M‬‬ ‫}‪O, AB ⧵ {B} ⟺ M ∈ 𝒞 (O, 1) ⧵ {B‬‬
‫‪2‬‬

‫‪Ŀ‬‬
‫و ﻧﺠﺪ اﻟﺪاﺋﺮة اﻟﺘﻲ ﻣﺮﻛﺰﻫﺎ ‪ O‬و ﻧﺼﻒ ﻗﻄﺮﻫﺎ ‪ 1‬ﺑﺎﺳﺘﺜﻨﺎء اﻟﻨﻘﻄﺔ )‪. B (1, 0‬‬
‫■‬

‫اﳌﺴﺘﻮي ﻣﻨﺴﻮب إﱃ ﻣﻌﻠﻢ ﻣﺘﻌﺎﻣﺪ و ﻣﺘﺠﺎﻧﺲ ‪.‬‬


‫‬‫‪.‬‬
‫𝟗𝟐‬ ‫‪.‬‬

‫‪ z‬ﻋﺪد ﻣﺮﻛﺐ ﺻﻮرﺗﻪ ‪ .M‬ﻧﻀﻊ‬


‫‪z−ı‬‬
‫=‪L‬‬
‫‪z+ı‬‬

‫‪ •1‬ﱢ‬
‫ﻋﲔ اﳌﺠﻤﻮﻋﺔ ‪ ℰ‬ﻟﻠﻨﻘﻂ ‪ M‬ﻣﻦ اﳌﺴﺘﻮي ﺑﺤﻴﺚ ﻳﻜﻮن ‪ L‬ﻋﺪ ًدا ﺣﻘﻴﻘﻴﺎ‪.‬‬

‫‪ •2‬ﱢ‬
‫ﻋﲔ اﳌﺠﻤﻮﻋﺔ ‪ ℰ‬ﻟﻠﻨﻘﻂ ‪ M‬ﻣﻦ اﳌﺴﺘﻮي ﺑﺤﻴﺚ ﻳﻜﻮن ‪ L‬ﺣﻘﻴﻘﻴﺎ ﺳﺎﻟﺒﺎ‪.‬‬

‫ﺍﳊﻞّ‪ .‬ﻧﻀﻊ )‪ A (−ı‬و ‪ z = x + ıy‬ﻣﻊ ‪ . x, y ∈ ℝ‬ﺣﺘﻰ ﻳﻜﻮن اﻟﻌﺪد ‪ُ L‬ﻣ ﱠ‬


‫ﻌﺮﻓ ًﺎ ﳚﺐ أن ﻳﻜﻮن ‪ z ≠ −ı‬أي ‪ . M ≠ A‬ﻟﺪﻳﻨﺎ‬
‫‪:‬‬
‫))‪z − ı x + ı (y − 1) (x + ı (y − 1)) (x − ı (y + 1‬‬
‫=‪L‬‬ ‫=‬ ‫=‬
‫)‪z + ı x + ı (y + 1‬‬ ‫)‪x + (y + 1‬‬
‫‪x − ıxy − ıx + ıxy − ıx + y − 1‬‬
‫‪ņőƱŀ Ţ‬‬
‫=‬
‫)‪x + (y + 1‬‬
‫‪x +y −1‬‬ ‫‪2ıx‬‬
‫=‬ ‫‪−‬‬
‫)‪x + (y + 1‬‬ ‫)‪x + (y + 1‬‬
‫‪VI‬‬

‫‪ •1‬ﻟﺘﻜﻦ ‪ . M ≠ A‬ﻟﺪﻳﻨﺎ ‪:‬‬


‫‪2x‬‬
‫‪M ∈ ℰ ⟺ Im (L) = 0 ⟺ −‬‬ ‫‪=0 ⟺ x=0‬‬
‫)‪x + (y + 1‬‬
‫و ﻫﻲ اﳌﺴﺘﻘﻴﻢ ‪ Δ‬اﻟﺬي ﻣﻌﺎدﻟﺘﻪ ‪) x = 0‬ﳏﻮر اﻟﱰاﺗﻴﺐ( ﺑﺎﺳﺘﺜﻨﺎء اﻟﻨﻘﻄﺔ )‪. A (−ı‬‬
‫‪ƕ‬‬

‫إذن ‪ ℰ = Δ ⧵ {A} :‬أو ﺑﺘﻌﺒﲑ آﺧﺮ ‪. ℰ = ıℝ ⧵ {A} :‬‬


‫‪ •2‬ﻟﺘﻜﻦ ‪ . M ≠ A‬ﻟﺪﻳﻨﺎ ‪:‬‬
‫‪ũŏ‬‬

‫⎧‬ ‫‪2x‬‬
‫⎪‬
‫⎪‬ ‫‪−‬‬ ‫‪=0‬‬
‫‪Im (L) = 0‬‬ ‫⎪‬ ‫)‪x + (y + 1‬‬ ‫‪x=0‬‬
‫‪M∈ℰ‬‬ ‫⟺‬ ‫⟺‬ ‫⟺‬
‫‪Re (L) ≤ 0‬‬ ‫⎨‬
‫⎪‬ ‫‪x +y −1‬‬ ‫‪x +y −1≤0‬‬
‫⎪‬
‫⎪‬ ‫≤‬ ‫‪0‬‬
‫)‪⎩ x + (y + 1‬‬
‫‪x=0‬‬ ‫‪x=0‬‬
‫⟺‬ ‫⟺‬
‫‪y ≤1‬‬ ‫‪−1 ≤ y ≤ 1‬‬

‫𝟳𝟵‬

‫‪http ://tinyurl.com/Malki1718‬‬ ‫‪0‬‬


‫‪ .1.VI‬ﶍﻮﻋﺔ اﻷﻋﺪاد اﳌﺮﻛﺒﺔ‬
‫‪.‬‬
‫و ﻫﻲ اﻟﻘﻄﻌﺔ اﳌﺴﺘﻘﻴﻤﺔ ]‪ [AB‬ﺑﺎﺳﺘﺜﻨﺎء اﻟﻨﻘﻄﺔ ‪ A‬ﺣﻴﺚ )‪. B (ı‬‬
‫إذن ‪. ℰ = AB :‬‬

‫‪Ŕž Ŧ Ľ‬‬
‫■‬

‫‪œǃ‬‬
‫‪𝟑𝟎.‬‬ ‫‪.‬‬
‫ﻟﻴﻜﻦ ‪ z‬ﻋﺪد ًاﻣﺮﻛﺒﺎ ﳜﺘﻠﻒ ﻋﻦ ‪ .1‬ﻧﻌﺘﱪ اﻟﻨﻘﻄﺔ ‪ M‬ﻣﻦ اﳌﺴﺘﻮي اﳌﺮﻛﺐ اﻟﺘﻲ ﻻﺣﻘﺘﻬﺎ ‪ z‬و ﻧﻀﻊ‬
‫‪z+ı‬‬

‫‪Ŀ‬‬
‫= ‪ .Z‬ﺣﺪّ د ‪:‬‬
‫‪z−1‬‬
‫‪ •1‬اﳌﺠﻤﻮﻋﺔ ‪ E‬ﻟﻠﻨﻘﻂ ‪ M‬ﺑﺤﻴﺚ ﻳﻜﻮن ‪ Z‬ﻋﺪد ًا ﺣﻘﻴﻘﻴﺎ‪.‬‬
‫‪ •2‬اﳌﺠﻤﻮﻋﺔ ‪ F‬ﻟﻠﻨﻘﻂ ‪ M‬ﺑﺤﻴﺚ ﻳﻜﻮن ‪.||Z|| = 1‬‬
‫𝜋‬
‫= )‪.arg (Z‬‬ ‫اﳌﺠﻤﻮﻋﺔ ‪ G‬ﻟﻠﻨﻘﻂ ‪ M‬ﺑﺤﻴﺚ ﻳﻜﻮن )𝜋‪(mod 2‬‬ ‫•‬ ‫‪3‬‬
‫‪2‬‬

‫ﺍﳊﻞّ‪ .‬ﻧﻀﻊ )‪ A (−ı‬و )‪ B (1‬ﻣﻨﻪ‬


‫‪z+ı‬‬ ‫‪z−z‬‬
‫= ‪.Z‬‬ ‫=‬
‫‪z−1‬‬ ‫‪z−z‬‬
‫•‬ ‫‪1‬‬
‫)𝜋 ‪ BM, AM = 0 (mod‬أو ‪ arg (Z) = 0 (mod 𝜋) ⟺ z = z‬أو ‪ Z ⟺ Z = 0‬ﺣﻘﻴﻘﻲ‬
‫ﻟﻜﻦ‬
‫‪ M‬ﺗﻨﺘﻤﻲ إﱃ اﳌﺴﺘﻘﻴﻢ )‪ (AB‬ﺑﺎﺳﺘﺜﻨﺎء ‪ A‬و ‪BM, AM = 0 (mod 𝜋) ⟺ B‬‬

‫ﻣﻨﻪ ﻧﺴﺘﻨﺘﺞ ّ‬
‫أن اﳌﺠﻤﻮﻋﺔ ‪ E‬ﻫﻲ اﳌﺴﺘﻘﻴﻢ )‪ (AB‬ﺑﺎﺳﺘﺜﻨﺎء اﻟﻨﻘﻄﺔ ‪.B‬‬
‫•‬ ‫‪2‬‬
‫ﻨﺼﻒ اﻟﻘﻄﻌﺔ ]‪||Z|| = 1 ⟺ ||z − z || = ||z − z || ⟺ AM = BM ⟺ [AB‬‬
‫‪ M‬ﺗﻨﺘﻤﻲ إﱃ ُﻣ ّ‬
‫ﻨﺼﻒ اﻟﻘﻄﻌﺔ ]‪.[AB‬‬
‫ﻓﺎﳌﺠﻤﻮﻋﺔ ‪ F‬ﻫﻲ ُﻣ ّ‬
‫‪ņőƱŀ Ţ‬‬
‫𝜋‬ ‫𝜋‬
‫•‬ ‫‪3‬‬
‫= )‪arg (Z‬‬ ‫⟺ )𝜋‪(mod 2‬‬ ‫= ‪BM, AM‬‬ ‫)𝜋‪(mod 2‬‬
‫‪2‬‬ ‫‪2‬‬
‫ﻓﺎﳌﺠﻤﻮﻋﺔ ‪ G‬ﻫﻲ ﻧﺼﻒ اﻟﺪاﺋﺮة اﻟﺘﻲ ﻗﻄﺮﻫﺎ ]‪ ،[AB‬ﺑﺎﺳﺘﺜﻨﺎء اﻟﻨﻘﻄﺔ ‪ ،B‬ﺑﺤﻴﺚ ﻳﻜﻮن اﳌﺜﻠﺚ ‪ AMB‬ﻣﺒﺎﴍ ًا‪.‬‬

‫‪VI‬‬
‫■‬

‫‪ z‬ﻋﺪد ﻣﺮﻛﺐ ﺻﻮرﺗﻪ ‪.M‬‬


‫‬ ‫‪𝟑𝟏.‬‬ ‫‪.‬‬
‫‪ƕ‬‬

‫‪ّ •1‬‬
‫ﻋﲔ ‪ ℰ‬ﳎﻤﻮﻋﺔ اﻟﻨﻘﻂ ‪ M‬ﺑﺤﻴﺚ ﻳﻜﻮن )‪ (z − 2ı) (z − 1‬ﲣﻴﻠﻴﺎ ﴏﻓﺎ‪.‬‬

‫‪ّ •2‬‬
‫ﻋﲔ ‪ ℰ‬ﳎﻤﻮﻋﺔ اﻟﻨﻘﻂ ‪ M‬ﺑﺤﻴﺚ ﻳﻜﻮن )‪ (z − 2ı) (z − 1‬ﺣﻘﻴﻘﻴﺎ‪.‬‬
‫‪ũŏ‬‬

‫ﺍﳊﻞّ‪ .‬ﻧﻀﻊ ‪ z = x + ıy‬ﻣﻊ ‪ . x, y ∈ ℝ‬ﻟﺪﻳﻨﺎ ‪:‬‬


‫‪(z − 2ı) (z − 1) = zz − z − 2ız + 2ı = (x + ıy) (x − ıy) − x − ıy − 2ı (x − ıy) + 2ı‬‬
‫)‪= x + y − x − ıy − 2ıx − 2y + 2ı = x + y − x − 2y + ı (−2x − y + 2‬‬
‫و ﺑﺎﻟﺘﺎﱄ ‪:‬‬

‫𝟴𝟵‬

‫‪http ://tinyurl.com/Malki1718‬‬ ‫‪0‬‬


‫‪ .VI‬ﲤﺎرﻳﻦ ﺗﻄﺒﻴﻘﻴﺔ‬
‫‪.‬‬
‫‪1‬‬
‫•‬

‫‪Ŕž Ŧ Ľ‬‬
‫‪M ∈ ℰ ⟺ Re ((z − 2ı) (z − 1)) = 0‬‬
‫‪⟺ x + y − x − 2y = 0‬‬

‫‪œǃ‬‬
‫‪1‬‬ ‫‪5‬‬ ‫‪√5‬‬
‫⟺‬ ‫‪x−‬‬ ‫= )‪+ (y − 1‬‬ ‫=‬
‫‪2‬‬ ‫‪4‬‬ ‫‪2‬‬

‫‪5‬‬ ‫‪5‬‬
‫‪ Ω‬و ﻧﺼﻒ ﻗﻄﺮﻫﺎ √ ‪.‬‬ ‫√ ‪ 𝒞 Ω,‬اﻟﺘﻲ ﻣﺮﻛﺰﻫﺎ‬

‫‪Ŀ‬‬
‫‪,1‬‬ ‫إذن ‪ ℰ‬ﻫﻲ اﻟﺪاﺋﺮة‬
‫‪2‬‬ ‫‪2‬‬

‫‪2‬‬
‫•‬

‫‪M∈ℰ‬‬ ‫‪⟺ Re ((z − 2ı) (z − 1)) = 0‬‬


‫‪⟺ −2x − y + 2 = 0‬‬
‫‪⟺ y = −2x + 2‬‬

‫إذن ‪ ℰ‬ﻫﻲ اﳌﺴﺘﻘﻴﻢ ‪ Δ‬اﻟﺬي ﻣﻌﺎدﻟﺘﻪ ‪. y = −2x + 2‬‬


‫■‬

‫اﳌﺴﺘﻮي ﻣﻨﺴﻮب إﱃ ﻣﻌﻠﻢ ﻣﺘﻌﺎﻣﺪ و ﻣﺘﺠﺎﻧﺲ ⃗ ‪. O, i,⃗ j‬‬


‫‬ ‫‪.‬‬
‫𝟐𝟑‬ ‫‪.‬‬

‫‪ z‬ﻋﺪد ﻣﺮﻛﺐ ﺻﻮرﺗﻪ ‪ .M‬ﻧﻀﻊ‬


‫‪z+2‬‬
‫=‪L‬‬
‫‪z+2‬‬

‫‪ّ •1‬‬
‫ﻋﲔ اﳉﺰءﻳﻦ اﳊﻘﻴﻘﻲ و اﻟﺘﺨﻴﲇ ﻟﻠﻌﺪد ‪. L‬‬
‫ﻋﲔ اﳌﺠﻤﻮﻋﺔ ‪ ℰ‬ﻟﻠﻨﻘﻂ ‪ M‬ﻣﻦ اﳌﺴﺘﻮي ﺑﺤﻴﺚ ﻳﻜﻮن ‪ L‬ﻋﺪ ًدا ﺣﻘﻴﻘﻴﺎ‪.‬‬‫‪ •2‬ﱢ‬
‫‪ņőƱŀ Ţ‬‬
‫ﻋﲔ اﳌﺠﻤﻮﻋﺔ ‪ ℰ‬ﻟﻠﻨﻘﻂ ‪ M‬ﻣﻦ اﳌﺴﺘﻮي ﺑﺤﻴﺚ ﻳﻜﻮن ‪ L‬ﲣﻴﻠﻴﺎ ِﴏ ًﻓﺎ‪.‬‬‫‪ •3‬ﱢ‬
‫‪VI‬‬

‫ﺍﳊﻞّ‪ .‬ﻧﻀﻊ )‪ A (−2‬و ‪ z = x + ıy‬ﻣﻊ ‪ .x, y ∈ ℝ‬ﺣﺘﻰ ﻳﻜﻮن اﻟﻌﺪد ‪ُ L‬ﻣ ﱠ‬


‫ﻌﺮﻓ ًﺎ ﳚﺐ أن ﻳﻜﻮن ‪ z ≠ −2‬أي ‪. M ≠ A‬‬
‫‪ •1‬ﻟﺪﻳﻨﺎ ‪:‬‬
‫‪z + 2 x + 2 − ıy‬‬ ‫)‪(x + 2 − ıy‬‬
‫=‪L‬‬
‫‪ƕ‬‬

‫=‬ ‫=‬
‫‪z + 2 x + 2 + ıy (x + 2) + y‬‬
‫)‪(x + 2) − 2ıy (x + 2) + (ıy‬‬
‫=‬
‫‪(x + 2) + y‬‬
‫‪ũŏ‬‬

‫‪x + 4x + 4 − 2ıxy − 4ıy − y‬‬


‫=‬
‫‪(x + 2) + y‬‬
‫‪x − y + 4x + 4‬‬ ‫‪2xy + 4y‬‬
‫=‬ ‫‪−ı‬‬
‫‪(x + 2) + y‬‬ ‫‪(x + 2) + y‬‬

‫‪2xy + 4y‬‬ ‫‪x − y + 4x + 4‬‬


‫‪. Im (L) = −‬‬ ‫= )‪ Re (L‬و‬ ‫و ﻫﻮ اﻟﺸﻜﻞ اﳉﱪي ﻟﻠﻌﺪد ‪ L‬ﻣﻨﻪ‬
‫‪(x + 2) + y‬‬ ‫‪(x + 2) + y‬‬

‫𝟵𝟵‬

‫‪http ://tinyurl.com/Malki1718‬‬ ‫‪0‬‬


‫‪ .1.VI‬ﶍﻮﻋﺔ اﻷﻋﺪاد اﳌﺮﻛﺒﺔ‬
‫‪.‬‬
‫‪ •2‬ﻟﺘﻜﻦ ‪ . M ≠ A‬ﻟﺪﻳﻨﺎ ‪:‬‬

‫‪Ŕž Ŧ Ľ‬‬
‫‪2xy + 4y‬‬
‫‪M ∈ ℰ ⟺ Im (L) = 0 ⟺ −‬‬ ‫‪= 0 ⟺ 2xy + 4y = 0‬‬
‫‪(x + 2) + y‬‬
‫أو ‪⟺ y (2x + 4) = 0 ⟺ y = 0‬‬ ‫‪2x + 4 = 0‬‬

‫‪œǃ‬‬
‫‪⟺ y=0‬‬ ‫أو‬ ‫‪x = −2‬‬

‫إذن ‪ ℰ‬ﻫﻲ اﲢﺎد اﳌﺴﺘﻘﻴﻢ ‪ Δ‬اﻟﺬي ﻣﻌﺎدﻟﺘﻪ ‪) y = 0‬ﳏﻮر اﻟﻔﻮاﺻﻞ( ﻣﻊ اﳌﺴﺘﻘﻴﻢ ‪ Δ‬اﻟﺬي ﻣﻌﺎدﻟﺘﻪ ‪ x = −2‬ﺑﺎﺳﺘﺜﻨﺎء‬

‫‪Ŀ‬‬
‫اﻟﻨﻘﻄﺔ )‪. A (−2‬‬
‫‪ •3‬ﻟﺘﻜﻦ ‪ . M ≠ A‬ﻟﺪﻳﻨﺎ ‪:‬‬
‫‪x − y + 4x + 4‬‬
‫‪M∈ℰ‬‬ ‫⟺ ‪⟺ Re (L) = 0‬‬ ‫‪= 0 ⟺ x − y + 4x + 4 = 0‬‬
‫‪(x + 2) + y‬‬
‫)‪⟺ (x + 2) − y = 0 ⟺ y = − (x + 2‬‬ ‫أو‬ ‫‪y=x+2‬‬

‫و ﺑﺎﻟﺘﺎﱄ ‪ ℰ‬ﻫﻲ اﲢﺎد اﳌﺴﺘﻘﻴﻢ ‪ Δ‬اﻟﺬي ﻣﻌﺎدﻟﺘﻪ ‪ y = −x−2‬ﺑﺎﺳﺘﺜﻨﺎء اﻟﻨﻘﻄﺔ )‪ A (−2‬ﻣﻊ اﳌﺴﺘﻘﻴﻢ ‪ Δ‬اﻟﺬي ﻣﻌﺎدﻟﺘﻪ‬
‫‪ y = x + 2‬ﺑﺎﺳﺘﺜﻨﺎء اﻟﻨﻘﻄﺔ )‪) A (−2‬اﻟﻨﻘﻄﺔ ‪ A‬ﻫﻲ ﻧﻘﻄﺔ ﺗﻘﺎﻃﻊ ﻫﺬﻳﻦ اﳌﺴﺘﻘﻴﻤﲔ( ‪.‬‬
‫■‬

‫= ‪.Z‬‬
‫‪2ız + 1‬‬
‫ﻟﻴﻜﻦ ‪ z‬ﻋﺪد ًا ﻣﺮﻛﺒﺎ ﳜﺘﻠﻒ ﻋﻦ ‪ 1‬و ‪ M‬ﺻﻮرﺗﻪ ﰲ اﳌﺴﺘﻮي اﳌﺮﻛﺐ‪ .‬ﻧﻀﻊ‬
‫‬ ‫‪𝟑𝟑.‬‬ ‫‪.‬‬
‫‪z−1‬‬
‫‪ •1‬ﺣﺪّ د ‪:‬‬
‫)ا( اﳌﺠﻤﻮﻋﺔ ‪ E‬ﻟﻠﻨﻘﻂ ‪ M‬ﺑﺤﻴﺚ ﻳﻜﻮن ‪ Z‬ﻋﺪد ًا ﺣﻘﻴﻘﻴﺎ‪.‬‬
‫)ب( اﳌﺠﻤﻮﻋﺔ ‪ F‬ﻟﻠﻨﻘﻂ ‪ M‬ﺑﺤﻴﺚ ﻳﻜﻮن ‪. ||Z|| = 2‬‬
‫‪ņőƱŀ Ţ‬‬
‫‪ •2‬ﻣ ّﺜﻞ اﳌﺠﻤﻮﻋﺘﲔ ‪ E‬و ‪ F‬ﰲ ﻣﻌﻠﻢ ﻣﺘﻌﺎﻣﺪ و ﻣﺘﺠﺎﻧﺲ‪.‬‬

‫ﺍﳊﻞّ‪ .‬ﻧﻀﻊ ‪ z = x + ıy‬ﻣﻊ ‪ . x, y ∈ ℝ‬ﺑﲈ ّ‬


‫أن ‪ّ z ≠ 1‬‬
‫ﻓﺈن )‪. (x, y) ≠ (0, 1‬‬
‫‪ •1‬ﻟﺪﻳﻨﺎ ‪:‬‬ ‫‪VI‬‬
‫)‪2ız + 1 2ı (x + ıy) + 1 −2y + 2ıx (1 − 2y + 2ıx) (x − 1 − ıy‬‬
‫=‪Z‬‬ ‫=‬ ‫=‬ ‫=‬
‫‪z−1‬‬ ‫‪x + ıy − 1‬‬ ‫‪x − 1 + ıy‬‬ ‫‪(x − 1) + y‬‬
‫‪ƕ‬‬

‫‪x − 1 − ıy − 2xy + 2y + 2ıy + 2ıx − 2ıx + 2xy‬‬


‫=‬
‫‪(x − 1) + y‬‬
‫‪x − 1 + 2y‬‬ ‫‪2x + 2y − 2x − y‬‬
‫=‬ ‫‪+ı‬‬
‫‪ũŏ‬‬

‫‪(x − 1) + y‬‬ ‫‪(x − 1) + y‬‬

‫و ﻫﻮ اﻟﺸﻜﻞ اﳉﱪي ﻟﻠﻌﺪد ‪. Z‬‬

‫𝟬𝟬𝟭‬

‫‪http ://tinyurl.com/Malki1718‬‬ ‫‪0‬‬


‫‪ .VI‬ﲤﺎرﻳﻦ ﺗﻄﺒﻴﻘﻴﺔ‬
‫‪.‬‬
‫‪F = C′‬‬

‫‪Ŕž Ŧ Ľ‬‬
‫‪1‬‬

‫‪œǃ‬‬
‫‪Ω′‬‬

‫‪Ŀ‬‬
‫‪Ω‬‬
‫‪A‬‬
‫‪−1‬‬ ‫‪1‬‬

‫}‪E = C \ {A‬‬

‫ﺷﻜﻞ ‪3.VI‬‬

‫)ا( ﻟﺪﻳﻨﺎ ‪:‬‬


‫‪2x + 2y − 2x − y‬‬
‫⟺ ‪Z ∈ ℝ ⟺ Im (Z) = 0‬‬ ‫‪=0‬‬
‫‪(x − 1) + y‬‬
‫‪1‬‬
‫‪⟺ 2x + 2y − 2x − y = 0 ⟺ x + y − x − y = 0‬‬
‫‪2‬‬
‫‪1‬‬ ‫‪1‬‬ ‫‪1‬‬ ‫‪1‬‬
‫‪⟺ x−‬‬ ‫‪− + y−‬‬ ‫‪−‬‬ ‫‪=0‬‬
‫‪2‬‬ ‫‪4‬‬ ‫‪4‬‬ ‫‪16‬‬

‫‪1‬‬ ‫‪1‬‬ ‫‪1‬‬ ‫‪1‬‬ ‫‪5‬‬ ‫‪√5‬‬


‫⟺‬ ‫‪x−‬‬ ‫‪+ y−‬‬ ‫=‬ ‫‪+‬‬ ‫=‬ ‫=‬
‫‪2‬‬ ‫‪4‬‬ ‫‪4 16 16‬‬ ‫‪4‬‬

‫‪5‬‬ ‫‪1 1‬‬


‫‪ Ω‬و ﻧﺼﻒ ﻗﻄﺮﻫﺎ √ ‪ .‬ﻟﻜﻦ اﻟﻨﻘﻄﺔ )‪ A (1, 0‬ﺗﻨﺘﻤﻲ إﱃ ﻫﺬه‬ ‫‪,‬‬ ‫و ﻫﻲ ﻣﻌﺎدﻟﺔ اﻟﺪاﺋﺮة 𝒞 اﻟﺘﻲ ﻣﺮﻛﺰﻫﺎ‬
‫‪ņőƱŀ Ţ‬‬
‫‪4‬‬ ‫‪2 4‬‬
‫اﻟﺪاﺋﺮة و ﺑﺎﻟﺘﺎﱄ ‪. E = 𝒞 ⧵ {A (1, 0)} :‬‬
‫)ب( ﻟﺪﻳﻨﺎ ‪:‬‬
‫‪VI‬‬

‫||‪||Z|| = 2 ⟺ ||2ız + 1|| = 2 ||z − 1|| ⟺ ||2ız + 1|| = 4 ||z − 1‬‬

‫||‪⟺ ||1 − 2y + 2ıx|| = 4 ||x − 1 + ıy‬‬


‫)‪⟺ (1 − 2y‬‬ ‫‪+ (2x) = (x − 1) + y‬‬
‫‪2‬‬ ‫‪4‬‬
‫‪⟺ 3x + 3y + 2x − 4y = 0 ⟺ x + y + x −‬‬ ‫‪y=0‬‬
‫‪3‬‬ ‫‪3‬‬
‫‪1‬‬ ‫‪1‬‬ ‫‪2‬‬ ‫‪4‬‬
‫‪ƕ‬‬

‫‪⟺ x+‬‬ ‫‪− + y−‬‬ ‫‪− =0‬‬


‫‪3‬‬ ‫‪9‬‬ ‫‪3‬‬ ‫‪9‬‬

‫‪1‬‬ ‫‪2‬‬ ‫‪5‬‬ ‫‪√5‬‬


‫⟺‬ ‫‪x+‬‬ ‫‪+ y−‬‬ ‫=‬ ‫=‬
‫‪3‬‬ ‫‪3‬‬ ‫‪9‬‬ ‫‪3‬‬
‫‪ũŏ‬‬

‫‪5‬‬ ‫‪1 2‬‬


‫‪ Ω − ,‬و ﻧﺼﻒ ﻗﻄﺮﻫﺎ √ ‪ .‬و ﺑﲈ ّ‬
‫أن اﻟﻨﻘﻄﺔ )‪ A (1, 0‬ﻻ ﺗﻨﺘﻤﻲ‬ ‫و ﻫﻲ ﻣﻌﺎدﻟﺔ اﻟﺪاﺋﺮة 𝒞 اﻟﺘﻲ ﻣﺮﻛﺰﻫﺎ‬
‫‪3‬‬ ‫‪3 3‬‬
‫إﱃ ﻫﺬه اﻟﺪاﺋﺮة ّ‬
‫ﻓﺈن ‪. F = 𝒞 :‬‬
‫‪ •2‬أﻧﻈﺮ اﻟﺸﻜﻞ ‪. 3.VI‬‬
‫■‬

‫𝟭𝟬𝟭‬

‫‪http ://tinyurl.com/Malki1718‬‬ ‫‪0‬‬


‫‪ .1.VI‬ﶍﻮﻋﺔ اﻷﻋﺪاد اﳌﺮﻛﺒﺔ‬
‫‪.‬‬
‫‪F =C‬‬
‫‪0.5‬‬

‫‪Ŕž Ŧ Ľ‬‬
‫‪A‬‬ ‫‪Ω‬‬ ‫}‪E = (Ox) \ {A‬‬

‫‪œǃ‬‬
‫‪−1.5‬‬ ‫‪−1‬‬ ‫‪0.5‬‬ ‫‪1‬‬

‫‪−0.5‬‬

‫‪Ŀ‬‬
‫ﺷﻜﻞ ‪4.VI‬‬

‫ﻟﻴﻜﻦ ‪ z = x + ıy‬ﻋﺪد ًا ﻣﺮﻛﺒﺎ ﳜﺘﻠﻒ ﻋﻦ ‪ x ) −1‬و ‪ y‬ﻋﺪدان ﺣﻘﻴﻘﻴﺎن( و ﻟﺘﻜﻦ ‪ M‬اﻟﻨﻘﻄﺔ اﻟﺘﻲ ﻻﺣﻘﺘﻬﺎ ‪.z‬‬
‫‬ ‫‪𝟑𝟒.‬‬ ‫‪.‬‬
‫‪ız‬‬
‫= ‪.Z‬‬ ‫ﻧﻀﻊ‬
‫‪z+1‬‬

‫‪ّ •1‬‬
‫ﻋﱪ ﻋﻦ اﳉﺰﺋﲔ اﳊﻘﻴﻘﻲ و اﻟﺘﺨﻴﲇ ﻟﻠﻌﺪد ‪ Z‬ﺑﺪﻻﻟﺔ ‪ x‬و ‪. y‬‬
‫‪ •2‬إﺳﺘﻨﺘﺞ اﳌﺠﻤﻮﻋﺔ ‪ E‬ﻟﻠﻨﻘﻂ ‪ M‬ﺑﺤﻴﺚ ‪ Re (Z) = 0‬و اﳌﺠﻤﻮﻋﺔ ‪ F‬ﻟﻠﻨﻘﻂ ‪ M‬ﺑﺤﻴﺚ ‪. Im (Z) = 0‬‬

‫ﺍﳊﻞّ‪ .‬ﺑﲈ ّ‬
‫أن ‪ّ z ≠ −1‬‬
‫ﻓﺈن )‪. (x, y) ≠ (−1, 0‬‬
‫‪ •1‬ﻟﺪﻳﻨﺎ ‪:‬‬
‫‪ız‬‬ ‫)‪ı (x + ıy‬‬ ‫‪−y + ıx‬‬ ‫)‪(−y + ıx) (x + 1 − ıy‬‬
‫=‪Z‬‬ ‫=‬ ‫=‬ ‫=‬
‫‪z + 1 x + ıy + 1 x + 1 + ıy‬‬ ‫‪(x + 1) + y‬‬
‫‪−xy − y + ıy + ıx + ıx + xy‬‬ ‫‪y‬‬ ‫‪x +y +x‬‬
‫=‬ ‫‪=−‬‬ ‫‪+ı‬‬
‫‪ņőƱŀ Ţ‬‬
‫‪(x + 1) + y‬‬ ‫‪(x + 1) + y‬‬ ‫‪(x + 1) + y‬‬

‫‪x +y +x‬‬ ‫‪y‬‬


‫= )‪. Im (Z‬‬ ‫و‬ ‫‪Re (Z) = −‬‬ ‫إذن‬
‫‪(x + 1) + y‬‬ ‫‪(x + 1) + y‬‬

‫‪VI‬‬
‫‪ •2‬ﻟﺪﻳﻨﺎ ‪:‬‬
‫‪y‬‬
‫‪Re (Z) = 0 ⟺ −‬‬ ‫‪=0 ⟺ y=0‬‬
‫‪(x + 1) + y‬‬

‫إذن ‪ E‬ﻫﻲ ﳎﻤﻮﻋﺔ اﻟﻨﻘﻂ اﻟﺘﻲ ﻟﻮاﺣﻘﻬﺎ ُﲢﻘﻖ ‪ Im (z) = 0‬أي ﻫﻲ اﳌﺴﺘﻘﻴﻢ )‪) (Ox‬ﳏﻮر اﻷﻋﺪاد اﳊﻘﻴﻘﻴﺔ( ﺑﺎﺳﺘﺜﻨﺎء‬
‫‪ƕ‬‬

‫اﻟﻨﻘﻄﺔ )‪ . A (−1, 0‬ﻣﻦ ﺟﻬﺔ أﺧﺮى ‪:‬‬


‫‪1‬‬ ‫‪1‬‬
‫⟺ ‪Im (Z) = 0 ⟺ x + y + x = 0‬‬ ‫‪x+‬‬ ‫= ‪+y‬‬
‫‪2‬‬ ‫‪2‬‬
‫‪ũŏ‬‬

‫‪1‬‬ ‫‪1‬‬
‫و ﻫﻲ ﻣﻌﺎدﻟﺔ اﻟﺪاﺋﺮة 𝒞 اﻟﺘﻲ ﻣﺮﻛﺰﻫﺎ ‪ Ω − , 0‬و ﻧﺼﻒ ﻗﻄﺮﻫﺎ ‪.‬‬
‫‪2‬‬ ‫‪2‬‬
‫ﻟﻜﻦ )‪ A (−1, 0‬ﺗﻨﺘﻤﻲ إﱃ ﻫﺬه اﻟﺪاﺋﺮة و ﺑﺎﻟﺘﺎﱄ })‪) F = 𝒞 ⧵ {A (−1, 0‬ﺷﻜﻞ ‪. (4.VI‬‬
‫■‬

‫𝟮𝟬𝟭‬

‫‪http ://tinyurl.com/Malki1718‬‬ ‫‪0‬‬


‫‪ .VI‬ﲤﺎرﻳﻦ ﺗﻄﺒﻴﻘﻴﺔ‬
‫‪.‬‬
‫‪C‬‬
‫‪2‬‬

‫‪Ŕž Ŧ Ľ‬‬
‫‪A‬‬ ‫‪1‬‬

‫‪œǃ‬‬
‫‪−1‬‬ ‫‪1‬‬ ‫‪2‬‬ ‫‪3‬‬

‫‪Ŀ‬‬
‫‪−1‬‬
‫‪B‬‬

‫‪−2‬‬
‫‪D‬‬

‫ﺷﻜﻞ ‪5.VI‬‬

‫‪.‬‬ ‫⃗ ‪O, i,⃗ j‬‬


‫اﳌﺴﺘﻮي اﳌﺮﻛﺐ ﻣﻨﺴﻮب إﱃ ﻣﻌﻠﻢ ﻣﺘﻌﺎﻣﺪ و ﻣﺘﺠﺎﻧﺲ‬
‫‬ ‫‪.‬‬
‫𝟓𝟑‬ ‫‪.‬‬

‫ﻧﻌﺘﱪ اﻟﻨﻘﻂ ‪ C ، B ، A‬و ‪ D‬اﻟﺘﻲ ﻟﻮاﺣﻘﻬﺎ ‪ c = 2ı ، b = −1 − ı ، a = −1 + ı‬و ‪ d = 2 − 2ı‬ﻋﲆ اﻟﱰﺗﻴﺐ‪.‬‬


‫‪ •1‬ﻋﻠﻢ اﻟﻨﻘﻂ ‪ C ، B ، A‬و ‪. D‬‬
‫‪c−b‬‬ ‫‪c−a‬‬
‫ﺛﻢ اﻷﳼ‪.‬‬
‫ﻋﲆ اﻟﺸﻜﻞ اﳉﱪي ّ‬ ‫و‬ ‫‪ •2‬أﻛﺘﺐ اﻟﻌﺪدﻳﻦ‬
‫‪d−b‬‬ ‫‪d−a‬‬
‫‪ •3‬إﺳﺘﻨﺘﺞ ﻃﺒﻴﻌﺔ اﳌﺜﻠﺜﲔ ‪ ACD‬و ‪. BCD‬‬

‫‪ •4‬أﺛﺒﺖ ّ‬
‫أن اﻟﻨﻘﻂ ‪ C ، B ، A‬و ‪ D‬ﺗﻘﻊ ﻋﲆ داﺋﺮة ُﻳﻄﻠﺐ ﺗﻌﻴﲔ ﻣﺮﻛﺰﻫﺎ و ﻧﺼﻒ ﻗﻄﺮﻫﺎ‪.‬‬
‫‪ņőƱŀ Ţ‬‬
‫‪ •1‬أﻧﻈﺮ اﻟﺸﻜﻞ ‪. 5.VI‬‬ ‫ﺍﳊﻞّ‪.‬‬
‫‪ •2‬ﻟﺪﻳﻨﺎ ‪:‬‬
‫‪VI‬‬

‫‪c−a‬‬ ‫‪2ı + 1 − ı‬‬ ‫‪1+ı‬‬ ‫)‪(1 + ı‬‬ ‫‪2ı 1‬‬


‫=‬ ‫=‬ ‫=‬ ‫=‬ ‫‪= ı‬‬ ‫)اﻟﺸﻜﻞ اﳉﱪي(‬
‫‪d − a 2 − 2ı + 1 − ı 3 − 3ı 3 1 + 1‬‬ ‫‪6‬‬ ‫‪3‬‬

‫‪1‬‬
‫‪= e /‬‬ ‫)اﻟﺸﻜﻞ اﻷﳼ(‬
‫‪3‬‬
‫‪ƕ‬‬

‫‪c−b‬‬ ‫‪2ı + 1 + ı‬‬ ‫)‪1 + 3ı ı (3 − ı‬‬


‫=‬ ‫=‬ ‫=‬ ‫‪=ı‬‬ ‫)اﻟﺸﻜﻞ اﳉﱪي(‬
‫‪d − b 2 − 2ı + 1 + ı‬‬ ‫‪3−ı‬‬ ‫‪3−ı‬‬
‫‪=e /‬‬ ‫)اﻟﺸﻜﻞ اﻷﳼ(‬
‫‪ũŏ‬‬

‫‪c−a‬‬ ‫𝜋‬
‫ّ‬
‫ﻓﺈن اﳌﺜﻠﺚ ‪ ACD‬ﻗﺎﺋﻢ اﻟﺰاوﻳﺔ ﻋﻨﺪ اﻟﺮأس ‪. A‬‬ ‫‪AD, AC = arg‬‬ ‫ﺑﲈ ّ‬
‫أن )‪= (mod 2ı‬‬ ‫•‬‫‪3‬‬
‫‪d−a‬‬ ‫‪2‬‬
‫‪c−b‬‬ ‫𝜋‬
‫‪ BD, BC = arg‬إذن ﻓﺎﳌﺜﻠﺚ ‪ BCD‬ﻗﺎﺋﻢ اﻟﺰاوﻳﺔ ﻋﻨﺪ اﻟﺮأس ‪. B‬‬ ‫ﺑﺎﳌﺜﻞ‪= (mod 2ı) ،‬‬
‫‪d−b‬‬ ‫‪2‬‬
‫‪c+d‬‬
‫= ‪ z‬و ‪. ID = ||d − z || = ||1 − 2ı|| = √5‬‬ ‫‪ •4‬ﻟﺘﻜﻦ ‪ I‬ﻣﻨﺘﺼﻒ اﻟﻘﻄﻌﺔ ]‪ . [CD‬ﻻﺣﻘﺘﻬﺎ ﻫﻲ ‪= 1‬‬
‫‪2‬‬

‫𝟯𝟬𝟭‬

‫‪http ://tinyurl.com/Malki1718‬‬ ‫‪0‬‬


‫‪ .1.VI‬ﶍﻮﻋﺔ اﻷﻋﺪاد اﳌﺮﻛﺒﺔ‬
‫‪.‬‬
‫اﻟﻘﻄﻌﺔ اﳌﺴﺘﻘﻴﻤﺔ ]‪ [CD‬وﺗﺮ ﰲ اﳌﺜﻠﺚ اﻟﻘﺎﺋﻢ ‪ ACD‬و ﺑﺎﻟﺘﺎﱄ ‪ IA = IC = ID‬أي ّ‬
‫أن اﻟﻨﻘﻂ ‪ C ، A‬و ‪ D‬ﺗﻨﺘﻤﻲ ﻛﻠﻬﺎ‬
‫إﱃ اﻟﺪاﺋﺮة اﻟﺘﻲ ﻣﺮﻛﺰﻫﺎ ‪ I‬و ﻧﺼﻒ ﻗﻄﺮﻫﺎ ‪. ID‬‬

‫‪Ŕž Ŧ Ľ‬‬
‫ﺑﺎﳌﺜﻞ‪ [CD] ،‬وﺗﺮ ﰲ اﳌﺜﻠﺚ اﻟﻘﺎﺋﻢ ‪ BCD‬ﻣﻨﻪ ‪ IB = IC = ID‬أي ّ‬
‫أن اﻟﻨﻘﻂ ‪ C ، B‬و ‪ D‬ﺗﻨﺘﻤﻲ إﱃ اﻟﺪاﺋﺮة اﻟﺘﻲ ﻣﺮﻛﺰﻫﺎ‬
‫‪ I‬و ﻧﺼﻒ ﻗﻄﺮﻫﺎ ‪. ID‬‬

‫‪œǃ‬‬
‫إذن اﻟﻨﻘﻂ ‪ C ، B ، A‬و ‪ D‬ﺗﻨﺘﻤﻲ إﱃ اﻟﺪاﺋﺮة اﻟﺘﻲ ﻣﺮﻛﺰﻫﺎ ‪ I‬و ﻧﺼﻒ ﻗﻄﺮﻫﺎ ‪. √5‬‬
‫)ﻳﻤﻜﻦ أﻳﻀ ًﺎ اﺳﺘﺨﺪام اﻟﺰواﻳﺎ ‪ CID = 𝜋 = 2CAD = 2CBD :‬إذن ﻓﺎﻟﺮﺑﺎﻋﻲ ‪ ABDC‬رﺑﺎﻋﻲ داﺋﺮي( ‪.‬‬
‫■‬

‫‪Ŀ‬‬
‫‪ ،‬ﻧﻌﺘﱪ اﻟﻨﻘﻂ ‪ B ، A‬و ‪ C‬اﻟﺘﻲ ﻟﻮاﺣﻘﻬﺎ ﻫﻲ‬ ‫⃗ ‪O, i,⃗ j‬‬
‫ﰲ اﳌﺴﺘﻮي اﳌﻨﺴﻮب إﱃ ﻣﻌﻠﻢ ﻣﺘﻌﺎﻣﺪ و ﻣﺘﺠﺎﻧﺲ‬
‫‬ ‫‪𝟑𝟔.‬‬ ‫‪.‬‬

‫‪ Z = √2 + √2ı ، Z = √3 + ı‬و ‪ Z = 1 + √3ı‬ﻋﲆ اﻟﱰﺗﻴﺐ‪.‬‬


‫أﺛﺒﺖ ّ‬
‫أن اﻟﻨﻘﻂ ‪ B ، A‬و ‪ C‬ﺗﻘﻊ ﻋﲆ داﺋﺮة ﻣﺮﻛﺰﻫﺎ ‪ُ O‬ﻳﻄﻠﺐ ﺗﻌﻴﲔ ﻧﺼﻒ ﻗﻄﺮﻫﺎ ‪. R‬‬

‫ﺍﳊﻞّ‪ .‬ﺣﺘﻰ ﻧﱪﻫﻦ ّ‬


‫أن اﻟﻨﻘﻂ ‪ B ، A‬و ‪ C‬ﺗﻨﺘﻤﻲ إﱃ داﺋﺮة ﻣﺮﻛﺰﻫﺎ ‪ O‬ﻳﻜﻔﻲ أن ﻧﱪﻫﻦ ّ‬
‫أن ‪ . OA = OB = OC‬ﻟﻜﻦ ‪:‬‬

‫= ||‪OA = ||Z − Z || = ||√3 + ı‬‬ ‫‪√3 + 1 = 2‬‬

‫= ||‪OB = ||Z − Z || = ||√2 + ı√2‬‬ ‫‪√2 + √2 = 2‬‬

‫= ||‪OC = ||Z − Z || = ||1 + ı√3‬‬ ‫‪1 + √3 = 2‬‬

‫■‬ ‫إذن ‪ OA = OB = OC‬و ﻫﺬا ﻳﻌﻨﻲ ّ‬


‫أن اﻟﻨﻘﻂ ‪ B ، A‬و ‪ C‬ﺗﻘﻊ ﻋﲆ اﻟﺪاﺋﺮة اﻟﺘﻲ ﻣﺮﻛﺰﻫﺎ ‪ O‬و ﻧﺼﻒ ﻗﻄﺮﻫﺎ ‪. R = 2‬‬

‫‬
‫‪𝟑𝟕. .‬‬
‫‪ņőƱŀ Ţ‬‬
‫ﰲ اﳌﺴﺘﻮي اﳌﻨﺴﻮب إﱃ ﻣﻌﻠﻢ ﻣﺘﻌﺎﻣﺪ و ﻣﺘﺠﺎﻧﺲ ⃗ ‪ ، O, i,⃗ j‬ﻧﻌﺘﱪ اﻟﻨﻘﻂ ‪ B ، A‬و ‪ C‬اﻟﺘﻲ ﻟﻮاﺣﻘﻬﺎ ﻫﻲ‬
‫‪ Z = 4 + ı ، Z = −ı‬و ‪ Z = 2 − 5ı‬ﻋﲆ اﻟﱰﺗﻴﺐ‪.‬‬
‫‪ •1‬ﻋﻠﻢ اﻟﻨﻘﻂ ‪ B ، A‬و ‪ C‬ﰲ ﻫﺬا اﳌﻌﻠﻢ‪.‬‬

‫‪VI‬‬
‫‪ •2‬أﺣﺴﺐ اﻷﻋﺪاد ‪ Z ، Z‬و ‪ Z‬ﻟﻮاﺣﻖ اﻷﺷﻌﺔ ‪ BC ، AB‬و ‪ CA‬ﻋﲆ اﻟﱰﺗﻴﺐ‪.‬‬
‫‪ •3‬أﺣﺴﺐ ﻃﻮﻳﻠﺔ ﻛﻞ ﻣﻦ ‪ Z ، Z‬و ‪ . Z‬ﻣﺎذا ﻳﻤﻜﻦ ﻗﻮﻟﻪ ﻋﻦ اﳌﺜﻠﺚ ‪ ABC‬؟‬
‫‪ƕ‬‬

‫‪ •1‬أﻧﻈﺮ اﻟﺸﻜﻞ ‪. 6.VI‬‬ ‫ﺍﳊﻞّ‪.‬‬


‫‪ •2‬ﻟﺪﻳﻨﺎ ‪:‬‬
‫‪ũŏ‬‬

‫‪Z =Z −Z‬‬ ‫‪= 4 + ı + ı = 4 + 2ı‬‬


‫‪Z = Z − Z = 2 − 5ı − 4 − ı = −2 − 6ı‬‬
‫‪Z = Z − Z = −ı − 2 + 5ı = −2 + 4ı‬‬

‫𝟰𝟬𝟭‬

‫‪http ://tinyurl.com/Malki1718‬‬ ‫‪0‬‬


‫‪ .VI‬ﲤﺎرﻳﻦ ﺗﻄﺒﻴﻘﻴﺔ‬
‫‪.‬‬
‫‪B‬‬
‫‪1‬‬

‫‪Ŕž Ŧ Ľ‬‬
‫‪1‬‬ ‫‪2‬‬ ‫‪3‬‬ ‫‪4‬‬

‫‪œǃ‬‬
‫‪A‬‬
‫‪−1‬‬

‫‪−2‬‬

‫‪Ŀ‬‬
‫‪−3‬‬

‫‪−4‬‬

‫‪−5‬‬
‫‪C‬‬

‫ﺷﻜﻞ ‪6.VI‬‬

‫‪ •3‬ﻟﺪﻳﻨﺎ ‪:‬‬

‫= || ‪||Z‬‬ ‫‪4 + 2 = √20 = 2√5‬‬

‫= || ‪||Z‬‬ ‫‪(−2) + (−6) = √40 = 2√10‬‬

‫= || ‪||Z‬‬ ‫‪(−2) + 4 = √20 = 2√5‬‬

‫ﻧﻼﺣﻆ ّ‬
‫أن || ‪ ||Z || = ||Z‬أي ‪ AB = AC‬و ﺑﺎﻟﺘﺎﱄ ﻓﺎﳌﺜﻠﺚ ‪ ABC‬ﻣﺘﺴﺎوي اﻟﺴﺎﻗﲔ ﻋﻨﺪ اﻟﺮأس ‪. A‬‬
‫ﻟﻜﻦ || ‪ ||Z || + ||Z || = ||Z‬أي ‪ AB + AC = BC‬إذن اﳌﺜﻠﺚ ‪ ABC‬ﻗﺎﺋﻢ اﻟﺰاوﻳﺔ ﻋﻨﺪ اﻟﺮأس ‪. A‬‬
‫‪ņőƱŀ Ţ‬‬
‫ﰲ اﻷﺧﲑ ‪ :‬اﳌﺜﻠﺚ ‪ ABC‬ﻗﺎﺋﻢ و ﻣﺘﺴﺎوي اﻟﺴﺎﻗﲔ ﻋﻨﺪ اﻟﺮأس ‪. A‬‬
‫■‬
‫‪VI‬‬

‫‪.‬‬
‫𝟖𝟑‬ ‫‪.‬‬
‫‪1 √3‬‬
‫ﻟﺘﻜﻦ ‪ B ، A‬و ‪ C‬اﻟﻨﻘﻂ اﻟﺘﻲ ﻟﻮاﺣﻘﻬﺎ ‪ 1 ، 2ı‬و ‪ + ı‬ﻋﲆ اﻟﱰﺗﻴﺐ‪.‬‬
‫‪2‬‬ ‫‪2‬‬
‫ﻟﺘﻜﻦ ‪ f‬اﻟﺪاﻟﺔ اﳌﻌﺮﻓﺔ ﻋﲆ ‪ ℂ‬ﺑﺎﻟﻌﺒﺎرة ‪ f (z) = e / z‬و ‪ T‬اﻟﺘﺤﻮﻳﻞ اﻟﻨﻘﻄﻲ اﳌﺮﻓﻖ ﲠﺎ‪.‬‬

‫‪ •1‬ﻋﻠﻢ اﻟﻨﻘﻂ ‪ B ، A‬و ‪ C‬ﰲ ﻣﻌﻠﻢ ⃗ ‪. O, i,⃗ j‬‬


‫‪ƕ‬‬

‫‪ •2‬ﻣﺎ ﻫﻲ ﻃﺒﻴﻌﺔ اﻟﺘﺤﻮﻳﻞ ‪ T‬؟ و ﻣﺎ ﻫﻲ ﻋﻨﺎﴏه اﳌﻤ ﱢﻴﺰه ؟‬


‫‪ •3‬ﻟﺘﻜﻦ ‪ Γ‬ﳎﻤﻮﻋﺔ اﻟﻨﻘﻂ ‪ M‬ذات اﻟﻼﺣﻘﺔ ‪ z‬ﺑﺤﻴﺚ ‪. ||z − 2ı|| = 2 :‬‬
‫‪ũŏ‬‬

‫ﺛﻢ أﻧﺸﺊ اﳌﺠﻤﻮﻋﺔ ‪ . Γ‬أﻛﺘﺐ ﻣﻌﺎدﻟﺘﻬﺎ‪.‬‬


‫)ا( ﺣﺪّ د ّ‬
‫ﺛﻢ أﻧﺸﺊ اﳌﺠﻤﻮﻋﺔ ‪ Γ‬ﺻﻮرة ‪ Γ‬ﺑﺎﻟﺘﺤﻮﻳﻞ ‪) T‬ﻻ ُﻳﻄﻠﺐ إﳚﺎد ﻣﻌﺎدﻟﺘﻬﺎ( ‪.‬‬
‫)ب( ﺣﺪّ د ّ‬
‫|‬ ‫‪1‬‬ ‫| ‪3‬‬
‫‪ •4‬ﻟﺘﻜﻦ ‪ Δ‬ﳎﻤﻮﻋﺔ اﻟﻨﻘﻂ ‪ M‬ذات اﻟﻼﺣﻘﺔ ‪ z‬ﺑﺤﻴﺚ ‪. ||z − 1|| = |z − − √ ı| :‬‬
‫||‬ ‫‪2‬‬ ‫|| ‪2‬‬

‫𝟱𝟬𝟭‬

‫‪http ://tinyurl.com/Malki1718‬‬ ‫‪0‬‬


‫‪ .1.VI‬ﶍﻮﻋﺔ اﻷﻋﺪاد اﳌﺮﻛﺒﺔ‬
‫‪.‬‬
‫)‪∆′ = (Oy‬‬

‫‪Ŕž Ŧ Ľ‬‬
‫‪4‬‬
‫‪Γ‬‬

‫‪œǃ‬‬
‫‪3‬‬
‫‪Γ′‬‬
‫‪A‬‬
‫‪2‬‬

‫‪Ŀ‬‬
‫‪A′‬‬ ‫‪C′‬‬ ‫‪C = B′‬‬
‫‪+ π3‬‬ ‫‪I‬‬
‫‪B‬‬
‫‪−4‬‬ ‫‪−3‬‬ ‫‪−2‬‬ ‫‪−1‬‬ ‫‪O‬‬ ‫‪1‬‬ ‫‪2‬‬ ‫‪3‬‬

‫‪−1‬‬
‫∆‬

‫ﺷﻜﻞ ‪7.VI‬‬

‫ﺛﻢ أﻧﺸﺊ اﳌﺠﻤﻮﻋﺔ ‪ . Δ‬أﻛﺘﺐ ﻣﻌﺎدﻟﺘﻬﺎ‪.‬‬


‫)ا( ﺣﺪّ د ّ‬
‫ﺛﻢ أﻧﺸﺊ اﳌﺠﻤﻮﻋﺔ ‪ Δ‬ﺻﻮرة ‪ Δ‬ﺑﺎﻟﺘﺤﻮﻳﻞ ‪) T‬ﻻ ُﻳﻄﻠﺐ إﳚﺎد ﻣﻌﺎدﻟﺘﻬﺎ( ‪.‬‬
‫)ب( ﺣﺪّ د ّ‬

‫ﺍﳊﻞّ‪ .‬ﻧُﺬﻛﱢﺮ ّ‬
‫أن اﻟﺪوران ﺗﺴﺎوي ﻗﻴﺎس و ﺑﺎﻟﺘﺎﱄ ﻓﺼﻮرة أي ﺷﻜﻞ ﻫﻨﺪﳼ ﻫﻮ ﺷﻜﻞ ﻫﻨﺪﳼ ُﳑَﺎﺛِﻞ )ﺻﻮرة داﺋﺮة ﻫﻲ داﺋﺮة و‬
‫ﺻﻮرة ﻣﺴﺘﻘﻴﻢ ﻫﻲ ﻣﺴﺘﻘﻴﻢ ‪ ...‬إﻟﺦ( ‪.‬‬
‫‪ņőƱŀ Ţ‬‬
‫‪ •1‬أﻧﻈﺮ اﻟﺸﻜﻞ ‪. 7.VI‬‬
‫‪/‬‬ ‫𝜋‬
‫‪ a = e‬و ﺑﺎﻟﺘﺎﱄ ‪ T‬دوران‪.‬‬ ‫‪= 1,‬‬ ‫‪ •2‬ﻋﺒﺎرة اﻟﺘﺤﻮﻳﻞ ‪ T‬ﻣﻦ اﻟﺸﻜﻞ ‪ f (z) = az + b‬ﺣﻴﺚ‬
‫‪3‬‬

‫‪VI‬‬
‫‪ 𝜔 = e‬أي ‪. 𝜔 = 0‬‬ ‫ﻻﺣﻘﺔ ﻣﺮﻛﺰ ﻫﺬا اﻟﺪوران 𝜔 ُﲢﻘﻖ ‪𝜔 :‬‬
‫‪/‬‬

‫𝜋‬ ‫𝜋‬
‫‪.‬‬ ‫إذن ‪ T‬ﻫﻮ اﻟﺪوران ‪ ℛ O,‬اﻟﺬي ﻣﺮﻛﺰه )‪ O (0, 0‬و زاوﻳﺘﻪ‬
‫‪3‬‬ ‫‪3‬‬
‫)ا( ﻟﺘﻜﻦ ‪ M‬ﺻﻮرة ‪ z‬ﰲ اﳌﺴﺘﻮي‪ .‬ﻟﺪﻳﻨﺎ ‪:‬‬ ‫•‬ ‫‪3‬‬
‫‪M ∈ Γ ⟺ MA = 2‬‬
‫‪ƕ‬‬

‫إذن ‪ Γ‬ﻫﻲ ﳎﻤﻮﻋﺔ اﻟﻨﻘﻂ اﻟﺘﻲ ﺗﺒﻌﺪ ﺑﻤﺴﺎﻓﺔ ‪ 2‬ﻋﻦ اﻟﻨﻘﻄﺔ ‪ A‬أي ‪ Γ‬ﻫﻲ اﻟﺪاﺋﺮة )‪ 𝒞 (A, 2‬اﻟﺘﻲ ﻣﺮﻛﺰﻫﺎ ‪ A‬و‬
‫ﻧﺼﻒ ﻗﻄﺮﻫﺎ ‪) 2‬ﺷﻜﻞ ‪. (7.VI‬‬
‫ﻣﻌﺎدﻟﺔ ‪ Γ‬ﻫﻲ ‪ (x − 0) + (y − 2) = 2 :‬أي ‪. x + y − 4y = 0‬‬
‫‪ũŏ‬‬

‫)ب( ‪ Γ‬ﻫﻲ اﻟﺪاﺋﺮة اﻟﺘﻲ ﻣﺮﻛﺰﻫﺎ )‪ A = T (A‬و ﻧﺼﻒ ﻗﻄﺮﻫﺎ ‪ . 2‬ﻻﺣﻘﺔ اﻟﻨﻘﻄﺔ ‪ A‬ﻫﻲ ‪:‬‬
‫‪/‬‬ ‫‪/‬‬ ‫‪/‬‬ ‫‪+‬‬ ‫‪/‬‬ ‫‪√3 1‬‬
‫‪e‬‬ ‫‪⋅ 2ı = 2e‬‬ ‫‪⋅e‬‬ ‫‪= 2e‬‬ ‫‪= 2e‬‬ ‫‪=2 −‬‬ ‫‪+ ı‬‬ ‫‪= −√ 3 + ı‬‬
‫‪2‬‬ ‫‪2‬‬

‫إذن ‪ Γ‬ﻫﻲ اﻟﺪاﺋﺮة ‪ 𝒞 A , 2‬اﻟﺘﻲ ﻣﺮﻛﺰﻫﺎ ‪ A −√3, 1‬و ﻧﺼﻒ ﻗﻄﺮﻫﺎ ‪. 2‬‬

‫𝟲𝟬𝟭‬

‫‪http ://tinyurl.com/Malki1718‬‬ ‫‪0‬‬


‫‪ .VI‬ﲤﺎرﻳﻦ ﺗﻄﺒﻴﻘﻴﺔ‬
‫‪.‬‬
‫)ا( ﻟﺘﻜﻦ ‪ M‬ﺻﻮرة ‪ z‬ﰲ اﳌﺴﺘﻮي‪ .‬ﻟﺪﻳﻨﺎ ‪:‬‬ ‫‪4‬‬
‫•‬
‫‪M ∈ Δ ⟺ MB = MC‬‬

‫‪Ŕž Ŧ Ľ‬‬
‫و ﻫﻲ ﳎﻤﻮﻋﺔ اﻟﻨﻘﻂ اﳌﺘﺴﺎوﻳﺔ اﻟﺒﻌﺪ ﻋﻦ اﻟﻨﻘﻄﺘﲔ ‪ B‬و ‪ C‬إذن ‪ Δ‬ﻫﻮ ﳏﻮر اﻟﻘﻄﻌﺔ اﳌﺴﺘﻘﻴﻤﺔ]‪. [BC‬‬
‫‪z +z‬‬ ‫‪3 √3‬‬

‫‪œǃ‬‬
‫= ‪.z‬‬ ‫=‬ ‫‪+‬‬ ‫ﻟﺘﻜﻦ ‪ I‬ﻣﻨﺘﺼﻒ ]‪ . [BC‬ﻻﺣﻘﺘﻬﺎ ﻫﻲ ‪ı‬‬
‫‪2‬‬ ‫‪4‬‬ ‫‪4‬‬
‫‪1‬‬ ‫‪3‬‬
‫إذن ‪ Δ‬ﻫﻮ اﳌﺴﺘﻘﻴﻢ اﻟﺬي ﻳﻤﺮ ﻣﻦ ‪ I‬و ُﻳﻌﺎﻣﺪ اﻟﺸﻌﺎع ‪ BC‬اﻟﺬي ﻻﺣﻘﺘﻪ √ ‪ . z − z = − + ı‬ﻟﺪﻳﻨﺎ ‪:‬‬
‫‪2‬‬ ‫‪2‬‬

‫‪Ŀ‬‬
‫‪M ∈ Δ ⟺ IM ⟂ BC‬‬
‫‪3‬‬ ‫‪1‬‬ ‫‪√3‬‬ ‫‪√3‬‬
‫⟺‬ ‫‪x−‬‬ ‫‪⋅ −‬‬ ‫‪+‬‬ ‫‪y−‬‬ ‫⋅‬ ‫‪=0‬‬
‫‪4‬‬ ‫‪2‬‬ ‫‪4‬‬ ‫‪2‬‬
‫‪⟺ x − √3y = 0‬‬

‫إذن ﻣﻌﺎدﻟﺔ ‪ Δ‬ﻫﻲ ‪. x − √3y = 0 :‬‬


‫)ب( ‪ Δ‬ﻫﻮ ﳏﻮر اﻟﻘﻄﻌﺔ ‪ B C‬ﺣﻴﺚ )‪ B = T (B‬و )‪ . C = T (C‬ﻟﺪﻳﻨﺎ ‪:‬‬

‫‪/‬‬ ‫‪/‬‬ ‫‪1‬‬ ‫‪√3‬‬


‫‪z‬‬ ‫‪′‬‬ ‫‪= f (z ) = e‬‬ ‫‪⋅1=e‬‬ ‫=‬ ‫‪+ı‬‬
‫‪2‬‬ ‫‪2‬‬
‫‪/‬‬ ‫‪1‬‬ ‫‪√3‬‬ ‫‪/‬‬ ‫‪/‬‬ ‫‪/‬‬ ‫‪1‬‬ ‫‪√3‬‬
‫‪z‬‬ ‫‪′‬‬ ‫‪= f (z ) = e‬‬ ‫⋅‬ ‫‪+ı‬‬ ‫‪=e‬‬ ‫‪⋅e‬‬ ‫‪=e‬‬ ‫‪=− +ı‬‬
‫‪2‬‬ ‫‪2‬‬ ‫‪2‬‬ ‫‪2‬‬

‫)أﻧﻈﺮ اﻟﺸﻜﻞ ‪. (7.VI‬‬


‫■‬

‫اﳌﺴﺘﻮي ﻣﻨﺴﻮب إﱃ ﻣﻌﻠﻢ ﻣﺘﻌﺎﻣﺪ ﻣﺘﺠﺎﻧﺲ و ﻣﺒﺎﴍ ⃗ ‪. O, i,⃗ j‬‬


‫‪.‬‬
‫𝟗𝟑‬ ‫‪.‬‬ ‫!‬
‫‪ņőƱŀ Ţ‬‬
‫ﻟﺘﻜﻦ ‪ A‬اﻟﻨﻘﻄﺔ ذات اﻟﻼﺣﻘﺔ ‪ . ı‬ﻧﺮﻓﻖ ﺑﻜﻞ ﻧﻘﻄﺔ ‪ M‬ﻣﻦ اﳌﺴﺘﻮي ﲣﺘﻠﻒ ﻋﻦ ‪ A‬و اﻟﺘﻲ ﻻﺣﻘﺘﻬﺎ ‪ ، z‬اﻟﻨﻘﻄﺔ ‪ M‬ذات‬
‫‪z‬‬
‫= ‪.z‬‬ ‫اﻟﻼﺣﻘﺔ ‪ z‬ﺑﺤﻴﺚ‬
‫‪ı−z‬‬
‫‪VI‬‬

‫‪ •1‬أوﺟﺪ اﻟﻨﻘﻂ ‪ M‬اﻟﺼﺎﻣﺪة أي ﺑﺤﻴﺚ ‪. M = M‬‬


‫‪ •2‬ﻟﻴﻜﻦ ‪ z‬ﻋﺪد ًا ﻣﺮﻛﺒﺎ ﳜﺘﻠﻒ ﻋﻦ ‪ . ı‬ﻧﻀﻊ ‪ z = x + ıy‬و ‪ z = x + ıy‬ﻣﻊ ‪ x ، y ، x‬و ‪ y‬أﻋﺪاد ﺣﻘﻴﻘﻴﺔ‪ .‬أﺛﺒﺖ ّ‬
‫أن ‪:‬‬

‫‪−x x + y − 2y‬‬
‫= ‪x‬‬
‫‪ƕ‬‬

‫)‪x + (1 − y‬‬

‫ﺛﻢ أﻧﺸﺊ اﳌﺠﻤﻮﻋﺔ ‪. E‬‬


‫إﺳﺘﻨﺘﺞ اﳌﺠﻤﻮﻋﺔ ‪ E‬ﻟﻠﻨﻘﻂ ‪ M‬اﻟﺘﻲ ﺗﻘﻊ ﺻﻮرﻫﺎ ‪ M‬ﻋﲆ ﳏﻮر اﻷﻋﺪاد اﻟﺘﺨﻴﻠﻴﺔ اﻟﴫﻓﺔ ّ‬
‫‪ũŏ‬‬

‫‪ •3‬أوﺟﺪ ﻋﻼﻗﺔ ﺑﺴﻴﻄﺔ ﺗﺮﺑﻂ ﺑﲔ اﻷﻃﻮال ‪ AM ، OM‬و ‪. OM‬‬


‫ﺛﻢ أﻧﺸﺊ اﳌﺠﻤﻮﻋﺔ ‪. F‬‬
‫إﺳﺘﻨﺘﺞ اﳌﺠﻤﻮﻋﺔ ‪ F‬ﻟﻠﻨﻘﻂ ‪ M‬ﺑﺤﻴﺚ ﺗﻘﻊ ‪ M‬و ‪ M‬ﻋﲆ داﺋﺮة ﻣﺮﻛﺰﻫﺎ ‪ّ O‬‬
‫‪1‬‬
‫‪ •4‬ﰲ ﻫﺬا اﻟﺴﺆال‪ ،‬ﻧﻌﺘﱪ ﻧﻘﻄﺔ ‪ M‬ﻻﺣﻘﺘﻬﺎ ‪ z‬و ﺗﻘﻊ ﻋﲆ اﻟﺪاﺋﺮة اﻟﺘﻲ ﻣﺮﻛﺰﻫﺎ ‪ A‬و ﻧﺼﻒ ﻗﻄﺮﻫﺎ ‪.‬‬
‫‪2‬‬
‫ﻟﺘﻜﻦ ‪ M‬اﻟﻨﻘﻄﺔ اﻟﺘﻲ ﻻﺣﻘﺘﻬﺎ ‪ z‬و ‪ G‬ﻣﺮﻛﺰ اﳌﺴﺎﻓﺎت اﳌﺘﺴﺎوﻳﺔ ﻟﻠﻨﻘﻂ ‪.M ، M ، A‬‬

‫𝟳𝟬𝟭‬

‫‪http ://tinyurl.com/Malki1718‬‬ ‫‪0‬‬


‫‪ .1.VI‬ﶍﻮﻋﺔ اﻷﻋﺪاد اﳌﺮﻛﺒﺔ‬
‫‪.‬‬

‫)ا( أﺣﺴﺐ ‪، z‬ﻻﺣﻘﺔ اﻟﻨﻘﻄﺔ ‪ ، G‬ﺑﺪﻻﻟﺔ ‪. z‬‬

‫‪Ŕž Ŧ Ľ‬‬
‫)ب( أﺛﺒﺖ ّ‬
‫أن ‪ G‬ﺗﻘﻊ ﻋﲆ داﺋﺮة ﻣﺮﻛﺰﻫﺎ ‪ُ O‬ﻳﻄﻠﺐ ﺗﻌﻴﲔ ﻧﺼﻒ ﻗﻄﺮﻫﺎ‪.‬‬
‫)ج( ﺑﻤﻘﺎرﻧﺔ اﻟﺰاوﻳﺘﲔ ‪ i,⃗ OG‬و ‪ ، i,⃗ AM‬إﻗﱰح إﻧﺸﺎء ًا ﻟﻠﻨﻘﻄﺔ ‪. G‬‬

‫‪œǃ‬‬
‫)د( إﺳﺘﻨﺘﺞ إﻧﺸﺎء ًا ﻟﻠﻨﻘﻄﺔ ‪. M‬‬

‫‪Ŀ‬‬
‫ﺍﳊﻞّ‪.‬‬
‫‪z‬‬
‫أي‬ ‫‪ •1‬ﻟﺘﻜﻦ )‪ . M (z‬ﻟﺪﻳﻨﺎ ‪ M = M :‬إذا و ﻓﻘﻂ إذا ﻛﺎن ‪ z = z‬أي إذا و ﻓﻘﻂ إذا ﻛﺎن ‪= z‬‬
‫‪ı−z‬‬
‫‪ı‬‬ ‫‪ı‬‬
‫‪ z = ız − z‬أي ‪ . z = z‬ﻣﻨﻪ ‪ z = 0‬أو = ‪. z‬‬
‫‪2‬‬ ‫‪2‬‬
‫‪1‬‬
‫‪.M‬‬ ‫إذن ﺗﻮﺟﺪ ﻧﻘﻄﺘﺎن ﺻﺎﻣﺪﺗﺎن ﳘﺎ )‪ M (0‬و ‪ı‬‬
‫‪2‬‬
‫‪ •2‬ﻟﺪﻳﻨﺎ ‪:‬‬
‫‪z‬‬ ‫)‪(x + ıy‬‬ ‫‪x − y + 2ıxy‬‬ ‫))‪x − y + 2ıxy (x + ı (1 − y‬‬
‫= ‪z‬‬ ‫=‬ ‫=‬ ‫=‬
‫‪ı − z ı − x − ıy‬‬ ‫)‪x + ı (1 − y‬‬ ‫)‪x + (1 − y‬‬
‫‪2xy − xy − x − ı y − y + x y + x‬‬ ‫‪−x x + y − 2y‬‬ ‫‪−y + y − x y − x‬‬
‫=‬ ‫=‬ ‫‪+ı‬‬
‫)‪x + (1 − y‬‬ ‫)‪x + (1 − y‬‬ ‫)‪x + (1 − y‬‬

‫‪−x x + y − 2y‬‬
‫= ‪. x = Re z‬‬ ‫ﻣﻨﻪ ‪:‬‬
‫)‪x + (1 − y‬‬
‫اﻟﻨﻘﻄﺔ ‪ M‬ﺗﻘﻊ ﻋﲆ ﳏﻮر اﻷﻋﺪاد اﻟﺘﺨﻴﻠﻴﺔ اﻟﴫﻓﺔ إذا و ﻓﻘﻂ إذا ﻛﺎن ‪ Re z = 0‬ﻣﻊ ‪ z ≠ ı‬أي إذا و ﻓﻘﻂ إذا ﻛﺎن‬
‫‪ −x x + y − 2y = 0‬ﻣﻊ )‪. (x, y) ≠ (0, 1‬‬
‫ﻟﻜﻦ ‪ −x x + y − 2y = 0‬إذا و ﻓﻘﻂ إذا ﻛﺎن ‪ x = 0‬أو ‪ x + y − 2y = 0‬أي إذا و ﻓﻘﻂ إذا ﻛﺎن ‪ x = 0‬أو‬
‫‪ x + (y − 1) = 1‬و ﺑﺎﻟﺘﺎﱄ اﳌﺠﻤﻮﻋﺔ ‪ E‬ﻫﻲ ّاﲢﺎد اﳌﺴﺘﻘﻴﻢ اﻟﺬي ﻣﻌﺎدﻟﺘﻪ ‪ ، x = 0‬ﺑﺎﺳﺘﺜﻨﺎء اﻟﻨﻘﻄﺔ )‪ ، A (0, 1‬ﻣﻊ‬
‫‪ņőƱŀ Ţ‬‬
‫اﻟﺪاﺋﺮة 𝒞 اﻟﺘﻲ ﻣﺮﻛﺰﻫﺎ )‪ A (0, 1‬و ﻧﺼﻒ ﻗﻄﺮﻫﺎ ‪. 1‬‬

‫||‪|z − 0‬‬ ‫‪z‬‬


‫| = ||‪ ||z − 0‬أي ||‪ ||z − 0|| = |ı − z| ⋅ ||z − 0‬و ﻫﺬا ﻳﻌﻨﻲ ّ‬
‫أن ‪:‬‬ ‫= ‪ z‬ﻣﻨﻪ‬ ‫‪ •3‬ﻟﺪﻳﻨﺎ ‪:‬‬
‫|‪|ı − z‬‬ ‫‪ı−z‬‬

‫‪VI‬‬
‫‪. OM = AM × OM‬‬
‫ﺣﺘﻰ ﺗﻘﻊ ‪ M‬و ‪ M‬ﻋﲆ داﺋﺮة ﻣﺮﻛﺰﻫﺎ ‪ O‬ﳚﺐ أن ﻳﻜﻮن ‪ OM = OM‬و ﺑﺎﻟﺘﺎﱄ ‪ OM = AM × OM‬و ﺑﲈ ّ‬
‫أن‬
‫أن اﻟﻨﻘﻄﺔ ‪ M‬ﺗﻨﺘﻤﻲ إﱃ ﳏﻮر اﻟﻘﻄﻌﺔ اﳌﺴﺘﻘﻴﻤﺔ ]‪) [OA‬اﻟﻨﻘﻄﺔ ‪ A‬ﻻ ﺗﻨﺘﻤﻲ‬ ‫أن ‪ OM = AM‬أي ّ‬ ‫‪ OM ≠ 0‬ﻓﻬﺬا ﻳﻌﻨﻲ ّ‬
‫إﻟﻴﻪ(‪.‬‬
‫‪ƕ‬‬

‫)ا( ﻟﺪﻳﻨﺎ ‪:‬‬ ‫•‬ ‫‪4‬‬


‫‪1‬‬ ‫‪1‬‬ ‫‪z‬‬ ‫‪(ı + z) (ı − z) + z‬‬ ‫‪−1‬‬
‫= ‪z‬‬ ‫‪z +z+z‬‬ ‫=‬ ‫‪ı+z+‬‬ ‫=‬ ‫=‬
‫‪3‬‬ ‫‪3‬‬ ‫‪ı−z‬‬ ‫)‪3 (ı − z‬‬ ‫)‪3 (ı − z‬‬
‫‪ũŏ‬‬

‫‪1‬‬
‫= |‪|z − ı‬‬ ‫أن ‪ M‬ﺗﻨﺘﻤﻲ إﱃ اﻟﺪاﺋﺮة اﻟﺘﻲ ﻣﺮﻛﺰﻫﺎ ‪ A‬و ﻧﺼﻒ ﻗﻄﺮﻫﺎ ‪ّ 1‬‬
‫ﻓﺈن‬ ‫)ب( ﺑﲈ ّ‬
‫‪2‬‬ ‫‪2‬‬
‫‪1‬‬
‫أي = )‪ x + (y − 1‬ﻣﻨﻪ ‪:‬‬
‫‪4‬‬
‫‪−1‬‬ ‫‪− (−x − ı (1 − y)) x + ı (1 − y) 4‬‬ ‫‪4‬‬
‫= ‪z‬‬ ‫=‬ ‫=‬ ‫)‪= x + ı (1 − y‬‬
‫)‪3 (ı − z) 3 x + (1 − y‬‬ ‫⋅‪3‬‬ ‫‪3‬‬ ‫‪3‬‬

‫𝟴𝟬𝟭‬

‫‪http ://tinyurl.com/Malki1718‬‬ ‫‪0‬‬


‫‪ .VI‬ﲤﺎرﻳﻦ ﺗﻄﺒﻴﻘﻴﺔ‬
‫‪.‬‬
‫‪1‬‬
‫ﻟﻜﻦ = )‪ x + (1 − y‬إذن ‪:‬‬
‫‪4‬‬

‫‪Ŕž Ŧ Ľ‬‬
‫‪4‬‬ ‫‪4‬‬ ‫‪16‬‬ ‫‪16‬‬ ‫‪16‬‬ ‫‪1‬‬ ‫‪4‬‬ ‫‪2‬‬
‫‪x‬‬ ‫‪+‬‬ ‫)‪(1 − y‬‬ ‫=‬ ‫‪x +‬‬ ‫= )‪(1 − y‬‬ ‫=‬ ‫=‬
‫‪3‬‬ ‫‪3‬‬ ‫‪9‬‬ ‫‪9‬‬ ‫‪9‬‬ ‫‪4‬‬ ‫‪9‬‬ ‫‪3‬‬

‫‪œǃ‬‬
‫‪2‬‬ ‫‪2‬‬
‫= ‪ x + y‬؛ و ﻫﻲ ﻣﻌﺎدﻟﺔ اﻟﺪاﺋﺮة اﻟﺘﻲ ﻣﺮﻛﺰﻫﺎ ‪ O‬و ﻧﺼﻒ ﻗﻄﺮﻫﺎ ‪.‬‬ ‫و ﻫﺬا ﻳﻌﻨﻲ ﺑﺎﻟﺘﺤﺪﻳﺪ ّ‬
‫أن ‪:‬‬
‫‪3‬‬ ‫‪3‬‬
‫‪2‬‬
‫إذن‪ ،‬اﻟﻨﻘﻄﺔ ‪ G‬ﺗﻨﺘﻤﻲ إﱃ اﻟﺪاﺋﺮة اﻟﺘﻲ ﻣﺮﻛﺰﻫﺎ ‪ O‬و ﻧﺼﻒ ﻗﻄﺮﻫﺎ ‪.‬‬
‫‪3‬‬

‫‪Ŀ‬‬
‫‪ّ 3z ⋅ (z − ı) = 1 = e‬‬
‫ﻓﺈن )𝜋‪arg (z − 0) + arg (z − z ) = 2𝜋 (mod 2‬‬ ‫)ج( ﺑﲈ ّ‬
‫أن‬
‫أي )𝜋‪. i,⃗ OG + i,⃗ AM = 2𝜋 (mod 2‬‬
‫‪1‬‬
‫و ﺗﺼﻨﻊ زاوﻳﺔ ﻗﻴﺴﻬﺎ )‪2𝜋 − arg (z − ı‬‬ ‫و ﺑﺎﻟﺘﺎﱄ ﻓﺎﻟﻨﻘﻄﺔ ‪ G‬ﺗﻘﻊ ﻋﲆ اﻟﺪاﺋﺮة اﻟﺘﻲ ﻣﺮﻛﺰﻫﺎ ‪ O‬و ﻧﺼﻒ ﻗﻄﺮﻫﺎ‬
‫‪3‬‬
‫ﻣﻊ ﳏﻮر اﻟﻔﻮاﺻﻞ )اﳌﺴﺘﻘﻴﻢ ‪. (y = 0‬‬
‫‪1‬‬
‫)د( ﻟﺪﻳﻨﺎ ‪ z = z + z + z‬ﻣﻨﻪ ‪ z = 3z − z − z‬و ﺑﺎﻟﺘﺎﱄ ﻓﺎﻟﻨﻘﻄﺔ ‪ M‬ﻫﻲ ﻣﺮﻛﺰ اﳌﺴﺎﻓﺎت اﳌﺘﻨﺎﺳﺒﺔ‬
‫‪3‬‬
‫ﻟﻠﺠﻤﻠﺔ })‪. {G (3) , A (−1) , M (−1‬‬
‫■‬

‫)ﻧﻔﺮض ّ‬
‫أن ‪.( z ≠ 1‬‬ ‫‪z‬‬ ‫‪ P ، N ، M ، A‬أرﺑﻊ ﻧﻘﻂ ﻣﻦ اﳌﺴﺘﻮي ﻟﻮاﺣﻘﻬﺎ ﻋﲆ اﻟﱰﺗﻴﺐ ‪، z ، z ، 1 :‬‬
‫‬ ‫‪.‬‬
‫𝟎𝟒‬ ‫‪.‬‬

‫‪ّ •1‬‬
‫ﻋﲔ ﳎﻤﻮﻋﺔ اﻟﻨﻘﻂ ‪ M‬ﺑﺤﻴﺚ ﻳﻜﻮن ‪ AM‬و ‪ MN‬ﻣﺘﻌﺎﻣﺪﻳﻦ‪.‬‬
‫‪ •2‬أوﺟﺪ اﻟﻨﻘﻂ ‪ M‬ﺑﺤﻴﺚ ﻳﻜﻮن اﻟﺮﺑﺎﻋﻲ ‪ AMNP‬ﻣﺮﺑﻌﺎ‪.‬‬

‫‪ •1‬ﻧﺴﺘﻌﺮض ﻓﻴﲈ ﻳﲇ ﻃﺮﻳﻘﺘﲔ ‪:‬‬ ‫ﺍﳊﻞّ‪.‬‬


‫‪ņőƱŀ Ţ‬‬
‫‪z−z‬‬
‫ﲣ ّﻴﻠﻴ ًﺎ‬ ‫اﻟﻄﺮﻳﻘﺔ اﻷوﱃ ‪ :‬ﺣﺴﺐ اﻟﻼزﻣﺔ ‪ 1‬ﺻﻔﺤﺔ ‪ ،53‬ﻳﻜﻮن ‪ AM‬و ‪ MN‬ﻣﺘﻌﺎﻣﺪﻳﻦ إذا و ﻓﻘﻂ إذا ﻛﺎن اﻟﻌﺪد‬
‫‪z−1‬‬
‫ِﴏﻓ ًﺎ ‪ .‬ﻟﻜﻦ ‪:‬‬
‫‪z−z‬‬ ‫)‪z (1 − z‬‬
‫‪VI‬‬

‫=‬ ‫‪= −z = −x − ıy‬‬


‫‪z−1‬‬ ‫‪z−1‬‬
‫‪z−z‬‬
‫ﻣﻌﺪوﻣ ًﺎ أي إذا و ﻓﻘﻂ إذا‬ ‫و ﺑﺎﻟﺘﺎﱄ‪ ،‬ﻳﻜﻮن ‪ AM‬و ‪ MN‬ﻣﺘﻌﺎﻣﺪﻳﻦ إذا و ﻓﻘﻂ إذا ﻛﺎن اﳉﺰء اﳊﻘﻴﻘﻲ ﻟﻠﻌﺪد‬
‫‪z−1‬‬
‫ﻛﺎن ‪ −x = 0‬أي إذا و ﻓﻘﻂ إذا ﻛﺎن ‪. x = 0‬‬
‫‪ƕ‬‬

‫إذن‪ ،‬اﳌﺠﻤﻮﻋﺔ اﻟﺘﻲ ﻧﺒﺤﺚ ﻋﻨﻬﺎ ﻫﻲ اﳌﺴﺘﻘﻴﻢ اﻟﺬي ﻣﻌﺎدﻟﺘﻪ ‪) x = 0‬ﳏﻮر اﻟﱰاﺗﻴﺐ( ‪.‬‬
‫اﻟﻄﺮﻳﻘﺔ اﻟﺜﺎﻧﻴﺔ ‪ :‬ﻧُﻄ ﱢﺒﻖ ﻧﺘﻴﺠﺔ اﳌﱪﻫﻨﺔ ‪ 25‬ﺻﻔﺤﺔ ‪.53‬‬
‫ﻻﺣﻘﺔ ‪ AM‬ﻫﻲ ‪ z − 1‬و ﻻﺣﻘﺔ ‪ MN‬ﻫﻲ ‪ ، z − z‬و ﺑﺎﻟﺘﺎﱄ ﻳﻜﻮن ‪ AM‬و ‪ MN‬ﻣﺘﻌﺎﻣﺪﻳﻦ إذا و ﻓﻘﻂ إذا ﻛﺎن‬
‫‪ũŏ‬‬

‫𝟵𝟬𝟭‬

‫‪http ://tinyurl.com/Malki1718‬‬ ‫‪0‬‬


‫ ﶍﻮﻋﺔ اﻷﻋﺪاد اﳌﺮﻛﺒﺔ‬.1.VI
.
: ‫ ﻟﻜﻦ‬. Re (z − 1) ⋅ z − z = 0 ‫ أي إذا و ﻓﻘﻂ إذا ﻛﺎن‬AM ⋅ MN = 0

Ŕž Ŧ Ľ
(z − 1) ⋅ z − z = (z − 1) ⋅ z − z = zz − zz − z + z

= (x + ıy) (x − ıy) − (x + ıy) (x − ıy) − (x − ıy) + x − ıy

œǃ
= (x + ıy) x − 2ıxy − y − x − y − x + y + 2ıxy + x − ıy
= x − 2ıx y − xy + ıx y + 2xy − ıy − 2x + 2ıxy + x − ıy
= x − 2x + x + xy + ı −y − x y + 2xy − y

Ŀ
Re (z − 1) ⋅ z − z = 0 ⟺ x − 2x + x + xy = 0 ⟺ x x + y − 2x + 1 = 0 : ‫ﻣﻨﻪ‬
⟺ x=0 ‫أو‬ x + y − 2x + 1 = 0
⟺ x=0 ‫أو‬ (x − 1) + y = 0
⟺ x=0 ‫أو‬ x=1 y=0
⟺ x=0 ‫أو‬ z=1
⟺ x=0 ّ
(z ≠ 1 ‫)ﻷن‬
.‫و ﻧﺼﻞ إﱃ ﻧﻔﺲ اﻟﻨﺘﻴﺠﺔ‬
: ‫ ﻟﻜﻦ‬.‫ ﻣﺘﻌﺎﻣﺪﻳﻦ و ﳍﲈ ﻧﻔﺲ اﻟﻄﻮل‬MP ‫ و‬AN ‫ ﻣﺮﺑﻌﺎ ﻳﻜﻔﻲ أن ﻳﻜﻮن ﻗﻄﺮاه‬AMNP ‫• ﺣﺘﻰ ﻳﻜﻮن اﻟﺮﺑﺎﻋﻲ‬2

AN = MP ⟺ ||z − 1|| = ||z − z|| ⟺ ||(z − 1) (z + 1)|| = ||z (z − 1) (z + 1)||


⟺ ||z − 1|| ⋅ ||z + 1|| − |z| ⋅ ||z − 1|| ⋅ ||z + 1|| = 0
⟺ ||z − 1|| ⋅ ||z + 1|| (1 − |z|) = 0
⟺ ||z − 1|| = 0 ‫أو‬ ||z + 1|| = 0 ‫أو‬ 1 − |z| = 0
⟺ z=1 ‫أو‬ z = −1 ‫أو‬ |z| = 1
⟺ |z| = 1 ّ
(||−1|| = 1 ‫ و‬z ≠ 1 ‫)ﻷن‬
ņőƱŀ Ţ
(AN) ⟂ (MP) ⟺ AN ⋅ MP = 0 ⟺ Re z −1 ⋅ z −z =0 ‫و‬
⟺ Re z −1 z − 1 ⋅ z = 0 ⟺ Re ||z − 1|| z = 0

⟺ ||z − 1|| ⋅ Re (z) = 0 (‫|| ﻋﺪد ﺣﻘﻴﻘﻲ‬z − 1|| ‫)ﻷن‬


ّ

VI
⟺ ||z − 1|| = 0 ‫أو‬ Re (z) = 0
⟺ z =1 ‫أو‬ Re (z) = 0
⟺ z = −1 ‫أو‬ Re (z) = 0 ّ
(z ≠ 1 ‫)ﻷن‬
ƕ

ّ z = −1 ‫ إذا ﻛﺎن‬،‫ﻟﻜﻦ‬
.‫ و ﺑﺎﻟﺘﺎﱄ ﻟﻴﺲ ﻟﺪﻳﻨﺎ ﻣﺮﺑﻊ‬P = M ‫ و‬N = A ‫ أي‬z = z ‫ و‬z = 1 ‫ﻓﺈن‬
: ‫ و ﺑﺎﻟﺘﺎﱄ‬. Re (z) = 0 ‫ ﻣﺘﻌﺎﻣﺪان إذا و ﻓﻘﻂ إذا ﻛﺎن‬MP ‫ و‬AN ،‫إذن‬

AN ⟂ MP |z| = 1 x +y =1
ũŏ

‫ ﻣﺮﺑﻊ‬AMNP ⟺ ⟺ ⟺
AN = MP Re (z) = 0 x=0
y =1 y = ±1
⟺ ⟺ ⟺ z = ±ı
x=0 x=0

. ı ‫ و‬−ı ‫إذن ﺗﻮﺟﺪ ﻧﻘﻄﺘﺎن ُﲢﻘﻘﺎن اﳌﻄﻠﻮب و ﳘﺎ اﻟﻨﻘﻄﺘﺎن اﻟﻠﺘﺎن ﻻﺣﻘﺘﺎﳘﺎ‬


𝟭𝟭𝟬

http ://tinyurl.com/Malki1718 0
‫‪ .VI‬ﲤﺎرﻳﻦ ﺗﻄﺒﻴﻘﻴﺔ‬
‫‪.‬‬

‫‪#‬‬ ‫‪.‬‬
‫𝟏𝟒‬ ‫‪.‬‬

‫‪Ŕž Ŧ Ľ‬‬
‫ﻣﺎ ﻫﻲ اﻟﻌﻼﻗﺔ )أو اﻟﻌﻼﻗﺎت( اﻟﺘﻲ ﺗﺮﺑﻂ ﺑﲔ اﻷﻋﺪاد اﳌﺮﻛﺒﺔ اﻟﺘﻲ ﺻﻮرﻫﺎ ﻫﻲ ‪:‬‬
‫‪ •1‬ﺛﻼﺛﺔ ﻧﻘﻂ ﻋﲆ اﺳﺘﻘﺎﻣﺔ واﺣﺪة ؟‬

‫‪œǃ‬‬
‫‪ •2‬رؤوس ﻣﺜﻠﺚ ﻣﺘﻘﺎﻳﺲ اﻷﺿﻼع ؟‬

‫‪ •3‬رؤوس ﻣﻀ ّﻠﻊ ﻣﻨﺘﻈﻢ ذي ‪ n‬ﺿﻠﻌ ًﺎ ؟‬

‫‪Ŀ‬‬
‫ﺍﳊﻞّ‪ .‬ﻧﻨﺴﺐ اﳌﺴﺘﻮي إﱃ ﻣﻌﻠﻢ ﻣﺘﻌﺎﻣﺪ و ﻣﺘﺠﺎﻧﺲ ⃗ ‪. O, i,⃗ j‬‬
‫‪ •1‬ﻟﺘﻜﻦ ‪ C ، B ، A‬اﻟﻨﻘﻂ اﻟﺘﻲ ﻟﻮاﺣﻘﻬﺎ ‪ c ، b ، a‬ﻋﲆ اﻟﱰﺗﻴﺐ‪.‬‬
‫اﻟﻨﻘﻂ ‪ B ، A‬و ‪ C‬ﻋﲆ اﺳﺘﻘﺎﻣﺔ واﺣﺪة إذا و ﻓﻘﻂ إذا ﻛﺎن ‪ AB, AC = 0 (mod 𝜋) :‬أي إذا و ﻓﻘﻂ إذا ﻛﺎن ‪:‬‬
‫)𝜋 ‪. arg (b − a) − arg (c − a) = 0 (mod‬‬
‫إذن اﻟﴩط اﻟﻼزم و اﻟﻜﺎﰲ ﺣﺘﻰ ﺗﻜﻮن اﻟﻨﻘﻂ ‪ B ، A‬و ‪ C‬ﻋﲆ اﺳﺘﻘﺎﻣﺔ واﺣﺪة ﻫﻮ أن ﻳﻜﻮن ‪:‬‬
‫‪b−a‬‬
‫‪arg‬‬ ‫)𝜋 ‪= 0 (mod‬‬
‫‪c−a‬‬

‫‪ •2‬ﻟﺘﻜﻦ ‪ C ، B ، A‬اﻟﻨﻘﻂ اﻟﺘﻲ ﻟﻮاﺣﻘﻬﺎ ‪ c ، b ، a‬ﻋﲆ اﻟﱰﺗﻴﺐ‪ .‬اﳌﺜﻠﺚ ‪ ABC‬ﻣﺘﻘﺎﻳﺲ اﻷﺿﻼع إذا و ﻓﻘﻂ إذا ﻛﺎن ‪:‬‬
‫𝜋‪2‬‬
‫= ‪ AB, BC‬ﻟﻜﻦ ‪:‬‬ ‫‖‪ ‖AB‖ = ‖BC‬و )𝜋 ‪(mod‬‬
‫‪3‬‬
‫|‪|c −b‬‬
‫| ⟺ ‖‪‖AB‖ = ‖BC‬‬ ‫‪=1‬‬
‫|| ‪| b − a‬‬
‫𝜋‪2‬‬ ‫‪c−b‬‬ ‫𝜋‪2‬‬
‫= ‪AB, BC‬‬ ‫‪(mod 𝜋) ⟺ arg‬‬ ‫=‬ ‫)𝜋 ‪(mod‬‬ ‫و‬
‫‪3‬‬ ‫‪b−a‬‬ ‫‪3‬‬

‫‪c−b‬‬
‫‪ņőƱŀ Ţ‬‬
‫‪.‬‬ ‫‪=e‬‬ ‫‪/‬‬ ‫إذن اﳌﺜﻠﺚ ‪ ABC‬ﻣﺘﻘﺎﻳﺲ اﻷﺿﻼع إذا و ﻓﻘﻂ إذا ﻛﺎن ‪:‬‬
‫‪b−a‬‬
‫‪ a‬ﻋﲆ اﻟﱰﺗﻴﺐ‪.‬‬ ‫‪−‬‬ ‫‪ A‬اﻟﻨﻘﻂ اﻟﺘﻲ ﻟﻮاﺣﻘﻬﺎ ‪، ⋯ ، a ، a ، a‬‬ ‫‪−‬‬ ‫‪ •3‬ﻟﺘﻜﻦ ‪، ⋯ ، A ، A ، A‬‬
‫‪ A‬رؤوس ﻣﻀ ّﻠﻊ ﻣﻨﺘﻈﻢ ﻫﻮ أن ﻳﻜﻮن ﻷﺟﻞ‬ ‫اﻟﴩط اﻟﻼزم و اﻟﻜﺎﰲ ﺣﺘﻰ ﺗﻜﻮن اﻟﻨﻘﻂ ‪، ⋯ ، A ، A ، A‬‬
‫‪VI‬‬

‫‪−‬‬
‫‪:1 ≤ k ≤ n−2‬‬

‫𝜋‪2‬‬
‫‪‖A‬‬ ‫‪−‬‬ ‫‪A ‖ = ‖A A‬‬ ‫‪+‬‬ ‫‖‬ ‫و‬ ‫‪A‬‬ ‫‪−‬‬ ‫‪A ,A A‬‬ ‫‪+‬‬ ‫=‬
‫‪n‬‬
‫إذن اﻟﻨﻘﻂ ‪ A − ، ⋯ ، A ، A ، A‬ﻫﻲ رؤوس ﻣﻀ ّﻠﻊ ﻣﻨﺘﻈﻢ إذا و ﻓﻘﻂ إذا ﻛﺎن ‪:‬‬
‫‪ƕ‬‬

‫‪a + −a‬‬
‫ﻷﺟﻞ ‪. 1 ≤ k ≤ n − 2‬‬ ‫‪=e /‬‬
‫‪a −a‬‬ ‫‪−‬‬

‫■‬
‫‪ũŏ‬‬

‫‪.‬‬
‫𝟐𝟒‬ ‫‪.‬‬
‫ﻣﺎ ﻫﻲ اﻷﻋﺪاد اﳌﺮﻛﺒﺔ ‪ z‬اﻟﺘﻲ ﻣﻦ أﺟﻠﻬﺎ ﻳﻜﻮن ‪:‬‬
‫)𝜋‪ arg (z) = arg (z + 3 + ı) (mod 2‬؟‬ ‫و‬ ‫||‪|z| = ||z − 2‬‬

‫𝟭𝟭𝟭‬

‫‪http ://tinyurl.com/Malki1718‬‬ ‫‪0‬‬


‫‪ .1.VI‬ﶍﻮﻋﺔ اﻷﻋﺪاد اﳌﺮﻛﺒﺔ‬
‫‪.‬‬
‫ﺍﳊﻞّ‪ .‬ﻧﻀﻊ ‪ z = x + ıy‬ﻣﻊ ‪ . x, y ∈ ℝ‬ﻟﺪﻳﻨﺎ ‪:‬‬

‫‪Ŕž Ŧ Ľ‬‬
‫⟺ ||‪|z| = ||z − 2‬‬ ‫= ‪x +y‬‬ ‫‪(x − 2) + y‬‬ ‫‪⟺ x + y = x − 4x + 4 + y‬‬

‫‪œǃ‬‬
‫‪⟺ −4x + 4 = 0 ⟺ x = 1‬‬

‫إذن ‪ z = 1 + ıy‬ﻣﻊ ‪. y ∈ ℝ‬‬


‫ﻣﻼﺣﻈﺔ ‪ ⧏ : 19‬ﻧﻀﻊ )‪ A (2) ، O (0‬و )‪ . M (z‬اﻟﴩط ||‪ |z| = ||z − 2‬أي ||‪ ||z − 0|| = ||z − 2‬ﻳﻌﻨﻲ ّ‬
‫أن اﻟﻨﻘﻄﺔ )‪M (z‬‬

‫‪Ŀ‬‬
‫ﻣﺘﺴﺎوﻳﺔ اﳌﺴﺎﻓﺔ )اﻟ ُﺒﻌﺪ( ﻋﻦ اﻟﻨﻘﻄﺘﲔ )‪ O (0‬و )‪ A (2‬أي أﳖﺎ ﺗﻨﺘﻤﻲ إﱃ ﳏﻮر اﻟﻘﻄﻌﺔ ]‪ . [OA‬ﻧﺘﺤﻘﻖ ﺑﺴﻬﻮﻟﺔ ّ‬
‫أن ﻣﻌﺎدﻟﺔ ﻫﺬا‬
‫⧐‬
‫اﳌﺤﻮر ﻫﻲ ‪. x = 1‬‬
‫ﻣﻦ ﺟﻬﺔ أﺧﺮى ‪:‬‬
‫)𝜋‪arg (z) = arg (z + 3 + ı) (mod 2𝜋) ⟺ arg (z) − arg (z + 3 + ı) = 0 (mod 2‬‬
‫‪z‬‬ ‫‪z‬‬ ‫∗‬
‫‪⟺ arg‬‬ ‫⟺ )𝜋‪= 0 (mod 2‬‬ ‫‪∈ ℝ+‬‬
‫‪z+3+ı‬‬ ‫‪z+3+ı‬‬
‫∗‬
‫‪⟺ z + 3 + ı = 𝜆z , 𝜆 ∈ ℝ+‬‬

‫إذن ‪:‬‬
‫||‪|z| = ||z − 2‬‬ ‫‪z = 1 + ıy, y ∈ ℝ‬‬
‫⟺‬ ‫∗‬
‫)𝜋‪arg (z) = arg (z + 3 + ı) (mod 2‬‬ ‫‪z + 3 + ı = 𝜆z, 𝜆 ∈ ℝ+‬‬
‫‪z = 1 + ıy, y ∈ ℝ‬‬
‫⟺‬ ‫∗‬
‫‪1 + ıy + 3 + ı = 𝜆 (1 + ıy) , 𝜆 ∈ ℝ+‬‬
‫‪z = 1 + ıy, y ∈ ℝ‬‬
‫⟺‬ ‫∗‬
‫‪4 + (1 + y) ı = 𝜆 + ı𝜆y, 𝜆 ∈ ℝ+‬‬
‫⎧‬
‫⎪‬ ‫‪z = 1 + ıy, y ∈ ℝ‬‬
‫⟺‬ ‫‪𝜆=4‬‬
‫‪ņőƱŀ Ţ‬‬
‫⎨‬
‫⎪‬ ‫∗‬
‫‪⎩ 1 + y = 4y, 𝜆 ∈ ℝ+‬‬
‫⎧‬ ‫‪1‬‬
‫⎪‬
‫⎪‬ ‫‪z=1+ ı‬‬
‫⎪‬
‫⎪‬ ‫‪3‬‬
‫⟺‬ ‫‪𝜆=4‬‬

‫‪VI‬‬
‫⎨‬
‫⎪‬
‫⎪‬
‫‪⎪y = 1‬‬
‫⎪‬
‫⎩‬ ‫‪3‬‬

‫‪1‬‬
‫‪ƕ‬‬

‫■‬ ‫أي أﻧﻪ ﻳﻮﺟﺪ ﻋﺪد ﻣﺮﻛﺐ وﺣﻴﺪ ُﳛﻘﻖ اﳌﻄﻠﻮب و ﻫﻮ ‪. z = 1 + ı :‬‬
‫‪3‬‬

‫‪𝟒𝟑. .‬‬
‫اﳌﺴﺘﻮي اﳌﺮﻛﺐ ﻣﻨﺴﻮب إﱃ ﻣﻌﻠﻢ ﻣﺘﻌﺎﻣﺪ و ﻣﺘﺠﺎﻧﺲ ⃗ ‪ a . O, i,⃗ j‬ﻋﺪد ﻣﺮﻛﺐ ُﻣﻌﻄﻰ‪.‬‬
‫‪ũŏ‬‬

‫أوﺟﺪ ﳎﻤﻮﻋﺔ اﻟﻨﻘﻂ ‪ M‬اﻟﺘﻲ ﻻﺣﻘﺘﻬﺎ ‪ z‬ﲢﻘﻖ ‪:‬‬


‫‪z−1−ı‬‬ ‫||‪|z − a| = ||2z − a‬‬ ‫•‬ ‫‪3‬‬ ‫‪(ız + 1) (z + ı − 1) ∈ ℝ‬‬ ‫•‬ ‫‪1‬‬
‫‪∈ ıℝ‬‬ ‫•‬ ‫‪5‬‬
‫‪z + 2ı‬‬
‫‪z−1−ı‬‬
‫‪(ız + 1) (z + ı − 1) ∈ ıℝ‬‬ ‫•‬ ‫‪6‬‬ ‫‪∈ℝ‬‬ ‫•‬ ‫‪4‬‬ ‫‪(ız + 1) (z + ı − 1) ∈ ıℝ‬‬ ‫•‬ ‫‪2‬‬
‫‪z + 2ı‬‬

‫𝟮𝟭𝟭‬

‫‪http ://tinyurl.com/Malki1718‬‬ ‫‪0‬‬


‫‪ .VI‬ﲤﺎرﻳﻦ ﺗﻄﺒﻴﻘﻴﺔ‬
‫‪.‬‬
‫ﺍﳊﻞّ‪ .‬ﻧﻀﻊ ‪ z = x + ıy‬ﻣﻊ ‪ . x, y ∈ ℝ‬ﻟﺪﻳﻨﺎ ‪:‬‬

‫‪Ŕž Ŧ Ľ‬‬
‫‪(ız + 1) (z + ı − 1) = ızz + ı (ı − 1) z + z + ı − 1 = ı |z| − (1 + ı) z + z + ı − 1‬‬
‫‪= ı |z| − (z − z) − ız + ı − 1 = ı x + y‬‬ ‫‪− 2ıy − ı (x + ıy) + ı − 1‬‬

‫‪œǃ‬‬
‫‪= (y − 1) + x + y − x − 2y + 1 ı‬‬

‫‪ •1‬إذن ‪:‬‬

‫‪Ŀ‬‬
‫‪(ız + 1) (z + ı − 1) ∈ ℝ ⟺ Im ((ız + 1) (z + ı − 1)) = 0‬‬
‫‪⟺ x + y − x − 2y + 1 = 0‬‬
‫‪1‬‬ ‫‪1‬‬
‫⟺‬ ‫‪x−‬‬ ‫‪−‬‬ ‫‪+ (y − 1) − 1 + 1 = 0‬‬
‫‪2‬‬ ‫‪4‬‬
‫‪1‬‬ ‫‪1‬‬
‫⟺‬ ‫‪x−‬‬ ‫= )‪+ (y − 1‬‬
‫‪2‬‬ ‫‪2‬‬

‫‪1‬‬ ‫‪1‬‬
‫‪ Ω‬و ﻧﺼﻒ ﻗﻄﺮﻫﺎ ‪.‬‬ ‫‪,1‬‬ ‫و ﻫﻲ ﻣﻌﺎدﻟﺔ اﻟﺪاﺋﺮة اﻟﺘﻲ ﻣﺮﻛﺰﻫﺎ‬
‫‪2‬‬ ‫‪2‬‬

‫‪ •2‬ﺑﺎﳌﺜﻞ ‪:‬‬
‫‪(ız + 1) (z + ı − 1) ∈ ıℝ ⟺ y − 1 = 0 ⟺ y = 1‬‬
‫و ﻫﻲ ﻣﻌﺎدﻟﺔ ﻣﺴﺘﻘﻴﻢ )ﻳﻮازي ﺣﺎﻣﻞ ﳏﻮر اﻟﻔﻮاﺻﻞ( ‪.‬‬
‫‪ •3‬ﻧﻀﻊ 𝛽‪ a = 𝛼 + ı‬ﻣﻨﻪ ‪:‬‬

‫||)𝛽 ‪|z − a| = ||2z − a|| ⟺ ||(x − 𝛼) + ı (y − 𝛽)|| = ||(2x − 𝛼) + ı (2y −‬‬


‫)𝛽 ‪⟺ (x − 𝛼) + (y − 𝛽) = (2x − 𝛼) + (2y −‬‬
‫𝛽 ‪⟺ x − 2𝛼x + 𝛼 + y − 2𝛽y + 𝛽 = 4x − 4𝛼x + 𝛼 + 4y − 4𝛽y +‬‬
‫‪ņőƱŀ Ţ‬‬
‫‪2‬‬ ‫‪2‬‬
‫‪⟺ 3x − 2𝛼x + 3y − 2𝛽y = 0 ⟺ x − 𝛼x + y − 𝛽y = 0‬‬
‫‪3‬‬ ‫‪3‬‬
‫𝛼‬ ‫𝛼‬ ‫𝛽‬ ‫𝛽‬
‫‪⟺ x−‬‬ ‫‪−‬‬ ‫‪+ y−‬‬ ‫‪−‬‬ ‫‪=0‬‬
‫‪3‬‬ ‫‪9‬‬ ‫‪3‬‬ ‫‪9‬‬
‫‪VI‬‬

‫𝛼‬ ‫𝛽‬ ‫𝛽‪𝛼 +‬‬


‫⟺‬ ‫‪x−‬‬ ‫‪+ y−‬‬ ‫⎛=‬ ‫⎞‬
‫‪3‬‬ ‫‪3‬‬ ‫‪3‬‬
‫⎝‬ ‫⎠‬

‫𝛽‪𝛼 +‬‬ ‫‪a‬‬ ‫𝛽 𝛼‬


‫‪ƕ‬‬

‫( و ﻧﺼﻒ ﻗﻄﺮﻫﺎ‬ ‫‪) Ω‬أي اﻟﻨﻘﻄﺔ ذات اﻟﻼﺣﻘﺔ‬ ‫‪,‬‬ ‫و ﻫﻲ ﻣﻌﺎدﻟﺔ اﻟﺪاﺋﺮة اﻟﺘﻲ ﻣﺮﻛﺰﻫﺎ‬
‫‪3‬‬ ‫‪3‬‬ ‫‪3 3‬‬
‫|‪|a‬‬
‫(‪.‬‬ ‫)أي‬
‫‪3‬‬
‫‪ũŏ‬‬

‫𝟯𝟭𝟭‬

‫‪http ://tinyurl.com/Malki1718‬‬ ‫‪0‬‬


‫‪ .1.VI‬ﶍﻮﻋﺔ اﻷﻋﺪاد اﳌﺮﻛﺒﺔ‬
‫‪.‬‬
‫ﻟﻴﻜﻦ ‪ . z ≠ −2ı‬ﻟﺪﻳﻨﺎ ‪:‬‬

‫‪Ŕž Ŧ Ľ‬‬
‫‪z − 1 − ı x − 1 + (y − 1) ı‬‬ ‫‪x − 1 + (y − 1) ı x − (y + 2) ı‬‬
‫=‬ ‫=‬
‫‪z + 2ı‬‬ ‫‪x + (y + 2) ı‬‬ ‫)‪x + (y + 2‬‬
‫‪x (x − 1) + (y − 1) (y + 2) − (x − 1) (y + 2) ı + x (y − 1) ı‬‬

‫‪œǃ‬‬
‫=‬
‫)‪x + (y + 2‬‬
‫‪x −x−y −y+2‬‬ ‫‪−xy − 2x + y + 2 + xy − x‬‬
‫=‬ ‫‪+ı‬‬
‫)‪x + (y + 2‬‬ ‫)‪x + (y + 2‬‬

‫‪Ŀ‬‬
‫‪x −x+y +y−2‬‬ ‫‪−3x + y + 2‬‬
‫=‬ ‫‪+ı‬‬
‫)‪x + (y + 2‬‬ ‫)‪x + (y + 2‬‬

‫‪ •4‬ﻷﺟﻞ ‪ ، z ≠ −2ı‬ﻟﺪﻳﻨﺎ ‪:‬‬


‫‪z−1−ı‬‬ ‫‪z−1−ı‬‬ ‫‪−3x + y + 2‬‬
‫‪∈ ℝ ⟺ Im‬‬ ‫⟺ ‪=0‬‬ ‫‪= 0 ⟺ −3x + y + 2 = 0‬‬
‫‪z + 2ı‬‬ ‫‪z + 2ı‬‬ ‫)‪x + (y + 2‬‬
‫و ﻫﻲ ﻣﻌﺎدﻟﺔ ﻣﺴﺘﻘﻴﻢ ‪.‬‬
‫ﻫﻞ اﻟﻨﻘﻄﺔ اﻟﺘﻲ ﻻﺣﻘﺘﻬﺎ ‪ −2ı‬ﺗﻨﺘﻤﻲ إﱃ ﻫﺬا اﳌﺴﺘﻘﻴﻢ ؟ ﻧﻌﻢ ّ‬
‫ﻷن ‪. (−3) × 0 + (−2) + 2 = 0‬‬
‫إذن اﳌﺠﻤﻮﻋﺔ اﻟﺘﻲ ﻧﺒﺤﺚ ﻋﻨﻬﺎ ﻫﻲ اﳌﺴﺘﻘﻴﻢ اﻟﺬي ﻣﻌﺎدﻟﺘﻪ ‪ y = 3x − 2‬ﺑﺎﺳﺘﺜﻨﺎء اﻟﻨﻘﻄﺔ )‪. (0, −2‬‬
‫‪ •5‬ﻷﺟﻞ ‪ ، z ≠ −2ı‬ﻟﺪﻳﻨﺎ ‪:‬‬
‫‪z−1−ı‬‬ ‫‪z−1−ı‬‬ ‫‪x −x+y +y−2‬‬
‫‪∈ ıℝ ⟺ Re‬‬ ‫⟺ ‪=0‬‬ ‫‪=0‬‬
‫‪z + 2ı‬‬ ‫‪z + 2ı‬‬ ‫)‪x + (y + 2‬‬
‫‪⟺ x −x+y −y−2=0‬‬
‫‪1‬‬ ‫‪1‬‬ ‫‪1‬‬ ‫‪1‬‬
‫⟺‬ ‫‪x−‬‬ ‫‪−‬‬ ‫‪+ y+‬‬ ‫‪−‬‬ ‫‪−2=0‬‬
‫‪2‬‬ ‫‪4‬‬ ‫‪2‬‬ ‫‪4‬‬

‫‪1‬‬ ‫‪1‬‬ ‫‪√5‬‬


‫‪ņőƱŀ Ţ‬‬
‫⟺‬ ‫‪x−‬‬ ‫‪+ y+‬‬ ‫=‬
‫‪2‬‬ ‫‪2‬‬ ‫‪2‬‬

‫‪5‬‬ ‫‪1 1‬‬


‫‪ Ω‬و ﻧﺼﻒ ﻗﻄﺮﻫﺎ √ ‪.‬‬ ‫‪,−‬‬ ‫و ﻫﻲ ﻣﻌﺎدﻟﺔ اﻟﺪاﺋﺮة اﻟﺘﻲ ﻣﺮﻛﺰﻫﺎ‬
‫‪2‬‬ ‫‪2 2‬‬

‫‪. 0−‬‬
‫‪1‬‬
‫‪2‬‬
‫‪+ −2 +‬‬
‫‪1‬‬
‫‪2‬‬
‫=‬
‫‪5‬‬
‫‪2‬‬
‫≠‬
‫‪√5‬‬
‫‪2‬‬
‫ﻫﻞ اﻟﻨﻘﻄﺔ اﻟﺘﻲ ﻻﺣﻘﺘﻬﺎ ‪ −2ı‬ﺗﻨﺘﻤﻲ إﱃ ﻫﺬه اﻟﺪاﺋﺮة ؟ ﻻ ّ‬
‫ﻷن‬ ‫‪VI‬‬
‫‪5‬‬ ‫‪1 1‬‬
‫‪ Ω‬و ﻧﺼﻒ ﻗﻄﺮﻫﺎ √ ‪.‬‬ ‫‪,−‬‬ ‫إذن اﳌﺠﻤﻮﻋﺔ اﻟﺘﻲ ﻧﺒﺤﺚ ﻋﻨﻬﺎ ﻫﻲ اﻟﺪاﺋﺮة اﻟﺘﻲ ﻣﺮﻛﺰﻫﺎ‬
‫‪2‬‬ ‫‪2 2‬‬
‫‪ƕ‬‬

‫‪ •6‬ﻟﺪﻳﻨﺎ ‪:‬‬
‫‪(ız + 1) (z + ı − 1) = ız + ı (ı − 1) z + z + ı − 1 = ız − ız + ı − 1‬‬
‫‪ũŏ‬‬

‫‪= ı (x + ıy) − ı (x + ıy) + ı − 1 = ı x − y + 2ıxy − ıx + y + ı − 1‬‬


‫‪= y − 2xy − 1 + ı x − y − x + 1‬‬

‫ﻣﻨﻪ ‪:‬‬
‫‪1‬‬
‫= ‪(ız + 1) (z + ı − 1) ∈ ıℝ ⟺ Re ((ız + 1) (z + ı − 1)) = 0 ⟺ y−2xy−1 = 0 ⟺ y‬‬
‫‪1 − 2x‬‬
‫‪1‬‬
‫= )‪. f (x‬‬ ‫⧵ ‪ ℝ‬ﺑـِـ ‪:‬‬ ‫اﳌﻌﺮﻓﺔ ﻋﲆ‬
‫إذن اﳌﺠﻤﻮﻋﺔ اﻟﺘﻲ ﻧﺒﺤﺚ ﻋﻨﻬﺎ ﻫﻲ ﻣﻨﺤﻨﻰ اﻟﺪاﻟﺔ ‪ّ f‬‬
‫‪1 − 2x‬‬

‫𝟰𝟭𝟭‬

‫‪http ://tinyurl.com/Malki1718‬‬ ‫‪0‬‬


‫‪ .VI‬ﲤﺎرﻳﻦ ﺗﻄﺒﻴﻘﻴﺔ‬
‫‪.‬‬
‫■‬

‫‪Ŕž Ŧ Ľ‬‬
‫ذات اﳌﺠﻬﻮل ‪. z ∈ ℂ‬‬ ‫‪||z + 1|| = |z| + 1‬‬ ‫ﺣﻞ ﰲ ‪ ℂ‬اﳌﻌﺎدﻟﺔ‬
‫‪#‬‬ ‫‪.‬‬
‫𝟒𝟒‬ ‫‪.‬‬

‫‪œǃ‬‬
‫ﺍﳊﻞّ‪ .‬ﻟﺪﻳﻨﺎ ‪:‬‬

‫‪Ŀ‬‬
‫‪||z + 1|| = (z + 1) z + 1 = (z + 1) (z + 1) = zz + z + z + 1 = |z| + 2 Re (z) + 1‬‬

‫و ‪ (|z| + 1) = |z| + 2 |z| + 1‬ﻣﻨﻪ ‪:‬‬


‫)‪||z + 1|| = |z| + 1 ⟺ ||z + 1|| = (|z| + 1‬‬
‫‪⟺ |z| + 2 Re z + 1 = |z| + 2 |z| + 1‬‬
‫|‪⟺ Re (z) = |z‬‬
‫‪+‬‬
‫‪⟺ z∈ℝ‬‬

‫■‬ ‫[∞‪. S = ℝ+ = [0, +‬‬ ‫إذن ﳎﻤﻮﻋﺔ اﳊﻠﻮل ﻫﻲ ‪:‬‬

‫ﺍﻟﺸﻜﻞ ﺍﳌﺜﻠﺜﻲ ﻟﻌﺪﺩ ﻣﺮﻛﺐ‬ ‫‪2.VI‬‬

‫أﻛﺘﺐ اﻷﻋﺪاد اﻟﺘﺎﻟﻴﺔ ﻋﲆ ﺷﻜﻠﻬﺎ اﳌﺜﻠﺜﻲ ‪:‬‬


‫‬ ‫‪.‬‬
‫𝟓𝟒‬ ‫‪.‬‬

‫‪ı‬‬
‫•‬ ‫‪10‬‬ ‫‪−√5 − √15ı‬‬ ‫•‬‫‪7‬‬ ‫‪−ı‬‬ ‫•‬ ‫‪4‬‬ ‫‪5‬‬ ‫‪1‬‬
‫•‬
‫‪2√3 + 2ı‬‬
‫‪−√6 + ı√2‬‬ ‫‪•8‬‬ ‫‪−2‬‬ ‫‪2‬‬
‫‪1+ı‬‬ ‫•‬ ‫‪5‬‬ ‫•‬
‫‪ņőƱŀ Ţ‬‬
‫‪√6‬‬ ‫‪2‬‬ ‫‪1‬‬
‫•‬ ‫‪11‬‬ ‫•‬ ‫‪9‬‬ ‫‪3 − 3ı‬‬ ‫•‬ ‫‪6‬‬ ‫‪ı‬‬ ‫‪3‬‬
‫•‬
‫‪1+ı‬‬ ‫‪1 + ı√3‬‬ ‫‪2‬‬
‫‪VI‬‬

‫ﺍﳊﻞّ‪ .‬ﺑﺪاﻳ ًﺔ‪ ،‬ﻧﻜﺘﺐ ّ‬


‫ﻛﻞ ﻋﺪد ﻋﲆ اﻟﺸﻜﻞ اﳉﱪي أي ﻋﲆ اﻟﺸﻜﻞ ‪ z = a + ıb‬ﻣﻊ ‪ . a, b ∈ ℝ‬ﻟﺪﻳﻨﺎ ‪:‬‬
‫‪a‬‬ ‫‪b‬‬
‫= ‪z = a + ıb‬‬ ‫‪a +b‬‬ ‫‪+‬‬ ‫‪ı‬‬
‫‪a +b‬‬ ‫‪a +b‬‬
‫‪b‬‬ ‫‪a‬‬
‫= 𝜃 ‪sin‬‬ ‫= 𝜃 ‪ cos‬و‬ ‫ﺑﻄﺒﻴﻌﺔ اﳊﺎل ‪ |z| = a + b‬و إذا ﻛﺎﻧﺖ 𝜃 ﻋﻤﺪ ًة ﻟﻠﻌﺪد اﳌﺮﻛﺐ ‪ّ z‬‬
‫ﻓﺈن‬
‫‪ƕ‬‬

‫‪a +b‬‬ ‫‪a +b‬‬


‫و ﺑﺎﻟﺘﺎﱄ ﻧﺒﺤﺚ ﻋﻦ زاوﻳﺔ ُﻋ ِﻠﻢ ﺟﻴﺒﻬﺎ و ﺟﻴﺐ ﲤﺎﻣﻬﺎ‪.‬‬
‫‪ũŏ‬‬

‫𝟱𝟭𝟭‬

‫‪http ://tinyurl.com/Malki1718‬‬ ‫‪0‬‬


‫ اﻟﺸﲁ اﳌﺜﻠﱻ ﻟﻌﺪد ﻣﺮﻛﺐ‬.2.VI
.

Ŕž Ŧ Ľ
5 = 5 (cos 0 + ı sin 0) = [5, 0] •1
−2 = 2 (cos 𝜋 + ı sin 𝜋) = [2, 𝜋] •2

1 1 𝜋 𝜋 1 𝜋

œǃ
ı= cos + ı sin = , • 3
2 2 2 2 2 2
𝜋 𝜋 𝜋
−ı = cos − + ı sin − = 1, − • 4
2 2 2

Ŀ
√2 √2 𝜋 𝜋 𝜋
1 + ı = √2 + ı sin = √2 cos + ı sin = √2, • 5
2 2 4 4 4

√2 √2 𝜋 𝜋 𝜋
3 − 3ı = 3√2 − ı sin = 3√2 cos − + ı sin − = 3√2, − • 6
2 2 4 4 4

1 √3 4𝜋 4𝜋 4𝜋
−√5 − √15ı = 2√5 − − ı = 2√5 cos + ı sin = 2√5, • 7
2 2 3 3 3

√3 1 5𝜋 5𝜋 5𝜋
−√6 + ı√2 = 2√2 − + ı = 2√2 cos + ı sin = 2√2, • 8
2 2 6 6 6

2 2 1 − ı√3 1 √3 𝜋 𝜋 𝜋
= = −ı = cos − + ı sin − = 1, − • 9
1 + ı√3 1 + √3 2 2 3 3 3

ı 1 ı √3 − ı 1 1 + ı√3 1 1 √3
= ⋅ = ⋅ = +ı 10

2√3 + 2ı 2 √3 + 1 2 4 4 2 2
1 𝜋 𝜋 1 𝜋
= cos + ı sin = ,
4 3 3 4 3
√6 √6 (1 − ı) √6 √2 √2
= = (1 − ı) = √3 − ı • 11
1+ı 1 +1 2 2 2
𝜋 𝜋 𝜋
ņőƱŀ Ţ
= √3 cos − + ı sin − = √3, −
4 4 4

VI
: ‫أﻛﺘﺐ ﻋﲆ اﻟﺸﻜﻞ اﳉﱪي‬
 𝟒𝟔. .

𝜋 𝜋
ƕ

.‫ ﻋﻤﺪة ﻟﻪ‬− ‫ و‬3 ‫ اﻟﺬي ﻃﻮﻳﻠﺘﻪ‬z ‫• اﻟﻌﺪد اﳌﺮﻛﺐ‬2 .‫ و ﻋﻤﺪة ﻟﻪ‬2 ‫ اﻟﺬي ﻃﻮﻳﻠﺘﻪ‬z ‫• اﻟﻌﺪد اﳌﺮﻛﺐ‬1
8 3

.ّ‫ﺍﳊﻞ‬
ũŏ

/ 𝜋 𝜋 1 √3
z = 2e = 2 cos + ı sin =2 +i = 1 + ı√3 : ‫ﻟﺪﻳﻨﺎ‬
3 3 2 2

𝜋 𝜋 𝜋 𝜋 3 2 + √2 3 2 − √2
z = 3e− / = 3 cos − + ı sin − = 3 cos − ı sin = −ı ‫و‬
8 8 8 8 2 2

𝟭𝟭𝟲

http ://tinyurl.com/Malki1718 0
‫‪ .VI‬ﲤﺎرﻳﻦ ﺗﻄﺒﻴﻘﻴﺔ‬
‫‪.‬‬
‫𝜋‬ ‫𝜋‬
‫ﺣﻴﺚ ﻹﳚﺎد ‪ cos‬و ‪ sin‬اﺗّﺒﻌﻨﺎ اﻟﻄﺮﻳﻘﺔ اﻟﺘﺎﻟﻴﺔ ‪: 1‬‬
‫‪8‬‬ ‫‪8‬‬
‫‪√2‬‬ ‫‪√2‬‬

‫‪Ŕž Ŧ Ľ‬‬
‫‪. ±e‬‬ ‫‪/‬‬ ‫ﺟﺬر ْﻳﻪ اﻟﱰﺑﻴﻌﻴﲔ ﳘﺎ‬ ‫‪ . e‬ﻣﻦ اﻟﻮاﺿﺢ ّ‬ ‫‪/‬‬
‫أن َ‬ ‫=‬
‫‪2‬‬
‫‪+ı‬‬
‫‪2‬‬
‫ﻟﺪﻳﻨﺎ‬
‫‪√2‬‬ ‫‪√2‬‬
‫= )‪(x + ıy‬‬ ‫‪+ı‬‬ ‫ﻟﻴﻜﻦ ‪ z = x + ıy‬ﺟﺬر ًا ﺗﺮﺑﻴﻌﻴ ًﺎ ﻟـِ ‪ e /‬ﺣﻴﺚ ‪ . x, y ∈ ℝ‬ﻟﺪﻳﻨﺎ‬

‫‪œǃ‬‬
‫‪2‬‬ ‫‪2‬‬
‫‪⎪ x − y = √2‬‬
‫⎧‬
‫⎪‬
‫⎪‬
‫⎪‬ ‫‪2‬‬
‫⎪‬ ‫‪2‬‬ ‫‪2‬‬
‫‪.‬‬ ‫‪√2‬‬ ‫أي √ ‪ x − y + 2ıxy = √ + ı‬و ﺑﺎﻟﺘﺎﱄ‬
‫⎨‬
‫⎪‬ ‫‪2xy‬‬ ‫=‬ ‫‪2‬‬ ‫‪2‬‬

‫‪Ŀ‬‬
‫⎪‬
‫⎪‬ ‫‪2‬‬
‫⎪‬
‫⎪‬
‫|‬ ‫‪/ |=1‬‬
‫‪⎩ x + y = |e‬‬ ‫|‬
‫‪2 + √2‬‬ ‫‪2 + √2‬‬ ‫‪2‬‬
‫‪.x = ±‬‬ ‫= ‪ x‬ﻣﻨﻪ‬ ‫ﺑﺠﻤﻊ اﳌﻌﺎدﻟﺘﲔ اﻷوﱃ و اﻟﺜﺎﻟﺜﺔ ﻳﻨﺘﺞ √ ‪ 2x = 1 +‬أي‬
‫‪2‬‬ ‫‪4‬‬ ‫‪2‬‬
‫‪2 − √2‬‬ ‫‪√2‬‬ ‫‪2 + √2‬‬
‫= ‪ y‬ﻣﻨﻪ‬ ‫‪ 2y = 1 −‬أي‬ ‫= ‪ . x‬ﺑﻄﺮح اﳌﻌﺎدﻟﺔ اﻷوﱃ ﻣﻦ اﻟﺜﺎﻟﺜﺔ ﻳﻨﺘﺞ‬ ‫ﻧﺤﺘﺎر ﻣﺜ ً‬
‫ﻼ‬
‫‪4‬‬ ‫‪2‬‬ ‫‪2‬‬
‫‪2 − √2‬‬
‫‪.y = ±‬‬
‫‪2‬‬
‫‪/‬‬ ‫‪2 − √2‬‬
‫‪e‬‬ ‫= ‪ y‬و ﺑﺎﻟﺘﺎﱄ ﻓﺎﳉﺬرﻳﻦ اﻟﱰﺑﻴﻌﻴﲔ ﻟﻠﻌﺪد‬ ‫أن ‪ y > 0‬أي‬ ‫ﺑﲈ أﻧﻨﺎ اﺧﱰﻧﺎ ‪ x > 0‬ﻓﻤﻦ اﳌﻌﺎدﻟﺔ اﻟﺜﺎﻧﻴﺔ ﻧﺴﺘﻨﺘﺞ ّ‬
‫‪2‬‬
‫𝜋‬ ‫𝜋‬ ‫𝜋 𝜋‬ ‫‪2 + √2‬‬ ‫‪2 − √2‬‬
‫‪ . ±‬ﻟﻜﻦ < < ‪ 0‬ﻣﻨﻪ ‪ cos > 0‬و ‪ sin > 0‬إذن ‪:‬‬ ‫‪+ı‬‬ ‫ﳘﺎ‬
‫‪8‬‬ ‫‪8‬‬ ‫‪8‬‬ ‫‪2‬‬ ‫‪2‬‬ ‫‪2‬‬

‫𝜋‬ ‫‪2 + √2‬‬ ‫𝜋‬ ‫‪2 − √2‬‬


‫‪cos‬‬ ‫=‬ ‫و‬ ‫‪sin‬‬ ‫=‬
‫‪8‬‬ ‫‪2‬‬ ‫‪8‬‬ ‫‪2‬‬
‫■‬

‫‬ ‫‪.‬‬
‫𝟕𝟒‬ ‫‪.‬‬
‫‪. 1 + ı√3‬‬ ‫أﻛﺘﺐ ﻋﲆ اﻟﺸﻜﻞ اﳌﺜﻠﺜﻲ ﺛﻢ اﳉﱪي اﻟﻌﺪد‬
‫‪ņőƱŀ Ţ‬‬
‫ﺍﳊﻞّ‪.‬‬

‫‪1‬‬ ‫‪√3‬‬ ‫𝜋‬ ‫𝜋‬


‫‪VI‬‬

‫‪1 + ı√3 = 2‬‬ ‫‪+ı‬‬ ‫‪= 2 cos‬‬ ‫‪+ ı sin‬‬ ‫‪= 2e‬‬ ‫‪/‬‬ ‫ﻟﺪﻳﻨﺎ ‪:‬‬
‫‪2‬‬ ‫‪2‬‬ ‫‪3‬‬ ‫‪3‬‬

‫‪1 + ı√3‬‬ ‫‪= 2e‬‬ ‫‪/‬‬ ‫‪=2 e‬‬ ‫‪= 2048e‬‬ ‫‪/‬‬ ‫‪− / = 2048e‬‬ ‫‪−‬‬ ‫‪/‬‬ ‫ﻣﻨﻪ ‪:‬‬
‫𝜋‬ ‫𝜋‬
‫‪= 2048e− / = 2048 cos −‬‬ ‫‪+ ı sin −‬‬ ‫)اﻟﺸﻜﻞ اﳌﺜﻠﺜﻲ(‬
‫‪3‬‬ ‫‪3‬‬
‫‪ƕ‬‬

‫‪1‬‬ ‫‪√3‬‬
‫‪= 2048‬‬ ‫‪−ı‬‬ ‫‪= 1024 1 − ı√3‬‬ ‫)اﻟﺸﻜﻞ اﳉﱪي(‬
‫‪2‬‬ ‫‪2‬‬
‫‪ũŏ‬‬

‫■‬ ‫أن اﻟﺸﻜﻞ اﳉﱪي أﻧﺴﺐ ﻟﻠﺠﻤﻊ و اﻟﻄﺮح و ّ‬


‫أن اﻟﺸﻜﻞ اﳌﺜﻠﺜﻲ )أو اﻷﳼ( أﻧﺴﺐ ﻟﻠﴬب و اﻟﻘﺴﻤﺔ‪.‬‬ ‫ﻧُﺬﻛﱢﺮ ّ‬

‫= ‪ ، sin‬ﻟﻜﻦ اﺧﱰﻧﺎ ﻫﺬه اﻟﻄﺮﻳﻘﺔ ﻷﳖﺎ ﺗﻌﺘﻤﺪ ﻋﲆ‬ ‫‪− cos‬‬ ‫= ‪ cos‬و‬ ‫‪+ cos‬‬ ‫ﻼ اﺳﺘﻌﲈل اﻟﻌﻼﻗﺘﲔ‬ ‫‪1‬ﻫﻨﺎك ﻋﺪة ﻃﺮق ﻣﺜ ً‬
‫اﻷﻋﺪاد اﳌﺮﻛﺒﺔ‪ .‬ﻳﻤﻜﻦ أﻳﻀ ًﺎ ﺗﻄﺒﻴﻖ ﻧﺘﻴﺠﺔ اﻟﺘﻤﺮﻳﻦ ‪ 69‬ﺻﻔﺤﺔ ‪.136‬‬

‫𝟳𝟭𝟭‬

‫‪http ://tinyurl.com/Malki1718‬‬ ‫‪0‬‬


‫‪ .2.VI‬اﻟﺸﲁ اﳌﺜﻠﱻ ﻟﻌﺪد ﻣﺮﻛﺐ‬
‫‪.‬‬

‫‬ ‫‪𝟒𝟖.‬‬ ‫‪.‬‬

‫‪Ŕž Ŧ Ľ‬‬
‫‪1 + ı√3‬‬
‫=‪ z‬؟‬ ‫ﻣﺎ ﻫﻮ اﻟﺸﻜﻞ اﳉﱪي ﻟﻠﻌﺪد‬
‫‪1−ı‬‬

‫‪œǃ‬‬
‫ﺍﳊﻞّ‪ .‬ﻟﺪﻳﻨﺎ ‪:‬‬

‫‪Ŀ‬‬
‫‪1 √3‬‬ ‫𝜋‬ ‫𝜋‬ ‫‪/‬‬
‫‪1 + ı√3 = 2‬‬ ‫‪+‬‬ ‫‪ı‬‬ ‫‪= 2 cos‬‬ ‫‪+ ı sin‬‬ ‫‪= 2e‬‬
‫‪2‬‬ ‫‪2‬‬ ‫‪3‬‬ ‫‪3‬‬

‫‪√2 √2‬‬ ‫𝜋‬ ‫𝜋‬


‫‪1 − ı = √2‬‬ ‫‪−‬‬ ‫‪ı‬‬ ‫‪= √2 cos −‬‬ ‫‪+ ı sin −‬‬ ‫‪= √2e−‬‬ ‫‪/‬‬ ‫و‬
‫‪2‬‬ ‫‪2‬‬ ‫‪4‬‬ ‫‪4‬‬

‫‪2e‬‬ ‫‪/‬‬
‫‪+‬‬ ‫‪/‬‬
‫=‪z‬‬ ‫‪= √2‬‬ ‫‪e‬‬ ‫‪=2 e‬‬ ‫‪= 1024e‬‬ ‫ﻣﻨﻪ ‪:‬‬
‫‪√2e−‬‬ ‫‪/‬‬

‫‪−‬‬ ‫𝜋‬ ‫𝜋‬


‫‪= 1024e‬‬ ‫‪= 1024e−‬‬ ‫‪/‬‬ ‫‪= 1024 cos −‬‬ ‫‪+ ı sin −‬‬
‫‪3‬‬ ‫‪3‬‬
‫‪1‬‬ ‫‪√3‬‬
‫‪= 1024‬‬ ‫‪−ı‬‬ ‫‪= −512 + 512ı√3‬‬
‫‪2‬‬ ‫‪2‬‬

‫■‬ ‫و ﻫﻮ اﻟﺸﻜﻞ اﳉﱪي ﻟﻠﻌﺪد ‪. z‬‬

‫‪1 + ı√3‬‬
‫‬ ‫‪𝟒𝟗.‬‬ ‫‪.‬‬
‫‪.A‬‬ ‫= ‪ . A‬أﻛﺘﺐ ﻋﲆ اﻟﺸﻜﻞ اﳉﱪي اﻟﻌﺪد‬
‫‪1+ı‬‬

‫‪ 1 + ı = √2e‬إذن ‪:‬‬ ‫‪/‬‬ ‫‪ 1 + ı√3 = 2e‬و‬ ‫‪/‬‬ ‫ﺍﳊﻞّ‪ .‬ﻟﺪﻳﻨﺎ ‪:‬‬


‫‪1 + ı√3‬‬ ‫‪2e‬‬ ‫‪/‬‬
‫‪ņőƱŀ Ţ‬‬
‫‪−‬‬ ‫‪/‬‬
‫=‪A‬‬ ‫=‬ ‫‪= √2e‬‬ ‫‪= √2e‬‬
‫‪1+ı‬‬ ‫‪√2e‬‬ ‫‪/‬‬

‫‪.A‬‬ ‫‪= √2 e‬‬ ‫ﻣﻨﻪ‬


‫أن ‪ّ 5 = 6 − 1 = 1 − 1‬‬
‫ﻓﺈن ‪:‬‬ ‫‪ . A = e‬و ﺑﲈ ّ‬ ‫‪/‬‬ ‫إذن‬
‫‪101‬‬
‫‪=4+‬‬
‫‪5‬‬
‫ﻟﻜﻦ‬
‫‪12‬‬ ‫‪12‬‬ ‫‪2 12‬‬ ‫‪12‬‬ ‫‪12‬‬

‫‪VI‬‬
‫‪A‬‬ ‫‪= √2 e −‬‬ ‫‪= √2 e / × e− / = ı√2‬‬ ‫‪e−‬‬ ‫‪/‬‬

‫ﻟﻜﻦ ‪:‬‬
‫‪/‬‬ ‫‪1 + ı√3 √3 + 1‬‬ ‫‪√3 − 1‬‬
‫‪√2e‬‬ ‫=‪=A‬‬ ‫=‬ ‫‪+ı‬‬
‫‪1+ı‬‬ ‫‪2‬‬ ‫‪2‬‬
‫‪ƕ‬‬

‫‪1‬‬ ‫‪√3 + 1‬‬ ‫‪√3 − 1‬‬


‫‪ e−‬و ﺑﺎﻟﺘﺎﱄ ‪:‬‬ ‫‪/‬‬ ‫=‬ ‫‪−ı‬‬ ‫إذن‬
‫‪√2‬‬ ‫‪2‬‬ ‫‪2‬‬
‫‪1‬‬ ‫‪√3 + 1‬‬ ‫‪√3 − 1‬‬
‫‪A‬‬ ‫‪= ı √2‬‬ ‫×‬ ‫‪−ı‬‬ ‫‪=2‬‬ ‫‪√3 − 1 + 2‬‬ ‫‪√3 + 1 ı‬‬
‫‪ũŏ‬‬

‫‪√2‬‬ ‫‪2‬‬ ‫‪2‬‬

‫■‬ ‫‪.A‬‬ ‫و ﻫﻮ اﻟﺸﻜﻞ اﳉﱪي ﻟﻠﻌﺪد‬

‫‪ z = 1 + ı√3‬ﻋﲆ أﺑﺴﻂ ﺷﻜﻞ‪.‬‬ ‫‪+ 1 − ı√3‬‬ ‫أﻛﺘﺐ اﻟﻌﺪد‬


‫‬ ‫‪𝟓𝟎.‬‬ ‫‪.‬‬

‫𝟴𝟭𝟭‬

‫‪http ://tinyurl.com/Malki1718‬‬ ‫‪0‬‬


‫ ﲤﺎرﻳﻦ ﺗﻄﺒﻴﻘﻴﺔ‬.VI
.
: ‫ و‬z = a + a : ‫ ﻟﺪﻳﻨﺎ‬. a = 1 + ı√3 ‫ ﻧﻀﻊ‬.ّ‫ﺍﳊﻞ‬

Ŕž Ŧ Ľ
1 √3 𝜋 𝜋 /
a = 1 + ı√3 = 2 +ı = 2 cos + ı sin = 2e
2 2 3 3

œǃ
/ n𝜋 + n𝜋
z = a + a = 2 Re (a ) = 2 Re 2 e = 2 2 cos =2 cos : ‫ﻣﻨﻪ‬
3 3

: ‫ ﻓﻴﻜﻮن‬0 ≤ r ≤ 5 ‫ ﻣﻊ‬n = 6q + r ‫ و ﻧﻜﺘﺐ‬6 ‫ ﻋﲆ‬n ‫ﻧﻌﺘﱪ اﻟﻘﺴﻤﺔ اﻹﻗﻠﻴﺪﻳﺔ ﻟﻠﻌﺪد‬

Ŀ
+ r𝜋 + r𝜋
z=2 cos 2q𝜋 + =2 cos
3 3

. ∀𝜃 ∈ ℝ : ||e − 1|| = 2 − 2 cos 𝜃 ّ ‫أﺛﺒﺖ‬


: ‫أن‬
 .
𝟓𝟏 .

: ‫ ﻟﺪﻳﻨﺎ‬. 𝜃 ∈ ℝ ‫ ﻟﻴﻜﻦ‬.ّ‫ﺍﳊﻞ‬

||e − 1|| = e − 1 e −1 = e −1 e −1 = e −1 e− − 1
= e e− − e − e− + 1 = 1 − e + e− + 1 = 2 − 2 cos 𝜃

ّ
. 𝜃 ∈ ℝ ‫ ﻟﻜﻞ‬cos 𝜃 ≤ 1 ‫ﻷن‬ 𝜃 ∈ ℝ ‫ ﻟﻜﻞ‬2 − 2 cos 𝜃 ≥ 0 ⧏ : 20 ‫ﻣﻼﺣﻈﺔ‬


ņőƱŀ Ţ
.
𝟓𝟐 .
: ‫ ﻣ ﱢﺜﻞ ﻫﻨﺪﺳﻴ ًﺎ ﻛﻼ ﻣﻦ‬. w = 2 − ı ‫ و‬z = 3 + 4ı ‫ ﻋﺪدﻳﻦ ﻣﺮﻛﺒﲔ ﺣﻴﺚ‬w ‫ و‬z ‫ﻟﻴﻜﻦ‬
w • 7 zw + wz • 5 wz • 3 z+w 1

VI

30 z +7 5z
• 8 • 6 • 4 w−z 2

1+z+w 12 2w

: ‫ ﻟﺪﻳﻨﺎ‬.ّ‫ﺍﳊﻞ‬
ƕ

. z + w = 3 + 4ı + 2 − ı = 5 + 3ı •1
. w − z = 2 − ı − 3 − 4ı = −1 − 5ı •2
. wz = (2 − ı) (3 + 4ı) = 6 + 8ı − 3ı − 4ı = 6 + 5ı + 4 = 10 + 5ı •3
ũŏ

5z 5 3 + 4ı 5 (3 + 4ı) (2 + ı) 5 6 + 8ı + 3ı + 4ı 5 2 + 11ı 2 + 11ı


. = ⋅ = ⋅ = ⋅ = ⋅ = 4

2w 2 2 − ı 2 (2 − ı) (2 + ı) 2 2 −ı 2 5 2
. zw + wz = (3 − 4ı) (2 − ı) + (2 + ı) (3 + 4ı) = 2 − 11ı + 2 + 11ı = 4 •5
z + 7 (3 + 4ı) + 7 3 + 2 × 3 × 4ı + (4ı) + 7 −7 + 24ı + 7
. = = = = 2ı 6

12 12 12 12

𝟭𝟭𝟵

http ://tinyurl.com/Malki1718 0
‫‪ .2.VI‬اﻟﺸﲁ اﳌﺜﻠﱻ ﻟﻌﺪد ﻣﺮﻛﺐ‬
‫‪.‬‬

‫‪Ŕž Ŧ Ľ‬‬
‫‪6‬‬ ‫‪5z‬‬
‫‪2w‬‬

‫‪œǃ‬‬
‫‪wz‬‬

‫‪z‬‬
‫‪4‬‬

‫‪Ŀ‬‬
‫‪z+w‬‬

‫‪z 2 +7‬‬
‫‪12‬‬
‫‪2‬‬

‫‪zw + wz‬‬
‫‪−2‬‬ ‫‪0‬‬ ‫‪2‬‬ ‫‪4‬‬ ‫‪6‬‬ ‫‪8‬‬ ‫‪10‬‬
‫‪w‬‬

‫‪30‬‬
‫‪1+z+w‬‬
‫‪−2‬‬

‫‪w2‬‬
‫‪−4‬‬

‫‪w−z‬‬

‫‪−6‬‬

‫ﺷﻜﻞ ‪8.VI‬‬
‫‪ņőƱŀ Ţ‬‬
‫‪. w = (2 − ı) = 2 − 2 × 2 × ı + ı = 4 − 4ı − 1 = 3 − 4ı •7‬‬
‫‪30‬‬ ‫‪30‬‬ ‫)‪30 (6 − 3ı) 30 × 3 (2 − ı‬‬
‫=‬ ‫=‬ ‫=‬ ‫‪= 4 − 2ı‬‬ ‫•‬ ‫‪8‬‬
‫‪1 + z + w 6 + 3ı‬‬ ‫‪45‬‬

‫‪VI‬‬
‫‪6 +3‬‬
‫و ﻣﻨﻪ اﻟﺘﻤﺜﻴﻞ اﻟﻨﻘﻄﻲ اﻟﺘﺎﱄ )ﺷﻜﻞ ‪: (8.VI‬‬
‫■‬

‫‬
‫‪ƕ‬‬

‫‪𝟓𝟑. .‬‬
‫‪ z‬و ‪ z‬ﻋﺪدان ﻣﺮﻛﺒﺎن ﺣﻴﺚ ‪ z = 4 + 4ı :‬و ‪. z = 1 − ı√3‬‬
‫ﻋﲔ اﻟﻄﻮﻳﻠﺔ و ﻋﻤﺪة ﻟﻜﻞ ﻣﻦ اﻷﻋﺪاد اﳌﺮﻛﺒﺔ اﻟﺘﺎﻟﻴﺔ ‪:‬‬
‫ّ‬
‫‪z‬‬ ‫‪z‬‬
‫‪3‬‬ ‫‪2‬‬ ‫‪1‬‬
‫‪ũŏ‬‬

‫•‬ ‫‪5‬‬ ‫•‬ ‫‪4‬‬ ‫‪z‬‬ ‫•‬ ‫‪z ⋅z‬‬ ‫•‬ ‫‪z‬‬ ‫•‬
‫‪z‬‬ ‫‪z‬‬

‫𝟬𝟮𝟭‬

‫‪http ://tinyurl.com/Malki1718‬‬ ‫‪0‬‬


‫ ﲤﺎرﻳﻦ ﺗﻄﺒﻴﻘﻴﺔ‬.VI
.
: ‫ ﻟﺪﻳﻨﺎ‬.ّ‫ﺍﳊﻞ‬

Ŕž Ŧ Ľ
√2 √2 𝜋 𝜋 𝜋
z = 4 + 4ı = 4√2 +ı = 4√2 cos + ı sin = 4√2,
2 2 4 4 4

œǃ
1 √3 𝜋 𝜋 𝜋
z = 1 − ı√3 = 2 −ı = 2 cos − + ı sin − = 2, −
2 2 3 3 3

. z ‫ و‬z ‫و ﻫﻮ اﻟﺸﻜﻞ اﳌﺜﻠﺜﻲ ﻟﻠﻌﺪدﻳﻦ‬

Ŀ
: ‫• ﻟﺪﻳﻨﺎ‬1
𝜋 𝜋 𝜋
z = 4√2, = 4√2 ,2 × = 32,
4 4 2
𝜋
. arg z = (mod 2𝜋) ‫و‬ ||z || = 32 : ‫و ﺑﺎﻟﺘﺎﱄ‬
2
: ‫• ﻟﺪﻳﻨﺎ‬2
𝜋 𝜋 𝜋 𝜋 𝜋
z ⋅ z = 4√2, ⋅ 2, − = 4√2 × 2, − = 8√2, −
4 3 4 3 12
𝜋
. arg (z ⋅ z ) = − (mod 2𝜋) ‫و‬ ||z ⋅ z || = 8√2 : ‫ﻣﻨﻪ‬
12
: ‫• ﻟﺪﻳﻨﺎ‬3
𝜋 𝜋 3𝜋
z = 4√2, = 4√2 ,3 × = 128√2,
4 4 4

3𝜋
. arg z = (mod 2𝜋) ‫و‬ ||z || = 128√2 : ‫إذن‬
4
: ‫• ﻟﺪﻳﻨﺎ‬4
ņőƱŀ Ţ
z 4√2, 4√2 𝜋 𝜋 7𝜋
= = , + = 2√2,
z 2, − 2 4 3 12
VI

z 7𝜋 |z |
. arg = (mod 2𝜋) ‫و‬ | | = 2√2 : ‫و ﺑﺎﻟﺘﺎﱄ‬
z 12 || z ||

: ‫• ﻟﺪﻳﻨﺎ‬5
ƕ

z 2, − 2 𝜋 𝜋 √2 7𝜋
= = ,− − = ,−
z 4√2, 4√2 3 4 4 12

z 1 1 1 7𝜋 √2 7𝜋
= = = ,− = ,− : ‫أو ﺑﻄﺮﻳﻘﺔ أﺧﺮى‬
ũŏ

z 2√2, 2 √2 12 4 12

z 7𝜋 |z | √2
. arg =− (mod 2𝜋) ‫و‬ | |= : ‫إذن‬
z 12 || z || 4

𝟭𝟮𝟭

http ://tinyurl.com/Malki1718 0
‫‪ .2.VI‬اﻟﺸﲁ اﳌﺜﻠﱻ ﻟﻌﺪد ﻣﺮﻛﺐ‬
‫‪.‬‬

‫‬ ‫‪𝟓𝟒.‬‬ ‫‪.‬‬

‫‪Ŕž Ŧ Ľ‬‬
‫ﻋﲔ اﻟﻄﻮﻳﻠﺔ و ﻋﻤﺪة ﻟﻠﻌﺪد اﳌﺮﻛﺐ ‪:‬‬
‫ّ‬
‫‪5 + 11ı√3‬‬
‫=‪z‬‬

‫‪œǃ‬‬
‫‪7 − 4ı√3‬‬

‫ﺍﳊﻞّ‪ .‬ﻟﺪﻳﻨﺎ ‪:‬‬

‫‪Ŀ‬‬
‫‪5 + 11ı√3‬‬ ‫‪5 + 11ı√3 7 + 4ı√3‬‬ ‫‪35 + 20ı√3 + 77ı√3 − 44 × 3‬‬
‫=‪z‬‬ ‫=‬ ‫=‬
‫‪7 − 4ı√3‬‬ ‫‪97‬‬
‫‪7 + 4√3‬‬

‫‪−97 + 97ı√3‬‬ ‫‪1‬‬ ‫‪√3‬‬ ‫𝜋‪2‬‬ ‫𝜋‪2‬‬


‫=‬ ‫‪= −1 + ı√3 = 2 − + ı‬‬ ‫‪= 2 cos‬‬ ‫‪+ ı sin‬‬
‫‪97‬‬ ‫‪2‬‬ ‫‪2‬‬ ‫‪3‬‬ ‫‪3‬‬

‫𝜋‪2‬‬
‫■‬ ‫= )‪. arg (z‬‬ ‫)𝜋‪(mod 2‬‬ ‫و‬ ‫‪|z| = 2‬‬ ‫و ﺑﺎﻟﺘﺎﱄ ‪:‬‬
‫‪3‬‬

‫اﻟﻜﺘﺎﺑﺎت اﳌﺨﺘﻠﻔﺔ ﻟﻸﻋﺪاد اﳌﺮﻛﺒﺔ‬


‫‬ ‫‪𝟓𝟓.‬‬ ‫‪.‬‬

‫‪ •1‬أﻛﻤﻞ اﳉﺪول اﻵﰐ ‪:‬‬

‫اﻟﺸﻜﻞ اﻷﳼ‬ ‫اﻟﺸﻜﻞ اﳌﺜﻠﺜﻲ‬ ‫اﻟﻌﺪد اﻟﺸﻜﻞ اﳉﱪي‬

‫‪4e‬‬ ‫‪/‬‬ ‫‪Z‬‬


‫‪ņőƱŀ Ţ‬‬
‫𝜋‪5‬‬ ‫𝜋‪5‬‬
‫‪4 cos‬‬ ‫‪+ ı sin‬‬ ‫‪Z‬‬
‫‪6‬‬ ‫‪6‬‬

‫‪2√3 − 2ı‬‬ ‫‪Z‬‬

‫‪ •2‬ﻋﻠﻢ اﻟﻨﻘﻂ ‪ B ، A‬و ‪ C‬ﺻﻮر اﻷﻋﺪاد ‪ Z ، Z‬و ‪ Z‬ﻋﲆ اﻟﱰﺗﻴﺐ ﰲ ﻣﻌﻠﻢ ﻣﺘﻌﺎﻣﺪ و ﻣﺘﺠﺎﻧﺲ ⃗ ‪. O, i,⃗ j‬‬ ‫‪VI‬‬
‫‪ •3‬أﺣﺴﺐ ‪ Z‬و ‪ ، Z‬ﻻﺣﻘﺘ َْﻲ اﻟﺸﻌﺎﻋﲔ ‪ AB‬و ‪ ، AC‬ﺛﻢ أﻛﺘﺒﻬﲈ ﻋﲆ اﻟﺸﻜﻠﲔ اﳉﱪي و اﻷﳼ‪.‬‬
‫‪ƕ‬‬

‫‪. Z = √3e‬‬ ‫‪/‬‬ ‫‪Z‬‬ ‫‪ •4‬أﺛﺒﺖ ّ‬


‫أن‬

‫ﺍﳊﻞّ‪.‬‬
‫‪ũŏ‬‬

‫‪ّ •1‬‬
‫ﻛﻞ اﻟﺰواﻳﺎ ﺷﻬﲑة و ﺑﺎﻟﺘﺎﱄ ‪:‬‬

‫𝟮𝟮𝟭‬

‫‪http ://tinyurl.com/Malki1718‬‬ ‫‪0‬‬


‫‪ .VI‬ﲤﺎرﻳﻦ ﺗﻄﺒﻴﻘﻴﺔ‬
‫‪.‬‬
‫‪4‬‬ ‫‪A‬‬

‫‪Ŕž Ŧ Ľ‬‬
‫‪3‬‬

‫‪B‬‬

‫‪œǃ‬‬
‫‪2‬‬

‫‪1‬‬

‫‪1‬‬ ‫‪2‬‬ ‫‪3‬‬ ‫‪4‬‬

‫‪Ŀ‬‬
‫‪−4‬‬ ‫‪−3‬‬ ‫‪−2‬‬ ‫‪−1‬‬ ‫‪0‬‬

‫‪−1‬‬

‫‪−2‬‬ ‫‪C‬‬

‫ﺷﻜﻞ ‪9.VI‬‬

‫اﻟﺸﻜﻞ اﻷﳼ‬ ‫اﻟﺸﻜﻞ اﳌﺜﻠﺜﻲ‬ ‫اﻟﻌﺪد اﻟﺸﻜﻞ اﳉﱪي‬

‫‪/‬‬ ‫𝜋‬ ‫𝜋‬


‫‪4e‬‬ ‫‪4 cos‬‬ ‫‪+ ı sin‬‬ ‫‪4ı‬‬ ‫‪Z‬‬
‫‪2‬‬ ‫‪2‬‬
‫‪/‬‬ ‫𝜋‪5‬‬ ‫𝜋‪5‬‬
‫‪4e‬‬ ‫‪4 cos‬‬ ‫‪+ ı sin‬‬ ‫‪−2√3 + 2ı‬‬ ‫‪Z‬‬
‫‪6‬‬ ‫‪6‬‬
‫𝜋‬ ‫𝜋‬
‫‪4e−‬‬ ‫‪/‬‬ ‫‪4 cos −‬‬ ‫‪+ ı sin −‬‬ ‫‪2√3 − 2ı‬‬ ‫‪Z‬‬
‫‪6‬‬ ‫‪6‬‬

‫‪ •2‬أﻧﻈﺮ اﻟﺸﻜﻞ ‪. 9.VI‬‬


‫‪ •3‬ﻟﺪﻳﻨﺎ ‪:‬‬
‫‪Z=Z −Z‬‬ ‫‪= −2√3 + 2ı − 4ı = −2√3 − 2ı‬‬ ‫)اﻟﺸﻜﻞ اﳉﱪي(‬
‫‪ņőƱŀ Ţ‬‬
‫‪√3 1‬‬ ‫𝜋‪7‬‬ ‫𝜋‪7‬‬ ‫‪/‬‬
‫‪=4 −‬‬ ‫‪− ı‬‬ ‫‪= 4 cos‬‬ ‫‪+ ı sin‬‬ ‫‪= 4e‬‬ ‫)اﻟﺸﻜﻞ اﻷﳼ(‬
‫‪2‬‬ ‫‪2‬‬ ‫‪6‬‬ ‫‪6‬‬
‫‪Z =Z −Z‬‬ ‫‪= 2√3 − 2ı − 4ı = 2√3 − 6ı‬‬ ‫)اﻟﺸﻜﻞ اﳉﱪي(‬
‫‪VI‬‬

‫‪1 √3‬‬ ‫𝜋‬ ‫𝜋‬


‫‪= 4 √3‬‬ ‫‪−‬‬ ‫‪= 4√3 cos −‬‬ ‫‪+ ı sin −‬‬ ‫‪= 4√3e−‬‬ ‫‪/‬‬ ‫)اﻟﺸﻜﻞ اﻷﳼ(‬
‫‪2‬‬ ‫‪2‬‬ ‫‪3‬‬ ‫‪3‬‬

‫‪ •4‬ﻟﺪﻳﻨﺎ ‪:‬‬
‫‪/‬‬ ‫‪/‬‬ ‫‪/‬‬ ‫‪+‬‬ ‫‪/‬‬
‫‪√3e‬‬ ‫‪Z = √3e‬‬ ‫‪⋅ 4e‬‬ ‫‪= 4√3e‬‬ ‫‪= 4√3e‬‬
‫‪ƕ‬‬

‫‪= 4√3e‬‬ ‫‪/ −‬‬ ‫‪= 4√3e−‬‬ ‫‪/‬‬ ‫‪=Z‬‬


‫𝜋‬
‫إذن اﻟﺰاوﻳﺔ ﺑﲔ اﻟﺸﻌﺎﻋﲔ ‪ AB‬و ‪ AC‬ﻫﻲ و ﻫﺬا ﻳﻌﻨﻲ أﳖﲈ ﻣﺘﻌﺎﻣﺪان )ﺷﻜﻞ ‪. (9.VI‬‬
‫‪2‬‬
‫■‬
‫‪ũŏ‬‬

‫‪6 − ı√2‬‬
‫‬ ‫‪.‬‬
‫𝟔𝟓‬ ‫‪.‬‬
‫√ = ‪ u‬و ‪ . v = 1 − ı‬إﺳﺘﻨﺘﺞ اﻟﻄﻮﻳﻠﺔ و ﻋﻤﺪة ﻟﻠﻌﺪد‬ ‫أﺣﺴﺐ اﻟﻄﻮﻳﻠﺔ و ﻋﻤﺪة ﻟﻠﻌﺪدﻳﻦ‬
‫‪2‬‬
‫‪u‬‬
‫= ‪.w‬‬
‫‪v‬‬

‫𝟯𝟮𝟭‬

‫‪http ://tinyurl.com/Malki1718‬‬ ‫‪0‬‬


‫ اﻟﺸﲁ اﳌﺜﻠﱻ ﻟﻌﺪد ﻣﺮﻛﺐ‬.2.VI
.
: ‫ ﻟﺪﻳﻨﺎ‬.ّ‫ﺍﳊﻞ‬

Ŕž Ŧ Ľ
√3 1 𝜋 𝜋
u = √2 − ı = √2 cos − ı sin = √2e− /
2 2 6 6

œǃ
√2 √2 𝜋 𝜋
v = √2 −ı = √2 cos − ı sin = √2e− / ‫و‬
2 2 4 4
u √2e− /
− + /

Ŀ
= =e =e : ‫ﻣﻨﻪ‬
v √2e− /

𝜋
■ . arg (w) = (mod 2𝜋) ‫| و‬w| = 1 ‫و ﺑﺎﻟﺘﺎﱄ‬
12

ّ ‫أﺛﺒﺖ‬
: ‫أن‬
 𝟓𝟕. .

𝜋 𝜋 1 − ı√3 5𝜋 5𝜋
cos + ı sin (1 + ı) = √2 cos + ı sin • 1
7 7 2 84 84

𝜋 𝜋 13𝜋 13𝜋
(1 − ı) cos + ı sin √3 − ı = 2√2 cos − ı sin • 2
5 5 60 60

1 𝜋 𝜋 √3 − ı
√2 cos + ı sin = • 3
1+ı 12 12 2

‫ و اﻟﻌﻼﻗﺘﲔ‬e = cos 𝜃 + ı sin 𝜃 : ‫ ﻧُﻄ ّﺒﻖ ﺻﻴﻐﺔ ﻣﻮﺍﻓﺮ‬.ّ‫ﺍﳊﻞ‬


e − +
. =e ‫و‬e e = e
e
: ‫• ﻟﺪﻳﻨﺎ‬1
ņőƱŀ Ţ
𝜋 𝜋 1 − ı√3 / 𝜋 𝜋 √2 √2
cos + ı sin (1 + ı) = e + ı sin −
cos − ⋅ √2 +ı
7 7 2 3 3 2 2
𝜋 𝜋
= √2e / e− / cos + ı sin

VI
4 4
= √2e / e− / e / = √2e − +
5𝜋 5𝜋
= √2e / = √2 cos + ı sin
84 84
ƕ

: ‫• ﻟﺪﻳﻨﺎ‬2
𝜋 𝜋 √2 √2 / √3 1
(1 − ı) cos + ı sin √3 − ı = √2 −ı e ⋅2 − ı
5 5 2 2 2 2
ũŏ

/ 𝜋 𝜋 𝜋 𝜋
= 2√2e cos − + ı sin − cos − + ı sin −
4 4 6 6
/ − / − / − −
= 2√2e e e = 2√2e
13𝜋 13𝜋
= 2√2e− / = 2√2 cos − + ı sin −
60 60
13𝜋 13𝜋
= 2√2 cos − ı sin
60 60

𝟭𝟮𝟰

http ://tinyurl.com/Malki1718 0
‫ ﲤﺎرﻳﻦ ﺗﻄﺒﻴﻘﻴﺔ‬.VI
.
: ‫• ﻟﺪﻳﻨﺎ‬3
1 𝜋 𝜋 / /
√2e e

Ŕž Ŧ Ľ
√2 cos + ı sin = =
1+ı 12 12 √2 +ı cos + ı sin

œǃ
e / 𝜋 𝜋

= /
=e = e− / = cos − + ı sin −
e 6 6
𝜋 𝜋 √3 1 √3 − ı
= cos − ı sin = − ı=
6 6 2 2 2

Ŀ

ً
. u = cos 𝜃 + ı sin 𝜃 ‫ ﻧﻀﻊ‬. 𝜃 ∈ [0, 2𝜋[ ‫ﻟﻴﻜﻦ 𝜃 ﻋﺪدا ﺣﻘﻴﻘﻴﺎ ﺣﻴﺚ‬
.
𝟓𝟖 . #
. z = ue / + ue / ‫أوﺟﺪ ﺣﺴﺐ ِﻗ َﻴﻢ 𝜃 اﻟﻄﻮﻳﻠﺔ و ﻋﻤﺪ ًة ﻟﻠﻌﺪد‬

: 2 ‫ ﻟﺪﻳﻨﺎ‬.ّ‫ﺍﳊﻞ‬
+ − + 𝜋 𝜋 𝜋 𝜋
z=e +e = cos 𝜃 +
+ cos −𝜃 + + ı sin 𝜃 + + sin −𝜃 +
4 6 4 6
1 𝜋 𝜋 1 𝜋 𝜋 1 𝜋 𝜋 1 𝜋 𝜋
= 2 cos 𝜃+ −𝜃+ cos 𝜃+ +𝜃− + 2ı sin 𝜃+ −𝜃+ cos 𝜃+ +𝜃−
2 4 6 2 4 6 2 4 6 2 4 6
5𝜋 𝜋 5𝜋 𝜋 𝜋 5𝜋 5𝜋
= 2 cos cos 𝜃 − + 2ı sin cos 𝜃 − = 2 cos 𝜃 − cos + ı sin
24 24 24 24 24 24 24
𝜋
= 2 cos 𝜃 − e /
24
: ‫ ﻧُﻤ ﱢﻴﺰ ﺛﻼث ﺣﺎﻻت‬، 20 ‫ ﺻﻔﺤﺔ‬8 ‫و ﺣﺴﺐ اﳌﱪﻫﻨﺔ‬
5𝜋 𝜋
. arg (z) = (mod 2𝜋) ‫| و‬z| = 2 cos 𝜃 − ّ cos 𝜃 − 𝜋 > 0 ‫إذا ﻛﺎن‬
‫ﻓﺈن‬ •
24 24 24
ņőƱŀ Ţ
ّ ، 𝜃 ∈ [0, 2𝜋[ ‫أن‬
: ‫ﻓﺈن‬ ّ ‫ و ﺑﲈ‬،‫ﻟﻜﻦ‬
𝜋 𝜋 𝜋 3𝜋 𝜋
cos 𝜃 − >0 ⟺ 0≤𝜃− < ‫أو‬ <𝜃− < 2𝜋
24 24 2 2 24
VI

𝜋 13𝜋 37𝜋 𝜋
⟺ ≤𝜃< ‫أو‬ < 𝜃 < 2𝜋 +
24 24 24 24
𝜋 13𝜋 37𝜋
⟺ ≤𝜃< ‫أو‬ < 𝜃 < 2𝜋
24 24 24
𝜋 13𝜋 37𝜋
ّ 𝜃∈
: ‫ﻓﺈن‬ , ∪ , 2𝜋 ‫ ﻷﺟﻞ‬،‫إذن‬
ƕ

24 24 24
5𝜋 𝜋
. arg (z) = (mod 2𝜋) ‫و‬ |z| = 2 cos 𝜃 −
24 24

ّ 𝜃 = 37𝜋 ‫ أي إذا ﻛﺎن 𝜋 = 𝜃 أو‬cos 𝜃 − 13𝜋 = 0 ‫إذا ﻛﺎن‬


ũŏ

.(‫ )و اﻟﻌﺪد اﳌﺮﻛﺐ اﳌﻌﺪوم ﻻ ﻋﻤﺪة ﻟﻪ‬z = 0 ‫ﻓﺈن‬ •


24 24 24
13𝜋 37𝜋 13𝜋
ّ
: ‫ﻓﺈن‬ <𝜃< ‫ أي إذا ﻛﺎن‬cos 𝜃 − < 0 ‫إذا ﻛﺎن‬ •
24 24 24
5𝜋 29𝜋 𝜋
. arg (z) = +𝜋= (mod 2𝜋) ‫و‬ |z| = −2 cos 𝜃 −
24 24 24
. sin + sin = sin +
cos −
‫و‬ cos + cos = cos +
cos − ّ ‫ﻧُﺬﻛﱢﺮ‬2
‫أن‬

𝟭𝟮𝟱

http ://tinyurl.com/Malki1718 0
‫‪ .2.VI‬اﻟﺸﲁ اﳌﺜﻠﱻ ﻟﻌﺪد ﻣﺮﻛﺐ‬
‫‪.‬‬
‫■‬

‫‪Ŕž Ŧ Ľ‬‬
‫‬ ‫‪𝟓𝟗.‬‬ ‫‪.‬‬

‫‪œǃ‬‬
‫‪ •1‬أﻛﺘﺐ ﻋﲆ اﻟﺸﻜﻞ اﳌﺜﻠﺜﻲ اﻟﻌﺪد ‪ 1 + e‬ﺣﻴﺚ [𝜋 ‪ِ . 𝜃 ∈ ]−𝜋,‬‬
‫أﻋﻂ ﺗﻔﺴﲑ ًا ﻫﻨﺪﺳﻴﺎ ﳍﺬه اﻟﻨﺘﻴﺠﺔ‪.‬‬
‫‪ •2‬أﺣﺴﺐ اﻟﻄﻮﻳﻠﺔ و ﻋﻤﺪة ﻟﻠﻌﺪد ‪ e − 1‬ﻣﻊ [𝜋 ‪. 𝜃 ∈ ]−𝜋,‬‬

‫‪Ŀ‬‬
‫‪e −1‬‬
‫ﻷﺟﻞ [𝜋 ‪. 𝜃 ∈ ]−𝜋,‬‬
‫‪e +1‬‬ ‫‪ّ •3‬‬
‫ﺑﺴﻂ اﻟﻌﺒﺎرة‬

‫‪ •1‬ﻟﺪﻳﻨﺎ ‪:‬‬ ‫ﺍﳊﻞّ‪.‬‬


‫‪/‬‬ ‫𝜃‬
‫‪1+e‬‬ ‫‪=e‬‬ ‫‪e−‬‬ ‫‪/‬‬ ‫‪+e‬‬ ‫‪/‬‬ ‫‪= 2e‬‬ ‫‪/‬‬ ‫‪cos‬‬
‫‪2‬‬
‫ﻓﺈن ‪ cos 𝜃 > 0‬و ﺣﺴﺐ اﳌﱪﻫﻨﺔ ‪ 8‬ﺻﻔﺤﺔ ‪ّ 20‬‬
‫ﻓﺈن اﻟﻜﺘﺎﺑﺔ اﻟﺴﺎﺑﻘﺔ ﻫﻲ اﻟﺸﻜﻞ اﻷﳼ ﻟﻠﻌﺪد‬ ‫أن [𝜋 ‪ّ 𝜃 ∈ ]−𝜋,‬‬ ‫ﺑﲈ ّ‬
‫‪2‬‬
‫𝜃 𝜃‬
‫‪. 1 + e = 2 cos ,‬‬ ‫‪ 1 + e‬و ﺑﺎﻟﺘﺎﱄ ﺷﻜﻠﻪ اﳌﺜﻠﺜﻲ ﻫﻮ‬
‫‪2 2‬‬
‫‪ B 1 + e‬و )‪ C (2‬؛ و اﻟﺪاﺋﺮة اﻟﺘﻲ ﻣﺮﻛﺰﻫﺎ )‪ (1, 0‬و ﻧﺼﻒ ﻗﻄﺮﻫﺎ‬‫اﻟﺘﻔﺴﲑ اﳍﻨﺪﳼ ‪ :‬ﻧﻌﺘﱪ اﻟﻨﻘﻂ )‪، A (1) ، O (0‬‬
‫‪) 1‬ﺷﻜﻞ ‪. (10.VI‬‬
‫أن ‪) CAB = 2COB‬أي أن اﻟﺰاوﻳﺔ اﳌﺮﻛﺰﻳﺔ ﺗﺴﺎوي ﺿﻌﻒ اﻟﺰاوﻳﺔ اﳌﺤﻴﻄﺔ( و ﻫﻲ ﻧﺘﻴﺠﺔ‬ ‫اﻟﻨﺘﻴﺠﺔ اﻟﺴﺎﺑﻘﺔ ﺗﻌﻨﻲ ّ‬
‫ﻫﻨﺪﺳﻴﺔ ﻣﻌﺮوﻓﺔ ‪.‬‬
‫‪ •2‬ﻟﺪﻳﻨﺎ ‪:‬‬
‫‪/‬‬ ‫‪/‬‬ ‫𝜃‬ ‫𝜃‬
‫‪e −1=e‬‬ ‫‪e‬‬ ‫‪−‬‬ ‫‪e− /‬‬ ‫‪= 2ıe‬‬ ‫‪/‬‬ ‫‪sin = 2e‬‬ ‫‪+‬‬ ‫‪/‬‬ ‫‪sin‬‬
‫‪2‬‬ ‫‪2‬‬
‫ﺣﺴﺐ اﳌﱪﻫﻨﺔ ‪ 8‬ﺻﻔﺤﺔ ‪: 20‬‬
‫‪ņőƱŀ Ţ‬‬
‫𝜃‬
‫إذا ﻛﺎن ‪ sin = 0‬أي إذا ﻛﺎن ‪ّ 𝜃 = 0‬‬
‫ﻓﺈن ‪. z = 0‬‬ ‫•‬
‫‪2‬‬
‫𝜋‪𝜃+‬‬ ‫𝜃‬
‫= )‪. arg (z‬‬ ‫ﻓﺈن ‪ |z| = 2 sin‬و )𝜋‪(mod 2‬‬‫إذا ﻛﺎن ‪ sin 𝜃 > 0‬أي إذا ﻛﺎن 𝜋 < 𝜃 < ‪ّ 0‬‬ ‫•‬
‫‪2‬‬ ‫‪2‬‬ ‫‪2‬‬

‫‪VI‬‬
‫𝜋‪𝜃+‬‬ ‫𝜃‬
‫= )‪. arg (z‬‬ ‫إذا ﻛﺎن ‪ sin 𝜃 < 0‬أي إذا ﻛﺎن ‪ّ −𝜋 < 𝜃 < 0‬‬
‫ﻓﺈن ‪ |z| = −2 sin‬و )𝜋‪+ 𝜋 (mod 2‬‬ ‫•‬
‫‪2‬‬ ‫‪2‬‬ ‫‪2‬‬
‫‪ •3‬ﻟﺪﻳﻨﺎ ‪:‬‬
‫‪/‬‬
‫‪e − 1 2ıe‬‬ ‫‪sin‬‬ ‫‪sin‬‬ ‫𝜃‬
‫=‬ ‫‪=ı‬‬ ‫‪= ı tan‬‬
‫‪e +1‬‬ ‫‪2e‬‬ ‫‪/‬‬ ‫‪cos‬‬ ‫‪cos‬‬ ‫‪2‬‬
‫‪ƕ‬‬

‫■‬

‫‬ ‫‪.‬‬
‫‪ũŏ‬‬

‫‪𝟔𝟎.‬‬
‫ﻟﻴﻜﻦ ‪ z = √3 − ı‬و ‪ . w = −2 + 2ı‬أوﺟﺪ اﳉﺰء اﳊﻘﻴﻘﻲ‪ ،‬اﳉﺰء اﻟﺘﺨﻴﲇ‪ ،‬اﻟﻄﻮﻳﻠﺔ و ﻋﻤﺪة ﻟﻸﻋﺪاد‬
‫اﻟﺘﺎﻟﻴﺔ ﺛﻢ اﻛﺘﺒﻬﺎ ﻋﲆ اﻟﺸﻜﻞ اﳉﱪي‪ ،‬اﻟﺸﻜﻞ اﳌﺜﻠﺜﻲ و اﻟﺸﻜﻞ اﻷﳼ ‪:‬‬
‫‪z‬‬
‫•‬ ‫‪zw‬‬ ‫•‬ ‫‪w‬‬ ‫•‬ ‫‪z‬‬ ‫•‬
‫‪w‬‬
‫ﻣ ّﺜﻞ اﻷﻋﺪاد أﻋﻼه ﰲ اﳌﺴﺘﻮي اﳌﺮﻛﺐ‪.‬‬

‫𝟲𝟮𝟭‬

‫‪http ://tinyurl.com/Malki1718‬‬ ‫‪0‬‬


‫ ﲤﺎرﻳﻦ ﺗﻄﺒﻴﻘﻴﺔ‬.VI
.
B 1 + eıθ

1

Ŕž Ŧ Ľ
œǃ
θ θ
2
O A (1) C (2)

Ŀ
−1

10.VI ‫ﺷﻜﻞ‬

.ّ‫ﺍﳊﻞ‬

(mod 2𝜋) 𝜃 ‫ﻋﻤﺪﺗﻪ‬ ‫ﻃﻮﻳﻠﺘﻪ‬ ‫ﺟﺰؤه اﻟﺘﺨﻴﲇ‬ ‫ﺟﺰؤه اﳊﻘﻴﻘﻲ‬ ‫اﻟﻌﺪد‬


𝜋
− 2 −1 √3 z = √3 − ı
6
3𝜋
2√2 2 −2 w = −2 + 2ı
4
7𝜋
4√2 2 √3 + 1 −2 √3 − 1 z⋅w
12
11𝜋 √2 √3 − 1 √3 + 1 z
− − −
12 2 4 4 w
ņőƱŀ Ţ
: ‫ﻓﻴﲈ ﻳﲇ ﺑﻌﺾ اﻟﺘﻔﺎﺻﻴﻞ‬

|z| = √3 + (−1) = 2 •
VI

Re (z) √3 Im (z) 1
𝜃 = arg (z) ⟺ cos 𝜃 = = ‫ و‬sin 𝜃 = =−
|z| 2 |z| 2
𝜋
⟺ 𝜃 = − (mod 2𝜋)
6
ƕ

𝜋 𝜋 𝜋
z = √3 − ı = 2 cos − + ı sin − = 2, − = 2e− / ‫أي‬
6 6 6

|w| = (−2) + 2 = √8 = 2√2 •


ũŏ

Re (z) −2 √2 Im (z) 2 √2
𝜃 = arg (w) ⟺ cos 𝜃 = = =− ‫ و‬sin 𝜃 = = =
|z| 2 √2 2 |z| 2 √2 2
3𝜋
⟺ 𝜃 = (mod 2𝜋)
4
3𝜋 3𝜋 3𝜋 /
w = −2 + 2ı = 2√2 cos + ı sin = 2√2, = 2√2e ‫أي‬
4 4 4

𝟭𝟮𝟳

http ://tinyurl.com/Malki1718 0
‫‪ .2.VI‬اﻟﺸﲁ اﳌﺜﻠﱻ ﻟﻌﺪد ﻣﺮﻛﺐ‬
‫‪.‬‬
‫ﻟﺪﻳﻨﺎ ‪:‬‬ ‫•‬

‫‪Ŕž Ŧ Ľ‬‬
‫‪z ⋅ w = √3 − ı (−2 + 2ı) = −2√3 + 2ı√3 + 2ı − 2ı‬‬

‫‪= −2√3 + 2ı√3 + 2ı + 2 = 2‬‬ ‫‪1 − √3 + ı 1 + √3‬‬

‫‪œǃ‬‬
‫ﻣﻨﻪ ‪ |z ⋅ w| = |z| ⋅ |w| = 2 × 2√2 = 4√2 :‬و‬
‫𝜋‪𝜋 3‬‬ ‫𝜋‪7‬‬
‫‪arg (z ⋅ w) = arg (z) + arg (w) (mod 2𝜋) = − +‬‬ ‫= )𝜋‪(mod 2‬‬ ‫)𝜋‪(mod 2‬‬
‫‪6‬‬ ‫‪4‬‬ ‫‪12‬‬

‫‪Ŀ‬‬
‫أي‬
‫𝜋‪7‬‬ ‫𝜋‪7‬‬ ‫𝜋‪7‬‬ ‫‪/‬‬
‫‪z⋅w = 2‬‬ ‫‪1 − √3 + ı 1 + √3‬‬ ‫‪= 4√2 cos‬‬ ‫‪+ ı sin‬‬ ‫‪= 4√2,‬‬ ‫‪= 4√2e‬‬
‫‪12‬‬ ‫‪12‬‬ ‫‪12‬‬

‫‪z‬‬ ‫‪√3 − ı‬‬ ‫)‪√3 − ı (−1 − ı‬‬ ‫‪1 + √3 1 − √3‬‬


‫=‬ ‫=‬ ‫‪=−‬‬ ‫‪+‬‬ ‫‪ı‬‬ ‫•‬
‫)‪w 2 (−1 + ı) 2 (−1 + ı) (−1 − ı‬‬ ‫‪4‬‬ ‫‪4‬‬
‫|‪|z‬‬ ‫|‪|z‬‬ ‫‪2‬‬ ‫‪√2‬‬
‫= |‪|| w || = |w‬‬ ‫=‬
‫‪2‬‬
‫‪2√2‬‬
‫‪z‬‬ ‫𝜋‪𝜋 3‬‬ ‫𝜋‪11‬‬
‫‪arg‬‬ ‫‪= arg (z) − arg (w) (mod 2𝜋) = − −‬‬ ‫‪(mod 2𝜋) = −‬‬ ‫)𝜋‪(mod 2‬‬
‫‪w‬‬ ‫‪6‬‬ ‫‪4‬‬ ‫‪12‬‬
‫أي‬

‫‪z‬‬ ‫‪1 + √3 1 − √3‬‬ ‫𝜋‪√2 11‬‬ ‫‪√2‬‬ ‫𝜋‪11‬‬ ‫𝜋‪11‬‬ ‫‪√2 −‬‬ ‫‪/‬‬
‫‪=−‬‬ ‫‪+‬‬ ‫=‪ı‬‬ ‫‪,−‬‬ ‫=‬ ‫‪cos −‬‬ ‫‪+ ı sin −‬‬ ‫=‬ ‫‪e‬‬
‫‪w‬‬ ‫‪4‬‬ ‫‪4‬‬ ‫‪2‬‬ ‫‪12‬‬ ‫‪2‬‬ ‫‪12‬‬ ‫‪12‬‬ ‫‪2‬‬

‫‪ u‬ﻻ ﺗﻨﺲ ّ‬
‫أن اﳉﺰء اﻟﺘﺨﻴﲇ ﻟﻠﻌﺪد اﳌﺮﻛﺐ ‪ z = a + ıb‬ﻫﻮ ‪) b‬و ﻫﻮ ﻋﺪد ﺣﻘﻴﻘﻲ( و ﻟﻴﺲ ‪. bı‬‬
‫ﲤﺜﻴﻞ اﻟﻨﻘﻂ اﻷرﺑﻌﺔ ﰲ اﳌﺴﺘﻮي اﳌﺮﻛﺐ ﻳﻜﻮن ﻛﲈ ﰲ اﻟﺸﻜﻞ ‪. 11.VI‬‬
‫■‬
‫‪ņőƱŀ Ţ‬‬
‫‪ z . 𝛼 ∈ − ,‬و ‪ z‬ﻋﺪدان ﻣﺮﻛﺒﺎن ﺣﻴﺚ‬
‫𝜋 𝜋‬
‫𝛼 ﻋﺪد ﺣﻘﻴﻘﻲ ﺣﻴﺚ‬
‫‬ ‫‪𝟔𝟏.‬‬ ‫‪.‬‬
‫‪2 2‬‬
‫‪1‬‬ ‫‪1‬‬
‫= ‪z‬‬ ‫و‬ ‫= ‪z‬‬

‫‪VI‬‬
‫𝛼 ‪1 − ı tan‬‬ ‫𝛼 ‪1 + ı tan‬‬
‫‪ •1‬أﻛﺘﺐ ‪ z‬ﻋﲆ ﺷﻜﻠﻪ اﳌﺜﻠﺜﻲ‪.‬‬

‫‪ •2‬أﺣﺴﺐ اﳉﺪاء ‪ّ z ⋅ z‬‬


‫ﺛﻢ اﺳﺘﻨﺘﺞ اﻟﺸﻜﻞ اﳌﺜﻠﺜﻲ ﻟﻠﻌﺪد ‪. z‬‬
‫‪ƕ‬‬

‫‪ •1‬ﻟﺪﻳﻨﺎ ‪:‬‬ ‫ﺍﳊﻞّ‪.‬‬


‫‪1‬‬ ‫𝛼 ‪1 + ı tan‬‬ ‫𝛼 ‪1 + ı tan‬‬
‫‪ũŏ‬‬

‫= ‪z‬‬ ‫=‬ ‫=‬ ‫)𝛼 ‪= cos 𝛼 (1 + ı tan‬‬


‫)𝛼 ‪1 − ı tan 𝛼 1 + (tan‬‬ ‫𝛼 ‪1 + tan‬‬
‫)𝛼 ‪= (cos 𝛼) (cos 𝛼 + ı cos 𝛼 tan 𝛼) = (cos 𝛼) (cos 𝛼 + ı sin‬‬
‫𝜋 𝜋‬
‫𝛼 ‪||z || = cos‬‬ ‫‪ّ 𝛼∈ − ,‬‬
‫ﻓﺈن ‪ cos 𝛼 > 0‬و ﺑﺎﻟﺘﺎﱄ ﻓﺎﻟﻜﺘﺎﺑﺔ اﻟﺴﺎﺑﻘﺔ ﻫﻲ اﻟﺸﻜﻞ اﳌﺜﻠﺜﻲ ﻟﻠﻌﺪد ‪ z‬أي ‪:‬‬ ‫و ﺑﲈ ّ‬
‫أن‬
‫‪2 2‬‬
‫‪.‬‬ ‫)𝜋‪arg (z ) = 𝛼 (mod 2‬‬ ‫و‬
‫]𝛼 ‪. z = [cos 𝛼,‬‬ ‫ﰲ اﻷﺧﲑ‪ ،‬اﻟﺸﻜﻞ اﳌﺜﻠﺜﻲ ﻟﻠﻌﺪد ‪ z‬ﻫﻮ ‪:‬‬

‫𝟴𝟮𝟭‬

‫‪http ://tinyurl.com/Malki1718‬‬ ‫‪0‬‬


‫ ﲤﺎرﻳﻦ ﺗﻄﺒﻴﻘﻴﺔ‬.VI
.

zw

Ŕž Ŧ Ľ
5

œǃ
4

Ŀ
2
w

−3 −2 −1 z 1 2
0
w
−1
z

11.VI ‫ﺷﻜﻞ‬

: ‫• ﻟﺪﻳﻨﺎ‬2
1 1 1 1
z ⋅z = = = = cos 𝛼
1 − ı tan 𝛼 1 + ı tan 𝛼 1 − (ı tan 𝛼) 1 + tan 𝛼

cos 𝛼
ņőƱŀ Ţ
: ‫ و ﺑﺎﻟﺘﺎﱄ‬z = ‫ﻣﻨﻪ‬
z
| cos 𝛼 | ||cos 𝛼|| cos 𝛼
||z || = | |= = = cos 𝛼
|| z || ||z || cos 𝛼
VI

arg (z ) = arg cos 𝛼 − arg (z ) = 0 − 𝛼 = −𝛼 (mod 2𝜋) ‫و‬


. z = [cos 𝛼, −𝛼] ‫ ﻫﻮ‬z ‫إذن ﻓﺎﻟﺸﻜﻞ اﳌﺜﻠﺜﻲ ﻟﻠﻌﺪد‬
ّ ‫ ⧏ ﻳﻤﻜﻦ ﻣﻼﺣﻈﺔ‬: 21 ‫ﻣﻼﺣﻈﺔ‬
. arg (z ) = − arg (z ) (mod 2𝜋) ‫|| و‬z || = ||z || : ‫ و ﺑﺎﻟﺘﺎﱄ‬z = z ‫أن‬

ƕ

# .
𝟔𝟐 .
ũŏ

: ( 𝛼, u, v ∈ ℝ ‫أﺣﺴﺐ اﻟﻄﻮﻳﻠﺔ و ﻋﻤﺪة ﻟﻸﻋﺪاد اﻟﺘﺎﻟﻴﺔ )ﻣﻊ‬


e +e • 3 e +e • 2 e •1

: ‫• ﻟﺪﻳﻨﺎ‬1 .ّ‫ﺍﳊﻞ‬
e = ecos + sin = ecos ⋅ e sin

𝟭𝟮𝟵

http ://tinyurl.com/Malki1718 0
‫‪ .2.VI‬اﻟﺸﲁ اﳌﺜﻠﱻ ﻟﻌﺪد ﻣﺮﻛﺐ‬
‫‪.‬‬
‫‪ e‬ﻣﻨﻪ ‪:‬‬ ‫أن ‪ ecos > 0‬ﻓﺤﺴﺐ اﳌﱪﻫﻨﺔ ‪ 8‬ﺻﻔﺤﺔ ‪ ، 20‬اﻟﻜﺘﺎﺑﺔ اﻟﺴﺎﺑﻘﺔ ﻫﻲ اﻟﺸﻜﻞ اﻷﳼ ﻟﻠﻌﺪد‬
‫ﺑﲈ ّ‬
‫‪. arg e‬‬ ‫)𝜋‪= sin 𝛼 (mod 2‬‬ ‫و‬ ‫‪||e‬‬ ‫‪|| = ecos‬‬

‫‪Ŕž Ŧ Ľ‬‬
‫‪+‬‬
‫‪ e‬ﻛﻌﺎﻣﻞ ﻣﺸﱰك ‪ .‬ﺑﺎﻟﺘﺤﺪﻳﺪ ‪:‬‬ ‫‪ •2‬ﻋﻨﺪ اﻟﺒﺤﺚ ﻋﻰ اﻟﺸﻜﻞ اﻷﳼ ﻟﻠﻌﺪد ‪ e + e‬ﻧﻀﻊ ﻋﺎﻣ ًﺔ‬
‫‪u−v‬‬

‫‪œǃ‬‬
‫‪+‬‬ ‫‪−‬‬ ‫‪−‬‬ ‫‪+‬‬
‫‪e‬‬ ‫‪+e‬‬ ‫‪=e‬‬ ‫‪e‬‬ ‫‪+ e−‬‬ ‫‪= 2e‬‬ ‫‪cos‬‬
‫‪2‬‬
‫و ﺣﺴﺐ اﳌﱪﻫﻨﺔ ‪ 8‬ﺻﻔﺤﺔ ‪ 20‬ﻓﺈﻧﻪ ‪:‬‬
‫𝜋 ‪u−v‬‬ ‫‪u−v‬‬
‫أي )𝜋‪u − v = 𝜋 (mod 2‬‬ ‫‪ cos‬أي إذا ﻛﺎن )𝜋 ‪= (mod‬‬ ‫إذا ﻛﺎن ‪= 0‬‬

‫‪Ŀ‬‬
‫•‬
‫‪2‬‬ ‫‪2‬‬ ‫‪2‬‬
‫ّ‬
‫ﻓﺈن ‪. e + e = 0 :‬‬
‫‪u−v‬‬ ‫𝜋‬ ‫𝜋‬ ‫‪u−v‬‬
‫ﻣﻊ ‪k ∈ ℤ‬‬ ‫‪ cos‬أي إذا ﻛﺎن 𝜋‪∈ − + 2k𝜋, + 2k‬‬ ‫إذا ﻛﺎن ‪> 0‬‬ ‫•‬
‫‪2‬‬ ‫‪2‬‬ ‫‪2‬‬ ‫‪2‬‬
‫أي [𝜋‪ u − v ∈ ]−𝜋 + 4k𝜋, 𝜋 + 4k‬ﻣﻊ ‪ّ k ∈ ℤ‬‬
‫ﻓﺈن ‪:‬‬
‫‪u+v‬‬ ‫‪u−v‬‬
‫‪. arg (e‬‬ ‫=) ‪+e‬‬ ‫)𝜋‪(mod 2‬‬ ‫و‬ ‫‪||e + e || = 2 cos‬‬
‫‪2‬‬ ‫‪2‬‬
‫‪u−v‬‬ ‫𝜋‬ ‫𝜋‪3‬‬ ‫‪u−v‬‬
‫ﻣﻊ ‪k ∈ ℤ‬‬ ‫∈‬ ‫‪+ 2k𝜋,‬‬ ‫‪ cos‬أي إذا ﻛﺎن 𝜋‪+ 2k‬‬ ‫إذا ﻛﺎن ‪< 0‬‬ ‫•‬
‫‪2‬‬ ‫‪2‬‬ ‫‪2‬‬ ‫‪2‬‬
‫أي [𝜋‪ u − v ∈ ]𝜋 + 4k𝜋, 3𝜋 + 4k‬ﻣﻊ ‪ّ k ∈ ℤ‬‬
‫ﻓﺈن ‪:‬‬
‫‪u+v‬‬ ‫‪u−v‬‬
‫= ) ‪. arg (e + e‬‬ ‫)𝜋‪+ 𝜋 (mod 2‬‬ ‫و‬ ‫‪||e‬‬ ‫‪+ e || = −2 cos‬‬
‫‪2‬‬ ‫‪2‬‬
‫ﻫﺬا اﳌﺜﺎل ﺣﺎﻟﺔ ﺧﺎﺻﺔ ﻣﻦ اﻟﺴﺎﺑﻖ ﻣﻊ ‪ v = 2u‬ﻣﻨﻪ ‪:‬‬ ‫•‬ ‫‪3‬‬
‫‪+‬‬ ‫‪u − 2u‬‬ ‫‪u‬‬
‫‪e‬‬ ‫‪+e‬‬ ‫‪= 2e‬‬ ‫‪cos‬‬ ‫‪= 2e / cos‬‬
‫‪2‬‬ ‫‪2‬‬
‫و ﺣﺴﺐ اﳌﱪﻫﻨﺔ ‪ 8‬ﺻﻔﺤﺔ ‪ 20‬ﻓﺈﻧﻪ ‪:‬‬
‫‪u‬‬
‫‪.e +e‬‬ ‫‪=0‬‬ ‫‪ cos‬أي إذا ﻛﺎن )𝜋‪ّ u = 𝜋 (mod 2‬‬
‫ﻓﺈن ‪:‬‬ ‫إذا ﻛﺎن ‪= 0‬‬ ‫•‬
‫‪2‬‬
‫إذا ﻛﺎن ‪ cos u > 0‬أي إذا ﻛﺎن [𝜋‪ u ∈ ]−𝜋 + 4k𝜋, 𝜋 + 4k‬ﻣﻊ ‪ّ k ∈ ℤ‬‬
‫ﻓﺈن ‪:‬‬ ‫•‬
‫‪2‬‬
‫‪3u‬‬ ‫‪u‬‬
‫‪ņőƱŀ Ţ‬‬
‫‪. arg e + e‬‬ ‫=‬ ‫)𝜋‪(mod 2‬‬ ‫و‬ ‫‪||e + e || = 2 cos‬‬
‫‪2‬‬ ‫‪2‬‬
‫‪u‬‬
‫ّ‬
‫إذا ﻛﺎن ‪ cos < 0‬أي إذا ﻛﺎن [𝜋‪ u ∈ ]𝜋 + 4k𝜋, 3𝜋 + 4k‬ﻣﻊ ‪ k ∈ ℤ‬ﻓﺈن ‪:‬‬ ‫•‬
‫‪2‬‬
‫‪3u‬‬ ‫‪u‬‬
‫‪. arg e + e‬‬ ‫=‬ ‫)𝜋‪+ 𝜋 (mod 2‬‬ ‫و‬ ‫|‬ ‫|‬
‫‪|e + e | = −2 cos‬‬
‫‪2‬‬ ‫‪2‬‬

‫‪VI‬‬
‫■‬

‫أﻛﺘﺐ ﻋﲆ ﺷﻜﻠﻪ اﳌﺜﻠﺜﻲ اﻟﻌﺪد ‪ z‬ﺣﻴﺚ ‪:‬‬


‫‬ ‫‪𝟔𝟑.‬‬ ‫‪.‬‬
‫‪ƕ‬‬

‫𝛼 ‪1 + ı tan‬‬ ‫𝜋 𝜋‬
‫=‪z‬‬ ‫‪,‬‬ ‫‪𝛼∈ − ,‬‬
‫𝛼 ‪1 − ı tan‬‬ ‫‪2 2‬‬

‫ﺍﳊﻞّ‪ .‬ﻟﺪﻳﻨﺎ ‪:‬‬


‫‪ũŏ‬‬

‫‪sin‬‬
‫‪1 + ı tan 𝛼 1 + ı cos‬‬ ‫𝛼 ‪cos 𝛼 + ı sin‬‬ ‫)𝛼 ‪1 ⋅ (cos 𝛼 + ı sin‬‬
‫=‪z‬‬ ‫=‬ ‫‪sin‬‬
‫=‬ ‫=‬
‫‪1 − ı tan 𝛼 1 − ı cos‬‬ ‫))𝛼‪cos 𝛼 − ı sin 𝛼 1 ⋅ (cos (−𝛼) + ı sin (−‬‬
‫]𝛼 ‪[1,‬‬ ‫‪1‬‬
‫=‬ ‫=‬ ‫]𝛼‪, 𝛼 − (−𝛼) = [1, 2‬‬
‫]𝛼‪[1, −‬‬ ‫‪1‬‬
‫■‬ ‫و ﻫﻮ اﻟﺸﻜﻞ اﳌﺜﻠﺜﻲ ﻟﻠﻌﺪد ‪. z‬‬

‫𝟬𝟯𝟭‬

‫‪http ://tinyurl.com/Malki1718‬‬ ‫‪0‬‬


‫‪ .VI‬ﲤﺎرﻳﻦ ﺗﻄﺒﻴﻘﻴﺔ‬
‫‪.‬‬

‫‪#‬‬ ‫‪.‬‬
‫𝟒𝟔‬ ‫‪.‬‬

‫‪Ŕž Ŧ Ľ‬‬
‫𝛼 ﻋﺪد ﺣﻘﻴﻘﻲ‪ z .‬و ‪ z‬ﻋﺪدان ﻣﺮﻛﺒﺎن ﺣﻴﺚ ‪:‬‬
‫‪1‬‬
‫𝛼 ‪z = 1 + ı tan‬‬ ‫‪,‬‬ ‫= ‪z‬‬

‫‪œǃ‬‬
‫‪1−z‬‬
‫ِ‬
‫ﻣﻌﺮﻓﲔ‪.‬‬ ‫‪ّ •1‬‬
‫ﻋﲔ ﻗ َﻴﻢ اﻟﻌﺪد 𝛼 ﺑﺤﻴﺚ ﻳﻜﻮن ‪ z‬و ‪ّ z‬‬

‫‪ّ •2‬‬
‫ﻋﲔ اﻟﻄﻮﻳﻠﺔ و ﻋﻤﺪة ﻟﻜﻞ ﻣﻦ ‪ z‬و ‪ z‬ﰲ اﳊﺎﻟﺘﲔ ‪:‬‬

‫‪Ŀ‬‬
‫𝜋‪2‬‬ ‫𝜋‬
‫=𝛼‬ ‫•‬ ‫=𝛼‬ ‫•‬
‫‪3‬‬ ‫‪3‬‬

‫‪ •3‬أﺣﺴﺐ ﺑﺪﻻﻟﺔ 𝛼 اﻟﻄﻮﻳﻠﺔ و ﻋﻤﺪة ﻟﻜﻞ ﻣﻦ ‪ z‬و ‪. z‬‬

‫ﻌﺮﻓ ًﺎ أي إذا و ﻓﻘﻂ إذا ﻛﺎن ‪ cos 𝛼 ≠ 0‬أي إذا و ﻓﻘﻂ إذا ﻛﺎن‬
‫ﻌﺮﻓ ًﺎ إذا و ﻓﻘﻂ إذا ﻛﺎن 𝛼 ‪ُ tan‬ﻣ ّ‬
‫‪ •1‬ﻳﻜﻮن ‪ُ z‬ﻣ ّ‬ ‫ﺍﳊﻞّ‪.‬‬
‫𝜋‬
‫)𝜋 ‪. 𝛼 ≠ (mod‬‬
‫‪2‬‬
‫𝜋‬
‫≠ 𝛼 و ‪ tan 𝛼 ≠ 0‬أي‬ ‫ﻌﺮﻓ ًﺎ و ‪ z ≠ 1‬أي إذا و ﻓﻘﻂ إذا ﻛﺎن )𝜋 ‪(mod‬‬
‫ﻌﺮﻓ ًﺎ إذا و ﻓﻘﻂ إذا ﻛﺎن ‪ُ z‬ﻣ ّ‬
‫ﻳﻜﻮن ‪ُ z‬ﻣ ّ‬
‫‪2‬‬
‫𝜋‬ ‫𝜋‬
‫إذا و ﻓﻘﻂ إذا ﻛﺎن )𝜋 ‪ 𝛼 ≠ (mod‬و ‪ sin 𝛼 ≠ 0‬أي إذا و ﻓﻘﻂ إذا ﻛﺎن )𝜋 ‪ 𝛼 ≠ (mod‬و )𝜋 ‪𝛼 ≠ 0 (mod‬‬
‫‪2‬‬ ‫‪2‬‬
‫𝜋‬
‫‪. 𝛼 ≠ 0 mod‬‬ ‫أي إذا و ﻓﻘﻂ إذا ﻛﺎن‬
‫‪2‬‬
‫𝜋‬
‫‪. 𝛼 ≠ 0 mod‬‬ ‫ﻣﻌﺮﻓﲔ إذا و ﻓﻘﻂ إذا ﻛﺎن‬
‫ﰲ اﻷﺧﲑ‪ ،‬ﻳﻜﻮن ‪ z‬و ‪ّ z‬‬
‫‪2‬‬
‫𝜋‬
‫• ﻷﺟﻞ = 𝛼 ﻳﻜﻮن ‪:‬‬ ‫•‬‫‪2‬‬
‫‪3‬‬
‫𝜋‬ ‫𝜋‬ ‫𝜋‬ ‫𝜋‬
‫‪= 1 + ı√3 = 2 cos + ı sin‬‬
‫‪z = 1 + ı tan‬‬ ‫‪= 2,‬‬
‫‪3‬‬ ‫‪3‬‬ ‫‪3‬‬ ‫‪3‬‬
‫‪ņőƱŀ Ţ‬‬
‫‪1‬‬ ‫‪1‬‬ ‫‪1‬‬ ‫‪√3‬‬ ‫𝜋 ‪√3‬‬
‫= ‪z‬‬ ‫=‬ ‫‪=−‬‬ ‫=‬ ‫=‪ı‬‬ ‫‪,‬‬
‫‪1−z‬‬ ‫‪1 − 1 − ı√3‬‬ ‫‪√3ı‬‬ ‫‪3‬‬ ‫‪3 2‬‬

‫𝜋‬ ‫‪3‬‬ ‫𝜋‬


‫= ) ‪. arg (z‬‬ ‫أي ‪ ||z || = 2 :‬؛ = ) ‪ arg (z‬؛ √ = || ‪ ||z‬و‬
‫‪VI‬‬

‫‪2‬‬ ‫‪3‬‬ ‫‪3‬‬


‫𝜋‪2‬‬
‫=𝛼 ّ‬
‫ﻓﺈن ‪:‬‬ ‫إذا ﻛﺎن‬ ‫•‬
‫‪3‬‬
‫𝜋‬ ‫𝜋‬ ‫𝜋‬ ‫𝜋‬
‫‪= 1 − ı√3 = 2 cos −‬‬
‫‪z = 1 + ı tan‬‬ ‫‪+ ı sin −‬‬ ‫‪= 2, −‬‬
‫‪3‬‬ ‫‪3‬‬ ‫‪3‬‬ ‫‪3‬‬
‫‪ƕ‬‬

‫‪1‬‬ ‫‪1‬‬ ‫‪1‬‬ ‫‪√3‬‬ ‫𝜋 ‪√3‬‬


‫= ‪z‬‬ ‫=‬ ‫=‬ ‫‪=−‬‬ ‫=‪ı‬‬ ‫‪,−‬‬
‫‪1−z‬‬ ‫‪1 − 1 + ı√3 √3ı‬‬ ‫‪3‬‬ ‫‪3‬‬ ‫‪2‬‬

‫𝜋‬ ‫‪3‬‬ ‫𝜋‬


‫أي ‪ ||z || = 2 :‬؛ ‪ arg (z ) = −‬؛ √ = || ‪ ||z‬و ‪. arg (z ) = −‬‬
‫‪ũŏ‬‬

‫‪2‬‬ ‫‪3‬‬ ‫‪3‬‬


‫𝜋‬
‫‪ . 𝛼 ≠ 0‬ﻟﺪﻳﻨﺎ ‪:‬‬ ‫‪mod‬‬ ‫‪ •3‬ﻟﻴﻜﻦ‬
‫‪2‬‬
‫‪1‬‬
‫= 𝛼 ‪z = 1 + ı tan‬‬ ‫)𝛼 ‪(cos 𝛼 + ı sin‬‬
‫𝛼 ‪cos‬‬
‫و ﺑﺎﻟﺘﺎﱄ ‪:‬‬

‫𝟭𝟯𝟭‬

‫‪http ://tinyurl.com/Malki1718‬‬ ‫‪0‬‬


‫‪ .2.VI‬اﻟﺸﲁ اﳌﺜﻠﱻ ﻟﻌﺪد ﻣﺮﻛﺐ‬
‫‪.‬‬
‫‪1‬‬
‫= || ‪ ||z‬و )𝜋‪ arg (z ) = 𝛼 (mod 2‬؛‬ ‫إذا ﻛﺎن ‪ّ cos 𝛼 > 0‬‬
‫ﻓﺈن‬ ‫•‬
‫𝛼 ‪cos‬‬

‫‪Ŕž Ŧ Ľ‬‬
‫‪1‬‬
‫‪ ||z || = −‬و )𝜋‪. arg (z ) = 𝛼 + 𝜋 (mod 2‬‬ ‫أ ّﻣﺎ إذا ﻛﺎن ‪ّ cos 𝛼 < 0‬‬
‫ﻓﺈن‬ ‫•‬
‫𝛼 ‪cos‬‬

‫‪œǃ‬‬
‫ﺑﺎﳌﺜﻞ ‪:‬‬
‫‪1‬‬ ‫‪1‬‬ ‫‪1‬‬
‫= ‪z‬‬ ‫=‬ ‫=‬ ‫𝛼 ‪ı = ı cotan‬‬
‫‪1−z‬‬ ‫𝛼 ‪−ı tan 𝛼 tan‬‬

‫‪Ŀ‬‬
‫و ﺑﺎﻟﺘﺎﱄ ‪:‬‬
‫𝜋‬
‫= ) ‪ arg (z‬؛‬ ‫إذا ﻛﺎن ‪ّ cotan 𝛼 > 0‬‬
‫ﻓﺈن 𝛼 ‪ ||z || = cotan‬و )𝜋‪(mod 2‬‬ ‫•‬
‫‪2‬‬
‫𝜋‬
‫و إذا ﻛﺎن ‪ّ cotan 𝛼 < 0‬‬
‫ﻓﺈن 𝛼 ‪ ||z || = − cotan‬و )𝜋‪. arg (z ) = − (mod 2‬‬ ‫•‬
‫‪2‬‬
‫■‬

‫ﻟﻴﻜﻦ 𝜑 ﻋﺪد ًا ﺣﻘﻴﻘﻴﺎ ﺑﺤﻴﺚ 𝜋 ≤ 𝜑 < 𝜋‪ . −‬أوﺟﺪ اﻟﻄﻮﻳﻠﺔ و ﻋﻤﺪة ﻟﻸﻋﺪاد اﳌﺮﻛﺒﺔ اﻵﺗﻴﺔ ‪:‬‬
‫‬ ‫‪𝟔𝟓.‬‬ ‫‪.‬‬

‫)𝜑 ‪c = cos 𝜑 + ı (1 + sin‬‬ ‫•‬ ‫‪3‬‬ ‫𝜑 ‪a = 1 + cos 𝜑 + ı sin‬‬ ‫•‬‫‪1‬‬


‫)𝜑 ‪b = sin 𝜑 + ı (1 + cos‬‬ ‫‪•2‬‬

‫‪ •1‬اﻟﻄﺮﻳﻘﺔ اﻷوﱃ ‪ :‬ﻟﺪﻳﻨﺎ ‪:‬‬ ‫ﺍﳊﻞّ‪.‬‬


‫‪/‬‬ ‫𝜑‬
‫‪a = 1 + cos 𝜑 + ı sin 𝜑 = 1 + e‬‬ ‫‪=e‬‬ ‫‪e−‬‬ ‫‪/‬‬ ‫‪+e‬‬ ‫‪/‬‬ ‫‪= 2e‬‬ ‫‪/‬‬ ‫‪cos‬‬
‫‪2‬‬
‫𝜋‬ ‫𝜋 𝜑‬
‫< ‪ −‬ﻣﻨﻪ ‪:‬‬ ‫≤‬ ‫أن 𝜋 ≤ 𝜑 < 𝜋‪ّ −‬‬
‫ﻓﺈن‬ ‫ﺑﲈ ّ‬
‫‪2‬‬ ‫‪2‬‬ ‫‪2‬‬
‫‪ņőƱŀ Ţ‬‬
‫𝜑‬
‫‪ cos‬أي إذا ﻛﺎن 𝜋 = 𝜑 ّ‬
‫ﻓﺈن ‪. a = 0‬‬ ‫• إذا ﻛﺎن ‪= 0‬‬
‫‪2‬‬
‫𝜑‬
‫• إذا ﻛﺎن ‪ cos 𝜑 ≠ 0‬أي إذا ﻛﺎن [𝜋 ‪ّ 𝜑 ∈ ]−𝜋,‬‬
‫ﻓﺈن ‪ cos > 0‬و ﺣﺴﺐ اﳌﱪﻫﻨﺔ �� ﺻﻔﺤﺔ �� ‪ ،‬اﻟﻜﺘﺎﺑﺔ‬
‫‪2‬‬ ‫‪2‬‬
‫𝜑‬ ‫𝜑‬ ‫𝜑‬
‫‪ 2e / cos‬ﻫﻲ اﻟﺸﻜﻞ اﻷﳼ ﻟﻠﻌﺪد ‪ a‬و ﺑﺎﻟﺘﺎﱄ ‪ |a| = 2 cos‬و )𝜋‪. arg (a) = (mod 2‬‬

‫‪VI‬‬
‫‪2‬‬ ‫‪2‬‬ ‫‪2‬‬
‫𝜑‬
‫أن ‪ّ 1 + cos 𝜑 = 2 cos‬‬
‫ﻓﺈن ‪:‬‬ ‫اﻟﻄﺮﻳﻘﺔ اﻟﺜﺎﻧﻴﺔ ‪ :‬ﺑﲈ ّ‬
‫‪2‬‬
‫𝜑‬ ‫𝜑‬ ‫𝜑‬ ‫𝜑‬ ‫𝜑‬ ‫𝜑‬ ‫𝜑‬
‫‪a = 2 cos‬‬ ‫‪+ 2ı sin cos = 2 cos‬‬ ‫‪cos + ı sin‬‬ ‫‪= 2e / cos‬‬
‫‪2‬‬ ‫‪2‬‬ ‫‪2‬‬ ‫‪2‬‬ ‫‪2‬‬ ‫‪2‬‬ ‫‪2‬‬
‫ﺛﻢ ﻧﺘﻤﻢ اﳊﻞ ﺑﻨﻔﺲ اﻟﻄﺮﻳﻘﺔ اﻟﺴﺎﺑﻘﺔ‪.‬‬
‫و ﻣﻦ ّ‬
‫‪ƕ‬‬

‫𝜋‬ ‫𝜋‬
‫‪sin (x) = cos‬‬ ‫‪−x‬‬ ‫‪ cos (x) = sin‬و‬ ‫‪−x‬‬ ‫أن 𝜑 ‪ 1 + cos 𝜑 = 2 cos‬؛ و ﺑﲈ ّ‬
‫أن‬ ‫‪ •2‬اﻟﻄﺮﻳﻘﺔ اﻷوﱃ ‪ :‬ﺑﲈ ّ‬
‫‪2‬‬ ‫‪2‬‬ ‫‪2‬‬
‫ّ‬
‫ﻓﺈن ‪:‬‬
‫‪ũŏ‬‬

‫𝜑‬ ‫𝜑‬ ‫𝜑‬ ‫𝜑‬ ‫𝜑‬ ‫𝜑‬


‫‪b = 2 cos‬‬ ‫‪sin + 2ı cos‬‬ ‫‪= 2 cos‬‬ ‫‪sin + ı cos‬‬
‫‪2‬‬ ‫‪2‬‬ ‫‪2‬‬ ‫‪2‬‬ ‫‪2‬‬ ‫‪2‬‬
‫𝜑‬ ‫𝜑 𝜋‬ ‫𝜑 𝜋‬ ‫𝜑‬ ‫‪−‬‬ ‫‪/‬‬
‫‪= 2 cos‬‬ ‫‪cos‬‬ ‫‪−‬‬ ‫‪+ ı sin‬‬ ‫‪−‬‬ ‫‪= 2 cos e‬‬
‫‪2‬‬ ‫‪2 2‬‬ ‫‪2 2‬‬ ‫‪2‬‬

‫ﺛﻢ ﻧﺘﻤﻢ اﳊﻞ ﺑﻨﻔﺲ ﻃﺮﻳﻘﺔ اﻟﺴﺆال اﻷول‪.‬‬


‫ّ‬
‫اﻟﻄﺮﻳﻘﺔ اﻟﺜﺎﻧﻴﺔ ‪ :‬ﻧﻼﺣﻆ ّ‬
‫أن ‪ b = ıa‬ﻣﻨﻪ ‪:‬‬

‫𝟮𝟯𝟭‬

‫‪http ://tinyurl.com/Malki1718‬‬ ‫‪0‬‬


‫‪ .VI‬ﲤﺎرﻳﻦ ﺗﻄﺒﻴﻘﻴﺔ‬
‫‪.‬‬
‫إذا ﻛﺎن 𝜋 = 𝜑 ّ‬
‫ﻓﺈن ‪. b = 0‬‬ ‫•‬

‫إذا ﻛﺎن [𝜋 ‪ّ 𝜑 ∈ ]−𝜋,‬‬


‫ﻓﺈن ‪:‬‬ ‫•‬

‫‪Ŕž Ŧ Ľ‬‬
‫𝜑‬
‫‪||b|| = ||ıa|| = |ı| ⋅ ||a|| = 1 × |a| = 2 cos‬‬
‫‪2‬‬

‫‪œǃ‬‬
‫𝜋‬ ‫𝜑‪𝜋−‬‬
‫= )‪arg (b) = arg (ıa) = arg (ı) + arg (a) = − arg (a‬‬ ‫)𝜋‪(mod 2‬‬ ‫و‬
‫‪2‬‬ ‫‪2‬‬
‫‪ •3‬اﻟﻄﺮﻳﻘﺔ اﻷوﱃ ﻟﺪﻳﻨﺎ ‪:‬‬
‫𝜋‬ ‫𝜋‬

‫‪Ŀ‬‬
‫‪c = cos 𝜑 + ı (1 + sin 𝜑) = sin‬‬ ‫‪− 𝜑 + ı 1 + cos‬‬ ‫𝜑‪−‬‬ ‫)𝜓 ‪= sin 𝜓 + ı (1 + cos‬‬
‫‪2‬‬ ‫‪2‬‬
‫𝜋‬
‫= 𝜓 ‪ .‬ﻧُﻄ ﱢﺒﻖ ﻧﺘﻴﺠﺔ اﻟﺴﺆال اﻟﺜﺎﲏ ﻓﻴﻜﻮن ‪:‬‬ ‫ﻣﻊ 𝜑 ‪−‬‬
‫‪2‬‬
‫𝜋‬
‫إذا ﻛﺎن 𝜋 = 𝜓 أي إذا ﻛﺎن ‪ّ 𝜑 = −‬‬
‫ﻓﺈن ‪. c = 0‬‬ ‫•‬
‫‪2‬‬
‫𝜑 𝜋‬ ‫𝜓‬
‫‪ c = 2 cos‬و ﻫﻨﺎ ﻧُﻤ ﱢﻴﺰ ﺣﺎﻟﺘﲔ ‪:‬‬
‫‪−‬‬ ‫ﻓﺈن ‪ c = 2 cos e − /‬أي ‪e / + /‬‬ ‫• و إﻻّ‪ّ ،‬‬
‫‪4 2‬‬ ‫‪2‬‬
‫𝜋‬ ‫𝜋‬ ‫𝜋‪3‬‬ ‫𝜋 𝜑 𝜋 𝜋‬ ‫𝜑 𝜋‬
‫< 𝜑 < ‪ −‬أي 𝜋 < 𝜑 < ‪−‬‬ ‫‪ cos −‬أي إذا ﻛﺎن < ‪ − < −‬أي‬ ‫➛ إذا ﻛﺎن ‪> 0‬‬
‫‪2‬‬ ‫‪2‬‬ ‫‪2‬‬ ‫‪2‬‬ ‫‪4 2‬‬ ‫‪2‬‬ ‫‪4 2‬‬
‫)ﻷن 𝜋 < 𝜑 < 𝜋‪ (−‬ﻓﺤﺴﺐ اﳌﱪﻫﻨﺔ ‪ 8‬ﺻﻔﺤﺔ ‪: 20‬‬ ‫ّ‬
‫𝜑 𝜋‬ ‫𝜑 𝜋‬
‫)𝜋‪. arg (c) = − (mod 2‬‬ ‫و‬ ‫‪|c| = 2 cos‬‬ ‫‪−‬‬
‫‪4 2‬‬ ‫‪4 2‬‬
‫➛ إذا ﻛﺎن ‪ cos 𝜋 − 𝜑 < 0‬أي إذا ﻛﺎن 𝜋 ‪ّ −𝜋 < 𝜑 < −‬‬
‫ﻓﺈن ‪:‬‬
‫‪2‬‬ ‫‪4 2‬‬
‫𝜑 𝜋‬ ‫𝜑 𝜋‪5‬‬ ‫𝜑 𝜋‬
‫‪ |c| = −2 cos −‬و )𝜋‪. arg (c) = 𝜋 + − (mod 2𝜋) = − (mod 2‬‬
‫‪4 2‬‬ ‫‪4‬‬ ‫‪2‬‬ ‫‪4 2‬‬
‫𝜋‬
‫اﻟﻄﺮﻳﻘﺔ اﻟﺜﺎﻧﻴﺔ ‪ :‬ﻧﻀﻊ 𝜑 ‪ 𝜓 = +‬ﻓﻴﻜﻮن ‪:‬‬
‫‪2‬‬
‫𝜋‬ ‫𝜋‬
‫‪c = cos 𝜑 + ı (1 + sin 𝜑) = cos 𝜓 −‬‬ ‫‪+ ı 1 + sin 𝜓 −‬‬ ‫)𝜓 ‪= sin 𝜓 + ı (1 − cos‬‬
‫‪2‬‬ ‫‪2‬‬
‫𝜓‬ ‫𝜓‬ ‫𝜓‬ ‫𝜓‬ ‫𝜓‬ ‫𝜓‬ ‫𝜓‬
‫‪= 2 sin cos + 2ı sin‬‬ ‫‪= 2 sin‬‬ ‫‪cos + ı sin‬‬ ‫‪= 2 sin e /‬‬
‫‪2‬‬ ‫‪2‬‬ ‫‪2‬‬ ‫‪2‬‬ ‫‪2‬‬ ‫‪2‬‬ ‫‪2‬‬
‫‪ņőƱŀ Ţ‬‬
‫𝜋‬
‫إذا ﻛﺎن ‪ 𝜓 = 0‬أي إذا ﻛﺎن ‪ّ 𝜑 = −‬‬
‫ﻓﺈن ‪. c = 0‬‬ ‫•‬
‫‪2‬‬
‫𝜑 𝜋‬ ‫𝜓‬
‫ﻓﺈن ‪ c = 2 sin e /‬أي ‪e / + /‬‬
‫‪ c = 2 sin‬و ﻫﻨﺎ ﻧُﻤ ﱢﻴﺰ ﺣﺎﻟﺘﲔ ‪:‬‬
‫‪+‬‬ ‫و إﻻّ‪ّ ،‬‬ ‫•‬
‫‪4 2‬‬ ‫‪2‬‬
‫‪VI‬‬

‫𝜋‬ ‫𝜋‪3‬‬ ‫𝜋‬ ‫𝜋 𝜑 𝜋 𝜋‬ ‫𝜑 𝜋‬


‫‪ −‬أي < 𝜑 < 𝜋‪−‬‬ ‫‪ sin‬أي إذا ﻛﺎن < ‪ − < +‬أي < 𝜑 <‬ ‫‪+‬‬ ‫➛ إذا ﻛﺎن ‪> 0‬‬
‫‪2‬‬ ‫‪2‬‬ ‫‪2‬‬ ‫‪2‬‬ ‫‪4 2‬‬ ‫‪2‬‬ ‫‪4 2‬‬
‫)ﻷن 𝜋 < 𝜑 < 𝜋‪ (−‬ﻓﺤﺴﺐ اﳌﱪﻫﻨﺔ ‪ 8‬ﺻﻔﺤﺔ ‪ّ 20‬‬
‫ﻓﺈن ‪:‬‬ ‫ّ‬
‫𝜑 𝜋‬ ‫𝜑 𝜋‬
‫)𝜋‪. arg (c) = + (mod 2‬‬ ‫و‬ ‫‪|c| = 2 sin‬‬ ‫‪+‬‬
‫‪4 2‬‬ ‫‪4 2‬‬
‫➛ إذا ﻛﺎن ‪ sin 𝜋 + 𝜑 < 0‬أي إذا ﻛﺎن 𝜋 < 𝜑 < 𝜋 ّ‬
‫ﻓﺈن ‪:‬‬
‫‪ƕ‬‬

‫‪2‬‬ ‫‪4 2‬‬


‫𝜑 𝜋‬ ‫𝜑 𝜋‪5‬‬ ‫𝜑 𝜋‬
‫‪ |c| = −2 sin +‬و )𝜋‪. arg (c) = 𝜋 + + (mod 2𝜋) = + (mod 2‬‬
‫‪4 2‬‬ ‫‪4‬‬ ‫‪2‬‬ ‫‪4 2‬‬
‫■‬
‫‪ũŏ‬‬

‫أوﺟﺪ اﻟﻄﻮﻳﻠﺔ و ﻋﻤﺪة ﻟﻸﻋﺪاد اﳌﺮﻛﺒﺔ اﻵﺗﻴﺔ ) ﻛﻠﲈ ﻛﺎﻧﺖ ﻣﻌﺮﻓﺔ ( ‪:‬‬
‫‬ ‫‪.‬‬
‫𝟔𝟔‬ ‫‪.‬‬

‫𝜃 ‪1 − cos 𝜃 + ı sin‬‬ ‫)𝜃 ‪(1 + ı tan‬‬


‫=‪b‬‬ ‫•‬ ‫=‪a‬‬ ‫•‬
‫𝜃 ‪1 + cos 𝜃 − ı sin‬‬ ‫𝜃 ‪1 + tan‬‬

‫𝟯𝟯𝟭‬

‫‪http ://tinyurl.com/Malki1718‬‬ ‫‪0‬‬


‫ اﻟﺸﲁ اﳌﺜﻠﱻ ﻟﻌﺪد ﻣﺮﻛﺐ‬.2.VI
.
.ّ‫ﺍﳊﻞ‬
1
: ‫ ﻣﻨﻪ‬1 + tan 𝜃 = ّ ‫• ﻧﻌﻠﻢ‬1
‫أن‬
cos 𝜃

Ŕž Ŧ Ľ
(1 + ı tan 𝜃) sin 𝜃
a= = cos 𝜃 1 + ı = (cos 𝜃 + ı sin 𝜃) = e
1 + tan 𝜃 cos 𝜃

œǃ
. arg (a) = 2𝜃 (mod 2𝜋) ‫و‬ |a| = 1 : ‫و ﺑﺎﻟﺘﺎﱄ‬
: ‫• ﻟﺪﻳﻨﺎ‬2
1 − cos 𝜃 + ı sin 𝜃 2 sin + 2ı sin cos 2 sin sin + ı cos

Ŀ
b= = = ⋅
1 + cos 𝜃 − ı sin 𝜃 2 cos − 2ı sin cos 2 cos cos − ı sin
((( (
𝜃 gray
ı −ı(sin((+(cos 𝜃
= tan ⋅   = ı tan
cos−ı 
2 gray sin 2

𝜃
: �� ‫ ؛ و إﻻّ ﻓﺤﺴﺐ اﳌﱪﻫﻨﺔ �� ﺻﻔﺤﺔ‬b = 0 ‫ﻓﺈن‬
ّ tan = 0 ‫ إذا ﻛﺎن‬،‫إذن‬
2
⎧ 𝜋 𝜃
⎪ (mod 2𝜋) ‫ ؛‬tan > 0 ‫إذا ﻛﺎن‬ | 𝜃|
. arg (b) = 2 2 ‫و‬ ||b|| = |tan |
⎪ − 𝜋 (mod 2𝜋)
⎨ 𝜃
‫ ؛‬tan < 0 ‫إذا ﻛﺎن‬ || 2 ||
⎩ 2 2

1+e 1 + cos 𝜃 − ı sin 𝜃 # 𝟔𝟕. .


. ‫و‬ ‫أﻛﺘﺐ ﻋﲆ اﻟﺸﻜﻞ اﳌﺜﻠﺜﻲ اﻷﻋﺪاد‬
1−e 1 − cos 𝜃 − ı sin 𝜃

ّ ‫ ﺑﲈ‬.ّ‫ﺍﳊﻞ‬
ّ 1 − cos 𝜃 − ı sin 𝜃 = 1 − e ‫أن‬
: ‫ﻓﺈن‬
1 − cos 𝜃 − ı sin 𝜃 = 0 ⟺ 1 − e =0 ⟺ e = 1 ⟺ 𝜃 = 0 (mod 2𝜋)
1+e 1 + cos 𝜃 − ı sin 𝜃
. 𝜃 ≠ 0 (mod 2𝜋) ‫ﻌﺮﻓﺎن إذا و ﻓﻘﻂ إذا ﻛﺎن‬
ّ ‫ُﻣ‬ ‫و‬ ‫إذن اﻟﻌﺪدان‬
1−e 1 − cos 𝜃 − ı sin 𝜃
ņőƱŀ Ţ
. 𝜃 ≠ 0 (mod 2𝜋) ‫أن‬ ّ ‫ﻧﻔﺮض إذن‬
: ‫• ﻟﺪﻳﻨﺎ‬1
1 + cos 𝜃 − ı sin 𝜃 1 + e− e− / e / + e− /

VI
= =
1 − cos 𝜃 − ı sin 𝜃 1−e e / e− / −e /

2 cos 𝜃 𝜃
= e− = ıe− cotan =e − cotan
−2ı sin 2 2

: ‫ﻣﻨﻪ ﻧُﻤ ﱢﻴﺰ ﺛﻼﺛﺔ ﺣﺎﻻت‬


ƕ

𝜃 𝜋 𝜃
(𝜃 ≠ 0 (mod 2𝜋) ‫( 𝜋 = 𝜃 )و ﺑﺎﻟﺘﺎﱄ‬mod 2𝜋) ‫= أي‬ (mod 𝜋) ‫ أي إذا ﻛﺎن‬cotan = 0 ‫إذا ﻛﺎن‬ •
2 2 2
ّ
: ‫ﻓﺈن‬
1 + cos 𝜃 − ı sin 𝜃
ũŏ

=0×e − =0
1 − cos 𝜃 − ı sin 𝜃
𝜃 𝜋 𝜃
ّ 𝜃∈
‫ﻓﺈن اﻟﺸﻜﻞ‬ ]2k𝜋, 𝜋 + 2k𝜋[ ‫∈ أي‬ k𝜋, + k𝜋 ‫ أي إذا ﻛﺎن‬cotan > 0 ‫إذا ﻛﺎن‬ •

2 ℤ
2 2
1 + cos 𝜃 − ı sin 𝜃
: ‫( ﻫﻮ‬20 ‫ ﺻﻔﺤﺔ‬8 ‫)ﻣﱪﻫﻨﺔ‬ ‫اﳌﺜﻠﺜﻲ ﻟﻠﻌﺪد‬
1 − cos 𝜃 − ı sin 𝜃
1 + cos 𝜃 − ı sin 𝜃 𝜃 𝜋
= cotan , − 𝜃
1 − cos 𝜃 − ı sin 𝜃 2 2

𝟭𝟯𝟰

http ://tinyurl.com/Malki1718 0
‫ ﲤﺎرﻳﻦ ﺗﻄﺒﻴﻘﻴﺔ‬.VI
.
𝜃 𝜋 𝜃
𝜃∈ ]𝜋 + 2k𝜋, 2 (k + 1) 𝜋[ ‫أي‬ ∈ + k𝜋, (k + 1) 𝜋 ‫ أي إذا ﻛﺎن‬cotan < 0 ‫إذا ﻛﺎن‬ •

2

2 2

Ŕž Ŧ Ľ
1 + cos 𝜃 − ı sin 𝜃
: ‫( ﻫﻮ‬20 ‫ ﺻﻔﺤﺔ‬8 ‫)ﻣﱪﻫﻨﺔ‬ ّ
‫ﻓﺈن اﻟﺸﻜﻞ اﳌﺜﻠﺜﻲ ﻟﻠﻌﺪد‬
1 − cos 𝜃 − ı sin 𝜃
1 + cos 𝜃 − ı sin 𝜃 𝜃 𝜋 𝜃 3𝜋

œǃ
= − cotan , 𝜋 + − 𝜃 = − cotan , −𝜃
1 − cos 𝜃 − ı sin 𝜃 2 2 2 2

: ‫• ﻟﺪﻳﻨﺎ‬2
/ e− / /

Ŀ
1+e e +e 2 cos 𝜃 𝜃
== = = ı cotan =e cotan
1−e e / e− / −e / −2ı sin 2 2

: ‫ﻣﻨﻪ ﻧُﻤ ﱢﻴﺰ ﺛﻼﺛﺔ ﺣﺎﻻت‬


𝜃 𝜋 𝜃
(𝜃 ≠ 0 (mod 2𝜋) ‫( 𝜋 = 𝜃 )و ﺑﺎﻟﺘﺎﱄ‬mod 2𝜋) ‫= أي‬ (mod 𝜋) ‫ أي إذا ﻛﺎن‬cotan = 0 ‫إذا ﻛﺎن‬ •
2 2 2
ّ
: ‫ﻓﺈن‬
1+e
=0×ı=0
1−e
𝜃 𝜋 𝜃
ّ 𝜃∈
‫ﻓﺈن اﻟﺸﻜﻞ‬ ]2k𝜋, 𝜋 + 2k𝜋[ ‫أي‬ ∈ k𝜋, + k𝜋 ‫ أي إذا ﻛﺎن‬cotan > 0 ‫إذا ﻛﺎن‬ •

2 ℤ
2 2
1+e
: ‫( ﻫﻮ‬20 ‫ ﺻﻔﺤﺔ‬8 ‫)ﻣﱪﻫﻨﺔ‬ ‫اﳌﺜﻠﺜﻲ ﻟﻠﻌﺪد‬
1−e
1+e 𝜃 𝜋
= cotan ,
1−e 2 2
𝜃 𝜋 𝜃
𝜃∈ ]𝜋 + 2k𝜋, 2 (k + 1) 𝜋[ ‫أي‬ ∈ + k𝜋, (k + 1) 𝜋 ‫ أي إذا ﻛﺎن‬cotan < 0 ‫إذا ﻛﺎن‬ •

2 ℤ
2 2
1+e
: ‫( ﻫﻮ‬20 ‫ ﺻﻔﺤﺔ‬8 ‫)ﻣﱪﻫﻨﺔ‬ ّ
‫ﻓﺈن اﻟﺸﻜﻞ اﳌﺜﻠﺜﻲ ﻟﻠﻌﺪد‬
1−e
1+e 𝜃 𝜋 𝜃 3𝜋 𝜃 𝜋
ņőƱŀ Ţ
= − cotan , 𝜋 + = − cotan , = − cotan , −
1−e 2 2 2 2 2 2

1 + cos 𝜃 − ı sin 𝜃 1+e


‫ﻣﻦ اﻟﺸﻜﻞ اﳌﺜﻠﺜﻲ ﻟﻠﻌﺪد‬ ‫ﻛﺎن ﺑﺈﻣﻜﺎﻧﻨﺎ أن ﻧﺴﺘﻨﺘﺞ اﻟﺸﻜﻞ اﳌﺜﻠﺜﻲ ﻟﻠﻌﺪد‬ ⧏ : 22 ‫ﻣﻼﺣﻈﺔ‬
1 − cos 𝜃 − ı sin 𝜃 1−e
VI

ّ ‫و ذﻟﻚ‬
: ‫ﻷن‬
1+e e e− + 1 1 + cos 𝜃 − ı sin 𝜃
= =e ⋅
1−e 1−e 1 − cos 𝜃 − ı sin 𝜃
|1 + e | | 1 + cos 𝜃 − ı sin 𝜃 | | 1 + cos 𝜃 − ı sin 𝜃 |
| | = ||e || ⋅ | |=| | : ‫و ﺑﺎﻟﺘﺎﱄ‬
ƕ

|| 1 − e || || 1 − cos 𝜃 − ı sin 𝜃 || || 1 − cos 𝜃 − ı sin 𝜃 ||


1+e 1 + cos 𝜃 − ı sin 𝜃
arg = arg e + arg (mod 2𝜋) ‫و‬
1−e 1 − cos 𝜃 − ı sin 𝜃
1 + cos 𝜃 − ı sin 𝜃
ũŏ

= 𝜃 + arg (mod 2𝜋)


1 − cos 𝜃 − ı sin 𝜃

𝟭𝟯𝟱

http ://tinyurl.com/Malki1718 0
‫‪ .2.VI‬اﻟﺸﲁ اﳌﺜﻠﱻ ﻟﻌﺪد ﻣﺮﻛﺐ‬
‫‪.‬‬

‫‬ ‫‪𝟔𝟖.‬‬ ‫‪.‬‬

‫‪Ŕž Ŧ Ľ‬‬
‫𝛼 ﻋﺪد ﺣﻘﻴﻘﻲ ﺣﻴﺚ ]𝜋 ‪ 𝛼 ∈ ]−𝜋,‬و ‪ z ، z‬ﻋﺪدان ﻣﺮﻛﺒﺎن ﺣﻴﺚ ‪:‬‬
‫𝛼 ‪z = cos 𝛼 + ı sin‬‬ ‫‪,‬‬ ‫‪z =1+z+z‬‬

‫‪œǃ‬‬
‫ﻋﲔ‪ ،‬ﺣﺴﺐ ِﻗ َﻴﻢ اﻟﻌﺪد 𝛼 ‪ ،‬اﻟﺸﻜﻞ اﳌﺜﻠﺜﻲ ﻟﻠﻌﺪد اﳌﺮﻛﺐ ‪. z‬‬
‫ّ‬

‫ﺍﳊﻞّ‪ .‬ﻟﺪﻳﻨﺎ ‪:‬‬

‫‪Ŀ‬‬
‫)𝛼 ‪z = 1 + z + z = 1 + cos 𝛼 + ı sin 𝛼 + (cos 𝛼 + ı sin‬‬
‫)𝛼‪= 1 + cos 𝛼 + ı sin 𝛼 + cos 𝛼 − sin 𝛼 + 2ı cos 𝛼 sin 𝛼 = 1 + cos 𝛼 + cos 2𝛼 + ı (sin 𝛼 + sin 2‬‬
‫𝛼 ‪= 1 + cos 𝛼 + 2 cos 𝛼 − 1 + ı (sin 𝛼 + 2 cos 𝛼 sin 𝛼) = (1 + 2 cos 𝛼) cos 𝛼 + ı (1 + 2 cos 𝛼) sin‬‬
‫)𝛼 ‪= (1 + 2 cos 𝛼) (cos 𝛼 + ı sin‬‬

‫و ﺑﺎﻟﺘﺎﱄ ‪:‬‬

‫ﻌﺮف(؛‬ ‫إذا ﻛﺎن ‪ 1 + 2 cos 𝛼 = 0‬أي إذا ﻛﺎن ‪ cos 𝛼 = − 1‬أي إذا ﻛﺎن 𝜋‪ّ 𝛼 = ± 2‬‬
‫ﻓﺈن ‪) z = 0‬اﻟﺸﻜﻞ اﳌﺜﻠﺜﻲ ﻏﲑ ُﻣ ّ‬ ‫•‬
‫‪3‬‬ ‫‪2‬‬
‫𝜋‪2‬‬ ‫𝜋‪2‬‬ ‫‪1‬‬
‫‪ّ −‬‬
‫ﻓﺈن ]𝛼 ‪ z = [1 + 2 cos 𝛼,‬؛‬ ‫<𝛼<‬ ‫إذا ﻛﺎن ‪ 1 + 2 cos 𝛼 > 0‬أي إذا ﻛﺎن ‪ cos 𝛼 > −‬أي إذا ﻛﺎن‬ ‫•‬
‫‪3‬‬ ‫‪3‬‬ ‫‪2‬‬
‫𝜋‪2‬‬ ‫𝜋‪2‬‬ ‫‪1‬‬
‫‪ّ 𝛼 ∈ −𝜋, −‬‬
‫ﻓﺈن‪:‬‬ ‫⋃‬ ‫𝜋‪,‬‬ ‫و إذا ﻛﺎن ‪ 1 + 2 cos 𝛼 < 0‬أي إذا ﻛﺎن ‪ cos 𝛼 < −‬أي إذا ﻛﺎن‬ ‫•‬
‫‪3‬‬ ‫‪3‬‬ ‫‪2‬‬
‫]𝛼 ‪. z = [−1 − 2 cos 𝛼, 𝜋 +‬‬
‫■‬

‫‬ ‫‪𝟔𝟗.‬‬ ‫‪.‬‬


‫‪ņőƱŀ Ţ‬‬
‫𝜋‬ ‫𝜋‬
‫أﺣﺴﺐ ﺑﻄﺮﻳﻘﺘﲔ ﳐﺘﻠﻔﺘﲔ اﳉﺬرﻳﻦ اﻟﱰﺑﻴﻌﻴﲔ ﻟﻠﻌﺪد ‪ . 1 + ı‬إﺳﺘﻨﺘﺞ ﻗﻴﻤﺔ ﻛﻼ ﻣﻦ ‪ cos‬و ‪. sin‬‬
‫‪8‬‬ ‫‪8‬‬

‫ﺍﳊﻞّ‪ .‬ﺑﺪاﻳ ًﺔ‪ ،‬ﻧﺴﺘﻌﻤﻞ اﻟﺸﻜﻞ اﻷﳼ ‪:‬‬

‫‪VI‬‬
‫‪𝜌 e‬‬ ‫ﻓﺈن ‪ u = 1 + ı‬أي ‪= √2e /‬‬ ‫ﻟﺪﻳﻨﺎ ‪ 1 + ı = √2e / :‬و إذا ﻛﺎن ‪ u = 𝜌e‬ﺟﺬر ًا ﺗﺮﺑﻴﻌﻴ ًﺎ ﻟﻠﻌﺪد ‪ّ 1 + ı‬‬
‫𝜋‬ ‫𝜋‬
‫ﻣﻨﻪ ‪ 𝜌 = √2‬و )𝜋‪ 2𝜃 = (mod 2‬أي ‪ 𝜌 = √2‬و )𝜋 ‪ 𝜃 = (mod‬إذن اﳉﺬرﻳﻦ اﻟﱰﺑﻴﻌﻴﲔ ﻟﻠﻌﺪد ‪ 1 + ı‬ﳘﺎ‬
‫‪8‬‬ ‫‪4‬‬
‫‪ u = √2e /‬و ‪. √2e + = − √2e / = −u‬‬
‫ﻓﺈن ‪ (x + ıy) = 1 + ı‬أي‬ ‫ﻣﻦ ﺟﻬﺔ أﺧﺮى )ﻧﺴﺘﻌﻤﻞ اﻟﺸﻜﻞ اﳉﱪي(‪ ،‬إذا ﻛﺎن ‪ x + ıy‬ﺟﺬر ًا ﺗﺮﺑﻴﻌﻴ ًﺎ ﻟﻠﻌﺪد ‪ّ 1 + ı‬‬
‫‪ƕ‬‬

‫⎧‬ ‫‪x −y =1‬‬


‫⎪‬
‫‪.‬‬ ‫‪2xy = 1‬‬ ‫‪ x − y + 2ıxy = 1 + ı‬ﻣﻨﻪ ‪:‬‬
‫⎨‬
‫⎪‬
‫‪⎩ x + y = √2‬‬
‫‪ũŏ‬‬

‫‪1 + √2‬‬
‫‪.x = ±‬‬ ‫ﺑﺠﻤﻊ اﳌﻌﺎدﻟﺘﲔ اﻷوﱃ و اﻟﺜﺎﻟﺜﺔ ﻳﻨﺘﺞ ‪ 2x = 1 + √2‬ﻣﻨﻪ‬
‫‪2‬‬
‫‪√2 − 1‬‬ ‫‪2−1‬‬
‫‪.y = ±‬‬ ‫√ = ‪ y = √2 − x‬ﻣﻨﻪ‬ ‫ﻣﻦ اﳌﻌﺎدﻟﺔ اﻟﺜﺎﻟﺜﺔ ﻧﺴﺘﻨﺘﺞ ّ‬
‫أن‬
‫‪2‬‬ ‫‪2‬‬
‫‪√2 + 1‬‬ ‫‪√2 − 1‬‬ ‫ﻣﻦ اﳌﻌﺎدﻟﺔ اﻟﺜﺎﻧﻴﺔ ﻧﺠﺪ ّ‬
‫و‬ ‫‪+ı‬‬ ‫أن ‪ x‬و ‪ y‬ﳍﲈ ﻧﻔﺲ اﻹﺷﺎرة إذن ﻓﺎﳉﺬرﻳﻦ اﻟﱰﺑﻴﻌﻴﲔ ﻟﻠﻌﺪد ‪ 1 + ı‬ﳘﺎ‬
‫‪2‬‬ ‫‪2‬‬
‫‪2+1‬‬ ‫‪√2 − 1‬‬
‫√ ‪.−‬‬ ‫‪−ı‬‬
‫‪2‬‬ ‫‪2‬‬

‫𝟲𝟯𝟭‬

‫‪http ://tinyurl.com/Malki1718‬‬ ‫‪0‬‬


‫ ﲤﺎرﻳﻦ ﺗﻄﺒﻴﻘﻴﺔ‬.VI
.
/ 𝜋
ّ ‫ ﻧﺴﺘﺨﻠﺺ ﳑّﺎ ﺳﺒﻖ‬، Re e
: ‫أن‬ = cos ّ ‫ﺑﲈ‬
> 0 ‫أن‬
8

Ŕž Ŧ Ľ
/ √2 + 1 √2 − 1
√2e = +ı
2 2

œǃ
/ 1 √2 + 1 ı √2 − 1
e = + : ‫ﻣﻨﻪ‬
√2 2 √2 2

/ √2 + 1 √2 − 1 2 + √2 2 − √2
e = +ı = +ı : ‫أي‬

Ŀ
2√2 2√2 2 2

𝜋 2 − √2 𝜋 2 + √2
■ . sin = ‫و‬ cos = : ‫ﰲ اﻷﺧﲑ‬
8 2 8 2

‫ و‬cos
𝜋
‫ﺛﻢ اﺳﺘﻨﺘﺞ ﻗﻴﻤﺔ ﻛﻞ ﻣﻦ‬
ّ ‫ ﻋﲆ ﺷﻜﻠ ْﻴﻪ اﳉﱪي و اﳌﺜﻠﺜﻲ‬z = (1 + ı) √3 + ı ‫أﻛﺘﺐ اﻟﻌﺪد‬
 .
𝟕𝟎 .
12
𝜋
. sin
12

: ‫اﻟﺸﻜﻞ اﳉﱪي‬ • .ّ‫ﺍﳊﻞ‬

z = (1 + ı) √3 + ı = √3 + ı + ı√3 + ı = √3 − 1 + ı √3 + 1

: ‫اﻟﺸﻜﻞ اﳌﺜﻠﺜﻲ‬ •

√2 √2 𝜋 𝜋 /
1 + ı = √2 +ı = √2 cos + ı sin = √2e
2 2 4 4

√3 1 𝜋 𝜋 /
√3 + ı = 2 + ı = 2 cos + ı sin = 2e ‫و‬
2 2 6 6
ņőƱŀ Ţ
/ / / + / / 5𝜋 5𝜋
z = √2e × 2e = 2√2e = 2√2e = 2√2 cos +ı : ‫ﻣﻨﻪ‬
12 12
: ‫√ ﻣﻨﻪ‬3 − 1 + ı √3 + 1 = 2√2e / ‫إذن‬
VI

5𝜋 5𝜋 / √3 − 1 √3 + 1
cos +ı =e = +ı
12 12 2√2 2√2
: ‫و ﺑﺎﻟﺘﺎﱄ‬

5𝜋 √3 − 1 √2 √3 − 1 √6 − √2
cos = = =
ƕ

12 2 √2 4
2 √2

5𝜋 √3 + 1 √2 √3 + 1 √6 + √2
sin = = = ‫و‬
12 2 √2 4
ũŏ

2 √2

5𝜋 𝜋 5𝜋 𝜋 5𝜋 𝜋 5𝜋 𝜋
: ‫ ﻣﻨﻪ‬sin = cos − = cos ‫ و‬cos = sin − = sin ‫ﻟﻜﻦ‬
12 2 12 12 12 2 12 12

𝜋 5𝜋 √6 + √2 𝜋 5𝜋 √6 − √2
cos = sin = ‫و‬ sin = cos =
12 12 4 12 12 4

𝟭𝟯𝟳

http ://tinyurl.com/Malki1718 0
‫‪ .2.VI‬اﻟﺸﲁ اﳌﺜﻠﱻ ﻟﻌﺪد ﻣﺮﻛﺐ‬
‫‪.‬‬

‫‬ ‫‪𝟕𝟏.‬‬ ‫‪.‬‬

‫‪Ŕž Ŧ Ľ‬‬
‫= ‪.z‬‬ ‫‪e−‬‬ ‫‪/‬‬ ‫‪z = e‬و‬ ‫‪/‬‬ ‫ﻧﻌﺘﱪ اﻟﻌﺪدﻳﻦ اﳌﺮﻛﺒﲔ‬
‫‪ •1‬أﻛﺘﺐ ‪ z‬و ‪ z‬ﻋﲆ اﻟﺸﻜﻞ اﳉﱪي‪.‬‬

‫‪œǃ‬‬
‫‪ •2‬أﻛﺘﺐ اﻟﻌﺪد ‪ z z‬ﻋﲆ اﻟﺸﻜﻞ اﳉﱪي‪ ،‬اﻷﳼ و اﳌﺜﻠﺜﻲ‪.‬‬
‫𝜋‬ ‫𝜋‬
‫‪. sin‬‬ ‫‪ cos‬و‬ ‫‪ •3‬إﺳﺘﻨﺘﺞ ﻗﻴﻤﺔ ﻛﻞ ﻣﻦ‬

‫‪Ŀ‬‬
‫‪12‬‬ ‫‪12‬‬

‫•‬ ‫‪1‬‬ ‫ﺍﳊﻞّ‪.‬‬


‫𝜋‬ ‫‪𝜋 1‬‬ ‫‪√3‬‬ ‫𝜋‬ ‫𝜋‬ ‫‪√2‬‬ ‫‪√2‬‬
‫‪z = cos‬‬ ‫‪+ ı sin = + ı‬‬ ‫و‬ ‫‪z = cos −‬‬ ‫‪+ ı sin −‬‬ ‫=‬ ‫‪−ı‬‬
‫‪3‬‬ ‫‪3 2‬‬ ‫‪2‬‬ ‫‪4‬‬ ‫‪4‬‬ ‫‪2‬‬ ‫‪2‬‬

‫‪1‬‬ ‫‪√3‬‬ ‫‪√2‬‬ ‫‪√2‬‬ ‫‪√6 + √2‬‬ ‫‪√6 − √2‬‬


‫= ‪z z‬‬ ‫‪+ı‬‬ ‫‪−ı‬‬ ‫=‬ ‫‪+ı‬‬ ‫‪ •2‬اﻟﺸﻜﻞ اﳉﱪي ﻟﻠﻌﺪد ‪: z z‬‬
‫‪2‬‬ ‫‪2‬‬ ‫‪2‬‬ ‫‪2‬‬ ‫‪4‬‬ ‫‪4‬‬
‫‪z z =e‬‬ ‫‪/‬‬ ‫‪⋅ e−‬‬ ‫‪/‬‬ ‫‪=e‬‬ ‫‪/‬‬ ‫اﻟﺸﻜﻞ اﻷﳼ ﻟﻠﻌﺪد ‪: z z‬‬
‫𝜋‬ ‫𝜋‬
‫‪z z = cos‬‬ ‫‪+ ı sin‬‬ ‫اﻟﺸﻜﻞ اﳌﺜﻠﺜﻲ ﻟﻠﻌﺪد ‪: z z‬‬
‫‪12‬‬ ‫‪12‬‬
‫‪ •3‬ﺑﺎﳌﻄﺎﺑﻘﺔ ﺑﲔ اﻟﺸﻜﻞ اﳉﱪي و اﻟﺸﻜﻞ اﳌﺜﻠﺜﻲ ﻟﻠﻌﺪد ‪ ،z z‬ﻧﺴﺘﻨﺘﺞ ّ‬
‫أن ‪:‬‬
‫𝜋‬ ‫‪√6 + √2‬‬ ‫𝜋‬ ‫‪√6 − √2‬‬
‫‪cos‬‬ ‫=‬ ‫و‬ ‫‪sin‬‬ ‫=‬
‫‪12‬‬ ‫‪4‬‬ ‫‪12‬‬ ‫‪4‬‬
‫■‬

‫‪𝟕𝟐.‬‬ ‫‪.‬‬
‫‪ņőƱŀ Ţ‬‬
‫‪ّ •1‬‬
‫ﻋﲔ اﻟﻄﻮﻳﻠﺔ و ﻋﻤﺪة ﻟﻠﻌﺪد ‪ z‬ﺣﻴﺚ ‪. z = 3 + 3ı‬‬
‫‪ •2‬ﻧﻌﺘﱪ اﻟﻌﺪد اﳌﺮﻛﺐ ‪ z‬اﳌﻌﺮف ﻛﲈ ﻳﲇ ‪:‬‬

‫‪VI‬‬
‫𝜋‪17‬‬ ‫𝜋‪17‬‬
‫‪z ⋅ z = 6√2 cos‬‬ ‫‪+ ı sin‬‬
‫‪12‬‬ ‫‪12‬‬
‫𝜋‪17‬‬ ‫𝜋‪17‬‬
‫‪. sin‬‬ ‫‪ cos‬و‬ ‫ﺛﻢ اﺳﺘﻨﺘﺞ ﻗﻴﻤﺘﻲ‬
‫ﻋﲔ اﻟﺸﻜﻞ اﳌﺜﻠﺜﻲ ﺛﻢ اﻟﺸﻜﻞ اﳉﱪي ﻟﻠﻌﺪد ‪ّ z‬‬
‫ّ‬
‫‪12‬‬ ‫‪12‬‬
‫‪ƕ‬‬

‫‪ •1‬ﻟﺪﻳﻨﺎ ‪:‬‬ ‫ﺍﳊﻞّ‪.‬‬

‫‪√2‬‬ ‫‪√2‬‬ ‫𝜋‬ ‫𝜋‬


‫‪ũŏ‬‬

‫‪z = 3 + 3ı = 3√2‬‬ ‫‪+ı‬‬ ‫‪= 3√2 cos‬‬ ‫‪+ ı sin‬‬


‫‪2‬‬ ‫‪2‬‬ ‫‪4‬‬ ‫‪4‬‬

‫𝜋‬
‫= ) ‪. arg (z‬‬ ‫)𝜋‪(mod 2‬‬ ‫و‬ ‫‪||z || = 3√2‬‬ ‫و ﺑﺎﻟﺘﺎﱄ ‪:‬‬
‫‪4‬‬

‫𝟴𝟯𝟭‬

‫‪http ://tinyurl.com/Malki1718‬‬ ‫‪0‬‬


‫‪ .VI‬ﲤﺎرﻳﻦ ﺗﻄﺒﻴﻘﻴﺔ‬
‫‪.‬‬
‫𝜋‪17‬‬ ‫𝜋‪17‬‬
‫‪ . z = 6√2 cos‬ﻟﺪﻳﻨﺎ ‪ z ⋅ z = z‬ﻣﻨﻪ ‪:‬‬ ‫‪+ ı sin‬‬ ‫‪ •2‬ﻧﻀﻊ‬
‫‪12‬‬ ‫‪12‬‬

‫‪Ŕž Ŧ Ľ‬‬
‫‪z‬‬ ‫‪6√2 cos‬‬ ‫‪+ ı sin‬‬ ‫‪6√2,‬‬
‫= ‪z‬‬ ‫=‬ ‫=‬
‫‪z‬‬ ‫‪3√2 cos‬‬ ‫‪+ ı sin‬‬

‫‪œǃ‬‬
‫‪3√2,‬‬

‫𝜋 𝜋‪6√2 17‬‬ ‫𝜋‪7‬‬ ‫𝜋‪7‬‬ ‫𝜋‪7‬‬


‫=‬ ‫‪,‬‬ ‫‪−‬‬ ‫‪= 2,‬‬ ‫‪= 2 cos‬‬ ‫‪+ ı sin‬‬
‫‪3√2‬‬ ‫‪12‬‬ ‫‪4‬‬ ‫‪6‬‬ ‫‪6‬‬ ‫‪6‬‬

‫‪Ŀ‬‬
‫و ﻫﻮ اﻟﺸﻜﻞ اﳌﺜﻠﺜﻲ ﻟﻠﻌﺪد ‪ . z‬ﻟﻜﻦ ‪ z = −√3 − ı‬ﻣﻨﻪ ‪:‬‬

‫𝜋‪17‬‬ ‫‪17𝜋 z ⋅ z‬‬ ‫‪(3 + 3ı) −√3 − ı‬‬ ‫‪−3√3 − 3ı − 3ı√3 + 3‬‬
‫‪cos‬‬ ‫‪+ ı sin‬‬ ‫=‬ ‫=‬ ‫=‬
‫‪12‬‬ ‫‪12‬‬ ‫‪6√2‬‬ ‫‪6√2‬‬ ‫‪6√2‬‬
‫‪3 − 3√3 − 3 1 + √3 ı‬‬ ‫‪√2 − √3 √2 + √3‬‬
‫=‬ ‫=‬ ‫‪−‬‬ ‫‪ı‬‬
‫‪6√2‬‬ ‫‪4‬‬ ‫‪4‬‬

‫𝜋‪17‬‬ ‫‪√2 + √3‬‬ ‫‪17𝜋 √2 − √3‬‬


‫‪. sin‬‬ ‫‪=−‬‬ ‫و‬ ‫‪cos‬‬ ‫=‬ ‫ﻣﻨﻪ ﻧﺴﺘﻨﺘﺞ ّ‬
‫أن ‪:‬‬
‫‪12‬‬ ‫‪4‬‬ ‫‪12‬‬ ‫‪4‬‬
‫■‬

‫‪ z‬ﻋﺪد ﻣﺮﻛﺐ ﺻﻮرﺗﻪ ‪. M‬‬


‫‪.‬‬
‫𝟑𝟕‬ ‫‬
‫‪.‬‬

‫ﻋﲔ ﳎﻤﻮﻋﺔ اﻟﻨﻘﻂ ‪ M‬ﻣﻦ اﳌﺴﺘﻮي ﺑﺤﻴﺚ ﻳﻜﻮن ‪. z + z = |z| :‬‬‫ّ‬

‫ﺍﳊﻞّ‪ .‬ﻧﻀﻊ ‪ z = x + ıy‬ﻣﻊ ‪ . x, y ∈ ℝ‬ﻟﺪﻳﻨﺎ ‪:‬‬

‫= ‪z + z = |z| ⟺ x + ıy + x − ıy‬‬ ‫‪x +y‬‬


‫‪ņőƱŀ Ţ‬‬
‫= ‪⟺ 2x‬‬ ‫‪x +y‬‬
‫‪⟺ x≥0‬‬ ‫و‬ ‫‪4x = x + y‬‬
‫‪⟺ x≥0‬‬ ‫و‬ ‫‪y = 3x‬‬
‫‪VI‬‬

‫‪⟺ x≥0‬‬ ‫و‬ ‫‪y = ±√3x‬‬


‫ِ‬
‫إذن ﳎﻤﻮﻋﺔ اﻟﻨﻘﻂ ‪ M‬اﻟﺘﻲ ُﲢﻘﻖ اﳌﻄﻠﻮب ﻫﻲ اﲢﺎد ﻧﺼ َﻔ ْﻲ اﳌﺴﺘﻘﻴﻤﲔ َذ َو ْ‬
‫ي اﳌﻌﺎدﻟﺘﲔ ‪ y = −√3x‬ﻣﻊ ‪ x ≥ 0‬و ‪y = √3x‬‬
‫■‬ ‫ﻣﻊ‪. x ≥ 0‬‬
‫‪ƕ‬‬

‫ُﻳﻌﻄﻰ ﻋﺪد ﻣﺮﻛﺐ 𝛼 ‪ ،‬و ‪ z‬ﻋﺪد ﻣﺮﻛﺐ ﺻﻮرﺗﻪ ‪. M‬‬


‫‪.‬‬
‫𝟒𝟕‬ ‫‪.‬‬ ‫‬
‫ﻋﲔ ﳎﻤﻮﻋﺔ اﻟﻨﻘﻂ ‪ M‬ﻣﻦ اﳌﺴﺘﻮي ﺑﺤﻴﺚ ﻳﻜﻮن ‪. (z − 𝛼) (z − 𝛼) = 4𝛼𝛼 :‬‬‫ﱢ‬
‫‪ũŏ‬‬

‫ﺍﳊﻞّ‪ .‬ﻟﺪﻳﻨﺎ ‪:‬‬

‫𝛼𝛼‪(z − 𝛼) (z − 𝛼) = 4𝛼𝛼 ⟺ (z − 𝛼) (z − 𝛼) = 4‬‬


‫|𝛼| ‪⟺ |z − 𝛼| = 4‬‬
‫|𝛼| ‪⟺ |z − 𝛼| = 2‬‬

‫𝟵𝟯𝟭‬

‫‪http ://tinyurl.com/Malki1718‬‬ ‫‪0‬‬


‫‪ .2.VI‬اﻟﺸﲁ اﳌﺜﻠﱻ ﻟﻌﺪد ﻣﺮﻛﺐ‬
‫‪.‬‬
‫و ﻫﻲ ﳎﻤﻮﻋﺔ اﻟﻨﻘﻂ ‪ M‬ذات اﻟﻼﺣﻘﺔ ‪ z‬اﻟﺘﻲ ﺗﺒﻌﺪ ﻋﻦ اﻟﻨﻘﻄﺔ ‪ Ω‬ذات اﻟﻼﺣﻘﺔ 𝛼 ﺑﻤﺴﺎﻓﺔ ﺛﺎﺑﺘﺔ ﺗﺴﺎوي |𝛼| ‪ 2‬أي ﻫﻲ اﻟﺪاﺋﺮة‬
‫■‬ ‫)|𝛼| ‪ 𝒞 (Ω, 2‬اﻟﺘﻲ ﻣﺮﻛﺰﻫﺎ )𝛼( ‪ Ω‬و ﻧﺼﻒ ﻗﻄﺮﻫﺎ |𝛼| ‪. 2‬‬

‫‪Ŕž Ŧ Ľ‬‬
‫‬ ‫‪.‬‬

‫‪œǃ‬‬
‫‪𝟕𝟓.‬‬
‫‪ z‬ﻋﺪد ﻣﺮﻛﺐ ﺣﻴﺚ ‪. z = cos 𝛼 + ı sin 𝛼 :‬‬
‫‪ •1‬ﻣﺎ ﻫﻲ ﳎﻤﻮﻋﺔ ﺻﻮر اﻷﻋﺪاد ‪ z‬ﻋﻨﺪﻣﺎ ّ‬
‫ﻳﺘﻐﲑ 𝛼 ؟‬

‫‪Ŀ‬‬
‫‪1 + ıx‬‬
‫‪.‬‬ ‫‪ . 1 + ıx‬ﻣﺎذا ﺗﺴﺘﻨﺘﺞ ؟ أﺛﺒﺖ ّ‬
‫أن ‪≠ −1 :‬‬ ‫‪ x •2‬ﻋﺪد ﺣﻘﻴﻘﻲ ‪ .‬أﺣﺴﺐ ﻃﻮﻳﻠﺔ اﻟﻌﺪد اﳌﺮﻛﺐ‬
‫‪1 − ıx‬‬ ‫‪1 − ıx‬‬
‫‪1 + ıx‬‬
‫ﺣﻴﺚ ‪. x ∈ ℝ‬‬ ‫‪ •3‬ﱢﺑﲔ أﻧﻪ إذا ﻛﺎن ‪ z ≠ −1‬ﻓﺈﻧﻪ ﻳﻤﻜﻦ ﻛﺘﺎﺑﺔ ‪ z‬ﻋﲆ اﻟﺸﻜﻞ‬
‫‪1 − ıx‬‬

‫‪ •1‬ﻟﺪﻳﻨﺎ ‪:‬‬ ‫ﺍﳊﻞّ‪.‬‬


‫= ||‪||z − 0‬‬ ‫‪(cos 𝛼) + (sin 𝛼) = √1 = 1‬‬
‫إذن‪ ،‬ﻣﻬﲈ ﻳﻜﻦ اﻟﻌﺪد اﳊﻘﻴﻘﻲ 𝛼 ّ‬
‫ﻓﺈن اﻟﻨﻘﻄﺔ اﻟﺘﻲ ﻻﺣﻘﺘﻬﺎ ‪ z‬ﺗﻨﺘﻤﻲ إﱃ اﻟﺪاﺋﺮة اﻟﺘﻲ ﻣﺮﻛﺰﻫﺎ )‪) O (0, 0‬ﻣﺒﺪأ اﳌﻌﻠﻢ( و‬
‫ﻧﺼﻒ ﻗﻄﺮﻫﺎ ‪. 1‬‬
‫ﻳﺘﻐﲑ 𝛼 ﻫﻲ اﻟﺪاﺋﺮة )‪. 𝒞 (O, 1‬‬
‫إذن‪ ،‬ﳎﻤﻮﻋﺔ ﺻﻮر اﻷﻋﺪاد ‪ z‬ﻋﻨﺪﻣﺎ ّ‬
‫‪1 + ıx‬‬
‫ﻌﺮف‪ .‬ﻋﻼو ًة ﻋﲆ ذﻟﻚ‪ ،‬ﻟﺪﻳﻨﺎ ‪:‬‬
‫ُﻣ ّ‬ ‫ﻟﻴﻜﻦ ‪ x‬ﻋﺪد ًا ﺣﻘﻴﻘﻴ ًﺎ‪ .‬ﻣﻦ اﻟﻮاﺿﺢ ّ‬
‫أن ‪ 1 − ıx ≠ 0‬إذن ﻓﺎﻟﻌﺪد‬ ‫•‬ ‫‪2‬‬
‫‪1 − ıx‬‬
‫||‪| 1 + ıx | ||1 + ıx‬‬ ‫‪√1 + x‬‬
‫|‬ ‫=|‬ ‫=‬ ‫‪=1‬‬
‫||‬ ‫‪1‬‬ ‫‪−‬‬ ‫‪ıx‬‬ ‫|‬
‫|‪|| |1 − ıx‬‬‫|‬
‫)‪1 + (−x‬‬
‫‪1 + ıx‬‬
‫‪.‬‬ ‫ﻧﺴﺘﻨﺘﺞ أﻧﻪ ﻳﻮﺟﺪ ‪ 𝛼 ∈ ℝ‬ﺑﺤﻴﺚ ‪= cos 𝛼 + ı sin 𝛼 :‬‬
‫‪1 − ıx‬‬
‫‪ņőƱŀ Ţ‬‬
‫ﻣﻦ ﺟﻬﺔ أﺧﺮى‪ ،‬ﻟﺪﻳﻨﺎ ‪:‬‬
‫‪1 + ıx −1 + ıx + 2 − (1 − ıx) + 2‬‬ ‫‪2‬‬
‫=‬ ‫=‬ ‫‪= −1 +‬‬ ‫‪≠ −1‬‬
‫‪1 − ıx‬‬ ‫‪1 − ıx‬‬ ‫‪1 − ıx‬‬ ‫‪1 − ıx‬‬
‫‪1 + ıx‬‬
‫‪.‬‬ ‫إذن‪ ،‬ﻣﻬﲈ ﻳﻜﻦ اﻟﻌﺪد اﳊﻘﻴﻘﻲ ‪ّ x‬‬
‫ﻓﺈن ‪≠ −1 :‬‬

‫‪VI‬‬
‫‪1 − ıx‬‬
‫𝛼‬
‫أن ‪ّ z ≠ −1‬‬
‫ﻓﺈن )𝜋‪ 𝛼 ≠ 𝜋 (mod 2‬ﻣﻨﻪ ‪ cos ≠ 0‬و ﺑﺎﻟﺘﺎﱄ ‪:‬‬ ‫ﻟﻴﻜﻦ ‪ . z = cos 𝛼 + ı sin 𝛼 = e‬ﺑﲈ ّ‬ ‫•‬ ‫‪3‬‬
‫‪2‬‬
‫‪e‬‬ ‫‪/‬‬ ‫‪cos + ı sin gray‬‬ ‫‬
‫‪cos  1 + tan‬‬ ‫‪1 + ı tan‬‬
‫= ‪z=e = − /‬‬ ‫  =‬ ‫=‬
‫‪e‬‬ ‫‪cos − ı sin gray‬‬ ‫‪cos 1 + tan‬‬ ‫‪1 − ı tan‬‬
‫‪ƕ‬‬

‫‪1 + ıx‬‬ ‫𝛼‬


‫= ‪.z‬‬ ‫‪ x = tan‬ﻓﻴﻜﻮن ‪ x ∈ ℝ :‬و‬ ‫ﻧﻀﻊ‬
‫‪1 − ıx‬‬ ‫‪2‬‬
‫اﳋﻼﺻﺔ ‪ :‬إذا ﻛﺎﻧﺖ 𝕌 ﳎﻤﻮﻋﺔ اﻷﻋﺪاد اﳌﺮﻛﺒﺔ اﻟﺘﻲ ﻃﻮﻳﻠﺘﻬﺎ ‪ 1‬و ﻛﺎن ‪ّ z ∈ ℂ‬‬
‫ﻓﺈن ‪:‬‬
‫‪1 + ıx‬‬
‫‪ũŏ‬‬

‫= ‪z ∈ 𝕌 ⧵ {−1} ⟺ ∃x ∈ ℝ ∣ z‬‬
‫‪1 − ıx‬‬
‫■‬

‫𝟬𝟰𝟭‬

‫‪http ://tinyurl.com/Malki1718‬‬ ‫‪0‬‬


‫ ﲤﺎرﻳﻦ ﺗﻄﺒﻴﻘﻴﺔ‬.VI
.

 .
𝟕𝟔 .

Ŕž Ŧ Ľ
ı 1 + ı√3
.z = ‫و‬z = ‫ ﻋﺪدان ﻣﺮﻛﺒﺎن ﺣﻴﺚ‬z ‫ و‬z
2ı√3 − 2 1 − ı√3
: ‫أﺣﺴﺐ ﻛﻼ ﻣﻦ اﻷﻋﺪاد اﻟﺘﺎﻟﻴﺔ و اﻛﺘﺒﻬﺎ ﻋﲆ اﻟﺸﻜﻞ اﳉﱪي‬

œǃ
z z
5 • 3 z +z 1

• z
z

Ŀ
z +1
• 6 z ⋅z 4 z ⋅z 2
z −1 • •

: ‫ ﻟﺪﻳﻨﺎ‬.ّ‫ﺍﳊﻞ‬

1 + ı√3 1 + ı√3 1 + 2ı√3 − 3 1 √3


z = = = =− +ı
1 − ı√3 1 + √3 4 2 2
2𝜋 2𝜋 2𝜋
= cos + ı sin = j = 1,
3 3 3

ı ı −2ı√3 − 2 2√3 − 2ı √3 1
z = = = = − ı
2ı√3 − 2 16 8 8
2√3 + 2

1 √3 1 1 𝜋 𝜋 1 𝜋
= − ı = cos − + ı sin − = ,−
4 2 2 4 6 6 4 6

: ‫و ﺑﺎﻟﺘﺎﱄ‬
ņőƱŀ Ţ
1 √3 √3 1 √3 − 4 4√3 − 1
z +z =− +ı + − ı= + ı • 1
2 2 8 8 8 8
2𝜋 1 𝜋 1 2𝜋 𝜋 1 𝜋 1
z ⋅ z = 1, ⋅ ,− = 1× , − = , = ı • 2
VI

3 4 6 4 3 6 4 2 4
z 1, 1 2𝜋 𝜋 5𝜋 5𝜋 5𝜋
= = , + = 4, = 4 cos + ı sin • 3
z ,− 3 6 6 6 6

√3 1
=4 − + ı = −2√3 + 2ı
2 2
ƕ

2𝜋 1 𝜋 2𝜋 1 𝜋
z ⋅z = 1, ⋅ ,− = 1 ,6 × ⋅ , 12 × − •4
3 4 6 3 4 6
1 1 1 1
ũŏ

= [1, 4𝜋] ⋅ , −2𝜋 = 1 × , 4𝜋 − 2𝜋 = , 2𝜋 =


2 2 2 2
z 1, 1 ,8 × 1, 1,
= = = = • 5
z ,− ,3 × − ,− ,−

cos + ı sin 1 √3
= = 64ı − − ı = 32√3 − 32ı
− ı 2 2

𝟭𝟰𝟭

http ://tinyurl.com/Malki1718 0
‫‪ .2.VI‬اﻟﺸﲁ اﳌﺜﻠﱻ ﻟﻌﺪد ﻣﺮﻛﺐ‬
‫‪.‬‬
‫‪z‬‬ ‫‪+1‬‬ ‫‪1,‬‬ ‫‪+1‬‬ ‫‪1,‬‬ ‫‪+1‬‬ ‫‪1,‬‬ ‫‪+1‬‬ ‫‪cos‬‬ ‫‪+ ı sin‬‬ ‫‪+1‬‬
‫=‬ ‫=‬ ‫=‬ ‫=‬ ‫•‬ ‫‪6‬‬
‫‪z‬‬ ‫‪−1‬‬ ‫‪1,‬‬ ‫‪−1‬‬ ‫‪1,‬‬ ‫‪−1‬‬ ‫‪cos‬‬ ‫‪+ ı sin‬‬ ‫‪−1‬‬

‫‪Ŕž Ŧ Ľ‬‬
‫‪1,‬‬ ‫‪−1‬‬

‫‪− +ı‬‬ ‫‪+1‬‬ ‫‪1 + ı√3‬‬ ‫‪1 + ı√3 −3 − ı√3‬‬


‫=‬ ‫=‬ ‫=‬

‫‪œǃ‬‬
‫‪− +ı‬‬ ‫‪−1‬‬ ‫‪−3 + ı√3‬‬ ‫‪3 + √3‬‬
‫‪−3 − ı√3 − 3ı√3 + 3‬‬ ‫‪√3‬‬
‫=‬ ‫‪=−‬‬ ‫‪ı‬‬
‫‪12‬‬ ‫‪3‬‬
‫■‬

‫‪Ŀ‬‬
‫‪2 + ı√6‬‬
‫‬ ‫‪𝟕𝟕.‬‬ ‫‪.‬‬
‫√ = ‪.z‬‬ ‫‪ z‬ﻋﺪد ﻣﺮﻛﺐ ﺣﻴﺚ ‪:‬‬
‫)‪2 (1 − ı‬‬

‫‪ . z‬أﻛﺘﺐ ﻛﻞ اﻟﻨﺘﺎﺋﺞ ﻋﲆ اﻟﺸﻜﻞ اﳉﱪي‪.‬‬ ‫‪z‬و‬ ‫‪ •1‬أﺣﺴﺐ ﻛﻼ ﻣﻦ اﻷﻋﺪاد ‪، z ، z :‬‬

‫ﻋﲔ ﻗﻴﻢ اﻟﻌﺪد اﻟﻄﺒﻴﻌﻲ )ﻏﲑ اﳌﻌﺪوم( ‪ n‬ﺣﺘﻰ ﻳﻜﻮن ‪ z‬ﻋﺪد ًا ﺣﻘﻴﻘﻴﺎ‪.‬‬
‫‪ •2‬ﱠ‬
‫أﺣﺴﺐ ‪ z‬ﻣﻦ أﺟﻞ أﺻﻐﺮ ﻗﻴﻤﺔ ﻟﻠﻌﺪد ‪ n‬اﳌﺘﺤﺼﻞ ﻋﻠﻴﻬﺎ‪.‬‬
‫ﻋﲔ ﻗﻴﻢ اﻟﻌﺪد اﻟﻄﺒﻴﻌﻲ ‪ n‬ﺣﺘﻰ ﻳﻜﻮن ‪ z‬ﲣﻴﻠﻴ ًﺎ ﴏﻓﺎ‪.‬‬
‫‪ •3‬ﱠ‬

‫ﺍﳊﻞّ‪ .‬ﻟﺪﻳﻨﺎ ‪:‬‬


‫‪+‬‬ ‫‪/‬‬
‫‪√2 1 + ı√3‬‬ ‫‪cos‬‬ ‫‪+ ı sin‬‬ ‫‪e‬‬ ‫‪+‬‬ ‫‪/‬‬
‫=‪z‬‬ ‫⋅‬ ‫=‬ ‫=‬ ‫=‬ ‫‪=e‬‬ ‫‪=e‬‬
‫‪2‬‬ ‫‪1−ı‬‬ ‫‪−‬‬ ‫‪cos −‬‬ ‫‪+ ı sin −‬‬ ‫‪e−‬‬ ‫‪/‬‬

‫‪ •1‬ﻣﻨﻪ ‪:‬‬
‫‪/‬‬ ‫×‬ ‫‪/‬‬ ‫‪/‬‬ ‫‪= e−‬‬ ‫‪/‬‬
‫‪ņőƱŀ Ţ‬‬
‫‪z = e‬‬ ‫‪=e‬‬ ‫‪=e‬‬ ‫‪= −ı‬‬
‫‪z‬‬ ‫‪= e‬‬ ‫‪/‬‬ ‫‪=e‬‬ ‫‪=e‬‬ ‫‪= −1‬‬
‫‪/‬‬ ‫×‬ ‫‪/‬‬ ‫‪−‬‬ ‫‪√3 1‬‬
‫‪z‬‬ ‫‪= e‬‬ ‫‪=e‬‬ ‫‪=e‬‬ ‫‪= e−‬‬ ‫‪/‬‬ ‫‪=−‬‬ ‫‪+ ı‬‬
‫‪2‬‬ ‫‪2‬‬

‫‪VI‬‬
‫‪z‬‬ ‫‪= e‬‬ ‫‪/‬‬ ‫‪=e‬‬ ‫×‬ ‫‪/‬‬ ‫‪=e‬‬ ‫‪=e‬‬ ‫‪=1‬‬

‫‪ّ z =e‬‬
‫ﻓﺈن ‪:‬‬ ‫‪/‬‬ ‫‪ •2‬ﺑﲈ ّ‬
‫أن‬
‫𝜋‪7n‬‬ ‫𝜋‪7n‬‬
‫‪z ∈ ℝ ⟺ Im (z ) = 0 ⟺ sin‬‬ ‫⟺ ‪=0‬‬ ‫)𝜋 ‪= 0 (mod‬‬
‫‪ƕ‬‬

‫‪12‬‬ ‫‪12‬‬
‫)‪⟺ 7n = 0 (mod 12) ⟺ n = 0 (mod 12‬‬ ‫ّ‬
‫)ﻷن ‪(gcd (7, 12) = 1‬‬
‫إذن‪ ،‬ﺣﺘﻰ ﻳﻜﻮن ‪ z‬ﻋﺪد ًا ﺣﻘﻴﻘﻴﺎ ﳚﺐ أن ﻳﻜﻮن ‪ n‬ﻣﻦ ﻣﻀﺎﻋﻔﺎت ‪. 12‬‬
‫‪ũŏ‬‬

‫أن ‪. z = −1‬‬ ‫ﻷﺟﻞ ‪ n = 12‬وﺟﺪﻧﺎ ّ‬


‫‪ •3‬ﺑﺎﳌﺜﻞ ‪:‬‬
‫𝜋‪7n‬‬ ‫𝜋 𝜋‪7n‬‬
‫‪z ∈ ıℝ ⟺ Re (z ) = 0 ⟺ cos‬‬ ‫⟺ ‪=0‬‬ ‫)𝜋 ‪= (mod‬‬
‫‪12‬‬ ‫‪12‬‬ ‫‪2‬‬
‫)‪⟺ 7n = 6 (mod 12) ⟺ 7 × 7n = 7 × 6 (mod 12‬‬
‫‪⟺ 49n = 42 (mod 12) ⟺ n = 6 (mod 12) ⟺ n = 12k + 6 = 6 (2k + 1) ,‬‬ ‫‪k∈ℕ‬‬

‫𝟮𝟰𝟭‬

‫‪http ://tinyurl.com/Malki1718‬‬ ‫‪0‬‬


‫‪ .VI‬ﲤﺎرﻳﻦ ﺗﻄﺒﻴﻘﻴﺔ‬
‫‪.‬‬
‫و ﺑﺎﻟﺘﺎﱄ‪ ،‬ﻟﻴﻜﻮن ‪ z‬ﲣﻴﻠﻴ ًﺎ ﴏﻓﺎ ﳚﺐ أن ﻳﻜﻮن ‪ n‬ﻣﻦ اﳌﻀﺎﻋﻔﺎت اﻟﻔﺮدﻳﺔ ﻟﻠﻌﺪد ‪) 6‬أي ﻣﻦ ﻣﻀﺎﻋﻔﺎت ‪ 6‬و ﻟﻴﺲ ﻣﻦ‬
‫ﻣﻀﺎﻋﻔﺎت ‪. (12‬‬

‫‪Ŕž Ŧ Ľ‬‬
‫■‬

‫‪œǃ‬‬
‫‬ ‫‪.‬‬
‫𝟖𝟕‬ ‫‪.‬‬

‫𝜋‬ ‫𝜋‬

‫‪Ŀ‬‬
‫‪. z = (x − 2) cos + ı sin‬‬
‫‪4‬‬ ‫‪4‬‬ ‫‪ •1‬ﱢ‬
‫ﻋﲔ‪ ،‬ﺣﺴﺐ ﻗﻴﻢ اﻟﻌﺪد اﳊﻘﻴﻘﻲ ‪ ، x‬اﻟﻄﻮﻳﻠﺔ و ﻋﻤﺪة ﻟﻠﻌﺪد اﳌﺮﻛﺐ ‪:‬‬

‫ﺛﻢ ﺣﺪﱢ د إﺷﺎرﺗﻪ‪.‬‬


‫‪ z‬ﺣﻘﻴﻘﻲ ّ‬ ‫‪ •2‬ﱠﺑﲔ ّ‬
‫أن اﻟﻌﺪد‬

‫‪ z = (x − 2) e‬؛ و ﺣﺴﺐ اﳌﱪﻫﻨﺔ ‪ 8‬ﺻﻔﺤﺔ ‪ّ 20‬‬


‫ﻓﺈن ‪:‬‬ ‫‪/‬‬ ‫‪ •1‬ﻟﺪﻳﻨﺎ ‪:‬‬ ‫ﺍﳊﻞّ‪.‬‬

‫إذا ﻛﺎن ‪ّ x = 2‬‬


‫ﻓﺈن ‪ z = 0‬؛‬ ‫•‬
‫𝜋‬
‫)𝜋‪ arg (z) = (mod 2‬؛‬ ‫و‬ ‫ﻓﺈن ‪|z| = x − 2 :‬‬‫إذا ﻛﺎن ‪ّ x > 2‬‬ ‫•‬
‫‪4‬‬
‫𝜋‪𝜋 5‬‬
‫= ‪. arg (z) = 𝜋 +‬‬ ‫)𝜋‪(mod 2‬‬ ‫و‬ ‫‪|z| = 2 − x‬‬ ‫و إذا ﻛﺎن ‪ّ x < 2‬‬
‫ﻓﺈن ‪:‬‬ ‫•‬
‫‪4‬‬ ‫‪4‬‬
‫أن ‪ّ 1980 = 4 × 495‬‬
‫ﻓﺈن ‪:‬‬ ‫‪ •2‬ﺑﲈ ّ‬
‫‪z‬‬ ‫)‪= (x − 2‬‬ ‫‪e‬‬ ‫‪/‬‬ ‫)‪= (x − 2‬‬ ‫‪e‬‬ ‫)‪= (x − 2‬‬ ‫‪e‬‬ ‫)‪= − (x − 2‬‬
‫)‪. − (x − 2‬‬ ‫و ﻫﻮ ﻋﺪد ﺣﻘﻴﻘﻲ‪ .‬ﻟﻜﻦ ‪ 1980‬ﻋﺪد زوﺟﻲ إذن ‪≤ 0‬‬
‫‪ z‬ﻋﺪد ﺣﻘﻴﻘﻲ ﺳﺎﻟﺐ‪.‬‬ ‫ﰲ اﻷﺧﲑ‪ ،‬اﻟﻌﺪد‬
‫■‬

‫‬
‫‪ņőƱŀ Ţ‬‬
‫‪.‬‬
‫𝟗𝟕‬ ‫‪.‬‬
‫)𝜃 ‪𝛼 (1 + ı tan‬‬
‫= ‪.z‬‬ ‫𝛼 و 𝜃 ﻋﺪدان ﺣﻘﻴﻘﻴﺎن ‪ .‬أوﺟﺪ اﻟﻄﻮﻳﻠﺔ و ﻋﻤﺪة ﻟﻠﻌﺪد اﳌﺮﻛﺐ ‪:‬‬
‫𝜃 ‪1 + tan‬‬

‫ﺍﳊﻞّ‪ .‬ﻟﺪﻳﻨﺎ ‪:‬‬


‫‪VI‬‬

‫)𝜃 ‪𝛼 (1 + ı tan‬‬ ‫)𝜃 ‪𝛼 (cos 𝜃 + ı sin‬‬


‫=‪z‬‬ ‫=‬ ‫‪× cos 𝜃 = e‬‬ ‫‪𝛼 cos 𝜃 = e‬‬ ‫𝜃 ‪𝛼 cos‬‬
‫𝜃 ‪1 + tan‬‬ ‫𝜃 ‪cos‬‬
‫‪ƕ‬‬

‫أن ‪ cos 𝜃 ≥ 0‬ﻓﺤﺴﺐ اﳌﱪﻫﻨﺔ ‪ 8‬ﺻﻔﺤﺔ ‪ّ 20‬‬


‫ﻓﺈن ‪:‬‬ ‫و ﺑﲈ ّ‬
‫‪z = 0‬؛‬ ‫إذا ﻛﺎن ‪ 𝛼 cos 𝜃 = 0‬أي إذا ﻛﺎن ‪ 𝛼 = 0‬أو ‪ّ cos 𝜃 = 0‬‬
‫ﻓﺈن ‪:‬‬ ‫•‬

‫إذا ﻛﺎن ‪ 𝛼 cos 𝜃 > 0‬أي إذا ﻛﺎن ‪ 𝛼 > 0‬و ‪ّ cos 𝜃 ≠ 0‬‬
‫ﻓﺈن ‪:‬‬
‫‪ũŏ‬‬

‫•‬
‫𝜃 ‪|z| = 𝛼 cos‬‬ ‫و‬ ‫)𝜋‪arg (z) = 10𝜃 (mod 2‬‬

‫و إذا ﻛﺎن ‪ 𝛼 cos 𝜃 < 0‬أي إذا ﻛﺎن ‪ 𝛼 < 0‬و ‪ّ cos 𝜃 ≠ 0‬‬
‫ﻓﺈن ‪:‬‬ ‫•‬
‫𝜃 ‪|z| = −𝛼 cos‬‬ ‫و‬ ‫)𝜋‪arg (z) = 𝜋 + 10𝜃 (mod 2‬‬

‫■‬

‫𝟯𝟰𝟭‬

‫‪http ://tinyurl.com/Malki1718‬‬ ‫‪0‬‬


‫‪ .2.VI‬اﻟﺸﲁ اﳌﺜﻠﱻ ﻟﻌﺪد ﻣﺮﻛﺐ‬
‫‪.‬‬

‫‪#‬‬ ‫‪𝟖𝟎.‬‬ ‫‪.‬‬

‫‪Ŕž Ŧ Ľ‬‬
‫‪ z‬و ‪ z‬ﻋﺪدان ﻣﺮﻛﺒﺎن ﺣﻴﺚ ‪ ||z || = ||z || = 1‬و ‪. z z + 1 ≠ 0‬‬
‫‪z +z‬‬
‫ﺣﻘﻴﻘﻲ‪.‬‬ ‫ﺑﺮﻫﻦ ّ‬
‫أن اﻟﻌﺪد‬
‫‪1+z z‬‬

‫‪œǃ‬‬
‫= ‪ z‬ﻣﻨﻪ ‪:‬‬
‫‪1‬‬
‫ﻓﺈن ‪ ، z = 1‬و ﺑﲈ ّ‬
‫أن ‪ّ ||z || = 1‬‬
‫ﻓﺈن‬ ‫ﺍﳊﻞّ‪ .‬ﺑﲈ ّ‬
‫أن ‪ّ ||z || = 1‬‬
‫‪z‬‬ ‫‪z‬‬

‫‪Ŀ‬‬
‫‪+‬‬
‫‪z +z‬‬ ‫‪z +z‬‬ ‫‪z +z‬‬ ‫‪+‬‬ ‫‪z +z‬‬
‫=‬ ‫=‬ ‫=‬ ‫=‬ ‫‪+‬‬
‫=‬
‫‪1+z z‬‬ ‫‪1+z z‬‬ ‫‪1+z z‬‬ ‫‪1+‬‬ ‫⋅‬ ‫‪1+z z‬‬

‫‪z +z‬‬
‫■‬ ‫ﺣﻘﻴﻘﻲ‪.‬‬ ‫و ﺑﺎﻟﺘﺎﱄ ﻓﺎﻟﻌﺪد‬
‫‪1+z z‬‬

‫= ‪ A‬ﲣﻴﲇ ﴏف إذا و ﻓﻘﻂ إذا ﻛﺎن ‪ |z| = 1‬أي ‪:‬‬


‫‪1+z‬‬
‫ﻟﻴﻜﻦ }‪ . z ∈ ℂ ⧵ {1‬أﺛﺒﺖ ّ‬
‫أن اﻟﻌﺪد‬
‫‪#‬‬ ‫‪𝟖𝟏.‬‬ ‫‪.‬‬
‫‪1−z‬‬
‫‪1+z‬‬
‫‪Re‬‬ ‫‪= 0 ⟺ |z| = 1‬‬
‫‪1−z‬‬

‫ﺍﳊﻞّ‪ .‬اﻟﻄﺮﻳﻘﺔ اﻷوﱃ ‪ :‬ﻧﻀﻊ ‪ z = x + ıy‬ﻣﻊ ‪ x, y ∈ ℝ‬ﻓﻴﻜﻮن ‪:‬‬

‫)‪1 + x + ıy (1 + x + ıy) (1 − x + ıy‬‬


‫=‪A‬‬ ‫=‬
‫‪1 − x − ıy‬‬ ‫‪(1 − x) + y‬‬
‫)‪(1 + x) (1 − x) − y + ı (1 + x + 1 − x‬‬
‫=‬
‫‪(1 − x) + y‬‬
‫‪ņőƱŀ Ţ‬‬
‫‪1−x −y‬‬ ‫‪2ı‬‬
‫=‬ ‫‪+‬‬
‫‪(1 − x) + y‬‬ ‫‪(1 − x) + y‬‬
‫‪1−x −y‬‬
‫⟺ ‪Re (A) = 0‬‬ ‫‪=0‬‬ ‫إذن ‪:‬‬
‫‪(1 − x) + y‬‬

‫‪VI‬‬
‫‪⟺ x + y = 1 ⟺ |z| = 1 ⟺ |z| = 1‬‬

‫اﻟﻄﺮﻳﻘﺔ اﻟﺜﺎﻧﻴﺔ ‪:‬‬

‫‪1+z‬‬ ‫‪1+z‬‬
‫‪ƕ‬‬

‫⟺ ‪A ∈ ıℝ ⟺ A = −A‬‬ ‫‪=−‬‬
‫‪1−z‬‬ ‫‪1−z‬‬
‫‪1+z‬‬ ‫‪1+z‬‬
‫⟺‬ ‫‪=−‬‬ ‫)‪⟺ (1 + z) (1 − z) = − (1 − z) (1 + z‬‬
‫‪1−z‬‬ ‫‪1−z‬‬
‫‪⟺ 2 − 2zz = 0 ⟺ zz = 1 ⟺ |z| = 1 ⟺ |z| = 1‬‬
‫‪ũŏ‬‬

‫■‬

‫‪ w ، v ، u‬أﻋﺪاد ﻣﺮﻛﺒﺔ ﺣﻴﺚ ‪ . |u| = |v| = |w| = 1‬أﺛﺒﺖ ّ‬


‫أن ‪. |uv + vw + wu| = |u + v + w| :‬‬
‫‪#‬‬ ‫‪𝟖𝟐.‬‬ ‫‪.‬‬

‫𝟰𝟰𝟭‬

‫‪http ://tinyurl.com/Malki1718‬‬ ‫‪0‬‬


‫‪ .VI‬ﲤﺎرﻳﻦ ﺗﻄﺒﻴﻘﻴﺔ‬
‫‪.‬‬
‫‪1‬‬
‫ﺍﳊﻞّ‪ .‬ﻧُﺬﻛﱢﺮ أﻧﻪ إذا ﻛﺎن ‪ّ |z| = 1‬‬
‫ﻓﺈن = ‪ z‬ﻣﻨﻪ ‪:‬‬
‫‪z‬‬

‫‪Ŕž Ŧ Ľ‬‬
‫‪|1 1‬‬ ‫|‪1‬‬
‫| ‪|u + v + w| = |u + v + w| = ||u + v + w|| = ||u + v + w|| = | + +‬‬
‫|| ‪|| u v w‬‬

‫‪œǃ‬‬
‫|‪| uv + uw + vw | |uv + uw + vw| |uv + uw + vw‬‬
‫|=‬ ‫= ||‬ ‫=‬
‫|‬ ‫‪uvw‬‬ ‫|‪|uvw‬‬ ‫|‪|u| ⋅ |v| ⋅ |w‬‬
‫|‪|uv + uw + vw‬‬
‫=‬ ‫|‪= |uv + uw + vw‬‬
‫‪1⋅1⋅1‬‬

‫‪Ŀ‬‬
‫■‬ ‫و ﻫﻮ اﳌﻄﻠﻮب‪.‬‬

‫ﻣﺎ ﻫﻲ ِﻗ َﻴﻢ اﻟﻌﺪد اﳌﺮﻛﺐ ‪ z‬اﻟﺘﻲ ﲡﻌﻞ اﻟﻌﺪد ‪ Z = z + z + 1‬ﺣﻘﻴﻘﻴ ًﺎ ؟ ﲣﻴﻠﻴﺎ ﴏﻓﺎ ؟‬
‫‪#‬‬ ‫‪.‬‬
‫𝟑𝟖‬ ‫‪.‬‬

‫ﺍﳊﻞّ‪ .‬ﻧﻀﻊ ‪ z = x + ıy‬ﻣﻊ ‪ . x, y ∈ ℝ‬ﻟﺪﻳﻨﺎ ‪:‬‬


‫)‪Z = (x + ıy) + (x + ıy) + 1 = x − y + 2ıxy + x + ıy + 1 = x − y + x + 1 + ıy (2x + 1‬‬
‫ﻣﻨﻪ ‪:‬‬
‫‪ Z ⟺ Im (Z) = 0 ⟺ y (2x + 1) = 0 ⟺ y = 0‬ﺣﻘﻴﻘﻲ‬ ‫أو‬ ‫‪2x + 1 = 0‬‬

‫‪1‬‬
‫ذو ْي اﳌﻌﺎدﻟﺘﲔ ‪ x = −‬و ‪. y = 0‬‬
‫إذن اﳌﺠﻤﻮﻋﺔ اﻟﺘﻲ ﻧﺒﺤﺚ ﻋﻨﻬﺎ ﻫﻲ اﲢﺎد اﳌﺴﺘﻘﻴﻤﲔ َ‬
‫‪2‬‬

‫‪1‬‬ ‫‪1‬‬
‫⟺ ‪ Z ⟺ Re (Z) = 0 ⟺ x − y + x + 1 = 0‬ﲣﻴﲇ ﴏف‬ ‫‪x+‬‬ ‫‪−‬‬ ‫‪−y +1=0‬‬
‫‪2‬‬ ‫‪4‬‬

‫‪1‬‬ ‫‪3‬‬ ‫‪√3‬‬


‫‪⟺ (y − 0) − x +‬‬ ‫=‬ ‫=‬
‫‪2‬‬ ‫‪4‬‬ ‫‪2‬‬
‫‪ņőƱŀ Ţ‬‬
‫‪1‬‬ ‫‪1‬‬
‫إذن اﳌﺠﻤﻮﻋﺔ اﻟﺘﻲ ﻧﺒﺤﺚ ﻋﻨﻬﺎ ﻫﻲ اﻟﻘﻄﻊ اﻟﺰاﺋﺪ اﻟﺬي ﻣﺮﻛﺰه ‪ ، Ω − , 0‬ﳏﻮره اﳌﺴﺘﻘﻴﻢ ذو اﳌﻌﺎدﻟﺔ ‪ x = −‬و ﻣﻌﺎدﻟﺘﺎ‬
‫‪2‬‬ ‫‪2‬‬
‫‪1‬‬
‫‪.y = ± x+‬‬ ‫ﻣﺴﺘﻘﻴﻤ ْﻴﻪ اﳌﻘﺎر َﺑ ْﲔ ﳘﺎ‬
‫َ‬
‫‪2‬‬
‫‪VI‬‬

‫■‬
‫‪ƕ‬‬

‫ﻟﺘﻜﻦ 𝕌 ﳎﻤﻮﻋﺔ اﳉﺬور اﻟﻨﻮﻧﻴﺔ ﻟﻠﻮﺣﺪة‪ .‬أﺛﺒﺖ ّ‬


‫أن ‪:‬‬
‫‪#‬‬ ‫‪.‬‬
‫𝟒𝟖‬ ‫‪.‬‬

‫∗‬
‫|‬ ‫|‪1 | |u − z‬‬
‫‪∀u ∈ 𝕌 ,‬‬ ‫‪∀z ∈ ℂ ,‬‬ ‫‪|u −‬‬ ‫=|‬
‫||‬ ‫|| ‪z‬‬ ‫|‪|z‬‬
‫‪ũŏ‬‬

‫ﺍﳊﻞّ‪.‬‬
‫‪1‬‬
‫= ‪ u‬و ﺑﺎﻟﺘﺎﱄ إذا ﻛﺎن ∗‪ّ z ∈ ℂ‬‬
‫ﻓﺈن ‪:‬‬ ‫أن 𝕌 ∈ ‪ّ u‬‬
‫ﻓﺈن ‪ |u| = 1‬ﻣﻨﻪ‬ ‫ﺑﲈ ّ‬
‫‪u‬‬
‫|‬ ‫|‪1| | 1 1‬‬ ‫|‪z − u | ||− (u − z)|| ||u − z|| |u − z‬‬
‫|| = | ‪|u − | = | −‬‬ ‫=‬ ‫=‬ ‫=‬
‫||‬ ‫|| ‪z || || u z || | u z‬‬ ‫||‪||u|| ⋅ ||z‬‬ ‫||‪1 × ||z‬‬ ‫|‪|z‬‬

‫■‬ ‫ّ‬
‫ﻷن |‪ |−v| = |v‬و |‪ ||v|| = |v‬ﻟﻜﻞ ﻋﺪد ﻣﺮﻛﺐ ‪. v ∈ ℂ‬‬

‫𝟱𝟰𝟭‬

‫‪http ://tinyurl.com/Malki1718‬‬ ‫‪0‬‬


‫‪ .2.VI‬اﻟﺸﲁ اﳌﺜﻠﱻ ﻟﻌﺪد ﻣﺮﻛﺐ‬
‫‪.‬‬

‫‪#‬‬ ‫‪𝟖𝟓.‬‬ ‫‪.‬‬

‫‪Ŕž Ŧ Ľ‬‬
‫)‪b (c − a‬‬
‫‪. A ∈ ℝ+‬‬ ‫= ‪ . A‬أﺛﺒﺖ ّ‬
‫أن‬ ‫ﻟﻴﻜﻦ 𝕌 ∈ ‪ a, b, c‬ﺑﺤﻴﺚ ‪ . b ≠ c‬ﻧﻀﻊ‬
‫)‪a (c − b‬‬

‫‪œǃ‬‬
‫ﺍﳊﻞّ‪ .‬ﻧُﺬﻛﱢﺮ ّ‬
‫أن 𝕌 ﻫﻲ ﳎﻤﻮﻋﺔ ّ‬
‫ﻛﻞ ﺟﺬور اﻟﻮﺣﺪة أي ﳎﻤﻮﻋﺔ اﻷﻋﺪاد اﳌﺮﻛﺒﺔ اﻟﺘﻲ ﻃﻮﻳﻠﺘﻬﺎ ﺗﺴﺎوي ‪. 1‬‬
‫)‪b (c − a‬‬
‫= ‪ُ A‬ﻣ َﻌ ﱠﺮف‪.‬‬ ‫ﻓﺈن ‪ a ≠ 0‬و ﺑﲈ ّ‬
‫أن ‪ b ≠ c‬ﻓﺎﳌﻘﺪار‬ ‫أن 𝕌 ∈ ‪ّ a‬‬
‫ﺑﲈ ّ‬

‫‪Ŀ‬‬
‫)‪a (c − b‬‬
‫‪c−a‬‬
‫= ‪ . u‬ﻟﺪﻳﻨﺎ ‪:‬‬ ‫ﻧﻀﻊ‬
‫‪c−b‬‬
‫‪1 1‬‬ ‫‪a−c‬‬
‫‪c−a c−a‬‬ ‫‪−‬‬ ‫‪b c−a b‬‬
‫=‪u‬‬ ‫=‬ ‫⋅ = ‪= c a = ac‬‬ ‫‪= u‬‬
‫‪c−b c−b‬‬ ‫‪1 1‬‬ ‫‪b‬‬ ‫‪−‬‬ ‫‪c‬‬ ‫‪a c−b a‬‬
‫‪−‬‬
‫‪c b‬‬ ‫‪bc‬‬
‫ﻣﻨﻪ ‪:‬‬
‫‪b c−a‬‬ ‫‪b‬‬ ‫‪b‬‬
‫=‪A‬‬ ‫=‬ ‫= ‪u‬‬ ‫‪u u = uu = |u| ∈ ℝ+‬‬
‫‪a c−b‬‬ ‫‪a‬‬ ‫‪a‬‬
‫■‬

‫𝛼 ﻋﺪد ﻣﺮﻛﺐ ﺣﻴﺚ ‪. 𝛼 = 2 − 2ı√3 :‬‬


‫‬ ‫‪𝟖𝟔.‬‬ ‫‪.‬‬

‫‪ •1‬أﻛﺘﺐ 𝛼 ﻋﲆ ﺷﻜﻠﻪ اﳌﺜﻠﺜﻲ ّ‬


‫ﺛﻢ ﺣﻞ ﰲ ‪ ℂ‬اﳌﻌﺎدﻟﺔ ‪. z = 𝛼 :‬‬
‫ﺛﻢ اﻷﻋﺪاد اﳌﺮﻛﺒﺔ ‪ z‬اﻟﺘﻲ ﲢﻘﻖ ‪. z = 𝛼 :‬‬ ‫‪ •2‬ﺑﺎﺳﺘﻌﲈل اﻟﻄﺮﻳﻘﺔ اﳉﱪﻳﺔ‪ ،‬ﱢ‬
‫ﻋﲔ اﻷﻋﺪاد اﳌﺮﻛﺒﺔ ‪ u‬اﻟﺘﻲ ﲢﻘﻖ 𝛼 = ‪ّ ، u‬‬
‫𝜋‪11‬‬ ‫𝜋‪11‬‬
‫‪. sin‬‬ ‫‪ cos‬و‬ ‫‪ •3‬إﺳﺘﻨﺘﺞ ﻗﻴﻤﺘ َْﻲ‬
‫‪12‬‬ ‫‪12‬‬
‫‪ņőƱŀ Ţ‬‬
‫‪ •1‬ﻟﺪﻳﻨﺎ ‪:‬‬ ‫ﺍﳊﻞّ‪.‬‬

‫‪VI‬‬
‫‪1‬‬ ‫‪√3‬‬ ‫𝜋‬ ‫𝜋‬ ‫𝜋‬
‫‪𝛼 = 2 − 2ı√3 = 4‬‬ ‫‪−ı‬‬ ‫‪= 4 cos −‬‬ ‫‪+ ı sin −‬‬ ‫‪= 4, −‬‬
‫‪2‬‬ ‫‪2‬‬ ‫‪3‬‬ ‫‪3‬‬ ‫‪3‬‬

‫و ﻫﻮ اﻟﺸﻜﻞ اﳌﺜﻠﺜﻲ ﻟﻠﻌﺪد 𝛼 ‪.‬‬


‫ﺣﻠﻮل اﳌﻌﺎدﻟﺔ 𝛼 = ‪ z‬ﻫﻲ اﳉﺬور اﻟﺮاﺑﻌﺔ ﻟﻠﻌﺪد 𝛼 ‪ .‬ﻟﻜﻦ ‪:‬‬
‫‪ƕ‬‬

‫𝜋‬
‫‪𝛼 = 4, −‬‬ ‫‪= 4e− / = √2e− /‬‬
‫‪3‬‬
‫‪ −ı√2e−‬أي‬ ‫‪/‬‬ ‫‪ ı√2e−‬و‬ ‫‪/‬‬ ‫و ﺑﺎﻟﺘﺎﱄ ﻓﺤﻠﻮل اﳌﻌﺎدﻟﺔ 𝛼 = ‪ z‬ﻫﻲ اﻷﻋﺪاد ‪، −√2e− / ، √2e− /‬‬
‫‪. √2e−‬‬ ‫‪/‬‬ ‫‪ √2e‬و‬ ‫‪/‬‬ ‫‪، √2e‬‬ ‫‪/‬‬ ‫‪، √2e−‬‬ ‫‪/‬‬ ‫اﻷﻋﺪاد‬
‫‪ũŏ‬‬

‫‪ •2‬ﻧﻀﻊ ‪ u = x + ıy‬ﻣﻊ ‪ . x, y ∈ ℝ‬ﻟﺪﻳﻨﺎ ‪:‬‬


‫⎧‬ ‫‪x −y =2‬‬
‫⎪‬
‫⎪‬
‫⎪‬
‫‪2xy = −2√3‬‬
‫⟺ ‪u = 𝛼 ⟺ x − y + 2ıxy = 2 − 2ı√3‬‬
‫⎨‬
‫⎪‬
‫⎪‬
‫‪⎪ x + y = 2 + 2√3‬‬
‫⎩‬ ‫‪=4‬‬

‫𝟲𝟰𝟭‬

‫‪http ://tinyurl.com/Malki1718‬‬ ‫‪0‬‬


‫‪ .VI‬ﲤﺎرﻳﻦ ﺗﻄﺒﻴﻘﻴﺔ‬
‫‪.‬‬
‫ُﻌﻮض ﰲ اﳌﻌﺎدﻟﺔ اﻟﺜﺎﻧﻴﺔ ﻟﻨﺠﺪ‬
‫ﺑﺠﻤﻊ اﳌﻌﺎدﻟﺘﲔ اﻷوﱃ و اﻟﺜﺎﻟﺜﺔ ﻳﻨﺘﺞ ‪ 2x = 6‬أي ‪ . x = ±√3‬ﻧﺨﺘﺎر ‪ x = √3‬و ﻧ ّ‬
‫‪. y = −1‬‬

‫‪Ŕž Ŧ Ľ‬‬
‫ﰲ اﻷﺧﲑ‪ ،‬ﻟﻠﻤﻌﺎدﻟﺔ 𝛼 = ‪ u‬ﺣﻼّن ﳘﺎ ‪ √3 − ı‬و ‪ . −√3 + ı‬ﻣﻦ ﺟﻬﺔ أﺧﺮى ﻟﺪﻳﻨﺎ ‪:‬‬
‫⟺ 𝛼= ‪z‬‬ ‫‪z‬‬ ‫‪= 𝛼 ⟺ z = ± √3 − ı‬‬

‫‪œǃ‬‬
‫و ﺑﺎﻟﺘﺎﱄ ﳚﺐ اﻟﺒﺤﺚ ﻋﻦ اﳉﺬور اﻟﱰﺑﻴﻌﻴﺔ ﻟﻠﻌﺪدﻳﻦ ‪. ± √3 − ı‬‬
‫ﻟﻴﻜﻦ ‪ v = a + ıb‬ﺟﺬر ًا ﺗﺮﺑﻴﻌﻴ ًﺎ ﻟﻠﻌﺪد ‪ . √3 − ı‬ﻟﺪﻳﻨﺎ ‪:‬‬

‫‪Ŀ‬‬
‫⎧‬
‫⎪‬ ‫‪a − b = √3‬‬
‫⎪‬
‫⎪‬
‫⟺ ‪v = √3 − ı ⟺ a − b + 2ıab = √3 − ı‬‬ ‫‪2ab = −1‬‬
‫⎨‬
‫⎪‬
‫⎪‬
‫⎪‬
‫‪⎩ a + b = √3 + 1 = 2‬‬

‫‪4 + 2√3‬‬ ‫‪2 + √3‬‬


‫‪.a = ±‬‬ ‫‪ a = ±‬أي‬ ‫ﺑﺠﻤﻊ اﳌﻌﺎدﻟﺘﲔ اﻷوﱃ و اﻟﺜﺎﻟﺜﺔ ﻳﻨﺘﺞ ‪ 2a = 2 + √3‬ﻣﻨﻪ‬
‫‪2‬‬ ‫‪2‬‬
‫‪4 − 2√3‬‬
‫‪.b = ±‬‬ ‫ﺑﻄﺮح اﳌﻌﺎدﻟﺔ اﻷوﱃ ﻣﻦ اﻟﺜﺎﻟﺜﺔ ﻳﻨﺘﺞ ‪ 2b = 2 − √3‬أي‬
‫‪2‬‬
‫ﻣﻦ اﳌﻌﺎدﻟﺔ اﻟﺜﺎﻧﻴﺔ ﻧﺴﺘﻨﺘﺞ ّ‬
‫أن ‪ a‬و ‪ b‬ﳐﺘﻠﻔﺎن ﰲ اﻹﺷﺎرة‪.‬‬
‫‪4 + 2√3‬‬ ‫‪4 − 2√3‬‬ ‫‪4 + 2√3‬‬ ‫‪4 − 2√3‬‬
‫‪.−‬‬ ‫‪+ı‬‬ ‫و‬ ‫‪−ı‬‬ ‫ﰲ اﻷﺧﲑ‪ ،‬ﺣﻠﻮل اﳌﻌﺎدﻟﺔ ‪ z = √3 − ı‬ﻫﻲ‬
‫‪2‬‬ ‫‪2‬‬ ‫‪2‬‬ ‫‪2‬‬
‫ﻹﳚﺎد اﳉﺬرﻳﻦ اﻟﱰﺑﻴﻌﻴﲔ ﻟﻠﻌﺪد ‪−√3 + ı‬ﻳﻤﻜﻦ اﺗّﺒﺎع ﻧﻔﺲ اﻟﻄﺮﻳﻘﺔ‪ ،‬أو ﻛﲈ ﻳﲇ ‪:‬‬

‫‪4 + 2 √3‬‬ ‫‪4 − 2√3‬‬


‫‪−√3 + ı = − √3 − ı = ı‬‬ ‫‪±‬‬ ‫‪−ı‬‬
‫‪2‬‬ ‫‪2‬‬

‫‪4 + 2√3‬‬ ‫‪4 − 2 √3‬‬


‫‪= ±ı‬‬ ‫‪−ı‬‬
‫‪ņőƱŀ Ţ‬‬
‫‪2‬‬ ‫‪2‬‬

‫‪4 − 2√3‬‬ ‫‪4 + 2√3‬‬


‫‪= ±‬‬ ‫‪+ı‬‬
‫‪2‬‬ ‫‪2‬‬
‫‪VI‬‬

‫‪4 − 2√3‬‬ ‫‪4 + 2√3‬‬ ‫‪4 − 2√3‬‬ ‫‪4 + 2 √3‬‬


‫‪.−‬‬ ‫‪−ı‬‬ ‫و‬ ‫‪+ı‬‬ ‫إذن‪ ،‬اﳉﺬران اﻟﱰﺑﻴﻌﻴﺎن ﻟﻠﻌﺪد ‪ −√3 + ı‬ﳘﺎ‬
‫‪2‬‬ ‫‪2‬‬ ‫‪2‬‬ ‫‪2‬‬
‫‪ 4 − 2√3 = √3 − 1‬و ﺑﺎﻟﺘﺎﱄ ﻓﺤﻠﻮل اﳌﻌﺎدﻟﺔ 𝛼 = ‪z‬‬ ‫‪ 4 + 2√3 = √3 + 1‬و‬ ‫ﻳﻤﻜﻦ أن ﻧﻼﺣﻆ ّ‬
‫أن‬
‫‪3−1‬‬ ‫‪√3 + 1‬‬ ‫‪3−1‬‬ ‫‪√3 + 1‬‬ ‫‪3+1‬‬ ‫‪√3 − 1 √3 + 1‬‬ ‫‪√3 − 1‬‬
‫√‪. −‬‬ ‫√و‬ ‫√‪، −‬‬
‫‪ƕ‬‬

‫‪−ı‬‬ ‫‪+ı‬‬ ‫‪+ı‬‬ ‫‪،‬‬ ‫‪−ı‬‬ ‫ﻫﻲ‪:‬‬


‫‪2‬‬ ‫‪2‬‬ ‫‪2‬‬ ‫‪2‬‬ ‫‪2‬‬ ‫‪2‬‬ ‫‪2‬‬ ‫‪2‬‬
‫𝜋‪7‬‬ ‫𝜋‪7‬‬ ‫𝜋‬ ‫𝜋‬
‫‪ cos −‬و ﻫﻲ أﻳﻀ ًﺎ ﻣﺘﻨﺎﻗﺼﺔ ﻋﲆ ]𝜋 ‪ [0,‬و‬ ‫‪= cos‬‬ ‫‪ cos −‬و‬ ‫‪= cos‬‬ ‫‪ •3‬اﻟﺪاﻟﺔ ‪ cos‬زوﺟﻴﺔ إذن‬
‫‪12‬‬ ‫‪12‬‬ ‫‪12‬‬ ‫‪12‬‬
‫‪ũŏ‬‬

‫ﺑﺎﻟﺘﺎﱄ ‪:‬‬
‫𝜋‬ ‫𝜋‪5‬‬ ‫𝜋‪7‬‬ ‫𝜋‪11‬‬
‫‪cos‬‬ ‫‪≥ cos‬‬ ‫‪≥ cos‬‬ ‫‪≥ cos‬‬
‫‪12‬‬ ‫‪12‬‬ ‫‪12‬‬ ‫‪12‬‬
‫ﻟﻜﻦ ‪:‬‬
‫‪√3 + 1 √3 − 1‬‬ ‫‪√3 − 1‬‬ ‫‪√3 + 1‬‬
‫≥‬ ‫‪≥−‬‬ ‫‪≥−‬‬
‫‪2‬‬ ‫‪2‬‬ ‫‪2‬‬ ‫‪2‬‬

‫𝟳𝟰𝟭‬

‫‪http ://tinyurl.com/Malki1718‬‬ ‫‪0‬‬


‫‪ .2.VI‬اﻟﺸﲁ اﳌﺜﻠﱻ ﻟﻌﺪد ﻣﺮﻛﺐ‬
‫‪.‬‬
‫ﻣﻨﻪ ﻧﺴﺘﻨﺘﺞ ّ‬
‫أن ‪:‬‬

‫‪Ŕž Ŧ Ľ‬‬
‫‪√3 + 1‬‬ ‫‪√3 − 1‬‬ ‫‪√3 − 1‬‬ ‫‪√3 + 1‬‬
‫‪e−‬‬ ‫‪/‬‬ ‫=‬ ‫‪−ı‬‬ ‫‪,‬‬ ‫‪e‬‬ ‫‪/‬‬ ‫=‬ ‫‪+ı‬‬
‫‪2‬‬ ‫‪2‬‬ ‫‪2‬‬ ‫‪2‬‬
‫‪√3 − 1‬‬ ‫‪√3 + 1‬‬ ‫‪√3 + 1‬‬ ‫‪√3 − 1‬‬

‫‪œǃ‬‬
‫‪e−‬‬ ‫‪/‬‬ ‫‪=−‬‬ ‫‪−ı‬‬ ‫‪,‬‬ ‫‪e‬‬ ‫‪/‬‬ ‫‪=−‬‬ ‫‪+ı‬‬
‫‪2‬‬ ‫‪2‬‬ ‫‪2‬‬ ‫‪2‬‬
‫و ﻣﻨﻪ اﳉﺪول اﻟﺘﺎﱄ ‪:‬‬

‫‪Ŀ‬‬
‫𝜃‬ ‫𝜃 ‪cos‬‬ ‫𝜃 ‪sin‬‬

‫𝜋‬ ‫‪√3 + 1‬‬ ‫‪√3 − 1‬‬


‫‪12‬‬ ‫‪2‬‬ ‫‪2‬‬
‫𝜋‪5‬‬ ‫‪√3 − 1‬‬ ‫‪√3 + 1‬‬
‫‪12‬‬ ‫‪2‬‬ ‫‪2‬‬
‫𝜋‪7‬‬ ‫‪√3 − 1‬‬ ‫‪√3 + 1‬‬
‫‪−‬‬
‫‪12‬‬ ‫‪2‬‬ ‫‪2‬‬
‫𝝅𝟏𝟏‬ ‫𝟏 ‪√𝟑 +‬‬ ‫𝟏 ‪√𝟑 −‬‬
‫‪−‬‬
‫𝟐𝟏‬ ‫𝟐‬ ‫𝟐‬
‫■‬

‫‬ ‫‪𝟖𝟕.‬‬ ‫‪.‬‬

‫‪x−e‬‬ ‫‪x − e−‬‬ ‫ﻟﻴﻜﻦ 𝜃 ﻋﺪد ًا ﺣﻘﻴﻘﻴﺎ‪ .‬أﺛﺒﺖ ّ‬


‫أن ‪= x − 2x cos 𝜃 + 1 :‬‬ ‫•‬ ‫‪1‬‬
‫‪.f(x) = x + 1‬‬ ‫‪ •2‬ﻟﺘﻜﻦ ‪ f‬داﻟﺔ ﻛﺜﲑ اﳊﺪود اﳌﻌﺮﻓﺔ ﻛﲈ ﻳﲇ ‪:‬‬
‫‪ņőƱŀ Ţ‬‬
‫ﻛﺜﲑ ْي ﺣﺪود ﻣﻦ اﻟﺪرﺟﺔ اﻟﺜﺎﻧﻴﺔ ﺑﻤﻌﺎﻣﻼت‬ ‫ﺛﻢ اﺳﺘﻨﺘﺞ ﲢﻠﻴ ً‬
‫ﻼ ﻟﻠﺪاﻟﺔ ‪ f‬ﻋﲆ ﺷﻜﻞ ﺟﺪاء َ‬ ‫أوﺟﺪ اﳉﺬور اﻟﺮاﺑﻌﺔ ﻟﻠﻌﺪد ‪ّ −1‬‬
‫ﺣﻘﻴﻘﻴﺔ‪.‬‬
‫‪ •3‬ﻟﻴﻜﻦ ‪ z‬ﻋﺪد ًا ﻣﺮﻛﺒﺎ ﺑﺤﻴﺚ ‪.1 + z + z = 0‬‬

‫‪VI‬‬
‫أن ‪ُ z‬ﳛﻘﻖ ‪) z = 1‬أي ّ‬
‫أن ‪ z‬ﺟﺬر إﺛﻨﺎ َﻋ َﴩي ﻟﻠﻮﺣﺪة(‬ ‫أﺛﺒﺖ ّ‬

‫‪ •1‬ﺑﺎﻟﻨﴩ و اﻟﺘﺒﺴﻴﻂ ﻧﺼﻞ إﱃ اﻟﻨﺘﻴﺠﺔ ‪:‬‬ ‫ﺍﳊﻞّ‪.‬‬


‫‪ƕ‬‬

‫‪x−e‬‬ ‫‪x−‬‬ ‫‪e−‬‬ ‫‪=x − e +‬‬ ‫‪e−‬‬ ‫‪x+e‬‬ ‫‪e−‬‬ ‫‪= x − 2x cos 𝜃 + 1‬‬

‫‪ •2‬ﻟﺪﻳﻨﺎ ‪:‬‬
‫‪z =1 ⟺ z =1‬‬ ‫‪ z = ±ı‬أو ‪ z = −1 ⟺ z = ±1‬أو‬
‫‪ũŏ‬‬

‫إذن‪ ،‬اﳉﺬور اﻟﺮاﺑﻌﺔ ﻟﻠﻮﺣﺪة ﻫﻲ ‪. 1, −1, ı, −ı :‬‬


‫ﻹﳚﺎد اﳉﺬور اﻟﺮاﺑﻌﺔ ﻟﻠﻌﺪد ‪ ، −1‬ﻳﻜﻔﻲ ﻣﻌﺮﻓﺔ إﺣﺪاﻫﺎ و ﴐﲠﺎ ﺑﺎﳉﺬور اﻟﺮاﺑﻌﺔ ﻟﻠﻮﺣﺪة‪.‬‬
‫ﻟﻜﻦ ‪ −1 = e = e−‬إذن إﺣﺪى اﳉﺬور اﻟﺮاﺑﻌﺔ ﻟﻠﻌﺪد ‪ −1‬ﻫﻲ ‪ e /‬ﻣﻨﻪ ﻧﺴﺘﻨﺘﺞ ّ‬
‫أن اﳉﺬور اﻟﺮاﺑﻌﺔ ﻟﻠﻌﺪد‬
‫أن ‪ı = e /‬‬ ‫‪ −1‬ﻫﻲ ‪ e / × 1, e / × −1, e / × ı, e / × −ı‬أي ‪ e / , −e / , ıe / , −ıe /‬و ﺑﲈ ّ‬
‫ﻓﺈن ﻫﺬه اﳉﺬور ﺗُﻜﺘﺐ ‪ e / , e / − , e / + / , e / − / :‬و ﺑﻌﺪ اﻟﺘﺒﺴﻴﻂ ﻧﺠﺪ ‪:‬‬ ‫و ‪ّ −ı = e− /‬‬
‫‪e‬‬ ‫‪/‬‬ ‫‪, e−‬‬ ‫‪/‬‬ ‫‪,e‬‬ ‫‪/‬‬ ‫‪, e−‬‬ ‫‪/‬‬

‫𝟴𝟰𝟭‬

‫‪http ://tinyurl.com/Malki1718‬‬ ‫‪0‬‬


‫‪ .VI‬ﲤﺎرﻳﻦ ﺗﻄﺒﻴﻘﻴﺔ‬
‫‪.‬‬
‫ﺟﺬور ﻛﺜﲑ اﳊﺪود ‪ f‬ﻫﻲ ﺑﺎﻟﺘﺤﺪﻳﺪ اﳉﺬور اﻟﺮاﺑﻌﺔ ﻟﻠﻌﺪد ‪ −1‬و ﺑﺎﻟﺘﺎﱄ ﻧﺴﺘﻨﺘﺞ اﻟﺘﺤﻠﻴﻞ اﻵﰐ ﻟﻪ ﻋﲆ ﺷﻜﻞ ﺟﺪاء ﻋﻮاﻣﻞ‬
‫‪f(x) = x + 1 = x − e /‬‬ ‫‪x − e− /‬‬ ‫‪x−e /‬‬ ‫ﻣﻦ اﻟﺪرﺟﺔ اﻷوﱃ ‪x − e− / :‬‬

‫‪Ŕž Ŧ Ľ‬‬
‫𝜋‬ ‫𝜋‪3‬‬
‫‪f(x) = x − 2x cos‬‬ ‫‪+1‬‬ ‫‪x − 2x cos‬‬ ‫‪+1‬‬ ‫)ﻣﺮﺗﲔ( ﻧﺠﺪ ‪:‬‬
‫اﻷول ّ‬
‫و ﺑﺘﻄﺒﻴﻖ ﻧﺘﻴﺠﺔ اﻟﺴﺆال ّ‬
‫‪4‬‬ ‫‪4‬‬

‫‪œǃ‬‬
‫‪f(x) = x + 1 = x − √2x + 1‬‬ ‫‪x + √2x + 1‬‬ ‫أي‬
‫ﻳﻤﻜﻦ اﳊﺼﻮل ﻋﲆ ﻫﺬه اﻟﻨﺘﻴﺠﺔ ﺑﺎﺗّﺒﺎع ﻃﺮﻳﻘﺔ ﺇﲤﺎﻡ ﺍﳌﺮﺑﻊ ‪:‬‬ ‫ﻣﻼﺣﻈﺔ ‪⧏ : 23‬‬

‫‪x + 1 = x + 2x + 1 − 2x = x + 1‬‬ ‫‪− √2x‬‬ ‫‪= x + 1 − √2x‬‬ ‫‪x + 1 + √2x‬‬

‫‪Ŀ‬‬
‫⧐‬

‫ﻣﻼﺣﻈﺔ ‪ ⧏ : 24‬ﰲ ﳎﻤﻮﻋﺔ اﻷﻋﺪاد اﳌﺮﻛﺒﺔ‪ ،‬ﻛﺜﲑات اﳊﺪود ﻏﲑ اﻟﻘﺎﺑﻠﺔ ﻟﻠﺘﺤﻠﻴﻞ ﻫﻲ ﻛﺜﲑات اﳊﺪود اﻟﺜﺎﺑﺘﺔ و‬
‫ﻛﺜﲑات اﳊﺪود ﻣﻦ اﻟﺪرﺟﺔ اﻷوﱃ‪ ،‬و ﰲ ﳎﻤﻮﻋﺔ اﻷﻋﺪاد اﳊﻘﻴﻘﻴﺔ‪ ،‬ﻛﺜﲑات اﳊﺪود ﻏﲑ اﻟﻘﺎﺑﻠﺔ ﻟﻠﺘﺤﻠﻴﻞ ﻫﻲ ﻛﺜﲑات‬
‫اﳊﺪود اﻟﺜﺎﺑﺘﺔ ‪ ،‬ﻛﺜﲑات اﳊﺪود ﻣﻦ اﻟﺪرﺟﺔ اﻷوﱃ و ﻛﺜﲑات اﳊﺪود ﻣﻦ اﻟﺪرﺟﺔ اﻟﺜﺎﻧﻴﺔ اﻟﺘﻲ ﳑ ّﻴﺰﻫﺎ ﺳﺎﻟﺐ ﲤﺎﻣ ًﺎ )ﺣﺘﻰ‬
‫أي ﺟﺬر ﺣﻘﻴﻘﻲ و ﻣﺜﺎل ذﻟﻚ اﻟﺪاﻟﺔ ‪ f‬اﻟﺘﻲ ﺟﺬورﻫﺎ ﻛﻠﻬﺎ ﻣﺮﻛﺒﺔ ﻟﻜﻨﻬﺎ ﻗﺎﺑﻠﺔ ﻟﻠﺘﺤﻠﻴﻞ إﱃ ﺟﺪاء ﻋﻮاﻣﻞ ﻣﻦ‬
‫و ﱂ ﻳﻜﻦ ﳍﺎ ّ‬
‫⧐‬ ‫اﻟﺪرﺟﺔ اﻟﺜﺎﻧﻴﺔ(‪.‬‬
‫ّ‬
‫)ﻷن ‪.(z = 1 ⟹ 1 + z + z = 1 + 1 + 1 = 3 ≠ 0 :‬‬ ‫ﻧﻼﺣﻆ ّأوﻻً ّ‬
‫أن ‪z ≠ 1‬‬ ‫‪3‬‬
‫•‬

‫‪1+z +z =0‬‬ ‫ﻧﻌﻠﻢ ّ‬


‫أن ‪:‬‬
‫‪z+z +z =0‬‬ ‫ﺑﴬب ﻃﺮﰲ اﳌﺴﺎواة ﺑﺎﻟﻌﺪد ‪: z‬‬
‫‪z +z +z‬‬ ‫‪=0‬‬ ‫ﺑﴬب ﻃﺮﰲ اﳌﺴﺎواة ﺑﺎﻟﻌﺪد ‪ z‬ﻣﺮة ﺛﺎﻧﻴﺔ ‪:‬‬
‫‪z +z +z‬‬ ‫‪=0‬‬ ‫و ﻣﺮة ﺛﺎﻟﺜﺔ ‪:‬‬

‫‪.‬‬ ‫‪z =0‬‬ ‫اﻵن‪ ،‬ﻧﺠﻤﻊ ﻫﺬه اﳌﺴﺎوﻳﺎت ﻃﺮﻓ ًﺎإﱃ ﻃﺮف ﻟﻨﺠﺪ ‪:‬‬
‫=‬
‫‪ņőƱŀ Ţ‬‬
‫‪1−z‬‬
‫ﻣﻨﻪ‬ ‫= ‪z‬‬ ‫‪=0‬‬ ‫و ﺑﲈ ّ‬
‫أن ‪ ، z ≠ 1‬ﻓﺒﺈﻣﻜﺎﻧﻨﺎ ﺗﻄﺒﻴﻖ اﻟﺼﻴﻐﺔ اﻟﻌﺎﻣﺔ ﻟﻠﻤﺠﻤﻮع اﳉﺰﺋﻲ ﳌﺘﺘﺎﻟﻴﺔ ﻫﻨﺪﺳﻴﺔ ‪:‬‬
‫=‬
‫‪1−z‬‬
‫‪.z = 1‬‬
‫ﻣﻼﺣﻈﺔ ‪ ⧏ : 25‬أﺛﺒﺘﻨﺎ ﻫﻨﺎ أﻧﻪ إذا ﻛﺎن ‪ z‬ﺟﺬرا ﻟﻜﺜﲑ اﳊﺪود ‪ X + X + 1‬ﻓﺈن ‪ z‬ﺟﺬر ﻟﻜﺜﲑ اﳊﺪود ‪X − 1‬‬
‫ّ‬ ‫ً‬
‫‪VI‬‬

‫أن ‪ X + X + 1‬ﻳﻘﺴﻢ ‪ ، X − 1‬و ﺑﺎﻟﺘﺤﺪﻳﺪ ﻟﺪﻳﻨﺎ ‪:‬‬ ‫و ﻫﺬا ﻳﻌﻨﻲ ّ‬


‫‪X‬‬ ‫‪−1= X +X +1‬‬ ‫‪X −1‬‬

‫و ﻧﺤﺼﻞ ﻋﻠﻴﻪ ﺑﺎﻟﺘﻌﻮﻳﺾ ‪ t = X‬ﰲ اﳌﺘﻄﺎﺑﻘﺔ ‪ ،t − 1 = (t − 1) t + t + 1‬ﻛﲈ ﻳﻤﻜﻦ ﺗﻌﻤﻴﻢ ﻫﺬه اﻟﻨﺘﻴﺠﺔ ‪:‬‬
‫‪ƕ‬‬

‫‪∀n ∈ ℕ , X‬‬ ‫‪−1= X‬‬ ‫‪+X +1‬‬ ‫‪X −1‬‬


‫⧐‬

‫■‬
‫‪ũŏ‬‬

‫ﺗﻄﺒﻴﻘﺎﺕ ﺍﻷﻋﺪﺍﺩ ﺍﳌﺮﻛﺒﺔ‬ ‫‪3.VI‬‬

‫𝟵𝟰𝟭‬

‫‪http ://tinyurl.com/Malki1718‬‬ ‫‪0‬‬


‫‪ .3.VI‬ﺗﻄﺒﻴﻘﺎت اﻷﻋﺪاد اﳌﺮﻛﺒﺔ‬
‫‪.‬‬

‫‬ ‫‪𝟖𝟖.‬‬ ‫‪.‬‬

‫‪Ŕž Ŧ Ľ‬‬
‫أﺣﺴﺐ اﳉﺬور اﻟﱰﺑﻴﻌﻴﺔ ﻟﻸﻋﺪاد اﻟﺘﺎﻟﻴﺔ ‪:‬‬
‫‪24 − 10ı‬‬ ‫‪7‬‬ ‫‪7 + 24ı‬‬ ‫‪5‬‬ ‫‪3 + 4ı‬‬ ‫‪3‬‬ ‫‪1‬‬ ‫‪1‬‬

‫‪œǃ‬‬
‫•‬ ‫•‬ ‫•‬ ‫•‬

‫‪48 + 14ı‬‬ ‫•‬ ‫‪8‬‬ ‫‪3 − 4ı‬‬ ‫‪6‬‬


‫•‬ ‫‪8 − 6ı‬‬ ‫•‬ ‫‪4‬‬ ‫‪ı‬‬ ‫•‬ ‫‪2‬‬

‫‪Ŀ‬‬
‫ﺍﳊﻞّ‪ .‬ﺑﲈ أﻧﻨﺎ ﻧﻜﺮر ﻧﻔﺲ اﻟﻌﻤﻠﻴﺔ ﻋﺪة ﻣﺮات‪ ،‬ﻓﻤﻦ اﻷﻓﻀﻞ اﻟﺒﺤﺚ ﻋﻦ ﺻﻴﻐﺔ ﻋﺎﻣﺔ ﻟﻠﺠﺬور اﻟﱰﺑﻴﻌﻴﺔ ﺛﻢ ﺗﻄﺒﻴﻖ ﻫﺬه اﻟﺼﻴﻐﺔ‬
‫ﻋﲆ اﳊﺎﻻت اﳌﺪروﺳﺔ‪.‬‬
‫ﻟﻴﻜﻦ ‪ z = a + ıb‬ﻋﺪد ًا ﻣﺮﻛﺒﺎ ﻣﻊ ‪ a, b ∈ ℝ‬و ﻟﻴﻜﻦ ‪ 𝜁 = x + ıy‬ﺟﺬر ًا ﺗﺮﺑﻴﻌﻴﺎ ﻟﻪ ﻣﻊ ‪. x, y ∈ ℝ‬‬
‫= ‪ x + y‬ﻣﻨﻪ ‪ x − y + 2ıxy = a + ıb‬و‬ ‫ﻟﺪﻳﻨﺎ ‪ 𝜁 = z‬و |‪ ||𝜁|| = |z‬أي ‪ (x + ıy) = a + ıb‬و ‪a + b‬‬
‫⎧‬ ‫‪x −y =a‬‬
‫⎪‬
‫⎪‬
‫‪.‬‬ ‫‪2xy = 2b‬‬ ‫‪ x + y = a + b‬و ﺑﺎﻟﺘﺎﱄ ‪:‬‬
‫⎨‬
‫⎪‬
‫⎪‬
‫‪⎩x + y = a + b‬‬
‫‪a+‬‬ ‫‪a +b‬‬
‫‪.x = ±‬‬ ‫‪ 2x = a +‬أي‬ ‫‪a +b‬‬ ‫ﺑﺠﻤﻊ اﳌﻌﺎدﻟﺘﲔ اﻷوﱃ و اﻟﺜﺎﻟﺜﺔ ﻳﻨﺘﺞ‬
‫‪2‬‬
‫‪−a +‬‬ ‫‪a +b‬‬
‫‪.y = ±‬‬ ‫ﺑﻄﺮح اﳌﻌﺎدﻟﺔ اﻷوﱃ ﻣﻦ اﻟﺜﺎﻟﺜﺔ ﻳﻨﺘﺞ ‪ 2y = −a + a + b‬أي‬
‫‪2‬‬
‫ﻧﺨﺘﺎر ‪ x‬و ‪ y‬ﻣﻊ ﻣﺮاﻋﺎة اﻟﴩط ‪ xy :‬و ‪ b‬ﳍﲈ ﻧﻔﺲ اﻹﺷﺎرة‪.‬‬
‫ﻟﺪﻳﻨﺎ ‪ z = 1‬أي ‪ a = 1‬و ‪ b = 0‬ﻣﻨﻪ ‪ x = ±1‬و ‪ y = ±0‬و ﺑﺎﻟﺘﺎﱄ ﻓﺎﳉﺬران اﻟﱰﺑﻴﻌﻴﺎن ﻟﻠﻌﺪد ‪ z = 1‬ﳘﺎ ‪𝜁 = −1‬‬ ‫•‬ ‫‪1‬‬
‫و‪.𝜁 = 1‬‬
‫‪1‬‬ ‫‪1‬‬
‫أن ‪ّ b > 0‬‬
‫ﻓﺈن ‪ x‬و ‪ y‬ﻣﻦ ﻧﻔﺲ اﻹﺷﺎرة و ﺑﺎﻟﺘﺎﱄ‬ ‫‪ . y = ±‬ﺑﲈ ّ‬ ‫‪x = ±‬و‬ ‫‪ z = ı •2‬أي ‪ a = 0‬و ‪ b = 1‬ﻣﻨﻪ‬
‫‪√2‬‬ ‫‪√2‬‬
‫‪−1 − ı‬‬ ‫‪1+ı‬‬
‫‪ņőƱŀ Ţ‬‬
‫= 𝜁‪.‬‬ ‫= 𝜁و‬ ‫اﳉﺬران اﻟﱰﺑﻴﻌﻴﺎن ﻟﻠﻌﺪد ‪ ı‬ﳘﺎ‬
‫‪√2‬‬ ‫‪√2‬‬
‫‪ z = 3 + 4ı •3‬أي ‪ a = 3‬و ‪ b = 4‬ﻣﻨﻪ ‪ x = ±2‬و ‪ . y = ±1‬ﻟﻜﻦ ‪ b > 0‬إذن ‪ x‬و ‪ y‬ﳍﲈ ﻧﻔﺲ اﻹﺷﺎرة و ﺑﺎﻟﺘﺎﱄ‬
‫ﻓﺎﳉﺬران اﻟﱰﺑﻴﻌﻴﺎن ﻟﻠﻌﺪد ‪ 3 + 4ı‬ﳘﺎ ‪ 𝜁 = 2 + ı‬و ‪. 𝜁 = −2 − ı‬‬

‫‪VI‬‬
‫أن ‪ّ b < 0‬‬
‫ﻓﺈن ‪ x‬و ‪ y‬ﻣﻦ ﻧﻔﺲ اﻹﺷﺎرة إذن‬ ‫‪ z = 8 − 6ı •4‬أي ‪ a = 8‬و ‪ b = −6‬ﻣﻨﻪ ‪ x = ±3‬و ‪ y = ±1‬و ﺑﲈ ّ‬
‫ﻓﺎﳉﺬران اﻟﱰﺑﻴﻌﻴﺎن ﻟﻠﻌﺪد ‪ 8 − 6ı‬ﳘﺎ ‪ 𝜁 = 3 − ı‬و ‪. 𝜁 = −3 + ı‬‬
‫أن ‪ّ b > 0‬‬
‫ﻓﺈن ‪ x‬و ‪ y‬ﻣﻦ ﻧﻔﺲ اﻹﺷﺎرة و ﺑﺎﻟﺘﺎﱄ‬ ‫‪ z = 7 + 24ı •5‬أي ‪ a = 7‬و ‪ b = 24‬ﻣﻨﻪ ‪ x = ±4‬و ‪ y = ±3‬و ﺑﲈ ّ‬
‫ﻓﺎﳉﺬران اﻟﱰﺑﻴﻌﻴﺎن ﻟﻠﻌﺪد ‪ 7 + 24ı‬ﳘﺎ ‪ 𝜁 = 4 + 3ı‬و ‪. 𝜁 = −4 − 3ı‬‬
‫‪ƕ‬‬

‫‪ z = 3 − 4ı •6‬أي ‪ a = 3‬و ‪ b = −4‬ﻣﻨﻪ ‪ x = ±2‬و ‪ .y = ±1‬ﻟﻜﻦ ‪ b < 0‬إذن ‪ x‬و ‪ y‬ﳍﲈ ﻧﻔﺲ اﻹﺷﺎرة و ﺑﺎﻟﺘﺎﱄ‬
‫ﻓﺎﳉﺬران اﻟﱰﺑﻴﻌﻴﺎن ﻟﻠﻌﺪد ‪ 3 − 4ı‬ﳘﺎ ‪ 𝜁 = 2 − ı‬و ‪. 𝜁 = −2 + ı‬‬
‫‪ũŏ‬‬

‫ﻳﻤﻜﻦ أﻳﻀ ًﺎ ﻣﻼﺣﻈﺔ ّ‬


‫أن ‪ 3 − 4ı = 3 + 4ı‬و ﺑﺎﻟﺘﺎﱄ ﻓﺎﳉﺬران اﻟﱰﺑﻴﻌﻴﺎن ﻟﻠﻌﺪد ‪ 3 − 4ı‬ﳘﺎ ﻣﺮاﻓ َﻘﺎ اﳉﺬرﻳﻦ اﻟﱰﺑﻴﻌﻴﲔ‬
‫ﻟﻠﻌﺪد ‪. 3 + 4ı‬‬
‫أن ‪ّ b < 0‬‬
‫ﻓﺈن ‪ x‬و ‪ y‬ﻣﻦ ﻧﻔﺲ اﻹﺷﺎرة و‬ ‫‪ z = 24 − 10ı •7‬أي ‪ a = 24‬و ‪ b = −10‬ﻣﻨﻪ ‪ x = ±5‬و ‪ y = ±1‬و ﺑﲈ ّ‬
‫ﺑﺎﻟﺘﺎﱄ ﻓﺎﳉﺬران اﻟﱰﺑﻴﻌﻴﺎن ﻟﻠﻌﺪد ‪ 24 − 10ı‬ﳘﺎ ‪ 𝜁 = 5 − ı‬و ‪. 𝜁 = −5 + ı‬‬
‫‪ z = 48 + 14ı •8‬أي ‪ a = 48‬و ‪ b = 14‬ﻣﻨﻪ ‪ x = ±7‬و ‪ . y = ±1‬ﻟﻜﻦ ‪ b > 0‬إذن ‪ x‬و ‪ y‬ﳍﲈ ﻧﻔﺲ اﻹﺷﺎرة و ﺑﺎﻟﺘﺎﱄ‬
‫ﻓﺎﳉﺬران اﻟﱰﺑﻴﻌﻴﺎن ﻟﻠﻌﺪد ‪ 48 + 14ı‬ﳘﺎ ‪ 𝜁 = 7 + ı‬و ‪. 𝜁 = −7 − ı‬‬

‫𝟬𝟱𝟭‬

‫‪http ://tinyurl.com/Malki1718‬‬ ‫‪0‬‬


‫‪ .VI‬ﲤﺎرﻳﻦ ﺗﻄﺒﻴﻘﻴﺔ‬
‫‪.‬‬
‫■‬ ‫ﲤﺮﻳﻦ ﺗﺪرﻳﺒﻲ ‪ :‬أﻋﺪ اﻟﺘﻤﺮﻳﻦ دون اﺳﺘﻌﲈل اﻟﺼﻴﻎ اﻟﺴﺎﺑﻘﺔ ﻟﻠﻌﺪدﻳﻦ ‪ x‬و ‪ y‬أي ﻣﻊ اﺗﺒﺎع اﻟﻄﺮﻳﻘﺔ اﻟﻌﺎﻣﺔ ﰲ ﻛﻞ ﻣﺮة‪.‬‬

‫‪Ŕž Ŧ Ľ‬‬
‫ﻋﲔ اﳉﺬرﻳﻦ اﻟﱰﺑﻴﻌﻴﲔ ﻟﻜﻞ ﻣﻦ اﻷﻋﺪاد اﻟﺘﺎﻟﻴﺔ ‪:‬‬
‫‬ ‫‪.‬‬
‫𝟗𝟖‬ ‫‪.‬‬

‫‪œǃ‬‬
‫ﱢ‬
‫‪2ı − 1‬‬ ‫‪20 − 21ı‬‬ ‫•‬ ‫‪3‬‬ ‫‪1 + ı√3‬‬ ‫•‬ ‫‪2‬‬ ‫‪8 − 6ı‬‬ ‫•‬‫‪1‬‬
‫•‬ ‫‪4‬‬
‫‪2+ı‬‬

‫‪Ŀ‬‬
‫‪ •1‬ﻟﺪﻳﻨﺎ ‪:‬‬ ‫ﺍﳊﻞّ‪.‬‬

‫)‪8 − 6ı = 9 − 1 − 2 × 3ı = 3 − 2 × 3 × ı + ı = (3 − ı‬‬

‫و ﺑﺎﻟﺘﺎﱄ‪ ،‬ﻓﺎﳉﺬران اﻟﱰﺑﻴﻌﻴﺎن ﻟﻠﻌﺪد ‪ 8 − 6ı‬ﳘﺎ ‪ 3 − ı‬و ‪. −3 + ı‬‬


‫‪ •2‬ﻟﺪﻳﻨﺎ ‪:‬‬

‫‪1‬‬ ‫‪√3‬‬ ‫‪/‬‬ ‫‪/‬‬ ‫‪√3 1‬‬ ‫‪√6‬‬ ‫‪√2‬‬


‫‪1 + ı√3 = 2‬‬ ‫‪+ı‬‬ ‫‪= 2e‬‬ ‫‪= √2e‬‬ ‫=‬ ‫‪√2‬‬ ‫‪+ ı‬‬ ‫=‬ ‫‪+ı‬‬
‫‪2‬‬ ‫‪2‬‬ ‫‪2‬‬ ‫‪2‬‬ ‫‪2‬‬ ‫‪2‬‬

‫‪6‬‬ ‫‪2‬‬ ‫‪6‬‬ ‫‪2‬‬


‫و ﺑﺎﻟﺘﺎﱄ ﻓﺎﳉﺬران اﻟﱰﺑﻴﻌﻴﺎن ﻟﻠﻌﺪد ‪ 1 + ı√3‬ﳘﺎ √ ‪ √ + ı‬و √ ‪. − √ − ı‬‬
‫‪2‬‬ ‫‪2‬‬ ‫‪2‬‬ ‫‪2‬‬
‫‪ •3‬ﻟﺪﻳﻨﺎ ‪:‬‬

‫‪1‬‬ ‫‪1‬‬ ‫‪1‬‬ ‫‪7 − 3ı‬‬


‫= ‪20 − 21ı‬‬ ‫= )‪(40 − 42ı‬‬ ‫)‪7 − 2 × 7 × 3ı + (3ı‬‬ ‫=‬ ‫= )‪(7 − 3ı‬‬
‫‪2‬‬ ‫‪2‬‬ ‫‪2‬‬ ‫‪√2‬‬

‫‪7 2‬‬ ‫‪3 2 7 2‬‬ ‫‪3 2‬‬


‫و ﺑﺎﻟﺘﺎﱄ ﻓﺎﳉﺬران اﻟﱰﺑﻴﻌﻴﺎن ﻟﻠﻌﺪد ‪ 20 − 21ı‬ﳘﺎ √ ‪ √ − ı‬و √ ‪. − √ + ı‬‬
‫‪2‬‬ ‫‪2‬‬ ‫‪2‬‬ ‫‪2‬‬
‫‪ņőƱŀ Ţ‬‬
‫‪ •4‬ﻟﺪﻳﻨﺎ ‪:‬‬

‫‪2ı − 1 (2ı − 1) (2 − ı) 4ı + 2 − 2 + ı‬‬ ‫‪/‬‬ ‫‪/‬‬ ‫‪√2‬‬ ‫‪√2‬‬


‫=‬ ‫=‬ ‫‪=ı=e‬‬ ‫‪= e‬‬ ‫=‬ ‫‪+ı‬‬
‫‪VI‬‬

‫‪2+ı‬‬ ‫‪2 +1‬‬ ‫‪5‬‬ ‫‪2‬‬ ‫‪2‬‬

‫‪2‬‬ ‫‪2‬‬ ‫‪2‬‬ ‫‪2‬‬ ‫‪2ı − 1‬‬


‫ﳘﺎ √ ‪ √ + ı‬و √ ‪. − √ − ı‬‬ ‫و ﺑﺎﻟﺘﺎﱄ ﻓﺎﳉﺬران اﻟﱰﺑﻴﻌﻴﺎن ﻟﻠﻌﺪد‬
‫‪2‬‬ ‫‪2‬‬ ‫‪2‬‬ ‫‪2‬‬ ‫‪2+ı‬‬
‫■‬
‫‪ƕ‬‬

‫= ‪.z‬‬
‫‪4‬‬
‫‪ z‬ﻋﺪد ﻣﺮﻛﺐ ﺣﻴﺚ ‪:‬‬
‫‬ ‫‪.‬‬
‫𝟎𝟗‬ ‫‪.‬‬
‫‪1 + ı√3‬‬
‫‪ũŏ‬‬

‫‪ •1‬ﱢ‬
‫ﻋﲔ اﻟﻄﻮﻳﻠﺔ و ﻋﻤﺪة ﻟﻠﻌﺪد ‪. z‬‬
‫‪ •2‬أﺣﺴﺐ ﺑﻄﺮﻳﻘﺘﲔ ﳐﺘﻠﻔﺘﲔ اﳉﺬرﻳﻦ اﻟﱰﺑﻴﻌﻴﲔ ﻟﻠﻌﺪد ‪. z‬‬

‫𝟭𝟱𝟭‬

‫‪http ://tinyurl.com/Malki1718‬‬ ‫‪0‬‬


‫‪ .3.VI‬ﺗﻄﺒﻴﻘﺎت اﻷﻋﺪاد اﳌﺮﻛﺒﺔ‬
‫‪.‬‬
‫‪ •1‬ﻟﺪﻳﻨﺎ ‪:‬‬ ‫ﺍﳊﻞّ‪.‬‬

‫‪Ŕž Ŧ Ľ‬‬
‫‪4‬‬ ‫‪4 1 − ı√3‬‬ ‫‪4 − 4ı√3‬‬ ‫‪1‬‬ ‫‪√3‬‬
‫=‪z‬‬ ‫=‬ ‫=‬ ‫‪= 1 − ı√3 = 2‬‬ ‫‪−ı‬‬ ‫‪= 2e−‬‬ ‫‪/‬‬
‫‪1 + ı√3‬‬ ‫‪4‬‬ ‫‪2‬‬ ‫‪2‬‬

‫‪œǃ‬‬
‫‪1 + √3‬‬
‫𝜋‬
‫و ﺑﺎﻟﺘﺎﱄ ‪ |z| = 2 :‬و )𝜋‪. arg (z) = − (mod 2‬‬
‫‪3‬‬

‫‪Ŀ‬‬
‫‪ √2e−‬و‬ ‫‪/‬‬ ‫‪ z = √2e−‬و ﺑﺎﻟﺘﺎﱄ ﻓﺎﳉﺬران اﻟﱰﺑﻴﻌﻴﺎن ﻟﻠﻌﺪد ‪ z‬ﳘﺎ‬ ‫‪/‬‬ ‫‪ •2‬ﺑﺎﺳﺘﺨﺪام اﻟﺸﻜﻞ اﳌﺜﻠﺜﻲ ‪ :‬ﻟﺪﻳﻨﺎ‬
‫‪6‬‬ ‫‪2‬‬ ‫‪6‬‬ ‫‪2‬‬
‫‪ −√2e−‬أي ‪ √ − √ ı‬و ‪. − √ + √ ı‬‬ ‫‪/‬‬
‫‪2‬‬ ‫‪2‬‬ ‫‪2‬‬ ‫‪2‬‬
‫ﺑﺎﺳﺘﺨﺪام اﻟﺸﻜﻞ اﳉﱪي ‪ :‬ﻟﻴﻜﻦ ‪ x + ıy‬ﺟﺬر ًا ﺗﺮﺑﻴﻌﻴﺎ ﻟﻠﻌﺪد ‪ z‬ﻣﻊ ‪ . x, y ∈ ℝ‬ﻟﺪﻳﻨﺎ ‪:‬‬
‫⎧‬ ‫‪x −y =1‬‬
‫⎪‬
‫⟺ ‪(x + ıy) = z ⟺ x − y + 2ıxy = 1 − ı√3‬‬ ‫‪2xy = −√3‬‬
‫⎨‬
‫⎪‬
‫‪⎩x + y = 2‬‬
‫‪3‬‬ ‫‪6‬‬
‫ﺑﺠﻤﻊ اﳌﻌﺎدﻟﺘﲔ اﻷوﱃ و اﻟﺜﺎﻟﺜﺔ ﻳﻨﺘﺞ ‪ 2x = 3 :‬أي √ ‪. x = ± √ = ±‬‬
‫‪√2‬‬ ‫‪2‬‬
‫‪2‬‬ ‫‪6‬‬
‫ُﻌﻮض ﰲ اﳌﻌﺎدﻟﺔ اﻟﺜﺎﻧﻴﺔ ﻟﻨﺠﺪ √ ‪. y = −‬‬ ‫ﻧﺨﺘﺎر √ = ‪ x‬ﻣﺜ ً‬
‫ﻼوﻧ ّ‬
‫‪2‬‬ ‫‪2‬‬
‫‪6‬‬ ‫‪2‬‬ ‫‪6‬‬ ‫‪2‬‬
‫ﰲ اﻷﺧﲑ‪ ،‬اﳉﺬران اﻟﱰﺑﻴﻌﻴﺎن ﻟﻠﻌﺪد ‪ z‬ﳘﺎ ‪ √ − √ ı‬و ‪. − √ + √ ı‬‬
‫‪2‬‬ ‫‪2‬‬ ‫‪2‬‬ ‫‪2‬‬
‫■‬

‫= ‪.z‬‬
‫𝜃 ‪1 − cos 𝜃 + ı sin‬‬
‫𝜃 ﻋﺪد ﺣﻘﻴﻘﻲ ﺣﻴﺚ [𝜋‪ 𝜃 ∈ ]𝜋, 2‬و ‪ z‬ﻋﺪد ﻣﺮﻛﺐ ﺣﻴﺚ ‪:‬‬
‫‬ ‫‪𝟗𝟏.‬‬ ‫‪.‬‬
‫𝜃 ‪1 + cos 𝜃 − ı sin‬‬
‫‪ņőƱŀ Ţ‬‬
‫‪ •1‬ﱢ‬
‫ﻋﲔ اﻟﻄﻮﻳﻠﺔ و ﻋﻤﺪة ﻟﻠﻌﺪد ‪. z‬‬
‫‪ •2‬أﺣﺴﺐ اﳉﺬرﻳﻦ اﻟﱰﺑﻴﻌﻴﲔ ﻟﻠﻌﺪد ‪. z‬‬

‫‪ •1‬ﻟﺪﻳﻨﺎ ‪:‬‬ ‫ﺍﳊﻞّ‪.‬‬


‫‪VI‬‬
‫‪1 − cos 𝜃 + ı sin 𝜃 1 − e−‬‬ ‫‪e−‬‬ ‫‪/‬‬ ‫‪e‬‬ ‫‪/‬‬ ‫‪− e−‬‬ ‫‪/‬‬
‫‪2ı sin‬‬ ‫𝜃‬ ‫‪/‬‬ ‫𝜃‬
‫=‪z‬‬ ‫=‬ ‫=‬ ‫=‬ ‫‪= ı tan‬‬ ‫‪=e‬‬ ‫‪tan‬‬
‫‪1 + cos 𝜃 − ı sin 𝜃 1 + e−‬‬ ‫‪e−‬‬ ‫‪/‬‬ ‫‪e‬‬ ‫‪/‬‬ ‫‪+ e−‬‬ ‫‪/‬‬ ‫‪2 cos‬‬ ‫‪2‬‬ ‫‪2‬‬
‫‪ƕ‬‬

‫𝜃‬ ‫𝜃‬
‫𝜋‬
‫∈ و ﺑﺎﻟﺘﺎﱄ ‪ tan < 0‬؛ ﻓﺤﺴﺐ اﳌﱪﻫﻨﺔ ‪ 8‬ﺻﻔﺤﺔ ‪ّ 20‬‬
‫ﻓﺈن ‪:‬‬ ‫𝜋‪,‬‬ ‫أن [𝜋‪ّ 𝜃 ∈ ]𝜋, 2‬‬
‫ﻓﺈن‬ ‫ﺑﲈ ّ‬
‫‪2‬‬ ‫‪2‬‬
‫‪2‬‬
‫𝜋‪𝜋 3‬‬ ‫𝜃‬
‫‪. arg (z) = 𝜋 +‬‬ ‫=‬ ‫)𝜋‪(mod 2‬‬ ‫و‬ ‫‪|z| = − tan‬‬
‫‪ũŏ‬‬

‫‪2‬‬ ‫‪2‬‬ ‫‪2‬‬


‫‪ •2‬ﻟﺪﻳﻨﺎ ‪:‬‬
‫𝜃‬ ‫‪/‬‬ ‫𝜃‬ ‫‪/‬‬ ‫‪1‬‬ ‫𝜃‬
‫‪z = − tan e‬‬ ‫=‬ ‫‪− tan e‬‬ ‫=‬ ‫)‪− tan (−1 + ı‬‬
‫‪2‬‬ ‫‪2‬‬ ‫‪2‬‬ ‫‪2‬‬

‫إذن ﻓﺎﳉﺬران اﻟﱰﺑﻴﻌﻴﺎن ﻟﻠﻌﺪد ‪ z‬ﳘﺎ ‪ (−1 + ı) − tan‬و ‪. (1 − ı) − tan‬‬


‫■‬

‫𝟮𝟱𝟭‬

‫‪http ://tinyurl.com/Malki1718‬‬ ‫‪0‬‬


‫‪ .VI‬ﲤﺎرﻳﻦ ﺗﻄﺒﻴﻘﻴﺔ‬
‫‪.‬‬

‫‪#‬‬ ‫‪.‬‬
‫𝟐𝟗‬ ‫‪.‬‬

‫‪Ŕž Ŧ Ľ‬‬
‫أوﺟﺪ اﳉﺬور اﻟﱰﺑﻴﻌﻴﺔ ﻟﻠﻌﺪدﻳﻦ اﻟﺘﺎﻟﻴﲔ ‪:‬‬

‫‪2‬‬ ‫‪7 + 24ı‬‬ ‫‪1‬‬

‫‪œǃ‬‬
‫)‪(a, b ∈ ℝ‬‬ ‫‪4ab + 2 a − b‬‬ ‫‪ı‬‬ ‫•‬ ‫•‬

‫‪ •1‬ﻟﻴﻜﻦ ‪ z = x + ıy‬ﺟﺬر ًا ﺗﺮﺑﻴﻌﻴﺎ ﻟﻠﻌﺪد ‪ . 7 + 24ı‬ﻟﺪﻳﻨﺎ ‪:‬‬ ‫ﺍﳊﻞّ‪.‬‬

‫‪Ŀ‬‬
‫⎧‬
‫⎪‬
‫= ||‪⎪ x + y = ||7 + 24ı‬‬ ‫‪7 + 24 = 25‬‬
‫⟺ ‪z = x − y + 2ıxy = 7 + 24ı‬‬ ‫‪x −y =7‬‬
‫⎨‬
‫⎪‬
‫⎪‬
‫⎩‬ ‫‪2xy = 24‬‬

‫ﺑﺠﻤﻊ اﳌﻌﺎدﻟﺘﲔ اﻷوﱃ و اﻟﺜﺎﻧﻴﺔ ﻳﻨﺘﺞ ‪ 2x = 25 + 7‬ﻣﻨﻪ ‪ . x = ±4‬ﻧﺄﺧﺬ ﻣﺜ ً‬


‫ﻼ ‪ x = 4‬ﻓﻴﻜﻮن ﻣﻦ اﳌﻌﺎدﻟﺔ اﻟﺜﺎﻟﺜﺔ‬
‫‪.y = 3‬‬
‫إذن اﳉﺬرﻳﻦ اﻟﱰﺑﻴﻌﻴﲔ ﻟﻠﻌﺪد ‪ 7 + 24ı‬ﳘﺎ ‪ 4 + 3ı‬و )‪. − (4 + 3ı‬‬
‫ﻣﻼﺣﻈﺔ ‪ ⧏ : 26‬ﻣﻦ اﳌﻤﻜﻦ‪ ،‬ﰲ ﺑﻌﺾ اﻷﺣﻴﺎن‪ ،‬إﳚﺎد ﺟﺬر ﺗﺮﺑﻴﻌﻲ ﻟﻌﺪد ﻣﺮﻛﺐ دون اﻟﻘﻴﺎم ﺑﻜﻞ ﻫﺬه اﳊﺴﺎﺑﺎت‬
‫ﻼ ‪ :‬ﻹﳚﺎد ﺟﺬر ﺗﺮﺑﻴﻌﻲ ﻟﻠﻌﺪد ‪ ،7 + 24ı‬ﻧﻨﻈﺮ إﱃ ﺟﺰﺋﻪ اﻟﺘﺨﻴﲇ ‪ 24‬و‬ ‫و ذﻟﻚ ﺑﺎﻟﻨﻈﺮ إﱃ اﳉﺰء اﻟﺘﺨﻴﲇ ﳍﺬا اﻟﻌﺪد‪ .‬ﻣﺜ ً‬
‫ﻧﺤﺎول ﻛﺘﺎﺑﺘﻪ ﻋﲆ ﺷﻜﻞ ﺿﻌﻒ ﺟﺪاء ﻋﺪدﻳﻦ ﺻﺤﻴﺤﲔ )و ﻫﺬا ُﻳﻜﺎﻓﺊ اﳌﻌﺎدﻟﺔ اﻟﺜﺎﻟﺜﺔ اﻟﺴﺎﺑﻘﺔ( ﻣﻊ ﻣﺮاﻋﺎة اﻟﴩوط‬
‫اﻟﺘﺎﻟﻴﺔ‪:‬‬
‫اﳉﺰء اﻟﺘﺨﻴﲇ ﻣﻮﺟﺐ‪ ،‬إذن ﻟﻠﻌﺪدﻳﻦ ﻧﻔﺲ اﻹﺷﺎرة و ﺑﺎﻟﺘﺎﱄ ﻧﺨﺘﺎرﳘﺎ ﻣﻮﺟﺒﲔ )أو ﺳﺎﻟﺒﲔ( ‪.‬‬ ‫•‬

‫اﳉﺰء اﳊﻘﻴﻘﻲ ‪ 7‬ﻓﺮدي إذن أﺣﺪﳘﺎ ﻓﺮدي و اﻵﺧﺮ زوﺟﻲ‪.‬‬ ‫•‬

‫اﳉﺰء اﳊﻘﻴﻘﻲ ‪ 7‬ﻣﻮﺟﺐ إذن اﳉﺰء اﳊﻘﻴﻘﻲ ﻟﻠﺠﺬر اﻟﱰﺑﻴﻌﻲ أﻛﱪ )ﺑﺎﻟﻘﻴﻤﺔ اﳌﻄﻠﻘﺔ( ﻣﻦ ﺟﺰﺋﻪ اﻟﺘﺨﻴﲇ‪.‬‬ ‫•‬

‫ﰲ اﻷﺧﲑ‪ ،‬ﻧﺒﺤﺚ ﻋﻦ ﻋﺪدﻳﻦ ﻣﻮﺟﺒﲔ )أو ﺳﺎﻟﺒﲔ( ‪ x‬و ‪ y‬ﺑﺤﻴﺚ ‪ |x| > ||y|| ، xy = 12‬و أﺣﺪﳘﺎ ﻓﺮدي و اﻵﺧﺮ‬
‫أن ‪ (4 + 3ı) = 7 + 24ı‬إذن اﳉﺬرﻳﻦ اﻟﱰﺑﻴﻌﻴﲔ‬‫زوﺟﻲ‪ .‬ﻟﻜﻦ ‪ 12 = 3 × 4‬ﻣﻨﻪ ‪ x = 4‬و ‪ . y = 3‬ﻧﺘﺤﻘﻖ ﺑﺴﻬﻮﻟﺔ ّ‬
‫‪ņőƱŀ Ţ‬‬
‫ﻟﻠﻌﺪد ﳘﺎ ‪ 4 + 3ı‬و ‪. −4 − 3ı‬‬

‫⧐‬
‫‪VI‬‬

‫‪ . 4ab + 2ı a − b‬ﻟﺪﻳﻨﺎ ‪:‬‬ ‫‪ •2‬اﻟﻄﺮﻳﻘﺔ اﻷوﱃ ‪ :‬ﻟﻴﻜﻦ ‪ z = x + ıy‬ﺟﺬر ًا ﺗﺮﺑﻴﻌﻴﺎ ﻟﻠﻌﺪد‬


‫⎧‬ ‫|‬ ‫|‬
‫⎪‬
‫⎪‬ ‫‪x + y = |4ab + 2ı a − b‬‬ ‫‪|| = 2 a + b‬‬
‫⎪‬ ‫|‬
‫‪z = 4ab + 2ı a − b‬‬ ‫⟺‬ ‫‪x − y = 4ab‬‬
‫⎨‬
‫⎪‬
‫⎪‬
‫⎪‬
‫⎩‬ ‫‪2xy = 2 a − b‬‬
‫‪ƕ‬‬

‫|‬ ‫|‬
‫‪||4ab + 2ı a − b‬‬ ‫= ||‬ ‫‪16a b + 4 a − b‬‬ ‫=‬ ‫‪4a + 8a b + 4b‬‬ ‫ﻣﻊ‬
‫‪ũŏ‬‬

‫‪= 2 a + 2a b + b = 2‬‬ ‫‪a +b‬‬ ‫‪=2 a +b‬‬

‫‪ 2x = 2 a + b‬ﻣﻨﻪ )‪. x = ± (a + b‬‬ ‫)‪+ 4ab = 2 (a + b‬‬ ‫ﺑﺠﻤﻊ اﳌﻌﺎدﻟﺘﲔ اﻷوﱃ و اﻟﺜﺎﻧﻴﺔ ﻳﻨﺘﺞ‬
‫‪ 2y = 2 a + b‬ﻣﻨﻪ )‪. y = ± (a − b‬‬ ‫)‪− 4ab = 2 (a − b‬‬ ‫ﺑﻄﺮح اﳌﻌﺎدﻟﺔ اﻟﺜﺎﻧﻴﺔ ﻣﻦ اﳌﻌﺎدﻟﺔ اﻷوﱃ ﻳﻨﺘﺞ‬

‫𝟯𝟱𝟭‬

‫‪http ://tinyurl.com/Malki1718‬‬ ‫‪0‬‬


‫‪ .3.VI‬ﺗﻄﺒﻴﻘﺎت اﻷﻋﺪاد اﳌﺮﻛﺒﺔ‬
‫‪.‬‬
‫اﳌﻌﺎدﻟﺔ اﻟﺜﺎﻟﺜﺔ ﺗُﻜﺘﺐ ‪:‬‬
‫])‪xy = a − b = (a + b) (a − b) = [− (a + b)] [− (a − b‬‬

‫‪Ŕž Ŧ Ľ‬‬
‫و ﺑﺎﻟﺘﺎﱄ ﻧﺨﺘﺎر ﻧﻔﺲ اﻹﺷﺎرة )ﻣﻮﺟﺒﺔ أو ﺳﺎﻟﺒﺔ( ﺑﺎﻟﻨﺴﺒﺔ ﻟﻠﻌﺪدﻳﻦ ‪ x‬و ‪. y‬‬

‫‪œǃ‬‬
‫إذن اﳉﺬرﻳﻦ اﻟﱰﺑﻴﻌﻴﲔ ﻟﻠﻌﺪد ‪ 4ab + 2 a − b ı‬ﳘﺎ ‪:‬‬
‫)‪(a + b) + ı (a − b‬‬ ‫و‬ ‫)‪− (a + b) − ı (a − b‬‬

‫اﻟﻄﺮﻳﻘﺔ اﻟﺜﺎﻧﻴﺔ ‪ :‬ﻟﺪﻳﻨﺎ ‪:‬‬

‫‪Ŀ‬‬
‫‪Z = 4ab + 2ı a − b‬‬ ‫𝜔‪= 2ı a − b − 2ıab = 2ı (a − ıb) = 2ı‬‬

‫ﻣﻊ ‪ . 𝜔 = a − ıb‬إذن ﻹﳚﺎد ﺟﺬر ﺗﺮﺑﻴﻌﻲ ﻟﻠﻌﺪد ‪ Z‬ﻳﻜﻔﻲ إﳚﺎد ﺟﺬر ﺗﺮﺑﻴﻌﻲ ﻟﻠﻌﺪد ‪ : u = 2ı = 0 + 2ı‬ﻧﺒﺤﺚ ﻋﻦ‬
‫)ﻷن ‪(Re(u) = 0 = x − y‬‬‫ﻷن ‪ (Im(u) > 0‬ﻣﺘﺴﺎوﻳﲔ ّ‬ ‫ﻋﺪدﻳﻦ ﺻﺤﻴﺤﲔ ‪ x‬و ‪ y‬ﻣﻮﺟﺒﲔ — أو ﺳﺎﻟﺒﲔ — ) ّ‬
‫و ﺣﺎﺻﻞ ﴐﲠﲈ ‪ . 1‬ﻧﺘﺤﻘﻖ ﺑﺴﻬﻮﻟﺔ ّ‬
‫أن ‪ x = y = 1‬ﺣﻞ ﻟﻠﻤﺴﺄﻟﺔ ﻣﻨﻪ )‪ 2ı = (1 + ı‬إذن ‪:‬‬

‫‪Z = 4ab + 2 a − b‬‬ ‫)‪ı = 2ı (a − ıb‬‬

‫))‪= (1 + ı) (a − ıb) = ((1 + ı) (a − ıb‬‬


‫))‪= ((a + b) + ı (a − b‬‬

‫و ﺑﺎﻟﺘﺎﱄ ﻓﺎﳉﺬران اﻟﱰﺑﻴﻌﻴﺎن ﻟﻠﻌﺪد ‪ Z‬ﳘﺎ‬


‫)‪(a + b) + ı (a − b‬‬ ‫و‬ ‫)‪− (a + b) − ı (a − b‬‬
‫■‬

‫‪𝟗𝟑.‬‬ ‫‪.‬‬
‫‪ •1‬ﺣﻞ ﰲ ‪ ℂ‬اﳌﻌﺎدﻟﺔ ‪. z + |z| = 0 :‬‬
‫‪ •2‬ﻟﻴﻜﻦ ‪ z‬ﻋﺪد ًا ﻣﺮﻛﺒﺎ ﺑﺤﻴﺚ ‪ . z ∉ ℝ−‬أﺛﺒﺖ ّ‬
‫أن ‪. 2 arg (z + |z|) = arg (z) (mod 2𝜋) :‬‬
‫‪ņőƱŀ Ţ‬‬
‫‪ •3‬ﻟﻴﻜﻦ ‪ z‬و ‪ z‬ﻋﺪدﻳﻦ ﻣﺮﻛﺒﲔ ﺑﺤﻴﺚ ‪ z + z ≠ 0‬و || ‪. ||z || = ||z‬‬
‫أن ‪. arg (z ) + arg (z ) = 2 arg (z + z ) (mod 2𝜋) :‬‬ ‫أﺛﺒﺖ ّ‬

‫‪VI‬‬
‫‪ •1‬ﻧﻀﻊ ‪ z = x + ıy‬ﺣﻴﺚ ‪ . x, y ∈ ℝ‬ﻟﺪﻳﻨﺎ ‪:‬‬ ‫ﺍﳊﻞّ‪.‬‬

‫‪⎧x +‬‬ ‫‪x +y =0‬‬ ‫‪x+‬‬ ‫‪x =0‬‬


‫‪z + |z| = 0 ⟺ x + ıy +‬‬ ‫⟺ ‪x +y =0‬‬ ‫⟺‬
‫⎨‬ ‫‪y=0‬‬ ‫‪y=0‬‬
‫⎩‬
‫‪ƕ‬‬

‫‪|x| = −x‬‬ ‫‪x ∈ ℝ−‬‬


‫⟺‬ ‫⟺‬ ‫‪⟺ z ∈ ℝ−‬‬
‫‪y=0‬‬ ‫‪y=0‬‬

‫إذن ﳎﻤﻮﻋﺔ اﳊﻠﻮل ﻫﻲ ‪. 𝒮 = ℝ− = ]−∞, 0] :‬‬


‫‪ũŏ‬‬

‫‪ •2‬ﺣﺴﺐ اﻟﺴﺆال اﻟﺴﺎﺑﻖ‪ ،‬إذا ﻛﺎن ‪ّ z ∉ ℝ−‬‬


‫ﻓﺈن ‪ z + |z| ≠ 0‬؛ و ﺣﺴﺐ ﺧﻮاص اﻟﻌﻤﺪة‪ ،‬ﻟﺪﻳﻨﺎ ‪:‬‬

‫)|‪2 arg (z + |z|) = arg (z) (mod 2𝜋) ⟺ arg (z + |z‬‬ ‫)𝜋‪= arg (z) (mod 2‬‬

‫)|‪(z + |z‬‬ ‫)|‪(z + |z‬‬ ‫∗‬


‫‪⟺ arg‬‬ ‫⟺ )𝜋‪= 0 (mod 2‬‬ ‫‪∈ ℝ+‬‬
‫‪z‬‬ ‫‪z‬‬

‫𝟰𝟱𝟭‬

‫‪http ://tinyurl.com/Malki1718‬‬ ‫‪0‬‬


‫‪ .VI‬ﲤﺎرﻳﻦ ﺗﻄﺒﻴﻘﻴﺔ‬
‫‪.‬‬
‫)|‪(z + |z‬‬ ‫∗‬
‫‪ .‬ﻟﻜﻦ ‪:‬‬ ‫‪∈ ℝ+‬‬ ‫إذن ﻳﻜﻔﻲ إﺛﺒﺎت ّ‬
‫أن ‪:‬‬
‫‪z‬‬

‫‪Ŕž Ŧ Ľ‬‬
‫)|‪(z + |z‬‬ ‫|‪z + 2z |z| + |z‬‬ ‫|‪|z‬‬ ‫‪zz‬‬
‫=‬ ‫‪= z + 2 |z| +‬‬ ‫‪= z + 2 |z| +‬‬
‫‪z‬‬ ‫‪z‬‬ ‫‪z‬‬ ‫‪z‬‬

‫‪œǃ‬‬
‫)|‪= z + z + 2 |z| = 2 Re (z) + 2 |z| = 2 (Re (z) + |z‬‬

‫)|‪(z + |z‬‬ ‫)|‪(z + |z‬‬


‫و ﺑﲈ ّ‬
‫أن ‪z ≠ 0‬‬ ‫أي ‪∈ ℝ+‬‬ ‫و ﻫﻮ ﻋﺪد ﺣﻘﻴﻘﻲ‪ .‬ﻟﻜﻦ |‪ Re (z) ≤ |z‬ﻟﻜﻞ ‪ ، z ∈ ℂ‬إذن ‪≥ 0‬‬
‫‪z‬‬ ‫‪z‬‬

‫‪Ŀ‬‬
‫)|‪(z + |z‬‬
‫و ﻫﻮ اﳌﻄﻠﻮب‪.‬‬ ‫ّ‬
‫ﻓﺈن ‪ Re (z) + |z| > 0‬ﻣﻨﻪ ‪> 0‬‬
‫‪z‬‬
‫أن ‪ّ z z ≠ 0‬‬
‫ﻓﺈن ‪:‬‬ ‫‪ •3‬ﺑﲈ ّ‬
‫) ‪arg (z ) + arg (z ) = 2 arg (z + z ) (mod 2𝜋) ⟺ arg (z z ) = arg (z + z‬‬ ‫)𝜋‪(mod 2‬‬

‫) ‪(z + z‬‬
‫‪⟺ arg‬‬ ‫)𝜋‪= 0 (mod 2‬‬
‫‪z z‬‬

‫) ‪(z + z‬‬ ‫∗‬


‫⟺‬ ‫‪∈ ℝ+‬‬
‫‪z z‬‬
‫) ‪(z + z‬‬ ‫∗‬
‫‪ .‬ﻟﻜﻦ ‪:‬‬ ‫‪∈ ℝ+‬‬ ‫إذن ﻳﻜﻔﻲ إﺛﺒﺎت ّ‬
‫أن ‪:‬‬
‫‪z z‬‬
‫) ‪(z + z‬‬ ‫‪z + 2z z + z‬‬ ‫‪z‬‬ ‫‪z‬‬ ‫‪z z‬‬ ‫‪z z‬‬
‫=‬ ‫=‬ ‫‪+2+‬‬ ‫=‬ ‫‪+2+‬‬
‫‪z z‬‬ ‫‪z z‬‬ ‫‪z‬‬ ‫‪z‬‬ ‫|| ‪||z‬‬ ‫|| ‪||z‬‬

‫أن || ‪ّ ||z || = ||z‬‬


‫ﻓﺈن || ‪ ||z || = ||z || ⋅ ||z || = ||z || ⋅ ||z‬ﻣﻨﻪ ‪:‬‬ ‫و ﺑﲈ ّ‬
‫) ‪(z + z‬‬ ‫‪z z +z z‬‬ ‫‪z z +z z‬‬
‫=‬ ‫=‪+2‬‬ ‫‪+2‬‬
‫‪z z‬‬ ‫|| ‪||z‬‬ ‫|| ‪||z‬‬
‫‪ņőƱŀ Ţ‬‬
‫|| ‪2 Re (z z ) + 2 ||z‬‬ ‫|| ‪Re (z z ) + ||z || ||z‬‬
‫=‬ ‫‪=2‬‬
‫|| ‪||z‬‬ ‫|| ‪||z‬‬
‫|| ‪Re (z z ) + ||z z‬‬
‫‪=2‬‬
‫‪VI‬‬

‫|| ‪||z‬‬
‫و ﻫﻮ ﻋﺪد ﺣﻘﻴﻘﻲ‪.‬‬
‫|| ‪Re (z z ) + ||z z‬‬ ‫|| ‪Re (z z ) + ||z z‬‬
‫‪ .‬و ﺑﲈ ّ‬
‫أن‬ ‫أي ‪∈ ℝ+‬‬ ‫ﻟﻜﻦ |‪ − Re (z) ≤ |z‬ﻟﻜﻞ ‪ ، z ∈ ℂ‬إذن ‪≥ 0‬‬
‫|| ‪||z‬‬ ‫|| ‪||z‬‬
‫|| ‪Re (z z ) + ||z z‬‬
‫‪ƕ‬‬

‫و ﻫﻮ اﳌﻄﻠﻮب‪.‬‬ ‫‪ z + z ≠ 0‬ﻓﺮﺿ ًﺎ ّ‬
‫ﻓﺈن ‪> 0‬‬
‫|| ‪||z‬‬
‫■‬
‫‪ũŏ‬‬

‫‬ ‫‪.‬‬
‫𝟒𝟗‬ ‫‪.‬‬
‫‪ •1‬ﻟﺘﻜﻦ ‪ z ، z ، z‬اﳉﺬور اﻟﺘﻜﻌﻴﺒﻴﺔ ﻟﻌﺪد ﻣﺮﻛﺐ ُﻣﻌﻄﻰ‪ .‬ﱢ‬
‫ﻋﱪ ﻋﻦ ‪ z‬و ‪ z‬ﺑﺪﻻﻟﺔ ‪. z‬‬
‫‪ •2‬أوﺟﺪ ﺣﻠﻮل اﳌﻌﺎدﻟﺔ ‪ z + (7 − ı) z − 8 − 8ı = 0‬ﺛﻢ أﻛﺘﺒﻬﺎ ﻋﲆ اﻟﺸﻜﻞ اﳌﺜﻠﺜﻲ و اﻟﺸﻜﻞ اﻷﳼ‪.‬‬
‫)إرﺷﺎد ‪ :‬ﺿﻊ ‪ Z = z‬و اﺣﺴﺐ )‪.( (9 + ı‬‬

‫𝟱𝟱𝟭‬

‫‪http ://tinyurl.com/Malki1718‬‬ ‫‪0‬‬


‫‪ .3.VI‬ﺗﻄﺒﻴﻘﺎت اﻷﻋﺪاد اﳌﺮﻛﺒﺔ‬
‫‪.‬‬
‫‪ •1‬ﻟﺘﻜﻦ ‪ z ، z‬و ‪ z‬ﺟﺬور ًا ﺗﻜﻌﻴﺒﻴﺔ ﻟﻌﺪد ﻣﺮﻛﺐ‪ ،‬ﳐﺘﻠﻔﺔ ﻣﺜﻨﻰ ﻣﺜﻨﻰ ‪ .‬إذن ‪. z = z = z‬‬ ‫ﺍﳊﻞّ‪.‬‬

‫‪Ŕž Ŧ Ľ‬‬
‫إذا ﻛﺎن ‪ّ z = 0‬‬
‫ﻓﺈن ‪ z = z = 0‬و ﺑﺎﻟﺘﺎﱄ ﻓﻬﺬه اﳉﺬور ﻟﻴﺴﺖ ﳐﺘﻠﻔﺔ و ﻫﺬا ﺗﻨﺎﻗﺾ ! إذن ‪ z ≠ 0‬ﻣﻨﻪ‬
‫‪z‬‬ ‫‪z‬‬
‫‪.‬‬ ‫=‬ ‫‪=1‬‬

‫‪œǃ‬‬
‫‪z‬‬ ‫‪z‬‬
‫‪z‬‬ ‫‪z‬‬
‫ﳐﺘﻠﻔﺔ ﻣﺜﻨﻰ ﻣﺜﻨﻰ و ﻣﻜﻌﺒﺎﲥﺎ ﺗﺴﺎوي ‪ 1‬إذن ﻓﻬﻲ اﳉﺬور اﻟﺘﻜﻌﻴﺒﻴﺔ ﻟﻠﻮﺣﺪة أي اﻷﻋﺪاد ‪ j ، 1‬و‬ ‫و‬ ‫اﻷﻋﺪاد ‪، 1‬‬
‫‪z‬‬ ‫‪z‬‬
‫‪z‬‬ ‫‪z‬‬ ‫‪z‬‬ ‫‪z‬‬
‫ﻐﲑ ﺷﻴﺌ ًﺎ( أي ‪ z = jz‬و ‪. z = j z‬‬
‫ﻟﻜﻦ ذﻟﻚ ﻻ ُﻳ ّ‬ ‫و‪= j‬‬ ‫‪=j‬‬ ‫)أو‬ ‫‪=j‬‬ ‫و‬ ‫‪ j‬و ﺑﺎﻟﺘﺎﱄ ‪= j‬‬

‫‪Ŀ‬‬
‫‪z‬‬ ‫‪z‬‬ ‫‪z‬‬ ‫‪z‬‬

‫ﻫﺬا ﻳﻌﻨﻲ أﻧﻪ ﺑﻤﻌﺮﻓﺔ أﺣﺪ اﳉﺬور اﻟﺘﻜﻌﻴﺒﻴﺔ ﻟﻌﺪد ﻣﺮﻛﺐ‪ ،‬ﻓﺈﻧﻨﺎ ﻧﻌﺮف اﳉﺬور اﻷﺧﺮى‪.‬‬

‫‪ •2‬ﻧﺘﺒﻊ اﻹرﺷﺎد ‪ :‬ﻧﺤﺴﺐ ‪ (9 + ı) = 81 + 18ı − 1 = 80 + 18ı‬و ﻧﻀﻊ‬


‫‪ Z = z‬ﻓﺘﺼﺒﺢ اﳌﻌﺎدﻟﺔ ‪Z + (7 − ı) z −‬‬
‫‪. 8 − 8ı = 0‬‬
‫ﳑ ّﻴﺰ ﻫﺬه اﳌﻌﺎدﻟﺔ ﻫﻮ )‪ Δ = (7 − ı) − 4 (−8 − 8ı) = 80 + 18ı = (9 + ı‬و ﺑﺎﻟﺘﺎﱄ ﳍﺎ ﺣﻼّن ‪Z = :‬‬
‫‪−7 + ı + 9 + ı‬‬ ‫‪−7 + ı − 9 − ı‬‬
‫= ‪.Z‬‬ ‫و‪= 1+ı‬‬ ‫‪= −8‬‬
‫‪2‬‬ ‫‪2‬‬
‫ﻹﲤﺎم اﳊﻞ‪ ،‬ﳚﺐ اﻟﺒﺤﺚ ﻋﻦ اﳉﺬور اﻟﺘﻜﻌﻴﺒﻴﺔ ﻟﻠﻌﺪدﻳﻦ ‪ −8‬و ‪. 1 + ı‬‬

‫‪ 1 + ı = √2e‬ﻓﺤﺴﺐ اﻟﺴﺆال اﻟﺴﺎﺑﻖ‪ ،‬اﳉﺬور اﻟﺘﻜﻌﻴﺒﻴﺔ ﻟﻠﻌﺪد ‪−8‬‬ ‫‪/‬‬ ‫‪/‬‬


‫=‬ ‫ﻟﻜﻦ )‪ −8 = (−2‬و‬
‫‪√2e‬‬
‫‪ 2e‬و‬ ‫‪/‬‬ ‫أن ‪ّ −1 = e‬‬
‫ﻓﺈن ﻫﺬه اﳉﺬور ﻫﻲ ‪، −2‬‬ ‫‪ −2e‬و ﺑﲈ ّ‬ ‫‪/‬‬ ‫ﻫﻲ ‪ −2j ، −2‬و ‪ −2j‬أي ‪ −2e / ، −2‬و‬
‫‪ 2e /‬أي ‪ 2e− / ، −2‬و ‪. 2e /‬‬
‫‪ √2e‬و‬ ‫‪/‬‬ ‫‪، √2e‬‬ ‫‪/‬‬ ‫‪ √2e‬أي‬ ‫‪/‬‬ ‫‪j‬‬ ‫‪ √2e‬و‬ ‫‪/‬‬ ‫‪j‬‬ ‫‪، √2e‬‬ ‫‪/‬‬ ‫اﳉﺬور اﻟﺘﻜﻌﻴﺒﻴﺔ ﻟﻠﻌﺪد ‪ 1 + ı‬ﻫﻲ‬
‫‪. √2e /‬‬
‫‪.𝒮 = −2, 2e− / , 2e / , √2e‬‬ ‫‪/‬‬ ‫‪, √2e‬‬ ‫‪/‬‬ ‫‪, √2e‬‬ ‫‪/‬‬ ‫ﰲ اﻷﺧﲑ‪ ،‬ﳎﻤﻮﻋﺔ ﺣﻠﻮل اﳌﻌﺎدﻟﺔ ﻫﻲ ‪:‬‬
‫■‬
‫‪ņőƱŀ Ţ‬‬
‫= ‪.Z‬‬
‫‪−1 + ı‬‬
‫أﺣﺴﺐ اﳉﺬور اﻟﺘﻜﻌﻴﺒﻴﺔ ﻟﻠﻌﺪد‬
‫‬ ‫‪𝟗𝟓.‬‬ ‫‪.‬‬
‫‪4‬‬

‫ﺍﳊﻞّ‪ .‬ﻧﻀﻊ ‪ j = e /‬و ﻧﺬﻛّﺮ ّ‬


‫أن ‪. j = 1‬‬
‫‪VI‬‬
‫ﻧُﻄ ّﺒﻖ ﻧﺘﻴﺠﺔ اﻟﺘﻤﺮﻳﻦ اﻟﺴﺎﺑﻖ )ﲤﺮﻳﻦ ‪ . (94‬ﻟﺪﻳﻨﺎ ‪:‬‬
‫‪ƕ‬‬

‫‪1‬‬ ‫‪√2‬‬ ‫‪√2‬‬ ‫‪√2‬‬ ‫‪− /‬‬ ‫𝜋‪3‬‬ ‫𝜋‪3‬‬ ‫‪√2‬‬ ‫‪/‬‬
‫= )‪(−1 + ı‬‬ ‫‪−‬‬ ‫‪+ı‬‬ ‫‪=2‬‬ ‫‪cos‬‬ ‫‪+ ı sin‬‬ ‫=‬ ‫‪e‬‬
‫‪4‬‬ ‫‪4‬‬ ‫‪2‬‬ ‫‪2‬‬ ‫‪4‬‬ ‫‪4‬‬ ‫‪2‬‬

‫إذن اﳉﺬور اﻟﺘﻜﻌﻴﺒﻴﺔ ﻟﻠﻌﺪد ‪ Z‬ﻫﻲ ‪:‬‬


‫‪√2‬‬ ‫‪√2‬‬ ‫‪√2‬‬ ‫‪√2‬‬ ‫‪√2‬‬
‫‪ũŏ‬‬

‫= ‪z‬‬ ‫‪e‬‬ ‫‪/‬‬ ‫‪,‬‬ ‫‪z =j‬‬ ‫‪e‬‬ ‫‪/‬‬ ‫=‬ ‫‪e‬‬ ‫‪/‬‬ ‫‪,‬‬ ‫‪z =j‬‬ ‫‪e‬‬ ‫‪/‬‬ ‫=‬ ‫‪e‬‬ ‫‪/‬‬
‫‪2‬‬ ‫‪2‬‬ ‫‪2‬‬ ‫‪2‬‬ ‫‪2‬‬
‫■‬

‫أﺣﺴﺐ اﳉﺬور اﻟﺘﻜﻌﻴﺒﻴﺔ ﻟﻠﻌﺪدﻳﻦ ‪ 2 − 2ı‬و ‪. 11 + 2ı‬‬


‫‬ ‫‪𝟗𝟔.‬‬ ‫‪.‬‬

‫𝟲𝟱𝟭‬

‫‪http ://tinyurl.com/Malki1718‬‬ ‫‪0‬‬


‫‪ .VI‬ﲤﺎرﻳﻦ ﺗﻄﺒﻴﻘﻴﺔ‬
‫‪.‬‬
‫ﺍﳊﻞّ‪ .‬ﻟﺪﻳﻨﺎ ‪:‬‬

‫‪Ŕž Ŧ Ľ‬‬
‫‪√2‬‬ ‫‪√2‬‬ ‫𝜋‬ ‫𝜋‬ ‫‪/‬‬
‫‪2 − 2ı = 2√2‬‬ ‫‪−ı‬‬ ‫‪= 2√2 cos‬‬ ‫‪− ı sin‬‬ ‫‪=2‬‬ ‫‪e−‬‬ ‫‪/‬‬ ‫‪= √2e−‬‬ ‫‪/‬‬
‫‪2‬‬ ‫‪2‬‬ ‫‪4‬‬ ‫‪4‬‬

‫‪œǃ‬‬
‫‪ j = e‬أي ﻫﻲ‬ ‫‪/‬‬ ‫‪ √2e−‬ﺣﻴﺚ‬ ‫‪/‬‬ ‫‪j‬‬ ‫‪ √2e−‬و‬ ‫‪/‬‬ ‫‪j ، √2e−‬‬ ‫‪/‬‬ ‫و ﺑﺎﻟﺘﺎﱄ ﻓﺎﳉﺬور اﻟﺘﻜﻌﻴﺒﻴﺔ ﻟﻠﻌﺪد ‪ 2 − 2ı‬ﻫﻲ‬
‫‪. √2e‬‬ ‫‪/‬‬ ‫اﻷﻋﺪاد ‪ √2e / ، √2e− /‬و ‪= −1 − ı‬‬
‫‪11√5‬‬ ‫‪2√5‬‬
‫‪ 11 + 2ı = 5√5‬إذن )‪ arg (11 + 2ı‬ﻟﻴﺴﺖ زاوﻳﺔ ﺷﻬﲑة و ﺑﺎﻟﺘﺎﱄ ﻻ‬ ‫‪+ı‬‬ ‫ﻟﺪﻳﻨﺎ ‪ ||11 + 2ı|| = 5√5‬ﻣﻨﻪ‬
‫‪25‬‬ ‫‪25‬‬

‫‪Ŀ‬‬
‫ﻧﺴﺘﻄﻴﻊ اﺳﺘﻌﲈل اﻟﺸﻜﻞ اﻷﳼ‪.‬‬
‫ﻟﻴﻜﻦ ‪ x + ıy‬ﺟﺬر ًا ﺗﻜﻌﻴﺒﻴ ًﺎ ﻟﻠﻌﺪد ‪ 11 + 2ı‬ﻣﻊ ‪. x, y ∈ ℝ‬‬
‫ﻟﺪﻳﻨﺎ ‪ (x + ıy) = 11 + 2ı‬أي ‪ x − 3xy + ı 3x y − y = 11 + 2ı‬و ‪ ||x + ıy|| = ||11 + 2ı|| = 5√5‬أي‬
‫أي‬ ‫و ﺑﺄﺧﺬ اﳉﺬر اﻟﺘﻜﻌﻴﺒﻲ )اﳊﻘﻴﻘﻲ( ﻟﻠﻄﺮﻓﲔ ﻳﻨﺘﺞ ‪x + y = √5‬‬ ‫‪x +y‬‬ ‫= ‪= 5√5‬‬ ‫‪5 = √5‬‬
‫⎧‬ ‫‪x x − 3y = 11‬‬
‫⎪‬
‫‪ . x + y = 5‬إذن ‪. y 3x − y = 2 :‬‬
‫⎨‬
‫⎪‬
‫‪⎩ x +y =5‬‬
‫ﻣﻦ اﳌﻌﺎدﻟﺔ اﻷﺧﲑة ﻧﺠﺪ ‪. y = 5 − x‬‬
‫ُﻌﻮض ﰲ اﳌﻌﺎدﻟﺔ اﻷوﱃ ﻟﻨﺠﺪ ‪ x 4x − 15 = 11‬أي ‪. 4x − 15x − 11 = 0‬‬
‫ﻧ ّ‬
‫ُﻌﻮض ﰲ اﳌﻌﺎدﻟﺔ اﻟﺜﺎﻧﻴﺔ ﻟﻨﺠﺪ ‪ y 15 − 4y = 2‬أي ‪. 4y − 15y + 2 = 0‬‬ ‫ﻣﻦ اﳌﻌﺎدﻟﺔ اﻷﺧﲑة ﻧﺠﺪ ‪ . x = 5 − y‬ﻧ ّ‬
‫أن ‪ّ y = −2‬‬
‫ﺣﻞ ﻟﻠﻤﻌﺎدﻟﺔ ‪) 4y − 15y + 2 = 0‬أو ﻧﺴﺘﻌﻤﻞ‬ ‫ﺣﻞ ﻟﻠﻤﻌﺎدﻟﺔ ‪ 4x − 15x − 11 = 0‬و ّ‬ ‫أن ‪ّ x = −1‬‬
‫ﻧﻼﺣﻆ ّ‬
‫اﳌﻌﺎدﻟﺔ اﻷﺧﲑة ﻣﻊ ‪ (x = −1‬و ﺑﺎﻟﺘﺎﱄ ‪ −1 − 2ı‬ﺟﺬر ﺗﻜﻌﻴﺒﻲ ﻟﻠﻌﺪد ‪ . 11 + 2ı‬اﳉﺬرﻳﻦ اﻟﺘﻜﻌﻴﺒﻴﲔ اﻵﺧﺮﻳﻦ ﳘﺎ ‪:‬‬
‫‪1 − 2√3‬‬ ‫‪2 + √3‬‬ ‫‪1 + 2√3‬‬ ‫‪2 − √3‬‬
‫= ‪(−1 − 2ı) j‬‬ ‫‪+ı‬‬ ‫و‬ ‫= ‪(−1 − 2ı) j‬‬ ‫‪+ı‬‬
‫‪2‬‬ ‫‪2‬‬ ‫‪2‬‬ ‫‪2‬‬
‫■‬ ‫‪) j = e‬ﺟﺬر ﺗﻜﻌﻴﺒﻲ ﻟﻠﻮﺣﺪة(‪.‬‬ ‫‪/‬‬ ‫ﻣﻊ‬

‫‬ ‫‪.‬‬
‫𝟕𝟗‬ ‫‪.‬‬
‫‪ņőƱŀ Ţ‬‬
‫ﻋﲔ اﳉﺬور اﻟﺘﻜﻌﻴﺒﻴﺔ ﻟﻸﻋﺪاد اﻟﺘﺎﻟﻴﺔ ‪:‬‬
‫ﱢ‬
‫)‪4√2 (1 + ı‬‬ ‫•‬ ‫‪3‬‬ ‫𝜃 ‪1 + ı tan‬‬ ‫‪ı + √3‬‬
‫•‬ ‫‪2‬‬ ‫•‬ ‫‪1‬‬
‫𝜃 ‪1 − ı tan‬‬ ‫‪ı − √3‬‬
‫‪VI‬‬

‫ﺍﳊﻞّ‪ .‬ﻧﻀﻊ ‪ j = e /‬و ﻧﺬﻛّﺮ ّ‬


‫أن ‪. j = 1‬‬
‫ﻧُﻄ ّﺒﻖ ﻧﺘﻴﺠﺔ اﻟﺘﻤﺮﻳﻦ اﻟﺴﺎﺑﻖ )ﲤﺮﻳﻦ ‪. (94‬‬
‫‪ •1‬ﻟﺪﻳﻨﺎ ‪:‬‬
‫‪ƕ‬‬

‫‪ı + √3‬‬ ‫‪+ ı‬‬ ‫‪e‬‬ ‫‪/‬‬


‫‪−‬‬
‫=‬ ‫=‬ ‫‪/‬‬
‫‪=e‬‬ ‫‪= e−‬‬ ‫‪/‬‬ ‫‪=e‬‬ ‫‪/‬‬ ‫‪= e‬‬ ‫‪/‬‬
‫‪ı − √3‬‬ ‫‪+ ı‬‬ ‫‪e‬‬
‫‪ũŏ‬‬

‫‪/‬‬ ‫‪/‬‬ ‫‪/‬‬ ‫‪ı + √3‬‬


‫‪.j e‬‬ ‫‪ je‬و‬ ‫‪،e‬‬ ‫ﻫﻲ اﻷﻋﺪاد‬ ‫و ﺑﺎﻟﺘﺎﱄ ﻓﺎﳉﺬور اﻟﺘﻜﻌﻴﺒﻴﺔ ﻟﻠﻌﺪد‬
‫‪ı − √3‬‬
‫‪ •2‬ﻟﺪﻳﻨﺎ ‪:‬‬
‫𝜃 ‪1 + ı tan 𝜃 cos 𝜃 + ı sin‬‬ ‫‪e‬‬ ‫‪/‬‬
‫=‬ ‫=‬ ‫‪=e‬‬ ‫‪= e‬‬
‫‪1 − ı tan 𝜃 cos 𝜃 − ı sin 𝜃 e−‬‬
‫‪/‬‬ ‫‪/‬‬ ‫‪/‬‬ ‫𝜃 ‪1 + ı tan‬‬
‫‪.j e‬‬ ‫‪ je‬و‬ ‫‪،e‬‬ ‫ﻫﻲ اﻷﻋﺪاد‬ ‫و ﺑﺎﻟﺘﺎﱄ ﻓﺎﳉﺬور اﻟﺘﻜﻌﻴﺒﻴﺔ ﻟﻠﻌﺪد‬
‫𝜃 ‪1 − ı tan‬‬

‫𝟳𝟱𝟭‬

‫‪http ://tinyurl.com/Malki1718‬‬ ‫‪0‬‬


‫‪ .3.VI‬ﺗﻄﺒﻴﻘﺎت اﻷﻋﺪاد اﳌﺮﻛﺒﺔ‬
‫‪.‬‬
‫‪ •3‬ﻟﺪﻳﻨﺎ ‪:‬‬
‫‪√2‬‬ ‫‪√2‬‬

‫‪Ŕž Ŧ Ľ‬‬
‫‪4√2 (1 + ı) = 8‬‬ ‫‪+ı‬‬ ‫‪= 8e‬‬ ‫‪/‬‬ ‫‪= 2e‬‬ ‫‪/‬‬
‫‪2‬‬ ‫‪2‬‬

‫‪œǃ‬‬
‫‪ . 2j e‬ﻟﺪﻳﻨﺎ ‪:‬‬ ‫‪/‬‬ ‫‪ 2je‬و‬ ‫‪/‬‬ ‫‪، 2e‬‬ ‫‪/‬‬ ‫و ﺑﺎﻟﺘﺎﱄ ﻓﺎﳉﺬور اﻟﺘﻜﻌﻴﺒﻴﺔ ﻟﻠﻌﺪد )‪ 4√2 (1 + ı‬ﻫﻲ اﻷﻋﺪاد‬
‫‪/‬‬ ‫‪/‬‬ ‫‪/‬‬ ‫‪+‬‬ ‫‪/‬‬
‫‪2je‬‬ ‫‪= 2e‬‬ ‫‪e‬‬ ‫‪= 2e‬‬ ‫‪= 2e‬‬
‫‪/‬‬ ‫‪/‬‬ ‫‪/‬‬ ‫‪+‬‬ ‫‪/‬‬
‫‪2j e‬‬ ‫‪= 2e‬‬ ‫‪e‬‬ ‫‪= 2e‬‬ ‫‪= 2e‬‬

‫‪Ŀ‬‬
‫■‬

‫‪. Z = −119 + 120ı‬‬ ‫أﺣﺴﺐ اﳉﺬور اﻟﺮاﺑﻌﺔ ﻟﻠﻌﺪد‬


‫‬ ‫‪𝟗𝟖.‬‬ ‫‪.‬‬

‫ﺍﳊﻞّ‪ .‬ﻧﻀﻊ ]𝜃 ‪ . Z = [r,‬ﻟﺪﻳﻨﺎ ‪ r = (−119) + 120 = √28561 = 169 :‬ﻣﻨﻪ‬


‫‪120‬‬ ‫‪119‬‬
‫= 𝜃 ‪sin‬‬ ‫‪ cos 𝜃 = −‬و‬
‫‪169‬‬ ‫‪169‬‬
‫إذن اﻟﺰاوﻳﺔ 𝜃 ﻟﻴﺴﺖ ﺷﻬﲑة و ﻫﺬا ﻳﻌﻨﻲ أﻧﻪ ﻻ ﻳﻤﻜﻦ إﺳﺘﻌﲈل اﻟﺸﻜﻞ اﻷﳼ ﻟﻠﻌﺪد ‪ Z‬ﻹﳚﺎد ﺟﺬوره اﻟﺮاﺑﻌﺔ ﻟﺬﻟﻚ ﻧﺒﺤﺚ ﻋﻦ‬
‫ﻃﺮﻳﻘﺔ أﺧﺮى‪ .‬ﻟﻜﻦ‪ ،‬اﳉﺬور اﻟﺮاﺑﻌﺔ ﻟﻠﻌﺪد ‪ Z‬ﻫﻲ اﳉﺬور اﻟﱰﺑﻴﻌﻴﺔ ﳉﺬوره اﻟﱰﺑﻴﻌﻴﺔ إذن ﻧﺒﺤﺚ ّأوﻻً ﻋﻦ ﺟﺬوره اﻟﱰﺑﻴﻌﻴﺔ‪.‬‬
‫‪⎪ x − y = −119‬‬
‫⎧‬
‫⟺ ‪.z = Z‬‬ ‫‪2xy = 120‬‬ ‫ﻟﻴﻜﻦ ‪ z = x + ıy‬ﺟﺬر ًا ﺗﺮﺑﻴﻌﻴ ًﺎ ﻟﻠﻌﺪد ‪ Z‬ﻣﻊ ‪ . x, y ∈ ℝ‬ﻟﺪﻳﻨﺎ ‪:‬‬
‫⎨‬
‫⎪‬
‫‪⎩ x + y = 169‬‬
‫ﺑﺠﻤﻊ اﳌﻌﺎدﻟﺘﲔ اﻷوﱃ و اﻟﺜﺎﻟﺜﺔ ﻳﻨﺘﺞ ‪ 2x = 50‬ﻣﻨﻪ ‪ . x = ±25‬ﻷﺟﻞ ‪ x = 5‬ﻳﻜﻮن )ﻣﻦ اﳌﻌﺎدﻟﺔ اﻟﺜﺎﻧﻴﺔ( ‪ y = 12‬أي‬
‫‪ z = 5 + 12ı‬و ﻷﺟﻞ ‪ x = −5‬ﻳﻜﻮن ‪ . z = −5 − 12ı‬ﻹﲤﺎم اﳊﻞ‪ ،‬ﳚﺐ اﻟﺒﺤﺚ ﻋﻦ اﳉﺬور اﻟﱰﺑﻴﻌﻴﺔ ﻟﻠﻌﺪدﻳﻦ ‪ z‬و‬
‫‪ z‬؛ أو ﻳﻜﻔﻲ ﴐب أﺣﺪ اﳉﺬور اﻟﱰﺑﻴﻌﻴﺔ ﳍﲈ ﺑﺎﳉﺬور اﻟﺮاﺑﻌﺔ ﻟﻠﻮﺣﺪة إذا ﻻﺣﻈﻨﺎ ّ‬
‫أن )‪. 5 + 12ı = (3 + 2ı‬‬
‫اﳉﺬور اﻟﺮاﺑﻌﺔ ﻟﻠﻮﺣﺪة ﻫﻲ ‪ ı ، −1 ، 1‬و ‪ . −ı‬إذن اﳉﺬور اﻟﺮاﺑﻌﺔ ﻟﻠﻌﺪد ‪ Z‬ﻫﻲ ‪ ı (3 + 2ı) ، − (3 + 2ı) ، 3 + 2ı‬و‬
‫■‬ ‫)‪ −ı (3 + 2ı‬أي اﻷﻋﺪاد ‪ −2 + 3ı ، −3 − 2ı ، 3 + 2ı‬و ‪. 2 − 3ı‬‬
‫‪ņőƱŀ Ţ‬‬
‫ﻋﲔ اﳉﺬور اﻟﻨﻮﻧﻴﺔ ﻟﻠﻌﺪد ‪ z‬ﰲ ﻛﻞ ﺣﺎﻟﺔ ﻣﻦ اﳊﺎﻻت اﻟﺘﺎﻟﻴﺔ ‪:‬‬
‫ّ‬
‫‬ ‫‪𝟗𝟗.‬‬ ‫‪.‬‬

‫‪VI‬‬
‫‪9 √3‬‬ ‫‪z = −7 − 24ı‬‬ ‫‪ n = 4 •1‬و‬
‫=‪z‬‬ ‫‪1 − ı√3‬‬ ‫‪ n = 5 •4‬و‬
‫‪2‬‬

‫‪z = 32 ı − √3‬‬ ‫‪ n = 6 •5‬و‬ ‫‪z = 8 − 8ı√3‬‬ ‫‪ n = 4 •2‬و‬

‫‪z = −ı‬‬ ‫‪ n = 6 •6‬و‬ ‫‪z = 32ı‬‬ ‫‪ n = 5 •3‬و‬


‫‪ƕ‬‬

‫ﺍﳊﻞّ‪ .‬ﻹﳚﺎد اﳉﺬور اﻟﻨﻮﻧﻴﺔ ﻟﻌﺪد ﻣﺮﻛﺐ ﻳﻜﻔﻲ ﻣﻌﺮﻓﺔ أﺣﺪ ﻫﺬه اﳉﺬور اﻟﻨﻮﻧﻴﺔ و ﴐﺑﻪ ﰲ اﳉﺬور اﻟﻨﻮﻧﻴﺔ ﻟﻠﻮﺣﺪة‪.‬‬
‫‪ũŏ‬‬

‫‪ •1‬ﻧﻼﺣﻆ ّ‬
‫أن )‪ −24ı = 2 × 3 × (−4ı‬و )‪ −7 = 9 − 16 = 3 + (−4ı‬و ﺑﺎﻟﺘﺎﱄ ‪:‬‬
‫)‪−7 − 24ı = 3 + 2 × 3 × (−4ı) + (−4ı) = (3 − 4ı‬‬
‫ﺑﺎﳌﺜﻞ‪ −4ı = 2 × 2 × (−ı) ،‬و )‪ 3 = 2 + (−ı‬إذن ‪:‬‬
‫)‪3 − 4ı = 2 + 2 × 2 × (−ı) + (−ı) = (2 − ı‬‬
‫ﻣﻨﻪ )‪ −7 − 24ı = (2 − ı‬و ﺑﺎﻟﺘﺎﱄ ﻓﺎﳉﺬور اﻟﺮاﺑﻌﺔ ﻟﻠﻌﺪد ‪ −7 − 24ı‬ﻫﻲ اﻷﻋﺪاد ‪ ı (2 − ı) ، − (2 − ı) ، 2 − ı‬و‬
‫)‪ −ı (2 − ı‬أي اﻷﻋﺪاد ‪ 1 + 2ı ، −2 + ı ، 2 − ı‬و ‪. −1 − 2ı‬‬

‫𝟴𝟱𝟭‬

‫‪http ://tinyurl.com/Malki1718‬‬ ‫‪0‬‬


‫‪ .VI‬ﲤﺎرﻳﻦ ﺗﻄﺒﻴﻘﻴﺔ‬
‫‪.‬‬
‫‪ •2‬ﻟﺪﻳﻨﺎ ‪:‬‬
‫‪1‬‬ ‫‪√3‬‬
‫‪8 − 8ı√3 = 16‬‬ ‫‪−ı‬‬ ‫‪= 16e−‬‬ ‫‪/‬‬ ‫‪= 2e−‬‬ ‫‪/‬‬

‫‪Ŕž Ŧ Ľ‬‬
‫‪2‬‬ ‫‪2‬‬

‫‪ −2ıe−‬أي‬ ‫‪/‬‬ ‫‪ 2ıe−‬و‬ ‫‪/‬‬ ‫‪، −2e−‬‬ ‫‪/‬‬ ‫‪، 2e−‬‬ ‫‪/‬‬
‫و ﺑﺎﻟﺘﺎﱄ ﻓﺎﳉﺬور اﻟﺮاﺑﻌﺔ ﻟﻠﻌﺪد ‪ 8 − 8ı√3‬ﻫﻲ اﻷﻋﺪاد‬

‫‪œǃ‬‬
‫اﻷﻋﺪاد ‪ 2e / ، 2e / ، 2e− /‬و ‪. 2e− /‬‬
‫‪ 2ıe‬و‬ ‫‪/‬‬ ‫‪، 2ıe‬‬ ‫‪/‬‬ ‫‪، 2ıe‬‬ ‫‪/‬‬ ‫‪ •3‬ﻧﻼﺣﻆ ّ‬
‫أن )‪ 32ı = (2ı‬و ﺑﺎﻟﺘﺎﱄ ﻓﺎﳉﺬور اﳋﺎﻣﺴﺔ ﻟﻠﻌﺪد ‪ 32ı‬ﻫﻲ اﻷﻋﺪاد ‪، 2ı‬‬
‫‪ 2ıe /‬أي اﻷﻋﺪاد ‪ 2e / ، 2e / ، 2e / ، 2ı‬و ‪. 2e /‬‬

‫‪Ŀ‬‬
‫‪ •4‬ﻟﺪﻳﻨﺎ ‪:‬‬
‫‪9√3‬‬ ‫‪/‬‬ ‫‪/‬‬
‫‪1 − ı√3 = 3‬‬ ‫‪e−‬‬ ‫‪/‬‬ ‫‪= 3‬‬ ‫‪e−‬‬ ‫‪/‬‬
‫‪2‬‬
‫‪9 3‬‬
‫و ﺑﺎﻟﺘﺎﱄ ﻓﺎﳉﺬور اﳋﺎﻣﺴﺔ ﻟﻠﻌﺪد ‪ √ 1 − ı√3‬ﻫﻲ اﻷﻋﺪاد ‪:‬‬
‫‪2‬‬
‫‪/‬‬ ‫‪/‬‬ ‫‪/‬‬ ‫‪/‬‬ ‫‪/‬‬
‫‪3‬‬ ‫‪e−‬‬ ‫‪/‬‬ ‫‪e‬‬ ‫‪/‬‬ ‫‪3‬و‬ ‫‪e−‬‬ ‫‪/‬‬ ‫‪e‬‬ ‫‪/‬‬ ‫‪،3‬‬ ‫‪e−‬‬ ‫‪/‬‬ ‫‪e‬‬ ‫‪/‬‬ ‫‪،3‬‬ ‫‪e−‬‬ ‫‪/‬‬ ‫‪e‬‬ ‫‪/‬‬ ‫‪،3‬‬ ‫‪e−‬‬ ‫‪/‬‬

‫‪/‬‬ ‫‪/‬‬ ‫‪/‬‬ ‫‪/‬‬ ‫‪/‬‬ ‫‪/‬‬ ‫‪/‬‬ ‫‪/‬‬ ‫‪/‬‬


‫‪.3‬‬ ‫‪e‬‬ ‫‪3‬و‬ ‫‪e‬‬ ‫‪،3‬‬ ‫‪e‬‬ ‫‪،3‬‬ ‫‪e‬‬ ‫‪،3‬‬ ‫‪e−‬‬ ‫‪/‬‬ ‫أي اﻷﻋﺪاد‬
‫‪ •5‬ﻟﺪﻳﻨﺎ ‪:‬‬
‫‪√3 1‬‬ ‫‪/‬‬ ‫‪/‬‬
‫‪32 ı − √3 = 64 −‬‬ ‫‪+‬‬ ‫‪= 64e‬‬ ‫‪= 2e‬‬
‫‪2‬‬ ‫‪2‬‬

‫و ﺑﺎﻟﺘﺎﱄ ﻓﺎﳉﺬور اﻟﺴﺎدﺳﺔ ﻟﻠﻌﺪد ‪ 32 ı − √3‬ﻫﻲ اﻷﻋﺪاد ‪:‬‬


‫‪2e‬‬ ‫‪/‬‬ ‫‪e‬‬ ‫‪/‬‬ ‫‪ 2e‬و‬ ‫‪/‬‬ ‫‪e‬‬ ‫‪/‬‬ ‫‪، 2e‬‬ ‫‪/‬‬ ‫‪e‬‬ ‫‪/‬‬ ‫‪، 2e‬‬ ‫‪/‬‬ ‫‪e‬‬ ‫‪/‬‬ ‫‪، 2e‬‬ ‫‪/‬‬ ‫‪e‬‬ ‫‪/‬‬ ‫‪، 2e‬‬ ‫‪/‬‬

‫‪. 2e‬‬ ‫‪/‬‬ ‫‪ 2e‬و‬ ‫‪/‬‬ ‫‪، 2e‬‬ ‫‪/‬‬ ‫‪، 2e‬‬ ‫‪/‬‬ ‫‪، 2e‬‬ ‫‪/‬‬ ‫‪، 2e‬‬ ‫‪/‬‬ ‫أي اﻷﻋﺪاد‬
‫‪ •6‬ﻟﺪﻳﻨﺎ ‪:‬‬
‫‪−ı = ı = e‬‬ ‫‪/‬‬ ‫‪= e‬‬ ‫‪/‬‬
‫‪ņőƱŀ Ţ‬‬
‫و ﺑﺎﻟﺘﺎﱄ ﻓﺎﳉﺬور اﻟﺴﺎدﺳﺔ ﻟﻠﻌﺪد ‪ −ı‬ﻫﻲ اﻷﻋﺪاد ‪:‬‬
‫‪e‬‬ ‫‪/‬‬ ‫‪e‬‬ ‫‪/‬‬ ‫‪e‬و‬ ‫‪/‬‬ ‫‪e‬‬ ‫‪/‬‬ ‫‪،e‬‬ ‫‪/‬‬ ‫‪e‬‬ ‫‪/‬‬ ‫‪،e‬‬ ‫‪/‬‬ ‫‪e‬‬ ‫‪/‬‬ ‫‪،e‬‬ ‫‪/‬‬ ‫‪e‬‬ ‫‪/‬‬ ‫‪،e‬‬ ‫‪/‬‬
‫‪VI‬‬

‫‪.e‬‬ ‫‪/‬‬ ‫‪e‬و‬ ‫‪/‬‬ ‫‪،e‬‬ ‫‪/‬‬ ‫‪،e‬‬ ‫‪/‬‬ ‫‪،e‬‬ ‫‪/‬‬ ‫‪،e‬‬ ‫‪/‬‬ ‫أي اﻷﻋﺪاد‬
‫■‬

‫‬ ‫‪.‬‬
‫‪ƕ‬‬

‫‪.‬‬
‫𝟎𝟎𝟏‬
‫ﺣﻞ ﰲ ﳎﻤﻮﻋﺔ اﻷﻋﺪاد اﳌﺮﻛﺒﺔ اﳌﻌﺎدﻟﺘﲔ اﻟﺘﺎﻟﻴﺘﲔ ‪:‬‬
‫‪1−ı‬‬ ‫‪1 + ı√3‬‬
‫= ‪z‬‬ ‫•‬ ‫‪2‬‬ ‫= ‪z‬‬ ‫•‬‫‪1‬‬
‫‪1 + ı√3‬‬ ‫‪1 − ı√3‬‬
‫‪ũŏ‬‬

‫‪ •1‬ﻟﺪﻳﻨﺎ ‪:‬‬ ‫ﺍﳊﻞّ‪.‬‬


‫‪1 + ı√3‬‬ ‫‪2 cos‬‬ ‫‪+ ı sin‬‬ ‫‪e‬‬ ‫‪/‬‬ ‫𝜋‪2‬‬ ‫𝜋‪2‬‬
‫=‬ ‫=‬ ‫‪=e‬‬ ‫‪/‬‬ ‫‪⋅e‬‬ ‫‪/‬‬ ‫‪=e‬‬ ‫‪/‬‬ ‫‪= cos‬‬ ‫‪+ ı sin‬‬
‫‪1 − ı√3 2 cos −‬‬ ‫‪+ ı sin −‬‬ ‫‪e−‬‬ ‫‪/‬‬ ‫‪3‬‬ ‫‪3‬‬

‫𝟵𝟱𝟭‬

‫‪http ://tinyurl.com/Malki1718‬‬ ‫‪0‬‬


‫‪ .3.VI‬ﺗﻄﺒﻴﻘﺎت اﻷﻋﺪاد اﳌﺮﻛﺒﺔ‬
‫‪.‬‬
‫و ﺑﺎﻟﺘﺎﱄ‪ ،‬ﻓﺤﺴﺐ اﻟﺼﻴﻐﺔ اﻟﻌﺎﻣﺔ ﻟﻠﺠﺬور اﻟﻨﻮﻧﻴﺔ )ﺻﻔﺤﺔ ‪ ،(35‬ﻳﻜﻮن ‪:‬‬

‫‪Ŕž Ŧ Ľ‬‬
‫𝜋‪2𝜋/3 2k‬‬ ‫𝜋‪2𝜋/3 2k‬‬
‫‪z = cos‬‬ ‫‪+‬‬ ‫‪+ ı sin‬‬ ‫‪+‬‬
‫‪6‬‬ ‫‪6‬‬ ‫‪6‬‬ ‫‪6‬‬
‫𝜋‪𝜋 k‬‬ ‫𝜋‪𝜋 k‬‬

‫‪œǃ‬‬
‫‪= cos‬‬ ‫‪+‬‬ ‫‪+ ı sin‬‬ ‫‪+‬‬ ‫‪, 0≤k≤5.‬‬
‫‪9‬‬ ‫‪3‬‬ ‫‪9‬‬ ‫‪3‬‬

‫‪ •2‬ﻟﺪﻳﻨﺎ ‪:‬‬

‫‪Ŀ‬‬
‫‪1−ı‬‬ ‫‪√2 cos − + ı sin −‬‬ ‫‪√2 e−‬‬ ‫‪/‬‬
‫‪√2 −‬‬ ‫‪/‬‬
‫=‬ ‫=‬ ‫⋅‬ ‫‪/‬‬
‫=‬ ‫‪e‬‬ ‫‪⋅ e−‬‬ ‫‪/‬‬
‫‪1 + ı√3‬‬ ‫‪2 cos + ı sin‬‬ ‫‪2‬‬ ‫‪e‬‬ ‫‪2‬‬
‫‪√2 −‬‬ ‫‪/‬‬ ‫‪√2‬‬ ‫‪−‬‬ ‫‪√2‬‬ ‫‪/‬‬
‫=‬ ‫‪e‬‬ ‫=‬ ‫‪e‬‬ ‫=‬ ‫‪e‬‬
‫‪2‬‬ ‫‪2‬‬ ‫‪2‬‬
‫و ﺑﺎﻟﺘﺎﱄ‪ ،‬ﻓﺤﺴﺐ اﻟﺼﻴﻐﺔ اﻟﻌﺎﻣﺔ ﻟﻠﺠﺬور اﻟﻨﻮﻧﻴﺔ )ﺻﻔﺤﺔ ‪ ،(35‬ﻳﻜﻮن ‪:‬‬

‫‪√2‬‬ ‫𝜋‪17𝜋/12 2k‬‬ ‫𝜋‪17𝜋/12 2k‬‬


‫= ‪z‬‬ ‫‪cos‬‬ ‫‪+‬‬ ‫‪+ ı sin‬‬ ‫‪+‬‬
‫‪2‬‬ ‫‪4‬‬ ‫‪4‬‬ ‫‪4‬‬ ‫‪4‬‬
‫‪1‬‬ ‫𝜋‪17𝜋 k‬‬ ‫𝜋‪17𝜋 k‬‬
‫=‬ ‫‪cos‬‬ ‫‪+‬‬ ‫‪+ ı sin‬‬ ‫‪+‬‬ ‫‪, 0≤k≤3.‬‬
‫‪√2‬‬ ‫‪48‬‬ ‫‪2‬‬ ‫‪48‬‬ ‫‪2‬‬

‫■‬

‫‪.‬‬
‫‪−32‬‬
‫أوﺟﺪ اﳉﺬور اﻟﺴﺎﺑﻌﺔ ﻟﻠﻌﺪد ‪ ı‬و اﳉﺬور اﻟﺜﺎﻣﻨﺔ ﻟﻠﻌﺪد‬
‫‬ ‫‪.‬‬
‫𝟏𝟎𝟏‬ ‫‪.‬‬
‫‪1 + ı√3‬‬

‫‪ e +‬أي اﻷﻋﺪاد ‪:‬‬ ‫‪ّ ı=e‬‬


‫ﻓﺈن اﳉﺬور اﻟﺴﺎﺑﻌﺔ ﻟﻠﻌﺪد ‪ ı‬ﻫﻲ اﻷﻋﺪاد‬ ‫‪/‬‬ ‫ﺍﳊﻞّ‪ .‬ﺑﲈ ّ‬
‫أن‬
‫‪ņőƱŀ Ţ‬‬
‫‪ e + /‬ﻣﻊ }‪. k ∈ {0, 1, 2, 3, 4, 5, 6‬‬
‫ﻣﻦ ﺟﻬﺔ أﺧﺮى ‪:‬‬
‫‪−32‬‬ ‫‪−16‬‬
‫=‬ ‫‪= −16e−‬‬ ‫‪/‬‬ ‫‪= 16e−‬‬ ‫‪+‬‬
‫‪= 16e‬‬ ‫‪/‬‬
‫‪1 + ı√3 e /‬‬

‫‪VI‬‬
‫‪+‬‬ ‫‪−32‬‬
‫‪ √16e‬أي اﻷﻋﺪاد ‪:‬‬ ‫ﻫﻲ اﻷﻋﺪاد‬ ‫إذن اﳉﺬور اﻟﺜﺎﻣﻨﺔ ﻟﻠﻌﺪد‬
‫‪1 + ı√3‬‬
‫}‪. k ∈ {0, 1, 2, 3, 4, 5, 6, 7‬‬ ‫ﻣﻊ‬ ‫‪+‬‬ ‫‪/‬‬
‫‪√2e‬‬
‫■‬
‫‪ƕ‬‬

‫ﺣﻞ ﰲ اﳌﺠﻤﻮﻋﺔ ‪ ℂ‬اﳌﻌﺎدﻟﺔ ‪. (z − 2) ⋅ z − 3ız + 4 = 0 :‬‬ ‫ّ‬


‫‪.‬‬
‫𝟐𝟎𝟏‬ ‫‪.‬‬ ‫‬
‫ﺛﻢ أﺛﺒﺖ ّ‬
‫أن اﳌﺜﻠﺚ ‪ ABC‬ﻗﺎﺋﻢ‪.‬‬ ‫ﻧﺴﻤﻲ ‪ B ، A‬و ‪ C‬ﺻﻮر ﺣﻠﻮل ﻫﺬه اﳌﻌﺎدﻟﺔ‪ .‬أﻧﺸﺊ اﻟﻨﻘﻂ ‪ّ C ، B ، A‬‬
‫‪ũŏ‬‬

‫ﺍﳊﻞّ‪ .‬ﻟﺪﻳﻨﺎ ‪:‬‬


‫‪(z − 2) ⋅ z − 3ız + 4 = 0 ⟺ z − 2 = 0‬‬ ‫أو‬ ‫‪z − 3ız + 4 = 0‬‬

‫ﻟﻠﻤﻌﺎدﻟﺔ ‪ّ z − 2‬‬
‫ﺣﻞ وﺣﻴﺪ ﻫﻮ ‪. z = 2‬‬ ‫•‬

‫𝟬𝟲𝟭‬

‫‪http ://tinyurl.com/Malki1718‬‬ ‫‪0‬‬


‫‪ .VI‬ﲤﺎرﻳﻦ ﺗﻄﺒﻴﻘﻴﺔ‬
‫‪.‬‬
‫ُﳑ َ ﱢﻴﺰ اﳌﻌﺎدﻟﺔ ‪ z − 3ız + 4 = 0‬ﻫﻮ ‪:‬‬ ‫•‬
‫)‪Δ = (−3ı) − 4 × 4 = −9 − 16 = −25 = (5ı‬‬

‫‪Ŕž Ŧ Ľ‬‬
‫‪3ı + 5ı‬‬ ‫‪3ı − 5ı‬‬
‫= ‪.z‬‬ ‫= ‪ z‬و ‪= 4ı‬‬ ‫و ﺑﺎﻟﺘﺎﱄ ﳍﺎ ﺣﻼّن ﳘﺎ ‪= −ı‬‬
‫‪2‬‬ ‫‪2‬‬

‫‪œǃ‬‬
‫إذن‪ ،‬ﻟﻠﻤﻌﺎدﻟﺔ ﺛﻼﺛﺔ ﺣﻠﻮل ﻫﻲ ‪ −ı ، 2‬و ‪. 4ı‬‬
‫‪C‬‬
‫‪4‬‬

‫‪Ŀ‬‬
‫‪3‬‬
‫ﻧﻀﻊ )‪ B (−ı) ، A (2‬و )‪ . C (4ı‬ﻟﺪﻳﻨﺎ ‪:‬‬
‫‪z −z‬‬ ‫‪4ı − 2‬‬ ‫‪(2 − 4ı) (2 − ı) 4 − 2ı − 8ı − 4‬‬
‫‪2‬‬ ‫=‬ ‫=‬ ‫=‬ ‫‪= −2ı‬‬
‫‪z −z‬‬ ‫‪−ı − 2‬‬ ‫‪2 +1‬‬ ‫‪5‬‬

‫‪1‬‬ ‫و ﻫﻮ ﻋﺪد ﲣﻴﲇ ِﴏف‪.‬‬


‫‪A‬‬
‫ﺣﺴﺐ اﻟﻼزﻣﺔ ‪ 1‬ﺻﻔﺤﺔ ‪ ، 53‬ﻓﺎﳌﺴﺘﻘﻴﲈن )‪ (AC‬و )‪ (AB‬ﻣﺘﻌﺎﻣﺪان أي ّ‬
‫أن اﳌﺜﻠﺚ‬
‫‪1‬‬ ‫‪2‬‬
‫‪ ABC‬ﻗﺎﺋﻢ ﻋﻨﺪ اﻟﺮأس ‪. A‬‬
‫‪−1‬‬
‫‪B‬‬

‫■‬

‫‪. f(z) = z − 2 √3 + ı z + 4 1 + ı√3 z − 8ı‬‬ ‫ﻧﻌﺘﱪ اﻟﺪاﻟﺔ ‪ f‬اﳌﻌﺮﻓﺔ ﻋﲆ ‪ ℂ‬ﺑﺎﻟﻌﺒﺎرة‬


‫‪#‬‬ ‫‪.‬‬
‫𝟑𝟎𝟏‬ ‫‪.‬‬

‫‪. f(z) = (z − 2ı) z − 2√3z + 4‬‬ ‫‪ •1‬ﲢﻘﻖ ﻣﻦ ّ‬


‫أن‬

‫‪ •2‬ﺣﻞ ﰲ ‪ ℂ‬اﳌﻌﺎدﻟﺔ ‪ . f(z) = 0‬أﻛﺘﺐ اﳊﻠﻮل ‪ z ، z‬و ‪ z‬ﻋﲆ اﻟﺸﻜﻠﲔ اﳉﱪي و اﻷﳼ‪.‬‬

‫‪ •3‬ﻣ ﱢﺜﻞ ﰲ اﳌﺴﺘﻮي اﳌﺮﻛﺐ اﻟﻨﻘﻂ ‪ M ، M‬و ‪ M‬ﺻﻮر اﻷﻋﺪاد ‪ z ، z‬و ‪ z‬ﻋﲆ اﻟﱰﺗﻴﺐ ﺛﻢ أﺛﺒﺖ أﳖﺎ ﺗﻘﻊ ﻋﲆ داﺋﺮة‬
‫‪ņőƱŀ Ţ‬‬
‫ﻣﺮﻛﺰﻫﺎ ‪. O‬‬
‫‪ •4‬ﻣﺎ ﻫﻲ ﻃﺒﻴﻌﺔ اﻟﺮﺑﺎﻋﻲ ‪ OM M M‬؟‬
‫‪VI‬‬

‫‪ •1‬ﻟﺪﻳﻨﺎ ‪:‬‬ ‫ﺍﳊﻞّ‪.‬‬

‫‪f (2ı) = (2ı) − 2 √3 + ı (2ı) + 4 1 + ı√3 (2ı) − 8ı‬‬


‫‪ƕ‬‬

‫‪= −8ı + 8 √3 + ı + 8 1 + ı√3 − 8ı = 0‬‬

‫و ﺑﺎﻟﺘﺎﱄ )‪ f (z‬ﻳﻘﺒﻞ اﻟﻘﺴﻤﺔ ﻋﲆ ‪ . z − 2ı‬ﻧﻜﺘﺐ ‪:‬‬


‫‪f (z) = (z − 2ı) az + bz + c = az + (b − 2aı) z + (c − 2bı) z − 2cı‬‬
‫‪ũŏ‬‬

‫ﺑﺎﳌﻄﺎﺑﻘﺔ ﻳﻨﺘﺞ ‪ c − 2bı = 4 1 + ı√3 ، b − 2aı = −2 √3 + ı ، a = 1 :‬و ‪ −2cı = −8ı‬أي ‪، a = 1‬‬


‫‪ b = −2√3‬و ‪. c = 4‬‬
‫‪. f (z) = (z − 2ı) z − 2√3z + 4‬‬ ‫إذن ‪:‬‬

‫‪ f (z) = 0 •2‬إذا و ﻓﻘﻂ إذا ﻛﺎن ‪ z − 2ı = 0‬أو ‪. z − 2√3z + 4 = 0‬‬

‫𝟭𝟲𝟭‬

‫‪http ://tinyurl.com/Malki1718‬‬ ‫‪0‬‬


‫‪ .3.VI‬ﺗﻄﺒﻴﻘﺎت اﻷﻋﺪاد اﳌﺮﻛﺒﺔ‬
‫‪.‬‬
‫‪M1‬‬
‫‪2‬‬

‫‪Ŕž Ŧ Ľ‬‬
‫‪M3‬‬

‫‪œǃ‬‬
‫‪1‬‬

‫‪O‬‬ ‫‪1‬‬ ‫‪2‬‬

‫‪Ŀ‬‬
‫‪−2‬‬ ‫‪−1‬‬

‫‪−1‬‬ ‫‪M2‬‬

‫‪−2‬‬

‫ﺷﻜﻞ ‪12.VI‬‬

‫ﻟﻠﻤﻌﺎدﻟﺔ اﻷوﱃ ﺣﻞ وﺣﻴﺪ ﻫﻮ ‪ z = 2ı‬؛‬ ‫•‬

‫‪ Δ = −√3‬و ﺑﺎﻟﺘﺎﱄ ﳍﺎ ﺣﻼّن ﳘﺎ ‪ z = √3 − ı‬و‬ ‫‪− 4 = −1 = ı‬‬ ‫ا ُﳌﻤ ﱢﻴﺰ اﳌﺨﺘﴫ ﻟﻠﻤﻌﺎدﻟﺔ اﻟﺜﺎﻧﻴﺔ ﻫﻮ‬ ‫•‬

‫‪. z = √3 + ı‬‬
‫اﻟﺸﻜﻞ اﻷﳼ ﳍﺬه اﳊﻠﻮل ﻫﻮ ‪:‬‬
‫𝜋‬ ‫𝜋‬
‫‪z = 2ı = 2 cos‬‬ ‫‪+ ı sin‬‬ ‫‪= 2e /‬‬
‫‪2‬‬ ‫‪2‬‬
‫‪√3‬‬ ‫‪1‬‬ ‫𝜋‬ ‫𝜋‬
‫‪z = √3 − ı = 2‬‬ ‫‪− sin ı = 2 cos − ı sin‬‬ ‫‪= 2e−‬‬ ‫‪/‬‬
‫‪2‬‬ ‫‪2‬‬ ‫‪6‬‬ ‫‪6‬‬

‫‪√3‬‬ ‫‪1‬‬ ‫𝜋‬ ‫𝜋‬


‫‪ņőƱŀ Ţ‬‬
‫‪z = √3 + ı = 2‬‬ ‫‪+ sin ı‬‬ ‫‪= 2 cos‬‬ ‫‪+ ı sin‬‬ ‫‪= 2e‬‬ ‫‪/‬‬
‫‪2‬‬ ‫‪2‬‬ ‫‪6‬‬ ‫‪6‬‬

‫‪ •3‬أﻧﻈﺮ اﻟﺸﻜﻞ ‪ . 12.VI‬ﻟﺪﻳﻨﺎ ‪:‬‬

‫‪VI‬‬
‫‪OM = ||z − 0|| = ||z || = 2‬‬
‫‪OM = ||z − 0|| = ||z || = 2‬‬
‫‪OM = ||z − 0|| = ||z || = 2‬‬

‫أي ‪ OM = OM = OM‬إذن اﻟﻨﻘﻂ ‪ M ، M‬و ‪ M‬ﺗﻘﻊ ﻋﲆ داﺋﺮة ﻣﺮﻛﺰﻫﺎ ‪) O‬و ﻧﺼﻒ ﻗﻄﺮﻫﺎ ‪. (2‬‬
‫‪ƕ‬‬

‫‪ •4‬ﻟﺪﻳﻨﺎ ‪ M OM = 𝜋 − − 𝜋 = 𝜋 (mod 2𝜋) :‬و ﺑﲈ ّ‬


‫أن ‪ OM = OM‬ﻓﺎﳌﺜﻠﺚ ‪ OM M‬ﻣﺘﻘﺎﻳﺲ‬
‫‪6‬‬ ‫‪6‬‬ ‫‪3‬‬
‫اﻷﺿﻼع أي ‪. M M = OM = OM‬‬
‫𝜋‬ ‫𝜋‬ ‫𝜋‬
‫‪ũŏ‬‬

‫ﺑﺎﳌﺜﻞ‪ M OM = − = (mod 2𝜋) ،‬ﻣﻊ ‪ OM = OM‬و ﺑﺎﻟﺘﺎﱄ ﻓﺎﳌﺜﻠﺚ ‪ OM M‬ﻣﺘﻘﺎﻳﺲ‬


‫‪2‬‬ ‫‪6‬‬ ‫‪3‬‬
‫اﻷﺿﻼع ﻣﻨﻪ ‪. M M = OM = OM‬‬
‫ﻣﻌﲔ )أو ﻣﺮ ّﺑﻊ( ‪.‬‬
‫ﰲ اﻷﺧﲑ‪ ،‬أﺿﻼع اﻟﺮﺑﺎﻋﻲ ‪ OM M M‬ﻛﻠﻬﺎ ﻣﺘﻘﺎﻳﺴﺔ إذن ﻓﻬﻮ ّ‬
‫𝜋‬ ‫𝜋‬ ‫𝜋‬
‫‪M M‬‬ ‫ﻣﻌﲔ و ﻟﻴﺲ ﻣﺮ ّﺑﻊ )و ﺑﺎﻟﺘﺎﱄ ُﻗﻄﺮاه ‪ OM‬و‬
‫= ‪ M OM‬إذن ‪ّ OM M M‬‬ ‫‪− −‬‬ ‫>‬ ‫ﻟﻜﻦ‬
‫‪2‬‬ ‫‪6‬‬ ‫‪2‬‬
‫ﻣﺘﻌﺎﻣﺪان( ‪.‬‬
‫■‬

‫𝟮𝟲𝟭‬

‫‪http ://tinyurl.com/Malki1718‬‬ ‫‪0‬‬


‫‪ .VI‬ﲤﺎرﻳﻦ ﺗﻄﺒﻴﻘﻴﺔ‬
‫‪.‬‬

‫‬ ‫‪.‬‬
‫𝟒𝟎𝟏‬ ‫‪.‬‬

‫‪Ŕž Ŧ Ľ‬‬
‫ّ‬
‫ﺣﻞ ﰲ اﳌﺠﻤﻮﻋﺔ ‪ ℂ‬ﻛﻼ ﻣﻦ اﳌﻌﺎدﻻت اﻟﺘﺎﻟﻴﺔ ‪:‬‬
‫‪z − 4 (6 + ı) z + 3 (63 + 16ı) = 0‬‬ ‫‪9‬‬ ‫‪z + 18z + 1681 = 0‬‬ ‫‪1‬‬

‫‪œǃ‬‬
‫•‬ ‫•‬

‫‪ız − 2z − 4ı − 4 = 0‬‬ ‫•‬ ‫‪10‬‬ ‫‪z + 6z + 25 = 0‬‬ ‫•‬‫‪2‬‬


‫‪z − 4 (6 + ı) z + 3 (8ı − 1) = 0‬‬ ‫•‬ ‫‪11‬‬ ‫‪z − 2z + 4 = 0‬‬ ‫•‬‫‪3‬‬

‫‪Ŀ‬‬
‫‪z + (10ı − 7) z − 11 − 41ı = 0‬‬ ‫•‬ ‫‪12‬‬ ‫‪z − (5 + 3ı) z + 7ı + 4 = 0‬‬ ‫•‬‫‪4‬‬
‫‪ız + (4ı − 3) z + ı − 5 = 0‬‬ ‫•‬ ‫‪13‬‬ ‫‪z + (9ı − 5) z − 14 − 23ı = 0‬‬ ‫•‬‫‪5‬‬
‫‪ız + (1 + ı) z + 1 = 0‬‬ ‫•‬ ‫‪14‬‬ ‫‪(4 − 3ı) z − (10 + 5ı) z + 6ı = 0‬‬ ‫•‬‫‪6‬‬
‫‪z + 2 (1 − ı) z − 1 = 0‬‬ ‫•‬ ‫‪15‬‬ ‫‪z − (1 + 2ı) z + 3 (1 + ı) = 0‬‬ ‫•‬‫‪7‬‬
‫‪z − 2ız − 1 + 2ı = 0‬‬ ‫•‬ ‫‪16‬‬ ‫‪z − (3 + 2ı) z + 5 + ı = 0‬‬ ‫•‬‫‪8‬‬

‫‪ُ •1‬ﳑ ﱢﻴﺰ اﳌﻌﺎدﻟﺔ ﻫﻮ ‪:‬‬ ‫ﺍﳊﻞّ‪.‬‬


‫)‪Δ = 18 − 4 × 1681 = −6400 = (80ı‬‬
‫إذن ﻟﻠﻤﻌﺎدﻟﺔ ﺣﻼّن ﳘﺎ ‪:‬‬
‫‪−18 − 80ı‬‬ ‫‪−1 + 40ı‬‬
‫= ‪z‬‬ ‫‪= −9 − 40ı‬‬ ‫و‬ ‫= ‪z‬‬ ‫‪= −9 + 40ı‬‬
‫‪2‬‬ ‫‪2‬‬
‫أي ﳎﻤﻮﻋﺔ اﳊﻠﻮل ﻫﻲ ‪. 𝒮 = {−9 − 40ı, −9 + 40ı} :‬‬

‫‪ •2‬ا ُﳌﻤ ﱢﻴﺰ اﳌﺨﺘﴫ ﻟﻠﻤﻌﺎدﻟﺔ ﻫﻮ ‪:‬‬


‫)‪Δ = 3 − 25 = −16 = (4ı‬‬
‫‪ņőƱŀ Ţ‬‬
‫إذن ﻟﻠﻤﻌﺎدﻟﺔ ﺣﻼّن ﳘﺎ ‪:‬‬
‫‪z = −3 − 4ı‬‬ ‫و‬ ‫‪z = −3 + 4ı‬‬
‫أي ﳎﻤﻮﻋﺔ اﳊﻠﻮل ﻫﻲ ‪. 𝒮 = {−3 − 4ı, −3 + 4ı} :‬‬
‫‪VI‬‬

‫‪ •3‬ا ُﳌﻤ ﱢﻴﺰ اﳌﺨﺘﴫ ﻟﻠﻤﻌﺎدﻟﺔ ﻫﻮ ‪:‬‬


‫‪Δ = (−1) − 4 = −3 = ı√3‬‬

‫إذن ﻟﻠﻤﻌﺎدﻟﺔ ﺣﻼّن ﳘﺎ ‪:‬‬


‫‪z = 1 − √3ı‬‬ ‫و‬ ‫‪z = 1 + √3ı‬‬
‫‪ƕ‬‬

‫أي ﳎﻤﻮﻋﺔ اﳊﻠﻮل ﻫﻲ ‪. 𝒮 = 1 − √3ı, 1 + √3ı :‬‬

‫‪ُ •4‬ﳑ ﱢﻴﺰ اﳌﻌﺎدﻟﺔ ﻫﻮ ‪:‬‬


‫‪ũŏ‬‬

‫‪/‬‬ ‫‪/‬‬ ‫‪1+ı‬‬


‫‪Δ = (− (5 + 3ı)) − 4 (4 + 7ı) = 2ı = 2e‬‬ ‫‪= √2e‬‬ ‫=‬ ‫‪√2‬‬ ‫)‪= (1 + ı‬‬
‫‪√2‬‬
‫إذن ﻟﻠﻤﻌﺎدﻟﺔ ﺣﻼّن ﳘﺎ ‪:‬‬
‫‪5 + 3ı − 1 − ı‬‬ ‫‪5 + 3ı + 1 + ı‬‬
‫= ‪z‬‬ ‫‪=2+ı‬‬ ‫و‬ ‫= ‪z‬‬ ‫‪= 3 + 2ı‬‬
‫‪2‬‬ ‫‪2‬‬
‫أي ﳎﻤﻮﻋﺔ اﳊﻠﻮل ﻫﻲ ‪. 𝒮 = {2 + ı, 3 + 2ı} :‬‬

‫𝟯𝟲𝟭‬

‫‪http ://tinyurl.com/Malki1718‬‬ ‫‪0‬‬


‫‪ .3.VI‬ﺗﻄﺒﻴﻘﺎت اﻷﻋﺪاد اﳌﺮﻛﺒﺔ‬
‫‪.‬‬
‫‪ُ •5‬ﳑ ﱢﻴﺰ اﳌﻌﺎدﻟﺔ ﻫﻮ ‪:‬‬
‫)‪Δ = (−5 + 9ı) − 4 (−14 − 23ı) = 2ı = (1 + ı‬‬

‫‪Ŕž Ŧ Ľ‬‬
‫إذن ﻟﻠﻤﻌﺎدﻟﺔ ﺣﻼّن ﳘﺎ ‪:‬‬
‫‪5 − 9ı − 1 − ı‬‬ ‫‪5 − 9ı + 1 + ı‬‬
‫= ‪z‬‬ ‫‪= 2 − 5ı‬‬ ‫و‬ ‫= ‪z‬‬ ‫‪= 3 − 4ı‬‬

‫‪œǃ‬‬
‫‪2‬‬ ‫‪2‬‬
‫أي ﳎﻤﻮﻋﺔ اﳊﻠﻮل ﻫﻲ ‪. 𝒮 = {2 − 5ı, 3 − 4ı} :‬‬

‫‪ُ •6‬ﳑ ﱢﻴﺰ اﳌﻌﺎدﻟﺔ ﻫﻮ ‪:‬‬

‫‪Ŀ‬‬
‫)‪Δ = (− (10 + 5ı)) − 4 (4 − 3ı) (6ı) = 3 + 4ı = 2 + 2 × 2 × ı + ı = (2 + ı‬‬
‫إذن ﻟﻠﻤﻌﺎدﻟﺔ ﺣﻼّن ﳘﺎ ‪:‬‬
‫‪10 + 5ı − 2 − ı 2 + 4ı‬‬ ‫‪10 + 5ı + 2 + ı 3 + 6ı‬‬
‫= ‪z‬‬ ‫=‬ ‫و‬ ‫= ‪z‬‬ ‫=‬
‫)‪2 (4 − 3ı‬‬ ‫‪5‬‬ ‫)‪2 (4 − 3ı‬‬ ‫‪5‬‬
‫‪2 4 3 6‬‬
‫= 𝒮‪.‬‬ ‫‪+ ı, + ı‬‬ ‫أي ﳎﻤﻮﻋﺔ اﳊﻠﻮل ﻫﻲ ‪:‬‬
‫‪5 5 5 5‬‬

‫‪ُ •7‬ﳑ ﱢﻴﺰ اﳌﻌﺎدﻟﺔ ﻫﻮ ‪:‬‬


‫)‪Δ = (− (1 + 2ı)) − 4 × 3 (1 + ı) = −15 − 8ı = 1 − 2 × 4ı + (4ı) = (1 − 4ı‬‬
‫إذن ﻟﻠﻤﻌﺎدﻟﺔ ﺣﻼّن ﳘﺎ ‪:‬‬
‫‪1 + 2ı − 1 + 4ı‬‬ ‫‪1 + 2ı + 1 − 4ı‬‬
‫= ‪z‬‬ ‫‪= 3ı‬‬ ‫و‬ ‫= ‪z‬‬ ‫‪=1−ı‬‬
‫‪2‬‬ ‫‪2‬‬
‫أي ﳎﻤﻮﻋﺔ اﳊﻠﻮل ﻫﻲ ‪. 𝒮 = {3ı, 1 − ı} :‬‬

‫‪ُ •8‬ﳑ ﱢﻴﺰ اﳌﻌﺎدﻟﺔ ﻫﻮ ‪:‬‬


‫)‪Δ = (3 + 2ı) − 4 (5 + ı) = 15 − 8ı = 1 + 2 × 4ı + (4ı) = (1 + 4ı‬‬
‫إذن ﻟﻠﻤﻌﺎدﻟﺔ ﺣﻼّن ﳘﺎ ‪:‬‬
‫‪3 + 2ı − 1 − 4ı‬‬ ‫‪3 + 2ı + 1 + 4ı‬‬
‫= ‪z‬‬ ‫‪=1−ı‬‬ ‫و‬ ‫= ‪z‬‬ ‫‪= 2 + 3ı‬‬
‫‪2‬‬ ‫‪2‬‬
‫‪ņőƱŀ Ţ‬‬
‫أي ﳎﻤﻮﻋﺔ اﳊﻠﻮل ﻫﻲ ‪. 𝒮 = {1 − ı, 2 + 3ı} :‬‬

‫‪ •9‬ا ُﳌﻤ ﱢﻴﺰ اﳌﺨﺘﴫ ﻟﻠﻤﻌﺎدﻟﺔ ﻫﻮ ‪:‬‬


‫)‪Δ = (−2 (6 + ı)) − 3 (63 + 16ı) = −49 = (7ı‬‬

‫‪VI‬‬
‫إذن ﻟﻠﻤﻌﺎدﻟﺔ ﺣﻼّن ﳘﺎ ‪:‬‬
‫‪z = 2 (6 + ı) − 7ı = 12 − 5ı‬‬ ‫و‬ ‫‪z = 2 (6 + ı) + 7ı = 12 + 9ı‬‬
‫أي ﳎﻤﻮﻋﺔ اﳊﻠﻮل ﻫﻲ ‪. 𝒮 = {12 − 5ı, 12 + 9ı} :‬‬
‫‪ƕ‬‬

‫‪ •10‬ا ُﳌﻤ ﱢﻴﺰ اﳌﺨﺘﴫ ﻟﻠﻤﻌﺎدﻟﺔ ﻫﻮ ‪:‬‬


‫)‪Δ = (−1) − ı (−4 − 4ı) = −3 + 4ı = 1 + 2 × 2ı + (2ı) = (1 + 2ı‬‬
‫إذن ﻟﻠﻤﻌﺎدﻟﺔ ﺣﻼّن ﳘﺎ ‪:‬‬
‫‪ũŏ‬‬

‫‪1 − 1 − 2ı‬‬ ‫‪1 + 1 + 2ı‬‬


‫= ‪z‬‬ ‫‪= −2‬‬ ‫و‬ ‫= ‪z‬‬ ‫‪= 2 − 2ı‬‬
‫‪ı‬‬ ‫‪ı‬‬
‫أي ﳎﻤﻮﻋﺔ اﳊﻠﻮل ﻫﻲ ‪. 𝒮 = {−2, 2 − 2ı} :‬‬

‫𝟰𝟲𝟭‬

‫‪http ://tinyurl.com/Malki1718‬‬ ‫‪0‬‬


‫‪ .VI‬ﲤﺎرﻳﻦ ﺗﻄﺒﻴﻘﻴﺔ‬
‫‪.‬‬
‫‪ •11‬ا ُﳌﻤ ﱢﻴﺰ اﳌﺨﺘﴫ ﻟﻠﻤﻌﺎدﻟﺔ ﻫﻮ ‪:‬‬
‫)‪Δ = (−2 (6 + ı)) − 3 (−1 + 8ı) = 143 + 24ı = 12 + 2 × 12 × ı + ı = (12 + ı‬‬

‫‪Ŕž Ŧ Ľ‬‬
‫إذن ﻟﻠﻤﻌﺎدﻟﺔ ﺣﻼّن ﳘﺎ ‪:‬‬
‫‪z = 2 (6 + ı) − 12 − ı = ı‬‬ ‫و‬ ‫‪z = 2 (6 + ı) + 12 + ı = 24 + 3ı‬‬

‫‪œǃ‬‬
‫أي ﳎﻤﻮﻋﺔ اﳊﻠﻮل ﻫﻲ ‪. 𝒮 = {ı, 24 + 3ı} :‬‬
‫‪ُ •12‬ﳑ ﱢﻴﺰ اﳌﻌﺎدﻟﺔ ﻫﻮ ‪:‬‬
‫)‪Δ = (−7 + 10ı) − 4 (−11 − 41ı) = −7 + 24ı = 3 + 2 × 3 × 4ı + (4ı) = (3 + 4ı‬‬

‫‪Ŀ‬‬
‫إذن ﻟﻠﻤﻌﺎدﻟﺔ ﺣﻼّن ﳘﺎ ‪:‬‬
‫‪7 − 10ı − 3 − 4ı‬‬ ‫‪7 − 10ı + 3 + 4ı‬‬
‫= ‪z‬‬ ‫‪= 2 − 7ı‬‬ ‫و‬ ‫= ‪z‬‬ ‫‪= 5 − 3ı‬‬
‫‪2‬‬ ‫‪2‬‬
‫أي ﳎﻤﻮﻋﺔ اﳊﻠﻮل ﻫﻲ ‪. 𝒮 = {2 − 7ı, 5 − 3ı} :‬‬
‫‪ُ •13‬ﳑ ﱢﻴﺰ اﳌﻌﺎدﻟﺔ ﻫﻮ ‪:‬‬
‫)‪Δ = (−3 + 4ı) − 4ı (−5 + ı) = −3 − 4ı = 1 − 2 × 2ı + (2ı) = (1 − 2ı‬‬
‫إذن ﻟﻠﻤﻌﺎدﻟﺔ ﺣﻼّن ﳘﺎ ‪:‬‬
‫‪3 − 4ı − 1 + 2ı‬‬ ‫‪3 − 4ı + 1 − 2ı‬‬
‫= ‪z‬‬ ‫‪= −1 − ı‬‬ ‫و‬ ‫= ‪z‬‬ ‫‪= −3 − 2ı‬‬
‫‪2ı‬‬ ‫‪2ı‬‬
‫أي ﳎﻤﻮﻋﺔ اﳊﻠﻮل ﻫﻲ ‪. 𝒮 = {−1 − ı, −3 − 2ı} :‬‬
‫‪ُ •14‬ﳑ ﱢﻴﺰ اﳌﻌﺎدﻟﺔ ﻫﻮ ‪:‬‬
‫)‪Δ = (1 + ı) − 4 × ı × 1 = 1 + 2ı + ı − 4ı = 1 − 2ı + ı = (1 − ı‬‬
‫إذن ﻟﻠﻤﻌﺎدﻟﺔ ﺣﻼّن ﳘﺎ ‪:‬‬
‫)‪− (1 + ı) − (1 − ı‬‬ ‫)‪− (1 + ı) + (1 − ı‬‬
‫= ‪z‬‬ ‫‪=ı‬‬ ‫و‬ ‫= ‪z‬‬ ‫‪= −1‬‬
‫‪2ı‬‬ ‫‪2ı‬‬
‫أي ﳎﻤﻮﻋﺔ اﳊﻠﻮل ﻫﻲ ‪. 𝒮 = {−1, ı} :‬‬
‫‪ُ •15‬ﳑ ﱢﻴﺰ اﳌﻌﺎدﻟﺔ ﻫﻮ ‪:‬‬
‫‪ņőƱŀ Ţ‬‬
‫‪Δ = 4 (1 − ı) − 4 × 1 × (−1) = 4 1 − 2ı − ı + 1 = 4 (1 − 2ı) = 4Δ‬‬
‫ﺣﻴﺚ ‪ . Δ = 1 − 2ı‬ﻟﻴﻜﻦ ‪ 𝛿 = x + ıy‬ﺟﺬر ًا ﺗﺮﺑﻴﻌﻴ ًﺎ ﻟﻠﻌﺪد ‪ . Δ‬ﻟﺪﻳﻨﺎ ‪ 𝛿 = Δ‬ﻣﻨﻪ ‪:‬‬
‫‪1 + √5‬‬
‫‪VI‬‬

‫⎧‬
‫⎪‬ ‫= ‪x‬‬ ‫‪=a‬‬
‫‪x −y =1‬‬ ‫⎧‬ ‫‪2x‬‬ ‫=‬ ‫‪1‬‬ ‫‪+‬‬ ‫√‬ ‫‪5‬‬ ‫⎪‬
‫⎪‬
‫⎧‬
‫⎪‬ ‫⎪‬ ‫⎪‬
‫⎪‬ ‫‪2‬‬
‫⟺ ‪𝛿 =Δ‬‬ ‫⟺ ‪2xy = −2‬‬ ‫‪xy = −1‬‬ ‫⟺‬ ‫‪xy = −1‬‬
‫⎨‬
‫⎪‬ ‫⎨‬
‫⎪‬ ‫⎨‬
‫⎪‬
‫⎪‬
‫⎪‬
‫‪⎩ x + y = √5‬‬ ‫‪⎩ x + y = √5‬‬ ‫‪⎪ y = −1 + √5 = b‬‬
‫⎪‬
‫⎩‬ ‫‪2‬‬
‫‪ƕ‬‬

‫ﻧﺨﺘﺎر ‪ x‬و ‪ y‬ﺑﺤﻴﺚ ﻳﻜﻮن ‪ xy < 0‬؛ ﻣﺜ ً‬


‫ﻼ ‪ x = −√a‬و ‪ y = √b‬و ﺑﺎﻟﺘﺎﱄ ‪:‬‬
‫‪Δ = 4Δ = 4𝛿 = 4 −√a + ı√b‬‬ ‫‪= 2 −√a + ı√b‬‬

‫إذن ﻟﻠﻤﻌﺎدﻟﺔ ﺣﻼّن ﳘﺎ ‪:‬‬


‫‪ũŏ‬‬

‫‪−2 (1 − ı) − 2 −√a + ı√b‬‬ ‫‪√10 + √2‬‬ ‫‪√10 − √2‬‬


‫= ‪z‬‬ ‫‪= −1 + ı + √a − ı√b = −1 −‬‬ ‫‪+‬‬ ‫‪1+‬‬ ‫‪ı‬‬
‫‪2‬‬ ‫‪2‬‬ ‫‪2‬‬

‫‪−2 (1 − ı) + 2 −√a + ı√b‬‬ ‫‪√10 + √2‬‬ ‫‪√10 − √2‬‬


‫= ‪z‬‬ ‫‪= −1 + ı − √a + ı√b = −1 +‬‬ ‫‪+‬‬ ‫‪1−‬‬ ‫‪ı‬‬
‫‪2‬‬ ‫‪2‬‬ ‫‪2‬‬

‫𝟱𝟲𝟭‬

‫‪http ://tinyurl.com/Malki1718‬‬ ‫‪0‬‬


‫‪ .3.VI‬ﺗﻄﺒﻴﻘﺎت اﻷﻋﺪاد اﳌﺮﻛﺒﺔ‬
‫‪.‬‬
‫‪+‬‬ ‫‪−‬‬ ‫‪+‬‬ ‫‪−‬‬
‫‪.𝒮 = −1 −‬‬ ‫‪+ 1+‬‬ ‫‪ı, −1 +‬‬ ‫‪+ 1−‬‬ ‫‪ı‬‬ ‫ﻣﻨﻪ ﳎﻤﻮﻋﺔ اﳊﻠﻮل ‪:‬‬

‫‪Ŕž Ŧ Ľ‬‬
‫‪ •16‬ﻟﺪﻳﻨﺎ ‪ . Δ = (−2ı) − 4 (−1 + 2ı) − 4 + 4 − 8ı = −8ı :‬ﻟﻨﺒﺤﺚ ﻋﻦ ﺟﺬر ﺗﺮﺑﻴﻌﻲ ‪ 𝛿 = a + ıb‬ﻟﻠﻌﺪد ‪ Δ‬أي‬
‫⎧‬
‫⎪‬ ‫‪a + b = ||Δ|| = 8‬‬
‫⎪‬

‫‪œǃ‬‬
‫‪. a −b =0‬‬ ‫ﺑﺤﻴﺚ ‪ . 𝛿 = Δ = −8ı‬ﻫﺬا اﻟﻌﺪد ُﳛﻘﻖ ‪:‬‬
‫⎨‬
‫⎪‬
‫⎪‬
‫⎩‬ ‫‪2ab = −8‬‬
‫ﻣﻦ اﳌﻌﺎدﻟﺔ اﻟﺜﺎﻧﻴﺔ ﻳﻨﺘﺞ ‪ b = a‬و ﺑﺎﻟﺘﻌﻮﻳﺾ ﰲ اﻷوﱃ ﻧﺠﺪ ‪ 2a = 8‬ﻣﻨﻪ ‪ . a = ±2‬ﻧﺨﺘﺎر ﻣﺜ ً‬
‫ﻼ ‪ a = 2‬ﻓﻴﻜﻮن‬

‫‪Ŀ‬‬
‫)ﻣﻦ اﳌﻌﺎدﻟﺔ اﻟﺜﺎﻟﺜﺔ( ‪. b = −2‬‬
‫‪2ı + 2 − 2ı‬‬ ‫‪2ı − 2 + 2ı‬‬
‫= ‪.z‬‬ ‫= ‪z‬و‪= 1‬‬ ‫أﺧﲑ ًا ‪= −1 + 2ı :‬‬
‫‪2‬‬ ‫‪2‬‬
‫ﳎﻤﻮﻋﺔ ﺣﻠﻮل اﳌﻌﺎدﻟﺔ اﻷوﱃ ﻫﻲ ‪. S = {1, −1 + 2ı} :‬‬
‫■‬

‫‪#‬‬ ‫‪.‬‬
‫𝟓𝟎𝟏‬ ‫‪.‬‬
‫‪ •1‬أوﺟﺪ اﳉﺬرﻳﻦ اﻟﱰﺑﻴﻌﻴﲔ اﳌﺮﻛﺒﲔ ﻟﻠﻌﺪد ‪. 5 − 12ı‬‬
‫‪ •2‬ﺣﻞ اﳌﻌﺎدﻟﺔ ‪ z − (1 + 2ı) z + 3 (1 + ı) z − 10 (1 + ı) = 0‬ﻋﻠ ًﲈ ّ‬
‫أن ﳍﺎ ﺣﻼ ﲣﻴﻠﻴﺎ ﴏﻓﺎ‪.‬‬
‫‪ •3‬ﻣﺎ ﻫﻲ ﻃﺒﻴﻌﺔ اﳌﺜﻠﺚ اﻟﺬي ﻟﻮاﺣﻖ رؤوﺳﻪ ﻫﻲ ﺣﻠﻮل اﳌﻌﺎدﻟﺔ اﻟﺴﺎﺑﻘﺔ ؟‬

‫‪ •1‬ﻟﻴﻜﻦ ‪ u‬ﺟﺬرا ﺗﺮﺑﻴﻌﻴﺎ ﻟﻠﻌﺪد ‪ . 5 − 12ı‬ﻧﻀﻊ ‪ u = a + ıb‬ﻣﻨﻪ ‪:‬‬ ‫ﺍﳊﻞّ‪.‬‬


‫⎧‬
‫⎪‬
‫= ||‪⎪ a + b = ||5 − 12ı‬‬ ‫‪5 + 12 = 13‬‬
‫⟺ ‪u = 5 − 12ı‬‬ ‫‪a −b =5‬‬
‫⎨‬
‫⎪‬
‫⎪‬
‫⎩‬ ‫‪2ab = −12‬‬
‫‪ņőƱŀ Ţ‬‬
‫ﺑﺠﻤﻊ اﳌﻌﺎدﻟﺘﲔ اﻷوﱃ و اﻟﺜﺎﻧﻴﺔ ﻳﻨﺘﺞ ‪ 2a = 13 + 5 = 18‬ﻣﻨﻪ ‪ . a = ±3‬ﻧﺨﺘﺎر ﻣﺜ ً‬
‫ﻼ ‪ a = 3‬ﻓﻴﻜﻮن )ﻣﻦ اﳌﻌﺎدﻟﺔ‬
‫اﻟﺜﺎﻟﺜﺔ( ‪ . b = −2‬أﺧﲑ ًا ‪ u = −3 + 2ı :‬و ‪. u = 3 − 2ı‬‬
‫إذن اﳉﺬرﻳﻦ اﻟﱰﺑﻴﻌﻴﲔ ﻟﻠﻌﺪد ‪ 5 − 12ı‬ﳘﺎ ‪ u = −3 + 2ı‬و ‪. u = 3 − 2ı‬‬
‫‪ •2‬ﻧﻀﻊ )‪ . P(z) = z − (1 + 2ı) z + 3 (1 + ı) z − 10 (1 + ı‬ﻟﻴﻜﻦ 𝛼‪ ، z = ı‬ﻣﻊ ‪ ، 𝛼 ∈ ℝ‬اﳊﻞ اﻟﺘﺨﻴﲇ‬ ‫‪VI‬‬
‫اﻟﴫف ﻟﻠﻤﻌﺎدﻟﺔ ‪ . P(z) = 0‬ﻟﺪﻳﻨﺎ ‪:‬‬
‫‪P (ı𝛼) = 0 ⟺ (ı𝛼) − (1 + 2ı) (ı𝛼) + 3 (1 + ı) (ı𝛼) − 10 (1 + ı) = 0‬‬
‫‪ƕ‬‬

‫‪⟺ −ı𝛼 + (1 + 2ı) 𝛼 + 3 (−1 + ı) 𝛼 − 10 (1 + ı) = 0‬‬


‫⟺‬ ‫‪𝛼 − 3𝛼 − 10 + ı −𝛼 + 2𝛼 + 3𝛼 − 10 = 0‬‬
‫‪⟺ 𝛼 − 3𝛼 − 10 = 0‬‬ ‫و‬ ‫‪− 𝛼 + 2𝛼 + 3𝛼 − 10 = 0‬‬
‫‪ũŏ‬‬

‫ﻟﻠﻤﻌﺎدﻟﺔ ‪ 𝛼 − 3𝛼 − 10 = 0‬ﺣﻼن ﳘﺎ ‪ 𝛼 = −2‬و ‪ 𝛼 = 5‬ﻟﻜﻦ ‪ 𝛼 = −2‬ﻫﻮ اﻟﻮﺣﻴﺪ اﻟﺬي ُﳛﻘﻖ اﳌﻌﺎدﻟﺔ اﻷﺧﺮى‪،‬‬
‫إذن ‪ z = −2ı‬ﻫﻮ اﳊﻞ اﻟﺘﺨﻴﲇ اﻟﴫف ﻟﻠﻤﻌﺎدﻟﺔ ‪. P(z) = 0‬‬
‫ﻧﺴﺘﻨﺘﺞ ّ‬
‫أن )‪ P(z‬ﻳﻘﺒﻞ اﻟﻘﺴﻤﺔ ﻋﲆ ‪ . x + 2ı‬ﺑﺎﻟﻘﺴﻤﺔ اﻹﻗﻠﻴﺪﻳﺔ )أو ﺑﻄﺮﻳﻘﺔ أﺧﺮى( ﻧﺠﺪ‬
‫‪. P(z) = (z + 2ı) z − (1 + 4ı)z − 5 + 5ı‬‬
‫اﳊﻠﻮل اﻷﺧﺮى ﻫﻲ ﺣﻠﻮل اﳌﻌﺎدﻟﺔ ‪. z − (1 + 4ı)z − 5 + 5ı = 0‬‬

‫𝟲𝟲𝟭‬

‫‪http ://tinyurl.com/Malki1718‬‬ ‫‪0‬‬


‫‪ .VI‬ﲤﺎرﻳﻦ ﺗﻄﺒﻴﻘﻴﺔ‬
‫‪.‬‬
‫ﻟﺪﻳﻨﺎ ‪ Δ = (−1 − 4ı) − 4 × 1 × (−5 + 5ı) = 5 − 12ı :‬و ﺣﺴﺐ اﻟﺴﺆال اﻟﺴﺎﺑﻖ ّ‬
‫ﻓﺈن أﺣﺪ ﺟﺬرﻳﻪ اﻟﱰﺑﻴﻌﻴﲔ‬
‫‪1 + 4ı + 3 − 2ı‬‬ ‫‪1 + 4ı − 3 + 2ı‬‬
‫= ‪.z‬‬ ‫= ‪z‬و‪= 2+ı‬‬ ‫ﻫﻮ ‪ 3 − 2ı‬ﻣﻨﻪ ‪= −1 + 3ı‬‬

‫‪Ŕž Ŧ Ľ‬‬
‫‪2‬‬ ‫‪2‬‬
‫}‪. S = {−2ı, −1 + 3ı, 2 + ı‬‬ ‫أﺧﲑ ًا‪ ،‬ﳎﻤﻮﻋﺔ ﺣﻠﻮل اﳌﻌﺎدﻟﺔ ‪ P(z) = 0‬ﻫﻲ‬

‫‪œǃ‬‬
‫•‬ ‫‪3‬‬
‫‪4‬‬
‫ﻫﻨﺎ‪ ،‬ﻣﻦ اﻷﻓﻀﻞ اﻹﺳﺘﻌﺎﻧﺔ ﺑﺮﺳﻢ‪.‬‬

‫‪Ŀ‬‬
‫‪−1 + 3ı‬‬ ‫ﻧﻼﺣﻆ ﻋﲆ اﻟﺸﻜﻞ ﺟﺎﻧﺒﻪ ّ‬
‫أن اﳌﺜﻠﺚ اﳌﺪروس ﻗﺎﺋﻢ ﻋﻨﺪ اﻟﺮأس اﻟﺬي‬
‫‪3‬‬
‫ﻻﺣﻘﺘﻪ ‪ 2 + ı‬و ﻣﺘﺴﺎوي اﻟﺴﺎﻗﲔ‪.‬‬
‫‪2‬‬ ‫ﻟﻨﺜﺒﺖ ذﻟﻚ‪ .‬ﻧﻀﻊ ‪ b = −1 + 3ı ، a = −2ı‬و ‪. c = 2 + ı‬‬
‫ﻟﺪﻳﻨﺎ ‪، c − b = 2 + ı + 1 − 3ı = 3 − 2ı :‬‬
‫‪2+ı‬‬
‫‪1‬‬ ‫‪c − a = 2 + ı + 2ı = 2 + 3ı‬‬
‫و ‪ b − a = −1 + 3ı + 2ı = −1 + 5ı‬ﻣﻨﻪ‬
‫‪1‬‬
‫‪−2‬‬ ‫‪−1‬‬ ‫‪0‬‬ ‫‪2‬‬ ‫‪3‬‬ ‫= ||‪، ||c − b‬‬ ‫‪3 + (−2) = √13‬‬

‫‪−1‬‬ ‫= |‪|c − a‬‬ ‫‪2 + 3 = √13‬‬


‫= ||‪ ||b − a‬أي ّ‬
‫أن ‪:‬‬ ‫و ‪(−1) + 5 = √26‬‬
‫‪−2‬‬
‫‪−2ı‬‬ ‫||‪||c − b|| + |c − a| = ||b − a‬‬ ‫|‪ ||c − b|| = |c − a‬و‬
‫إذن ﻫﺬا اﳌﺜﻠﺚ ﻗﺎﺋﻢ و ﻣﺘﺴﺎوي اﻟﺴﺎﻗﲔ‪.‬‬
‫‪−3‬‬

‫■‬

‫‪1 √3‬‬
‫‪#‬‬ ‫‪.‬‬
‫𝟔𝟎𝟏‬ ‫‪.‬‬
‫‪. u = a + bj + cj‬‬ ‫‪ j = − +‬و ‪a + bj + cj‬‬ ‫ﻟﺘﻜﻦ ‪ b ، a‬و ‪ c‬ﺛﻼﺛﺔ أﻋﺪاد ﻣﺮﻛﺒﺔ‪ .‬ﻧﻀﻊ ‪ı‬‬
‫‪2‬‬ ‫‪2‬‬
‫‪ •1‬أﻛﺘﺐ ﻋﲆ اﻟﺸﻜﻞ اﳉﱪي اﻷﻋﺪاد ‪ j ، j :‬و ‪. 1 + j + j‬‬
‫‪ņőƱŀ Ţ‬‬
‫‪ •2‬أﺛﺒﺖ ّ‬
‫أن ‪. u = a + b + c − (ab + ac + bc) :‬‬
‫‪ّ •3‬ﺑﲔ ّ‬
‫أن ‪. a + b + c − 3abc = u (a + b + c) :‬‬
‫‪VI‬‬

‫)‪. (𝛼, 𝛽 ∈ ℂ‬‬ ‫إﺳﺘﻨﺘﺞ ﺣﻠﻮل اﳌﻌﺎدﻟﺔ ‪ z − 3𝛼𝛽z + 𝛼 + 𝛽 = 0‬ﰲ ‪ℂ‬‬ ‫•‬ ‫‪4‬‬

‫‪ j = e‬ﻹﺟﺮاء اﳊﺴﺎﺑﺎت( ‪:‬‬ ‫‪/‬‬ ‫‪ •1‬ﻟﺪﻳﻨﺎ )ﻳﻤﻜﻦ إﺳﺘﻌﲈل اﻟﺸﻜﻞ اﻷﳼ‬ ‫ﺍﳊﻞّ‪.‬‬
‫‪ƕ‬‬

‫‪1 √3‬‬ ‫‪1‬‬ ‫‪1‬‬ ‫‪√3‬‬ ‫‪3‬‬ ‫‪1 √3‬‬


‫= ‪j‬‬ ‫‪− +‬‬ ‫‪ı‬‬ ‫=‬ ‫‪−2 −‬‬ ‫‪ı‬‬ ‫‪−‬‬ ‫‪=− −‬‬ ‫‪ı=j‬‬
‫‪2‬‬ ‫‪2‬‬ ‫‪4‬‬ ‫‪2‬‬ ‫‪2‬‬ ‫‪4‬‬ ‫‪2‬‬ ‫‪2‬‬
‫‪1 3‬‬
‫‪ũŏ‬‬

‫= ||‪j = j × j = j × j = ||j‬‬ ‫‪+ =1‬‬


‫‪4 4‬‬
‫‪1‬‬
‫‪1 + j + j = 1 + j + j = 1 + 2 Re (j) = 1 + 2 −‬‬ ‫‪=0‬‬
‫‪2‬‬

‫أن ‪ّ 1 + j + j = 1 − j = 0 :‬‬
‫ﻷن ‪. j = 1‬‬ ‫ﻳﻤﻜﻦ أﻳﻀ ًﺎ ﻣﻼﺣﻈﺔ ّ‬
‫‪1−j‬‬

‫𝟳𝟲𝟭‬

‫‪http ://tinyurl.com/Malki1718‬‬ ‫‪0‬‬


‫ ﺗﻄﺒﻴﻘﺎت اﻷﻋﺪاد اﳌﺮﻛﺒﺔ‬.3.VI
.
ّ ‫• ﻟﺪﻳﻨﺎ )ﻧُﺬﻛﱢﺮ‬2
: (j + j = −1 ‫ و‬j = j ⋅ j = j ‫أن‬

Ŕž Ŧ Ľ
a + bj + cj a + bj + cj = a + abj + acj + abj + b j + bcj + acj + bcj + c j

= a + b + c + (ab + ac + bc) j + (ab + ac + bc) j

œǃ
= a + b + c + (ab + ac + bc) j + j

= a + b + c − (ab + ac + bc)

Ŀ
: ‫• ﻟﺪﻳﻨﺎ‬3
u (a + b + c) = a + b + c − (ab + ac + bc) (a + b + c)
a  
ab +  bc+
c
= a gray
+a bgray
+ cgray
+  b gray
+ cbgray
+ acgray
+

ab − abcgray abcgray 
gray
−a bgray
−  abcgray
−a c− − bc− −b  − bc
cgray
= a + b + c − 3abc

: ‫( ﻳﻜﻮن‬c = 𝛽 ‫ و‬b = 𝛼 ، a = z ‫• ﺣﺴﺐ اﻟﺴﺆال اﻟﺴﺎﺑﻖ )ﺑﺄﺧﺬ‬4


z − 3𝛼𝛽z + 𝛼 + 𝛽 = 0 ⟺ (z + 𝛼 + 𝛽) z + 𝛼j + 𝛽j z + 𝛼j + 𝛽j = 0

⟺ z+𝛼+𝛽=0 ‫أو‬ z + 𝛼j + 𝛽j = 0 ‫أو‬ z + 𝛼j + 𝛽j = 0


⟺ z = −𝛼 − 𝛽 ‫أو‬ z = −𝛼j − 𝛽j ‫أو‬ z = −𝛼j − 𝛽j

. 𝒮 = −𝛼 − 𝛽, −𝛼j − 𝛽j , −𝛼j − 𝛽j : ‫إذن ﳎﻤﻮﻋﺔ ﺣﻠﻮل اﳌﻌﺎدﻟﺔ اﳌﻘﱰﺣﺔ ﻫﻲ‬


ّ
: ‫ ﻛﻼ ﻣﻦ اﳌﻌﺎدﻻت اﻟﺘﺎﻟﻴﺔ‬ℂ ‫ﺣﻞ ﰲ اﳌﺠﻤﻮﻋﺔ‬
 .
𝟏𝟎𝟕 .
ņőƱŀ Ţ
(z − ı) = ı (z + ı) 5
• z +1=0 • 1
(z + 1) + ı z + z =0 6
• z + 16 = 0 • 2

VI
z − (5 − 14ı) z − 2 (5ı + 12) = 0 7
• z +z +1=0 • 3
z − (1 − ı) z − ı = 0 8
• (z + 1) + (z + 2) = 0 • 4

.ّ‫ﺍﳊﻞ‬
ƕ

ّ ‫• ﻳﻤﻜﻦ‬1
: ‫ أو ﻛﲈ ﻳﲇ‬، −1 ‫ﺣﻞ اﳌﻌﺎدﻟﺔ ﺑﺈﳚﺎد اﳉﺬور اﻟﺮاﺑﻌﺔ ﻟﻠﻌﺪد‬
ّ ‫ﻧﻼﺣﻆ‬
: ‫ و ﺑﺎﻟﺘﺎﱄ‬−2ı = ı (1 + ı) = (1 − ı) ‫ ﻣﻨﻪ‬2ı = (1 + ı) ‫أن‬
z +1=0 ⟺ z −ı =0 ⟺ z −ı z +ı =0
ũŏ

1+ı 1−ı
⟺ z − z − =0
√2 √2
√2 + ı√2 √2 + ı√2 √2 − ı√2 √2 − ı√2
⟺ z− z+ z− z+ =0
2 2 2 2
√2 + ı√2 / √2 − ı√2 /
⟺ z=± = ±e ‫أو‬ z=± = ±e
2 2

𝟭𝟲𝟴

http ://tinyurl.com/Malki1718 0
‫‪ .VI‬ﲤﺎرﻳﻦ ﺗﻄﺒﻴﻘﻴﺔ‬
‫‪.‬‬
‫‪2 + ı√2 √2 + ı√2 √2 − ı√2 −√2 + ı√2‬‬
‫√ = 𝒮‪.‬‬ ‫‪,−‬‬ ‫‪,‬‬ ‫‪,‬‬ ‫إذن ﳎﻤﻮﻋﺔ اﳊﻠﻮل ﻫﻲ ‪:‬‬
‫‪2‬‬ ‫‪2‬‬ ‫‪2‬‬ ‫‪2‬‬

‫‪Ŕž Ŧ Ľ‬‬
‫‪ •2‬ﻳﻤﻜﻦ ّ‬
‫ﺣﻞ اﳌﻌﺎدﻟﺔ ﺑﺈﳚﺎد اﳉﺬور اﻟﺮاﺑﻌﺔ ﻟﻠﻌﺪد ‪ ، −16‬أو ﻛﲈ ﻳﲇ )ﺑﺎﺳﺘﻌﲈل اﻟﻨﺘﻴﺠﺔ اﻟﺴﺎﺑﻘﺔ( ‪:‬‬

‫‪œǃ‬‬
‫‪z‬‬ ‫‪z‬‬ ‫‪z‬‬
‫⟺ ‪z + 16 = 0‬‬ ‫⟺ ‪+1=0‬‬ ‫⟺ ‪+1=0‬‬ ‫𝒮∈‬
‫‪16‬‬ ‫‪2‬‬ ‫‪2‬‬
‫إذن ﳎﻤﻮﻋﺔ اﳊﻠﻮل ﻫﻲ ‪. 𝒮 = √2 + ı√2, −√2 − ı√2, √2 − ı√2, −√2 + ı√2 :‬‬

‫‪Ŀ‬‬
‫‪ •3‬ﻫﺬه اﳌﻌﺎدﻟﺔ ﻣﻀﺎﻋﻔﺔ اﻟﱰﺑﻴﻊ‪ .‬ﳊ ّﻠﻬﺎ‪ ،‬ﻧﻀﻊ ‪ Z = z‬ﻓﺘﺼﺒﺢ اﳌﻌﺎدﻟﺔ ‪ . Z + Z + 1 = 0 :‬ﳍﺬه اﻷﺧﲑة ﺣﻼّن ﳘﺎ‬
‫‪ z = ±e‬أي‬ ‫‪/‬‬ ‫‪ z = j = e‬ﻣﻨﻪ‬ ‫‪/‬‬ ‫‪= e‬‬ ‫‪/‬‬ ‫‪ j‬و ‪) j‬اﳉﺬور اﻟﺘﻜﻌﻴﺒﻴﺔ ﻏﲑ اﳊﻘﻴﻘﻴﺔ ﻟﻠﻮﺣﺪة( و ﺑﺎﻟﺘﺎﱄ ‪:‬‬
‫‪1‬‬ ‫‪√3‬‬
‫‪z = ±‬؛‬ ‫‪+ı‬‬
‫‪2‬‬ ‫‪2‬‬

‫‪1‬‬ ‫‪3‬‬
‫√‪. z = ± − + ı‬‬ ‫أو ‪ z = j‬ﻣﻨﻪ ‪ z = ±j‬أي‬
‫‪2‬‬ ‫‪2‬‬

‫‪1‬‬ ‫‪√3 1‬‬ ‫‪√3 1‬‬ ‫‪√3 1‬‬ ‫‪√3‬‬


‫= 𝒮‪.‬‬ ‫‪+ı‬‬ ‫‪,− − ı‬‬ ‫‪,− + ı‬‬ ‫‪, −ı‬‬ ‫ﰲ اﻷﺧﲑ‪ ،‬ﳎﻤﻮﻋﺔ اﳊﻠﻮل ﻫﻲ ‪:‬‬
‫‪2‬‬ ‫‪2‬‬ ‫‪2‬‬ ‫‪2‬‬ ‫‪2‬‬ ‫‪2 2‬‬ ‫‪2‬‬

‫‪ •4‬ﻟﺪﻳﻨﺎ ‪:‬‬
‫)‪(z + 1) + (z + 2) = 0 ⟺ (z + 1) = − (z + 2‬‬
‫)‪⟺ (z + 1) = ı (z + 2‬‬

‫⟺‬ ‫)‪(z + 1‬‬ ‫))‪= (ı (z + 2‬‬

‫))‪⟺ (z + 1) = ± (ı (z + 2‬‬

‫و ﺑﺎﻟﺘﺎﱄ ‪:‬‬
‫‪ņőƱŀ Ţ‬‬
‫إﻣ ّﺎ )‪ (z + 1) = ı (z + 2‬أي ‪. z + (2 − ı) z + 1 − 2ı = 0‬‬ ‫•‬

‫اﳌﻤ ّﻴﺰ ﻫﻮ ‪. Δ = (2 − ı) − 4 (1 − 2ı) = −1 + 4ı‬‬


‫‪−2 + 2√17‬‬ ‫‪2 + 2√17‬‬
‫ﺟﺬر ﺗﺮﺑﻴﻌﻲ ﻟﻠﻌﺪد ‪ Δ‬ﻣﻨﻪ اﳊﻼّن ‪:‬‬ ‫‪+ı‬‬ ‫ﺑﺎﺳﺘﻌﲈل اﻟﻄﺮﻳﻘﺔ اﳉﱪﻳﺔ ﻧﺠﺪ ّ‬
‫أن‬
‫‪VI‬‬

‫‪2‬‬ ‫‪2‬‬
‫‪− +‬‬ ‫‪+‬‬
‫‪−2 + ı −‬‬ ‫‪−ı‬‬ ‫‪−4 −‬‬ ‫‪−2 + 2√17‬‬ ‫‪4−‬‬ ‫‪−2 + 2√17‬‬
‫= ‪z‬‬ ‫=‬ ‫‪+ı‬‬
‫‪2‬‬ ‫‪4‬‬ ‫‪4‬‬
‫‪− +‬‬ ‫‪+‬‬
‫‪−2 + ı +‬‬ ‫‪+ı‬‬ ‫‪−4 +‬‬ ‫‪−2 + 2√17‬‬ ‫‪4+‬‬ ‫‪−2 + 2√17‬‬
‫= ‪z‬‬ ‫‪+ı‬‬
‫‪ƕ‬‬

‫=‬
‫‪2‬‬ ‫‪4‬‬ ‫‪4‬‬

‫أو )‪ (z + 1) = −ı (z + 2‬أي ‪. z + (2 + ı) z + 1 + 2ı = 0‬‬ ‫•‬

‫اﳌﻤ ّﻴﺰ ﻫﻮ ‪. Δ = (2 + ı) − 4 (1 + 2ı) = −1 − 4ı‬‬


‫‪ũŏ‬‬

‫‪−2 + 2√17‬‬ ‫‪2 + 2√17‬‬


‫ﺟﺬر ﺗﺮﺑﻴﻌﻲ ﻟﻠﻌﺪد ‪ Δ‬ﻣﻨﻪ اﳊﻼّن ‪:‬‬ ‫‪−ı‬‬ ‫ﺑﺎﳌﺜﻞ‪ ،‬ﻧﺠﺪ ّ‬
‫أن‬
‫‪2‬‬ ‫‪2‬‬
‫‪− +‬‬ ‫‪+‬‬
‫‪−2 − ı −‬‬ ‫‪+ı‬‬ ‫‪−4 −‬‬ ‫‪−2 + 2√17‬‬ ‫‪4+‬‬ ‫‪−2 + 2√17‬‬
‫= ‪z‬‬ ‫=‬ ‫‪+ı‬‬
‫‪2‬‬ ‫‪4‬‬ ‫‪4‬‬
‫‪− +‬‬ ‫‪+‬‬
‫‪−2 − ı +‬‬ ‫‪−ı‬‬ ‫‪−4 +‬‬ ‫‪−2 + 2√17‬‬ ‫‪4−‬‬ ‫‪−2 + 2√17‬‬
‫= ‪z‬‬ ‫=‬ ‫‪+ı‬‬
‫‪2‬‬ ‫‪4‬‬ ‫‪4‬‬

‫𝟵𝟲𝟭‬

‫‪http ://tinyurl.com/Malki1718‬‬ ‫‪0‬‬


‫‪ .3.VI‬ﺗﻄﺒﻴﻘﺎت اﻷﻋﺪاد اﳌﺮﻛﺒﺔ‬
‫‪.‬‬
‫‪.𝒮 = z ,z ,z ,z‬‬ ‫ﰲ اﻷﺧﲑ‪ ،‬ﳎﻤﻮﻋﺔ اﳊﻠﻮل ﻫﻲ ‪:‬‬

‫‪Ŕž Ŧ Ľ‬‬
‫ﻼ ﻟﻠﻤﻌﺎدﻟﺔ ّ‬
‫ﻓﺈن ‪:‬‬ ‫‪ •5‬ﺑﲈ ّ‬
‫أن ‪ z = −ı‬ﻟﻴﺲ ﺣ ً‬
‫‪z−ı‬‬
‫)‪(z − ı) = ı (z + ı‬‬ ‫⟺‬ ‫‪=ı‬‬

‫‪œǃ‬‬
‫‪z+ı‬‬
‫‪z−ı‬‬
‫⟺‬ ‫)‪= (−ı‬‬
‫‪z+ı‬‬
‫‪z−ı‬‬ ‫‪z−ı‬‬ ‫‪z−ı‬‬
‫⟺‬ ‫أو ‪= −ı‬‬ ‫أو ‪= −jı‬‬ ‫‪= −j ı‬‬

‫‪Ŀ‬‬
‫‪z+ı‬‬ ‫‪z+ı‬‬ ‫‪z+ı‬‬
‫‪−ı + 1‬‬ ‫‪−jı + 1‬‬ ‫‪−j ı + 1‬‬
‫‪⟺ z=−‬‬ ‫‪ z = −‬أو ‪ı‬‬ ‫‪ı‬‬ ‫‪ z = −‬أو‬ ‫‪ı‬‬
‫‪−ı − 1‬‬ ‫‪−jı − 1‬‬ ‫‪−j ı − 1‬‬
‫‪1‬‬ ‫‪1‬‬
‫‪⟺ z=1‬‬ ‫أو‬ ‫=‪z‬‬ ‫‪ z = −‬أو ‪= −√3 − 2‬‬ ‫‪= √3 − 2‬‬
‫‪√3 − 2‬‬ ‫‪√3 + 2‬‬
‫ﰲ اﻷﺧﲑ‪ ،‬ﳎﻤﻮﻋﺔ اﳊﻠﻮل ﻫﻲ ‪. 𝒮 = 1, −√3 − 2, √3 − 2 :‬‬
‫‪ •6‬ﻟﺪﻳﻨﺎ ‪:‬‬
‫‪1‬‬
‫‪(z + 1) + ı z + z‬‬ ‫⟺ ‪=0‬‬ ‫‪z +z‬‬ ‫)‪= − (z + 1‬‬
‫‪ı‬‬
‫‪z +z‬‬ ‫‪= ı (z + 1) = e‬‬ ‫‪/‬‬
‫⟺‬ ‫)‪(z + 1‬‬
‫‪z +z‬‬ ‫‪= e‬‬ ‫‪/‬‬
‫⟺‬ ‫)‪(z + 1‬‬
‫‪⟺ z +z=e‬‬ ‫‪/‬‬ ‫‪/‬‬
‫)‪(z + 1‬‬ ‫أو‬ ‫‪z + z = −e‬‬ ‫)‪(z + 1‬‬
‫‪⟺ z = −1‬‬ ‫أو‬ ‫‪z=e‬‬ ‫‪/‬‬ ‫أو‬ ‫‪z = −1‬‬ ‫أو‬ ‫‪z = −e‬‬ ‫‪/‬‬

‫‪√2‬‬ ‫‪√2‬‬ ‫‪√2‬‬ ‫‪√2‬‬


‫‪⟺ z = −1‬‬ ‫أو‬ ‫=‪z‬‬ ‫‪+ı‬‬ ‫أو‬ ‫‪z=−‬‬ ‫‪−ı‬‬
‫‪2‬‬ ‫‪2‬‬ ‫‪2‬‬ ‫‪2‬‬
‫‪2‬‬ ‫‪2‬‬ ‫‪2‬‬ ‫‪2‬‬
‫√ ‪. 𝒮 = −1, √ + ı √ , − √ − ı‬‬ ‫ﰲ اﻷﺧﲑ‪ ،‬ﳎﻤﻮﻋﺔ اﳊﻠﻮل ﻫﻲ ‪:‬‬
‫‪2‬‬ ‫‪2‬‬ ‫‪2‬‬ ‫‪2‬‬
‫‪ņőƱŀ Ţ‬‬
‫⧏ ﻟﻠﻤﻌﺎدﻟﺔ ‪ 4‬ﺣﻠﻮل ّ‬
‫ﻷن اﳊﻞ ‪ُ z = −1‬ﻣﻀﺎﻋﻒ‪.‬‬ ‫ﻣﻼﺣﻈﺔ ‪: 27‬‬
‫ﻳﻤﻜﻦ أﻳﻀ ًﺎ ّ‬
‫ﺣﻞ اﳌﻌﺎدﻟﺔ ﺑﻤﻼﺣﻈﺔ أﳖﺎ ﺗُﻜﺎﻓﺊ ‪. (z + 1) 1 + ız = 0 :‬‬
‫⧐‬

‫‪VI‬‬
‫‪ •7‬ﻫﺬه اﳌﻌﺎدﻟﺔ ﻣﻀﺎﻋﻔﺔ اﻟﱰﺑﻴﻊ‪ .‬ﳊ ّﻠﻬﺎ‪ ،‬ﻧﻀﻊ ‪ Z = z‬ﻓﺘﺼﺒﺢ اﳌﻌﺎدﻟﺔ ‪. Z − (5 − 14ı) Z − 2 (5ı + 12) = 0 :‬‬
‫اﳌﻤ ّﻴﺰ ﻫﻮ ‪:‬‬
‫‪Δ = (− (5 − 14ı)) − 4 (−2 (5ı + 12)) = −75 − 100ı‬‬

‫ﻟﻨﺒﺤﺚ ﻋﻦ ﺟﺬر ﺗﺮﺑﻴﻌﻲ ‪ 𝛿 = a + ıb‬ﻟﻠﻌﺪد ‪ Δ‬أي ﺑﺤﻴﺚ ‪. 𝛿 = Δ = −75 − 100ı‬‬


‫‪ƕ‬‬

‫⎧‬
‫⎪‬
‫= ||‪⎪ a + b = ||Δ‬‬ ‫‪100 + 75 = 125‬‬
‫‪.‬‬ ‫‪a − b = −75‬‬ ‫ﻫﺬا اﻟﻌﺪد ُﳛﻘﻖ ‪:‬‬
‫⎨‬
‫⎪‬
‫⎪‬
‫⎩‬ ‫‪2ab = −100‬‬
‫‪ũŏ‬‬

‫ﺑﺠﻤﻊ اﳌﻌﺎدﻟﺘﲔ اﻷوﱃ و اﻟﺜﺎﻧﻴﺔ ﻳﻨﺘﺞ ‪ 2a = 125 − 75 = 50‬ﻣﻨﻪ ‪ . a = ±5‬ﻧﺨﺘﺎر ﻣﺜ ً‬


‫ﻼ ‪ a = 5‬ﻓﻴﻜﻮن )ﻣﻦ اﳌﻌﺎدﻟﺔ‬
‫اﻟﺜﺎﻟﺜﺔ( ‪. b = −10‬‬
‫و ﺑﺎﻟﺘﺎﱄ ﻟﻠﻤﻌﺎدﻟﺔ ﺣﻼّن ﳘﺎ ‪:‬‬
‫‪5 − 14ı − 5 + 10ı‬‬ ‫‪5 − 14ı + 5 − 10ı‬‬
‫= ‪Z‬‬ ‫‪= −2ı‬‬ ‫‪,‬‬ ‫= ‪Z‬‬ ‫‪= 5 − 12ı‬‬
‫‪2‬‬ ‫‪2‬‬
‫اﻵن ﻧﺤﻞ اﳌﻌﺎدﻟﺘﲔ ‪ z = −2ı‬و ‪ . z = 5 − 12ı‬ﳍﺬا اﻟﻐﺮض‪ ،‬ﻧﻀﻊ ‪. z = a + ıb‬‬

‫𝟬𝟳𝟭‬

‫‪http ://tinyurl.com/Malki1718‬‬ ‫‪0‬‬


‫‪ .VI‬ﲤﺎرﻳﻦ ﺗﻄﺒﻴﻘﻴﺔ‬
‫‪.‬‬
‫•‬
‫⎧‬
‫⎪‬ ‫‪a + b = ||−2ı|| = 2‬‬
‫⎪‬

‫‪Ŕž Ŧ Ľ‬‬
‫⟺ ‪z = −2ı‬‬ ‫‪a −b =0‬‬
‫⎨‬
‫⎪‬
‫⎪‬
‫⎩‬ ‫‪2ab = −2‬‬

‫‪œǃ‬‬
‫‪ b = a‬و ﺑﺎﻟﺘﻌﻮﻳﺾ ﰲ اﻷوﱃ ﻧﺠﺪ ‪ 2a = 2‬ﻣﻨﻪ ‪ . a = ±1‬ﻧﺨﺘﺎر ﻣﺜ ً‬
‫ﻼ‪a = 1‬‬ ‫ﻣﻦ اﳌﻌﺎدﻟﺔ اﻟﺜﺎﻧﻴﺔ ﻳﻨﺘﺞ‬
‫ﻓﻴﻜﻮن )ﻣﻦ اﳌﻌﺎدﻟﺔ اﻟﺜﺎﻟﺜﺔ( ‪ . b = −1‬أﺧﲑ ًا ‪ z = −1 + ı :‬و ‪. z = 1 − ı‬‬
‫•‬

‫‪Ŀ‬‬
‫⎧‬
‫⎪‬
‫= ||‪⎪ a + b = ||5 − 12ı‬‬ ‫‪5 + 12 = 13‬‬
‫⟺ ‪z = 5 − 12ı‬‬ ‫‪a −b =5‬‬
‫⎨‬
‫⎪‬
‫⎪‬
‫⎩‬ ‫‪2ab = −12‬‬

‫ﺑﺠﻤﻊ اﳌﻌﺎدﻟﺘﲔ اﻷوﱃ و اﻟﺜﺎﻧﻴﺔ ﻳﻨﺘﺞ ‪ 2a = 13 + 5 = 18‬ﻣﻨﻪ ‪ . a = ±3‬ﻧﺨﺘﺎر ﻣﺜ ً‬


‫ﻼ ‪ a = 3‬ﻓﻴﻜﻮن )ﻣﻦ‬
‫اﳌﻌﺎدﻟﺔ اﻟﺜﺎﻟﺜﺔ( ‪ . b = −2‬أﺧﲑ ًا ‪ z = −3 + 2ı :‬و ‪. z = 3 − 2ı‬‬
‫ﰲ اﻷﺧﲑ‪ ،‬ﳎﻤﻮﻋﺔ اﳊﻠﻮل ﻫﻲ ‪. 𝒮 = {1 − ı, −1 + ı, 3 − 2ı, −3 + 2ı} :‬‬

‫‪ •8‬ﻧﻀﻊ ‪ Z = z‬ﻓﺘﺼﺒﺢ اﳌﻌﺎدﻟﺔ ‪ . Z − (1 − ı) Z − ı = 0 :‬اﳌﻤ ّﻴﺰ ﻫﻮ ‪:‬‬


‫)‪Δ = (− (1 − ı)) + 4ı = 2ı = (1 + ı‬‬
‫و ﺑﺎﻟﺘﺎﱄ اﳊﻼّن ‪:‬‬
‫‪1−ı−1−ı‬‬ ‫‪1−ı+1+ı‬‬
‫= ‪Z‬‬ ‫)‪= −ı = (ı‬‬ ‫‪,‬‬ ‫= ‪Z‬‬ ‫‪=1‬‬
‫‪2‬‬ ‫‪2‬‬

‫‪√3 1‬‬ ‫‪3‬‬ ‫‪1‬‬


‫=‪z=j ı‬‬ ‫‪− ı‬‬ ‫ﻣﻨﻪ ‪ z = ı :‬أو ‪ z = jı = − √ − ı‬أو‬
‫‪2‬‬ ‫‪2‬‬ ‫‪2‬‬ ‫‪2‬‬
‫‪1‬‬ ‫‪3‬‬ ‫‪1 √3‬‬
‫أو ‪. z = j = − − √ ı‬‬ ‫‪z=j=− +‬‬ ‫‪ı‬‬ ‫‪ z = 1‬أو‬ ‫أو‬
‫‪2‬‬ ‫‪2‬‬ ‫‪2‬‬ ‫‪2‬‬
‫‪ņőƱŀ Ţ‬‬
‫‪1‬‬ ‫‪3‬‬ ‫‪1‬‬ ‫‪3‬‬ ‫‪3‬‬ ‫‪1‬‬ ‫‪3‬‬ ‫‪1‬‬
‫ﰲ اﻷﺧﲑ‪ ،‬ﳎﻤﻮﻋﺔ اﳊﻠﻮل ﻫﻲ ‪. 𝒮 = 1, ı, − + √ − − √ ı, − √ − ı, √ − ı :‬‬
‫‪2‬‬ ‫‪2‬‬ ‫‪2‬‬ ‫‪2‬‬ ‫‪2‬‬ ‫‪2‬‬ ‫‪2‬‬ ‫‪2‬‬
‫■‬
‫‪VI‬‬

‫ّ‬
‫ﺣﻞ ﰲ ‪ ℂ‬اﳌﻌﺎدﻻت اﻵﺗﻴﺔ‪ ،‬ﺣﻴﺚ ‪ z‬ﻫﻮ اﳌﺠﻬﻮل و 𝜃 ﻋﺪد ﺣﻘﻴﻘﻲ ﺛﺎﺑﺖ ‪:‬‬
‫‬ ‫‪.‬‬
‫𝟖𝟎𝟏‬ ‫‪.‬‬

‫‪z − 2z + sin 𝜃 = 0‬‬ ‫•‬ ‫‪4‬‬ ‫‪z +z+1=0‬‬ ‫•‬‫‪1‬‬


‫‪ƕ‬‬

‫‪z − (6 + ı) z + (11 + 13ı) = 0‬‬ ‫•‬ ‫‪5‬‬ ‫‪2z + 2z + 1 = 0‬‬ ‫•‬‫‪2‬‬


‫‪2z − (7 + 3ı) z + (2 + 4ı) = 0‬‬ ‫•‬ ‫‪6‬‬ ‫‪z − 2z cos 𝜃 + 1 = 0‬‬ ‫•‬‫‪3‬‬
‫‪ũŏ‬‬

‫ﺍﳊﻞّ‪.‬‬
‫‪−1 − ı√3‬‬
‫و‬ ‫‪= j‬‬ ‫‪ Δ = (−1) − 4 × 1 = −3 = ı√3‬إذن ﳍﺎ ﺣﻼّن ﳘﺎ‬ ‫‪ُ •1‬ﳑ َ ﱢﻴﺰ اﳌﻌﺎدﻟﺔ ﻫﻮ‬
‫‪2‬‬
‫‪/‬‬ ‫‪−1 + ı√3‬‬
‫‪.j = e‬‬ ‫ﻣﻊ‬ ‫‪=j‬‬
‫‪2‬‬
‫ﻧﻼﺣﻆ ّ‬
‫أن ‪ z = 1‬ﻟﻴﺲ ﺣﻼ ﻟﻠﻤﻌﺎدﻟﺔ إذن ﺑﴬب ﻃﺮﻓﻴﻬﺎ ﺑﺎﳌﻘﺪار ‪ z − 1‬ﺗﺼﺒﺢ ‪z − 1 = 0‬‬ ‫ﻣﻼﺣﻈﺔ ‪⧏ : 28‬‬
‫⧐‬ ‫إذن اﳊﻼّن ﳘﺎ اﳉﺬران اﻟﺘﻜﻌﻴﺒﻴﺎن ﻏﲑ اﳊﻘﻴﻘﻴﲔ ﻟﻠﻮﺣﺪة أي ‪ j‬و ‪. j‬‬

‫𝟭𝟳𝟭‬

‫‪http ://tinyurl.com/Malki1718‬‬ ‫‪0‬‬


‫‪ .3.VI‬ﺗﻄﺒﻴﻘﺎت اﻷﻋﺪاد اﳌﺮﻛﺒﺔ‬
‫‪.‬‬
‫‪−1 + ı‬‬ ‫‪−1 − ı‬‬
‫‪.‬‬ ‫و‬ ‫‪ •2‬اﳌﻤ ّﻴﺰ اﳌﺨﺘﴫ ﻫﻮ ‪ Δ = (−1) − 2 × 1 = −1 = ı‬إذن ﻟﻠﻤﻌﺎدﻟﺔ ﺣﻼّن ﳘﺎ‬
‫‪2‬‬ ‫‪2‬‬

‫‪Ŕž Ŧ Ľ‬‬
‫‪ •3‬اﳌﻤ ّﻴﺰ اﳌﺨﺘﴫ ﻫﻮ )𝜃 ‪ Δ = (cos 𝜃) − 1 × 1 = cos 𝜃 − 1 = − sin 𝜃 = (ı sin‬إذن ﻟﻠﻤﻌﺎدﻟﺔ ﺣﻼّن ﳘﺎ‬
‫‪ cos 𝜃 − ı sin 𝜃 = e−‬و ‪. cos 𝜃 + ı sin 𝜃 = e‬‬

‫‪œǃ‬‬
‫‪ •4‬اﳌﻤ ّﻴﺰ اﳌﺨﺘﴫ ﻫﻮ 𝜃 ‪ Δ = 1 − 1 × sin 𝜃 = cos‬إذن ﻟﻠﻤﻌﺎدﻟﺔ ﺣﻼّن ﳘﺎ ‪:‬‬
‫𝜃‬ ‫𝜃‬
‫‪.1 + cos 𝜃 = 2 cos‬‬ ‫و‬ ‫‪1 − cos 𝜃 = 2 sin‬‬
‫‪2‬‬ ‫‪2‬‬

‫‪Ŀ‬‬
‫أن ‪ −9 = 16 − 25 = 4 − 5‬و‬ ‫‪ُ •5‬ﳑ َ ﱢﻴﺰ اﳌﻌﺎدﻟﺔ ﻫﻮ ‪ . Δ = (6 + ı) − 4 (11 + 13ı) = −9 − 40ı‬ﻳﻤﻜﻦ ﻣﻼﺣﻈﺔ ّ‬
‫‪ 40 = 2 × 4 × 5‬ﻣﻨﻪ )‪ ، Δ = 4 − 2 × 4 × 5ı + (5ı) = (4 − 5ı‬ﻟﻜﻦ ﺳﻨﺘّﺒﻊ اﻟﻄﺮﻳﻘﺔ اﻟﻌﺎﻣﺔ ‪:‬‬
‫⎧‬ ‫‪x − y = −9‬‬
‫⎪‬
‫⎪‬
‫‪.‬‬ ‫‪2xy = −40‬‬ ‫ﻟﻴﻜﻦ ‪ 𝛿 = x + ıy‬ﺟﺬر ًا ﺗﺮﺑﻴﻌﻴ ًﺎ ﻟﻠﻌﺪد ‪ . Δ‬ﻟﺪﻳﻨﺎ ‪ 𝛿 = Δ :‬ﻣﻨﻪ ‪:‬‬
‫⎨‬
‫⎪‬
‫⎪‬
‫‪⎩ x + y = (−9) + (−40) = 41‬‬
‫ُﻌﻮض ﰲ اﳌﻌﺎدﻟﺔ‬ ‫ﺑﺠﻤﻊ اﳌﻌﺎدﻟﺘﲔ اﻷوﱃ و اﻟﺜﺎﻟﺜﺔ ﻳﻨﺘﺞ ‪ 2x = 41 − 9 = 32‬أي ‪ . x = ±4‬ﻧﺨﺘﺎر ‪ x = 4‬ﻣﺜ ً‬
‫ﻼوﻧ ﱢ‬
‫اﻟﺜﺎﻧﻴﺔ ﻟﻨﺠﺪ ّ‬
‫أن ‪ . y = −5‬إذن ‪ 𝛿 = 4 − 5ı‬أي )‪ Δ = (4 − 5ı‬و ﺑﺎﻟﺘﺎﱄ ﻟﻠﻤﻌﺎدﻟﺔ ﺣﻼّن ﳘﺎ ‪:‬‬
‫‪6 + ı + 4 − 5ı‬‬ ‫‪6 + ı − 4 + 5ı‬‬
‫‪.‬‬ ‫‪= 5 − 2ı‬‬ ‫و‬ ‫‪= 1 + 3ı‬‬
‫‪2‬‬ ‫‪2‬‬
‫أن ‪ 24 = 25 − 1 = 5 − 1‬و‬ ‫‪ُ •6‬ﳑ َ ﱢﻴﺰ اﳌﻌﺎدﻟﺔ ﻫﻮ ‪ . Δ = (7 + 3ı) − 4 × 2 (2 + 4ı) = 24 + 10ı‬ﻳﻤﻜﻦ ﻣﻼﺣﻈﺔ ّ‬
‫‪ 10 = 2 × 5 × 1‬ﻣﻨﻪ )‪ ، Δ = 5 + 2 × 5 × ı + ı = (5 + ı‬ﻟﻜﻦ ﺳﻨﺘّﺒﻊ اﻟﻄﺮﻳﻘﺔ اﻟﻌﺎﻣﺔ ‪:‬‬
‫⎧‬ ‫‪x − y = 24‬‬
‫⎪‬
‫⎪‬
‫‪.‬‬ ‫‪2xy = 10‬‬ ‫ﻟﻴﻜﻦ ‪ 𝛿 = x + ıy‬ﺟﺬر ًا ﺗﺮﺑﻴﻌﻴ ًﺎ ﻟﻠﻌﺪد ‪ . Δ‬ﻟﺪﻳﻨﺎ ‪ 𝛿 = Δ :‬ﻣﻨﻪ ‪:‬‬
‫⎨‬
‫⎪‬
‫⎪‬
‫‪⎩ x + y = 24 + 10 = 26‬‬
‫ُﻌﻮض ﰲ اﳌﻌﺎدﻟﺔ‬ ‫ﺑﺠﻤﻊ اﳌﻌﺎدﻟﺘﲔ اﻷوﱃ و اﻟﺜﺎﻟﺜﺔ ﻳﻨﺘﺞ ‪ 2x = 24 + 26 = 50‬أي ‪ . x = ±5‬ﻧﺨﺘﺎر ‪ x = 5‬ﻣﺜ ً‬
‫ﻼوﻧ ﱢ‬
‫‪ņőƱŀ Ţ‬‬
‫اﻟﺜﺎﻧﻴﺔ ﻟﻨﺠﺪ ّ‬
‫أن ‪ . y = 1‬إذن ‪ 𝛿 = 5 + ı‬أي )‪ Δ = (5 + ı‬و ﺑﺎﻟﺘﺎﱄ ﻟﻠﻤﻌﺎدﻟﺔ ﺣﻼّن ﳘﺎ ‪:‬‬
‫‪7 + 3ı + 5 + ı‬‬ ‫‪7 + 3ı − 5 − ı 1 + ı‬‬
‫‪.‬‬ ‫‪=3+ı‬‬ ‫و‬ ‫=‬
‫‪4‬‬ ‫‪4‬‬ ‫‪2‬‬
‫■‬

‫‪VI‬‬
‫ﻟﺪت ﰲ ‪ 11‬أﻛﺘﻮﺑﺮ ّ‬
‫ﻓﺈن‬ ‫ﻓﻴﲈ ﻳﲇ‪ ،‬اﳊﺮف ‪ J‬ﻳﺮﻣﺰ إﱃ ﻳﻮم ﻣﻴﻼدك و اﳊﺮف ‪ M‬إﱃ ﺷﻬﺮ ﻣﻴﻼدك‪ .‬ﻣﺜﻼً‪ ،‬إذا ُو َ‬
‫‪.‬‬
‫𝟗𝟎𝟏‬ ‫‪.‬‬ ‫‪#‬‬
‫‪ J = 11‬و ‪. M = 10‬‬
‫‪ƕ‬‬

‫‪1‬‬
‫= ‪.Z‬‬ ‫‪ •1‬أﺣﺴﺐ اﳉﺰء اﻟﺘﺨﻴﲇ ﻟﻠﻌﺪد ‪:‬‬
‫‪M+ı‬‬

‫‪1 + ı√3‬‬
‫= ‪.Z‬‬ ‫‪ •2‬ﺣﺪّ د ﻋﻤﺪ ًة ﻟﻠﻌﺪد ‪:‬‬
‫‪ũŏ‬‬

‫‪1+ı‬‬

‫‪. z + (2 − J) z − (J − 1) = 0‬‬ ‫‪ •3‬ﺣﻞ ﰲ اﳌﺠﻤﻮﻋﺔ ‪ ℂ‬اﳌﻌﺎدﻟﺔ ‪:‬‬

‫ﺍﳊﻞّ‪ّ .‬‬
‫ﳊﻞ اﻟﺘﻤﺮﻳﻦ‪ ،‬ﻧﺨﺘﺎر ‪ J = 11‬و ‪. M = 10‬‬

‫𝟮𝟳𝟭‬

‫‪http ://tinyurl.com/Malki1718‬‬ ‫‪0‬‬


‫‪ .VI‬ﲤﺎرﻳﻦ ﺗﻄﺒﻴﻘﻴﺔ‬
‫‪.‬‬
‫‪ •1‬ﻟﺪﻳﻨﺎ ‪:‬‬
‫‪1‬‬ ‫‪10 − ı‬‬ ‫‪10 − ı‬‬ ‫‪10‬‬ ‫‪1‬‬
‫=‪M‬‬ ‫=‬ ‫=‬ ‫=‬ ‫‪−‬‬ ‫‪ı‬‬

‫‪Ŕž Ŧ Ľ‬‬
‫‪10 + ı 10 + 1‬‬ ‫‪101‬‬ ‫‪101 101‬‬
‫‪1‬‬
‫‪.−‬‬ ‫و ﺑﺎﻟﺘﺎﱄ ﻓﺎﳉﺰء اﻟﺘﺨﻴﲇ ﻟﻠﻌﺪد ‪ Z‬ﻫﻮ‬

‫‪œǃ‬‬
‫‪101‬‬
‫‪ •2‬ﻟﺪﻳﻨﺎ ‪:‬‬
‫‪1 + ı√3‬‬ ‫‪2 cos + ı sin‬‬ ‫‪e‬‬ ‫‪/‬‬
‫‪−‬‬ ‫‪/‬‬
‫=‬ ‫‪= √2‬‬ ‫‪/‬‬
‫‪= √2e‬‬ ‫‪= √2e‬‬
‫‪1+ı‬‬ ‫‪√2 cos + ı sin‬‬ ‫‪e‬‬

‫‪Ŀ‬‬
‫ﻣﻨﻪ ‪:‬‬
‫‪Z = √2e‬‬ ‫‪/‬‬ ‫‪=2 e‬‬ ‫‪/‬‬ ‫‪= 32e‬‬ ‫‪/‬‬

‫𝜋‪5‬‬
‫= )‪. arg (Z‬‬ ‫و ﺑﺎﻟﺘﺎﱄ ‪(mod 2𝜋) :‬‬
‫‪6‬‬
‫‪ •3‬اﳌﻌﺎدﻟﺔ ﻫﻲ ‪ z − 9z − 10 = 0 :‬و ﻫﻲ ﻣﻌﺎدﻟﺔ ﻣﻀﺎﻋﻔﺔ اﻟﱰﺑﻴﻊ‪.‬‬
‫ﳊ ّﻠﻬﺎ ﻧﻀﻊ ‪ Z = z‬ﻓﻴﻜﻮن ‪. Z − 9Z − 10 = 0 :‬‬
‫ﻟﺪﻳﻨﺎ ‪ Δ = (−9) − 4 (−10) = 121 = 11‬إذن ﻟﻠﻤﻌﺎدﻟﺔ ﺣﻼّن ﳘﺎ ‪:‬‬
‫‪9 + 11‬‬ ‫‪9 − 11‬‬
‫= ‪.Z‬‬ ‫‪= 10‬‬ ‫و‬ ‫= ‪Z‬‬
‫‪= −1‬‬
‫‪2‬‬ ‫‪2‬‬
‫ﰲ اﻷﺧﲑ ‪ z = −1 :‬أو ‪ z = 10‬أي ‪ z = ±ı‬أو ‪. z = ±√10‬‬
‫‪. −ı, ı, −√10, √10‬‬ ‫إذن‪ ،‬ﳎﻤﻮﻋﺔ اﳊﻠﻮل ﻫﻲ ‪:‬‬
‫■‬

‫ﺣﻞ ﰲ ‪ ℂ‬اﳌﻌﺎدﻻت اﻟﺘﺎﻟﻴﺔ ذات اﳌﺠﻬﻮل ‪: z‬‬


‫‬ ‫‪.‬‬
‫𝟎𝟏𝟏‬ ‫‪.‬‬

‫‪z − 2ız + 2 − 4ı = 0‬‬ ‫‪3‬‬ ‫‪z − (5 − 14ı) z − 2 (5ı + 12) = 0‬‬ ‫‪1‬‬
‫‪ņőƱŀ Ţ‬‬
‫•‬ ‫•‬

‫)‪(𝜃 ∈ ℝ‬‬ ‫‪z − 2z cos 3𝜃 + 1 = 0‬‬ ‫•‬ ‫‪4‬‬ ‫)‪z − ı = 6 (z + ı‬‬ ‫‪2‬‬
‫•‬
‫‪VI‬‬

‫ﺍﳊﻞّ‪ •1 .‬ﻫﺬه اﳌﻌﺎدﻟﺔ ﻣﻀﺎﻋﻔﺔ اﻟﱰﺑﻴﻊ‪ .‬ﳊ ّﻠﻬﺎ‪ ،‬ﻧﻀﻊ ‪ Z = z‬ﻣﻨﻪ ‪ . Z − (5 − 14ı) Z − 2 (5ı + 12) = 0‬ﳑ ّﻴﺰ ﻫﺬه‬
‫اﳌﺴﺎﻋﺪة ﻫﻮ ‪ . Δ = (− (5 − 14ı)) − 4 × 1 × −2 (5ı + 12) = −75 − 100ı‬ﻟﻴﻜﻦ ‪ 𝛿 = x + ıy‬ﺟﺬر ًا‬‫ِ‬ ‫اﳌﻌﺎدﻟﺔ‬
‫⎧‬ ‫‪x − y = −75‬‬
‫⎪‬
‫⎪‬
‫‪.‬‬ ‫‪2xy = −100‬‬ ‫ﺗﺮﺑﻴﻌﻴ ًﺎ ﻟﻠﻌﺪد ‪ . Δ‬ﻟﺪﻳﻨﺎ ‪ (x + ıy) = −75 − 100ı :‬ﻣﻨﻪ ‪:‬‬
‫⎨‬
‫⎪‬
‫‪ƕ‬‬

‫⎪‬
‫‪⎩ x + y = (−75) + (−100) = 125‬‬
‫ﻧﻌﻮض ﰲ اﳌﻌﺎدﻟﺔ اﻟﺜﺎﻧﻴﺔ ﻟﻨﺠﺪ‬
‫ﻼ‪x = 5‬و ّ‬‫ﺑﺠﻤﻊ اﳌﻌﺎدﻟﺘﲔ اﻷوﱃ و اﻟﺜﺎﻟﺜﺔ ﻧﺠﺪ ‪ 2x = 50‬ﻣﻨﻪ ‪ . x = ±5‬ﻧﺨﺘﺎر ﻣﺜ ً‬
‫)‪5 − 14ı − (5 − 10ı‬‬ ‫ِ‬
‫= ‪Z‬و‬ ‫‪ y = −10‬إذن ‪ . 𝛿 = 5 − 10ı‬ﻣﻨﻪ ﻧﺴﺘﻨﺘﺞ ﺣﻠﻮل اﳌﻌﺎدﻟﺔ اﳌﺴﺎﻋﺪة و ﻫﻲ ‪= −2ı‬‬
‫‪2‬‬
‫‪ũŏ‬‬

‫)‪5 − 14ı + (5 − 10ı‬‬


‫= ‪.Z‬‬ ‫‪= 5 − 12ı‬‬
‫‪2‬‬
‫ﺣﻠﻮل اﳌﻌﺎدﻟﺔ اﻷﺻﻠﻴﺔ ﻫﻲ اﳉﺬور اﻟﱰﺑﻴﻌﻴﺔ ﻟﻠﻌﺪدﻳﻦ ‪ Z‬و ‪ Z‬ﻹﳚﺎدﻫﺎ‪ ،‬ﻧﻌﺘﱪ اﻟﻌﺪدﻳﻦ ‪ a + ıb‬و ‪ c + ıd‬ﺑﺤﻴﺚ‬
‫⎧‬
‫⎪‬ ‫‪a −b =0‬‬
‫⎪‬
‫ﺑﺠﻤﻊ اﳌﻌﺎدﻟﺘﲔ‬ ‫‪.‬‬ ‫‪2ab = −2‬‬ ‫‪ (a + ıb) = 2ı‬و ‪ . (c + ıd) = 5 − 12ı‬ﻟﺪﻳﻨﺎ ‪:‬‬
‫⎨‬
‫⎪‬
‫⎪‬
‫‪⎩ a + b = 0 + (−2) = 2‬‬

‫𝟯𝟳𝟭‬

‫‪http ://tinyurl.com/Malki1718‬‬ ‫‪0‬‬


‫‪ .3.VI‬ﺗﻄﺒﻴﻘﺎت اﻷﻋﺪاد اﳌﺮﻛﺒﺔ‬
‫‪.‬‬
‫ﻧﻌﻮض ﰲ اﳌﻌﺎدﻟﺔ اﻟﺜﺎﻧﻴﺔ ﻟﻨﺠﺪ ‪ . b = −1‬إذن ﻟﻠﻌﺪد‬
‫اﻷوﱃ و اﻟﺜﺎﻟﺜﺔ ﻳﻨﺘﺞ ‪ 2a = 2‬أي ‪ . a = ±1‬ﻧﺨﺘﺎر ‪ a = 1‬و ّ‬
‫‪ Z‬ﺟﺬران ﺗﺮﺑﻴﻌﻴﺎن ﳘﺎ )‪. ± (1 − ı‬‬

‫‪Ŕž Ŧ Ľ‬‬
‫⎧‬
‫⎪‬ ‫‪c −d =5‬‬
‫⎪‬
‫‪ .‬ﺑﺠﻤﻊ اﳌﻌﺎدﻟﺘﲔ اﻷوﱃ و اﻟﺜﺎﻟﺜﺔ ﻳﻨﺘﺞ ‪ 2c = 18‬أي ‪. c = ±3‬‬ ‫‪2cd = −12‬‬ ‫ﺑﺎﳌﺜﻞ ‪:‬‬

‫‪œǃ‬‬
‫⎨‬
‫⎪‬
‫⎪‬
‫‪⎩ c + d = 5 + (−12) = 13‬‬
‫ﻧﻌﻮض ﰲ اﳌﻌﺎدﻟﺔ اﻟﺜﺎﻧﻴﺔ ﻟﻨﺠﺪ ‪ . d = −2‬إذن ﻟﻠﻌﺪد ‪ Z‬ﺟﺬران ﺗﺮﺑﻴﻌﻴﺎن ﳘﺎ )‪. ± (3 − 2ı‬‬
‫ﻧﺨﺘﺎر ‪ c = 3‬و ّ‬
‫ﰲ اﻷﺧﲑ‪ ،‬ﻟﻠﻤﻌﺎدﻟﺔ ا ُﳌﻘﱰﺣﺔ أرﺑﻌﺔ ﺣﻠﻮل و ﻫﻲ ‪ 3 − 2ı ، −1 + ı ، 1 − ı :‬و ‪. −3 + 2ı‬‬

‫‪Ŀ‬‬
‫‪ •2‬ﻣﻔﺘﺎح اﳊﻞ ﻫﻮ ﻣﻼﺣﻈﺔ ّ‬
‫أن ‪ −ı = ı‬ﻣﻨﻪ ‪:‬‬
‫‪z − ı = z + ı = (z + ı) z − ız + ı‬‬ ‫‪= (z + ı) z − ız − 1‬‬
‫إذن ‪:‬‬
‫‪z − ı = 6 (z + ı) ⟺ (z + ı) z − ız − 1 − 6 (z + ı) = 0‬‬
‫‪⟺ (z + ı) z − ız − 1 − 6 = 0‬‬
‫‪⟺ (z + ı) z − ız − 7 = 0‬‬
‫‪⟺ z+ı=0‬‬ ‫أو‬ ‫‪z − ız − 7 = 0‬‬

‫‪ Δ = (−ı) − 4 × 1 × −7 = 27 = 3√3‬إذن ﳍﺬه‬ ‫ﻣﻦ اﳌﻌﺎدﻟﺔ اﻷوﱃ ﻧﺠﺪ ‪ . z = −ı‬ﳑ ّﻴﺰ اﳌﻌﺎدﻟﺔ اﻟﺜﺎﻧﻴﺔ ﻫﻮ‬
‫‪ı + 3√3‬‬ ‫‪ı − 3√3‬‬
‫= ‪.z‬‬ ‫= ‪z‬و‬ ‫اﳌﻌﺎدﻟﺔ ﺣﻼّن ﳘﺎ‬
‫‪2‬‬ ‫‪2‬‬
‫‪1‬‬ ‫‪1‬‬
‫‪.‬‬ ‫‪3√3 + ı‬‬ ‫و‬ ‫‪−3√3 + ı‬‬ ‫ﰲ اﻷﺧﲑ‪ ،‬ﻟﻠﻤﻌﺎدﻟﺔ ﺛﻼﺛﺔ ﺣﻠﻮل ﻫﻲ ‪، −ı :‬‬
‫‪2‬‬ ‫‪2‬‬
‫‪ •3‬اﳌﻤ ﱢﻴﺰ اﳌﺨﺘﴫ ﳍﺬه اﳌﻌﺎدﻟﺔ ﻫﻮ ‪:‬‬
‫)‪Δ = (−ı) − (2 − 4ı) = −3 + 4ı = 1 + (2ı) + 2 × 1 × 2ı = (1 + 2ı‬‬
‫إذن ﻟﻠﻤﻌﺎدﻟﺔ ﺣﻼّن ﳘﺎ ‪ z = ı − (1 + 2ı) = −1 − ı :‬و ‪. z = ı + (1 + 2ı) = 1 + 3ı‬‬
‫‪ņőƱŀ Ţ‬‬
‫‪ •4‬ﺳﻨﻌﺮض ﻃﺮﻳﻘﺘﲔ ّ‬
‫ﳊﻞ ﻫﺬه اﳌﻌﺎدﻟﺔ ‪:‬‬
‫اﻟﻄﺮﻳﻘﺔ اﻷوﱃ ‪ :‬ﻧﻀﻊ ‪ Z = z‬ﻣﻨﻪ اﳌﻌﺎدﻟﺔ ﺗﺼﺒﺢ ‪ . Z − 2Z cos 3𝜃 + 1 = 0 :‬ﳑ ّﻴﺰﻫﺎ اﳌﺨﺘﴫ ﻫﻮ ‪:‬‬
‫)𝜃‪Δ = (− cos 3𝜃) − 1 = cos 3𝜃 − 1 = − sin 3𝜃 = (ı sin 3‬‬

‫‪VI‬‬
‫‪ Z = Z = e‬ﻣﻨﻪ ‪:‬‬ ‫‪ Z = cos 3𝜃 − ı sin 3𝜃 = e−‬و‬ ‫ِ‬
‫اﳌﺴﺎﻋﺪة ﺣﻼّن ﳘﺎ ‪:‬‬ ‫إذن ﻟﻠﻤﻌﺎدﻟﺔ‬

‫‪ z ∈ e− , je− , j e−‬؛‬ ‫‪ z = e−‬و ﺑﺎﻟﺘﺎﱄ ‪:‬‬ ‫•‬

‫‪. z ∈ e , je , j e‬‬ ‫‪ z = e‬و ﺑﺎﻟﺘﺎﱄ ‪:‬‬ ‫أو‬ ‫•‬


‫‪ƕ‬‬

‫𝜋‪2ık‬‬ ‫𝜋‪2ık‬‬
‫‪z = exp 𝜃 +‬‬ ‫‪ z = exp −𝜃 +‬و‬ ‫ﰲ اﻷﺧﲑ‪ ،‬ﺣﻠﻮل اﳌﻌﺎدﻟﺔ )و ﻋﺪدﻫﺎ ﺳﺘﺔ( ﻫﻲ اﻷﻋﺪاد ‪:‬‬
‫‪3‬‬ ‫‪3‬‬
‫ﻣﻊ ‪. 0 ≤ k ≤ 2‬‬
‫‪ũŏ‬‬

‫اﻟﻄﺮﻳﻘﺔ اﻟﺜﺎﻧﻴﺔ ‪ :‬ﻧﻼﺣﻆ ّ‬


‫أن ‪:‬‬

‫‪z − 2z cos 3𝜃 + 1 = z‬‬ ‫‪− e−‬‬ ‫‪+e‬‬ ‫‪z + e−‬‬ ‫‪⋅e‬‬ ‫‪= z − e−‬‬ ‫‪z−e‬‬
‫‪z − 2z cos 3𝜃 + 1 =0 ⟺ z −‬‬ ‫‪e−‬‬ ‫‪=0‬‬ ‫أو‬ ‫‪z −e‬‬ ‫‪=0‬‬ ‫ﻣﻨﻪ ‪:‬‬
‫ﺛﻢ ﻧﺘﻤﻢ اﳊﻞ ﻛﲈ ﰲ اﻟﻄﺮﻳﻘﺔ اﻷوﱃ‪.‬‬
‫ّ‬
‫■‬

‫𝟰𝟳𝟭‬

‫‪http ://tinyurl.com/Malki1718‬‬ ‫‪0‬‬


‫‪ .VI‬ﲤﺎرﻳﻦ ﺗﻄﺒﻴﻘﻴﺔ‬
‫‪.‬‬

‫‬ ‫‪.‬‬
‫𝟏𝟏𝟏‬ ‫‪.‬‬

‫‪Ŕž Ŧ Ľ‬‬
‫ﺣﻞ ﰲ ‪ ℂ‬اﳌﻌﺎدﻻت اﻟﺘﺎﻟﻴﺔ ذات اﳌﺠﻬﻮل ‪: z‬‬
‫‪4z − 2z + 1 = 0‬‬ ‫‪6‬‬ ‫‪z +z+1=0‬‬ ‫‪1‬‬

‫‪œǃ‬‬
‫•‬ ‫•‬

‫‪z + 10z + 169 = 0‬‬ ‫•‬ ‫‪7‬‬ ‫‪z − (1 + 2ı) z + ı − 1 = 0‬‬ ‫‪2‬‬
‫•‬

‫‪z + 2z + 4 = 0‬‬ ‫•‬ ‫‪8‬‬ ‫‪z − √3z − ı = 0‬‬ ‫‪3‬‬


‫•‬

‫‪Ŀ‬‬
‫‪z − (11 − 5ı) z + 24 − 27ı = 0‬‬ ‫•‬ ‫‪9‬‬ ‫‪z − (5 − 14ı) z − 2 (5ı + 12) = 0‬‬ ‫‪4‬‬
‫•‬

‫‪z + 3z − 2ı = 0‬‬ ‫•‬ ‫‪10‬‬ ‫‪z − (3 + 4ı) z − 1 + 5ı = 0‬‬ ‫‪5‬‬


‫•‬

‫ﻌﺘﱪ ﻣﻦ اﳌﻌﺎدﻻت اﻟﺘﻲ ﻗﻤﻨﺎ ﺑﺤ ّﻠﻬﺎ ﰲ اﻟﺘﲈرﻳﻦ اﻟﺴﺎﺑﻘﺔ‪ ،‬ﻧﺮﺟﻮ أن ﺗﻜﻮن ﻗﺪ اِﺗﱠﻀﺤﺖ اﻟﻄﺮﻳﻘﺔ و ﻧﻜﺘﻔﻲ‬ ‫ﺍﳊﻞّ‪ .‬ﻣﻊ اﻟﻜﻢ ا ُﳌ َ‬
‫ﻛﻞ ﻣﻦ ‪ :‬اﳌﻤ ّﻴﺰ ‪) Δ‬أو اﳌﻤ ّﻴﺰ اﳌﺨﺘﴫ ‪ ، (Δ‬أﺣﺪ اﳉﺬرﻳﻦ اﻟﱰﺑﻴﻌﻴﲔ 𝛿 ﻟﻠﻤﻤ ّﻴﺰ )أو اﳌﻤ ّﻴﺰ اﳌﺨﺘﴫ( ‪ ،‬و اﳊﻠﻮل‪.‬‬‫ﺑﺈﻋﻄﺎء ﻗﻴﻤﺔ ّ‬
‫‪1‬‬ ‫‪3‬‬ ‫‪1‬‬ ‫‪3‬‬
‫‪. z = − + √ ı ، z = − − √ ı ، 𝛿 = ı√3 ، Δ = −3 •1‬‬
‫‪2‬‬ ‫‪2‬‬ ‫‪2‬‬ ‫‪2‬‬
‫‪. z = 1 + ı ، z = ı ، 𝛿 = 1 ، Δ = 1 •2‬‬
‫‪3+2 1‬‬ ‫‪√3 − 2 1‬‬
‫√ = ‪.z‬‬ ‫= ‪+ ı ، z‬‬ ‫‪− ı ، 𝛿 = 2 + ı ، Δ = 3 + 4ı •3‬‬
‫‪2‬‬ ‫‪2‬‬ ‫‪2‬‬ ‫‪2‬‬
‫‪. z = 5 − 12ı ، z = −2ı ، 𝛿 = 5 − 10ı ، Δ = −75 − 100ı •4‬‬
‫‪. z = 2 + 3ı ، z = 1 + ı ، 𝛿 = 1 + 2ı ، Δ = −3 + 4ı •5‬‬
‫‪1 √3‬‬ ‫‪1 √3‬‬
‫= ‪.z‬‬ ‫‪+‬‬ ‫‪ı‬‬ ‫= ‪، z‬‬ ‫‪−‬‬ ‫‪ı‬‬ ‫‪، 𝛿 = ı√3 ، Δ = −3 •6‬‬
‫‪4‬‬ ‫‪4‬‬ ‫‪4‬‬ ‫‪4‬‬
‫‪ •7‬ﻧﻀﻊ ‪ Z = z‬ﻓﻴﻜﻮن ‪. Z + 10Z + 169 = 0‬‬
‫‪ņőƱŀ Ţ‬‬
‫‪. Z = −5 + 12ı ، Z = −5 − 12ı ، 𝛿 = 12ı ، Δ = −144‬‬
‫اﳉﺬران اﻟﱰﺑﻴﻌﻴﺎن ﻟﻠﻌﺪد ‪ −5 − 12ı‬ﳘﺎ ‪ 2 − 3ı‬و ‪ −2 + 3ı‬و اﳉﺬران اﻟﱰﺑﻴﻌﻴﺎن ﻟﻠﻌﺪد ‪ −5 + 12ı‬ﳘﺎ ‪ 2 + 3ı‬و‬
‫‪. −2 − 3ı‬‬
‫‪VI‬‬

‫إذن‪ ،‬ﻟﻠﻤﻌﺎدﻟﺔ أرﺑﻌﺔ ﺣﻠﻮل و ﻫﻲ ‪ 2 + 3ı ، −2 + 3ı ، 2 − 3ı :‬و ‪. −2 − 3ı‬‬

‫‪ •8‬ﻧﻀﻊ ‪ Z = z‬ﻓﻴﻜﻮن ‪. Z + 2Z + 4 = 0‬‬


‫‪. Z = −1 + ı√3 ، Z = −1 − ı√3 ، 𝛿 = ı√3 ، Δ = −3‬‬
‫‪ƕ‬‬

‫‪2‬‬ ‫‪6‬‬ ‫‪2‬‬ ‫‪6‬‬


‫اﳉﺬران اﻟﱰﺑﻴﻌﻴﺎن ﻟﻠﻌﺪد ‪ −1 − ı√3‬ﳘﺎ ‪ √ − √ ı‬و ‪ − √ + √ ı‬و اﳉﺬران اﻟﱰﺑﻴﻌﻴﺎن ﻟﻠﻌﺪد ‪ −1 + ı√3‬ﳘﺎ‬
‫‪2‬‬ ‫‪2‬‬ ‫‪2‬‬ ‫‪2‬‬
‫‪2‬‬ ‫‪6‬‬ ‫‪2‬‬ ‫‪6‬‬
‫‪ √ + √ ı‬و ‪. −√ − √ ı‬‬
‫‪2‬‬ ‫‪2‬‬ ‫‪2‬‬ ‫‪2‬‬
‫‪ũŏ‬‬

‫‪2‬‬ ‫‪6‬‬ ‫‪2‬‬ ‫‪6‬‬ ‫‪2‬‬ ‫‪6‬‬ ‫‪2‬‬ ‫‪6‬‬


‫إذن‪ ،‬ﻟﻠﻤﻌﺎدﻟﺔ أرﺑﻌﺔ ﺣﻠﻮل و ﻫﻲ ‪ √ + √ ı − √ + √ ı ، √ − √ ı :‬و ‪. − √ − √ ı‬‬
‫‪2‬‬ ‫‪2‬‬ ‫‪2‬‬ ‫‪2‬‬ ‫‪2‬‬ ‫‪2‬‬ ‫‪2‬‬ ‫‪2‬‬
‫‪. z = 6 − 3ı ، z = 5 − 2ı ، 𝛿 = 1 − ı ، Δ = −2ı •9‬‬
‫أن ‪. z + 3z − 2ı = (z − ı) z + ız + 2‬‬ ‫أن ‪ّ z = ı‬‬
‫ﺣﻞ ﻟﻠﻤﻌﺎدﻟﺔ ‪ .‬ﻧﺠﺪ ّ‬ ‫‪ •10‬ﻧﻼﺣﻆ ّ‬
‫ُﳑ ﱢﻴﺰ اﳌﻌﺎدﻟﺔ ‪ z + ız + 2 = 0‬ﻫﻮ ‪ Δ = −9‬ﻣﻨﻪ ‪ z = −2ı ، 𝛿 = 3ı‬و ‪. z = ı‬‬

‫𝟱𝟳𝟭‬

‫‪http ://tinyurl.com/Malki1718‬‬ ‫‪0‬‬


‫‪ .3.VI‬ﺗﻄﺒﻴﻘﺎت اﻷﻋﺪاد اﳌﺮﻛﺒﺔ‬
‫‪.‬‬
‫■‬

‫‪Ŕž Ŧ Ľ‬‬
‫ﺣﻞ ﰲ ‪ ℂ‬اﳌﻌﺎدﻟﺘﲔ اﻟﺘﺎﻟﻴﺘﲔ ﺛﻢ ﻣ ﱢﺜﻞ اﳊﻠﻮل ﰲ اﳌﺴﺘﻮي اﳌﺮﻛﺐ ‪:‬‬
‫‬ ‫‪.‬‬
‫𝟐𝟏𝟏‬ ‫‪.‬‬

‫‪œǃ‬‬
‫‪z + 2ız + 3 = 0‬‬ ‫•‬ ‫‪2‬‬ ‫‪z = 4ı‬‬ ‫•‬ ‫‪1‬‬

‫‪Ŀ‬‬
‫•‬ ‫‪1‬‬ ‫ﺍﳊﻞّ‪.‬‬

‫||‪z = 4ı ⟺ ||z || = ||4ı‬‬ ‫و‬ ‫‪arg z‬‬ ‫)𝜋‪= arg (4ı) (mod 2‬‬
‫‪⟺ |z| = 4‬‬ ‫و‬ ‫)𝜋‪4 arg (z) = arg (4) + arg (ı) (mod 2‬‬
‫𝜋‬
‫‪⟺ |z| = 4‬‬ ‫و‬ ‫)𝜋‪4 arg (z) = 0 + (mod 2‬‬
‫‪2‬‬
‫𝜋‬
‫‪⟺ |z| = √2‬‬ ‫و‬ ‫‪4 arg (z) = + 2k𝜋 , k ∈ ℤ‬‬
‫‪2‬‬
‫𝜋‬ ‫𝜋‬
‫‪⟺ |z| = √2‬‬ ‫و‬ ‫‪arg (z) = + k , k ∈ ℤ‬‬
‫‪8‬‬ ‫‪2‬‬

‫أي ّ‬
‫أن اﳌﻌﺎدﻟﺔ ﺗﻘﺒﻞ أرﺑﻌﺔ ﺣﻠﻮل ﻫﻲ ‪:‬‬
‫‪/‬‬ ‫‪/‬‬ ‫‪/‬‬ ‫‪/‬‬
‫‪√2e‬‬ ‫‪,‬‬ ‫‪√2e‬‬ ‫‪,‬‬ ‫‪√2e‬‬ ‫‪,‬‬ ‫‪√2e‬‬
‫) و ﻧﺤﺼﻞ ﻋﻠﻴﻬﺎ ﺑﺄﺧﺬ ‪.( k = 0, 1, 2, 3‬‬
‫‪ّ •2‬‬
‫ﳊﻞ ﻫﺬه اﳌﻌﺎدﻟﺔ ) و ﻫﻲ ﻣﻌﺎدﻟﺔ ﻣﻀﺎﻋﻔﺔ اﻟﱰﺑﻴﻊ ( ﻧﻀﻊ ‪ Z = z‬ﻓﻴﻜﻮن ‪. Z + 2ıZ + 3 = 0 :‬‬
‫اﳌﻤ ّﻴﺰ ﻳﺴﺎوي )‪ Δ = (2ı) − 4 × 1 × 3 = −4 − 12 = −16 = (4ı‬ﻣﻨﻪ ‪:‬‬
‫‪−2ı − 4ı‬‬ ‫‪−2ı + 4ı‬‬
‫= ‪Z‬‬ ‫‪= −3ı ,‬‬ ‫= ‪Z‬‬ ‫‪=ı‬‬
‫‪2‬‬ ‫‪2‬‬
‫ﻣﻨﻪ‬
‫‪ņőƱŀ Ţ‬‬
‫||‪z = −3ı ⟺ ||z || = ||−3i‬‬ ‫و‬ ‫‪arg z‬‬ ‫)𝜋‪= arg (−3ı) (mod 2‬‬
‫𝜋‬
‫‪⟺ |z| = 3‬‬ ‫و‬ ‫)𝜋‪2 arg (z) = 𝜋 + (mod 2‬‬
‫‪2‬‬
‫𝜋‪3‬‬

‫‪VI‬‬
‫‪⟺ |z| = √3‬‬ ‫و‬ ‫= )‪arg (z‬‬ ‫)𝜋 ‪(mod‬‬
‫‪4‬‬
‫‪⟺ z = ±e‬‬ ‫‪/‬‬

‫أو‬
‫‪ƕ‬‬

‫||‪z = ı ⟺ ||z || = ||i‬‬ ‫و‬ ‫‪arg z‬‬ ‫)𝜋‪= arg (ı) (mod 2‬‬
‫𝜋‬
‫‪⟺ |z| = 1‬‬ ‫و‬ ‫)𝜋‪2 arg (z) = (mod 2‬‬
‫‪2‬‬
‫𝜋‬
‫‪⟺ |z| = 1‬‬ ‫و‬ ‫)𝜋 ‪arg (z) = (mod‬‬
‫‪4‬‬
‫‪ũŏ‬‬

‫‪⟺ z = ±e‬‬ ‫‪/‬‬

‫■‬ ‫اﻟﺸﻜﻞ ‪ 13.VI‬ﻳﻤ ّﺜﻞ ﺣﻠﻮل اﳌﻌﺎدﻟﺔ اﻷوﱃ و اﻟﺸﻜﻞ ‪ 14.VI‬ﻳﻤ ّﺜﻞ ﺣﻠﻮل اﳌﻌﺎدﻟﺔ اﻟ ّﺜﺎﻧﻴﺔ‪.‬‬

‫𝟲𝟳𝟭‬

‫‪http ://tinyurl.com/Malki1718‬‬ ‫‪0‬‬


‫‪ .VI‬ﲤﺎرﻳﻦ ﺗﻄﺒﻴﻘﻴﺔ‬
‫‪.‬‬

‫‪Ŕž Ŧ Ľ‬‬
‫‪œǃ‬‬
‫‪.‬‬ ‫‪.‬‬

‫‪Ŀ‬‬
‫ﺷﻜﻞ ‪14.VI‬‬ ‫ﺷﻜﻞ ‪13.VI‬‬

‫‪.‬‬
‫𝟑𝟏𝟏‬ ‫‪.‬‬
‫ﺣﻞ ﰲ ‪ ℂ‬اﳌﻌﺎدﻻت اﻟﺘﺎﻟﻴﺔ ذات اﳌﺠﻬﻮل ‪: z‬‬
‫‪27 (z − 1) + (z + 1) = 0‬‬ ‫•‬ ‫‪3‬‬ ‫‪z=z‬‬ ‫‪1‬‬
‫•‬

‫‪1+ı‬‬ ‫‪1‬‬
‫= ‪z‬‬ ‫•‬ ‫‪4‬‬ ‫= ‪z‬‬ ‫‪2‬‬
‫•‬
‫‪√3 − ı‬‬ ‫‪z‬‬

‫‪z = 𝜌e‬‬ ‫ﺣﻞ ﻟﻠﻤﻌﺎدﻟﺔ‪ .‬ﻧﺒﺤﺚ إذن ﻋﻦ اﳊﻠﻮل ﻏﲑ اﳌﻌﺪوﻣﺔ ‪ :‬ﻟﻴﻜﻦ ‪ z ≠ 0‬ﺣﻴﺚ‬‫أن ‪ّ z = 0‬‬ ‫ﺍﳊﻞّ‪ •1 .‬ﺑﺪاﻳ ًﺔ‪ ،‬ﻧﻼﺣﻆ ّ‬
‫ﻣﻊ 𝜌 ﻋﺪد ﺣﻘﻴﻘﻲ ﻣﻮﺟﺐ ﲤﺎﻣ ًﺎ )‪ (𝜌 > 0‬و ‪ . 𝜃 ∈ ℝ‬ﻟﺪﻳﻨﺎ ‪:‬‬

‫𝜌=𝜌‬ ‫‪𝜌 =1‬‬


‫‪z=z‬‬ ‫‪⟺ 𝜌e−‬‬ ‫‪=𝜌 e‬‬
‫‪ņőƱŀ Ţ‬‬
‫⟺‬ ‫⟺‬
‫)𝜋‪−𝜃 ≡ 3𝜃 (mod 2‬‬ ‫)𝜋‪4𝜃 ≡ 0 (mod 2‬‬
‫‪⎧𝜌 = 1‬‬ ‫‪⎧𝜌 = 1‬‬
‫⟺‬ ‫𝜋‬ ‫⟺‬ ‫𝜋‬
‫‪⎨ 𝜃 ≡ 0 mod‬‬ ‫‪⎨𝜃=k ,‬‬ ‫}‪k ∈ {0, 1, 2, 3‬‬
‫⎩‬ ‫‪2‬‬ ‫⎩‬ ‫‪2‬‬
‫‪VI‬‬

‫إذن اﳊﻠﻮل ﻏﲑ اﳌﻌﺪوﻣﺔ ﻟﻠﻤﻌﺎدﻟﺔ ا ُﳌﻘﱰﺣﺔ ﻫﻲ ‪ −1 ، ı ، 1 :‬و ‪. −ı‬‬


‫}‪. {0, 1, −1, ı, −ı‬‬ ‫ﰲ اﻷﺧﲑ‪ ،‬ﳎﻤﻮﻋﺔ ﺣﻠﻮل اﳌﻌﺎدﻟﺔ ﻫﻲ ‪:‬‬
‫‪ •2‬ﺣﺘﻰ ﺗﻜﻮن اﳌﻌﺎدﻟﺔ ﻣﻌﺮﻓﺔ‪ ،‬ﳚﺐ أن ﻳﻜﻮن ‪. z ≠ 0‬‬
‫‪ƕ‬‬

‫‪1‬‬ ‫|‬ ‫‪1‬‬ ‫|‬


‫= |‪ |z‬ﻣﻨﻪ ‪ |z| = 1‬ﻣﻨﻪ ‪|z| = 1‬‬ ‫ﻃﺮﰲ اﳌﻌﺎدﻟﺔ ﻳﻨﺘﺞ | | = || ‪ ||z‬أي‬ ‫ﺑﺪاﻳ ًﺔ‪ ،‬ﻧُﺬﻛّﺮ ّ‬
‫أن |‪ . ||z|| = |z‬ﺑﺄﺧﺬ ﻃﻮﻳﻠﺔ َ ْ‬
‫|‪|z‬‬ ‫|| ‪|| z‬‬
‫‪1‬‬ ‫‪1‬‬ ‫‪1‬‬
‫أي ‪. z = 1‬‬ ‫=‬ ‫إذن = ‪ . z‬و ﺑﺎﻟﺘﺎﱄ‪ ،‬اﳌﻌﺎدﻟﺔ ﺗﻜﺎﻓﺊ ‪:‬‬
‫‪ũŏ‬‬

‫‪z‬‬ ‫‪z‬‬ ‫‪z‬‬


‫‪ z = e‬ﻣﻊ }‪. k ∈ {0, 1, 2, 3, 4‬‬ ‫‪/‬‬ ‫إذن ﺣﻠﻮل اﳌﻌﺎدﻟﺔ ﻫﻲ اﳉﺬور اﳋﺎﻣﺴﺔ ﻟﻠﻮﺣﺪة أي اﻷﻋﺪاد ‪:‬‬

‫‪z+1‬‬
‫‪ .‬ﻧﻀﻊ‬ ‫‪= −27‬‬ ‫‪ •3‬ﺑﲈ ّ‬
‫أن ‪ z = 1‬ﻟﻴﺲ ﺣﻼ ﻟﻠﻤﻌﺎدﻟﺔ ﻓﺒﺈﻣﻜﺎﻧﻨﺎ ﻗﺴﻤﺔ ﻃﺮﻓﻴﻬﺎ ﻋﲆ )‪ (z − 1‬و ﻧﺠﺪ ‪:‬‬
‫‪z−1‬‬
‫𝜋‪ı𝜋 + 2ık‬‬ ‫‪z+1‬‬
‫‪u = √27 exp‬‬ ‫= ‪ u‬ﻓﻴﻜﻮن ‪ u = −27 = 27e‬و ﺑﺎﻟﺘﺎﱄ ّ‬
‫ﻓﺈن اﳊﻠﻮل ﻫﻲ اﻷﻋﺪاد ‪:‬‬
‫‪6‬‬ ‫‪z−1‬‬

‫𝟳𝟳𝟭‬

‫‪http ://tinyurl.com/Malki1718‬‬ ‫‪0‬‬


‫ ﺗﻄﺒﻴﻘﺎت اﻷﻋﺪاد اﳌﺮﻛﺒﺔ‬.3.VI
.
: ‫ أي‬0 ≤ k ≤ 5 ‫ﻣﻊ‬

Ŕž Ŧ Ľ
/ √3 1 3 √3
u = √3e = √3 + ı = + ı
2 2 2 2
ı𝜋 ı𝜋

œǃ
u = √3 exp + = √3e / = ı√3
6 3
ı𝜋 2ı𝜋 −√3 1 3 √3
u = √3 exp + = √3e / = √3 + ı =− + ı
6 3 2 2 2 2

Ŀ
ı𝜋 / −√3 1 3 √3
u = √3 exp + 𝜋 = √3e = √3 − ı =− − ı
6 2 2 2 2
ı𝜋 4ı𝜋 / /
u = √3 exp + = √3e = √3e = −ı√3
6 3
ı𝜋 5ı𝜋 / √3 1 3 √3
u = √3 exp + = √3e = √3 − ı = − ı
6 3 2 2 2 2

z+1
ّ ‫ و ﻧﻼﺣﻆ‬u = x + ıy ‫ ﻧﻀﻊ‬. 0 ≤ k ≤ 5 ‫ﻣﻊ‬
: ‫ ﻣﻨﻪ‬0 ≤ k ≤ 5 ‫ ﻷﺟﻞ‬u ≠ −1 ‫أن‬ =u ‫إذن‬
z−1
z+1 u +1 (u + 1) (u − 1)
=u ⟺ z + 1 = (z − 1) u ⟺ z= ⟺ z=
z−1 u −1 ||u − 1||
||u || − 2ı Im (u ) − 1 3 − 2ıy − 1 2 − 2ıy 1 − ıy
⟺ z= = = =
||u || − 2 Re (u × 1) + ||1|| 3 − 2x + 1 4 − 2x 2−x

: ‫و ﺑﺎﻟﺘﺎﱄ‬
1 − ıy 1−
z = = = 2 − ı√3
2−x 2−
1 − ıy 1 − ı√3 1 √3
ņőƱŀ Ţ
z = = = − ı
2−x 2−0 2 2
1 − ıy 1− ı 2 √3
z = = = − ı
2−x 2+ 7 7

VI
1 − ıy 1+ ı 2 √3
z = = = + ı
2−x 2+ 7 7
1 − ıy 1 + ı√3 1 √3
z = = = + ı
2−x 2−0 2 2
ƕ

1 − ıy 1+
z = = = 2 + ı√3
2−x 2−

.‫و ﻫﻲ ﺣﻠﻮل اﳌﻌﺎدﻟﺔ اﳌﻘﱰﺣﺔ‬


ũŏ

: ‫• ﻫﻨﺎ ﻣﻦ اﻷﻓﻀﻞ إﺳﺘﻌﲈل اﻟﺸﻜﻞ اﻷﳼ‬4

√2 √2 𝜋 𝜋 /
1 + ı = √2 +ı = √2 cos + ı sin = √2e
2 2 4 4

√3 1 𝜋 𝜋
√3 − ı = 2 − ı = 2 cos − + ı sin − = 2e− /
2 2 6 6

𝟭𝟳𝟴

http ://tinyurl.com/Malki1718 0
‫‪ .VI‬ﲤﺎرﻳﻦ ﺗﻄﺒﻴﻘﻴﺔ‬
‫‪.‬‬
‫ﻣﻨﻪ ‪:‬‬
‫‪1+ı‬‬ ‫‪/‬‬
‫‪√2e‬‬ ‫‪√2‬‬ ‫𝜋‪ı𝜋 ı‬‬ ‫‪√2‬‬ ‫‪/‬‬
‫= ‪z‬‬ ‫= ‪⟺ z‬‬ ‫=‬ ‫‪exp‬‬ ‫‪+‬‬ ‫=‬ ‫‪e‬‬

‫‪Ŕž Ŧ Ľ‬‬
‫‪√3 − ı‬‬ ‫‪2e−‬‬ ‫‪/‬‬ ‫‪2‬‬ ‫‪4‬‬ ‫‪6‬‬ ‫‪2‬‬
‫‪2‬‬
‫‪ √ e‬أي اﻷﻋﺪاد ‪:‬‬ ‫‪/‬‬ ‫و ﺑﺎﻟﺘﺎﱄ ّ‬
‫ﻓﺈن اﳊﻠﻮل ﻫﻲ اﳉﺬور اﻟﺜﺎﻣﻨﺔ ﻟﻠﻌﺪد‬

‫‪œǃ‬‬
‫‪2‬‬
‫‪√2‬‬ ‫𝜋‪5ı𝜋 2ık‬‬ ‫‪− /‬‬ ‫𝜋‪5ı𝜋 ık‬‬
‫= ‪z‬‬ ‫‪exp‬‬ ‫‪+‬‬ ‫‪=2‬‬ ‫‪exp‬‬ ‫‪+‬‬ ‫‪,‬‬ ‫‪0≤k≤7‬‬
‫‪2‬‬ ‫‪96‬‬ ‫‪8‬‬ ‫‪96‬‬ ‫‪4‬‬
‫■‬

‫‪Ŀ‬‬
‫‪. z − 3z + 5 + (z − ı) = 0‬‬ ‫ﺣﻞ ﰲ ‪ ℂ‬اﳌﻌﺎدﻟﺔ ذات اﳌﺠﻬﻮل ‪: z‬‬
‫‬ ‫‪.‬‬
‫𝟒𝟏𝟏‬ ‫‪.‬‬

‫ﺍﳊﻞّ‪ .‬ﻧﺴﻤﻲ )‪ (E‬ﻫﺬه اﳌﻌﺎدﻟﺔ‪ .‬ﻟﺪﻳﻨﺎ ‪:‬‬

‫⟺ )‪(E‬‬ ‫‪z − 3z + 5‬‬ ‫)‪= − (z − ı) = ı(z − ı‬‬


‫)‪⟺ z − 3z + 5 = ı(z − ı‬‬ ‫)‪ z − 3z + 5 = −ı(z − ı‬أو‬
‫‪⟺ z − (3 + ı)z + 4 = 0‬‬ ‫‪ z − (3 − ı)z + 6 = 0‬أو‬

‫ُﳑ ﱢﻴﺰ اﳌﻌﺎدﻟﺔ اﻷوﱃ ﻫﻮ )‪ Δ = (3 + ı) − 4 × 4 = −8 + 6ı = (1 + 3ı‬إذن ﳍﺎ ﺣﻼّن ﳘﺎ ‪:‬‬ ‫•‬


‫‪3 + ı − 1 − 3ı‬‬ ‫‪3 + ı + 1 + 3ı‬‬
‫= ‪z‬‬ ‫‪=1−ı‬‬ ‫;‬ ‫= ‪z‬‬ ‫‪= 2 + 2ı‬‬
‫‪2‬‬ ‫‪2‬‬

‫ُﳑ ﱢﻴﺰ اﳌﻌﺎدﻟﺔ اﻟﺜﺎﻧﻴﺔ ‪ . Δ = (3 − ı) − 4 × 6 = −16 − 6ı‬ﻟﻴﻜﻦ ‪ 𝛿 = x + ıy‬أﺣﺪ اﳉﺬرﻳﻦ اﻟﱰﺑﻴﻌﻴﲔ ﳍﺬا ا ُﳌﻤ ﱢﻴﺰ ‪.‬‬ ‫•‬

‫⎧‬ ‫‪x − y = −16‬‬


‫⎪‬
‫⎪‬
‫‪ .‬ﺑﺠﻤﻊ اﳌﻌﺎدﻟﺘﲔ اﻷوﱃ و اﻟﺜﺎﻟﺜﺔ ﻳﻨﺘﺞ ‪ 2x = −16 + 2√73‬ﻣﻨﻪ‬ ‫‪2xy = −6‬‬ ‫ﻟﺪﻳﻨﺎ ‪:‬‬
‫⎨‬
‫⎪‬
‫⎪‬
‫‪ņőƱŀ Ţ‬‬
‫‪⎩ x + y = 16 + 6 = 2√73‬‬
‫ٌﻌﻮض ﰲ اﳌﻌﺎدﻟﺔ اﻷوﱃ ﻓﻴﻜﻮن‬
‫ﻼ ‪ x = −8 + √73‬و ﻧ ّ‬ ‫‪ . x = ± −8 + √73‬ﻧﺨﺘﺎر ﻣﺜ ً‬
‫ﻓﺈن ‪. y = − 8 + √73‬‬‫أن ‪) y < 0‬ﻣﻦ اﳌﻌﺎدﻟﺔ اﻟﺜﺎﻧﻴﺔ( ّ‬
‫‪ . y = x + 16 = 8 + √73‬و ﺑﲈ ّ‬
‫‪VI‬‬

‫= 𝛿 = ‪ Δ‬ﻣﻨﻪ اﳊﻼّن ‪:‬‬ ‫‪−8 + √73 − ı 8 + √73‬‬ ‫إذن‬

‫‪3−ı−‬‬ ‫‪−8 + √73 + ı 8 + √73 3 −‬‬ ‫‪−8 + √73 1 −‬‬ ‫‪8 + √73‬‬
‫= ‪z‬‬ ‫=‬ ‫‪−‬‬ ‫‪ı‬‬
‫‪2‬‬ ‫‪2‬‬ ‫‪2‬‬
‫‪3−ı+‬‬ ‫‪−8 + √73 − ı 8 + √73 3 +‬‬ ‫‪−8 + √73 1 +‬‬ ‫‪8 + √73‬‬
‫‪ƕ‬‬

‫= ‪z‬‬ ‫=‬ ‫‪−‬‬ ‫‪ı‬‬


‫‪2‬‬ ‫‪2‬‬ ‫‪2‬‬

‫■‬ ‫‪. z ,z ,z ,z‬‬ ‫ﰲ اﻷﺧﲑ‪ ،‬ﳎﻤﻮﻋﺔ ﺣﻠﻮل اﳌﻌﺎدﻟﺔ )‪ (E‬ﻫﻲ ‪:‬‬


‫‪ũŏ‬‬

‫‪. (z − 1) + (z − 1) (z + 1) + (z − 1) (z + 1) + (z + 1) = 0‬‬ ‫ﺣﻞ ﰲ ‪ ℂ‬اﳌﻌﺎدﻟﺔ ‪:‬‬


‫‬ ‫‪.‬‬
‫𝟓𝟏𝟏‬ ‫‪.‬‬

‫𝟵𝟳𝟭‬

‫‪http ://tinyurl.com/Malki1718‬‬ ‫‪0‬‬


‫‪ .3.VI‬ﺗﻄﺒﻴﻘﺎت اﻷﻋﺪاد اﳌﺮﻛﺒﺔ‬
‫‪.‬‬
‫ﺍﳊﻞّ‪ .‬ﻧﺴﻤﻲ )‪ (E‬ﻫﺬه اﳌﻌﺎدﻟﺔ‪.‬‬

‫‪Ŕž Ŧ Ľ‬‬
‫ﺑﲈ ّ‬
‫أن ‪ z = −1‬ﻟﻴﺲ ﺣﻼ ﻟﻠﻤﻌﺎدﻟﺔ )‪ (E‬ﻓﺒﺈﻣﻜﺎﻧﻨﺎ ﻗﺴﻤﺔ ﻃﺮﻓﻴﻬﺎ ﻋﲆ )‪ (z + 1‬ﻓﻴﻨﺘﺞ ‪:‬‬
‫‪z−1‬‬ ‫‪z−1‬‬ ‫‪z−1‬‬
‫⟺ )‪(E‬‬ ‫‪+‬‬ ‫‪+‬‬ ‫‪+1=0‬‬

‫‪œǃ‬‬
‫‪z+1‬‬ ‫‪z+1‬‬ ‫‪z+1‬‬
‫‪z−1‬‬
‫إذن ‪:‬‬ ‫‪ . z = 1 + Z‬ﻧﻼﺣﻆ أﻧﻪ ﻟﻜﻞ ﻋﺪد ﻣﺮﻛﺐ ‪ّ z‬‬
‫ﻓﺈن ‪≠ 1‬‬ ‫= ‪ Z‬ﻣﻨﻪ‬
‫‪z−1‬‬
‫ﻧﻀﻊ‬
‫‪z+1‬‬ ‫‪1−Z‬‬ ‫‪z+1‬‬
‫‪Z −1‬‬

‫‪Ŀ‬‬
‫⟺ ‪(E) ⟺ Z + Z + Z + 1 = 0‬‬ ‫‪=0 ⟺ Z =1‬‬
‫‪Z−1‬‬
‫‪1+Z‬‬
‫=‪z‬‬ ‫ﻣﻨﻪ }‪) Z ∈ {1, −1, ı, −ı‬اﳉﺬور اﻟﺮاﺑﻌﺔ ﻟﻠﻮﺣﺪة( ‪ .‬ﻟﻜﻦ ‪ Z ≠ 1‬إذن اﳊﻠﻮل اﳌﻤﻜﻨﺔ ﻫﻲ ‪ ı ، −1 :‬و ‪ −ı‬و ﺑﲈ ّ‬
‫أن‬
‫‪1−Z‬‬
‫ّ‬
‫ﻓﺈن ‪:‬‬
‫)‪1 + (−1‬‬
‫=‪z‬‬ ‫‪=0‬‬
‫)‪1 − (−1‬‬
‫)‪1 + ı (1 + ı‬‬ ‫‪1 + 2ı − 1‬‬
‫=‪z‬‬ ‫=‬ ‫=‬ ‫‪=ı‬‬ ‫أو‬
‫‪1−ı‬‬ ‫‪1 +1‬‬ ‫‪2‬‬
‫)‪1 + (−ı) (1 − ı‬‬ ‫‪1 − 2ı − 1‬‬
‫=‪z‬‬ ‫=‬ ‫=‬ ‫‪= −ı‬‬ ‫أو‬
‫)‪1 − (−ı‬‬ ‫‪1 +1‬‬ ‫‪2‬‬

‫■‬ ‫‪ ı ، 0‬و ‪. −ı‬‬ ‫ﰲ اﻷﺧﲑ‪ ،‬ﺣﻠﻮل اﳌﻌﺎدﻟﺔ اﳌﻘﱰﺣﺔ ﻫﻲ ‪:‬‬

‫‬ ‫‪.‬‬
‫𝟔𝟏𝟏‬ ‫‪.‬‬

‫‪ •1‬ﺣﻞ ﰲ ‪ ℂ‬اﳌﻌﺎدﻟﺔ ذات اﳌﺠﻬﻮل ‪: z‬‬


‫‪z − (16 − ı) z + (89 − 16ı) z + 89ı = 0‬‬ ‫)𝟑(‬
‫‪ņőƱŀ Ţ‬‬
‫‪ •2‬ﻟﺘﻜﻦ ‪ c ، b ، a‬ﺣﻠﻮل اﳌﻌﺎدﻟﺔ )𝟑( و ‪ُ C ، B ، A‬ﺻ َﻮر ﻫﺬه اﳊﻠﻮل ﰲ اﳌﺴﺘﻮي اﳌﺮﻛﺐ ‪ .‬ﻣﺎ ﻫﻲ ﻃﺒﻴﻌﺔ اﳌﺜﻠﺚ ‪ ABC‬؟‬

‫‪ •1‬ﻟﺪﻳﻨﺎ ‪:‬‬ ‫ﺍﳊﻞّ‪.‬‬

‫‪(𝟑) ⟺ z + ız − 16z − 16ız + 89 + 89ı‬‬ ‫‪VI‬‬


‫‪⟺ z (z + ı) − 16z (z + ı) + 89 (z + ı) = 0‬‬
‫‪⟺ (z + ı) z − 16z + 89 = 0‬‬
‫‪ƕ‬‬

‫إذن ﺣﻠﻮل اﳌﻌﺎدﻟﺔ )𝟑( ﻫﻲ اﻷﻋﺪاد ‪ z‬ﺑﺤﻴﺚ ‪ z + ı = 0‬أو ‪. z − 16z + 89 = 0‬‬


‫ﻟﻠﻤﻌﺎدﻟﺔ اﻷوﱃ ّ‬
‫ﺣﻞ وﺣﻴﺪ ﻫﻮ ‪ z = −ı‬؛‬ ‫•‬
‫‪ũŏ‬‬

‫اﳌﻤ ﱢﻴﺰ اﳌﺨﺘﴫ ﻟﻠﻤﻌﺎدﻟﺔ اﻟﺜﺎﻧﻴﺔ ﻫﻮ )‪ Δ = 8 − 89 = −25 = (5ı‬إذن ﳍﺎ ﺣﻼّن ﳘﺎ ‪ z = 8 − 5ı‬و‬ ‫•‬
‫‪. z = 8 + 5ı‬‬
‫ﰲ اﻷﺧﲑ‪ ،‬ﳎﻤﻮﻋﺔ ﺣﻠﻮل اﳌﻌﺎدﻟﺔ )𝟑( ﻫﻲ ‪. {−ı, 8 − 5ı, 8 + 5ı} :‬‬
‫•‬ ‫‪2‬‬

‫𝟬𝟴𝟭‬

‫‪http ://tinyurl.com/Malki1718‬‬ ‫‪0‬‬


‫‪ .VI‬ﲤﺎرﻳﻦ ﺗﻄﺒﻴﻘﻴﺔ‬
‫‪.‬‬
‫‪8 + 5ı‬‬
‫‪5‬‬
‫ﻧﻀﻊ ‪ b = 8 − 5ı ، a = −ı‬و ‪. c = 8 + 5ı‬‬
‫‪4‬‬
‫ﰲ ﻫﺬا اﻟﻨﻮع ﻣﻦ اﻷﺳﺌﻠﺔ‪ ،‬ﻣﻦ اﻷﻓﻀﻞ اﻹﺳﺘﻌﺎﻧﺔ ﺑﺸﻜﻞ‪.‬‬

‫‪Ŕž Ŧ Ľ‬‬
‫‪3‬‬

‫‪2‬‬
‫أن اﳌﺜﻠﺚ اﳌﻄﻠﻮب ﻣﺘﺴﺎوي‬ ‫ﻧﻼﺣﻆ ﻋﲆ اﻟﺸﻜﻞ ﺟﺎﻧﺒﻪ ّ‬
‫اﻟﺴﺎﻗﲔ ﻋﻨﺪ اﻟﺮأس ‪C‬ذي اﻟﻼﺣﻘﺔ ‪ c = 8 + 5ı‬و ﻫﺬا‬

‫‪œǃ‬‬
‫‪1‬‬

‫‪−1‬‬ ‫‪0‬‬ ‫‪1‬‬ ‫‪2‬‬ ‫‪3‬‬ ‫‪4‬‬ ‫‪5‬‬ ‫‪6‬‬ ‫‪7‬‬ ‫‪8‬‬
‫ﻣﺎ ﻳﺜﺒﺘﻪ اﳊﺴﺎب ‪:‬‬
‫‪−1‬‬ ‫‪−ı‬‬ ‫||)‪||(8 + 5ı) − (−ı‬‬ ‫‪= ||8 + 6ı|| = 10‬‬
‫‪−2‬‬
‫‪||(8 + 5ı) − (8 − 5ı)|| = ||10ı|| = 10‬‬

‫‪Ŀ‬‬
‫‪−3‬‬

‫‪−4‬‬
‫إذن اﳌﺜﻠﺚ اﻟﺬي ﻟﻮاﺣﻖ رؤوﺳﻪ ﻫﻲ ﺣﻠﻮل اﳌﻌﺎدﻟﺔ )𝟑(‬
‫‪−5‬‬ ‫ﻣﺘﺴﺎوي اﻟﺴﺎﻗﲔ ﻋﻨﺪ اﻟﺮأس ذي اﻟﻼﺣﻘﺔ ‪. 8 + 5ı‬‬
‫‪8 − 5ı‬‬
‫■‬

‫‪ . P(z) = z + 4z + 16‬ﻧﻌﺘﱪ ﰲ ‪ ℂ‬اﳌﻌﺎدﻟﺔ ‪:‬‬ ‫ﻟﻜﻞ ﻋﺪد ﻣﺮﻛﺐ ‪ z‬ﻧﻀﻊ ‪:‬‬


‫‬ ‫‪.‬‬
‫𝟕𝟏𝟏‬ ‫‪.‬‬

‫‪P(z) = 0‬‬ ‫)𝟒(‬


‫‪. P(z) = z + az + 4 z − az + 4‬‬ ‫‪ّ •1‬‬
‫ﻋﲔ اﻟﻌﺪد اﳊﻘﻴﻘﻲ اﳌﻮﺟﺐ ‪ a‬ﺑﺤﻴﺚ ﻳﻜﻮن ‪:‬‬

‫‪ •2‬أوﺟﺪ ﺣﻠﻮل اﳌﻌﺎدﻟﺔ )𝟒( ‪ .‬ﻧﺮﻣﺰ ﺑـِ ‪ z‬إﱃ اﳊﻞ اﻟﺬي ﺟﺰآه اﳊﻘﻴﻘﻲ و اﻟﺘﺨﻴﲇ ﻣﻮﺟﺒﺎن‪.‬‬
‫ﲢﻘﻖ ﻣﻦ ّ‬
‫أن اﳊﻠﻮل ﻫﻲ ‪ z ، −z ، z‬و ‪ . −z‬أﻛﺘﺐ ﻫﺬه اﳊﻠﻮل ﻋﲆ اﻟﺸﻜﻞ اﳌﺜﻠﺜﻲ‪.‬‬

‫‪ •3‬اﳌﺴﺘﻮي ﻣﻨﺴﻮب إﱃ ﻣﻌﻠﻢ ﻣﺘﻌﺎﻣﺪ و ﻣﺘﺠﺎﻧﺲ ⃗ ‪) O, i,⃗ j‬وﺣﺪة اﻟﻄﻮل ‪. (4 cm :‬‬

‫)ا( ﻋﻠﻢ اﻟﻨﻘﻂ ‪ C ، B ، A‬و ‪ D‬ﻟﻮاﺣﻖ اﻷﻋﺪاد ‪ −z ، −z ، z‬و ‪ z‬ﻋﲆ اﻟﱰﺗﻴﺐ‪.‬‬


‫)ب( أﺛﺒﺖ ّ‬
‫أن ﻫﺬه اﻟﻨﻘﻂ اﻷرﺑﻌﺔ ﺗﻘﻊ ﻋﲆ اﻟﺪاﺋﺮة اﻟﺘﻲ ﻣﺮﻛﺰﻫﺎ ‪ O‬و ﻧﺼﻒ ﻗﻄﺮﻫﺎ ‪. 2‬‬
‫‪ņőƱŀ Ţ‬‬
‫‪ •1‬ﻟﺪﻳﻨﺎ ﺑﻌﺪ اﻟﻨﴩ و اﻟﱰﺗﻴﺐ ‪:‬‬ ‫ﺍﳊﻞّ‪.‬‬
‫‪z + az + 4‬‬ ‫‪z − az + 4 = z + 8 − a‬‬ ‫‪z + 16‬‬
‫‪VI‬‬

‫و ﺑﺎﻟﺘﺎﱄ ‪ P(z) = z + az + 4 z − az + 4 :‬إذا و ﻓﻘﻂ إذا ﻛﺎن ‪ 8−a = 4‬أي إذا و ﻓﻘﻂ إذا ﻛﺎن ‪.a = ±2‬‬
‫ﻟﻜﻦ ‪ a ≥ 0‬إذن ‪. a = 2‬‬
‫ﰲ اﻷﺧﲑ ‪. P(z) = z + 2z + 4 z − 2z + 4 :‬‬
‫‪ P (z) = 0 •2‬إذا و ﻓﻘﻂ إذا ﻛﺎن ‪ z + 2z + 4 = 0‬أو ‪. z − 2z + 4 = 0‬‬
‫‪ƕ‬‬

‫‪ Δ = 1 − 4 = −3 = ı√3‬و ﺑﺎﻟﺘﺎﱄ ﳍﺎ ﺣﻼّن ﳘﺎ ‪:‬‬ ‫اﳌﻤ ﱢﻴﺰ اﳌﺨﺘﴫ ﻟﻠﻤﻌﺎدﻟﺔ ‪ z + 2z + 4 = 0‬ﻫﻮ‬ ‫•‬

‫‪ −1 ± ı√3‬؛‬
‫‪ũŏ‬‬

‫‪ Δ = (−1) − 4 = −3 = ı√3‬إذن ﳍﺎ ﺣﻼّن ﳘﺎ ‪:‬‬ ‫و اﳌﻤ ﱢﻴﺰ اﳌﺨﺘﴫ ﻟﻠﻤﻌﺎدﻟﺔ ‪ z − 2z + 4 = 0‬ﻫﻮ‬ ‫•‬

‫‪. 1 ± ı√3‬‬
‫أن ‪ Re (z ) ≥ 0‬و ‪ّ Im (z ) ≥ 0‬‬
‫ﻓﺈن ‪ z = 1 + ı√3‬ﻣﻨﻪ ‪:‬‬ ‫ﺑﲈ ّ‬
‫‪1 − ı√3 = z‬‬ ‫‪,‬‬ ‫‪−1 − ı√3 = −z‬‬ ‫‪,‬‬ ‫‪−1 + ı√3 = −z‬‬

‫𝟭𝟴𝟭‬

‫‪http ://tinyurl.com/Malki1718‬‬ ‫‪0‬‬


‫‪ .3.VI‬ﺗﻄﺒﻴﻘﺎت اﻷﻋﺪاد اﳌﺮﻛﺒﺔ‬
‫‪.‬‬

‫‪2‬‬

‫‪Ŕž Ŧ Ľ‬‬
‫‪B‬‬ ‫‪A‬‬

‫‪œǃ‬‬
‫‪1‬‬

‫‪Ŀ‬‬
‫‪O‬‬
‫‪−2‬‬ ‫‪−1‬‬ ‫‪1‬‬ ‫‪2‬‬

‫‪−1‬‬

‫‪C‬‬ ‫‪D‬‬
‫‪−2‬‬

‫ﺷﻜﻞ ‪15.VI‬‬

‫اﻟﺸﻜﻞ اﳌﺜﻠﺜﻲ ﻟﻠﺤﻠﻮل ﻫﻮ ‪:‬‬

‫‪1‬‬ ‫‪√3‬‬ ‫𝜋‬ ‫𝜋‬ ‫𝜋‬


‫‪z =2‬‬ ‫‪+ı‬‬ ‫‪= 2 cos‬‬ ‫‪+ ı sin‬‬ ‫‪= 2,‬‬
‫‪2‬‬ ‫‪2‬‬ ‫‪3‬‬ ‫‪3‬‬ ‫‪3‬‬
‫𝜋‬ ‫𝜋‬ ‫𝜋‪4‬‬ ‫𝜋‪2‬‬
‫‪−z = − 2,‬‬ ‫‪= 2, + 𝜋 = 2,‬‬ ‫‪= 2, −‬‬
‫‪3‬‬ ‫‪3‬‬ ‫‪3‬‬ ‫‪3‬‬
‫𝜋‬ ‫𝜋‬
‫‪z = 2,‬‬ ‫‪= 2, −‬‬
‫‪ņőƱŀ Ţ‬‬
‫‪3‬‬ ‫‪3‬‬
‫𝜋‪2‬‬ ‫𝜋‪2‬‬
‫‪−z = (−z ) = 2, −‬‬ ‫‪= 2,‬‬
‫‪3‬‬ ‫‪3‬‬

‫‪) •3‬ا( أﻧﻈﺮ اﻟﺸﻜﻞ ‪. 15.VI‬‬

‫‪OA = ||z − 0|| = 2 , OB = ||−z − 0|| = 2 , OC = ||−z − 0|| = 2 , OD = ||z − 0|| = 2‬‬
‫)ب( ﻟﺪﻳﻨﺎ ‪:‬‬
‫‪VI‬‬
‫أي ‪ OA = OB = OC = OD‬و ﻫﺬا ﻳﻌﻨﻲ ّ‬
‫أن اﻟﻨﻘﻂ ‪ C ، B ، A‬و ‪ D‬ﺗﻨﺘﻤﻲ ﻛﻠﻬﺎ إﱃ اﻟﺪاﺋﺮة اﻟﺘﻲ ﻣﺮﻛﺰﻫﺎ ‪O‬‬
‫و ﻧﺼﻒ ﻗﻄﺮﻫﺎ ‪. OA = 2‬‬
‫‪ƕ‬‬

‫■‬

‫‪.‬‬
‫𝟖𝟏𝟏‬ ‫‪.‬‬
‫‪ũŏ‬‬

‫ّ‬
‫ﺣﻞ ﰲ ﳎﻤﻮﻋﺔ اﻷﻋﺪاد اﳌﺮﻛﺒﺔ اﳌﻌﺎدﻟﺔ ‪ z − 2ze + 1 = 0‬ﺣﻴﺚ 𝜃 وﺳﻴﻂ ﺣﻘﻴﻘﻲ و ‪ z ∈ ℂ‬ﻫﻮ‬
‫اﳌﺠﻬﻮل‪.‬‬
‫ﻣﺎ ﻫﻮ اﳌﺤﻞ اﳍﻨﺪﳼ ﻟﻠﺤﻠﻮل ﰲ اﳌﺴﺘﻮي اﳌﺮﻛﺐ ﻋﻨﺪﻣﺎ ﻳﻤﺴﺢ اﻟﻮﺳﻴﻂ 𝜃 اﳌﺠﻤﻮﻋﺔ ‪ ℝ‬؟‬

‫ﺍﳊﻞّ‪ .‬اﳌﻌﺎدﻟﺔ ﺗُﻜﺎﻓﺊ ‪:‬‬


‫‪z−e‬‬ ‫‪=e‬‬ ‫‪− 1 = 2ıe sin 𝜃 = 2e‬‬ ‫‪+ /‬‬ ‫𝜃 ‪sin‬‬

‫𝟮𝟴𝟭‬

‫‪http ://tinyurl.com/Malki1718‬‬ ‫‪0‬‬


‫‪ .VI‬ﲤﺎرﻳﻦ ﺗﻄﺒﻴﻘﻴﺔ‬
‫‪.‬‬
‫‪. z = e ± (1 + ı) e‬‬ ‫‪/‬‬ ‫‪/‬‬ ‫‪/‬‬ ‫إذا ﻛﺎن ‪ّ sin 𝜃 ≥ 0‬‬
‫‪ z = e ± √2e‬أي 𝜃 ‪√sin‬‬ ‫‪e‬‬ ‫ﻓﺈن اﳊﻠﻮل ﻫﻲ 𝜃 ‪√sin‬‬ ‫•‬

‫‪Ŕž Ŧ Ľ‬‬
‫‪/‬‬
‫‪.z = e ± (1 − ı) e‬‬ ‫‪ z = e ± √2e−‬أي 𝜃 ‪√− sin‬‬ ‫‪/‬‬ ‫‪e‬‬ ‫‪/‬‬ ‫إذا ﻛﺎن ‪ّ sin 𝜃 < 0‬‬
‫ﻓﺈن اﳊﻠﻮل ﻫﻲ 𝜃 ‪√− sin‬‬ ‫•‬

‫اﳌﺤﻞ اﳍﻨﺪﳼ ﻟﻠﺤﻠﻮل ‪ :‬اﻟﻌﺪد اﳌﺮﻛﺐ ‪ z‬ﻳﻨﺘﻤﻲ إﱃ ﳎﻤﻮﻋﺔ اﳊﻠﻮل إذا و ﻓﻘﻂ إذا ُوﺟﺪ ﻋﺪد ﺣﻘﻴﻘﻲ 𝜃 ﺑﺤﻴﺚ ‪:‬‬

‫‪œǃ‬‬
‫|‬‫‪z +1‬‬ ‫|‬
‫| ‪ .‬ﻫﺬا اﻟﴩط ُﻳﻜﺘﺐ أﻳﻀ ًﺎ |‪. ||z + 1|| = 2 |z‬‬ ‫‪ ، z − 2ze + 1 = 0‬و ﻫﺬا اﻟﴩط ُﻳﻜﺎﻓﺊ ‪ z ≠ 0‬و ‪| = ||e || = 1‬‬
‫|| ‪|| 2z‬‬
‫ﻧﻀﻊ ‪ z = x + ıy‬ﻣﻊ ‪ . x, y ∈ ℝ‬ﻟﺪﻳﻨﺎ ‪:‬‬

‫‪Ŀ‬‬
‫||‪||z + 1|| = 2 |z| ⟺ ||x − y + 1 + 2ıxy|| = 2 ||x + ıy‬‬

‫⟺‬ ‫‪x −y +1‬‬ ‫‪+ 4x y = 2 x + y‬‬

‫⟺‬ ‫‪x −y +1‬‬ ‫‪+ 4x y = 4 x + y‬‬

‫‪ . x + y‬ﻟﻜﻦ ‪:‬‬ ‫ﺑﻌﺪ اﻟﻨﴩ و اﻟﺘﺒﺴﻴﻂ ﻳﻨﺘﺞ ‪+ 1 − 2x − 6y = 0 :‬‬

‫‪x +y‬‬ ‫‪+ 1 − 2x − 6y = x + y‬‬ ‫‪−2 x +y‬‬ ‫‪+ 1 − 4y‬‬


‫‪= x +y −1‬‬ ‫‪− 4y = x + y − 2y − 1‬‬ ‫‪x + y + 2y − 1‬‬
‫‪= x + (y − 1) − 2‬‬ ‫‪x + (y + 1) − 2‬‬

‫ﻣﻨﻪ‬
‫‪x +y‬‬ ‫‪+ 1 − 2x − 6y = 0 ⟺ x + (y − 1) − 2 = 0‬‬ ‫أو‬ ‫‪x + (y + 1) − 2 = 0‬‬

‫ﻓﺈن اﳌﺤﻞ اﳍﻨﺪﳼ ﳊﻠﻮل اﳌﻌﺎدﻟﺔ اﳌﻌﻄﺎة ﻫﻮ اﲢﺎد اﻟﺪاﺋﺮﺗﲔ ‪ 𝒞 (0, 1) , √2‬و ‪. 𝒞 (0, −1) , √2‬‬‫و ﺑﺎﻟﺘﺎﱄ ّ‬
‫■‬ ‫ﻫﺎﺗﺎن اﻟﺪاﺋﺮﺗﺎن ﺗﺘﻘﺎﻃﻌﺎن ﰲ اﻟﻨﻘﻄﺘﲔ )‪ (1, 0‬و )‪. (−1, 0‬‬

‫‪#‬‬ ‫‪.‬‬
‫𝟗𝟏𝟏‬ ‫‪.‬‬
‫‪ņőƱŀ Ţ‬‬
‫‪1 + ız‬‬ ‫𝛼 ‪1 + ı tan‬‬
‫‪.‬‬ ‫=‬ ‫𝜋 ‪ّ . 𝛼 ∈ −𝜋,‬‬
‫ﺣﻞ ﰲ ‪ ℂ‬اﳌﻌﺎدﻟﺔ ‪:‬‬ ‫ﻟﻴﻜﻦ‬
‫‪1 − ız‬‬ ‫𝛼 ‪1 − ı tan‬‬ ‫‪2 2‬‬

‫ﺍﳊﻞّ‪ .‬ﻟﻴﻜﻦ‬
‫𝜋 𝜋‬
‫‪VI‬‬

‫‪ . 𝛼 ∈ − ,‬ﻟﺪﻳﻨﺎ ‪:‬‬
‫‪2 2‬‬
‫‪sin‬‬
‫‪1 + ı tan 𝛼 1 +‬‬ ‫‪ı cos‬‬ ‫𝛼 ‪cos 𝛼 + ı sin‬‬ ‫‪e‬‬
‫=‬ ‫‪sin‬‬
‫=‬ ‫‪= −‬‬ ‫‪=e‬‬
‫‪1 − ı tan 𝛼 1 −‬‬ ‫‪ı cos‬‬ ‫‪cos 𝛼 − ı sin 𝛼 e‬‬
‫‪1 + ız‬‬
‫= ‪ . Z‬ﻫﻞ ﻳﻤﻜﻦ أن ﻳﻜﻮن ‪ Z = −1‬؟ إذا ﻛﺎن ‪ّ Z = −1‬‬
‫ﻓﺈن )‪ 1 + ız = − (1 − ız‬أي ‪ 0z = 2‬و ﻫﺬا‬ ‫ﻧﻀﻊ‬
‫‪1 − ız‬‬
‫‪ƕ‬‬

‫‪1−Z‬‬
‫‪ . z = ı‬إذن ‪:‬‬ ‫ﻣﺴﺘﺤﻴﻞ إذن ‪ Z ≠ −1‬و ﺑﺎﻟﺘﺎﱄ ‪ Z − ızZ = 1 + ız‬ﻣﻨﻪ‬
‫‪1+Z‬‬

‫‪1 + ız‬‬ ‫𝛼 ‪1 + ı tan‬‬


‫‪⟺ Z =e‬‬
‫‪ũŏ‬‬

‫=‬
‫‪1 − ız‬‬ ‫𝛼 ‪1 − ı tan‬‬
‫‪⟺ Z=j z‬‬ ‫‪/‬‬ ‫‪,‬‬ ‫‪k ∈ {0, 1, 2} ,‬‬ ‫‪j=e‬‬ ‫‪/‬‬

‫‪⟺ Z=e‬‬ ‫‪+‬‬ ‫‪/‬‬ ‫‪,‬‬ ‫}‪k ∈ {0, 1, 2‬‬


‫‪1−e‬‬ ‫‪+‬‬ ‫‪/‬‬
‫‪⟺ z=ı‬‬ ‫‪+‬‬ ‫‪/‬‬
‫‪,‬‬ ‫}‪k ∈ {0, 1, 2‬‬
‫‪1+e‬‬

‫𝟯𝟴𝟭‬

‫‪http ://tinyurl.com/Malki1718‬‬ ‫‪0‬‬


‫‪ .3.VI‬ﺗﻄﺒﻴﻘﺎت اﻷﻋﺪاد اﳌﺮﻛﺒﺔ‬
‫‪.‬‬
‫𝜋‪2𝛼 + 2k‬‬
‫= ‪ . t‬ﻟﺪﻳﻨﺎ ‪:‬‬ ‫ﻧﻀﻊ‬
‫‪3‬‬

‫‪Ŕž Ŧ Ľ‬‬
‫‪−e‬‬ ‫‪/‬‬ ‫‪e‬‬ ‫‪/‬‬ ‫‪− e−‬‬ ‫‪/‬‬
‫‪1−e‬‬ ‫‪−2ı sin‬‬ ‫‪t‬‬
‫=‬ ‫=‬ ‫‪= −ı tan‬‬
‫‪1+e‬‬ ‫‪e‬‬ ‫‪/‬‬ ‫‪e‬‬ ‫‪/‬‬ ‫‪+ e−‬‬ ‫‪/‬‬ ‫‪2 cos‬‬ ‫‪2‬‬

‫‪œǃ‬‬
‫‪1 + ız‬‬ ‫𝛼 ‪1 + ı tan‬‬ ‫𝜋‪1 2𝛼 + 2k‬‬
‫=‬ ‫‪⟺ z = ı ⋅ (−ı) tan‬‬ ‫‪,‬‬ ‫}‪k ∈ {0, 1, 2‬‬ ‫ﻣﻨﻪ ‪:‬‬
‫‪1 − ız‬‬ ‫𝛼 ‪1 − ı tan‬‬ ‫‪2‬‬ ‫‪3‬‬
‫𝜋‪𝛼 + k‬‬
‫‪⟺ z = tan‬‬ ‫}‪, k ∈ {0, 1, 2‬‬

‫‪Ŀ‬‬
‫‪3‬‬

‫𝛼‬ ‫𝜋‪𝛼+‬‬ ‫𝜋‪𝛼 + 2‬‬


‫■‬ ‫‪. tan , tan‬‬ ‫‪, tan‬‬ ‫إذن ﳎﻤﻮﻋﺔ ﺣﻠﻮل اﳌﻌﺎدﻟﺔ ﻫﻲ ‪:‬‬
‫‪3‬‬ ‫‪3‬‬ ‫‪3‬‬

‫‪.‬‬
‫𝟎𝟐𝟏‬ ‫‪.‬‬
‫أوﺟﺪ ﺛﻼﺛﺔ أﻋﺪاد ﻣﺮﻛﺒﺔ ﻃﻮﻳﻠﺘﻬﺎ ‪ 1‬ﺑﺤﻴﺚ ﻳﻜﻮن ﺟﺪاؤﻫﺎ ‪ 1‬و ﳎﻤﻮﻋﻬﺎ ‪. 1‬‬

‫ﺍﳊﻞّ‪ .‬ﻟﺘﻜﻦ ‪ z ، z ، z‬ﻫﺬه اﻷﻋﺪاد‪ .‬ﻟﻨﺒﺤﺚ ﻋﻦ اﻟﴩط اﻟﻼزم اﻟﺬي ﳚﺐ أن ُﲢﻘﻘﻪ ﻫﺬه اﻷﻋﺪاد‪.‬‬
‫ﻓﺈن ‪ z = 1 ، z = 1‬و ‪ z = 1‬ﻣﻨﻪ )ﻋﻠ ًﲈ ّ‬
‫أن ‪: (z z z = 1‬‬ ‫أن ﻃﻮﻳﻠﺘﻬﺎ ‪ّ 1‬‬
‫ﺑﲈ ّ‬
‫‪z‬‬ ‫‪z‬‬ ‫‪z‬‬
‫‪1‬‬ ‫‪1‬‬ ‫‪1‬‬ ‫‪z z +z z +z z‬‬
‫= ‪1=z +z +z‬‬ ‫‪+‬‬ ‫‪+‬‬ ‫=‬ ‫‪=z z +z z +z z‬‬
‫‪z‬‬ ‫‪z‬‬ ‫‪z‬‬ ‫‪z z z‬‬
‫و ﺑﺎﻟﺘﺎﱄ ّ‬
‫ﻓﺈن ‪:‬‬

‫‪z +z +z =1‬‬
‫‪z z +z z +z z =1‬‬
‫‪z z z =1‬‬
‫‪ņőƱŀ Ţ‬‬
‫ﻣﻨﻪ ‪:‬‬
‫‪(X − z ) (X − z ) (X − z ) = X − (z + z + z ) X + (z z + z z + z z ) X − z z z‬‬
‫‪= X − X + X − 1 = X (X − 1) + 1 ⋅ (X − 1) = (X − 1) X + 1‬‬

‫‪. z ,z ,z‬‬ ‫‪ (X − 1) X + 1‬و ﺑﺎﻟﺘﺎﱄ ّ‬


‫ﻓﺈن }‪= {1, ı, −ı‬‬ ‫أن اﻷﻋﺪاد ‪ z ، z‬و ‪ z‬ﻫﻲ ﺟﺬور ﻛﺜﲑ اﳊﺪود‬ ‫أي ّ‬
‫‪VI‬‬
‫ﺣﻞ ﻟﻠﻤﺴﺄﻟﺔ ّ‬
‫ﻷن ‪:‬‬ ‫ﻣﻦ ﺟﻬﺔ أﺧﺮى‪ ،‬اﻷﻋﺪاد ‪ ı ، 1‬و ‪ّ −ı‬‬
‫ﻃﻮﻳﻠﺔ ّ‬
‫ﻛﻞ ﻣﻨﻬﺎ ﺗﺴﺎوي ‪، 1‬‬ ‫•‬
‫‪ƕ‬‬

‫ﳎﻤﻮﻋﻬﺎ ‪، 1 + ı − ı = 1‬‬ ‫•‬

‫و ﺟﺪاؤﻫﺎ ‪. 1 ⋅ ı ⋅ (−ı) = 1‬‬ ‫•‬


‫‪ũŏ‬‬

‫أي ّ‬
‫أن اﻷﻋﺪاد اﻟﺘﻲ ﻧﺒﺤﺚ ﻋﻨﻬﺎ ﻫﻲ )دون اﻋﺘﺒﺎر اﻟﱰﺗﻴﺐ( ‪ ı ، 1‬و ‪. −ı‬‬
‫ﰲ اﻷﺧﲑ‪ ،‬ﻋﺪد اﳊﻠﻮل ﻫﻮ ﻋﺪد ﺗﺒﺪﻳﻼت ﳎﻤﻮﻋﺔ ذات ﺛﻼث ﻋﻨﺎﴏ أي ‪ 3! = 6‬و ﻫﺬه اﳊﻠﻮل ﻫﻲ ‪:‬‬
‫)‪(1, ı, −ı) , (1, −ı, ı) , (ı, 1, −ı) , (ı, −ı, 1) , (−ı, 1, ı) , (−ı, ı, 1‬‬

‫أن ﳎﻤﻮع اﻷﻋﺪاد اﳌﺮﻛﺒﺔ اﻟﺘﻲ ﻧﺒﺤﺚ ﻋﻨﻬﺎ ﻳﺴﺎوي ‪ 1‬ﻓﺒﺈﻣﻜﺎﻧﻨﺎ أن ﻧﻔﺮض ّ‬
‫أن ﻫﺬه اﻷﻋﺪاد ﻫﻲ ‪ z ، 1‬و ‪.−z‬‬ ‫ّ‬
‫ﺣﻞ آﺧﺮ ‪ :‬ﺑﺎﻋﺘﺒﺎر ّ‬
‫‪1‬‬
‫أن ﺣﺎﺻﻞ ﴐﲠﺎ ﻳﺴﺎوي ‪ 1‬ﻓﺒﺈﻣﻜﺎﻧﻨﺎ أن ﻧﻔﺮض ّ‬
‫أن ﻫﺬه اﻷﻋﺪاد ﻫﻲ ‪ z ، 1‬و ‪.‬‬ ‫و ﺑﺎﻋﺘﺒﺎر ّ‬
‫‪z‬‬

‫𝟰𝟴𝟭‬

‫‪http ://tinyurl.com/Malki1718‬‬ ‫‪0‬‬


‫‪ .VI‬ﲤﺎرﻳﻦ ﺗﻄﺒﻴﻘﻴﺔ‬
‫‪.‬‬
‫‪1‬‬ ‫‪1‬‬ ‫‪1‬‬ ‫‪1‬‬
‫‪ {1, z, −z} = 1, z,‬ﻣﻨﻪ ‪ z = 1 :‬أو ‪ −z = 1‬أو ‪ z = −z‬أو ‪ = 1‬أو ‪ = z‬أو ‪. = −z‬‬ ‫و ﺑﺎﻟﺘﺎﱄ ّ‬
‫ﻓﺈن‬
‫‪z‬‬ ‫‪z‬‬ ‫‪z‬‬ ‫‪z‬‬

‫‪Ŕž Ŧ Ľ‬‬
‫‪1‬‬
‫ﻧﺘﺤﻘﻖ ﺑﺴﻬﻮﻟﺔ ّ‬
‫أن اﳊﺎﻟﺔ اﳌﻤﻜﻨﺔ اﻟﻮﺣﻴﺪة ﻫﻲ ‪ = −z‬ﻣﻨﻪ ‪ z = −1‬أي ‪ z = ±ı‬و ﻧﺼﻞ إﱃ ﻧﻔﺲ اﳋﻼﺻﺔ‪.‬‬
‫‪z‬‬
‫■‬

‫‪œǃ‬‬
‫‪.‬‬
‫𝟏𝟐𝟏‬ ‫‪.‬‬
‫‪ •1‬ﺣﻞ ﰲ ‪ ℂ‬اﳌﻌﺎدﻟﺔ ‪:‬‬

‫‪Ŀ‬‬
‫‪1+z+z +z +z +z +z =0‬‬ ‫)𝟓(‬
‫‪ •2‬ﻧﻌﺘﱪ ﰲ اﳌﺠﻤﻮﻋﺔ ‪ ℂ‬اﳌﻌﺎدﻟﺔ ‪:‬‬
‫‪z + z − 2z − 1 = 0‬‬ ‫)𝟔(‬
‫‪1‬‬
‫‪ a +‬ﺣﻼ ﻟﻠﻤﻌﺎدﻟﺔ )𝟔( ‪.‬‬ ‫ﺣﻞ ﻟﻠﻤﻌﺎدﻟﺔ )𝟓( إذا و ﻓﻘﻂ إذا ﻛﺎن‬ ‫)ا( ﻟﻴﻜﻦ ∗‪ . a ∈ ℂ‬أﺛﺒﺖ ّ‬
‫أن ‪ّ a‬‬
‫‪a‬‬
‫)ب( إﺳﺘﻨﺘﺞ ﺣﻠﻮل اﳌﻌﺎدﻟﺔ )𝟔( ‪.‬‬
‫𝜋‪6‬‬ ‫𝜋‪4‬‬ ‫𝜋‪2‬‬
‫‪ cos‬ﻻ ﺗﻨﺘﻤﻲ إﱃ ‪) ℚ‬أي ﻟﻴﺴﺖ أﻋﺪاد ًا ﻧﺎﻃﻘﺔ(‪.‬‬ ‫‪ cos‬و‬ ‫‪، cos‬‬ ‫‪ •3‬أﺛﺒﺖ ّ‬
‫أن اﻷﻋﺪاد‬
‫‪7‬‬ ‫‪7‬‬ ‫‪7‬‬

‫‪ •1‬ﻧﻼﺣﻆ ّ‬
‫أن ‪ z = 1‬ﻟﻴﺲ ﺣﻼ ﻟﻠﻤﻌﺎدﻟﺔ )𝟓( إذن ‪:‬‬ ‫ﺍﳊﻞّ‪.‬‬

‫‪z −1‬‬ ‫‪/‬‬


‫⟺ )𝟓(‬ ‫‪z≠1‬‬ ‫و‬ ‫‪=0‬‬ ‫⟺‬ ‫‪z≠1‬‬ ‫و‬ ‫‪z =1‬‬ ‫‪⟺ z=z‬‬ ‫‪,‬‬ ‫}‪k ∈ {1, 2, 3, 4, 5, 6‬‬
‫‪z−1‬‬

‫أي ّ‬
‫أن اﳊﻠﻮل ﻫﻲ اﳉﺬور اﻟﺴﺎﺑﻌﺔ ﻟﻠﻮﺣﺪة )ﺑﺎﺳﺘﺜﻨﺎء ‪(1‬‬
‫𝜋‪2k‬‬ ‫𝜋‪2k‬‬
‫‪ z = cos‬ﻣﻊ }‪. k ∈ {1, 2, 3, 4, 5, 6‬‬
‫‪+ ı sin‬‬ ‫و ﻫﻲ اﻷﻋﺪاد ‪:‬‬
‫‪7‬‬ ‫‪7‬‬
‫‪ņőƱŀ Ţ‬‬
‫أن ﻃﻮﻳﻠﺔ ﻛﻞ ﻣﻨﻬﺎ ﻫﻲ ‪ّ 1‬‬
‫ﻓﺈن‬ ‫ﻣﻼﺣﻈﺔ ‪ ⧏ : 29‬ﺑﲈ ّ‬
‫أن ﻣﻌﺎﻣﻼت اﳌﻌﺎدﻟﺔ )𝟓(ﺣﻘﻴﻘﻴﺔ ﻓﺈﳖﺎ ﻣﱰاﻓﻘﺔ ﻣﺜﻨﻰ ﻣﺜﻨﻰ و ﺑﲈ ّ‬
‫‪1‬‬
‫= ‪ z‬ﻣﻨﻪ ‪:‬‬
‫‪z‬‬
‫‪1‬‬ ‫‪1‬‬ ‫‪−‬‬
‫= ‪z‬‬ ‫=‬ ‫‪= e−‬‬ ‫‪/ =e‬‬ ‫‪=e −‬‬ ‫‪/ =z‬‬
‫‪VI‬‬

‫‪/‬‬ ‫‪−‬‬
‫‪z‬‬ ‫‪e‬‬
‫‪.z = z‬‬ ‫‪−‬‬ ‫‪=z‬‬ ‫و‬ ‫‪z =z‬‬ ‫‪−‬‬ ‫‪=z‬‬ ‫‪،‬‬ ‫‪z =z‬‬ ‫‪−‬‬ ‫‪=z‬‬ ‫إذن ‪:‬‬
‫⧐‬

‫‪ •2‬ﻟﺪﻳﻨﺎ ‪:‬‬
‫‪ƕ‬‬

‫‪1‬‬ ‫‪1‬‬ ‫‪1‬‬ ‫‪1‬‬


‫‪ّ a+‬‬
‫ﺣﻞ ﻟﻠﻤﻌﺎدﻟﺔ )𝟔(‬ ‫⟺‬ ‫‪a+‬‬ ‫‪+ a+‬‬
‫‪−2 a+‬‬ ‫‪−1=0‬‬
‫‪a‬‬ ‫‪a‬‬ ‫‪a‬‬ ‫‪a‬‬
‫‪1‬‬ ‫‪1‬‬ ‫‪1‬‬ ‫‪1‬‬ ‫‪1‬‬ ‫‪2‬‬
‫‪⟺ a + 3a ⋅ + 3a‬‬ ‫‪+‬‬ ‫‪+ a + 2a ⋅ +‬‬ ‫‪− 2a − − 1 = 0‬‬
‫‪ũŏ‬‬

‫‪a‬‬ ‫‪a‬‬ ‫‪a‬‬ ‫‪a a‬‬ ‫‪a‬‬


‫‪1‬‬ ‫‪1‬‬ ‫‪1‬‬
‫‪⟺ a +a +a+1+ +‬‬ ‫‪+‬‬ ‫‪=0‬‬
‫‪a a‬‬ ‫‪a‬‬
‫‪1‬‬
‫⟺‬ ‫‪a +a +a +a +a +a+1 =0‬‬
‫‪a‬‬
‫‪⟺ a +a +a +a +a +a+1=0‬‬
‫‪ّ a‬‬
‫ﺣﻞ ﻟﻠﻤﻌﺎدﻟﺔ )𝟓( ⟺‬

‫𝟱𝟴𝟭‬

‫‪http ://tinyurl.com/Malki1718‬‬ ‫‪0‬‬


‫‪ .3.VI‬ﺗﻄﺒﻴﻘﺎت اﻷﻋﺪاد اﳌﺮﻛﺒﺔ‬
‫‪.‬‬
‫)ا( ﻟﻴﻜﻦ }‪ . k ∈ {1, 2, 3, 4, 5, 6‬ﻟﺪﻳﻨﺎ ‪:‬‬ ‫•‬ ‫‪3‬‬

‫‪Ŕž Ŧ Ľ‬‬
‫‪ّ z + 1‬‬
‫ﺣﻞ ﻟﻠﻤﻌﺎدﻟﺔ )𝟔( ⟺ ‪ّ z‬‬
‫ﺣﻞ ﻟﻠﻤﻌﺎدﻟﺔ )𝟓(‬
‫‪z‬‬
‫‪ّ z +z‬‬
‫ﺣﻞ ﻟﻠﻤﻌﺎدﻟﺔ )𝟔( ⟺‬

‫‪œǃ‬‬
‫) ‪ّ 2 Re (z‬‬
‫ﺣﻞ ﻟﻠﻤﻌﺎدﻟﺔ )𝟔( ⟺‬

‫إذن ﺣﻠﻮل اﳌﻌﺎدﻟﺔ )𝟔( ﻫﻲ اﻷﻋﺪاد ) ‪ 2 Re (z‬ﺣﻴﺚ }‪ . k ∈ {1, 2, 3, 4, 5, 6‬ﻟﻜﻦ ‪:‬‬

‫‪Ŀ‬‬
‫𝜋‪2‬‬
‫‪Re (z ) = cos‬‬ ‫) ‪= Re (z ) = Re (z‬‬
‫‪7‬‬
‫𝜋‪4‬‬
‫‪Re (z ) = cos‬‬ ‫) ‪= Re (z ) = Re (z‬‬
‫‪7‬‬
‫𝜋‪6‬‬
‫‪Re (z ) = cos‬‬ ‫) ‪= Re (z ) = Re (z‬‬
‫‪7‬‬

‫𝜋‪2‬‬ ‫𝜋‪4‬‬ ‫𝜋‪6‬‬


‫‪. 𝒮 = 2 cos‬‬ ‫‪, 2 cos , 2 cos‬‬ ‫أي ّ‬
‫أن ﳎﻤﻮﻋﺔ ﺣﻠﻮل اﳌﻌﺎدﻟﺔ )𝟔( ﻫﻲ‬
‫‪7‬‬ ‫‪7‬‬ ‫‪7‬‬
‫ﻣﻼﺣﻈﺔ ‪ ⧏ : 30‬ﺑﲈ أﻧﻨﺎ وﺟﺪﻧﺎ ‪ِ 6‬ﻗ َﻴﻢ ﻓﻬﺬا ﻻ ﻳﻌﻨﻲ ّ‬
‫أن اﳌﻌﺎدﻟﺔ )𝟔( ﺗﻘﺒﻞ ‪ 6‬ﺣﻠﻮل )و ﻫﻮ ﳾء ﻣﺴﺘﺤﻴﻞ‬
‫أن ﺗﻜﺮار ﺑﻌﺾ اﻟﻌﻨﺎﴏ ﻻ ُﻳ ِ‬
‫ﻐﲑ اﳌﺠﻤﻮﻋﺔ أي ‪:‬‬ ‫ﻷن درﺟﺘﻬﺎ ﻫﻲ ‪ (3‬ﻟﻜﻦ ﻣﻦ ﺧﻮاص اﳌﺠﻤﻮﻋﺎت ّ‬ ‫ّ‬
‫𝜋‪2‬‬ ‫𝜋‪4‬‬ ‫𝜋‪6‬‬ ‫𝜋‪2‬‬ ‫𝜋‪4‬‬ ‫𝜋‪6‬‬ ‫𝜋‪2‬‬ ‫𝜋‪4‬‬ ‫𝜋‪6‬‬
‫= 𝒮‬ ‫‪2 cos‬‬ ‫‪, 2 cos , 2 cos , 2 cos , 2 cos , 2 cos‬‬ ‫=‬ ‫‪2 cos‬‬ ‫‪, 2 cos , 2 cos‬‬
‫‪7‬‬ ‫‪7‬‬ ‫‪7‬‬ ‫‪7‬‬ ‫‪7‬‬ ‫‪7‬‬ ‫‪7‬‬ ‫‪7‬‬ ‫‪7‬‬
‫⧐‬
‫𝜋‪4‬‬ ‫𝜋‪2‬‬
‫‪2 cos‬‬ ‫أن اﻷﻋﺪاد ‪، 2 cos‬‬‫أن اﻷﻋﺪاد 𝜋‪ cos 4𝜋 ، cos 2‬و 𝜋‪ cos 6‬ﻻ ﺗﻨﺘﻤﻲ إﱃ ‪ ℚ‬ﻳﻜﻔﻲ أن ﻧﺜﺒﺖ ّ‬ ‫ﺣﺘﻰ ﻧﺜﺒﺖ ّ‬ ‫•‬ ‫‪4‬‬
‫‪7‬‬ ‫‪7‬‬ ‫‪7‬‬ ‫‪7‬‬ ‫‪7‬‬
‫و 𝜋‪ 2 cos 6‬ﻻ ﺗﻨﺘﻤﻲ إﱃ ‪ ℚ‬و ﻫﺬا ﻳﻜﺎﻓﺊ إﺛﺒﺎت ّ‬
‫أن اﳌﻌﺎدﻟﺔ )𝟔( ﻻ ﺗﻘﺒﻞ ﺣﻼ ﻧﺎﻃﻘ ًﺎ )أي ﰲ ‪.( ℚ‬‬
‫‪7‬‬
‫‪ņőƱŀ Ţ‬‬
‫أن اﳌﻌﺎدﻟﺔ )𝟔( ﺗﻘﺒﻞ ﺣﻼ ﻧﺎﻃﻘ ًﺎ و ﻧﱪﻫﻦ ّ‬
‫أن ﻫﺬا ﻳﺆدي إﱃ ﺗﻨﺎﻗﺾ‪.‬‬ ‫ﻧﻔﺮض إذن ّ‬
‫‪p‬‬
‫ﻟﻴﻜﻦ = ‪ z‬ﺣﻼ ﻧﺎﻃﻘ ًﺎ ﻟﻠﻤﻌﺎدﻟﺔ ﻣﻊ ‪ q ∈ ℕ∗ ، p ∈ ℤ‬و ‪ . pgcd (p, q) = 1‬ﻟﺪﻳﻨﺎ ‪:‬‬
‫‪q‬‬

‫‪VI‬‬
‫‪p‬‬ ‫‪p‬‬ ‫‪p‬‬
‫‪+‬‬ ‫‪−2‬‬ ‫‪− 1 = 0 ⟺ p + p q − 2pq − q = 0‬‬
‫‪q‬‬ ‫‪q‬‬ ‫‪q‬‬

‫ﻣﻨﻪ ﻧﺴﺘﻨﺘﺞ ّ‬
‫أن ‪ q = p p + pq − 2q‬و ‪ p = q q + 2pq − p‬و ﺑﺎﻟﺘﺎﱄ ‪ p ∣ q‬و ‪. q ∣ p‬‬
‫أن اﳌﻌﺎدﻟﺔ )𝟔( ﺗﻘﺒﻞ ‪ −1‬و ‪ 1‬ﺣ ّﻠﲔ ﳍﺎ و‬
‫ﻟﻜﻦ ‪ pgcd (p, q) = 1‬إذن ‪ ||p|| = 1‬و ‪ ||q|| = 1‬أي ‪ z = ±1‬و ﻫﺬا ﻳﻌﻨﻲ ّ‬
‫‪ƕ‬‬

‫ﻫﺬا ﺗﻨﺎﻗﺾ!‬
‫إذن ﺣﻠﻮل اﳌﻌﺎدﻟﺔ )𝟔( ﻻ ﺗﻨﺘﻤﻲ إﱃ ‪. ℚ‬‬
‫■‬
‫‪ũŏ‬‬

‫أوﺟﺪ ﺟﺬور ﻛﺜﲑ اﳊﺪود ‪. P (X) = 1 + X + X + X + X + X‬‬


‫‬ ‫‪.‬‬
‫𝟐𝟐𝟏‬ ‫‪.‬‬

‫𝟲𝟴𝟭‬

‫‪http ://tinyurl.com/Malki1718‬‬ ‫‪0‬‬


‫‪ .VI‬ﲤﺎرﻳﻦ ﺗﻄﺒﻴﻘﻴﺔ‬
‫‪.‬‬
‫= ‪) 1 + X + X + ⋯ + X‬ﳎﻤﻮع اﳊﺪود اﻷوﱃ ﳌﺘﺘﺎﻟﻴﺔ ﻫﻨﺪﺳﻴﺔ(‪ .‬ﻟﺪﻳﻨﺎ ‪:‬‬
‫‪1−X‬‬
‫ﺍﳊﻞّ‪ .‬ﻣﻔﺘﺎح ّ‬
‫اﳊﻞ ﻫﻮ ﻣﻼﺣﻈﺔ ّ‬
‫أن‬
‫‪1−X‬‬

‫‪Ŕž Ŧ Ľ‬‬
‫‪(1 − X) P (X) = (1 − X) 1 + X + X + X + X + X‬‬

‫‪=1−X‬‬

‫‪œǃ‬‬
‫‪= 1−X‬‬ ‫‪1+X‬‬

‫‪= (1 − X) 1 + X + X‬‬ ‫‪(1 + X) 1 − X + X‬‬

‫‪Ŀ‬‬
‫ﻟﻜﻦ ‪ P(1) = 6 ≠ 0‬أي ‪ 1‬ﻟﻴﺲ ﺟﺬر ًا ﻟﻜﺜﲑ اﳊﺪود )‪ P (X‬و ﺑﻘﺴﻤﺔ اﻟﻄﺮﻓﲔ ﻋﲆ ‪ 1 − X‬ﻳﻨﺘﺞ ‪:‬‬
‫‪P (X) = 1 + X + X‬‬ ‫‪(1 + X) 1 − X + X‬‬

‫ﳑ ﱢﻴﺰ ‪ 1 + X + X‬ﻫﻮ ‪ Δ = 1 − 4 = −3 < 0‬إذن ﻟﻴﺲ ﻟﻪ ﺟﺬور ﺣﻘﻴﻘﻴﺔ‪ .‬ﺑﺎﳌﺜﻞ‪ 1 − X + X ،‬ﻟﻴﺲ ﻟﻪ ﺟﺬور ﰲ ‪ّ ℝ‬‬
‫ﻷن‬
‫ﳑ ﱢﻴﺰه ‪ Δ = 1 − 4 = −3 < 0‬و ﺑﺎﻟﺘﺎﱄ ﻓﺎﳉﺬر اﳊﻘﻴﻘﻲ اﻟﻮﺣﻴﺪ ﻟـِ )‪ P (X‬ﻫﻮ ‪. −1‬‬
‫ﰲ اﳌﺠﻤﻮﻋﺔ ‪ ، ℂ‬ﺟﺬر َا ‪ 1 + X + X‬ﳘﺎ ‪ j = e /‬و ‪ ، j = j‬و ﺟﺬر َا ‪ 1 − X + X‬ﳘﺎ ‪ −j‬و ‪ −j‬و ﺑﺎﻟﺘﺎﱄ ﻓﻠﻜﺜﲑ اﳊﺪود‬
‫)‪ P (X‬ﲬﺴﺔ ﺟﺬور ﻣﺮﻛﺒﺔ ﻫﻲ ‪ −j ، j ، j ، −1‬و ‪ −j‬أي ‪:‬‬
‫‪P (X) = (1 + X) 1 + X + X‬‬ ‫‪1−X+X‬‬ ‫‪= (X + 1) (X − j) X − j (X + 1) (X + j) X + j‬‬
‫■‬

‫ﺛﻢ ﰲ ‪. ℝ‬‬
‫ﺣﻠﻞ ﻛﺜﲑ اﳊﺪود ‪ P (X) = 1 + X + X + X + X‬ﰲ اﳌﺠﻤﻮﻋﺔ ‪ّ ℂ‬‬
‫‪#‬‬ ‫‪.‬‬
‫𝟑𝟐𝟏‬ ‫‪.‬‬

‫ﺍﳊﻞّ‪ .‬ﻟﺪﻳﻨﺎ ‪:‬‬

‫‪(X − 1) P (X) = (X − 1) 1 + X + X + X + X‬‬ ‫‪=X −1‬‬


‫‪ņőƱŀ Ţ‬‬
‫إذن‪ ،‬ﺟﺬور )‪ (X − 1) P (X‬ﻫﻲ اﳉﺬور اﳋﺎﻣﺴﺔ ﻟﻠﻮﺣﺪة أي اﻷﻋﺪاد ‪ e /‬ﻣﻊ }‪. k ∈ {0, 1, 2, 3, 4‬‬
‫‪e‬‬ ‫‪ P‬ﻫﻲ اﻷﻋﺪاد ‪/‬‬ ‫ﻟﻜﻦ ‪ P(1) = 5 ≠ 0‬أي ‪ 1‬ﻟﻴﺲ ﺟﺬر ًا ﻟﻜﺜﲑ اﳊﺪود )‪ P (X‬و ﻫﺬا ﻳﻌﻨﻲ ّ‬
‫أن ﺟﺬور )‪(X‬‬
‫‪ e / = e− / ، e / ، e /‬و ‪ e / = e− /‬و ﺑﺎﻟﺘﺎﱄ ّ‬
‫ﻓﺈن ‪:‬‬ ‫ﻣﻊ }‪ k ∈ {1, 2, 3, 4‬أي‬
‫‪VI‬‬

‫‪P (X) = X − e‬‬ ‫‪/‬‬ ‫‪X − e−‬‬ ‫‪/‬‬ ‫‪X−e‬‬ ‫‪/‬‬ ‫‪X − e−‬‬ ‫‪/‬‬

‫و ﻫﻮ ﲢﻠﻴﻞ )‪ P (X‬ﰲ اﳌﺠﻤﻮﻋﺔ ‪. ℂ‬‬


‫ﻟﻠﺤﺼﻮل ﻋﲆ ﲢﻠﻴﻞ )‪ P (X‬ﰲ اﳌﺠﻤﻮﻋﺔ ‪ ℝ‬ﻳﻜﻔﻲ أﺧﺬ ّ‬
‫ﻛﻞ ﺣﺪّ ﻳﻦ ﻣﱰاﻓﻘﲔ ‪:‬‬
‫‪ƕ‬‬

‫‪X−e‬‬ ‫‪/‬‬ ‫‪X − e−‬‬ ‫‪/‬‬ ‫‪=X − e‬‬ ‫‪/‬‬ ‫‪+ e−‬‬ ‫‪/‬‬ ‫‪+e‬‬ ‫‪/‬‬ ‫‪⋅ e−‬‬ ‫‪/‬‬

‫𝜋‪2‬‬
‫‪= X − 2 cos‬‬ ‫‪+1‬‬
‫‪5‬‬
‫‪ũŏ‬‬

‫‪X−e‬‬ ‫‪/‬‬ ‫‪X − e−‬‬ ‫‪/‬‬ ‫‪=X − e‬‬ ‫‪/‬‬ ‫‪+ e−‬‬ ‫‪/‬‬ ‫‪+e‬‬ ‫‪/‬‬ ‫‪⋅ e−‬‬ ‫‪/‬‬ ‫و‬
‫𝜋‪4‬‬
‫‪= X − 2 cos‬‬ ‫‪+1‬‬
‫‪5‬‬
‫𝜋‪2‬‬ ‫𝜋‪4‬‬
‫‪P (X) = X − 2 cos‬‬ ‫‪+ 1 ⋅ X − 2 cos‬‬ ‫‪+1‬‬ ‫ﻣﻨﻪ‬
‫‪5‬‬ ‫‪5‬‬

‫■‬ ‫و ﻫﻮ ﲢﻠﻴﻞ )‪ P (X‬ﰲ اﳌﺠﻤﻮﻋﺔ ‪. ℝ‬‬

‫𝟳𝟴𝟭‬

‫‪http ://tinyurl.com/Malki1718‬‬ ‫‪0‬‬


‫‪ .3.VI‬ﺗﻄﺒﻴﻘﺎت اﻷﻋﺪاد اﳌﺮﻛﺒﺔ‬
‫‪.‬‬

‫‬ ‫‪.‬‬
‫𝟒𝟐𝟏‬ ‫‪.‬‬

‫‪Ŕž Ŧ Ľ‬‬
‫ﻓﻴﲈ ﻳﲇ‪ f(z) ،‬ﻛﺜﲑ ﺣﺪود ﻟﻠﻤﺘﻐﲑ اﳌﺮﻛﺐ ‪. z‬‬

‫‪œǃ‬‬
‫أن اﳌﻌﺎدﻟﺔ ‪ f(z) = 0‬ﺗﻘﺒﻞ ﺣﻼ ﺣﻘﻴﻘﻴ ًﺎ 𝛼 ‪.‬‬
‫أﺛﺒﺖ ّ‬ ‫•‬

‫أوﺟﺪ اﻷﻋﺪاد اﳌﺮﻛﺒﺔ ‪ c ، b ، a‬اﻟﺘﻲ ﲢﻘﻖ ﻣﻦ أﺟﻞ ﻛﻞ ﻋﺪد ﻣﺮﻛﺐ ‪.f(z) = (z − 𝛼) az + bz + c : z‬‬ ‫•‬

‫ّ‬
‫ﺣﻞ ﰲ اﳌﺠﻤﻮﻋﺔ ‪ ℂ‬اﳌﻌﺎدﻟﺔ ‪. f(z) = 0 :‬‬

‫‪Ŀ‬‬
‫•‬

‫ﰲ ﻛﻞ ﺣﺎﻟﺔ ﻣﻦ اﳊﺎﻻت اﻵﺗﻴﺔ ‪:‬‬


‫‪f(z) = z + (3 + 5ı) z + (6 + 15ı) z + 18‬‬ ‫•‬ ‫‪1‬‬
‫‪f(z) = z − (11 + 2ı) z + 2 (17 + 7ı) z − 42‬‬ ‫•‬ ‫‪2‬‬
‫‪f(z) = z + (2ı − 5) z + 7 (1 − ı) z − 2 + 6ı‬‬ ‫•‬ ‫‪3‬‬
‫)‪f(z) = ız + (2ı − 1) z − (ı + 4) z + 3 (2ı − 1‬‬ ‫•‬ ‫‪4‬‬
‫‪f(z) = z + 2 (3 − 2ı) z + (8 − 15ı) z + 3 − 11ı‬‬ ‫•‬ ‫‪5‬‬
‫‪f(z) = 4z − 6ı√3z − 3 3 + ı√3 z − 4‬‬ ‫•‬ ‫‪6‬‬

‫ﺍﳊﻞّ‪ .‬ﻹﳚﺎد اﳊﻞ اﳊﻘﻴﻘﻲ‪ ،‬ﻧﺴﺘﻌﻤﻞ اﳋﺎﺻﻴﺔ اﻟﺘﺎﻟﻴﺔ ‪:‬‬


‫إذا ﻛﺎن ‪ z = u + ıv‬ﻋﺪد ًا ﻣﺮﻛﺒ ًﺎ ﺑﺤﻴﺚ ‪ّ u, v ∈ ℝ‬‬
‫ﻓﺈن ‪ z = 0 :‬إذا و ﻓﻘﻂ إذا ﻛﺎن ‪. u = v = 0‬‬
‫ﻟﻴﻜﻦ 𝛼 ﺣﻼ ﺣﻘﻴﻘﻴ ًﺎ ﻟﻠﻤﻌﺎدﻟﺔ ‪ . f (z) = 0 :‬ﻟﺪﻳﻨﺎ ‪:‬‬ ‫•‬ ‫•‬ ‫‪1‬‬
‫‪f (𝛼) = 0 ⟺ 𝛼 + (3 + 5ı) 𝛼 + (6 + 15ı) 𝛼 + 18 = 0‬‬
‫‪ņőƱŀ Ţ‬‬
‫‪⟺ 𝛼 + 3𝛼 + 6𝛼 + 18 + 5𝛼 + 15𝛼 ı = 0‬‬
‫‪𝛼 + 3𝛼 + 6𝛼 + 18 = 0‬‬
‫⟺‬
‫‪5𝛼 + 15𝛼 = 0‬‬

‫‪VI‬‬
‫أن ‪ 𝛼 = 0‬أو ‪ 𝛼 = −3‬و ﻋﻨﺪ اﻟﺘﻌﻮﻳﺾ ﻧﺠﺪ ّ‬
‫أن ‪ 𝛼 = −3‬ﻫﻮ اﻟﻮﺣﻴﺪ اﻟﺬي ُﳛﻘﻖ‬ ‫ﻣﻦ اﳌﻌﺎدﻟﺔ اﻟﺜﺎﻧﻴﺔ ﻧﺠﺪ ّ‬
‫اﳌﻌﺎدﻟﺔ اﻷوﱃ‪.‬‬
‫إذن ‪ ،‬اﳌﻌﺎدﻟﺔ ‪ f (z) = 0‬ﺗﻘﺒﻞ ﺣﻼ ﺣﻘﻴﻘﻴ ًﺎ و ﻫﻮ ‪. 𝛼 = −3‬‬
‫ﻟﺪﻳﻨﺎ ‪:‬‬ ‫•‬
‫‪ƕ‬‬

‫‪f (z) = (z + 3) az + bz + c‬‬


‫‪= az + (b + 3a) z + (c + 3b) z + 3c‬‬
‫‪= z + (3 + 5ı) z + (6 + 15ı) z + 18‬‬
‫‪ũŏ‬‬

‫أن ‪ a = 1‬و ‪ b + 3a = 3 + 5ı‬و ‪ c + 3b = 6 + 15ı‬و ‪ 3c = 18‬أي ‪ a = 1‬و ‪ b = 5ı‬و‬‫ﺑﺎﳌﻄﺎﺑﻘﺔ ﻧﺴﺘﻨﺘﺞ ّ‬


‫‪.c = 6‬‬
‫إذن ‪. f (z) = (z + 3) z + 5ız + 6 :‬‬
‫ﻟﺪﻳﻨﺎ ‪:‬‬ ‫•‬
‫‪f (z) = 0 ⟺ z + 3 = 0‬‬ ‫أو‬ ‫‪z + 5ız + 6 = 0‬‬
‫ﻟﻠﻤﻌﺎدﻟﺔ اﻷوﱃ ّ‬
‫ﺣﻞ وﺣﻴﺪ ﻫﻮ ‪) z = −3‬و ﻫﻮ اﳊﻞ اﳊﻘﻴﻘﻲ اﻟﺬي وﺟﺪﻧﺎه أﻋﻼه(‪.‬‬

‫𝟴𝟴𝟭‬

‫‪http ://tinyurl.com/Malki1718‬‬ ‫‪0‬‬


‫‪ .VI‬ﲤﺎرﻳﻦ ﺗﻄﺒﻴﻘﻴﺔ‬
‫‪.‬‬
‫ُﳑ ﱢﻴﺰ اﳌﻌﺎدﻟﺔ اﻟﺜﺎﻧﻴﺔ ﻫﻮ ‪:‬‬
‫)‪Δ = (5ı) − 4 × 6 = −49 = (7ı‬‬

‫‪Ŕž Ŧ Ľ‬‬
‫‪−5ı + 7ı‬‬ ‫‪−5ı − 7ı‬‬
‫= ‪.z‬‬ ‫= ‪ z‬و ‪=ı‬‬ ‫و ﺑﺎﻟﺘﺎﱄ ﳍﺎ ﺣﻼّن ﳘﺎ ‪= −6ı :‬‬
‫‪2‬‬ ‫‪2‬‬

‫‪œǃ‬‬
‫ﰲ اﻷﺧﲑ‪ ،‬ﳎﻤﻮﻋﺔ ﺣﻠﻮل اﳌﻌﺎدﻟﺔ ‪ f (z) = 0‬ﻫﻲ ‪. 𝒮 = {−3, ı, −6ı} :‬‬

‫ﻟﻴﻜﻦ 𝛼 ﺣﻼ ﺣﻘﻴﻘﻴ ًﺎ ﻟﻠﻤﻌﺎدﻟﺔ ‪ . f (z) = 0 :‬ﻟﺪﻳﻨﺎ ‪:‬‬ ‫•‬ ‫‪2‬‬


‫•‬

‫‪Ŀ‬‬
‫‪f (𝛼) = 0 ⟺ 𝛼 − (11 + 2ı) 𝛼 + 2 (17 + 7ı) 𝛼 − 42 = 0‬‬
‫‪⟺ 𝛼 − 11𝛼 + 34𝛼 − 42 + −2𝛼 + 14𝛼 ı = 0‬‬
‫‪𝛼 + −11𝛼 + 34𝛼 − 42 = 0‬‬
‫⟺‬
‫‪−2𝛼 + 14𝛼 = 0‬‬

‫أن ‪ 𝛼 = 0‬أو ‪ 𝛼 = 7‬و ﻋﻨﺪ اﻟﺘﻌﻮﻳﺾ ﻧﺠﺪ ّ‬


‫أن ‪ 𝛼 = 7‬ﻫﻮ اﻟﻮﺣﻴﺪ اﻟﺬي ُﳛﻘﻖ اﳌﻌﺎدﻟﺔ‬ ‫ﻣﻦ اﳌﻌﺎدﻟﺔ اﻟﺜﺎﻧﻴﺔ ﻧﺠﺪ ّ‬
‫اﻷوﱃ‪.‬‬
‫إذن ‪ ،‬اﳌﻌﺎدﻟﺔ ‪ f (z) = 0‬ﺗﻘﺒﻞ ﺣﻼ ﺣﻘﻴﻘﻴ ًﺎ و ﻫﻮ ‪. 𝛼 = 7‬‬
‫ﻟﺪﻳﻨﺎ ‪:‬‬ ‫•‬

‫‪f (z) = (z − 7) az + bz + c‬‬


‫‪= az + (b − 7a) z + (c − 7b) z − 7c‬‬
‫‪= z − (11 + 2ı) z + 2 (17 + 7ı) z − 42‬‬

‫أن ‪ a = 1‬و ‪ b − 7a = −11 − 2ı‬و ‪ c + 3b = 34 + 14ı‬و ‪ −7c = −42‬أي ‪ a = 1‬و‬ ‫ﺑﺎﳌﻄﺎﺑﻘﺔ ﻧﺴﺘﻨﺘﺞ ّ‬
‫‪ b = −4 − 2ı‬و ‪. c = 6‬‬
‫إذن ‪. f (z) = (z − 7) z − (4 + 2ı) z + 6 :‬‬
‫ﻟﺪﻳﻨﺎ ‪:‬‬ ‫•‬
‫‪f (z) = 0 ⟺ z + 3 = 0‬‬ ‫أو‬ ‫‪z − (4 + 2ı) z + 6 = 0‬‬
‫‪ņőƱŀ Ţ‬‬
‫ﻟﻠﻤﻌﺎدﻟﺔ اﻷوﱃ ّ‬
‫ﺣﻞ وﺣﻴﺪ ﻫﻮ ‪) z = 7‬و ﻫﻮ اﳊﻞ اﳊﻘﻴﻘﻲ اﻟﺬي وﺟﺪﻧﺎه أﻋﻼه(‪.‬‬
‫ُﳑ ﱢﻴﺰ اﳌﻌﺎدﻟﺔ اﻟﺜﺎﻧﻴﺔ ﻫﻮ ‪:‬‬
‫)‪Δ = (− (4 + 2ı)) − 4 × 6 = −12 + 16ı = 4 (−3 + 4ı) = 4 (1 + 2ı) = (2 + 4ı‬‬
‫‪VI‬‬

‫‪4 + 2ı − 2 − 4ı‬‬ ‫‪4 + 2ı + 2 + 4ı‬‬


‫= ‪.z‬‬ ‫= ‪ z‬و ‪= 1−ı‬‬ ‫و ﺑﺎﻟﺘﺎﱄ ﳍﺎ ﺣﻼّن ﳘﺎ ‪= 3 + 3ı :‬‬
‫‪2‬‬ ‫‪2‬‬
‫ﰲ اﻷﺧﲑ‪ ،‬ﳎﻤﻮﻋﺔ ﺣﻠﻮل اﳌﻌﺎدﻟﺔ ‪ f (z) = 0‬ﻫﻲ ‪. 𝒮 = {7, 1 − ı, 3 + 3ı} :‬‬

‫ﻟﻴﻜﻦ 𝛼 ﺣﻼ ﺣﻘﻴﻘﻴ ًﺎ ﻟﻠﻤﻌﺎدﻟﺔ ‪ . f (z) = 0 :‬ﻟﺪﻳﻨﺎ ‪:‬‬ ‫•‬ ‫‪3‬‬


‫•‬
‫‪ƕ‬‬

‫‪f (𝛼) = 0 ⟺ 𝛼 + (−5 + 2ı) 𝛼 + 7 (1 − ı) 𝛼 − 2 + 6ı = 0‬‬


‫‪⟺ 𝛼 − 5𝛼 + 7𝛼 − 2 + 2𝛼 − 7𝛼 + 6 ı = 0‬‬
‫‪𝛼 − 5𝛼 + 7𝛼 − 2 = 0‬‬
‫‪ũŏ‬‬

‫⟺‬
‫‪2𝛼 − 7𝛼 + 6 = 0‬‬

‫‪3‬‬
‫= 𝛼 و ﻋﻨﺪ اﻟﺘﻌﻮﻳﺾ ﻧﺠﺪ ّ‬
‫أن ‪ 𝛼 = 2‬ﻫﻮ اﻟﻮﺣﻴﺪ اﻟﺬي ُﳛﻘﻖ اﳌﻌﺎدﻟﺔ‬ ‫ﻣﻦ اﳌﻌﺎدﻟﺔ اﻟﺜﺎﻧﻴﺔ ﻧﺠﺪ ّ‬
‫أن ‪ 𝛼 = 2‬أو‬
‫‪2‬‬
‫اﻷوﱃ‪.‬‬
‫إذن ‪ ،‬اﳌﻌﺎدﻟﺔ ‪ f (z) = 0‬ﺗﻘﺒﻞ ﺣﻼ ﺣﻘﻴﻘﻴ ًﺎ و ﻫﻮ ‪. 𝛼 = 2‬‬

‫𝟵𝟴𝟭‬

‫‪http ://tinyurl.com/Malki1718‬‬ ‫‪0‬‬


‫‪ .3.VI‬ﺗﻄﺒﻴﻘﺎت اﻷﻋﺪاد اﳌﺮﻛﺒﺔ‬
‫‪.‬‬
‫ﻟﺪﻳﻨﺎ ‪:‬‬ ‫•‬

‫‪Ŕž Ŧ Ľ‬‬
‫‪f (z) = (z − 2) az + bz + c‬‬
‫‪= az + (b − 2a) z + (c − 2b) z − 2c‬‬

‫‪œǃ‬‬
‫‪= z + (−5 + 2ı) z + 7 (1 − ı) z − 2 + 6ı‬‬

‫أن ‪ a = 1‬و ‪ b − 2a = −5 + 2ı‬و ‪ c − 2b = 7 − 7ı‬و ‪ −2c = −2 + 6ı‬أي ‪ a = 1‬و‬‫ﺑﺎﳌﻄﺎﺑﻘﺔ ﻧﺴﺘﻨﺘﺞ ّ‬


‫‪ b = −3 + 2ı‬و ‪. c = 1 − 3ı‬‬

‫‪Ŀ‬‬
‫إذن ‪. f (z) = (z − 2) z + 5 (−3 + 2ı) z + 1 − 3ı :‬‬
‫ﻟﺪﻳﻨﺎ ‪:‬‬ ‫•‬
‫‪f (z) = 0 ⟺ z − 2 = 0‬‬ ‫أو‬ ‫‪z + 5 (−3 + 2ı) z + 1 − 3ı = 0‬‬
‫ﻟﻠﻤﻌﺎدﻟﺔ اﻷوﱃ ّ‬
‫ﺣﻞ وﺣﻴﺪ ﻫﻮ ‪) z = 2‬و ﻫﻮ اﳊﻞ اﳊﻘﻴﻘﻲ اﻟﺬي وﺟﺪﻧﺎه أﻋﻼه(‪.‬‬
‫ُﳑ ﱢﻴﺰ اﳌﻌﺎدﻟﺔ اﻟﺜﺎﻧﻴﺔ ﻫﻮ ‪:‬‬
‫‪Δ = (−3 + 2ı) − 4 (1 − 3ı) = 1‬‬
‫‪3 − 2ı + 1‬‬ ‫‪3 − 2ı − 1‬‬
‫= ‪.z‬‬ ‫= ‪ z‬و ‪=2−ı‬‬ ‫و ﺑﺎﻟﺘﺎﱄ ﳍﺎ ﺣﻼّن ﳘﺎ ‪= 1 − ı :‬‬
‫‪2‬‬ ‫‪2‬‬
‫ﰲ اﻷﺧﲑ‪ ،‬ﳎﻤﻮﻋﺔ ﺣﻠﻮل اﳌﻌﺎدﻟﺔ ‪ f (z) = 0‬ﻫﻲ ‪. 𝒮 = {2, 1 − ı, 2 − ı} :‬‬

‫ﻟﻴﻜﻦ 𝛼 ﺣﻼ ﺣﻘﻴﻘﻴ ًﺎ ﻟﻠﻤﻌﺎدﻟﺔ ‪ . f (z) = 0 :‬ﻟﺪﻳﻨﺎ ‪:‬‬ ‫•‬ ‫•‬ ‫‪4‬‬


‫‪f (𝛼) = 0 ⟺ ı𝛼 + (−1 + 2ı) 𝛼 − (4 + ı) 𝛼 + 3 (−1 + 2ı) = 0‬‬
‫‪⟺ −𝛼 − 4𝛼 − 3 + 𝛼 + 2𝛼 − 𝛼 + 6 ı = 0‬‬
‫‪−𝛼 − 4𝛼 − 3 = 0‬‬
‫⟺‬
‫‪𝛼 + 2𝛼 − 𝛼 + 6 = 0‬‬

‫أن ‪ 𝛼 = −1‬أو ‪ 𝛼 = −3‬و ﻋﻨﺪ اﻟﺘﻌﻮﻳﺾ ﻧﺠﺪ ّ‬


‫أن ‪ 𝛼 = −3‬ﻫﻮ اﻟﻮﺣﻴﺪ اﻟﺬي ُﳛﻘﻖ‬ ‫ﻣﻦ اﳌﻌﺎدﻟﺔ اﻷوﱃ ﻧﺠﺪ ّ‬
‫اﳌﻌﺎدﻟﺔ اﻟﺜﺎﻧﻴﺔ‪.‬‬
‫‪ņőƱŀ Ţ‬‬
‫إذن ‪ ،‬اﳌﻌﺎدﻟﺔ ‪ f (z) = 0‬ﺗﻘﺒﻞ ﺣﻼ ﺣﻘﻴﻘﻴ ًﺎ و ﻫﻮ ‪. 𝛼 = −3‬‬
‫ﻟﺪﻳﻨﺎ ‪:‬‬ ‫•‬

‫‪f (z) = (z + 3) az + bz + c‬‬

‫‪VI‬‬
‫‪= az + (b + 3a) z + (c + 3b) z + 3c‬‬
‫)‪= ız + (−1 + 2ı) z − (4 + ı) z + 3 (−1 + 2ı‬‬

‫أن ‪ a = ı‬و ‪ b + 3a = −1 + 2ı‬و ‪ c + 3b = −4 − ı‬و ‪ 3c = −3 + 6ı‬أي ‪ a = ı‬و‬ ‫ﺑﺎﳌﻄﺎﺑﻘﺔ ﻧﺴﺘﻨﺘﺞ ّ‬


‫‪ƕ‬‬

‫‪ b = −1 − ı‬و ‪. c = −1 + 2ı‬‬
‫إذن ‪. f (z) = (z + 3) z − (1 + ı) z − 1 + 2ı :‬‬
‫ﻟﺪﻳﻨﺎ ‪:‬‬ ‫•‬
‫‪f (z) = 0 ⟺ z + 3 = 0‬‬ ‫أو‬ ‫‪z − (1 + ı) z − 1 + 2ı = 0‬‬
‫‪ũŏ‬‬

‫ﻟﻠﻤﻌﺎدﻟﺔ اﻷوﱃ ّ‬
‫ﺣﻞ وﺣﻴﺪ ﻫﻮ ‪) z = −3‬و ﻫﻮ اﳊﻞ اﳊﻘﻴﻘﻲ اﻟﺬي وﺟﺪﻧﺎه أﻋﻼه(‪.‬‬
‫ُﳑ ﱢﻴﺰ اﳌﻌﺎدﻟﺔ اﻟﺜﺎﻧﻴﺔ ﻫﻮ ‪:‬‬
‫)‪Δ = (−1 − ı) − 4ı (−1 + 2ı) = 8 + 6ı = (3 + ı‬‬
‫‪1+ı+3+ı‬‬ ‫‪1+ı−3−ı‬‬ ‫‪1‬‬
‫= ‪.z‬‬ ‫= ‪ z‬و ‪= 1 − 2ı‬‬ ‫و ﺑﺎﻟﺘﺎﱄ ﳍﺎ ﺣﻼّن ﳘﺎ ‪= − = ı :‬‬
‫‪2ı‬‬ ‫‪2ı‬‬ ‫‪ı‬‬
‫ﰲ اﻷﺧﲑ‪ ،‬ﳎﻤﻮﻋﺔ ﺣﻠﻮل اﳌﻌﺎدﻟﺔ ‪ f (z) = 0‬ﻫﻲ ‪. 𝒮 = {−3, ı, 1 − 2ı} :‬‬

‫𝟬𝟵𝟭‬

‫‪http ://tinyurl.com/Malki1718‬‬ ‫‪0‬‬


‫‪ .VI‬ﲤﺎرﻳﻦ ﺗﻄﺒﻴﻘﻴﺔ‬
‫‪.‬‬
‫ﻟﻴﻜﻦ 𝛼 ﺣﻼ ﺣﻘﻴﻘﻴ ًﺎ ﻟﻠﻤﻌﺎدﻟﺔ ‪ . f (z) = 0 :‬ﻟﺪﻳﻨﺎ ‪:‬‬ ‫•‬ ‫‪5‬‬
‫•‬

‫‪f (𝛼) = 0 ⟺ 𝛼 + 2 (3 − 2ı) 𝛼 + (8 − 15ı) 𝛼 + 3 − 11ı = 0‬‬

‫‪Ŕž Ŧ Ľ‬‬
‫‪⟺ 𝛼 + 6𝛼 + 8𝛼 + 3 + −4𝛼 − 15𝛼 − 11 ı = 0‬‬
‫‪𝛼 + 6𝛼 + 8𝛼 + 3 = 0‬‬

‫‪œǃ‬‬
‫⟺‬
‫‪−4𝛼 − 15𝛼 − 11 = 0‬‬
‫‪11‬‬
‫‪ 𝛼 = −‬و ﻋﻨﺪ اﻟﺘﻌﻮﻳﺾ ﻧﺠﺪ ّ‬
‫أن ‪ 𝛼 = −1‬ﻫﻮ اﻟﻮﺣﻴﺪ اﻟﺬي ُﳛﻘﻖ‬ ‫ﻣﻦ اﳌﻌﺎدﻟﺔ اﻟﺜﺎﻧﻴﺔ ﻧﺠﺪ ّ‬
‫أن ‪ 𝛼 = −1‬أو‬
‫‪4‬‬

‫‪Ŀ‬‬
‫اﳌﻌﺎدﻟﺔ اﻷوﱃ‪.‬‬
‫إذن ‪ ،‬اﳌﻌﺎدﻟﺔ ‪ f (z) = 0‬ﺗﻘﺒﻞ ﺣﻼ ﺣﻘﻴﻘﻴ ًﺎ و ﻫﻮ ‪. 𝛼 = −1‬‬
‫ﻟﺪﻳﻨﺎ ‪:‬‬ ‫•‬

‫‪f (z) = (z + 1) az + bz + c‬‬


‫‪= az + (b + a) z + (c + b) z + c‬‬
‫‪= z + 2 (3 − 2ı) z + (8 − 15ı) z + 3 − 11ı‬‬
‫ﺑﺎﳌﻄﺎﺑﻘﺔ ﻧﺴﺘﻨﺘﺞ ّ‬
‫أن ‪ a = 1‬و ‪ b + a = 6 − 4ı‬و ‪ c + b = 8 − 15ı‬و ‪ c = 3 − 11ı‬أي ‪ a = 1‬و ‪b = 5 − 4ı‬‬
‫و ‪. c = 3 − 11ı‬‬
‫إذن ‪. f (z) = (z + 1) z + (5 − 4ı) z + 3 − 11ı :‬‬
‫ﻟﺪﻳﻨﺎ ‪:‬‬ ‫•‬
‫‪f (z) = 0 ⟺ z + 1 = 0‬‬ ‫أو‬ ‫‪z + (5 − 4ı) z + 3 − 11ı = 0‬‬
‫ﺣﻞ وﺣﻴﺪ ﻫﻮ ‪) z = −1‬و ﻫﻮ اﳊﻞ اﳊﻘﻴﻘﻲ اﻟﺬي وﺟﺪﻧﺎه أﻋﻼه(‪.‬‬‫ﻟﻠﻤﻌﺎدﻟﺔ اﻷوﱃ ّ‬
‫ُﳑ ﱢﻴﺰ اﳌﻌﺎدﻟﺔ اﻟﺜﺎﻧﻴﺔ ﻫﻮ ‪:‬‬
‫)‪Δ = (5 − 4ı) − 4 (3 − 11ı) = −3 + 4ı = (1 + 2ı‬‬
‫و ﺑﺎﻟﺘﺎﱄ ﳍﺎ ﺣﻼّن ﳘﺎ ‪:‬‬
‫‪−5 + 4ı + 1 + 2ı‬‬ ‫‪−5 + 4ı − 1 − 2ı‬‬
‫= ‪.z‬‬ ‫و ‪= −2 + 3ı‬‬ ‫= ‪z‬‬ ‫‪= −3 + ı‬‬
‫‪2‬‬ ‫‪2‬‬
‫}‪. 𝒮 = {−1, −3 + ı, −2 + 3ı‬‬ ‫ﰲ اﻷﺧﲑ‪ ،‬ﳎﻤﻮﻋﺔ ﺣﻠﻮل اﳌﻌﺎدﻟﺔ ‪ f (z) = 0‬ﻫﻲ ‪:‬‬
‫‪ņőƱŀ Ţ‬‬
‫ﻟﻴﻜﻦ 𝛼 ﺣﻼ ﺣﻘﻴﻘﻴ ًﺎ ﻟﻠﻤﻌﺎدﻟﺔ ‪ . f (z) = 0 :‬ﻟﺪﻳﻨﺎ ‪:‬‬ ‫•‬ ‫‪6‬‬
‫•‬

‫‪f (𝛼) = 0 ⟺ 4𝛼 − 6ı√3𝛼 − 3 3 + ı√3 𝛼 − 4 = 0‬‬


‫‪VI‬‬

‫‪⟺ 4𝛼 − 9𝛼 − 4 + −6√3𝛼 − 3√3𝛼 ı = 0‬‬


‫‪4𝛼 − 9𝛼 − 4 = 0‬‬
‫⟺‬
‫‪−6√3𝛼 − 3√3𝛼 = 0‬‬
‫‪1‬‬
‫أن ‪ 𝛼 = 0‬أو ‪ 𝛼 = − 1‬و ﻋﻨﺪ اﻟﺘﻌﻮﻳﺾ ﻧﺠﺪ ّ‬
‫‪ƕ‬‬

‫أن ‪ 𝛼 = −‬ﻫﻮ اﻟﻮﺣﻴﺪ اﻟﺬي ُﳛﻘﻖ‬ ‫ﻣﻦ اﳌﻌﺎدﻟﺔ اﻟﺜﺎﻧﻴﺔ ﻧﺠﺪ ّ‬


‫‪2‬‬ ‫‪2‬‬
‫اﳌﻌﺎدﻟﺔ اﻷوﱃ‪.‬‬
‫‪1‬‬
‫إذن ‪ ،‬اﳌﻌﺎدﻟﺔ ‪ f (z) = 0‬ﺗﻘﺒﻞ ﺣﻼ ﺣﻘﻴﻘﻴ ًﺎ و ﻫﻮ ‪. 𝛼 = −‬‬
‫‪2‬‬
‫‪ũŏ‬‬

‫ﻟﺪﻳﻨﺎ ‪:‬‬ ‫•‬

‫‪1‬‬
‫= )‪f (z‬‬ ‫‪z+‬‬ ‫‪az + bz + c‬‬
‫‪2‬‬
‫‪a‬‬ ‫‪b‬‬ ‫‪c‬‬
‫‪= az + b +‬‬ ‫‪z + c+‬‬ ‫‪z+‬‬
‫‪2‬‬ ‫‪2‬‬ ‫‪2‬‬
‫‪= 4z − 6ı√3z − 3 3 + ı√3 z − 4‬‬

‫𝟭𝟵𝟭‬

‫‪http ://tinyurl.com/Malki1718‬‬ ‫‪0‬‬


‫‪ .3.VI‬ﺗﻄﺒﻴﻘﺎت اﻷﻋﺪاد اﳌﺮﻛﺒﺔ‬
‫‪.‬‬
‫‪c‬‬ ‫‪b‬‬ ‫‪a‬‬
‫أي ‪ a = 4‬و‬ ‫‪ c +‬و ‪= −4‬‬ ‫‪ b +‬و ‪= −9 − 3ı√3‬‬ ‫ﺑﺎﳌﻄﺎﺑﻘﺔ ﻧﺴﺘﻨﺘﺞ ّ‬
‫أن ‪ a = 4‬و ‪= −6ı√3‬‬
‫‪2‬‬ ‫‪2‬‬ ‫‪2‬‬
‫‪ b = −2 − 6ı√3‬و ‪. c = −8‬‬

‫‪Ŕž Ŧ Ľ‬‬
‫‪1‬‬
‫‪. f (z) = z +‬‬ ‫إذن ‪4z − 2 + 6ı√3 z − 8 :‬‬
‫‪2‬‬

‫‪œǃ‬‬
‫ﻟﺪﻳﻨﺎ ‪:‬‬ ‫•‬
‫‪1‬‬
‫‪f (z) = 0 ⟺ z + = 0‬‬ ‫أو‬ ‫‪4z − 2 + 6ı√3 z − 8 = 0‬‬
‫‪2‬‬
‫‪1‬‬

‫‪Ŀ‬‬
‫ﻟﻠﻤﻌﺎدﻟﺔ اﻷوﱃ ّ‬
‫ﺣﻞ وﺣﻴﺪ ﻫﻮ ‪) z = −‬و ﻫﻮ اﳊﻞ اﳊﻘﻴﻘﻲ اﻟﺬي وﺟﺪﻧﺎه أﻋﻼه(‪.‬‬
‫‪2‬‬
‫ُﳑ ﱢﻴﺰ اﳌﻌﺎدﻟﺔ اﻟﺜﺎﻧﻴﺔ ﻫﻮ ‪:‬‬
‫‪Δ = −2 − 6ı√3‬‬ ‫‪− 4 × 4 (−8) = 24 + 24√3 = 6 + 2ı√3‬‬

‫و ﺑﺎﻟﺘﺎﱄ ﳍﺎ ﺣﻼّن ﳘﺎ ‪:‬‬


‫‪2 + 6ı√3 + 6 + 2ı√3‬‬ ‫‪2 + 6ı√3 − 6 − 2ı√3‬‬ ‫‪1 √3‬‬
‫= ‪.z‬‬ ‫و ‪= 1 + ı√3‬‬ ‫= ‪z‬‬ ‫‪=− +‬‬ ‫‪ı‬‬
‫‪2×4‬‬ ‫‪2×4‬‬ ‫‪2‬‬ ‫‪2‬‬
‫‪1‬‬ ‫‪1‬‬ ‫‪3‬‬
‫‪. 𝒮 = − , 1 + ı√3, − + √ ı‬‬ ‫ﰲ اﻷﺧﲑ‪ ،‬ﳎﻤﻮﻋﺔ ﺣﻠﻮل اﳌﻌﺎدﻟﺔ ‪ f (z) = 0‬ﻫﻲ ‪:‬‬
‫‪2‬‬ ‫‪2‬‬ ‫‪2‬‬

‫■‬

‫أﻛﺘﺐ ﻛﻼ ﻣﻦ اﻟﻌﺒﺎرات اﻵﺗﻴﺔ ﻋﲆ ﺷﻜﻞ ﻋﺒﺎرة ﺧﻄﻴﺔ ‪:‬‬


‫‬ ‫‪.‬‬
‫𝟓𝟐𝟏‬ ‫‪.‬‬

‫‪cos x ⋅ sin x + sin x ⋅ cos x‬‬ ‫‪3‬‬


‫•‬ ‫‪sin x‬‬ ‫‪ cos x •1‬و‬
‫‪16 cos x sin x + 8 cos x sin x‬‬ ‫‪4‬‬
‫•‬ ‫‪sin x‬‬ ‫‪ cos x •2‬و‬
‫‪ņőƱŀ Ţ‬‬
‫ﺍﳊﻞّ‪ .‬ﻧﻀﻊ ‪ z = e‬و ﻧُﻄ ّﺒﻖ ِﺻ َﻴﻎ ﺃﻭﻟﺮ ‪.‬‬
‫)ا( ﻟﺪﻳﻨﺎ ‪:‬‬ ‫•‬ ‫‪1‬‬

‫‪VI‬‬
‫‪1‬‬ ‫‪1‬‬
‫= ‪cos x‬‬ ‫‪z+‬‬
‫‪2‬‬ ‫‪z‬‬
‫‪1‬‬ ‫‪1‬‬ ‫‪1‬‬ ‫‪1‬‬ ‫‪1‬‬ ‫‪1‬‬
‫=‬ ‫× ‪z + 5z × + 10z‬‬ ‫× ‪+ 10z‬‬ ‫× ‪+ 5z‬‬ ‫‪+‬‬
‫‪32‬‬ ‫‪z‬‬ ‫‪z‬‬ ‫‪z‬‬ ‫‪z‬‬ ‫‪z‬‬
‫‪1‬‬ ‫‪1‬‬ ‫‪1‬‬ ‫‪1‬‬
‫‪ƕ‬‬

‫=‬ ‫‪z +‬‬ ‫‪+5 z +‬‬ ‫‪+ 10 z +‬‬


‫‪32‬‬ ‫‪z‬‬ ‫‪z‬‬ ‫‪z‬‬
‫‪1‬‬
‫=‬ ‫]‪[2 cos 5x + 5 × 2 cos 3x + 10 × 2 cos x‬‬
‫‪32‬‬
‫‪ũŏ‬‬

‫‪1‬‬ ‫‪5‬‬ ‫‪5‬‬


‫= ‪cos x‬‬ ‫‪cos 5x +‬‬ ‫‪cos 3x + cos x‬‬ ‫ﻣﻨﻪ ‪:‬‬
‫‪16‬‬ ‫‪16‬‬ ‫‪8‬‬

‫𝟮𝟵𝟭‬

‫‪http ://tinyurl.com/Malki1718‬‬ ‫‪0‬‬


‫ ﲤﺎرﻳﻦ ﺗﻄﺒﻴﻘﻴﺔ‬.VI
.
: ‫)ب( ﺑﺎﳌﺜﻞ‬

Ŕž Ŧ Ľ
1 1
sin x = z−
2ı z
1 1 1 1 1 1

œǃ
= z − 5z × + 10z × − 10z × + 5z × −
32ı z z z z z
1 1 1 1
= z − −5 z − + 10 z −
32ı z z z

Ŀ
1
= [2ı sin 5x − 5 × 2ı sin 3x + 10 × 2ı cos x]
32ı
1 5 5
sin x = sin 5x − sin 3x + sin x : ‫ﻣﻨﻪ‬
16 16 8

: ‫)ا( ﻟﺪﻳﻨﺎ‬ 2

1 1
cos x = z+
2 z
1 1 1 1 1 1 1
= z + 6z × + 15z × + 20z × + 15z × + 6z × +
64 z z z z z z
1 1 1 1
= z + +6 z + + 15 z + + 20
64 z z z
1
= [2 cos 6x + 6 × 2 cos 4x + 15 × 2 cos 2x + 20]
64
1 3 15 5
cos x = cos 6x + cos 4x + cos 2x + : ‫ﻣﻨﻪ‬
32 16 32 16

: ‫)ب( ﺑﺎﳌﺜﻞ‬
1 1
ņőƱŀ Ţ
sin x = z−
2ı z
1 1 1 1 1 1 1
=− z − 6z × + 15z × − 20z × + 15z × − 6z × +
64 z z z z z z
1 1 1 1
VI

=− z + −6 z + + 15 z + − 20
64 z z z
1
= − [2 cos 6x − 6 × 2 cos 4x + 15 × 2 cos 2x − 20]
64
1 3 15 5
sin x = − cos 6x + cos 4x − cos 2x + : ‫ﻣﻨﻪ‬
32 16 32 16
ƕ

: ‫ﻟﺪﻳﻨﺎ ﻣﻦ ﺟﻬﺔ‬ • 3

1 1 1 1
ũŏ

cos x ⋅ sin x = z+ ⋅ z−
8 z 16 z
1 1 3 1 4
= z + + 3z + z + − 4z − +6
128 z z z z
1 1 1 1 1
= z + − z + −3 z + +3 z+
128 z z z z
1
= [cos 7x − cos 5x − 3 cos 3x + 3 cos x]
64

𝟭𝟵𝟯

http ://tinyurl.com/Malki1718 0
‫ ﺗﻄﺒﻴﻘﺎت اﻷﻋﺪاد اﳌﺮﻛﺒﺔ‬.3.VI
.
: ‫و ﻣﻦ ﺟﻬﺔ أﺧﺮى‬ •

Ŕž Ŧ Ľ
1 1 1 1
sin x ⋅ cos x = − z− ⋅ z+
8ı z 16 z

œǃ
1 1 3 1 4
=− z − + 3z − z + + 4z + +6
128ı z z z z
1 1 1 1 1
=− z − + z − −3 z − −3 z−
128ı z z z z

Ŀ
1
= − [sin 7x + sin 5x − 3 sin 3x − 3 sin x]
64

: ‫و ﺑﺎﻟﺘﺎﱄ‬
cos x ⋅ sin x + sin x ⋅ cos x = [cos 7x − sin 7x − cos 5x − sin 5x + 3 cos 3x − 3 sin 3x − 3 cos x − 3 sin x]

ّ ‫وﺟﺪﻧﺎ ﰲ اﻟﺴﺆال اﻟﺴﺎﺑﻖ‬


: ‫أن‬ • • 4
1
sin x ⋅ cos x = − [sin 7x + sin 5x − 3 sin 3x − 3 sin x]
64
1
16 cos x ⋅ sin x = − [sin 7x + sin 5x − 3 sin 3x − 3 sin x] : ‫ﻣﻨﻪ‬
4

: ‫و ﻣﻦ ﺟﻬﺔ أﺧﺮى‬ •

1 1 1 1
cos x ⋅ sin x = z+ ⋅ − z−
8 z 4 z
1 1 3 1
=− z + + 3z + z + −2
32 z z z
1 1 1 1
ņőƱŀ Ţ
=− z + + z + −2 z+
32 z z z
1
= − [2 cos 5x + 2 cos 3x − 4 cos x]
32
1

VI
= − [cos 5x + cos 3x − 2 cos x]
16
1
8 cos x ⋅ sin x = − [cos 5x + cos 3x − 2 cos x] : ‫ﻣﻨﻪ‬
2

: ‫ﰲ اﻷﺧﲑ‬
ƕ

16 cos x ⋅ sin x + 8 cos x ⋅ sin x = − [sin 7x + sin 5x + 2 cos 5x − 3 sin 3x + 2 cos 3x − 3 sin x − 4 cos x]


ũŏ

ّ ‫ﺛﻢ اﺳﺘﻨﺘﺞ‬
‫أن‬ ّ ‫ أﺛﺒﺖ‬n = 4 ‫ﺑﺎﺳﺘﻌﲈل دﺳﺘﻮر ﻣﻮﺍﻓﺮ ﻣﻊ اﻟﻘﻴﻤﺔ‬
ّ cos 4𝜃 = 8 cos 𝜃 − 8 cos 𝜃 + 1 ‫أن‬
 .
𝟏𝟐𝟔 .

𝜋 2 + √2
. cos =
8 2

𝟭𝟵𝟰

http ://tinyurl.com/Malki1718 0
‫ ﲤﺎرﻳﻦ ﺗﻄﺒﻴﻘﻴﺔ‬.VI
.
: ‫ ﺣﺴﺐ ﻣﻔﻜﻮك ﺛﻨﺎﺋﻲ اﳊﺪ‬،‫ و ﻣﻦ ﺟﻬﺔ أﺧﺮى‬، e = cos 4𝜃 + ı sin 4𝜃 ‫ ﺣﺴﺐ دﺳﺘﻮر ﻣﻮﺍﻓﺮ‬،‫ﻟﺪﻳﻨﺎ ﻣﻦ ﺟﻬﺔ‬ .ّ‫ﺍﳊﻞ‬

Ŕž Ŧ Ľ
e = (cos 𝜃 + ı sin 𝜃)
= cos 𝜃 + 4ı cos 𝜃 sin 𝜃 − 6 cos 𝜃 sin 𝜃 − 4ı cos 𝜃 sin 𝜃 + sin 𝜃

œǃ
: ‫ﻣﻨﻪ‬
cos 4𝜃 = Re e

Ŀ
= Re cos 𝜃 + 4ı cos 𝜃 sin 𝜃 − 6 cos 𝜃 sin 𝜃 − 4ı cos 𝜃 sin 𝜃 + sin 𝜃

= cos 𝜃 − 6 cos 𝜃 sin 𝜃 + sin 𝜃


= cos 𝜃 − 6 cos 1 − cos 𝜃 + 1 − cos 𝜃
= 8 cos 𝜃 − 8 cos 𝜃 + 1
𝜋 𝜋
: ‫( ﻧﺠﺪ‬c = cos ‫ و ﺑﺎﻟﺘﻌﻮﻳﺾ ﻋﻦ 𝜃 ﺑﺎﻟﻘﻴﻤﺔ )و ﺑﻮﺿﻊ‬،‫اﻵن‬
8 8
𝜋 4𝜋
0 = cos = cos = 8c − 8c + 1
2 8
4 ± √16 − 8
c =x= ‫ ﻣﻨﻪ‬8x −8x+1 = 0 ‫ ﻓﻴﻜﻮن‬x = c ‫ ﻧﻀﻊ‬،‫ ﳊﻠﻬﺎ‬. c ‫و ﻫﻲ ﻣﻌﺎدﻟﺔ ﻣﻀﺎﻋﻔﺔ اﻟﱰﺑﻴﻊ ﰲ اﳌﺠﻬﻮل‬
8
𝜋 𝜋 𝜋 2 + √2 2 + √2 2 + √2
‫ أي‬c > 0 ‫ ﻣﻨﻪ‬0 < < ‫ ﻟﻜﻦ‬. c = cos = ± = ± ‫ و ﺑﺎﻟﺘﺎﱄ‬c = ‫أي‬
8 2 8 4 2 4
𝜋 2 + √2
■ . cos =
8 2

. cos
𝜋
‫ إﺳﺘﻨﺘﺞ ﻗﻴﻤﺔ‬. cos 𝜃 ‫ ﻋﲆ ﺷﻜﻞ ﻛﺜﲑ ﺣﺪود ﰲ‬cos 5𝜃 ‫أﻛﺘﺐ‬
# .
𝟏𝟐𝟕 .
10
ņőƱŀ Ţ
: ( 47 ‫ ﻧﺘّﺒﻊ اﻟﻄﺮﻳﻘﺔ اﳌﻌﺮوﺿﺔ ﰲ اﻟﺪرس )ﺻﻔﺤﺔ‬.ّ‫ﺍﳊﻞ‬
cos 5𝜃 + ı sin 5𝜃 = (cos 𝜃 + ı sin 𝜃)
VI

= cos 𝜃 + 5 (cos 𝜃) (ı sin 𝜃) + 10 (cos 𝜃) (ı sin 𝜃)


+10 (cos 𝜃) (ı sin 𝜃) + 5 (cos 𝜃) (ı sin 𝜃) + (ı sin 𝜃)
= cos 𝜃 − 10 cos 𝜃 ⋅ sin 𝜃 + 5 cos 𝜃 ⋅ sin 𝜃
+ı 5 cos 𝜃 ⋅ sin 𝜃 − 10 cos 𝜃 ⋅ sin 𝜃 + sin 𝜃
ƕ

: ‫إذن‬
cos 5𝜃 = cos 𝜃 − 10 cos 𝜃 ⋅ sin 𝜃 + 5 cos 𝜃 ⋅ sin 𝜃
ũŏ

= cos 𝜃 − 10 cos 𝜃 1 − cos 𝜃 + 5 cos 𝜃 1 − cos 𝜃


= 16 cos 𝜃 − 20 cos 𝜃 + 5 cos 𝜃

𝜋 𝜋
: ‫ ﻓﻴﻜﻮن‬x = cos ‫= 𝜃 و ﻧﻀﻊ‬ ‫ﻧﺄﺧﺬ‬
10 10
𝜋 𝜋
0 = cos = cos 5 = 16x − 20x + 5x = x 16x − 20x + 5
2 10

𝟭𝟵𝟱

http ://tinyurl.com/Malki1718 0
‫ ﺗﻄﺒﻴﻘﺎت اﻷﻋﺪاد اﳌﺮﻛﺒﺔ‬.3.VI
.
ّ ‫ و ﺑﲈ‬. ± 5 − √5 ≈ ±0.59 ‫ و‬± 5 + √5 ≈ ±0.95 ، 0 ‫ ﻫﻲ‬16x − 20x + 5x = 0 ‫ﺣﻠﻮل اﳌﻌﺎدﻟﺔ‬
‫أن‬
8 8

Ŕž Ŧ Ľ
5 + √5 2 𝜋
: ‫ ﻫﻮ اﳊﻞ اﻟﻮﺣﻴﺪ اﻟﺬي ُﳛﻘﻖ ﻫﺬه اﻟﴩوط إذن‬x = ّ 0< 𝜋 <𝜋
‫ و‬0.71 ≈ √ < cos < 1 ‫ﻓﺈن‬
8 2 10 10 4

œǃ
𝜋 10 + 2√5 𝜋 5 + √5
■ . cos = ‫أي‬ cos =
10 4 10 8

Ŀ
.
𝟏𝟐𝟖 .
n𝜋 tan 𝜃 − 10 tan 𝜃 + 5 tan 𝜃
tan ‫ﺛﻢ اﺳﺘﻨﺘﺞ ِﻗ َﻴﻢ‬
ّ tan 5𝜃 = ّ ‫ أﺛﺒﺖ‬،‫ﺑﺎﺳﺘﻌﲈل دﺳﺘﻮر ﻣﻮﺍﻓﺮ‬
: ‫أن‬
10 5 tan 𝜃 − 10 tan 𝜃 + 1
. n ∈ {1, 2, 3, 4} ‫ﻷﺟﻞ‬

: ‫ ﺣﺴﺐ ﻣﻔﻜﻮك ﺛﻨﺎﺋﻲ اﳊﺪ‬،‫ و ﻣﻦ ﺟﻬﺔ أﺧﺮى‬، e = cos 5𝜃 + ı sin 5𝜃 ‫ ﺣﺴﺐ دﺳﺘﻮر ﻣﻮﺍﻓﺮ‬،‫ﻟﺪﻳﻨﺎ ﻣﻦ ﺟﻬﺔ‬ .ّ‫ﺍﳊﻞ‬

e = (cos 𝜃 + ı sin 𝜃)
= cos 𝜃 + 5ı cos 𝜃 sin 𝜃 − 10 cos 𝜃 sin 𝜃 − 10ı cos 𝜃 sin 𝜃 + 5 cos 𝜃 sin 𝜃 + ı sin 𝜃
: ‫ﻣﻨﻪ‬
cos 5𝜃 = Re e
= cos 𝜃 − 10 cos 𝜃 sin 𝜃 + 5 cos 𝜃 sin 𝜃
= cos 𝜃 − 10 cos 𝜃 1 − cos 𝜃 + 5 cos 𝜃 1 − cos 𝜃
= 16 cos 𝜃 − 20 cos 𝜃 + 5 cos 𝜃
sin 5𝜃 = Im e
= 5 cos 𝜃 sin 𝜃 − 10 cos 𝜃 sin 𝜃 + sin 𝜃

= 5 1 − sin 𝜃 sin 𝜃 − 10 1 − sin 𝜃 sin 𝜃 + sin 𝜃


ņőƱŀ Ţ
= 16 sin 𝜃 − 20 sin 𝜃 + 5 sin 𝜃

sin 5𝜃
tan 5𝜃 = : ‫و ﺑﺎﻟﺘﺎﱄ‬
cos 5𝜃

VI
16 sin 𝜃 − 20 sin 𝜃 + 5 sin 𝜃
=
16 cos 𝜃 − 20 cos 𝜃 + 5 cos 𝜃
16 tan 𝜃 − 20 tan 𝜃 cos− 𝜃 + 5 tan 𝜃 cos− 𝜃
=
16 − 20 cos− 𝜃 + 5 cos− 𝜃
ƕ

(cos 𝜃 ‫)ﺑﻘﺴﻤﺔ اﻟﺒﺴﻂ و اﳌﻘﺎم ﻋﲆ‬


16 tan 𝜃 − 20 tan 𝜃 1 + tan 𝜃 + 5 tan 𝜃 1 + tan 𝜃
=
16 − 20 1 + tan 𝜃 + 5 1 + tan 𝜃
ũŏ

1
( ّ
= 1 + tan x ‫)ﻷن‬
cos x
tan 𝜃 − 10 tan 𝜃 + 5 tan 𝜃
=
5 tan 𝜃 − 10 tan 𝜃 + 1
3𝜋 3𝜋 𝜋
ّ ‫ و ﺑﺎﻟﺘﺎﱄ‬tan 5𝜃 = tan
‫ﻓﺈن ﻣﻘﺎم‬ = −∞ ‫= 𝜃 ﻳﻜﻮن‬ ‫ و ﻋﻨﺪﻣﺎ‬tan 5𝜃 = tan = +∞ ‫ﻓﺈن‬ّ 𝜃 = 𝜋 ‫ﻋﻨﺪﻣﺎ‬ •
2 10 2 10
‫ﻫﺬه‬ ّ ‫اﻟﻜﴪ اﻟﺴﺎﺑﻖ ﳚﺐ أن ﻳﻜﻮن ﻣﻌﺪوﻣ ًﺎ‬
. t = tan 𝜃 ‫ ﺣﻴﺚ‬5t − 10t + 1 = 0 ‫)ﻷن ﻗﻴﻤﺔ اﻟﺒﺴﻂ ﻣﻨﺘﻬﻴﺔ( أي‬

𝟭𝟵𝟲

http ://tinyurl.com/Malki1718 0
‫‪ .VI‬ﲤﺎرﻳﻦ ﺗﻄﺒﻴﻘﻴﺔ‬
‫‪.‬‬
‫√‬ ‫‪5±2 5‬‬ ‫√‬ ‫‪5±‬‬ ‫‪25 − 5‬‬
‫‪ t = ±‬ﻟﻜﻦ و ﺑﲈ ّ‬
‫أن ﻫﺎﺗﲔ اﻟﺰاوﻳﺘﲔ‬ ‫= ‪ t‬ﻣﻨﻪ‬ ‫اﳌﻌﺎدﻟﺔ ﻣﻀﺎﻋﻔﺔ اﻟﱰﺑﻴﻊ‪ ،‬ﺣﻠﻮﳍﺎ ﻫﻲ‬
‫‪5‬‬ ‫‪5‬‬

‫‪Ŕž Ŧ Ľ‬‬
‫ﻓﺈن اﳉﻴﻮب )‪ (sin‬و ﺟﻴﻮب اﻟﺘﲈم )‪ (cos‬ﻣﻮﺟﺒﺔ ﻣﻨﻪ ‪ t = tan 𝜃 ≥ 0‬و ﺑﲈ ّ‬
‫أن داﻟﺔ‬ ‫ﺗﻘﻌﺎن ﰲ اﻟﺮﺑﻊ اﻷول ﻣﻦ اﳌﺴﺘﻮي ّ‬
‫𝜋‪3‬‬ ‫𝜋‬
‫‪ tan‬ﻣﻨﻪ ‪:‬‬ ‫‪> tan‬‬ ‫اﻟﻈﻞ )‪ (tan‬ﻣﺘﺰاﻳﺪة ّ‬
‫ﻓﺈن‬
‫‪10‬‬ ‫‪10‬‬

‫‪œǃ‬‬
‫𝜋‬ ‫‪5 − 2√5‬‬ ‫‪25 − 10√5‬‬ ‫𝜋‪3‬‬ ‫‪5 + 2√5‬‬ ‫‪25 + 10√5‬‬
‫‪tan‬‬ ‫=‬ ‫=‬ ‫و‬ ‫‪tan‬‬ ‫=‬ ‫=‬
‫‪10‬‬ ‫‪5‬‬ ‫‪5‬‬ ‫‪10‬‬ ‫‪5‬‬ ‫‪5‬‬

‫ﻋﻨﺪﻣﺎ 𝜋‪ 𝜃 = 2‬أو 𝜋‪ّ 𝜃 = 4‬‬

‫‪Ŀ‬‬
‫ﻓﺈن ﺑﺴﻂ اﻟﻜﴪ اﻟﺴﺎﺑﻖ ﳚﺐ أن ﻳﻜﻮن ﻣﻌﺪوﻣ ًﺎ ّ‬
‫)ﻷن اﳌﻘﺎم ﻏﲑ‬ ‫ﻓﺈن ‪ tan 5𝜃 = 0‬و ﺑﺎﻟﺘﺎﱄ ّ‬ ‫•‬
‫‪10‬‬ ‫‪10‬‬
‫ﻣﻌﺪوم( أي ‪ t − 10t + 5t = 0‬ﺣﻴﺚ 𝜃 ‪. t = tan‬‬

‫‪t − 10t + 5t = 0 ⟺ t t − 10t + 5 = 0‬‬


‫‪⟺ t=0‬‬ ‫أو‬ ‫‪t − 10t + 5 = 0‬‬
‫‪⟺ t=0‬‬ ‫أو‬ ‫‪t = 5 ± √25 − 5‬‬

‫‪⟺ t=0‬‬ ‫أو‬ ‫‪t=±‬‬ ‫‪5 ± 2√5‬‬

‫أن ﻫﺎﺗﲔ اﻟﺰاوﻳﺘﲔ ﺗﻘﻌﺎن ﰲ اﻟﺮﺑﻊ اﻷول ﻣﻦ اﳌﺴﺘﻮي ّ‬


‫ﻓﺈن اﳉﻴﻮب )‪ (sin‬و ﺟﻴﻮب اﻟﺘﲈم )‪ (cos‬ﻣﻮﺟﺒﺔ ﻣﻨﻪ‬ ‫ﻟﻜﻦ و ﺑﲈ ّ‬
‫𝜋‪4‬‬ ‫𝜋‪2‬‬
‫‪ tan‬ﻣﻨﻪ‬ ‫‪> tan‬‬ ‫أن داﻟﺔ اﻟﻈﻞ )‪ (tan‬ﻣﺘﺰاﻳﺪة ّ‬
‫ﻓﺈن‬ ‫‪ t = tan 𝜃 ≥ 0‬و ﺑﲈ ّ‬
‫‪10‬‬ ‫‪10‬‬
‫𝜋‪2‬‬ ‫𝜋‪4‬‬
‫‪tan‬‬ ‫=‬ ‫‪5 − 2√5‬‬ ‫و‬ ‫‪tan‬‬ ‫=‬ ‫‪5 + 2√5‬‬
‫‪10‬‬ ‫‪10‬‬
‫𝜋‪2𝜋 4‬‬
‫∈ 𝜃 ﻟﺬا ّ‬
‫ﻓﺈن اﳊﻞ ‪ t = 0‬ﻣﺴﺘﺒﻌﺪ( ‪.‬‬ ‫‪,‬‬ ‫)‪ tan 𝜃 ≠ 0‬ﻷﺟﻞ‬
‫‪10 10‬‬
‫■‬

‫‪#‬‬
‫‪ņőƱŀ Ţ‬‬
‫‪.‬‬
‫𝟗𝟐𝟏‬ ‫‪.‬‬
‫ﻟﻴﻜﻦ ‪ m ≥ 1‬ﻋﺪد ًا ﻃﺒﻴﻌﻴﺎ ﻏﲑ ﻣﻌﺪوم؛ ‪ x‬و ‪ a‬ﻋﺪدﻳﻦ ﺣﻘﻴﻘﻴﲔ‪ .‬أﺛﺒﺖ ّ‬
‫أن ‪:‬‬
‫‪+‬‬ ‫‪+‬‬ ‫𝜋‪2k‬‬
‫‪x‬‬ ‫‪−a‬‬ ‫)‪= (x − a‬‬ ‫‪x − 2ax cos‬‬ ‫‪+a‬‬
‫=‬
‫‪2m + 1‬‬
‫‪VI‬‬

‫ﺍﳊﻞّ‪ .‬ﻧﻀﻊ ‪ f(x) = x + − a +‬و‬


‫‪2𝜋ı‬‬
‫‪) Ω = exp‬ﺟﺬر ﻣﻦ اﻟﺮﺗﺒﺔ ‪ 2m + 1‬ﻟﻠﻮﺣﺪة(‪.‬‬
‫‪2m + 1‬‬
‫ﻓﺈن اﳊﻠﻮل‬ ‫‪ ، a +‬و ﺑﲈ ّ‬
‫أن ‪ x = a‬ﻫﻲ إﺣﺪى ﻫﺬه اﳊﻠﻮل ّ‬ ‫ﺣﻠﻮل اﳌﻌﺎدﻟﺔ ‪ f(x) = 0‬ﻫﻲ اﳉﺬور ﻣﻦ اﻟﺮﺗﺒﺔ ‪ 2m + 1‬ﻟﻠﻌﺪد‬
‫‪ƕ‬‬

‫ﻫﻲ ‪ x = aΩ‬ﺣﻴﺚ }‪ . k ∈ {0, 1, 2, ⋯ , 2m‬و ﺑﲈ ّ‬


‫أن درﺟﺔ ﻛﺜﲑ اﳊﺪود )‪ f(x‬ﻫﻲ ‪ 2m + 1‬و ﻋﺪد ﻫﺬه اﳊﻠﻮل ﻫﻮ‬

‫⋅ ‪f(x) = c‬‬ ‫‪ 2m + 1‬ﻓﺈﻧﻨﺎ ﻧﻜﻮن ﻗﺪ وﺟﺪﻧﺎ ﲨﻴﻊ اﳊﻠﻮل؛ و ﺑﺎﻟﺘﺎﱄ ﺑﺈﻣﻜﺎﻧﻨﺎ ﻛﺘﺎﺑﺔ ﻛﺜﲑ اﳊﺪود )‪ f(x‬ﻋﲆ اﻟﺸﻜﻞ ) ‪(x − x‬‬
‫=‬
‫‪ũŏ‬‬

‫ﺣﻴﺚ ‪ c‬ﻫﻮ ﻣﻌﺎﻣﻞ وﺣﻴﺪ اﳊﺪ ذي اﻷﻛﱪ درﺟﺔ أي ‪ c = 1‬ﻣﻨﻪ ‪:‬‬


‫‪+‬‬
‫‪f(x) = (x − a) (x − aΩ) x − aΩ‬‬ ‫‪⋯ x − aΩ‬‬ ‫‪x − aΩ‬‬ ‫‪⋯ x − aΩ‬‬
‫⎭‬
‫⎪‬
‫⎪‬
‫⎪‬
‫⎪‬
‫⎪‬
‫⎪‬
‫⎪‬
‫⎪‬
‫⎪‬
‫⎪‬
‫⎪‬
‫⎪‬
‫⎪‬
‫⎪‬
‫⎪‬
‫⎪‬
‫⎪‬
‫⎪‬
‫⎪‬
‫⎬‬
‫⎪‬
‫⎪‬
‫⎪‬
‫⎪‬
‫⎪‬
‫⎪‬
‫⎪‬
‫⎪‬
‫⎪‬
‫⎪‬
‫⎪‬
‫⎪‬
‫⎪‬
‫⎪‬
‫⎪‬
‫⎪‬
‫⎪‬
‫⎪‬
‫⎪‬
‫⎫‬
‫⎭‬
‫⎪‬
‫⎪‬
‫⎪‬
‫⎪‬
‫⎪‬
‫⎪‬
‫⎪‬
‫⎪‬
‫⎪‬
‫⎪‬
‫⎪‬
‫⎪‬
‫⎪‬
‫⎪‬
‫⎪‬
‫⎬‬
‫⎪‬
‫⎪‬
‫⎪‬
‫⎪‬
‫⎪‬
‫⎪‬
‫⎪‬
‫⎪‬
‫⎪‬
‫⎪‬
‫⎪‬
‫⎪‬
‫⎪‬
‫⎪‬
‫⎪‬
‫⎫‬

‫ﺣﺪّ ًا‬ ‫ﺣﺪّ ًا‬

‫𝟳𝟵𝟭‬

‫‪http ://tinyurl.com/Malki1718‬‬ ‫‪0‬‬


‫ ﺗﻄﺒﻴﻘﺎت اﻷﻋﺪاد اﳌﺮﻛﺒﺔ‬.3.VI
.
+ −
‫ و‬k ∈ {1, 2, ⋯ m − 1, m} ‫ ﺣﻴﺚ‬x − aΩ ّ ‫ﻓﺈن‬
ّ‫ ُﻳﻘﺎﺑﻠﻪ اﳊﺪ‬x − aΩ ‫ﻛﻞ ﺣﺪّ ﻣﻦ اﻟﺸﻜﻞ‬ ّ ، x − a ‫ﺑﺎﺳﺘﺜﻨﺎء اﳊﺪ‬
: (Ω = Ω ‫ و‬Ω + = 1 ‫أن‬

ّ ‫ﺑﺎﻟﺘﺎﱄ )ﻧُﺬﻛّﺮ‬

Ŕž Ŧ Ľ
+ −
f(x) = (x − a) x − aΩ x − aΩ

œǃ
=

+ − +
= (x − a) x − ax Ω + Ω +a Ω
=

Ŀ
= (x − a) x − ax Ω + Ω− +a ×1
=

= (x − a) x − ax Ω + Ω +a
=

= (x − a) x − 2ax Re Ω +a
=

= (x − a) x − 2ax Re e / + +a
=
2k𝜋
= (x − a) x − 2ax cos +a
=
2m + 1

■ .‫و ﻫﻮ اﳌﻄﻠﻮب‬

: ‫أﻛﺘﺐ ﻋﲆ ﺷﻜﻞ ﺟﺪاء ﻛﻼ ﻣﻦ‬


 .
𝟏𝟑𝟎 .

sin x + sin 2x + sin 7x + sin 8x • 2 cos x + 2 cos 2x + cos 3x • 1

.ّ‫ﺍﳊﻞ‬
ņőƱŀ Ţ
• 1
x + 3x x − 3x
cos x + cos 3x = 2 cos cos = 2 cos 2x cos (−x) = 2 cos x cos 2x : ‫ﻟﺪﻳﻨﺎ‬
2 2
cos x + 2 cos 2x + cos 3x = 2 cos 2x + 2 cos x cos 2x = 2 cos 2x (1 + cos x) : ‫ﻣﻨﻪ‬

VI
x x
= 2 cos 2x 2 cos = 4 cos 2x cos
2 2
• 2
x + 8x x − 8x 9x 7x
sin x + sin 8x = 2 sin cos = 2 sin cos : ‫ﻟﺪﻳﻨﺎ‬
ƕ

2 2 2 2
2x + 7x 2x − 7x 9x 5x
sin 2x + sin 7x = 2 sin cos = 2 sin cos ‫و‬
2 2 2 2
9x 7x 9x 5x
sin x + sin 2x + sin 7x + sin 8x = 2 sin cos + 2 sin cos : ‫ﻣﻨﻪ‬
ũŏ

2 2 2 2
9x 7x 5x
= 2 sin cos + cos
2 2 2
9x + −
= 2 sin 2 cos cos
2 2 2
9x x
= 4 sin cos 3x cos
2 2

𝟭𝟵𝟴

http ://tinyurl.com/Malki1718 0
‫ ﲤﺎرﻳﻦ ﺗﻄﺒﻴﻘﻴﺔ‬.VI
.

Ŕž Ŧ Ľ
: ‫ﺑﺴﻂ ﻛﻼ ﻣﻦ‬
‫ﱢ‬
 .
𝟏𝟑𝟏 .

œǃ

P = (2 cos x − 1) (2 cos 2x − 1) ⋯ 2 cos 2 x −1 •1
cos 6x + 6 cos 4x + 15 cos 2x + 10
2

Ŀ
Q= •
cos 5x + 5 cos 3x + 10 cos x

: ‫• ﻟﺪﻳﻨﺎ‬1 .ّ‫ﺍﳊﻞ‬
+
1 + cos 2 x
+
2 cos 2 x − 1 2 cos 2 x + 1 = 4 cos 2 x −1 = 4 −1 = 2 cos 2 x +1
2
+
2 cos 2 x +1
: ‫ ﻳﻜﻮن‬a = 2 cos 2 x + 1 ‫ ﺑﻮﺿﻊ‬. 2 cos 2 x − 1 = ‫ﻣﻨﻪ‬
2 cos 2 x + 1

− −
a + gray a −  a
 a  gray
a gray a 2 cos 2 x + 1
P= 2 cos 2 x − 1 = = × × ⋯ ×  × = =
a a 
a gray a 
 gray −
 a 
gray 
− a 2 cos x + 1
= =

: ‫• ﻟﺪﻳﻨﺎ‬2
(2 cos x) (cos 5x + 5 cos 3x + 10 cos x) = 2 cos x cos 5x + 10 cos x cos 3x + 20 cos x
= (cos (x + 5x) + cos (x − 5x)) + 5 (cos (x + 3x) + cos (x − 3x)) + 10 (1 + cos 2x)
= cos 6x + cos 4x + 5 cos 4x + 5 cos 2x + 10 + 10 cos 2x
= cos 6x + 6 cos 4x + 15 cos 2x + 10
ņőƱŀ Ţ
cos 6x + 6 cos 4x + 15 cos 2x + 10
Q= = 2 cos x : ‫ﻣﻨﻪ‬
cos 5x + 5 cos 3x + 10 cos x
VI

: ‫أوﺟﺪ داﻟﺔ أﺻﻠﻴﺔ ﻟﻜﻞ ﻣﻦ اﻟﺪوال اﻵﺗﻴﺔ‬


 .
𝟏𝟑𝟐 .
ƕ

f (x) = cos x sin x • 7 f (x) = cos x sin x • 4 f (x) = cos x •1


f (x) = sin x • 5 f (x) = cos x •2
f (x) = cos x 8 f (x) = cos x sin x 6 f (x) = sin x 3
ũŏ

• • •

.ّ‫ﺍﳊﻞ‬
1 1
. sin x = (e − e− ) ‫ و‬cos x = (e + e− ) : ‫ﻣﻔﺘﺎح اﳊﻞ ﻫﻮ ﺻﻴﻎ ﺃﻭﻟﺮ‬
2ı 2

𝟭𝟵𝟵

http ://tinyurl.com/Malki1718 0
‫ ﺗﻄﺒﻴﻘﺎت اﻷﻋﺪاد اﳌﺮﻛﺒﺔ‬.3.VI
.
: ‫• ﻟﺪﻳﻨﺎ‬1

Ŕž Ŧ Ľ
e + e− 1
cos x = = e + 2e e− + e−
2 4
+ e−

œǃ
1 1 e 1 1
= e + e− +2 = + = (1 + cos 2x)
4 2 2 2 2

1 1
.x ↦ x+ sin 2x ‫ ﻫﻲ اﻟﺪاﻟﺔ‬x ↦ cos x ‫و ﺑﺎﻟﺘﺎﱄ إﺣﺪى اﻟﺪوال اﻷﺻﻠﻴﺔ ﻟﻠﺪاﻟﺔ‬
2 2

Ŀ
: ‫• ﻟﺪﻳﻨﺎ‬2

e + e− 1
cos x = = e + 4e e− + 6e e− + 4e e− + e−
2 16
1 1 e + e− 1 e + e− 6
= e + 4e + 6 + 4e− + e− = + +
16 8 2 2 2 16
1 1 3
= cos 4x + cos 2x +
8 2 8
1 1 3
.x ↦ sin 4x + sin 2x + x ‫ ﻫﻲ اﻟﺪاﻟﺔ‬x ↦ cos x ‫إذن إﺣﺪى اﻟﺪوال اﻷﺻﻠﻴﺔ ﻟﻠﺪاﻟﺔ‬
32 4 8
: ‫• ﻟﺪﻳﻨﺎ‬3

e − e− 1
sin x = = e − 4e e− + 6e e− − 4e e− + e−
2ı 16
1 1 e + e− 1 e + e− 6
= e − 4e + 6 − 4e− + e− = − +
16 8 2 2 2 16
1 1 3
= cos 4x − cos 2x +
8 2 8
ņőƱŀ Ţ
1 1 3
.x ↦ sin 4x − sin 2x + x ‫ ﻫﻲ اﻟﺪاﻟﺔ‬x ↦ cos x ‫إذن إﺣﺪى اﻟﺪوال اﻷﺻﻠﻴﺔ ﻟﻠﺪاﻟﺔ‬
32 4 8
ّ ‫ﻳﻤﻜﻦ أﻳﻀ ًﺎ ﻣﻼﺣﻈﺔ‬
: ‫أن‬

VI
sin x = sin x = 1 − cos x = 1 − 2 cos x + cos x

.‫و اﺳﺘﻌﲈل ﻧﺘﻴﺠﺔ اﻟﺴﺆاﻟﲔ اﻟﺴﺎﺑﻘﲔ‬


1
: cos x sin x = sin 2x ‫• ﻟﺪﻳﻨﺎ‬4
4
ƕ

e − e− 1
sin x = =− e − 2e e− + e−
2ı 4
1 1 e + e− 1 1
=− e + e− −2 =− + = (1 − cos 4x)
ũŏ

4 2 2 2 2

1
‫ ﻫﻲ اﻟﺪاﻟﺔ‬x ↦ cos x sin x ‫ و ﺑﺎﻟﺘﺎﱄ إﺣﺪى اﻟﺪوال اﻷﺻﻠﻴﺔ ﻟﻠﺪاﻟﺔ‬cos x sin x = (1 − cos 4x) ‫ﻣﻨﻪ‬
8
1 1
.x ↦ x − sin 2x
8 4

𝟮𝟬𝟬

http ://tinyurl.com/Malki1718 0
‫‪ .VI‬ﲤﺎرﻳﻦ ﺗﻄﺒﻴﻘﻴﺔ‬
‫‪.‬‬
‫‪ •5‬ﻟﺪﻳﻨﺎ ‪:‬‬

‫‪Ŕž Ŧ Ľ‬‬
‫‪e‬‬ ‫‪− e−‬‬ ‫‪1‬‬
‫= ‪sin x‬‬ ‫‪=−‬‬ ‫‪e‬‬ ‫‪− 6e‬‬ ‫‪+ 15e‬‬ ‫‪− 20 + 15e−‬‬ ‫‪− 6e−‬‬ ‫‪+ e−‬‬
‫‪2ı‬‬ ‫‪64‬‬
‫‪1 e‬‬ ‫‪+ e−‬‬ ‫‪6 e‬‬ ‫‪+ e−‬‬ ‫‪15‬‬ ‫‪e‬‬ ‫‪+ e−‬‬ ‫‪20‬‬

‫‪œǃ‬‬
‫‪=−‬‬ ‫‪+‬‬ ‫‪−‬‬ ‫‪+‬‬
‫‪32‬‬ ‫‪2‬‬ ‫‪32‬‬ ‫‪2‬‬ ‫‪32‬‬ ‫‪2‬‬ ‫‪64‬‬
‫‪1‬‬ ‫‪3‬‬ ‫‪15‬‬ ‫‪5‬‬
‫‪= − cos 6x +‬‬ ‫‪cos 4x −‬‬ ‫‪cos 2x +‬‬
‫‪32‬‬ ‫‪16‬‬ ‫‪32‬‬ ‫‪16‬‬

‫‪Ŀ‬‬
‫‪1‬‬ ‫‪3‬‬ ‫‪15‬‬ ‫‪5‬‬
‫‪.x ↦ −‬‬ ‫و ﺑﺎﻟﺘﺎﱄ إﺣﺪى اﻟﺪوال اﻷﺻﻠﻴﺔ ﻟﻠﺪاﻟﺔ ‪ x ↦ sin x‬ﻫﻲ اﻟﺪاﻟﺔ ‪sin 6x + sin 4x − sin 2x + x‬‬
‫‪192‬‬ ‫‪64‬‬ ‫‪64‬‬ ‫‪16‬‬
‫‪ •6‬ﻧﻀﻊ ‪ u (x) = sin x‬ﻣﻨﻪ ‪ u (x) = cos x‬و )‪ cos x sin x = u (x) u (x‬و ﺑﺎﻟﺘﺎﱄ إﺣﺪى اﻟﺪوال اﻷﺻﻠﻴﺔ ﻟﻠﺪاﻟﺔ‬
‫‪1‬‬
‫‪ x ↦ cos x sin x‬ﻫﻲ اﻟﺪاﻟﺔ ‪. x ↦ sin x‬‬
‫‪7‬‬
‫‪ •7‬ﻟﺪﻳﻨﺎ ‪:‬‬

‫‪cos x sin x = cos x 1 − sin x‬‬ ‫‪sin x = cos x sin x − 2 sin x + sin x‬‬
‫)‪= u (x) u (x) − 2u (x) u (x) + u (x) u (x‬‬

‫ﺣﻴﺚ ‪. u (x) = sin x‬‬


‫‪1‬‬ ‫‪2‬‬ ‫‪1‬‬
‫و ﺑﺎﻟﺘﺎﱄ إﺣﺪى اﻟﺪوال اﻷﺻﻠﻴﺔ ﻟﻠﺪاﻟﺔ ‪ x ↦ cos x sin x‬ﻫﻲ اﻟﺪاﻟﺔ ‪. x ↦ sin x − sin x + sin x‬‬
‫‪3‬‬ ‫‪5‬‬ ‫‪7‬‬
‫‪ •8‬ﻟﺪﻳﻨﺎ ‪:‬‬
‫‪e‬‬ ‫‪+ e−‬‬ ‫‪1‬‬
‫= ‪cos x‬‬ ‫=‬ ‫‪e‬‬ ‫‪+ 3e‬‬ ‫‪+ 3e−‬‬ ‫‪+ e−‬‬
‫‪2‬‬ ‫‪8‬‬
‫‪1‬‬ ‫‪e‬‬ ‫‪+ e−‬‬ ‫‪3‬‬ ‫‪e‬‬ ‫‪+ e−‬‬ ‫‪1‬‬
‫=‬ ‫‪+‬‬ ‫=‬ ‫)‪(cos 3x + 3 cos x‬‬
‫‪4‬‬ ‫‪2‬‬ ‫‪4‬‬ ‫‪2‬‬ ‫‪4‬‬
‫‪ņőƱŀ Ţ‬‬
‫‪1‬‬ ‫‪1‬‬
‫↦ ‪.x‬‬ ‫‪sin 3x + 3 sin x‬‬ ‫و ﺑﺎﻟﺘﺎﱄ إﺣﺪى اﻟﺪوال اﻷﺻﻠﻴﺔ ﻟﻠﺪاﻟﺔ ‪ x ↦ cos x‬ﻫﻲ اﻟﺪاﻟﺔ‬
‫‪4‬‬ ‫‪3‬‬
‫ﻳﻤﻜﻦ أﻳﻀ ًﺎ ﻣﻼﺣﻈﺔ ّ‬
‫أن ‪ cos x = 1 − sin x cos x = cos x − sin x cos x‬و ﺑﺎﻟﺘﺎﱄ إﺣﺪى اﻟﺪوال اﻷﺻﻠﻴﺔ‬
‫‪VI‬‬

‫‪1‬‬
‫ﻟﻠﺪاﻟﺔ ‪ x ↦ cos x‬ﻫﻲ اﻟﺪاﻟﺔ ‪. x ↦ sin x − sin x‬‬
‫‪3‬‬
‫‪1‬‬ ‫‪1‬‬ ‫‪1‬‬
‫↦ ‪ g: x‬و ‪ h: x ↦ sin x − sin x‬داﻟﺔ‬ ‫‪sin 3x + 3 sin x‬‬ ‫ﻼ ﻣﻦ‬ ‫وﺟﺪﻧﺎ ّ‬
‫أن ﻛ ً‬ ‫ﻣﻼﺣﻈﺔ ‪⧏ : 31‬‬
‫‪3‬‬ ‫‪4‬‬ ‫‪3‬‬
‫أﺻﻠﻴﺔ ﻟﻠﺪاﻟﺔ ‪ x ↦ cos x‬ﻟﻜﻦ ﻫﺬا ﻻ ﻳﻌﻨﻲ ﺑﺎﻟﴬورة أﳖﲈ ﻣﺘﺴﺎوﻳﺘﺎن ﺑﻞ ّ‬
‫ﻛﻞ ﻣﺎ ﻧﺴﺘﻄﻴﻊ ﻗﻮﻟﻪ ﻫﻮ أﻧﻪ ﻳﻮﺟﺪ ‪c ∈ ℝ‬‬
‫‪ƕ‬‬

‫ﺑﺤﻴﺚ ‪ . g (x) = h (x) + c‬ﻹﳚﺎد ﻗﻴﻤﺔ ‪ c‬ﻳﻜﻔﻲ إﻋﻄﺎء ‪ x‬ﻗﻴﻤﺔ ﻣﻌﻴﻨﺔ ﻣﺜ ً‬


‫ﻼ ‪ x = 0‬و ﻧﺠﺪ ‪ g (0) = 0‬و ‪h (0) = 0‬‬

‫⧐‬
‫ﻣﻨﻪ ‪) c = 0‬أي ﺻﺪﻓ ًﺔ ‪. (! g = h‬‬
‫‪ũŏ‬‬

‫■‬

‫‪/‬‬ ‫‪/‬‬ ‫‬ ‫‪.‬‬


‫𝟑𝟑𝟏‬ ‫‪.‬‬
‫= ‪.J‬‬ ‫‪cos x sin x dx‬‬ ‫و‬ ‫=‪I‬‬ ‫‪cos x sin x dx‬‬ ‫أﺣﺴﺐ ‪:‬‬
‫‪/‬‬ ‫‪/‬‬

‫𝟭𝟬𝟮‬

‫‪http ://tinyurl.com/Malki1718‬‬ ‫‪0‬‬


‫ ﺗﻄﺒﻴﻘﺎت اﻷﻋﺪاد اﳌﺮﻛﺒﺔ‬.3.VI
.
ّ ‫ ﻧﻼﺣﻆ‬،‫• ﺑﺪاﻳ ًﺔ‬1 .ّ‫ﺍﳊﻞ‬
‫ زوﺟﻴﺔ و ﺑﺎﻟﺘﺎﱄ ﻋﺒﺎرﲥﺎ اﳋﻄﻴﺔ زوﺟﻴﺔ أﻳﻀ ًﺎ أي أﳖﺎ ﻻ ﲢﺘﻮي‬x ↦ cos x sin x ‫أن اﻟﺪاﻟﺔ‬
: ‫ ﻟﺪﻳﻨﺎ‬. sin ‫ﻋﲆ داﻟﺔ اﳉﻴﺐ‬

Ŕž Ŧ Ľ
e + e− e − e−

œǃ
cos x sin x =
2 2ı
1
= e + 4e + 6 + 4e− + e− ×
16
−1

Ŀ
× e − 6e + 15e − 20 + 15e− − 6e− + e−
64
1
=− e − 2e − 3e + 8e + 2e − 12+
1024
+2e− + 8e− − 3e− − 2e− + e−

1 e + e− e + e− e + e−
=− −2⋅ −3⋅ +
512 2 2 2

e + e− e + e−
+8 ⋅ +2⋅ −6
2 2
1
=− (cos 10x − 2 cos 8x − 3 cos 6x + 8 cos 4x + 2 cos 2x − 6)
512

/
1
I= − (cos 10x − 2 cos 8x − 3 cos 6x + 8 cos 4x + 2 cos 2x − 6) dx : ‫ﻣﻨﻪ‬
512
/
/
1 1 1 1
=− sin 10x − sin 8x − sin 6x + 2 sin 4x + sin 2x − 6x
512 10 4 2 /
1 1 √3 √3 1 √3 √3 1
=− − + − + − (0 − 0) +
ņőƱŀ Ţ
512 10 2 2 4 2 2 2

√3 √3 √3 √3 𝜋 𝜋
+2 − − + − −6 −
2 2 2 2 3 6

VI
1 √3 9√3 + 4𝜋
=− − − 2√3 − 𝜋 = ≃ 0.014
512 4 2048

: ‫• ﻟﺪﻳﻨﺎ‬2
ƕ

cos x sin x = cos x sin x sin x = cos x 1 − cos x sin x


= cos x 1 − 3 cos x + 3 cos x − cos x sin x
= cos x sin x − 3 cos x sin x + 3 cos x sin x − cos x sin x
ũŏ

= −u (x) u (x) + 3u (x) u (x) − 3u (x) u (x) + u (x) u (x)

: ‫ ﻣﻨﻪ‬u (x) = cos x ‫ﺣﻴﺚ‬


/

J= −u (x) u (x) + 3u (x) u (x) − 3u (x) u (x) + u (x) u (x) dx


/

𝟮𝟬𝟮

http ://tinyurl.com/Malki1718 0
‫ ﲤﺎرﻳﻦ ﺗﻄﺒﻴﻘﻴﺔ‬.VI
.
/
1 3 1 1
= − u (x) + u (x) − u (x) + u (x)
5 7 3 11

Ŕž Ŧ Ľ
/
/
1 3 1 1
= − cos x + cos x − cos x + cos x
5 7 3 11

œǃ
/
1 1 − 9√3 3 1 − 27√3 1 1 − 81√3 1 1 − 243√3
=− + − +
5 32 7 128 3 512 11 2048
−8299 + 18441√3

Ŀ
= ≃ 0.01
2365440

: ‫ﻣﺒﺴﻄﺔ ﻟﻠﺠﺪاء‬
1 tan 2x
ّ ‫أﺛﺒﺖ‬
# .
𝟏𝟑𝟒 .
ّ ‫ﺛﻢ اﺳﺘﻨﺘﺞ ﻛﺘﺎﺑﺔ‬
ّ 1+
cos 2x
=
tan x
‫أن‬

⎛1 + 1 ⎞
P =
= ⎝ cos 2 x ⎠

: ‫ ﻟﺪﻳﻨﺎ‬. cos 2x ≠ 0 ‫ و‬sin x ≠ 0 ، cos x ≠ 0 ‫ ﺑﺤﻴﺚ‬x ∈ ℝ ‫ﻟﻴﻜﻦ‬ • .ّ‫ﺍﳊﻞ‬


1 1 + cos 2x 2 cos x 2 cos x sin x cos x sin 2x cos x
1+ = = = =
cos 2x cos 2x cos 2x cos 2x sin x cos 2x sin x
sin 2x cos x 1 tan 2x
= × = tan 2x × =
cos 2x sin x tan x tan x
1 tan x
.1+ = : ‫و ﻛﺤﺎﻟﺔ ﺧﺎﺻﺔ‬
cos x tan (x/2)
ņőƱŀ Ţ
ّ k ∈ {0, 1, ⋯ , n} ‫ﻧﻔﺮض أﻧﻪ ﻣﻬﲈ ﻛﺎن‬
. cos 2 x ≠ 0 ‫ﻓﺈن‬ •

ّ cos (x/2) = 0 ‫➛ إذا ﻛﺎن‬


. (‫ )اﳊﺪّ اﻷول ﰲ اﳉﺪاء ﻣﻌﺪوم‬P = 0 ‫ ﻣﻨﻪ‬cos x = 2 cos (x/2) − 1 = −1 ‫ﻓﺈن‬
ّ sin (x/2) = 0 ‫➛ إذا ﻛﺎن‬
‫ و ﺑﺼﻔﺔ‬cos 2x = 2 cos x − 1 = 2 − 1 = 1 ، cos x = 1 − 2 sin x = 1 ‫ﻓﺈن‬
VI

: ‫ ﻣﻨﻪ‬cos 2 x = 1 ‫ﻓﺈن‬ ّ k ∈ {0, 1, ⋯ , n} ‫ ﻣﻬﲈ ﻛﺎن‬،‫ﻋﺎﻣﺔ‬

⎛1 + 1 ⎞= 1 +
P = 1+ = 2=2
= cos 2 x ⎠ =
1 =

ƕ

− −
‫ ﻓﺈﻧّﻪ‬sin 2 x = 2 sin 2 x cos 2 ّ ‫ ﺑﲈ‬. sin (x/2) ≠ 0 ‫ و‬cos (x/2) ≠ 0 ‫أن‬
‫أن‬
x ّ ‫➛ ﻧﻔﺮض‬
ّ
: ‫ و ﺑﺘﻄﺒﻴﻖ اﳌﺴﺎواة اﳌﱪﻫﻨﺔ أﻋﻼه ﻳﻨﺘﺞ‬sin 2 x ≠ 0 ‫ ﻳﻜﻮن‬k ∈ {0, 1, ⋯ , n} ‫ﻟﻜﻞ‬

tan
gray xgray
tan2x tan 2 x tan 2 x
ũŏ

P = × ×⋯×
  =

tan
tan (x/2) gray x gray 2  x tan (x/2)
tan

.
𝟏𝟑𝟓 .

. k ∈ {1, 2, ⋯ , n} ‫ ﻟﻜﻞ‬cos (kx) ≠ 0 ‫ ﺑﺤﻴﺚ‬n ∈ ℕ ‫ ﻋﺪد ًا ﺣﻘﻴﻘﻴﺎ و‬x ‫ﻟﻴﻜﻦ‬

𝟮𝟬𝟯

http ://tinyurl.com/Malki1718 0
‫ ﺗﻄﺒﻴﻘﺎت اﻷﻋﺪاد اﳌﺮﻛﺒﺔ‬.3.VI
.
sin x
. tan (k + 1) x − tan kx = ّ ‫• أﺛﺒﺖ‬1
: ‫أن‬

Ŕž Ŧ Ľ
cos (k + 1) x cos kx

1
.S = ‫• ﱢ‬2
: ‫ﺑﺴﻂ اﻟﻌﺒﺎرة‬

œǃ
=
cos kx cos (k + 1) x

: ‫• ﻟﺪﻳﻨﺎ‬1 .ّ‫ﺍﳊﻞ‬

Ŀ
sin (k + 1) x sin kx
tan (k + 1) x − tan kx = −
cos (k + 1) x cos kx
sin (k + 1) x cos kx − cos (k + 1) x sin kx
=
cos kx cos (k + 1) x
sin ((k + 1)x − kx) sin x
= =
cos kx cos (k + 1) x cos kx cos (k + 1) x

1 tan (k + 1) x − tan kx
: ‫ﻣﻨﻪ‬ = ّ sin x ≠ 0 ‫)ا( إذا ﻛﺎن‬
‫ﻓﺈن‬ • 2
cos kx cos (k + 1) x sin x

1 tan (k + 1) x − tan kx 1
S = = = (tan (k + 1) x − tan kx)
=
cos kx cos (k + 1) x =
sin x sin x =
1
= [(tan 2x − tan x) + (tan 3x − tan 2x) + ⋯ + (tan (n + 1) x − tan nx)]
sin x
1 tan (n + 1) x − tan x
= [tan (n + 1) x − tan x] =
sin x sin x

: ‫ ﻟﺪﻳﻨﺎ‬k ∈ ℕ ‫ إذن ﻟﻜﻞ‬، sin hx = 0 ‫ ﻳﻜﻮن‬h ∈ ℤ ‫ ﻓﺈﻧﻪ ﻟﻜﻞ‬sin x = 0 ‫)ب( إذا ﻛﺎن‬
cos x = cos ((k + 1)x − kx) = cos (k + 1) x cos kx − sin (k + 1) x sin kx
ņőƱŀ Ţ
= cos (k + 1) x cos kx
1 1 n
S = = = ‫ﻣﻨﻪ‬
=
cos kx cos (k + 1) x =
cos x cos x

VI

.
𝟏𝟑𝟔 .
: ‫ ﻧﻀﻊ‬. k ∈ {0, 1, 2, ⋯ , n − 1} ‫ ﻟﻜﻞ‬cos 2 x ≠ 0 ‫ ﺑﺤﻴﺚ‬n ∈ ℕ∗ ‫ ﻋﺪد ًا ﺣﻘﻴﻘﻴﺎ و‬x ‫ﻟﻴﻜﻦ‬
ƕ

1
u =

2 cos x cos 2x ⋯ cos 2 x

: ‫ ﻋﲆ اﻟﺸﻜﻞ‬k ≥ 2 ‫ ﻷﺟﻞ‬u ‫ﻧﺮﻳﺪ ﻛﺘﺎﺑﺔ اﻟﻌﺪد‬


ũŏ

a − a
u = −
− − −
2 cos x cos 2x ⋯ cos 2 x 2 cos x cos 2x ⋯ cos 2 x


. 2 cos 2 x a − − a = 1 : ‫أﺛﺒﺖ أﻧﻪ ﰲ ﻫﺬه اﳊﺎﻟﺔ‬ • 1

.‫ ُﲢﻘﻖ ﻫﺬا اﻟﴩط‬a = cos 2 x ‫ﻓﺈن اﻷﻋﺪاد‬


ّ h ≥ 1 ‫• أﺛﺒﺖ أﻧﻪ ﻷﺟﻞ‬2

𝟮𝟬𝟰

http ://tinyurl.com/Malki1718 0
‫ ﲤﺎرﻳﻦ ﺗﻄﺒﻴﻘﻴﺔ‬.VI
.
a
. u = cos x − ّ ‫• ﲢﻘﻖ ﻣﻦ‬3
‫أن‬

Ŕž Ŧ Ľ
2 cos x
1
.S = : ‫• إﺳﺘﻨﺘﺞ ﻛﺘﺎﺑﺔ ﻣﺒﺴﻄﺔ ﻟﻠﻤﺠﻤﻮع‬4

œǃ

= 2 cos x cos 2x ⋯ cos 2 x

.ّ‫ﺍﳊﻞ‬

Ŀ
ّ ‫ ﺣﻴﺚ‬،‫• ﻳﻜﻔﻲ ﺗﻮﺣﻴﺪ اﳌﻘﺎﻣﺎت ﻟﻠﺤﺼﻮل ﻋﲆ اﳌﻄﻠﻮب‬1
‫أن اﳌﻘﺎم اﳌﺸﱰك ﻫﻮ‬
− −
2 cos 2 x (‫ ﻓﺒﴬب اﳊﺪ اﻷول ﺑﺴﻄ ًﺎ و ﻣﻘﺎﻣ ًﺎ ﺑﺎﻟﻌﺪد )ﻏﲑ اﳌﻌﺪوم‬، 2 cos x cos 2x ⋯ cos 2 x
: ‫ﻳﻨﺘﺞ‬
a − a
u = −
− − −
2 cos x cos 2x ⋯ cos 2 x 2 cos x cos 2x ⋯ cos 2 x

2 cos 2 x a − −a 1
= =
− −
2 cos x cos 2x ⋯ cos 2 x 2 cos x cos 2x ⋯ cos 2 x


. 2 cos 2 x a − −a =1 : ‫ﻣﻨﻪ‬

: ‫ ﻣﻨﻪ‬a − = cos 2 x ّ a = cos 2 x ‫• إذا ﻛﺎن‬2
‫ﻓﺈن‬
− − −
2 cos 2 x a − − a = 2 cos 2 x cos 2 x − cos 2 x

= 2 cos 2 x − cos 2 x

= cos 2 × 2 x + 1 − cos 2 x

= cos 2 x + 1 − cos 2 x = 1
ņőƱŀ Ţ
: ‫• ﻟﺪﻳﻨﺎ‬3
1 2 cos x − cos 2x cos 2x a
u = = = cos x − = cos x −
VI

2 cos x 2 cos x 2 cos x 2 cos x


: ‫ ﻟﺪﻳﻨﺎ‬،‫• ﺣﺴﺐ ﻣﺎ ﺳﺒﻖ‬4
1
S =

= 2 cos x cos 2x ⋯ cos 2 x
ƕ

⎛ a − a ⎞
= −
− − −
= ⎝2 cos x cos 2x ⋯ cos 2 x 2 cos x cos 2x ⋯ cos 2 x ⎠
ũŏ

a   a  a 
= cos xgray
−  +gray  −
gray    + ⋯ +
2 cos
x x 2 
2 cos cos x cos 2x


a −  a
+⎛   − ⎞
gray − ⋯ cos 2 − x −
⎝ 2 cos  2x
x cos 2 cos x cos 2x ⋯ cos 2 x ⎠

a
= cos x −

2 cos x cos 2x ⋯ cos 2 x

𝟮𝟬𝟱

http ://tinyurl.com/Malki1718 0
‫‪ .3.VI‬ﺗﻄﺒﻴﻘﺎت اﻷﻋﺪاد اﳌﺮﻛﺒﺔ‬
‫‪.‬‬
‫‪cos 2 x‬‬
‫‪= cos x −‬‬
‫‪−‬‬
‫‪2 cos x cos 2x ⋯ cos 2‬‬ ‫‪x‬‬

‫‪Ŕž Ŧ Ľ‬‬
‫■‬

‫‪œǃ‬‬
‫ﻧﻘﺒﻞ ﺑﺪون ﺑﺮﻫﺎن اﻟﻨﺘﻴﺠﺔ اﻵﺗﻴﺔ ‪:‬‬
‫!‬ ‫‪.‬‬
‫𝟕𝟑𝟏‬ ‫‪.‬‬

‫‪Ŀ‬‬
‫ﻛﺜﲑات اﳊﺪود )ﺑﻤﻌﺎﻣﻼت ﺣﻘﻴﻘﻴﺔ( ﻏﲑ اﻟﻘﺎﺑﻠﺔ ﻟﻠﺘﺤﻠﻴﻞ ﻋﲆ ‪ ℝ‬ﻫﻲ وﺣﻴﺪات اﳊﺪ و ﻛﺜﲑات اﳊﺪود ﻣﻦ اﻟﺪرﺟﺔ اﻟﺜﺎﻧﻴﺔ و‬
‫اﻟﺘﻲ ﳑﻴﺰﻫﺎ ﺳﺎﻟﺐ ﲤﺎﻣﺎ أي ‪:‬‬
‫‪ P(X) = aX + b‬و ‪ P(X) = uX + vX + w‬ﺑﺤﻴﺚ ‪. v − 4uw < 0‬‬

‫ﻟﻴﻜﻦ )‪ P (X‬ﻛﺜﲑ ﺣﺪود ﺑﺤﻴﺚ ‪ P (x) ≥ 0‬ﻟﻜﻞ ‪ . x ∈ ℝ‬أﺛﺒﺖ أﻧﻪ ﻳﻮﺟﺪ ﻛﺜﲑا ﺣﺪود )‪ A (X‬و )‪ B (X‬ﺑﺤﻴﺚ‬
‫‪.P = A +B‬‬

‫ﺍﳊﻞّ‪ .‬رأﻳﻨﺎ ﰲ اﳌﻼﺣﻈﺔ ‪ 7‬ﺻﻔﺤﺔ ‪ 17‬أﻧﻪ ﻣﻬﲈ ﺗﻜﻦ اﻷﻋﺪاد اﳊﻘﻴﻘﻴﺔ ‪ّ D ، C ، B ، A‬‬
‫ﻓﺈن ‪:‬‬
‫‪A +B‬‬ ‫‪C +D‬‬ ‫)‪= (AC + BD) + (AD − BC‬‬ ‫)𝟕(‬
‫ﻫﺬه اﳌﺘﻄﺎﺑﻘﺔ ﺻﺤﻴﺤﺔ أﻳﻀ ًﺎ إذا ﻛﺎﻧﺖ ‪ D ، C ، B ، A‬ﻛﺜﲑات ﺣﺪود )أو دوال أو ‪ ...‬إﻟﺦ( و ﻳﻤﻜﻦ إﺛﺒﺎت ذﻟﻚ إﻣﺎ ﺑﺎﺗّﺒﺎع‬
‫ﻧﻔﺲ اﻟﻄﺮﻳﻘﺔ أو ﺑﻨﴩ اﻟﻄﺮﻓﲔ‪.‬‬
‫ﻓﺈن ﲢﻠﻴﻞ )‪ P(X‬إﱃ ﺟﺪاء ﻋﻮاﻣﻞ‬ ‫ﻟﻴﻜﻦ )‪ P(X‬ﻛﺜﲑ ﺣﺪود ﺑﻤﻌﺎﻣﻼت ﺣﻘﻴﻘﻴﺔ‪ .‬ﺣﺴﺐ اﻟﻨﺘﻴﺠﺔ اﻟﺘﻲ ﻗﺒﻠﻨﺎﻫﺎ ﺑﺪون ﺑﺮﻫﺎن‪ّ ،‬‬
‫أوﻟﻴﺔ ﻫﻮ ‪:‬‬
‫𝜆 = )‪P(X‬‬ ‫) ‪(X − a‬‬ ‫𝛽‪X +𝛼 X+‬‬ ‫‪,‬‬ ‫‪𝛼 − 4𝛽 < 0‬‬
‫=‬ ‫=‬

‫أن )‪ P(X‬ﻣﻮﺟﺐ داﺋ ًﲈ ّ‬


‫ﻓﺈن ‪ 𝜆 ≥ 0‬و ﻛﻞ ‪ m‬زوﺟﻲ ‪.‬‬ ‫و ﺑﲈ ّ‬
‫‪ņőƱŀ Ţ‬‬
‫ﻧﻀﻊ 𝜇 = 𝜆 ‪ m = 2n ،‬و ) ‪ . A = (X − a‬ﻟﺪﻳﻨﺎ ‪:‬‬
‫) ‪. (X − a‬‬ ‫) ‪= (X − a‬‬ ‫‪=A =A +0‬‬ ‫‪ 𝜆=𝜇 +0‬و‬
‫ﻣﻦ ﺟﻬﺔ أﺧﺮى‪ ،‬ﻟﺪﻳﻨﺎ ‪:‬‬
‫𝛼‬ ‫𝛼 ‪4𝛽 −‬‬
‫‪X +𝛼 X+𝛽 = X+‬‬ ‫‪+‬‬

‫‪VI‬‬
‫‪2‬‬ ‫‪4‬‬
‫𝛼 ‪4𝛽 −‬‬ ‫𝛼 ‪4𝛽 −‬‬
‫= 𝜈 ‪ .‬و ﺑﺎﻟﺘﺎﱄ‪ّ ،‬‬
‫ﻓﺈن ‪:‬‬ ‫ﺣﻴﺚ‬ ‫و ﺑﲈ أن اﳌﻤ ّﻴﺰ ﺳﺎﻟﺐ ﲤﺎﻣﺎ ّ‬
‫ﻓﺈن 𝜈 =‬
‫‪2‬‬ ‫‪4‬‬
‫𝛼‬
‫‪P(X) = 𝜇 + 0‬‬ ‫‪A +0‬‬ ‫‪X+‬‬ ‫𝜈‪+‬‬
‫‪2‬‬
‫‪ƕ‬‬

‫=‬ ‫=‬

‫أي ﻋﺪد ﻣﻦ‬ ‫ﻼ ﻣﻦ اﻟﺸﻜﻞ ‪ ، U + V‬و ﻣﻦ اﻟﺴﻬﻞ اﻹﺛﺒﺎت )ﺑﺎﻟﱰاﺟﻊ( ّ‬


‫أن ﺟﺪاء ّ‬ ‫ﻫﺬا اﻟﺘﺤﻠﻴﻞ ﳛﺘﻮي ﻋﲆ ‪ 1 + m + p‬ﻋﺎﻣ ً‬
‫اﻷﻋﺪاد )أو ﻛﺜﲑات اﳊﺪود أو ‪ (...‬اﻟﺘﻲ ﺗُﻜﺘﺐ ﻋﲆ ﺷﻜﻞ ﳎﻤﻮع ﻣﺮﺑﻌﲔ‪ ،‬ﻫﻮ ﻋﺪد )أو ﻛﺜﲑ ﺣﺪود أو ‪ُ (...‬ﻳﻜﺘﺐ ﻋﲆ ﺷﻜﻞ‬
‫■‬ ‫ﳎﻤﻮع ﻣﺮﺑﻌﲔ‪.‬‬
‫‪ũŏ‬‬

‫اﳌﻌﺮف ﺑﺎﻟﺼﻴﻐﺔ اﳌﺮﻛﺒﺔ ‪:‬‬


‫ﻋﲔ ﰲ ﻛﻞ ﺣﺎﻟﺔ ﻃﺒﻴﻌﺔ و ﻋﻨﺎﴏ اﻟﺘﺤﻮﻳﻞ ّ‬
‫ﱢ‬
‫‬ ‫‪.‬‬
‫𝟖𝟑𝟏‬ ‫‪.‬‬

‫𝟲𝟬𝟮‬

‫‪http ://tinyurl.com/Malki1718‬‬ ‫‪0‬‬


‫‪ .VI‬ﲤﺎرﻳﻦ ﺗﻄﺒﻴﻘﻴﺔ‬
‫‪.‬‬

‫‪f : z ↦ ız + 1‬‬ ‫•‬ ‫‪f : z ↦ (1 + ı) z + 2 − ı‬‬ ‫•‬

‫‪Ŕž Ŧ Ľ‬‬
‫‪f : z↦z+ı−1‬‬ ‫•‬ ‫‪f : z ↦ (−3 + 4ı) z + 12 + 6ı‬‬ ‫•‬

‫‪œǃ‬‬
‫ﺍﳊﻞّ‪ .‬ﻛﻞ ﻫﺬه اﻟﺘﺤﻮﻳﻼت ﻣﻦ اﻟﺸﻜﻞ ‪ z ↦ az + b‬ﺣﻴﺚ ‪. a, b ∈ ℂ‬‬
‫𝜋‬
‫‪ f‬ﺗﺸﺎﺑﻪ ّ‬

‫‪Ŀ‬‬
‫ﻷن ‪. a = 1 + ı = √2,‬‬ ‫•‬
‫‪4‬‬
‫‪2−ı‬‬
‫= 𝜔 ﻣﻨﻪ‬ ‫إذا ﻛﺎﻧﺖ 𝜔 ﻻﺣﻘﺔ ‪ Ω‬اﻟﻨﻘﻄﺔ اﻟﺼﺎﻣﺪة اﻟﻮﺣﻴﺪة ّ‬
‫ﻓﺈن ‪ 𝜔 = (1 + ı) 𝜔 + 2 − ı‬أي ‪= 1 + 2ı‬‬
‫)‪1 − (1 + ı‬‬
‫)‪. Ω (1, 2‬‬
‫𝜋‬ ‫𝜋‬
‫‪ f‬ﻫﻮ اﻟﺘﺸﺎﺑﻪ ‪) 𝒮 (1, 2) , √2,‬و ﻫﻮ ﺗﺮﻛﻴﺐ �ﺗﺒﺪﻳﲇ� ﻟﻠﺘﺤﺎﻛﻲ ‪ ℋ (1, 2) , √2‬ﻣﻊ اﻟﺪوران ‪(ℛ (1, 2) ,‬‬ ‫إذن‬
‫‪4‬‬ ‫‪4‬‬
‫‪.‬‬
‫‪4‬‬ ‫‪3‬‬
‫= 𝜃 ‪sin‬‬ ‫= |‪ |a‬و إذا ﻛﺎﻧﺖ 𝜃 ﻋﻤﺪ ًة ﻟﻠﻌﺪد ‪ّ a‬‬
‫ﻓﺈن ‪ cos 𝜃 = −‬و‬ ‫ﻟﺪﻳﻨﺎ ‪ a = −3 + 4ı‬ﻣﻊ ‪(−3) + 4 = 5‬‬ ‫•‬
‫‪5‬‬ ‫‪5‬‬
‫)اﻟﺰاوﻳﺔ 𝜃 ﻟﻴﺴﺖ ﺷﻬﲑة( ‪ .‬إذن ‪ f‬ﺗﺸﺎﺑﻪ‪.‬‬
‫‪3 9‬‬ ‫‪12 + 6ı‬‬ ‫‪3 9‬‬
‫‪.Ω‬‬ ‫‪,‬‬ ‫= 𝜔 أي‬ ‫إذا ﻛﺎﻧﺖ 𝜔 ﻻﺣﻘﺔ ‪ Ω‬اﻟﻨﻘﻄﺔ اﻟﺼﺎﻣﺪة اﻟﻮﺣﻴﺪة ّ‬
‫ﻓﺈن ‪= + ı‬‬
‫‪4 4‬‬ ‫‪1 − (−3 + 4ı) 4 4‬‬
‫‪4‬‬ ‫‪3‬‬ ‫‪3 9‬‬
‫= 𝜃 ‪) sin‬و ﻫﻮ ﺗﺮﻛﻴﺐ �ﺗﺒﺪﻳﲇ� ﻟﻠﺘﺤﺎﻛﻲ‬ ‫𝒮 ﺣﻴﺚ ‪ cos 𝜃 = −‬و‬ ‫‪,‬‬ ‫إذن ‪ f‬ﻫﻮ اﻟﺘﺸﺎﺑﻪ 𝜃 ‪, 5,‬‬
‫‪5‬‬ ‫‪5‬‬ ‫‪4 4‬‬
‫)‪ ℋ ((1, 2) , 5‬ﻣﻊ اﻟﺪوران )𝜃 ‪. (ℛ ((1, 2) ,‬‬
‫𝜋‬
‫‪. a = ı = 1,‬‬ ‫‪ f‬دوران ّ‬
‫ﻷن‬ ‫•‬
‫‪2‬‬
‫‪1 1‬‬ ‫‪1‬‬ ‫‪1 1‬‬
‫‪.Ω‬‬ ‫‪,‬‬ ‫= 𝜔 أي اﻟﻨﻘﻄﺔ‬ ‫ﻣﺮﻛﺰ ﻫﺬا اﻟﺪوران ﻫﻮ اﻟﻨﻘﻄﺔ ‪ Ω‬اﻟﺘﻲ ﻻﺣﻘﺘﻬﺎ 𝜔 ﺑﺤﻴﺚ ‪= + ı‬‬
‫‪2 2‬‬ ‫‪1−ı 2 2‬‬
‫‪1 1‬‬ ‫𝜋‬
‫‪.ℛ‬‬ ‫‪,‬‬ ‫‪,‬‬ ‫إذن ‪ f‬ﻫﻮ اﻟﺪوران‬
‫‪ņőƱŀ Ţ‬‬
‫‪2 2‬‬ ‫‪2‬‬

‫‪ f‬إﻧﺴﺤﺎب ّ‬
‫ﻷن ‪ . a = 1‬ﻻﺣﻘﺔ ﺷﻌﺎع ﻫﺬا اﻹﻧﺴﺤﺎب ﻫﻲ ‪. b = ı − 1‬‬ ‫•‬

‫إذن ‪ f‬ﻫﻮ اﻹﻧﺴﺤﺎب ⃗ 𝒯 اﻟﺬي ﻻﺣﻘﺔ ﺷﻌﺎﻋﻪ ⃗‪ V‬ﻫﻲ ‪. −1 + ı‬‬


‫‪V‬‬
‫‪VI‬‬

‫■‬

‫ﺣﺪﱢ د ﻃﺒﻴﻌﺔ اﻟﺘﺤﻮﻳﻼت اﻵﺗﻴﺔ و ﻋﻨﺎﴏﻫﺎ اﳌﻤﻴﺰة ‪:‬‬


‫‬ ‫‪.‬‬
‫𝟗𝟑𝟏‬ ‫‪.‬‬
‫‪ƕ‬‬

‫‪z = ız + 1‬‬ ‫•‬ ‫‪3‬‬ ‫‪z =z+3−ı‬‬ ‫‪1‬‬


‫•‬

‫‪z = (1 − ı) z + 2 + ı‬‬ ‫•‬ ‫‪4‬‬ ‫‪z = 2z + 3‬‬ ‫‪2‬‬


‫•‬
‫‪ũŏ‬‬

‫ﺍﳊﻞّ‪ .‬ﻟﻴﻜﻦ ‪ f‬اﻟﺘﺤﻮﻳﻞ اﻟﻨﻘﻄﻲ ا ُﳌﻌﺘ ََﱪ‪ .‬ﻋﺒﺎرﺗﻪ ﻣﻦ اﻟﺸﻜﻞ ‪. f (z) = az + b‬‬
‫‪3‬‬
‫= ⃗‪. u‬‬ ‫أن ‪ّ a = 1‬‬
‫ﻓﺈن ‪ f‬ﻫﻮ اﻹﻧﺴﺤﺎب اﻟﺬي ﻻﺣﻘﺔ ﺷﻌﺎﻋﻪ ﻫﻲ ‪ 3 − ı‬أي اﻹﻧﺴﺤﺎب اﻟﺬي ﺷﻌﺎﻋﻪ‬ ‫‪ •1‬ﺑﲈ ّ‬
‫‪−1‬‬

‫𝟳𝟬𝟮‬

‫‪http ://tinyurl.com/Malki1718‬‬ ‫‪0‬‬


‫‪ .3.VI‬ﺗﻄﺒﻴﻘﺎت اﻷﻋﺪاد اﳌﺮﻛﺒﺔ‬
‫‪.‬‬
‫ﻷن ∗‪ . a = 2 ∈ ℝ‬إذا ﻛﺎﻧﺖ 𝜔 ﻻﺣﻘﺔ ‪ Ω‬ﻣﺮﻛﺰ ﻫﺬا اﻟﺘﺤﺎﻛﻲ ّ‬
‫ﻓﺈن ‪ 𝜔 = 2𝜔 + 3 :‬أي ‪. 𝜔 = −3‬‬ ‫‪ f •2‬ﲢﺎﻛﻲ ّ‬
‫إذن ‪ f‬ﻫﻮ اﻟﺘﺤﺎﻛﻲ )‪ ℋ ((−3, 0) , 2‬اﻟﺬي ﻣﺮﻛﺰه )‪ Ω (−3, 0‬و ﻧﺴﺒﺘﻪ ‪. 2‬‬

‫‪Ŕž Ŧ Ľ‬‬
‫𝜋‬
‫‪. a = ı = 1,‬‬ ‫‪ f •3‬دوران ّ‬
‫ﻷن‬
‫‪2‬‬

‫‪œǃ‬‬
‫‪1‬‬ ‫‪1‬‬
‫= 𝜔‪.‬‬ ‫ﻓﺈن ﻻﺣﻘﺘﻪ 𝜔 ُﲢﻘﻖ ‪ 𝜔 = ı𝜔 + 1 :‬أي )‪= (1 + ı‬‬
‫إذا ﻛﺎن ‪ Ω‬ﻣﺮﻛﺰ ﻫﺬا اﻟﺪوران ّ‬
‫‪1−ı 2‬‬
‫𝜋‬ ‫‪1 1‬‬ ‫‪1 1‬‬ ‫𝜋‬
‫‪ Ω ,‬و زاوﻳﺘﻪ ‪.‬‬ ‫‪ ℛ‬اﻟﺬي ﻣﺮﻛﺰه‬ ‫‪,‬‬ ‫‪,‬‬ ‫إذن ‪ f‬ﻫﻮ اﻟﺪوران‬

‫‪Ŀ‬‬
‫‪2‬‬ ‫‪2 2‬‬ ‫‪2 2‬‬ ‫‪2‬‬
‫𝜋‬
‫‪ f •4‬ﺗﺸﺎﺑﻪ ّ‬
‫ﻷن ‪. a = 1 − ı = √2, −‬‬
‫‪4‬‬
‫‪2+ı‬‬
‫= 𝜔‪.‬‬ ‫ﻓﺈن ﻻﺣﻘﺘﻪ 𝜔 ُﲢﻘﻖ ‪ 𝜔 = (1 − ı) 𝜔 + 2 + ı :‬أي ‪= 1 − 2ı‬‬
‫إذا ﻛﺎن ‪ Ω‬ﻣﺮﻛﺰ ﻫﺬا اﻟﺘﺸﺎﺑﻪ ّ‬
‫‪ı‬‬
‫𝜋‬ ‫𝜋‬
‫إذن ‪ f‬ﻫﻮ اﻟﺘﺸﺎﺑﻪ ‪ 𝒮 (1, −2) , √2, −‬اﻟﺬي ﻣﺮﻛﺰه )‪ ، Ω (1, −2‬ﻧﺴﺒﺘﻪ ‪ √2‬و زاوﻳﺘﻪ ‪. −‬‬
‫‪4‬‬ ‫‪4‬‬
‫■‬

‫‬ ‫‪.‬‬
‫𝟎𝟒𝟏‬ ‫‪.‬‬

‫ﻟﺘﻜﻦ ‪ E‬ﳎﻤﻮﻋﺔ اﻷﻋﺪاد اﳌﺮﻛﺒﺔ ‪ z‬اﻟﺘﻲ ﲢﻘﻖ ‪ ||z − 1|| = 1‬و ‪ f‬اﻟﺪاﻟﺔ اﳌﻌﺮﻓﺔ ﻋﲆ ‪ ℂ‬ﺑﺎﻟﻌﺒﺎرة ‪. f(z) = ız‬‬
‫‪ •1‬ﺣﺪّ د اﳌﺠﻤﻮﻋﺔ ‪ E‬و أﻧﺸﺊ ﲤﺜﻴﻠﻬﺎ اﻟﺒﻴﺎﲏ 𝒞 ﰲ اﳌﺴﺘﻮي اﳌﺮﻛﺐ‪.‬‬
‫‪ •2‬ﻣﺎ ﻫﻮ اﻟﺘﺤﻮﻳﻞ اﳍﻨﺪﳼ ‪ T‬اﳌﺮﻓﻖ ﺑﺎﻟﺪاﻟﺔ ‪ f‬؟‬
‫‪ •3‬ﻣﺎ ﻫﻲ ﺻﻮرة 𝒞 ﺑﺎﻟﺘﺤﻮﻳﻞ ‪ T‬؟‬
‫‪ •4‬إﺳﺘﻨﺘﺞ ّ‬
‫أن ﺻﻮرة ‪ E‬ﺑﺎﻟﺪاﻟﺔ ‪ f‬ﻫﻲ اﳌﺠﻤﻮﻋﺔ ‪ E‬ﻟﻸﻋﺪاد اﳌﺮﻛﺒﺔ ‪ z‬ﺑﺤﻴﺚ ‪. ||z − ı|| = 1‬‬
‫‪ņőƱŀ Ţ‬‬
‫‪ •1‬ﻟﺘﻜﻦ )‪ M (z‬و )‪ . A (1‬ﻟﺪﻳﻨﺎ ‪:‬‬ ‫ﺍﳊﻞّ‪.‬‬
‫‪M ∈ E ⟺ AM = 1‬‬

‫‪VI‬‬
‫𝒞 اﻟﺘﻲ ﻣﺮﻛﺰﻫﺎ ‪ A‬و ﻧﺼﻒ ﻗﻄﺮﻫﺎ ‪1‬‬ ‫إذن ‪ E‬ﻫﻲ ﳎﻤﻮﻋﺔ اﻟﻨﻘﻂ ‪ M‬اﻟﺘﻲ ﺗﺒﻌﺪ ﺑﻤﺴﺎﻓﺔ ‪ 1‬ﻋﻦ اﻟﻨﻘﻄﺔ ‪ A‬أي ﻫﻲ اﻟﺪاﺋﺮة‬
‫)ﺷﻜﻞ ‪.(16.VI‬‬
‫𝜋‬
‫‪ a = ı = 1,‬و ﺑﺎﻟﺘﺎﱄ ‪ T‬دوران‪ .‬ﻻﺣﻘﺔ ﻣﺮﻛﺰ ﻫﺬا اﻟﺪوران‬ ‫‪ •2‬ﻋﺒﺎرة اﻟﺘﺤﻮﻳﻞ ‪ T‬ﻣﻦ اﻟﺸﻜﻞ ‪ f (z) = az + b‬ﺣﻴﺚ‬
‫‪2‬‬
‫𝜔 ُﲢﻘﻖ ‪ 𝜔 = ı𝜔 :‬أي ‪. 𝜔 = 0‬‬
‫‪ƕ‬‬

‫𝜋‬ ‫𝜋‬
‫‪ ℛ O,‬اﻟﺬي ﻣﺮﻛﺰه )‪ O (0, 0‬و زاوﻳﺘﻪ ‪.‬‬ ‫إذن ‪ T‬ﻫﻮ اﻟﺪوران‬
‫‪2‬‬ ‫‪2‬‬
‫‪ T‬ﻫﻲ اﻟﺪاﺋﺮة اﻟﺘﻲ ﻣﺮﻛﺰﻫﺎ )‪ B = T (A‬و ﻧﺼﻒ ﻗﻄﺮﻫﺎ ‪ . 1‬ﻟﻜﻦ ‪ f (1) = ı‬إذن )‪T (A‬‬ ‫‪ •3‬ﺻﻮرة اﻟﺪاﺋﺮة 𝒞 ﺑﺎﻟﺪوران‬
‫‪ũŏ‬‬

‫ﻫﻲ اﻟﻨﻘﻄﺔ )‪. B (ı‬‬


‫ﰲ اﻷﺧﲑ‪ ،‬ﺻﻮرة 𝒞 ﺑﺎﻟﺪوران ‪ T‬ﻫﻲ اﻟﺪاﺋﺮة 𝒞 اﻟﺘﻲ ﻣﺮﻛﺰﻫﺎ )‪ B (ı‬و ﻧﺼﻒ ﻗﻄﺮﻫﺎ ‪. 1‬‬
‫‪ •4‬اﻟﺪاﺋﺮة 𝒞 ﻫﻲ ﳎﻤﻮﻋﺔ اﻟﻨﻘﻂ ‪ M z‬اﻟﺘﻲ ﺗﺒﻌﺪ ﺑﻤﺴﺎﻓﺔ ‪ 1‬ﻋﻦ اﻟﻨﻘﻄﺔ )‪ B (ı‬و ﻫﺬا ﻳﻌﻨﻲ ّ‬
‫أن اﳌﺠﻤﻮﻋﺔ ‪ E‬ﻫﻲ ﳎﻤﻮﻋﺔ‬
‫اﻷﻋﺪاد اﳌﺮﻛﺒﺔ ‪ z‬ﺑﺤﻴﺚ ‪. ||z − ı|| = 1 :‬‬
‫■‬

‫𝟴𝟬𝟮‬

‫‪http ://tinyurl.com/Malki1718‬‬ ‫‪0‬‬


‫‪ .VI‬ﲤﺎرﻳﻦ ﺗﻄﺒﻴﻘﻴﺔ‬
‫‪.‬‬

‫‪Ŕž Ŧ Ľ‬‬
‫‪2‬‬
‫‪C′‬‬

‫‪œǃ‬‬
‫‪B‬‬
‫‪1‬‬
‫‪C‬‬

‫‪Ŀ‬‬
‫‪A‬‬
‫‪−1‬‬ ‫‪O‬‬ ‫‪1‬‬ ‫‪2‬‬

‫‪−1‬‬

‫ﺷﻜﻞ ‪16.VI‬‬

‫‬ ‫‪.‬‬
‫𝟏𝟒𝟏‬ ‫‪.‬‬

‫‪ •1‬ﺣﻞ ﰲ ‪ ℂ‬اﳌﻌﺎدﻟﺔ ‪ z + 2√3z + 4 = 0‬و اﻛﺘﺐ ﺣﻠﻮﳍﺎ ﻋﲆ اﻟﺸﻜﻞ اﻷﳼ‪.‬‬


‫اﳌﺴﺘﻮي ﻣﻨﺴﻮب إﱃ ﻣﻌﻠﻢ ﻣﺘﻌﺎﻣﺪ ﻣﺘﺠﺎﻧﺲ و ﻣﺒﺎﴍ ⃗ ‪ . O, i,⃗ j‬ﻟﻴﻜﻦ ‪ T‬اﻟﺘﺤﻮﻳﻞ اﻟﻨﻘﻄﻲ ﻟﻠﻤﺴﺘﻮي اﳌﺮﻛﺐ و اﻟﺬي‬
‫‪ņőƱŀ Ţ‬‬
‫ﻳﺮﻓﻖ ﺑﻜﻞ ﻧﻘﻄﺔ ‪ M‬ذات اﻟﻼﺣﻘﺔ ‪ z‬اﻟﻨﻘﻄﺔ ‪ M‬ذات اﻟﻼﺣﻘﺔ ‪ z‬ﺑﺤﻴﺚ ‪. z = e / z :‬‬
‫‪ •2‬ﻣﺎ ﻫﻲ ﻃﺒﻴﻌﺔ اﻟﺘﺤﻮﻳﻞ ‪ T‬؟‬
‫‪ •3‬ﻟﺘﻜﻦ ‪ M‬اﻟﻨﻘﻄﺔ اﻟﺘﻲ ﻻﺣﻘﺘﻬﺎ ‪. z = −√3 + ı‬‬
‫‪VI‬‬

‫ﺣﺪّ د ‪ z‬و ‪ z‬ﻻﺣﻘﺘ َْﻲ اﻟﻨﻘﻄﺘﲔ ‪ M‬و ‪ M‬ﺑﺤﻴﺚ ) ‪ M = T (M‬و ) ‪. M = T (M‬‬


‫‪ •4‬أﻧﺸﺊ اﻟﻨﻘﻂ ‪ M ، M‬و ‪. M‬‬
‫‪z −z‬‬
‫ﺛﻢ اﺳﺘﻨﺘﺞ ﻃﺒﻴﻌﺔ اﳌﺜﻠﺚ ‪. M M M‬‬ ‫‪ •5‬أﺣﺴﺐ‬
‫‪ƕ‬‬

‫ّ‬ ‫‪z −z‬‬

‫‪ •1‬اﳌﻤ ﱢﻴﺰ اﳌﺨﺘﴫ ﻫﻮ ‪ Δ = √3 − 4 = −1 = ı‬و ﺑﺎﻟﺘﺎﱄ ﻟﻠﻤﻌﺎدﻟﺔ ﺣﻼّن ﳘﺎ ‪ −√3 − ı‬و ‪. −√3 + ı‬‬ ‫ﺍﳊﻞّ‪.‬‬
‫‪ũŏ‬‬

‫اﻟﺸﻜﻞ اﻷﳼ ﳍﺬﻳﻦ اﳊ ّﻠﲔ ﻫﻮ ‪:‬‬

‫‪√3 1‬‬ ‫𝜋‪7‬‬ ‫𝜋‪7‬‬ ‫‪/‬‬


‫‪−√3 − ı = 2 −‬‬ ‫‪− ı‬‬ ‫‪= 2 cos‬‬ ‫‪+ ı sin‬‬ ‫‪= 2e‬‬
‫‪2‬‬ ‫‪2‬‬ ‫‪6‬‬ ‫‪6‬‬

‫‪−√3 + ı = −√3 − ı = 2e‬‬ ‫‪/‬‬ ‫‪= 2e−‬‬ ‫‪/‬‬ ‫‪= 2e−‬‬ ‫‪/ +‬‬ ‫‪= 2e‬‬ ‫‪/‬‬

‫𝟵𝟬𝟮‬

‫‪http ://tinyurl.com/Malki1718‬‬ ‫‪0‬‬


‫‪ .3.VI‬ﺗﻄﺒﻴﻘﺎت اﻷﻋﺪاد اﳌﺮﻛﺒﺔ‬
‫‪.‬‬
‫‪M1‬‬ ‫‪M3‬‬
‫‪1‬‬

‫‪Ŕž Ŧ Ľ‬‬
‫‪+ π3‬‬

‫‪œǃ‬‬
‫‪−2‬‬ ‫‪−1‬‬ ‫‪1‬‬ ‫‪2‬‬

‫‪−1‬‬

‫‪Ŀ‬‬
‫‪−2‬‬ ‫‪M2‬‬

‫ﺷﻜﻞ ‪17.VI‬‬

‫‪/‬‬ ‫𝜋‪2‬‬
‫‪ ، a = e‬إذن ‪ T‬دوران‪.‬‬ ‫‪= 1,‬‬ ‫‪ •2‬ﻋﺒﺎرة اﻟﺘﺤﻮﻳﻞ ‪ T‬ﻣﻦ اﻟﺸﻜﻞ ‪ z = az + b‬ﻣﻊ‬
‫‪3‬‬
‫‪ 𝜔 = e‬أي ‪. 𝜔 = 0‬‬ ‫‪/‬‬ ‫ﻻﺣﻘﺔ ﻣﺮﻛﺰ ﻫﺬا اﻟﺪوران 𝜔 ُﲢﻘﻖ ‪𝜔 :‬‬
‫𝜋‪2‬‬ ‫𝜋‪2‬‬
‫‪.‬‬ ‫‪ ℛ O,‬اﻟﺬي ﻣﺮﻛﺰه )‪ O (0, 0‬و زاوﻳﺘﻪ‬ ‫إذن ‪ T‬ﻫﻮ اﻟﺪوران‬
‫‪3‬‬ ‫‪3‬‬

‫‪ •3‬ﻟﺪﻳﻨﺎ ‪:‬‬

‫‪/‬‬ ‫‪1‬‬ ‫‪√3‬‬


‫‪z =e‬‬ ‫= ‪z‬‬ ‫‪− +ı‬‬ ‫‪−√3 + ı = −2ı‬‬
‫‪2‬‬ ‫‪2‬‬

‫‪/‬‬ ‫‪1‬‬ ‫‪√3‬‬


‫‪z =e‬‬ ‫= ‪z‬‬ ‫‪− +ı‬‬ ‫‪(−2ı) = √3 + ı‬‬
‫‪2‬‬ ‫‪2‬‬
‫‪ņőƱŀ Ţ‬‬
‫‪ •4‬أﻧﻈﺮ اﻟﺸﻜﻞ ‪. 17.VI‬‬
‫‪ •5‬ﻟﺪﻳﻨﺎ ‪:‬‬

‫‪VI‬‬
‫‪z −z‬‬ ‫‪−2ı − √3 − ı‬‬ ‫‪1‬‬ ‫‪√3‬‬ ‫‪/‬‬
‫=‬ ‫‪= +ı‬‬ ‫‪=e‬‬
‫‪z −z‬‬ ‫‪−√3 + ı − √3 − ı 2‬‬ ‫‪2‬‬

‫أن ‪ّ ||e / || = 1‬‬


‫ﻓﺈن || ‪ ||z − z || = ||z − z‬أي ‪ M M = M M‬إذن اﳌﺜﻠﺚ ‪ M M M‬ﻣﺘﺴﺎوي‬ ‫و ﺑﲈ ّ‬
‫اﻟﺴﺎﻗﲔ ﻋﻨﺪ اﻟﺮأس ‪. M‬‬
‫‪ƕ‬‬

‫‪z −z‬‬ ‫𝜋‬


‫‪ M M , M M‬و ﺑﺎﻟﺘﺎﱄ ﻓﺎﳌﺜﻠﺚ ‪ M M M‬ﻣﺘﻘﺎﻳﺲ‬ ‫‪= arg‬‬ ‫=‬ ‫)𝜋‪(mod 2‬‬ ‫ﻟﻜﻦ‬
‫‪z −z‬‬ ‫‪3‬‬
‫اﻷﺿﻼع )و ﻣﺒﺎﴍ( ‪.‬‬
‫■‬
‫‪ũŏ‬‬

‫‪.‬‬
‫𝟐𝟒𝟏‬ ‫‪.‬‬
‫ﻣﺎ ﻫﻮ اﻟﴩط اﻟﺬي ﳚﺐ أن ﳛﻘﻘﻪ اﻟﻮﺳﻴﻂ ‪ a‬ﺣﺘﻰ ﻳﻜﻮن ﻟﻠﻤﻌﺎدﻟﺔ ‪ z + z = a‬ﺣﻼ ﺣﻘﻴﻘﻴﺎ ؟ ﰲ ﻫﺬه‬
‫اﳊﺎﻟﺔ )أي إذا ﲢﻘﻖ ﻫﺬا اﻟﴩط( أوﺟﺪ ﺣﻠﻮل ﻫﺬه اﳌﻌﺎدﻟﺔ‪.‬‬

‫𝟬𝟭𝟮‬

‫‪http ://tinyurl.com/Malki1718‬‬ ‫‪0‬‬


‫‪ .VI‬ﲤﺎرﻳﻦ ﺗﻄﺒﻴﻘﻴﺔ‬
‫‪.‬‬
‫ﺍﳊﻞّ‪ .‬ﻟﻴﻜﻦ ‪ x‬ﻋﺪد ًا ﺣﻘﻴﻘﻴﺎ‪ x .‬ﺣﻞ ﻟﻠﻤﻌﺎدﻟﺔ اﳌﻌﻄﺎة إذا و ﻓﻘﻂ إذا ﻛﺎن ‪ x + x − a = 0‬أي إذا و ﻓﻘﻂ إذا ﻛﺎن ‪ a ∈ ℝ‬و‬
‫‪1‬‬

‫‪Ŕž Ŧ Ľ‬‬
‫‪ .Δ = 1 − 4 × (−a) ≥ 0‬إذن اﻟﴩط اﳌﻄﻠﻮب ﻫﻮ أن ﻳﻜﻮن ‪ a‬ﻋﺪدا ﺣﻘﻴﻘﻴﺎ ﺑﺤﻴﺚ ‪. a ≥ −‬‬
‫‪4‬‬
‫‪1‬‬

‫‪œǃ‬‬
‫ﻧﻔﺮض إذن ّ‬
‫أن ‪ a‬ﻋﺪد ﺣﻘﻴﻘﻲ ﺑﺤﻴﺚ ‪. a ≥ −‬‬
‫‪4‬‬

‫ﻟﻴﻜﻦ ‪ z = x + ıy‬ﺣﻼ ﻟﻠﻤﻌﺎدﻟﺔ اﻟﺴﺎﺑﻘﺔ )ﻣﻊ ‪ . (x, y ∈ ℝ‬ﻟﺪﻳﻨﺎ ‪:‬‬


‫)‪z + z = x + x − y + ı (y − 2xy‬‬ ‫و‬ ‫‪z = x − y + 2ıxy‬‬

‫‪Ŀ‬‬
‫ﻣﻨﻪ ‪:‬‬
‫‪z + z = a ⟺ x + x − y + ı (y − 2xy) = a + 0ı‬‬
‫‪x+x −y =a‬‬ ‫‪x+x −y =a‬‬
‫⟺‬ ‫⟺‬
‫‪y − 2xy = 0‬‬ ‫‪y (1 − 2x) = 0‬‬

‫⎧‬ ‫‪y=0‬‬‫‪ x+x −y =a‬و‬


‫⎪‬
‫⟺‬ ‫أو‬
‫⎨‬
‫⎪‬
‫‪ x + x − y = a‬و ‪⎩ 1 − 2x = 0‬‬

‫⎧‬ ‫‪y=0‬‬ ‫و‬ ‫‪x+x −a=0‬‬


‫⎪‬
‫⎪‬
‫⎪‬
‫⎪‬ ‫أو‬
‫⟺‬
‫⎨‬
‫⎪‬ ‫‪1‬‬ ‫‪1‬‬ ‫‪1‬‬
‫⎪‬
‫⎪‬
‫=‪⎪ x‬‬ ‫و‬ ‫‪+‬‬ ‫‪−y =a‬‬
‫⎩‬ ‫‪2‬‬ ‫‪2‬‬ ‫‪2‬‬

‫⎧‬
‫⎪‬ ‫‪y=0‬‬ ‫‪ x+x −a=0‬و‬
‫⎪‬
‫⎪‬
‫⟺‬ ‫أو‬
‫‪ņőƱŀ Ţ‬‬
‫⎨‬
‫⎪‬
‫‪⎪ x= 1‬‬
‫⎪‬
‫و‬ ‫= ‪y‬‬
‫‪3‬‬
‫‪−a‬‬
‫⎩‬ ‫‪2‬‬ ‫‪4‬‬
‫‪3‬‬ ‫‪3‬‬
‫اﳊﺎﻟﺔ اﻷوﱃ ‪ :‬إذا ﻛﺎن > ‪ّ a‬‬
‫ﻓﺈن اﳌﻌﺎدﻟﺔ ‪ y = − a‬ﻣﺴﺘﺤﻴﻠﺔ و ﺑﺎﻟﺘﺎﱄ ﻳﺒﻘﻰ ‪ y = 0‬و ‪ x + x − a = 0‬؛ ﰲ ﻫﺬه اﳊﺎﻟﺔ‬
‫‪VI‬‬

‫‪4‬‬ ‫‪4‬‬
‫‪−1 ± √1 + 4a‬‬
‫= ‪.z‬‬ ‫ﻟﻠﻤﻌﺎدﻟﺔ ﺣﻼن ﺣﻘﻴﻘﻴﺎن ﳘﺎ‬
‫‪2‬‬
‫‪3‬‬
‫اﳊﺎﻟﺔ اﻟﺜﺎﻧﻴﺔ ‪ :‬إذا ﻛﺎن ‪ّ a = 3‬‬
‫ﻓﺈن اﳌﻌﺎدﻟﺔ ‪ y = − a‬ﺗﺼﺒﺢ ‪ y = 0‬ﻣﻨﻪ ‪:‬‬
‫‪4‬‬ ‫‪4‬‬
‫‪3‬‬ ‫‪1‬‬
‫‪y=0‬‬ ‫و‬ ‫‪x +x− =0‬‬ ‫أو‬ ‫=‪x‬‬ ‫‪ y=0‬و‬
‫‪ƕ‬‬

‫‪4‬‬ ‫‪2‬‬
‫‪1‬‬ ‫‪3‬‬
‫ﰲ ﻫﺬه اﳊﺎﻟﺔ ﻟﻠﻤﻌﺎدﻟﺔ ﺣﻼن ﺣﻘﻴﻘﻴﺎن ‪ z = − :‬و = ‪. z‬‬
‫‪2‬‬ ‫‪2‬‬
‫‪−1 ± √1 + 4a‬‬ ‫‪1‬‬ ‫‪3‬‬
‫‪ũŏ‬‬

‫= ‪ z‬و ﺣﻼن‬ ‫<‪ّ − ≤a‬‬


‫ﻓﺈن اﳌﻌﺎدﻟﺔ ﺗﻘﺒﻞ أرﺑﻌﺔ ﺣﻠﻮل ‪ :‬ﺣﻼن ﺣﻘﻴﻘﻴﺎن‬ ‫اﳊﺎﻟﺔ اﻟﺜﺎﻟﺜﺔ ‪ :‬إذا ﻛﺎن‬
‫‪2‬‬ ‫‪4‬‬ ‫‪4‬‬
‫‪1‬‬ ‫‪3‬‬
‫■‬ ‫‪.z = ±ı‬‬ ‫آﺧﺮان ﻏﲑ ﺣﻘﻴﻘﻴﲔ ‪− a‬‬
‫‪2‬‬ ‫‪4‬‬

‫𝟭𝟭𝟮‬

‫‪http ://tinyurl.com/Malki1718‬‬ ‫‪0‬‬


‫‪ .3.VI‬ﺗﻄﺒﻴﻘﺎت اﻷﻋﺪاد اﳌﺮﻛﺒﺔ‬
‫‪.‬‬

‫‬ ‫‪.‬‬
‫𝟑𝟒𝟏‬ ‫‪.‬‬

‫‪Ŕž Ŧ Ľ‬‬
‫‪ .‬ﻧﻌﺘﱪ اﻟﺘﺤﻮﻳﻞ اﻟﻨﻘﻄﻲ ‪ f‬ﻟﻠﻤﺴﺘﻮي اﳌﺮﻛﺐ و‬ ‫⃗ ‪O, i,⃗ j‬‬ ‫اﳌﺴﺘﻮي ﻣﻨﺴﻮب إﱃ ﻣﻌﻠﻢ ﻣﺘﻌﺎﻣﺪ و ﻣﺘﺠﺎﻧﺲ‬
‫‪1‬‬ ‫‪3‬‬
‫√‪. z = − + ı‬‬

‫‪œǃ‬‬
‫‪z‬‬ ‫اﻟﺬي ﻳﺮﻓﻖ ﺑﺎﻟﻨﻘﻄﺔ ‪ M‬ذات اﻟﻼﺣﻘﺔ ‪ z‬اﻟﻨﻘﻄﺔ ‪ M‬ذات اﻟﻼﺣﻘﺔ ‪ z‬ﺑﺤﻴﺚ‬
‫‪2‬‬ ‫‪2‬‬
‫‪ z = 3e−‬ﻋﲆ اﻟﱰﺗﻴﺐ ‪.‬‬ ‫‪/‬‬ ‫ﻟﺘﻜﻦ ‪ A‬و ‪ B‬اﻟﻨﻘﻄﺘﲔ ا ّﻟﻠﺘﲔ ﻻﺣﻘﺘﺎﳘﺎ ‪ z = 3e /‬و‬
‫ﻧﻀﻊ )‪ A = f (A‬و )‪ B = f (B‬ﺻﻮرﺗﻴﻬﲈ ﺑﺎﻟﺘﺤﻮﻳﻞ ‪. f‬‬

‫‪Ŀ‬‬
‫‪ •1‬أﺛﺒﺖ ّ‬
‫أن ‪ f‬دوران ُﻳﻄﻠﺐ ﺗﻌﻴﲔ ﻣﺮﻛﺰه و زاوﻳﺘﻪ‪.‬‬
‫‪ •2‬ﺣﺪّ د اﻟﻌﺪدﻳﻦ ‪ z ′‬و ‪ z ′‬ﻻﺣﻘﺘ َْﻲ ‪ A‬و ‪ B‬و ﺿﻌﻬﲈ ﻋﲆ اﻟﺸﻜﻞ اﻷﳼ‪.‬‬

‫‪ •3‬أﺛﺒﺖ ّ‬
‫أن اﻟﻨﻘﻂ ‪ A ، B ، A‬و ‪ B‬ﺗﻘﻊ ﻋﲆ داﺋﺮة ‪ Γ‬ﻣﺮﻛﺰﻫﺎ ‪ O‬و ﻧﺼﻒ ﻗﻄﺮﻫﺎ ‪. 3‬‬
‫‪z‬‬ ‫‪′‬‬
‫ﺛﻢ اﺳﺘﻨﺘﺞ ّ‬
‫أن اﻟﻨﻘﻂ ‪ B ، A‬و ‪ O‬ﻋﲆ اﺳﺘﻘﺎﻣﺔ واﺣﺪة‪.‬‬ ‫‪ّ arg‬‬ ‫‪ •4‬أﺣﺴﺐ‬
‫‪z‬‬
‫ﻣﺎ ﻫﻲ ﻃﺒﻴﻌﺔ اﳌﺜﻠﺚ ‪ ABA‬؟‬

‫‪ •1‬ﻋﺒﺎرة اﻟﺘﺤﻮﻳﻞ ‪ f‬ﻣﻦ اﻟﺸﻜﻞ ‪ z = az + b‬ﺣﻴﺚ ‪:‬‬ ‫ﺍﳊﻞّ‪.‬‬


‫‪1‬‬ ‫‪√3‬‬ ‫𝜋‪2‬‬ ‫𝜋‪2‬‬ ‫‪/‬‬ ‫𝜋‪2‬‬
‫‪a=− +ı‬‬ ‫‪= cos‬‬ ‫‪+ ı sin‬‬ ‫‪=e‬‬ ‫‪= 1,‬‬
‫‪2‬‬ ‫‪2‬‬ ‫‪3‬‬ ‫‪3‬‬ ‫‪3‬‬
‫‪/‬‬ ‫𝜋‪2‬‬
‫‪ 𝜔 = e‬أي ‪. 𝜔 = 0‬‬ ‫‪ .‬إذا ﻛﺎﻧﺖ 𝜔 ﻻﺣﻘﺔ ﻣﺮﻛﺰ ﻫﺬا اﻟﺪوران ّ‬
‫ﻓﺈن 𝜔‬ ‫إذن ‪ f‬دوران زاوﻳﺘﻪ‬
‫‪3‬‬
‫𝜋‪2‬‬ ‫𝜋‪2‬‬
‫‪.‬‬ ‫‪ ℛ‬اﻟﺬي ﻣﺮﻛﺰه )‪ O (0, 0‬و زاوﻳﺘﻪ‬ ‫‪O,‬‬ ‫إذن ‪ f‬ﻫﻮ اﻟﺪوران‬
‫‪3‬‬ ‫‪3‬‬
‫‪ •2‬ﻟﺪﻳﻨﺎ ‪:‬‬
‫‪ņőƱŀ Ţ‬‬
‫‪1‬‬ ‫‪√3‬‬ ‫‪/‬‬ ‫‪/‬‬ ‫‪+‬‬ ‫‪/‬‬
‫‪z‬‬ ‫‪′‬‬ ‫=‬ ‫‪− +ı‬‬ ‫‪z‬‬ ‫‪=e‬‬ ‫‪⋅ 3e‬‬ ‫‪= 3e‬‬ ‫‪= 3e‬‬
‫‪2‬‬ ‫‪2‬‬

‫‪1‬‬ ‫‪√3‬‬ ‫‪/‬‬ ‫‪−‬‬


‫‪⋅ 3e−‬‬ ‫‪/‬‬ ‫‪/‬‬

‫‪VI‬‬
‫‪z‬‬ ‫‪′‬‬ ‫=‬ ‫‪− +ı‬‬ ‫‪z‬‬ ‫‪=e‬‬ ‫‪= 3e‬‬ ‫‪= 3e‬‬
‫‪2‬‬ ‫‪2‬‬

‫أن ‪ ||z || = ||z || = ||z ′ || = ||z ′ || = 3‬أي ‪ّ ||z − 0|| = ||z − 0|| = ||z ′ − 0|| = ||z ′ − 0|| = 3‬‬
‫ﻓﺈن‬ ‫‪ •3‬ﺑﲈ ّ‬
‫‪ OA = OB = OA = OB = 3‬و ﻫﺬا ﻳﻌﻨﻲ ّ‬
‫أن اﻟﻨﻘﻂ ‪ A ، B ، A‬و ‪ B‬ﺗﻘﻊ ﻋﲆ اﻟﺪاﺋﺮة ‪ Γ‬اﻟﺘﻲ ﻣﺮﻛﺰﻫﺎ ‪ O‬و‬
‫ﻧﺼﻒ ﻗﻄﺮﻫﺎ ‪. 3‬‬
‫‪ƕ‬‬

‫‪ •4‬ﻟﺪﻳﻨﺎ ‪:‬‬
‫‪z‬‬ ‫‪′‬‬ ‫‪3e‬‬ ‫‪/‬‬
‫‪+‬‬
‫=‬ ‫‪=e‬‬ ‫‪=e‬‬ ‫‪= −1‬‬
‫‪ũŏ‬‬

‫‪z‬‬ ‫‪3e−‬‬ ‫‪/‬‬

‫‪z ′ −0‬‬
‫‪B،A‬و‪O‬‬ ‫‪ OB, OA‬و ﻫﺬا ﻳﻌﻨﻲ ّ‬
‫أن اﻟﻨﻘﻂ‬ ‫‪ arg‬أي )𝜋‪= 𝜋 (mod 2‬‬ ‫إذن )𝜋‪= 𝜋 (mod 2‬‬
‫‪z −0‬‬
‫ﻋﲆ اﺳﺘﻘﺎﻣﺔ واﺣﺪة )ﺷﻜﻞ ‪. (18.VI‬‬
‫ﰲ اﻟﺪاﺋﺮة ‪ Γ‬اﻟﺰاوﻳﺔ ‪ A OB‬ﻣﺮﻛﺰﻳﺔ و اﻟﺰاوﻳﺔ ‪ A AB‬ﳏﻴﻄﺔ و ﳘﺎ ﺗﺸﺪّ ان ﻧﻔﺲ اﻟﻮﺗﺮ و ﺑﺎﻟﺘﺎﱄ ‪ A OB = 2A AB‬أي‬
‫𝜋‬
‫= ‪. A AB‬‬ ‫)𝜋‪(mod 2‬‬
‫‪2‬‬

‫𝟮𝟭𝟮‬

‫‪http ://tinyurl.com/Malki1718‬‬ ‫‪0‬‬


‫‪ .VI‬ﲤﺎرﻳﻦ ﺗﻄﺒﻴﻘﻴﺔ‬
‫‪.‬‬
‫‪3‬‬
‫‪B′‬‬

‫‪Ŕž Ŧ Ľ‬‬
‫‪2‬‬
‫‪A′‬‬ ‫‪A‬‬

‫‪œǃ‬‬
‫‪1‬‬

‫‪−3‬‬ ‫‪−2‬‬ ‫‪−1‬‬ ‫‪O‬‬ ‫‪1‬‬ ‫‪2‬‬ ‫‪3‬‬

‫‪Ŀ‬‬
‫‪−1‬‬
‫‪B‬‬
‫‪−2‬‬

‫‪−3‬‬

‫ﺷﻜﻞ ‪18.VI‬‬

‫ﻧﺴﺘﻨﺘﺞ ّ‬
‫أن اﳌﺜﻠﺚ ‪ ABA‬ﻗﺎﺋﻢ اﻟﺰاوﻳﺔ ﻋﻨﺪ اﻟﺮأس ‪. A‬‬
‫■‬

‫‬ ‫‪.‬‬
‫𝟒𝟒𝟏‬ ‫‪.‬‬

‫‪ •1‬ﻟﻴﻜﻦ ‪ f‬اﻟﺘﺤﻮﻳﻞ اﻟﻨﻘﻄﻲ ﻟﻠﻤﺴﺘﻮي اﳌﺮﻛﺐ ﰲ ﻧﻔﺴﻪ و اﻟﺬي ﻳﺮﻓﻖ ﺑﺎﻟﻨﻘﻄﺔ )‪ M (z‬اﻟﻨﻘﻄﺔ ‪ M z‬ﺑﺤﻴﺚ ‪:‬‬
‫‪z = az + 3ı‬‬
‫ﺛﻢ ﻋﻨﺪﻣﺎ ‪.a = −ı‬‬
‫ﺣﺪﱢ د ﻃﺒﻴﻌﺔ اﻟﺘﺤﻮﻳﻞ ‪ f‬و ﻋﻨﺎﴏه اﳌﻤ ّﻴﺰة ﻋﻨﺪﻣﺎ ‪ّ a = 2‬‬
‫‪ُ •2‬ﻳﻌﻄﻰ )‪ A (2ı) ، B (2 + ı) ، A (1‬و )‪.B (1 + ı‬‬
‫‪ņőƱŀ Ţ‬‬
‫أن ‪.AB = A B‬‬ ‫)ا( ﲢﻘﻖ ﻣﻦ ّ‬
‫ﺛﻢ ﺣﺪّ د ﻫﺬا اﻟﺪوران‪.‬‬
‫)ب( أﺛﺒﺖ أﻧّﻪ ﻳﻮﺟﺪ دوران ‪ r‬واﺣﺪ و واﺣﺪ ﻓﻘﻂ ﺑﺤﻴﺚ ‪ r (A) = A‬و ‪ّ r (B) = B‬‬
‫‪VI‬‬

‫‪ : a = 2‬اﻟﻨﻘﻄﺔ اﻟﺼﺎﻣﺪة‬ ‫•‬ ‫‪1‬‬


‫•‬ ‫ﺍﳊﻞّ‪.‬‬
‫‪f (𝜔) = 𝜔 ⟺ 2𝜔 + 3ı = 𝜔 ⟺ 𝜔 = −3ı‬‬
‫اﻟﺘﺤﻮﻳﻞ ‪ f‬ﻳﻘﺒﻞ ﻧﻘﻄﺔ ﺻﺎﻣﺪة وﺣﻴﺪة و ﻫﻲ اﻟﻨﻘﻄﺔ ‪ Ω‬ذات اﻟﻼّﺣﻘﺔ ‪.𝜔 = −3ı‬‬
‫‪ƕ‬‬

‫‪z = 2z + 3ı‬‬
‫‪.‬‬ ‫ﻟﺘﺤﺪﻳﺪ ﻃﺒﻴﻌﺔ اﻟﺘﺤﻮﻳﻞ ‪ ،f‬ﻧﺒﺤﺚ ﻋﻦ ﻋﺒﺎرة 𝜔 ‪ z −‬ﺑﺪﻻﻟﺔ 𝜔 ‪ .z −‬ﻟﺪﻳﻨﺎ ‪:‬‬
‫‪𝜔 = 2𝜔 + 3ı‬‬
‫ﺑﻄﺮح اﳌﺴﺎوﺗﲔ ﻃﺮﻓ ًﺎ إﱃ ﻃﺮف ﻳﻨﺘﺞ ‪z − 𝜔 = 2 (z − 𝜔) :‬‬
‫‪ũŏ‬‬

‫و ﻫﻲ اﻟﻜﺘﺎﺑﺔ اﳌﺮﻛﺒﺔ ﻟﻠﺘﺤﺎﻛﻲ اﻟﺬي ﻣﺮﻛﺰه )‪ Ω (−3ı‬و ﻧﺴﺒﺘﻪ ‪.k = 2‬‬


‫‪ : a = −ı‬اﻟﻨﻘﻄﺔ اﻟﺼﺎﻣﺪة‬ ‫•‬
‫‪3ı‬‬ ‫‪3 3‬‬
‫= 𝜔 ⟺ 𝜔 = ‪f (𝜔) = 𝜔 ⟺ −ı𝜔 + 3ı‬‬ ‫‪= + ı‬‬
‫‪1+ı 2 2‬‬
‫‪3 3‬‬
‫اﻟﺘﺤﻮﻳﻞ ‪ f‬ﻳﻘﺒﻞ ﻧﻘﻄﺔ ﺻﺎﻣﺪة وﺣﻴﺪة و ﻫﻲ اﻟﻨﻘﻄﺔ ‪ Ω‬ذات اﻟﻼّﺣﻘﺔ ‪.𝜔 = + ı‬‬
‫‪2 2‬‬

‫𝟯𝟭𝟮‬

‫‪http ://tinyurl.com/Malki1718‬‬ ‫‪0‬‬


‫‪ .3.VI‬ﺗﻄﺒﻴﻘﺎت اﻷﻋﺪاد اﳌﺮﻛﺒﺔ‬
‫‪.‬‬
‫‪z = −ız + 3ı‬‬
‫‪.‬‬ ‫ﻟﺘﺤﺪﻳﺪ ﻃﺒﻴﻌﺔ اﻟﺘﺤﻮﻳﻞ ‪ ،f‬ﻧﺒﺤﺚ ﻋﻦ ﻋﺒﺎرة 𝜔 ‪ z −‬ﺑﺪﻻﻟﺔ 𝜔 ‪ .z −‬ﻟﺪﻳﻨﺎ ‪:‬‬
‫‪𝜔 = −ı𝜔 + 3ı‬‬

‫‪Ŕž Ŧ Ľ‬‬
‫ﺑﻄﺮح اﳌﺴﺎوﺗﲔ ﻃﺮﻓ ًﺎ إﱃ ﻃﺮف ﻳﻨﺘﺞ ‪z − 𝜔 = −ı (z − 𝜔) :‬‬
‫𝜋‬ ‫‪3 3‬‬

‫‪œǃ‬‬
‫‪ Ω‬و زاوﻳﺘﻪ )𝜋‪.𝜃 = arg (−ı) = − (mod 2‬‬ ‫‪+ ı‬‬ ‫و ﻫﻲ اﻟﻜﺘﺎﺑﺔ اﳌﺮﻛﺒﺔ ﻟﻠﺪوران اﻟﺬي ﻣﺮﻛﺰه‬
‫‪2‬‬ ‫‪2 2‬‬

‫= ‪ ،AB‬و ﻣﻦ ﺟﻬﺔ أﺧﺮى‬ ‫ﻟﺪﻳﻨﺎ ﻣﻦ ﺟﻬﺔ ‪(2 − 1) + (1 − 0) = √2 :‬‬ ‫)ا(‬ ‫•‬ ‫‪2‬‬

‫‪Ŀ‬‬
‫= ‪ A B‬أي ‪.AB = A B‬‬ ‫‪(1 − 2) + (1 − 0) = √2‬‬
‫)ب( ﻟﻴﻜﻦ )𝜃 ‪ r (Ω,‬دوراﻧ ًﺎ ﻣﺮﻛﺰه ‪ Ω‬و زاوﻳﺘﻪ 𝜃‪ .‬اﻟﻜﺘﺎﺑﺔ اﳌﺮﻛﺒﺔ ﻟﻪ ﻫﻲ ‪.z − 𝜔 = e (z − 𝜔) :‬‬
‫ﻟﻨﺜﺒﺖ أﻧﻪ ﻳﻮﺟﺪ زوج وﺣﻴﺪ )𝜃 ‪ (𝜔,‬ﻣﻦ اﻷﻋﺪاد ﺑﺤﻴﺚ ‪ 𝜔 ∈ ℂ‬و [𝜋‪ُ 𝜃 ∈ [0, 2‬ﳛﻘﻖ ‪:‬‬
‫‪ r (A) = A‬و ‪.r (B) = B‬‬
‫اﻟﴩط ‪ُ r (A) = A‬ﻳﻜﺘﺐ ‪ 2ı − 𝜔 = e (1 − 𝜔) :‬؛‬
‫و اﻟﴩط ‪ُ r (B) = B‬ﻳﻜﺘﺐ ‪.1 + ı − 𝜔 = e (2 + ı − 𝜔) :‬‬
‫ﺑﻄﺮح اﳌﺴﺎوﺗﲔ ﻃﺮﻓ ًﺎ إﱃ ﻃﺮف ﻳﻨﺘﺞ ‪ ı − 1 = e (−1 − ı) :‬ﻣﻨﻪ ‪e = −ı = e− /‬‬
‫𝜋‬
‫أي )𝜋‪.𝜃 = − (mod 2‬‬
‫‪2‬‬
‫‪3 3‬‬
‫= 𝜔‪.‬‬ ‫ﻧﺴﺘﻨﺘﺞ ّ‬
‫أن ‪ 2ı − 𝜔 = −ı (1 − 𝜔) :‬أي ‪+ ı‬‬
‫‪2 2‬‬
‫‪3 3‬‬
‫‪Ω‬‬ ‫‪+ ı‬‬ ‫إذن ﻳﻮﺟﺪ دوران وﺣﻴﺪ ‪ُ r‬ﳛ ّﻮل اﻟﻨﻘﻄﺔ ‪ A‬إﱃ ‪ A‬و ُﳛ ّﻮل اﻟﻨﻘﻄﺔ ‪ B‬إﱃ ‪ B‬و ﻫﻮ اﻟﺪوران اﻟﺬي ﻣﺮﻛﺰه‬
‫‪2 2‬‬
‫𝜋‬
‫و زاوﻳﺘﻪ ‪.𝜃 = −‬‬
‫‪2‬‬
‫■‬

‫اﳌﺴﺘﻮي ﻣﻨﺴﻮب إﱃ ﻣﻌﻠﻢ ﻣﺘﻌﺎﻣﺪ و ﻣﺘﺠﺎﻧﺲ ⃗ ‪ B ، A . O, i,⃗ j‬و ‪ C‬ﺛﻼﺛﺔ ﻧﻘﻂ ﻣﻦ اﳌﺴﺘﻮي ﻟﻮاﺣﻘﻬﺎ ‪، a‬‬
‫‪#‬‬ ‫‪.‬‬
‫𝟓𝟒𝟏‬ ‫‪.‬‬
‫‪ņőƱŀ Ţ‬‬
‫‪ b‬و ‪ c‬ﻋﲆ اﻟﱰﺗﻴﺐ‪ .‬ﻧﺮﻓﻖ ﺑﺎﻟﺜﻼﺛﻴﺔ ‪ C ، B ، A‬اﻷﻋﺪاد اﻵﺗﻴﺔ ‪:‬‬

‫‪u (A, B, C) = a + bj + cj‬‬ ‫‪,‬‬ ‫‪v (A, B, C) = a + bj + cj‬‬


‫)‪u (A, B, C‬‬ ‫‪/‬‬

‫‪VI‬‬
‫= )‪w (A, B, C‬‬ ‫‪(j = e‬‬ ‫)ﺣﻴﺚ‬
‫)‪v (A, B, C‬‬

‫ﺑﺼ َﻮرﻫﺎ ‪:‬‬


‫ُﻌﻮض اﻟﻨﻘﻂ ‪ُ C ، B ، A‬‬
‫ﺗﺘﺤﻮل اﻷﻋﺪاد )‪ v (A, B, C) ، u (A, B, C‬و )‪ w (A, B, C‬ﻋﻨﺪﻣﺎ ﻧ ّ‬
‫ﻛﻴﻒ ّ‬
‫ٍ‬
‫ﺑﺘﺤﺎك ﻣﺮﻛﺰه ‪ O‬؟‬ ‫‪•3‬‬ ‫‪ •2‬ﺑﺪوران ﻣﺮﻛﺰه ‪ O‬؟‬ ‫‪ •1‬ﺑﺎﻧﺴﺤﺎب؟‬
‫‪ƕ‬‬

‫‪ •1‬ﻟﻴﻜﻦ 𝜏 اﻹﻧﺴﺤﺎب اﻟﺬي ﺷﻌﺎﻋﻪ ‪ OM‬و ‪ m‬ﻻﺣﻘﺔ اﻟﻨﻘﻄﺔ ‪ . M‬ﺣﺴﺐ اﳌﱪﻫﻨﺔ ‪ 26‬ﺻﻔﺤﺔ ‪ّ 55‬‬
‫ﻓﺈن ﻻﺣﻘﺔ‬ ‫ﺍﳊﻞّ‪.‬‬
‫‪ũŏ‬‬

‫𝟰𝟭𝟮‬

‫‪http ://tinyurl.com/Malki1718‬‬ ‫‪0‬‬


‫‪ .VI‬ﲤﺎرﻳﻦ ﺗﻄﺒﻴﻘﻴﺔ‬
‫‪.‬‬
‫اﻟﻨﻘﻄﺔ )‪ 𝜏 (A‬ﻫﻲ ‪ ، a + m‬ﻻﺣﻘﺔ )‪ 𝜏 (B‬ﻫﻲ ‪ b + m‬و ﻻﺣﻘﺔ )‪ 𝜏 (C‬ﻫﻲ ‪ c + m‬إذن ‪:‬‬

‫‪Ŕž Ŧ Ľ‬‬
‫‪u (𝜏(A), 𝜏(B), 𝜏(C)) = (a + m) + (b + m)j + (c + m)j‬‬
‫‪= a + bj + cj + m 1 + j + j‬‬

‫‪œǃ‬‬
‫‪1−j‬‬
‫= ‪1+j+j‬‬ ‫‪=0‬‬ ‫ﻟﻜﻦ ‪ j‬ﺟﺬر ﺗﻜﻌﻴﺒﻲ ﻟﻠﻮﺣﺪة إذن ‪:‬‬
‫‪1−j‬‬
‫)‪u (𝜏(A), 𝜏(B), 𝜏(C)) = a + bj + cj = u (A, B, C‬‬ ‫ﻣﻨﻪ ‪:‬‬

‫‪Ŀ‬‬
‫)‪v (𝜏(A), 𝜏(B), 𝜏(C)) = a + bj + cj = v (A, B, C‬‬ ‫ﻧﺜﺒﺖ ﺑﻨﻔﺲ اﻟﻄﺮﻳﻘﺔ ّ‬
‫أن ‪:‬‬
‫)‪w (𝜏(A), 𝜏(B), 𝜏(C)) = w (A, B, C‬‬ ‫و ﻧﺴﺘﻨﺘﺞ ّ‬
‫أن ‪:‬‬

‫‪ •2‬ﻟﻴﻜﻦ 𝜌 اﻟﺪوران اﻟﺬي ﻣﺮﻛﺰه ‪ O‬و 𝛼 زاوﻳﺘﻪ ‪.‬ﺣﺴﺐ اﳌﱪﻫﻨﺔ ‪ 27‬ﺻﻔﺤﺔ ‪ّ 57‬‬
‫ﻓﺈن ﻻﺣﻘﺔ اﻟﻨﻘﻄﺔ )‪ 𝜌 (A‬ﻫﻲ ‪، ae‬‬
‫ﻻﺣﻘﺔ )‪ 𝜌 (B‬ﻫﻲ ‪ be‬و ﻻﺣﻘﺔ )‪ 𝜌 (C‬ﻫﻲ ‪ ce‬إذن ‪:‬‬

‫‪u (𝜌(A), 𝜌(B), 𝜌(C)) = ae‬‬ ‫‪+ be j + ce j = u (A, B, C) ⋅ e‬‬


‫‪v (𝜌(A), 𝜌(B), 𝜌(C)) = v (A, B, C) ⋅ e‬‬ ‫ﺑﻨﻔﺲ اﻟﻄﺮﻳﻘﺔ ﻧﺜﺒﺖ ّ‬
‫أن ‪:‬‬
‫)‪w (𝜌(A), 𝜌(B), 𝜌(C)) = w (A, B, C‬‬ ‫ﻣﻨﻪ ‪:‬‬

‫‪ •3‬ﻟﻴﻜﻦ 𝜎 اﻟﺘﺤﺎﻛﻲ اﻟﺬي ﻣﺮﻛﺰه ‪ O‬و ﻧﺴﺒﺘﻪ ‪. k‬ﺣﺴﺐ اﳌﱪﻫﻨﺔ ‪ 26‬ﺻﻔﺤﺔ ‪ّ 55‬‬
‫ﻓﺈن ﻻﺣﻘﺔ اﻟﻨﻘﻄﺔ )‪ 𝜎 (A‬ﻫﻲ ‪، k ⋅ a‬‬
‫ﻻﺣﻘﺔ )‪ 𝜎 (B‬ﻫﻲ ‪ k ⋅ b‬و ﻻﺣﻘﺔ )‪ 𝜎 (C‬ﻫﻲ ‪ k ⋅ c‬إذن ‪:‬‬

‫)‪u (𝜌(A), 𝜌(B), 𝜌(C)) = ka + kbj + kcj = k ⋅ u (A, B, C‬‬


‫)‪v (𝜎(A), 𝜎(B), 𝜎(C)) = k ⋅ v (A, B, C‬‬ ‫ﺑﻨﻔﺲ اﻟﻄﺮﻳﻘﺔ ﻧﺜﺒﺖ ّ‬
‫أن ‪:‬‬
‫)‪w (𝜎(A), 𝜎(B), 𝜎(C)) = w (A, B, C‬‬ ‫ﻣﻨﻪ ‪:‬‬
‫■‬
‫‪ņőƱŀ Ţ‬‬
‫‪.‬‬
‫𝟔𝟒𝟏‬ ‫‪.‬‬
‫اﳌﻌﺮف ﻋﲆ ‪ ℂ‬ﺑﺎﻟﻌﺒﺎرة ‪.U (X) = X + (1 − 2ı) X − 2ı :‬‬
‫ﻟﻴﻜﻦ )‪ U (X‬ﻛﺜﲑ اﳊﺪود ﱠ‬
‫‪VI‬‬

‫‪ •1‬أوﺟﺪ اﳉﺬرﻳﻦ اﻟﱰﺑﻴﻌﻴﲔ ﻟﻠﻌﺪد ‪. −3 + 4ı‬‬


‫‪ •2‬أوﺟﺪ اﳉﺬرﻳﻦ اﳌﺮﻛﺒﲔ ﻟﻜﺜﲑ اﳊﺪود )‪. U (X‬‬
‫‪ •3‬ﻟﻴﻜﻦ ‪ z‬ﻋﺪد ًا ﻣﺮﻛﺒ ًﺎ ﺣﻴﺚ ‪ z = x + ıy‬ﻣﻊ ‪ x‬و ‪ y‬ﻋﺪدان ﺣﻘﻴﻘﻴﺎن‪.‬‬
‫‪ƕ‬‬

‫)ا( أﺣﺴﺐ اﳉﺰء اﳊﻘﻴﻘﻲ و اﳉﺰء اﻟﺘﺨﻴﲇ ﻟـِ )‪ U (X‬ﺑﺪﻻﻟﺔ ‪ x‬و ‪. y‬‬

‫)ب( ﻧﻨﺴﺐ اﳌﺴﺘﻮي إﱃ اﳌﻌﻠﻢ اﳌﺘﻌﺎﻣﺪ و اﳌﺘﺠﺎﻧﺲ ⃗ ‪) ℛ = O, i,⃗ j‬ﻣﻊ ‪.(‖‖i‖‖⃗ = ‖‖j‖‖⃗ = 1 cm‬‬
‫‪ũŏ‬‬

‫)‪ (i‬ﻟﺘﻜﻦ ‪ Γ‬ﳎﻤﻮﻋﺔ اﻟﻨﻘﻂ ‪ M‬ذات اﻹﺣﺪاﺛﻴﲔ )‪ (x, y‬ﺑﺤﻴﺚ ﻳﻜﻮن )‪ U (x + ıy‬ﲣﻴﻠﻴ ًﺎ ﴏﻓ ًﺎ‪ .‬ﺣﺪﱢ د ﻃﺒﻴﻌﺔ‬
‫ﺛﻢ أوﺟﺪ ﻣﺮﻛﺰﻫﺎ و اﺧﺘﻼﻓﻬﺎ اﳌﺮﻛﺰي‪ .‬أﻧﺸﺊ اﳌﺠﻤﻮﻋﺔ ‪. Γ‬‬
‫‪ّ Γ‬‬
‫)‪ (ii‬ﻟﺘﻜﻦ ‪ Γ‬ﳎﻤﻮﻋﺔ اﻟﻨﻘﻂ ‪ M‬ذات اﻹﺣﺪاﺛﻴﲔ )‪ (x, y‬ﺑﺤﻴﺚ ﻳﻜﻮن )‪ U (x + ıy‬ﺣﻘﻴﻘﻴ ًﺎ‪ .‬ﺣﺪﱢ د ﻃﺒﻴﻌﺘﻬﺎ و‬
‫ﻣﺮﻛﺰﻫﺎ‪ .‬أﻧﺸﺊ ‪ Γ‬ﻋﲆ ﻧﻔﺲ اﻟﺮﺳﻢ ﻣﻊ ‪. Γ‬‬

‫𝟱𝟭𝟮‬

‫‪http ://tinyurl.com/Malki1718‬‬ ‫‪0‬‬


‫‪ .3.VI‬ﺗﻄﺒﻴﻘﺎت اﻷﻋﺪاد اﳌﺮﻛﺒﺔ‬
‫‪.‬‬

‫ﺗﺬﻛﲑ ‪ :‬ﻟﺘﻜﻦ 𝒞 ﳎﻤﻮﻋﺔ اﻟﻨﻘﻂ )‪ ، M (x, y‬ﰲ اﳌﺴﺘﻮي اﳌﻨﺴﻮب إﱃ ﻣﻌﻠﻢ ﻣﺘﻌﺎﻣﺪ و ﻣﺘﺠﺎﻧﺲ‪ ،‬ﺑﺤﻴﺚ ‪:‬‬

‫‪Ŕž Ŧ Ľ‬‬
‫‪ ax + bxy + cy + dx + ey + f = 0‬ﻣﻊ ‪ a, b, c, d, e, f ∈ ℝ‬و )‪. (a, b, c) ≠ (0, 0, 0‬‬

‫إذا ﻛﺎن ‪ّ b − 4ac < 0‬‬


‫ﻓﺈن اﳌﺠﻤﻮﻋﺔ 𝒞 ﻫﻲ إ ّﻣﺎ ﺧﺎﻟﻴﺔ أو ﻧﻘﻄﺔ أو ﻗﻄﻊ ﻧﺎﻗﺺ )أو داﺋﺮة(‪.‬‬

‫‪œǃ‬‬
‫•‬

‫إذا ﻛﺎن ‪ّ b − 4ac = 0‬‬


‫ﻓﺈن اﳌﺠﻤﻮﻋﺔ 𝒞 ﻫﻲ إ ّﻣﺎ ﺧﺎﻟﻴﺔ أو ﻣﺴﺘﻘﻴﻢ أو إﲢﺎد ﻣﺴﺘﻘﻴﻤﲔ ﻣﺘﻮازﻳﲔ أو ﻗﻄﻊ ﻣﻜﺎﻓﺊ‪.‬‬ ‫•‬

‫إذا ﻛﺎن ‪ّ b − 4ac > 0‬‬


‫ﻓﺈن اﳌﺠﻤﻮﻋﺔ 𝒞 ﻫﻲ إ ّﻣﺎ إﲢﺎد ﻣﺴﺘﻘﻴﻤﲔ ﻣﺘﻘﺎﻃﻌﲔ أو ﻗﻄﻊ زاﺋﺪ‪.‬‬

‫‪Ŀ‬‬
‫•‬

‫‪⎪ x − y = Re (−3 + 4ı) = −3‬‬


‫⎧‬
‫‪.‬‬ ‫‪2xy = Im (−3 + 4ı) = 4‬‬ ‫‪ •1‬ﻟﻴﻜﻦ ‪ z = x + ıy‬ﺟﺬر ًا ﺗﺮﺑﻴﻌﻴﺎ ﻟﻠﻌﺪد ‪ . −3 + 4ı‬ﻟﺪﻳﻨﺎ ‪:‬‬ ‫ﺍﳊﻞّ‪.‬‬
‫⎨‬
‫⎪‬
‫‪⎩ x + y = ||−3 + 4ı|| = 5‬‬
‫ﻧﻌﻮض ﰲ اﳌﻌﺎدﻟﺔ اﻟﺜﺎﻧﻴﺔ ﻟﻨﺠﺪ‬ ‫ﺑﺠﻤﻊ اﳌﻌﺎدﻟﺘﲔ اﻷوﱃ و اﻟﺜﺎﻟﺜﺔ ﻧﺠﺪ ‪ 2x = 2‬أي ‪ . x = ±1‬ﻧﺄﺧﺬ ﻣﺜ ً‬
‫ﻼ‪x = 1‬و ّ‬
‫‪.y = 2‬‬
‫و ﺑﺎﻟﺘﺎﱄ ﻓﺎﳉﺬران اﻟﱰﺑﻴﻌﻴﺎن ﻟﻠﻌﺪد ‪ −3 + 4ı‬ﳘﺎ ‪ 1 + 2ı‬و )‪. − (1 + 2ı‬‬
‫ﻓﺠﺬرا ﻛﺜﲑ اﳊﺪود )‪U (X‬‬
‫َ‬ ‫ﻧﺘّﺒﻊ ﻃﺮﻳﻘﺔ اﳌﻤ ﱢﻴﺰ ‪ . Δ = (1 − 2ı) − 4 × (−2ı) = −3 + 4ı = (1 + 2ı) :‬و ﺑﺎﻟﺘﺎﱄ‬ ‫•‬ ‫‪2‬‬
‫)‪− (1 − 2ı) + (1 + 2ı‬‬ ‫)‪− (1 − 2ı) − (1 + 2ı‬‬
‫‪.‬‬ ‫و ‪= 2ı‬‬ ‫ﳘﺎ ‪= −1‬‬
‫‪2‬‬ ‫‪2‬‬
‫)ا( ﻟﺪﻳﻨﺎ ‪:‬‬ ‫•‬ ‫‪3‬‬
‫)‪U (z) = (x + ıy) + (1 − 2ı) (x + ıy) − 2ı = x − y + x + 2y + ı (2xy − 2x + y − 2‬‬
‫‪ Re (U (z)) = x − y + x + 2y‬و ‪. Im (U (z)) = 2xy − 2x + y − 2‬‬ ‫ﻣﻨﻪ ‪:‬‬
‫)ب( )‪ (i‬ﻣﻌﺎدﻟﺔ ‪ Γ‬ﻫﻲ ‪ . x − y + x + 2y = 0‬ﻟﺪﻳﻨﺎ ‪ b = 0 ، a = 1‬و ‪ c = −1‬ﻣﻨﻪ ‪b − 4ac = 4 > 0‬‬
‫و ﺑﺎﻟﺘﺎﱄ ّ‬
‫ﻓﺈن ‪ Γ‬ﻫﻲ إ ّﻣﺎ إﲢﺎد ﻣﺴﺘﻘﻴﻤﲔ ﻣﺘﻘﺎﻃﻌﲔ أو ﻗﻄﻊ زاﺋﺪ‪ .‬ﻟﻜﻦ ‪:‬‬
‫‪ņőƱŀ Ţ‬‬
‫‪1‬‬
‫× ‪x − y + x + 2y = x + 2‬‬ ‫‪× x − y − 2 × (−1) × y‬‬
‫‪2‬‬
‫‪1‬‬ ‫‪1‬‬
‫=‬ ‫‪x+‬‬ ‫‪−‬‬ ‫)‪− (y − 1) − (−1‬‬
‫‪2‬‬ ‫‪2‬‬

‫‪VI‬‬
‫‪1‬‬ ‫‪3‬‬
‫=‬ ‫‪x+‬‬ ‫‪− (y − 1) −‬‬
‫‪2‬‬ ‫‪4‬‬
‫ﻣﻨﻪ ‪:‬‬
‫‪1‬‬ ‫‪3‬‬ ‫‪x+‬‬ ‫‪y−1‬‬
‫⟺ ‪x −y +x+2y = 0‬‬ ‫‪x+‬‬ ‫⟺ ‪−(y − 1) − = 0‬‬ ‫‪−‬‬ ‫‪=1‬‬
‫‪ƕ‬‬

‫‪2‬‬ ‫‪4‬‬

‫‪1‬‬
‫و ﻫﻲ ﻣﻌﺎدﻟﺔ ﻗﻄﻊ زاﺋﺪ ﻣﺮﻛﺰه اﻟﻨﻘﻄﺔ ‪ Ω − , 1‬و ﻣﻌﺎدﻟﺘﺎ ﳏﻮرﻳﻪ ﳘﺎ ‪) x = 0‬ﳏﻮر اﻟﱰاﺗﻴﺐ( و‬
‫‪2‬‬
‫‪) y = 0‬ﳏﻮر اﻟﻔﻮاﺻﻞ(‪ .‬و ﻫﻮ ﻗﻄﻊ زاﺋﺪ ﻣﺘﺴﺎوي اﻟﺴﺎﻗﲔ )اﻟﻀﻠﻌﲔ( و ﺑﺎﻟﺘﺎﱄ ّ‬
‫ﻓﺈن اﺧﺘﻼﻓﻪ اﳌﺮﻛﺰي‬
‫‪ũŏ‬‬

‫ﻳﻤﺮان ﺑﺎﳌﺮﻛﺰ ‪ Ω‬و ﻣﻴﻼﳘﺎ ﻳﺴﺎوﻳﺎن ‪. ±1‬‬


‫ﻳﺴﺎوي ‪ √2‬و ﳏﻮرﻳﻪ اﳌﻘﺎرﺑﲔ ّ‬
‫)‪ (ii‬ﻣﻌﺎدﻟﺔ ‪ Γ‬ﻫﻲ ‪ . 2xy − 2x + y − 2 = 0‬ﻟﺪﻳﻨﺎ ‪ b = 1 ، a = 0‬و ‪ c = 0‬ﻣﻨﻪ ‪ b − 4ac = 1 > 0‬و‬
‫ﺑﺎﻟﺘﺎﱄ ّ‬
‫ﻓﺈن ‪ Γ‬ﻫﻲ إ ّﻣﺎ إﲢﺎد ﻣﺴﺘﻘﻴﻤﲔ ﻣﺘﻘﺎﻃﻌﲔ أو ﻗﻄﻊ زاﺋﺪ‪ .‬ﻟﺪﻳﻨﺎ ‪:‬‬
‫‪1‬‬ ‫‪1‬‬ ‫‪1‬‬
‫⟺ ‪2xy − 2x + y − 2 = 0 ⟺ xy − x + y − 1 = 0‬‬ ‫‪x+‬‬ ‫= )‪(y − 1‬‬
‫‪2‬‬ ‫‪2‬‬ ‫‪2‬‬

‫𝟲𝟭𝟮‬

‫‪http ://tinyurl.com/Malki1718‬‬ ‫‪0‬‬


‫‪ .VI‬ﲤﺎرﻳﻦ ﺗﻄﺒﻴﻘﻴﺔ‬
‫‪.‬‬
‫‪1‬‬
‫ﻧُﺠﺮي إﻧﺴﺤﺎﺑ ًﺎ ﺷﻌﺎﻋﻪ ⃗ ‪ V⃗ = − i ⃗ + j‬ﻟﻠﻤﻌﻠﻢ ⃗ ‪ ℛ = O, i,⃗ j‬و ﻟﻴﻜﻦ ⃗ ‪ ℛ = Ω, i,⃗ j‬اﳌﻌﻠﻢ‬
‫‪2‬‬
‫‪1‬‬ ‫‪1‬‬

‫‪Ŕž Ŧ Ľ‬‬
‫اﳉﺪﻳﺪ ﺣﻴﺚ ‪ .Ω = − , 1‬ﰲ اﳌﻌﻠﻢ ‪ ، ℛ‬ﻣﻌﺎدﻟﺔ ‪ Γ‬ﻫﻲ = ‪. XY‬‬
‫‪2‬‬ ‫‪2‬‬
‫ﳌﺤﻮري اﳌﻌﻠﻢ ‪.ℛ‬‬
‫ْ‬ ‫ﺑﺪاﻳ ًﺔ‪ ،‬ﻧﺘﺨ ّﻠﺺ ﻣﻦ اﳊﺪ ‪ XY‬و ذﻟﻚ ﺑﺘﻄﺒﻴﻖ دوران‬

‫‪œǃ‬‬
‫ﻟﺘﻜﻦ ‪ z = X + ıY‬و ‪ z = X + ıY‬ﻧﻘﻄﺘﲔ ﻣﻦ اﳌﺴﺘﻮي اﳌﺮﻛﺐ و ﻟﻴﻜﻦ )𝜃 ‪ ℛ (Ω,‬دوراﻧ ًﺎ ﻣﺮﻛﺰه ‪ Ω‬و‬
‫زاوﻳﺘﻪ 𝜃 ‪ .‬ﻟﺪﻳﻨﺎ ‪:‬‬

‫‪z = e z ⟺ X + ıY = (cos 𝜃 + ı sin 𝜃) X + ıY‬‬

‫‪Ŀ‬‬
‫𝜃 ‪⟺ X + ıY = X cos 𝜃 − Y sin 𝜃 + ı X sin 𝜃 + Y cos‬‬
‫𝜃 ‪X = X cos 𝜃 − Y sin‬‬
‫⟺‬
‫𝜃 ‪Y = X sin 𝜃 + Y cos‬‬

‫ﻟﻜﻦ ‪:‬‬
‫‪1‬‬ ‫‪1‬‬
‫= ‪XY‬‬ ‫⟺‬ ‫𝜃 ‪X cos 𝜃 − Y sin‬‬ ‫= 𝜃 ‪X sin 𝜃 + Y cos‬‬
‫‪2‬‬ ‫‪2‬‬
‫‪1‬‬
‫‪⟺ X cos 𝜃 sin 𝜃 − Y cos 𝜃 sin 𝜃 + X Y‬‬ ‫= 𝜃 ‪cos 𝜃 − sin‬‬
‫‪2‬‬

‫ﻧﻼﺣﻆ أﻧﻪ ﻟﻠﺘﺨﻠﺺ ﻣﻦ اﳊﺪ 𝜃 ‪ 2X Y cos 𝜃 − sin‬ﻳﻜﻔﻲ أن ﻳﻜﻮن ‪ cos 𝜃 − sin 𝜃 = 0‬أي‬
‫𝜋‬
‫= 𝜃 ﻓﻴﻜﻮن ‪:‬‬ ‫𝜃 ‪ . cos 𝜃 = ± sin‬ﻧﺄﺧﺬ‬
‫‪4‬‬
‫‪1‬‬ ‫‪√2 √2‬‬ ‫‪√2 √2 1‬‬
‫= ‪XY‬‬ ‫‪⟺ X‬‬ ‫⋅‬ ‫‪−Y‬‬ ‫⋅‬ ‫=‬
‫‪2‬‬ ‫‪2‬‬ ‫‪2‬‬ ‫‪2‬‬ ‫‪2‬‬ ‫‪2‬‬
‫‪X −Y =1‬‬
‫‪ņőƱŀ Ţ‬‬
‫و ﻫﻲ ﻣﻌﺎدﻟﺔ ﻗﻄﻊ زاﺋﺪ ﻣﺮﻛﺰه اﻟﻨﻘﻄﺔ ‪ Ω‬و ﻣﻌﺎدﻟﺘﺎ ﳏﻮرﻳﻪ ﳘﺎ ‪ X = 0‬و ‪ . Y = 0‬و ﻫﻮ ﻗﻄﻊ زاﺋﺪ‬
‫ﻳﻤﺮان ﺑﺎﳌﺮﻛﺰ‬ ‫ﻣﺘﺴﺎوي اﻟﺴﺎﻗﲔ )اﻟﻀﻠﻌﲔ( و ﺑﺎﻟﺘﺎﱄ ّ‬
‫ﻓﺈن اﺧﺘﻼﻓﻪ اﳌﺮﻛﺰي ﻳﺴﺎوي ‪ √2‬و ﳏﻮرﻳﻪ اﳌﻘﺎرﺑﲔ ّ‬
‫‪ Ω‬و ﻣﻴﻼﳘﺎ ﻳﺴﺎوﻳﺎن ‪. ±1‬‬
‫ﻣﻼﺣﻈﺔ ‪ ⧏ : 32‬ﻧﻘﻂ ﺗﻘﺎﻃﻊ ‪ Γ‬و ‪ُ Γ‬ﲢﻘﻖ ‪ Re (U (z)) = Im (U (z)) = 0‬أي ‪ U (z) = 0‬إذن ﻫﻲ ﺟﺬور ﻛﺜﲑ‬
‫‪VI‬‬

‫⧐‬
‫اﳊﺪود )‪ U (X‬و ﻫﻲ ‪ −1‬و ‪ 2ı‬أو ﺑﺘﻌﺒﲑ آﺧﺮ )‪ (−1, 0‬و )‪. (0, 2‬‬
‫■‬

‫ﲤﺎﺭﻳﻦ ﻣﺘﻨﻮﻋﺔ‬ ‫‪4.VI‬‬


‫‪ũŏ‬‬ ‫‪ƕ‬‬

‫‪.‬‬
‫𝟕𝟒𝟏‬ ‫‪.‬‬
‫ﻟﻴﻜﻦ ‪ f: ℂ → ℂ‬ﺗﻄﺒﻴﻘ ًﺎ ﻣﺮﻛﺒﺎ ﺑﺤﻴﺚ ‪:‬‬
‫‪∀z , z ∈ ℂ ,‬‬ ‫) ‪f (z + z ) = f (z ) + f (z‬‬
‫‪∀𝜆 ∈ ℝ, ∀z ∈ ℂ ,‬‬ ‫)‪f (𝜆z) = 𝜆f (z‬‬

‫‪. ∀z ∈ ℂ ,‬‬ ‫‪f (z) = 𝛼z + 𝛽 z‬‬ ‫أﺛﺒﺖ أﻧﻪ ﻳﻮﺟﺪ ﻋﺪدان ﻣﺮﻛﺒﺎن 𝛼 و 𝛽 ﺑﺤﻴﺚ ‪:‬‬

‫𝟳𝟭𝟮‬

‫‪http ://tinyurl.com/Malki1718‬‬ ‫‪0‬‬


‫‪ .4.VI‬ﲤﺎرﻳﻦ ﻣﺘﻨﻮﻋﺔ‬
‫‪.‬‬
‫‪5‬‬

‫‪Ŕž Ŧ Ľ‬‬ ‫‪x = − 21‬‬


‫‪4‬‬

‫‪y‬‬
‫=‬
‫‪−‬‬
‫‪x‬‬

‫‪œǃ‬‬
‫‪+‬‬
‫‪Γ2‬‬

‫‪1‬‬
‫‪2‬‬
‫‪3‬‬

‫‪2‬‬
‫‪π‬‬
‫‪4‬‬

‫‪Ŀ‬‬
‫‪y=1‬‬ ‫‪Ω‬‬
‫‪b‬‬
‫‪1‬‬ ‫‪π‬‬
‫‪2‬‬

‫‪b‬‬

‫‪O‬‬
‫‪−6‬‬ ‫‪−5‬‬ ‫‪−4‬‬ ‫‪−3‬‬ ‫‪−2‬‬ ‫‪−1‬‬ ‫‪1‬‬ ‫‪2‬‬ ‫‪3‬‬ ‫‪4‬‬ ‫‪5‬‬

‫‪−1‬‬
‫‪Γ1‬‬

‫‪−2‬‬
‫‪3‬‬
‫‪2‬‬
‫‪+‬‬
‫‪x‬‬
‫=‬
‫‪y‬‬

‫‪−3‬‬

‫‪−4‬‬

‫ﺷﻜﻞ ‪19.VI‬‬

‫ﻟﻴﻜﻦ ‪ z = x + ıy‬ﻋﺪد ًا ﻣﺮﻛﺒﺎ ﺣﻴﺚ ‪ . x, y ∈ ℝ‬ﺣﺴﺐ اﳋﺎﺻﻴﺔ اﻷوﱃ ﻟﺪﻳﻨﺎ ‪f (z) = f (x + ıy) = f (x) + :‬‬ ‫ﺍﳊﻞّ‪.‬‬
‫)‪ f (ıy‬و ﺑﲈ ّ‬
‫أن ‪ x, y ∈ ℝ‬و ‪ x = x × 1‬ﻓﺤﺴﺐ اﳋﺎﺻﻴﺔ اﻟﺜﺎﻧﻴﺔ ﻟﺪﻳﻨﺎ ‪ . f (z) = f (x) + f (ıy) = xf (1) + y (ı) :‬ﻧﻀﻊ‬
‫‪z−z‬‬ ‫‪z+z‬‬
‫= )‪ y = Im (z‬ﻣﻨﻪ ‪:‬‬ ‫= )‪ x = Re (z‬و‬ ‫)‪ a = f (1‬و )‪ b = f (ı‬ﻓﻴﻜﻮن ‪ . f (z) = ax + by‬ﻟﻜﻦ‬
‫‪2ı‬‬ ‫‪2‬‬
‫‪z+z‬‬ ‫‪z−z‬‬
‫‪f (z) = ax + by = a‬‬ ‫‪+b‬‬
‫‪ņőƱŀ Ţ‬‬
‫‪2‬‬ ‫‪2ı‬‬
‫‪a b‬‬ ‫‪a b‬‬
‫=‬ ‫‪+‬‬ ‫‪z+‬‬ ‫‪−‬‬ ‫‪z‬‬
‫‪2 2ı‬‬ ‫‪2 2ı‬‬
‫‪1‬‬ ‫‪1‬‬
‫‪= (a − bı) z + (a + bı) z‬‬

‫‪VI‬‬
‫‪2‬‬ ‫‪2‬‬
‫‪= 𝛼z + 𝛽 z‬‬
‫‪1‬‬ ‫‪1‬‬
‫ﺣﻴﺚ )‪ 𝛼 = (a − bı‬و )‪ 𝛽 = (a + bı‬و ﻫﻮ اﳌﻄﻠﻮب‪.‬‬
‫‪2‬‬ ‫‪2‬‬
‫ﻣﻼﺣﻈﺔ ‪ ⧏ : 33‬ﺑﻮﺿﻊ )‪ u = a + bı = f (1) + ıf (ı‬ﻳﻜﻮن ‪ 2f (z) = uz + uz‬و ﻫﺬا اﻟﻌﺪد ﺣﻘﻴﻘﻲ و ُﻳﺪﻋﻰ‬
‫‪ƕ‬‬

‫⧐‬
‫اﳉﺪاء اﳊﻘﻴﻘﻲ ﻟﻠﻌﺪدﻳﻦ ‪ u‬و ‪.z‬‬
‫■‬
‫‪ũŏ‬‬

‫‪.‬‬ ‫⃗ ‪O, i,⃗ j‬‬ ‫اﳌﺴﺘﻮي اﳌﺮﻛﺐ ﻣﻨﺴﻮب إﱃ ﻣﻌﻠﻢ ﻣﺘﻌﺎﻣﺪ و ﻣﺘﺠﺎﻧﺲ‬
‫‬ ‫‪.‬‬
‫𝟖𝟒𝟏‬ ‫‪.‬‬

‫‪−1 + ı√3‬‬ ‫‪3+ı‬‬


‫= 𝛽‪.‬‬ ‫√ = 𝛼و‬ ‫𝛼 و 𝛽 ﻋﺪدان ﻣﺮﻛﺒﺎن ﺣﻴﺚ ‪:‬‬
‫‪2‬‬ ‫‪2‬‬
‫ﺻﻮرﰐ اﻟﻌﺪدﻳﻦ 𝛼 و 𝛽 ﻋﲆ اﻟﱰﺗﻴﺐ‪.‬‬
‫َْ‬ ‫‪ •1‬أﻛﺘﺐ ﻛﻼ ﻣﻦ اﻟﻌﺪدﻳﻦ 𝛼 و 𝛽 ﻋﲆ ﺷﻜﻠﻪ اﳌﺜﻠﺜﻲ و ﻋﻠﻢ اﻟﻨﻘﻄﺘﲔ ‪ A‬و ‪B‬‬

‫𝟴𝟭𝟮‬

‫‪http ://tinyurl.com/Malki1718‬‬ ‫‪0‬‬


‫‪ .VI‬ﲤﺎرﻳﻦ ﺗﻄﺒﻴﻘﻴﺔ‬
‫‪.‬‬
‫‪1.5‬‬ ‫‪M2‬‬

‫‪Ŕž Ŧ Ľ‬‬
‫‪B‬‬ ‫‪1‬‬

‫‪œǃ‬‬
‫‪A‬‬
‫‪M1‬‬ ‫‪0.5‬‬

‫‪Ŀ‬‬
‫‪−1.5‬‬ ‫‪−1‬‬ ‫‪−0.5‬‬ ‫‪O‬‬ ‫‪0.5‬‬ ‫‪1‬‬

‫ﺷﻜﻞ ‪20.VI‬‬

‫أن ﺣ ّﻠ ْﻴﻬﺎ ‪ z‬و ‪ُ z‬ﻳﻜﺘﺒﺎن ﺑﺪﻻﻟﺔ‬ ‫‪ّ •2‬‬


‫ﺣﻞ ﰲ اﳌﺠﻤﻮﻋﺔ ‪ ℂ‬اﳌﻌﺎدﻟﺔ ‪ z + 1 − ı√3 z − 1 + ı√3 = 0 :‬و ﲢﻘﻖ ﻣﻦ ّ‬
‫𝛼و𝛽‪.‬‬
‫ﺻﻮرﰐ اﻟﻌﺪدﻳﻦ ‪ z‬و ‪ z‬ﻋﲆ اﻟﱰﺗﻴﺐ‪.‬‬
‫َْ‬ ‫‪ •3‬إﺳﺘﻨﺘﺞ ﳑّﺎ ﺳﺒﻖ إﻧﺸﺎء اﻟﻨﻘﻄﺘﲔ ‪ M‬و ‪M‬‬

‫أﻛﺘﺐ ‪ z‬و ‪ z‬ﻋﲆ اﻟﺸﻜﻞ اﳌﺜﻠﺜﻲ‪.‬‬


‫𝜋‪11‬‬ ‫𝜋‪11‬‬ ‫𝜋‪5‬‬ ‫𝜋‪5‬‬
‫‪. sin‬‬ ‫‪ cos‬و‬ ‫‪، sin ، cos‬‬ ‫‪ •4‬إﺳﺘﻨﺘﺞ ﻗﻴﻤﺔ ﻛﻞ ﻣﻦ ‪:‬‬
‫‪12‬‬ ‫‪12‬‬ ‫‪12‬‬ ‫‪12‬‬

‫‪ •1‬ﻟﺪﻳﻨﺎ ‪:‬‬ ‫ﺍﳊﻞّ‪.‬‬


‫‪ņőƱŀ Ţ‬‬
‫‪√3 1‬‬ ‫𝜋‬ ‫𝜋‬ ‫𝜋‬
‫=𝛼‬ ‫‪+ ı = cos + ı sin = 1,‬‬
‫‪2‬‬ ‫‪2‬‬ ‫‪6‬‬ ‫‪6‬‬ ‫‪6‬‬
‫‪1 √3‬‬ ‫𝜋‪2‬‬ ‫𝜋‪2‬‬ ‫𝜋‪2‬‬
‫‪𝛽=− +‬‬ ‫‪ı = cos‬‬ ‫‪+ ı sin‬‬ ‫‪= 1,‬‬ ‫و‬
‫‪2‬‬ ‫‪2‬‬ ‫‪3‬‬ ‫‪3‬‬ ‫‪3‬‬
‫‪VI‬‬

‫أﻧﻈﺮ اﻟﺸﻜﻞ ‪. 20.VI‬‬


‫‪ُ •2‬ﳑ ﱢﻴﺰ اﳌﻌﺎدﻟﺔ ﻫﻮ ‪:‬‬
‫‪1 √3‬‬ ‫‪/‬‬ ‫‪/‬‬
‫‪Δ = 1 − ı√3‬‬ ‫‪+4 1 + ı√3 = 2+2ı√3 = 4‬‬ ‫‪+‬‬ ‫‪ı‬‬ ‫‪= 4e‬‬ ‫‪= 2e‬‬ ‫‪= √3 + ı‬‬
‫‪2‬‬ ‫‪2‬‬
‫‪ƕ‬‬

‫و ﺑﺎﻟﺘﺎﱄ ﳍﺎ ﺣﻼّن ‪:‬‬

‫‪−1 + ı√3 − √3 − ı‬‬


‫= ‪z‬‬ ‫𝛼‪=𝛽−‬‬
‫‪ũŏ‬‬

‫‪2‬‬
‫‪−1 + ı√3 + √3 + ı‬‬
‫= ‪z‬‬ ‫𝛼‪=𝛽+‬‬ ‫و‬
‫‪2‬‬

‫ﻓﺈن ‪) . OM = OB + OA‬ﺷﻜﻞ ‪(20.VI‬‬


‫أن 𝛼 ‪ّ z = 𝛽 +‬‬
‫ﻓﺈن ‪ ، OM = OB − OA‬و ﺑﲈ ّ‬
‫أن 𝛼 ‪ّ z = 𝛽 −‬‬
‫‪ •3‬ﺑﲈ ّ‬
‫‪.‬‬

‫𝟵𝟭𝟮‬

‫‪http ://tinyurl.com/Malki1718‬‬ ‫‪0‬‬


‫‪ .4.VI‬ﲤﺎرﻳﻦ ﻣﺘﻨﻮﻋﺔ‬
‫‪.‬‬
‫ﻟﺪﻳﻨﺎ ‪:‬‬
‫𝜋‬

‫‪Ŕž Ŧ Ľ‬‬
‫‪z =𝛽−𝛼=e‬‬ ‫‪/‬‬ ‫‪−e‬‬ ‫‪/‬‬ ‫‪=e‬‬ ‫‪/‬‬ ‫‪e‬‬ ‫‪/‬‬ ‫‪− e−‬‬ ‫‪/‬‬ ‫‪=e‬‬ ‫‪/‬‬ ‫‪× 2ı sin‬‬
‫‪4‬‬
‫‪/‬‬ ‫‪/‬‬ ‫‪/‬‬ ‫𝜋‪11‬‬
‫‪= √2e‬‬ ‫‪e‬‬ ‫‪= √2e‬‬ ‫‪= √2,‬‬

‫‪œǃ‬‬
‫‪12‬‬
‫‪/‬‬ ‫‪/‬‬ ‫‪/‬‬ ‫‪/‬‬ ‫𝜋‬
‫‪z =𝛽+𝛼=e‬‬ ‫‪+e‬‬ ‫‪=e‬‬ ‫‪e‬‬ ‫‪+ e−‬‬ ‫‪/‬‬ ‫‪=e‬‬ ‫‪/‬‬ ‫‪× 2 cos‬‬ ‫و‬
‫‪4‬‬
‫‪/‬‬ ‫𝜋‪5‬‬
‫‪= √2e‬‬ ‫‪= √2,‬‬

‫‪Ŀ‬‬
‫‪12‬‬

‫‪ •4‬ﻟﺪﻳﻨﺎ ‪:‬‬
‫‪−‬‬
‫) ‪5𝜋 Re (z‬‬ ‫‪√6 − √2‬‬
‫‪cos‬‬ ‫=‬ ‫=‬ ‫=‬
‫‪12‬‬ ‫|| ‪||z‬‬ ‫‪√2‬‬ ‫‪4‬‬
‫‪+‬‬
‫) ‪5𝜋 Im (z‬‬ ‫‪√6 + √2‬‬
‫‪sin‬‬ ‫=‬
‫|| ‪||z‬‬
‫=‬ ‫=‬ ‫و‬
‫‪12‬‬ ‫‪√2‬‬ ‫‪4‬‬
‫‪−‬‬ ‫‪−‬‬
‫) ‪11𝜋 Re (z‬‬ ‫‪√6 + √2‬‬
‫‪cos‬‬ ‫=‬
‫|| ‪||z‬‬
‫=‬ ‫‪=−‬‬ ‫ﺑﺎﳌﺜﻞ ‪:‬‬
‫‪12‬‬ ‫‪√2‬‬ ‫‪4‬‬
‫‪−‬‬
‫) ‪11𝜋 Im (z‬‬ ‫‪√6 − √2‬‬
‫‪sin‬‬ ‫=‬
‫|| ‪||z‬‬
‫=‬ ‫=‬ ‫و‬
‫‪12‬‬ ‫‪√2‬‬ ‫‪4‬‬

‫ﻻ داﻋﻲ ﳊﺴﺎب || ‪ ||z‬و || ‪ ||z‬ﺑﺎﻟﻄﺮﻳﻘﺔ اﳉﱪﻳﺔ ﻷﻧﻨﺎ وﺟﺪﻧﺎ ﻗﻴﻤ َﺘ ْﻴ ِﻬﲈ ﺑﺎﻟﻄﺮﻳﻘﺔ اﳌﺜﻠﺜﻴﺔ ﰲ اﻟﺴﺆال‬ ‫ﻣﻼﺣﻈﺔ ‪⧏ : 34‬‬
‫اﻟﺴﺎﺑﻖ‪.‬‬
‫⧐‬

‫■‬

‫‬ ‫‪.‬‬
‫𝟗𝟒𝟏‬ ‫‪.‬‬
‫‪ņőƱŀ Ţ‬‬
‫𝛼 ﻋﺪد ﺣﻘﻴﻘﻲ ﻳﻨﺘﻤﻲ إﱃ اﳌﺠﺎل ]𝜋 ‪. [0,‬‬

‫‪ •1‬أﻛﺘﺐ ﺑﺪﻻﻟﺔ 𝛼 ‪ cos‬اﻟﻌﺪد ‪. A = sin 2𝛼 − 2 (1 + cos 2𝛼) :‬‬

‫‪VI‬‬
‫‪ّ •2‬‬
‫ﺣﻞ ﰲ اﳌﺠﻤﻮﻋﺔ ‪ ℂ‬اﳌﻌﺎدﻟﺔ ذات اﳌﺠﻬﻮل ‪. 2z (1 + cos 2𝛼) − 2z sin 2𝛼 + 1 = 0 : z‬‬
‫ﻳﺘﻐﲑ 𝛼 ‪.‬‬ ‫‪ •3‬ﱢ‬
‫ﻋﲔ ﳎﻤﻮﻋﺔ ﺻﻮر ﺣﻠﻮل ﻫﺬه اﳌﻌﺎدﻟﺔ ﻋﻨﺪﻣﺎ ّ‬

‫ﺍﳊﻞّ‪.‬‬
‫‪ƕ‬‬

‫‪ •1‬ﻟﺪﻳﻨﺎ ‪:‬‬

‫𝛼 ‪A = sin 2𝛼 − 2 (1 + cos 2𝛼) = (2 sin 𝛼 cos 𝛼) − 2 2 cos 𝛼 = 4 sin 𝛼 cos 𝛼 − 4 cos‬‬


‫𝛼 ‪= 4 cos 𝛼 sin 𝛼 − 1 = 4 cos 𝛼 − cos 𝛼 = −4 cos‬‬
‫‪ũŏ‬‬

‫𝜋‬
‫)ﻷن ]𝜋 ‪ّ (𝛼 ∈ [0,‬‬
‫ﻓﺈن اﳌﻌﺎدﻟﺔ ﺗﺼﺒﺢ ‪ 1 = 0‬و ﻫﺬا ﻣﺴﺘﺤﻴﻞ ‪ ،‬إذن‬ ‫=𝛼 ّ‬ ‫إذا ﻛﺎن ‪ 1 + cos 2𝛼 = 0‬أي إذا ﻛﺎن‬ ‫•‬ ‫‪2‬‬
‫‪2‬‬
‫𝜋‬
‫≠ 𝛼 ﺑﺎﻟﴬورة أي ‪ 1 + cos 2𝛼 ≠ 0‬و ﺑﺎﻟﺘﺎﱄ ﻓﺎﳌﻌﺎدﻟﺔ ﻣﻦ اﻟﺪرﺟﺔ اﻟﺜﺎﻧﻴﺔ‪ُ .‬ﳑ َ ﱢﻴﺰﻫﺎ ﻫﻮ ‪:‬‬
‫‪2‬‬
‫𝛼 ‪Δ = (sin 2𝛼) − 2 (1 + cos 2𝛼) = A = −4 cos 𝛼 = 2ı cos‬‬

‫𝟬𝟮𝟮‬

‫‪http ://tinyurl.com/Malki1718‬‬ ‫‪0‬‬


‫‪ .VI‬ﲤﺎرﻳﻦ ﺗﻄﺒﻴﻘﻴﺔ‬
‫‪.‬‬
‫و ﺑﺎﻟﺘﺎﱄ ﳍﺎ ﺣﻼّن ‪:‬‬

‫‪Ŕž Ŧ Ľ‬‬
‫𝛼 ‪sin 2𝛼 − 2ı cos 𝛼 2 sin 𝛼 cos 𝛼 − 2ı cos‬‬
‫= ‪z‬‬ ‫=‬ ‫𝛼 ‪= −ı + tan‬‬
‫)𝛼‪2 (1 + cos 2‬‬ ‫𝛼 ‪2 cos‬‬
‫𝛼 ‪sin 2𝛼 + 2ı cos 𝛼 2 sin 𝛼 cos 𝛼 + 2ı cos‬‬

‫‪œǃ‬‬
‫= ‪z‬‬ ‫=‬ ‫𝛼 ‪= ı + tan‬‬
‫)𝛼‪2 (1 + cos 2‬‬ ‫𝛼 ‪2 cos‬‬

‫‪ •3‬ﻧﻼﺣﻆ أﻧﻪ ﻣﻬﲈ ﺗﻜﻦ ﻗﻴﻤﺔ 𝛼 ّ‬


‫ﻓﺈن ‪ Im (z ) = −1‬و ﺑﺎﻟﺘﺎﱄ ﻓﺎﻟﻨﻘﻄﺔ ذات اﻟﻼﺣﻘﺔ ‪ z‬ﺗﻨﺘﻤﻲ إﱃ اﳌﺴﺘﻘﻴﻢ اﻟﺬي ﻣﻌﺎدﻟﺘﻪ‬

‫‪Ŀ‬‬
‫ﻳﺘﻐﲑ 𝛼 ‪ّ ،‬‬
‫ﻓﺈن اﻟﻨﻘﻄﺔ ذات اﻟﻼﺣﻘﺔ ‪z‬‬ ‫‪) Im (z) = −1‬أي ‪ . (y = −1‬ﻟﻜﻦ ‪ Re (z ) ⟶ ±∞ :‬و ﺑﺎﻟﺘﺎﱄ ﻋﻨﺪﻣﺎ ّ‬
‫→‬
‫ﲤﺴﺢ ّ‬
‫ﻛﻞ اﳌﺴﺘﻘﻴﻢ اﻟﺬي ﻣﻌﺎدﻟﺘﻪ ‪. y = −1‬‬
‫ﰲ اﻷﺧﲑ‪ ،‬ﳎﻤﻮﻋﺔ ﺻﻮر اﻟﻨﻘﻄﺔ ذات اﻟﻼﺣﻘﺔ ‪ z‬ﻫﻲ اﳌﺴﺘﻘﻴﻢ اﻟﺬي ﻣﻌﺎدﻟﺘﻪ ‪) Im (z) = −1‬أي ‪. ( y = −1‬‬
‫ﺑﺎﳌﺜﻞ‪ ،‬ﻣﻬﲈ ﺗﻜﻦ ﻗﻴﻤﺔ 𝛼 ّ‬
‫ﻓﺈن ‪ Im (z ) = 1‬و ﺑﺎﻟﺘﺎﱄ ﻓﺎﻟﻨﻘﻄﺔ ذات اﻟﻼﺣﻘﺔ ‪ z‬ﺗﻨﺘﻤﻲ إﱃ اﳌﺴﺘﻘﻴﻢ اﻟﺬي ﻣﻌﺎدﻟﺘﻪ‬
‫ﻳﺘﻐﲑ 𝛼 ‪ّ ،‬‬
‫ﻓﺈن اﻟﻨﻘﻄﺔ ذات اﻟﻼﺣﻘﺔ ‪ z‬ﲤﺴﺢ‬ ‫‪) Im (z) = 1‬أي ‪ . (y = 1‬ﻟﻜﻦ ‪ Re (z ) ⟶ ±∞ :‬و ﺑﺎﻟﺘﺎﱄ ﻋﻨﺪﻣﺎ ّ‬
‫→‬
‫ّ‬
‫ﻛﻞ اﳌﺴﺘﻘﻴﻢ اﻟﺬي ﻣﻌﺎدﻟﺘﻪ ‪. y = 1‬‬
‫ﰲ اﻷﺧﲑ‪ ،‬ﳎﻤﻮﻋﺔ ﺻﻮر اﻟﻨﻘﻄﺔ ذات اﻟﻼﺣﻘﺔ ‪ z‬ﻫﻲ اﳌﺴﺘﻘﻴﻢ اﻟﺬي ﻣﻌﺎدﻟﺘﻪ ‪) Im (z) = 1‬أي ‪. ( y = 1‬‬
‫■‬

‫‪. f(z) = 8z − 8z + 27z − 27‬‬ ‫)‪ f(z‬ﻛﺜﲑ ﺣﺪود ﻟﻠﻤﺘﻐﲑ اﳌﺮﻛﺐ ‪ z‬ﺣﻴﺚ ‪:‬‬
‫‬ ‫‪.‬‬
‫𝟎𝟓𝟏‬ ‫‪.‬‬

‫‪ •1‬ﱢﺑﲔ أﻧﻪ ﻳﻤﻜﻦ ﻛﺘﺎﺑﺔ )‪ f(z‬ﻋﲆ اﻟﺸﻜﻞ )‪ (z − 1) ⋅ g(z‬ﺣﻴﺚ )‪ g(z‬ﻛﺜﲑ ﺣﺪود ﻣﻦ اﻟﺪرﺟﺔ اﻟﺜﺎﻟﺜﺔ ُﻳﻄﻠﺐ ﺗﻌﻴﻴﻨﻪ‪.‬‬
‫ﻋﲔ اﻟﻄﻮﻳﻠﺔ و ﻋﻤﺪة ﻟﻜﻞ ّ‬
‫ﺣﻞ ﻣﻦ ﻫﺬه اﳊﻠﻮل‪.‬‬ ‫‪ّ •2‬‬
‫ﺣﻞ ﰲ اﳌﺠﻤﻮﻋﺔ ‪ ℂ‬اﳌﻌﺎدﻟﺔ ‪ . f(z) = 0 :‬ﱢ‬
‫‪ņőƱŀ Ţ‬‬
‫أن ‪ f(1) = 0‬و ﻣﻦ َﺛ ّﻢ ﻧُﺠﺮي اﻟﻘﺴﻤﺔ اﻹﻗﻠﻴﺪﻳﺔ ﻟـِ )‪ f(z‬ﻋﲆ ‪ z − 1‬ﻹﳚﺎد ﺣﺎﺻﻞ‬ ‫ﺍﳊﻞّ‪ •1 .‬ﻳﻤﻜﻦ اﻟﺘﺤﻘﻖ ﺑﺴﻬﻮﻟﺔ ﻣﻦ ّ‬
‫اﻟﻘﺴﻤﺔ )‪ g(z‬؛ ﻟﻜﻦ ﻣﻦ اﳌﻤﻜﻦ إﺗﺒﺎع اﻟﻄﺮﻳﻘﺔ اﻟﺘﺎﻟﻴﺔ ‪:‬‬
‫‪f(z) = 8z − 8z + 27z − 27 = 8z (z − 1) + 27 (z − 1) = (z − 1) 8z + 27‬‬
‫‪VI‬‬

‫إذن ‪. g(z) = 8z + 27‬‬


‫‪ •2‬ﻟﺪﻳﻨﺎ ‪:‬‬
‫‪f(z) = 0 ⟺ (z − 1) 8z + 27 = 0‬‬
‫‪ƕ‬‬

‫‪⟺ z=1‬‬ ‫أو‬ ‫‪8z + 27 = 0‬‬


‫‪3‬‬
‫‪⟺ z=1‬‬ ‫أو‬ ‫= ‪z‬‬ ‫‪−‬‬
‫‪2‬‬
‫‪3‬‬ ‫‪3‬‬ ‫‪3‬‬
‫‪ũŏ‬‬

‫‪⟺ z=1‬‬ ‫أو‬ ‫‪z=−‬‬ ‫أو‬ ‫‪z=− j‬‬ ‫أو‬ ‫‪z=− j‬‬
‫‪2‬‬ ‫‪2‬‬ ‫‪2‬‬

‫𝟭𝟮𝟮‬

‫‪http ://tinyurl.com/Malki1718‬‬ ‫‪0‬‬


‫‪ .4.VI‬ﲤﺎرﻳﻦ ﻣﺘﻨﻮﻋﺔ‬
‫‪.‬‬
‫‪ j = z‬ﺟﺬر ﺗﻜﻌﻴﺒﻲ ﻟﻠﻮﺣﺪة ؛ و ﺑﺎﻟﺘﺎﱄ ‪:‬‬ ‫‪/‬‬ ‫ﺣﻴﺚ‬

‫‪Ŕž Ŧ Ľ‬‬
‫‪z=1=e‬‬ ‫]‪= [1, 0‬‬
‫‪3 3‬‬ ‫‪3‬‬
‫= ‪z=− = e‬‬ ‫𝜋‪,‬‬ ‫أو‬
‫‪2 2‬‬ ‫‪2‬‬

‫‪œǃ‬‬
‫‪3‬‬ ‫‪3‬‬ ‫‪3‬‬ ‫‪3‬‬ ‫𝜋 ‪3‬‬
‫‪z=− j= e e / = e‬‬ ‫‪/‬‬ ‫‪= e−‬‬ ‫‪/‬‬ ‫=‬ ‫‪,−‬‬ ‫أو‬
‫‪2‬‬ ‫‪2‬‬ ‫‪2‬‬ ‫‪2‬‬ ‫‪2 3‬‬
‫‪3‬‬ ‫‪3‬‬ ‫‪3‬‬ ‫‪/ = 3e‬‬ ‫𝜋 ‪3‬‬
‫‪z=− j = e e / = e‬‬ ‫‪/‬‬ ‫=‬ ‫‪,‬‬ ‫أو‬

‫‪Ŀ‬‬
‫‪2‬‬ ‫‪2‬‬ ‫‪2‬‬ ‫‪2‬‬ ‫‪2 3‬‬
‫■‬

‫ﻟﺘﻜﻦ اﳌﻌﺎدﻟﺔ ‪. z + (−1 + 5ı) z + (−7 − 4ı) z + 3 − 3ı = 0 :‬‬


‫‬ ‫‪.‬‬
‫𝟏𝟓𝟏‬ ‫‪.‬‬

‫‪ •1‬ﱢﺑﲔ ّ‬
‫أن ﻫﺬه اﳌﻌﺎدﻟﺔ ﺗﻘﺒﻞ ﺣﻼ ﲣﻴﻠﻴﺎ ﴏﻓﺎ ُﻳﻄﻠﺐ ﺗﻌﻴﻴﻨﻪ‪.‬‬
‫‪ّ •2‬‬
‫ﺣﻞ ﰲ اﳌﺠﻤﻮﻋﺔ ‪ ℂ‬ﻫﺬه اﳌﻌﺎدﻟﺔ‪.‬‬
‫‪ •3‬ﻧﺴﻤﻲ ‪ z ، z ، z‬ﺣﻠﻮل ﻫﺬه اﳌﻌﺎدﻟﺔ ﺣﻴﺚ || ‪. ||z || < ||z || < ||z‬‬
‫ﻋﲔ اﻟﺸﻜﻞ اﳌﺜﻠﺜﻲ ﻟﻜﻞ ﻣﻦ ‪ z ، z‬و ‪. z‬‬
‫)ا( ﱢ‬
‫‪z‬‬
‫‪.‬‬ ‫)ب( أﺣﺴﺐ اﻟﻌﺪد ‪:‬‬
‫‪√2‬‬

‫ﺍﳊﻞّ‪ .‬ﻧﻀﻊ ‪. f (z) = z + (−1 + 5ı) z + (−7 − 4ı) z + 3 − 3ı :‬‬

‫‪ •1‬ﻟﻴﻜﻦ 𝛼‪ z = ı‬ﺣﻼ ﲣﻴﻠﻴﺎ ﴏﻓﺎ ﻟﻠﻤﻌﺎدﻟﺔ‪ .‬ﻟﺪﻳﻨﺎ ‪:‬‬


‫‪ņőƱŀ Ţ‬‬
‫‪z + (−1 + 5ı) z + (−7 − 4ı) z + 3 − 3ı = (ı𝛼) + (−1 + 5ı) (ı𝛼) + (−7 − 4ı) (ı𝛼) + 3 − 3ı‬‬
‫𝛼‪= −ı𝛼 + (−1 + 5ı) −‬‬ ‫‪− 7ı𝛼 + 4𝛼 + 3 − 3ı‬‬

‫‪VI‬‬
‫‪= 𝛼 + 4𝛼 + 3 + ı −𝛼 − 5𝛼 − 7𝛼 − 3‬‬

‫و ﺑﺎﻟﺘﺎﱄ ‪:‬‬
‫‪𝛼 + 4𝛼 + 3 = 0‬‬
‫⟺ ‪z + (−1 + 5ı) z + (−7 − 4ı) z + 3 − 3ı = 0‬‬
‫‪−𝛼 − 5𝛼 − 7𝛼 − 3 = 0‬‬
‫‪ƕ‬‬

‫أن ِﻛ َﻠ ْﻴﻬﲈ ّ‬
‫ﺣﻞ ﻟﻠﻤﻌﺎدﻟﺔ اﻟﺜﺎﻧﻴﺔ‪.‬‬ ‫ﻟﻠﻤﻌﺎدﻟﺔ اﻷوﱃ ﺣﻼّن ﳘﺎ ‪ 𝛼 = −3‬و ‪ 𝛼 = −1‬و ﻧﺘﺤﻘﻖ ﺑﺴﻬﻮﻟﺔ ّ‬
‫و ﺑﺎﻟﺘﺎﱄ‪ ،‬ﻟﻠﻤﻌﺎدﻟﺔ ‪ f (z) = 0‬ﺣﻼّن ﲣﻴﻠﻴﺎن ﴏﻓﺎن ﳘﺎ ‪ z = −ı :‬و ‪. z = −3ı‬‬
‫‪ •2‬ﻧﻜﺘﺐ ‪ ، f (z) = a (z + ı) (z + 3ı) (z − 𝛽) :‬ﻧﻨﴩ و ﻧﺮﺗﺐ ‪:‬‬
‫‪ũŏ‬‬

‫)𝛽 ‪f (z) = a (z + ı) (z + 3ı) (z −‬‬


‫‪= az + (−ab + 4aı) z + (−3a − 4ıab) z + 3ab‬‬
‫‪= z + (−1 + 5ı) z + (−7 − 4ı) z + 3 − 3ı‬‬

‫ﺑﺎﳌﻄﺎﺑﻘﺔ ﻳﻨﺘﺞ ‪ −3a − 4ıab = −7 − 4ı ، −ab + 4aı = −1 + 5ı ، a = 1 :‬و ‪ 3ab = 3 − 3ı‬؛ أي ‪ a = 1‬و‬
‫‪.b = 1−ı‬‬

‫𝟮𝟮𝟮‬

‫‪http ://tinyurl.com/Malki1718‬‬ ‫‪0‬‬


‫‪ .VI‬ﲤﺎرﻳﻦ ﺗﻄﺒﻴﻘﻴﺔ‬
‫‪.‬‬
‫و ﺑﺎﻟﺘﺎﱄ ‪. f (z) = (z + ı) (z + 3ı) (z − 1 + ı) :‬‬
‫إذن‪ ،‬ﺣﻠﻮل اﳌﻌﺎدﻟﺔ ‪ f (z) = 0‬ﻫﻲ ‪ z = 1 − ı ، z = ı :‬و ‪ . z = 3ı‬ﻟﺪﻳﻨﺎ ‪. ||z || < ||z || < ||z || :‬‬

‫‪Ŕž Ŧ Ľ‬‬
‫)ا( ﻟﺪﻳﻨﺎ ‪:‬‬

‫‪œǃ‬‬
‫𝜋‬ ‫𝜋‬ ‫𝜋‬
‫‪z = −ı = cos −‬‬ ‫‪+ ı sin −‬‬ ‫‪= 1, −‬‬
‫‪2‬‬ ‫‪2‬‬ ‫‪2‬‬
‫‪√2‬‬ ‫‪√2‬‬ ‫𝜋‬ ‫𝜋‬ ‫𝜋‬
‫‪z = 1 − ı = √2‬‬ ‫‪−ı‬‬ ‫‪= √2 cos −‬‬ ‫‪+ ı sin −‬‬ ‫‪= √2, −‬‬
‫‪2‬‬ ‫‪2‬‬ ‫‪4‬‬ ‫‪4‬‬ ‫‪4‬‬

‫‪Ŀ‬‬
‫𝜋‬ ‫𝜋‬ ‫𝜋‬
‫‪z = −3ı = 3 cos −‬‬ ‫‪+ ı sin −‬‬ ‫‪= 3, −‬‬
‫‪2‬‬ ‫‪2‬‬ ‫‪2‬‬
‫)ب( ﻧﻼﺣﻆ ّ‬
‫أن ‪ 2014 = 4 × 504 − 2‬و ﺑﺎﻟﺘﺎﱄ ‪:‬‬

‫‪z‬‬ ‫‪√2e−‬‬ ‫‪/‬‬


‫=‬ ‫‪= e−‬‬ ‫‪/‬‬
‫‪√2‬‬ ‫‪√2‬‬
‫‪= e−‬‬ ‫‪+‬‬ ‫‪=e‬‬ ‫‪/‬‬ ‫‪=ı‬‬

‫■‬

‫= ‪.z‬‬ ‫‪2 + √2 + ı‬‬ ‫أﺣﺴﺐ اﻟﻄﻮﻳﻠﺔ و ﻋﻤﺪة ﻟﻠﻌﺪد ‪2 − √2 :‬‬


‫‬ ‫‪.‬‬
‫𝟐𝟓𝟏‬ ‫‪.‬‬

‫= |‪ |z‬ﻣﻨﻪ ‪. |z| = 2‬‬ ‫‪2 + √2‬‬ ‫‪+‬‬ ‫‪2 − √2‬‬ ‫اﻟﻄﻮﻳﻠﺔ ‪= 2 + √2 + 2 − √2 = 4 :‬‬ ‫ﺍﳊﻞّ‪.‬‬
‫𝜋‬
‫‪.‬‬ ‫‪0,‬‬ ‫اﻟﻌﻤﺪة ‪ :‬ﻟﻴﻜﻦ 𝜃 ﻋﻤﺪ ًة ﻟﻠﻌﺪد ‪ . z‬ﺑﲈ ّ‬
‫أن ‪ Re (z) ≥ 0‬و ‪ Im (z) ≥ 0‬ﻓﺒﺈﻣﻜﺎﻧﻨﺎ إﺧﺘﻴﺎر 𝜃 ﰲ اﳌﺠﺎل‬
‫‪2‬‬
‫‪Re (z) 1‬‬
‫= 𝜃 ‪ cos‬ﻣﻨﻪ ‪:‬‬
‫=‬ ‫ﻟﺪﻳﻨﺎ ‪2 + √2‬‬
‫|‪|z‬‬ ‫‪2‬‬
‫‪1‬‬ ‫‪√2‬‬
‫‪ņőƱŀ Ţ‬‬
‫= ‪cos (2𝜃) = 2 cos 𝜃 − 1‬‬ ‫= ‪2 + √2 − 1‬‬
‫‪2‬‬ ‫‪2‬‬
‫𝜋‬ ‫𝜋‬ ‫𝜋‬
‫ﻣﻊ ]𝜋 ‪ 2𝜃 ∈ [0,‬إذن = 𝜃‪ 2‬أي = 𝜃 ‪ .‬ﰲ اﻷﺧﲑ ‪. arg (z) = (mod 2𝜋) :‬‬
‫‪8‬‬ ‫‪8‬‬ ‫‪4‬‬
‫)‪𝜋 Im (z‬‬ ‫)‪𝜋 Re (z‬‬
‫= ‪ sin‬أي ‪:‬‬ ‫= ‪ cos‬و‬ ‫ﻧﺴﺘﻨﺘﺞ ّ‬
‫أن ‪:‬‬
‫‪VI‬‬

‫‪8‬‬ ‫|‪|z‬‬ ‫‪8‬‬ ‫|‪|z‬‬


‫𝜋‬ ‫‪2 − √2‬‬ ‫𝜋‬ ‫‪2 + √2‬‬
‫■‬ ‫‪sin‬‬ ‫=‬ ‫و‬ ‫‪cos‬‬ ‫=‬
‫‪8‬‬ ‫‪2‬‬ ‫‪8‬‬ ‫‪2‬‬

‫‬ ‫‪.‬‬
‫‪ƕ‬‬

‫‪.‬‬
‫𝟑𝟓𝟏‬
‫= 𝛼‪.‬‬ ‫‪2 − √2 − ı‬‬ ‫𝛼 ﻋﺪد ﻣﺮﻛﺐ ﺣﻴﺚ ‪2 + √2 :‬‬

‫‪ •1‬أﺣﺴﺐ 𝛼 و 𝛼 ّ‬
‫ﺛﻢ اﻛﺘﺐ اﻟﻌﺪد 𝛼 ﻋﲆ ﺷﻜﻠﻪ اﳌﺜﻠﺜﻲ‪.‬‬
‫‪ũŏ‬‬

‫‪ •2‬إﺳﺘﻨﺘﺞ اﻟﻄﻮﻳﻠﺔ و ﻋﻤﺪة ﻟﻠﻌﺪد 𝛼 ‪.‬‬


‫𝜋‪13‬‬ ‫𝜋‪13‬‬
‫‪. sin‬‬ ‫‪ cos‬و‬ ‫‪ •3‬أﺣﺴﺐ ﻛﻼ ﻣﻦ ‪:‬‬
‫‪8‬‬ ‫‪8‬‬
‫‪ •4‬ﻟﺘﻜﻦ ‪ M‬ﺻﻮرة اﻟﻌﺪد ‪ z‬ﰲ اﳌﺴﺘﻮي اﳌﻨﺴﻮب إﱃ ﻣﻌﻠﻢ ﻣﺘﻌﺎﻣﺪ و ﻣﺘﺠﺎﻧﺲ‪ .‬ﱢ‬
‫ﻋﲔ ﳎﻤﻮﻋﺔ اﻟﻨﻘﻂ ‪ M‬ﺑﺤﻴﺚ ﻳﻜﻮن ‪:‬‬
‫‪. |𝛼z| = 8‬‬

‫𝟯𝟮𝟮‬

‫‪http ://tinyurl.com/Malki1718‬‬ ‫‪0‬‬


‫‪ .4.VI‬ﲤﺎرﻳﻦ ﻣﺘﻨﻮﻋﺔ‬
‫‪.‬‬
‫‪ •1‬ﻟﺪﻳﻨﺎ ‪:‬‬ ‫ﺍﳊﻞّ‪.‬‬

‫‪Ŕž Ŧ Ľ‬‬
‫= 𝛼‬ ‫‪2 − √2 − ı 2 + √2‬‬

‫‪œǃ‬‬
‫=‬ ‫‪2 − √2 − 2ı 2 − √2‬‬ ‫‪2 + √2 −‬‬ ‫‪2 + √2‬‬

‫‪= 2 − √2 − 2ı‬‬ ‫‪2 − √2‬‬ ‫‪2 + √2 − 2 − √2‬‬

‫‪Ŀ‬‬
‫‪= −2√2 − 2ı√4 − 2 = −2√2 − 2ı√2‬‬

‫‪𝛼 = −2√2 − 2ı√2‬‬ ‫‪= −2√2‬‬ ‫)‪(1 + ı‬‬ ‫و‬


‫‪= 8 (1 + 2ı − 1) = 16ı‬‬
‫𝜋‬
‫‪= 16e / = 16,‬‬ ‫و اﻟﺸﻜﻞ اﳌﺜﻠﺜﻲ ‪:‬‬
‫‪2‬‬
‫𝜋‬ ‫𝜋‬
‫ﻧﻀﻊ ]𝜃 ‪ 𝛼 = [𝜌,‬ﻓﻴﻜﻮن ‪ 𝛼 = 𝜌 , 4𝜃 = 16,‬و ﺑﺎﻟﺘﺎﱄ ‪ 𝜌 = 16‬و )𝜋‪ 4𝜃 = (mod 2‬أي ‪ 𝜌 = 2‬و‬ ‫•‬ ‫‪2‬‬
‫‪2‬‬ ‫‪2‬‬
‫𝜋‪3‬‬ ‫𝜋‬ ‫𝜋‬
‫= 𝜃‪ .‬ﻟﻜﻦ ‪ Re (𝛼) > 0‬و ‪ Im (𝛼) < 0‬ﻣﻨﻪ ‪ cos 𝜃 > 0‬و ‪ sin 𝜃 < 0‬و ﺑﺎﻟﺘﺎﱄ 𝜋‪ < 𝜃 < 2‬ﻣﻨﻪ‬ ‫‪mod‬‬
‫‪2‬‬ ‫‪8‬‬ ‫‪2‬‬
‫𝜋‪13‬‬ ‫𝜋 𝜋‬
‫= 𝜃‪.‬‬ ‫)𝜋‪ 𝜃 = + (mod 2‬أي )𝜋‪(mod 2‬‬
‫‪8‬‬ ‫‪8 2‬‬
‫𝜋‪13‬‬
‫‪. 𝛼 = 2,‬‬ ‫ﰲ اﻷﺧﲑ‪،‬‬
‫‪8‬‬
‫‪ •3‬ﻟﺪﻳﻨﺎ ‪:‬‬
‫𝜋‪13‬‬ ‫𝜋‪13‬‬
‫=𝛼‬ ‫‪2 − √2 − ı 2 + √2 = 2 cos‬‬ ‫‪+ ı sin‬‬
‫‪8‬‬ ‫‪8‬‬

‫𝜋‪13‬‬ ‫‪2 + √2‬‬ ‫𝜋‪13‬‬ ‫‪2 − √2‬‬


‫‪. sin‬‬ ‫‪=−‬‬ ‫و‬ ‫‪cos‬‬ ‫=‬ ‫ﻣﻨﻪ ‪:‬‬
‫‪8‬‬ ‫‪2‬‬ ‫‪8‬‬ ‫‪2‬‬
‫‪ņőƱŀ Ţ‬‬
‫‪ •4‬ﻟﺪﻳﻨﺎ ‪:‬‬
‫‪|𝛼z| = 8 ⟺ |𝛼| ⋅ |z| = 8 ⟺ 2 |z| = 8 ⟺ |z| = 4‬‬
‫و ﻫﻲ ﻣﻌﺎدﻟﺔ اﻟﺪاﺋﺮة )‪ 𝒞 (O, 4‬اﻟﺘﻲ ﻣﺮﻛﺰﻫﺎ )‪ O (0, 0‬و ﻧﺼﻒ ﻗﻄﺮﻫﺎ ‪. 4‬‬
‫■‬

‫‪VI‬‬
‫‬ ‫‪.‬‬
‫𝟒𝟓𝟏‬ ‫‪.‬‬

‫‪ •1‬أﻛﺘﺐ اﻟﻌﺪد اﳌﺮﻛﺐ )‪ (−1 − ı‬ﻋﲆ ﺷﻜﻠﻪ اﳌﺜﻠﺜﻲ‪.‬‬


‫‪ƕ‬‬

‫‪(1 − 3ı) z + 3 + ı‬‬


‫‪.‬‬ ‫‪=z‬‬ ‫‪ّ •2‬‬
‫ﺣﻞ ﰲ اﳌﺠﻤﻮﻋﺔ ‪ ℂ‬اﳌﻌﺎدﻟﺔ ذات اﳌﺠﻬﻮل اﳌﺮﻛﺐ ‪: z‬‬
‫‪z−ı‬‬
‫‪ũŏ‬‬

‫‪ •3‬ﻟﻴﻜﻦ ‪ّ z‬‬
‫اﳊﻞ ذا أﺻﻐﺮ ﻃﻮﻳﻠﺔ ﻟﻠﻤﻌﺎدﻟﺔ اﻟﺴﺎﺑﻘﺔ‪.‬‬
‫‪z‬‬
‫‪.‬‬ ‫)ا( أﺣﺴﺐ اﻟﻌﺪد اﳌﺮﻛﺐ ‪:‬‬
‫‪√2‬‬
‫‪z‬‬
‫ﻋﺪد ًا ﺣﻘﻴﻘﻴ ًﺎ‪.‬‬ ‫ﻋﲔ ِﻗ َﻴﻢ اﻟﻌﺪد اﻟﻄﺒﻴﻌﻲ ‪ n‬ﺣﺘﻰ ﻳﻜﻮن‬
‫)ب( ﱢ‬
‫‪√2‬‬

‫𝟰𝟮𝟮‬

‫‪http ://tinyurl.com/Malki1718‬‬ ‫‪0‬‬


‫‪ .VI‬ﲤﺎرﻳﻦ ﺗﻄﺒﻴﻘﻴﺔ‬
‫‪.‬‬
‫‪ •1‬ﻟﺪﻳﻨﺎ ‪:‬‬ ‫ﺍﳊﻞّ‪.‬‬

‫‪Ŕž Ŧ Ľ‬‬
‫‪√2‬‬ ‫‪√2‬‬ ‫𝜋‪3‬‬ ‫𝜋‪3‬‬ ‫𝜋‪3‬‬
‫‪−1 − ı = √2 −‬‬ ‫‪−ı‬‬ ‫‪= √2 cos −‬‬ ‫‪+ ı sin −‬‬ ‫‪= √2, −‬‬
‫‪2‬‬ ‫‪2‬‬ ‫‪4‬‬ ‫‪4‬‬ ‫‪4‬‬

‫‪œǃ‬‬
‫ﻣﻌﺮﻓﺔ إذا و ﻓﻘﻂ إذا ﻛﺎن ‪ . z ≠ ı‬ﻧﻔﺮض إذن ّ‬
‫أن ‪ z ≠ ı‬ﻓﻴﻜﻮن ‪:‬‬ ‫‪ •2‬اﳌﻌﺎدﻟﺔ ّ‬
‫‪(1 − 3ı) z + 3 + ı‬‬
‫‪= z ⟺ z (z − ı) = (1 − 3ı) z + 3 + ı‬‬
‫‪z−ı‬‬

‫‪Ŀ‬‬
‫‪⟺ z − ız − (1 − 3ı) z − 3 − ı = 0‬‬
‫‪⟺ z − (1 − 2ı) z − 3 − ı = 0‬‬

‫و ﻫﻲ ﻣﻌﺎدﻟﺔ ﻣﻦ اﻟﺪرﺟﺔ اﻟﺜﺎﻧﻴﺔ‪ُ ،‬ﳑ َ ﱢﻴﺰﻫﺎ ‪:‬‬


‫‪Δ = (− (1 − 2ı)) − 4 (−3 − ı) = 9 = 3‬‬
‫‪1 − 2ı + 3‬‬ ‫‪1 − 2ı − 3‬‬
‫= ‪.z‬‬ ‫= ‪z‬و‪= 2−ı‬‬ ‫و ﺑﺎﻟﺘﺎﱄ ﳍﺎ ﺣﻼّن ﳘﺎ ‪= −1 − ı :‬‬
‫‪2‬‬ ‫‪2‬‬
‫ﻟﺪﻳﻨﺎ ‪ ||z || = √2 :‬و ‪. ||z || = √5‬‬
‫أن ‪ّ 2014 = 4 × 504 − 2‬‬
‫ﻓﺈن ‪:‬‬ ‫)ا( ﺑﲈ ّ‬ ‫•‬‫‪3‬‬
‫‪z‬‬ ‫‪√2e−‬‬ ‫‪/‬‬
‫=‬ ‫‪= e−‬‬ ‫×‬ ‫‪/‬‬ ‫‪= e−‬‬ ‫‪−‬‬ ‫‪/‬‬ ‫‪=e‬‬ ‫‪/‬‬ ‫‪= −ı‬‬
‫‪√2‬‬ ‫‪√2‬‬

‫)ب( ﻟﻴﻜﻦ ‪ n‬ﻋﺪد ًا ﻃﺒﻴﻌﻴ ًﺎ‪ .‬ﻟﺪﻳﻨﺎ ‪:‬‬


‫‪z‬‬ ‫‪√2e−‬‬ ‫‪/‬‬ ‫𝜋‪3n‬‬ ‫𝜋‪3n‬‬
‫=‬ ‫‪= e−‬‬ ‫‪/‬‬ ‫‪= cos −‬‬ ‫‪+ ı sin −‬‬
‫‪√2‬‬ ‫‪√2‬‬ ‫‪4‬‬ ‫‪4‬‬

‫𝜋‪3n‬‬ ‫‪z‬‬
‫‪ sin −‬أي ﳚﺐ أن ﻳﻜﻮن‬ ‫ﻋﺪد ًا ﺣﻘﻴﻘﻴ ًﺎ ﳚﺐ أن ﻳﻜﻮن ‪= 0‬‬ ‫إذن‪ ،‬ﺣﺘﻰ ﻳﻜﻮن‬
‫‪4‬‬ ‫‪√2‬‬
‫𝜋‪3n‬‬
‫‪ņőƱŀ Ţ‬‬
‫أي )‪ 3n = 0 (mod 4‬أي )‪. n = 0 (mod 4‬‬ ‫)𝜋 ‪= 0 (mod‬‬
‫‪4‬‬
‫‪z‬‬
‫ﻋﺪد ًا ﺣﻘﻴﻘﻴ ًﺎ ﻫﻲ ﻣﻀﺎﻋﻔﺎت اﻟﻌﺪد ‪. 4‬‬ ‫إذن‪ ،‬اﻷﻋﺪاد اﻟﻄﺒﻴﻌﻴﺔ ‪ n‬اﻟﺘﻲ ﻣﻦ أﺟﻠﻬﺎ ﻳﻜﻮن‬
‫‪√2‬‬
‫■‬
‫‪VI‬‬

‫= ‪.L‬‬
‫𝛽‪𝛼ı − 4‬‬
‫𝛼 و 𝛽 ﻋﺪدان ﺣﻘﻴﻘﻴﺎن‪ L .‬ﻋﺪد ﻣﺮﻛﺐ ﺣﻴﺚ ‪:‬‬
‫‬ ‫‪.‬‬
‫𝟓𝟓𝟏‬ ‫‪.‬‬
‫‪5 + 3ı‬‬
‫‪ƕ‬‬

‫𝜋‪3‬‬
‫= )‪. arg (L‬‬ ‫‪ •1‬أوﺟﺪ 𝛼 و 𝛽 ﺑﺤﻴﺚ ﻳﻜﻮن ‪ ||L|| = 1‬و‬
‫‪4‬‬
‫‪. 𝛼 = 𝛽 = √2‬‬ ‫أن‬
‫ﻧﻔﺮض ﻓﻴﲈ ﻳﲇ ّ‬
‫‪ũŏ‬‬

‫‪.L +L‬‬ ‫‪ •2‬أﺣﺴﺐ ‪:‬‬


‫‪.L‬‬ ‫‪+L‬‬ ‫‪ n‬ﻋﺪد ﻃﺒﻴﻌﻲ زوﺟﻲ و ‪ m‬ﻋﺪد ﻃﺒﻴﻌﻲ ﻓﺮدي‪ .‬أﺛﺒﺖ ّ‬
‫أن ‪= 0 :‬‬ ‫•‬‫‪3‬‬
‫‪ n •4‬ﻋﺪد ﻃﺒﻴﻌﻲ‪ّ .‬‬
‫ﺣﻞ ﰲ ‪ ℂ‬اﳌﻌﺎدﻟﺔ ذات اﳌﺠﻬﻮل اﳌﺮﻛﺐ ‪: z‬‬
‫‪+‬‬ ‫‪+‬‬
‫‪z ⋅L‬‬ ‫‪− 2z ⋅ L‬‬ ‫‪−L‬‬ ‫‪=0‬‬

‫𝟱𝟮𝟮‬

‫‪http ://tinyurl.com/Malki1718‬‬ ‫‪0‬‬


‫‪ .4.VI‬ﲤﺎرﻳﻦ ﻣﺘﻨﻮﻋﺔ‬
‫‪.‬‬
‫ﺍﳊﻞّ‪ .‬ﻟﺪﻳﻨﺎ ‪:‬‬
‫𝛽‪𝛼ı − 4𝛽 (𝛼ı − 4𝛽) (5 − 3ı) 5𝛼ı + 3𝛼 − 20𝛽 + 12𝛽ı 3𝛼 − 20𝛽 5𝛼 + 12‬‬

‫‪Ŕž Ŧ Ľ‬‬
‫=‪L‬‬ ‫=‬ ‫=‬ ‫=‬ ‫‪+‬‬ ‫‪ı‬‬
‫‪5 + 3ı‬‬ ‫‪5 +3‬‬ ‫‪34‬‬ ‫‪34‬‬ ‫‪34‬‬
‫𝜋‪3‬‬

‫‪œǃ‬‬
‫ﻋﻤﺪة ﻟﻪ ﻫﻮ ‪:‬‬
‫اﻟﻌﺪد اﳌﺮﻛﺐ اﻟﺬي ﻃﻮﻳﻠﺘﻪ ‪ 1‬و‬ ‫•‬ ‫‪1‬‬
‫‪4‬‬
‫‪/‬‬ ‫𝜋‪3‬‬ ‫𝜋‪3‬‬ ‫‪√2‬‬ ‫‪√2‬‬
‫‪1⋅e‬‬ ‫‪= cos‬‬ ‫‪+ ı sin‬‬ ‫‪=−‬‬ ‫‪+ı‬‬
‫‪4‬‬ ‫‪4‬‬ ‫‪2‬‬ ‫‪2‬‬

‫‪Ŀ‬‬
‫و ﺑﺎﻟﺘﺎﱄ ‪:‬‬

‫‪√2‬‬ ‫‪√2‬‬ ‫𝛽‪3𝛼 − 20𝛽 5𝛼 + 12‬‬ ‫‪√2‬‬ ‫‪√2‬‬


‫‪L=−‬‬ ‫‪+ı‬‬ ‫⟺‬ ‫‪+‬‬ ‫‪ı=−‬‬ ‫‪+ı‬‬
‫‪2‬‬ ‫‪2‬‬ ‫‪34‬‬ ‫‪34‬‬ ‫‪2‬‬ ‫‪2‬‬
‫⎧‬
‫⎪‬ ‫𝛼‪3‬‬ ‫‪−‬‬ ‫𝛽‪20‬‬ ‫√‬ ‫‪2‬‬
‫⎪‬ ‫‪=−‬‬ ‫‪3𝛼 − 20𝛽 = −17√2‬‬
‫⟺‬ ‫‪34‬‬ ‫⟺ ‪2‬‬
‫⎨‬
‫‪⎪ 5𝛼 + 12𝛽 √2‬‬ ‫‪5𝛼 + 12𝛽 = 17√2‬‬
‫⎪‬ ‫=‬
‫⎩‬ ‫‪34‬‬ ‫‪2‬‬

‫ﺑﺎﳊﻞ‪ ،‬ﻧﺠﺪ أﻧﻪ ﻟﻠﺠﻤﻠﺔ ّ‬


‫ﺣﻞ وﺣﻴﺪ و ﻫﻮ ‪. (𝛼, 𝛽) = √2, √2 :‬‬
‫𝜋‪3‬‬
‫‪ L = 1,‬ﻣﻨﻪ ‪:‬‬ ‫‪ •2‬ﺑﲈ ّ‬
‫أن ‪ 𝛼 = √2‬و ‪ 𝛽 = √2‬ﻓﺤﺴﺐ اﻟﺴﺆال اﻟﺴﺎﺑﻖ‬
‫‪4‬‬

‫‪L‬‬ ‫‪+L‬‬ ‫‪= e‬‬ ‫‪/‬‬ ‫‪+ e‬‬ ‫‪/‬‬ ‫‪=e‬‬ ‫‪+e‬‬ ‫‪= −1 + 1 = 0‬‬

‫‪ •3‬إذا ﻛﺎن ‪ n = 2k‬و ‪ m = 2ℓ + 1‬ﻣﻊ ‪ّ k, ℓ ∈ ℕ‬‬


‫ﻓﺈن ‪:‬‬
‫‪+‬‬ ‫‪/‬‬ ‫‪+‬‬ ‫‪/‬‬ ‫‪+‬‬
‫‪L‬‬ ‫‪+L‬‬ ‫‪=L‬‬ ‫‪+L‬‬ ‫‪=e‬‬ ‫‪+e‬‬ ‫‪=e‬‬ ‫‪+e‬‬
‫‪= e‬‬ ‫‪+ e‬‬ ‫‪+‬‬ ‫‪= 1 + (−1) = 1 − 1 = 0‬‬
‫‪ņőƱŀ Ţ‬‬
‫‪ •4‬ﻟﻴﻜﻦ ‪ n‬ﻋﺪد ًا ﻃﺒﻴﻌﻴ ًﺎ‪ .‬اﳌﻤ ّﻴﺰ اﳌﺨﺘﴫ ﻟﻠﻤﻌﺎدﻟﺔ ﻫﻮ ‪:‬‬
‫‪+‬‬ ‫‪+‬‬ ‫‪+‬‬
‫‪Δ = L‬‬ ‫‪+L‬‬ ‫‪⋅L‬‬ ‫‪=L‬‬ ‫‪+L‬‬

‫‪VI‬‬
‫ﻓﺈن ‪ n + 1‬ﻓﺮدي‪ ،‬و إذا ﻛﺎن ‪ n‬ﻓﺮدﻳ ًﺎ ّ‬
‫ﻓﺈن ‪ n + 1‬زوﺟﻲ(‬ ‫ﻟﻜﻦ ‪ n‬و ‪ n + 1‬ﳐﺘﻠﻔﺎن ﰲ اﻟﺰوﺟﻴﺔ )إذا ﻛﺎن ‪ n‬زوﺟﻴ ًﺎ ّ‬
‫ﺣﻞ ﻣﻀﺎﻋﻒ ‪:‬‬‫ﻓﺈن ‪ Δ = 0‬و ﺑﺎﻟﺘﺎﱄ ﻟﻠﻤﻌﺎدﻟﺔ ّ‬
‫ﻓﺤﺴﺐ اﻟﺴﺆال اﻟﺴﺎﺑﻖ ّ‬
‫‪L‬‬ ‫‪−‬‬ ‫‪/‬‬ ‫‪−‬‬
‫= ‪z =z‬‬ ‫‪+‬‬
‫‪=L‬‬ ‫‪= e‬‬ ‫‪= e−‬‬ ‫‪/‬‬ ‫‪= −ı‬‬
‫‪L‬‬
‫‪ƕ‬‬

‫■‬

‫‬ ‫‪.‬‬ ‫‪.‬‬


‫‪ũŏ‬‬

‫𝟔𝟓𝟏‬
‫اﳌﺴﺘﻮي اﳌﺮﻛﺐ ﻣﻨﺴﻮب إﱃ ﻣﻌﻠﻢ ﻣﺘﻌﺎﻣﺪ و ﻣﺘﺠﺎﻧﺲ ⃗ ‪. O, i,⃗ j‬‬
‫‪z − 𝛼ı‬‬
‫= ‪ L‬ﺣﻴﺚ 𝛼 وﺳﻴﻂ ﺣﻘﻴﻘﻲ ﻏﲑ ﻣﻌﺪوم‪.‬‬ ‫‪ z‬ﻋﺪد ﻣﺮﻛﺐ و ‪ M‬ﺻﻮرﺗﻪ‪ .‬ﻟﻴﻜﻦ ‪ L‬اﻟﻌﺪد اﳌﺮﻛﺐ اﳌﻌﺮف ﺑـِ ‪:‬‬
‫‪z + 𝛼ı‬‬

‫‪ •1‬ﱢ‬
‫ﻋﲔ ﳎﻤﻮﻋﺔ اﻟﻨﻘﻂ ‪ M‬ﺑﺤﻴﺚ ﻳﻜﻮن ‪:‬‬
‫)ا( ‪ L‬ﻋﺪد ًا ﺣﻘﻴﻘﻴﺎ‪.‬‬

‫𝟲𝟮𝟮‬

‫‪http ://tinyurl.com/Malki1718‬‬ ‫‪0‬‬


‫‪ .VI‬ﲤﺎرﻳﻦ ﺗﻄﺒﻴﻘﻴﺔ‬
‫‪.‬‬

‫)ب( ‪ L‬ﻋﺪد ًا ﲣﻴﻠﻴﺎ ﴏﻓﺎ )ﻧﺎﻗﺶ ﺣﺴﺐ ِﻗ َﻴﻢ اﻟﻮﺳﻴﻂ 𝛼( ‪.‬‬

‫‪Ŕž Ŧ Ľ‬‬
‫𝜋‬
‫ﻋﲔ ﳎﻤﻮﻋﺔ اﻟﻨﻘﻂ ‪ M‬ﺑﺤﻴﺚ ﻳﻜﻮن ﻋﻤﺪ ًة ﻟﻠﻌﺪد ‪. L‬‬ ‫‪ •2‬ﻧﻔﺮض ّ‬
‫أن ‪ . 𝛼 = 2‬ﱢ‬
‫‪2‬‬

‫‪œǃ‬‬
‫ﺍﳊﻞّ‪ .‬ﻧﻀﻊ ‪ z = x + ıy‬ﻣﻊ ‪ . x, y ∈ ℝ‬ﺣﺘﻰ ﻳﻜﻮن ‪ّ L‬‬
‫ﻣﻌﺮﻓ ًﺎ‪ ،‬ﳚﺐ أن ﻳﻜﻮن ‪ z + 𝛼ı ≠ 0‬أي ‪. x + (y + 𝛼) ≠ 0‬‬
‫ﻧﻔﺮض إذن ّ‬
‫أن ‪ . x + (y + 𝛼) ≠ 0‬ﻟﺪﻳﻨﺎ ‪:‬‬

‫‪Ŀ‬‬
‫))𝛼 ‪x + ıy − ı𝛼 (x + ı (y − 𝛼)) (x − ı (y +‬‬
‫=‪L‬‬ ‫=‬
‫𝛼‪x + ıy + ı‬‬ ‫)𝛼 ‪x + (y +‬‬
‫)𝛼 ‪x − ı (xy + 𝛼x) + ı (xy − 𝛼x) + (y − 𝛼) (y +‬‬
‫=‬
‫)𝛼 ‪x + (y +‬‬
‫‪x + y − 𝛼 − 2ı𝛼x‬‬ ‫𝛼‪x +y −‬‬ ‫‪2𝛼x‬‬
‫=‬ ‫=‬ ‫‪−‬‬ ‫‪ı‬‬
‫)𝛼 ‪x + (y +‬‬ ‫)𝛼 ‪x + (y +‬‬ ‫)𝛼 ‪x + (y +‬‬
‫‪2𝛼x‬‬
‫‪ −‬أي ‪ . 𝛼x = 0‬ﻟﻜﻦ ‪ 𝛼 ≠ 0‬ﻣﻨﻪ ‪. x = 0‬‬ ‫ﺣﺘﻰ ﻳﻜﻮن ‪ L‬ﺣﻘﻴﻘﻴﺎ ﳚﺐ أن ﻳﻜﻮن ‪= 0‬‬ ‫)ا(‬ ‫‪1‬‬
‫•‬
‫)𝛼 ‪x + (y +‬‬
‫و ﺑﺎﻟﺘﺎﱄ ﳎﻤﻮﻋﺔ اﻟﻨﻘﻂ ‪ M‬اﻟﺘﻲ ﻣﻦ أﺟﻠﻬﺎ ﻳﻜﻮن ‪ L‬ﺣﻘﻴﻘﻴﺎ ﻫﻲ اﳌﺴﺘﻘﻴﻢ اﻟﺬي ﻣﻌﺎدﻟﺘﻪ ‪) x = 0‬ﳏﻮر اﻟﱰاﺗﻴﺐ(‬
‫ﺑﺎﺳﺘﺜﻨﺎء اﻟﻨﻘﻄﺔ ذات اﻹﺣﺪاﺛﻴﲔ )𝛼‪. (0, −‬‬
‫𝛼‪x +y −‬‬
‫أي 𝛼 = ‪ x + y‬و ﻫﻲ ‪:‬‬ ‫ﺣﺘﻰ ﻳﻜﻮن ‪ L‬ﲣﻴﻠﻴﺎ ﴏﻓﺎ ﳚﺐ أن ﻳﻜﻮن ‪= 0‬‬ ‫)ب(‬
‫)𝛼 ‪x + (y +‬‬
‫• ﻣﻌﺎدﻟﺔ اﻟﺪاﺋﺮة )𝛼 ‪ 𝒞 (O,‬اﻟﺘﻲ ﻣﺮﻛﺰﻫﺎ )‪ O (0, 0‬و ﻧﺼﻒ ﻗﻄﺮﻫﺎ 𝛼 ﺑﺎﺳﺘﺜﻨﺎء اﻟﻨﻘﻄﺔ ذات اﻹﺣﺪاﺛﻴﲔ‬
‫)𝛼‪ (0, −‬إذا ﻛﺎن ‪ 𝛼 > 0‬؛‬
‫• و ﻣﻌﺎدﻟﺔ اﻟﺪاﺋﺮة )𝛼‪ 𝒞 (O, −‬اﻟﺘﻲ ﻣﺮﻛﺰﻫﺎ )‪ O (0, 0‬و ﻧﺼﻒ ﻗﻄﺮﻫﺎ 𝛼‪ −‬ﺑﺎﺳﺘﺜﻨﺎء اﻟﻨﻘﻄﺔ ذات اﻹﺣﺪاﺛﻴﲔ‬
‫)𝛼‪ (0, −‬إذا ﻛﺎن ‪. 𝛼 < 0‬‬
‫ﰲ ﻛﻞ اﳊﺎﻻت‪ ،‬ﳎﻤﻮﻋﺔ اﻟﻨﻘﻂ ‪ M‬اﻟﺘﻲ ﻣﻦ أﺟﻠﻬﺎ ﻳﻜﻮن ‪ L‬ﲣﻴﻠﻴﺎ ﴏﻓﺎ ﻫﻲ اﻟﺪاﺋﺮة )|𝛼| ‪ 𝒞 (O,‬اﻟﺘﻲ ﻣﺮﻛﺰﻫﺎ‬
‫‪ņőƱŀ Ţ‬‬
‫)‪ O (0, 0‬و ﻧﺼﻒ ﻗﻄﺮﻫﺎ |𝛼| ﺑﺎﺳﺘﺜﻨﺎء اﻟﻨﻘﻄﺔ ذات اﻹﺣﺪاﺛﻴﲔ )𝛼‪. (0, −‬‬
‫‪ •2‬إذا ﻛﺎن ‪ّ 𝛼 = 2‬‬
‫ﻓﺈن ‪:‬‬
‫‪x +y −4‬‬ ‫‪4x‬‬
‫=‪L‬‬ ‫‪−‬‬ ‫‪ı‬‬
‫‪VI‬‬

‫)‪x + (y + 2‬‬ ‫)‪x + (y + 2‬‬


‫𝜋‬
‫ﻋﻤﺪ ًة ﻟﻠﻌﺪد ‪ L‬ﳚﺐ أن ﻳﻜﻮن ‪ Re (L) = 0‬و ‪ Im (L) > 0‬أي ‪ x + y = 4‬و ‪ −4x > 0‬أي‬ ‫ﺣﺘﻰ ﻳﻜﻮن‬
‫‪2‬‬
‫‪x +y = 2‬و‪.x < 0‬‬
‫𝜋‬
‫ﻋﻤﺪ ًة ﻟﻠﻌﺪد ‪ L‬ﻫﻲ ﻧﺼﻒ اﻟﺪاﺋﺮة )‪ 𝒞 (O, 2‬اﳌﻌﺮﻓﺔ ﻷﺟﻞ ‪x < 0‬‬ ‫و ﺑﺎﻟﺘﺎﱄ ﳎﻤﻮﻋﺔ اﻟﻨﻘﻂ ‪ M‬اﻟﺘﻲ ﻣﻦ أﺟﻠﻬﺎ ﻳﻜﻮن‬
‫‪2‬‬
‫‪ƕ‬‬

‫)اﻟﻨﻘﻄﺔ ذات اﻹﺣﺪاﺛﻴﲔ )‪ (0, −2‬ﻻ ﺗﻨﺘﻤﻲ إﻟﻴﻬﺎ( ‪.‬‬


‫■‬
‫‪ũŏ‬‬

‫‬ ‫‪.‬‬
‫𝟕𝟓𝟏‬ ‫‪.‬‬

‫𝒮‬ ‫‪ •1‬ﻧﻌﺘﱪ ﰲ اﳌﺠﻤﻮﻋﺔ ‪ ℂ‬اﳌﻌﺎدﻟﺔ ‪ (z − m) = m − 1 :‬ﺣﻴﺚ ‪ z‬ﻫﻮ اﳌﺠﻬﻮل و ‪ m‬وﺳﻴﻂ ﺣﻘﻴﻘﻲ‪ .‬ﻧﺴﻤﻲ‬
‫ﳎﻤﻮﻋﺔ ﺣﻠﻮل ﻫﺬه اﳌﻌﺎدﻟﺔ‪.‬‬
‫ﻋﲔ ﺗِﺒﻌ ًﺎ ﻟﻘﻴﻢ اﻟﻮﺳﻴﻂ اﳊﻘﻴﻘﻲ ‪ m‬ﻋﻨﺎﴏ اﳌﺠﻤﻮﻋﺔ 𝒮 ‪.‬‬
‫ﱢ‬

‫𝟳𝟮𝟮‬

‫‪http ://tinyurl.com/Malki1718‬‬ ‫‪0‬‬


‫‪ .4.VI‬ﲤﺎرﻳﻦ ﻣﺘﻨﻮﻋﺔ‬
‫‪.‬‬
‫𝜋‬
‫≤ 𝜃 ≤ ‪ . 0‬ﻧﻌﺘﱪ ﰲ ‪ ℂ‬اﳌﻌﺎدﻟﺔ ‪:‬‬ ‫‪ m‬وﺳﻴﻂ ﻣﺮﻛﺐ و 𝜃 وﺳﻴﻂ ﺣﻘﻴﻘﻲ ﺣﻴﺚ‬ ‫•‬ ‫‪2‬‬
‫‪2‬‬

‫‪Ŕž Ŧ Ľ‬‬
‫𝜃‪(z − m) = cos 2𝜃 + ı sin 2‬‬

‫و ﻧﺮﻣﺰ ﺑـِ ‪ z‬و ‪ z‬إﱃ ﺣ ﱠﻠ ْﻴﻬﺎ‪.‬‬

‫‪œǃ‬‬
‫)ا( أﺣﺴﺐ ‪ z‬و ‪. z‬‬
‫)ب( ﻧﻌﺘﱪ اﻟﻨﻘﻂ ‪ M‬ﻣﻦ اﳌﺴﺘﻮي اﻟﺘﻲ ﻟﻮاﺣﻘﻬﺎ ‪ z − z‬أو ‪. z − z‬‬

‫‪Ŀ‬‬
‫أوﺟﺪ ﳎﻤﻮﻋﺔ اﻟﻨﻘﻂ ‪ M‬ﻋﻨﺪﻣﺎ ﻳﺘﻐﲑ 𝜃 ‪.‬‬
‫‪z +z‬‬
‫‪.‬‬ ‫)ج( ﻧﻔﺮض ّ‬
‫أن ‪ m‬ﻋﺪد ﻣﺮﻛﺐ ﻏﲑ ﻣﻌﺪوم ﻋﻤﺪﺗﻪ ﺛﺎﺑﺘﺔ 𝜃 ‪ .‬أوﺟﺪ ﳎﻤﻮﻋﺔ اﻟﻨﻘﻂ اﻟﺘﻲ ﻟﻮاﺣﻘﻬﺎ‬
‫‪2‬‬

‫‪ •1‬ﻧُﻤ ﱢﻴﺰ ﺛﻼث ﺣﺎﻻت ‪:‬‬ ‫ﺍﳊﻞّ‪.‬‬

‫ﻼ ﻣﻀﺎﻋﻔ ًﺎ ‪ z = m‬ﻣﻨﻪ‬ ‫إذا ﻛﺎن ‪ m = 1‬أي إذا ﻛﺎن ‪ّ m = ±1‬‬


‫ﻓﺈن اﳌﻌﺎدﻟﺔ ﺗﺼﺒﺢ ‪ (z − m) = 0‬ﺗﻘﺒﻞ ﺣ ّ‬ ‫•‬
‫}‪. 𝒮 = {m‬‬
‫‪ (z − m) = √m − 1‬ﻣﻨﻪ اﳊﻼّن‬ ‫إذا ﻛﺎن ‪ m − 1 > 0‬أي إذا ﻛﺎن [∞‪ّ m ∈ ]−∞, −1[ ∪ ]1, +‬‬
‫ﻓﺈن‬ ‫•‬

‫‪ z = m − √m − 1‬و ‪ z = m + √m − 1‬أي ‪. 𝒮 = m − √m − 1, m + √m − 1‬‬

‫= )‪ (z − m‬ﻣﻨﻪ اﳊﻼّن ‪z = m −‬‬ ‫‪ı√1 − m‬‬ ‫إذا ﻛﺎن ‪ m − 1 < 0‬أي [‪ّ m ∈ ]−1, 1‬‬
‫ﻓﺈن‬ ‫•‬

‫‪. 𝒮 = m − ı√1 − m , m + ı√1 − m‬‬ ‫‪ ı√1 − m‬و ‪ z = m + ı√1 − m‬أي‬

‫‪ (z − m) = e‬و ﺑﺎﻟﺘﺎﱄ ﳍﺎ ﺣﻼن ﳘﺎ ‪ z = m − e :‬و ‪. z = m + e‬‬ ‫‪= e‬‬ ‫‪) •2‬ا( اﳌﻌﺎدﻟﺔ ﺗﻜﺎﻓﺊ ‪:‬‬
‫)ب( ﻟﺪﻳﻨﺎ ‪:‬‬
‫‪z −z =m+e −m+e‬‬ ‫‪= 2e‬‬ ‫)𝜃 ‪= 2 (cos 𝜃 + ı sin‬‬
‫ﻋﻨﺪﻣﺎ ﻳﺘﻐﲑ 𝜃 ﰲ اﳌﺠﺎل 𝜋 ‪ّ 0,‬‬
‫ﻓﺈن ‪ ||z − z || = 2‬و ‪ 2 cos 𝜃 ≥ 0‬و ‪ ، 2 sin 𝜃 ≥ 0‬إذن ﻋﻨﺪﻣﺎ ﻳﺘﻐﲑ 𝜃 ﰲ‬
‫‪ņőƱŀ Ţ‬‬
‫‪2‬‬
‫ﻓﺈن ﳎﻤﻮﻋﺔ اﻟﻨﻘﻂ ‪ M‬اﻟﺘﻲ ﻟﻮاﺣﻘﻬﺎ ‪ z − z‬ﻫﻲ رﺑﻊ اﻟﺪاﺋﺮة اﻟﺘﻲ ﻣﻌﺎدﻟﺘﻬﺎ ‪ x + y = 4‬ﻣﻊ‬ ‫اﳌﺠﺎل 𝜋 ‪ّ 0,‬‬
‫‪2‬‬
‫‪x ≥ 0‬و‪y ≥ 0‬؛‬
‫𝜋‬
‫ّ‬
‫ﻓﺈن ‪ ||z − z || = 2‬و ‪ −2 cos 𝜃 ≤ 0‬و‬ ‫‪0,‬‬ ‫ﺑﺎﳌﺜﻞ‪ z − z = −2e ،‬ﻣﻨﻪ ﻋﻨﺪﻣﺎ ﻳﺘﻐﲑ 𝜃 ﰲ اﳌﺠﺎل‬

‫‪VI‬‬
‫‪2‬‬
‫ﻓﺈن ﳎﻤﻮﻋﺔ اﻟﻨﻘﻂ ‪ M‬اﻟﺘﻲ ﻟﻮاﺣﻘﻬﺎ ‪ z − z‬ﻫﻲ رﺑﻊ‬ ‫‪ ، −2 sin 𝜃 ≤ 0‬إذن ﻋﻨﺪﻣﺎ ﻳﺘﻐﲑ 𝜃 ﰲ اﳌﺠﺎل 𝜋 ‪ّ 0,‬‬
‫‪2‬‬
‫اﻟﺪاﺋﺮة اﻟﺘﻲ ﻣﻌﺎدﻟﺘﻬﺎ ‪ x + y = 4‬ﻣﻊ ‪ x ≤ 0‬و ‪y ≤ 0‬‬
‫‪ z − z‬ﻫﻲ رﺑﻌﺎ اﻟﺪاﺋﺮة اﻟﺘﻲ ﻣﻌﺎدﻟﺘﻬﺎ ‪x + y = 4‬‬ ‫ﰲ اﻷﺧﲑ‪ ،‬ﳎﻤﻮﻋﺔ اﻟﻨﻘﻂ ‪ M‬اﻟﺘﻲ ﻟﻮاﺣﻘﻬﺎ ‪ z − z‬أو‬
‫ﻣﻊ ‪) xy ≥ 0‬ﺷﻜﻞ ‪.(21.VI‬‬
‫‪ƕ‬‬

‫‪z +z‬‬
‫‪ .‬ﻧﻀﻊ ‪. m = x + ıy‬‬ ‫ﻟﺪﻳﻨﺎ ‪= m :‬‬ ‫)ج(‬
‫‪2‬‬
‫‪z +z‬‬
‫ﻫﻲ‬ ‫ﻓﺈن ‪ m = ıy‬و ﻫﻲ ﻣﻌﺎدﻟﺔ ﳏﻮر اﻟﱰاﺗﻴﺐ و ﺑﺎﻟﺘﺎﱄ ﳎﻤﻮﻋﺔ اﻟﻨﻘﻂ اﻟﺘﻲ ﻟﻮاﺣﻘﻬﺎ‬ ‫إذا ﻛﺎن ‪ّ x = 0‬‬
‫‪2‬‬
‫‪ũŏ‬‬

‫ﳏﻮر اﻟﱰاﺗﻴﺐ )اﳌﺴﺘﻘﻴﻢ ‪ (y = 0‬ﺑﺎﺳﺘﺜﻨﺎء اﻟﻨﻘﻄﺔ )‪. O (0, 0‬‬


‫‪y‬‬
‫ﻓﺈن 𝜃 ‪ = tan‬و ﻫﻮ ﻋﺪد ﺛﺎﺑﺖ ‪ ،‬أي 𝜃 ‪ y = x tan‬و ﻫﻲ ﻣﻌﺎدﻟﺔ اﳌﺴﺘﻘﻴﻢ اﻟﺬي ﻳﻤﺮ ﺑﺎﳌﺮﻛﺰ‬ ‫و إذا ﻛﺎن ‪ّ x ≠ 0‬‬
‫‪x‬‬
‫و ﻳﺼﻨﻊ زاوﻳﺔ ﺗﺴﺎوي 𝜃 ﻣﻊ ﳏﻮر اﻟﻔﻮاﺻﻞ ﺑﺎﺳﺘﺜﻨﺎء اﳌﺮﻛﺰ )‪. O (0, 0‬‬
‫‪z +z‬‬
‫ﻫﻲ اﳌﺴﺘﻘﻴﻢ اﻟﺬي ﻳﻤﺮ ﺑﺎﳌﺮﻛﺰ و ﻳﺼﻨﻊ زاوﻳﺔ ﺗﺴﺎوي‬ ‫ﰲ ﻛﻞ اﳊﺎﻻت‪ ،‬ﳎﻤﻮﻋﺔ اﻟﻨﻘﻂ اﻟﺘﻲ ﻟﻮاﺣﻘﻬﺎ‬
‫‪2‬‬
‫𝜃 ) = 𝜃 إذا ﻛﺎن ‪ (x = 0‬ﻣﻊ ﳏﻮر اﻟﻔﻮاﺻﻞ ﺑﺎﺳﺘﺜﻨﺎء اﳌﺮﻛﺰ )‪. O (0, 0‬‬
‫■‬

‫𝟴𝟮𝟮‬

‫‪http ://tinyurl.com/Malki1718‬‬ ‫‪0‬‬


‫‪ .VI‬ﲤﺎرﻳﻦ ﺗﻄﺒﻴﻘﻴﺔ‬
‫‪.‬‬
‫‪2‬‬

‫‪Ŕž Ŧ Ľ‬‬
‫‪1‬‬

‫‪œǃ‬‬
‫‪−2‬‬ ‫‪−1‬‬ ‫‪1‬‬ ‫‪2‬‬

‫‪−1‬‬

‫‪Ŀ‬‬
‫‪−2‬‬

‫ﺷﻜﻞ ‪21.VI‬‬

‫‪.‬‬
‫𝟖𝟓𝟏‬ ‫‪.‬‬
‫‪ •1‬أﻛﺘﺐ اﻟﻌﺪد )‪ 4√2 (−1 + ı‬ﻋﲆ اﻟﺸﻜﻞ اﳌﺜﻠﺜﻲ‪.‬‬
‫‪ •2‬ﻧﻌﺘﱪ ﺛﻼﺛﺔ أﻋﺪاد ﻣﺮﻛﺒﺔ ﺟﺪاؤﻫﺎ )‪. 4√2 (−1 + ı‬‬
‫ﻃﻮﻳﻼت ﻫﺬه اﻷﻋﺪاد ﺗُﺸﻜﱢﻞ ﺣﺪود ﻣﺘﺘﺎﺑﻌﺔ ﳌﺘﺘﺎﻟﻴﺔ ﻫﻨﺪﺳﻴﺔ أﺳﺎﺳﻬﺎ ‪ 2‬و ﻋﻤﺪاﲥﺎ ﺣﺪود ﻣﺘﺘﺎﺑﻌﺔ ﳌﺘﺘﺎﻟﻴﺔ ﺣﺴﺎﺑﻴﺔ أﺳﺎﺳﻬﺎ‬
‫𝜋‬
‫‪.‬‬
‫‪4‬‬
‫ﻟﺘﻜﻦ ‪ z ، z‬و ‪ z‬ﻫﺬه اﻷﻋﺪاد ﻣﻊ || ‪. ||z || ≤ ||z || ≤ ||z‬‬
‫𝜋‬
‫‪ ،‬أﺣﺴﺐ ﻃﻮﻳﻠﺔ و ﻋﻤﺪ ًة ﻟﻜﻞ ﻣﻦ ‪ z ، z‬و ‪. z‬‬ ‫ﻋﻠ ًﲈ ّ‬
‫أن 𝜋 ≤ ) ‪≤ arg (z‬‬
‫‪2‬‬
‫‪ •3‬أﻧﺸﺊ اﻟﻨﻘﻂ ‪ M ، M‬و ‪ M‬ﺻﻮر اﻷﻋﺪاد ‪ z ، z‬و ‪ z‬ﻋﲆ اﻟﱰﺗﻴﺐ ﰲ اﳌﺴﺘﻮي اﳌﺮﻛﺐ‪.‬‬

‫ﺍﳊﻞّ‪.‬‬
‫‪ņőƱŀ Ţ‬‬
‫‪ •1‬ﻧﻀﻊ ]𝜃 ‪ . 4√2 (−1 + ı) = [𝜌,‬ﻟﺪﻳﻨﺎ ‪:‬‬

‫‪||4√2 (−1 + ı)|| = ||4√2|| ⋅ ||(−1 + ı)|| = 4√2 × √2 = 8‬‬


‫‪−4√2‬‬ ‫‪√2‬‬ ‫‪4 2‬‬ ‫‪2‬‬
‫√ = √ = 𝜃 ‪ sin‬و‬
‫‪VI‬‬

‫= 𝜃 ‪cos‬‬ ‫‪=−‬‬
‫‪8‬‬ ‫‪2‬‬ ‫‪8‬‬ ‫‪2‬‬
‫𝜋‪3‬‬
‫=𝜃‬ ‫)𝜋‪(mod 2‬‬ ‫ﻣﻨﻪ‬
‫‪4‬‬

‫‪. 4√2 (−1 + ı) = 8e‬‬ ‫‪/‬‬ ‫إذن ‪:‬‬


‫‪ƕ‬‬

‫‪ •2‬ﻟﺘﻜﻦ 𝜌 ‪ 𝜌 ،‬و 𝜌 ﻃﻮﻳﻼت اﻷﻋﺪاد ‪ z ، z‬و ‪ z‬ﻋﲆ اﻟﱰﺗﻴﺐ و ‪ b ، a‬و ‪ c‬ﻋﻤﺪاﲥﺎ )ﻟﻴﺲ ﺑﺎﻟﴬورة ﻋﲆ اﻟﱰﺗﻴﺐ(‪.‬‬
‫ﻟﺪﻳﻨﺎ ‪ 𝜌 = 2𝜌 :‬و 𝜌‪. 𝜌 = 4‬‬
‫𝜋‬ ‫𝜋‬
‫‪ . c = a +‬ﻣﻨﻪ ‪:‬‬ ‫ﻟﺪﻳﻨﺎ أﻳﻀ ًﺎ ‪ b = a + (mod 2𝜋) :‬و )𝜋‪(mod 2‬‬
‫‪ũŏ‬‬

‫‪2‬‬ ‫‪4‬‬

‫‪⎧𝜌 ⋅ 𝜌 ⋅ 𝜌 = 8‬‬ ‫⎧‬ ‫‪8𝜌 = 8‬‬ ‫‪𝜌 =1‬‬


‫⟺‬ ‫⟺‬
‫)𝜋‪⎨ a + b + c = 3𝜋 (mod 2‬‬ ‫)𝜋‪⎨ 3a + 3𝜋 = 3𝜋 (mod 2‬‬ ‫)𝜋‪a = 0 (mod 2‬‬
‫⎩‬ ‫‪4‬‬ ‫⎩‬ ‫‪4‬‬ ‫‪4‬‬
‫𝜋‬ ‫𝜋‬
‫= ‪.c‬‬ ‫= ‪b‬و‬ ‫إذن ‪ 𝜌 = 2 ، 𝜌 = 1‬و ‪ 𝜌 = 4‬؛ ‪، a = 0‬‬
‫‪2‬‬ ‫‪4‬‬

‫𝟵𝟮𝟮‬

‫‪http ://tinyurl.com/Malki1718‬‬ ‫‪0‬‬


‫‪ .4.VI‬ﲤﺎرﻳﻦ ﻣﺘﻨﻮﻋﺔ‬
‫‪.‬‬
‫‪2‬‬ ‫‪2‬‬
‫‪M3‬‬ ‫‪M2‬‬

‫‪Ŕž Ŧ Ľ‬‬
‫‪M1‬‬ ‫‪M1‬‬
‫‪1‬‬ ‫‪1‬‬

‫‪œǃ‬‬
‫‪M2‬‬ ‫‪M3‬‬
‫‪O‬‬ ‫‪O‬‬
‫‪1‬‬ ‫‪2‬‬ ‫‪3‬‬ ‫‪4‬‬ ‫‪1‬‬ ‫‪2‬‬ ‫‪3‬‬ ‫‪4‬‬

‫ﺷﻜﻞ ‪23.VI‬‬ ‫ﺷﻜﻞ ‪22.VI‬‬

‫‪Ŀ‬‬
‫𝜋‬
‫= ) ‪ arg (z‬و ‪ arg (z ) = 0‬؛ أو ‪ arg (z ) = 0‬و‬ ‫ﻟﻜﻦ 𝜋 ≤ ) ‪ 𝜋 ≤ arg (z‬إذن 𝜋 = ) ‪ arg (z‬و ﺑﺎﻟﺘﺎﱄ ّ‬
‫ﻓﺈن‬
‫‪4‬‬ ‫‪2‬‬ ‫‪2‬‬
‫𝜋‬
‫= ) ‪ arg (z‬إذن ‪:‬‬
‫‪4‬‬
‫‪ z = 2e / ، z = e /‬و ‪ z = 4e = 4‬؛‬ ‫‪=ı‬‬
‫أو ‪ z = 4e = 4 ، z = e / = ı‬و ‪. z = 2e /‬‬
‫‪ •3‬أﻧﻈﺮ اﻟﺸﻜﻠﲔ ‪ 22.VI‬و ‪23.VI‬‬
‫■‬

‫اﳌﺴﺘﻮي اﳌﺮﻛﺐ ﻣﻨﺴﻮب إﱃ ﻣﻌﻠﻢ ﻣﺘﻌﺎﻣﺪ‪ ،‬ﻣﺘﺠﺎﻧﺲ و ﻣﺒﺎﴍ ⃗‪. O, u,⃗ v‬‬
‫‪.‬‬
‫𝟗𝟓𝟏‬ ‫‪.‬‬ ‫‬
‫ﻟﺘﻜﻦ ‪ A‬اﻟﻨﻘﻄﺔ ذات اﻟﻼﺣﻘﺔ ‪ z = 1 + 3ı‬و ‪ B‬اﻟﻨﻘﻄﺔ ذات اﻟﻼﺣﻘﺔ ‪. z = 2‬‬
‫‪ •1‬أﻧﺸﺊ اﳌﺜﻠﺚ ‪. OAB‬‬
‫‪ •2‬إﴍح ﳌﺎذا اﳌﺜﻠﺚ ‪ OAB‬ﻣﺘﺴﺎوي اﻟﺴﺎﻗﲔ ﻋﻨﺪ اﻟﺮأس ‪. A‬‬
‫‪ •3‬ﻧﻌﺘﱪ اﻟﻨﻘﻄﺔ ‪ C‬ﻧﻈﲑة اﻟﻨﻘﻄﺔ ‪ O‬ﺑﺎﻟﻨﺴﺒﺔ إﱃ ‪ A‬و اﻟﻨﻘﻄﺔ ‪ D‬ﻧﻈﲑة ‪ B‬ﺑﺎﻟﻨﺴﺒﺔ إﱃ ‪. O‬‬
‫ﻋﻠﻢ اﻟﻨﻘﻄﺘﲔ ‪ C‬و ‪ D‬ﻋﲆ اﻟﺸﻜﻞ ‪24.VI‬‬‫)ا(‬
‫‪ņőƱŀ Ţ‬‬
‫)ب( أوﺟﺪ اﻟﻌﺪدﻳﻦ ‪ z‬و ‪ z‬ﻻﺣﻘﺘ َْﻲ ‪ C‬و ‪ D‬ﻋﲆ اﻟﱰﺗﻴﺐ‪.‬‬
‫‪1‬‬
‫‪ •4‬ﻟﺘﻜﻦ ‪ E‬ﺻﻮرة ‪ A‬ﺑﺎﻟﺘﺤﺎﻛﻲ اﻟﺬي ﻣﺮﻛﺰه ‪ O‬و ﻧﺴﺒﺘﻪ ‪.‬‬
‫‪3‬‬
‫ﻋﻠﻢ اﻟﻨﻘﻄﺔ ‪ E‬ﻋﲆ اﻟﺸﻜﻞ ‪24.VI‬‬ ‫)ا(‬
‫)ب( أوﺟﺪ اﻟﻌﺪد ‪ z‬ﻻﺣﻘﺔ اﻟﻨﻘﻄﺔ ‪. E‬‬
‫‪VI‬‬
‫‪ •5‬ﻟﺘﻜﻦ ‪ F‬ﻣﺮﻛﺰ اﳌﺴﺎﻓﺎت اﳌﺘﻨﺎﺳﺒﺔ ﻟﻠﺠﻤﻠﺔ })‪. {(A, 2) , (B, −3) , (D, 2‬‬
‫)ا( أوﺟﺪ ‪ z‬ﻻﺣﻘﺔ اﻟﻨﻘﻄﺔ ‪. F‬‬
‫‪ƕ‬‬

‫)ب( ﻋﻠﻢ اﻟﻨﻘﻄﺔ ‪ F‬ﻋﲆ اﻟﺸﻜﻞ ‪24.VI‬‬

‫‪ •6‬أﺛﺒﺖ ّ‬
‫أن اﻟﻨﻘﻂ ‪ E ، B‬و ‪ F‬ﻋﲆ اﺳﺘﻘﺎﻣﺔ واﺣﺪة‪.‬‬
‫‪z −z‬‬
‫‪ũŏ‬‬

‫= ‪.Z‬‬ ‫‪ •7‬ﻟﻴﻜﻦ ‪ Z‬اﻟﻌﺪد اﳌﺮﻛﺐ اﳌﻌﺮف ﺑـِ ‪:‬‬


‫‪z −z‬‬

‫ﻓﺼﻞ إﺟﺎﺑﺘﻚ‪.‬‬
‫)ا( أوﺟﺪ اﻟﻌﺪد اﳊﻘﻴﻘﻲ ‪ a‬ﺑﺤﻴﺚ ‪ . Z = aı‬ﱢ‬
‫)ب( أوﺟﺪ اﻟﻄﻮﻳﻠﺔ ||‪ ||Z‬و ﻋﻤﺪ ًة )‪ arg (Z‬ﻟﻠﻌﺪد ‪. Z‬‬
‫)ج( إﺳﺘﻨﺘﺞ ﻃﺒﻴﻌﺔ اﳌﺜﻠﺚ ‪ . CBF‬ﻋ ﱢﻠﻞ‪.‬‬

‫𝟬𝟯𝟮‬

‫‪http ://tinyurl.com/Malki1718‬‬ ‫‪0‬‬


‫‪ .VI‬ﲤﺎرﻳﻦ ﺗﻄﺒﻴﻘﻴﺔ‬
‫‪.‬‬
‫‪F‬‬ ‫‪6‬‬
‫‪C‬‬

‫‪Ŕž Ŧ Ľ‬‬
‫‪4‬‬

‫‪œǃ‬‬
‫‪A‬‬

‫‪2‬‬

‫‪Ŀ‬‬
‫‪E‬‬
‫~‬
‫‪v‬‬
‫‪D‬‬ ‫‪B‬‬
‫‪−8‬‬ ‫‪−6‬‬ ‫‪−4‬‬ ‫‪−2‬‬ ‫‪O‬‬ ‫~‬
‫‪u‬‬ ‫‪2‬‬

‫ﺷﻜﻞ ‪24.VI‬‬

‫‪ •1‬أﻧﻈﺮ اﻟﺸﻜﻞ ‪24.VI‬‬ ‫ﺍﳊﻞّ‪.‬‬

‫‪ •2‬ﻟﺪﻳﻨﺎ ‪ OA = ||z − z || = ||1 + 3ı|| = √10 :‬و ‪ AB = ||z − z || = ||1 − 3ı|| = √10‬ﻣﻨﻪ ‪ OA = AB‬إذن‬
‫اﳌﺜﻠﺚ ‪ OAB‬ﻣﺘﺴﺎوي اﻟﺴﺎﻗﲔ ﻋﻨﺪ اﻟﺮأس ‪. A‬‬
‫أﻧﻈﺮ اﻟﺸﻜﻞ ‪24.VI‬‬
‫‪) •3‬ا(‬
‫)ب( ﻟﺪﻳﻨﺎ ‪ OA = AC :‬ﻣﻨﻪ ‪ z − z = z − z‬أي ‪. z = 2z − z = 2 + 6ı‬‬
‫ﺑﺎﳌﺜﻞ‪ BO = OD ،‬ﻣﻨﻪ ‪ z − z = z − z‬أي ‪. z = 2z − z = −2‬‬
‫أﻧﻈﺮ اﻟﺸﻜﻞ ‪24.VI‬‬ ‫)ا(‬ ‫•‬‫‪4‬‬
‫‪1‬‬ ‫‪1‬‬ ‫‪1‬‬
‫= ‪) z = z‬أﻧﻈﺮ أﻳﻀ ًﺎ اﻟﻼزﻣﺔ ‪ 1‬ﺻﻔﺤﺔ ‪. (53‬‬ ‫‪ z − z‬ﻣﻨﻪ ‪+ ı‬‬ ‫=‬ ‫ﻟﺪﻳﻨﺎ ‪(z − z ) :‬‬ ‫)ب(‬
‫‪3‬‬ ‫‪3‬‬ ‫‪3‬‬
‫‪) •5‬ا(‬ ‫أﻧﻈﺮ اﻟﺸﻜﻞ ‪24.VI‬‬
‫)ب( ﻟﺪﻳﻨﺎ ‪ 2 (z − z ) − 3 (z − z ) + 2 (z − z ) = 0 :‬ﻣﻨﻪ ‪(2 − 3 + 2) z = 2z − 3z + 2z‬‬
‫‪ņőƱŀ Ţ‬‬
‫أي ‪. z = −8 + 6ı‬‬
‫‪1‬‬ ‫‪5‬‬
‫= ‪ z − z‬و ﻻﺣﻘﺔ اﻟﺸﻌﺎع ‪ BF‬ﻫﻲ = ‪z − z = −8 + 6ı − 2‬‬ ‫ﻻﺣﻘﺔ اﻟﺸﻌﺎع ‪ BE‬ﻫﻲ ‪+ ı − 2 = − + ı‬‬ ‫•‬‫‪6‬‬
‫‪3‬‬ ‫‪3‬‬
‫‪ −10 + 6ı = 6 − 5 + ı‬إذن ‪ BF = 6BE‬و ﻫﺬا ﻳﻌﻨﻲ ّ‬
‫‪VI‬‬

‫أن اﻟﻨﻘﻂ ‪ E ، B‬و ‪ F‬ﻋﲆ اﺳﺘﻘﺎﻣﺔ واﺣﺪة‪.‬‬


‫‪3‬‬

‫)ا( ﻟﺪﻳﻨﺎ ‪:‬‬ ‫•‬‫‪7‬‬


‫‪z −z‬‬ ‫‪−8 + 6ı − 2 − 6ı −10‬‬ ‫‪5‬‬
‫=‪Z‬‬ ‫=‬ ‫=‬ ‫‪=− ı‬‬
‫‪z −z‬‬ ‫‪2 − 2 − 6ı‬‬ ‫‪−6ı‬‬ ‫‪3‬‬
‫‪ƕ‬‬

‫‪5‬‬
‫إذن ‪. a = −‬‬
‫‪3‬‬
‫‪5‬‬ ‫‪5‬‬
‫)ب( ﻟﺪﻳﻨﺎ ‪ ||Z|| = |||− ı||| = :‬و‬
‫‪3‬‬ ‫‪3‬‬
‫‪ũŏ‬‬

‫‪5‬‬ ‫‪5‬‬ ‫𝜋‪𝜋 3‬‬ ‫𝜋‬


‫‪arg (Z) = arg − ı‬‬ ‫‪= arg −‬‬ ‫= ‪+ arg (ı) = 𝜋 +‬‬ ‫)𝜋‪(mod 2𝜋) = − (mod 2‬‬
‫‪3‬‬ ‫‪3‬‬ ‫‪2‬‬ ‫‪2‬‬ ‫‪2‬‬
‫𝜋‬
‫ّ‬
‫ﻓﺈن ‪ CBF‬ﻗﺎﺋﻢ اﻟﺰاوﻳﺔ ﰲ ‪. C‬‬ ‫)ج( ﺑﲈ ّ‬
‫أن )𝜋‪CB, CF = arg (Z) = − (mod 2‬‬
‫‪2‬‬
‫■‬

‫𝟭𝟯𝟮‬

‫‪http ://tinyurl.com/Malki1718‬‬ ‫‪0‬‬


‫‪ .4.VI‬ﲤﺎرﻳﻦ ﻣﺘﻨﻮﻋﺔ‬
‫‪.‬‬

‫‬ ‫‪.‬‬
‫𝟎𝟔𝟏‬ ‫‪.‬‬

‫‪Ŕž Ŧ Ľ‬‬
‫اﳌﺴﺘﻮي اﳌﺮﻛﺐ ﻣﻨﺴﻮب إﱃ ﻣﻌﻠﻢ ﻣﺘﻌﺎﻣﺪ‪ ،‬ﻣﺘﺠﺎﻧﺲ و ﻣﺒﺎﴍ ⃗‪. O, u,⃗ v‬‬
‫ﻟﺘﻜﻦ ‪ A‬اﻟﻨﻘﻄﺔ ذات اﻟﻼﺣﻘﺔ ‪ z = 1‬و ‪ B‬اﻟﻨﻘﻄﺔ ذات اﻟﻼﺣﻘﺔ ‪. z = 3 − 2ı‬‬

‫‪œǃ‬‬
‫ﻟﻜﻞ ﻋﺪد ﻣﺮﻛﺐ ‪ ، z‬ﻧﻀﻊ ‪ z = ız + 1 − ı :‬و ﻧﻌﺘﱪ اﻟﺘﺤﻮﻳﻞ ‪ F‬اﻟﺬي ﻳﺮﻓﻖ ﺑﻜﻞ ﻧﻘﻄﺔ ‪ M‬ﻻﺣﻘﺘﻬﺎ ‪ ، z‬اﻟﻨﻘﻄﺔ ‪ M‬ذات‬
‫اﻟﻼﺣﻘﺔ ‪. z‬‬
‫‪ •1‬ﻋﻠﻢ اﻟﻨﻘﻄﺘﲔ ‪ A‬و ‪ B‬ﻋﲆ اﻟﺸﻜﻞ ‪25.VI‬‬

‫‪Ŀ‬‬
‫‪ •2‬ﰲ ﻫﺬا اﻟﺴﺆال‪ ،‬ﻧﻌﺘﱪ ﻧﻘﻄﺔ ‪ M‬ﲣﺘﻠﻒ ﻋﻦ ‪ A‬ﻻﺣﻘﺘﻬﺎ ‪. z ≠ 1‬‬
‫‪z −1‬‬
‫= ‪ Z‬ﻋﲆ اﻟﺸﻜﻞ اﳉﱪي‪.‬‬ ‫أﻛﺘﺐ اﻟﻌﺪد‬ ‫)ا(‬
‫‪z−1‬‬
‫أوﺟﺪ اﻟﻄﻮﻳﻠﺔ ||‪ ||Z‬و ﻋﻤﺪ ًة )‪ arg (Z‬ﻟﻠﻌﺪد ‪. Z‬‬ ‫)ب(‬
‫ﺛﻢ أوﺟﺪ ‪. AM, AM‬‬‫ﻋﱪ ﻋﻦ اﻟﻄﻮل ‪ AM‬ﺑﺪﻻﻟﺔ ‪ّ ، AM‬‬ ‫)ج( ﱢ‬
‫)د( إﺳﺘﻨﺘﺞ ﻃﺒﻴﻌﺔ اﻟﺘﺤﻮﻳﻞ ‪ F‬و ﻋﻨﺎﴏه اﳌﻤ ﱢﻴﺰة‪.‬‬

‫ﺻﻮرﰐ ‪ A‬و ‪ B‬ﺑﺎﻟﺘﺤﻮﻳﻞ ‪. F‬‬


‫َْ‬ ‫‪ •3‬أوﺟﺪ ‪ z ′‬و ‪ z ′‬ﻻﺣﻘﺘ َْﻲ اﻟﻨﻘﻄﺘﲔ ‪ A‬و ‪، B‬‬
‫‪ •4‬ﻟﺘﻜﻦ ‪ C‬اﻟﻨﻘﻄﺔ اﻟﺘﻲ ﺻﻮرﲥﺎ ﺑﺎﻟﺘﺤﻮﻳﻞ ‪ F‬ﻫﻲ اﻟﻨﻘﻄﺔ ‪ C‬ذات اﻟﻼﺣﻘﺔ ‪. z ′ = −3 − 3ı‬‬
‫)ا( أوﺟﺪ ‪ z‬ﻻﺣﻘﺔ اﻟﻨﻘﻄﺔ ‪. C‬‬
‫)ب( أﻧﺸﺊ اﳌﺜﻠﺜﲔ ‪ ABC‬و ‪ ACB‬ﻋﲆ اﻟﺸﻜﻞ ‪25.VI‬‬

‫‪ •5‬ﻟﺘﻜﻦ ‪ I‬ﻣﻨﺘﺼﻒ اﻟﻘﻄﻌﺔ ‪. BC‬‬

‫اﳌﺘﻮﺳﻂ 𝒟 اﳌﺘﻌﻠﻖ ﺑﺎﻟﺮأس ‪. A‬‬


‫ّ‬ ‫)ا( أوﺟﺪ ‪ z‬ﻻﺣﻘﺔ اﻟﻨﻘﻄﺔ ‪ . I‬أﻧﺸﺊ‪ ،‬ﰲ اﳌﺜﻠﺚ ‪، ABC‬‬
‫)ب( أوﺟﺪ ﻻﺣﻘﺘ َْﻲ اﻟﺸﻌﺎﻋﲔ ‪ AI‬و ‪. CB‬‬
‫‪ņőƱŀ Ţ‬‬
‫)ج( ﻣﺎ ﻫﻲ اﻟﻮﺿﻌﻴﺔ اﻟﻨﺴﺒﻴﺔ ﻟﻠﻤﺴﺘﻘﻴﻤﲔ )‪ (AI‬و ‪ CB‬؟ ﱢﺑﺮر إﺟﺎﺑﺘﻚ‪.‬‬
‫)د( ﻣﺎذا ُﻳﻤ ّﺜﻞ اﳌﺴﺘﻘﻴﻢ 𝒟 ﺑﺎﻟﻨﺴﺒﺔ إﱃ اﳌﺜﻠﺚ ‪ ACB‬؟‬

‫‪VI‬‬
‫𝒟 و أﻧﺸﺌﻪ ﻋﲆ اﻟﺸﻜﻞ ‪25.VI‬‬ ‫‪ •6‬ﻟﻴﻜﻦ 𝒟 ﺻﻮرة اﳌﺴﺘﻘﻴﻢ )‪ (AI‬ﺑﺎﻟﺘﺤﻮﻳﻞ ‪ . F‬أوﺟﺪ‬

‫‪ •1‬أﻧﻈﺮ اﻟﺸﻜﻞ ‪. 25.VI‬‬ ‫ﺍﳊﻞّ‪.‬‬


‫‪ƕ‬‬

‫)ا( ﻟﺪﻳﻨﺎ ‪:‬‬ ‫•‬ ‫‪2‬‬


‫)‪z − 1 ız + 1 − ı − 1 ız − ı ı (z − 1‬‬
‫=‪Z‬‬ ‫=‬ ‫=‬ ‫=‬ ‫‪=ı‬‬
‫‪z−1‬‬ ‫‪z−1‬‬ ‫‪z−1‬‬ ‫‪z−1‬‬
‫‪ũŏ‬‬

‫𝜋‬
‫أن ‪ّ Z = ı = e /‬‬
‫ﻓﺈن ‪ ||Z|| = 1‬و )𝜋‪. arg (Z) = (mod 2‬‬ ‫ﺑﲈ ّ‬ ‫)ب(‬
‫‪2‬‬
‫ﻟﺪﻳﻨﺎ إذن )‪ z − 1 = ı (z − 1‬ﻣﻨﻪ ‪:‬‬ ‫)ج(‬

‫‪AM = ||z − z || = ||z − 1|| = ||ı (z − 1)|| = |ı| ⋅ ||z − 1|| = 1 ⋅ ||z − z || = ||z − z || = AM‬‬
‫‪z −z‬‬ ‫‪z −1‬‬ ‫𝜋‬
‫‪AM, AM‬‬ ‫‪= arg‬‬ ‫‪= arg‬‬ ‫= )‪= arg (Z‬‬ ‫)𝜋‪(mod 2‬‬
‫‪z−z‬‬ ‫‪z−1‬‬ ‫‪2‬‬

‫𝟮𝟯𝟮‬

‫‪http ://tinyurl.com/Malki1718‬‬ ‫‪0‬‬


‫‪ .VI‬ﲤﺎرﻳﻦ ﺗﻄﺒﻴﻘﻴﺔ‬
‫‪.‬‬
‫‪C‬‬
‫‪4‬‬ ‫‪D‬‬

‫‪Ŕž Ŧ Ľ‬‬
‫‪3‬‬

‫‪œǃ‬‬
‫‪B′‬‬
‫‪′‬‬ ‫‪2‬‬
‫‪D‬‬

‫‪1‬‬

‫‪Ŀ‬‬
‫‪A = A′‬‬
‫‪−3‬‬ ‫‪−2‬‬ ‫‪−1‬‬ ‫‪1‬‬ ‫‪2‬‬ ‫‪3‬‬

‫‪I′‬‬
‫‪−1‬‬

‫‪B‬‬
‫‪−2‬‬

‫‪C′‬‬ ‫‪I‬‬
‫‪−3‬‬

‫ﺷﻜﻞ ‪25.VI‬‬

‫𝜋‬ ‫‪/‬‬
‫‪ّ z −z =e‬‬
‫ﻓﺈن ‪ F‬ﻫﻮ اﻟﺪوران اﻟﺬي ﻣﺮﻛﺰه اﻟﻨﻘﻄﺔ ‪ A‬و زاوﻳﺘﻪ ‪.‬‬ ‫ﺑﲈ ّ‬
‫أن ) ‪(z − z‬‬ ‫)د(‬
‫‪2‬‬
‫‪ •3‬ﻟﺪﻳﻨﺎ ‪:‬‬
‫‪z‬‬ ‫‪′‬‬ ‫‪= ız + 1 − ı = ı + 1 − ı = 1‬‬
‫‪z‬‬ ‫‪′‬‬ ‫‪= ız + 1 − ı = ı (3 − 2ı) + 1 − ı = 3ı + 2 + 1 − ı = 3 + 2ı‬‬

‫‪ّ z‬‬
‫ﻷن ‪ A‬ﻫﻲ ﻣﺮﻛﺰ اﻟﺪوران ‪ F‬و ﺑﺎﻟﺘﺎﱄ ﻓﻬﻲ ﻧﻘﻄﺔ ﺻﺎﻣﺪة )و ﻫﻲ وﺣﻴﺪة(‪.‬‬ ‫‪′‬‬ ‫‪=z‬‬ ‫ﻳﻤﻜﻦ أﻳﻀ ًﺎ ﻣﻼﺣﻈﺔ ّ‬
‫أن‬
‫أن ‪ّ z ′ = ız + 1 − ı = −3 − 3ı‬‬
‫ﻓﺈن ‪ız = −3 − 3ı − 1 + ı = −4 − 2ı‬‬ ‫ﺑﲈ ّ‬ ‫)ا(‬ ‫‪4‬‬
‫•‬
‫‪1‬‬
‫‪ņőƱŀ Ţ‬‬
‫أي ‪. z = (−4 − 2ı) = −ı (−4 − 2ı) = −2 + 4ı‬‬
‫‪ı‬‬
‫إذن ﻻﺣﻘﺔ اﻟﻨﻘﻄﺔ ‪ C‬ﻫﻲ ‪. z = −2 + 4ı‬‬
‫)ب( أﻧﻈﺮ اﻟﺸﻜﻞ ‪. 25.VI‬‬
‫‪VI‬‬

‫)ا( ﻟﺪﻳﻨﺎ ‪:‬‬ ‫‪5‬‬


‫•‬
‫‪z +z‬‬ ‫‪′‬‬ ‫‪3 − 2ı − 3 − 3ı‬‬ ‫‪5‬‬
‫= ‪z‬‬ ‫=‬ ‫‪=− ı‬‬
‫‪2‬‬ ‫‪2‬‬ ‫‪2‬‬
‫أﻧﻈﺮ اﻟﺸﻜﻞ ‪. 25.VI‬‬
‫‪5‬‬
‫)ب( ﻻﺣﻘﺔ اﻟﺸﻌﺎع ‪ AI‬ﻫﻲ ‪ ، z − z = −1 − ı‬و ﻻﺣﻘﺔ اﻟﺸﻌﺎع ‪ CB‬ﻫﻲ ‪. z ′ − z = 5 − 2ı‬‬
‫‪ƕ‬‬

‫‪2‬‬
‫‪5‬‬ ‫‪−1‬‬
‫‪ CB‬ﻣﺘﻌﺎﻣﺪان و ﻫﺬا ﻣﺎ ُﻳﺒ ﱢﻴﻨﻪ اﳊﺴﺎب‪:‬‬ ‫‪ AI‬و‬ ‫)ج( اﳌﺴﺘﻘﻴﲈن )‪ (AI‬و ‪ CB‬ﻣﺘﻌﺎﻣﺪان ّ‬
‫ﻷن اﻟﺸﻌﺎﻋﺎن‬
‫‪−2‬‬ ‫‪−‬‬
‫‪5‬‬
‫‪AI ⋅ CB = −1 × 5 + (−2) −‬‬ ‫‪=0‬‬
‫‪ũŏ‬‬

‫‪2‬‬
‫)د( اﳌﺴﺘﻘﻴﻢ 𝒟 ﻫﻮ اﻟﻌﻤﻮد اﻟﻨﺎزل ﻣﻦ اﻟﺮأس ‪ A‬ﰲ اﳌﺜﻠﺚ ‪ ACB‬ﻷﻧﻪ ﻳﻤﺮ ﺑﺎﻟﺮأس ‪ A‬و ُﻳﻌﺎﻣﺪ اﻟﻀﻠﻊ ‪) CB‬اﳌﺴﺘﻘﻴﻢ‬
‫𝒟 ﻫﻮ اﳌﺴﺘﻘﻴﻢ )‪. ((AI‬‬
‫‪ •6‬ﺻﻮرة اﳌﺴﺘﻘﻴﻢ )‪ (AI‬ﺑﺎﻟﺪوران ‪ F‬ﻫﻮ اﳌﺴﺘﻘﻴﻢ ‪ A I‬ﺣﻴﺚ ‪ A = F (A) = A‬و )‪ . I = F (I‬و ﺑﲈ ّ‬
‫أن زاوﻳﺔ ﻫﺬا‬
‫اﻟﺪوران ﻫﻲ 𝜋 ّ‬
‫ﻓﺈن اﳌﺴﺘﻘﻴﻢ 𝒟 ﻫﻲ اﳌﺴﺘﻘﻴﻢ اﻟﺬي ﻳﻤﺮ ﺑﺎﻟﻨﻘﻄﺔ ‪ A‬و ﻋﻤﻮدي ﻋﲆ )‪ . (AI‬أﻧﻈﺮ اﻟﺸﻜﻞ ‪. 25.VI‬‬
‫‪2‬‬
‫■‬

‫𝟯𝟯𝟮‬

‫‪http ://tinyurl.com/Malki1718‬‬ ‫‪0‬‬


‫‪ .4.VI‬ﲤﺎرﻳﻦ ﻣﺘﻨﻮﻋﺔ‬
‫‪.‬‬

‫‬ ‫‪.‬‬
‫𝟏𝟔𝟏‬ ‫‪.‬‬

‫‪Ŕž Ŧ Ľ‬‬
‫اﳌﺴﺘﻮي اﳌﺮﻛﺐ 𝒫 ﻣﻨﺴﻮب إﱃ ﻣﻌﻠﻢ ﻣﺘﻌﺎﻣﺪ و ﻣﺘﺠﺎﻧﺲ ⃗‪. O, u,⃗ v‬‬
‫اﳉﺰء اﻷول‬

‫‪œǃ‬‬
‫ﻧﻌﺘﱪ اﻟﺘﺤﻮﻳﻞ اﻟﻨﻘﻄﻲ ‪ T‬اﻟﺬي ﻳﺮﻓﻖ ﺑﻜﻞ ﻧﻘﻄﺔ ‪ M‬ﻣﻦ 𝒫 ﻻﺣﻘﺘﻬﺎ ‪ z‬اﻟﻨﻘﻄﺔ ‪ M‬اﻟﺘﻲ ﻻﺣﻘﺘﻬﺎ ‪ z‬ﺣﻴﺚ ‪:‬‬
‫‪√2‬‬ ‫‪√2‬‬
‫= ‪z‬‬ ‫‪ız +‬‬

‫‪Ŀ‬‬
‫‪2‬‬ ‫‪2‬‬
‫‪ •1‬أوﺟﺪ ‪ b‬ﻻﺣﻘﺔ اﻟﻨﻘﻄﺔ اﻟﺼﺎﻣﺪة اﻟﻮﺣﻴﺪة ‪ B‬ﺑﺎﻟﺘﺤﻮﻳﻞ ‪ T‬أي اﻟﺘﻲ ﲢﻘﻖ ‪. T (B) = B :‬‬
‫‪z −b‬‬
‫= ‪.Z‬‬ ‫‪ •2‬ﻧﻔﺮض ّ‬
‫أن ‪ M‬ﲣﺘﻠﻒ ﻋﻦ ‪ B‬و ﻧﻀﻊ ‪:‬‬
‫‪z−b‬‬
‫)ا( أوﺟﺪ اﻟﻌﺪد اﳊﻘﻴﻘﻲ ‪ R‬ﺑﺤﻴﺚ ‪. Z = ıR‬‬
‫)ب( أوﺟﺪ اﻟﻄﻮﻳﻠﺔ ||‪ ||Z‬و ﻋﻤﺪ ًة )‪ arg (Z‬ﻟﻠﻌﺪد ‪. Z‬‬
‫)ج( إﺳﺘﻨﺘﺞ ﻗﻴﺴ ًﺎ ﻟﻠﺰاوﻳﺔ ‪ BM, BM‬و ﻋﺒﺎرة ‪ BM‬ﺑﺪﻻﻟﺔ ‪. BM‬‬

‫اﳉﺰء اﻟﺜﺎﲏ‬

‫ﻟﻴﻜﻦ 𝛼 ﻋﺪد ًا ﻣﺮﻛﺒﺎ و ‪ A‬اﻟﻨﻘﻄﺔ ذات اﻟﻼﺣﻘﺔ 𝛼 ‪.‬‬


‫ﻧﻌﺘﱪ اﻟﺘﺤﻮﻳﻞ اﻟﻨﻘﻄﻲ ‪ T‬اﻟﺬي ﻳﺮﻓﻖ ﺑﻜﻞ ﻧﻘﻄﺔ ‪ M‬ﻣﻦ اﳌﺴﺘﻮي‪ ،‬ذات اﻟﻼﺣﻘﺔ ‪ ، z‬اﻟﻨﻘﻄﺔ ‪ M‬ذات اﻟﻼﺣﻘﺔ ‪ z‬ﺑﺤﻴﺚ ‪:‬‬
‫‪√2‬‬ ‫‪√2‬‬
‫= ‪z‬‬ ‫‪(𝛼 + ı) z +‬‬ ‫)𝛼 ‪(1 −‬‬
‫‪2‬‬ ‫‪2‬‬
‫‪ •1‬ﻧﻔﺮض ﰲ ﻫﺬا اﻟﺴﺆال ّ‬
‫أن ‪. 𝛼 = √2 − ı :‬‬

‫)ا( أوﺟﺪ ﻻﺣﻘﺔ اﻟﺸﻌﺎع ‪. MM‬‬


‫‪ņőƱŀ Ţ‬‬
‫‪ T‬ﲢﻮﻳﻞ ﻫﻨﺪﳼ ﺑﺴﻴﻂ ُﻳﻄﻠﺐ ﺗﻌﻴﲔ ﻋﻨﺎﴏه اﳌﻤ ﱢﻴﺰة‪.‬‬ ‫‪−‬‬
‫)ب( إﺳﺘﻨﺘﺞ ّ‬
‫أن‬

‫‪ T‬دوراﻧ ًﺎ ﻣﺮﻛﺰه ‪ . O‬ﻣﺎ ﻫﻲ ﻫﺬه اﻟﻘﻴﻤﺔ ؟‬ ‫)ا( ﺗﻮﺟﺪ ﻗﻴﻤﺔ وﺣﻴﺪة 𝛼 ﻟﻠﻌﺪد 𝛼 اﻟﺘﻲ ﻣﻦ أﺟﻠﻬﺎ ﻳﻜﻮن اﻟﺘﺤﻮﻳﻞ‬ ‫•‬ ‫‪2‬‬
‫)ب( أوﺟﺪ 𝜃 زاوﻳﺔ ﻫﺬا اﻟﺪوران ﻣﻊ اﻟﺘﻌﻠﻴﻞ‪.‬‬

‫‪VI‬‬
‫ﺍﳊﻞّ‪.‬‬
‫‪ƕ‬‬

‫اﳉﺰء اﻷول‬

‫‪ •1‬ﻹﳚﺎد اﻟﻨﻘﻂ اﻟﺼﺎﻣﺪة‪ ،‬ﻧﺤﻞ اﳌﻌﺎدﻟﺔ ‪ z = z‬ذات اﳌﺠﻬﻮل ‪: z‬‬


‫‪ũŏ‬‬

‫‪√2‬‬ ‫‪√2‬‬
‫⟺ ‪z =z‬‬ ‫‪ız +‬‬ ‫‪=z‬‬
‫‪2‬‬ ‫‪2‬‬
‫‪⟺ z 2 − √2ı = √2‬‬

‫‪√2‬‬ ‫‪√2 2 + √2ı‬‬ ‫‪√2 + ı‬‬


‫=‪⟺ z‬‬ ‫=‬ ‫=‬
‫‪2 − √2ı‬‬ ‫‪4+2‬‬ ‫‪3‬‬

‫𝟰𝟯𝟮‬

‫‪http ://tinyurl.com/Malki1718‬‬ ‫‪0‬‬


‫‪ .VI‬ﲤﺎرﻳﻦ ﺗﻄﺒﻴﻘﻴﺔ‬
‫‪.‬‬
‫‪2 1‬‬
‫إذن ﻻﺣﻘﺔ اﻟﻨﻘﻄﺔ اﻟﺼﺎﻣﺪة اﻟﻮﺣﻴﺪة ﻫﻲ ‪. b = √ + ı‬‬
‫‪3‬‬ ‫‪3‬‬

‫‪Ŕž Ŧ Ľ‬‬
‫‪2‬‬ ‫‪2‬‬
‫ﻓﺈن √ ‪ b = √ ıb +‬ﻣﻨﻪ ‪:‬‬
‫أن ‪ّ b = b‬‬
‫)ا( ﺑﲈ ّ‬ ‫‪2‬‬
‫•‬
‫‪2‬‬ ‫‪2‬‬

‫‪œǃ‬‬
‫‪√2‬‬ ‫‪√2‬‬ ‫‪√2‬‬ ‫‪√2‬‬
‫‪ız +‬‬ ‫‪−‬‬ ‫‪ıb +‬‬ ‫‪√2‬‬
‫‪z −b‬‬ ‫‪2‬‬ ‫‪2‬‬ ‫‪2‬‬ ‫‪2‬‬ ‫‪ı (z − b) √2‬‬
‫=‪Z‬‬ ‫=‬ ‫‪= 2‬‬ ‫=‬ ‫‪ı‬‬
‫‪z−b‬‬ ‫‪z−b‬‬ ‫‪z−b‬‬ ‫‪2‬‬
‫‪2‬‬
‫إذن √ = ‪. R‬‬

‫‪Ŀ‬‬
‫‪2‬‬
‫| ‪| 2‬‬ ‫‪2‬‬
‫√ = |‪ ||Z|| = | √ ı‬و‬ ‫)ب( ﻟﺪﻳﻨﺎ ‪:‬‬
‫|| ‪|| 2‬‬ ‫‪2‬‬
‫‪√2‬‬ ‫‪√2‬‬ ‫𝜋 𝜋‬
‫‪arg (Z) = arg‬‬ ‫‪ı‬‬ ‫‪= arg‬‬ ‫)𝜋‪+ arg (ı) = 0 + = (mod 2‬‬
‫‪2‬‬ ‫‪2‬‬ ‫‪2‬‬ ‫‪2‬‬

‫)ج( ﻟﺪﻳﻨﺎ ‪:‬‬


‫‪z −b‬‬ ‫𝜋‬
‫‪BM, BM‬‬ ‫‪= arg‬‬ ‫= )‪= arg (Z‬‬ ‫)𝜋‪(mod 2‬‬
‫‪z−b‬‬ ‫‪2‬‬
‫‪BM‬‬ ‫‪√2‬‬ ‫‪√2‬‬
‫= ||‪= ||Z‬‬ ‫= ‪⟹ BM‬‬ ‫‪BM‬‬
‫‪BM‬‬ ‫‪2‬‬ ‫‪2‬‬

‫اﳉﺰء اﻟﺜﺎﲏ‬

‫)ا( ﻻﺣﻘﺔ اﻟﺸﻌﺎع ‪ MM‬ﻫﻲ ‪:‬‬ ‫‪1‬‬


‫•‬

‫‪√2‬‬ ‫‪√2‬‬ ‫‪√2‬‬


‫=‪z −z‬‬ ‫‪√2 − ı + ı z +‬‬ ‫= ‪1 − √2 + ı‬‬ ‫‪1 − √2 + ı‬‬
‫‪2‬‬ ‫‪2‬‬ ‫‪2‬‬
‫‪2‬‬
‫√‪.‬‬ ‫‪1 − √2 + ı‬‬ ‫‪ T‬ﻫﻮ اﻹﻧﺴﺤﺎب اﻟﺬي ﻻﺣﻘﺔ ﺷﻌﺎﻋﻪ‬ ‫‪−‬‬
‫)ﻷن ﻻﺣﻘﺘﻪ ﺛﺎﺑﺘﺔ( ّ‬
‫ﻓﺈن‬ ‫أن اﻟﺸﻌﺎع ‪ MM‬ﺛﺎﺑﺖ ّ‬
‫)ب( ﺑﲈ ّ‬
‫‪2‬‬
‫‪ņőƱŀ Ţ‬‬
‫‪| 2‬‬ ‫|‬
‫)ا( ﺣﺘﻰ ﻳﻜﻮن اﻟﺘﺤﻮﻳﻞ ‪ T‬دوراﻧ ًﺎ‪ ،‬ﳚﺐ أن ﻳﻜﻮن ‪ . | √ (𝛼 + ı)| = 1‬ﻟﻜﻦ ‪:‬‬ ‫‪2‬‬
‫•‬
‫‪|| 2‬‬ ‫||‬

‫‪| √2‬‬ ‫|‬ ‫‪√2‬‬


‫‪VI‬‬

‫|‬ ‫⟺ ‪(𝛼 + ı)| = 1‬‬ ‫‪𝛼 +1=1‬‬


‫||‬ ‫‪2‬‬ ‫||‬ ‫‪2‬‬
‫⟺‬ ‫‪𝛼 + 1 = √2 ⟺ 𝛼 = 1 ⟺ 𝛼 = ±1‬‬

‫و ﺣﺘﻰ ﺗﻜﻮن اﻟﻨﻘﻄﺔ ‪ O‬ﻣﺮﻛﺰ اﻟﺪوران ‪ T‬ﳚﺐ أن ﻳﻜﻮن ‪:‬‬


‫‪ƕ‬‬

‫‪√2‬‬ ‫‪√2‬‬
‫‪.𝛼 = 1‬‬ ‫ﻣﻨﻪ‬ ‫=‪0‬‬ ‫‪(𝛼 + ı) × 0 +‬‬ ‫)𝛼 ‪(1 −‬‬ ‫أي‬ ‫‪T (O) = O‬‬
‫‪2‬‬ ‫‪2‬‬
‫‪ T‬دوراﻧ ًﺎ ﻣﺮﻛﺰه ‪. O‬‬ ‫إذن‪ ،‬ﺗﻮﺟﺪ ﻗﻴﻤﺔ وﺣﻴﺪة ﻟﻠﻌﺪد 𝛼 ‪ ،‬و ﻫﻲ ‪ ، 𝛼 = 1‬و اﻟﺘﻲ ﻣﻦ أﺟﻠﻬﺎ ﻳﻜﻮن اﻟﺘﺤﻮﻳﻞ‬
‫ﻷﺟﻞ ‪ T− ، 𝛼 = −1‬دوران ﻣﺮﻛﺰه ﻫﻮ اﻟﻨﻘﻄﺔ اﻟﺼﺎﻣﺪة اﻟﻮﺣﻴﺪة اﻟﺘﻲ ﲢﻘﻖ ‪:‬‬ ‫ﻣﻼﺣﻈﺔ ‪⧏ : 35‬‬
‫‪ũŏ‬‬

‫‪1 + √2 + ı‬‬
‫‪.‬‬ ‫‪ . T (M) = M‬ﻧﺠﺪ ّ‬
‫أن ﻻﺣﻘﺔ ﻫﺬا اﳌﺮﻛﺰ ﻫﻲ‬
‫⧐‬ ‫‪2 + √2‬‬
‫𝜋‬
‫= 𝜃‪.‬‬ ‫‪ z = √2 (1 + ı) z = e‬و ﺑﺎﻟﺘﺎﱄ ّ‬
‫ﻓﺈن زاوﻳﺔ ﻫﺬا اﻟﺪوران ﻫﻲ )𝜋‪(mod 2‬‬ ‫‪/‬‬ ‫ﰲ ﻫﺬه اﳊﺎﻟﺔ‪ ،‬ﻟﺪﻳﻨﺎ ‪z :‬‬ ‫)ب(‬
‫‪4‬‬ ‫‪2‬‬
‫■‬

‫𝟱𝟯𝟮‬

‫‪http ://tinyurl.com/Malki1718‬‬ ‫‪0‬‬


‫‪ .4.VI‬ﲤﺎرﻳﻦ ﻣﺘﻨﻮﻋﺔ‬
‫‪.‬‬

‫‪.‬‬
‫𝟐𝟔𝟏‬ ‫‪.‬‬

‫‪Ŕž Ŧ Ľ‬‬
‫ﰲ اﳌﺴﺘﻮي اﳌﺮﻛﺐ اﳌﻨﺴﻮب إﱃ ﻣﻌﻠﻢ ﻣﺘﻌﺎﻣﺪ و ﻣﺘﺠﺎﻧﺲ ⃗‪ ، O, u,⃗ v‬ﻧﻌﺘﱪ اﻟﻨﻘﻂ ‪ C ، B ، A‬و ‪ F‬ذوات‬
‫اﻟﻠﻮاﺣﻖ ‪:‬‬

‫‪œǃ‬‬
‫‪z‬‬ ‫‪= 4 + 4ı ,‬‬ ‫‪z =4,‬‬ ‫‪z =7,‬‬ ‫‪z = 4 + 3ı‬‬
‫ﻟﺘﻜﻦ ‪ E‬ﻧﻘﻄﺔ ﺗﻘﺎﻃﻊ اﳌﺴﺘﻘﻴﻤﲔ )‪ (AC‬و )‪. (OF‬‬
‫اﳉﺰء اﻷول‬

‫‪Ŀ‬‬
‫‪ •1‬أﻧﺸﺊ اﻟﺸﻜﻞ‪.‬‬
‫‪z‬‬
‫ﻋﲆ اﻟﺸﻜﻞ اﳉﱪي‪.‬‬ ‫)ا( أﻛﺘﺐ اﻟﻌﺪد‬ ‫•‬ ‫‪2‬‬
‫‪z −z‬‬
‫)ب( ﻣﺎ اﻟﺬي ﺗﺴﺘﻨﺘﺠﻪ ﻣﻦ اﻟﺴﺆال اﻟﺴﺎﺑﻖ ﻓﻴﲈ ﳜﺺ اﳌﺴﺘﻘﻴﻤﲔ )‪ (OF‬و )‪ (AC‬؟ ﻋ ﱢﻠﻞ‪.‬‬
‫‪ •3‬ﻧﺮﻳﺪ اﻟﺘﺤﻘﻖ ﻣﻦ ّ‬
‫أن اﻟﻨﻘﻄﺔ ‪ F‬ﻫﻲ ﻣﺮﻛﺰ اﳌﺴﺎﻓﺎت اﳌﺘﻨﺎﺳﺒﺔ ﻟﻠﺠﻤﻠﺔ })𝛼 ‪ {(O, 3) , (A, 𝛼) , (C, 25 −‬ﺣﻴﺚ 𝛼 ﻋﺪد‬
‫ﺣﻘﻴﻘﻲ‪.‬‬

‫ﻋﱪ ﻋﻦ اﻟﺸﻌﺎع ‪ OF‬ﺑﺪﻻﻟﺔ 𝛼 ‪ OA ،‬و ‪. OC‬‬‫)ا( ﱢ‬


‫)ب( إﺳﺘﻨﺘﺞ ﻗﻴﻤﺔ 𝛼 ‪.‬‬

‫‪ •4‬اﻟﻨﻘﻄﺔ ‪ ، E‬ﻧﻘﻄﺔ ﺗﻘﺎﻃﻊ )‪ (AC‬و )‪ ، (OF‬ﻫﻲ ﻣﺮﻛﺰ اﳌﺴﺎﻓﺎت اﳌﺘﻨﺎﺳﺒﺔ ﻟﻠﺠﻤﻠﺔ })‪ . {(A, 𝛽) , (C, 4‬أوﺟﺪ ﻗﻴﻤﺔ 𝛽 ‪.‬‬

‫اﳉﺰء اﻟﺜﺎﲏ‬

‫ﻧﻌﺘﱪ اﻟﺪواﺋﺮ اﻵﺗﻴﺔ ‪:‬‬


‫اﻟﺪاﺋﺮة 𝒞 ‪ ،‬اﳌﺤﻴﻄﺔ ﺑﺎﳌﺜﻠﺚ ‪ ، BOF‬و اﻟﺘﻲ ﻣﺮﻛﺰﻫﺎ ‪ Ω‬و ﻧﺼﻒ ﻗﻄﺮﻫﺎ ‪. r‬‬ ‫•‬

‫اﻟﺪاﺋﺮة 𝒞 ‪ ،‬اﳌﺤﻴﻄﺔ ﺑﺎﳌﺜﻠﺚ ‪ ، OEC‬و اﻟﺘﻲ ﻣﺮﻛﺰﻫﺎ ‪ Ω‬و ﻧﺼﻒ ﻗﻄﺮﻫﺎ ‪. r‬‬
‫‪ņőƱŀ Ţ‬‬
‫•‬

‫اﻟﺪاﺋﺮة 𝒞 ‪ ،‬اﳌﺤﻴﻄﺔ ﺑﺎﳌﺜﻠﺚ ‪ ، ABC‬و اﻟﺘﻲ ﻣﺮﻛﺰﻫﺎ ‪ Ω‬و ﻧﺼﻒ ﻗﻄﺮﻫﺎ ‪. r‬‬ ‫•‬

‫اﻟﺪاﺋﺮة 𝒞 ‪ ،‬اﳌﺤﻴﻄﺔ ﺑﺎﳌﺜﻠﺚ ‪ ، AEF‬و اﻟﺘﻲ ﻣﺮﻛﺰﻫﺎ ‪ Ω‬و ﻧﺼﻒ ﻗﻄﺮﻫﺎ ‪. r‬‬ ‫•‬

‫‪ •1‬أوﺟﺪ اﻷﻋﺪاد ‪ z ، z ، z‬و ‪ z‬ﻟﻮاﺣﻖ اﻟﻨﻘﻂ ‪ Ω ، Ω ، Ω‬و ‪ Ω‬ﻋﲆ اﻟﱰﺗﻴﺐ‪.‬‬


‫‪VI‬‬
‫أﺣﺴﺐ ‪ r ، r ، r‬و ‪. r‬‬
‫‪ •2‬أﺣﺴﺐ اﻟﻌﺪدﻳﻦ ‪ z − z‬و ‪ z − z‬ﻋﲆ اﻟﺸﻜﻞ اﳉﱪي‪.‬‬
‫‪ƕ‬‬

‫ﻣﺎ اﻟﺬي ﺗﺴﺘﻨﺘﺠﻪ ﺑﺎﻟﻨﺴﺒﺔ ﻟﻠﺮﺑﺎﻋﻲ ‪ Ω Ω Ω Ω‬؟‬


‫‪z −z‬‬
‫ﻋﲆ اﻟﺸﻜﻞ اﳉﱪي‪.‬‬ ‫‪ •3‬أﺣﺴﺐ اﻟﻌﺪد‬
‫‪z −z‬‬
‫‪ũŏ‬‬

‫‪Ω Ω Ω Ω‬‬ ‫ﻣﺎ اﻟﺬي ﺗﺴﺘﻨﺘﺠﻪ ﺑﺎﻟﻨﺴﺒﺔ ﻟﻠﺮﺑﺎﻋﻲ‬

‫ﺍﳊﻞّ‪.‬‬
‫اﳉﺰء اﻷول‬

‫𝟲𝟯𝟮‬

‫‪http ://tinyurl.com/Malki1718‬‬ ‫‪0‬‬


‫ ﲤﺎرﻳﻦ ﺗﻄﺒﻴﻘﻴﺔ‬.VI
.

Ŕž Ŧ Ľ
4
Ω4 E

œǃ
F

2
Ω3
Ω1
1

Ŀ
B C
O 1 2 3 Ω2 4 5 6 7

−1

26.VI ‫ﺷﻜﻞ‬

. 26.VI ‫• أﻧﻈﺮ اﻟﺸﻜﻞ‬1


: ‫• ﻟﺪﻳﻨﺎ‬2
z 4 + 3ı 4 + 3ı 4 + 3ı 1
= = = = = −ı
z −z (4 + 4ı) − 7 −3 + 4ı ı (3ı + 4) ı

z −z 𝜋 z −z
CA, OF = arg =− (mod 2𝜋) ‫ﻣﻨﻪ‬ ّ ‫ ﻣﺘﻌﺎﻣﺪان‬OF ‫ و‬CA ‫اﻟﺸﻌﺎﻋﺎن‬
= −ı ‫ﻷن‬ •3
z −z 2 z −z
. (53 ‫ ﺻﻔﺤﺔ‬1 ‫ )أﻧﻈﺮ أﻳﻀ ًﺎ اﻟﻼزﻣﺔ‬CA ⟂ OF ‫أي‬
(3 + 𝛼 + (25 − 𝛼)) OF = 𝛼OA+(25 − 𝛼) OC ‫ ﲢﻘﻖ‬F ‫ اﻟﻨﻘﻄﺔ‬،‫ﺣﺴﺐ ﺗﻌﺮﻳﻒ ﻣﺮﻛﺰ اﳌﺴﺎﻓﺎت اﳌﺘﻨﺎﺳﺒﺔ‬ (‫)ا‬ •4
𝛼 25 − 𝛼
. OF = OA + OC ‫ ﻣﻨﻪ‬28OF = 𝛼OA + (25 − 𝛼) OC ‫أي‬
28 28
: ‫)ب( ﻟﺪﻳﻨﺎ‬
ņőƱŀ Ţ
𝛼 25 − 𝛼
CA ⋅ OF = 0 ⟺ CA ⋅ OA + CA ⋅ OC = 0
28 28
4−7 4−0 4−7 7−0
⟺ 𝛼 ⋅ + (25 − 𝛼) ⋅ =0
4−0 4−0 4−0 0−0
VI

⟺ 𝛼 (−3 × 4 + 4 × 4) + (25 − 𝛼) (−3 × 7 + 0 × 0) = 0


⟺ 4𝛼 − 21 (25 − 𝛼) = 0 ⟺ 𝛼 = 21

ّ CA ⟂ OF ‫ و‬E ∈ (OF) ‫أن‬


: ‫ ﻣﻨﻪ‬CA ⟂ OE ‫ﻓﺈن‬ ّ ‫ 𝛽( و ﺑﲈ‬+ 4) OE = 𝛽OA + 4OC : ‫)ج( ﻟﺪﻳﻨﺎ‬
ƕ

0 = CA ⋅ OE = CA ⋅ (𝛽 + 4) OE
= CA ⋅ 𝛽OA + 4OC = 𝛽CA ⋅ OA + 4CA ⋅ OC
= 𝛽 (−3 × 4 + 4 × 4) + 4 (−3 × 7 + 0 × 0)
ũŏ

= 4𝛽 − 4 × 21

ّ ‫و ﺑﺎﻟﺘﺎﱄ‬
. 𝛽 = 21 ‫ﻓﺈن‬
‫اﳉﺰء اﻟﺜﺎﲏ‬

𝟮𝟯𝟳

http ://tinyurl.com/Malki1718 0
‫‪ .4.VI‬ﲤﺎرﻳﻦ ﻣﺘﻨﻮﻋﺔ‬
‫‪.‬‬
‫‪ •1‬ﻣﺮﻛﺰ اﻟﺪاﺋﺮة اﳌﺤﻴﻄﺔ ﺑﻤﺜﻠﺚ ﻫﻮ ﻧﻘﻄﺔ ﺗﻘﺎﻃﻊ ﱢ‬
‫ﻣﻨﺼﻔﺎت أﺿﻼﻋﻪ ﻣﻨﻪ ‪:‬‬

‫‪Ŕž Ŧ Ľ‬‬
‫‪3‬‬ ‫‪7‬‬ ‫‪11‬‬ ‫‪7‬‬
‫‪z = 2 + ı,‬‬ ‫‪z = ,‬‬ ‫= ‪z‬‬ ‫‪+ 2ı,‬‬ ‫‪z =4+ ı‬‬
‫‪2‬‬ ‫‪2‬‬ ‫‪2‬‬ ‫‪2‬‬
‫‪5‬‬ ‫‪7‬‬ ‫‪5‬‬ ‫‪1‬‬

‫‪œǃ‬‬
‫‪r = ,‬‬ ‫‪r = ,‬‬ ‫‪r = ,‬‬ ‫= ‪r‬‬
‫‪2‬‬ ‫‪2‬‬ ‫‪2‬‬ ‫‪2‬‬

‫‪3 3‬‬ ‫‪3 3‬‬


‫= ‪ z − z‬ﻣﻨﻪ ‪ Ω Ω = Ω Ω‬أي ‪ Ω Ω Ω Ω‬ﻣﺘﻮازي أﺿﻼع‪.‬‬ ‫ﻟﺪﻳﻨﺎ ‪ z − z = − ı :‬و ‪− ı‬‬ ‫•‬ ‫‪2‬‬
‫‪2 2‬‬ ‫‪2 2‬‬

‫‪Ŀ‬‬
‫ﻟﺪﻳﻨﺎ ‪:‬‬
‫‪z −z‬‬ ‫‪− ı‬‬ ‫‪3‬‬ ‫‪1−ı‬‬ ‫‪3‬‬ ‫‪3‬‬
‫=‬ ‫‪=−‬‬ ‫‪= − (1 − ı) = ı‬‬
‫‪z −z‬‬ ‫‪−2 − 2ı‬‬ ‫‪4‬‬ ‫‪1+ı‬‬ ‫‪8‬‬ ‫‪4‬‬
‫𝜋‬
‫■‬ ‫‪ Ω Ω , Ω Ω‬أي ‪ Ω Ω Ω Ω‬ﻣﺴﺘﻄﻴﻞ )أﻧﻈﺮ أﻳﻀ ًﺎ اﻟﻼزﻣﺔ ‪ 1‬ﺻﻔﺤﺔ ‪. (53‬‬ ‫=‬ ‫ﻣﻨﻪ )𝜋‪(mod 2‬‬
‫‪2‬‬

‫‪.‬‬
‫𝟑𝟔𝟏‬ ‫‪.‬‬
‫‪U‬‬ ‫اﳌﺴﺘﻮي ﻣﻨﺴﻮب إﱃ ﻣﻌﻠﻢ ﻣﺘﻌﺎﻣﺪ‪ ،‬ﻣﺘﺠﺎﻧﺲ و ﻣﺒﺎﴍ‬
‫‪3‬‬ ‫⃗ ‪. O, i,⃗ j‬‬
‫ﻟﺘﻜﻦ ‪ A‬اﻟﻨﻘﻄﺔ اﻟﺘﻲ ﻻﺣﻘﺘﻬﺎ ‪ 1‬و ‪ B‬اﻟﻨﻘﻄﺔ ذات اﻟﻼﺣﻘﺔ‬
‫‪2‬‬ ‫‪T‬‬ ‫‪.2‬‬
‫ﻧﻌﺘﱪ اﻟﺪاﺋﺮة 𝒞 اﻟﺘﻲ ﻣﺮﻛﺰﻫﺎ ‪ A‬و ﻧﺼﻒ ﻗﻄﺮﻫﺎ ‪ 1‬و‬
‫‪1‬‬ ‫‪M‬‬ ‫‪C‬‬
‫اﳌﺴﺘﻘﻴﻢ ‪ ، T‬اﳌﲈس ﻟﻠﺪاﺋﺮة 𝒞 ﰲ اﻟﻨﻘﻄﺔ ‪ ، B‬و اﻟﺬي‬
‫‪A‬‬ ‫‪B‬‬ ‫ﻣﻌﺎدﻟﺘﻪ ‪. x = 2‬‬
‫‪−1‬‬ ‫‪O‬‬ ‫‪1‬‬ ‫‪2‬‬ ‫‪3‬‬ ‫ﻟﻜﻞ ﻋﺪد ﺣﻘﻴﻘﻲ 𝜑 ﺑﺤﻴﺚ 𝜋 ≤ 𝜑 ≤ 𝜋‪ −‬ﻧﻀﻊ ‪:‬‬
‫)𝜑(‪z = (cos(𝜑) + 1) + ı sin‬‬
‫‪−1‬‬ ‫‪N‬‬ ‫‪V‬‬
‫أﺧﲑ ًا‪ ،‬ﻟﺘﻜﻦ ‪ M‬اﻟﻨﻘﻄﺔ اﻟﺘﻲ ﻻﺣﻘﺘﻬﺎ ‪. z‬‬
‫‪ņőƱŀ Ţ‬‬
‫‪ •1‬أﻳﻦ ﺗﺘﻮاﺟﺪ اﻟﻨﻘﻄﺔ ‪ M‬ﻋﻨﺪﻣﺎ ‪ 𝜑 = −𝜋 :‬؟ 𝜋 = 𝜑 ؟ ‪ 𝜑 = 0‬؟‬
‫‪ •2‬ﻟﻴﻜﻦ [𝜋 ‪. 𝜑 ∈ ]−𝜋,‬‬
‫)ا( أوﺟﺪ‪ ،‬ﺑﺪﻻﻟﺔ 𝜑 ‪ ،‬ﻻﺣﻘﺔ اﻟﺸﻌﺎع ‪. AM‬‬

‫‪VI‬‬
‫)ب( ﱢﺑﺮر ّ‬
‫أن اﻟﻨﻘﻄﺔ ‪ M‬ﺗﻨﺘﻤﻲ إﱃ اﻟﺪاﺋﺮة 𝒞 ‪.‬‬
‫𝝅𝟐‬
‫= 𝝋‪.‬‬ ‫‪ •3‬ﻓﻴﲈ ﻳﲇ‪ ،‬ﻧﻔﺮض ّ‬
‫أن ‪:‬‬
‫𝟑‬
‫‪1‬‬ ‫‪√3‬‬
‫‪ƕ‬‬

‫= ‪ z‬؛ و ﻟﺘﻜﻦ ‪ N‬ﻧﻈﲑة ‪ M‬ﺑﺎﻟﻨﺴﺒﺔ إﱃ ‪. A‬‬ ‫‪+ı‬‬ ‫ﻟﺘﻜﻦ ‪ M‬اﻟﻨﻘﻄﺔ اﻟﺘﻲ ﻻﺣﻘﺘﻬﺎ ‪:‬‬
‫‪2‬‬ ‫‪2‬‬
‫أوﺟﺪ ‪ Z‬ﻻﺣﻘﺔ اﻟﻨﻘﻄﺔ ‪. N‬‬
‫‪ •4‬ﺣﺪﱢ د ﻣﻌﺎدﻟﺔ دﻳﻜﺎرﺗﻴﺔ ﻟﻠﻤﺴﺘﻘﻴﻢ 𝒟 ﱢ‬
‫ﻣﻨﺼﻒ اﻟﻘﻄﻌﺔ ]‪. [MN‬‬
‫‪ũŏ‬‬

‫‪ •5‬ﻧﻌﺘﱪ اﻟﻨﻘﻄﺔ ‪ U‬ﻧﻘﻄﺔ ﺗﻘﺎﻃﻊ اﳌﺴﺘﻘﻴﻢ )‪ (OM‬ﻣﻊ اﳌﲈس ‪ T‬و ‪ V‬ﻧﻘﻄﺔ ﺗﻘﺎﻃﻊ اﳌﺴﺘﻘﻴﻢ )‪ (ON‬ﻣﻊ اﳌﲈس ‪. T‬‬
‫)ا( أوﺟﺪ ‪ u‬ﻻﺣﻘﺔ اﻟﻨﻘﻄﺔ ‪ U‬و ‪ v‬ﻻﺣﻘﺔ اﻟﻨﻘﻄﺔ ‪. V‬‬
‫)ب( أوﺟﺪ ‪ k‬ﻻﺣﻘﺔ اﻟﻨﻘﻄﺔ ‪ K‬ﻣﻨﺘﺼﻒ اﻟﻘﻄﻌﺔ ] ‪. [UV‬‬
‫ﻣﻨﺼﻒ اﻟﻘﻄﻌﺔ ] ‪. [UV‬‬
‫)ج( إﺳﺘﻨﺘﺞ ﻣﻌﺎدﻟﺔ دﻳﻜﺎرﺗﻴﺔ ﻟﻠﻤﺴﺘﻘﻴﻢ ‪ Δ‬ﱢ‬

‫𝟴𝟯𝟮‬

‫‪http ://tinyurl.com/Malki1718‬‬ ‫‪0‬‬


‫‪ .VI‬ﲤﺎرﻳﻦ ﺗﻄﺒﻴﻘﻴﺔ‬
‫‪.‬‬

‫)ا( ﺣﺪﱢ د 𝜔 ﻻﺣﻘﺔ ‪ Ω‬ﻧﻘﻄﺔ ﺗﻘﺎﻃﻊ اﳌﺴﺘﻘﻴﻤﲔ 𝒟 و ‪. Δ‬‬ ‫‪6‬‬


‫•‬

‫‪Ŕž Ŧ Ľ‬‬
‫ﲤﺮ ﺑﺎﻟﻨﻘﻄﺔ ‪. M‬‬‫)ب( أرﺳﻢ اﳌﺴﺘﻘﻴﻤﲔ 𝒟 و ‪ Δ‬و اﻟﺪاﺋﺮة 𝒞 اﻟﺘﻲ ﻣﺮﻛﺰﻫﺎ ‪ Ω‬و ّ‬
‫ﻣﺎ ﻫﻲ اﻟﻨﻘﻂ اﻟﺜﻼث اﳌﺬﻛﻮرة أﻋﻼه‪ ،‬ﺑﺎﺳﺘﺜﻨﺎء اﻟﻨﻘﻄﺔ ‪ ، M‬و اﻟﺘﻲ ﺗﻨﺘﻤﻲ إﱃ اﻟﺪاﺋﺮة 𝒞 ؟‬

‫‪œǃ‬‬
‫ﺍﳊﻞّ‪ •1 .‬ﻟﺪﻳﻨﺎ ‪ z− = (1 + cos (−𝜋)) + ı sin (−𝜋) = 0 :‬ﻣﻨﻪ ‪ . M− = O‬ﺑﺎﳌﺜﻞ‪ z = 0 ،‬ﻣﻨﻪ ‪ M = O‬و‬

‫‪Ŀ‬‬
‫‪ z = 1‬ﻣﻨﻪ ‪. M = B‬‬
‫‪.z‬‬ ‫‪−z‬‬ ‫‪= z − 1 = cos 𝜑 + ı sin 𝜑 = e‬‬ ‫)ا( ﻻﺣﻘﺔ اﻟﺸﻌﺎع ‪ AM‬ﻫﻲ‬ ‫‪2‬‬
‫•‬

‫أن ‪ّ AM = ||z − 1|| = ||e || = 1‬‬


‫ﻓﺈن 𝒞 ∈ ‪. M‬‬ ‫)ب( ﺑﲈ ّ‬
‫‪ •3‬ﻟﺪﻳﻨﺎ ‪ AN = −AM :‬ﻣﻨﻪ ) ‪ Z − z = − (z − z‬ﻣﻨﻪ ‪:‬‬
‫‪1‬‬ ‫‪√3‬‬ ‫‪3‬‬ ‫‪√3‬‬
‫‪Z = −z + 2z‬‬ ‫‪=−‬‬ ‫‪+ı‬‬ ‫=‪+2‬‬ ‫‪−ı‬‬
‫‪2‬‬ ‫‪2‬‬ ‫‪2‬‬ ‫‪2‬‬

‫اﳌﻨﺼﻒ 𝒟 ﻫﻮ ﳎﻤﻮﻋﺔ اﻟﻨﻘﻂ ‪ P‬ﺑﺤﻴﺚ ‪ . PM = PN‬ﻟﻴﻜﻦ ‪ x‬و ‪ y‬إﺣﺪاﺛ َﻴ ْﻲ اﻟﻨﻘﻄﺔ ‪ . P‬ﻟﺪﻳﻨﺎ ‪:‬‬
‫ﱢ‬ ‫‪•4‬‬

‫‪1‬‬ ‫‪√3‬‬ ‫‪3‬‬ ‫‪√3‬‬


‫⟺ ‪PM = PN‬‬ ‫‪x−‬‬ ‫‪+‬‬ ‫‪y−‬‬ ‫=‬ ‫‪x−‬‬ ‫‪+‬‬ ‫‪y+‬‬
‫‪2‬‬ ‫‪2‬‬ ‫‪2‬‬ ‫‪2‬‬

‫‪1‬‬ ‫‪√3‬‬ ‫‪3‬‬ ‫‪√3‬‬


‫⟺‬ ‫‪x−‬‬ ‫‪+‬‬ ‫‪y−‬‬ ‫=‬ ‫‪x−‬‬ ‫‪+‬‬ ‫‪y+‬‬
‫‪2‬‬ ‫‪2‬‬ ‫‪2‬‬ ‫‪2‬‬
‫‪⟺ x + y − x − √3y + 1 = x + y − 3x + √3y + 3‬‬
‫‪⟺ x − √3y − 1 = 0‬‬

‫إذن ﻣﻌﺎدﻟﺔ 𝒟 ﻫﻲ ‪. x − √3y − 1 = 0‬‬


‫‪ņőƱŀ Ţ‬‬
‫)ا( ﻣﻌﺎدﻟﺔ )‪ (OM‬ﻣﻦ اﻟﺸﻜﻞ ‪ y = ax‬ﺣﻴﺚ ‪ a‬ﻋﺪد ﺣﻘﻴﻘﻲ‪.‬‬ ‫‪5‬‬
‫•‬

‫‪3‬‬ ‫‪1‬‬
‫ﻓﺈن × ‪ √ = a‬ﻣﻨﻪ ‪ a = √3‬و ﺑﺎﻟﺘﺎﱄ‪ ،‬ﻣﻌﺎدﻟﺔ )‪ (OM‬ﻫﻲ ‪. y = √3x‬‬
‫أن )‪ّ M ∈ (OM‬‬
‫ﺑﲈ ّ‬
‫‪2‬‬ ‫‪2‬‬
‫أن )‪ّ U ∈ (OM‬‬
‫ﻓﺈن ‪y = √3x = 2√3‬‬ ‫ﻓﺈن ‪ x = 2‬و ﺑﲈ ّ‬
‫‪ّ U∈T‬‬ ‫ﻟﻴﻜﻦ ‪ x‬و ‪ y‬إﺣﺪاﺛ َﻴ ْﻲ اﻟﻨﻘﻄﺔ ‪ . U‬ﺑﲈ ّ‬
‫أن‬
‫‪VI‬‬

‫إذن ‪ U = 2, 2√3‬أي ‪. u = 2 + 2ı√3‬‬


‫ﺑﺎﳌﺜﻞ‪ ،‬ﻣﻌﺎدﻟﺔ )‪ (ON‬ﻣﻦ اﻟﺸﻜﻞ ‪ y = bx‬ﺣﻴﺚ ‪ b‬ﻋﺪد ﺣﻘﻴﻘﻲ‪.‬‬
‫‪3‬‬ ‫‪3‬‬ ‫‪3‬‬ ‫‪3‬‬
‫ﻓﺈن × ‪ − √ = b‬ﻣﻨﻪ √ ‪ b = −‬إذن ﻣﻌﺎدﻟﺔ )‪ (ON‬ﻫﻲ ‪. y = − √ x‬‬
‫أن )‪ّ N ∈ (ON‬‬
‫ﺑﲈ ّ‬
‫‪3‬‬ ‫‪3‬‬ ‫‪2‬‬ ‫‪2‬‬
‫‪ƕ‬‬

‫‪√3‬‬ ‫‪2 3‬‬


‫√ ‪ y = − x = −‬ﻣﻨﻪ‬ ‫أن )‪ّ V ∈ (ON‬‬
‫ﻓﺈن‬ ‫ﻓﺈن ‪ x = 2‬و ﺑﲈ ّ‬
‫أن ‪ّ V ∈ T‬‬
‫ﻟﻴﻜﻦ ‪ x‬و ‪ y‬إﺣﺪاﺛ َﻴ ْﻲ ‪ . V‬ﺑﲈ ّ‬
‫‪3‬‬ ‫‪3‬‬
‫‪2ı 3‬‬ ‫‪2 3‬‬
‫√ ‪ V = 2, −‬أي √ ‪. v = 2 −‬‬
‫‪3‬‬ ‫‪3‬‬
‫‪ũŏ‬‬

‫)ب( ﻟﺪﻳﻨﺎ ‪:‬‬


‫‪u+v 1‬‬ ‫‪2ı√3‬‬ ‫‪2ı√3‬‬
‫=‪k‬‬ ‫=‬ ‫‪2 + 2ı√3 + 2 −‬‬ ‫‪=2+‬‬
‫‪2‬‬ ‫‪2‬‬ ‫‪3‬‬ ‫‪3‬‬

‫ﻣﻨﺼﻒ اﻟﻘﻄﻌﺔ ] ‪ [UV‬أي ‪ Δ‬ﻫﻲ ﻣﻦ اﻟﺸﻜﻞ ‪y = c‬‬ ‫ﻣﻌﺎدﻟﺔ اﳌﺴﺘﻘﻴﻢ )‪ (UV‬ﻫﻲ ‪ x = 2‬و ﺑﺎﻟﺘﺎﱄ ّ‬
‫ﻓﺈن ﻣﻌﺎدﻟﺔ ﱢ‬ ‫)ج(‬
‫‪2 3‬‬
‫ﺣﻴﺚ ‪ . c ∈ ℝ‬ﻟﻜﻦ ‪ K ∈ Δ‬إذن ﻣﻌﺎدﻟﺔ ‪ Δ‬ﻫﻲ √ = ‪. y‬‬
‫‪3‬‬

‫𝟵𝟯𝟮‬

‫‪http ://tinyurl.com/Malki1718‬‬ ‫‪0‬‬


‫‪ .4.VI‬ﲤﺎرﻳﻦ ﻣﺘﻨﻮﻋﺔ‬
‫‪.‬‬

‫‪4‬‬

‫‪Ŕž Ŧ Ľ‬‬
‫‪U‬‬ ‫‪C′‬‬
‫‪3‬‬

‫‪œǃ‬‬
‫‪T‬‬
‫‪2‬‬
‫‪D‬‬

‫‪K‬‬ ‫‪Ω‬‬
‫‪1‬‬ ‫‪M‬‬ ‫‪C‬‬

‫‪Ŀ‬‬
‫‪A‬‬ ‫‪B‬‬
‫‪−1‬‬ ‫‪O‬‬ ‫‪1‬‬ ‫‪2‬‬ ‫‪3‬‬ ‫‪4‬‬ ‫‪5‬‬ ‫‪6‬‬

‫‪−1‬‬
‫‪N‬‬ ‫‪V‬‬

‫‪−2‬‬

‫ﺷﻜﻞ ‪27.VI‬‬

‫‪2√3‬‬
‫× ‪x − √3‬‬ ‫𝒟∈‪ّ Ω‬‬
‫ﻓﺈن ‪− 1 = 0‬‬ ‫ﻓﺈن ‪ . y = 2√3‬و ﺑﲈ ّ‬
‫أن‬ ‫أن ‪ّ Ω ∈ Δ‬‬
‫)ا( ﻟﻴﻜﻦ ‪ x‬و ‪ y‬إﺣﺪاﺛ َﻴ ْﻲ ‪ . Ω‬ﺑﲈ ّ‬ ‫•‬ ‫‪6‬‬
‫‪3‬‬ ‫‪3‬‬
‫‪2ı 3‬‬
‫ﻣﻨﻪ ‪ x = 3‬إذن √ ‪. 𝜔 = 3 +‬‬
‫‪3‬‬
‫أﻧﻈﺮ اﻟﺸﻜﻞ ‪27.VI‬‬ ‫)ب(‬
‫|‬ ‫‪2ı 3‬‬ ‫| ‪2ı 3‬‬ ‫‪57‬‬
‫ﻟﺪﻳﻨﺎ ‪ Ω ∈ Δ :‬ﻣﻨﻪ ‪. ΩU = ΩV = | 2 − √ − 3 + √ | = √ :‬‬
‫||‬ ‫‪3‬‬ ‫‪3‬‬ ‫||‬ ‫‪3‬‬
‫‪| 1‬‬ ‫‪3‬‬ ‫‪2ı√3‬‬ ‫‪| √57‬‬
‫√ ‪. ΩM = ΩN = | + ı‬‬ ‫‪−‬‬ ‫‪3+‬‬ ‫=|‬ ‫ﺑﺎﳌﺜﻞ‪ Ω ∈ 𝒟 ،‬ﻣﻨﻪ ‪:‬‬
‫‪|| 2‬‬ ‫‪2‬‬ ‫‪3‬‬ ‫||‬ ‫‪3‬‬
‫إذن ‪ ΩM = ΩN = ΩU = ΩV‬أي ّ‬
‫أن اﻟﻨﻘﻂ ‪ U ، N‬و ‪ V‬ﺗﻨﺘﻤﻲ إﱃ اﻟﺪاﺋﺮة 𝒞 ‪.‬‬
‫■‬
‫‪ņőƱŀ Ţ‬‬
‫‪.‬‬
‫𝟒𝟔𝟏‬ ‫‪.‬‬
‫ﰲ اﳌﺴﺘﻮي اﳌﺮﻛﺐ اﳌﻨﺴﻮب إﱃ ﻣﻌﻠﻢ ﻣﺘﻌﺎﻣﺪ و ﻣﺘﺠﺎﻧﺲ ⃗‪ ، O, u,⃗ v‬ﻧﻌﺘﱪ اﻟﻨﻘﻂ ‪ I ، A‬و ‪ B‬ذوات‬

‫‪VI‬‬
‫اﻟﻠﻮاﺣﻖ ‪:‬‬
‫‪z‬‬ ‫‪=1,‬‬ ‫‪z =2,‬‬ ‫‪z =3‬‬
‫‪1‬‬
‫= ‪ z‬و ﻧﻌﺘﱪ اﻟﺪاﻟﺔ ‪ F‬اﻟﺘﻲ ﺗﺮﻓﻖ ﺑﻜﻞ ﻧﻘﻄﺔ ‪ M‬ﻣﻦ اﳌﺴﺘﻮي‪ ،‬ﲣﺘﻠﻒ‬ ‫ﻟﻜﻞ ﻋﺪد ﻣﺮﻛﺐ ‪ z‬ﳜﺘﻠﻒ ﻋﻦ ‪ 2‬ﻧﻀﻊ ‪+ 2 :‬‬
‫‪z−2‬‬
‫ﻋﻦ ‪ I‬و اﻟﺘﻲ ﻻﺣﻘﺘﻬﺎ ‪ ، z‬اﻟﻨﻘﻄﺔ ‪ M‬ذات اﻟﻼﺣﻘﺔ ‪. z‬‬
‫‪ƕ‬‬

‫‪ •1‬أوﺟﺪ اﳌﺠﻤﻮﻋﺔ ‪ ℰ‬ﻟﻠﻨﻘﻂ ‪ M‬ﺑﺤﻴﺚ ‪ . F (M) = M‬ﻋ ﱢﻠﻞ إﺟﺎﺑﺘﻚ‪.‬‬

‫)ا( أﺣﺴﺐ‪ ،‬ﺑﺪﻻﻟﺔ ‪ ، z‬ﻻﺣﻘﺘ َْﻲ اﻟﺸﻌﺎﻋﲔ ‪ IM‬و ‪. IM‬‬ ‫•‬ ‫‪2‬‬
‫‪ũŏ‬‬

‫)ب( إﺳﺘﻨﺘﺞ ﻋﻼﻗﺔ ﺗﺮﺑﻂ ﺑﲔ اﻟﻄﻮﻟﲔ ‪ IM‬و ‪ IM‬و ﻋﻼﻗﺔ ﺑﲔ اﻟﺰاوﻳﺘﲔ ‪ u,⃗ IM‬و ‪. u,⃗ IM‬‬
‫‪ •3‬ﻧﻌﺘﱪ ﻧﻘﻄﺔ ‪ M‬ﲣﺘﻠﻒ ﻋﻦ ‪ ، I‬ﻋﻦ ‪ A‬و ﻋﻦ ‪ B‬و ﻟﺘﻜﻦ ‪ z‬ﻻﺣﻘﺘﻬﺎ‪.‬‬
‫‪1−z‬‬ ‫‪1−z‬‬
‫‪.‬‬ ‫𝛽=‬ ‫)ا( أوﺟﺪ اﻟﻌﺪد اﳊﻘﻴﻘﻲ 𝛽 ﺑﺤﻴﺚ ‪:‬‬
‫‪3−z‬‬ ‫‪3−z‬‬

‫𝟬𝟰𝟮‬

‫‪http ://tinyurl.com/Malki1718‬‬ ‫‪0‬‬


‫‪ .VI‬ﲤﺎرﻳﻦ ﺗﻄﺒﻴﻘﻴﺔ‬
‫‪.‬‬

‫‪MA‬‬ ‫‪MA‬‬
‫‪. MB, MA‬‬ ‫‪M B, M A‬‬ ‫و ﻋﻼﻗﺔ ﺑﲔ اﻟﺰاوﻳﺘﲔ‬ ‫و‬
‫إﺳﺘﻨﺘﺞ ﻋﻼﻗﺔ ﺑﲔ‬ ‫)ب(‬

‫‪Ŕž Ŧ Ľ‬‬
‫‪MB‬‬ ‫‪MB‬‬
‫ﻣﻨﺼﻒ اﻟﻘﻄﻌﺔ ]‪ . [AB‬ﻣﺎ اﻟﺬي ﺗﺴﺘﻨﺘﺠﻪ ﺑﺎﻟﻨﺴﺒﺔ ﻟﻠﻨﻘﻄﺔ ‪ M‬؟ ﱢﺑﺮر‬ ‫ﻧﻔﺮض ّ‬
‫أن اﻟﻨﻘﻄﺔ ‪ M‬ﺗﻨﺘﻤﻲ إﱃ اﳌﺴﺘﻘﻴﻢ ‪ Δ‬ﱢ‬ ‫)ج(‬

‫‪œǃ‬‬
‫إﺟﺎﺑﺘﻚ‪.‬‬

‫‪ •1‬ﻟﺪﻳﻨﺎ ‪:‬‬ ‫ﺍﳊﻞّ‪.‬‬

‫‪Ŀ‬‬
‫‪1‬‬
‫⟺ ‪F (M) = M ⟺ z = z‬‬ ‫‪+2=z‬‬
‫‪z−2‬‬
‫‪1‬‬
‫=‪⟺ z−2‬‬ ‫‪⟺ (z − 2) = 1 ⟺ z − 2 = ±1‬‬
‫‪z−2‬‬
‫‪ M = B‬أو ‪ z = 3 ⟺ M = A‬أو ‪⟺ z = 1‬‬
‫إذن ‪. ℰ = {A, B} :‬‬
‫)ا( ﻻﺣﻘﺔ اﻟﺸﻌﺎع ‪ IM‬ﻫﻲ ‪ z − z‬أي ‪ z − 2‬؛‬ ‫•‬‫‪2‬‬
‫‪1‬‬
‫= ‪.z −2‬‬ ‫‪ z‬أي‬ ‫‪′‬‬ ‫‪−z‬‬ ‫و ﻻﺣﻘﺔ اﻟﺸﻌﺎع ‪ IM‬ﻫﻲ‬
‫‪z−2‬‬
‫‪1‬‬
‫= ‪ . IM‬ﻣﻦ ﺟﻬﺔ أﺧﺮى‪ ،‬ﻟﺪﻳﻨﺎ ‪:‬‬ ‫)ب( ﻣﻦ اﻟﺴﺆال اﻟﺴﺎﺑﻖ ﻧﺴﺘﻨﺘﺞ ّ‬
‫أن ‪:‬‬
‫‪IM‬‬

‫‪u,⃗ IM‬‬ ‫‪= arg z‬‬ ‫‪′‬‬ ‫‪−z‬‬ ‫)𝜋‪(mod 2𝜋) = arg z − 2 (mod 2𝜋) = arg z − 2 (mod 2‬‬
‫‪1‬‬
‫‪= arg‬‬ ‫‪(mod 2𝜋) = − arg (z − 2) (mod 2𝜋) = − u,⃗ IM‬‬
‫‪z−2‬‬

‫)ا( ﻟﺪﻳﻨﺎ ‪:‬‬ ‫•‬‫‪3‬‬


‫‪1−z‬‬ ‫‪−1 −‬‬ ‫‪−‬‬ ‫‪− (z − 2) − 1‬‬ ‫‪z−1‬‬ ‫‪1−z‬‬
‫=‬ ‫=‬ ‫‪=−‬‬ ‫‪=−‬‬
‫‪3−z‬‬ ‫‪(z − 2) − 1‬‬ ‫‪z−3‬‬ ‫‪3−z‬‬
‫‪ņőƱŀ Ţ‬‬
‫‪1−‬‬ ‫‪−‬‬

‫إذن ‪. 𝛽 = −1 :‬‬
‫‪M A MA‬‬
‫‪ .‬ﻣﻦ ﺟﻬﺔ أﺧﺮى ﻟﺪﻳﻨﺎ ‪:‬‬ ‫=‬ ‫)ب( ﻣﻦ اﻟﺴﺆال اﻟﺴﺎﺑﻖ ﻧﺴﺘﻨﺘﺞ ّ‬
‫أن ‪:‬‬
‫‪MB‬‬ ‫‪MB‬‬
‫‪VI‬‬

‫‪1−z‬‬ ‫‪1−z‬‬
‫‪M B, M A = arg‬‬ ‫‪(mod 2𝜋) = arg −‬‬ ‫)𝜋‪(mod 2‬‬
‫‪3−z‬‬ ‫‪3−z‬‬
‫‪1−z‬‬
‫=‬ ‫‪arg (−1) + arg‬‬ ‫)𝜋‪(mod 2‬‬
‫‪3−z‬‬
‫‪ƕ‬‬

‫‪= 𝜋 + MB, MA‬‬ ‫)𝜋‪(mod 2‬‬

‫)ج( ﻟﺪﻳﻨﺎ ‪:‬‬


‫‪MA‬‬
‫‪ũŏ‬‬

‫⟹ ‪M ∈ Δ ⟹ MA = MB‬‬ ‫‪=1‬‬
‫‪MB‬‬
‫‪MA‬‬
‫⟹‬ ‫‪=1‬‬ ‫)ﺣﺴﺐ اﻟﺴﺆال اﻟﺴﺎﺑﻖ(‬
‫‪MB‬‬
‫‪⟹ MA=MB ⟹ M ∈Δ‬‬

‫إذن اﻟﻨﻘﻄﺔ ‪ M‬أﻳﻀ ًﺎ ﺗﻨﺘﻤﻲ إﱃ ‪ Δ‬ﱢ‬


‫ﻣﻨﺼﻒ اﻟﻘﻄﻌﺔ ]‪.[AB‬‬
‫■‬

‫𝟭𝟰𝟮‬

‫‪http ://tinyurl.com/Malki1718‬‬ ‫‪0‬‬


‫‪ .4.VI‬ﲤﺎرﻳﻦ ﻣﺘﻨﻮﻋﺔ‬
‫‪.‬‬
‫‪C‬‬ ‫‪M1‬‬
‫‪1‬‬

‫‪Ŕž Ŧ Ľ‬‬
‫‪0.5‬‬

‫‪œǃ‬‬
‫‪B‬‬ ‫‪O‬‬ ‫‪A‬‬
‫‪−1‬‬ ‫‪−0.5‬‬ ‫‪0.5‬‬ ‫‪1‬‬ ‫‪1.5‬‬ ‫‪2‬‬

‫‪−0.5‬‬

‫‪Ŀ‬‬
‫‪−1‬‬ ‫‪M2‬‬

‫ﺷﻜﻞ ‪28.VI‬‬

‫‪.‬‬
‫𝟓𝟔𝟏‬ ‫‪.‬‬
‫اﳌﺴﺘﻮي اﳌﺮﻛﺐ ﻣﻨﺴﻮب إﱃ ﻣﻌﻠﻢ ﻣﺘﻌﺎﻣﺪ و ﻣﺘﺠﺎﻧﺲ ⃗‪. O, u,⃗ v‬‬
‫‪1‬‬
‫ﻟﺘﻜﻦ ‪ F‬اﻟﺪاﻟﺔ اﻟﺘﻲ ﺗﺮﻓﻖ ﺑﻜﻞ ﻧﻘﻄﺔ ‪ M‬ﻻﺣﻘﺘﻬﺎ ‪ z‬اﻟﻨﻘﻄﺔ ‪ M‬ذات اﻟﻼﺣﻘﺔ ‪ z‬ﺑﺤﻴﺚ ‪. z = − :‬‬
‫‪z‬‬

‫‪) •1‬ا( ﱢ‬
‫ﻋﱪ ﻋﻦ || ‪ ||z‬ﺑﺪﻻﻟﺔ |‪. |z‬‬
‫ﻋﲔ ﻋﻤﺪ ًة ﻟﻠﻌﺪد ‪ z‬ﺑﺪﻻﻟﺔ )‪ ، arg (z‬ﻋﻤﺪة ‪. z‬‬ ‫)ب( ﱢ‬
‫ﻣﺎ اﻟﺬي ﻳﻤﻜﻦ اﺳﺘﻨﺘﺎﺟﻪ ﺑﺎﻟﻨﺴﺒﺔ ﻟﻠﻨﻘﻂ ‪ M ، O‬و ‪ M‬؟‬
‫‪ •2‬ﻧﻌﺘﱪ اﻟﺪاﺋﺮة ‪ Γ‬اﻟﺘﻲ ﻣﺮﻛﺰﻫﺎ ‪ O‬و ﻧﺼﻒ ﻗﻄﺮﻫﺎ ‪ . 1‬ﻟﺘﻜﻦ ‪ M‬ﻧﻘﻄﺔ ﻣﻦ ‪ Γ‬ﻻﺣﻘﺘﻬﺎ ‪. z‬‬
‫)ا( أوﺟﺪ || ‪. ||z‬‬
‫)ب( ﻣﺎ ﻫﻲ ﺻﻮرة اﻟﺪاﺋﺮة ‪ Γ‬ﺑﺎﻟﺪاﻟﺔ ‪ F‬؟‬
‫‪ņőƱŀ Ţ‬‬
‫𝒞 اﻟﺪاﺋﺮة اﻟﺘﻲ ﻣﺮﻛﺰﻫﺎ ‪ A‬و ﲤﺮ ﺑﺎﻟﻨﻘﻄﺔ ‪ . O‬ﻟﺘﻜﻦ ‪M‬‬ ‫ﺻﻮرﰐ اﻟﻌﺪدﻳﻦ ‪ 1‬و ‪ −1‬ﻋﲆ اﻟﱰﺗﻴﺐ و ﻟﺘﻜﻦ‬
‫َْ‬ ‫‪ •3‬ﻟﺘﻜﻦ ‪ A‬و ‪B‬‬
‫ﻧﻘﻄﺔ ﻣﻦ اﳌﺴﺘﻮي ﻻﺣﻘﺘﻬﺎ ‪. z‬‬
‫)ا( ﻣﺎ ﻫﻮ اﻟﴩط اﻟﺬي ﳚﺐ أن ُﳛﻘﻘﻪ اﻟﻌﺪد ‪ z‬ﺣﺘﻰ ﺗﻜﻮن ‪ M‬ﻧﻘﻄﺔ ﻣﻦ 𝒞 ؟‬

‫‪VI‬‬
‫)ب( ﻧﻔﺮض ّ‬
‫أن ‪ M‬ﻧﻘﻄﺔ ﻣﻦ 𝒞 ﲣﺘﻠﻒ ﻋﻦ ‪. O‬‬
‫|‪|z + 1‬‬
‫| ُﻣﺒ ﱢﻴﻨ ًﺎ ﺗﻔﺎﺻﻴﻞ اﳊﺴﺎب‪.‬‬ ‫)‪ (i‬أﺣﺴﺐ |‬
‫|| ‪|| z‬‬
‫)‪ (ii‬ﻗﺎرن اﻟﻄﻮﻟﲔ ‪ BM‬و ‪. OM‬‬
‫‪ƕ‬‬

‫)‪ (iii‬إﺳﺘﻨﺘﺞ ّ‬
‫أن اﻟﻨﻘﻄﺔ ‪ M‬ﺗﻨﺘﻤﻲ إﱃ ﻣﺴﺘﻘﻴﻢ ﺛﺎﺑﺖ )‪ُ (D‬ﻳﻄﻠﺐ ﺗﻌﻴﻴﻨﻪ‪.‬‬
‫ﺻﻮرﰐ ‪ M‬و ‪ M‬ﻋﲆ اﻟﱰﺗﻴﺐ‪.‬‬
‫َْ‬ ‫)ج( أﻧﺸﺊ ﻋﲆ اﻟﺸﻜﻞ ‪ 28.VI‬اﻟﻨﻘﻄﺘﲔ ‪ M‬و ‪M‬‬
‫)د( ﻣﺎ ﻫﻲ ﺻﻮرة اﻟﺪاﺋﺮة 𝒞 ‪ ،‬ﺑﺎﺳﺘﺜﻨﺎء اﻟﻨﻘﻄﺔ ‪ ، O‬ﺑﺎﻟﺪاﻟﺔ ‪ F‬؟‬
‫‪ũŏ‬‬

‫|‪| 1| |1‬‬
‫ﺍﳊﻞّ‪.‬‬
‫‪1‬‬ ‫‪1‬‬
‫)ا( ﻟﺪﻳﻨﺎ ‪. ||z || = |− | = | | = | | = :‬‬ ‫•‬ ‫‪1‬‬
‫|| ‪|| z‬‬ ‫|| ‪|| z‬‬ ‫|‪|z‬‬ ‫|‪|z‬‬
‫)ب( ﻟﺪﻳﻨﺎ ‪:‬‬
‫‪arg z‬‬ ‫)𝜋‪= arg (−1)−arg (z) (mod 2𝜋) = 𝜋−(− arg (z)) (mod 2𝜋) = arg (z)+𝜋 (mod 2‬‬

‫𝟮𝟰𝟮‬

‫‪http ://tinyurl.com/Malki1718‬‬ ‫‪0‬‬


‫‪ .VI‬ﲤﺎرﻳﻦ ﺗﻄﺒﻴﻘﻴﺔ‬
‫‪.‬‬
‫ﺣﺴﺐ اﳌﱪﻫﻨﺔ ‪ 24‬ﺻﻔﺤﺔ ‪ّ 52‬‬
‫ﻓﺈن ‪:‬‬
‫‪z −0‬‬
‫‪OM , OM = arg‬‬ ‫)𝜋‪(mod 2𝜋) = arg z − arg (z) (mod 2𝜋) = 𝜋 (mod 2‬‬

‫‪Ŕž Ŧ Ľ‬‬
‫‪z−0‬‬
‫أن اﻟﻨﻘﻂ ‪ M ، O‬و ‪ M‬ﻋﲆ اﺳﺘﻘﺎﻣﺔ واﺣﺪة و ّ‬
‫أن اﻟﻨﻘﻄﺔ ‪ O‬ﺗﻨﺘﻤﻲ إﱃ اﻟﻘﻄﻌﺔ اﳌﺴﺘﻘﻴﻤﺔ ‪. MM‬‬ ‫و ﻫﺬا ﻳﻌﻨﻲ ّ‬

‫‪œǃ‬‬
‫‪1‬‬
‫= || ‪. ||z‬‬ ‫‖‪‖OM‬‬
‫‖ = |‪ |z‬ﻣﻨﻪ ‪= 1‬‬ ‫إذا ﻛﺎﻧﺖ )‪ّ M ∈ Γ (O, 1‬‬
‫ﻓﺈن ‪‖ = 1‬‬ ‫)ا(‬ ‫‪2‬‬
‫•‬
‫|‪|z‬‬
‫أن )‪ M ∈ Γ (O, 1‬أي ‪. F (Γ) ⊂ Γ‬‬ ‫ﻓﺈن ‪ ‖‖OM ‖‖ = 1‬و ﻫﺬا ﻳﻌﻨﻲ ّ‬
‫أن ‪ّ ||z − 0|| = ||z || = 1‬‬
‫ﺑﲈ ّ‬ ‫)ب(‬

‫‪Ŀ‬‬
‫ﻓﺈن ‪ |u| = 1‬و )‪ P ∈ F (Γ‬إذا و ﻓﻘﻂ إذا ُوﺟﺪت ﻧﻘﻄﺔ ‪M‬‬ ‫ﻣﻦ ﺟﻬﺔ أﺧﺮى‪ ،‬إذا ﻛﺎﻧﺖ ‪ P ∈ Γ‬ﻻﺣﻘﺘﻬﺎ ‪ّ u‬‬
‫ﻻﺣﻘﺘﻬﺎ ‪ z‬ﺑﺤﻴﺚ )‪ . P = F (M‬ﻟﻜﻦ ‪:‬‬
‫‪1‬‬ ‫‪1‬‬ ‫‪1‬‬
‫‪u=− ⟺ z=− ⟺ z=−‬‬
‫‪z‬‬ ‫‪u‬‬ ‫‪u‬‬
‫‪1‬‬
‫إذن اﻟﻨﻘﻄﺔ ‪ P‬ذات اﻟﻼﺣﻘﺔ ‪ u‬ﻫﻲ ﺻﻮرة اﻟﻨﻘﻄﺔ ‪ M‬ذات اﻟﻼﺣﻘﺔ ‪ −‬ﺑﺎﻟﺪاﻟﺔ ‪ F‬أي )‪ P ∈ F (Γ‬و ﻫﺬا ﻳﻌﻨﻲ‬
‫‪u‬‬
‫أن )‪. Γ ⊂ F (Γ‬‬ ‫ّ‬
‫ﰲ اﻷﺧﲑ ‪ F (Γ) ⊂ Γ :‬و )‪ Γ ⊂ F (Γ‬إذن ‪ F (Γ) = Γ‬أي ‪ :‬ﺻﻮرة ‪ Γ‬ﺑﺎﻟﺪاﻟﺔ ‪ F‬ﻫﻲ ‪ Γ‬ﻧﻔﺴﻬﺎ‪.‬‬
‫)ا( ﻟﺪﻳﻨﺎ ‪:‬‬ ‫‪3‬‬
‫•‬
‫‖ ⟺ )‪M ∈ 𝒞 (A, 1‬‬
‫‖‪‖MA‬‬
‫‪‖ = 1 ⟺ ||z‬‬ ‫‪− z || = 1 ⟺ ||z − 1|| = 1‬‬

‫ﻌﺮف‪.‬‬ ‫ﻓﺈن ‪ z ≠ 0‬و ﺑﺎﻟﺘﺎﱄ ّ‬


‫ﻓﺈن ‪ُ z‬ﻣ ﱠ‬ ‫أن ‪ّ M ≠ O‬‬
‫)ب( ﺑﲈ ّ‬
‫أن 𝒞 ∈ ‪ M‬ﻓﺤﺴﺐ اﻟﺴﺆال اﻟﺴﺎﺑﻖ ّ‬
‫ﻓﺈن ‪ ||1 − z|| = 1‬ﻣﻨﻪ ‪:‬‬ ‫)‪ (i‬ﺑﲈ ّ‬
‫| |‪|z + 1‬‬ ‫|‪1‬‬
‫|‬ ‫‪| = |1 + | = ||1 − z|| = ||1 − z|| = ||1 − z|| = 1‬‬
‫|| || ‪|| z‬‬ ‫|| ‪z‬‬
‫)‪ (ii‬ﻟﺪﻳﻨﺎ ‪:‬‬
‫|‪|z + 1‬‬
‫|‬ ‫‖ ‪| = 1 ⟺ ||z + 1|| = ||z − 0|| ⟺ ‖BM ‖ = ‖OM‬‬
‫|| ‪|| z‬‬ ‫‖‬ ‫‖ ‖‬ ‫‖‬
‫‪ņőƱŀ Ţ‬‬
‫أي ‪. BM = OM :‬‬
‫ﻨﺼﻒ اﻟﻘﻄﻌﺔ اﳌﺴﺘﻘﻴﻤﺔ‬ ‫)‪ (iii‬ﳎﻤﻮﻋﺔ اﻟﻨﻘﻂ ‪ُ M‬ﲢﻘﻖ ‪ BM = OM‬أي ّ‬
‫أن ‪ M‬ﺗﻨﺘﻤﻲ إﱃ اﳌﺴﺘﻘﻴﻢ )‪ُ (D‬ﻣ ﱢ‬
‫‪1‬‬
‫]‪ [OB‬و اﻟﺬي ﻣﻌﺎدﻟﺘﻪ اﻟﺪﻳﻜﺎرﺗﻴﺔ ﻫﻲ ‪. x = − :‬‬
‫‪2‬‬
‫‪VI‬‬

‫‪ M‬ﺻﻮرة ‪ M‬ﺑﺎﻟﺪاﻟﺔ ‪ُ F‬ﲢﻘﻖ ‪ M ∈ [OM] :‬و )‪ M ∈ (D‬إذن ‪ M‬ﻫﻲ ﻧﻘﻄﺔ ﺗﻘﺎﻃﻊ اﳌﺴﺘﻘﻴﻤﲔ‬ ‫)‪ (iv‬اﻟﻨﻘﻄﺔ‬
‫)‪ (D‬و )‪ . (OM‬ﻫﻜﺬا ﻧ ِ‬
‫ُﻨﺸﺊ اﻟﻨﻘﻄﺘﲔ ‪ M‬و ‪) M‬ﺷﻜﻞ ‪.(29.VI‬‬
‫■‬
‫‪ƕ‬‬

‫‪.‬‬
‫𝟔𝟔𝟏‬ ‫‪.‬‬
‫اﳌﺴﺘﻮي اﳌﺮﻛﺐ 𝒫 ﻣﻨﺴﻮب إﱃ ﻣﻌﻠﻢ ﻣﺘﻌﺎﻣﺪ‪ ،‬ﻣﺘﺠﺎﻧﺲ و ﻣﺒﺎﴍ ⃗‪. O, u,⃗ v‬‬
‫اﳉﺰء اﻷول‬
‫‪ũŏ‬‬

‫‪. √2 − ı√2‬‬ ‫‪ •1‬أﻛﺘﺐ ﻋﲆ اﻟﺸﻜﻞ اﳉﱪي اﻟﻌﺪد‬

‫‪ •2‬ﻟﺘﻜﻦ ‪ A‬اﻟﻨﻘﻄﺔ ذات اﻟﻼﺣﻘﺔ ‪ z = √2 − ı√2‬و ‪ B‬اﻟﻨﻘﻄﺔ ذات اﻟﻼﺣﻘﺔ ‪. z = −z‬‬


‫)ا( أﻛﺘﺐ ‪ z‬و ‪ z‬ﻋﲆ اﻟﺸﻜﻞ اﻷﳼ‪.‬‬

‫𝟯𝟰𝟮‬

‫‪http ://tinyurl.com/Malki1718‬‬ ‫‪0‬‬


‫‪ .4.VI‬ﲤﺎرﻳﻦ ﻣﺘﻨﻮﻋﺔ‬
‫‪.‬‬
‫) ‪(D‬‬
‫‪C‬‬ ‫‪M1‬‬
‫‪M2′‬‬ ‫‪1‬‬

‫‪Ŕž Ŧ Ľ‬‬
‫‪0.5‬‬

‫‪œǃ‬‬
‫‪B‬‬ ‫‪O‬‬ ‫‪A‬‬
‫‪−1‬‬ ‫‪−0.5‬‬ ‫‪0.5‬‬ ‫‪1‬‬ ‫‪1.5‬‬ ‫‪2‬‬

‫‪M1′‬‬ ‫‪−0.5‬‬

‫‪Ŀ‬‬
‫‪−1‬‬ ‫‪M2‬‬

‫ﺷﻜﻞ ‪29.VI‬‬

‫)ب( أﻧﺸﺊ ‪ A‬و ‪ B‬ﻋﲆ اﻟﺸﻜﻞ ‪. 30.VI‬‬


‫𝜋‬ ‫𝜋‬
‫‪ •3‬ﻟﺘﻜﻦ ‪ C‬ﺻﻮرة اﻟﻨﻘﻄﺔ ‪ B‬ﺑﺎﻟﺪوران اﻟﺬي ﻣﺮﻛﺰه ‪ O‬و زاوﻳﺘﻪ و ‪ D‬ﺻﻮرة ‪ C‬ﺑﺎﻟﺪوران اﻟﺬي ﻣﺮﻛﺰه ‪ A‬و زاوﻳﺘﻪ ‪.‬‬
‫‪2‬‬ ‫‪2‬‬
‫)ا( أﻧﺸﺊ اﻟﻨﻘﻄﺘﲔ ‪ C‬و ‪ D‬ﻋﲆ اﻟﺸﻜﻞ ‪. 30.VI‬‬
‫ﻔﺼ ً‬
‫ﻼ اﻹﺟﺎﺑﺔ‪.‬‬ ‫)ب( أﺣﺴﺐ ‪ z‬ﻻﺣﻘﺔ اﻟﻨﻘﻄﺔ ‪ C‬و اﻛﺘﺒﻪ ﻋﲆ اﻟﺸﻜﻞ اﳉﱪي ُﻣ ﱢ‬
‫ﻋﱪ ﻋﻦ ‪ z‬ﻻﺣﻘﺔ اﻟﻨﻘﻄﺔ ‪ D‬ﺑﺪﻻﻟﺔ ‪ z‬و ‪. z‬‬ ‫)ج( ﱢ‬
‫أن ‪. z = √2 − 3ı√2‬‬ ‫ﱢﺑﺮر ّ‬

‫‪ •4‬ﻣﺎ ﻫﻲ ﻃﺒﻴﻌﺔ اﻟﺮﺑﺎﻋﻲ ‪ ABCD‬؟ ﻋ ﱢﻠﻞ‪.‬‬

‫اﳉﺰء اﻟﺜﺎﲏ‬
‫‪ņőƱŀ Ţ‬‬
‫اﳌﻌﺮﻓﺘﲔ ﰲ اﳉﺰء اﻷول و ﻟﺘﻜﻦ ‪ E‬اﻟﻨﻘﻄﺔ ذات‬
‫ﻟﺘﻜﻦ اﻟﻨﻘﻄﺘﲔ ‪ A‬و ‪ B‬اﻟﻠﺘﲔ ﻻﺣﻘﺘﺎﳘﺎ ‪ z = √2 − ı√2‬و ‪ z = −z‬و ﱠ‬
‫اﻟﻼﺣﻘﺔ ‪. z = −√2 − ı√2‬‬
‫ﻟﻴﻜﻦ ‪ m‬ﻋﺪد ًا ﺣﻘﻴﻘﻴ ًﺎ ﻏﲑ ﻣﻌﺪوم و ‪ G‬ﻣﺮﻛﺰ اﳌﺴﺎﻓﺎت اﳌﺘﻨﺎﺳﺒﺔ ﻟﻠﺠﻤﻠﺔ })‪. {(A, m) , (B, −2) , (E, 2‬‬

‫‪VI‬‬
‫‪ •1‬أوﺟﺪ ﻋﻼﻗﺔ ﺗﺮﺑﻂ ﺑﲔ ‪ AG‬و ‪. BE‬‬
‫‪ •2‬ﰲ ﻫﺬا اﻟﺴﺆال‪ ،‬ﻧﺄﺣﺬ ‪. m = −4‬‬
‫ﻟﺘﻜﻦ ‪ ℰ‬ﳎﻤﻮﻋﺔ اﻟﻨﻘﻂ ‪ M‬ﻣﻦ اﳌﺴﺘﻮي 𝒫 ﺑﺤﻴﺚ ‪. ‖‖−4MA − 2MB + 2ME‖‖ = 4√2 :‬‬
‫‪ƕ‬‬

‫)ا( ﺑﺎﺳﺘﻌﲈل اﻟﺴﺆال )‪ ، (•1‬أﻛﺘﺐ اﻟﻌﻼﻗﺔ اﻟﺘﻲ ﺗﺮﺑﻂ ﺑﲔ ‪ AG−‬و ‪ . BE‬أﻧﺸﺊ اﻟﻨﻘﻄﺔ ‪ G−‬ﻋﲆ اﻟﺸﻜﻞ ‪. 30.VI‬‬
‫)ب( أﺛﺒﺖ ّ‬
‫أن اﻟﻨﻘﻄﺔ ‪ A‬ﺗﻨﺘﻤﻲ إﱃ اﳌﺠﻤﻮﻋﺔ ‪. ℰ‬‬
‫)ج( ﻣﺎ ﻫﻲ اﳌﺠﻤﻮﻋﺔ ‪ ℰ‬؟‬
‫‪ũŏ‬‬

‫)د( أﻧﺸﺊ ‪ ℰ‬ﻋﲆ اﻟﺸﻜﻞ ‪. 30.VI‬‬


‫‪ •3‬ﰲ ﻫﺬا اﻟﺴﺆال‪ ،‬ﻧﺄﺣﺬ ‪. m = 2‬‬
‫ﻟﺘﻜﻦ ‪ ℱ‬ﳎﻤﻮﻋﺔ اﻟﻨﻘﻂ ‪ M‬ﻣﻦ اﳌﺴﺘﻮي 𝒫 ﺑﺤﻴﺚ ‪ 2MA − 2MB + 2ME ⋅ DA = −16‬ﺣﻴﺚ ‪ D‬ﻫﻲ اﻟﻨﻘﻄﺔ‬
‫اﳌﻌﺮﻓﺔ ﰲ اﻟﺴﺆال )‪•3‬ج( ‪.‬‬
‫ذات اﻟﻼﺣﻘﺔ ‪ z = √2 − 3ı√2‬و ﱠ‬

‫𝟰𝟰𝟮‬

‫‪http ://tinyurl.com/Malki1718‬‬ ‫‪0‬‬


‫‪ .VI‬ﲤﺎرﻳﻦ ﺗﻄﺒﻴﻘﻴﺔ‬
‫‪.‬‬
‫‪2‬‬

‫‪Ŕž Ŧ Ľ‬‬
‫‪1‬‬

‫‪œǃ‬‬
‫‪O‬‬
‫‪−3‬‬ ‫‪−2‬‬ ‫‪−1‬‬ ‫‪0‬‬ ‫‪1‬‬ ‫‪2‬‬ ‫‪3‬‬ ‫‪4‬‬

‫‪−1‬‬

‫‪Ŀ‬‬
‫‪−2‬‬

‫‪−3‬‬

‫‪−4‬‬

‫ﺷﻜﻞ ‪30.VI‬‬

‫)ا( ﺑﺎﺳﺘﻌﲈل اﻟﺴﺆال )‪ ، (•1‬أﻛﺘﺐ اﻟﻌﻼﻗﺔ اﻟﺘﻲ ﺗﺮﺑﻂ ﺑﲔ ‪ AG‬و ‪ . BE‬ﺣﺪﱢ د اﻟﻨﻘﻄﺔ ‪. G‬‬
‫)ب( ﻧُﱪﻫﻦ ّ‬
‫أن اﻟﻨﻘﻄﺔ ‪ M‬ﺗﻨﺘﻤﻲ إﱃ ‪ ℱ‬إذا و ﻓﻘﻂ إذا ﻛﺎن ‪ MG ⋅ DA = a‬ﺣﻴﺚ ‪ a‬ﻋﺪد ﺣﻘﻴﻘﻲ‪ .‬أوﺟﺪ ﻗﻴﻤﺔ ‪.a‬‬
‫)ج( أﺛﺒﺖ ّ‬
‫أن ‪ A‬ﺗﻨﺘﻤﻲ إﱃ ‪. ℱ‬‬
‫‪ ℱ‬إذا و ﻓﻘﻂ إذا ﻛﺎن ‪ AD ⋅ DA − MD ⋅ DA = b‬و ‪AM ⋅ DA = c‬‬ ‫)د( ﻧﺴﺘﻨﺘﺞ ّ‬
‫أن اﻟﻨﻘﻄﺔ ‪ M‬ﺗﻨﺘﻤﻲ إﱃ‬
‫ﺣﻴﺚ ‪ b‬و ‪ c‬ﻋﺪدان ﺣﻘﻴﻘﻴﺎن‪ .‬ﺣﺪﱢ د ﻗﻴﻤﺔ ‪ b‬و ‪. c‬‬
‫ﰲ ﻫﺬه اﳊﺎﻟﺔ‪ ،‬ﻣﺎذا ﻳﻤﻜﻦ اﺳﺘﻨﺘﺎﺟﻪ ﺑﺎﻟﻨﺴﺒﺔ ﻟﻠﺸﻌﺎﻋﲔ ‪ AM‬و ‪ DA‬؟‬
‫)ﻫ( ﻣﺎ ﻫﻲ اﳌﺠﻤﻮﻋﺔ ‪ ℱ‬؟ أﻧﺸﺌﻬﺎ ﻋﲆ اﻟﺸﻜﻞ‪. 30.VI‬‬
‫‪ņőƱŀ Ţ‬‬
‫ﺍﳊﻞّ‪.‬‬
‫اﳉﺰء اﻷول‬
‫‪VI‬‬

‫‪ •1‬ﻟﺪﻳﻨﺎ ‪:‬‬
‫‪√2 − ı√2‬‬ ‫‪= √2‬‬ ‫‪− 2 × √2 × ı√2 + ı√2‬‬ ‫‪= 2 − 4ı − 2 = −4ı‬‬
‫‪ƕ‬‬

‫‪√2‬‬ ‫)ا( ﻟﺪﻳﻨﺎ ‪ z = √2 − ı√2‬ﻣﻨﻪ ‪ ||z || = √2 + 2 = 2‬و إذا ﻛﺎﻧﺖ )𝜋‪ّ 𝜃 = arg (z) (mod 2‬‬
‫= 𝜃 ‪cos‬‬ ‫ﻓﺈن‬ ‫•‬‫‪2‬‬
‫‪2‬‬
‫𝜋‬ ‫‪2‬‬
‫‪. z = 2e−‬‬ ‫‪/‬‬ ‫و √ ‪ sin 𝜃 = −‬ﻣﻨﻪ )𝜋‪ arg (z ) = 𝜃 = − (mod 2‬أي‬
‫‪4‬‬ ‫‪2‬‬
‫‪. z = −z = −2e−‬‬ ‫‪/‬‬ ‫‪= 2e e−‬‬ ‫‪/‬‬ ‫‪= 2e‬‬ ‫‪−‬‬ ‫‪= 2e‬‬ ‫‪/‬‬ ‫ﻣﻨﻪ‬
‫‪ũŏ‬‬

‫)ب( أﻧﻈﺮ اﻟﺸﻜﻞ ‪. 31.VI‬‬


‫‪) •3‬ا( أﻧﻈﺮ اﻟﺸﻜﻞ ‪. 31.VI‬‬
‫)ب( ﻟﺪﻳﻨﺎ ‪:‬‬
‫‪z =e‬‬ ‫‪/‬‬ ‫‪z = ız = −ız‬‬ ‫‪= ı −√2 + ı√2 = −√2 − ı√2‬‬

‫𝟱𝟰𝟮‬

‫‪http ://tinyurl.com/Malki1718‬‬ ‫‪0‬‬


‫‪ .4.VI‬ﲤﺎرﻳﻦ ﻣﺘﻨﻮﻋﺔ‬
‫‪.‬‬
‫‪ z = e‬ﻣﻨﻪ ‪:‬‬ ‫‪/‬‬ ‫‪/‬‬
‫‪(z − z ) + z‬‬ ‫‪= ı (z − z ) + z‬‬ ‫‪ z − z = e‬أي‬ ‫ﻟﺪﻳﻨﺎ ‪(z − z ) :‬‬ ‫)ج(‬

‫‪Ŕž Ŧ Ľ‬‬
‫‪z‬‬ ‫‪= ı (z − z ) + z‬‬ ‫‪= ı −√2 − ı√2 − √2 + ı√2 + √2 − ı√2 = √2 − 3ı√2‬‬

‫‪ •4‬ﺑﺎﻹﺳﺘﻌﺎﻧﺔ ﺑﺎﻟﺸﻜﻞ ‪ ، 31.VI‬ﻧﻼﺣﻆ ّ‬

‫‪œǃ‬‬
‫أن اﻟﺮﺑﺎﻋﻲ ‪ ABCD‬ﻣﺘﻮازي أﺿﻼع‪ .‬ﻟﻨﺜﺒﺖ ذﻟﻚ ‪:‬‬
‫‪z −z‬‬ ‫‪= −2z‬‬ ‫ﻻﺣﻘﺔ اﻟﺸﻌﺎع ‪ AB‬ﻫﻲ ‪= −2√2 + 2ı√2‬‬
‫‪z −z‬‬ ‫و ﻻﺣﻘﺔ اﻟﺸﻌﺎع ‪ DC‬ﻫﻲ ‪= −√2 − ı√2 − √2 + 3ı√2 = −2√2 + 2ı√2‬‬

‫‪Ŀ‬‬
‫ﻣﻨﻪ ‪ AB = DC‬و ﻫﺬا ﻳﻌﻨﻲ ّ‬
‫أن ‪ ABCD‬ﻣﺘﻮازي أﺿﻼع‪.‬‬
‫اﳉﺰء اﻟﺜﺎﲏ‬

‫‪ُ G‬ﲢﻘﻖ ‪mG A − 2G B + 2G E = 0⃗ :‬‬ ‫‪ •1‬ﺣﺴﺐ ﺗﻌﺮﻳﻒ ﻣﺮﻛﺰ اﳌﺴﺎﻓﺎت اﳌﺘﻨﺎﺳﺒﺔ‪ ،‬اﻟﻨﻘﻄﺔ‬
‫‪mG A + 2 BG‬‬ ‫‪ mG A + 2BG‬ﻣﻨﻪ ⃗‪+ G E = 0‬‬ ‫أي ⃗‪+ 2G E = 0‬‬
‫‪2‬‬ ‫‪2‬‬
‫= ‪. AG‬‬ ‫ﻣﻨﻪ ⃗‪ mG A + 2BE = 0‬ﻣﻨﻪ ‪ G A = − BE‬أي ‪BE‬‬
‫‪m‬‬ ‫‪m‬‬
‫‪2‬‬ ‫‪1‬‬
‫= ‪ . AG−‬أﻧﻈﺮ اﻟﺸﻜﻞ ‪. 31.VI‬‬ ‫ﺑﺄﺧﺬ ‪ m = −4‬ﰲ اﳌﺴﺎواة اﻟﺴﺎﺑﻘﺔ ﻳﻨﺘﺞ ‪BE = − BE‬‬ ‫)ا(‬ ‫•‬ ‫‪2‬‬
‫‪−4‬‬ ‫‪2‬‬
‫)ب( ﻟﺪﻳﻨﺎ ‪:‬‬
‫‖ ⟺ ‪A∈ℰ‬‬
‫‖‪‖−4AA − 2AB + 2AE‬‬
‫‪‖ = 4√2‬‬

‫‖=‖‬ ‫⃗‬
‫‖‬
‫‖‪‖−4AA − 2AB + 2AE‬‬ ‫‖‪‖−40 + 2BA + 2AE‬‬ ‫‖=‖‬ ‫‖‪‖2BE‬‬
‫‖‬ ‫ﻟﻜﻦ ‪:‬‬
‫‪= 2 ||z − z || = 2 ||−√2 − ı√2 = √2 − ı√2|| = 2 ||−2ı√2|| = 4√2‬‬

‫إذن اﻟﻨﻘﻄﺔ ‪ A‬ﺗﻨﺘﻤﻲ إﱃ اﳌﺠﻤﻮﻋﺔ ‪. ℰ‬‬


‫)ج( ﻟﺪﻳﻨﺎ ‪:‬‬
‫‪ņőƱŀ Ţ‬‬
‫‖ ⟺ ‪M∈ℰ‬‬
‫‖‪‖−4MA − 2MB + 2ME‬‬‫‪‖ = 4√2‬‬
‫‖ ⟺‬ ‫‖‬
‫‪‖−4 MG− + G− A − 2 MG− + G− B + 2 MG− + G− E ‖ = 4√2‬‬
‫‖ ⟺‬
‫‖‪‖−4MG− − 4G− A − 2G− B + 2G− E‬‬
‫‪‖ = 4√2‬‬

‫‪VI‬‬
‫‖‪‖−4MG− − 4G− A + 2BG− + 2G− E‬‬
‫‖ ⟺‬ ‫‪‖ = 4√2‬‬
‫‖‪‖−4MG− − 4G− A + 2BE‬‬
‫‖ ⟺‬ ‫‪‖ = 4√2‬‬
‫‖ ⟺‬ ‫‖‬
‫‪‖−4MG− − 4G− A + 2 −2AG− ‖ = 4√2‬‬
‫‪ƕ‬‬

‫‖ ⟺‬
‫‖ ‪‖−4MG− − 4G− A − 4AG−‬‬
‫‪‖ = 4√2‬‬
‫‖ ⟺‬
‫‖ ‪‖−4MG−‬‬‫‖ ‪‖ = 4√2 ⟺ 4‬‬
‫‖ ‪‖MG−‬‬
‫‪‖ = 4√2‬‬
‫‖ ⟺‬
‫‖ ‪‖MG−‬‬
‫‪‖ = √2 ⟺ MG− = √2‬‬
‫‪ũŏ‬‬

‫إذن ‪ ℰ‬ﻫﻲ اﻟﺪاﺋﺮة اﻟﺘﻲ ﻣﺮﻛﺰﻫﺎ ‪ G−‬و ﻧﺼﻒ ﻗﻄﺮﻫﺎ ‪. √2‬‬


‫)د( أﻧﻈﺮ اﻟﺸﻜﻞ ‪. 31.VI‬‬
‫‪2‬‬
‫)ا( ﺑﺄﺧﺬ ‪ m = 2‬ﰲ ﻧﺘﻴﺠﺔ اﻟﺴﺆال )‪ (•1‬ﻧﺠﺪ ‪ . AG = BE = BE :‬ﻟﻜﻦ ‪ z = z = −√2 − ı√2‬ﻣﻨﻪ‬ ‫•‬ ‫‪3‬‬
‫‪2‬‬
‫ﻓﺈن ‪ AG = BE = BC = AD‬أي ‪. G = D‬‬‫‪ E = C‬و ﺑﺎﻟﺘﺎﱄ ّ‬
‫أن ‪ّ E = C‬‬
‫ﻓﺈن اﻟﺮﺑﺎﻋﻲ ‪ ABED = ABCD‬ﻣﺘﻮازي أﺿﻼع‪.‬‬ ‫و ﺑﲈ ّ‬

‫𝟲𝟰𝟮‬

‫‪http ://tinyurl.com/Malki1718‬‬ ‫‪0‬‬


‫‪ .VI‬ﲤﺎرﻳﻦ ﺗﻄﺒﻴﻘﻴﺔ‬
‫‪.‬‬
‫‪2‬‬
‫‪B‬‬ ‫‪E‬‬

‫‪Ŕž Ŧ Ľ‬‬
‫‪1‬‬

‫‪G −4‬‬

‫‪œǃ‬‬
‫‪−3‬‬ ‫‪−2‬‬ ‫‪−1‬‬ ‫‪O‬‬ ‫‪1‬‬ ‫‪2‬‬ ‫‪3‬‬ ‫‪4‬‬
‫‪0‬‬

‫‪−1‬‬
‫‪A‬‬
‫‪F C=E‬‬

‫‪Ŀ‬‬
‫‪−2‬‬

‫‪−3‬‬

‫‪−4‬‬
‫‪D = G2‬‬

‫ﺷﻜﻞ ‪31.VI‬‬

‫)ب( ﻟﺪﻳﻨﺎ ‪:‬‬

‫‪2MA − 2MB + 2ME = 2 MG + G A − 2 MG + G B + 2 MG + G E‬‬


‫‪= 2MG + 2G A − 2G B + 2G E‬‬
‫‪= 2MG + 2G A + 2BG + 2G E‬‬
‫‪= 2MG + 2G A + 2BE‬‬
‫‪= 2MG + 2 −BE + 2BE‬‬
‫‪= 2MG‬‬

‫ﻣﻨﻪ ‪:‬‬
‫‪ņőƱŀ Ţ‬‬
‫‪2MA − 2MB + 2ME ⋅ DA = −16 ⟺ 2MG ⋅ DA = −16 ⟺ MG ⋅ DA = −8‬‬

‫أي ‪. a = −8‬‬
‫)ج( ﻟﺪﻳﻨﺎ ‪ . A ∈ ℱ ⟺ AG ⋅ DA = −8 :‬ﻟﻜﻦ ‪ G = D‬ﻣﻨﻪ ‪:‬‬
‫‪VI‬‬

‫‪AG ⋅ DA = AD ⋅ DA = −DA = − ||z − z || = − −2√2‬‬ ‫‪= −8‬‬

‫إذن اﻟﻨﻘﻄﺔ ‪ A‬ﺗﻨﺘﻤﻲ إﱃ اﳌﺠﻤﻮﻋﺔ ‪. ℱ‬‬


‫أن ‪ AD ⋅ DA = −8‬و ﺣﺴﺐ اﻟﺴﺆال )‪•3‬ب( ّ‬
‫ﻓﺈن ‪:‬‬ ‫)د( ﻣﻦ اﻟﺴﺆال اﻟﺴﺎﺑﻖ ﻧﺴﺘﻨﺘﺞ ّ‬
‫‪ƕ‬‬

‫)ﻧُﺬﻛﱢﺮ ّ‬
‫أن ‪(G = D‬‬ ‫‪M ∈ ℱ ⟺ MD ⋅ DA = −8‬‬
‫‪. b=0‬‬ ‫أي‬ ‫‪M ∈ ℱ ⟺ AD ⋅ DA − MD ⋅ DA = −8 + 8 = 0‬‬ ‫ﻣﻨﻪ ‪:‬‬
‫ﻣﻦ ﺟﻬﺔ أﺧﺮى‪ ،‬ﻟﺪﻳﻨﺎ ‪:‬‬
‫‪ũŏ‬‬

‫‪AM ⋅ DA = AD − MD ⋅ DA = AD ⋅ DA − MD ⋅ DA = 0‬‬

‫إذن ‪ c = 0‬و ﻧﺴﺘﻨﺘﺞ ّ‬


‫أن اﻟﺸﻌﺎﻋﲔ ‪ AM‬و ‪ DA‬ﻣﺘﻌﺎﻣﺪان‪.‬‬
‫)ﻫ( ﺣﺴﺐ ﻣﺎ ﺳﺒﻖ‪ ،‬اﳌﺠﻤﻮﻋﺔ ‪ ℱ‬ﻫﻲ اﳌﺴﺘﻘﻴﻢ اﻟﺬي ﻳﻤﺮ ﺑﺎﻟﻨﻘﻄﺔ ‪ A‬و ُﻳﻌﺎﻣﺪ اﳌﺴﺘﻘﻴﻢ )‪ (AD‬إذن ﻓﻬﻮ اﳌﺴﺘﻘﻴﻢ‬
‫)‪ (AE‬أي )‪) ℱ = (AE‬ﺷﻜﻞ ‪. (31.VI‬‬
‫■‬

‫𝟳𝟰𝟮‬

‫‪http ://tinyurl.com/Malki1718‬‬ ‫‪0‬‬


‫‪ .4.VI‬ﲤﺎرﻳﻦ ﻣﺘﻨﻮﻋﺔ‬
‫‪.‬‬

‫‪.‬‬
‫𝟕𝟔𝟏‬ ‫‪.‬‬

‫‪Ŕž Ŧ Ľ‬‬
‫اﳌﺴﺘﻮي اﳌﺮﻛﺐ ﻣﻨﺴﻮب إﱃ ﻣﻌﻠﻢ ﻣﺘﻌﺎﻣﺪ و ﻣﺘﺠﺎﻧﺲ ⃗‪. O, u,⃗ v‬‬
‫ﻧﻘﻮل ﻋﻦ داﻟﺔ ‪ f : ℂ ⟶ ℂ‬أﳖﺎ ﺗﻀﺎﻣﻨﻴﺔ إذا ﻛﺎن ﻟﻜﻞ ‪ّ z ∈ ℂ‬‬
‫ﻓﺈن ‪ (f ∘ f) (z) = z‬ﺣﻴﺚ ‪ f ∘ f‬ﻫﻲ ﺗﺮﻛﻴﺐ اﻟﺪاﻟﺔ ‪ f‬ﻣﻊ‬

‫‪œǃ‬‬
‫ﻧﻔﺴﻬﺎ‪.‬‬
‫اﳉﺰء اﻷول‬

‫‪Ŀ‬‬
‫ﻟﻴﻜﻦ ‪ A‬و ‪ B‬ﻋﺪدﻳﻦ ﺣﻘﻴﻘﻴﲔ‪.‬‬
‫ﻧﻌﺘﱪ اﻟﺪاﻟﺔ ‪ F : ℂ ⟶ ℂ‬اﳌﻌﺮﻓﺔ ﺑـِ ‪. F (z) = Az + Bz :‬‬

‫‪ •1‬أﺣﺴﺐ )‪ F (1‬و )‪. F (ı‬‬


‫ﻓﺈن ‪ . F (z) = 0‬أﺛﺒﺖ ّ‬
‫أن ‪. A = B = 0 :‬‬ ‫‪ •2‬ﻧﻔﺮض أﻧﻪ ﻟﻜﻞ ﻋﺪد ﻣﺮﻛﺐ ‪ّ z‬‬
‫‪. F (z) = z‬‬ ‫‪ •3‬ﱢ‬
‫ﻋﲔ ‪ A‬و ‪ B‬ﺣﺘﻰ ﻳﻜﻮن‪ ،‬ﻟﻜﻞ ﻋﺪد ﻣﺮﻛﺐ ‪: z‬‬
‫اﳉﺰء اﻟﺜﺎﲏ‬

‫ﻟﻴﻜﻦ ‪ a‬و ‪ b‬ﻋﺪدﻳﻦ ﺣﻘﻴﻘﻴﲔ‪.‬‬


‫‪.f‬‬ ‫‪ f‬اﳌﻌﺮﻓﺔ ﺑـِ ‪(z) = az + bz :‬‬ ‫ﻧﻌﺘﱪ اﻟﺪاﻟﺔ ‪: ℂ ⟶ ℂ‬‬

‫‪. f‬‬ ‫‪∘f‬‬ ‫أوﺟﺪ‪ ،‬ﻟﻜﻞ ﻋﺪد ﻣﺮﻛﺐ ‪ ، z‬ﻋﺒﺎرة )‪(z‬‬ ‫•‬ ‫‪1‬‬
‫‪ f‬ﺗﻀﺎﻣﻨﻴ ًﺔ إذا و ﻓﻘﻂ إذا ﺣﻘﻖ اﻟﻌﺪدان ‪ a‬و ‪ b‬ﲨﻠﺔ ﻣﻌﺎدﻟﺘﲔ‪ .‬أوﺟﺪ ﻫﺬه اﳉﻤﻠﺔ‪.‬‬ ‫‪ •2‬ﺗﻜﻮن اﻟﺪاﻟﺔ‬
‫‪ f‬اﻟﺘﻀﺎﻣﻨﻴﺔ‪.‬‬ ‫‪ •3‬أوﺟﺪ ﲨﻴﻊ اﻟﺪوال‬

‫اﳉﺰء اﻟﺜﺎﻟﺚ‬

‫‪√2‬‬
‫‪ņőƱŀ Ţ‬‬
‫‪. g (z) = e‬‬ ‫‪/‬‬ ‫=‪z‬‬ ‫ﻧﻌﺘﱪ اﻟﺪاﻟﺔ ‪ g : ℂ ⟶ ℂ‬اﳌﻌﺮﻓﺔ ﺑـِ ‪(1 + ı) z :‬‬
‫‪2‬‬

‫‪ •1‬ﻫﻞ اﻟﺪاﻟﺔ ‪ g‬ﺗﻀﺎﻣﻨﻴﺔ ؟ ﻋ ﱢﻠﻞ إﺟﺎﺑﺘﻚ‪.‬‬


‫ﻟﺘﻜﻦ ‪ M‬ﻧﻘﻄﺔ ﻣﻦ اﳌﺴﺘﻮي ﻻﺣﻘﺘﻬﺎ ‪. z = x + ıy‬‬
‫ﻧﻌﺘﱪ اﻟﻨﻘﻄﺔ ‪ M‬ذات اﻟﻼﺣﻘﺔ ‪ z = x + ıy‬ﺑﺤﻴﺚ ‪. z = g (z) :‬‬
‫‪VI‬‬
‫ﻋﱪ ﻋﻦ ‪ x‬و ‪ y‬ﺑﺪﻻﻟﺔ ‪ x‬و ‪. y‬‬
‫)ا( ﱢ‬
‫)ب( )‪ (i‬أوﺟﺪ ﲨﻠﺔ اﳌﻌﺎدﻟﺘﲔ اﻟﺘﻲ ﳚﺐ أن ﳛﻘﻘﻬﲈ ‪ x‬و ‪ y‬ﺣﺘﻰ ﻳﻜﻮن ‪. M = M‬‬
‫‪ƕ‬‬

‫)‪ (ii‬ﻟﺘﻜﻦ ‪ Δ‬ﳎﻤﻮﻋﺔ اﻟﻨﻘﻂ ‪ M‬ﺑﺤﻴﺚ ‪. M = M‬‬


‫أﺛﺒﺖ ّ‬
‫أن ‪ Δ‬ﻫﻮ اﳌﺴﺘﻘﻴﻢ اﻟﺬي ﻣﻌﺎدﻟﺘﻪ ‪ . y = √2 − 1 x‬أوﺟﺪ ﺷﻌﺎع ﺗﻮﺟﻴﻪ ⃗‪ u‬ﳍﺬا اﳌﺴﺘﻘﻴﻢ‪.‬‬
‫)ج( أﺛﺒﺖ أﻧﻪ ﻟﻜﻞ ﻧﻘﻄﺔ ‪ّ M‬‬
‫ﻓﺈن اﻟﺸﻌﺎع ‪ MM‬ﻳﻌﺎﻣﺪ اﻟﺸﻌﺎع ⃗‪. u‬‬
‫‪ũŏ‬‬

‫)د( )‪ (i‬أوﺟﺪ اﻹﺣﺪاﺛﻴﲔ ‪ x‬و ‪ y‬ﻟﻠﻨﻘﻄﺔ ‪ I‬ﻣﻨﺘﺼﻒ اﻟﻘﻄﻌﺔ ‪. MM‬‬


‫)‪ (ii‬أﺛﺒﺖ ّ‬
‫أن اﻟﻨﻘﻄﺔ ‪ I‬ﺗﻨﺘﻤﻲ إﱃ اﳌﺴﺘﻘﻴﻢ ‪. Δ‬‬
‫)ﻫ( ﻣﺎ ﻫﻮ اﻟﺘﺤﻮﻳﻞ اﻟﻨﻘﻄﻲ اﻟﺬي ﺗﻜﻮن ﺑﻪ ‪ M‬ﺻﻮر َة ‪ M‬؟‬

‫𝟴𝟰𝟮‬

‫‪http ://tinyurl.com/Malki1718‬‬ ‫‪0‬‬


‫‪ .VI‬ﲤﺎرﻳﻦ ﺗﻄﺒﻴﻘﻴﺔ‬
‫‪.‬‬
‫ﺍﳊﻞّ‪.‬‬

‫‪Ŕž Ŧ Ľ‬‬
‫اﳉﺰء اﻷول‬
‫•‬‫‪1‬‬

‫‪œǃ‬‬
‫‪F (1) = A × 1 + B × 1 = A + B‬‬ ‫ﻟﺪﻳﻨﺎ ‪:‬‬
‫‪F (ı) = A × ı + B × ı = Aı − Bı = (A − B) ı‬‬ ‫و‬

‫‪Ŀ‬‬
‫‪ •2‬ﻟﻴﻜﻦ ‪ z = x + ıy‬ﻋﺪد ًا ﻣﺮﻛﺒﺎ‪ .‬ﻟﺪﻳﻨﺎ ‪:‬‬
‫‪∀z ∈ ℂ , F (z) = 0 ⟺ ∀x, y ∈ ℝ , A (x + ıy) + B(x + ıy) = 0‬‬
‫‪⟺ ∀x, y ∈ ℝ , A (x + ıy) + B (x − ıy) = 0‬‬
‫‪⟺ ∀x, y ∈ ℝ , (A + B) x + ıy (A − B) = 0‬‬
‫‪⟺ ∀x, y ∈ ℝ , (A + B) x = 0‬‬ ‫و‬ ‫‪y (A − B) = 0‬‬
‫‪ A−B=0‬و ‪⟺ A+B=0‬‬
‫‪⟺ A=B=0‬‬

‫‪ •3‬ﻟﻴﻜﻦ ‪ z = x + ıy‬ﻋﺪد ًا ﻣﺮﻛﺒﺎ‪ .‬ﻟﺪﻳﻨﺎ ‪:‬‬


‫‪∀z ∈ ℂ , F (z) = z ⟺ ∀x, y ∈ ℝ , A (x + ıy) + B(x + ıy) = x + ıy‬‬
‫‪⟺ ∀x, y ∈ ℝ , (A + B) x + ıy (A − B) = x + ıy‬‬
‫‪A+B=1‬‬ ‫‪A=1‬‬
‫⟺‬ ‫⟺‬
‫‪A−B=1‬‬ ‫‪B=0‬‬

‫ﻗﺪ ﻳﺘﺴﺎءل اﻟﻘﺎرئ اﻟﻜﺮﻳﻢ ﻋﻦ ﺳﺒﺐ ﻫﺬا اﻟﺘﻔﺼﻴﻞ )اﳌﻤﻞ؟( ﰲ اﳉﻮاب إذ ﺑﺎﻹﻣﻜﺎن إﺗﱢﺒﺎع‬ ‫⧏‬ ‫ﻣﻼﺣﻈﺔ ‪: 36‬‬
‫اﻟﻄﺮﻳﻘﺔ اﻟﺘﺎﻟﻴﺔ ‪:‬‬
‫‪∀z ∈ ℂ , F (z) = z ⟺ ∀z ∈ ℂ , Az + Bz = 1 × z + 0 × z ⟺ A = 1‬‬ ‫‪ B=0‬و‬
‫‪ņőƱŀ Ţ‬‬
‫ﻟﻜﻦ ﻣﻜﺘﺴﺒﺎﺗﻨﺎ ﻻ ﺗﺴﻤﺢ ﻟﻨﺎ ﺑﺎﻋﺘﺒﺎر ﻫﺬه اﻟﻄﺮﻳﻘﺔ ﻷﳖﺎ ﺗﻌﺘﻤﺪ ﻋﲆ ﺧﺎﺻﻴﺔ أﺳﺎﺳﻴﺔ ﰲ ﻣﻴﺪان ﺍﳉﱪ ﺍﳋﻄﻲ )ﺧﺎرج ﻋﻦ‬
‫اﻟﱪﻧﺎﻣﺞ( و ﻫﻲ ﺧﺎﺻﻴﺔ اﻹﺳﺘﻘﻼل اﳋﻄﻲ ‪.‬‬
‫⧐‬
‫‪VI‬‬

‫اﳉﺰء اﻟﺜﺎﲏ‬

‫أن ‪ّ a, b ∈ ℝ‬‬
‫ﻓﺈن ‪ a = a‬و ‪ b = b‬ﻣﻨﻪ ‪:‬‬ ‫‪ •1‬ﺑﲈ ّ‬
‫‪ƕ‬‬

‫‪f‬‬ ‫‪∘f‬‬ ‫‪(z) = a (az + bz) + b(az + bz) = a (az + bz) + b a ⋅ z + b ⋅ z‬‬
‫‪= a z + abz + b (az + bz) = a + b‬‬ ‫‪z + 2abz‬‬
‫‪ũŏ‬‬

‫‪ f‬ﺗﻀﺎﻣﻨﻴﺔ إذا و ﻓﻘﻂ إذا ‪:‬‬ ‫‪ •2‬اﻟﺪاﻟﺔ‬


‫‪a +b =1‬‬
‫‪∀z ∈ ℂ , f‬‬ ‫‪∘f‬‬ ‫‪(z) = z ⟺ ∀z ∈ ℂ ,‬‬ ‫‪a +b‬‬ ‫⟺ ‪z + 2abz = z‬‬
‫‪2ab = 0‬‬

‫‪ A = a + b ، F = f‬و ‪. B = 2ab‬‬ ‫‪∘f‬‬ ‫ﺣﻴﺚ اﺳﺘﻌﻤﻠﻨﺎ ﻧﺘﻴﺠﺔ اﻟﺴﺆال )‪ (•3‬ﻣﻦ اﳉﺰء اﻷول ﺑﺄﺧﺬ‬

‫𝟵𝟰𝟮‬

‫‪http ://tinyurl.com/Malki1718‬‬ ‫‪0‬‬


‫‪ .4.VI‬ﲤﺎرﻳﻦ ﻣﺘﻨﻮﻋﺔ‬
‫‪.‬‬
‫‪ f‬ﺗﻀﺎﻣﻨﻴﺔ إذا و ﻓﻘﻂ إذا ‪:‬‬ ‫‪ •3‬اﻟﺪاﻟﺔ‬

‫‪Ŕž Ŧ Ľ‬‬
‫‪a +b =1‬‬ ‫‪a +b =1‬‬
‫⟺‬
‫‪2ab = 0‬‬ ‫‪a=0‬‬ ‫‪ b = 0‬أو‬

‫‪œǃ‬‬
‫‪a +b =1‬‬ ‫‪a +b =1‬‬
‫⟺‬ ‫أو‬
‫‪a=0‬‬ ‫‪b=0‬‬
‫‪b =1‬‬ ‫‪a =1‬‬
‫⟺‬ ‫أو‬

‫‪Ŀ‬‬
‫‪a=0‬‬ ‫‪b=0‬‬
‫‪b = ±1‬‬ ‫‪a = ±1‬‬
‫⟺‬ ‫أو‬
‫‪a=0‬‬ ‫‪b=0‬‬

‫‪ f‬ﺑﺤﻴﺚ ‪. (a, b) ∈ {(0, −1) , (0, 1) , (−1, 0) , (1, 0)} :‬‬ ‫‪ f‬اﻟﺘﻀﺎﻣﻨﻴﺔ ﻫﻲ اﻟﺪوال‬ ‫إذن اﻟﺪوال‬
‫‪.f‬‬ ‫‪−‬‬ ‫‪: z ↦ −z ,‬‬ ‫‪f‬‬ ‫‪:z ↦ z ,‬‬ ‫‪f−‬‬ ‫‪: z ↦ −z‬‬ ‫‪f‬‬ ‫أي ‪: z ↦ z :‬‬

‫اﳉﺰء اﻟﺜﺎﻟﺚ‬

‫‪ •1‬ﻟﺪﻳﻨﺎ ‪:‬‬
‫‪∀z ∈ ℂ , (g ∘ g) (z) = e‬‬ ‫‪/‬‬ ‫‪e‬‬ ‫‪/‬‬ ‫‪z =e‬‬ ‫‪/‬‬ ‫‪e−‬‬ ‫‪/‬‬ ‫‪z=e‬‬ ‫‪/ − /‬‬ ‫‪z=e z=z‬‬
‫إذن اﻟﺪاﻟﺔ ‪ g‬ﺗﻀﺎﻣﻨﻴﺔ‪.‬‬
‫‪ •2‬ﻟﺪﻳﻨﺎ ‪:‬‬
‫‪z = g (z) ⟺ x + ıy = e‬‬ ‫‪/‬‬
‫)‪(x − ıy‬‬
‫‪√2‬‬ ‫‪√2‬‬
‫= ‪⟺ x + ıy‬‬ ‫‪+ı‬‬ ‫)‪(x − ıy‬‬
‫‪2‬‬ ‫‪2‬‬
‫‪√2‬‬ ‫‪√2‬‬
‫= ‪⟺ x + ıy‬‬ ‫‪(x + y) + ı‬‬ ‫)‪(x − y‬‬
‫‪ņőƱŀ Ţ‬‬
‫‪2‬‬ ‫‪2‬‬
‫⎧‬
‫⎪‬ ‫‪√2‬‬
‫= ‪⎪x‬‬ ‫)‪(x + y‬‬
‫⟺‬ ‫‪2‬‬
‫⎨‬
‫)‪⎪ y = √2 (x − y‬‬
‫⎪‬
‫⎩‬

‫‪VI‬‬
‫‪2‬‬

‫⎧‬
‫⎪‬ ‫‪√2‬‬
‫⎪‬ ‫‪(x + y) = x‬‬ ‫‪⎧ √2 − 2 x + √2y = 0‬‬
‫⟺ ‪M =M ⟺ z =z‬‬ ‫‪2‬‬ ‫⟺‬ ‫ﻟﺪﻳﻨﺎ ‪:‬‬ ‫)ا(‬ ‫•‬ ‫‪3‬‬
‫⎨‬ ‫‪⎨ √2x − √2 − 2 y = 0‬‬
‫‪⎪ √2 (x − y) = y‬‬
‫⎪‬
‫⎩‬
‫‪⎩ 2‬‬
‫‪ƕ‬‬

‫ﻟﺪﻳﻨﺎ ‪ M = M‬إذا و ﻓﻘﻂ إذا ‪:‬‬ ‫)ب(‬


‫‪⎧ √2 − 2 x + √2y = 0‬‬ ‫⎧‬
‫‪⎪ y = √2 − 1 x‬‬
‫⟺‬ ‫‪1‬‬ ‫‪⟺ y = √2 − 1 x‬‬
‫‪⎨ √2x − √2 − 2 y = 0‬‬ ‫⎨‬
‫‪ũŏ‬‬

‫= ‪⎪y‬‬ ‫‪x‬‬ ‫=‬ ‫√‬ ‫‪2‬‬ ‫‪−‬‬ ‫‪1‬‬ ‫‪x‬‬


‫⎩‬ ‫⎩‬ ‫‪1 + √2‬‬
‫إذن ‪ Δ‬ﻫﻮ اﳌﺴﺘﻘﻴﻢ اﻟﺬي ﻣﻌﺎدﻟﺘﻪ ‪. y = √2 − 1 x :‬‬
‫‪1‬‬ ‫)‪− (−1‬‬
‫‪.‬‬ ‫أي‬ ‫ﻣﻌﺎدﻟﺔ ‪ Δ‬ﺗُﻜﺘﺐ أﻳﻀ ًﺎ ‪ √2 − 1 x − y = 0‬إذن أﺣﺪ أﺷﻌﺔ ﺗﻮﺟﻴﻬﻪ ﻫﻮ‬
‫‪√2 − 1‬‬ ‫‪√2 − 1‬‬
‫إذن اﻟﺸﻌﺎع ‪ u⃗ = 1, √2 − 1‬ﻫﻮأﺣﺪ أﺷﻌﺔ ﺗﻮﺟﻴﻪ اﳌﺴﺘﻘﻴﻢ ‪. Δ‬‬

‫𝟬𝟱𝟮‬

‫‪http ://tinyurl.com/Malki1718‬‬ ‫‪0‬‬


‫‪ .VI‬ﲤﺎرﻳﻦ ﺗﻄﺒﻴﻘﻴﺔ‬
‫‪.‬‬
‫‪ •4‬ﻟﺪﻳﻨﺎ ‪:‬‬

‫‪Ŕž Ŧ Ľ‬‬
‫‪MM ⋅ u⃗ = x − x ⋅ 1 + y − y ⋅ √2 − 1‬‬

‫‪√2‬‬ ‫‪√2‬‬

‫‪œǃ‬‬
‫=‬ ‫‪(x + y) − x‬‬ ‫‪+ √2 − 1‬‬ ‫‪(x − y) − y‬‬
‫‪2‬‬ ‫‪2‬‬
‫‪√2‬‬
‫=‬ ‫‪1 − √2 x + y + √2 − 1 x − y = 0‬‬
‫‪2‬‬

‫‪Ŀ‬‬
‫إذن‪ ،‬ﻟﻜﻞ ﻧﻘﻄﺔ ‪ M‬ﻣﻦ اﳌﺴﺘﻮي‪ ،‬اﻟﺸﻌﺎع ‪ُ MM‬ﻳﻌﺎﻣﺪ ⃗‪. u‬‬
‫‪y‬‬ ‫‪+y‬‬ ‫‪′‬‬ ‫‪x‬‬ ‫‪+x‬‬ ‫‪′‬‬
‫= ‪ y‬ﻣﻨﻪ ‪:‬‬ ‫= ‪x‬و‬ ‫)ا( إذا ﻛﺎﻧﺖ ‪ I‬ﻣﻨﺘﺼﻒ اﻟﻘﻄﻌﺔ ‪ّ MM‬‬
‫ﻓﺈن ‪:‬‬ ‫•‬‫‪5‬‬
‫‪2‬‬ ‫‪2‬‬

‫‪x‬‬ ‫‪+x‬‬ ‫‪′‬‬ ‫‪1‬‬ ‫‪√2‬‬ ‫‪√2‬‬


‫= ‪x‬‬ ‫=‬ ‫‪x+‬‬ ‫= )‪(x + y‬‬ ‫‪1 + √2 x + y‬‬
‫‪2‬‬ ‫‪2‬‬ ‫‪2‬‬ ‫‪2‬‬
‫‪y‬‬ ‫‪+y‬‬ ‫‪′‬‬ ‫‪1‬‬ ‫‪√2‬‬ ‫‪√2‬‬
‫= ‪y‬‬ ‫=‬ ‫‪y+‬‬ ‫= )‪(x − y‬‬ ‫‪x + √2 − 1 y‬‬
‫‪2‬‬ ‫‪2‬‬ ‫‪2‬‬ ‫‪2‬‬

‫)ب( ﻟﺪﻳﻨﺎ ‪ . I ∈ Δ ⟺ y = √2 − 1 x :‬ﻟﻜﻦ ‪:‬‬


‫‪√2‬‬ ‫‪√2‬‬
‫⋅ ‪√2 − 1 x = √2 − 1‬‬ ‫= ‪1 + √2 x + y‬‬ ‫‪x + √2 − 1 y = y‬‬
‫‪2‬‬ ‫‪2‬‬
‫إذن اﻟﻨﻘﻄﺔ ‪ I‬ﺗﻨﺘﻤﻲ إﱃ اﳌﺴﺘﻘﻴﻢ ‪. Δ‬‬

‫‪ •6‬ﺣﺴﺐ ﻣﺎ ﺳﺒﻖ‪ ،‬اﻟﻘﻄﻌﺔ اﳌﺴﺘﻘﻴﻤﺔ ‪ MM‬ﺗُﻌﺎﻣﺪ اﳌﺴﺘﻘﻴﻢ ‪ Δ‬و ﻣﻨﺘﺼﻔﻬﺎ ‪ I‬ﻳﻨﺘﻤﻲ إﱃ ‪ Δ‬إذن اﻟﻨﻘﻄﺔ ‪ M‬ﻫﻲ ﺻﻮرة‬
‫اﻟﻨﻘﻄﺔ ‪ M‬ﺑﺎﻟﺘﻨﺎﻇﺮ اﳌﺤﻮري )اﻟﻌﻤﻮدي( ﺣﻮل اﳌﺴﺘﻘﻴﻢ ‪. Δ‬‬
‫■‬
‫‪ņőƱŀ Ţ‬‬
‫‪.‬‬
‫𝟖𝟔𝟏‬ ‫‪.‬‬
‫اﳌﺴﺘﻮي اﳌﺮﻛﺐ ﻣﻨﺴﻮب إﱃ ﻣﻌﻠﻢ ﻣﺘﻌﺎﻣﺪ و ﻣﺘﺠﺎﻧﺲ ⃗ ‪. O, i,⃗ j‬‬
‫ﻟﺘﻜﻦ )‪ A (a‬و )‪ B (b‬ﻧﻘﻄﺘﲔ ﻣﻦ اﳌﺴﺘﻮي ﺑﺤﻴﺚ ‪ a ≠ b‬و ﻟﻴﻜﻦ ‪ k ∈ ℝ‬ﺣﻴﺚ ‪. k > 0‬‬
‫‪VI‬‬

‫ﻟﻴﻜﻦ ‪ p‬و ‪ q‬ﻋﺪدﻳﻦ ﻣﺮﻛﺒﲔ ﻏﲑ ﻣﻌﺪوﻣﲔ و ∗‪. n ∈ ℕ‬‬


‫‪ •1‬أوﺟﺪ ‪ ℰ‬ﳎﻤﻮﻋﺔ اﻟﻨﻘﻂ ‪ M‬ﺑﺤﻴﺚ ‪. MA = kMB :‬‬
‫‪ •2‬إﺳﺘﻨﺘﺞ ّ‬
‫أن ﺻﻮر ﺣﻠﻮل اﳌﻌﺎدﻟﺔ ‪ p (z − a) + q (z − b) = 0‬ﺗﻨﺘﻤﻲ إﱃ داﺋﺮة أو ﻣﺴﺘﻘﻴﻢ ُﻳﻄﻠﺐ ﺗﻌﻴﻴﻨﻬﲈ‪.‬‬
‫‪ƕ‬‬

‫‪ •1‬ﻟﺘﻜﻦ )‪ M (z‬ﻧﻘﻄﺔ ﻣﻦ اﳌﺴﺘﻮي‪ .‬ﻧﻤ ﱢﻴﺰ ﺣﺎﻟﺘﲔ ‪:‬‬ ‫ﺍﳊﻞّ‪.‬‬


‫‪ũŏ‬‬

‫إذا ﻛﺎن ‪ّ k = 1‬‬


‫ﻓﺈن ‪:‬‬ ‫•‬
‫‪M ∈ ℰ ⟺ MA = MB‬‬
‫ﻣﻨﺼﻒ اﻟﻘﻄﻌﺔ اﳌﺴﺘﻘﻴﻤﺔ ]‪. [AB‬‬ ‫أي ّ‬
‫أن ‪ ℰ‬ﻫﻲ ﱢ‬

‫𝟭𝟱𝟮‬

‫‪http ://tinyurl.com/Malki1718‬‬ ‫‪0‬‬


‫ ﲤﺎرﻳﻦ ﻣﺘﻨﻮﻋﺔ‬.4.VI
.
ّ k ≠ 1 ‫إذا ﻛﺎن‬
: ‫ﻓﺈن‬ •

Ŕž Ŧ Ľ
M ∈ ℰ ⟺ MA = kMB ⟺ |z − a| = k ||z − b|| ⟺ |z − a| = k ||z − b||
⟺ (z − a) (z − a) = k (z − b) (z − b)

œǃ
⟺ 1−k |z| − a − k b z − a − k b z + |a| − k ||b|| = 0

a−k b a−k b |a| − k ||b||


⟺ |z| − z− z+ =0
1−k 1−k 1−k

Ŀ
|a| − k ||b|| a−k b
: ‫ ﻳﻜﻮن‬r = − ‫= 𝛼و‬ ‫ﺑﻮﺿﻊ‬
1−k 1−k

M ∈ ℰ ⟺ |z| − 𝛼z − 𝛼z − r = 0
⟺ (z − 𝛼) (z − 𝛼) − 𝛼𝛼 − r = 0
⟺ |z − 𝛼| − |𝛼| − r = 0 ⟺ |z − 𝛼| = |𝛼| + r
a−k b a−k b |a| − k ||b||
⟺ |z − 𝛼| = −
1−k 1−k

k ||a − b||
⟺ |z − 𝛼| =
1−k

: ‫ ﻟﺪﻳﻨﺎ‬. 𝛼 ‫ ﺻﻮرة‬Ω ‫ﻟﺘﻜﻦ‬

k ||a − b|| k ||a − b||


M ∈ ℰ ⟺ ΩM = ⟺ ΩM =
1−k | |
||1 − k ||
k
⟺ ΩM = AB
ņőƱŀ Ţ
| |
||1 − k ||

k
.| |
AB ‫ و ﻧﺼﻒ ﻗﻄﺮﻫﺎ‬Ω (𝛼) ‫ ﻫﻲ اﻟﺪاﺋﺮة اﻟﺘﻲ ﻣﺮﻛﺰﻫﺎ‬ℰ ‫إذن‬
||1 − k ||

‫• ﻧ ﱢ‬2
. p (z − a) + q (z − b) ‫ ﳎﻤﻮﻋﺔ ﺻﻮر ﺣﻠﻮل اﳌﻌﺎدﻟﺔ‬ℱ ‫ُﺴﻤﻲ‬ VI
: ‫ ﻟﺪﻳﻨﺎ‬. z ≠ b ‫ و‬z ≠ a ‫ ﻧﻘﻄﺔ ﻣﻦ اﳌﺴﺘﻮي ﺑﺤﻴﺚ‬M (z) ‫ﻟﺘﻜﻦ‬
M ∈ ℱ ⟹ p (z − a) + q (z − b) = 0 ⟹ p (z − a) = −q (z − b)
ƕ

z−a q |z − a| | q| |z − a| |q|
⟹ =− ⟹ | | = |− | ⟹ | = | |
z−b p | z−b | || p || | z − b || || p ||
ũŏ

|q|
: ‫ ﻳﻜﻮن‬k = | | ‫ﺑﻮﺿﻊ‬
|| p ||
MA
M∈ℱ ⟹ = k ⟹ MA = kMB
MB
: ‫و ﺣﺴﺐ اﻟﺴﺆال اﻟﺴﺎﺑﻖ‬
. [AB] ‫ﻨﺼﻒ اﻟﻘﻄﻌﺔ اﳌﺴﺘﻘﻴﻤﺔ‬ ّ ||p|| = ||q|| ‫ أي إذا ﻛﺎن‬k = 1 ‫إذا ﻛﺎن‬
‫ ُﳏﺘﻮاة ﰲ )ﺟﺰء ﻣﻦ( ُﻣ ﱢ‬ℱ ‫ﻓﺈن‬ •

𝟮𝟱𝟮

http ://tinyurl.com/Malki1718 0
‫‪ .VI‬ﲤﺎرﻳﻦ ﺗﻄﺒﻴﻘﻴﺔ‬
‫‪.‬‬
‫إذا ﻛﺎن ‪ k ≠ 1‬أي إذا ﻛﺎن ||‪ّ ||p|| ≠ ||q‬‬
‫ﻓﺈن ‪ُ ℱ‬ﳏﺘﻮاة ﰲ )ﺟﺰء ﻣﻦ( اﻟﺪاﺋﺮة اﻟﺘﻲ ﻣﺮﻛﺰﻫﺎ )𝛼( ‪ Ω‬و ﻧﺼﻒ ﻗﻄﺮﻫﺎ‬ ‫•‬

‫|‪|q‬‬ ‫‪a−k b‬‬ ‫‪k‬‬

‫‪Ŕž Ŧ Ľ‬‬
‫= ‪.k‬‬ ‫= 𝛼و| |‬ ‫| ﻣﻊ‬ ‫‪AB‬‬
‫|| ‪|| p‬‬ ‫‪1−k‬‬ ‫|‬
‫|| ‪||1 − k‬‬

‫‪œǃ‬‬
‫■‬

‫‪.‬‬
‫𝟗𝟔𝟏‬ ‫‪.‬‬

‫‪Ŀ‬‬
‫ﻋﲔ‪ ،‬ﺑﺎﳊﺴﺎب و ﻫﻨﺪﺳﻴ ًﺎ ‪ ،‬اﻷﻋﺪاد اﳌﺮﻛﺒﺔ ‪ z‬اﻟﺘﻲ ُﲢﻘﻖ ‪:‬‬
‫‪ •1‬ﱢ‬
‫‪| z − 3 | √2‬‬ ‫|‪|z − 3‬‬
‫|‬ ‫=|‬ ‫)ب(‬ ‫|‬ ‫‪|=1‬‬ ‫)ا(‬
‫|| ‪|| z − 5‬‬ ‫‪2‬‬ ‫|| ‪|| z − 5‬‬

‫‪ •2‬ﱢ‬
‫ﻋﻤﻢ ﻫﺬه اﻟﻨﺘﻴﺠﺔ إﱃ )‪ a, b, k ∈ ℝ‬ﻣﻊ ‪ k > 0 ، a ≠ b‬و ‪: (k ≠ 1‬‬
‫|‪|z − a‬‬ ‫|‪|z − a‬‬
‫‪|| z − b || = k‬‬ ‫)ب(‬ ‫‪|| z − b || = 1‬‬ ‫)ا(‬

‫)ا( ﺑﺎﳊﺴﺎب ‪ :‬ﻧﻀﻊ ‪ z = x + ıy‬ﻣﻨﻪ ‪:‬‬ ‫‪1‬‬


‫•‬ ‫ﺍﳊﻞّ‪.‬‬
‫‪||z − 3|| = (x − 3) + y‬‬ ‫‪,‬‬ ‫‪||z − 5|| = (x − 5) + y‬‬
‫|‪|z − 3‬‬
‫| ُﻳﻜﺎﻓﺊ ||‪ ||z − 3|| = ||z − 5‬أي ‪ x − 6x + 9 + y = x − 10x + 25 + y‬ﻣﻨﻪ‬ ‫و اﻟﴩط ‪| = 1‬‬
‫|| ‪|| z − 5‬‬
‫‪.x = 4‬‬
‫ﻓﺎﻷﻋﺪاد اﳌﺮﻛﺒﺔ اﻟﺘﻲ ﻧﺒﺤﺚ ﻋﻨﻬﺎ ﻫﻲ ﻛﻞ اﻷﻋﺪاد اﳌﺮﻛﺒﺔ اﻟﺘﻲ ﺟﺰؤﻫﺎ اﳊﻘﻴﻘﻲ ﻳﺴﺎوي ‪. x = 4‬‬
‫ﻫﻨﺪﺳﻴ ًﺎ ‪ :‬ﻧﻨﺴﺐ اﳌﺴﺘﻮي إﱃ ﻣﻌﻠﻢ ﻣﺘﻌﺎﻣﺪ و ﻣﺘﺠﺎﻧﺲ ⃗ ‪ . O, i,⃗ j‬ﻟﺘﻜﻦ ‪ M ، B ، A‬اﻟﻨﻘﻂ اﻟﺘﻲ ﻟﻮاﺣﻘﻬﺎ ‪، 3‬‬
‫‪ņőƱŀ Ţ‬‬
‫‪ z ، 5‬ﻋﲆ اﻟﱰﺗﻴﺐ‪ .‬ﻟﺪﻳﻨﺎ ‪:‬‬
‫|‪|z − 3‬‬
‫|‬ ‫‖‪| = 1 ⟺ ‖MA‖ = ‖MB‬‬
‫|| ‪|| z − 5‬‬

‫أن ‪ M‬ﺗﻨﺘﻤﻲ إﱃ ﳏﻮر اﻟﻘﻄﻌﺔ ]‪ . [AB‬ﻫﺬا اﳌﺤﻮر ﻫﻮ اﳌﺴﺘﻘﻴﻢ اﻟﺬي ﻣﻌﺎدﻟﺘﻪ ‪ ،x = 4‬إذن اﻟﻌﺪد‬‫و ﻫﺬا ﻳﻌﻨﻲ ّ‬
‫‪VI‬‬

‫اﳌﺮﻛﺐ ‪ُ z‬ﳛﻘﻖ اﳌﻄﻠﻮب إذا و ﻓﻘﻂ إذا ﻛﺎن ‪. Re (z) = 4‬‬


‫)ب( ﺑﺎﳊﺴﺎب ‪ :‬ﻧﻀﻊ ‪ z = x + ıy‬ﻣﻨﻪ ‪:‬‬
‫‪||z − 3|| = (x − 3) + y‬‬ ‫‪,‬‬ ‫‪||z − 5|| = (x − 5) + y‬‬
‫‪ƕ‬‬

‫‪1‬‬ ‫‪1‬‬ ‫‪| z − 3 | √2‬‬


‫= ‪x −6x+9+y‬‬ ‫‪x − 10x + 25 + y‬‬ ‫| ُﻳﻜﺎﻓﺊ ||‪ ||z − 3|| = ||z − 5‬أي‬ ‫=|‬ ‫و اﻟﴩط‬
‫‪2‬‬ ‫‪2‬‬ ‫|| ‪|| z − 5‬‬ ‫‪2‬‬
‫ﻣﻨﻪ ‪. (x − 1) + y = 8‬‬
‫‪ũŏ‬‬

‫‪(x − 1) +y = 8 = 2√2‬‬ ‫ﻓﺎﻷﻋﺪاد اﳌﺮﻛﺒﺔ اﻟﺘﻲ ﻧﺒﺤﺚ ﻋﻨﻬﺎ ﻫﻲ اﻷﻋﺪاد اﳌﺮﻛﺒﺔ ‪ z = x+ıy‬اﻟﺘﻲ ُﲢﻘﻖ‬
‫أي ﻫﻲ اﻟﺪاﺋﺮة اﻟﺘﻲ ﻣﺮﻛﺰﻫﺎ )‪ (1, 0‬و ﻧﺼﻒ ﻗﻄﺮﻫﺎ ‪. 2√2‬‬

‫𝟯𝟱𝟮‬

‫‪http ://tinyurl.com/Malki1718‬‬ ‫‪0‬‬


‫‪ .4.VI‬ﲤﺎرﻳﻦ ﻣﺘﻨﻮﻋﺔ‬
‫‪.‬‬
‫ﻫﻨﺪﺳﻴ ًﺎ ‪ :‬ﻧﻨﺴﺐ اﳌﺴﺘﻮي إﱃ ﻣﻌﻠﻢ ﻣﺘﻌﺎﻣﺪ و ﻣﺘﺠﺎﻧﺲ ⃗ ‪ . O, i,⃗ j‬ﻟﺪﻳﻨﺎ ‪:‬‬

‫‪Ŕž Ŧ Ľ‬‬
‫‪| z − 3 | √2‬‬ ‫‪1‬‬ ‫‪1‬‬
‫|‬ ‫=|‬ ‫||‪⟺ ||z − 3|| = ||z − 5‬‬ ‫= )‪⟺ (z − 3) (z − 3‬‬ ‫)‪(z − 5) (z − 5‬‬
‫|| ‪|| z − 5‬‬ ‫‪2‬‬ ‫‪2‬‬ ‫‪2‬‬

‫‪œǃ‬‬
‫‪1‬‬
‫= ‪⟺ zz − 3z − 3z + 9‬‬ ‫‪(zz − 5z − 5z + 25) ⟺ zz − (z + z) = 7‬‬
‫‪2‬‬
‫‪⟺ (z − 1) (z − 1) − 1 = 7 ⟺ ||z − 1|| = 8 ⟺ ||z − 1|| = 2√2‬‬

‫‪Ŀ‬‬
‫و ﺑﺎﻟﺘﺎﱄ ﻓﺎﳌﺠﻤﻮﻋﺔ اﻟﺘﻲ ﻧﺒﺤﺚ ﻋﻨﻬﺎ ﻫﻲ ﳎﻤﻮﻋﺔ اﻟﻨﻘﻂ اﻟﺘﻲ ﺗﺒﻌﺪ ﻋﻦ اﻟﻨﻘﻄﺔ )‪ (1, 0‬ﺑﻤﺴﺎﻓﺔ ‪ 2√2‬أي ﻫﻲ‬
‫اﻟﺪاﺋﺮة اﻟﺘﻲ ﻣﺮﻛﺰﻫﺎ )‪ (1, 0‬و ﻧﺼﻒ ﻗﻄﺮﻫﺎ ‪. 2√2‬‬
‫)ا( ﺑﺎﳊﺴﺎب ‪ :‬ﻧﻀﻊ ‪ z = x + ıy‬ﻣﻨﻪ ‪:‬‬ ‫•‬ ‫‪2‬‬
‫|‪|z − a‬‬
‫||‪|| z − b || = 1 ⟺ |z − a| = ||z − b‬‬
‫‪⟺ (x − a) + y = (x − b) + y‬‬
‫‪⟺ −2ax + a = −2bx + b‬‬
‫‪⟺ 2ax − 2bx + b − a = 0‬‬
‫‪a+b‬‬
‫‪⟺ 2 (a − b) x −‬‬ ‫‪=0‬‬
‫‪2‬‬
‫‪a+b‬‬
‫=‪⟺ x‬‬ ‫ّ‬
‫)ﻷن ‪(a ≠ b‬‬
‫‪2‬‬
‫و ﻫﻲ ﻣﻌﺎدﻟﺔ ﳏﻮر اﻟﻘﻄﻌﺔ اﳌﺴﺘﻘﻴﻤﺔ ]‪ [AB‬ﺣﻴﺚ )‪ A (a, 0‬و )‪. B (b, 0‬‬
‫ﻫﻨﺪﺳﻴ ًﺎ ‪ :‬ﻧﻨﺴﺐ اﳌﺴﺘﻮي إﱃ ﻣﻌﻠﻢ ﻣﺘﻌﺎﻣﺪ و ﻣﺘﺠﺎﻧﺲ ⃗ ‪. O, i,⃗ j‬‬
‫ﻟﺘﻜﻦ ‪ M ، B ، A‬اﻟﻨﻘﻂ اﻟﺘﻲ ﻟﻮاﺣﻘﻬﺎ ‪ z ، b ، a‬ﻋﲆ اﻟﱰﺗﻴﺐ‪ .‬ﻟﺪﻳﻨﺎ ‪:‬‬
‫|‪|z − a‬‬
‫‖‪|| z − b || = 1 ⟺ ‖MA‖ = ‖MB‬‬
‫‪ņőƱŀ Ţ‬‬
‫‪a+b‬‬
‫= ‪ ،x‬إذن‬ ‫و ﻫﺬا ﻳﻌﻨﻲ ّ‬
‫أن ‪ M‬ﺗﻨﺘﻤﻲ إﱃ ﳏﻮر اﻟﻘﻄﻌﺔ ]‪ . [AB‬ﻫﺬا اﳌﺤﻮر ﻫﻮ اﳌﺴﺘﻘﻴﻢ اﻟﺬي ﻣﻌﺎدﻟﺘﻪ‬
‫‪2‬‬
‫‪a+b‬‬
‫= )‪. Re (z‬‬ ‫اﻟﻌﺪد اﳌﺮﻛﺐ ‪ُ z‬ﳛﻘﻖ اﳌﻄﻠﻮب إذا و ﻓﻘﻂ إذا ﻛﺎن‬
‫‪2‬‬

‫‪VI‬‬
‫)ب( ﺑﺎﳊﺴﺎب ‪ :‬ﻧﻀﻊ ‪ z = x + ıy‬ﻣﻨﻪ ‪:‬‬
‫|‪|z − a‬‬
‫||‪|| z − b || = k ⟺ |z − a| = k ||z − b‬‬
‫‪⟺ (x − a) + y = k (x − b) + y‬‬
‫‪ƕ‬‬

‫‪⟺ x − 2ax + a = k‬‬ ‫‪x − 2bx + b‬‬

‫⟺‬ ‫‪k −1 x +2 a−k b x+ k −1 y =a −k b‬‬

‫‪a−k b‬‬ ‫‪a −k b‬‬


‫‪ũŏ‬‬

‫‪⟺ x +2‬‬ ‫= ‪x+y‬‬ ‫ّ‬


‫)ﻷن ‪(k ≠ 1‬‬
‫‪k −1‬‬ ‫‪k −1‬‬
‫‪a−k b‬‬ ‫‪a −k b‬‬ ‫‪a−k b‬‬
‫⟺‬ ‫‪x+‬‬ ‫= ‪+y‬‬ ‫‪+‬‬
‫‪k −1‬‬ ‫‪k −1‬‬ ‫‪k −1‬‬

‫‪a−k b‬‬ ‫‪k‬‬


‫⟺‬ ‫‪x+‬‬ ‫= ‪+y‬‬ ‫)‪(a − b‬‬
‫‪k −1‬‬ ‫‪k −1‬‬

‫𝟰𝟱𝟮‬

‫‪http ://tinyurl.com/Malki1718‬‬ ‫‪0‬‬


‫‪ .VI‬ﲤﺎرﻳﻦ ﺗﻄﺒﻴﻘﻴﺔ‬
‫‪.‬‬
‫‪a−k b‬‬ ‫)‪k (a − b‬‬
‫⟺‬ ‫‪x+‬‬ ‫= ‪+y‬‬
‫‪k −1‬‬ ‫‪k −1‬‬

‫‪Ŕž Ŧ Ľ‬‬
‫| )‪| k (a − b‬‬ ‫‪a−k b‬‬
‫| )ﻗﻴﻤﺔ ﻣﻄﻠﻘﺔ( ‪.‬‬ ‫‪ Ω −‬و ﻧﺼﻒ ﻗﻄﺮﻫﺎ |‬ ‫و ﻫﻲ ﻣﻌﺎدﻟﺔ اﻟﺪاﺋﺮة اﻟﺘﻲ ﻣﺮﻛﺰﻫﺎ ‪, 0‬‬

‫‪œǃ‬‬
‫|| ‪|| k − 1‬‬ ‫‪k −1‬‬
‫ﻫﻨﺪﺳﻴ ًﺎ ‪ :‬ﻧﻨﺴﺐ اﳌﺴﺘﻮي إﱃ ﻣﻌﻠﻢ ﻣﺘﻌﺎﻣﺪ و ﻣﺘﺠﺎﻧﺲ ⃗ ‪. O, i,⃗ j‬‬

‫|‪|z − a‬‬

‫‪Ŀ‬‬
‫||‪|| z − b || = k ⟺ |z − a| = k ||z − b‬‬ ‫ﻟﺪﻳﻨﺎ ‪:‬‬
‫)‪⟺ (z − a) (z − a) = k (z − b) (z − b‬‬
‫‪⟺ zz − az − az + a = k‬‬ ‫‪zz − bz − bz + b‬‬

‫⟺‬ ‫‪k − 1 zz + a − k b (z + z) + k b − a = 0‬‬

‫‪a−k b‬‬ ‫‪k b −a‬‬


‫‪⟺ zz +‬‬ ‫‪(z + z) +‬‬ ‫‪=0‬‬ ‫ّ‬
‫)ﻷن ‪(k ≠ 1‬‬
‫‪k −1‬‬ ‫‪k −1‬‬
‫‪a−k b‬‬ ‫‪a−k b‬‬ ‫‪a−k b‬‬ ‫‪k b −a‬‬
‫⟺‬ ‫‪z+‬‬ ‫‪z+‬‬ ‫‪−‬‬ ‫‪+‬‬ ‫‪=0‬‬
‫‪k −1‬‬ ‫‪k −1‬‬ ‫‪k −1‬‬ ‫‪k −1‬‬
‫‪a−k b‬‬ ‫‪a−k b‬‬ ‫‪k‬‬
‫⟺‬ ‫‪z+‬‬ ‫‪z+‬‬ ‫‪−‬‬ ‫‪(a − b) = 0‬‬
‫‪k −1‬‬ ‫‪k −1‬‬ ‫‪k −1‬‬

‫|‬ ‫|‪a − k b‬‬ ‫)‪k (a − b‬‬


‫‪⟺ |z +‬‬ ‫= |‬
‫||‬ ‫|| ‪k − 1‬‬ ‫‪k −1‬‬
‫|‬ ‫| )‪a − k b | | k (a − b‬‬
‫‪⟺ |z +‬‬ ‫|=|‬ ‫|‬
‫||‬ ‫|| ‪k − 1 || || k − 1‬‬
‫| )‪| k (a − b‬‬ ‫‪a−k b‬‬
‫‪ņőƱŀ Ţ‬‬
‫| )ﻗﻴﻤﺔ ﻣﻄﻠﻘﺔ( ‪.‬‬ ‫‪ Ω −‬و ﻧﺼﻒ ﻗﻄﺮﻫﺎ |‬ ‫‪,0‬‬ ‫و ﻫﻲ ﻣﻌﺎدﻟﺔ اﻟﺪاﺋﺮة اﻟﺘﻲ ﻣﺮﻛﺰﻫﺎ‬
‫|| ‪|| k − 1‬‬ ‫‪k −1‬‬
‫■‬
‫‪VI‬‬

‫‪.‬‬
‫𝟎𝟕𝟏‬ ‫‪.‬‬
‫أن ﳎﻤﻮﻋﺔ اﻟﻨﻘﻂ ‪ z = x + ıy‬ﻣﻦ اﳌﺴﺘﻮي اﳌﺮﻛﺐ و اﻟﺘﻲ ُﲢﻘﻖ ‪:‬‬
‫أﺛﺒﺖ ّ‬
‫‪|z − ıa| = 𝜆 |z + ıa| ,‬‬ ‫‪𝜆 > 0, a ∈ ℂ ,‬‬
‫| 𝜆‪| 2a‬‬ ‫𝜆‪1+‬‬
‫| إذا ﻛﺎن ‪. 𝜆 ≠ 1‬‬ ‫‪ z = ıa‬و ﻧﺼﻒ ﻗﻄﺮﻫﺎ |‬ ‫ﻫﻲ اﻟﺪاﺋﺮة اﻟﺘﻲ ﻣﺮﻛﺰﻫﺎ اﻟﻨﻘﻄﺔ‬
‫|| 𝜆 ‪|| 1 −‬‬ ‫𝜆‪1−‬‬
‫‪ƕ‬‬

‫أدرس اﳊﺎﻟﺔ ‪. 𝜆 = 1‬‬

‫ﺍﳊﻞّ‪ .‬ﺑﲈ أﻧﻨﺎ ﻧﺒﺤﺚ ﻋﻦ ﳎﻤﻮﻋﺔ ﻧﻘﻂ ﰲ اﳌﺴﺘﻮي ‪ ، xOy‬ﻓﺒﺪاﻳ ًﺔ ﳚﺐ اﻟﺘﻌﺒﲑ ﻋﻦ اﳌﻘﺪار ْﻳﻦ |‪ |z + ıa‬و |‪ |z − ıa‬ﺑﺪﻻﻟﺔ‬
‫‪ũŏ‬‬

‫ﺗﻨﺲ ّ‬
‫أن ‪ a‬ﻋﺪد ﻣﺮﻛﺐ( ‪:‬‬ ‫اﻹﺣﺪاﺛﻴﲔ ‪ x‬و ‪) y‬ﻻ َ‬
‫ْ‬
‫)‪|z + ıa| = ||x + ıy + ıa|| = (x + ıy + ıa) (x − ıy − ıa‬‬
‫)‪= x + y + |a| + ıa (x − ıy) − ıa (x + ıy‬‬
‫)‪|z − ıa| = ||x + ıy − ıa|| = (x + ıy − ıa) (x − ıy + ıa‬‬
‫)‪= x + y + |a| − ıa (x − ıy) + ıa (x + ıy‬‬

‫𝟱𝟱𝟮‬

‫‪http ://tinyurl.com/Malki1718‬‬ ‫‪0‬‬


‫ ﲤﺎرﻳﻦ ﻣﺘﻨﻮﻋﺔ‬.4.VI
.
: ‫ﻳﻨﺘﺞ‬ ||x + ıy − ıa|| = 𝜆 ||x + ıy + ıa|| ‫و ﺑﺎﻟﺘﻌﻮﻳﺾ ﰲ اﳌﻌﺎدﻟﺔ‬

Ŕž Ŧ Ľ
|z − ıa| = 𝜆 |z + ıa| ⟺ ||x + ıy − ıa|| = 𝜆 ||x + ıy + ıa||
⟺ ||x + ıy − ıa|| = 𝜆 ||x + ıy + ıa||

œǃ
⟺ x + y + |a| − ıa (x − ıy) + ıa (x + ıy)
=𝜆 x + y + |a| + ıa (x − ıy) − ıa (x + ıy)

⟺ 1−𝜆 x −ı 1+𝜆 (a − a) x+

Ŀ
+ 1−𝜆 y − 1+𝜆 (a + a) y + 1 − 𝜆 |a| = 0

ّ 𝜆 = 1 ‫ أي إذا ﻛﺎن‬1 − 𝜆 = 0 ‫إذا ﻛﺎن‬


2ı (a − a) x − 2 (a + a) y = 0 : ‫ﻓﺈن اﳌﻌﺎدﻟﺔ ﺗﺼﺒﺢ‬ •

‫ و ﻫﻲ ﻣﻌﺎدﻟﺔ ﻣﺴﺘﻘﻴﻢ )و اﳌﺴﺘﻘﻴﻢ ﻋﺒﺎرة‬Im (a) ⋅ x − Re (a) ⋅ y = 0 ‫ أي‬4 Im (a) ⋅ x − 4 Re (a) ⋅ y = 0 ‫أي‬
. (+∞ ‫ﻋﻦ داﺋﺮة ﻧﺼﻒ ﻗﻄﺮﻫﺎ ﻳﺴﺎوي‬
: ‫ ﻳﻨﺘﺞ‬1 − 𝜆 ‫ﻃﺮﰲ اﳌﻌﺎدﻟﺔ اﻷﺧﲑة ﻋﲆ‬
ْ ‫ ﻓﺒﻘﺴﻤﺔ‬، 𝜆 ≠ 1 ‫ أي إذا ﻛﺎن‬1 − 𝜆 ≠ 0 ‫إذا ﻛﺎن‬ •

1+𝜆 1+𝜆
|z − ıa| = 𝜆 |z + ıa| ⟺ x − ı (a − a) x + y − (a + a) y + |a| = 0
1−𝜆 1−𝜆
1 + 𝜆 (a − a) 1 + 𝜆 (a + a)
⟺ x +2 x+y −2 y + |a| = 0
1−𝜆 2ı 1−𝜆 2
1+𝜆 1+𝜆
⟺ x + 2 Im (a) x + y − 2 Re (a) y + |a| = 0
1−𝜆 1−𝜆
1+𝜆 1+𝜆
⟺ x+ Im (a) + y− Re (a) + |a|
1−𝜆 1−𝜆

1+𝜆
− (Im (a)) + (Re (a)) =0
ņőƱŀ Ţ
1−𝜆

1+𝜆 1+𝜆
⟺ x+ Im (a) + y− Re (a) + |a|
1−𝜆 1−𝜆

VI
1+𝜆
− |a| = 0
1−𝜆

1+𝜆 1+𝜆 1+𝜆


⟺ x+ Im (a) + y− Re (a) =⎡ − 1⎤ |a|
1−𝜆 1−𝜆 ⎣ 1−𝜆
ƕ


‫و ﻫﻲ ﻣﻌﺎدﻟﺔ ﻣﻦ اﻟﺸﻜﻞ‬
1+𝜆 4𝜆
(x − 𝛼) + (y − 𝛽) = ⎡ − 1⎤ |a| = |a|
ũŏ

⎣ 1−𝜆 ⎦ 1−𝜆
‫ﺣﻴﺚ‬
1+𝜆 1+𝜆
𝛼 + ı𝛽 = (− Im (a) + ı Re (a)) = ıa
1−𝜆 1−𝜆
| 2a𝜆 |
.| | ‫ و ﻧﺼﻒ ﻗﻄﺮﻫﺎ‬z = 𝛼 + ı𝛽 ‫أي ﻫﻲ ﻣﻌﺎدﻟﺔ اﻟﺪاﺋﺮة اﻟﺘﻲ ﻣﺮﻛﺰﻫﺎ اﻟﻨﻘﻄﺔ‬
|| 1 − 𝜆 ||

𝟮𝟱𝟲

http ://tinyurl.com/Malki1718 0
‫‪ .VI‬ﲤﺎرﻳﻦ ﺗﻄﺒﻴﻘﻴﺔ‬
‫‪.‬‬
‫‪γ‬‬
‫‪b‬‬

‫‪Ŕž Ŧ Ľ‬‬
‫‪a‬‬

‫‪œǃ‬‬
‫‪α‬‬

‫‪β‬‬ ‫‪c‬‬

‫ﺷﻜﻞ ‪32.VI‬‬

‫‪Ŀ‬‬
‫■‬

‫!‬ ‫‪.‬‬
‫𝟏𝟕𝟏‬ ‫‪.‬‬
‫‪ •1‬ﻟﺘﻜﻦ ‪ b ، a‬و ‪ c‬أﻃﻮال أﺿﻼع ﻣﺜﻠﺚ ﻣﺴﺎﺣﺘﻪ ‪ . s‬أﺛﺒﺖ ّ‬
‫أن ‪:‬‬
‫)‪16s = (a + b + c) (a + b − c) (a + c − b) (b + c − a‬‬
‫|‬ ‫‪1‬‬ ‫|‬
‫| 𝛼𝛾 ‪. s = | Im 𝛼𝛽 + 𝛽𝛾 +‬‬ ‫‪ •2‬ﻟﺘﻜﻦ 𝛼 ‪ 𝛽 ،‬و 𝛾 ﻟﻮاﺣﻖ رؤوس ﻣﺜﻠﺚ ﻣﺴﺎﺣﺘﻪ ‪ . s‬أﺛﺒﺖ ّ‬
‫أن ‪:‬‬
‫‪|| 2‬‬ ‫||‬

‫ﺍﳊﻞّ‪.‬‬
‫‪1‬‬
‫‪ •1‬ﻟﺪﻳﻨﺎ ‪ s = bc ||sin 𝛼|| :‬و 𝛼 ‪) a = b + c − 2bc cos‬ﺷﻜﻞ ‪ (32.VI‬ﻣﻨﻪ ‪:‬‬
‫‪2‬‬
‫‪1‬‬
‫)‪16s = 16 × b c sin 𝛼 = (2bc‬‬ ‫)𝛼 ‪1 − cos 𝛼 = (2bc) − (2bc cos‬‬
‫‪4‬‬
‫‪= (2bc) − a − b − c‬‬ ‫‪= 2bc + a − b − c‬‬ ‫‪2bc − a + b + c‬‬

‫)‪= a − (b − c‬‬ ‫‪(b + c) − a‬‬


‫‪ņőƱŀ Ţ‬‬
‫)‪= (a + b − c) (a − b + c) (b + c − a) (b + c + a‬‬

‫و ﻫﻮ اﳌﻄﻠﻮب‪.‬‬
‫‪VI‬‬

‫‪′‬‬
‫‪ •2‬ﻟﻴﻜﻦ ‪ z = 𝜌e‬و ‪ z = 𝜌e‬ﻋﺪدﻳﻦ ﻣﺮﻛﺒﲔ‪ .‬اﳌﻘﺪار ‪:‬‬
‫‪1‬‬ ‫|‬ ‫‪| |1‬‬ ‫‪′‬‬
‫‪−‬‬
‫‪| |1‬‬ ‫‪′‬‬
‫‪| = | Im 𝜌 e 𝜌e−‬‬
‫‪| |1‬‬ ‫|‬
‫‪𝜌𝜌 |sin 𝜃 − 𝜃 | = | Im 𝜌𝜌 e‬‬ ‫‪| = | Im z z‬‬ ‫|‬
‫‪2‬‬ ‫|‬ ‫‪| || 2‬‬ ‫‪|| || 2‬‬ ‫‪|| || 2‬‬ ‫||‬

‫ﻳﻤ ّﺜﻞ ﻣﺴﺎﺣﺔ اﳌﺜﻠﺜﺎت اﻟﺘﻲ ﻟﻮاﺣﻖ رؤوﺳﻬﺎ ﻫﻲ اﻷﻋﺪاد ‪ w + z ، w‬و ‪ w + z‬ﺣﻴﺚ ‪ w‬ﻋﺪد ﻣﺮﻛﺐ‪.‬‬
‫‪ƕ‬‬

‫ﰲ ﺣﺎﻟﺘﻨﺎ ‪ w + z = 𝛽 ، w = 𝛼 :‬و 𝛾 = ‪ w + z‬أي 𝛼 ‪ z = 𝛽 − w = 𝛽 −‬و 𝛼 ‪ z = 𝛾 − w = 𝛾 −‬و ﺑﺎﻟﺘﻌﻮﻳﺾ‬


‫أن )‪ Im Z = − Im (Z‬و ّ‬
‫أن ‪ Im (t) = 0‬إذا ﻛﺎن ‪: (t ∈ ℝ‬‬ ‫ﰲ اﳌﺴﺎواة اﻟﺴﺎﺑﻘﺔ ﻧﺠﺪ )ﻧُﺬﻛّﺮ ّ‬
‫‪|1‬‬ ‫‪| |1‬‬ ‫|‬
‫‪ũŏ‬‬

‫| )𝛼 ‪s = | Im z z | = | Im (𝛽 − 𝛼) (𝛾 −‬‬
‫‪|| 2‬‬ ‫‪|| || 2‬‬ ‫||‬
‫‪|1‬‬ ‫‪| |1‬‬ ‫|‬
‫| 𝛼𝛼 ‪= | Im 𝛽 − 𝛼 (𝛾 − 𝛼) | = | Im 𝛽𝛾 − 𝛽𝛼 − 𝛼𝛾 +‬‬
‫‪|| 2‬‬ ‫‪|| || 2‬‬ ‫||‬
‫‪|1‬‬ ‫|‬
‫| )𝛼𝛼( ‪= | Im 𝛽𝛾 − Im 𝛽𝛼 − Im (𝛼𝛾) + Im‬‬
‫‪|| 2‬‬ ‫||‬

‫𝟳𝟱𝟮‬

‫‪http ://tinyurl.com/Malki1718‬‬ ‫‪0‬‬


‫ ﲤﺎرﻳﻦ ﻣﺘﻨﻮﻋﺔ‬.4.VI
.
|1 |
= | Im 𝛽𝛾 + Im 𝛽𝛼 + Im 𝛼𝛾 + Im |𝛼| |
|| 2 ||

Ŕž Ŧ Ľ
|1 |
= | Im 𝛽𝛾 + Im (𝛽𝛼) + Im (𝛼𝛾) + 0 |
|| 2 ||

œǃ
|1 |
= | Im 𝛼𝛽 + 𝛽𝛾 + 𝛾𝛼 |
|| 2 ||

.‫و ﻫﻮ اﳌﻄﻠﻮب‬

Ŀ

ņőƱŀ Ţ
VI
ũŏ ƕ

𝟮𝟱𝟴

http ://tinyurl.com/Malki1718 0
‫‪.‬‬

‫‪7‬‬
‫‪Ŕž Ŧ Ľ‬‬
‫‪œǃ‬‬
‫‪otttttttttttttttttttttttttttttttttttttttttttp‬‬
‫‪s‬‬ ‫‪u‬‬
‫‪s‬‬ ‫ﺗﻤﺎرﻳﻦ ﻟﻠﺘﻌﻤﻖ‬
‫‪u‬‬

‫‪Ŀ‬‬
‫‪s‬‬ ‫‪u‬‬
‫‪qvvvvvvvvvvvvvvvvvvvvvvvvvvvvvvvvvvvvvvvvvvvr‬‬
‫‪.‬‬

‫ﻣ ﱢﺜﻞ ﺑﻴﺎﻧﻴ ًﺎ ﰲ اﳌﺴﺘﻮي اﳌﺮﻛﺐ ﳎﻤﻮﻋﺎت اﻟﻨﻘﻂ اﻟﺘﺎﻟﻴﺔ ‪:‬‬


‫‬ ‫‪.‬‬
‫𝟐𝟕𝟏‬ ‫‪.‬‬

‫‪G = z ∈ ℂ : 1 ≤ ||z − 2|| < 2‬‬ ‫•‬ ‫‪4‬‬ ‫}‪D = {z ∈ ℂ : |z| = 2‬‬ ‫‪1‬‬
‫•‬

‫||‪H = z ∈ ℂ : ||z − 2|| = ||z + 2ı‬‬ ‫•‬ ‫‪5‬‬ ‫‪E = z ∈ ℂ : ||z − 2|| = 1‬‬ ‫‪2‬‬
‫•‬

‫||‪I = z ∈ ℂ : ||z − 2|| < ||z + 2ı‬‬ ‫•‬ ‫‪6‬‬ ‫‪F = z ∈ ℂ : ||z − 2|| ≤ 1‬‬ ‫‪3‬‬
‫•‬

‫ﺍﳊﻞّ‪ .‬اﻟﻨﻘﻄﺔ اﻷﺳﺎﺳﻴﺔ ﻫﻨﺎ ﻫﻲ ﻣﻌﺮﻓﺔ ّ‬


‫أن |‪ |z − w‬ﻫﻲ اﳌﺴﺎﻓﺔ ﺑﲔ اﻟﻨﻘﻄﺘﲔ ‪ z‬و ‪ w‬ﰲ اﳌﺴﺘﻮي اﳌﺮﻛﺐ‪.‬‬

‫‪ |z| = 2 : (D) •1‬ﻫﻲ ﳎﻤﻮﻋﺔ اﻟﻨﻘﻂ اﻟﺘﻲ ﺗﺒﻌﺪ ﺑﻤﺴﺎﻓﺔ ‪ 2‬ﻋﻦ اﳌﺒﺪأ ‪ O‬أي ﻫﻲ اﻟﺪاﺋﺮة اﻟﺘﻲ ﻣﺮﻛﺰﻫﺎ )‪ O(0, 0‬و ﻧﺼﻒ‬
‫‪ņőƱŀ Ţ‬‬
‫ﻗﻄﺮﻫﺎ ‪.2‬‬
‫‪ ||z − 2|| = 1 : (E) •2‬ﻫﻲ ﳎﻤﻮﻋﺔ اﻟﻨﻘﻂ اﻟﺘﻲ ﺗﺒﻌﺪ ﺑﻤﺴﺎﻓﺔ ‪ 1‬ﻋﻦ اﻟﻨﻘﻄﺔ ‪ 2‬أي ﻫﻲ اﻟﺪاﺋﺮة اﻟﺘﻲ ﻣﺮﻛﺰﻫﺎ )‪ (2, 0‬و ﻧﺼﻒ‬
‫ﻗﻄﺮﻫﺎ ‪.1‬‬
‫‪ ||z − 2|| ≤ 1 : (F) •3‬ﻫﻲ ﳎﻤﻮﻋﺔ اﻟﻨﻘﻂ اﻟﺘﻲ ﺗﺒﻌﺪ ﺑﻤﺴﺎﻓﺔ ‪ 1‬أو ّ‬
‫أﻗﻞ ﻋﻦ اﻟﻨﻘﻄﺔ ‪ 2‬أي ﻫﻲ اﻟﻘﺮص اﻟﺬي ﻣﺮﻛﺰه )‪ (2, 0‬و‬
‫ﻧﺼﻒ ﻗﻄﺮه ‪.1‬‬
‫)‪ 1 ≤ ||z − 2|| < 2 : (G‬ﻫﻲ ﳎﻤﻮﻋﺔ اﻟﻨﻘﻂ اﻟﺘﻲ ﺗﺒﻌﺪ ﺑﻤﺴﺎﻓﺔ أﻛﱪ ﻣﻦ أو ﺗﺴﺎوي ‪ 1‬و أﺻﻐﺮ ﲤﺎﻣ ًﺎﻣﻦ ‪ ،2‬ﻋﻦ اﻟﻨﻘﻄﺔ ‪2‬‬ ‫‪4‬‬
‫•‬
‫‪ƕ‬‬

‫أي ﻫﻲ اﳊﻠﻘﺔ اﻟﺘﻲ ﻣﺮﻛﺰﻫﺎ )‪ (2, 0‬و ﻧﺼﻒ ﻗﻄﺮﻫﺎ اﻷﺻﻐﺮ ‪ 1‬و ﻧﺼﻒ ﻗﻄﺮﻫﺎ اﻷﻛﱪ ‪ 2‬ﺑﺎﺳﺘﺜﻨﺎء اﻟﺪاﺋﺮة اﻟﺘﻲ ﻣﺮﻛﺰﻫﺎ‬
‫‪VII‬‬

‫)‪ (2, 0‬و ﻧﺼﻒ ﻗﻄﺮﻫﺎ ‪.2‬‬


‫)‪ ||z − 2|| = ||z + 2ı|| : (H‬ﻫﻲ ﳎﻤﻮﻋﺔ اﻟﻨﻘﻂ اﳌﺘﺴﺎوﻳﺔ اﳌﺴﺎﻓﺔ ﻣﻦ اﻟﻴﻘﻄﺘﲔ ‪ 2‬و ‪ −2ı‬أي ﻫﻲ ّ‬
‫ﻣﻨﺼﻒ اﻟﻘﻄﻌﺔ ]‪[AB‬‬ ‫‪5‬‬
‫•‬
‫‪ũŏ‬‬

‫ﺣﻴﺚ )‪ A(2, 0‬و )‪. B(0, −2‬‬


‫‪ ||z − 2|| < ||z + 2ı|| : (I) •6‬ﻫﻲ ﳎﻤﻮﻋﺔ اﻟﻨﻘﻂ اﻷﻗﺮب ﻣﻦ اﻟﻨﻘﻄﺔ ‪ 2‬ﻣﻨﻪ ﻣﻦ اﻟﻨﻘﻄﺔ ‪ −2ı‬أي ﻫﻲ ﻧﺼﻒ اﳌﺴﺘﻮي اﻟﻮاﻗﻊ‬
‫اﳌﻨﺼﻒ‪.‬‬
‫اﳌﻨﺼﻒ ﻟﻠﻘﻄﻌﺔ ]‪ [AB‬ﺑﺎﺳﺘﺜﻨﺎء ﻫﺬا ّ‬
‫ﻋﲆ ﻳﻤﲔ ) ﻓﻮق ( اﳌﺴﺘﻘﻴﻢ ّ‬
‫■‬

‫𝟵𝟱𝟮‬

‫‪http ://tinyurl.com/Malki1718‬‬ ‫‪0‬‬


.

Ŕž Ŧ Ľ
y y
(D)

œǃ
(E)
2
. 2 .
x x

Ŀ (G)
y

.
x
y

. 2
(F)

x
ņőƱŀ Ţ
y y

(I) (H)
ƕ

. 2 . 2
VII

x x
ũŏ

−2ı −2ı

𝟮𝟲𝟬

http ://tinyurl.com/Malki1718 0
‫‪ .VII‬ﲤﺎرﻳﻦ ﻟﻠﺘﻌﻤﻖ‬
‫‪.‬‬

‫‪#‬‬ ‫‪.‬‬
‫𝟑𝟕𝟏‬ ‫‪.‬‬

‫‪Ŕž Ŧ Ľ‬‬
‫ﻟﻴﻜﻦ ‪ n‬ﻋﺪد ًا ﻃﺒﻴﻌﻴﺎ‪.‬‬
‫ﺛﻢ أوﺟﺪ ﻃﻮﻳﻠﺘﻪ ﺣﺴﺐ ِﻗ َﻴﻢ ‪. n‬‬ ‫‪ •1‬أﺛﺒﺖ ّ‬
‫أن اﻟﻌﺪد )‪ z = (1 + ı) + (1 − ı‬ﺣﻘﻴﻘﻲ ّ‬

‫‪œǃ‬‬
‫ﺛﻢ أوﺟﺪ ﻃﻮﻳﻠﺘﻪ ﺣﺴﺐ ِﻗ َﻴﻢ ‪. n‬‬ ‫ِ‬ ‫‪ •2‬أﺛﺒﺖ ّ‬
‫أن اﻟﻌﺪد )‪ z = (1 + ı) − (1 − ı‬ﲣﻴﲇ ﴏف ّ‬

‫‪Ŀ‬‬
‫ﺍﳊﻞّ‪ .‬ﻧﻀﻊ ‪ a = 1 + ı‬ﻣﻨﻪ ‪ a = 1 − ı‬و ﺑﺎﻟﺘﺎﱄ ‪:‬‬
‫‪ •1‬ﻟﺪﻳﻨﺎ ‪:‬‬
‫‪z =a +a =a +a =a +a =a +a =a +a =z‬‬

‫أن ‪ّ z = z‬‬
‫ﻓﺈن ‪:‬‬ ‫إذن ‪ z‬ﺣﻘﻴﻘﻲ‪ .‬و ﺑﲈ ّ‬

‫‪||z || = z ⋅ z = z = a + a‬‬ ‫‪=a‬‬ ‫‪+a‬‬ ‫‪+ 2a a‬‬

‫‪= 2 Re a‬‬ ‫‪+ 2 (aa) = 2 Re‬‬ ‫‪a‬‬ ‫|‪+ 2 |a‬‬

‫ﻟﻜﻦ ‪ a = 2ı‬ﻣﻨﻪ ‪ |a| = ||a || = 2‬و ﺑﺎﻟﺘﺎﱄ ‪:‬‬

‫‪||z || = 2 Re (2ı) + 2 × 2‬‬

‫أن )‪ ı = (−1‬إذا ﻛﺎن ‪ n‬زوﺟﻴ ًﺎ و ‪ ı = 0‬إذا ﻛﺎن ‪ n‬ﻓﺮدﻳ ًﺎ ّ‬


‫ﻓﺈن ‪:‬‬ ‫و ﺑﲈ ّ‬
‫‪+‬‬
‫)‪2 (−2‬‬ ‫‪+2×2‬‬ ‫‪ ⎧ 2‬إذا ﻛﺎن ‪ n‬زوﺟﻴ ًﺎ ؛‬ ‫)‪(−1‬‬ ‫‪+2‬‬ ‫إذا ﻛﺎن ‪ n‬زوﺟﻴ ًﺎ ؛‬
‫= || ‪||z‬‬ ‫=‬
‫‪2×0+2×2‬‬ ‫⎨ إذا ﻛﺎن ‪ n‬ﻓﺮدﻳ ًﺎ ؛‬‫⎩‬ ‫‪2‬‬
‫‪+‬‬
‫إذا ﻛﺎن ‪ n‬ﻓﺮدﻳ ًﺎ ؛‬
‫‪ņőƱŀ Ţ‬‬
‫ﻣﻨﻪ ‪:‬‬
‫⎧‬
‫⎪‬
‫⎪‬ ‫‪2‬‬
‫‪+‬‬
‫‪+2‬‬ ‫إذا ﻛﺎن ‪ n‬زوﺟﻴ ًﺎ ؛‬ ‫‪⎧2‬‬ ‫‪+‬‬
‫)‪(−1‬‬ ‫)‪(−1‬‬ ‫‪+2‬‬ ‫إذا ﻛﺎن ‪ n‬زوﺟﻴ ًﺎ ؛‬
‫= || ‪||z‬‬ ‫=‬
‫⎨‬
‫⎪‬
‫⎪‬ ‫⎨‬ ‫‪+‬‬

‫‪2‬‬
‫‪+‬‬
‫إذا ﻛﺎن ‪ n‬ﻓﺮدﻳ ًﺎ ؛‬ ‫⎩‬ ‫‪2‬‬ ‫إذا ﻛﺎن ‪ n‬ﻓﺮدﻳ ًﺎ ؛‬
‫⎩‬

‫‪ •2‬ﻟﺪﻳﻨﺎ ‪:‬‬
‫‪ƕ‬‬

‫‪z =a −a =a −a =a −a =a −a =− a −a‬‬ ‫‪= −z‬‬


‫‪VII‬‬

‫إذن ‪ z‬ﲣﻴﲇ ِﴏف‪ .‬و ﺑﲈ ّ‬


‫أن ‪ّ z = −z‬‬
‫ﻓﺈن ‪:‬‬
‫‪ũŏ‬‬

‫‪||z || = z ⋅ z = −z = − a − a‬‬ ‫‪= −a‬‬ ‫‪−a‬‬ ‫‪+ 2a a‬‬

‫‪= −2 Re a‬‬ ‫‪+ 2 (aa) = −2 Re‬‬ ‫‪a‬‬ ‫|‪+ 2 |a‬‬

‫ﻟﻜﻦ ‪ a = 2ı‬ﻣﻨﻪ ‪ |a| = ||a || = 2‬و ﺑﺎﻟﺘﺎﱄ ‪:‬‬

‫)‪||z || = 2 × 2 − 2 Re (2ı‬‬

‫𝟭𝟲𝟮‬

‫‪http ://tinyurl.com/Malki1718‬‬ ‫‪0‬‬


‫‪.‬‬
‫أن )‪ ı = (−1‬إذا ﻛﺎن ‪ n‬زوﺟﻴ ًﺎ و ‪ ı = 0‬إذا ﻛﺎن ‪ n‬ﻓﺮدﻳ ًﺎ ّ‬
‫ﻓﺈن ‪:‬‬ ‫و ﺑﲈ ّ‬
‫‪+‬‬

‫‪Ŕž Ŧ Ľ‬‬
‫)‪2 × 2 − 2 (−2‬‬ ‫‪ ⎧ 2‬إذا ﻛﺎن ‪ n‬زوﺟﻴ ًﺎ ؛‬ ‫‪2‬‬ ‫)‪− (−1‬‬ ‫إذا ﻛﺎن ‪ n‬زوﺟﻴ ًﺎ ؛‬
‫= || ‪||z‬‬ ‫=‬
‫⎨ إذا ﻛﺎن ‪ n‬ﻓﺮدﻳ ًﺎ ؛ ‪2 × 2 − 2 × 0‬‬‫⎩‬ ‫‪2‬‬
‫‪+‬‬
‫إذا ﻛﺎن ‪ n‬ﻓﺮدﻳ ًﺎ ؛‬

‫‪œǃ‬‬
‫ﻣﻨﻪ ‪:‬‬
‫⎧‬
‫⎪‬
‫⎪‬ ‫‪2‬‬
‫‪+‬‬
‫‪2‬‬ ‫)‪− (−1‬‬ ‫إذا ﻛﺎن ‪ n‬زوﺟﻴ ًﺎ ؛‬ ‫‪⎧2‬‬ ‫‪+‬‬

‫= || ‪||z‬‬
‫‪2‬‬ ‫)‪− (−1‬‬ ‫إذا ﻛﺎن ‪ n‬زوﺟﻴ ًﺎ ؛‬
‫=‬
‫⎨‬ ‫⎨‬

‫‪Ŀ‬‬
‫‪+‬‬
‫⎪‬
‫⎪‬ ‫‪2‬‬ ‫إذا ﻛﺎن ‪ n‬ﻓﺮدﻳ ًﺎ ؛‬
‫⎩‬ ‫‪2‬‬
‫‪+‬‬
‫إذا ﻛﺎن ‪ n‬ﻓﺮدﻳ ًﺎ ؛‬ ‫⎩‬

‫ﻳﻤﻜﻦ ﺗﺒﺴﻴﻂ اﻟﻨﺘﺎﺋﺞ اﻟﺴﺎﺑﻘﺔ و ذﻟﻚ ﺑﺎﻋﺘﺒﺎر ِﻗﻴﻢ ﺑﻮاﻗﻲ اﻟﻘﺴﻤﺔ اﻹﻗﻠﻴﺪﻳﺔ ﻟـِ ‪ n‬ﻋﲆ ‪: 4‬‬ ‫ﻣﻼﺣﻈﺔ ‪⧏ : 37‬‬

‫)‪ (−1) = (−1‬ﻣﻨﻪ ‪:‬‬ ‫إذا ﻛﺎن ‪ّ n = 4k‬‬


‫ﻓﺈن ‪= 1‬‬ ‫•‬

‫‪+‬‬
‫‪+‬‬ ‫‪+‬‬
‫‪||z || = 2‬‬ ‫‪1+2‬‬ ‫‪=2‬‬ ‫‪1+2‬‬ ‫‪= 2 √2 1 + 2‬‬ ‫‪=2‬‬ ‫‪2‬‬ ‫‪+2‬‬
‫‪+‬‬
‫‪+‬‬ ‫‪+‬‬
‫‪||z || = 2‬‬ ‫‪2‬‬ ‫‪−1=2‬‬ ‫‪2‬‬ ‫‪− 1 = 2 √2 2‬‬ ‫‪−1=2‬‬ ‫‪2‬‬ ‫‪−2‬‬ ‫و‬
‫‪+‬‬
‫‪. ||z || = 2‬‬ ‫‪=2‬‬ ‫|| ‪√2 = ||z‬‬ ‫إذا ﻛﺎن ‪ّ n = 4k + 1‬‬
‫ﻓﺈن ‪:‬‬ ‫•‬

‫‪+‬‬
‫)‪ (−1) = (−1‬ﻣﻨﻪ ‪:‬‬ ‫إذا ﻛﺎن ‪ّ n = 4k + 2‬‬
‫ﻓﺈن ‪= −1‬‬ ‫•‬

‫‪+‬‬ ‫‪+‬‬
‫‪+‬‬ ‫‪+‬‬
‫‪||z || = 2‬‬ ‫‪−1 + 2‬‬ ‫‪=2‬‬ ‫‪−1 + 2‬‬
‫‪+‬‬ ‫‪+‬‬
‫‪+‬‬ ‫‪+‬‬
‫‪||z || = 2‬‬ ‫‪2‬‬ ‫‪−1=2‬‬ ‫‪2‬‬ ‫‪−1‬‬ ‫و‬
‫‪+‬‬
‫‪+‬‬
‫‪. ||z || = 2‬‬ ‫‪=2‬‬ ‫|| ‪= ||z‬‬ ‫إذا ﻛﺎن ‪ّ n = 4k + 3‬‬
‫ﻓﺈن ‪:‬‬ ‫•‬

‫⧐‬
‫‪ņőƱŀ Ţ‬‬
‫■‬

‫‪x+y=1+ı‬‬
‫‬ ‫‪.‬‬
‫𝟒𝟕𝟏‬ ‫‪.‬‬
‫‪.‬‬ ‫ﺣﻞ ﰲ اﳌﺠﻤﻮﻋﺔ ‪ ℂ‬اﳉﻤﻠﺔ ‪:‬‬
‫‪xy = 2 − ı‬‬

‫ﺍﳊﻞّ‪ x .‬و ‪ y‬ﺣﻼن ﻟﻠﻤﻌﺎدﻟﺔ ‪ z − (x + y) z + xy = 0‬أي ‪. z − (1 + ı) + 2 − ı = 0‬‬


‫‪ƕ‬‬

‫ﻟﺪﻳﻨﺎ ‪. Δ = (−1 − ı) − 4 × 1 × (2 − ı) = −8 + 6ı :‬‬


‫‪VII‬‬

‫ﻟﻴﻜﻦ ‪ 𝛿 = a + ıb‬ﺟﺬر ًا ﺗﺮﺑﻴﻌﻴﺎ ﻟﻠﻌﺪد ‪ Δ‬ﻣﻊ ‪ . a, b ∈ ℝ‬ﻟﺪﻳﻨﺎ ‪:‬‬


‫⎧‬
‫= ||‪⎪ a + b = ||−8 + 6ı‬‬
‫⎪‬ ‫‪(−8) + 6 = 10‬‬
‫‪ũŏ‬‬

‫⟺ ‪𝛿 = Δ = −8 + 6ı‬‬ ‫‪a − b = −8‬‬


‫⎨‬
‫⎪‬
‫⎪‬
‫⎩‬ ‫‪2ab = 6‬‬
‫ﺑﺠﻤﻊ اﳌﻌﺎدﻟﺘﲔ اﻷوﱃ و اﻟﺜﺎﻧﻴﺔ ﻳﻨﺘﺞ ‪ 2a = 10 − 8 = 2‬ﻣﻨﻪ ‪ . a = ±1‬ﻧﺨﺘﺎر ﻣﺜ ً‬
‫ﻼ ‪ a = 1‬ﻓﻴﻜﻮن )ﻣﻦ اﳌﻌﺎدﻟﺔ اﻟﺜﺎﻟﺜﺔ(‬
‫‪1 + ı + 1 + 3ı‬‬ ‫‪1 + ı − 1 − 3ı‬‬
‫= ‪ z‬أي )‪ (x, y) = (−ı, 1 + 2ı‬أو‬ ‫= ‪ z‬و ‪= 1 + 2ı‬‬ ‫‪ . b = 3‬ﻣﻨﻪ ‪= −ı‬‬
‫‪2‬‬ ‫‪2‬‬
‫)‪. (x, y) = (1 + 2ı, −ı‬‬
‫■‬ ‫ﳎﻤﻮﻋﺔ اﳊﻠﻮل ﻫﻲ إذن ‪. S = {(−ı, 1 + 2ı) , (1 + 2ı, −ı)} :‬‬

‫𝟮𝟲𝟮‬

‫‪http ://tinyurl.com/Malki1718‬‬ ‫‪0‬‬


‫‪ .VII‬ﲤﺎرﻳﻦ ﻟﻠﺘﻌﻤﻖ‬
‫‪.‬‬

‫‪#‬‬ ‫‪.‬‬
‫𝟓𝟕𝟏‬ ‫‪.‬‬

‫‪Ŕž Ŧ Ľ‬‬
‫‪xy‬‬ ‫‪=3‬‬
‫ﻟﻴﻜﻦ ‪ x‬و ‪ y‬ﻋﺪدﻳﻦ ﻣﺮﻛﺒﲔ ‪ّ .‬‬
‫ﺣﻞ ﰲ ‪ ℂ‬اﳉﻤﻠﺔ ‪:‬‬
‫‪x + y = 10‬‬

‫‪œǃ‬‬
‫ﺍﳊﻞّ‪ .‬ﻟﺪﻳﻨﺎ ‪ّ . x + y = (x + y) − 2xy :‬‬
‫ﳊﻞ اﳉﻤﻠﺔ ﻧﻀﻊ ‪ S = x + y‬و ‪ P = xy‬ﻣﻨﻪ ‪:‬‬
‫‪xy‬‬ ‫‪=3‬‬ ‫‪P‬‬ ‫‪=3‬‬ ‫‪P=3‬‬ ‫‪P=3‬‬

‫‪Ŀ‬‬
‫⟺‬ ‫⟺‬ ‫⟺‬
‫‪x + y = 10‬‬ ‫‪S − 2P = 10‬‬ ‫‪S = 10 + 2 × 3 = 16‬‬ ‫‪S = ±4‬‬

‫ﻹﳚﺎد ‪ x‬و ‪ّ y‬‬


‫ﻧﺤﻞ اﳌﻌﺎدﻟﺔ ‪ t − St + P = 0‬ﰲ اﳊﺎﻟﺘﲔ ‪:‬‬

‫ﻼ اﳌﻌﺎدﻟﺔ ‪ . t + 4t + 3 = 0‬ﻟﺪﻳﻨﺎ ‪ Δ = 2 − 3 = 1‬إذن ﻟﻠﻤﻌﺎدﻟﺔ‬ ‫إذا ﻛﺎن ‪ S = −4‬و ‪ّ P = 3‬‬


‫ﻓﺈن ‪ x‬و ‪ y‬ﳘﺎ َﺣ ﱠ‬ ‫•‬
‫ﺣﻼّن ﳘﺎ ‪ t = −2 − 1 = −3‬و ‪ . t = −2 + 1 = −1‬أي ّ‬
‫أن })‪. (x, y) ∈ {(−3, −1) , (−1, −3‬‬

‫ﻼ اﳌﻌﺎدﻟﺔ ‪ . t − 4t + 3 = 0‬ﻟﺪﻳﻨﺎ ‪ Δ = 2 − 3 = 1‬إذن ﻟﻠﻤﻌﺎدﻟﺔ‬ ‫إذا ﻛﺎن ‪ S = 4‬و ‪ّ P = 3‬‬


‫ﻓﺈن ‪ x‬و ‪ y‬ﳘﺎ َﺣ ﱠ‬ ‫•‬
‫ﺣﻼّن ﳘﺎ ‪ . t = 2 + 1 = 3 t = 2 − 1 = 1‬أي })‪. (x, y) ∈ {(1, 3) , (3, 1‬‬
‫■‬ ‫})‪. 𝒮 = {(−3, −1) , (−1, −3) , (1, 3) , (3, 1‬‬ ‫ﰲ اﻷﺧﲑ‪ ،‬ﳎﻤﻮﻋﺔ ﺣﻠﻮل اﳉﻤﻠﺔ ﻫﻲ ‪:‬‬

‫‪#‬‬ ‫‪.‬‬
‫𝟔𝟕𝟏‬ ‫‪.‬‬
‫‪ •1‬أﺛﺒﺖ أﻧﻪ إذا ﻛﺎن ‪ xy + yz + zx = b ، x + y + z = a‬و ‪ xyz = c‬ﻓﺈن ‪ z ، y ، x‬ﺣﻠﻮل ﻟﻠﻤﻌﺎدﻟﺔ ‪:‬‬
‫‪Z − aZ + bZ − c = 0‬‬
‫أوﺟﺪ ‪ y ، x‬و ‪ z‬إذا ﻛﺎن ‪ a = b = 0‬و ‪. c = −8‬‬
‫⎧‬ ‫‪x+y+z=4‬‬
‫⎪‬
‫‪.‬‬ ‫‪x +y +z =4‬‬ ‫‪ •2‬ﺣﻞ ﰲ ‪ ℝ‬اﳉﻤﻠﺔ ‪:‬‬
‫⎨‬
‫‪ņőƱŀ Ţ‬‬
‫⎪‬
‫‪⎩x + y + z = 1‬‬

‫ﻧﻌﺘﱪ ﻛﺜﲑ اﳊﺪود ‪ y ، x . P (Z) = Z − aZ + bZ − c‬و ‪ z‬ﺣﻠﻮل ﻟﻠﻤﻌﺎدﻟﺔ ‪Z − aZ + bZ − c = 0‬‬ ‫•‬‫‪1‬‬ ‫ﺍﳊﻞّ‪.‬‬
‫إذا و ﻓﻘﻂ إذا ﻛﺎﻧﺖ ﺟﺬور ًا ﻟـِ ‪ P‬أي إذا و ﻓﻘﻂ إذا ﻛﺎن )‪ . P (Z) = (Z − x) (Z − y) (Z − z‬ﻟﻜﻦ ‪:‬‬
‫‪(Z − x) (Z − y) (Z − z) = Z − (x + y + z) Z + (xy + yz + zx) Z − xyz‬‬
‫و ﺑﺎﻟﺘﺎﱄ إذا ﻛﺎن ‪ b = xy + yz + zx ، a = x + y + z‬و ‪ّ c = xyz‬‬
‫ﻓﺈن ‪ y ، x‬و ‪ z‬ﺟﺬور ﻟﻜﺜﲑ اﳊﺪود‬
‫‪ƕ‬‬

‫‪ P (Z) = Z − aZ + bZ − c‬أي ‪ y ، x‬و ‪ z‬ﺣﻠﻮل ﻟﻠﻤﻌﺎدﻟﺔ ‪. Z − aZ + bZ − c = 0‬‬


‫‪VII‬‬

‫إذا ﻛﺎن ‪ a = b = 0‬و ‪ّ c = −8‬‬


‫ﻓﺈن ‪ y ، x‬و ‪ z‬ﻫﻲ ﺣﻠﻮل اﳌﻌﺎدﻟﺔ ‪ Z = −8‬أي ﻫﻲ اﻷﻋﺪاد ‪ −2j ، −2‬و ‪. −2j‬‬
‫‪. ( −2, −2j, −2j‬‬ ‫‪) {x, y, z} = −2, −2j, −2j‬ﺗﻮﺟﺪ ‪ 3! = 6‬ﺣﻠﻮل و ﻫﻲ ﺗﺒﺪﻳﻼت اﳌﺠﻤﻮﻋﺔ‬ ‫إذن ‪:‬‬
‫‪ũŏ‬‬

‫𝟯𝟲𝟮‬

‫‪http ://tinyurl.com/Malki1718‬‬ ‫‪0‬‬


‫‪.‬‬
‫ِ‬
‫اﻷول‪ ،‬ﻛﲈ ﻳﲇ ‪:‬‬ ‫‪ •2‬ﺑﺪاﻳﺔ ﻧﺒﺤﺚ ﻋﻦ ﻗﻴﻢ اﳌﻘﺎدﻳﺮ ‪ b ، a‬و ‪ ، c‬ا ُﳌ ّ‬
‫ﻌﺮﻓﺔ ﰲ اﻟﺴﺆال ّ‬

‫‪Ŕž Ŧ Ľ‬‬
‫)‪x + y + z = (x + y + z) − 2 (xy + yz + zx‬‬ ‫ﻟﺪﻳﻨﺎ ‪:‬‬
‫)‪4 = 4 − 2 (xy + yz + zx‬‬ ‫أي‬

‫‪œǃ‬‬
‫‪1‬‬
‫= ‪xy + yz + zx‬‬‫‪(16 − 4) = 6‬‬ ‫ﻣﻨﻪ ‪:‬‬
‫‪2‬‬
‫)‪x + y + z = (x + y + z) − 3 (x + y) (x + z) (y + z‬‬ ‫ﺑﺎﳌﺜﻞ ‪:‬‬
‫)‪= 4 − 3 (4 − z) (4 − y) (4 − x‬‬

‫‪Ŀ‬‬
‫‪= 48 (x + y + z) − 12 (xy + yz + zx) + 3xyz − 128‬‬
‫‪= 48 (4) − 12 (6) + 3xyz − 128‬‬
‫‪= −8 + 3xyz‬‬
‫‪1‬‬
‫= ‪xyz‬‬ ‫‪x +y +z‬‬ ‫‪+8‬‬ ‫ﻣﻨﻪ ‪:‬‬
‫‪3‬‬
‫‪1‬‬
‫‪= (9) = 3‬‬
‫‪3‬‬
‫و ﺑﺎﻟﺘﺎﱄ ‪ y ، x‬و ‪ z‬ﻫﻲ ﺣﻠﻮل اﳌﻌﺎدﻟﺔ ‪. Z − 4Z + 6Z − 3 = 0‬‬
‫أن ‪ّ Z = 1‬‬
‫ﺣﻞ ﳍﺎ إذن ‪. Z − 4Z + 6Z − 3 = (Z − 1) Z − 3Z + 3‬‬ ‫ﻧﻼﺣﻆ ّ‬
‫‪3 ± ı√3‬‬
‫‪ .‬و ﺑﺎﻟﺘﺎﱄ ‪:‬‬ ‫ﻟﻠﻤﻌﺎدﻟﺔ ‪ Z − 3Z + 3 = 0‬ﺣﻼّن ﳘﺎ‬
‫‪2‬‬
‫‪3 − ı√3 3 + ı√3‬‬ ‫‪3 − ı√3 3 + ı√3‬‬
‫‪.( 1,‬‬ ‫‪,‬‬ ‫‪) {x, y, z} = 1,‬ﺗﻮﺟﺪ ‪ 3! = 6‬ﺣﻠﻮل و ﻫﻲ ﺗﺒﺪﻳﻼت اﳌﺠﻤﻮﻋﺔ‬ ‫‪,‬‬
‫‪2‬‬ ‫‪2‬‬ ‫‪2‬‬ ‫‪2‬‬
‫■‬

‫‪x − 15x y + 15x y − y = 1‬‬


‫‪#‬‬ ‫‪.‬‬
‫𝟕𝟕𝟏‬ ‫‪.‬‬
‫‪.‬‬ ‫ّ‬
‫ﺣﻞ ﰲ ‪ ℝ‬اﳉﻤﻠﺔ ‪:‬‬
‫‪ņőƱŀ Ţ‬‬
‫‪3x y − 10x y + 3xy = 0‬‬

‫ﺍﳊﻞّ‪ .‬ﺑﲈ ّ‬
‫أن اﻟﻌﺪدﻳﻦ ‪ x‬و ‪ y‬ﺣﻘﻴﻘﻴﺎن‪ّ ،‬‬
‫ﻓﺈن اﳉﻤﻠﺔ اﻟﺴﺎﺑﻘﺔ ﺗُﻜﺎﻓﺊ ‪:‬‬
‫‪x − 15x y + 15x y − y‬‬ ‫‪+ 2ı 3x y − 10x y + 3xy‬‬ ‫‪= 1 + 0ı = 1‬‬
‫أي‬
‫‪x + 6x (ıy) + 15x (ıy) + 20x (ıy) + 15x (ıy) + 6x(ıy) + (ıy) = 1‬‬
‫و ﻫﺬه اﻷﺧﲑة ﺗُﻜﺎﻓﺊ ‪. (x + ıy) = 1‬‬
‫𝜋‪k‬‬ ‫𝜋‪k‬‬
‫‪ƕ‬‬

‫ﻣﻊ }‪ k ∈ {0, 1, 2, 3, 4, 5‬و ﺑﺎﻟﺘﺎﱄ ّ‬


‫ﻓﺈن‬ ‫‪𝜔 = cos‬‬ ‫‪+ ı sin‬‬ ‫ﻟﻜﻨﻨﺎ ﻧﻌﻠﻢ ّ‬
‫أن اﳉﺬور اﻟﺴﺎدﺳﺔ ﻟﻠﻮﺣﺪة ﻫﻲ اﻷﻋﺪاد‬
‫‪VII‬‬

‫‪3‬‬ ‫‪3‬‬
‫ﺣﻠﻮل اﳉﻤﻠﺔ اﳌﻌﻄﺎة ﻫﻲ ‪:‬‬
‫𝜋‪k‬‬ ‫𝜋‪k‬‬
‫‪x = cos‬‬ ‫‪, y = sin‬‬ ‫‪,‬‬ ‫}‪k ∈ {0, 1, 2, 3, 4, 5‬‬
‫‪3‬‬ ‫‪3‬‬
‫‪ũŏ‬‬

‫■‬

‫‪.‬‬
‫𝟖𝟕𝟏‬ ‫‪.‬‬
‫)‪(1 + t) = (1 − t‬‬
‫ﺣﻴﺚ ∗‪ n ∈ ℕ‬ﻋﺪد ﻃﺒﻴﻌﻲ ﻏﲑ ﻣﻌﺪوم ُﻣﻌﻄﻰ‪.‬‬ ‫ّ‬
‫ﺣﻞ ﰲ ‪ ℂ‬اﳉﻤﻠﺔ‬
‫‪z +t =1‬‬

‫𝟰𝟲𝟮‬

‫‪http ://tinyurl.com/Malki1718‬‬ ‫‪0‬‬


‫‪ .VII‬ﲤﺎرﻳﻦ ﻟﻠﺘﻌﻤﻖ‬
‫‪.‬‬
‫ﺍﳊﻞّ‪ .‬ﺑﺪاﻳ ًﺔ‪ّ ،‬‬
‫ﻧﺤﻞ اﳌﻌﺎدﻟﺔ )‪ . (1 + t) = (1 − t‬ﻳﻜﻮن اﻟﻌﺪد اﳌﺮﻛﺐ ‪ t ∈ ℂ‬ﺣﻼ ﳍﺬه اﳌﻌﺎدﻟﺔ إذا و ﻓﻘﻂ إذا ُوﺟﺪ‬
‫ﻋﺪد ﻣﺮﻛﺐ ‪ u‬ﺑﺤﻴﺚ ‪ u = 1‬و )‪ 1 + t = u (1 − t‬أي إذا و ﻓﻘﻂ إذا ُوﺟﺪ ﻋﺪد ﻃﺒﻴﻌﻲ }‪ k ∈ {0, 1, ⋯ , n − 1‬ﺑﺤﻴﺚ‬

‫‪Ŕž Ŧ Ľ‬‬
‫𝜋‪k‬‬ ‫𝜋‪k‬‬ ‫‪/‬‬ ‫‪/‬‬ ‫‪/‬‬
‫‪. t cos‬‬ ‫‪= ı sin‬‬ ‫‪ . 1 + e‬ﰲ اﻷﺧﲑ ‪:‬‬ ‫‪t=e‬‬ ‫‪ 1 + t = e‬أي ﺑﺤﻴﺚ ‪− 1‬‬ ‫)‪(1 − t‬‬
‫‪n‬‬ ‫‪n‬‬

‫‪œǃ‬‬
‫𝜋‪k‬‬
‫‪ cos‬و ﺑﺎﻟﺘﺎﱄ ﻟﻠﻤﻌﺎدﻟﺔ ‪ n‬ﺣﻼ ﻫﻲ اﻷﻋﺪاد‬ ‫إذا ﻛﺎن ‪ n‬ﻓﺮدﻳ ًﺎ‪ ،‬ﻓﺈﻧﻪ ﻟﻜﻞ }‪ k ∈ {0, 1, ⋯ , n − 1‬ﻳﻜﻮن ‪≠ 0‬‬ ‫•‬
‫‪n‬‬
‫𝜋‪k‬‬
‫‪ ı tan‬ﻣﻊ }‪. k ∈ {0, 1, ⋯ , n − 1‬‬
‫‪n‬‬

‫‪Ŀ‬‬
‫إذا ﻛﺎن ‪ n‬زوﺟﻴ ًﺎ‪ ،‬أي ‪ n = 2p‬ﻓﺈﻧﻪ ﻷﺟﻞ ‪ k = p‬ﻳﻜﻮن ‪:‬‬ ‫•‬
‫𝜋‪p‬‬ ‫𝜋‬ ‫𝜋‪p‬‬ ‫𝜋‪p‬‬ ‫𝜋‬
‫‪sin‬‬ ‫‪= sin ≠ 0‬‬ ‫و‬ ‫‪cos‬‬ ‫‪= cos‬‬ ‫‪= cos = 0‬‬
‫‪n‬‬ ‫‪2‬‬ ‫‪n‬‬ ‫‪2p‬‬ ‫‪2‬‬
‫و ﺑﺎﻟﺘﺎﱄ ﻓﺎﳌﻌﺎدﻟﺔ ﻻ ّ‬
‫ﺣﻞ ﳍﺎ؛ ﻟﻜﻦ ﻟﻜﻞ }‪ k ∈ {0, 1, ⋯ , p − 1, p + 1, ⋯ , n − 1‬ﻳﻜﻮن ﻟﻠﻤﻌﺎدﻟﺔ اﳊﻞ اﻟﺬي ﻋﺒﺎرﺗﻪ‬
‫𝜋‪k‬‬
‫‪ ı tan‬و ﰲ ﻫﺬه اﳊﺎﻟﺔ‪ ،‬اﳌﻌﺎدﻟﺔ ﺗﻘﺒﻞ ‪ n − 1‬ﺣﻼ‪.‬‬ ‫ﻫﻲ‬
‫‪n‬‬
‫𝜋‪k‬‬
‫‪ ، ı tan‬ﻳﻜﻮن اﻟﻌﺪد اﳌﺮﻛﺐ ‪ z‬ﺣﻼ ﻟﻠﻤﻌﺎدﻟﺔ اﻟﺜﺎﻧﻴﺔ إذا و ﻓﻘﻂ إذا ﻛﺎن ‪:‬‬ ‫ﻟﻜﻞ ّ‬
‫ﺣﻞ ‪ t‬ﻟﻠﻤﻌﺎدﻟﺔ اﻟﺴﺎﺑﻘﺔ ﻣﻦ اﻟﺸﻜﻞ‬
‫‪n‬‬
‫𝜋‪k‬‬ ‫‪1‬‬
‫‪z = 1 − t = 1 + tan‬‬ ‫=‬
‫‪n‬‬ ‫‪cos‬‬
‫‪1‬‬
‫‪.z = ±‬‬ ‫أي إذا و ﻓﻘﻂ إذا ﻛﺎن‬
‫‪cos‬‬
‫ﰲ اﻷﺧﲑ‪ ،‬ﻟﻠﺠﻤﻠﺔ ‪ 2 × (n − 1) = 2n − 2‬ﺣﻼ إذا ﻛﺎن ‪ n‬زوﺟﻴ ًﺎ و ‪ 2n‬ﺣﻼ إذا ﻛﺎن ‪ n‬ﻓﺮدﻳ ًﺎ‪.‬‬
‫■‬

‫ﻫﻞ اﻟﻨﻘﻂ ‪ D ، C ، B ، A‬اﻟﺘﻲ ﻟﻮاﺣﻘﻬﺎ ‪ 2 + ı ، 1 − 3ı ، −2 + 2ı ، −3 − 2ı‬ﻋﲆ اﻟﱰﺗﻴﺐ ﺗﻨﺘﻤﻲ إﱃ ﻧﻔﺲ‬


‫‪#‬‬ ‫‪.‬‬
‫𝟗𝟕𝟏‬ ‫‪.‬‬

‫اﻟﺪاﺋﺮة ؟‬
‫‪ņőƱŀ Ţ‬‬
‫ﺍﳊﻞّ‪ .‬ﺗﻨﺘﻤﻲ اﻟﻨﻘﻂ ‪ C ، B ، A‬و ‪ D‬إﱃ ﻧﻔﺲ اﻟﺪاﺋﺮة إذا و ﻓﻘﻂ إذا ُو ِﺟﺪت ﻧﻘﻄﺔ ‪ Ω‬ﻣﻦ اﳌﺴﺘﻮي ﺑﺤﻴﺚ ‪:‬‬
‫‪ ΩA = ΩB = ΩC = ΩD‬؛ و ﰲ ﻫﺬه اﳊﺎﻟﺔ ‪ Ω‬ﻫﻲ ﻣﺮﻛﺰ ﻫﺬه اﻟﺪاﺋﺮة و ‪) ΩA‬أو ‪ ΩB‬أو ‪ ΩC‬أو ‪ ( ΩD‬ﻧﺼﻒ ﻗﻄﺮﻫﺎ‪.‬‬
‫ﻟﺘﻜﻦ ‪ Ω‬ﻧﻘﻄﺔ ﻣﻦ اﳌﺴﺘﻮي ﻻﺣﻘﺘﻬﺎ‪ z = x + ıy‬ﻣﻊ ‪ . x, y ∈ ℝ‬ﻟﺪﻳﻨﺎ ‪:‬‬

‫⎧‬ ‫‪ΩB = ΩA‬‬ ‫⎧‬ ‫)‪(x + 2) + (y − 2) = (x + 3) + (y + 2‬‬


‫‪⎪ ΩB = ΩA‬‬
‫⎧‬ ‫⎪‬
‫⎪‬ ‫⎪‬
‫⎪‬
‫⟺ ‪ΩC = ΩA‬‬ ‫‪ΩC = ΩA‬‬ ‫⟺‬ ‫)‪(x − 1) + (y + 3) = (x + 3) + (y + 2‬‬
‫⎨‬
‫⎪‬ ‫⎨‬
‫⎪‬
‫⎪‬ ‫⎨‬
‫⎪‬
‫⎪‬
‫‪⎩ ΩD = ΩA‬‬ ‫‪⎩ ΩD = ΩA‬‬ ‫)‪⎩ (x − 2) + (y − 1) = (x + 3) + (y + 2‬‬
‫‪ƕ‬‬

‫‪x + y + 4x − 4y + 8 = x + y + 6x + 4y + 13‬‬
‫‪VII‬‬

‫⎧‬
‫⎪‬
‫⟺‬ ‫‪x + y − 2x + 6y + 10 = x + y + 6x + 4y + 13‬‬
‫⎨‬
‫⎪‬
‫‪⎩ x + y − 4x − 2y + 5 = x + y + 6x + 4y + 13‬‬
‫‪ũŏ‬‬

‫‪⎪ 2x + 8y + 5 = 0‬‬
‫⎧‬ ‫‪⎧ 2x + 8y + 5 = 0‬‬ ‫)اﳌﻌﺎدﻟﺔ اﻟﺜﺎﻟﺜﺔ ﻫﻲ‬
‫⟺‬ ‫⟺ ‪8x − 2y + 3 = 0‬‬
‫⎨‬
‫⎪‬ ‫‪⎨ 8x − 2y + 3 = 0‬‬ ‫ﳎﻤﻮع اﳌﻌﺎدﻟﺘﲔ اﻷُو َﻟ َﻴ ْﲔ(‬
‫‪⎩ 10x + 6y + 8 = 0‬‬ ‫⎩‬

‫⎧‬ ‫‪1‬‬
‫‪2x = −8y − 5‬‬ ‫‪⎧ 2x = −8y − 5‬‬ ‫‪⎪x=−‬‬
‫⟺‬ ‫⟺‬ ‫⟺‬ ‫‪2‬‬
‫‪4 (−8y − 5) − 2y + 3 = 0‬‬ ‫‪⎨ y = −1‬‬ ‫‪⎪ y = −1‬‬
‫⎨‬
‫⎩‬ ‫‪2‬‬ ‫⎩‬ ‫‪2‬‬

‫𝟱𝟲𝟮‬

‫‪http ://tinyurl.com/Malki1718‬‬ ‫‪0‬‬


‫‪.‬‬
‫‪1 1‬‬
‫ﻫﺬا ﻳﻌﻨﻲ أﻧﻪ ﺗﻮﺟﺪ ﻧﻘﻄﺔ ‪) Ω‬وﺣﻴﺪة( ‪ ،‬و ﻫﻲ اﻟﻨﻘﻄﺔ ذات اﻟﻼﺣﻘﺔ ‪ ، z = − − ı‬ﺑﺤﻴﺚ ﻳﻜﻮن ‪ΩA = ΩB = ΩC = ΩD‬‬
‫‪2 2‬‬

‫‪Ŕž Ŧ Ľ‬‬
‫و ﺑﺎﻟﺘﺎﱄ ﻓﺎﻟﻨﻘﻂ ‪ C ، B ، A‬و ‪ D‬ﺗﻨﺘﻤﻲ ﻛ ّﻠﻬﺎ إﱃ اﻟﺪاﺋﺮة اﻟﺘﻲ ﻣﺮﻛﺰﻫﺎ ‪ Ω‬و ﻧﺼﻒ ﻗﻄﺮﻫﺎ ‪ ΩA‬ﻣﻊ ‪:‬‬
‫‪1‬‬ ‫‪1‬‬ ‫‪√34‬‬
‫= ‪ΩA‬‬ ‫‪−3 +‬‬ ‫‪+ −2 +‬‬ ‫=‬

‫‪œǃ‬‬
‫‪2‬‬ ‫‪2‬‬ ‫‪2‬‬
‫‪34‬‬
‫■‬ ‫أن √ = ‪. ΩB = ΩC = ΩD‬‬‫ﻧﺘﺤﻘﻖ ﺑﺴﻬﻮﻟﺔ ﻣﻦ ّ‬
‫‪2‬‬

‫‪Ŀ‬‬
‫ﻣﺎ ﻫﻲ ِﻗ َﻴﻢ ‪ a ∈ ℝ‬اﻟﺘﻲ ﻣﻦ أﺟﻠﻬﺎ ﻳﻜﻮن ‪ x = 1‬ﺣﻼ ﻟﻠﻤﻌﺎدﻟﺔ ‪ x + 2x + 2ax − a = 0‬؟‬
‫‬ ‫‪.‬‬
‫𝟎𝟖𝟏‬ ‫‪.‬‬

‫ﰲ ﻫﺬه اﳊﺎﻟﺔ‪ ،‬ﻣﺎ ﻫﻲ اﳊﻠﻮل اﻷﺧﺮى ؟‬

‫ﺍﳊﻞّ‪ x = 1 .‬ﺣﻞ ﻟﻠﻤﻌﺎدﻟﺔ ‪ x + 2x + 2ax − a = 0‬إذا و ﻓﻘﻂ إذا ﻛﺎن ‪ 1 + 2 × 1 + 2a × 1 − a = 0‬أي‬


‫‪. a − 2a − 3 = 0‬‬
‫‪2+4‬‬ ‫‪2−4‬‬
‫= ‪.a‬‬ ‫= ‪a‬و‪= 3‬‬ ‫ﻟﺪﻳﻨﺎ ‪ Δ = (−2) − 4 × 1 × (−3) = 16 = 4 :‬ﻣﻨﻪ ‪= −1‬‬
‫‪2‬‬ ‫‪2‬‬
‫أن ‪ x = 1‬ﺣﻞ ﳍﺬه اﳌﻌﺎدﻟﺔ ّ‬
‫ﻓﺈن‬ ‫ﻓﺈن ‪ x + 2x − 2x − 1 = 0‬و ﺑﲈ ّ‬ ‫إذا ﻛﺎن ‪ّ a = −1‬‬ ‫•‬

‫‪ x + 2x − 2x − 1‬ﻳﻘﺒﻞ اﻟﻘﺴﻤﺔ ﻋﲆ ‪ x − 1‬و ﺑﺎﻟﻘﺴﻤﺔ اﻹﻗﻠﻴﺪﻳﺔ )أوأي ﻃﺮﻳﻘﺔ أﺧﺮى( ﻧﺠﺪ‬


‫‪. x + 2x − 2x − 1 = (x − 1) x + 3x + 1‬‬
‫‪−3 − √5‬‬
‫= ‪x‬‬ ‫اﳊﻠﻮل اﻷﺧﺮى ﻫﻲ ﺣﻠﻮل اﳌﻌﺎدﻟﺔ ‪ . x + 3x + 1 = 0‬ﻟﺪﻳﻨﺎ ‪ Δ = 3 − 4 × 1 × 1 = 5 :‬ﻣﻨﻪ‬
‫‪2‬‬
‫‪−3 + √5‬‬
‫= ‪.x‬‬ ‫و‬
‫‪2‬‬
‫أن ‪ x = 1‬ﺣﻞ ﳍﺬه اﳌﻌﺎدﻟﺔ ّ‬
‫ﻓﺈن‬ ‫ﻓﺈن ‪ x + 2x + 6x − 9 = 0‬و ﺑﲈ ّ‬ ‫إذا ﻛﺎن ‪ّ a = 3‬‬ ‫•‬

‫‪ x + 2x − 2x − 1‬ﻳﻘﺒﻞ اﻟﻘﺴﻤﺔ ﻋﲆ ‪ x − 1‬و ﺑﺎﻟﻘﺴﻤﺔ اﻹﻗﻠﻴﺪﻳﺔ )أوأي ﻃﺮﻳﻘﺔ أﺧﺮى( ﻧﺠﺪ‬


‫‪ņőƱŀ Ţ‬‬
‫‪. x + 2x − 2x − 1 = (x − 1) x + 3x + 9‬‬
‫‪ Δ = 3 − 4 × 1 × 9 = −27 = 3ı√3‬ﻣﻨﻪ‬ ‫اﳊﻠﻮل اﻷﺧﺮى ﻫﻲ ﺣﻠﻮل اﳌﻌﺎدﻟﺔ ‪ . x + 3x + 9‬ﻟﺪﻳﻨﺎ ‪:‬‬
‫‪−3 + 3ı√3‬‬ ‫‪−3 − 3ı√3‬‬
‫= ‪.x‬‬ ‫= ‪x‬و‬
‫‪2‬‬ ‫‪2‬‬
‫■‬

‫‪.‬‬
‫𝟏𝟖𝟏‬ ‫‪.‬‬
‫ﺣﻞ ﰲ ‪ ℂ‬اﳌﻌﺎدﻟﺔ ذات اﳌﺠﻬﻮل ‪: z‬‬
‫‪ƕ‬‬

‫‪VII‬‬

‫‪z (z − 3) (z + 2) (z + 5) = 54‬‬ ‫)𝟏(‬

‫ﺍﳊﻞّ‪ .‬ﻟﺪﻳﻨﺎ ‪:‬‬


‫‪ũŏ‬‬

‫‪(𝟏) ⟺ (z (z + 2)) ((z − 3) (z + 5)) = 54 ⟺ z + 2z z + 2z − 15 = 54‬‬


‫ﻧﻀﻊ ‪ Z = z + 2z‬ﻓﻴﻜﻮن ‪:‬‬
‫}‪(𝟏) ⟺ Z (Z − 15) = 54 ⟺ Z − 15Z − 54 = 0 ⟺ Z ∈ {−3, 18‬‬
‫إذن ‪ Z = −3‬أو ‪ Z = 18‬ﻣﻊ ‪ Z = z + 2z‬ﻣﻨﻪ ‪:‬‬

‫𝟲𝟲𝟮‬

‫‪http ://tinyurl.com/Malki1718‬‬ ‫‪0‬‬


‫‪ .VII‬ﲤﺎرﻳﻦ ﻟﻠﺘﻌﻤﻖ‬
‫‪.‬‬
‫‪ Δ = (−1) − 3 = −2 = ı√2‬ﻣﻨﻪ ‪ z = −1 − ı√2‬أو‬ ‫إذا ﻛﺎن ‪ Z = −3‬أي ‪ّ z + 2z + 3 = 0‬‬
‫ﻓﺈن‬ ‫•‬

‫‪Ŕž Ŧ Ľ‬‬
‫‪ z = −1 + ı√2‬؛‬
‫و إذا ﻛﺎن ‪ Z = 18‬أي ‪ّ z + 2z − 18 = 0‬‬
‫ﻓﺈن ‪ Δ = (−1) + 18 = 19‬ﻣﻨﻪ ‪ z = −1 − √19‬أو ‪.z = −1 + √19‬‬ ‫•‬

‫‪œǃ‬‬
‫■‬ ‫‪. 𝒮 = −1 − ı√2, −1 + ı√2, −1 − √19, −1 + √19‬‬ ‫ﰲ اﻷﺧﲑ‪ ،‬ﳎﻤﻮﻋﺔ ﺣﻠﻮل اﳌﻌﺎدﻟﺔ )𝟏( ﻫﻲ ‪:‬‬

‫‪.‬‬

‫‪Ŀ‬‬
‫‪.‬‬
‫𝟐𝟖𝟏‬
‫ّ‬
‫ﺣﻞ ﰲ ﳎﻤﻮﻋﺔ اﻷﻋﺪاد اﳌﺮﻛﺒﺔ اﳌﻌﺎدﻟﺔ ‪:‬‬
‫‪z − 4z + 6z − 6z + 6z − 12z + 8z + 4 = 0‬‬

‫‪ •1‬ﺑﺎﻟﺘﻌﻮﻳﺾ ﻋﻦ ‪ z‬ﺑﺎﻟﻘﻴﻤﺔ ‪ ، 2‬و ﻣﻦ َﺛ ّﻢ‬

‫‪ •2‬ﺑﺎﻟﺘﺤﻠﻴﻞ و ﺑﺎﺳﺘﺨﺪام ﻣﻔﻜﻮك )ﻧﴩ( اﳌﻘﺪار )‪ (z + a‬ﺣﻴﺚ ‪. a ∈ ℂ‬‬

‫ﺍﳊﻞّ‪ .‬ﻧﻀﻊ ‪. f(z) = z − 4z + 6z − 6z + 6z − 12z + 8z + 4‬‬


‫‪ •1‬إذا ﻛﺎن ‪ّ z = 2‬‬
‫ﻓﺈن ‪:‬‬
‫‪z = z‬‬ ‫‪⋅ z = 2 z = 4z‬‬ ‫‪ z = z‬و‬ ‫‪=2 =4‬‬ ‫‪، z = z ⋅ z = 2z ، z = z ⋅ z = 2z‬‬
‫و ﺑﺎﻟﺘﻌﻮﻳﺾ ﰲ اﳌﻌﺎدﻟﺔ ﻧﺠﺪ ‪:‬‬
‫‪4z − 16 + 12z − 12z + 12 − 12z + 8z + 4 = 0‬‬
‫و ﻫﻲ ُﳏﻘﻘﺔ ﺗﻄﺎ ُﺑﻘ ًﺎ و ﺑﺎﻟﺘﺎﱄ ّ‬
‫ﻓﺈن )‪ f(z‬ﻳﻘﺒﻞ اﻟﻘﺴﻤﺔ ﻋﲆ ‪. z − 2‬‬
‫ِ‬
‫اﳌﻌﺎﻣﻼت ﻣﺜﻼً‪ ،‬ﻳﻤﻜﻦ أن ﻧﻜﺘﺐ ‪:‬‬ ‫‪ •2‬ﺑﺎﻟﻘﺴﻤﺔ اﻹﻗﻠﻴﺪﻳﺔ أو ﺑﻄﺮﻳﻘﺔ ﻣﺴﺎواة‬
‫‪f(z) = z − 2‬‬ ‫‪az + bz + cz + dz + e‬‬
‫و ﺑﻌﺪ اﻟﻨﴩ و اﻟﱰﺗﻴﺐ ﻳﻨﺘﺞ ‪:‬‬
‫‪f(z) = az + bz + cz + (d − 2a)z + (e − 2b)z − 2cz − 2dz − 2e‬‬
‫‪ņőƱŀ Ţ‬‬
‫ﻣﻨﻪ ‪ −2d = 8 ، −2c = −12 ، e − 2b = 6 ، d − 2a = −6 ، c = 6 ، b = −4 ، a = 1‬و ‪ −2e = 4‬أي ‪،a = 1‬‬
‫‪ d = −4 ، c = 6 ، b = −4‬و ‪ e = −2‬و ﺑﺎﻟﺘﺎﱄ ‪:‬‬
‫‪f(z) = z − 2‬‬ ‫‪z − 4z + 6z − 4z − 2‬‬
‫ﺑﺎﺗﺒﺎع اﻹﻗﱰاح‪ ،‬ﻧﻨﴩ اﳌﻘﺪار )‪ (z + a‬و ﻧﺠﺪ ‪:‬‬
‫‪(z + a) = a + 4az + 6a z + 4a z + a‬‬
‫و ﺑﺎﳌﻘﺎرﻧﻪ ﻣﻊ اﻟﻌﺒﺎرة اﻷﺧﲑة ﻟﻜﺜﲑ اﳊﺪود )‪ ، f(z‬ﻧﺴﺘﻨﺘﺞ أﻧّﻪ ﻷﺟﻞ ‪ a = −1‬ﻳﻜﻮن ‪:‬‬
‫‪(z − 1) = z − 4z + 6z − 4z + 1 = z − 4z + 6z − 4z − 2 + 3‬‬
‫‪ƕ‬‬
‫‪VII‬‬

‫ﻣﻨﻪ‬
‫‪f(z) = z − 2‬‬ ‫‪z − 4z + 6z − 4z − 2 = z − 2‬‬ ‫‪(z − 1) − 3‬‬
‫و ﻣﻨﻪ ‪:‬‬
‫‪ũŏ‬‬

‫⟺ ‪f(z) = 0‬‬ ‫‪z −2‬‬ ‫‪(z − 1) − 3 = 0‬‬

‫‪⟺ z −2=0‬‬ ‫أو‬ ‫‪(z − 1) − 3 = 0‬‬


‫‪⟺ z =2‬‬ ‫أو‬ ‫‪(z − 1) = 3‬‬
‫‪⟺ z = √2e‬‬ ‫‪/‬‬ ‫‪, k = 0, 1, 2‬‬ ‫أو‬ ‫‪z − 1 = √3e‬‬ ‫‪/‬‬ ‫‪, k = 0, 1, 2, 3‬‬
‫‪⟺ z = √2e‬‬ ‫‪/‬‬ ‫‪, k = 0, 1, 2‬‬ ‫أو‬ ‫‪z = 1 + √3e‬‬ ‫‪/‬‬ ‫‪, k = 0, 1, 2, 3‬‬

‫𝟳𝟲𝟮‬

‫‪http ://tinyurl.com/Malki1718‬‬ ‫‪0‬‬


‫‪.‬‬
‫أي ّ‬
‫أن اﳊﻠﻮل اﻟﺴﺒﻌﺔ ﻟﻠﻤﻌﺎدﻟﺔ ‪ f(z) = 0‬ﻫﻲ ‪:‬‬
‫‪−1 ± ı√3‬‬

‫‪Ŕž Ŧ Ľ‬‬
‫‪√2 , √2‬‬ ‫‪, 1 ± √3 , 1 ± ı √3‬‬
‫‪2‬‬

‫أن اﳊﻠﻮل اﳌﺮﻛﺒﺔ )ﻏﲑ اﳊﻘﻴﻘﻴﺔ( ﺗُﺸﻜﱢﻞ أزواﺟ ًﺎ ﻣﱰاﻓﻘﺔ و ﻫﺬا ُﻣﺘﻮ ﱠﻗﻊ ّ‬
‫ﻧﻼﺣﻆ ّ‬

‫‪œǃ‬‬
‫ﻷن ﻣﻌﺎﻣﻼت اﳌﻌﺎدﻟﺔ ﺣﻘﻴﻘﻴﺔ‪.‬‬
‫■‬

‫‪#‬‬

‫‪Ŀ‬‬
‫‪.‬‬
‫𝟑𝟖𝟏‬ ‫‪.‬‬
‫‪.z +1 = 0 :‬‬ ‫‪ n‬ﻋﺪد ﻃﺒﻴﻌﻲ ﻏﲑ ﻣﻌﺪوم‪ .‬ﺣﻞ ﰲ ‪ ℂ‬اﳌﻌﺎدﻟﺔ‬

‫إذا ﻛﺎن ‪ّ n = 1‬‬


‫ﻓﺈن اﳌﻌﺎدﻟﺔ ﺗﺼﺒﺢ ‪ z + 1 = 0‬أي ‪. z = −1‬‬ ‫•‬ ‫ﺍﳊﻞّ‪.‬‬
‫ﻧﻔﺮض ّ‬
‫أن ‪ n ≥ 2‬ﻣﻨﻪ اﳌﻌﺎدﻟﺔ ﺗﺼﺒﺢ ‪. z = −1 = e‬‬ ‫•‬

‫ﻓﺈن اﳊﻠﻮل اﻷﺧﺮى ﺗُﺴﺘﻨﺘﺞ ﺑﴬب ﻫﺬا اﳊﻞ اﳋﺎص ﺑﺎﳉﺬور‬ ‫أن ‪ّ z = e /‬‬
‫ﺣﻞ ﳍﺬه اﳌﻌﺎدﻟﺔ و ﺑﺎﻟﺘﺎﱄ ّ‬ ‫ﻧﻼﺣﻆ ّ‬
‫اﻟﻨﻮﻧﻴﺔ ﻟﻠﻮﺣﺪة أي ﳎﻤﻮﻋﺔ اﳊﻠﻮل ﻫﻲ ‪:‬‬
‫‪S= z 𝜔 ,‬‬ ‫‪k = 0, 1, ⋯ , n − 1 = e‬‬ ‫‪/‬‬ ‫‪e‬‬ ‫‪/‬‬ ‫‪,‬‬ ‫‪k = 0, 1, ⋯ , n − 1‬‬
‫‪+‬‬
‫‪.S = e‬‬ ‫‪,‬‬ ‫‪k = 0, 1, 2, ⋯ , n − 1‬‬ ‫أﺧﲑ ًا ‪:‬‬
‫■‬

‫ﻟﻴﻜﻦ ‪ n ≥ 2‬ﻋﺪد ًا ﻃﺒﻴﻌﻴﺎ أﻛﱪ ﻣﻦ أو ﻳﺴﺎوي ‪ . 2‬ﻧﻌﺘﱪ اﳌﻌﺎدﻟﺔ ‪:‬‬


‫‪#‬‬ ‫‪.‬‬
‫𝟒𝟖𝟏‬ ‫‪.‬‬

‫‪(z − 1) − (z + 1) = 0‬‬ ‫)𝟐(‬


‫‪ •1‬أﺛﺒﺖ ّ‬
‫أن ﺣﻠﻮل اﳌﻌﺎدﻟﺔ )𝟐( ﲣﻴﻠﻴﺔ ﴏﻓﺔ‪.‬‬
‫ﻓﺈن ‪ّ −z‬‬
‫ﺣﻞ ﳍﺎ أﻳﻀ ًﺎ‪.‬‬ ‫‪ •2‬أﺛﺒﺖ أﻧّﻪ إذا ﻛﺎن ‪ z‬ﺣﻼ ﻟﻠﻤﻌﺎدﻟﺔ )𝟐( ّ‬
‫‪ņőƱŀ Ţ‬‬
‫‪ •3‬ﺣﻞ ﰲ ‪ ℂ‬اﳌﻌﺎدﻟﺔ )𝟐( ‪.‬‬

‫‪ •1‬ﻟﻴﻜﻦ ‪ . z ∈ ℂ‬ﻟﺘﻜﻦ ‪ A ، M‬و ‪ B‬اﻟﻨﻘﻂ اﻟﺘﻲ ﻟﻮاﺣﻘﻬﺎ ‪ 1 ، z‬و ‪ −1‬ﻋﲆ اﻟﱰﺗﻴﺐ‪ .‬ﻟﺪﻳﻨﺎ ‪:‬‬ ‫ﺍﳊﻞّ‪.‬‬
‫|| )‪ّ z ⟹ (z − 1) = (z + 1) ⟹ ||(z − 1) || = ||(z + 1‬‬
‫ﺣﻞ ﻟﻠﻤﻌﺎدﻟﺔ )𝟐(‬
‫||‪⟹ ||z − 1|| = ||z + 1|| ⟹ ||z − 1|| = ||z + 1‬‬
‫‪ M‬ﺗﻨﺘﻤﻲ إﱃ ﳏﻮر اﻟﻘﻄﻌﺔ اﳌﺴﺘﻘﻴﻤﺔ ]‪⟹ AM = BM ⟹ [AB‬‬
‫‪ƕ‬‬

‫‪VII‬‬

‫‪⟹ M ∈ (Oy) ⟹ z ∈ ıℝ‬‬


‫إذن ﺣﻠﻮل اﳌﻌﺎدﻟﺔ )𝟐( ﲣﻴﻠﻴﺔ ﴏﻓﺔ‪.‬‬
‫‪ũŏ‬‬

‫‪ •2‬ﻟﻴﻜﻦ ‪ . z ∈ ℂ‬ﻟﺪﻳﻨﺎ ‪:‬‬


‫)‪(−z − 1) − (−z + 1) = (−1‬‬ ‫)‪(z + 1) − (z − 1‬‬ ‫)‪= − (−1‬‬ ‫)‪(z − 1) − (z + 1‬‬
‫‪ّ z ⟺ (z − 1) − (z + 1) = 0‬‬
‫ﺣﻞ ﻟﻠﻤﻌﺎدﻟﺔ )𝟐(‬ ‫ﻣﻨﻪ ‪:‬‬
‫)‪⟺ − (−1‬‬ ‫)‪(z − 1) − (z + 1‬‬ ‫‪=0‬‬
‫‪⟺ (−z − 1) − (−z + 1) = 0‬‬
‫‪ّ −z‬‬
‫ﺣﻞ ﻟﻠﻤﻌﺎدﻟﺔ )𝟐( ⟺‬

‫𝟴𝟲𝟮‬

‫‪http ://tinyurl.com/Malki1718‬‬ ‫‪0‬‬


‫‪ .VII‬ﲤﺎرﻳﻦ ﻟﻠﺘﻌﻤﻖ‬
‫‪.‬‬
‫‪ •3‬ﻟﻴﻜﻦ ‪ . z ∈ ℂ‬ﻟﺪﻳﻨﺎ ‪:‬‬

‫‪Ŕž Ŧ Ľ‬‬
‫)‪ّ z ⟺ (z − 1) = (z + 1‬‬
‫ﺣﻞ ﻟﻠﻤﻌﺎدﻟﺔ )𝟐(‬
‫‪⟺ z + 1 = (z − 1) e‬‬ ‫‪/‬‬ ‫ﻣﻊ }‪k ∈ {0, 1, 2, ⋯ , n − 1‬‬

‫‪œǃ‬‬
‫‪⟺ z + 1 = (z − 1) e‬‬ ‫‪/‬‬ ‫ﻣﻊ }‪ ) k ∈ {1, 2, ⋯ , n − 1‬اﳌﻌﺎدﻟﺔ ﻣﺴﺘﺤﻴﻠﺔ ﻷﺟﻞ ‪(k = 0‬‬
‫‪e‬‬ ‫‪/‬‬ ‫‪−1‬‬
‫=‪⟺ z‬‬ ‫‪/‬‬
‫ﻣﻊ }‪k ∈ {1, 2, ⋯ , n − 1‬‬
‫‪e‬‬ ‫‪+1‬‬
‫‪/‬‬ ‫‪− e−‬‬ ‫‪/‬‬

‫‪Ŀ‬‬
‫‪e‬‬
‫=‪⟺ z‬‬ ‫ﻣﻊ }‪k ∈ {1, 2, ⋯ , n − 1‬‬
‫‪e /‬‬ ‫‪+ e−‬‬ ‫‪/‬‬

‫‪2ı sin‬‬
‫=‪⟺ z‬‬ ‫ﻣﻊ }‪k ∈ {1, 2, ⋯ , n − 1‬‬
‫‪2 cos‬‬
‫𝜋‪k‬‬
‫‪⟺ z = ı cotan‬‬ ‫ﻣﻊ }‪k ∈ {1, 2, ⋯ , n − 1‬‬
‫‪n‬‬
‫𝜋‪k‬‬
‫‪. 𝒮 = ı cotan‬‬ ‫‪,‬‬ ‫‪k = 1, 2, ⋯ , n − 1‬‬ ‫إذن ﳎﻤﻮﻋﺔ اﳊﻠﻮل ﻫﻲ ‪:‬‬
‫‪n‬‬
‫ﺣﻞ ﻟﻠﻤﻌﺎدﻟﺔ )𝟐( و ﻧﺤﺼﻞ ﻋﻠﻴﻪ ﻷﺟﻞ ‪) k = n‬إذا ﻛﺎن ‪ n‬زوﺟﻴ ًﺎ ّ‬
‫ﻓﺈن‬ ‫ﻓﺈن ‪ّ z = 0‬‬
‫إذا ﻛﺎن ‪ n‬زوﺟﻴ ًﺎ ّ‬ ‫ﻣﻼﺣﻈﺔ ‪⧏ : 38‬‬
‫‪2‬‬
‫‪n‬‬
‫‪.( ∈ ℕ‬‬
‫‪2‬‬
‫⧐‬
‫ﻼ ﻷﳖﺎ ﻣﺴﺘﻤﺮة و ﻣﺘﻨﺎﻗﺼﺔ ﲤﺎﻣ ًﺎ ﻋﲆ ﻫﺬا‬
‫ﻇﻞ اﻟﺘﲈم ‪ cotan‬ﻣﺘﺒﺎﻳﻨﺔ ﻋﲆ اﳌﺠﺎل [𝜋 ‪) ]0,‬ﻣﺜ ً‬ ‫ﻣﻼﺣﻈﺔ ‪ ⧏ : 39‬ﺑﲈ ّ‬
‫أن داﻟﺔ ّ‬
‫أن اﳌﻌﺎدﻟﺔ )𝟐( ﺗﻘﺒﻞ ‪ n − 1‬ﺣﻼ ﻓﻘﻂ ‪ .‬ﻫﻞ ﻫﺬا ﺗﻨﺎﻗﺾ ؟ اﳉﻮاب ‪ :‬ﻻ‪،‬‬ ‫ﻓﺈن ﻫﺬه اﳊﻠﻮل ﳐﺘﻠﻔﺔ ﻣﺜﻨﻰ ﻣﺜﻨﻰ ﺑﻤﻌﻨﻰ ّ‬
‫اﳌﺠﺎل( ّ‬
‫⧐‬ ‫ﻷن درﺟﺔ ﻛﺜﲑ اﳊﺪود )‪ P (X) = (X − 1) − (X + 1‬ﻫﻲ ‪ n − 1‬و ﻟﻴﺲ ‪) n‬اﳊﺪ ‪ X‬ﳜﺘﻔﻲ ﺑﻌﺪ اﻟﻨﴩ( ‪.‬‬ ‫ّ‬
‫■‬

‫‪.‬‬
‫‪#‬‬
‫𝟓𝟖𝟏‬ ‫‪.‬‬
‫‪ņőƱŀ Ţ‬‬
‫‪ n‬ﻋﺪد ﻃﺒﻴﻌﻲ ﺣﻴﺚ ‪ . n ≥ 2‬ﺣﻞ ﰲ ‪ ℂ‬اﳌﻌﺎدﻟﺔ ‪. (z + ı) = (z − ı) :‬‬
‫ﻻﺣﻆ ّ‬
‫أن ﻫﺬه اﳌﻌﺎدﻟﺔ ﺗﻘﺒﻞ ‪ n − 1‬ﺣﻼّ‪ ،‬ﻛﻠﻬﺎ ﺣﻘﻴﻘﻴﺔ‪.‬‬

‫ﺍﳊﻞّ‪ .‬ﺑﺪاﻳ ًﺔ‪ ،‬ﻧﻼﺣﻆ ّ‬


‫أن ‪ z = ı‬ﻟﻴﺲ ﺣ ّ‬
‫ﻼ ﳍﺬه اﳌﻌﺎدﻟﺔ ﻣﻨﻪ‬
‫‪z+ı‬‬
‫)‪(z + ı) = (z − ı‬‬ ‫⟺‬ ‫‪=1‬‬
‫‪z−ı‬‬
‫‪Z+1‬‬ ‫‪z+ı‬‬
‫‪ z = ı‬ﻓﻴﻜﻮن ‪:‬‬ ‫= ‪ Z‬أي‬ ‫ﻧﻀﻊ‬
‫‪Z−1‬‬ ‫‪z−ı‬‬
‫‪ƕ‬‬

‫)‪(z + ı) = (z − ı‬‬ ‫‪⟺ Z =1 ⟺ Z=𝜔 ,‬‬ ‫‪k = 0, ⋯ , n − 1‬‬


‫‪VII‬‬

‫‪ 𝜔 = e‬ﻫﻲ اﳉﺬور اﻟﻨﻮﻧﻴﺔ ﻟﻠﻮﺣﺪة‪.‬‬ ‫‪/‬‬ ‫ﺣﻴﺚ‬

‫ﻣﻦ أﺟﻞ ‪ 𝜔 = 1 ، k = 0‬و اﳌﻌﺎدﻟﺔ ‪ z + ı = 1‬ﻻ ّ‬


‫ﺣﻞ ﳍﺎ‪.‬‬ ‫•‬
‫‪ũŏ‬‬

‫‪z−ı‬‬
‫‪𝜔 +1‬‬
‫‪.z = ı‬‬ ‫ﻣﻦ أﺟﻞ }‪ 𝜔 ≠ 1 ، k ∈ {1, 2, ⋯ , n − 1‬و اﳌﻌﺎدﻟﺔ 𝜔 = ‪ z + ı‬ﺗﻘﺒﻞ ّ‬
‫اﳊﻞ ‪:‬‬ ‫•‬
‫‪𝜔 −1‬‬ ‫‪z−ı‬‬

‫أي )أﻧﻈﺮ اﻟﺘﻤﺮﻳﻦ ‪ 59‬ﺻﻔﺤﺔ ‪: (126‬‬


‫‪𝜔 +1‬‬ ‫‪2 cos‬‬ ‫𝜋‪k‬‬
‫‪z =ı‬‬ ‫‪=ı‬‬ ‫‪= cotan‬‬ ‫‪∈ ℝ , k = 1, 2, ⋯ , n − 1‬‬
‫‪𝜔 −1‬‬ ‫‪2ı sin‬‬ ‫‪n‬‬

‫𝟵𝟲𝟮‬

‫‪http ://tinyurl.com/Malki1718‬‬ ‫‪0‬‬


‫‪.‬‬
‫أن ﻫﺬه اﻷﻋﺪاد ﺣﻠﻮل ﻟﻠﻤﻌﺎدﻟﺔ اﳌﻌﻄﺎة‪.‬‬ ‫ﺑﺎﻟﻌﻜﺲ‪ ،‬ﺑﺎﺗﺒﺎع ﻧﻔﺲ اﳊﺴﺎب ﻧﺘﺤﻘﻖ ﺑﺴﻬﻮﻟﺔ ّ‬
‫‪. S = cotan , k = 1, 2, ⋯ , n − 1‬‬ ‫إذن‪ ،‬ﳎﻤﻮﻋﺔ اﳊﻠﻮل ﻫﻲ ‪:‬‬

‫‪Ŕž Ŧ Ľ‬‬
‫ﻼ ﻷﳖﺎ ﻣﺴﺘﻤﺮة و ﻣﺘﻨﺎﻗﺼﺔ ﲤﺎﻣ ًﺎ ﻋﲆ ﻫﺬا اﳌﺠﺎل( ّ‬
‫ﻓﺈن ﻫﺬه اﳊﻠﻮل‬ ‫أن داﻟﺔ ّ‬
‫ﻇﻞ اﻟﺘﲈم ‪ cotan‬ﻣﺘﺒﺎﻳﻨﺔ ﻋﲆ اﳌﺠﺎل [𝜋 ‪) ]0,‬ﻣﺜ ً‬ ‫و ﺑﲈ ّ‬
‫أن ﻋﺪد اﳊﻠﻮل ﻫﻮ ‪ n − 1‬و ﻫﻲ ﻛﻠﻬﺎ ﺣﻘﻴﻘﻴﺔ‪.‬‬ ‫ﳐﺘﻠﻔﺔ ﻣﺜﻨﻰ ﻣﺜﻨﻰ ﺑﻤﻌﻨﻰ ّ‬

‫‪œǃ‬‬
‫ﻣﻼﺣﻈﺔ ‪ ⧏ : 40‬ﻧﺴﻤﻲ )‪ (E‬ﻫﺬه اﳌﻌﺎدﻟﺔ‪ .‬ﻟﺪﻳﻨﺎ ‪:‬‬

‫)‪ّ z ⟺ (z + ı) = (z − ı‬‬
‫ﺣﻞ ﻟﻠﻤﻌﺎدﻟﺔ )‪(E‬‬

‫‪Ŀ‬‬
‫)‪⟺ (−ı) (ız − 1) = (−ı) (ız + 1‬‬
‫)‪⟺ (ız − 1) = (ız + 1‬‬
‫⟺‬ ‫‪ّ ız‬‬
‫ﺣﻞ ﻟﻠﻤﻌﺎدﻟﺔ )𝟐( )ﲤﺮﻳﻦ ‪ 184‬ﺻﻔﺤﺔ ‪(268‬‬

‫⧐‬
‫و ﺑﺎﻟﺘﺎﱄ ﻳﻤﻜﻦ إﺳﺘﻨﺘﺎج ﺣﻠﻮل اﳌﻌﺎدﻟﺔ )‪ (E‬ﻣﻦ اﻟﺘﻤﺮﻳﻦ ‪. 184‬‬
‫■‬

‫)‪cos (kx‬‬
‫!‬ ‫‪.‬‬
‫𝟔𝟖𝟏‬ ‫‪.‬‬
‫‪.‬‬ ‫‪=0‬‬ ‫ﻟﻴﻜﻦ ‪ n ∈ ℕ‬ﻋﺪد ًا ﻃﺒﻴﻌﻴﺎ‪ .‬ﺣﻞ ﰲ ‪ ℝ‬اﳌﻌﺎدﻟﺔ اﻟﺘﺎﻟﻴﺔ ‪:‬‬
‫=‬
‫‪cos x‬‬

‫ﺍﳊﻞّ‪ .‬ﻟﻠﻤﻌﺎدﻟﺔ ﻣﻌﻨﻰ إذا و ﻓﻘﻂ إذا ﻛﺎن ‪ ∀k ∈ {0, 1, 2, ⋯ , n} , cos x ≠ 0 :‬أي إذا و ﻓﻘﻂ إذا ﻛﺎن ‪:‬‬
‫𝜋‬
‫≠ ‪.x‬‬ ‫)𝜋 ‪(mod‬‬
‫‪2‬‬
‫𝜋‬
‫≠ ‪ . x‬ﻟﺪﻳﻨﺎ ‪:‬‬ ‫ﻓﻴﲈ ﻳﲇ ﻧﻔﺮض إذن ّ‬
‫أن )𝜋 ‪(mod‬‬
‫‪2‬‬

‫)‪cos (kx‬‬ ‫‪Re e‬‬


‫⟺ ‪=0‬‬ ‫‪=0‬‬
‫=‬
‫‪cos x‬‬ ‫=‬
‫‪cos x‬‬
‫‪ņőƱŀ Ţ‬‬
‫‪e‬‬
‫⟺‬ ‫‪Re‬‬ ‫‪=0‬‬
‫=‬
‫‪cos x‬‬

‫‪e‬‬
‫‪⟺ Re‬‬ ‫‪=0‬‬
‫=‬
‫‪cos x‬‬

‫‪e‬‬
‫أي إذا ﻛﺎن ‪ sin x = 0‬أي )𝜋 ‪ّ x = 0 (mod‬‬
‫ﻓﺈن ‪:‬‬ ‫إذا ﻛﺎن ‪= 1‬‬ ‫•‬
‫‪cos x‬‬
‫)‪cos (kx‬‬
‫=‬ ‫‪1=n+1‬‬
‫‪ƕ‬‬

‫=‬
‫‪cos x‬‬ ‫=‬
‫‪VII‬‬

‫و اﳌﻌﺎدﻟﺔ اﻟﺴﺎﺑﻘﺔ ﻻ ّ‬
‫ﺣﻞ ﳍﺎ‪.‬‬
‫‪ũŏ‬‬

‫𝟬𝟳𝟮‬

‫‪http ://tinyurl.com/Malki1718‬‬ ‫‪0‬‬


‫ ﲤﺎرﻳﻦ ﻟﻠﺘﻌﻤﻖ‬.VII
.
e
ّ x ≠ 0 (mod 𝜋) ‫ أي‬sin x ≠ 0 ‫أي إذا ﻛﺎن‬
: ‫ﻓﺈن‬ ≠ 1 ‫إذا ﻛﺎن‬ •
cos x

Ŕž Ŧ Ľ
+
cos (kx) 1−
= 0 ⟺ Re ⎛ ⎞=0
cos
⎜ ⎟

œǃ
=
cos x 1 − cos
⎝ ⎠
1 cos + x−e +
⟺ Re =0
cos x −ı sin x

Ŀ
1 cos + x − cos ((n + 1)x) + ı sin ((n + 1)x)
⟺ =0
cos x −ı sin x
ı cos + x − cos ((n + 1)x) − sin ((n + 1)x)
1
⟺ =0
cos x sin x
⟺ sin ((n + 1)x) = 0
⟺ (n + 1)x = 0 (mod 𝜋)
𝜋
⟺ x = 0 mod
n+1

𝜋 k𝜋
.x ≠ (mod 𝜋) ‫ و‬x ≠ 0 (mod 𝜋) ‫ و‬k ∈ ℤ ‫ ﻣﻊ‬x = ّ ‫أي‬
‫أن‬
2 n+1
k𝜋
■ .S = , k∈ℤ ‫( و‬n + 1) ∤ k ‫ و‬n + 1 ≠ ±2k : ‫إذن ﳎﻤﻮﻋﺔ اﳊﻠﻮل ﻫﻲ‬
n+1

: ‫ اﳌﻌﺎدﻟﺘﲔ‬ℂ ‫ ﺣﻞ ﰲ‬. n ∈ ℕ∗ ‫𝛼 و‬, 𝜃 ∈ ℝ ‫ﻟﻴﻜﻦ‬


! .
𝟏𝟖𝟕 .

n n
sin (𝜃 + k𝛼) z = 0 • 2 cos (𝜃 + k𝛼) z = 0 1

=
k =
k
ņőƱŀ Ţ
‫ ﻧﻀﻊ‬.ّ‫ﺍﳊﻞ‬
n n
S = sin (𝜃 + k𝛼) z ‫و‬C = cos (𝜃 + k𝛼) z
=
k =
k

e + e−
: ‫ ﻣﻨﻪ‬cos x = ّ x ∈ ℝ ‫• ﻧُﺬﻛﱢﺮ أﻧﻪ ﻟﻜﻞ‬1
‫ﻓﺈن‬
2
n n e + + e− +
C = cos (𝜃 + k𝛼) z = z
=
k =
k 2
1 n 1 n −
ƕ

= e + z + e + z
VII

2 =
k 2 =
k
1 n 1 n −
= e e z + e− e z
2 =
k 2 =
k
ũŏ

1 n 1 n
= e (e ) z + e− (e− ) z
2 =
k 2 =
k
1 n 1 n
= e (ze ) + e− (ze− )
2 =
k 2 =
k
1 1
= e (1 + ze ) + e (1 + ze− )
2 2

𝟮𝟳𝟭

http ://tinyurl.com/Malki1718 0
‫‪.‬‬
‫و ﺑﺎﻟﺘﺎﱄ ّ‬
‫ﻓﺈن ‪:‬‬

‫‪Ŕž Ŧ Ľ‬‬
‫‪1‬‬ ‫‪1‬‬
‫⟺ ‪C =0‬‬ ‫‪e (1 + ze ) + e− (1 + ze− ) = 0‬‬
‫‪2‬‬ ‫‪2‬‬
‫‪⟺ e (1 + ze ) + e− (1 + ze− ) = 0‬‬

‫‪œǃ‬‬
‫‪⟺ e (1 + ze ) + e− (1 + ze− ) = 0‬‬
‫) ‪⟺ e (1 + ze ) = −e− (1 + ze−‬‬

‫‪Ŀ‬‬
‫و ﻫﻨﺎ ﻧُﻤ ﱢﻴﺰ ﺛﻼث ﺣﺎﻻت ‪:‬‬
‫ﻓﺈن ‪ e = e− = 1‬ﻣﻨﻪ ‪:‬‬
‫إذا ﻛﺎن )𝜋‪ّ 𝛼 = 0 (mod 2‬‬ ‫•‬
‫= )‪C = 0 ⟺ e (1 + z‬‬ ‫‪−e−‬‬ ‫)‪(1 + z‬‬ ‫‪⟺ (1 + z) = 0 ⟺ 1+z = 0 ⟺ z = −1‬‬
‫إذن ﻟﻠﻤﻌﺎدﻟﺔ ّ‬
‫ﺣﻞ و ﺣﻴﺪ و ﻫﻮ ‪. z = −1‬‬
‫ﻣﻼﺣﻈﺔ ‪ ⧏ : 41‬ﺑﺎﻟﺘﻌﻮﻳﺾ ﻋﻦ ‪ 𝛼 = 0‬ﰲ اﳌﻌﺎدﻟﺔ‪ ،‬و ﺑﲈ ّ‬
‫أن ‪ z = −1‬ﺣﻞ ﳍﺎ‪ ،‬ﻧﺴﺘﻨﺘﺞ ّ‬
‫أن ‪:‬‬
‫‪n‬‬
‫‪(−1) = 0‬‬
‫=‬
‫‪k‬‬

‫‪n‬‬
‫= )‪. 0 = (1 − 1‬‬ ‫)‪(−1‬‬ ‫و ﻫﻲ ﻧﺘﻴﺠﺔ ﻣﻌﺮوﻓﺔ ﻳﻤﻜﻦ اﳊﺼﻮل ﻋﻠﻴﻬﺎ أﻳﻀ ًﺎ ﺑﻤﻼﺣﻈﺔ ّ‬
‫أن ‪:‬‬
‫‪k‬‬
‫⧐‬ ‫=‬

‫ﻓﺈن ‪ e = e− = −1‬ﻣﻨﻪ ‪:‬‬


‫إذا ﻛﺎن )𝜋‪ّ 𝛼 = 𝜋 (mod 2‬‬ ‫•‬
‫= )‪C = 0 ⟺ e (1 − z‬‬ ‫‪−e−‬‬ ‫)‪(1 − z‬‬ ‫‪⟺ (1 − z) = 0 ⟺ 1−z = 0 ⟺ z = 1‬‬
‫إذن ﻟﻠﻤﻌﺎدﻟﺔ ّ‬
‫ﺣﻞ و ﺣﻴﺪ و ﻫﻮ ‪. z = 1‬‬
‫إذا ﻛﺎن )𝜋‪ 𝛼 ≠ 0 (mod 2‬و )𝜋‪) 𝛼 ≠ 𝜋 (mod 2‬أي إذا ﻛﺎن )𝜋 ‪ّ (𝛼 ≠ 0 (mod‬‬
‫ﻓﺈن ‪:‬‬ ‫•‬

‫) ‪C = 0 ⟺ e (1 + ze ) = −e− (1 + ze−‬‬
‫‪1 + ze‬‬ ‫‪1 + ze‬‬
‫⟺‬ ‫‪= −e‬‬ ‫⟺‬ ‫‪= e−‬‬ ‫‪−‬‬
‫‪ņőƱŀ Ţ‬‬
‫‪1 + ze−‬‬ ‫‪1 + ze−‬‬
‫‪1 + ze‬‬ ‫|‬ ‫‪−‬‬ ‫‪−‬‬ ‫‪+‬‬
‫⟺‬ ‫‪−‬‬
‫‪=z‬‬ ‫‪|| z‬‬ ‫‪=e‬‬ ‫‪,‬‬ ‫}‪k ∈ {0, 1, 2, ⋯ , n − 1‬‬
‫‪1 + ze‬‬
‫‪z −1‬‬ ‫|‬ ‫‪−‬‬ ‫‪−‬‬ ‫‪+‬‬
‫=‪⟺ z‬‬ ‫‪−‬‬ ‫||‬ ‫‪z =e‬‬ ‫‪,‬‬ ‫}‪k ∈ {0, 1, 2, ⋯ , n − 1‬‬
‫‪e −z e‬‬
‫‪z −1‬‬ ‫|‬ ‫‪−‬‬ ‫‪+‬‬ ‫‪+‬‬
‫‪⟺ z=e‬‬ ‫||‬ ‫‪z =e‬‬ ‫‪,‬‬ ‫}‪k ∈ {0, 1, 2, ⋯ , n − 1‬‬
‫‪e‬‬ ‫‪−z‬‬

‫و ﰲ ﻫﺬه اﳊﺎﻟﺔ‪ ،‬ﻟﻠﻤﻌﺎدﻟﺔ ‪ n‬ﺣﻼ‪.‬‬


‫‪ƕ‬‬

‫‪− e−‬‬
‫‪VII‬‬

‫‪e‬‬
‫= ‪ (sin x‬ﻟﻜﻦ ﻳﻤﻜﻦ إﺳﺘﻌﲈل اﻟﻨﺘﻴﺠﺔ اﻟﺴﺎﺑﻘﺔ‬ ‫ﳊﻞ اﳌﻌﺎدﻟﺔ اﻟﺜﺎﻧﻴﺔ‪ ،‬ﻳﻤﻜﻦ إﺗﺒﺎع ﻧﻔﺲ اﻟﻄﺮﻳﻘﺔ )ﻧُﺬﻛﱢﺮ ّ‬
‫أن‬ ‫•‬ ‫‪2‬‬
‫‪2ı‬‬
‫𝜋‬ ‫𝜋‬
‫أن ‪ sin (𝜃 + k𝛼) = cos − 𝜃 − k𝛼 :‬أي ﺑﺎﻟﺘﻌﻮﻳﺾ ‪ 𝜃 = 𝜃 −‬و 𝛼‪ . 𝛼 = −‬إذن ‪:‬‬ ‫ﺑﻌﺪ ﻣﻼﺣﻈﺔ ّ‬
‫‪2‬‬ ‫‪2‬‬
‫‪ũŏ‬‬

‫ﻟﻠﻤﻌﺎدﻟﺔ ّ‬
‫ﺣﻞ وﺣﻴﺪ و ﻫﻮ ‪ z = −1‬إذا ﻛﺎن )𝜋‪ −𝛼 = 0 (mod 2‬أي إذا ﻛﺎن )𝜋‪. 𝛼 = 0 (mod 2‬‬ ‫•‬

‫ﺣﻞ وﺣﻴﺪ و ﻫﻮ ‪ z = 1‬إذا ﻛﺎن )𝜋‪ −𝛼 = 𝜋 (mod 2‬أي إذا ﻛﺎن )𝜋‪) 𝛼 = −𝜋 (mod 2‬أي إذا‬‫ﻟﻠﻤﻌﺎدﻟﺔ ّ‬ ‫•‬
‫ﻛﺎن )𝜋‪. (𝛼 = 𝜋 (mod 2‬‬
‫أ ّﻣﺎ إذا ﻛﺎن )𝜋 ‪ −𝛼 ≠ 0 (mod‬أي إذا ﻛﺎن )𝜋 ‪ّ 𝛼 ≠ 0 (mod‬‬
‫ﻓﺈن اﳌﻌﺎدﻟﺔ ﺗﻘﺒﻞ ‪ n‬ﺣﻼ و ﻫﻲ ‪:‬‬ ‫•‬
‫‪z −1‬‬ ‫|‬ ‫‪−‬‬ ‫‪−‬‬ ‫‪+‬‬ ‫‪+‬‬ ‫‪+‬‬
‫‪z = e−‬‬ ‫||‬ ‫‪z =e‬‬ ‫‪=e‬‬ ‫‪,‬‬ ‫}‪k ∈ {0, 1, 2, ⋯ , n − 1‬‬
‫‪e−‬‬ ‫‪−z‬‬

‫𝟮𝟳𝟮‬

‫‪http ://tinyurl.com/Malki1718‬‬ ‫‪0‬‬


‫‪ .VII‬ﲤﺎرﻳﻦ ﻟﻠﺘﻌﻤﻖ‬
‫‪.‬‬
‫■‬

‫‪Ŕž Ŧ Ľ‬‬
‫‪ 1 + z + z + ⋯ + z‬ﳍﺎ ﻃﻮﻳﻠﺔ أﺻﻐﺮ ﻣﻦ أو ﺗﺴﺎوي ‪. 1‬‬ ‫‪−‬‬ ‫‪− nz = 0‬‬ ‫أﺛﺒﺖ ّ‬
‫أن ﺣﻠﻮل اﳌﻌﺎدﻟﺔ‬
‫!‬ ‫‪.‬‬
‫𝟖𝟖𝟏‬ ‫‪.‬‬

‫‪œǃ‬‬
‫ﺍﳊﻞّ‪ .‬ﻟﻴﻜﻦ ‪ . z ∈ ℂ‬ﻧﻔﺮض ّ‬
‫أن ‪ . |z| > 1‬إذن‪ ،‬ﺣﺴﺐ اﳌﺘﺒﺎﻳﻨﺔ اﳌﺜﻠﺜﻴﺔ ‪:‬‬

‫‪Ŀ‬‬
‫‪||1 + z + z + ⋯ + z‬‬ ‫‪−‬‬ ‫|‪|| ≤ 1 + |z| + ||z || + ⋯ + |z‬‬ ‫‪−‬‬
‫|| ‪< |z| + |z| + ⋯ + |z| = n |z| = ||nz‬‬

‫‪.1+z+z +⋯+z‬‬ ‫‪−‬‬


‫أي || ‪ ||1 + z + z + ⋯ + z − || < ||nz‬و ﺑﺎﻟﺘﺎﱄ‬
‫‪≠ nz‬‬
‫ﻓﺈن ‪ z‬ﻟﻴﺲ ﺣﻼ ﻟﻠﻤﻌﺎدﻟﺔ ‪. 1 + z + z + ⋯ + z − = nz‬‬ ‫ﺑﺮﻫﻨّﺎ إذن أﻧﻪ إذا ﻛﺎن ‪ّ |z| > 1‬‬
‫■‬ ‫ﺑﺄﺧﺬ اﻟﻌﻜﺲ اﻟﻨﻘﻴﺾ‪ ،‬ﻧﺴﺘﻨﺘﺞ أﻧﻪ إذا ﻛﺎن ‪ z‬ﺣﻼ ﻟﻠﻤﻌﺎدﻟﺔ ‪ّ 1 + z + z + ⋯ + z − = nz‬‬
‫ﻓﺈن ‪. |z| ≤ 1‬‬

‫‪.‬‬
‫𝟗𝟖𝟏‬ ‫‪.‬‬
‫| ‪a−b‬‬
‫||| ‪.‬‬ ‫ﻟﻴﻜﻦ ‪ a‬و ‪ b‬ﻋﺪدﻳﻦ ﻣﺮﻛﺒﲔ ﺣﻴﺚ ‪ |a| < 1‬و ‪ . ||b|| < 1‬أﺛﺒﺖ ّ‬
‫أن ‪< 1‬‬
‫|| ‪1 − ab‬‬
‫أﻧﻈﺮ اﻟﺘﻤﺮﻳﻦ ‪ 190‬ﺻﻔﺤﺔ ‪273‬‬ ‫‬
‫ﺍﳊﻞّ‪ .‬إذا ﻛﺎن ‪ |a| < 1‬و ‪ّ ||b|| < 1‬‬
‫ﻓﺈن ‪ ||ab|| = |a| ⋅ ||b|| < 1‬ﻣﻨﻪ ‪ ||ab|| = ||a|| ⋅ ||b|| = |a| ⋅ ||b|| < 1‬و ﻫﺬا ﻳﺴﺘﻠﺰم ّ‬
‫أن ‪ab ≠ 1‬‬
‫‪a−b‬‬
‫ﻌﺮف‪.‬‬‫و ﺑﺎﻟﺘﺎﱄ ﻓﺎﳌﻘﺪار ‪ُ 1 − ab‬ﻣ ﱠ‬
‫ﻟﻴﻜﻦ ‪ a‬و ‪ b‬ﻋﺪدﻳﻦ ﻣﺮﻛﺒﲔ ﺣﻴﺚ ‪ |a| < 1‬و ‪ . ||b|| < 1‬ﻟﺪﻳﻨﺎ ‪:‬‬
‫| ‪| a−b‬‬
‫||‪|| 1 − ab || < 1 ⟺ ||a − b|| < ||1 − ab|| ⟺ ||a − b|| < ||1 − ab‬‬
‫‪⟺ a − b (a − b) < (1 − ab) 1 − ab‬‬
‫‪ņőƱŀ Ţ‬‬
‫‪⟺ aa − ab − ba + bb < 1 − ab − ba + aabb‬‬
‫‪⟺ 1 − |a| − ||b|| + |a| ||b|| > 0‬‬
‫⟺‬ ‫|‪1 − |a‬‬ ‫||‪1 − ||b‬‬ ‫‪>0‬‬

‫ﻷن ‪ |a| < 1‬و ‪. ||b|| < 1‬‬‫و اﳌﱰاﺟﺤﺔ اﻷﺧﲑة ﺻﺤﻴﺤﺔ ّ‬
‫| ‪a−b‬‬
‫■‬ ‫|| ‪.‬‬
‫| ‪| 1 − ab‬‬ ‫‪|<1‬‬ ‫ﻧﺴﺘﻨﺘﺞ ّ‬
‫أن ‪:‬‬
‫‪ƕ‬‬

‫‪.‬‬
‫‪VII‬‬

‫‪.‬‬
‫𝟎𝟗𝟏‬
‫| ‪z−a‬‬
‫|||‪.‬‬ ‫‪≤1‬‬ ‫‪ a‬ﻋﺪد ﻣﺮﻛﺐ ﺑﺤﻴﺚ ‪ . |a| < 1‬ﺣﺪّ د ﳎﻤﻮﻋﺔ اﻷﻋﺪاد اﳌﺮﻛﺒﺔ ‪ z‬اﻟﺘﻲ ُﲢﻘﻖ ‪:‬‬
‫|| ‪1 − az‬‬
‫أﻧﻈﺮ اﻟﺘﻤﺮﻳﻦ ‪ 189‬ﺻﻔﺤﺔ ‪273‬‬ ‫‬
‫‪ũŏ‬‬

‫ﺍﳊﻞّ‪ .‬اﳌﻘﺪار‬
‫‪1‬‬ ‫‪z−a‬‬
‫≠ ‪ z‬و ﰲ ﻫﺬه اﳊﺎﻟﺔ ‪:‬‬‫ﻌﺮف إذا و ﻓﻘﻂ إذا ﻛﺎن‬
‫ّ‬ ‫ُﻣ‬
‫‪a‬‬ ‫‪1 − az‬‬
‫| ‪| z−a‬‬ ‫|‬ ‫|‬
‫|‪|| 1 − az || ≤ 1 ⟺ |z − a| ≤ |1 − az‬‬

‫𝟯𝟳𝟮‬

‫‪http ://tinyurl.com/Malki1718‬‬ ‫‪0‬‬


‫‪.‬‬
‫ﻓﺈن )ﻧُﺬﻛّﺮ ّ‬
‫أن‬ ‫و ﺑﲈ أﻧﻪ ﻟﻜﻞ ﻋﺪدﻳﻦ ﻣﺮﻛﺒﲔ ‪ u‬و ‪ v‬ﻳﻜﻮن |‪) |u + v| = |u| + 2 Re (uv) + |v‬ﲤﺮﻳﻦ ‪ 248‬ﺻﻔﺤﺔ ‪ّ (348‬‬
‫|‪: (||a|| = |a‬‬

‫‪Ŕž Ŧ Ľ‬‬
‫| ‪| z−a‬‬
‫||‪|| 1 − az || ≤ 1 ⟺ |z − a| ≤ ||1 − az‬‬

‫‪œǃ‬‬
‫||‪⟺ |z| + 2ℜ (−a)z + |−a| ≤ ||1|| + 2ℜ 1(−az) + ||−az‬‬

‫|‪⟺ |z| + |a| ≤ 1 + ||a|| ⋅ |z‬‬


‫‪⟺ |z| + |a| − 1 − ||a|| ⋅ |z| ≤ 0‬‬

‫‪Ŀ‬‬
‫⟺‬ ‫‪|a| − 1‬‬ ‫‪|z| − 1 ≥ 0‬‬

‫‪⟺ |z| − 1 ≤ 0‬‬ ‫) ّ‬


‫ﻷن ‪(|a| < 1‬‬
‫‪⟺ |z| ≤ 1‬‬

‫■‬ ‫إذن ‪ :‬اﳌﺠﻤﻮﻋﺔ اﻟﺘﻲ ﻧﺒﺤﺚ ﻋﻨﻬﺎ ﻫﻲ ﳎﻤﻮﻋﺔ اﻷﻋﺪاد اﳌﺮﻛﺒﺔ اﻟﺘﻲ ﻃﻮﻳﻠﺘﻬﺎ أﺻﻐﺮ ﻣﻦ أو ﺗﺴﺎوي ‪. 1‬‬

‫ً‬
‫ﻟﻴﻜﻦ ‪ n‬ﻋﺪدا ﻃﺒﻴﻌﻴﺎ ﺣﻴﺚ ‪. n ≥ 1‬‬
‫‬ ‫‪.‬‬
‫𝟏𝟗𝟏‬ ‫‪.‬‬

‫) ‪ (v‬ﻛﲈ ﻳﲇ ‪:‬‬ ‫≥‬


‫ُﻌﺮف اﳌﺘﺘﺎﻟﻴﺎت ≥ ) ‪ (u ) ≥ ، (b ) ≥ ، (a‬و‬‫ﻧ ّ‬

‫‪a = 1 + nı‬‬ ‫‪,‬‬ ‫‪b = 1 − nı‬‬


‫‪1 + n (n + 1) + ı‬‬
‫= ‪u‬‬ ‫‪,‬‬ ‫= ‪v‬‬ ‫‪u =u ×u ×⋯×u‬‬
‫‪1 + n (n + 1) − ı‬‬ ‫=‬

‫‪a + b‬‬
‫‪. ∀n ≥ 1 ,‬‬ ‫= ‪u‬‬ ‫‪ •1‬أﺛﺒﺖ ّ‬
‫أن ‪:‬‬
‫‪a b +‬‬
‫‪1 − ı 1 + (n + 1) ı‬‬
‫‪ņőƱŀ Ţ‬‬
‫‪. ∀n ≥ 1 ,‬‬ ‫= ‪v‬‬ ‫⋅‬ ‫‪ •2‬إﺳﺘﻨﺘﺞ ّ‬
‫أن ‪:‬‬
‫‪1 + ı 1 − (n + 1) ı‬‬

‫‪ •3‬أﻛﺘﺐ ‪ v‬ﻋﲆ اﻟﺸﻜﻞ اﳉﱪي ‪.‬‬


‫‪ •4‬ﻟﻴﻜﻦ 𝜃 ﻋﻤﺪة ﻟﻠﻌﺪد ‪ . (0 ≤ 𝜃 < 2𝜋) v‬إﺳﺘﻨﺘﺞ ﳑﺎ ﺳﺒﻖ ﻗﻴﻤﺔ ﻛﻞ ﻣﻦ 𝜃 ‪ cos‬و 𝜃 ‪. sin‬‬

‫‪. lim‬‬ ‫‪ •5‬ﻧﻀﻊ 𝜃 ‪ . w = tan‬أوﺟﺪ اﻟﻨﻬﺎﻳﺔ ‪w :‬‬


‫∞‪→+‬‬

‫‪ •6‬إﺳﺘﻨﺘﺞ اﻟﻨﻬﺎﻳﺔ ‪. lim 𝜃 :‬‬


‫∞‪→+‬‬
‫‪ƕ‬‬

‫‪VII‬‬

‫‪ •1‬ﻟﻴﻜﻦ ‪ . n ≥ 1‬ﻟﺪﻳﻨﺎ ‪:‬‬ ‫ﺍﳊﻞّ‪.‬‬


‫‪ũŏ‬‬

‫‪a + b‬‬ ‫)‪(1 + (n + 1) ı) (1 − nı) 1 − nı + (n + 1) ı + n (n + 1‬‬


‫=‬ ‫=‬
‫‪a b +‬‬ ‫)‪(1 + nı) (1 − (n + 1) ı) 1 − (n + 1) ı + nı + n (n + 1‬‬
‫‪n + n + 1 + ı 1 + n (n + 1) + ı‬‬
‫=‬ ‫=‬ ‫‪=u‬‬
‫‪n + n + 1 + ı 1 + n (n + 1) − ı‬‬

‫𝟰𝟳𝟮‬

‫‪http ://tinyurl.com/Malki1718‬‬ ‫‪0‬‬


‫‪ .VII‬ﲤﺎرﻳﻦ ﻟﻠﺘﻌﻤﻖ‬
‫‪.‬‬
‫‪ •2‬ﻟﻴﻜﻦ ‪ . n ≥ 1‬ﻟﺪﻳﻨﺎ ‪:‬‬
‫‬

‫‪Ŕž Ŧ Ľ‬‬
‫ ‪a‬‬
‫‪gray‬‬ ‫‪b gray‬‬
‫ ‪a‬‬‫‪b‬‬
‫‪gray‬‬ ‫‪a‬‬
‫‪gray‬‬
‫‪gray‬‬‫‬ ‫‪−  a gray‬‬
‫‪b‬‬ ‫ ‪+ b‬‬‫‬
‫‪v =u ×u ×⋯×u‬‬ ‫×‬ ‫‪u‬‬ ‫=‬ ‫×‬ ‫×‬ ‫⋯‬ ‫×‬ ‫×‬
‫‪−‬‬
‫‪a‬‬ ‫ ‪a‬‬
‫‪b gray‬‬
‫‪gray‬‬ ‫‪b‬‬
‫‪gray‬‬ ‫‪gray‬‬‫‬‫‪−‬‬
‫‬
‫‪a gray‬‬
‫ ‪b‬‬‫ ‪a‬‬
‫‪gray‬‬ ‫‪b +‬‬
‫‪b a‬‬ ‫‪1 − ı 1 + (n + 1) ı‬‬

‫‪œǃ‬‬
‫‪+‬‬
‫=‬ ‫⋅‬ ‫=‬ ‫⋅‬
‫‪a b‬‬ ‫‪+‬‬ ‫‪1 + ı 1 − (n + 1) ı‬‬

‫‪ •3‬ﻟﻴﻜﻦ ‪ . n ≥ 1‬ﻟﺪﻳﻨﺎ ‪:‬‬

‫‪Ŀ‬‬
‫)‪1 − ı 1 + (n + 1) ı (1 − ı) (1 + (n + 1) ı‬‬
‫= ‪v‬‬ ‫⋅‬ ‫=‬ ‫⋅‬
‫‪1 + ı 1 − (n + 1) ı‬‬ ‫‪1 +1‬‬ ‫)‪1 + (n + 1‬‬
‫)‪1 − 2ı − 1 1 + 2 (n + 1) ı − (n + 1‬‬
‫=‬ ‫⋅‬
‫‪2‬‬ ‫‪n + 2n + 2‬‬
‫‪−n − 2n + 2 (n + 1) ı‬‬ ‫‪2n + 2 + n + 2n ı‬‬ ‫‪2n + 2‬‬ ‫‪n + 2n‬‬
‫‪= −ı‬‬ ‫=‬ ‫=‬ ‫‪+‬‬ ‫‪ı‬‬
‫‪n + 2n + 2‬‬ ‫‪n + 2n + 2‬‬ ‫‪n + 2n + 2 n + 2n + 2‬‬

‫و ﻫﻮ اﻟﺸﻜﻞ اﳉﱪي ﻟﻠﻌﺪد ‪. v‬‬


‫‪ •4‬ﻟﻴﻜﻦ ‪ . n ≥ 1‬ﻟﺪﻳﻨﺎ ‪:‬‬

‫‪(2n + 2) + n + 2n‬‬ ‫‪= n + 4n + 8n + 8n + 4 = n + 2n + 2‬‬

‫ﻣﻨﻪ ‪:‬‬

‫‪(2n + 2) + n + 2n‬‬ ‫‪n + 2n + 2‬‬


‫= || ‪||v‬‬ ‫=‬ ‫‪=1‬‬
‫‪n + 2n + 2‬‬ ‫‪n + 2n + 2‬‬

‫ﻣﻨﻪ ‪:‬‬
‫‪ņőƱŀ Ţ‬‬
‫) ‪Re (v‬‬ ‫‪2n + 2‬‬ ‫) ‪Im (v‬‬ ‫‪n + 2n‬‬
‫= 𝜃 ‪cos‬‬ ‫=‬ ‫و‬ ‫= 𝜃 ‪sin‬‬ ‫=‬
‫|| ‪||v‬‬ ‫‪n + 2n + 2‬‬ ‫|| ‪||v‬‬ ‫‪n + 2n + 2‬‬

‫أن ‪ n + 4n + 8n + 8n + 4 = n + 2n + 2 :‬ﻓﻴﻜﻤﻦ اﻹﻛﺘﻔﺎء‬ ‫ﻣﻼﺣﻈﺔ ‪ ⧏ : 42‬إن ﱂ ﻧﻼﺣﻆ ّ‬


‫ﺑﺎﻟﺸﻜﻞ اﳌﻨﺸﻮر ﻟﻪ و ﻹﳚﺎد اﻟﻄﻮﻳﻠﺔ‪ ،‬ﻧﻘﻮم ﺑﻨﴩ ﻋﺒﺎرة اﳌﻘﺎم ‪ n + 2n + 2‬و ﻧﺠﺪ ‪:‬‬
‫‪(2n + 2) + n + 2n‬‬ ‫‪n + 4n + 8n + 8n + 4‬‬
‫= || ‪||v‬‬ ‫=‬ ‫‪=1‬‬
‫‪n + 2n + 2‬‬ ‫‪n + 4n + 8n + 8n + 4‬‬
‫⧐‬
‫‪ƕ‬‬

‫ﻣﻼﺣﻈﺔ ‪ ⧏ : 43‬ﻟﻴﻜﻦ ∗‪ . n ∈ ℕ‬ﺑﲈ ّ‬


‫‪VII‬‬

‫أن ‪ (2n + 2) + n + 2n = n + 2n + 2‬ﻓﻬﺬا ﻳﻌﻨﻲ ّ‬


‫أن‬
‫اﻟﺜﻼﺛﻴﺔ ‪ 2n + 2, n + 2, n + 2n + 2‬ﺣﻞ ﻟﻠﻤﻌﺎدﻟﺔ اﻟﺪﻳﻮﻓﺎﻧﺘﻴﺔ ‪ . x + y = z‬و ﺑﲈ ّ‬
‫أن ‪ n‬ﻛﻴﻔﻲ ﻓﻬﺬا ﻳﻌﻨﻲ‬
‫أن ﻫﺬه اﳌﻌﺎدﻟﺔ ﺗﻘﺒﻞ ﻋﺪد ًا ﻻﳖﺎﺋﻴ ًﺎ ﻣﻦ اﳊﻠﻮل‪.‬‬
‫ّ‬
‫‪ũŏ‬‬

‫ﰲ اﳊﻘﻴﻘﺔ‪ ،‬ﻳﻤﻜﻦ إﺛﺒﺎت ّ‬


‫أن ﺣﻠﻮل اﳌﻌﺎدﻟﺔ اﻟﺪﻳﻮﻓﺎﻧﺘﻴﺔ ‪ x + y = z‬ﻫﻲ ﲨﻴﻊ اﻟﺜﻼﺛﻴﺎت‬
‫‪𝛼 u −v‬‬ ‫‪𝛼 u +v‬‬
‫‪ 𝛼uv,‬ﺣﻴﺚ 𝛼 ﻋﺪد ﻃﺒﻴﻌﻲ ﻛﻴﻔﻲ‪ ،‬و ‪ u‬و ‪ v‬ﻋﺪدان ّأوﻟﻴﺎن ﻓﺮدﻳﺎن و ﳐﺘﻠﻔﺎن ﻣﻊ‬ ‫‪,‬‬
‫‪2‬‬ ‫‪2‬‬
‫‪.v < u‬‬
‫⧐‬

‫𝟱𝟳𝟮‬

‫‪http ://tinyurl.com/Malki1718‬‬ ‫‪0‬‬


‫‪.‬‬
‫ﻣﻼﺣﻈﺔ ‪ ⧏ : 44‬ﰲ اﻟﻘﺮن اﻟﺴﺎﺑﻊ ﻋﴩ‪ ،‬ﻛﺘﺐ اﻟﻌﺎﱂ اﻟﻔﺮﻧﴘ ﺑﻴﻴﺮ ﻓﻴﺮﻣﺎ ﴿‪ّ ﴾P FERMAT‬‬
‫أن اﳌﻌﺎدﻟﺔ‬
‫اﻟﺪﻳﻮﻓﺎﻧﺘﻴﺔ ‪ ، x + y = z‬ﺣﻴﺚ ‪ ، n ≥ 3‬ﺗﻘﺒﻞ ﺣﻼ وﺣﻴﺪ ًا و ﻫﻮ اﻟﺜﻼﺛﻴﺔ )‪ (0, 0, 0‬ﻟﻜﻨﻪ ﱂ ﻳﻘﺪﱢ م إﺛﺒﺎﺗ ًﺎ ﻟﺬﻟﻚ ﺑﻞ‬

‫‪Ŕž Ŧ Ľ‬‬
‫اﻛﺘﻔﻰ ﺑﺎﻟﻘﻮل ﰲ )رﺳﺎﻟﺔ ﻛﺘﺒﻬﺎ ﻷﺣﺪ زﻣﻼﺋﻪ( ‪” :‬ﺇﻥ‪ ‬ﻫﺎﻣﺶ ﻫﺬﻩ ﺍﻟﺮﺳﺎﻟﺔ ﻻ ﻳﺘّﺴﻊ ﻟﻜﺘﺎﺑﺔ ﺍﻟﱪﻫﺎﻥ“‪ .‬و ﻇﻠﺖ ﻫﺬه »اﻟﻨﻈﺮﻳﺔ«ﺑﺪون‬

‫‪œǃ‬‬
‫ﺑﺮﻫﺎن ﻟﻔﱰة ﻃﻮﻳﻠﺔ رﻏﻢ ﻛﻞ اﳉﻬﻮد اﻟﺘﻲ ﺑﺬﳍﺎ اﻟﻌﻠﲈء )و ﻫﻢ ُﻛ ُﺜﺮ( ﺑﻌﺪ ﻓﻴﺮﻣﺎ إﱃ ﻏﺎﻳﺔ أواﺧﺮ اﻟﻘﺮن اﻟﻌﴩﻳﻦ )و‬
‫ﺑﺎﻟﺘﺤﺪﻳﺪ ﻧﻮﻓﻤﱪ ‪ (1994‬ﺣﻴﺚ ﲤﻜﻦ اﻟﻌﺎﱂ اﻟﱪﻳﻄﺎﲏ ﺃﻧﺪﺭﻭ ﻭﺍﻳﻠﺰ ﴿‪ ﴾A WILES‬ﻣﻦ إﺛﺒﺎﲥﺎ ﻣﺴﺘﻌﻤ ً‬
‫ﻼ ًّ‬
‫ﻛﲈ‬
‫اﻟﺘﺪرج( ‪.‬‬
‫ُﺪرس ﻓﻴﲈ ﺑﻌﺪ ﱡ‬ ‫ﻣﻌﺘﱪ ًا ﻣﻦ اﻟﺮﻳﺎﺿﻴﺎت اﳌﺘﻘﺪﻣﺔ )و اﻟﺘﻲ ﺗ ﱠ‬
‫⧐‬

‫‪Ŀ‬‬
‫‪ •5‬ﻷﺟﻞ ‪ n ≥ 1‬ﻟﺪﻳﻨﺎ ‪:‬‬
‫‪+‬‬
‫𝜃 ‪sin‬‬ ‫‪+ +‬‬ ‫‪n + 2n‬‬
‫= 𝜃 ‪w = tan‬‬ ‫=‬ ‫‪+‬‬
‫=‬
‫𝜃 ‪cos‬‬ ‫‪2n + 2‬‬
‫‪+ +‬‬

‫‪n + 2n‬‬
‫‪. lim‬‬ ‫‪w = lim‬‬ ‫إذن ∞‪= +‬‬
‫∞‪→+‬‬ ‫∞‪→+‬‬ ‫‪2n + 2‬‬
‫𝜋‬ ‫𝜋‬
‫‪0,‬‬ ‫∪‬ ‫𝜋‪,‬‬ ‫أن 𝜋‪ 0 ≤ 𝜃 < 2‬و ∞‪ ، lim tan 𝜃 = +‬و ﺑﲈ ّ‬
‫أن داﻟﺔ اﻟﻈﻞ )‪ (tan‬ﻣﺴﺘﻤﺮة ﻋﲆ اﳌﺠﺎل‬ ‫‪ •6‬ﺑﲈ ّ‬
‫‪2‬‬ ‫‪2‬‬ ‫∞‪→+‬‬
‫𝜋‬
‫‪. lim‬‬ ‫𝜃 ‪ lim tan 𝜃 = tan lim‬ﻣﻨﻪ ‪𝜃 = + :‬‬ ‫ّ‬
‫ﻓﺈن ∞‪= +‬‬
‫∞‪→+‬‬ ‫‪2‬‬ ‫∞‪→+‬‬ ‫∞‪→+‬‬
‫𝜋‬
‫‪ ،‬ﳍﺬا ﻧﻜﺘﺐ ‪:‬‬ ‫ﻣﻼﺣﻈﺔ ‪ ⧏ : 45‬ﻟﺪﻳﻨﺎ ‪ ||v || = 1 :‬ﻣﻨﻪ ‪ ، lim ||v || = 1‬و ﻟﺪﻳﻨﺎ ‪lim arg (v ) = + :‬‬
‫∞‪→+‬‬ ‫‪2‬‬ ‫∞‪→+‬‬
‫‪. lim v = 1e / = ı‬‬
‫∞‪→+‬‬
‫⧐‬

‫■‬

‫𝜋‪k‬‬
‫‬ ‫‪.‬‬
‫𝟐𝟗𝟏‬ ‫‪.‬‬
‫= ‪.S‬‬ ‫‪sin‬‬ ‫) ‪ (S‬اﳌﻌﺮﻓﺔ ﻟﻜﻞ ‪ n ≥ 2‬ﺑﺎﻟﻌﺒﺎرة ‪:‬‬ ‫≥‬
‫اﳍﺪف ﻣﻦ اﻟﺘﻤﺮﻳﻦ ﻫﻮ دراﺳﺔ اﳌﺘﺘﺎﻟﻴﺔ‬
‫‪n‬‬
‫‪ņőƱŀ Ţ‬‬
‫=‬

‫‪−‬‬
‫‪.‬‬ ‫‪z‬‬ ‫‪ . z = e‬أﺣﺴﺐ اﳌﺠﻤﻮع‬ ‫‪/‬‬ ‫‪ •1‬ﻧﻀﻊ‪ ،‬ﻷﺟﻞ ‪، n ≥ 2‬‬
‫=‬

‫‪2‬‬ ‫‪1‬‬
‫‪.‬‬ ‫‪=1+ı‬‬ ‫‪ •2‬أﺛﺒﺖ أﻧﻪ ﻟﻜﻞ ‪: n ≥ 2‬‬
‫‪1−z‬‬ ‫‪tan‬‬
‫‪1‬‬
‫= ‪.S‬‬ ‫‪ •3‬إﺳﺘﻨﺘﺞ أﻧﻪ ﻟﻜﻞ ‪: n ≥ 2‬‬
‫‪tan‬‬
‫‪ƕ‬‬

‫‪S‬‬
‫‪VII‬‬

‫= ‪.u‬‬ ‫) ‪ (u‬اﳌﻌﺮﻓﺔ ﻟﻜﻞ ‪ n ≥ 2‬ﺑﺎﻟﻌﺒﺎرة ‪:‬‬ ‫≥‬


‫‪ •4‬أدرس ﳖﺎﻳﺔ اﳌﺘﺘﺎﻟﻴﺔ‬
‫‪n‬‬
‫‪ũŏ‬‬

‫‪ •1‬ﳎﻤﻮع ﺟﺰﺋﻲ ﳌﺘﺘﺎﻟﻴﺔ ﻫﻨﺪﺳﻴﺔ أﺳﺎﺳﻬﺎ ‪: z ≠ 1‬‬ ‫ﺍﳊﻞّ‪.‬‬


‫‪−‬‬
‫‪1−z‬‬ ‫‪1−e‬‬ ‫)‪1 − (−1‬‬ ‫‪2‬‬
‫= ‪z‬‬ ‫=‬ ‫=‬ ‫=‬
‫=‬
‫‪1−z‬‬ ‫‪1−z‬‬ ‫‪1−z‬‬ ‫‪1−z‬‬

‫‪ •2‬ﻟﺪﻳﻨﺎ ﻟﻜﻞ ‪: n ≥ 2‬‬


‫𝜋‬
‫‪1−z=1−e‬‬ ‫‪/‬‬ ‫‪=e‬‬ ‫‪e−‬‬ ‫‪−e‬‬ ‫‪= −2ıe‬‬ ‫‪sin‬‬
‫‪2n‬‬

‫𝟲𝟳𝟮‬

‫‪http ://tinyurl.com/Malki1718‬‬ ‫‪0‬‬


‫ ﲤﺎرﻳﻦ ﻟﻠﺘﻌﻤﻖ‬.VII
.
‫ﻣﻨﻪ‬
2 ıe− ı cos − ı sin 1
= = =1+ı

Ŕž Ŧ Ľ
1 − z sin sin tan

ّ ‫ ﻧﻼﺣﻆ‬،‫• ﺑﺪاﻳﺔ‬3
‫أن‬

œǃ
− −
k𝜋 k𝜋 k𝜋 k𝜋
S = sin = sin = sin = sin
=
n =
n =
n =
n

k𝜋

Ŀ
. (k = n ‫ و‬k = 0 ‫ ﻣﻦ أﺟﻞ‬sin = 0)
n
: ‫ﻟﺪﻳﻨﺎ‬
− −
k𝜋 /
S = sin = Im e
=
n =
− −
2
= Im z = Im z = Im
= =
1−z

1 1
= Im 1 + ı =
tan tan

: ‫ ﻟﺪﻳﻨﺎ‬n ≥ 2 ‫• ﻟﻜﻞ‬4
S 1 cos
u = = =
n n tan n sin
2 𝜋
= ⋅ ⋅ cos
𝜋 sin 2n

‫ﻟﻜﻦ‬
x 𝜋
lim = lim =1 x= ⟶ 0
ņőƱŀ Ţ
→+∞ sin → sin x 2n →+∞

𝜋
‫ ﻣﻨﻪ‬lim cos = lim cos x = 1 ‫و‬
→+∞ 2n →
2 2
lim u = × 1 × 1 =
→+∞ 𝜋 𝜋
2
. ‫( ﻧﺤﻮ‬converge en moyenne) ‫( ﺗﺘﻘﺎرب َو َﺳﻄﻴ ًﺎ‬S ) ‫أن اﳌﺘﺘﺎﻟﻴﺔ‬
ّ ‫⧏ ﻧﻘﻮل‬ : 46 ‫ﻣﻼﺣﻈﺔ‬
𝜋


ƕ
VII

ّ ‫ أﺛﺒﺖ‬. zz ‫ أﺣﺪ اﳉﺬرﻳﻦ اﻟﱰﺑﻴﻌﻴﲔ ﻟﻠﻌﺪد‬u ‫ ﻋﺪدﻳﻦ ﻣﺮﻛﺒﲔ و‬z ‫ و‬z ‫ﻟﻴﻜﻦ‬
: ‫أن‬
$ .
𝟏𝟗𝟑 .
ũŏ

|z + z | |z + z |
|z| + ||z || = | − u| + | + u|
|| 2 || || 2 ||

‫ ﻣﻨﻪ‬b = z ‫ و‬a = z ‫ ﻟﺪﻳﻨﺎ‬. z ِ‫ ﺟﺬر ًا ﺗﺮﺑﻴﻌﻴﺎ ﻟـ‬b ‫ و‬z ِ‫ ﺟﺬر ًا ﺗﺮﺑﻴﻌﻴﺎ ﻟـ‬a ‫ ﻟﻴﻜﻦ‬: ‫اﻟﻄﺮﻳﻘﺔ اﻷوﱃ‬ • .ّ‫ﺍﳊﻞ‬
. u = ±ab ‫( أي‬u − ab) (u + ab) = 0 ‫( ﻣﻨﻪ‬ab) = zz = u

𝟮𝟳𝟳

http ://tinyurl.com/Malki1718 0
.
ّ ‫ إذا‬1 ‫أن ﻧﺺ اﻟﺘﻤﺮﻳﻦ ﻻ ﻳﺘﺒﺪّ ل‬
: ‫ و ﺑﺎﻟﺘﺎﱄ‬u = ab ‫ ﻓﺒﺈﻣﻜﺎﻧﻨﺎ أن ﻧﺄﺧﺬ‬−u ِ‫ ﺑــ‬u ‫ﻋﻮﺿﻨﺎ ﻋﻦ‬ ّ ‫ﺑﲈ‬

Ŕž Ŧ Ľ
|z + z | |z + z | |a + b | |a + b | | a + b − 2ab | | a + b + 2ab |
| − u| + | + u| = | − ab| + | + ab| = | |+| |
|| 2 || || 2 || || 2 || || 2 || || 2 || || 2 ||

œǃ
| (a − b) | | (a + b) | | 1 | | |1|
=| |+| | = | | |(a − b) || + | | ||(a + b) ||
|| 2 || || 2 || || 2 || | | || 2 || | |
1 1 1
= ||a − b|| + ||a + b|| = (a − b) a − b + (a + b) a + b

Ŀ
2 2 2
1
= aa − ab − bb + bb + aa + ab + bb + bb
2
| |
= aa + bb = ||a || + ||b|| = ||a || + |b | = |z| + ||z ||
| |
.‫و ﻫﻮ اﳌﻄﻠﻮب‬
ّ ‫ و ﻧُﺬﻛﱢﺮ أﻧﻪ‬،‫ ﻧﻘﻮم ﺑﱰﺑﻴﻊ اﻟﻄﺮﻓﲔ‬: ‫اﻟﻄﺮﻳﻘﺔ اﻟﺜﺎﻧﻴﺔ‬
: ‫ ﻟﺪﻳﻨﺎ‬v ‫ و‬u ‫ﻟﻜﻞ ﻋﺪدﻳﻦ ﻣﺮﻛﺒﲔ‬ •


⎪ |u + v| = (u + v) (u + v) = |u| + 2 Re (uv) + |v|

|u − v| = (u − v) (u − v) = |u| − 2 Re (uv) + |v|



⎩ |u + v| + |u − v| = 2 |u| + 2 |v| (‫)ﺑﺠﻤﻊ اﳌﺴﺎوﺗﲔ أﻋﻼه‬

: ‫ﻓﻴﻜﻮن‬

|z + z | |z + z | |z + z | |z + z | |z + z | |z + z |
| − u| + | + u| =| − u| + 2 | − u| ⋅ | + u| + | + u|
|| 2 || || 2 || || 2 || || 2 || || 2 || || 2 ||
|z + z | | z+z z+z | |z + z |
=| − u| + 2 | −u +u |+| + u|
|| 2 || || 2 2 || || 2 ||
| |
ņőƱŀ Ţ
|z + z | |z + z | | z+z |
=| − u| + | + u| + 2 | −u |
|| 2 || || 2 || || 2 ||
|z + z | | |
| z+z |
= 2| | + 2 ||zz || + 2 | − zz |
|| 2 || || 2 ||
| | | z + 2zz + z − 4zz |
| z+z | | | | |
= 2| | + 2 |z| ⋅ |z | + 2
|| 4 || || 4 ||
| |
ƕ

| z+z | | z − 2zz + z |
VII

=| | + 2 |z| ⋅ |
|z |
| + | |
|| 2 || || 2 ||
| | | |
| z+z | | z−z |
| + 2 |z| ⋅ ||z ||
ũŏ

=| |+|
|| 2 || || 4 ||
1 | | | |
= z + z | + | z − z | + 2 |z| ⋅ ||z ||
2 || | | |
1
= ||z + z || + ||z − z || + 2 |z| ⋅ ||z ||
2

!‫أي اﳉﺬرﻳﻦ ﻧﺄﺧﺬ و إﻻّ ﻟﻜﺎن اﻟﺘﻤﺮﻳﻦ ﺧﺎﻃﺌ ًﺎ‬


ّ ‫ ﻷﻧﻪ إن ﱂ ﻳﻜﻦ اﻷﻣﺮ ﻛﺬﻟﻚ ﳊﺪد اﻟﻨﺺ‬،‫و ﻫﺬا ﻳﻤﻜﻦ ﺗﻮ ّﻗﻌﻪ‬1

𝟮𝟳𝟴

http ://tinyurl.com/Malki1718 0
‫ ﲤﺎرﻳﻦ ﻟﻠﺘﻌﻤﻖ‬.VII
.
1
= 2 |z| + 2 ||z || + 2 |z| ⋅ ||z ||
2

Ŕž Ŧ Ľ
= |z| + ||z || + 2 |z| ⋅ ||z || = |z| + ||z ||

.(x = y ⟺ x = y : ℝ+ ‫و ﺑﺄﺧﺬ اﳉﺬر اﻟﱰﺑﻴﻌﻲ ﻟﻠﻄﺮﻓﲔ ﻧﺼﻞ إﱃ اﻟﻨﺘﻴﺠﺔ اﳌﻄﻠﻮﺑﺔ )ﻧُﺬﻛﱢﺮ أﻧﻪ ﰲ‬

œǃ

! .

Ŀ
.
𝟏𝟗𝟒
cos a + cos b + cos c = 0
. : ‫( اﻟﺘﻲ ﲢﻘﻖ اﳉﻤﻠﺔ‬a, b, c) ∈ ℝ ‫أوﺟﺪ اﻟﺜﻼﺛﻴﺎت‬
sin a + sin b + sin c = 0
280 ‫ ﺻﻔﺤﺔ‬195 ‫أﻧﻈﺮ اﻟﺘﻤﺮﻳﻦ‬ 
300 ‫ ﺻﻔﺤﺔ‬210 ‫أﻧﻈﺮ اﻟﺘﻤﺮﻳﻦ‬ 
‫ ﻟﺪﻳﻨﺎ‬: ‫ اﻟﻄﺮﻳﻘﺔ اﻷوﱃ ﺑﺎﳊﺴﺎب‬.ّ‫ﺍﳊﻞ‬

cos a + cos b + cos c = 0


⟺ (cos a + cos b + cos c) + ı (sin a + sin b + sin c) = 0
sin a + sin b + sin c = 0
⟺ e +e +e =0

. ||e + e || = ||−e || = 1 ‫ ﺑﺄﺧﺬ اﻟﻄﻮﻳﻠﺔ ﻳﻨﺘﺞ‬. e + e = −e ‫ﻣﻨﻪ‬


: ‫ ﻣﻨﻪ‬e + e = e / e / e − / + e− − / ‫ﻟﻜﻦ‬
1 = ||e / e / e − / + e− − / ||
a − b|
= ||e / || ⋅ ||e / || ⋅ ||e − / + e− − / || = 1 ⋅ 1 ⋅ ||2 cos
| 2 ||
2𝜋 a−b 𝜋 | a − b| 1 a − b|
‫ ﻣﻨﻪ‬a − b = ± (mod 2𝜋) ‫أي‬ = ± (mod 𝜋) ‫| و ﺑﺎﻟﺘﺎﱄ‬cos = ‫||| ﻣﻨﻪ‬2 cos = 1 ‫أي‬
2 || 2 2 ||
ņőƱŀ Ţ
3 2 3 |
/ ± / 2𝜋
: ‫ ﻓﻴﻜﻮن‬j = e ‫ ﻧﻀﻊ‬. e = e ‫ أي‬b = a ± (mod 2𝜋)
3
‫؛‬e = − e +e = − (1 + j) e =j e ‫ ﻣﻨﻪ‬e = je •

e =− e +e =− 1+j e = je ‫ ﻣﻨﻪ‬e = j e ‫أو‬ •

4𝜋 2𝜋
c=a+ (mod 2𝜋) ‫ و‬b=a+ (mod 2𝜋)
3 3 : ‫و ﺑﺎﻟﺘﺎﱄ‬
ƕ
VII

2𝜋 4𝜋
.c = a + (mod 2𝜋) ‫ و‬b=a+ (mod 2𝜋) ‫أو‬
3 3
: ‫ﺑﺎﻟﻌﻜﺲ‬
ũŏ

ّ e = j e ‫ و‬e = je ‫إذا ﻛﺎن‬


: ‫ﻓﺈن‬ •

e +e +e =e 1+j+j =0

ّ e = je ‫ و‬e = j e ‫و إذا ﻛﺎن‬


: ‫ﻓﺈن‬ •

e +e +e =e 1+j +j =0

𝟮𝟳𝟵

http ://tinyurl.com/Malki1718 0
‫‪.‬‬
‫ﰲ اﻷﺧﲑ ‪ ،‬ﳎﻤﻮﻋﺔ اﳊﻠﻮل ﻫﻲ ‪:‬‬

‫‪Ŕž Ŧ Ľ‬‬
‫𝜋‪2‬‬ ‫𝜋‪4‬‬
‫= 𝒮‬ ‫‪a, a +‬‬ ‫‪+ 2k𝜋, a +‬‬ ‫‪+ 2ℓ𝜋 ; (a, k, ℓ) ∈ ℝ × ℤ × ℤ‬‬
‫‪3‬‬ ‫‪3‬‬
‫𝜋‪4‬‬ ‫𝜋‪2‬‬

‫‪œǃ‬‬
‫‪a, a +‬‬ ‫‪+ 2k𝜋, a +‬‬ ‫‪+ 2ℓ𝜋 ; (a, k, ℓ) ∈ ℝ × ℤ × ℤ‬‬
‫‪3‬‬ ‫‪3‬‬

‫اﻟﻄﺮﻳﻘﺔ اﻟﺜﺎﻧﻴﺔ ‪ :‬ﻳﻤﻜﻦ اﻟﺒﺤﺚ ﺑﻄﺮﻳﻘﺔ ﻫﻨﺪﺳﻴﺔ‪ .‬ﻧﻀﻊ ‪ v = e ، u = e‬و ‪ w = e‬و ﻟﺘﻜﻦ ‪ C ، B ، A‬ﻟﻮاﺣﻘﻬﺎ ﻋﲆ‬
‫أن ‪ّ ||u − 0|| = ||v − 0|| = ||w − 0|| = 1‬‬
‫ﻓﺈن اﻷﻋﺪاد ‪ v ، u‬و ‪ w‬ﻫﻲ ﻟﻮاﺣﻖ ﺛﻼﺛﺔ ﻧﻘﻂ ﺗﻘﻊ ﻋﲆ اﻟﺪاﺋﺮة اﻟﺘﻲ‬ ‫اﻟﱰﺗﻴﺐ‪ .‬ﺑﲈ ّ‬

‫‪Ŀ‬‬
‫ﻣﺮﻛﺰﻫﺎ اﻟﻨﻘﻄﺔ ‪) O‬ﻣﺒﺪأ اﳌﻌﻠﻢ( و ﻧﺼﻒ ﻗﻄﺮﻫﺎ ‪. 1‬‬
‫‪1‬‬
‫أن اﻟﻨﻘﻄﺔ ‪) O‬ﺻﻮرة اﻟﻌﺪد ‪ (0‬ﻫﻲ ﻣﺮﻛﺰ اﳌﺴﺎﻓﺎت اﳌﺘﺴﺎوﻳﺔ‬ ‫ﻟﺪﻳﻨﺎ ‪ u + v + w = 0‬أي ‪ (u + v + w) = 0‬و ﻫﺬا ﻳﻌﻨﻲ ّ‬
‫‪3‬‬
‫ﻣﺘﻮﺳﻄﺎت ﻫﺬا اﳌﺜﻠﺚ ﻫﻲ أﻳﻀ ًﺎ ﳏﺎور أﺿﻼﻋﻪ ‪.‬‬
‫ﱢ‬ ‫ّ‬
‫أن‬ ‫ﻳﻌﻨﻲ‬ ‫ﻫﺬا‬ ‫و‬ ‫‪ABC‬‬ ‫اﳌﺜﻠﺚ‬ ‫ﻟﻠﻨﻘﻂ ‪ B ، A‬و ‪ C‬أي ‪ O = G‬ﻫﻲ ﻣﺮﻛﺰ ﺛﻘﻞ‬
‫إذن ﻓﺎﳌﺜﻠﺚ ‪ ABC‬ﻣﺘﻘﺎﻳﺲ اﻷﺿﻼع‪.‬‬
‫أن ﻣﺮﻛﺰ ﺛﻘﻠﻪ ﻫﻮ ﻣﺮﻛﺰ اﻟﺪاﺋﺮة اﳌﺤﻴﻄﺔ ﺑﻪ أي ّ‬
‫أن‬ ‫ﺑﺎﻟﻌﻜﺲ‪ ،‬إذا ﻛﺎن اﳌﺜﻠﺚ ‪ ABC‬ﻣﺘﻘﺎﻳﺲ اﻷﺿﻼع ﻓﻤﻦ اﻟﻮاﺿﺢ ّ‬
‫‪.u+v+w = 0‬‬
‫ﺣﻞ ﻟﻠﻤﻌﺎدﻟﺔ ‪ e + e + e = 0‬إذا و ﻓﻘﻂ إذا ﻛﺎن اﳌﺜﻠﺚ‪ ،‬اﻟﺬي ﻟﻮاﺣﻖ رؤوﺳﻪ ﻫﻲ ‪،e‬‬ ‫أن اﻟﺜﻼﺛﻴﺔ )‪ّ (a, b, c‬‬
‫ﻧﺴﺘﻨﺘﺞ ّ‬
‫‪ e‬و ‪ ، e‬ﻣﺘﻘﺎﻳﺲ اﻷﺿﻼع أي ﳎﻤﻮﻋﺔ اﳊﻠﻮل ﻫﻲ ‪:‬‬
‫𝜋‪2‬‬ ‫𝜋‪4‬‬
‫= 𝒮‬ ‫‪a, a +‬‬ ‫‪+ 2k𝜋, a +‬‬ ‫‪+ 2ℓ𝜋 ; (a, k, ℓ) ∈ ℝ × ℤ × ℤ‬‬
‫‪3‬‬ ‫‪3‬‬
‫𝜋‪4‬‬ ‫𝜋‪2‬‬
‫‪a, a +‬‬ ‫‪+ 2k𝜋, a +‬‬ ‫‪+ 2ℓ𝜋 ; (a, k, ℓ) ∈ ℝ × ℤ × ℤ‬‬
‫‪3‬‬ ‫‪3‬‬

‫■‬ ‫ﻫﺬا اﻟﺘﻤﺮﻳﻦ ﺣﺎﻟﺔ ﺧﺎﺻﺔ ﻣﻦ اﻟﺘﻤﺮﻳﻦ ‪ 210‬ﺻﻔﺤﺔ ‪.300‬‬

‫ﻟﺘﻜﻦ ‪ y ، x‬و ‪ z‬أﻋﺪاد ًا ﺣﻘﻴﻘﻴﺔ ُﲢﻘﻖ ‪ . e + e + e = 0 :‬أﺛﺒﺖ ّ‬


‫أن ‪:‬‬
‫!‬ ‫‪.‬‬
‫𝟓𝟗𝟏‬ ‫‪.‬‬
‫‪ņőƱŀ Ţ‬‬
‫‪e‬‬ ‫‪+e‬‬ ‫‪+e‬‬ ‫‪=0‬‬
‫أﻧﻈﺮ اﻟﺘﻤﺮﻳﻦ ‪ 210‬ﺻﻔﺤﺔ ‪300‬‬ ‫‬
‫أﻧﻈﺮ اﻟﺘﻤﺮﻳﻦ ‪ 194‬ﺻﻔﺤﺔ ‪279‬‬ ‫‬
‫ﺍﳊﻞّ‪ .‬اﻟﻄﺮﻳﻘﺔ اﻷوﱃ ‪ :‬ﻧﻘﺴﻢ ﻃﺮﰲ اﳌﺴﺎواة ‪ e + e + e = 0‬ﻋﲆ ‪ e‬ﻓﻨﺠﺪ ‪ 1 + e + e = 0‬ﻣﻊ ‪ 𝛼 = y − x‬و‬
‫‪ . 𝛽 = z − x‬ﻣﻨﻪ ‪:‬‬
‫‪e‬‬ ‫‪+e‬‬ ‫‪= −1 ⟺ (cos 𝛼 + cos 𝛽) + ı (sin 𝛼 + sin 𝛽) = −1 + 0 ⋅ ı‬‬
‫‪ƕ‬‬

‫‪cos 𝛼 + cos 𝛽 = −1‬‬


‫‪VII‬‬

‫⟺‬
‫‪sin 𝛼 + sin 𝛽 = 0‬‬

‫أن )𝜋‪ 𝛼 = −𝛽 (mod 2‬أو )𝜋‪. 𝛼 = 𝜋 + 𝛽 (mod 2‬‬ ‫ﻣﻦ اﳌﻌﺎدﻟﺔ ‪ sin 𝛼 + sin 𝛽 = 0‬ﻧﺴﺘﻨﺘﺞ ّ‬
‫‪ũŏ‬‬

‫إذا ﻛﺎن )𝜋‪ 𝛼 = 𝜋 + 𝛽 (mod 2‬ﻓﺈﻧّﻪ ﺑﺎﻟﺘﻌﻮﻳﺾ ﰲ اﳌﻌﺎدﻟﺔ ‪ cos 𝛼 + cos 𝛽 = −1‬ﻧﺠﺪ ‪ 0 = −1‬و ﻫﺬا ﺗﻨﺎﻗﺾ ‪.‬‬
‫إذا ﻛﺎن )𝜋‪ 𝛼 = −𝛽 (mod 2‬ﻓﺈﻧّﻪ ﺑﺎﻟﺘﻌﻮﻳﺾ ﰲ اﳌﻌﺎدﻟﺔ ‪ cos 𝛼 + cos 𝛽 = −1‬ﻧﺠﺪ ‪2 cos 𝛼 = −1‬‬
‫𝜋‪2‬‬
‫‪ e = e−‬ﻣﻨﻪ ‪:‬‬ ‫‪/‬‬ ‫‪=j‬‬ ‫‪ e = e‬أو‬ ‫‪/‬‬ ‫‪ .𝛼 = ±‬و ﺑﺎﻟﺘﺎﱄ ّ‬
‫ﻓﺈن ‪= j‬‬ ‫ﻣﻨﻪ )𝜋‪(mod 2‬‬
‫‪3‬‬
‫‪1+e‬‬ ‫‪+e‬‬ ‫‪=1+e‬‬ ‫‪/‬‬ ‫‪+ e−‬‬ ‫‪/‬‬ ‫‪=1+j+j =0‬‬
‫‪1+e‬‬ ‫‪+e‬‬ ‫‪= 1 + e−‬‬ ‫‪/‬‬ ‫‪+e‬‬ ‫‪/‬‬ ‫‪=1+j +j=0‬‬ ‫أو‬

‫𝟬𝟴𝟮‬

‫‪http ://tinyurl.com/Malki1718‬‬ ‫‪0‬‬


‫‪ .VII‬ﲤﺎرﻳﻦ ﻟﻠﺘﻌﻤﻖ‬
‫‪.‬‬
‫‪ e‬أي ‪. e + e + e = 0‬‬ ‫‪1+e‬‬ ‫‪+e‬‬ ‫‪ 1 + e‬ﻣﻨﻪ ‪= 0‬‬ ‫‪+e‬‬ ‫أي ‪= 0‬‬
‫أن ‪ (e ) = e‬و ﺑﺎﻟﺘﺎﱄ ﻳﻤﻜﻦ ﺗﺮﺑﻴﻊ ﻃﺮﰲ اﳌﺴﺎواة اﻷوﱃ ﻟﻠﺤﺼﻮل ﻋﲆ ﺣﺪود اﻟﻄﺮف اﻷول‬ ‫اﻟﻄﺮﻳﻘﺔ اﻟﺜﺎﻧﻴﺔ ‪ :‬ﻧﻼﺣﻆ ّ‬

‫‪Ŕž Ŧ Ľ‬‬
‫ﻣﻦ اﳌﺴﺎواة اﻟﺜﺎﻧﻴﺔ‪ .‬ﻟﻜﻦ ‪:‬‬
‫‪+e‬‬ ‫‪+e ) =e‬‬ ‫‪+e‬‬ ‫‪+e‬‬ ‫‪+2 e‬‬ ‫‪+‬‬ ‫‪+e‬‬ ‫‪+‬‬ ‫‪+e‬‬ ‫‪+‬‬
‫‪(e‬‬

‫‪œǃ‬‬
‫ﻛﻴﻒ ﻳﻤﻜﻦ اﻟﺘﺨﻠﺺ ﻣﻦ اﳊﺪود اﻹﺿﺎﻓﻴﺔ ؟ اﳉﻮاب ‪ :‬ﺑﺎﺳﺘﻐﻼل اﳋﺎﺻﻴﺔ ‪ ||e || = ||e || = ||e || = 1‬ﻋﻠ ًﲈ أﻧﻪ إذا ﻛﺎن‬
‫‪1‬‬
‫= ‪ . u‬ﻟﺪﻳﻨﺎ ‪:‬‬ ‫‪ّ |u| = 1‬‬
‫ﻓﺈن‬
‫‪u‬‬

‫‪Ŀ‬‬
‫‪e‬‬ ‫‪+e‬‬ ‫‪+e‬‬ ‫‪=0 ⟹ e‬‬ ‫‪+e‬‬ ‫‪+e‬‬ ‫‪=0‬‬
‫‪⟹ e‬‬ ‫‪+e‬‬ ‫‪+e‬‬ ‫‪=0‬‬
‫‪1‬‬ ‫‪1‬‬ ‫‪1‬‬
‫⟹‬ ‫‪+‬‬ ‫‪+‬‬ ‫‪=0‬‬
‫‪e‬‬ ‫‪e‬‬ ‫‪e‬‬
‫‪e + +e + +e‬‬ ‫‪+‬‬
‫⟹‬ ‫‪=0‬‬
‫‪e + +‬‬
‫‪⟹ e‬‬ ‫‪+‬‬ ‫‪+e + +e‬‬ ‫‪+‬‬ ‫‪=0‬‬

‫و ﺑﺎﻟﺘﺎﱄ ‪:‬‬
‫‪0 = (e‬‬ ‫‪+e‬‬ ‫‪+e ) =e‬‬ ‫‪+e‬‬ ‫‪+e‬‬ ‫‪+2 e‬‬ ‫‪+‬‬ ‫‪+e‬‬ ‫‪+‬‬ ‫‪+e‬‬ ‫‪+‬‬ ‫‪=e‬‬ ‫‪+e‬‬ ‫‪+e‬‬
‫اﻟﻄﺮﻳﻘﺔ اﻟﺜﺎﻟﺜﺔ ‪ :‬ﻳﻤﻜﻦ اﻟﺒﺤﺚ ﺑﻄﺮﻳﻘﺔ ﻫﻨﺪﺳﻴﺔ‪ .‬ﻧﻀﻊ ‪ b = e ، a = e‬و ‪ c = e‬و ﻟﺘﻜﻦ ‪ C ، B ، A‬ﻟﻮاﺣﻘﻬﺎ ﻋﲆ‬
‫أن ‪ّ ||a − 0|| = ||b − a|| = ||c − 0|| = 1‬‬
‫ﻓﺈن اﻷﻋﺪاد ‪ b ، a‬و ‪ c‬ﻫﻲ ﻟﻮاﺣﻖ ﺛﻼﺛﺔ ﻧﻘﻂ ﺗﻘﻊ ﻋﲆ اﻟﺪاﺋﺮة اﻟﺘﻲ‬ ‫اﻟﱰﺗﻴﺐ‪ .‬ﺑﲈ ّ‬
‫ﻣﺮﻛﺰﻫﺎ اﻟﻨﻘﻄﺔ ‪) O‬ﻣﺒﺪأ اﳌﻌﻠﻢ( و ﻧﺼﻒ ﻗﻄﺮﻫﺎ ‪. 1‬‬
‫‪1‬‬
‫ﻟﺪﻳﻨﺎ ‪ a + b + c = 0‬أي ‪ (a + b + c) = 0‬و ﻫﺬا ﻳﻌﻨﻲ ّ‬
‫أن اﻟﻨﻘﻄﺔ ‪) O‬ﺻﻮرة اﻟﻌﺪد ‪ (0‬ﻫﻲ ﻣﺮﻛﺰ اﳌﺴﺎﻓﺎت اﳌﺘﺴﺎوﻳﺔ‬
‫‪3‬‬
‫ﻣﺘﻮﺳﻄﺎت اﳌﺜﻠﺚ ﻫﻲ أﻳﻀ ًﺎ ﳏﺎور أﺿﻼﻋﻪ و‬
‫ّ‬ ‫ّ‬
‫أن‬ ‫ﻳﻌﻨﻲ‬ ‫ﻫﺬا‬ ‫و‬ ‫‪ABC‬‬ ‫اﳌﺜﻠﺚ‬ ‫ﻟﻠﻨﻘﻂ ‪ B ، A‬و ‪ C‬أي ‪ O = G‬ﻫﻲ ﻣﺮﻛﺰ ﺛﻘﻞ‬
‫ﺑﺎﻟﺘﺎﱄ ﻓﺎﳌﺜﻠﺚ ‪ ABC‬ﻣﺘﻘﺎﻳﺲ اﻷﺿﻼع‪.‬‬
‫أن ‪ j = 1‬و ‪ . (1 + j + j = 0‬ﺣﺴﺐ ﺧﻮاص اﳌﺜﻠﺚ اﳌﺘﻘﺎﻳﺲ اﻷﺿﻼع ّ‬
‫ﻓﺈن ‪ b = ja‬و ‪c = j a‬‬ ‫ﻧﻀﻊ ‪) j = e /‬ﻧُﺬﻛﱢﺮ ّ‬
‫‪ņőƱŀ Ţ‬‬
‫)أو ‪ b = j a‬و ‪. (c = ja‬‬
‫ﻧﻔﺮض ّ‬
‫أن ‪ b = ja‬و ‪ c = j a‬ﻣﻨﻪ‬
‫‪e‬‬ ‫‪+e‬‬ ‫‪+e‬‬ ‫‪=a +b +c =a +j a +j a =a‬‬ ‫‪1+j +j‬‬ ‫‪=0‬‬

‫ﻷن ‪ j = j‬و ‪ 1 + j + j = 0‬و ﻫﻮ اﳌﻄﻠﻮب‪.‬‬ ‫ّ‬


‫أن اﳌﺜﻠﺚ ‪ ABC‬ﻣﺘﻘﺎﻳﺲ اﻷﺿﻼع‪.‬‬ ‫ﻳﻤﻜﻦ أﻳﻀ ًﺎ ﺗﻄﺒﻴﻖ ﻧﺘﻴﺠﺔ اﻟﺘﻤﺮﻳﻦ ‪ 210‬ﺻﻔﺤﺔ ‪ 300‬ﻹﺛﺒﺎت ّ‬
‫اﻟﻄﺮﻳﻘﺔ اﻟﺮاﺑﻌﺔ ‪ :‬ﻟﺪﻳﻨﺎ ‪ e + e + e = 0‬ﻣﻨﻪ ‪ e + e = −e‬و ﻫﺬا ﻳﻌﻨﻲ ّ‬
‫أن ‪ . ||e + e || = ||−e || = 1‬ﻟﻜﻦ ‪:‬‬

‫‪||e‬‬ ‫‪+ e || = (e‬‬ ‫‪+ e ) (e‬‬ ‫‪+ e ) = (e‬‬ ‫‪+ e ) (e−‬‬ ‫) ‪+ e−‬‬
‫‪ƕ‬‬

‫‪−‬‬ ‫‪+ e−‬‬ ‫‪−‬‬


‫‪VII‬‬

‫‪=1+e‬‬ ‫)‪+ 1 = 2 + 2 cos (y − x‬‬

‫و ﺑﺎﻟﺘﺎﱄ ‪:‬‬
‫‪ũŏ‬‬

‫‪||e‬‬ ‫‪+ e || = 1 ⟺ ||e‬‬ ‫‪+ e || = 1 ⟺ 2 + 2 cos (y − x) = 1‬‬


‫‪1‬‬ ‫𝜋‪2‬‬
‫‪⟺ cos (y − x) = − ⟺ y − x = ±‬‬ ‫)𝜋‪(mod 2‬‬
‫‪2‬‬ ‫‪3‬‬

‫‪e‬‬ ‫‪= −e‬‬ ‫‪−e‬‬ ‫‪= (−1 − j) e‬‬ ‫‪e‬و=‬ ‫‪=e‬‬ ‫إذا ﻛﺎن )𝜋‪ّ y − x = 2𝜋 (mod 2‬‬
‫‪+‬‬
‫ﻓﺈن‬ ‫‪= je‬‬ ‫•‬
‫‪3‬‬
‫‪ j e‬و ﻫﺬا ﻳﻌﻨﻲ ّ‬
‫أن اﳌﺜﻠﺚ اﻟﺬي ﻟﻮاﺣﻖ رؤوﺳﻪ ‪ e ، e ، e‬ﻫﻮ ﻣﺜﻠﺚ ﻣﺘﻘﺎﻳﺲ اﻷﺿﻼع‪.‬‬

‫𝟭𝟴𝟮‬

‫‪http ://tinyurl.com/Malki1718‬‬ ‫‪0‬‬


‫‪.‬‬
‫‪y‬‬
‫‪P‬‬

‫‪Ŕž Ŧ Ľ‬‬
‫‪M‬‬

‫‪œǃ‬‬
‫‪O‬‬ ‫‪x‬‬

‫‪Ŀ‬‬
‫‪N‬‬

‫ﺷﻜﻞ ‪1.VII‬‬

‫‪e‬‬ ‫‪= −e −e‬‬ ‫‪= −1 − j‬‬ ‫‪e‬‬ ‫‪e‬و=‬ ‫‪=e‬‬ ‫‪−‬‬ ‫إذا ﻛﺎن )𝜋‪ّ y−x = − 2𝜋 (mod 2‬‬
‫ﻓﺈن‬ ‫‪=j e‬‬ ‫•‬
‫‪3‬‬
‫‪ je‬و ﻫﺬا ﻳﻌﻨﻲ ّ‬
‫أن اﳌﺜﻠﺚ اﻟﺬي ﻟﻮاﺣﻖ رؤوﺳﻪ ‪ e ، e ، e‬ﻫﻮ ﻣﺜﻠﺚ ﻣﺘﻘﺎﻳﺲ اﻷﺿﻼع‪.‬‬

‫■‬ ‫ﰲ ﻛﻞ اﳊﺎﻻت‪ ،‬ﻫﺬا اﳌﺜﻠﺚ ﻣﺘﻘﺎﻳﺲ اﻷﺿﻼع و ﻧﺘﻤﻢ اﳊﻞ ﺑﻨﻔﺲ اﻟﻄﺮﻳﻘﺔ اﻟﺴﺎﺑﻘﺔ‪.‬‬

‫‪.‬‬
‫𝟔𝟗𝟏‬ ‫‪.‬‬
‫∗‬
‫ﻟﻴﻜﻦ ‪ z ∈ ℂ‬و ‪ v ، u‬اﳉﺬرﻳﻦ اﻟﱰﺑﻴﻌﻴﲔ ﻟﻠﻌﺪد ‪. z‬‬
‫أوﺟﺪ ﳎﻤﻮﻋﺔ اﻷﻋﺪاد ∗‪ z ∈ ℂ‬ﺑﺤﻴﺚ ﺗُﺸﻜّﻞ اﻟﻨﻘﻂ اﻟﺘﻲ ﻟﻮاﺣﻘﻬﺎ ‪ v ، u ، z‬ﻣﺜﻠﺜ ًﺎ ﻗﺎﺋﲈ ﰲ اﻟﻨﻘﻄﺔ اﻟﺘﻲ ﻻﺣﻘﺘﻬﺎ ‪. z‬‬

‫ﺍﳊﻞّ‪ .‬اﻟﻄﺮﻳﻘﺔ اﻷوﱃ ‪) :‬ﻫﻨﺪﺳﻴﺔ( ﻧﺴﺘﻌﲔ ﺑﺎﻟﺸﻜﻞ ‪.1.VII‬‬


‫ﻟﺘﻜﻦ ‪ P‬اﻟﻨﻘﻄﺔ ذات اﻟﻼﺣﻘﺔ ‪ M ، z‬اﻟﻨﻘﻄﺔ ذات اﻟﻼﺣﻘﺔ ‪ u‬و ‪ N‬اﻟﻨﻘﻄﺔ ذات اﻟﻼﺣﻘﺔ ‪. v‬‬
‫ﻟﺪﻳﻨﺎ ‪ z = u = v‬و ‪ v = −u‬إذن ﻣﺒﺪأ اﳌﻌﻠﻢ ‪ O‬ﻫﻮ ﻣﻨﺘﺼﻒ اﻟﻘﻄﻌﺔ ]‪. [MN‬‬
‫اﳌﺜﻠﺚ ‪ MNP‬ﻗﺎﺋﻢ اﻟﺰاوﻳﺔ ﰲ اﻟﺮأس ‪ P‬إذا و ﻓﻘﻂ إذا إﻧﺘﻤﺖ اﻟﻨﻘﻄﺔ ‪ P‬إﱃ اﻟﺪاﺋﺮة اﻟﺘﻲ ﻣﺮﻛﺰﻫﺎ ‪ O‬و ‪ MN‬ﻗﻄﺮ ﳍﺎ أي إذا و‬
‫‪ņőƱŀ Ţ‬‬
‫ﻓﻘﻂ إذا ﻛﺎن ‪ . OM = OP‬ﻟﻜﻦ ‪:‬‬

‫|‪OM = 0P ⟺ |u| = |z| ⟺ |u| = ||u || ⟺ |u| = |u‬‬


‫‪⟺ |u| = 0‬‬ ‫أو‬ ‫‪|u| = 1‬‬
‫‪⟺ |u| = 1‬‬ ‫)‪ّ u ≠ 0‬‬
‫ﻷن ‪(z ≠ 0‬‬
‫‪⟺ |u| = 1‬‬
‫‪⟺ |z| = 1‬‬
‫‪ƕ‬‬

‫‪VII‬‬

‫إذن اﳌﺠﻤﻮﻋﺔ اﻟﺘﻲ ﻧﺒﺤﺚ ﻋﻨﻬﺎ ﻫﻲ ﳎﻤﻮﻋﺔ اﻷﻋﺪاد اﳌﺮﻛﺒﺔ اﻟﺘﻲ ﻃﻮﻳﻠﺘﻬﺎ ‪. 1‬‬
‫اﻟﻄﺮﻳﻘﺔ اﻟﺜﺎﻧﻴﺔ ‪) :‬ﺣﺴﺎﺑﻴﺔ( اﳌﺜﻠﺚ ‪ MNP‬ﻗﺎﺋﻢ ﰲ ‪ P‬إذا و ﻓﻘﻂ إذا ﻛﺎن ‪ PM ⟂ PN‬و ﺣﺴﺐ اﳌﱪﻫﻨﺔ ‪25‬ﺻﻔﺤﺔ ‪ ، 53‬ﻳﺘﺤﻘﻖ‬
‫ﻫﺬا إذا و ﻓﻘﻂ إذا ﻛﺎن ‪ . Re (u − z) ⋅ (v − z) = 0‬ﻟﻜﻦ ‪:‬‬
‫‪ũŏ‬‬

‫‪Re (u − z) ⋅ (v − z) = 0 ⟺ Re‬‬ ‫‪u−u‬‬ ‫‪−u − u‬‬ ‫‪=0‬‬


‫‪1‬‬
‫⟺‬ ‫‪u−u‬‬ ‫‪−u − u‬‬ ‫‪+‬‬ ‫‪u−u‬‬ ‫‪−u − u‬‬ ‫‪=0‬‬
‫‪2‬‬
‫⟺‬ ‫‪u−u‬‬ ‫‪−u − u‬‬ ‫‪+ u−u‬‬ ‫‪−u − u‬‬ ‫‪=0‬‬

‫‪⟺ −uu − uu + u u + u u − uu − uu + u u + u u = 0‬‬

‫𝟮𝟴𝟮‬

‫‪http ://tinyurl.com/Malki1718‬‬ ‫‪0‬‬


‫‪ .VII‬ﲤﺎرﻳﻦ ﻟﻠﺘﻌﻤﻖ‬
‫‪.‬‬
‫‪⟺ − |u| + ||u || − |u| + ||u || = 0‬‬
‫‪⟺ −2 |u| + 2 |u| = 0‬‬

‫‪Ŕž Ŧ Ľ‬‬
‫‪⟺ |u| = 0‬‬ ‫أو‬ ‫‪|u| = 1‬‬

‫‪œǃ‬‬
‫‪⟺ |u| = 1‬‬ ‫)‪ّ u ≠ 0‬‬
‫ﻷن ‪(z ≠ 0‬‬
‫‪⟺ |z| = 1‬‬

‫■‬ ‫و ﻧﺼﻞ إﱃ ﻧﻔﺲ اﻟﻨﺘﻴﺠﺔ‪.‬‬

‫‪Ŀ‬‬
‫‪ c ، b ، a‬و ‪ d‬أﻋﺪاد ﺣﻘﻴﻘﻴﺔ‪ّ .‬‬
‫ﺣﻞ ﰲ ‪ ℂ‬اﳌﻌﺎدﻟﺔ ذات اﳌﺠﻬﻮل ‪ z‬ﰲ اﳊﺎﻟﺘﲔ ‪:‬‬
‫‪$‬‬ ‫‪.‬‬
‫𝟕𝟗𝟏‬ ‫‪.‬‬

‫‪z − |z| = c + ıd‬‬ ‫•‬ ‫‪2‬‬ ‫‪z + |z| = a + ıb‬‬ ‫•‬‫‪1‬‬

‫‪ •1‬ﻧُﻤ ﱢﻴﺰ ﺣﺎﻟﺘﲔ ‪:‬‬ ‫ﺍﳊﻞّ‪.‬‬

‫إذا ﻛﺎن ‪ a + ıb = 0‬أي إذا ﻛﺎن ‪ّ a = b = 0‬‬


‫ﻓﺈن اﳌﻌﺎدﻟﺔ ﺗُﺼﺒﺢ ‪ . z = − |z| :‬و ﺑﺎﻟﺘﺎﱄ‪ ،‬إ ّﻣﺎ ‪ z = 0‬أو‬ ‫•‬

‫‪ |z| e arg = − |z| = |z| e‬أي إ ّﻣﺎ ‪ z = 0‬أو )𝜋‪. arg (z) = 𝜋 (mod 2‬‬
‫إذن ﳎﻤﻮﻋﺔ اﳊﻠﻮل ﻫﻲ ﳎﻤﻮﻋﺔ اﻷﻋﺪاد اﳊﻘﻴﻘﻴﺔ اﻟﺴﺎﻟﺒﺔ ‪) ℝ−‬و ‪. (0 ∈ ℝ−‬‬
‫ﻓﺈن ‪. z ∉ ℝ−‬‬‫إذا ﻛﺎن ‪ a + ıb ≠ 0‬ﻓﺤﺴﺐ ﻣﺎ ﺳﺒﻖ‪ّ ،‬‬ ‫•‬

‫ﻧﻀﻊ ‪ a + ıb = re‬ﻣﻊ ‪ r > 0‬و 𝜋 ≤ 𝜑 < 𝜋‪ −‬و ﻧﺒﺤﺚ ﻋﻦ اﳊﻠﻮل ﻋﲆ اﻟﺸﻜﻞ ‪ z = 𝜌e‬ﻣﻊ ‪ 𝜌 > 0‬و‬
‫𝜋 < 𝜃 < 𝜋‪ّ 𝜃 ≠ ±𝜋) −‬‬
‫ﻷن ‪ . (z ∉ ℝ−‬ﻟﺪﻳﻨﺎ ‪:‬‬
‫‪z + |z| = a + ıb ⟺ 𝜌e + 𝜌 = re‬‬ ‫‪⟺ 𝜌 1+e‬‬ ‫‪= re‬‬
‫‪/‬‬ ‫𝜃‬
‫‪⟺ 𝜌e‬‬ ‫‪e−‬‬ ‫‪/‬‬ ‫‪+e‬‬ ‫‪/‬‬ ‫‪= re‬‬ ‫‪⟺ 2𝜌e‬‬ ‫‪/‬‬ ‫‪cos‬‬ ‫‪= re‬‬
‫‪2‬‬
‫‪ņőƱŀ Ţ‬‬
‫𝜃‬
‫‪ 2𝜌e‬ﻣﻜﺘﻮب‬ ‫‪/‬‬ ‫ﻓﺈن ‪ cos 𝜃 > 0‬و ﺣﺴﺐ اﳌﱪﻫﻨﺔ ‪ 8‬ﺻﻔﺤﺔ ‪ّ 20‬‬
‫ﻓﺈن اﻟﻌﺪد ‪cos‬‬ ‫أن 𝜋 < 𝜃 < 𝜋 ‪ّ −‬‬ ‫ﺑﲈ ّ‬
‫‪2‬‬ ‫‪2‬‬ ‫‪2‬‬ ‫‪2‬‬ ‫‪2‬‬
‫𝜋‬ ‫𝜃‬ ‫𝜋‬ ‫𝜃‬ ‫𝜃‬
‫ﻋﲆ ﺷﻜﻠﻪ اﻷﳼ إذن ‪ 2𝜌 cos = r :‬و )𝜋‪ . = 𝜑 (mod 2‬ﻟﻜﻦ < < ‪ ، −‬إذن إذا ﻛﺎن‬
‫‪2‬‬ ‫‪2‬‬ ‫‪2‬‬ ‫‪2‬‬ ‫‪2‬‬
‫𝜋 𝜋‬
‫أي ّ‬
‫ﺣﻞ‪ .‬ﻟﻜﻦ ‪:‬‬ ‫‪ 𝜑 ∉ − 2 , 2‬ﻓﺎﳌﻌﺎدﻟﺔ ﻻ ﺗﻘﺒﻞ ّ‬
‫𝜋 𝜋‬ ‫𝜃‬
‫‪𝜑∉ − ,‬‬ ‫‪⟺ cos ≤ 0 ⟺ Re (a + ıb) ≤ 0 ⟺ a ≤ 0‬‬
‫‪2 2‬‬ ‫‪2‬‬
‫أي ّ‬
‫ﺣﻞ إذا ﻛﺎن ‪. a ≤ 0‬‬ ‫إذن اﳌﻌﺎدﻟﺔ ﻻ ﺗﻘﺒﻞ ّ‬
‫‪ƕ‬‬

‫‪r‬‬
‫= 𝜌 ﻣﻨﻪ ‪:‬‬ ‫ﻓﺈن )𝜋‪ 𝜃 = 2𝜑 (mod 2‬و‬‫إذا ﻛﺎن ‪ّ a > 0‬‬
‫‪VII‬‬

‫𝜑 ‪2 cos‬‬
‫‪r‬‬ ‫‪r‬‬
‫=‪z‬‬ ‫‪e‬‬ ‫=‬ ‫))𝜑‪(cos (2𝜑) + ı sin (2‬‬
‫𝜑 ‪2 cos‬‬ ‫𝜑 ‪2 cos‬‬
‫‪ũŏ‬‬

‫‪r‬‬ ‫‪r‬‬
‫=‬ ‫= 𝜑 ‪cos 𝜑 − sin 𝜑 + 2ı cos 𝜑 sin‬‬ ‫𝜑 ‪cos 𝜑 − sin 𝜑 + 2ı cos 𝜑 sin‬‬
‫𝜑 ‪2 cos‬‬ ‫𝜑 ‪2r cos‬‬
‫‪1‬‬ ‫‪1‬‬ ‫‪a −b‬‬
‫=‬ ‫= )𝜑 ‪(r cos 𝜑) − (r sin 𝜑) + 2ı (r cos 𝜑) (r sin‬‬ ‫= ‪a − b + 2ıab‬‬ ‫‪+ ıb‬‬
‫‪2a‬‬ ‫‪2a‬‬ ‫‪2a‬‬

‫‪a −b‬‬
‫= ‪.z‬‬ ‫إذن ﻟﻠﻤﻌﺎدﻟﺔ ﺣﻞ وﺣﻴﺪ و ﻫﻮ ‪+ ıb :‬‬
‫‪2a‬‬

‫𝟯𝟴𝟮‬

‫‪http ://tinyurl.com/Malki1718‬‬ ‫‪0‬‬


‫‪.‬‬

‫‪Ŕž Ŧ Ľ‬‬
‫‪y‬‬

‫‪p‬‬
‫=‬
‫‪a‬‬
‫‪2‬‬
‫‪−‬‬
‫‪2a‬‬
‫‪a2 − b2‬‬
‫‬ ‫‬

‫‪œǃ‬‬
‫‪x‬‬
‫‪M‬‬ ‫‪,b‬‬
‫‪2a‬‬ ‫)‪A (a + ıb‬‬

‫‪Ŀ‬‬
‫)|‪N (|z‬‬
‫‪O‬‬

‫ﺷﻜﻞ ‪2.VII‬‬

‫ﻣﻼﺣﻈﺔ ‪ ⧏ : 47‬ﻳﻤﻜﻦ أﻳﻀ ًﺎ إﺳﺘﻌﲈل ااﻟﺸﻜﻞ اﳉﱪي ﻹﳚﺎد اﳊﻠﻮل ‪ :‬ﻧﻀﻊ ‪ z = x + ıy‬و ﻧُﺬﻛﱢﺮ ّ‬
‫أن‬
‫ﻓﺈن ‪ x + x + y > 0‬ﻣﻨﻪ ‪:‬‬‫أن ‪ّ z ∉ ℝ−‬‬
‫|‪ − Re (z) ≤ |z‬أي ‪ x + x + y ≥ 0‬و ﺑﲈ ّ‬
‫‪⎧x +‬‬ ‫‪x +y =a‬‬
‫‪z + |z| = a + ıb ⟺ x +‬‬ ‫⟺ ‪x + y + ıy = a + ıb‬‬
‫⎨‬ ‫‪y=b‬‬
‫⎩‬
‫‪ . (x +‬إذن ‪ a > 0‬ﻣﻨﻪ ‪:‬‬ ‫ﺣﻞ ّ‬
‫)ﻷن ‪x + y > 0‬‬ ‫أي ّ‬
‫إذن‪ ،‬إذا ﻛﺎن ‪ a ≤ 0‬ﻓﺎﳌﻌﺎدﻟﺔ ﻻ ﺗﻘﺒﻞ ّ‬

‫‪⎧x +‬‬ ‫⎧‬


‫‪⎪ x+‬‬
‫‪x +y =a‬‬ ‫‪x +y‬‬ ‫‪=a‬‬
‫⟺‬
‫⎨‬ ‫‪y=b‬‬ ‫⎨‬
‫⎪‬
‫‪ņőƱŀ Ţ‬‬
‫⎩‬ ‫⎩‬ ‫‪y=b‬‬
‫=‬
‫⎧‬
‫⎪‬
‫⎣‬
‫⎢‬
‫⎢‬
‫⎢‬
‫⎢‬
‫⎡‬

‫⎪‬ ‫⎧‬
‫‪⎪x= a −b‬‬
‫⟺‬ ‫‪2x x +‬‬ ‫‪x +y‬‬ ‫‪=a −y‬‬ ‫⟺‬ ‫‪2a‬‬
‫⎨‬
‫⎪‬
‫⎪‬ ‫⎨‬
‫⎪‬
‫‪y=b‬‬ ‫⎩‬ ‫‪y‬‬ ‫=‬ ‫‪b‬‬
‫⎩‬

‫‪a −b‬‬
‫= ‪.z‬‬ ‫و ﻧﺼﻞ إﱃ ﻧﻔﺲ اﻟﻨﺘﻴﺠﺔ ‪ :‬ﻟﻠﻤﻌﺎدﻟﺔ ﺣﻞ وﺣﻴﺪ و ﻫﻮ ‪+ ıb‬‬
‫‪2a‬‬
‫⧐‬
‫اﻟﺸﻜﻞ ‪ُ 2.VII‬ﻳ ﱢﺒﲔ ﻣﻮﺿﻊ اﻟﻨﻘﻄﺔ )‪ M (z‬ﺑﺪﻻﻟﺔ ﻣﻮﺿﻊ اﻟﻨﻘﻄﺔ )‪) A (a + ıb‬ﻓﺎﺻﻠﺘﻬﺎ ‪ a‬ﻣﻮﺟﺒﺔ ﲤﺎﻣ ًﺎ( ‪ .‬اﻟﺮﺑﺎﻋﻲ‬
‫‪ƕ‬‬

‫‪VII‬‬

‫أدق‪ ،‬اﻟﻨﻘﻄﺔ )|‪ N (|z‬ﻫﻲ ﻧﻘﻄﺔ ﺗﻘﺎﻃﻊ ﻧﺼﻒ اﳌﺴﺘﻘﻴﻢ ‪ x ≥ 0, y = 0‬ﻣﻊ ﳏﻮر اﻟﻘﻄﻌﺔ‬ ‫ﻌﲔ‪ .‬ﺑﺘﻌﺒﲑ ّ‬ ‫‪ُ ONAM‬ﻣ ﱠ‬
‫اﳌﺴﺘﻘﻴﻤﺔ ]‪ [OA‬و اﻟﻨﻘﻄﺔ )‪ M (z‬ﻫﻲ ﻧﻈﲑة ‪ N‬ﺑﺎﻟﻨﺴﺒﺔ إﱃ اﳌﺴﺘﻘﻴﻢ )‪ . (OA‬ﻋﻼو ًة ﻋﲆ ذﻟﻚ‪ ،‬ﻟﺪﻳﻨﺎ ‪:‬‬
‫‪ũŏ‬‬

‫‪x+‬‬ ‫‪x + y = a ⟺ (x − a) = x + y‬‬ ‫‪⟺ y = a − 2ax‬‬


‫إذن اﻟﻨﻘﻄﺔ )‪ M (z‬ﺗﻨﺘﻤﻲ إﱃ اﳌﻨﺤﻨﻰ )ﻗﻄﻊ ﻣﻜﺎﻓﺊ( اﻟﺬي ﻣﻌﺎدﻟﺘﻪ ‪. y = a − 2ax :‬‬
‫‪ •2‬ﻟﺪﻳﻨﺎ ‪:‬‬
‫‪z − |z| = c + ıd ⟺ −z + |z| = −c − ıd ⟺ −z + |−z| = −c − ıd ⟺ Z + ||Z|| = a + ıb‬‬
‫ﺣﻴﺚ ‪ a = −c ، Z = −z‬و ‪. b = −d‬‬
‫و ﺣﺴﺐ اﻟﺴﺆال اﻷول ‪:‬‬

‫𝟰𝟴𝟮‬

‫‪http ://tinyurl.com/Malki1718‬‬ ‫‪0‬‬


‫‪ .VII‬ﲤﺎرﻳﻦ ﻟﻠﺘﻌﻤﻖ‬
‫‪.‬‬
‫إذا ﻛﺎن ‪ a + ıb = 0‬أي إذا ﻛﺎن ‪ −c − ıd = 0‬أي إذا ﻛﺎن ‪ّ c = d = 0‬‬
‫ﻓﺈن اﳊﻠﻮل ﻫﻲ اﻷﻋﺪاد ‪−z ∈ ℝ−‬‬ ‫•‬

‫أي ‪. z ∈ ℝ+‬‬

‫‪Ŕž Ŧ Ľ‬‬
‫ﳎﻤﻮﻋﺔ اﳊﻠﻮل ﻫﻲ ﳎﻤﻮﻋﺔ اﻷﻋﺪاد اﳊﻘﻴﻘﻴﺔ اﳌﻮﺟﺒﺔ ‪) ℝ+‬و ‪. (0 ∈ ℝ+‬‬
‫أي ّ‬

‫‪œǃ‬‬
‫ﺣﻞ إذا‬ ‫إذا ﻛﺎن ‪ a + ıb ≠ 0‬أي إذا ﻛﺎن ‪ −c − ıd ≠ 0‬أي إذا ﻛﺎن ‪ c + ıd ≠ 0‬ﻓﺈﻧﻪ ﻟﻴﺲ ﻟﻠﻤﻌﺎدﻟﺔ ّ‬ ‫•‬

‫ﻛﺎن ‪ a ≤ 0‬أي إذا ﻛﺎن ‪ c ≥ 0‬؛ و ﻟﻠﻤﻌﺎدﻟﺔ ّ‬


‫ﺣﻞ وﺣﻴﺪ إذا ﻛﺎن ‪ a > 0‬أي إذا ﻛﺎن ‪ c < 0‬و ﻫﺬا اﳊﻞ ﻫﻮ‬
‫‪d −c‬‬ ‫‪a −b‬‬ ‫‪c −d‬‬
‫= ‪.z‬‬ ‫= ‪ z‬أي ‪− ıd‬‬ ‫= ‪+ ıb‬‬ ‫‪− ıb‬‬
‫‪2c‬‬ ‫‪2a‬‬ ‫‪−2c‬‬

‫‪Ŀ‬‬
‫■‬

‫اﳌﺴﺘﻮي اﳌﺮﻛﺐ ﻣﻨﺴﻮب إﱃ ﻣﻌﻠﻢ ﻣﺘﻌﺎﻣﺪ و ﻣﺘﺠﺎﻧﺲ ⃗ ‪. O, i,⃗ j‬‬


‫!‬ ‫‪.‬‬
‫𝟖𝟗𝟏‬ ‫‪.‬‬

‫ﻟﺘﻜﻦ 𝒞 اﻟﺪاﺋﺮة اﻟﺘﻲ ﻣﺮﻛﺰﻫﺎ )𝜔( ‪ Ω‬و ﻧﺼﻒ ﻗﻄﺮﻫﺎ ‪) r‬ﺣﻴﺚ ‪. ( r > 0‬‬
‫‪ •1‬أﺛﺒﺖ أﻧﻪ ﻣﻬﲈ ﺗﻜﻦ اﻟﻨﻘﻄﺔ )‪ M (z‬ﻣﻦ اﳌﺴﺘﻮي ّ‬
‫ﻓﺈن ‪:‬‬
‫‪M ∈ 𝒞 ⟺ ∃𝜃 ∈ ℝ , z = 𝜔 + re‬‬

‫‪ •2‬إﺳﺘﻨﺘﺞ أﻧﻪ إذا ﻛﺎﻧﺖ )‪ C (c) ، B (b) ، A (a‬و )‪ D (d‬ﻧﻘﻄ ًﺎ ﻣﻦ 𝒞 ّ‬


‫ﻓﺈن ‪:‬‬
‫‪c−a‬‬ ‫‪d−a‬‬
‫‪arg‬‬ ‫‪= arg‬‬ ‫)𝜋 ‪(mod‬‬
‫‪c−b‬‬ ‫‪d−b‬‬

‫‪ •3‬أﺛﺒﺖ أﻧﻪ ﻣﻬﲈ ﺗﻜﻦ اﻟﻨﻘﻄﺔ )‪M (z‬ﻣﻦ اﳌﺴﺘﻮي ّ‬


‫ﻓﺈن ‪:‬‬
‫‪M ∈ 𝒞 ⟺ |z| − 𝜔z − 𝜔z + |𝜔| = r‬‬

‫‪ •1‬ﻟﺘﻜﻦ )‪ M (z‬ﻧﻘﻄﺔ ﻣﻦ اﳌﺴﺘﻮي‪ .‬ﻟﺪﻳﻨﺎ ‪:‬‬ ‫ﺍﳊﻞّ‪.‬‬


‫‪ņőƱŀ Ţ‬‬
‫‪M ∈ 𝒞 ⟺ ΩM = r ⟺ |z − 𝜔| = r ⟺ ∃𝜃 ∈ ℝ , z − 𝜔 = re‬‬ ‫‪⟺ ∃𝜃 ∈ ℝ , z = 𝜔 + re‬‬

‫‪ •2‬ﻟﺘﻜﻦ )‪ C (c) ، B (b) ، A (a‬و )‪ D (d‬ﻧﻘﻄ ًﺎ ﻣﻦ 𝒞 ‪ .‬ﺣﺴﺐ اﻟﺴﺆال اﻟﺴﺎﺑﻖ ﻟﺪﻳﻨﺎ ‪:‬‬
‫‪ c = 𝜔 + re‬و ‪d = 𝜔 + re‬‬ ‫‪، b = 𝜔 + re ، a = 𝜔 + re‬‬
‫ﺣﻴﺚ 𝛼 ‪ 𝛾 ، 𝛽 ،‬و 𝛿 أﻋﺪاد ﺣﻘﻴﻘﻴﺔ‪ .‬ﻣﻨﻪ ‪:‬‬
‫‪c − a r (e − e ) e − e‬‬
‫=‬ ‫=‬
‫‪c−b r e −e‬‬ ‫‪e −e‬‬
‫‪e −e‬‬ ‫‪c−a‬‬
‫= ‪ . 𝜌e‬ﺑﺄﺧﺬ اﳌﺮاﻓﻖ ﰲ َ ْ‬
‫ﻃﺮﰲ ﻫﺬه اﳌﺴﺎواة ﻳﻨﺘﺞ ‪:‬‬ ‫)ﺣﻴﺚ ‪ (𝜌 > 0‬ﻣﻨﻪ ‪:‬‬ ‫‪= 𝜌e‬‬ ‫ﻧﻀﻊ‬
‫‪ƕ‬‬

‫‪e −e‬‬ ‫‪c−b‬‬


‫‪VII‬‬

‫‪e− − e−‬‬ ‫‪−‬‬ ‫‪e −e‬‬ ‫‪e ⋅e‬‬


‫‪𝜌e−‬‬ ‫=‬ ‫=‬ ‫=‬ ‫⋅‬ ‫‪= 𝜌r ⋅ e‬‬ ‫‪−‬‬
‫‪e− − e−‬‬ ‫‪−‬‬ ‫‪e ⋅e‬‬ ‫‪e −e‬‬
‫‪ũŏ‬‬

‫‪ e‬ﻣﻨﻪ )𝜋‪ 2𝜃 = 𝛼 − 𝛽 (mod 2‬و ﺑﻘﺴﻤﺔ اﻟﻄﺮﻓﲔ ﻋﲆ‬ ‫ﻓﺈن ‪= e −‬‬ ‫ﻣﻨﻪ ‪ 𝜌e = 𝜌e −‬و ﺑﲈ ّ‬
‫أن ‪ّ 𝜌 > 0‬‬
‫‪c−a‬‬ ‫𝛽‪𝛼−‬‬ ‫𝛽‪𝛼−‬‬
‫‪. arg‬‬ ‫=‬ ‫= 𝜃 أي )𝜋 ‪(mod‬‬ ‫‪ 2‬ﻳﻨﺘﺞ ‪(mod 𝜋) :‬‬
‫‪c−b‬‬ ‫‪2‬‬ ‫‪2‬‬
‫‪d−a‬‬ ‫𝛽‪𝛼−‬‬
‫‪. arg‬‬ ‫=‬ ‫ﻧﱪﻫﻦ ﺑﺎﳌﺜﻞ ّ‬
‫أن ‪(mod 𝜋) :‬‬
‫‪d−b‬‬ ‫‪2‬‬
‫‪c−a‬‬ ‫‪d−a‬‬
‫‪. arg‬‬ ‫‪= arg‬‬ ‫ﰲ اﻷﺧﲑ ‪(mod 𝜋) :‬‬
‫‪c−b‬‬ ‫‪d−b‬‬

‫𝟱𝟴𝟮‬

‫‪http ://tinyurl.com/Malki1718‬‬ ‫‪0‬‬


‫‪.‬‬
‫‪ •3‬ﻟﺘﻜﻦ )‪ M (z‬ﻧﻘﻄﺔ ﻣﻦ اﳌﺴﺘﻮي‪ .‬ﻟﺪﻳﻨﺎ ‪:‬‬

‫‪Ŕž Ŧ Ľ‬‬
‫‪M ∈ 𝒞 ⟺ |z − 𝜔| = r ⟺ |z − 𝜔| = r‬‬ ‫‪⟺ (z − 𝜔) (z − 𝜔) = r‬‬
‫‪⟺ (z − 𝜔) (z − 𝜔) = r‬‬ ‫‪⟺ zz − z𝜔 − 𝜔z + 𝜔𝜔 = r‬‬

‫‪œǃ‬‬
‫‪⟺ |z| − 𝜔z − 𝜔z + |𝜔| = r‬‬

‫■‬

‫‪Ŀ‬‬
‫‪.‬‬ ‫⃗ ‪O, i,⃗ j‬‬ ‫اﳌﺴﺘﻮي اﳌﺮﻛﺐ ﻣﻨﺴﻮب إﱃ ﻣﻌﻠﻢ ﻣﺘﻌﺎﻣﺪ و ﻣﺘﺠﺎﻧﺲ‬
‫!‬ ‫‪.‬‬
‫𝟗𝟗𝟏‬ ‫‪.‬‬

‫‪1‬‬
‫= ‪. OM ⋅ OM‬‬ ‫‪zz + zz‬‬ ‫‪ •1‬ﻟﺘﻜﻦ )‪ M (z‬و ‪ M z‬ﻧﻘﻄﺘﲔ ﻣﻦ اﳌﺴﺘﻮي‪ .‬أﺛﺒﺖ ّ‬
‫أن ‪:‬‬
‫‪2‬‬
‫‪1‬‬
‫= ‪. AM ⋅ AB‬‬ ‫)‪(z − a) b − a + (z − a) (b − a‬‬ ‫‪ •2‬ﻧﻌﺘﱪ اﻟﻨﻘﻂ )‪ B (b) ، A (a‬و )‪ . M (z‬أﺛﺒﺖ ّ‬
‫أن ‪:‬‬
‫‪2‬‬
‫‪ •3‬ﺗﻄﺒﻴﻖ ‪:‬‬
‫)ا( ﻟﻴﻜﻦ ‪ a‬و ‪ b‬ﻋﺪدﻳﻦ ﻣﺮﻛﺒﲔ ﻃﻮﻳﻠﺘﻬﲈ ‪ 1‬ﺑﺤﻴﺚ ‪ a ≠ b‬و ﻟﺘﻜﻦ ‪ A‬و ‪ B‬ﻻﺣﻘﺘ ْﻴﻬﲈ ﻋﲆ اﻟﱰﺗﻴﺐ‪ .‬ﻟﺘﻜﻦ ‪ C‬ﻧﻘﻄﺔ‬
‫ﻻﺣﻘﺘﻬﺎ ‪ .c‬ﻧﻌﺘﱪ اﳌﺴﺘﻘﻴﻢ )‪ (Δ‬اﳌﺎر ﺑﺎﻟﻨﻘﻄﺔ )‪ C (c‬و اﻟﻌﻤﻮدي ﻋﲆ اﳌﺴﺘﻘﻴﻢ )‪ . (AB‬أﺛﺒﺖ أﻧﻪ ﻣﻬﲈ ﺗﻜﻦ اﻟﻨﻘﻄﺔ‬
‫)‪ّ M (z‬‬
‫ﻓﺈن ‪:‬‬
‫‪M ∈ (Δ) ⟺ z − abz = c − abc‬‬

‫)ب( ﻧﻌﺘﱪ اﻟﻨﻘﻂ )‪ C (c) ، B (b) ، A (a‬و )‪ D (d‬ﺑﺤﻴﺚ ‪ . ||a − b|| = ||c − d|| = 1‬أﺛﺒﺖ ّ‬
‫أن ‪:‬‬
‫‪(AB) ⟂ (CD) ⟺ (b − a) + (d − c) = 0‬‬
‫ﻧﻌﺘﱪ اﻟﻨﻘﻂ )‪ B (b) ، A (a‬و )‪ C (c‬ﺑﺤﻴﺚ ‪ a ≠ b‬و ||‪ . |a| = ||b‬ﻟﺘﻜﻦ اﻟﻨﻘﻄﺔ ‪ H‬اﳌﺴﻘﻂ اﻟﻌﻤﻮدي ﻟﻠﻨﻘﻄﺔ ‪C‬‬ ‫)ج(‬
‫ﻋﲆ اﳌﺴﺘﻘﻴﻢ )‪ . (AB‬أﺛﺒﺖ ّ‬
‫أن ‪:‬‬
‫‪AH 1‬‬ ‫‪c − abc‬‬
‫‪1+‬‬
‫‪ņőƱŀ Ţ‬‬
‫=‬
‫‪AB‬‬ ‫‪2‬‬ ‫‪b−a‬‬

‫‪ •1‬ﻻﺣﻘﺔ ‪ OM‬ﻫﻲ ‪ z − 0 = z‬و ﻻﺣﻘﺔ ‪ OM‬ﻫﻲ ‪ ، z − 0 = z‬و ﺣﺴﺐ اﳌﱪﻫﻨﺔ ‪ 25‬ﺻﻔﺤﺔ ‪ 53‬ﻟﺪﻳﻨﺎ ‪:‬‬ ‫ﺍﳊﻞّ‪.‬‬
‫‪1‬‬ ‫‪1‬‬
‫‪OM ⋅ OM = Re z ⋅ z‬‬ ‫=‬ ‫‪z⋅z +z⋅z‬‬ ‫=‬ ‫‪z⋅z +z⋅z‬‬
‫‪2‬‬ ‫‪2‬‬

‫‪ •2‬ﻳﻤﻜﻦ إﺛﺒﺎت ﻫﺬه اﻟﻨﺘﻴﺠﺔ ﺑﻨﻔﺲ اﻟﻄﺮﻳﻘﺔ اﻟﺴﺎﺑﻘﺔ أو ﻛﲈ ﻳﲇ ‪ :‬ﻟﺘﻜﻦ ‪ C‬اﻟﻨﻘﻄﺔ اﻟﺘﻲ ﲢﻘﻖ ‪ AB = OC‬و ‪ D‬اﻟﻨﻘﻄﺔ اﻟﺘﻲ‬
‫ﻓﺈن ‪ c−0 = b−a‬أي ‪ c = b−a‬و ‪d−0 = z−a‬‬ ‫ﲢﻘﻖ ‪ . AM = OD‬إذا ﻛﺎﻧﺖ ‪ c‬ﻻﺣﻘﺔ ‪ C‬و ﻛﺎﻧﺖ ‪ d‬ﻻﺣﻘﺔ ‪ّ D‬‬
‫‪ƕ‬‬

‫‪VII‬‬

‫أي ‪ d = z − a‬و ﺣﺴﺐ اﻟﺴﺆال اﻟﺴﺎﺑﻖ ‪:‬‬


‫‪1‬‬ ‫‪1‬‬
‫= ‪AM ⋅ AB = OD ⋅ OC‬‬ ‫= ‪cd + dc‬‬ ‫)‪(b − a) (z − a) + (z − a) (b − a‬‬
‫‪2‬‬ ‫‪2‬‬
‫‪ũŏ‬‬

‫‪1‬‬
‫=‬ ‫)‪(z − a) b − a + (z − a) (b − a‬‬
‫‪2‬‬

‫𝟲𝟴𝟮‬

‫‪http ://tinyurl.com/Malki1718‬‬ ‫‪0‬‬


‫‪ .VII‬ﲤﺎرﻳﻦ ﻟﻠﺘﻌﻤﻖ‬
‫‪.‬‬
‫‪1‬‬ ‫‪1‬‬
‫= ‪ . b‬ﻟﺘﻜﻦ ‪ M‬ﻧﻘﻄﺔ ﻣﻦ اﳌﺴﺘﻮي ﻻﺣﻘﺘﻬﺎ ‪ . z‬ﻟﺪﻳﻨﺎ ‪:‬‬ ‫= ‪a‬و‬ ‫أن ‪ّ |a| = ||b|| = 1‬‬
‫ﻓﺈن‬ ‫)ا( ﺑﲈ ّ‬ ‫‪3‬‬
‫•‬
‫‪b‬‬ ‫‪a‬‬

‫‪Ŕž Ŧ Ľ‬‬
‫‪1‬‬
‫⟺ ‪M ∈ (Δ) ⟺ CM ⋅ AB = 0‬‬ ‫‪(z − c) b − a + (z − c) (b − a) = 0‬‬
‫‪2‬‬

‫‪œǃ‬‬
‫‪1 1‬‬
‫)‪⟺ (z − c) b − a + (z − c) (b − a) = 0 = ⟺ (z − c‬‬ ‫‪−‬‬ ‫‪+ (z − c) (b − a) = 0‬‬
‫‪b a‬‬
‫‪a−b‬‬
‫⟺‬ ‫‪[z − c − ab (z − c)] = 0 ⟺ z − c − abz + abc = 0‬‬
‫‪ab‬‬

‫‪Ŀ‬‬
‫‪⟺ z − abz = c − abc‬‬

‫‪1‬‬ ‫‪1‬‬
‫= ‪ d − c‬ﻣﻨﻪ ‪:‬‬ ‫= ‪b−a‬و‬ ‫أن ‪ّ ||b − a|| = ||d − c|| = 1‬‬
‫ﻓﺈن‬ ‫)ب( ﺑﲈ ّ‬
‫‪d−c‬‬ ‫‪b−a‬‬
‫‪1‬‬
‫⟺ ‪(AB) ⟂ (CD) ⟺ AB ⋅ CD = 0‬‬ ‫‪(b − a) d − c + b − a (d − c) = 0‬‬
‫‪2‬‬
‫‪b−a d−c‬‬
‫⟺ ‪⟺ (b − a) (d − c) + (b − a) (d − c) = 0‬‬ ‫‪+‬‬ ‫‪=0‬‬
‫‪d−c b−a‬‬
‫‪b−a‬‬ ‫‪d−c‬‬
‫⟺‬ ‫‪=−‬‬ ‫)‪⟺ (b − a) = − (d − c‬‬
‫‪d−c‬‬ ‫‪b−a‬‬
‫‪⟺ (b − a) + (d − c) = 0‬‬

‫)ج( ﻟﺘﻜﻦ ‪ h‬ﻻﺣﻘﺔ ‪ . H‬ﺑﲈ ّ‬


‫أن ‪ C‬ﺗﻨﺘﻤﻲ إﱃ اﳌﺴﺘﻘﻴﻢ اﻟﻌﻤﻮدي ﻋﲆ )‪ (AB‬ﰲ ‪ H‬ﻓﺤﺴﺐ اﻟﺴﺆال )‪•3‬ا( ﻳﻜﻮن ‪:‬‬
‫‪AH‬‬ ‫‪AH‬‬
‫= ‪ h − a‬أي‬ ‫= ‪ AH‬ﻣﻨﻪ )‪(b − a‬‬ ‫أن )‪ّ H ∈ (AB‬‬
‫ﻓﺈن ‪AB‬‬ ‫‪ . c − abc = h − abh‬و ﺑﲈ ّ‬
‫‪AB‬‬ ‫‪AB‬‬
‫‪h−a h−a‬‬ ‫‪h−a‬‬ ‫‪h−a‬‬ ‫‪h−a‬‬
‫‪.‬‬ ‫=‬ ‫أي‬ ‫=‬ ‫ﻣﻨﻪ‬ ‫‪ AH = h − a‬و ﻫﺬا ﻳﻌﻨﻲ ّ‬
‫أن ‪∈ ℝ‬‬
‫‪b−a b−a‬‬ ‫‪b−a‬‬ ‫‪b−a‬‬ ‫‪b−a‬‬ ‫‪AB‬‬ ‫‪b−a‬‬
‫‪h−a‬‬ ‫‪h−a‬‬ ‫‪h−a‬‬ ‫‪h−a‬‬ ‫‪h−a‬‬ ‫‪1‬‬ ‫‪1‬‬
‫⋅‪= ab‬‬ ‫أي‬ ‫=‬ ‫⋅‪= ab‬‬ ‫ﻟﻜﻦ ‪ |a| = ||b|| = 1‬إذن = ‪ a‬و = ‪ b‬ﻣﻨﻪ‬
‫‪b−a‬‬ ‫‪a−b‬‬ ‫‪b−a‬‬ ‫‪−‬‬ ‫‪a−b‬‬ ‫‪b‬‬ ‫‪a‬‬
‫ﻣﻨﻪ ‪ h − a = −ab h − a‬أي ‪ّ h + abh = a + aab = a + b‬‬
‫)ﻷن ‪. (aa = |a| = 1‬‬
‫‪ņőƱŀ Ţ‬‬
‫ﰲ اﻷﺧﲑ‪ ،‬ﻟﺪﻳﻨﺎ ‪ c − abc = h − abh :‬و ‪ . h + abh = a + b‬ﺑﺠﻤﻊ ﻫﺎﺗﲔ اﳌﺴﺎوﺗﲔ ﻃﺮﻓ ًﺎ إﱃ ﻃﺮف‬
‫‪1‬‬
‫ﻳﻨﺘﺞ ‪ 2h = a + b + c − abc :‬ﻣﻨﻪ )‪ (h − a) = (b − a + c − abc‬و ﺑﺎﻟﺘﺎﱄ ‪:‬‬
‫‪2‬‬
‫‪AH‬‬ ‫‪(b − a + c − abc) 1‬‬ ‫‪c − abc‬‬
‫=‬ ‫=‬ ‫‪1+‬‬
‫‪AB‬‬ ‫‪b−a‬‬ ‫‪2‬‬ ‫‪b−a‬‬
‫■‬

‫!‬ ‫‪.‬‬
‫𝟎𝟎𝟐‬ ‫‪.‬‬
‫‪ƕ‬‬

‫أي ﳾء ﻋﻦ‬
‫ﰲ ﻫﺬا اﻟﺘﻤﺮﻳﻦ‪ b ، a ،‬و ‪ c‬ﺛﻼﺛﺔ أﻋﺪاد ﻣﺮﻛﺒﺔ ﳐﺘﻠﻔﺔ ﻣﺜﻨﻰ ﻣﺜﻨﻰ‪ .‬ﺳﻨﻔﺮض أﻧﻨﺎ ﻻ ﻧﻌﺮف ّ‬
‫‪VII‬‬

‫اﳌﺜﻠﺜﺎت اﳌﺘﻘﺎﻳﺴﺔ اﻷﺿﻼع ﺳﻮى اﻟﺘﻌﺮﻳﻒ اﻟﺘﺎﱄ ‪:‬‬


‫ﺗﻌﺮﻳﻒ ‪ :‬ﻧﻘﻮل ﻋﻦ اﳌﺜﻠﺚ ‪ abc‬أﻧﻪ ‪:‬‬
‫‪ũŏ‬‬

‫ﻣﺒﺎﺷﺮ إذا ﻛﺎن اﻷﺳﺎس ‪ ab, ac‬ﻣﺒﺎﴍ ًا أي إذا ﻛﺎن ﻟﻠﺰاوﻳﺔ ‪ ab, ac‬ﻗﻴﺎس ﰲ [𝜋 ‪. ]0,‬‬ ‫•‬

‫ﻣﺘﻘﺎﻳﺲ ﺍﻷﺿﻼﻉ إذا ﻛﺎن |‪. ||a − b|| = ||b − c|| = |c − a‬‬ ‫•‬

‫و ﻫﺪﻓﻨﺎ ﻫﻮ إﺛﺒﺎت ﺑﻌﺾ اﻟﻨﺘﺎﺋﺞ اﳌﻌﺮوﻓﺔ و اﳌﺴﺘﺨﺪﻣﺔ )دون إﺛﺒﺎت( ﺣﻮل اﳌﺜﻠﺜﺎت اﳌﺘﻘﺎﻳﺴﺔ اﻷﺿﻼع‪.‬‬

‫𝟳𝟴𝟮‬

‫‪http ://tinyurl.com/Malki1718‬‬ ‫‪0‬‬


‫‪.‬‬
‫‪c−a‬‬ ‫‪c−a‬‬
‫‪ .‬ﺑﺎﳌﺜﻞ‪ ،‬ﻧﻌﺘﱪ اﻷﻋﺪاد اﳊﻘﻴﻘﻴﺔ ‪ 𝛽 ، C ، B‬و 𝛾 اﳌﻌﺮﻓﺔ‬ ‫‪= Ae‬‬ ‫ﺑﻤﻌﻨﻰ‬ ‫ﻟﻴﻜﻦ ‪ A‬ﻃﻮﻳﻠﺔ و 𝛼 ﻋﻤﺪ ًة ﻟﻠﻌﺪد‬

‫‪Ŕž Ŧ Ľ‬‬
‫‪b−a‬‬ ‫‪b−a‬‬
‫‪b−c‬‬ ‫‪a−b‬‬
‫‪.‬‬ ‫و ‪= Ce‬‬ ‫ﺑﺎﻟﻌﻼﻗﺎت ‪= Be :‬‬
‫‪a−c‬‬ ‫‪c−b‬‬

‫‪œǃ‬‬
‫أﺧﲑ ًا‪ ،‬ﻧﻀﻊ ‪) j = e /‬و ﻧُﺬﻛّﺮ ّ‬
‫أن ‪ j = 1‬و ‪.(1 + j + j = 0‬‬

‫أن ‪ ABC = 1 :‬و )𝜋‪ . 𝛼 + 𝛽 + 𝛾 = 𝜋 (mod 2‬ﻫﻞ ُﻳﻌﺘﱪ ﻫﺬا إﻛﺘﺸﺎﻓ ًﺎ ﺟﺪﻳﺪ ًا ؟‬
‫)ا( أﺛﺒﺖ ّ‬ ‫•‬ ‫‪1‬‬
‫‪1‬‬ ‫‪1‬‬ ‫‪1‬‬
‫‪. Ce +‬‬ ‫‪ Be‬و ‪= 1‬‬ ‫‪+‬‬ ‫‪= 1 ، Ae‬‬ ‫‪+‬‬ ‫ﲢﻘﻖ ﻣﻦ ّ‬
‫أن ‪= 1 :‬‬ ‫)ب(‬

‫‪Ŀ‬‬
‫‪Ae‬‬ ‫‪Ce‬‬ ‫‪Be‬‬
‫‪ •2‬ﻓﻴﲈ ﻳﲇ‪ ،‬ﻧُﺜﺒﺖ اﻟﺘﻜﺎﻓﺆ ‪:‬‬
‫اﳌﺜﻠﺚ ‪ abc‬ﻣﺘﻘﺎﻳﺲ اﻷﺿﻼع ⟺ ﻟﻠﺰواﻳﺎ ‪ bc, ba ، ab, ac‬و ‪ ca, cb‬ﻧﻔﺲ اﻟﻘﻴﺎس ﺑﱰدﻳﺪ 𝜋‪2‬‬

‫)ا( ﻧﻔﺮض ّ‬
‫أن اﳌﺜﻠﺚ ‪ abc‬ﻣﺘﻘﺎﻳﺲ اﻷﺿﻼع‪.‬‬
‫)‪ (i‬أﺛﺒﺖ ّ‬
‫أن ‪. cos 𝛼 = cos 𝛽 = cos 𝛾 :‬‬
‫)‪ (ii‬ﻣﺎذا ﺗﺴﺘﻨﺘﺞ ؟‬
‫‪ ca, cb‬ﻧﻔﺲ اﻟﻘﻴﺎس ﺑﱰدﻳﺪ 𝜋‪2‬‬ ‫)ب( ﻧﻔﺮض أﻧﻪ ﻟﻠﺰواﻳﺎ ‪ bc, ba ، ab, ac‬و‬
‫𝜋‬‫𝜋‬
‫)‪ (i‬أﺛﺒﺖ ّ‬
‫أن 𝛼 ﻳﺴﺎوي ‪ ، −‬أو 𝜋 ﺑﱰدﻳﺪ 𝜋‪. 2‬‬
‫‪3‬‬‫‪3‬‬
‫)‪ (ii‬أﺛﺒﺖ أﻧﻪ ﺑﺎﻟﴬورة )𝜋‪. 𝛼 ≠ 𝜋 (mod 2‬‬
‫)‪ (iii‬إﺳﺘﻨﺘﺞ ّ‬
‫أن ‪ . A ⋅ B = B ⋅ C = C ⋅ A = 1 :‬ﻣﺎذا ﻳﻤﻜﻦ إﺳﺘﺨﻼﺻﻪ ؟‬

‫‪.a−b = e‬‬ ‫‪/‬‬ ‫‪3‬‬


‫⟺ )‪(c − b‬‬ ‫)ا( أﺛﺒﺖ اﻟﺘﻜﺎﻓﺆ ‪ :‬اﳌﺜﻠﺚ ‪ abc‬ﻣﺘﻘﺎﻳﺲ اﻷﺿﻼع و ﻣﺒﺎﴍ‬ ‫•‬

‫)ب( أﺛﺒﺖ اﻟﺘﻜﺎﻓﺆ ‪ :‬اﳌﺜﻠﺚ ‪ abc‬ﻣﺘﻘﺎﻳﺲ اﻷﺿﻼع و ﻣﺒﺎﴍ ⟺ ‪. a + bj + cj = 0‬‬

‫ﺍﳊﻞّ‪.‬‬
‫‪ņőƱŀ Ţ‬‬
‫)ا( ﻟﺪﻳﻨﺎ ‪:‬‬ ‫•‬ ‫‪1‬‬
‫‪+ +‬‬ ‫‪c−a a−b b−c‬‬
‫‪(Ae ) Be‬‬ ‫‪(Ce ) = ABCe‬‬ ‫=‬ ‫×‬ ‫×‬ ‫‪= −1 = e‬‬
‫‪b−a c−b a−c‬‬
‫ﻣﻨﻪ ‪ ABC = 1‬و )𝜋‪. 𝛼 + 𝛽 + 𝛾 = 𝜋 (mod 2‬‬
‫ﰲ ﺣﺎﻟﺔ ﻣﺎ إذا ﻛﺎن )𝜋‪ bc, ba = 𝛽 (mod 2𝜋) ، ab, ac = 𝛼 (mod 2‬و )𝜋‪ca, cb = 𝛾 (mod 2‬‬
‫ﻮﺟﻬﺔ ﻛﲈ ﻳﻨﺒﻐﻲ ﰲ ﺣﺎﻟﺘﻨﺎ ﻫﺬه( ﻣﺜﻠﺚ ﻳﺴﺎوي 𝜋 )أي ∘‪ (180‬ﺑﱰدﻳﺪ 𝜋‪ 2‬و ﻫﻲ ﻧﺘﻴﺠﺔ‬ ‫ﻓﻬﺬا ﻳﻌﻨﻲ ّ‬
‫أن ﳎﻤﻮع زواﻳﺎ ) ُﻣ ّ‬
‫ﻣﻌﺮوﻓﺔ‪.‬‬
‫)ب( ﺑﺎﳊﺴﺎب ‪:‬‬
‫‪ƕ‬‬

‫‪VII‬‬

‫‪1‬‬ ‫‪c−a c−b‬‬ ‫‪c−a c−b‬‬ ‫‪c−a−c+b‬‬


‫‪Ae‬‬ ‫‪+‬‬ ‫=‬ ‫‪+‬‬ ‫=‬ ‫‪−‬‬ ‫=‬ ‫‪=1‬‬
‫‪Be‬‬ ‫‪b−a a−b b−a b−a‬‬ ‫‪b−a‬‬
‫‪1‬‬ ‫‪a−b a−c a−b a−c a−b−a+c‬‬
‫‪Be +‬‬ ‫=‬ ‫‪+‬‬ ‫=‬ ‫‪−‬‬ ‫=‬ ‫‪=1‬‬
‫‪Ce‬‬ ‫‪c−b b−c‬‬ ‫‪c−b c−b‬‬ ‫‪c−b‬‬
‫‪ũŏ‬‬

‫‪1‬‬ ‫‪b−c b−a b−c b−a b−c−b+a‬‬


‫‪Ce +‬‬ ‫=‬ ‫‪+‬‬ ‫=‬ ‫‪−‬‬ ‫=‬ ‫‪=1‬‬
‫‪Ae‬‬ ‫‪a−c c−a‬‬ ‫‪a−c a−c‬‬ ‫‪a−c‬‬

‫)ا( )‪ (i‬اﳌﺜﻠﺚ ‪ abc‬ﻣﺘﻘﺎﻳﺲ اﻷﺿﻼع إذا و ﻓﻘﻂ إذا ﻛﺎن ‪ A = B = C = 1‬و ﺑﺎﻟﺘﺎﱄ ﻓﺎﳌﺴﺎوﻳﺎت اﳌﱪﻫﻨﺔ ﰲ‬ ‫•‬ ‫‪2‬‬
‫اﻟﺴﺆال )‪•1‬ب( ﺗﺼﺒﺢ ‪ e + e− = 1 ، e + e− = 1 :‬و ‪.e + e− = 1‬‬

‫𝟴𝟴𝟮‬

‫‪http ://tinyurl.com/Malki1718‬‬ ‫‪0‬‬


‫‪ .VII‬ﲤﺎرﻳﻦ ﻟﻠﺘﻌﻤﻖ‬
‫‪.‬‬
‫ﺑﺄﺧﺬ اﳉﺰء اﳊﻘﻴﻘﻲ ﻟﻄﺮﰲ ﻛﻞ ﻣﺴﺎواة ﻣﻦ ﻫﺬه اﳌﺴﺎوﻳﺎت ﻳﻨﺘﺞ ‪:‬‬

‫‪Ŕž Ŧ Ľ‬‬
‫‪⎪ cos 𝛼 + cos 𝛽 = 1‬‬
‫⎧‬
‫‪cos 𝛽 + cos 𝛾 = 1‬‬
‫⎨‬
‫⎪‬

‫‪œǃ‬‬
‫‪⎩ cos 𝛾 + cos 𝛼 = 1‬‬

‫ﺑﻄﺮح اﳌﻌﺎدﻟﺔ اﻟﺜﺎﻧﻴﺔ ﻣﻦ اﻷوﱃ ﻧﺠﺪ ‪، cos 𝛼 − cos 𝛾 = 0‬‬


‫و ﺑﻄﺮح اﳌﻌﺎدﻟﺔ اﻟﺜﺎﻟﺜﺔ ﻣﻦ اﻟﺜﺎﻧﻴﺔ ﻧﺠﺪ ‪ cos 𝛽 − cos 𝛼 = 0‬ﻣﻨﻪ 𝛾 ‪. cos 𝛼 = cos 𝛽 = cos‬‬

‫‪Ŀ‬‬
‫)‪ (ii‬ﺑﺄﺧﺬ اﳉﺰء اﻟﺘﺨﻴﲇ ﻟﻠﻤﺴﺎوﻳﺎت اﻟﺴﺎﺑﻘﺔ ﻧﺠﺪ ‪:‬‬
‫‪ sin 𝛽 − sin 𝛾 = 0 ، sin 𝛼 − sin 𝛽 = 0‬و ‪ sin 𝛾 − sin 𝛼 = 0‬أي 𝛾 ‪. sin 𝛼 = sin 𝛽 = sin‬‬
‫)ﻷن ﳍﺎ ﻧﻔﺲ اﳉﻴﺐ‬‫أن ‪ّ 𝛼 = 𝛽 = 𝛾 (mod 2𝜋) :‬‬ ‫ﻣﻦ ﻫﺬه اﻟﻨﺘﻴﺠﺔ و ﻧﺘﻴﺠﺔ اﻟﺴﺆال )‪ (i‬اﻟﺴﺎﺑﻖ ﻧﺴﺘﻨﺘﺞ ّ‬
‫‪ ، sin‬و ﻧﻔﺲ ﺟﻴﺐ اﻟﺘﲈم ‪ ( cos‬و ﻫﻮ اﳌﻄﻠﻮب‪.‬‬
‫ّ‬
‫]ﻷن‪ ،‬ﺑﺎﻟﺘﻌﺮﻳﻒ ‪:‬‬
‫)𝜋‪ bc, ba = 𝛽 (mod 2𝜋) ، ab, ac = 𝛼 (mod 2‬و )𝜋‪. [ ca, cb = 𝛾 (mod 2‬‬
‫)ب( )‪ (i‬ﻟﺪﻳﻨﺎ ﻓﺮﺿ ًﺎ ‪ 𝛼 = 𝛽 = 𝛾 (mod 2𝜋) :‬و ﺣﺴﺐ اﻟﺴﺆال اﻷول ‪ 𝛼 + 𝛽 + 𝛾 = 𝜋 (mod 2𝜋) :‬ﻣﻨﻪ‬
‫𝜋‪2‬‬ ‫𝜋‬
‫= 𝛾 = 𝛽 = 𝛼‪.‬‬ ‫‪mod‬‬ ‫)𝜋‪ 3𝛼 = 3𝛽 = 3𝛾 = 𝜋 (mod 2‬أي‬
‫‪3‬‬ ‫‪3‬‬
‫𝜋‬ ‫𝜋‬
‫أن 𝛼 ‪ 𝛽 ،‬و 𝛾 ﺗﺴﺎوي ﻛﻠﻬﺎ ‪ −‬أو أو 𝜋 ‪.‬‬‫ﺑﱰدﻳﺪ 𝜋‪ 2‬ﻫﺬا ﻳﻌﻨﻲ ّ‬
‫‪3‬‬ ‫‪3‬‬
‫ﻓﺈن )𝜋‪ 𝛽 = 𝜋 (mod 2‬أﻳﻀ ًﺎ ﻣﻨﻪ ‪:‬‬
‫)‪ (ii‬إذا ﻛﺎن )𝜋‪ّ 𝛼 = 𝜋 (mod 2‬‬
‫‪1‬‬ ‫‪1‬‬ ‫‪1‬‬
‫‪1 = Ae‬‬ ‫‪+‬‬ ‫‪= −A − = − A +‬‬ ‫‪≤0‬‬
‫‪Be‬‬ ‫‪B‬‬ ‫‪B‬‬
‫و ﻫﺬا ﺗﻨﺎﻗﺾ‪ ،‬إذن ﻻ ﻳﻤﻜﻦ أن ﻳﻜﻮن )𝜋‪. 𝛼 = 𝜋 (mod 2‬‬
‫)‪ (iii‬ﺑﺄﺧﺬ اﳉﺰء اﻟﺘﺨﻴﲇ ﻟﻠﻤﺴﺎوﻳﺎت اﳌﱪﻫﻨﺔ ﰲ اﻟﺴﺆال )‪•1‬ب( ﻧﺠﺪ ‪:‬‬
‫‪1‬‬ ‫‪1‬‬ ‫‪1‬‬
‫‪A−‬‬ ‫= 𝛼 ‪sin‬‬ ‫‪B−‬‬ ‫= 𝛼 ‪sin‬‬ ‫‪C−‬‬ ‫‪sin 𝛼 = 0‬‬
‫‪B‬‬ ‫‪C‬‬ ‫‪A‬‬
‫‪ņőƱŀ Ţ‬‬
‫‪1‬‬ ‫‪1‬‬ ‫‪1‬‬
‫ﻓﺈن ‪ sin 𝛼 ≠ 0‬ﻣﻨﻪ ‪ A − = B − = C − = 0‬ﻣﻨﻪ‬ ‫ﻟﻜﻦ ﺣﺴﺐ اﻟﺴﺆاﻟﲔ )‪ (i‬و )‪ (ii‬اﻟﺴﺎﺑﻘﲔ ّ‬
‫‪B‬‬ ‫‪C‬‬ ‫‪A‬‬
‫‪ . AB = BC = CA = 1‬ﻟﻜﻦ ‪ ABC‬إذن ‪ A = B = C = 1‬أي |‪ ||a − b|| = ||b − c|| = |c − a‬و‬
‫ﺣﺴﺐ اﻟﺘﻌﺮﻳﻒ‪ ،‬ﻓﺎﳌﺜﻠﺚ ‪ abc‬ﻣﺘﻘﺎﻳﺲ اﻷﺿﻼع‪.‬‬

‫أن اﳌﺜﻠﺚ ‪ abc‬ﻣﺘﻘﺎﻳﺲ اﻷﺿﻼع و ﻣﺒﺎﴍ‪ .‬إذن ﻓﻠﻠﺰاوﻳﺔ ‪ ، bc, ba‬و اﻟﺘﻲ ﺗﺴﺎوي 𝛽 ﺑﱰدﻳﺪ 𝜋‪، 2‬‬ ‫)ا( ﻧﻔﺮض ّ‬ ‫‪3‬‬
‫•‬
‫𝜋‬
‫ﻓﺈن ﻫﺬا اﻟﻘﻴﺎس ﻫﻮ )𝜋‪ . 𝛽 = (mod 2‬ﻣﻦ‬ ‫ﻗﻴﺎس ﰲ اﳌﺠﺎل [𝜋 ‪ . ]0,‬و ﺣﺴﺐ ﻧﺘﻴﺠﺔ اﻟﺴﺆاﻟﲔ )‪ (i‬و )‪ّ (ii‬‬
‫‪3‬‬
‫|‪|a − b‬‬ ‫)ﻷن اﳌﺜﻠﺚ ‪ abc‬ﻣﺘﻘﺎﻳﺲ اﻷﺿﻼع(‪ّ ،‬‬‫أن ||‪ّ ||a − b|| = ||c − b‬‬
‫ﺟﻬﺔ أﺧﺮى‪ ،‬و ﺑﲈ ّ‬
‫|| = ‪ B‬ﻣﻨﻪ‬ ‫ﻓﺈن ‪= 1‬‬
‫|| ‪c − b‬‬
‫‪ƕ‬‬

‫‪a−b‬‬
‫‪.‬‬ ‫‪= Be = e /‬‬
‫‪VII‬‬

‫‪c−b‬‬
‫أن اﳌﺜﻠﺚ ‪abc‬‬ ‫أن )𝜋‪) bc, ba = 𝜋 (mod 2‬أي ّ‬ ‫أن ‪ a − b = e /‬ﻓﻬﺬا ﻳﻌﻨﻲ ّ‬ ‫ﺑﺎﻟﻌﻜﺲ‪ ،‬إذا ﻓﺮﺿﻨﺎ ّ‬
‫‪3‬‬ ‫‪c−b‬‬
‫‪a − c (a − b) + (b − c) a − b b − c‬‬
‫‪ũŏ‬‬

‫=‬ ‫=‬ ‫‪+‬‬ ‫أن ||‪ . ||a − b|| = ||c − b‬ﻟﻜﻦ ‪= e / − 1‬‬
‫ﻣﺒﺎﴍ( و ّ‬
‫‪c−b‬‬ ‫‪c−b‬‬ ‫‪c−b c−b‬‬
‫و‬
‫‪|a − c| | /‬‬ ‫‪/‬‬ ‫‪/‬‬ ‫‪−‬‬ ‫‪/‬‬ ‫|‬ ‫‪/‬‬ ‫|𝜋‬
‫‪|| c − b || = |e‬‬ ‫‪− 1|| = ||e‬‬ ‫‪e‬‬ ‫‪−e‬‬ ‫‪|| = |2ıe‬‬ ‫‪sin | = 1‬‬
‫|‬ ‫|‪6‬‬
‫ﻣﻨﻪ |‪ ||b − c|| = |c − a‬أي اﳌﺜﻠﺚ ‪ abc‬ﻣﺘﻘﺎﻳﺲ اﻷﺿﻼع و ﻣﺒﺎﴍ‪.‬‬

‫𝟵𝟴𝟮‬

‫‪http ://tinyurl.com/Malki1718‬‬ ‫‪0‬‬


‫‪.‬‬
‫‪ e‬و ‪ 1 + j + j = 0‬ﻧﺠﺪ ‪:‬‬ ‫‪/‬‬ ‫‪= −j‬‬ ‫)ب( ﺑﺘﻄﺒﻴﻖ اﻟﻨﺘﻴﺠﺔ اﻟﺴﺎﺑﻘﺔ و اﻟﻌﻼﻗﺘﲔ‬

‫‪Ŕž Ŧ Ľ‬‬
‫‪ ⟺ a − b = e‬اﳌﺜﻠﺚ ‪ abc‬ﻣﺘﻘﺎﻳﺲ اﻷﺿﻼع و ﻣﺒﺎﴍ‬ ‫‪/‬‬
‫)‪(c − b‬‬
‫)‪⟺ a − b = −j (c − b‬‬

‫‪œǃ‬‬
‫‪⟺ a + (−1 − j )b + cj = 0‬‬
‫‪⟺ a + bj + cj = 0‬‬

‫■‬

‫‪Ŀ‬‬
‫ﻣﺎ ﻫﻮ اﻟﴩط اﻟﻼزم و اﻟﻜﺎﰲ اﻟﺬي ﳚﺐ أن ﲢﻘﻘﻪ اﻷﻋﺪاد اﳌﺮﻛﺒﺔ ‪ c ، b ، a‬ﻟﻮاﺣﻖ اﻟﻨﻘﻂ ‪ C ، B ، A‬ﻋﲆ‬
‫‪.‬‬
‫𝟏𝟎𝟐‬ ‫‪.‬‬ ‫‪#‬‬
‫أﻧﻈﺮ اﻟﺘﻤﺮﻳﻦ ‪ 202‬ﺻﻔﺤﺔ ‪290‬‬ ‫‬ ‫اﻟﱰﺗﻴﺐ ﺣﺘﻰ ﻳﻜﻮن اﳌﺜﻠﺚ ‪ ABC‬ﻣﺘﻘﺎﻳﺲ اﻷﺿﻼع ؟‬

‫ﺍﳊﻞّ‪ .‬ﺑﺪاﻳ ًﺔ‪ ،‬ﻧُﺬﻛﱢﺮ ّ‬


‫أن ‪:‬‬

‫‪1+j+j =0,‬‬ ‫‪j =1‬‬


‫‪e‬‬ ‫‪/‬‬ ‫‪=e‬‬ ‫‪/ +‬‬ ‫‪=e‬‬ ‫‪+‬‬ ‫‪/‬‬ ‫‪=e‬‬ ‫‪e‬‬ ‫‪/‬‬ ‫‪= −j‬‬
‫‪e−‬‬ ‫‪/‬‬ ‫=‬ ‫‪e−‬‬ ‫‪/ +‬‬ ‫‪=e‬‬ ‫‪−‬‬ ‫‪/‬‬ ‫‪=e‬‬ ‫‪⋅e‬‬ ‫‪/‬‬ ‫‪= −j‬‬

‫ﻧُﻤ ّﻴﺰ ﺣﺎﻟﺘﲔ ‪ :‬اﳌﺜﻠﺚ ‪ ABC‬ﻣﺒﺎﴍ أو ﻏﲑ ﻣﺒﺎﴍ‪.‬‬


‫𝜋‬
‫اﳌﺜﻠﺚ ‪ ABC‬ﻣﺘﻘﺎﻳﺲ اﻷﺿﻼع و ﻣﺒﺎﴍ إذا و ﻓﻘﻂ إذا ﻛﺎن ‪ BA‬ﺻﻮرة ‪ BC‬ﺑﺎﻟﺪوران اﻟﺬي ﻣﺮﻛﺰه ‪ B‬و زاوﻳﺘﻪ أي‬ ‫•‬
‫‪3‬‬
‫إذا و ﻓﻘﻂ إذا ﻛﺎن )‪ a − b = e / (c − b‬أي إذا و ﻓﻘﻂ إذا ﻛﺎن )‪ a − b = −j (c − b‬أي إذا و ﻓﻘﻂ إذا ﻛﺎن‬
‫‪ a − (1 + j )b + j c = 0‬أي إذا و ﻓﻘﻂ إذا ﻛﺎن ‪. a + bj + cj = 0‬‬
‫اﳌﺜﻠﺚ ‪ ABC‬ﻣﺘﻘﺎﻳﺲ اﻷﺿﻼع و ﻏﲑ ﻣﺒﺎﴍ إذا و ﻓﻘﻂ إذا ﻛﺎن ‪ BA‬ﺻﻮرة ‪ BC‬ﺑﺎﻟﺪوران اﻟﺬي ﻣﺮﻛﺰه ‪ B‬و زاوﻳﺘﻪ‬ ‫•‬
‫‪ņőƱŀ Ţ‬‬
‫𝜋‬
‫‪ −‬أي إذا و ﻓﻘﻂ إذا ﻛﺎن )‪ a − b = e− / (c − b‬أي إذا و ﻓﻘﻂ إذا ﻛﺎن )‪ a − b = −j (c − b‬أي إذا و ﻓﻘﻂ إذا‬
‫‪3‬‬
‫ﻛﺎن ‪ a − (1 + j)b + jc = 0‬أي إذا و ﻓﻘﻂ إذا ﻛﺎن ‪. a + bj + cj = 0‬‬
‫إذن اﳌﺜﻠﺚ ‪ ABC‬ﻣﺘﻘﺎﻳﺲ اﻷﺿﻼع إذا و ﻓﻘﻂ إذا ﻛﺎن ‪ a + bj + cj = 0‬أو ‪ a + bj + cj = 0‬أي إذا و ﻓﻘﻂ إذا ﻛﺎن‬
‫‪ . a + bj + cj‬ﻟﻜﻦ ‪:‬‬ ‫‪a + bj + cj = 0‬‬

‫‪a + bj + cj‬‬ ‫)‪a + bj + cj = a + b + c + j (ab + ac + bc) + j (ab + ac + bc‬‬


‫‪ƕ‬‬

‫‪=a +b +c + j+j‬‬ ‫)‪j (ab + ac + bc‬‬


‫‪VII‬‬

‫)‪= a + b + c − (ab + ac + bc‬‬

‫إذن اﳌﺜﻠﺚ ‪ ABC‬ﻣﺘﻘﺎﻳﺲ اﻷﺿﻼع إذا و ﻓﻘﻂ إذا ﻛﺎن ‪ a + b + c − (ab + ac + bc) = 0‬أي إذا و ﻓﻘﻂ إذا ﻛﺎن‬
‫‪ũŏ‬‬

‫■‬ ‫‪. a + b + c = ab + ac + bc‬‬

‫ﻟﺘﻜﻦ ‪ B ، A‬و ‪ C‬ﺛﻼﺛﺔ ﻧﻘﻂ ﻣﻦ اﳌﺴﺘﻮي ﳐﺘﻠﻔﺔ ﻣﺜﻨﻰ ﻣﺜﻨﻰ ‪ ،‬ﻟﻮاﺣﻘﻬﺎ ‪ b ، a‬و ‪ c‬ﻋﲆ اﻟﱰﺗﻴﺐ‪.‬‬
‫‪#‬‬ ‫‪.‬‬
‫𝟐𝟎𝟐‬ ‫‪.‬‬

‫𝟬𝟵𝟮‬

‫‪http ://tinyurl.com/Malki1718‬‬ ‫‪0‬‬


‫ ﲤﺎرﻳﻦ ﻟﻠﺘﻌﻤﻖ‬.VII
.

ّ ‫ أﺛﺒﺖ‬. 1 + j + j = 0 ‫ و‬j = 1 ‫أن‬


: ‫أن‬ ّ ‫ و ﻧُﺬﻛﱢﺮ‬j = e / ‫ﻧﻀﻊ‬

Ŕž Ŧ Ľ
ّ j ‫ أو‬j
‫ ﻣﺘﻘﺎﻳﺲ اﻷﺿﻼع‬ABC ⟺ az + bz + c = 0 ‫ﺣﻞ ﻟﻠﻤﻌﺎدﻟﺔ‬

œǃ
⟺ a + b + c = ab + ac + bc
⟺ (b − a) + (c − b) + (a − c) = 0
1 1 1
⟺ + + =0

Ŀ
b−c c−a a−b

290 ‫ ﺻﻔﺤﺔ‬201 ‫أﻧﻈﺮ اﻟﺘﻤﺮﻳﻦ‬ 


: ‫ ﻟﺪﻳﻨﺎ‬.ّ‫ﺍﳊﻞ‬

‫ ﻣﺘﻘﺎﻳﺲ اﻷﺿﻼع‬ABC ⟺ C = ℛ / (B) ‫أو‬ C=ℛ − / (B)

⟺ c − a = −j (b − a) ‫أو‬ c − a = (−j) (b − a)

⟺ −1 − j a+j b+c=0 ‫أو‬ (−1 − j) a + jb + c = 0

⟺ ja + j b + c = 0 ‫أو‬ j + jb + c = 0

⟺ j a+j b+c=0 ‫أو‬ j a + jb + c = 0


ّ j
⟺ az + bz + c = 0 ‫ﺣﻞ ﻟﻠﻤﻌﺎدﻟﺔ‬ ‫ أو‬j
: ‫ﻣﻨﻪ‬
‫ ﻣﺘﻘﺎﻳﺲ اﻷﺿﻼع‬ABC ⟺ ja + j + c = 0 ‫أو‬ j + jb + c = 0

⟺ ja + j b + c j + jb + c = 0
ņőƱŀ Ţ
⟺ a +b +c + j+j (ab + ac + bc) = 0

⟺ a + b + c − (ab + ac + bc) = 0
⟺ a + b + c = ab + ac + bc

: ‫ﻣﻨﻪ‬
‫ ﻣﺘﻘﺎﻳﺲ اﻷﺿﻼع‬ABC ⟺ a + b + c − ab − ac − bc = 0
⟺ 2a + 2b + 2c − 2ab − 2ac − 2bc = 0
ƕ
VII

⟺ b − 2ab + a + c − 2bc + b + a − 2ac + c = 0


⟺ (b − a) + (c − b) + (a − c) = 0
ũŏ

: ‫و أﻳﻀ ًﺎ‬

‫ ﻣﺘﻘﺎﻳﺲ اﻷﺿﻼع‬ABC ⟺ a + b + c − ab − ac − bc = 0
⟺ −a + ab + ac − bc − b + bc + ba − ac − c + ca + cb − ab = 0
⟺ (c − a) (a − b) + (a − b) (b − c) + (b − c) (c − a) = 0
(c − a) (a − b) + (a − b) (b − c) + (b − c) (c − a)
⟺ =0
(b − c) (c − a) (a − b)

𝟮𝟵𝟭

http ://tinyurl.com/Malki1718 0
‫‪.‬‬
‫‪1‬‬ ‫‪1‬‬ ‫‪1‬‬
‫⟺‬ ‫‪+‬‬ ‫‪+‬‬ ‫‪=0‬‬
‫‪b−c c−a a−b‬‬

‫‪Ŕž Ŧ Ľ‬‬
‫‪1‬‬ ‫‪1‬‬ ‫‪1‬‬
‫ُﻮﺣﺪ اﳌﻘﺎﻣﺎت و‬
‫ﺛﻢ ﻧ ﱢ‬ ‫‪+‬‬ ‫‪+‬‬ ‫ﻣﻼﺣﻈﺔ ‪ ⧏ : 48‬ﰲ اﻟﺘﻜﺎﻓﺆ اﻷﺧﲑ‪ ،‬ﻣﻦ اﻷﺳﻬﻞ اﻹﻧﻄﻼق ﻣﻦ ‪= 0‬‬

‫‪œǃ‬‬
‫‪b−c c−a a−b‬‬
‫⧐‬ ‫ُﺒﺴﻂ اﳊﺴﺎﺑﺎت ﺣﺘّﻰ ﻧﺼﻞ إﱃ ‪. a + b + c − ab − ac − bc = 0‬‬
‫ﻧ ﱢ‬
‫■‬

‫‪Ŀ‬‬
‫ﻧﻀﻊ ‪:‬‬
‫‬ ‫‪.‬‬
‫𝟑𝟎𝟐‬ ‫‪.‬‬
‫‪n‬‬ ‫‪n‬‬ ‫‪n‬‬ ‫‪n‬‬ ‫‪n‬‬ ‫‪n‬‬ ‫‪n‬‬ ‫‪n‬‬ ‫‪n‬‬
‫=‪S‬‬ ‫‪+‬‬ ‫‪+‬‬ ‫‪+⋯ ,‬‬ ‫=‪T‬‬ ‫‪+‬‬ ‫‪+‬‬ ‫‪+⋯ ,‬‬ ‫=‪U‬‬ ‫‪+‬‬ ‫‪+‬‬ ‫⋯‪+‬‬
‫‪0‬‬ ‫‪3‬‬ ‫‪6‬‬ ‫‪1‬‬ ‫‪4‬‬ ‫‪7‬‬ ‫‪2‬‬ ‫‪5‬‬ ‫‪8‬‬
‫‪.1+j+j = 0‬‬ ‫‪ •1‬أﺛﺒﺖ ّ‬
‫أن ‪:‬‬
‫⎧‬
‫⎪‬ ‫)‪S + T + U = (1 + 1‬‬
‫⎪‬
‫⎪‬
‫)‪S + jT + j U = (1 + j‬‬ ‫‪ •2‬ﺑﺎﺳﺘﻌﲈل دﺳﺘﻮر ﺛﻨﺎﺋﻲ اﳊﺪ‪ ،‬أﺛﺒﺖ ّ‬
‫أن ‪:‬‬
‫⎨‬
‫⎪‬
‫⎪‬
‫‪⎪ S + j T + jU = 1 + j‬‬
‫⎩‬
‫‪ •3‬إﺳﺘﻨﺘﺞ ﻗﻴﻤﺔ ﻛﻞ ﻣﻦ ‪ T ، S‬و ‪. U‬‬

‫أﻧﻈﺮ اﻟﺘﻤﺮﻳﻦ ‪ 215‬ﺻﻔﺤﺔ ‪306‬‬ ‫‬


‫أن ‪ّ j ≠ 1‬‬
‫ﻓﺈن ‪. 1 + j + j = 0 :‬‬ ‫‪ (j − 1) 1 + j + j‬و ﺑﲈ ّ‬ ‫ﻧﻌﻠﻢ ّ‬
‫أن ‪ j = 1‬أي ‪= 0‬‬ ‫•‬ ‫‪1‬‬ ‫ﺍﳊﻞّ‪.‬‬

‫‪ •2‬ﻧﺴﺘﻌﻤﻞ دﺳﺘﻮر ﺛﻨﺎﺋﻲ اﳊﺪ ﻟﻨﴩ )‪ (1 + x‬ﻣﻊ ‪ x = j ، x = 1‬و ‪: x = j‬‬


‫‪n‬‬ ‫‪n‬‬ ‫‪n‬‬ ‫‪n‬‬ ‫‪n‬‬ ‫‪n‬‬
‫‪ņőƱŀ Ţ‬‬
‫= )‪(1 + 1‬‬ ‫‪+‬‬ ‫‪+‬‬ ‫‪+‬‬ ‫‪+‬‬ ‫‪+‬‬ ‫‪+⋯=S+T +U‬‬
‫‪0‬‬ ‫‪1‬‬ ‫‪2‬‬ ‫‪3‬‬ ‫‪4‬‬ ‫‪5‬‬
‫‪n‬‬ ‫‪n‬‬ ‫‪n‬‬ ‫‪n‬‬ ‫‪n‬‬ ‫‪n‬‬
‫= )‪(1 + j‬‬ ‫‪+j‬‬ ‫‪+j‬‬ ‫‪+j‬‬ ‫‪+j‬‬ ‫‪+j‬‬ ‫⋯‪+‬‬
‫‪0‬‬ ‫‪1‬‬ ‫‪2‬‬ ‫‪3‬‬ ‫‪4‬‬ ‫‪5‬‬
‫‪n‬‬ ‫‪n‬‬ ‫‪n‬‬ ‫‪n‬‬ ‫‪n‬‬ ‫‪n‬‬
‫=‬ ‫‪+j‬‬ ‫‪+j‬‬ ‫‪+‬‬ ‫‪+j‬‬ ‫‪+j‬‬ ‫⋯‪+‬‬
‫‪0‬‬ ‫‪1‬‬ ‫‪2‬‬ ‫‪3‬‬ ‫‪4‬‬ ‫‪5‬‬
‫‪n‬‬ ‫‪n‬‬ ‫‪n‬‬ ‫‪n‬‬ ‫‪n‬‬ ‫‪n‬‬
‫=‬ ‫‪+‬‬ ‫⋯‪+‬‬ ‫‪+j‬‬ ‫‪+‬‬ ‫‪+⋯ +j‬‬ ‫‪+‬‬ ‫⋯‪+‬‬
‫‪0‬‬ ‫‪3‬‬ ‫‪1‬‬ ‫‪4‬‬ ‫‪2‬‬ ‫‪5‬‬
‫‪= S + jT + j U‬‬
‫‪n‬‬ ‫‪n‬‬ ‫‪n‬‬ ‫‪n‬‬ ‫‪n‬‬ ‫‪n‬‬
‫‪1+j‬‬ ‫=‬ ‫‪+j‬‬ ‫‪+j‬‬ ‫‪+j‬‬ ‫‪+j‬‬ ‫‪+j‬‬ ‫⋯‪+‬‬
‫‪ƕ‬‬

‫‪0‬‬ ‫‪1‬‬ ‫‪2‬‬ ‫‪3‬‬ ‫‪4‬‬ ‫‪5‬‬


‫‪n‬‬ ‫‪n‬‬ ‫‪n‬‬ ‫‪n‬‬ ‫‪n‬‬ ‫‪n‬‬
‫‪VII‬‬

‫=‬ ‫‪+j‬‬ ‫‪+j‬‬ ‫‪+‬‬ ‫‪+j‬‬ ‫‪+j‬‬ ‫⋯‪+‬‬


‫‪0‬‬ ‫‪1‬‬ ‫‪2‬‬ ‫‪3‬‬ ‫‪4‬‬ ‫‪5‬‬
‫‪n‬‬ ‫‪n‬‬ ‫‪n‬‬ ‫‪n‬‬ ‫‪n‬‬ ‫‪n‬‬
‫=‬ ‫‪+‬‬ ‫⋯‪+‬‬ ‫‪+j‬‬ ‫‪+‬‬ ‫‪+⋯ +j‬‬ ‫‪+‬‬ ‫⋯‪+‬‬
‫‪0‬‬ ‫‪3‬‬ ‫‪1‬‬ ‫‪4‬‬ ‫‪2‬‬ ‫‪5‬‬
‫‪ũŏ‬‬

‫‪= S + j T + jU‬‬

‫‪ •3‬ﻧُﻄ ﱢﺒﻖ ﻧﺘﻴﺠﺔ اﻟﺘﻤﺮﻳﻦ ‪ 14‬ﺻﻔﺤﺔ ‪ 80‬ﻣﻊ ‪:‬‬


‫‪c= 1+j‬‬ ‫)‪= (−j‬‬ ‫و‬ ‫‪b = (1 + j) = −j‬‬ ‫‪، a = (1 + 1) = 2‬‬

‫𝟮𝟵𝟮‬

‫‪http ://tinyurl.com/Malki1718‬‬ ‫‪0‬‬


‫‪ .VII‬ﲤﺎرﻳﻦ ﻟﻠﺘﻌﻤﻖ‬
‫‪.‬‬
‫‪ −j = e ⋅ e‬ﻓﻴﻜﻮن ‪:‬‬ ‫‪/‬‬ ‫‪= e−‬‬ ‫‪/‬‬ ‫و ﻧُﺬﻛﱢﺮ ّ‬
‫أن ‪ j = j‬و‬

‫‪Ŕž Ŧ Ľ‬‬
‫‪1‬‬ ‫‪1‬‬ ‫‪1‬‬
‫=‪S‬‬ ‫= ]‪[a + b + c‬‬ ‫‪2 + −j‬‬ ‫= )‪+ (−j‬‬ ‫)‪2 + −j + (−j‬‬
‫‪3‬‬ ‫‪3‬‬ ‫‪3‬‬
‫‪1‬‬ ‫‪1‬‬ ‫‪1‬‬ ‫𝜋‪n‬‬

‫‪œǃ‬‬
‫=‬ ‫)‪2 + 2 Re (−j‬‬ ‫=‬ ‫‪2 + 2 Re e− /‬‬ ‫=‬ ‫‪2 + 2 cos −‬‬
‫‪3‬‬ ‫‪3‬‬ ‫‪3‬‬ ‫‪3‬‬
‫‪1‬‬ ‫𝜋‬
‫=‬ ‫‪2 + 2 cos n‬‬
‫‪3‬‬ ‫‪3‬‬
‫‪1‬‬ ‫‪1‬‬ ‫‪1‬‬

‫‪Ŀ‬‬
‫=‪T‬‬ ‫= ‪a + bj + cj‬‬ ‫‪2 + j −j‬‬ ‫= )‪+ j (−j‬‬ ‫)‪2 + j −j + j (−j‬‬
‫‪3‬‬ ‫‪3‬‬ ‫‪3‬‬
‫‪1‬‬ ‫‪1‬‬ ‫‪1‬‬ ‫𝜋)‪(n − 2‬‬
‫=‬ ‫)‪2 + 2 Re j (−j‬‬ ‫=‬ ‫‪2 + 2 Re e / e− /‬‬ ‫=‬ ‫‪2 + 2 cos −‬‬
‫‪3‬‬ ‫‪3‬‬ ‫‪3‬‬ ‫‪3‬‬
‫‪1‬‬ ‫𝜋)‪(n − 2‬‬
‫=‬ ‫‪2 + 2 cos‬‬
‫‪3‬‬ ‫‪3‬‬
‫‪1‬‬ ‫‪1‬‬ ‫‪1‬‬
‫=‪U‬‬ ‫= ‪a + bj + cj‬‬ ‫‪2 + j −j‬‬ ‫= )‪+ j (−j‬‬ ‫)‪2 + j −j + j (−j‬‬
‫‪3‬‬ ‫‪3‬‬ ‫‪3‬‬
‫‪1‬‬ ‫‪1‬‬ ‫‪1‬‬ ‫𝜋)‪(n − 4‬‬
‫=‬ ‫)‪2 + 2 Re j (−j‬‬ ‫=‬ ‫‪2 + 2 Re e / e− /‬‬ ‫=‬ ‫‪2 + 2 cos −‬‬
‫‪3‬‬ ‫‪3‬‬ ‫‪3‬‬ ‫‪3‬‬
‫‪1‬‬ ‫𝜋)‪(n − 4‬‬
‫=‬ ‫‪2 + 2 cos‬‬
‫‪3‬‬ ‫‪3‬‬

‫■‬

‫‪ ABC‬ﻣﺜﻠﺚ ﻣﺒﺎﴍ‪.‬‬
‫‬‫‪.‬‬
‫𝟒𝟎𝟐‬ ‫‪.‬‬

‫‪ DBA‬ﻣﺜﻠﺚ ﻣﺒﺎﴍ‪،‬ﻣﺘﺴﺎوي اﻟﺴﺎﻗﲔ و ﻗﺎﺋﻢ ﰲ ‪.D‬‬


‫‪ ACE‬ﻣﺜﻠﺚ ﻣﺒﺎﴍ‪ ،‬ﻣﺘﺴﺎوي اﻟﺴﺎﻗﲔ و ﻗﺎﺋﻢ ﰲ ‪.E‬‬
‫ﻧُﻨﺸﺊ اﻟﻨﻘﻄﺔ ‪ L‬ﺑﺤﻴﺚ ‪.CL = DB‬‬
‫‪ņőƱŀ Ţ‬‬
‫‪ •1‬أرﺳﻢ اﻟﺸﻜﻞ‪.‬‬
‫‪ •2‬أﺛﺒﺖ ّ‬
‫أن اﳌﺜﻠﺚ ‪ EDL‬ﻣﺒﺎﴍ‪ ،‬ﻣﺘﺴﺎوي اﻟﺴﺎﻗﲔ و ﻗﺎﺋﻢ ﰲ ‪.E‬‬

‫ﺍﳊﻞّ‪ •1 .‬اﻟﺸﻜﻞ ‪ :‬ﻹﻧﺸﺎء اﳌﺜﻠﺚ اﳌﺒﺎﴍ‪ ،‬اﻟﻘﺎﺋﻢ ﰲ ‪ D‬و اﳌﺘﺴﺎوي اﻟﺴﺎﻗﲔ ‪ ،DBA‬ﻧُﻨﺸﺊ )ﻧﺤﻮ اﳉﻬﺔ اﳋﺎرﺟﻴﺔ( اﳌﺮﺑﻊ‬
‫اﻟﺬي أﺣﺪ أﺿﻼﻋﻪ ‪ ،AB‬ﻓﺘﻜﻮن ﻧﻘﻄﺔ ﺗﻘﺎﻃﻊ ُﻗﻄﺮﻳﻪ ﻫﻲ اﻟﺮأس ‪. D‬‬
‫‪ƕ‬‬

‫ﻧُﻨﺸﺊ ﺑﻨﻔﺲ اﻟﻄﺮﻳﻘﺔ اﳌﺜﻠﺚ ‪. ACE‬‬


‫‪VII‬‬

‫ﺛﻢ ﻧﺼﻒ اﳌﺴﺘﻘﻴﻢ‬


‫ﻹﻧﺸﺎء اﻟﻨﻘﻄﺔ ‪ ،L‬ﻧُﻨﺸﺊ ﻣﺘﻮازي اﻷﺿﻼع ‪ DBLC‬ﻛﲈ ﻳﲇ ‪ :‬ﻧُﻨﺸﺊ اﻟﻨﻘﻄﺔ ‪ I‬ﻣﻨﺘﺼﻒ اﻟﻘﻄﻌﺔ ]‪ّ [BC‬‬
‫)‪ . [DI‬اﻟﻨﻘﻄﺔ ‪ I‬ﻫﻲ أﻳﻀ ًﺎ ﻣﻨﺘﺼﻒ اﻟﻘﻄﻌﺔ ]‪. [DL‬‬
‫‪ũŏ‬‬

‫أﻧﻈﺮ اﻟﺸﻜﻞ ‪. 3.VII‬‬


‫‪ •2‬ﻧﻨﺴﺐ اﳌﺴﺘﻮي إﱃ ﻣﻌﻠﻢ ﻣﺘﻌﺎﻣﺪ ﻣﺘﺠﺎﻧﺲ و ﻣﺒﺎﴍ ⃗ ‪ . O, i,⃗ j‬ﻟﺘﻜﻦ ‪ e ،d ،c ،b ،a‬و ‪ ℓ‬ﻟﻮاﺣﻖ اﻟﻨﻘﻂ ‪،C ،B ،A‬‬
‫‪ E ،D‬و ‪ L‬ﻋﲆ اﻟﱰﺗﻴﺐ‪.‬‬
‫‪/‬‬ ‫𝜋‬
‫‪.(e‬‬ ‫)ﻧُﺬﻛﺮ ّ‬
‫أن ‪= ı‬‬ ‫‪ّ ℛ D,‬‬
‫ﻓﺈن ‪a − d = ı (b − d) :‬‬ ‫ﺑﲈ ّ‬
‫أن ‪ A‬ﻫﻲ ﺻﻮرة ‪ B‬ﺑﺎﻟﺪوران‬
‫‪2‬‬
‫ﺑﺎﳌﺜﻞ ﰲ اﳌﺜﻠﺚ ‪.c − e = ı (a − e) : ACE‬‬

‫𝟯𝟵𝟮‬

‫‪http ://tinyurl.com/Malki1718‬‬ ‫‪0‬‬


‫‪.‬‬

‫‪Ŕž Ŧ Ľ‬‬
‫‪œǃ‬‬
‫‪E‬‬

‫‪D‬‬

‫‪Ŀ‬‬
‫‪A‬‬

‫‪B‬‬ ‫‪I‬‬ ‫‪C‬‬

‫‪L‬‬
‫ﺷﻜﻞ ‪3.VII‬‬

‫أن ‪ L‬ﻫﻲ ﺻﻮرة ‪ C‬ﺑﺎﻹﻧﺴﺤﺎب ذي اﻟﺸﻌﺎع ‪ّ DB‬‬


‫ﻓﺈن ‪ ℓ − c = b − d :‬أي ‪. ℓ = c + b − d‬‬ ‫و ﺑﲈ ّ‬
‫أن اﳌﺜﻠﺚ ‪ EDL‬ﻣﺒﺎﴍ‪ ،‬ﻣﺘﺴﺎوي اﻟﺴﺎﻗﲔ و ﻗﺎﺋﻢ ﰲ ‪ E‬ﻳﻜﻔﻲ أن ﻧُﺜﺒﺖ ّ‬
‫أن اﻟﻨﻘﻄﺔ ‪ L‬ﻫﻲ ﺻﻮرة ‪ D‬ﺑﺎﻟﺪوران‬ ‫ﻹﺛﺒﺎت ّ‬
‫𝜋 ‪ ℛ E,‬أي ّ‬
‫أن )‪. ℓ − e = ı (d − e‬‬
‫‪2‬‬
‫ﻟﻜﻦ ‪:‬‬
‫)‪ℓ − e = (c + b − d) − e = c − e + b − d = ı (a − e) − ı (a − d) = ı (d − e‬‬
‫ﻧﺴﺘﻨﺘﺞ ّ‬
‫أن اﳌﺜﻠﺚ ‪ EDL‬ﻣﺒﺎﴍ‪ ،‬ﻣﺘﺴﺎوي اﻟﺴﺎﻗﲔ و ﻗﺎﺋﻢ ﰲ ‪. E‬‬
‫■‬

‫‬
‫‪ņőƱŀ Ţ‬‬
‫‪.‬‬
‫𝟓𝟎𝟐‬ ‫‪.‬‬

‫‪ ABCD‬رﺑﺎﻋﻲ ﻛﻴﻔﻲ و ﻣﺒﺎﴍ ‪ .‬ﻧُﻨﺸﺊ‪ ،‬ﻧﺤﻮ اﳉﻬﺔ اﳋﺎرﺟﻴﺔ ﳍﺬا اﻟﺮﺑﺎﻋﻲ‪ ،‬اﳌﺮﺑﻌﺎت اﻟﺘﻲ ﻣﺮاﻛﺰﻫﺎ ‪ S ، R ، Q ، P‬و أﺣﺪ‬
‫أﺿﻼﻋﻬﺎ ]‪ [DA] ، [CD] ، [BC] ، [AB‬ﻋﲆ اﻟﱰﺗﻴﺐ‪) .‬ﺷﻜﻞ ‪(4.VII‬‬
‫ﻄﺮي اﻟﺮﺑﺎﻋﻲ ‪ PQRS‬ﻣﺘﻌﺎﻣﺪان و ﳍﲈ ﻧﻔﺲ اﻟﻄﻮل‪.‬‬ ‫أن ُﻗ َ‬
‫اﳍﺪف ﻣﻦ اﻟﺘﻤﺮﻳﻦ ﻫﻮ إﺛﺒﺎت ّ‬
‫ﻧﻨﺴﺐ اﳌﺴﺘﻮي إﱃ ﻣﻌﻠﻢ ﻣﺘﻌﺎﻣﺪ ﻣﺘﺠﺎﻧﺲ و ﻣﺒﺎﴍ ⃗ ‪ . O, i,⃗ j‬ﻟﺘﻜﻦ ‪ r ،q ،p ،d ،c ،b ،a‬و ‪ s‬ﻟﻮاﺣﻖ اﻟﻨﻘﻂ ‪،C ،B ،A‬‬
‫‪ R ،Q ،P ،D‬و ‪ S‬ﻋﲆ اﻟﱰﺗﻴﺐ‪.‬‬
‫‪a − ıb‬‬
‫= ‪.p‬‬ ‫‪ •1‬أﺛﺒﺖ أﻧﻪ ﰲ اﳌﺮﺑﻊ اﳌﺮﺳﻮم ﻋﲆ ]‪ [AB‬ﻟﺪﻳﻨﺎ ‪:‬‬
‫‪ƕ‬‬

‫‪1−ı‬‬
‫‪VII‬‬

‫أوﺟﺪ ﻋﺒﺎرة ﻣﺸﺎﲠﺔ ﻟﻜﻞ ﻣﻦ ‪ r ،q‬و ‪ s‬ﺑﺎﺳﺘﻌﲈل اﳌﺮﺑﻌﺎت اﻟﺜﻼﺛﺔ اﻷﺧﺮى‪.‬‬


‫‪s−q‬‬
‫‪ .‬ﻣﺎذا ﺗﺴﺘﻨﺘﺞ ؟‬ ‫‪ •2‬أﺣﺴﺐ ‪:‬‬
‫‪r−p‬‬
‫‪ũŏ‬‬

‫أن اﻟﺮﺑﺎﻋﻲ ‪ ABCD‬ﻣﺒﺎﴍ و ‪ P‬ﻣﺮﻛﺰ اﳌﺜﻠﺚ اﳌﺮﺳﻮم ﺧﺎرﺟﻴ ًﺎ ﻋﲆ ]‪ّ [AB‬‬


‫ﻓﺈن ‪ A‬ﻫﻲ ﺻﻮرة ‪ B‬ﺑﺎﻟﺪوران‬ ‫‪ •1‬ﺑﲈ ّ‬ ‫ﺍﳊﻞّ‪.‬‬
‫‪a − ıb‬‬ ‫𝜋‬
‫= ‪.p‬‬ ‫‪ ℛ P,‬ﻣﻨﻪ )‪ a − p = ı (b − p‬أي ‪ a − ıb = p − ıp‬ﻣﻨﻪ‬
‫‪1−ı‬‬ ‫‪2‬‬
‫‪d − ıa‬‬ ‫‪c − ıd‬‬ ‫‪b − ıc‬‬
‫= ‪.s‬‬ ‫= ‪r‬و‬ ‫= ‪،q‬‬ ‫ﺑﺎﳌﺜﻞ ‪:‬‬
‫‪1−ı‬‬ ‫‪1−ı‬‬ ‫‪1−ı‬‬

‫𝟰𝟵𝟮‬

‫‪http ://tinyurl.com/Malki1718‬‬ ‫‪0‬‬


‫‪ .VII‬ﲤﺎرﻳﻦ ﻟﻠﺘﻌﻤﻖ‬
‫‪.‬‬

‫‪Ŕž Ŧ Ľ‬‬
‫‪R‬‬

‫‪S‬‬

‫‪œǃ‬‬
‫‪D‬‬

‫‪C‬‬
‫‪Q‬‬

‫‪Ŀ‬‬
‫‪A‬‬
‫‪B‬‬

‫‪P‬‬

‫ﺷﻜﻞ ‪4.VII‬‬

‫‪ •2‬ﳑﺎ ﺳﺒﻖ ﻧﺴﺘﻨﺘﺞ ّ‬


‫أن ‪:‬‬
‫]‪s − q d − b + ı (c − a) ı [−ı (d − b) + c − a‬‬
‫=‬ ‫=‬ ‫‪=ı‬‬
‫)‪r − p c − a + ı (b − d‬‬ ‫)‪c − a + ı (b − d‬‬
‫|‪|s − q‬‬
‫| ّ‬
‫ﻓﺈن ‪ PR = QS‬أي ‪:‬‬ ‫أن اﳌﺴﺘﻘﻴﻤﲔ )‪ (PR‬و )‪ (QS‬ﻣﺘﻌﺎﻣﺪان‪ .‬و ﺑﲈ ّ‬
‫أن ‪| = |ı| = 1‬‬ ‫ﻫﺬا ُﻳﺜﺒﺖ ّ‬
‫|| ‪|| r − p‬‬
‫‪ņőƱŀ Ţ‬‬
‫ﻗُﻄﺮﺍ ﺍﻟﺮﺑﺎﻋﻲ ‪ PQRS‬ﻣﺘﻌﺎﻣﺪﺍﻥ ﻭ ﳍﻤﺎ ﻧﻔﺲ ﺍﻟﻄﻮﻝ‪.‬‬

‫⧐‬
‫ﻫﺬه اﻟﻨﺘﻴﺠﺔ ﺗُﻌﺮف ﺑﺎﺳﻢ »ﻧﻈﺮﻳﺔ ﻓﻮﻥ ﺃﻭﺑﻞ«)‪. (Théorème de V A‬‬ ‫ﻣﻼﺣﻈﺔ ‪⧏ : 49‬‬

‫■‬

‫‪ ABCD‬ﻣﺘﻮازي أﺿﻼع ‪ .‬ﻧُﻨﺸﺊ ﻋﲆ اﻟﻀﻠﻌﲔ ‪ BC‬و ‪ ، CD‬ﻧﺤﻮ اﳉﻬﺔ اﳋﺎرﺟﻴﺔ‪ ،‬اﳌﺜﻠﺜﲔ ‪ BCE‬و ‪CDF‬‬
‫‪#‬‬ ‫‪.‬‬
‫𝟔𝟎𝟐‬ ‫‪.‬‬
‫‪ƕ‬‬

‫اﳌﺘﻘﺎﻳﺴﲔ اﻷﺿﻼع‪ .‬أﺛﺒﺖ ّ‬


‫أن اﳌﺜﻠﺚ ‪ AEF‬ﻣﺘﻘﺎﻳﺲ اﻷﺿﻼع‪.‬‬
‫‪VII‬‬

‫أن ‪ j = 1 :‬و ‪. 1 + j + j = 0‬‬‫ﺍﳊﻞّ‪ .‬ﻧﻀﻊ ‪ j = e /‬و ﻧﺬﻛّﺮ ّ‬


‫‪ũŏ‬‬

‫ﻟﺘﻜﻦ ‪ e ، d ، c ، b ، a‬و ‪ f‬ﻟﻮاﺣﻖ اﻟﻨﻘﻂ ‪ E ، D ، C ، B ، A‬و ‪ F‬ﻋﲆ اﻟﱰﺗﻴﺐ‪.‬‬


‫ﻓﺈن ‪ b − a = c − d‬أي ‪. a + c = b + d‬‬ ‫أن ‪ ABCD‬ﻣﺘﻮازي أﺿﻼع ّ‬ ‫ﺑﲈ ّ‬
‫𝜋‬
‫اﳌﺜﻠﺚ ‪ BCE‬ﻣﺘﻘﺎﻳﺲ اﻷﺿﻼع و ﻏﲑ ﻣﺒﺎﴍ إذن ‪ E‬ﻫﻲ ﺻﻮرة ‪ C‬ﺑﺎﻟﺪوران اﻟﺬي ﻣﺮﻛﺰه ‪ B‬و زاوﻳﺘﻪ ‪ −‬أي‬
‫‪3‬‬
‫)‪ e − b = e− / (c − b) = −j (c − b‬ﻣﻨﻪ ‪:‬‬
‫‪e = b − j (c − b) = (1 + j) b − jc = −j b − jc‬‬
‫𝜋‬
‫ﺑﺎﳌﺜﻞ ‪ CDF‬ﻣﺘﻘﺎﻳﺲ اﻷﺿﻼع و ﻏﲑ ﻣﺒﺎﴍ إذن ‪ F‬ﻫﻲ ﺻﻮرة ‪ D‬ﺑﺎﻟﺪوران اﻟﺬي ﻣﺮﻛﺰه ‪ C‬و زاوﻳﺘﻪ ‪ −‬ﻣﻨﻪ ‪.f = −j c − jd‬‬
‫‪3‬‬

‫𝟱𝟵𝟮‬

‫‪http ://tinyurl.com/Malki1718‬‬ ‫‪0‬‬


‫‪.‬‬
‫‪F‬‬

‫‪Ŕž Ŧ Ľ‬‬
‫‪œǃ‬‬
‫‪D‬‬ ‫‪C‬‬

‫‪Ŀ‬‬
‫‪E‬‬

‫‪A‬‬ ‫‪B‬‬
‫ﺷﻜﻞ ‪5.VII‬‬

‫أن ‪) f + ja + j e = 0‬و ﰲ ﻫﺬه اﳊﺎﻟﺔ ‪ AEF‬ﻣﺒﺎﴍ( أو‬ ‫أن اﳌﺜﻠﺚ ‪ AEF‬ﻣﺘﻘﺎﻳﺲ اﻷﺿﻼع‪ ،‬ﻳﻜﻔﻲ أن ﻧﱪﻫﻦ ّ‬
‫ﺣﺘﻰ ﻧﱪﻫﻦ ّ‬
‫‪) f + j a + je = 0‬و ﰲ ﻫﺬه اﳊﺎﻟﺔ ‪ AEF‬ﻏﲑ ﻣﺒﺎﴍ( و ﻫﺬا ﺣﺴﺐ اﻟﺘﻤﺮﻳﻦ ‪ 201‬ﺻﻔﺤﺔ ‪.290‬‬
‫ﻟﻜﻦ ﻧﻼﺣﻆ ﻋﲆ اﻟﺸﻜﻞ ‪ّ 5.VII‬‬
‫أن ‪ AEF‬ﻣﺒﺎﴍ إذن ﻳﻜﻔﻲ أن ﻧﱪﻫﻦ ّ‬
‫أن ‪ . f + ja + j e = 0 :‬ﻟﺪﻳﻨﺎ ‪:‬‬

‫‪f + ja + j e = −j c − jd + ja + j‬‬ ‫‪−j b − jc = −j c − jd + ja − j b − j c‬‬

‫‪= ja − jb − 1 + j‬‬ ‫‪c − jd = ja − jb + jc − jd = j (a − b + c − d) = 0‬‬

‫ّ‬
‫)ﻷن ‪ (a + c = b + d‬إذن اﳌﺜﻠﺚ ‪ AEF‬ﻣﺘﻘﺎﻳﺲ اﻷﺿﻼع )و ﻣﺒﺎﴍ(‪.‬‬
‫■‬

‫‬
‫‪ņőƱŀ Ţ‬‬
‫‪.‬‬
‫𝟕𝟎𝟐‬ ‫‪.‬‬
‫‪ ABC‬ﻣﺜﻠﺚ‪ .‬ﻧُﻨﺸﺊ اﳌﺮﺑﻊ ﻏﲑ اﳌﺒﺎﴍ ‪ ABDE‬و اﳌﺮﺑﻊ اﳌﺒﺎﴍ ‪ . ACFG‬ﻟﻴﻜﻦ ‪ M‬ﻣﻨﺘﺼﻒ اﻟﻘﻄﻌﺔ ]‪.[BC‬‬
‫)ﺷﻜﻞ‪.(6.VII‬‬ ‫أﺛﺒﺖ ّ‬
‫أن ‪. (AM) ⟂ (EG) :‬‬

‫ﺍﳊﻞّ‪ .‬ﻟﺘﻜﻦ ‪ g ، f ، e ، d ، c ، b ، a‬و ‪ m‬ﻟﻮاﺣﻖ اﻟﻨﻘﻂ ‪ G ، F ، E ، D ، C ، B ، A‬و ‪ M‬ﻋﲆ اﻟﱰﺗﻴﺐ‪.‬‬


‫‪b+c‬‬
‫= ‪.m‬‬ ‫أن ‪ M‬ﻣﻨﺘﺼﻒ اﻟﻘﻄﻌﺔ ]‪ّ [BC‬‬
‫ﻓﺈن‬ ‫ﺑﲈ ّ‬
‫‪2‬‬
‫𝜋‬
‫اﳌﺮﺑﻊ ‪ ABDE‬ﻏﲑ ﻣﺒﺎﴍ إذن اﻟﺸﻌﺎع ‪ AE‬ﻫﻮ ﺻﻮرة اﻟﺸﻌﺎع ‪ AB‬ﺑﺎﻟﺪوران اﻟﺬي ﻣﺮﻛﺰه ‪ A‬و زاوﻳﺘﻪ ‪ −‬أي‬
‫‪2‬‬
‫‪ƕ‬‬

‫)‪ e − a = e− / (b − a‬ﻣﻨﻪ )‪. e = a − ı (b − a‬‬


‫‪VII‬‬

‫𝜋‬
‫اﳌﺮﺑﻊ ‪ ACFG‬ﻣﺒﺎﴍ إذن اﻟﺸﻌﺎع ‪ AG‬ﻫﻮ ﺻﻮرة اﻟﺸﻌﺎع ‪ AC‬ﺑﺎﻟﺪوران اﻟﺬي ﻣﺮﻛﺰه ‪ A‬و زاوﻳﺘﻪ أي‬
‫‪2‬‬
‫)‪ g − a = e / (c − a‬ﻣﻨﻪ )‪ . g = a + ı (c − a‬ﻣﻨﻪ ‪:‬‬
‫‪ũŏ‬‬

‫‪b+c‬‬
‫‪g − e = (a + ı (c − a)) − (a − ı (b − a)) = ı (c + b − 2a) = 2ı‬‬ ‫‪−a‬‬
‫‪2‬‬
‫‪= 2ı (m − a) = 2e‬‬ ‫‪/‬‬
‫)‪(m − a‬‬
‫𝜋‬
‫و ﻫﺬا ﻳﻌﻨﻲ ّ‬
‫أن اﻟﺸﻌﺎع ‪ GE‬ﻫﻮ ﺻﻮرة اﻟﺸﻌﺎع ‪ MA‬ﺑﺘﺸﺎﺑﻪ ﻣﺒﺎﴍ )ﻧﺴﺒﺘﻪ ‪ 2‬و( زاوﻳﺘﻪ ‪ ،‬و ﺑﺎﻟﺘﺎﱄ )‪. (MA) ⟂ (GE‬‬
‫‪2‬‬
‫■‬

‫𝟲𝟵𝟮‬

‫‪http ://tinyurl.com/Malki1718‬‬ ‫‪0‬‬


‫‪ .VII‬ﲤﺎرﻳﻦ ﻟﻠﺘﻌﻤﻖ‬
‫‪.‬‬
‫‪F‬‬

‫‪C‬‬

‫‪Ŕž Ŧ Ľ‬‬
‫‪M‬‬

‫‪œǃ‬‬
‫‪G‬‬
‫‪A‬‬ ‫‪B‬‬

‫‪Ŀ‬‬ ‫‪E‬‬

‫ﺷﻜﻞ ‪6.VII‬‬
‫‪D‬‬

‫‪ ABC‬ﻣﺜﻠﺚ )ﻛﻴﻔﻲ( ﻣﺒﺎﴍ‪ .‬ﻧُﻨﺸﺊ‪ ،‬ﻧﺤﻮ اﳉﻬﺔ اﳋﺎرﺟﻴﺔ ﳍﺬا اﳌﺜﻠﺚ‪ ،‬اﳌﺮﺑﻌﺎت اﻟﺘﻲ ﻣﺮاﻛﺰﻫﺎ ‪ Q ، P‬و ‪ R‬و أﺣﺪ أﺿﻼﻋﻬﺎ‬
‫]‪ [BC] ، [AB‬و ]‪ [CA‬ﻋﲆ اﻟﱰﺗﻴﺐ‪) .‬ﺷﻜﻞ ‪(7.VII‬‬
‫ﻧﻨﺴﺐ اﳌﺴﺘﻮي إﱃ ﻣﻌﻠﻢ ﻣﺘﻌﺎﻣﺪ ﻣﺘﺠﺎﻧﺲ و ﻣﺒﺎﴍ ⃗ ‪. O, i, j‬‬
‫⃗‬
‫‪.‬‬
‫𝟖𝟎𝟐‬ ‫‪.‬‬

‫ﻟﺘﻜﻦ ‪ q ،p ،c ،b ،a‬و ‪ r‬ﻟﻮاﺣﻖ اﻟﻨﻘﻂ ‪ Q ،P ،C ،B ،A‬و ‪ R‬ﻋﲆ اﻟﱰﺗﻴﺐ‪.‬‬


‫‪ •1‬أﺛﺒﺖ ّ‬
‫أن اﳌﺜﻠﺜﲔ ‪ ABC‬و ‪ PQR‬ﳍﲈ ﻧﻔﺲ ﻣﺮﻛﺰ اﻟﺜﻘﻞ‪.‬‬
‫‪ņőƱŀ Ţ‬‬
‫‪a − ıb‬‬
‫= ‪.p‬‬ ‫‪ •2‬أﺛﺒﺖ أﻧﻪ ﰲ اﳌﺮﺑﻊ اﳌﺮﺳﻮم ﻋﲆ ]‪ [AB‬ﻟﺪﻳﻨﺎ ‪:‬‬
‫‪1−ı‬‬
‫أوﺟﺪ ﻋﺒﺎرة ﻣﺸﺎﲠﺔ ﻟﻜﻞ ﻣﻦ ‪ q‬و ‪ r‬ﺑﺎﺳﺘﻌﲈل اﳌﺮﺑﻌﲔ اﻵﺧﺮﻳﻦ‪.‬‬
‫‪ •3‬أﺛﺒﺖ ّ‬
‫أن اﳌﺴﺘﻘﻴﻤﲔ )‪ (AQ‬و )‪ (PR‬ﻣﺘﻌﺎﻣﺪان‪.‬‬
‫أن اﳌﺴﺘﻘﻴﲈت )‪ (BR) ، (AQ‬و )‪ (CP‬ﻣﺘﻘﺎﻃﻌﺔ ﰲ ﻧﻘﻄﺔ واﺣﺪة )ﺗُﺪﻋﻰ ﻧﻘﻄﺔ ﻓ‪‬ﻜﺘ‪‬ﻦ — ‪.(Point de V‬‬
‫إﺳﺘﻨﺘﺞ ّ‬

‫ﺍﳊﻞّ‪.‬‬
‫𝜋‬
‫)‪. a − p = ı (b − p‬‬ ‫‪ّ ℛ P,‬‬
‫ﻓﺈن ‪:‬‬ ‫‪ •1‬ﺑﲈ ّ‬
‫أن ‪ A‬ﻫﻲ ﺻﻮرة ‪ B‬ﺑﺎﻟﺪوران‬
‫‪ƕ‬‬

‫‪2‬‬
‫‪VII‬‬

‫)‪. c − r = ı (a − r‬‬ ‫و‬ ‫)‪b − q = ı (c − q‬‬ ‫ﺑﺎﳌﺜﻞ ‪:‬‬


‫ﺑﺠﻤﻊ ﻫﺬه اﳌﺴﺎوﻳﺎت ﻃﺮﻓ ًﺎإﱃ ﻃﺮف ﻳﻨﺘﺞ ‪:‬‬
‫))‪a + b + c − (p + q + r) = ı (a + b + c − (p + q + r‬‬
‫‪ũŏ‬‬

‫‪a+b+c p+q+r‬‬
‫و ﻫﺬا ﻳﻌﻨﻲ أﻧّﻪ ﻟﻠﻤﺜﻠﺜﲔ ‪ ABC‬و ‪ PQR‬ﻧﻔﺲ ﻣﺮﻛﺰ‬ ‫=‬ ‫أي ‪ a + b + c = p + q + r :‬ﻣﻨﻪ‬
‫‪3‬‬ ‫‪3‬‬
‫اﻟﺜﻘﻞ‪.‬‬
‫‪a − ıb‬‬
‫= ‪.p‬‬ ‫‪ •2‬ﻣﻦ اﳌﺴﺎواة )‪ a − p = ı (b − p‬ﻧﺴﺘﻨﺘﺞ ّ‬
‫أن‬
‫‪1−ı‬‬
‫‪c − ıa‬‬ ‫‪b − ıc‬‬
‫= ‪.r‬‬ ‫و‬ ‫=‪q‬‬ ‫ﺑﺎﳌﺜﻞ ‪:‬‬
‫‪1−ı‬‬ ‫‪1−ı‬‬

‫𝟳𝟵𝟮‬

‫‪http ://tinyurl.com/Malki1718‬‬ ‫‪0‬‬


‫‪.‬‬

‫‪Ŕž Ŧ Ľ‬‬
‫‪R‬‬
‫‪P‬‬
‫‪A‬‬

‫‪œǃ‬‬
‫‪C‬‬

‫‪Ŀ‬‬
‫‪B‬‬

‫‪Q‬‬

‫ﺷﻜﻞ ‪7.VII‬‬

‫‪r−p‬‬ ‫)‪c − a + ı (b − a‬‬


‫أن اﳌﺴﺘﻘﻴﻤﲔ )‪ (PR‬و )‪ (AQ‬ﻣﺘﻌﺎﻣﺪان أي ّ‬
‫أن ‪:‬‬ ‫‪ .‬ﻧﺴﺘﻨﺘﺞ ّ‬ ‫=‬ ‫ﻟﺪﻳﻨﺎ ‪= ı :‬‬ ‫•‬ ‫‪3‬‬
‫)‪q − a b − a + ı (a − c‬‬

‫)‪ (AQ‬ﻫﻮ ﺍﻟﻌﻤﻮﺩ ﺍﻟﻨﺎﺯﻝ ﻣﻦ ‪ A‬ﰲ ﺍﳌﺜﻠﺚ ‪PQR‬‬

‫أي ﻣﺜﻠﺚ ﺗﺘﻘﺎﻃﻊ‬ ‫أن )‪ (BR‬و )‪ (CP‬ﳘﺎ اﻟﻌﻤﻮدﻳﻦ اﻵﺧﺮﻳﻦ ﰲ اﳌﺜﻠﺚ ‪ . PQR‬و ﺑﲈ ّ‬
‫أن أﻋﻤﺪة ﱢ‬ ‫ﺑﺎﺗﺒﺎع ﻧﻔﺲ اﻟﻄﺮﻳﻘﺔ ﻧُﺜﺒﺖ ّ‬
‫ﰲ ﻧﻘﻄﺔ واﺣﺪة ﻫﻲ )ﻣﺮﻛﺰ ﺛﻘﻞ ﻫﺬا اﳌﺜﻠﺚ( ّ‬
‫ﻓﺈن ‪:‬‬
‫ﺍﳌﺴﺘﻘﻴﻤﺎﺕ )‪ (BR) ، (AQ‬ﻭ )‪ (CP‬ﻣﺘﻘﺎﻃﻌﺔ ﰲ ﻧﻘﻄﺔ ﻭﺍﺣﺪﺓ‬
‫‪ņőƱŀ Ţ‬‬
‫■‬

‫‪.‬‬
‫𝟗𝟎𝟐‬ ‫‪.‬‬
‫اﳌﺴﺘﻮي ﻣﻨﺴﻮب إﱃ ﻣﻌﻠﻢ ﻣﺘﻌﺎﻣﺪ ﻣﺘﺠﺎﻧﺲ و ﻣﺒﺎﴍ ⃗ ‪. O, i,⃗ j‬‬

‫اﻷول ‪ :‬ﺧﺼﻮﺻﻴﺎﺕ ﺍﳌﺜﻠﺚ ﺍﳌﺘﻘﺎﻳﺲ ﺍﻷﺿﻼﻉ‬


‫اﳉﺰء ّ‬
‫‪ j = e‬و ﻟﺘﻜﻦ ‪ V ، U‬و ‪ W‬ﺛﻼﺛﺔ ﻧﻘﻂ ﻣﻦ اﳌﺴﺘﻮي ﻟﻮاﺣﻘﻬﺎ ‪ v ، u‬و ‪ w‬ﻋﲆ اﻟﱰﺗﻴﺐ‪.‬‬ ‫ﻧﻀﻊ‬
‫‪ƕ‬‬

‫‪VII‬‬

‫‪ UVW‬ﻣﺘﻘﺎﻳﺲ اﻷﺿﻼع و ﻣﺒﺎﴍ ⟺ )‪. u − v = −j (w − v‬‬ ‫‪ •1‬أﺛﺒﺖ ّ‬


‫أن ‪:‬‬

‫‪ UVW‬ﻣﺘﻘﺎﻳﺲ اﻷﺿﻼع و ﻣﺒﺎﴍ ⟺ ‪. u + jv + j w = 0‬‬ ‫‪ •2‬أﺛﺒﺖ ّ‬


‫أن ‪:‬‬
‫‪ũŏ‬‬

‫اﳉﺰء اﻟﺜﺎﲏ ‪ :‬ﻧﻈﺮﻳﺔ ﻧﺎﺑﻮﻟﻴﻮﻥ )‪(Théorème de N‬‬

‫‪ ABC‬ﻣﺜﻠﺚ )ﻛﻴﻔﻲ( ﻣﺒﺎﴍ‪ .‬ﻧُﻨﺸﺊ‪ ،‬اﻟﻨﻘﻂ ‪ Q ، P‬و ‪ R‬ﺑﺤﻴﺚ ﺗﻜﻮن اﳌﺜﻠﺜﺎت ‪ CQA ، BPC‬و ‪ ARB‬ﻣﺘﻘﺎﻳﺴﺔ اﻷﺿﻼع و‬
‫ﻣﺒﺎﴍة‪ .‬ﻟﺘﻜﻦ ‪ V ، U‬و ‪ W‬ﻣﺮاﻛﺰ ﺛﻘﻞ ‪ CQA ، BPC‬و ‪ ARB‬ﻋﲆ اﻟﱰﺗﻴﺐ‪.‬‬
‫أن اﳌﺜﻠﺚ ‪ UVW‬ﻣﺘﻘﺎﻳﺲ اﻷﺿﻼع و ﻟﻪ ﻧﻔﺲ ﻣﺮﻛﺰ اﻟﺜﻘﻞ ﻣﻊ ‪. ABC‬‬ ‫ﺑﺮﻫﻦ ّ‬

‫𝟴𝟵𝟮‬

‫‪http ://tinyurl.com/Malki1718‬‬ ‫‪0‬‬


‫‪ .VII‬ﲤﺎرﻳﻦ ﻟﻠﺘﻌﻤﻖ‬
‫‪.‬‬
‫‪R‬‬

‫‪Q‬‬
‫‪A‬‬

‫‪Ŕž Ŧ Ľ‬‬
‫‪W‬‬
‫‪V‬‬

‫‪œǃ‬‬
‫‪B‬‬ ‫‪C‬‬

‫‪Ŀ‬‬
‫‪U‬‬

‫‪P‬‬

‫ﺷﻜﻞ ‪8.VII‬‬

‫ﺍﳊﻞّ‪.‬‬
‫اﻷول ‪ :‬ﺧﺼﻮﺻﻴﺎﺕ ﺍﳌﺜﻠﺚ ﺍﳌﺘﻘﺎﻳﺲ ﺍﻷﺿﻼﻉ‬
‫اﳉﺰء ّ‬
‫𝜋‬
‫‪ℛ V,‬‬ ‫‪ •1‬إذا ﻛﺎن اﳌﺜﻠﺚ ‪ UVW‬ﻣﺘﻘﺎﻳﺲ اﻷﺿﻼع و ﻣﺒﺎﴍ ًا ّ‬
‫ﻓﺈن ‪ U‬ﻫﻲ ﺻﻮرة ‪ W‬ﺑﺎﻟﺪوران‬
‫‪3‬‬
‫‪.u−v = e‬‬ ‫‪/‬‬
‫أي ‪(w − v) :‬‬
‫‪ e‬ﻣﻨﻪ )‪. u − v = −j (w − v‬‬ ‫‪/‬‬ ‫‪= e e−‬‬ ‫‪/‬‬ ‫‪= −e−‬‬ ‫‪/‬‬ ‫‪= −e‬‬ ‫‪/‬‬ ‫‪= −j‬‬ ‫ﻟﻜﻦ ‪:‬‬
‫ﻓﺈن ‪ U‬ﻫﻲ ﺻﻮرة ‪ W‬ﺑﺎﻟﺪوران 𝜋 ‪ ℛ V,‬أي ّ‬
‫أن‬ ‫ﺑﺎﻟﻌﻜﺲ‪ ،‬إذا ﻛﺎن )‪ّ u − v = −j (w − v) = e / (w − v‬‬
‫‪3‬‬
‫اﳌﺜﻠﺚ ‪ UVW‬ﻣﺘﻘﺎﻳﺲ اﻷﺿﻼع و ﻣﺒﺎﴍ‪.‬‬
‫‪ņőƱŀ Ţ‬‬
‫‪ •2‬إذا ﻛﺎن اﳌﺜﻠﺚ ‪ UVW‬ﻣﺘﻘﺎﻳﺲ اﻷﺿﻼع و ﻣﺒﺎﴍ ًا ﻓﺤﺴﺐ ﻣﺎ ﺳﺒﻖ ﻟﺪﻳﻨﺎ ‪:‬‬
‫‪. u + −1 − j‬‬ ‫)‪ u − v = −j (w − v‬أي ‪v + j w = 0‬‬

‫ﻟﻜﻦ ‪ 1 + j + j = 0‬أي ‪ −1 − j = j‬ﻣﻨﻪ ‪. u + jv + j w = 0‬‬


‫‪ u + −1 − j‬أي )‪ ، u − v = −j (w − v‬و ﺣﺴﺐ‬ ‫ﺑﺎﻟﻌﻜﺲ‪ ،‬إذا ﻛﺎن ‪ّ u + jv + j w = 0‬‬
‫ﻓﺈن ‪v + j w = 0‬‬
‫اﻷول ّ‬
‫ﻓﺈن اﳌﺜﻠﺚ ‪ UVW‬ﻣﺘﻘﺎﻳﺲ اﻷﺿﻼع و ﻣﺒﺎﴍ‪.‬‬ ‫اﻟﺴﺆال ّ‬
‫اﳉﺰء اﻟﺜﺎﲏ ‪ :‬ﺇﺛﺒﺎﺕ ﻧﻈﺮﻳﺔ ﻧﺎﺑﻮﻟﻴﻮﻥ‬
‫‪ƕ‬‬
‫‪VII‬‬

‫ﻓﺮﺿ ًﺎ ﻟﺪﻳﻨﺎ ‪:‬‬


‫)‪a − w = j (b − w‬‬ ‫)𝟑(‬
‫‪ũŏ‬‬

‫)‪b − u = j (c − u‬‬ ‫)𝟒(‬


‫)‪c − v = j (a − v‬‬ ‫)𝟓(‬

‫ﺑﺠﻤﻊ ﻫﺬه اﳌﺴﺎوﻳﺎت ﻃﺮﻓ ًﺎإﱃ ﻃﺮف ﻳﻨﺘﺞ ‪a + b + c − (u + v + w) = j (a + b + c − (u + v + w)) :‬‬


‫ﻣﻨﻪ ‪ a + b + c = u + v + w‬و ﻫﺬا ﻳﻌﻨﻲ ّ‬
‫أن اﳌﺜﻠﺚ ‪ UVW‬ﻟﻪ ﻧﻔﺲ ﻣﺮﻛﺰ اﻟﺜﻘﻞ ﻣﻊ اﳌﺜﻠﺚ ‪. ABC‬‬
‫‪a − jb‬‬
‫= ‪.w‬‬ ‫ﻣﻦ اﳌﻌﺎدﻟﺔ )𝟑( ﻧﺴﺘﻨﺘﺞ ّ‬
‫أن ‪:‬‬
‫‪1−j‬‬

‫𝟵𝟵𝟮‬

‫‪http ://tinyurl.com/Malki1718‬‬ ‫‪0‬‬


‫‪.‬‬
‫‪c − ja‬‬ ‫‪b − jc‬‬
‫ﻣﻨﻪ ‪:‬‬ ‫=‪ v‬؛‬
‫=‪ u‬و‬ ‫ﺑﺎﳌﺜﻞ ﻣﻊ )𝟒( و )𝟓( ‪:‬‬
‫‪1−j‬‬ ‫‪1−j‬‬

‫‪Ŕž Ŧ Ľ‬‬
‫‪a − jb + j (b − jc) + j (c − ja) a − jb + jb − j c + j c − a‬‬
‫= ‪u + jv + j w‬‬ ‫=‬ ‫‪=0‬‬
‫‪1−j‬‬ ‫‪1−j‬‬

‫‪œǃ‬‬
‫اﻷول ﻣﻦ اﻟﺘﻤﺮﻳﻦ ّ‬
‫ﻓﺈن اﳌﺜﻠﺚ ‪ UVW‬ﻣﺘﻘﺎﻳﺲ اﻷﺿﻼع و ﻣﺒﺎﴍ‪.‬‬ ‫و ﺣﺴﺐ اﳉﺰء ّ‬
‫‪R‬‬

‫‪A‬‬ ‫‪Q‬‬
‫أن اﳌﺴﺘﻘﻴﲈت )‪،(AP‬‬ ‫ﻣﻼﺣﻈﺔ ‪ ⧏ : 50‬ﻳﻤﻜﻦ إﺛﺒﺎت ّ‬

‫‪Ŀ‬‬
‫‪W‬‬
‫‪V‬‬ ‫)‪ (BQ‬و )‪ (CR‬ﺗﺘﻘﺎﻃﻊ ﰲ ﻧﻘﻄﺔ واﺣﺪة ‪ T‬ﺗُﺪﻋﻰ ﻧﻘﻄﺔ‬
‫‪r‬‬
‫ﻃﻮﺭﻳﺘﺸﻠﹼﻲ )‪ .(Point de T‬ﳍﺬه اﻟﻨﻘﻄﺔ ﻋﺪة ﺧﻮاص‬
‫ﻣﻨﻬﺎ ‪ :‬اﻟﻨﻘﻄﺔ ‪ T‬ﻫﻲ ﻧﻘﻄﺔ ﺗﻘﺎﻃﻊ اﻟﺪواﺋﺮ اﳌﺤﻴﻄﺔ ﺑﺎﳌﺜﻠﺜﺎت‬
‫‪B‬‬ ‫‪C‬‬
‫‪ ACQ ، ABR‬و ‪ BCP‬؛ ﻫﻲ أﻳﻀ ًﺎ اﻟﻨﻘﻄﺔ اﻟﺘﻲ ﲡﻌﻞ اﳌﺴﺎﻓﺔ‬
‫‪U‬‬ ‫‪ MA+MB+MC‬أﺻﻐﺮ ﻣﺎ ﻳﻤﻜﻦ )ﻋﻨﺪﻣﺎ ﺗﻜﻮن زواﻳﺎ اﳌﺜﻠﺚ‬
‫⧐‬
‫أﺻﻐﺮ ﻣﻦ ∘‪ — (120‬ﻛﲈ ﰲ اﻟﺸﻜﻞ اﳌﻘﺎﺑﻞ‪.‬‬

‫‪P‬‬
‫■‬

‫اﳌﺴﺘﻮي 𝒫 ﻣﻨﺴﻮب إﱃ ﻣﻌﻠﻢ ﻣﺘﻌﺎﻣﺪ ﻣﺘﺠﺎﻧﺲ و ﻣﺒﺎﴍ ⃗ ‪. O, i,⃗ j‬‬


‫‪.‬‬
‫𝟎𝟏𝟐‬ ‫!‬
‫‪.‬‬

‫ﻟﺘﻜﻦ ‪ B ، A‬و ‪ C‬ﺛﻼﺛﺔ ﻧﻘﻂ ﻣﻦ 𝒫 ﻟﻮاﺣﻘﻬﺎ ‪ b ، a‬و ‪ c‬ﻋﲆ اﻟﱰﺗﻴﺐ ﺑﺤﻴﺚ |‪. |a| = ||b|| = |c‬‬
‫أﻧﻈﺮ اﻟﺘﻤﺮﻳﻦ ‪ 195‬ﺻﻔﺤﺔ ‪280‬‬ ‫‬ ‫أن اﳌﺜﻠﺚ ‪ ABC‬ﻣﺘﻘﺎﻳﺲ اﻷﺿﻼع إذا و ﻓﻘﻂ إذا ﻛﺎن ‪. a + b + c = 0‬‬‫أﺛﺒﺖ ّ‬

‫ﺍﳊﻞّ‪ .‬ﻧﻀﻊ |‪ b = Re ، a = Re ، R = |a| = ||b|| = |c‬و ‪. c = Re‬‬


‫‪−‬‬ ‫‪−‬‬
‫‪ņőƱŀ Ţ‬‬
‫‪ 1 + e‬و ﺑﺎﳌﺴﺎواة ﺑﲔ اﻷﺟﺰاء‬ ‫‪+e‬‬ ‫‪ Re‬ﻣﻨﻪ ‪= 0‬‬ ‫ﻓﺈن ‪+ Re + Re = 0‬‬ ‫إذا ﻛﺎن ‪ّ a + b + c = 0‬‬ ‫•‬ ‫‪1‬‬
‫اﻟﺘﺨﻴﻠﻴﺔ ّ‬
‫ﻓﺈن ‪ sin (𝛽 − 𝛼) + sin (𝛾 − 𝛼) = 0‬أي ‪:‬‬
‫)𝜋‪. 𝛽 − 𝛼 = 𝜋 − (−(𝛾 − 𝛼)) (mod 2‬‬ ‫أو‬ ‫)𝜋‪𝛽 − 𝛼 = −(𝛾 − 𝛼) (mod 2‬‬

‫إذا ﻛﺎن )𝜋‪ 𝛽 − 𝛼 = −(𝛾 − 𝛼) (mod 2‬ﻓﺒﺎﳌﺴﺎواة ﺑﲔ اﻷﺟﺰاء اﳊﻘﻴﻘﻴﺔ ﻳﻨﺘﺞ ‪:‬‬ ‫•‬

‫‪ . 1 + cos (𝛽 − 𝛼) + cos (𝛾 − 𝛼) = 0‬ﻟﻜﻦ ‪ 𝛽 − 𝛼 = −(𝛾 − 𝛼) (mod 2𝜋) :‬ﻣﻨﻪ ‪:‬‬


‫‪1‬‬
‫‪. cos (𝛽 − 𝛼) = cos (𝛾 − 𝛼) = −‬‬
‫‪ 1 + 2 cos (𝛾 − 𝛼) = 0‬أي‬
‫‪2‬‬
‫إذا ﻛﺎن )𝜋‪ّ 𝛽 − 𝛼 = 𝜋 + (𝛾 − 𝛼) (mod 2‬‬
‫ﻓﺈن ‪ 1 + cos (𝛽 − 𝛼) + cos (𝛾 − 𝛼) = 0‬ﻣﻨﻪ‬ ‫•‬
‫‪ƕ‬‬

‫‪ 1 + cos (𝛽 − 𝛼) − cos (𝛽 − 𝛼) = 0‬أي ‪1 = 0‬؛ إذن ﻻ ﻳﻤﻜﻦ أن ﻳﻜﻮن )𝜋‪.𝛽 − 𝛼 = −(𝛾 − 𝛼) (mod 2‬‬
‫‪VII‬‬

‫𝜋‪2‬‬ ‫‪1‬‬
‫‪ 𝛽 − 𝛼 = 𝛼 − 𝛾 = ±‬ﻣﻨﻪ ‪:‬‬ ‫ﰲ اﻷﺧﲑ‪ cos (𝛽 − 𝛼) = cos (𝛼 − 𝛾) = − ،‬أي‬
‫‪3‬‬ ‫‪2‬‬
‫‪ũŏ‬‬

‫)‪ b = j a‬و ‪.(c = ja‬‬ ‫أو‬ ‫)‪ b = ja‬و ‪(c = j a‬‬


‫و ﻫﺬا ﻳﻌﻨﻲ ّ‬
‫أن اﳌﺜﻠﺚ ‪ ABC‬ﻣﺘﻘﺎﻳﺲ اﻷﺿﻼع‪.‬‬
‫‪ •2‬ﻧﻔﺮض ّ‬
‫أن ‪ ABC‬ﻣﺘﻘﺎﻳﺲ اﻷﺿﻼع ‪.‬‬
‫ﻣﻦ اﳌﺴﺎواة ||‪ ||a − 0|| = ||b − 0|| = ||c − 0‬ﻧﺴﺘﻨﺘﺞ ّ‬
‫أن اﻟﻨﻘﻂ ‪ B ، A‬و ‪ C‬ﻣﺘﺴﺎوﻳﺔ اﳌﺴﺎﻓﺔ ﻋﻦ اﻟﻨﻘﻄﺔ ‪ ، O‬ﻣﺒﺪأ اﳌﻌﻠﻢ‪،‬‬
‫أن ‪ O‬ﻫﻲ ﻣﺮﻛﺰ اﻟﺪاﺋﺮة اﳌﺤﻴﻄﺔ ﺑﺎﳌﺜﻠﺚ ‪. ABC‬‬ ‫أي ّ‬

‫𝟬𝟬𝟯‬

‫‪http ://tinyurl.com/Malki1718‬‬ ‫‪0‬‬


‫‪ .VII‬ﲤﺎرﻳﻦ ﻟﻠﺘﻌﻤﻖ‬
‫‪.‬‬
‫‪B‬‬

‫‪M‬‬

‫‪Ŕž Ŧ Ľ‬‬
‫‪A‬‬
‫‪Ω‬‬
‫‪N‬‬

‫‪œǃ‬‬
‫‪P‬‬

‫‪Ŀ‬‬
‫‪C‬‬

‫ﺷﻜﻞ ‪9.VII‬‬

‫‪1‬‬
‫ﻓﺈن ‪ O‬ﻫﻲ أﻳﻀ ًﺎ ﻣﺮﻛﺰ ﺛﻘﻞ ﻫﺬا اﳌﺜﻠﺚ ﻣﻨﻪ ‪(a + b + c) = 0‬‬
‫أن اﳌﺜﻠﺚ ‪ ABC‬ﻣﺘﻘﺎﻳﺲ اﻷﺿﻼع ّ‬
‫و ﺑﲈ ّ‬
‫‪3‬‬
‫أي ‪. a + b + c = 0‬‬
‫■‬

‫‪.‬‬
‫𝟏𝟏𝟐‬ ‫‪.‬‬
‫‪.‬‬ ‫⃗ ‪O, i,⃗ j‬‬ ‫اﳌﺴﺘﻮي اﳌﺮﻛﺐ ﻣﻨﺴﻮب إﱃ ﻣﻌﻠﻢ ﻣﺘﻌﺎﻣﺪ و ﻣﺘﺠﺎﻧﺲ‬
‫ﻧﻌﺘﱪ اﻟﻨﻘﻂ )‪ N (u) ، M (z) ، C (c) ، B (b) ، A (a‬و )‪ P (v‬ﺑﺤﻴﺚ ﲤ ﱢﺜﻞ اﻟﻨﻘﻂ ‪ N ، M‬و ‪ P‬ﻧﻘﻂ ﺗﻘﺎﻃﻊ اﳌﻨﺼﻔﺎت‬
‫اﻟﺪاﺧﻠﻴﺔ ﻟﺰواﻳﺎ اﳌﺜﻠﺚ ‪ ABC‬ﻣﻊ ]‪ [BC] ، [AB‬و ]‪ [AC‬ﻋﲆ اﻟﱰﺗﻴﺐ )ﺷﻜﻞ ‪. (9.VII‬‬
‫ﻧﻀﻊ ‪ 𝛽 = AC ، 𝛼 = BC‬و ‪. 𝛾 = AB‬‬
‫𝛽‬ ‫𝛾‬
‫= ‪.u‬‬ ‫‪b+‬‬ ‫أﺛﺒﺖ ّ‬
‫أن ‪c :‬‬ ‫•‬‫‪1‬‬
‫𝛾‪𝛽+‬‬ ‫𝛾‪𝛽+‬‬
‫‪𝛼a + 𝛽b + 𝛾c‬‬
‫= 𝜔‪.‬‬ ‫‪ •2‬ﻟﻴﻜﻦ )𝜔( ‪ Ω‬ﻣﺮﻛﺰ اﻟﺪاﺋﺮة اﻟﺘﻲ ﲤﺲ داﺧﻠﻴ ًﺎ أﺿﻼع اﳌﺜﻠﺚ ‪ . ABC‬أﺛﺒﺖ ّ‬
‫أن ‪:‬‬
‫‪ņőƱŀ Ţ‬‬
‫𝛾‪𝛼+𝛽+‬‬

‫‪ •3‬ﱢﺑﲔ ّ‬
‫أن اﳌﺜﻠﺚ ‪ ABC‬ﻣﺘﻘﺎﻳﺲ اﻷﺿﻼع إذا و ﻓﻘﻂ ﻛﺎن ‪. OA + OB + OC = OM + ON + OP :‬‬

‫أﻧﻈﺮ اﻟﺘﻤﺮﻳﻦ ‪ 212‬ﺻﻔﺤﺔ ‪302‬‬ ‫‬


‫ﺍﳊﻞّ‪.‬‬
‫‪BN‬‬
‫= ‪ . BN‬ﻟﻜﻦ ‪ N‬ﻫﻲ ﻧﻘﻄﺔ ﺗﻘﺎﻃﻊ اﳌﻨﺼﻒ اﻟﺪاﺧﲇ ﻟﻠﺰاوﻳﺔ ‪ A‬ﻣﻊ اﻟﻘﻄﻌﺔ‬ ‫أن ]‪ّ N ∈ [BC‬‬
‫ﻓﺈن ‪BC‬‬ ‫‪ •1‬ﺑﲈ ّ‬
‫‪BC‬‬
‫‪AB‬‬ ‫‪BN‬‬ ‫‪AB‬‬
‫= ‪. BN‬‬ ‫أي ‪CN‬‬ ‫=‬ ‫اﳌﺴﺘﻘﻴﻤﺔ ]‪ [BC‬إذن‬
‫‪ƕ‬‬

‫‪AC‬‬ ‫‪CN AC‬‬


‫‪VII‬‬

‫‪AB‬‬ ‫‪AB + AC‬‬


‫= ‪BC‬‬ ‫= ‪CN + CN‬‬ ‫أن ‪ّ BC = BN + NC‬‬
‫ﻓﺈن ‪CN‬‬ ‫و ﺑﲈ ّ‬
‫‪AC‬‬ ‫‪AC‬‬
‫‪AC‬‬ ‫𝛾‬
‫= ‪ . BN‬ﻟﻜﻦ اﻟﺸﻌﺎﻋﲔ ‪ BN‬و ‪ BC‬ﻣﺘﻮازﻳﺎن و ﳍﲈ ﻧﻔﺲ اﻹﲡﺎه‬ ‫= ‪BC‬‬ ‫ﻣﻨﻪ ‪BC‬‬
‫‪ũŏ‬‬

‫‪AB + AC‬‬ ‫𝛾‪𝛽+‬‬


‫𝛽‬ ‫𝛾‬ ‫𝛾‬ ‫𝛾‬
‫= ‪.u‬‬ ‫‪b+‬‬ ‫= ‪ u − b‬ﻣﻨﻪ ‪c :‬‬ ‫= ‪ BN‬أي )‪(c − b‬‬ ‫إذن ‪BC‬‬
‫𝛾‪𝛽+‬‬ ‫𝛾‪𝛽+‬‬ ‫𝛾‪𝛽+‬‬ ‫𝛾‪𝛽+‬‬

‫أن ﻣﺮﻛﺰ اﻟﺪاﺋﺮة اﻟﺘﻲ ﲤﺲ داﺧﻠﻴ ًﺎ أﺿﻼع ﻣﺜﻠﺚ ﻫﻲ ﻧﻘﻄﺔ ﺗﻘﺎﻃﻊ اﳌﻨﺼﻔﺎت اﻟﺪاﺧﻠﻴﺔ ﻟﺰواﻳﺎه ّ‬
‫ﻓﺈن ]‪Ω ∈ [AN‬‬ ‫ﺑﲈ ّ‬ ‫•‬‫‪2‬‬
‫‪AΩ‬‬ ‫‪CA‬‬ ‫‪AΩ‬‬
‫أي‬ ‫=‬ ‫= ‪ . AΩ‬ﻟﻜﻦ )‪ (CΩ‬ﻫﻮ اﳌﻨﺼﻒ اﻟﺪاﺧﲇ ﻟﻠﺰاوﻳﺔ ‪ C‬ﰲ اﳌﺜﻠﺚ ‪ ACN‬إذن‬ ‫ﻣﻨﻪ ‪AN‬‬
‫‪NΩ‬‬ ‫‪CN‬‬ ‫‪AN‬‬

‫𝟭𝟬𝟯‬

‫‪http ://tinyurl.com/Malki1718‬‬ ‫‪0‬‬


‫‪.‬‬
‫‪CN‬‬ ‫‪CA + CN‬‬
‫‪ AN = AΩ +‬ﻣﻨﻪ‬ ‫= ‪AΩ‬‬ ‫أن ‪ّ AN = AΩ + ΩN‬‬
‫ﻓﺈن ‪AΩ‬‬ ‫‪ . NΩ = CN AΩ‬و ﺑﲈ ّ‬
‫‪AC‬‬ ‫‪CA‬‬ ‫‪CA‬‬
‫‪CA‬‬ ‫𝛽‬

‫‪Ŕž Ŧ Ľ‬‬
‫= ‪. AΩ‬‬ ‫= ‪AN‬‬ ‫‪AN‬‬
‫‪CA + CN‬‬ ‫‪𝛽 + CN‬‬
‫𝛼𝛾‬ ‫𝛾‬ ‫𝛽‬

‫‪œǃ‬‬
‫= ‪ BN‬إذن ‪:‬‬ ‫=‬ ‫ﻟﻜﻦ‪ ،‬و ﻣﻦ اﻟﺴﺆال اﻟﺴﺎﺑﻖ‪ ،‬ﻟﺪﻳﻨﺎ ‪ CN = BN :‬و‬
‫𝛾‪𝛽+𝛾 𝛽+‬‬ ‫𝛾‬
‫𝛽‬ ‫𝛽‬ ‫‪1‬‬ ‫𝛾‪𝛽+‬‬
‫= ‪AΩ‬‬ ‫= ‪AN‬‬ ‫= ‪AN‬‬ ‫= ‪AN‬‬ ‫‪AN‬‬
‫‪𝛽 + CN‬‬ ‫‪𝛽+‬‬ ‫‪1+ +‬‬ ‫𝛾‪𝛼+𝛽+‬‬
‫‪+‬‬

‫‪Ŀ‬‬
‫𝛾‪𝛽+‬‬ ‫𝛾‪𝛽+‬‬
‫= ‪.𝜔−a‬‬ ‫= ‪ AΩ‬أي )‪(u − a‬‬ ‫أن اﻟﺸﻌﺎﻋﲔ ‪ AΩ‬و ‪ AN‬ﻣﺘﻮازﻳﺎن و ﳍﲈ ﻧﻔﺲ اﻹﲡﺎه ّ‬
‫ﻓﺈن ‪AN‬‬ ‫و ﺑﲈ ّ‬
‫𝛾‪𝛼+𝛽+‬‬ ‫𝛾‪𝛼+𝛽+‬‬
‫‪𝛼a + 𝛽b + 𝛾c‬‬ ‫𝛽‬ ‫𝛾‬
‫= 𝜔‪.‬‬ ‫= ‪ u‬إذن ‪:‬‬ ‫‪b+‬‬ ‫ﻟﻜﻦ ‪c‬‬
‫𝛾‪𝛼+𝛽+‬‬ ‫𝛾‪𝛽+‬‬ ‫𝛾‪𝛽+‬‬
‫‪ •3‬ﻟﺪﻳﻨﺎ ‪:‬‬
‫)‪OA + OB + OC = OM + ON + OP ⟺ (a − 0) + (b − 0) + (c − 0) = (z − 0) + (u − 0) + (v − 0‬‬
‫)‪⟺ a + b + c = z + u + v ⟺ (z − a) + (u − b) + (v − c‬‬

‫𝛼‬ ‫𝛽‬ ‫𝛾‬


‫= ‪ v − c‬ﻣﻨﻪ ‪:‬‬ ‫= ‪ z − a‬و )‪(a − c‬‬ ‫= ‪(b − a) ، u − b‬‬ ‫ﻟﻜﻦ ‪(c − b) :‬‬
‫𝛼‪𝛾+‬‬ ‫𝛽‪𝛼+‬‬ ‫𝛾‪𝛽+‬‬
‫𝛽‬ ‫𝛾‬ ‫𝛼‬
‫⟺ ‪OA + OB + OC = OM + ON + OP‬‬ ‫‪(b − a) +‬‬ ‫‪(c − b) +‬‬ ‫‪(a − c) = 0‬‬
‫𝛽‪𝛼+‬‬ ‫𝛾‪𝛽+‬‬ ‫𝛼‪𝛾+‬‬
‫𝛽‬ ‫𝛾‬ ‫𝛼‬
‫⟺‬ ‫‪AB +‬‬ ‫‪BC +‬‬ ‫⃗‪CA = 0‬‬
‫𝛽‪𝛼+‬‬ ‫𝛾‪𝛽+‬‬ ‫𝛼‪𝛾+‬‬
‫𝛽‬ ‫𝛾‬ ‫𝛼‬
‫⟺‬ ‫‪AB +‬‬ ‫‪AC − AB −‬‬ ‫⃗‪AC = 0‬‬
‫𝛽‪𝛼+‬‬ ‫𝛾‪𝛽+‬‬ ‫𝛼‪𝛾+‬‬
‫𝛽‬ ‫𝛾‬ ‫𝛾‬ ‫𝛼‬
‫⟺‬ ‫‪−‬‬ ‫‪AB +‬‬ ‫‪−‬‬ ‫⃗‪AC = 0‬‬
‫𝛾‪𝛼+𝛽 𝛽+‬‬ ‫𝛼‪𝛽+𝛾 𝛾+‬‬
‫⎧‬ ‫𝛽‬ ‫𝛾‬ ‫⎧‬ ‫𝛽‬ ‫𝛾‬
‫⎪‬
‫⎪‬ ‫‪−‬‬ ‫‪=0‬‬ ‫⎪‬
‫⎪‬ ‫=‬
‫‪ņőƱŀ Ţ‬‬
‫𝛾‪𝛼+𝛽 𝛽+‬‬ ‫𝛾‪𝛼+𝛽 𝛽+‬‬
‫⟺‬ ‫⟺‬
‫⎨‬
‫⎪‬ ‫𝛾‬ ‫𝛼‬ ‫⎨‬
‫⎪‬ ‫𝛾‬ ‫𝛼‬
‫⎪‬ ‫‪−‬‬ ‫‪=0‬‬ ‫⎪‬ ‫=‬
‫𝛼‪⎩ 𝛽+𝛾 𝛾+‬‬ ‫𝛼‪⎩ 𝛽+𝛾 𝛾+‬‬
‫⎧‬ ‫𝛾‬
‫𝛾𝛼 = 𝛽‬
‫⎪‬
‫=𝛼 ⎪‬
‫𝛽‬ ‫𝛾 =𝛼 ⎪‬
‫⎧‬
‫⟺‬ ‫⟺‬ ‫⟺‬ ‫𝛽‬
‫𝛽𝛼 = 𝛾‬ ‫⎨‬
‫⎪‬ ‫𝛾‬ ‫⎨‬
‫⎪‬
‫= 𝛽⎪‬ ‫𝛾‬ ‫𝛾 = 𝛽⎩‬
‫⎩‬ ‫𝛽‬
‫𝛾 = 𝛼⎪‬
‫⎧‬
‫⟺‬ ‫𝛾=𝛽=𝛼 ⟺ 𝛽‬
‫⎨‬
‫‪ƕ‬‬

‫⎪‬
‫𝛾=𝛽⎩‬
‫‪VII‬‬

‫⟺‬ ‫اﳌﺜﻠﺚ ‪ ABC‬ﻣﺘﻘﺎﻳﺲ اﻷﺿﻼع‬


‫■‬
‫‪ũŏ‬‬

‫‪ ABC‬ﻣﺜﻠﺚ‪ .‬ﻧﻀﻊ ‪ b = CA ، a = BC‬و ‪ . c = AB‬ﻟﻴﻜﻦ ‪ I‬ﻣﺮﻛﺰ اﻟﺪاﺋﺮة اﳌﺮﺳﻮﻣﺔ داﺧﻞ ﻫﺬا اﳌﺜﻠﺚ‬
‫!‬ ‫‪.‬‬
‫𝟐𝟏𝟐‬ ‫‪.‬‬

‫)ﻧﻘﻄﺔ ﺗﻘﺎﻃﻊ ﻣﻨﺼﻔﺎت زواﻳﺎه اﻟﺪاﺧﻠﻴﺔ(‪ .‬أﺛﺒﺖ ّ‬


‫أن اﻟﻨﻘﻄﺔ ‪ I‬ﻫﻲ ﻣﺮﻛﺰ اﳌﺴﺎﻓﺎت اﳌﺘﻨﺎﺳﺒﺔ ﻟﻠﺠﻤﻠﺔ })‪. {A(a), B(b), C(c‬‬
‫أﻧﻈﺮ اﻟﺘﻤﺮﻳﻦ ‪ 211‬ﺻﻔﺤﺔ ‪301‬‬ ‫‬
‫𝟮𝟬𝟯‬

‫‪http ://tinyurl.com/Malki1718‬‬ ‫‪0‬‬


‫‪ .VII‬ﲤﺎرﻳﻦ ﻟﻠﺘﻌﻤﻖ‬
‫‪.‬‬
‫‪A‬‬

‫‪Ŕž Ŧ Ľ‬‬
‫‪œǃ‬‬
‫‪I‬‬

‫‪C‬‬ ‫‪I1‬‬ ‫‪B‬‬

‫‪Ŀ‬‬
‫‪A1‬‬

‫ﺷﻜﻞ ‪10.VII‬‬

‫ﺍﳊﻞّ‪ .‬ﻟﻴﻜﻦ ‪ Δ‬ﻣﻨﺼﻒ اﻟﺰاوﻳﺔ ‪ BAC‬و ‪ I‬ﻧﻘﻄﺔ ﺗﻘﺎﻃﻊ ‪ Δ‬و )‪ . (BC‬ﻣﻦ اﻟﻮاﺿﺢ ّ‬
‫أن اﳌﺴﺘﻘﻴﻤﲔ ‪ Δ‬و )‪ (AC‬ﻏﲑ‬
‫ﻣﺘﻮازﻳﲔ )ﻷﳖﲈ ﻳﺘﻘﺎﻃﻌﺎن ﰲ اﻟﻨﻘﻄﺔ ‪. (A‬‬
‫اﳌﺴﺘﻘﻴﻢ اﳌﻮازي ﻟـِ )‪ (AC‬و اﳌﺎر ﺑﺎﻟﻨﻘﻄﺔ ‪ B‬ﻳﻘﻄﻊ ‪ Δ‬ﰲ اﻟﻨﻘﻄﺔ ‪. A‬‬
‫ﻓﺈن ‪ CAA = A AB‬ﻣﻨﻪ ‪ AA B = A AB‬و‬ ‫اﻟﺰاوﻳﺘﺎن ‪ CAA‬و ‪ AA B‬ﻣﺘﺒﺎدﻟﺘﺎن داﺧﻠﻴ ًﺎ إذن ﻣﺘﻘﺎﻳﺴﺘﺎن و ﺑﺎﻟﺘﺎﱄ ّ‬
‫أن اﳌﺜﻠﺚ ‪ ABA‬ﻣﺘﺴﺎوس اﻟﺴﺎﻗﲔ ﻋﻨﺪ اﻟﺮأس ‪ . B‬ﺣﺴﺐ ﻧﻈﺮﻳﺔ ﻃﺎﻟﻴﺲ )‪ّ (T‬‬
‫ﻓﺈن ‪:‬‬ ‫ﻫﺬا ﻳﻌﻨﻲ ّ‬
‫‪I B A B AB‬‬ ‫‪c‬‬
‫=‬ ‫=‬ ‫=‬
‫‪I C‬‬ ‫‪AC‬‬ ‫‪AC b‬‬
‫أن ‪ I‬ﻫﻲ ﻣﺮﻛﺰ اﳌﺴﺎﻓﺎت اﳌﺘﻨﺎﺳﺒﺔ‬ ‫أن ‪ bI B + cI C = 0⃗ :‬أي ّ‬ ‫أن اﻟﻨﻘﻄﺔ ‪ I‬ﺗﻘﻊ ﺑﲔ اﻟﻨﻘﻄﺘﲔ ‪ B‬و ‪ C‬ﻓﻬﺬا ﻳﻌﻨﻲ ّ‬ ‫و ﺑﲈ ّ‬
‫ﻟﻠﺠﻤﻠﺔ })‪. {B (b) , C (c‬‬
‫ﻟﺘﻜﻦ ‪ I‬ﻧﻘﻄﺔ ﺗﻘﺎﻃﻊ ﻣﻨﺼﻒ اﻟﺰاوﻳﺔ ‪ ABC‬ﻣﻊ )‪ (AC‬و ‪ I‬ﻧﻘﻄﺔ ﺗﻘﺎﻃﻊ ﻣﻨﺼﻒ اﻟﺰاوﻳﺔ ‪ ACB‬ﻣﻊ )‪ . (AB‬ﺑﻨﻔﺲ اﻟﻄﺮﻳﻘﺔ‬
‫أن ‪ aI A + cI C = 0⃗ :‬أي ّ‬
‫أن اﻟﻨﻘﻄﺔ ‪ I‬ﻫﻲ ﻣﺮﻛﺰ اﳌﺴﺎﻓﺎت اﳌﺘﻨﺎﺳﺒﺔ ﻟﻠﺠﻤﻠﺔ })‪ {A (a) , C (c‬؛‬ ‫ﻧﱪﻫﻦ ّ‬
‫‪ņőƱŀ Ţ‬‬
‫أن ⃗‪ aI A + bI B = 0‬أي ّ‬
‫أن اﻟﻨﻘﻄﺔ ‪ I‬ﻫﻲ ﻣﺮﻛﺰ اﳌﺴﺎﻓﺎت اﳌﺘﻨﺎﺳﺒﺔ ﻟﻠﺠﻤﻠﺔ })‪. {A (a) , B (b‬‬ ‫و ّ‬
‫ﻟﺘﻜﻦ ‪ I‬ﻣﺮﻛﺰ اﳌﺴﺎﻓﺎت اﳌﺘﻨﺎﺳﺒﺔ ﻟﻠﺠﻤﻠﺔ })‪ . {A(a), B(b), C(c‬ﻧﻌﻠﻢ أﻧﻪ ﺑﺈﻣﻜﺎﻧﻨﺎ اﻟﺘﻌﻮﻳﺾ ﻋﻦ اﻟﻨﻘﻄﺘﲔ )‪ B (b‬و )‪C (c‬‬
‫ﺑﻤﺮﻛﺰ ﻣﺴﺎﻓﺘﻴﻬﲈ اﳌﺘﻨﺎﺳﺒﺔ )‪ I (b + c‬و ﺑﺎﻟﺘﺎﱄ ‪ I‬ﻫﻲ ﻣﺮﻛﺰ اﳌﺴﺎﻓﺎت اﳌﺘﻨﺎﺳﺒﺔ ﻟﻠﺠﻤﻠﺔ })‪. {A (a) , I (b + c‬‬
‫ﺑﺎﳌﺜﻞ‪ I ،‬ﻫﻲ ﻣﺮﻛﺰ اﳌﺴﺎﻓﺎت اﳌﺘﻨﺎﺳﺒﺔ ﻟﻠﺠﻤﻠﺔ })‪ {B (b) , I (a + c‬و ﻟﻠﺠﻤﻠﺔ )‪. C (c) , I (a + b‬‬
‫أن ‪ I‬ﺗﻨﺘﻤﻲ إﱃ اﳌﺴﺘﻘﻴﲈت ) ‪ (BI ) ، (AI‬و ) ‪ . (CI‬ﻟﻜﻦ ﻫﺬه اﳌﺴﺘﻘﻴﲈت ﻫﻲ ﻣﻨﺼﻔﺎت اﻟﺰواﻳﺎ اﻟﺪاﺧﻠﻴﺔ ﻟﻠﻤﺜﻠﺚ‬ ‫ﻫﺬا ﻳﻌﻨﻲ ّ‬
‫‪ ABC‬و ﻧﻌﻠﻢ أﳖﺎ ﺗﺘﻘﺎﻃﻊ ﰲ ﻧﻘﻄﺔ واﺣﺪة ﻫﻲ ‪ I‬إذن ‪. I = I‬‬
‫■‬ ‫ﻫﻜﺬا ﻧﻜﻮن ﻗﺪ أﺛﺒﺘﻨﺎ ّ‬
‫أن اﻟﻨﻘﻄﺔ ‪ I‬ﻫﻲ ﻣﺮﻛﺰ اﳌﺴﺎﻓﺎت اﳌﺘﻨﺎﺳﺒﺔ ﻟﻠﺠﻤﻠﺔ })‪. {A(a), B(b), C(c‬‬
‫‪ƕ‬‬
‫‪VII‬‬

‫‪.‬‬
‫𝟑𝟏𝟐‬ ‫‪.‬‬

‫⎧‬ ‫‪1‬‬
‫‪⎪u + v = −‬‬
‫‪ũŏ‬‬

‫‪2‬‬ ‫‪ •1‬ﺣﻞ ﰲ ‪ ℂ × ℂ‬اﳉﻤﻠﺔ ‪:‬‬


‫‪⎪ uv = − 1‬‬
‫⎨‬
‫⎩‬ ‫‪4‬‬
‫‪ .𝜔 = e‬أﺛﺒﺖ ّ‬
‫أن ‪.𝜔 + 𝜔 + 𝜔 + 𝜔 + 𝜔 = 0 :‬‬ ‫‪/‬‬ ‫‪ •2‬ﻧﻀﻊ‬
‫𝜋‪2‬‬ ‫𝜋‪4‬‬ ‫‪1‬‬ ‫إﺳﺘﻨﺘﺞ )ﺑﺎﻹﺳﺘﻌﺎﻧﺔ ِ‬
‫‪cos‬‬ ‫‪+ cos‬‬ ‫‪=−‬‬ ‫ﺑﺼﻴﻎ ﺃﻭﻟﺮ( ّ‬
‫أن ‪:‬‬
‫‪5‬‬ ‫‪5‬‬ ‫‪2‬‬

‫𝟯𝟬𝟯‬

‫‪http ://tinyurl.com/Malki1718‬‬ ‫‪0‬‬


‫‪.‬‬

‫‪ •3‬أﺛﺒﺖ ّ‬
‫أن ‪:‬‬

‫‪Ŕž Ŧ Ľ‬‬
‫𝜋‪2‬‬ ‫𝜋‪4‬‬ ‫𝜋‪2‬‬ ‫𝜋‪4‬‬ ‫𝜋‪2‬‬
‫‪cos‬‬ ‫‪cos‬‬ ‫‪+ sin‬‬ ‫‪sin‬‬ ‫‪= cos‬‬
‫‪5‬‬ ‫‪5‬‬ ‫‪5‬‬ ‫‪5‬‬ ‫‪5‬‬

‫‪œǃ‬‬
‫𝜋‪2‬‬ ‫𝜋‪4‬‬ ‫𝜋‪2‬‬ ‫𝜋‪4‬‬ ‫𝜋‪4‬‬
‫‪cos‬‬ ‫‪cos‬‬ ‫‪− sin‬‬ ‫‪sin‬‬ ‫‪= cos‬‬ ‫و‬
‫‪5‬‬ ‫‪5‬‬ ‫‪5‬‬ ‫‪5‬‬ ‫‪5‬‬

‫𝜋‪2‬‬ ‫𝜋‪4‬‬ ‫‪1‬‬

‫‪Ŀ‬‬
‫‪.cos‬‬ ‫‪cos‬‬ ‫‪=−‬‬ ‫‪ •4‬إﺳﺘﻨﺘﺞ ّ‬
‫أن ‪:‬‬
‫‪5‬‬ ‫‪5‬‬ ‫‪4‬‬

‫𝜋‪4‬‬ ‫‪−1 − √5‬‬ ‫𝜋‪2‬‬ ‫‪−1 + √5‬‬


‫‪. cos‬‬ ‫=‬ ‫و‬ ‫‪cos‬‬ ‫=‬ ‫‪ •5‬أﺛﺒﺖ ّ‬
‫أن ‪:‬‬
‫‪5‬‬ ‫‪4‬‬ ‫‪5‬‬ ‫‪4‬‬

‫ﺍﳊﻞّ‪.‬‬
‫‪1‬‬
‫‪ v = −u −‬؛ ﺑﺎﻟﺘﻌﻮﻳﺾ ﰲ اﳌﻌﺎدﻟﺔ اﻟﺜﺎﻧﻴﺔ ﻧﺠﺪ ‪:‬‬ ‫‪ •1‬ﻧﺘّﺒﻊ ﻃﺮﻳﻘﺔ اﻟﺘﻌﻮﻳﺾ ‪ :‬ﻣﻦ اﳌﻌﺎدﻟﺔ اﻷوﱃ ﻧﺠﺪ ‪:‬‬
‫‪2‬‬
‫‪−1 − √5‬‬ ‫‪1‬‬ ‫‪1‬‬
‫= ‪u‬‬ ‫‪ u −u −‬أي ‪ 4u + 2u − 1 = 0‬و ﻫﻲ ﻣﻌﺎدﻟﺔ ﻣﻦ اﻟﺪرﺟﺔ اﻟﺜﺎﻧﻴﺔ ﺣﻠﻮﳍﺎ ‪:‬‬
‫‪= −‬‬
‫‪4‬‬ ‫‪2‬‬ ‫‪4‬‬
‫‪1 −1 − √5‬‬ ‫‪1 −1 + √5‬‬ ‫‪−1 + √5‬‬
‫= ‪. v = −u −‬‬ ‫= ‪ v = −u −‬و‬ ‫= ‪ ، u‬ﻣﻨﻪ‬ ‫و‬
‫‪2‬‬ ‫‪4‬‬ ‫‪2‬‬ ‫‪4‬‬ ‫‪4‬‬
‫‪−1 + √5 −1 − √5‬‬ ‫‪−1 − √5 −1 + √5‬‬
‫= )‪. (u, v‬‬ ‫‪,‬‬ ‫= )‪ (u, v‬و‬ ‫‪,‬‬ ‫أن اﳉﻤﻠﺔ ﺗﻘﺒﻞ ﺣ ّﻠﲔ ‪:‬‬
‫ﻧﺴﺘﻨﺘﺞ ّ‬
‫‪4‬‬ ‫‪4‬‬ ‫‪4‬‬ ‫‪4‬‬

‫‪ •2‬ﳎﻤﻮع ‪ 5‬ﺣﺪود اﻷوﱃ ﳌﺘﺘﺎﻟﻴﺔ ﻫﻨﺪﺳﻴﺔ أﺳﺎﺳﻬﺎ ‪ r = 𝜔 ≠ 1‬إذن ‪:‬‬


‫𝜔‪1−‬‬
‫= 𝜔‪𝜔 +𝜔 +𝜔 +𝜔 +‬‬ ‫‪=0‬‬ ‫) ّ‬
‫ﻷن ‪(𝜔 = 1‬‬
‫𝜔‪1−‬‬
‫‪1+e‬‬ ‫‪/‬‬ ‫‪+e‬‬ ‫‪/‬‬ ‫‪+e‬‬ ‫‪/‬‬ ‫‪+e‬‬ ‫‪/‬‬ ‫‪=0‬‬ ‫أي‬
‫𝜋‪8‬‬ ‫𝜋‪2‬‬ ‫𝜋‪6‬‬ ‫𝜋‪4‬‬
‫ﻟﻜﻦ )𝜋‪ = − (mod 2‬و )𝜋‪ = − (mod 2‬ﻣﻨﻪ‬
‫‪ņőƱŀ Ţ‬‬
‫‪5‬‬ ‫‪5‬‬ ‫‪5‬‬ ‫‪5‬‬
‫ِ‬
‫‪ 1 + e‬و ﺑﺘﻄﺒﻴﻖ ﺻﻴﻎ ﺃﻭﻟﺮ ﻳﻨﺘﺞ ‪:‬‬ ‫‪/‬‬ ‫‪+e‬‬ ‫‪/‬‬ ‫‪+e‬‬ ‫‪−‬‬ ‫‪/ + e−‬‬ ‫‪/ =0‬‬

‫𝜋‪2‬‬ ‫𝜋‪4‬‬ ‫‪1‬‬ ‫𝜋‪2‬‬ ‫𝜋‪4‬‬


‫‪. cos‬‬ ‫‪+ cos‬‬ ‫‪=−‬‬ ‫‪ 1 + 2 cos‬أي‬ ‫‪+ 2 cos‬‬ ‫‪=0‬‬
‫‪5‬‬ ‫‪5‬‬ ‫‪2‬‬ ‫‪5‬‬ ‫‪5‬‬

‫‪ •3‬ﻟﺪﻳﻨﺎ ‪:‬‬
‫𝜋‪2‬‬ ‫𝜋‪4𝜋 2‬‬ ‫𝜋‪2‬‬ ‫𝜋‪4‬‬ ‫𝜋‪2‬‬ ‫𝜋‪4‬‬
‫‪cos‬‬ ‫‪= cos‬‬ ‫‪−‬‬ ‫‪= cos‬‬ ‫‪cos‬‬ ‫‪+ sin‬‬ ‫‪sin‬‬
‫‪5‬‬ ‫‪5‬‬ ‫‪5‬‬ ‫‪5‬‬ ‫‪5‬‬ ‫‪5‬‬ ‫‪5‬‬
‫‪ƕ‬‬

‫𝜋‪4‬‬ ‫𝜋‪4‬‬ ‫𝜋‪6‬‬ ‫𝜋‪4𝜋 2‬‬ ‫𝜋‪2‬‬ ‫𝜋‪4‬‬ ‫𝜋‪2‬‬ ‫𝜋‪4‬‬


‫‪cos‬‬ ‫‪= cos −‬‬ ‫‪= cos‬‬ ‫‪= cos‬‬ ‫‪+‬‬ ‫‪= cos‬‬ ‫‪cos‬‬ ‫‪− sin‬‬ ‫‪sin‬‬
‫‪VII‬‬

‫‪5‬‬ ‫‪5‬‬ ‫‪5‬‬ ‫‪5‬‬ ‫‪5‬‬ ‫‪5‬‬ ‫‪5‬‬ ‫‪5‬‬ ‫‪5‬‬

‫𝜋‪2‬‬ ‫𝜋‪4‬‬ ‫𝜋‪2‬‬ ‫𝜋‪4‬‬


‫‪cos‬‬ ‫‪+ cos‬‬ ‫‪= 2 cos‬‬ ‫‪cos‬‬ ‫‪ •4‬ﻧﺠﻤﻊ اﳌﺴﺎوﺗﲔ اﻟﺴﺎﺑﻘﺘﲔ ﻃﺮﻓ ًﺎإﱃ ﻃﺮف ﻟﻨﺠﺪ ‪:‬‬
‫‪ũŏ‬‬

‫‪5‬‬ ‫‪5‬‬ ‫‪5‬‬ ‫‪5‬‬


‫𝜋‪2‬‬ ‫𝜋‪4‬‬ ‫‪1‬‬ ‫𝜋‪2‬‬ ‫𝜋‪4‬‬ ‫‪1‬‬ ‫‪1‬‬ ‫‪1‬‬
‫‪cos‬‬ ‫‪cos‬‬ ‫=‬ ‫‪cos‬‬ ‫‪+ cos‬‬ ‫=‬ ‫‪× −‬‬ ‫‪=−‬‬ ‫أي‬
‫‪5‬‬ ‫‪5‬‬ ‫‪2‬‬ ‫‪5‬‬ ‫‪5‬‬ ‫‪2‬‬ ‫‪2‬‬ ‫‪4‬‬

‫⎧‬ ‫‪1‬‬
‫‪⎪u + v = −‬‬ ‫𝜋‪4‬‬ ‫𝜋‪2‬‬
‫‪.‬‬ ‫‪2‬‬ ‫‪ . v = cos‬ﺣﺴﺐ ﻣﺎ ﺳﺒﻖ ﻟﺪﻳﻨﺎ ‪:‬‬ ‫‪ u = cos‬و‬ ‫‪ •5‬ﻧﻀﻊ‬
‫‪⎪ uv = − 1‬‬
‫⎨‬ ‫‪5‬‬ ‫‪5‬‬
‫⎩‬ ‫‪4‬‬

‫𝟰𝟬𝟯‬

‫‪http ://tinyurl.com/Malki1718‬‬ ‫‪0‬‬


‫‪ .VII‬ﲤﺎرﻳﻦ ﻟﻠﺘﻌﻤﻖ‬
‫‪.‬‬
‫𝜋‪4‬‬ ‫𝜋‬
‫(‬ ‫∈‬ ‫𝜋‪,‬‬ ‫)ﻷن 𝜋 ‪ ( 2𝜋 ∈ 0,‬و ‪ّ cos 4𝜋 < 0‬‬
‫)ﻷن‬ ‫ﻟﻜﻦ ‪ّ cos 2𝜋 > 0‬‬
‫‪5‬‬ ‫‪2‬‬ ‫‪5‬‬ ‫‪5‬‬ ‫‪2‬‬ ‫‪5‬‬

‫‪Ŕž Ŧ Ľ‬‬
‫𝜋‪4‬‬ ‫‪−1 − √5‬‬ ‫𝜋‪2‬‬ ‫‪−1 + √5‬‬
‫‪. cos‬‬ ‫=‬ ‫و‬ ‫‪cos‬‬ ‫=‬ ‫اﻷول( ‪:‬‬
‫ﻣﻨﻪ )اﻟﺴﺆال ّ‬
‫‪5‬‬ ‫‪4‬‬ ‫‪5‬‬ ‫‪4‬‬
‫■‬

‫‪œǃ‬‬
‫‪.‬‬
‫𝟒𝟏𝟐‬ ‫‪.‬‬
‫إﻧﺸﺎء اﳋﲈﳼ اﳌﻨﺘﻈﻢ ﺑﺎﳌﺴﻄﺮة و اﳌﺪور‬

‫‪Ŀ‬‬
‫‪ a = z + z ، z = e‬و ‪ . b = z + z‬أوﺟﺪ اﳌﻌﺎدﻟﺔ ﻣﻦ اﻟﺪرﺟﺔ اﻟﺜﺎﻧﻴﺔ اﻟﺘﻲ ﺣﻠﻮﳍﺎ ﻫﻲ ‪ a‬و ‪ b‬ﺛﻢ‬ ‫‪/‬‬ ‫‪ •1‬ﻧﻀﻊ‬
‫𝜋‬ ‫𝜋‬ ‫𝜋‪4‬‬ ‫𝜋‪4‬‬ ‫𝜋‪2‬‬ ‫𝜋‪2‬‬
‫‪. sin‬‬ ‫‪ cos‬و‬ ‫‪، sin‬‬ ‫‪، cos‬‬ ‫‪، sin‬‬ ‫‪، cos‬‬ ‫اﺳﺘﻨﺘﺞ ِﻗ َﻴﻢ ّ‬
‫ﻛﻞ ﻣﻦ‬
‫‪5‬‬ ‫‪5‬‬ ‫‪5‬‬ ‫‪5‬‬ ‫‪5‬‬ ‫‪5‬‬
‫‪1‬‬
‫‪ •2‬اﻟﺪاﺋﺮة اﻟﺘﻲ ﻣﺮﻛﺰﻫﺎ اﻟﻨﻘﻄﺔ ‪ Ω‬ذات اﻟﻼﺣﻘﺔ ‪ −‬و اﻟﺘﻲ ﲤﺮ ﺑﺎﻟﻨﻘﻄﺔ ‪ M‬ذات اﻟﻼﺣﻘﺔ ‪ ı‬ﺗﻘﻄﻊ اﳌﺤﻮر )‪ (Ox‬ﰲ اﻟﻨﻘﻄﺘﲔ‬
‫‪2‬‬
‫اﳌﺪرﺟﺔ(‪ ،‬ﻟﻠﺨﲈﳼ‬
‫ّ‬ ‫)ﻏﲑ‬ ‫اﳌﺴﻄﺮة‬ ‫و‬ ‫ﺑﺎﳌﺪور‬ ‫ًا‪،‬‬ ‫ء‬ ‫إﻧﺸﺎ‬ ‫اﺳﺘﻨﺘﺞ‬ ‫ﺛﻢ‬
‫ّ‬ ‫‪OI‬‬ ‫‪+‬‬ ‫أن ‪OJ = OI ⋅ OJ = −1‬‬‫‪ I‬و ‪ . J‬أﺛﺒﺖ ّ‬
‫اﳌﻨﺘﻈﻢ اﻟﺬي ﺗﻘﻊ رؤوﺳﻪ ﻋﲆ اﻟﺪاﺋﺮة ذات اﳌﺮﻛﺰ ‪ O‬و ﻧﺼﻒ اﻟﻘﻄﺮ ‪ ، 1‬و أﺣﺪ رؤوﺳﻪ اﻟﻨﻘﻄﺔ ذات اﻟﻼﺣﻘﺔ ‪. 1‬‬

‫ﺍﳊﻞّ‪.‬‬
‫𝜋‪2‬‬ ‫‪1 z‬‬
‫‪a = z + z = 2 Re (z) = 2 cos‬‬ ‫‪ •1‬ﻟﺪﻳﻨﺎ ‪ z = 1‬و ‪ ||1|| = 1‬ﻣﻨﻪ ‪ z = = = z‬ﺑﺎﳌﺜﻞ ‪ z = z‬إذن‬
‫‪5‬‬ ‫‪z‬‬ ‫‪z‬‬
‫𝜋‪4‬‬
‫و ‪. b = z + z = 2 Re z = 2 Re e / = 2 cos‬‬
‫‪5‬‬
‫أن ‪ z ، z ، z ، 1‬و ‪ z‬ﻫﻲ اﳉﺬور اﳋﺎﻣﺴﺔ ﻟﻠﻮﺣﺪة ﰲ ‪ّ ℂ‬‬
‫ﻓﺈن ‪ 1 + z + z + z + z = 0‬ﻣﻨﻪ‬ ‫ﺑﲈ ّ‬
‫‪ . a + b = z + z + z + z = −1‬ﻣﻦ ﺟﻬﺔ أﺧﺮى ‪:‬‬
‫‪ab = z + z‬‬ ‫‪z +z‬‬ ‫‪=z +z +z +z =z +z +z⋅z +z ⋅z‬‬
‫‪= z + z + z + z = a + b = −1‬‬
‫‪ņőƱŀ Ţ‬‬
‫‪−1 ± √5‬‬
‫‪ .‬ﻟﻜﻦ‬ ‫أي ‪ a + b = −1‬و ‪ ab = −1‬إذن ‪ a‬و ‪ b‬ﳘﺎ ﺣ ﱠ‬
‫ﻼ اﳌﻌﺎدﻟﺔ ‪ X + X − 1 = 0‬أي ﻳﺴﺎوﻳﺎن‬
‫‪2‬‬
‫𝜋‪2‬‬ ‫‪−1 + √5‬‬ ‫‪−1 − √5‬‬ ‫‪−1 + √5‬‬ ‫𝜋‪2‬‬ ‫𝜋‬
‫= ‪ cos‬و‬ ‫= ‪ b‬ﻣﻨﻪ‬ ‫= ‪a‬و‬ ‫إذن‬ ‫‪∈ 0,‬‬ ‫‪ّ a = 2 cos 2𝜋 > 0‬‬
‫ﻷن‬
‫‪5‬‬ ‫‪4‬‬ ‫‪2‬‬ ‫‪2‬‬ ‫‪5‬‬ ‫‪2‬‬ ‫‪5‬‬
‫‪4𝜋 −1 − √5‬‬
‫= ‪. cos‬‬
‫‪5‬‬ ‫‪4‬‬
‫أن ‪ّ sin 2𝜋 > 0‬‬
‫ﻓﺈن ‪:‬‬ ‫ﺑﲈ ّ‬
‫‪5‬‬
‫𝜋‪2‬‬ ‫𝜋‪2‬‬ ‫‪−1 + √5‬‬ ‫‪10 + 2√5‬‬
‫‪ƕ‬‬

‫‪sin‬‬ ‫‪=+‬‬ ‫‪1 − cos‬‬ ‫=‬ ‫‪1−‬‬ ‫=‬


‫‪5‬‬ ‫‪5‬‬ ‫‪4‬‬ ‫‪4‬‬
‫‪VII‬‬

‫𝜋‪4‬‬
‫‪ّ sin‬‬
‫ﻓﺈن ‪:‬‬ ‫و ﺑﲈ ّ‬
‫أن ‪> 0‬‬
‫‪5‬‬
‫‪ũŏ‬‬

‫𝜋‪4‬‬ ‫𝜋‪4‬‬ ‫‪−1 − √5‬‬ ‫‪10 − 2√5‬‬


‫‪sin‬‬ ‫‪=+‬‬ ‫‪1 − cos‬‬ ‫=‬ ‫‪1−‬‬ ‫=‬
‫‪5‬‬ ‫‪5‬‬ ‫‪4‬‬ ‫‪4‬‬

‫𝜋‬ ‫𝜋‬ ‫‪4𝜋 1 + √5‬‬


‫‪cos‬‬ ‫‪= − cos 𝜋 −‬‬ ‫‪= − cos‬‬ ‫=‬ ‫ﻣﻦ ﺟﻬﺔ أﺧﺮى ‪:‬‬
‫‪5‬‬ ‫‪5‬‬ ‫‪5‬‬ ‫‪4‬‬
‫𝜋‬ ‫𝜋‬ ‫𝜋‪4‬‬ ‫‪10 − 2√5‬‬
‫‪sin = sin 𝜋 −‬‬ ‫‪= sin‬‬ ‫=‬ ‫و‬
‫‪5‬‬ ‫‪5‬‬ ‫‪5‬‬ ‫‪4‬‬

‫𝟱𝟬𝟯‬

‫‪http ://tinyurl.com/Malki1718‬‬ ‫‪0‬‬


‫‪.‬‬
‫ﰲ اﻷﺧﲑ ‪:‬‬

‫‪Ŕž Ŧ Ľ‬‬
‫‪𝜋 1 + √5‬‬ ‫‪2𝜋 −1 + √5‬‬ ‫‪4𝜋 −1 − √5‬‬
‫‪cos‬‬ ‫=‬ ‫;‬ ‫‪cos‬‬ ‫=‬ ‫;‬ ‫‪cos‬‬ ‫=‬
‫‪5‬‬ ‫‪4‬‬ ‫‪5‬‬ ‫‪4‬‬ ‫‪5‬‬ ‫‪4‬‬

‫‪œǃ‬‬
‫𝜋‬ ‫‪10 − 2√5‬‬ ‫𝜋‪2‬‬ ‫‪10 + 2√5‬‬ ‫𝜋‪4‬‬ ‫‪10 − 2√5‬‬
‫= ‪sin‬‬ ‫;‬ ‫‪sin‬‬ ‫=‬ ‫;‬ ‫‪sin‬‬ ‫=‬
‫‪5‬‬ ‫‪4‬‬ ‫‪5‬‬ ‫‪4‬‬ ‫‪5‬‬ ‫‪4‬‬

‫‪Ŀ‬‬
‫‪ •2‬ﻧﺼﻒ ﻗﻄﺮ اﻟﺪاﺋﺮة اﻟﺘﻲ ﻣﺮﻛﺰﻫﺎ ‪ Ω‬و ﲤﺮ ﺑﺎﻟﻨﻘﻄﺔ ‪ M‬ﻫﻮ ‪:‬‬

‫‪1‬‬ ‫‪√5‬‬
‫= ‪ΩM‬‬ ‫= ‪ΩO + OM‬‬ ‫=‪+1‬‬
‫‪4‬‬ ‫‪2‬‬
‫‪1 √5 −1 + √5‬‬
‫‪OI = OΩ + ΩI = − +‬‬ ‫=‬ ‫إذن ‪:‬‬
‫‪2‬‬ ‫‪2‬‬ ‫‪2‬‬
‫‪1 √5 −1 − √5‬‬
‫‪OJ = OΩ + ΩJ = − −‬‬ ‫=‬ ‫و‬
‫‪2‬‬ ‫‪2‬‬ ‫‪2‬‬
‫𝜋‪2‬‬
‫‪OI = 2 cos‬‬ ‫‪=a‬‬ ‫أي ‪:‬‬
‫‪5‬‬
‫𝜋‪4‬‬
‫‪OJ = 2 cos‬‬ ‫‪=b‬‬ ‫و‬
‫‪5‬‬
‫‪OI + OJ = a + b = −1‬‬ ‫ﻣﻨﻪ ‪:‬‬
‫‪OI ⋅ OJ = ab = −1‬‬ ‫و‬
‫‪1‬‬ ‫𝜋‪2‬‬
‫‪ x = OI = cos‬و ﺑﺎﻟﺘﺎﱄ ﻓﻬﺬا اﳌﺤﻮر ﻳﻘﻄﻊ اﻟﺪاﺋﺮة اﻟﺘﻲ ﻣﺮﻛﺰﻫﺎ‬ ‫ﻣﻌﺎدﻟﺔ ﳏﻮر اﻟﻘﻄﻌﺔ اﳌﺴﺘﻘﻴﻤﺔ ]‪ [OI‬ﻫﻲ‬
‫‪2‬‬ ‫‪5‬‬
‫𝜋‪2‬‬ ‫𝜋‪2‬‬ ‫𝜋‪2‬‬ ‫𝜋‪2‬‬
‫‪ cos , − sin‬و‬ ‫‪ O‬و ﻧﺼﻒ ﻗﻄﺮﻫﺎ ‪1‬ﰲ ﻧﻘﻄﺘﲔ ﻓﺎﺻﻠﺘﺎﳘﺎ ‪ cos‬و ﺗﺮﺗﻴﺒﺘﺎﳘﺎ ‪ ± sin‬أي ﰲ اﻟﻨﻘﻄﺘﲔ‬
‫‪5‬‬ ‫‪5‬‬ ‫‪5‬‬ ‫‪5‬‬
‫𝜋‪2‬‬ ‫𝜋‪2‬‬
‫‪ cos , sin‬اﻟﻠﺘﲔ ﳘﺎ رأﺳﺎن ﻟﻠﺨﲈﳼ اﳌﻨﺘﻈﻢ‪.‬‬
‫‪5‬‬ ‫‪5‬‬
‫‪ņőƱŀ Ţ‬‬
‫‪1‬‬ ‫𝜋‪4‬‬
‫‪ x = OJ = cos‬و ﺑﺎﻟﺘﺎﱄ ﻓﻬﺬا اﳌﺤﻮر ﻳﻘﻄﻊ اﻟﺪاﺋﺮة اﻟﺘﻲ ﻣﺮﻛﺰﻫﺎ ‪O‬‬ ‫ﺑﺎﳌﺜﻞ‪ ،‬ﻣﻌﺎدﻟﺔ ﳏﻮر اﻟﻘﻄﻌﺔ ‪ OJ‬ﻫﻲ‬
‫‪2‬‬ ‫‪5‬‬
‫𝜋‪4‬‬ ‫𝜋‪4‬‬ ‫𝜋‪4‬‬ ‫𝜋‪4‬‬
‫‪ cos , − sin‬و‬ ‫و ﻧﺼﻒ ﻗﻄﺮﻫﺎ ‪ 1‬ﰲ ﻧﻘﻄﺘﲔ ﻓﺎﺻﻠﺘﻬﲈ ‪ cos‬و ﺗﺮﺗﻴﺒﺘﺎﳘﺎ ‪ ± sin‬أي ﰲ اﻟﻨﻘﻄﺘﲔ‬
‫‪5‬‬ ‫‪5‬‬ ‫‪5‬‬ ‫‪5‬‬
‫𝜋‪4‬‬ ‫𝜋‪4‬‬
‫‪ cos , sin‬اﻟﻠﺘﲔ ﳘﺎ رأﺳﺎن ﻟﻠﺨﲈﳼ اﳌﻨﺘﻈﻢ‪.‬‬
‫‪5‬‬ ‫‪5‬‬
‫)ﺷﻜﻞ ‪.(11.VII‬‬ ‫ﻫﻜﺬا ﻧﻜﻮن ﻗﺪ ﲢﺼﻠﻨﺎ ﻋﲆ أرﺑﻌﺔ رؤوس ﻟﻠﺨﲈﳼ اﳌﻨﺘﻈﻢ‪ .‬اﻟﺮأس اﳋﺎﻣﺲ ﻫﻮ اﻟﻨﻘﻄﺔ )‪. (1, 0‬‬
‫■‬
‫‪ƕ‬‬

‫‪VII‬‬

‫‪.‬‬
‫𝟓𝟏𝟐‬ ‫‪.‬‬
‫ﺑﺎﺗﺒﺎع ﻧﻔﺲ ﻃﺮﻳﻘﺔ اﻟﺘﻤﺮﻳﻦ ‪ 203‬ﺻﻔﺤﺔ ‪ ، 292‬أﺣﺴﺐ اﳌﺠﺎﻣﻴﻊ اﻵﺗﻴﺔ ‪:‬‬
‫‪n‬‬ ‫‪n‬‬ ‫‪n‬‬
‫⎧‬
‫⎪‬ ‫=‪S‬‬ ‫‪+‬‬ ‫‪+‬‬ ‫⋯‪+‬‬
‫⎪‬
‫⎪‬ ‫‪0‬‬ ‫‪4‬‬ ‫‪8‬‬
‫‪ũŏ‬‬

‫⎪‬
‫⎪‬ ‫‪n‬‬ ‫‪n‬‬ ‫‪n‬‬
‫⎪‬
‫⎪‬
‫=‪⎪T‬‬ ‫‪+‬‬ ‫‪+‬‬ ‫⋯‪+‬‬
‫‪.‬‬ ‫‪1‬‬ ‫‪5‬‬ ‫‪9‬‬
‫⎨‬ ‫‪n‬‬ ‫‪n‬‬ ‫‪n‬‬
‫⎪‬
‫⎪‬ ‫=‪U‬‬ ‫‪+‬‬ ‫‪+‬‬ ‫⋯‪+‬‬
‫⎪‬
‫⎪‬ ‫‪2‬‬ ‫‪6‬‬ ‫‪10‬‬
‫⎪‬
‫⎪‬
‫‪⎪V = n +‬‬
‫⎪‬ ‫‪n‬‬
‫‪+‬‬
‫‪n‬‬
‫⋯‪+‬‬
‫⎩‬ ‫‪3‬‬ ‫‪7‬‬ ‫‪11‬‬

‫𝟲𝟬𝟯‬

‫‪http ://tinyurl.com/Malki1718‬‬ ‫‪0‬‬


‫ ﲤﺎرﻳﻦ ﻟﻠﺘﻌﻤﻖ‬.VII
.

Ŕž Ŧ Ľ
œǃ
J O I

Ŀ
Ω 1

11.VII ‫ﺷﻜﻞ‬

ّ ،‫( ﺑﺎﺳﺘﻌﲈل ﻣﻔﻜﻮك ﺛﻨﺎﺋﻲ اﳊﺪ‬1 + x) ‫ ﻧﻨﴩ‬.ّ‫ﺍﳊﻞ‬


: x = −ı ‫ و‬x = −1 ، x = ı ، x = 1 ‫ﺛﻢ ﻧُﻄ ّﺒﻖ اﻟﻨﺘﻴﺠﺔ ﻋﲆ‬
n n n n n n
(1 + 1) = + + + + + +⋯=S+T +U+V
0 1 2 3 4 5
n n n n n n
(1 + ı) = +ı +ı +ı +ı +ı +⋯
0 1 2 3 4 5
n n n n n n
= +ı − −ı + +ı + ⋯ = S + ıT − U − ıV
0 1 2 3 4 5
n n n n n n
(1 − 1) = + (−1) + (−1) + (−1) + (−1) + (−1) +⋯
0 1 2 3 4 5
ņőƱŀ Ţ
n n n n n n
= − + − + − +⋯=S−T +U−V
0 1 2 3 4 5
n n n n n n
(1 − ı) = + (−ı) + (−ı) + (−ı) + (−ı) + (−ı) +⋯
0 1 2 3 4 5
n n n n n n
= −ı − +ı + −ı + ⋯ = S − ıT − U + ıV
0 1 2 3 4 5

⎧ S+T +U+V =2 (1)



ƕ



⎪ S + ıT − U − ıV = (1 + ı)
VII

(2)
⎨ S−T +U−V =0
⎪ (3)



⎩ S − ıT − U + ıV = (1 − ı) (4)
ũŏ

: ‫ ﻣﻨﻪ‬4S = 2 + (1 + ı) + (1 − ı) ‫( ﻧﺠﺪ‬1) + (2) + (3) + (4) ‫ﺑﺠﻤﻊ‬

2 + (1 + ı) + (1 + ı) 2 + 2 Re (1 + ı)
S= =
4 4
2 + 2 Re √2 e /
2 + 2√2 cos n
= =
4 4

𝟯𝟬𝟳

http ://tinyurl.com/Malki1718 0
‫‪.‬‬
‫ﺑﺠﻤﻊ )‪ (1) − ı × (2) − (3) + ı × (4‬ﻧﺠﺪ ‪:‬‬

‫‪Ŕž Ŧ Ľ‬‬
‫)‪2 − ı (1 + ı) + ı (1 − ı‬‬ ‫)‪2 − 2 Re ı (1 + ı‬‬
‫=‪T‬‬ ‫=‬
‫‪4‬‬ ‫‪4‬‬

‫‪œǃ‬‬
‫‪2 − 2√2 cos‬‬ ‫‪+n‬‬ ‫‪2 + 2√2 sin n‬‬
‫=‬ ‫=‬
‫‪4‬‬ ‫‪4‬‬
‫ﺑﺠﻤﻊ )‪ (1) − (2) + (3) − (4‬ﻧﺠﺪ ‪:‬‬

‫‪Ŀ‬‬
‫)‪2 − (1 + ı) − (1 − ı‬‬ ‫)‪2 − 2 Re (1 + ı‬‬
‫=‪U‬‬ ‫=‬
‫‪4‬‬ ‫‪4‬‬
‫‪2 − 2 Re √2 e‬‬ ‫‪/‬‬
‫‪2 − 2√2 cos n‬‬
‫=‬ ‫=‬
‫‪4‬‬ ‫‪4‬‬
‫ﺑﺠﻤﻊ )‪ (1) + ı × (2) − (3) − ı × (4‬ﻧﺠﺪ ‪:‬‬

‫)‪2 + ı (1 + ı) − ı (1 − ı‬‬ ‫)‪2 + 2 Re ı (1 + ı‬‬


‫=‪V‬‬ ‫=‬
‫‪4‬‬ ‫‪4‬‬
‫‪2 + 2√2 cos‬‬ ‫‪+n‬‬ ‫‪2 − 2√2 sin n‬‬
‫=‬ ‫=‬
‫‪4‬‬ ‫‪4‬‬
‫■‬

‫‪.‬‬
‫𝟔𝟏𝟐‬ ‫‪.‬‬
‫‪n‬‬ ‫!‪n‬‬ ‫‪n‬‬
‫‪.‬‬ ‫=‬ ‫= )‪) (1 + x‬ﻣﻔﻜﻮك ﺛﻨﺎﺋﻲ اﳊﺪ( ﺣﻴﺚ‬ ‫‪x‬‬ ‫ﻧُﺬﻛّﺮ ّ‬
‫أن ‪:‬‬
‫‪k‬‬ ‫!)‪k! (n − k‬‬ ‫=‬
‫‪k‬‬

‫‪ •1‬ﺑﺎﺳﺘﻌﲈل دﺳﺘﻮر ﻣﻮﺍﻓﺮ ‪ ،‬أﺛﺒﺖ ّ‬


‫أن ‪:‬‬
‫‪n‬‬ ‫‪n‬‬ ‫‪n‬‬ ‫‪n‬‬ ‫‪/‬‬ ‫𝜋‪n‬‬
‫= )‪S (n‬‬ ‫‪−‬‬ ‫‪+‬‬ ‫)‪− ⋯ + (−1‬‬ ‫‪=2‬‬ ‫‪cos‬‬ ‫‪,‬‬ ‫‪n − 1 ≤ 2m ≤ n‬‬
‫‪0‬‬ ‫‪2‬‬ ‫‪4‬‬ ‫‪2m‬‬ ‫‪4‬‬
‫‪ņőƱŀ Ţ‬‬
‫‪ •2‬أوﺟﺪ ﻋﺒﺎرة ُﳑَﺎﺛِﻠﺔ ﻟﻠﻤﺠﻤﻮع ‪:‬‬
‫‪n‬‬ ‫‪n‬‬ ‫‪n‬‬ ‫‪n‬‬
‫= )‪S (n‬‬ ‫‪−‬‬ ‫‪+‬‬ ‫)‪− ⋯ + (−1‬‬ ‫‪,‬‬ ‫‪n − 1 ≤ 2m + 1 ≤ n‬‬
‫‪1‬‬ ‫‪3‬‬ ‫‪5‬‬ ‫‪2m + 1‬‬
‫ﺛﻢ َﲢﻘﻖ ﻣﻦ ﻫﺬه اﻟﻨﺘﻴﺠﺔ ﻷﺟﻞ ‪ n = 7 ، n = 6‬و ‪. n = 8‬‬
‫ّ‬

‫ﺍﳊﻞّ‪ .‬ﺑﲈ أﻧﻨﺎ ﺑﺼﺪد اﻟﺒﺤﺚ ﻋﻦ ﳎﻤﻮع ﻣﻌﺎﻣﻼت ﺛﻨﺎﺋﻲ اﳊﺪ ُرﺗَﺒﻬﺎ ﻛﻠﻬﺎ زوﺟﻴﺔ أو ﻛﻠﻬﺎ ﻓﺮدﻳﺔ‪ ،‬ﻓﻌﻠﻴﻨﺎ اﺧﺘﻴﺎر ﻗﻴﻤﺔ ﺧﺎﺻﺔ‬
‫‪ƕ‬‬

‫ﻟﻠﻤﺘﻐﲑ ‪ x‬ﺑﺤﻴﺚ ﻳﻜﻮن ﻟﻠﻤﻘﺪار ‪ x‬ﺧﺼﻮﺻﻴﺎت ﺗﺮﺗﺒﻂ ﺑﺸﻔﻌﻴﺔ )زوﺟﻴﺔ أو ﻓﺮدﻳﺔ( اﻟﻌﺪد اﻟﻄﺒﻴﻌﻲ ‪ . k‬و ﻣﻦ اﻟﺴﻬﻞ ﻣﻼﺣﻈﺔ‬ ‫ّ‬
‫‪VII‬‬

‫أن اﻟﻮﺣﺪة اﻟﺘﺨﻴﻠﻴﺔ ‪ ı‬ﺗﺘﻤﺘﻊ ﲠﺬه اﳌﻴﺰة ‪ :‬ﻓﺎﻟﻌﺪد ‪ ı‬ﺣﻘﻴﻘﻲ إذا ﻛﺎن ‪ k‬زوﺟﻴ ًﺎ ‪ ،‬و ﲣﻴﲇ ﴏف إذا ﻛﺎن ‪ k‬ﻓﺮدﻳ ًﺎ‪.‬‬
‫ّ‬

‫ﺛﻢ ﻧُﻄ ّﺒﻖ دﺳﺘﻮر ﻣﻮﺍﻓﺮ ﻓﻨﺠﺪ ‪:‬‬


‫‪ّ 1 + ı = √2e‬‬ ‫‪/‬‬ ‫‪ •1‬ﻧﺄﺧﺬ إذن ‪ x = ı‬و ﻧﻜﺘﺐ‬
‫‪ũŏ‬‬

‫𝜋‪n‬‬ ‫𝜋‪n‬‬
‫‪√2‬‬ ‫‪cos‬‬ ‫‪+ ı sin‬‬ ‫‪= √2e /‬‬ ‫)‪= (1 + ı‬‬
‫‪4‬‬ ‫‪4‬‬
‫‪n‬‬ ‫‪n‬‬ ‫‪n‬‬ ‫‪n‬‬ ‫‪n‬‬ ‫‪n‬‬
‫=‬ ‫‪+ı‬‬ ‫‪+ı‬‬ ‫‪+ı‬‬ ‫‪+ı‬‬ ‫‪+ı‬‬ ‫⋯‪+‬‬
‫‪0‬‬ ‫‪1‬‬ ‫‪2‬‬ ‫‪3‬‬ ‫‪4‬‬ ‫‪5‬‬
‫‪n‬‬ ‫‪n‬‬ ‫‪n‬‬ ‫‪n‬‬ ‫‪n‬‬ ‫‪n‬‬
‫=‬ ‫‪−‬‬ ‫‪+‬‬ ‫‪−⋯ +ı‬‬ ‫‪−‬‬ ‫‪+‬‬ ‫⋯‪−‬‬
‫‪0‬‬ ‫‪2‬‬ ‫‪4‬‬ ‫‪1‬‬ ‫‪3‬‬ ‫‪5‬‬

‫𝟴𝟬𝟯‬

‫‪http ://tinyurl.com/Malki1718‬‬ ‫‪0‬‬


‫‪ .VII‬ﲤﺎرﻳﻦ ﻟﻠﺘﻌﻤﻖ‬
‫‪.‬‬
‫‪n‬‬ ‫‪n‬‬ ‫‪n‬‬ ‫‪n‬‬
‫= )‪ S (n‬ﺣﻴﺚ ‪ n − 1 ≤ 2m ≤ n‬ﻳﺴﺎوي اﳉﺰء‬ ‫‪−‬‬ ‫‪+‬‬ ‫)‪− ⋯ + (−1‬‬ ‫أي ّ‬
‫أن اﳌﺠﻤﻮع‬
‫‪0‬‬ ‫‪2‬‬ ‫‪4‬‬ ‫‪2m‬‬

‫‪Ŕž Ŧ Ľ‬‬
‫‪ √2e‬ﻣﻨﻪ ‪:‬‬ ‫‪/‬‬ ‫اﳊﻘﻴﻘﻲ ﻟﻠﻌﺪد‬
‫‪n‬‬ ‫‪n‬‬ ‫‪n‬‬ ‫‪n‬‬ ‫‪/‬‬ ‫𝜋‪n‬‬
‫= )‪S (n‬‬ ‫‪−‬‬ ‫‪+‬‬ ‫)‪− ⋯ + (−1‬‬ ‫‪= 2 cos‬‬ ‫‪, n − 1 ≤ 2m ≤ n‬‬
‫‪0‬‬ ‫‪2‬‬ ‫‪4‬‬ ‫‪2m‬‬ ‫‪4‬‬

‫‪œǃ‬‬
‫‪n‬‬ ‫‪n‬‬ ‫‪n‬‬ ‫‪n‬‬
‫= )‪ S (n‬ﺣﻴﺚ‬ ‫‪−‬‬ ‫‪+‬‬ ‫)‪− ⋯ + (−1‬‬ ‫ﳑّﺎ ﺳﺒﻖ‪ ،‬ﻧﺴﺘﺨﻠﺺ ﻣﺒﺎﴍ ًة ّ‬
‫أن اﳌﺠﻤﻮع‬ ‫•‬‫‪2‬‬
‫‪1‬‬ ‫‪3‬‬ ‫‪5‬‬ ‫‪2m + 1‬‬
‫‪ √2e /‬أي ‪:‬‬ ‫‪ n − 1 ≤ 2m ≤ n‬ﻳﺴﺎوي اﳉﺰء اﻟﺘﺨﻴﲇ ﻟﻠﻌﺪد‬

‫‪Ŀ‬‬
‫‪n‬‬ ‫‪n‬‬ ‫‪n‬‬ ‫‪n‬‬ ‫‪/‬‬ ‫𝜋‪n‬‬
‫= )‪S (n‬‬ ‫‪−‬‬ ‫‪+‬‬ ‫)‪− ⋯ + (−1‬‬ ‫‪= 2 sin‬‬ ‫‪, n − 1 ≤ 2m + 1 ≤ n‬‬
‫‪1‬‬ ‫‪3‬‬ ‫‪5‬‬ ‫‪2m + 1‬‬ ‫‪4‬‬
‫ﻷﺟﻞ ‪ n = 7 ، n = 6‬و ‪ ، n = 8‬و ﺑﺎﳊﺴﺎب اﳌﺒﺎﴍ ﻧﺠﺪ ‪:‬‬
‫‪6‬‬ ‫‪6‬‬ ‫‪6‬‬ ‫‪/‬‬ ‫𝜋‪6‬‬
‫= )‪S (6‬‬ ‫‪−‬‬ ‫‪+‬‬ ‫‪= 6 − 20 + 6 = −8 = 2 sin‬‬
‫‪1‬‬ ‫‪3‬‬ ‫‪5‬‬ ‫‪4‬‬
‫‪7‬‬ ‫‪7‬‬ ‫‪7‬‬ ‫‪7‬‬ ‫‪/‬‬ ‫𝜋‪7‬‬
‫= )‪S (7‬‬ ‫‪−‬‬ ‫‪+‬‬ ‫‪−‬‬ ‫‪= 7 − 35 + 21 − 1 = −8 = 2 sin‬‬
‫‪1‬‬ ‫‪3‬‬ ‫‪5‬‬ ‫‪7‬‬ ‫‪4‬‬
‫‪8‬‬ ‫‪8‬‬ ‫‪8‬‬ ‫‪8‬‬ ‫‪/‬‬ ‫𝜋‪8‬‬
‫= )‪S (8‬‬ ‫‪−‬‬ ‫‪+‬‬ ‫‪−‬‬ ‫‪= 8 − 56 + 56 − 8 = 0 = 2 sin‬‬
‫‪1‬‬ ‫‪3‬‬ ‫‪5‬‬ ‫‪7‬‬ ‫‪4‬‬

‫■‬

‫‪.‬‬
‫𝟕𝟏𝟐‬ ‫‪.‬‬
‫‪ •1‬أوﺟﺪ ﺟﺬور ﻛﺜﲑ اﳊﺪود )‪ P (X) = (1 + ıX) − (1 − ıX‬ﻷﺟﻞ ∗‪. n ∈ ℕ‬‬
‫‪−‬‬
‫𝜋‪k‬‬
‫‪.‬‬ ‫‪tan‬‬ ‫‪ •2‬إﺳﺘﻨﺘﺞ‪ ،‬ﻋﻨﺪﻣﺎ ﻳﻜﻮن ‪ n‬ﻓﺮدﻳ ًﺎ‪ ،‬ﻗﻴﻤﺔ اﳌﺠﻤﻮع‬
‫=‬
‫‪n‬‬
‫‪ņőƱŀ Ţ‬‬
‫‪ •1‬ﻟﺪﻳﻨﺎ ‪:‬‬ ‫ﺍﳊﻞّ‪.‬‬
‫)‪P (z) = 0 ⟺ (1 + ız) = (1 − ız‬‬
‫ﻓﺈن ‪ (1 + ız) = 2‬و ‪ (1 − ız) = 0‬أي ّ‬
‫أن ‪ z = −ı‬ﻟﻴﺲ‬ ‫ﻟﻜﻦ ‪ 1 − ız = 0 ⟺ z = −ı‬و ﰲ ﻫﺬه اﳊﺎﻟﺔ ّ‬
‫ﺣﻼ ﻟﻠﻤﻌﺎدﻟﺔ )‪ (1 + ız) = (1 − ız‬و ﺑﺎﻟﺘﺎﱄ ﺑﺈﻣﻜﺎﻧﻨﺎ ﻗﺴﻤﺔ ﻃﺮﻓﻴﻬﺎ ﻋﲆ )‪ (1 − ız‬ﻓﻴﻜﻮن ‪:‬‬
‫)‪(1 + ız‬‬
‫)‪P (z) = 0 ⟺ (1 + ız) = (1 − ız‬‬ ‫⟺‬ ‫‪=1‬‬
‫)‪(1 − ız‬‬
‫‪ƕ‬‬

‫‪1 + ız‬‬
‫‪VII‬‬

‫⟺‬ ‫‪=1=e /‬‬


‫‪1 − ız‬‬
‫‪1 + ız‬‬ ‫‪/ ,‬‬
‫⟺‬ ‫‪=e‬‬ ‫}‪k ∈ {0, 1, 2, ⋯ , n − 1‬‬
‫‪1 − ız‬‬
‫‪ũŏ‬‬

‫‪⟺ ız 1 + e‬‬ ‫‪/‬‬ ‫‪=e‬‬ ‫‪/ −1,‬‬ ‫}‪k ∈ {0, 1, 2, ⋯ , n − 1‬‬


‫𝜋‪k‬‬
‫= 𝜃 ﻳﻨﺘﺞ ‪:‬‬ ‫‪ . e‬ﺑﺄﺧﺬ‬ ‫‪ 1 + e‬و 𝜃 ‪− 1 = 2ıe sin‬‬ ‫ﻟﻜﻦ 𝜃 ‪= 2e cos‬‬
‫‪n‬‬
‫‪P (z) = 0 ⟺ ız 1 + e‬‬ ‫‪/‬‬ ‫‪=e‬‬ ‫‪/‬‬ ‫‪−1,‬‬ ‫}‪k ∈ {0, 1, 2, ⋯ , n − 1‬‬
‫‪/‬‬ ‫𝜋‪k‬‬ ‫‪/‬‬ ‫𝜋‪k‬‬
‫‪⟺ 2ıze‬‬ ‫‪cos‬‬ ‫‪= 2ıe‬‬ ‫‪sin‬‬ ‫‪,‬‬ ‫}‪k ∈ {0, 1, 2, ⋯ , n − 1‬‬
‫‪n‬‬ ‫‪n‬‬

‫𝟵𝟬𝟯‬

‫‪http ://tinyurl.com/Malki1718‬‬ ‫‪0‬‬


‫‪.‬‬
‫ﻟﻜﻦ ‪:‬‬
‫𝜋‪k‬‬ ‫𝜋 𝜋‪k‬‬ ‫‪n‬‬
‫‪cos‬‬ ‫⟺ ‪=0‬‬ ‫=‬ ‫=‪⟺ k‬‬ ‫‪ n‬زوﺟﻲ و‬

‫‪Ŕž Ŧ Ľ‬‬
‫‪n‬‬ ‫‪n‬‬ ‫‪2‬‬ ‫‪2‬‬
‫ﻣﻨﻪ ‪:‬‬

‫‪œǃ‬‬
‫𝜋‪k‬‬
‫‪z = tan‬‬ ‫إذا ﻛﺎن ‪ n‬زوﺟﻴ ًﺎ ّ‬
‫ﻓﺈن ﺟﺬور ﻛﺜﲑ اﳊﺪود )‪ P (X‬ﻫﻲ اﻷﻋﺪاد‬ ‫•‬
‫‪n‬‬
‫‪n‬‬
‫≠ ‪) k‬ﻋﺪد اﳊﻠﻮل ﻫﻮ ‪.(n − 1‬‬ ‫ﻣﻊ }‪ k ∈ {0, 1, 2, ⋯ , n − 1‬و‬
‫‪2‬‬

‫‪Ŀ‬‬
‫𝜋‪k‬‬
‫‪ z = tan‬ﻣﻊ }‪k ∈ {0, 1, 2, ⋯ , n−1‬‬ ‫إذا ﻛﺎن ‪ n‬ﻓﺮدﻳ ًﺎ ّ‬
‫ﻓﺈن ﺟﺬور ﻛﺜﲑ اﳊﺪود )‪ P (X‬ﻫﻲ اﻷﻋﺪاد‬ ‫•‬
‫‪n‬‬
‫)ﻋﺪد اﳊﻠﻮل ﻫﻮ ‪.(n‬‬

‫= )‪ P (X‬ﻛﺜﲑ ﺣﺪود ﺑﻤﻌﺎﻣﻼت ﻣﺮﻛﺒﺔ درﺟﺘﻪ ‪ n ≥ 1‬ﻓﺈﻧﻪ‬ ‫‪a X‬‬ ‫‪ •2‬ﻧﻌﻠﻢ أﻧﻪ )ﻣﱪﻫﻨﺔ ‪ 46‬ﺻﻔﺤﺔ ‪ (383‬إذا ﻛﺎن‬
‫=‬
‫‪a −‬‬
‫‪ . 𝜎 = −‬ﰲ ﻣﺜﺎﻟﻨﺎ ‪:‬‬ ‫ﻳﻘﺒﻞ ‪ n‬ﺟﺬر ًا ﰲ ‪ ℂ‬و ّ‬
‫أن ﳎﻤﻮع ﻫﺬه اﳉﺬور ﻫﻮ‬
‫‪a‬‬
‫‪−‬‬ ‫‪−‬‬ ‫‪−‬‬ ‫‪−‬‬
‫‪P (X) = ı z + nı‬‬ ‫‪z‬‬ ‫)‪+ ⋯ − (−ı) z + n (−ı‬‬ ‫‪z‬‬ ‫⋯‪+‬‬
‫‪−‬‬ ‫‪−‬‬
‫)‪ (−ı‬ﻣﻨﻪ ‪:‬‬ ‫‪=ı‬‬ ‫إذا ﻛﺎن ‪ n‬ﻓﺮدﻳ ًﺎ ّ‬
‫ﻓﺈن ‪ n − 1‬زوﺟﻲ و ﺑﺎﻟﺘﺎﱄ ‪ (−ı) = −ı‬و‬
‫‪P (X) = ı z + nı‬‬ ‫‪−‬‬ ‫‪z‬‬ ‫‪−‬‬ ‫‪+ ⋯ − −ı z + nı‬‬ ‫‪−‬‬ ‫‪z‬‬ ‫‪−‬‬ ‫‪+ ⋯ = 2ı z + 0 ⋅ z‬‬ ‫‪−‬‬ ‫⋯‪+‬‬
‫إذن ‪:‬‬
‫‪−‬‬
‫𝜋‪k‬‬ ‫‪0‬‬
‫‪tan‬‬ ‫‪=𝜎 =−‬‬ ‫‪=0‬‬
‫‪n‬‬ ‫‪2ı‬‬
‫■‬

‫‪.‬‬
‫𝟖𝟏𝟐‬ ‫‪.‬‬
‫‪ı‬‬ ‫‪ı‬‬
‫‪ņőƱŀ Ţ‬‬
‫‪. 1+‬‬ ‫‪− 1−‬‬ ‫أوﺟﺪ ﻛﻞ اﻷﻋﺪاد اﻟﻄﺒﻴﻌﻴﺔ ‪ n ∈ ℕ‬ﺑﺤﻴﺚ ‪= 0 :‬‬ ‫•‬ ‫‪1‬‬
‫‪√3‬‬ ‫‪√3‬‬
‫‪−‬‬
‫‪n‬‬ ‫‪−‬‬
‫‪.‬‬ ‫إﺳﺘﻨﺘﺞ أﻧﻪ ﰲ ﻫﺬه اﳊﺎﻟﺔ ‪(−1) 3 = 0 :‬‬ ‫•‬ ‫‪2‬‬
‫=‬
‫‪2k + 1‬‬

‫إذا ﺣﺎوﻟﻨﺎ ﻧﴩ اﳊﺪ اﻷول ﻣﻦ ﻣﺴﺎواة اﻟﺴﺆال )‪ (1‬ﺑﺘﻄﺒﻴﻖ دﺳﺘﻮر ﺛﻨﺎﺋﻲ اﳊﺪ‪ ،‬ﻓﺈﻧﻨﺎ ﻧﺤﺼﻞ ﻋﲆ ﻣﺴﺎواة اﻟﺴﺆال )‪(2‬‬ ‫ﺍﳊﻞّ‪.‬‬
‫و ﻫﺬه اﻷﺧﲑة ﻳﺼﻌﺐ إﺛﺒﺎﲥﺎ ﺑﺼﻔﺔ ﻣﺒﺎﴍة و ﺑﺎﻟﺘﺎﱄ ﳚﺐ اﻟﺒﺤﺚ ﻋﻦ ﻃﺮﻳﻘﺔ أﺧﺮى‪.‬‬
‫‪ƕ‬‬

‫ﻟﻜﻦ‪ ،‬اﻟﻜﺘﺎﺑﺔ اﻷﺳﻴﺔ )أو اﳌﺜﻠﺜﻴﺔ( ﻫﻲ اﻷﻧﺴﺐ ﻋﻨﺪ اﻟﺘﻌﺎﻣﻞ ﻣﻊ ﺟﺪاء أﻋﺪاد ﻣﺮﻛﺒﺔ ‪ :‬ﻧﺴﺘﺨﺪم إذن اﻟﺸﻜﻞ اﻷﳼ ‪ ...‬و ﻧﺘﺬﻛﺮ‬
‫‪VII‬‬

‫‪1‬‬ ‫𝜋‬
‫‪.‬‬ ‫‪= tan‬‬ ‫ّ‬
‫أن‬
‫‪√3‬‬ ‫‪6‬‬
‫‪ •1‬ﻟﺪﻳﻨﺎ ‪:‬‬
‫‪ũŏ‬‬

‫‪ı‬‬ ‫‪2‬‬ ‫‪√3 1‬‬ ‫‪2‬‬ ‫‪/‬‬


‫‪1+‬‬ ‫=‬ ‫‪+ ı‬‬ ‫=‬ ‫‪e‬‬
‫‪√3‬‬ ‫‪√3‬‬ ‫‪2‬‬ ‫‪2‬‬ ‫‪√3‬‬
‫‪ı‬‬ ‫‪2‬‬ ‫‪√3 1‬‬ ‫‪2 −‬‬ ‫‪/‬‬
‫‪1−‬‬ ‫=‬ ‫‪− ı‬‬ ‫=‬ ‫‪e‬‬ ‫و‬
‫‪√3‬‬ ‫‪√3‬‬ ‫‪2‬‬ ‫‪2‬‬ ‫‪√3‬‬

‫𝟬𝟭𝟯‬

‫‪http ://tinyurl.com/Malki1718‬‬ ‫‪0‬‬


‫‪ .VII‬ﲤﺎرﻳﻦ ﻟﻠﺘﻌﻤﻖ‬
‫‪.‬‬
‫ﻣﻨﻪ‬
‫‪ı‬‬ ‫‪ı‬‬ ‫‪2‬‬ ‫‪2 − /‬‬

‫‪Ŕž Ŧ Ľ‬‬
‫‪1+‬‬ ‫‪− 1−‬‬ ‫⟺ ‪=0‬‬ ‫‪e /‬‬ ‫‪−‬‬ ‫‪e‬‬ ‫‪=0‬‬
‫‪√3‬‬ ‫‪√3‬‬ ‫‪√3‬‬ ‫‪√3‬‬
‫‪2‬‬

‫‪œǃ‬‬
‫⟺‬ ‫‪e / − e− / = 0‬‬
‫‪√3‬‬
‫‪2‬‬ ‫𝜋‪n‬‬
‫⟺‬ ‫‪2ı sin‬‬ ‫‪=0‬‬
‫‪√3‬‬ ‫‪6‬‬

‫‪Ŀ‬‬
‫𝜋‪n‬‬
‫⟺‬ ‫‪sin‬‬ ‫‪=0‬‬
‫‪6‬‬
‫⟺‬ ‫)‪n = 0 (mod 6‬‬

‫إذن اﻷﻋﺪاد اﻟﺘﻲ ﲢﻘﻖ اﳌﻄﻠﻮب ﻫﻲ ﻣﻀﺎﻋﻔﺎت اﻟﻌﺪد ‪. 6‬‬

‫‪ •2‬ﻧﻄﺒﻖ دﺳﺘﻮر ﺛﻨﺎﺋﻲ اﳊﺪ ‪:‬‬

‫‪ı‬‬ ‫‪n‬‬ ‫‪ı‬‬


‫‪1+‬‬ ‫=‬
‫‪√3‬‬ ‫=‬
‫‪k‬‬ ‫‪√3‬‬
‫‪ı‬‬ ‫‪n‬‬ ‫‪ı‬‬
‫‪1−‬‬ ‫=‬ ‫‪−‬‬ ‫و‬
‫‪√3‬‬ ‫=‬
‫‪k‬‬ ‫‪√3‬‬
‫ﻣﻨﻪ‬
‫‪ı‬‬ ‫‪ı‬‬ ‫‪n‬‬ ‫)‪ı − (−ı‬‬
‫‪1+‬‬ ‫‪− 1−‬‬ ‫=‬
‫‪√3‬‬ ‫‪√3‬‬ ‫‪k‬‬
‫=‬ ‫‪√3‬‬

‫ﻓﺈن ‪ ı − (−ı) = 0‬و ﺑﺎﻟﺘﺎﱄ ﻻ ﺗﺒﻘﻰ إﻻّ اﳊﺪود ذات اﻟﺮﺗﺒﺔ اﻟﻔﺮدﻳﺔ و ﺑﲈ أﻧﻨﺎ‬
‫ﰲ اﳌﺠﻤﻮع اﻟﺴﺎﺑﻖ‪ ،‬إذا ﻛﺎن ‪ k‬زوﺟﻴ ًﺎ ّ‬
‫‪n‬‬
‫ﻓﺈن اﳊﺪ اﻷﺧﲑ ﰲ اﳌﺠﻤﻮع اﻟﺴﺎﺑﻖ ﻳﻨﻌﺪم( ‪:‬‬ ‫أن ‪ n‬زوﺟﻲ‪ّ ،‬‬ ‫ﻓﺈن ‪ ∈ ℕ‬ﻣﻨﻪ )ﺑﲈ ّ‬‫ﻧﻌﺘﱪ اﳊﺎﻟﺔ )‪ّ n = 0 (mod 6‬‬
‫‪2‬‬
‫‪ņőƱŀ Ţ‬‬
‫‪ı‬‬ ‫‪ı‬‬ ‫‪n‬‬ ‫)‪ı − (−ı‬‬
‫‪1+‬‬ ‫‪− 1−‬‬ ‫⟺ ‪=0‬‬ ‫‪=0‬‬
‫‪√3‬‬ ‫‪√3‬‬ ‫‪k‬‬
‫=‬ ‫‪√3‬‬
‫‪−‬‬ ‫‪+‬‬ ‫‪+‬‬
‫‪n‬‬ ‫‪ı‬‬ ‫)‪− (−ı‬‬
‫⟺‬ ‫‪+‬‬
‫‪=0‬‬
‫‪2m + 1‬‬
‫=‬ ‫‪√3‬‬
‫‪−‬‬ ‫)‪ı × (ı ) − (−ı) × (−ı‬‬
‫‪n‬‬ ‫⎡‬ ‫‪⎤=0‬‬
‫⟺‬ ‫⎢‬ ‫⎥‬
‫‪2m + 1‬‬ ‫‪/‬‬
‫=‬
‫⎣‬ ‫‪√3 × 3‬‬ ‫⎦‬
‫‪ƕ‬‬

‫‪−‬‬
‫‪VII‬‬

‫‪ı‬‬ ‫‪n‬‬ ‫)‪(−1) + (−1‬‬


‫⟺‬ ‫‪=0‬‬
‫‪√3‬‬ ‫=‬
‫‪2m + 1‬‬ ‫‪3‬‬
‫‪−‬‬
‫‪n‬‬
‫‪ũŏ‬‬

‫‪−‬‬
‫⟺‬ ‫‪2(−1) 3‬‬ ‫‪=0‬‬
‫=‬
‫‪2m + 1‬‬
‫‪−‬‬
‫‪n‬‬ ‫‪−‬‬
‫⟺‬ ‫‪(−1) 3‬‬ ‫‪=0‬‬
‫=‬
‫‪2m + 1‬‬

‫و ﻫﻮ اﳌﻄﻠﻮب‪.‬‬

‫𝟭𝟭𝟯‬

‫‪http ://tinyurl.com/Malki1718‬‬ ‫‪0‬‬


‫‪.‬‬
‫■‬

‫‪Ŕž Ŧ Ľ‬‬
‫‪.𝜔 = e‬‬ ‫‪/‬‬ ‫‪ n‬ﻋﺪد ﻃﺒﻴﻌﻲ ﺑﺤﻴﺚ ‪ n ≥ 2‬و‬
‫!‬ ‫‪.‬‬
‫𝟗𝟏𝟐‬ ‫‪.‬‬

‫‪œǃ‬‬
‫‪−‬‬ ‫‪−‬‬
‫‪.‬‬ ‫𝜔‪z−‬‬ ‫=‬ ‫‪z‬‬ ‫‪ •1‬أﺛﺒﺖ أﻧﻪ ﻟﻜﻞ ﻋﺪد ﻣﺮﻛﺐ ‪ z ∈ ℂ‬ﺑﺤﻴﺚ ‪ّ z ≠ 1‬‬
‫ﻓﺈن ‪:‬‬
‫=‬ ‫=‬

‫‪Ŀ‬‬
‫‪ •2‬ﱢﺑﺮر ّ‬
‫أن اﳌﺴﺎواة اﻟﺴﺎﺑﻘﺔ ﺗﺒﻘﻰ ﺻﺤﻴﺤﺔ ﻷﺟﻞ ‪. z = 1‬‬
‫‪−‬‬
‫𝜋‪k‬‬ ‫‪n‬‬
‫‪.‬‬ ‫‪sin‬‬ ‫‪= −‬‬ ‫‪ •3‬إﺳﺘﻨﺘﺞ اﳌﺴﺎواة ‪:‬‬
‫=‬
‫‪n‬‬ ‫‪2‬‬

‫𝜔 ّ‬
‫ﻓﺈن ‪:‬‬ ‫‪−‬‬ ‫‪ •1‬ﺑﲈ ّ‬
‫أن ﺣﻠﻮل اﳌﻌﺎدﻟﺔ ‪ z − 1 = 0‬ﻫﻲ اﻷﻋﺪاد ‪، ⋯ ، 𝜔 ، 𝜔 ، 1‬‬ ‫ﺍﳊﻞّ‪.‬‬
‫‪−‬‬
‫)‪z − 1 = (z − 1‬‬ ‫𝜔‪z−‬‬
‫=‬

‫ﻣﻨﻪ ‪:‬‬ ‫‪z − 1 = (z − 1) 1 + z + z + ⋯ z‬‬ ‫‪−‬‬ ‫ﻟﻜﻦ ‪:‬‬


‫‪−‬‬ ‫‪−‬‬
‫)‪z − 1 = (z − 1‬‬ ‫𝜔‪z−‬‬ ‫)‪= (z − 1‬‬ ‫‪z‬‬
‫=‬ ‫=‬
‫‪−‬‬ ‫‪−‬‬
‫‪.‬‬ ‫𝜔‪z−‬‬ ‫=‬ ‫‪z‬‬ ‫إذا ﻛﺎن ‪ ، z ≠ 1‬ﻓﺒﺎﺧﺘﺰال اﳊﺪ ‪ z − 1‬ﻳﻨﺘﺞ ‪:‬‬
‫=‬ ‫=‬

‫‪−‬‬ ‫‪−‬‬
‫)ﻛﺜﲑا ﺣﺪود( و ﻣﺘﻄﺎﺑﻘﺘﺎن‬
‫َ‬ ‫⟼ ‪ x‬ﻣﺴﺘﻤﺮﺗﺎن )ﻣﺘﺼﻠﺘﺎن( ﻋﲆ ‪ℝ‬‬ ‫‪x‬‬ ‫⟼ ‪x‬و‬ ‫𝜔‪x−‬‬ ‫‪ •2‬اﻟﺪاﻟﺘﺎن‬
‫=‬ ‫=‬
‫‪ņőƱŀ Ţ‬‬
‫اﳌﺘﻐﲑ ‪ x‬إﱃ ‪ 1‬أي ‪:‬‬
‫)ﻣﺘﺴﺎوﻳﺘﺎن( ﻋﲆ }‪ ℝ ⧵ {1‬و ﺑﺎﻟﺘﺎﱄ ﳍﲈ ﻧﻔﺲ اﻟﻨﻬﺎﻳﺔ ﻋﻨﺪﻣﺎ ﻳﺆول ّ‬
‫‪−‬‬ ‫‪−‬‬
‫‪lim‬‬ ‫𝜔‪x−‬‬ ‫‪= lim‬‬ ‫‪x‬‬
‫→‬ ‫→‬
‫=‬ ‫=‬

‫أن ﻫﺬه اﻟﻨﻬﺎﻳﺔ ﻣﻮﺟﻮدة و ﻣﻨﺘﻬﻴﺔ )ﺑﺎﻹﺳﺘﻤﺮارﻳﺔ( ّ‬


‫ﻓﺈن اﳌﺴﺎواة اﻟﺴﺎﺑﻘﺔ ﺗﺒﻘﻰ ﺻﺤﻴﺤﺔ ﻷﺟﻞ ‪. z = 1‬‬ ‫و ﺑﲈ ّ‬
‫‪−‬‬ ‫‪−‬‬
‫‪ .‬ﻟﻜﻦ ‪:‬‬ ‫𝜔‪z−‬‬ ‫=‬ ‫ﻣﻦ أﺟﻞ ‪ ، z = 1‬ﻧﺘﻴﺠﺔ اﻟﺴﺆال اﻷول ﺗُﻜﺘﺐ ‪z = n :‬‬ ‫•‬ ‫‪3‬‬
‫=‬ ‫=‬
‫‪/‬‬ ‫‪/‬‬ ‫𝜋‪k‬‬
‫‪1−𝜔 =1−e‬‬ ‫‪= −2ıe‬‬ ‫‪sin‬‬
‫‪n‬‬
‫‪ƕ‬‬

‫‪VII‬‬

‫و‬
‫‪−‬‬ ‫‪−‬‬ ‫‪−‬‬
‫‪/‬‬ ‫∑‬ ‫⋅‬ ‫‪−‬‬ ‫‪/‬‬ ‫‪−‬‬
‫‪e‬‬ ‫‪=e‬‬ ‫=‬ ‫‪=e‬‬ ‫‪=e‬‬ ‫‪=ı‬‬
‫=‬
‫‪ũŏ‬‬

‫𝟮𝟭𝟯‬

‫‪http ://tinyurl.com/Malki1718‬‬ ‫‪0‬‬


‫‪ .VII‬ﲤﺎرﻳﻦ ﻟﻠﺘﻌﻤﻖ‬
‫‪.‬‬
‫ﻣﻨﻪ ‪:‬‬
‫‪−‬‬ ‫‪−‬‬

‫‪Ŕž Ŧ Ľ‬‬
‫‪/‬‬ ‫𝜋‪k‬‬
‫=‪n‬‬ ‫𝜔‪1−‬‬ ‫=‬ ‫‪−2ıe‬‬ ‫‪sin‬‬
‫=‬ ‫=‬
‫‪n‬‬

‫‪œǃ‬‬
‫‪−‬‬ ‫‪−‬‬
‫‪−‬‬ ‫‪−‬‬ ‫‪−‬‬ ‫‪/‬‬ ‫𝜋‪k‬‬
‫)‪= (−1‬‬ ‫‪⋅2‬‬ ‫‪⋅ı‬‬ ‫‪e‬‬ ‫‪sin‬‬
‫=‬ ‫=‬
‫‪n‬‬
‫‪−‬‬
‫‪−‬‬ ‫‪−‬‬ ‫‪−‬‬ ‫‪−‬‬ ‫𝜋‪k‬‬
‫)‪= (−1‬‬ ‫‪⋅2‬‬ ‫‪⋅ı‬‬ ‫‪⋅ı‬‬ ‫⋅‬ ‫‪sin‬‬

‫‪Ŀ‬‬
‫=‬
‫‪n‬‬
‫‪−‬‬
‫‪−‬‬ ‫‪−‬‬ ‫‪−‬‬ ‫𝜋‪k‬‬
‫)‪= (−1‬‬ ‫‪⋅2‬‬ ‫‪⋅ı‬‬ ‫‪sin‬‬
‫=‬
‫‪n‬‬
‫‪−‬‬
‫‪−‬‬ ‫‪−‬‬ ‫‪−‬‬ ‫𝜋‪k‬‬
‫)‪= (−1‬‬ ‫‪⋅2‬‬ ‫)‪⋅ (−1‬‬ ‫‪sin‬‬
‫=‬
‫‪n‬‬
‫‪−‬‬
‫‪−‬‬ ‫𝜋‪k‬‬
‫‪=2‬‬ ‫‪sin‬‬
‫=‬
‫‪n‬‬

‫‪−‬‬ ‫‪−‬‬ ‫‪−‬‬ ‫‪−‬‬ ‫‪−‬‬


‫)‪.((−1‬‬ ‫)‪⋅ (−1‬‬ ‫‪ı‬و‪= 1‬‬ ‫‪= ı‬‬ ‫)‪= (−1‬‬ ‫ّ‬
‫)ﻷن‬
‫‪−‬‬
‫𝜋‪k‬‬ ‫‪n‬‬
‫‪.‬‬ ‫‪sin‬‬ ‫‪= −‬‬ ‫ﰲ اﻷﺧﲑ ‪:‬‬
‫=‬
‫‪n‬‬ ‫‪2‬‬
‫■‬

‫‪$‬‬ ‫‪.‬‬
‫𝟎𝟐𝟐‬ ‫‪.‬‬
‫= )‪. P(z‬‬ ‫‪a z‬‬ ‫ﻟﻴﻜﻦ ‪ n ∈ ℕ‬و ‪ . a , a , ⋯ , a ∈ ℂ‬ﻧﻀﻊ‪ ،‬ﻟﻜﻞ ‪: z ∈ ℂ‬‬
‫=‬
‫ﻟﺘﻜﻦ 𝕌 ﳎﻤﻮﻋﺔ اﻷﻋﺪاد اﳌﺮﻛﺒﺔ اﻟﺘﻲ ﻃﻮﻳﻠﺘﻬﺎ ‪ 1‬؛ و ﻟﻴﻜﻦ ‪ M ∈ ℝ+‬ﺑﺤﻴﺚ ﻣﻬﲈ ﻳﻜﻦ 𝕌 ∈ ‪ّ u‬‬
‫ﻓﺈن ‪.||P(u)|| ≤ M :‬‬
‫‪ņőƱŀ Ţ‬‬
‫أﺧﲑ ًا‪ ،‬ﻧﻀﻊ ‪. 𝜔 = e +‬‬

‫ﻟﻜﻞ ‪. p ∈ ℤ‬‬ ‫𝜔‬ ‫‪ •1‬أﺣﺴﺐ اﳌﺠﻤﻮع‬


‫=‬

‫ﺛﻢ اﺳﺘﻨﺘﺞ اﳌﺘﺒﺎﻳﻨﺔ ‪. ||a || ≤ M‬‬


‫ّ‬ ‫𝜔 ‪P‬‬ ‫‪𝜔−‬‬ ‫‪ •2‬ﻟﻴﻜﻦ }‪ . q ∈ {0, 1, ⋯ , n‬ﱢ‬
‫ﺑﺴﻂ اﳌﺠﻤﻮع‬
‫=‬
‫‪ƕ‬‬

‫أن 𝜔 ﺟﺬر ﻣﻦ اﻟﺮﺗﺒﺔ ‪ n + 1‬ﻟﻠﻮﺣﺪة ّ‬


‫ﻓﺈن ‪ 𝜔 = 1‬إذا ﻛﺎن )‪ p = 0 (mod n + 1‬ﻣﻨﻪ ‪:‬‬ ‫‪ •1‬ﻟﻴﻜﻦ ‪ . p ∈ ℤ‬ﺑﲈ ّ‬ ‫ﺍﳊﻞّ‪.‬‬
‫‪VII‬‬

‫𝜔‬ ‫=‬ ‫) 𝜔(‬


‫‪ũŏ‬‬

‫=‬ ‫=‬

‫⎧‬
‫⎪‬ ‫= ‪1‬‬ ‫‪1=n+1‬‬ ‫إذا ﻛﺎن )‪) p = 0 (mod n + 1‬ﻷﻧﻪ ﰲ ﻫﺬه اﳊﺎﻟﺔ ‪ (𝜔 = 1‬؛‬
‫⎪‬
‫⎪‬
‫=‬ ‫=‬ ‫=‬
‫⎨‬
‫⎪‬ ‫‪+‬‬
‫⎪‬
‫) 𝜔( ⎪‬ ‫‪−1 e‬‬ ‫‪−1‬‬
‫‪⎩ 𝜔 −1‬‬ ‫=‬ ‫‪=0‬‬ ‫إذا ﻛﺎن )‪) p ≠ 0 (mod n + 1‬ﻷﻧﻪ ﰲ ﻫﺬه اﳊﺎﻟﺔ ‪ (𝜔 ≠ 1‬؛‬
‫‪𝜔 −1‬‬

‫𝟯𝟭𝟯‬

‫‪http ://tinyurl.com/Malki1718‬‬ ‫‪0‬‬


‫‪.‬‬
‫‪ •2‬ﻟﺪﻳﻨﺎ ‪:‬‬

‫‪Ŕž Ŧ Ľ‬‬
‫𝜔 ‪P‬‬ ‫‪𝜔−‬‬ ‫=‬ ‫‪a‬‬ ‫𝜔‬ ‫‪𝜔−‬‬ ‫=‬ ‫𝜔 ‪a‬‬ ‫‪𝜔−‬‬
‫=‬ ‫=‬ ‫=‬ ‫=‬ ‫=‬

‫‪œǃ‬‬
‫=‬ ‫𝜔 ‪a‬‬ ‫‪−‬‬ ‫=‬ ‫𝜔 ‪a‬‬ ‫‪−‬‬

‫=‬ ‫=‬ ‫=‬ ‫=‬

‫=‬ ‫‪a‬‬ ‫𝜔‬ ‫‪−‬‬ ‫=‬ ‫‪a ×1‬‬

‫‪Ŀ‬‬
‫=‬ ‫=‬ ‫=‬
‫‪= (n + 1) a‬‬

‫ﺣﻴﺚ ‪ p = m − q‬و اﳊﺎﻟﺔ‬ ‫𝜔‬ ‫اﳌﺴﺎواة ﻣﺎ ﻗﺒﻞ اﻷﺧﲑة ﻧﺎﲡﺔ ﻋﻦ ﺗﻄﺒﻴﻖ ﻧﺘﻴﺠﺔ اﻟﺴﺆال اﻷول ﻋﲆ اﳌﺠﻤﻮع‬
‫=‬
‫اﻟﻮﺣﻴﺪة اﻟﺘﻲ ﻳﻜﻮن ﻓﻴﻬﺎ )‪ m − q = 0 (mod n + 1‬ﻫﻲ ﻋﻨﺪﻣﺎ ﻳﻜﻮن ‪ m − q = 0‬أي ﻋﻨﺪﻣﺎ ﻳﻜﻮن ‪ّ m = q‬‬
‫ﻷن ‪:‬‬
‫}‪m ∈ {0, 1, 2, ⋯ , n} ⟹ m − q ∈ {−q, −q + 1, −q + 2, ⋯ , −q + n‬‬
‫إذن ﻟﻜﻞ }‪ّ m ∈ {1, 2, ⋯ , n‬‬
‫ﻓﺈن ‪ 0 < ||m − q|| < n + 1‬ﻣﻨﻪ ‪:‬‬
‫|‬ ‫|‬
‫|| = || ‪||(n + 1) a‬‬ ‫𝜔 ‪P‬‬ ‫‪𝜔−‬‬
‫|‬
‫|‬
‫||‬ ‫=‬ ‫||‬

‫≤‬ ‫𝜔 ‪||P‬‬ ‫‪𝜔−‬‬ ‫||‬ ‫)ﺑﺘﻄﺒﻴﻖ اﳌﺘﺒﺎﻳﻨﺔ اﳌﺜﻠﺜﻴﺔ اﳌﻌﻤﻤﺔ(‬


‫=‬

‫=‬ ‫𝜔 ‪||P‬‬ ‫‪|| ⋅ ||𝜔−‬‬ ‫||‬ ‫)ﺣﺴﺐ ﺧﻮاص اﻟﻄﻮﻳﻠﺔ(‬


‫=‬

‫=‬ ‫𝜔 ‪||P‬‬ ‫||‬ ‫)ﻷن ‪(||𝜔− || = 1‬‬


‫ّ‬
‫=‬
‫‪ņőƱŀ Ţ‬‬
‫≤‬ ‫‪M‬‬ ‫ّ‬
‫)ﻷن 𝕌 ∈ 𝜔(‬
‫=‬
‫‪= (n + 1) M‬‬

‫و ﺑﺎﺧﺘﺰال اﻟﻌﺎﻣﻞ اﳌﺸﱰك )اﳌﻮﺟﺐ ﲤﺎﻣ ًﺎ ( )‪ (n + 1‬ﻳﻨﺘﺞ ‪. ||a || ≤ M :‬‬


‫■‬

‫‪$‬‬ ‫‪.‬‬
‫𝟏𝟐𝟐‬ ‫‪.‬‬
‫‪ƕ‬‬

‫‪VII‬‬

‫‪ّ •1‬‬
‫ﺣﻞ ﰲ ‪ ℂ‬اﳌﻌﺎدﻟﺔ اﻟﺘﺎﻟﻴﺔ ) ‪ z‬ﻫﻮ اﳌﺠﻬﻮل‪ a ،‬ﻋﺪد ﺣﻘﻴﻘﻲ و ‪ n‬ﻋﺪد ﻃﺒﻴﻌﻲ ( ‪:‬‬
‫‪(z + 1) = cos 2na + ı sin 2na‬‬
‫ﻟﺘﻜﻦ ‪ k = 0, 1, 2, ⋯ , n − 1 , z‬ﻫﺬه اﳊﻠﻮل‪.‬‬
‫‪ũŏ‬‬

‫‪−‬‬
‫= )‪. A (z‬‬ ‫) ‪(z − z‬‬ ‫‪ . A (z) = (z + 1) − e‬أﺛﺒﺖ ّ‬
‫أن ‪:‬‬ ‫‪ •2‬ﻧﻀﻊ‬
‫=‬

‫‪−‬‬
‫؟ و ﰲ )‪ A (z‬؟‬ ‫) ‪(z − z‬‬ ‫‪ •3‬ﻣﺎ ﻫﻮ اﳌﻌﺎﻣﻞ اﻟﺜﺎﺑﺖ ﰲ ﻧﴩ‬
‫=‬

‫𝟰𝟭𝟯‬

‫‪http ://tinyurl.com/Malki1718‬‬ ‫‪0‬‬


‫‪ .VII‬ﲤﺎرﻳﻦ ﻟﻠﺘﻌﻤﻖ‬
‫‪.‬‬
‫‪ •4‬ﻧﻀﻊ‬

‫‪Ŕž Ŧ Ľ‬‬
‫‪−‬‬
‫𝜋‪k‬‬ ‫𝜋‬ ‫‪n−1‬‬
‫= ‪P‬‬ ‫‪sin a +‬‬ ‫‪= sin a × sin a +‬‬ ‫‪× ⋯ × sin a +‬‬ ‫𝜋‬
‫=‬
‫‪n‬‬ ‫‪n‬‬ ‫‪n‬‬

‫‪œǃ‬‬
‫‪−‬‬
‫‪+‬‬
‫‪.‬‬ ‫)‪z = (−1‬‬ ‫‪2 ıe‬‬ ‫‪P‬‬ ‫أﺛﺒﺖ ّ‬
‫أن ‪:‬‬
‫=‬

‫‪ •5‬إﺳﺘﻨﺘﺞ ﻋﺒﺎرة ‪ P‬ﺑﺪﻻﻟﺔ ‪. n‬‬

‫‪Ŀ‬‬
‫‪ ، 𝜔 = e‬ﻣﻊ ‪ ، 0 ≤ k ≤ n − 1‬اﳉﺬور اﻟﻨﻮﻧﻴﺔ ﻟﻠﻮﺣﺪة‪.‬‬ ‫‪/‬‬ ‫ﺍﳊﻞّ‪ .‬ﻟﺘﻜﻦ‬

‫‪ z +1 = 𝜔 e‬ﻣﻊ ‪0 ≤ k ≤ n−1‬‬ ‫‪ (z + 1) = e‬إذن اﳊﻠﻮل ﻫﻲ اﻷﻋﺪاد ‪ z‬ﺑﺤﻴﺚ ‪:‬‬ ‫‪ •1‬اﳌﻌﺎدﻟﺔ ﺗﻜﺎﻓﺊ ‪:‬‬
‫ﻣﻨﻪ ‪:‬‬
‫𝜋‪2ık‬‬ ‫𝜋‪ık‬‬ ‫𝜋‪ık‬‬ ‫𝜋‪ık‬‬
‫‪z =𝜔 e‬‬ ‫‪= exp 2ıa +‬‬ ‫‪− 1 = exp ıa +‬‬ ‫‪exp ıa +‬‬ ‫‪− exp −ıa −‬‬
‫‪n‬‬ ‫‪n‬‬ ‫‪n‬‬ ‫‪n‬‬
‫𝜋‪ık‬‬ ‫𝜋‪k‬‬
‫‪= exp ıa +‬‬ ‫‪× 2ı sin a +‬‬
‫‪n‬‬ ‫‪n‬‬

‫‪ (z + 1) = e‬أي ﻫﻲ اﻷﻋﺪاد ‪ z‬ﻣﻊ ‪. 0 ≤ k ≤ n − 1‬‬ ‫‪ •2‬ﺟﺬور ﻛﺜﲑ اﳊﺪود )‪ A (z‬ﻫﻲ ﺣﻠﻮل اﳌﻌﺎدﻟﺔ ‪:‬‬
‫أن اﳌﻌﺎﻣﻞ اﳌﻬﻴﻤﻦ ﰲ )‪ A (z‬ﻫﻮ ‪ّ 1‬‬
‫ﻓﺈن ‪:‬‬ ‫و ﺑﲈ ّ‬
‫‪−‬‬ ‫‪−‬‬
‫× ‪A (z) = 1‬‬ ‫= ) ‪(z − z‬‬ ‫) ‪(z − z‬‬
‫=‬ ‫=‬

‫‪−‬‬ ‫‪−‬‬
‫أي‬ ‫ﺑﺎﻟﺘﻌﻮﻳﺾ ‪ z = 0‬إذن ﻫﺬا اﳌﻌﺎﻣﻞ اﻟﺜﺎﺑﺖ ﻫﻮ ) ‪(−z‬‬ ‫ﻧﺤﺼﻞ ﻋﲆ اﳌﻌﺎﻣﻞ اﻟﺜﺎﺑﺖ ﰲ ) ‪(z − z‬‬ ‫‪3‬‬
‫•‬
‫‪ņőƱŀ Ţ‬‬
‫=‬ ‫=‬
‫‪−‬‬
‫)‪ . (−1‬ﻣﻦ ﺟﻬﺔ أﺧﺮى‪ ،‬اﳌﻌﺎﻣﻞ اﻟﺜﺎﺑﺖ ﰲ )‪ (z + 1‬ﻫﻮ ‪) 1‬و ﻧﺤﺼﻞ ﻋﻠﻴﻪ ﺑﺄﺧﺬ ‪ (z = 0‬إذن اﳌﻌﺎﻣﻞ‬ ‫‪z‬‬
‫=‬
‫‪.1−e‬‬ ‫اﻟﺜﺎﺑﺖ ﰲ )‪ A (z‬ﻫﻮ‬
‫ﻟﻜﻦ ‪:‬‬
‫‪1−e‬‬ ‫‪=− e‬‬ ‫‪− 1 = − [e‬‬ ‫‪(e‬‬ ‫‪− e−‬‬ ‫‪)] = −e‬‬ ‫‪× 2ı sin na = −2ıe‬‬ ‫‪sin na‬‬

‫‪−2ıe‬‬ ‫إذن اﳌﻌﺎﻣﻞ اﻟﺜﺎﺑﺖ ﰲ )‪ A (z‬ﻫﻮ ‪sin na‬‬


‫‪ƕ‬‬
‫‪VII‬‬

‫‪−‬‬ ‫‪−‬‬
‫‪+‬‬
‫‪.‬‬ ‫)‪z = (−1‬‬ ‫‪2ıe‬‬ ‫)‪ (−1‬ﻣﻨﻪ )‪sin (na‬‬ ‫‪z = −2ıe‬‬ ‫)‪sin (na‬‬ ‫‪ •4‬ﺣﺴﺐ اﻟﺴﺆال اﻟﺴﺎﺑﻖ ‪:‬‬
‫=‬ ‫=‬
‫‪ũŏ‬‬

‫𝟱𝟭𝟯‬

‫‪http ://tinyurl.com/Malki1718‬‬ ‫‪0‬‬


.
: ‫ﻟﻜﻦ‬

Ŕž Ŧ Ľ
− − − −
ık𝜋 k𝜋 ık𝜋 k𝜋
z = exp ıa + × 2ı sin a + = exp ıa + 2ı sin a +
= =
n n =
n =
n

œǃ
− −
k𝜋 k𝜋
= exp ıa + ı × (2ı) sin a +
=
2 =
n

𝜋
= exp (ına) exp ı k ×2 ×ı P

Ŀ
n =
𝜋 n (n − 1) (n − 1) 𝜋
=e × exp ı × ×2 ×ı P =e × exp ı ×2 ×ı ×P
n 2 2
/ − −
=e × e ×2 ×ı P =e ×ı ×2 ×ı ×P
− 1
=e ×2 ×ı P =e ×2 × ı × ×P
ı
+
= (−1) 2 ıe P
+ +
: ‫ ﻣﻨﻪ‬2 P = 2 sin (na) ‫( أي‬−1) 2 ıe P = (−1) 2ıe sin (na) : ‫إذن‬
sin (na)
P = −
2

.
𝟐𝟐𝟐 .
.‫ أﺣﺴﺐ ﺣﺎﺻﻞ ﴐب اﳉﺬور اﻟﻨﻮﻧﻴﺔ ﻟﻠﻮﺣﺪة‬. n ≥ 2 ‫ ﺑﺤﻴﺚ‬n ∈ ℕ ‫ﻟﻴﻜﻦ‬

: ‫ إذن‬. k = 0, 1, ⋯ , n − 1 ‫ ﻣﻊ‬e / ّ ‫ ﻧﻌﻠﻢ‬.ّ‫ﺍﳊﻞ‬


‫أن اﳉﺬور اﻟﻨﻮﻧﻴﺔ ﻟﻠﻮﺣﺪة ﻫﻲ اﻷﻋﺪاد‬
ņőƱŀ Ţ
− − −
/ 2ık𝜋 2ı𝜋
e = exp = exp k
= =
n n =
2ı𝜋 n(n − 1) − − −
= exp × =e = (e ) = (−1)
n 2

!
ƕ

.
𝟐𝟐𝟑 .
VII

. ‫ ﳎﻤﻮﻋﺔ اﳉﺬور اﻟﻨﻮﻧﻴﺔ ﻟﻠﻮﺣﺪة‬U ‫ ﻟﺘﻜﻦ‬. n ≥ 2 ‫ ﺑﺤﻴﺚ‬n ∈ ℕ ‫ﻟﻴﻜﻦ‬


. ||z − 1|| : ‫أﺣﺴﺐ اﳌﺠﻤﻮع‬
ũŏ

: ‫ ﻟﺪﻳﻨﺎ‬. k = 0, 1, ⋯ , n − 1 ‫ = 𝜔 ﻣﻊ‬e / ‫ اﳉﺬور اﻟﻨﻮﻧﻴﺔ ﻟﻠﻮﺣﺪة ﻫﻲ اﻷﻋﺪاد‬.ّ‫ﺍﳊﻞ‬


k𝜋
𝜔 −1=e / −1=e e − e− = 2ıe sin
n

𝟯𝟭𝟲

http ://tinyurl.com/Malki1718 0
‫‪ .VII‬ﲤﺎرﻳﻦ ﻟﻠﺘﻌﻤﻖ‬
‫‪.‬‬
‫𝜋‪k‬‬
‫≤ ‪ (0‬ﻣﻨﻪ ‪:‬‬ ‫𝜋‪) ||𝜔 − 1|| = 2 ||sin k𝜋 || = 2 sin k‬ﻣﻮﺟﺐ ّ‬
‫ﻷن 𝜋 <‬ ‫ﻣﻨﻪ‬
‫‪n‬‬ ‫|‬ ‫|‪n‬‬ ‫‪n‬‬

‫‪Ŕž Ŧ Ľ‬‬
‫‪−‬‬ ‫‪−‬‬
‫𝜋‪k‬‬ ‫‪/‬‬
‫= ||‪||z − 1‬‬ ‫‪2 sin‬‬ ‫‪=2‬‬ ‫‪Im e‬‬
‫=‬
‫‪n‬‬ ‫=‬

‫‪œǃ‬‬
‫‪−‬‬ ‫‪/‬‬
‫‪/‬‬
‫‪1− e‬‬
‫‪= 2 Im‬‬ ‫‪e‬‬ ‫‪= 2 Im‬‬ ‫‪/‬‬
‫=‬
‫‪1−e‬‬
‫‪1−e‬‬ ‫‪/‬‬ ‫)‪1 − (−1‬‬ ‫‪1‬‬

‫‪Ŀ‬‬
‫‪= 2 Im‬‬ ‫‪/‬‬
‫‪= 2 Im‬‬ ‫‪= 4 Im‬‬
‫‪1−e‬‬ ‫‪1−e /‬‬ ‫‪1−e‬‬ ‫‪/‬‬

‫‪1‬‬ ‫‪e−‬‬
‫‪= 4 Im‬‬ ‫‪= 4 Im‬‬
‫‪e‬‬ ‫‪e−‬‬ ‫‪−e‬‬ ‫‪−2ı sin‬‬
‫‪ıe−‬‬ ‫‪cos‬‬ ‫𝜋‬
‫‪= 2 Im‬‬ ‫‪=2‬‬ ‫‪= 2 cotan‬‬
‫‪sin‬‬ ‫‪sin‬‬ ‫‪2n‬‬
‫■‬

‫‪.‬‬
‫𝟒𝟐𝟐‬ ‫‪.‬‬
‫‪−‬‬
‫= ‪.S‬‬ ‫𝜔 )‪(k + 1‬‬ ‫ﻟﻴﻜﻦ 𝜔 ﺟﺬر ًا ﻧﻮﻧﻴﺎ ﻟﻠﻮﺣﺪة ﳜﺘﻠﻒ ﻋﻦ ‪ . 1‬ﻧﻀﻊ‬
‫=‬
‫ﺑﺤﺴﺎب ‪ ، (1 − 𝜔) S‬أوﺟﺪ ﻗﻴﻤﺔ ‪. S‬‬

‫ﺍﳊﻞّ‪ .‬اﻟﻄﺮﻳﻘﺔ اﻷوﱃ ‪ :‬ﻟﺪﻳﻨﺎ ‪:‬‬


‫‪−‬‬ ‫‪−‬‬
‫‪+‬‬
‫= ‪(1 − 𝜔) S‬‬ ‫= 𝜔 )𝜔 ‪(k + 1) (1 −‬‬ ‫𝜔 ‪(k + 1) 𝜔 −‬‬
‫=‬ ‫=‬
‫‪−‬‬ ‫‪−‬‬
‫‪+‬‬
‫=‬ ‫‪(k + 1) 𝜔 −‬‬ ‫𝜔 )‪(k + 1‬‬
‫‪ņőƱŀ Ţ‬‬
‫=‬ ‫=‬

‫ﰲ اﳌﺠﻤﻮع اﻷﺧﲑ ﻧﻘﻮم ﺑﺎﻟﺘﻌﻮﻳﺾ ‪ ℓ = k + 1‬ﻣﻨﻪ )‪ (k = 0 ↔ ℓ = 1‬و )‪ (k = n − 1 ↔ ℓ = n‬إذن ‪:‬‬


‫‪−‬‬
‫= ‪(1 − 𝜔) S‬‬ ‫‪(k + 1) 𝜔 −‬‬ ‫𝜔‪ℓ‬‬
‫=‬ ‫=‬
‫‪−‬‬ ‫‪−‬‬
‫=‬ ‫‪k𝜔 +‬‬ ‫‪𝜔 −‬‬ ‫𝜔‪ℓ‬‬
‫=‬ ‫=‬ ‫=‬
‫‪−‬‬
‫‪ƕ‬‬

‫=‬ ‫𝜔)‪𝜔 + 0 + 𝜔 + 2𝜔 + ⋯ + (n − 1‬‬ ‫‪−‬‬


‫‪VII‬‬

‫=‬

‫𝜔)‪− 𝜔 + 2𝜔 + ⋯ (n − 1‬‬ ‫‪−‬‬ ‫𝜔‪+ n‬‬


‫‪ũŏ‬‬

‫‪−‬‬
‫=‬ ‫𝜔‬ ‫𝜔‪− n‬‬
‫=‬

‫‪−‬‬
‫)ﻣﱪﻫﻨﺔ ‪ 14‬ﺻﻔﺤﺔ ‪ 33‬ﻣﻊ ‪ k = 1‬؛ أو ﲤﺮﻳﻦ ‪ 225‬ﺻﻔﺤﺔ ‪ 318‬ﻣﻊ ‪ ( p = 1‬و ‪ 𝜔 = 1‬ﻣﻨﻪ‬ ‫𝜔‬ ‫ﻟﻜﻦ ‪= 0‬‬
‫=‬
‫‪n‬‬
‫= ‪.S‬‬ ‫‪ (1 − 𝜔) S = −n‬أي‬
‫‪𝜔−1‬‬

‫𝟳𝟭𝟯‬

‫‪http ://tinyurl.com/Malki1718‬‬ ‫‪0‬‬


‫‪.‬‬
‫‪−‬‬
‫))‪n (1 + (n − 1 + 1‬‬
‫=‪S‬‬ ‫= ‪ S‬ﻣﻨﻪ‬ ‫إذا ﻛﺎن ‪ّ 𝜔 = 1‬‬
‫ﻓﺈن )‪(k + 1‬‬ ‫ﻣﻼﺣﻈﺔ ‪⧏ : 51‬‬
‫‪2‬‬

‫‪Ŕž Ŧ Ľ‬‬
‫=‬
‫)‪n(n + 1‬‬
‫⧐‬
‫= ‪.S‬‬ ‫أي‬
‫‪2‬‬

‫‪œǃ‬‬
‫اﻟﻄﺮﻳﻘﺔ اﻟﺜﺎﻧﻴﺔ ‪ :‬ﻧﻼﺣﻆ ّ‬
‫أن ‪ S‬ﻳﺴﺎوي ﳎﻤﻮع اﻷﻋﺪاد اﳌﻮﺟﻮدة ﰲ اﳌﺜﻠﺚ اﻟﺘﺎﱄ ‪:‬‬

‫𝜔 𝜔 ‪1‬‬ ‫𝜔‬ ‫𝜔 ⋯‬ ‫‪−‬‬ ‫𝜔‬ ‫‪−‬‬

‫‪Ŀ‬‬
‫𝜔 𝜔‬ ‫𝜔‬ ‫𝜔 ⋯‬ ‫‪−‬‬ ‫𝜔‬ ‫‪−‬‬

‫𝜔‬ ‫𝜔‬ ‫𝜔 ⋯‬ ‫‪−‬‬ ‫𝜔‬ ‫‪−‬‬

‫𝜔‬ ‫𝜔 ⋯‬ ‫‪−‬‬ ‫𝜔‬ ‫‪−‬‬

‫⋱‬ ‫⋮‬ ‫⋮‬

‫𝜔‬ ‫‪−‬‬ ‫𝜔‬ ‫‪−‬‬

‫𝜔‬ ‫‪−‬‬

‫و ﺑﺎﳉﻤﻊ ﺳﻄﺮ ًا ﺳﻄﺮ ًا ﻧﺠﺪ ‪:‬‬


‫‪−‬‬ ‫‪−‬‬ ‫‪−‬‬
‫‪+‬‬ ‫‪−‬‬ ‫𝜔‪𝜔 −‬‬ ‫‪𝜔 −1‬‬
‫=‪S‬‬ ‫𝜔‪𝜔 +‬‬ ‫𝜔‪+⋯+‬‬ ‫=‬ ‫=‬
‫=‬ ‫=‬
‫𝜔‪1−‬‬ ‫=‬
‫𝜔‪1−‬‬
‫‪−‬‬ ‫‪−‬‬ ‫‪−‬‬
‫‪1‬‬ ‫‪1‬‬ ‫‪1‬‬
‫=‬ ‫= ‪𝜔 −1‬‬ ‫‪𝜔 −‬‬ ‫‪1‬‬ ‫=‬ ‫)‪(0 − n‬‬
‫𝜔‪1−‬‬ ‫=‬
‫𝜔‪1−‬‬ ‫=‬ ‫=‬
‫𝜔‪1−‬‬
‫‪ņőƱŀ Ţ‬‬
‫‪n‬‬
‫= ‪.S‬‬ ‫ﻣﻨﻪ‬
‫‪𝜔−1‬‬
‫‪+‬‬
‫‪1−X‬‬
‫= ‪. P(X) = 1 + X + X + ⋯ + X‬‬ ‫اﻟﻄﺮﻳﻘﺔ اﻟﺜﺎﻟﺜﺔ ‪ :‬ﻧﻌﺘﱪ ﻛﺜﲑ اﳊﺪود‬
‫‪1−X‬‬
‫‪+‬‬ ‫‪+‬‬
‫‪1−X‬‬ ‫‪nX‬‬ ‫‪− (n + 1)X + 1‬‬
‫= )‪P (X‬‬ ‫=‬ ‫ﻟﺪﻳﻨﺎ ﻣﻦ ﺟﻬﺔ ‪:‬‬
‫‪1−X‬‬ ‫)‪(1 − X‬‬
‫‪−‬‬
‫‪P (X) = 1 + X + X + ⋯ + X‬‬ ‫‪= 1 + 2X + 3X + ⋯ + nX‬‬ ‫و ﻣﻦ ﺟﻬﺔ أﺧﺮى ‪:‬‬
‫ﻣﻨﻪ ‪:‬‬
‫𝜔‪n‬‬ ‫‪+‬‬ ‫‪− (n + 1)𝜔 + 1‬‬ ‫‪n𝜔 − (n + 1) + 1‬‬ ‫)‪n (𝜔 − 1‬‬ ‫‪n‬‬
‫= )𝜔( ‪S = P‬‬ ‫=‬ ‫=‬ ‫=‬
‫‪ƕ‬‬

‫)𝜔 ‪(1 −‬‬ ‫)𝜔 ‪(1 −‬‬ ‫)𝜔 ‪(1 −‬‬ ‫‪𝜔−1‬‬
‫‪VII‬‬

‫■‬
‫‪ũŏ‬‬

‫𝜔 اﳉﺬور اﻟﻨﻮﻧﻴﺔ ﻟﻠﻮﺣﺪة و ‪. p ∈ ℤ‬‬ ‫ﻟﺘﻜﻦ 𝜔 ‪، ⋯ ، 𝜔 ،‬‬


‫!‬ ‫‪.‬‬
‫𝟓𝟐𝟐‬ ‫‪.‬‬
‫‪−‬‬
‫‪−‬‬ ‫‪−‬‬
‫= ‪.P‬‬ ‫) 𝜔 ‪(2 −‬‬ ‫= ‪ S‬ﺛﻢ اﳉﺪاء‬ ‫𝜔‬ ‫أﺣﺴﺐ اﳌﺠﻤﻮع‬
‫=‬ ‫=‬

‫‪.𝜔 = e‬‬ ‫‪/‬‬ ‫𝜔=‬ ‫‪𝜔 = e‬و‬ ‫‪/‬‬ ‫ﺍﳊﻞّ‪ .‬ﻟﺪﻳﻨﺎ ‪، 𝜔 = 1 :‬‬

‫𝟴𝟭𝟯‬

‫‪http ://tinyurl.com/Malki1718‬‬ ‫‪0‬‬


‫‪ .VII‬ﲤﺎرﻳﻦ ﻟﻠﺘﻌﻤﻖ‬
‫‪.‬‬
‫‪ •1‬ﻟﺪﻳﻨﺎ ‪:‬‬
‫‪−‬‬ ‫‪−‬‬ ‫‪−‬‬

‫‪Ŕž Ŧ Ľ‬‬
‫= ‪S‬‬ ‫= 𝜔‬ ‫𝜔‬ ‫=‬ ‫𝜔‬
‫=‬ ‫=‬ ‫=‬

‫‪œǃ‬‬
‫إذن ‪ S‬ﻫﻮ ﳎﻤﻮع ‪ n‬ﺣﺪا اﻷوﱃ ﳌﺘﺘﺎﻟﻴﺔ ﻫﻨﺪﺳﻴﺔ ﺣﺪّ ﻫﺎ اﻷول ‪ 𝜔 = 𝜔 = 1‬و أﺳﺎﺳﻬﺎ 𝜔 = ‪ . q‬إذن ‪:‬‬
‫‪−‬‬
‫= ‪.S‬‬ ‫‪ e‬أي إذا ﻛﺎن اﻟﻌﺪد ‪ p‬ﻣﻀﺎﻋﻔ ًﺎ ﻟﻠﻌﺪد‪ّ n‬‬
‫ﻓﺈن ‪1 = n :‬‬ ‫‪/‬‬ ‫إذا ﻛﺎن ‪ q = 1‬أي إذا ﻛﺎن ‪= 1‬‬ ‫•‬
‫=‬

‫‪Ŀ‬‬
‫‪q −1‬‬
‫𝜔 = ‪ّ q‬‬
‫ﻷن 𝜔 ﺟﺬر‬ ‫= ‪ . S‬ﻟﻜﻦ ‪= 1‬‬ ‫إذا ﻛﺎن ‪ q ≠ 1‬أي إذا ﱂ ﻳﻜﻦ ‪ p‬ﻣﻀﺎﻋﻔ ًﺎ ﻟـِ ‪ّ n‬‬
‫ﻓﺈن ‪:‬‬ ‫•‬
‫‪q−1‬‬
‫ﻧﻮﲏ ﻟﻠﻮﺣﺪة‪ .‬إذن ‪. S = 0‬‬

‫⎧‬
‫⎪‬
‫⎪‬
‫‪−‬‬ ‫⎪‬
‫‪⎪n‬‬ ‫إذا ﻛﺎن ‪ p‬ﻣﻀﺎﻋﻔ ًﺎ ﻟـِ ‪ n‬؛‬
‫= ‪.S‬‬ ‫= 𝜔‬ ‫ﰲ اﻷﺧﲑ ‪:‬‬
‫=‬
‫⎨‬
‫⎪‬
‫⎪‬
‫⎪‬
‫‪⎪0‬‬ ‫إذا ﱂ ﻳﻜﻦ ‪ p‬ﻣﻀﺎﻋﻔ ًﺎ ﻟـِ ‪ n‬؛‬
‫⎩‬

‫‪ •2‬ﻧﻌﺘﱪ ﻛﺜﲑ اﳊﺪود ‪ . P (X) = X − 1 :‬ﺟﺬور ‪ P‬ﻫﻲ اﻷﻋﺪاد 𝜔 ﻣﻊ ‪ 0 ≤ k ≤ n − 1‬و ﻣﻌﺎﻣﻠﻪ اﳌﻬﻴﻤﻦ ﻫﻮ ‪ 1‬إذن ‪:‬‬
‫‪−‬‬ ‫‪−‬‬
‫× ‪P (X) = X − 1 = 1‬‬ ‫= ) 𝜔 ‪(X −‬‬ ‫) 𝜔 ‪(X −‬‬
‫=‬ ‫=‬

‫ﺑﺄﺧﺬ ‪ X = 2‬ﻳﻨﺘﺞ ‪:‬‬


‫‪−‬‬
‫= ‪P‬‬ ‫‪(2 − 𝜔 ) = P (2) = 2 − 1‬‬
‫=‬

‫■‬
‫‪ņőƱŀ Ţ‬‬
‫‪−‬‬ ‫!‬ ‫‪.‬‬
‫𝟔𝟐𝟐‬ ‫‪.‬‬
‫= ‪ . S‬أﺣﺴﺐ ‪. S‬‬ ‫𝜔‪z+‬‬ ‫‪𝜔 = e‬و‬ ‫ﻟﻴﻜﻦ }‪ n ∈ ℕ ⧵ {0, 1‬و ‪ . z ∈ ℂ‬ﻧﻀﻊ‬
‫=‬

‫أن اﳉﻤﻊ ﻋﻤﻠﻴﺔ ﺗﺒﺪﻳﻠﻴﺔ إذن ﻳﻤﻜﻦ ﺗﺒﺪﻳﻞ رﻣﻮز اﳉﻤﻊ )∑( ﻷﻧﻨﺎ ﻧﺠﻤﻊ ﻋﺪد ًا ﻣﻨﺘﻬﻴ ًﺎ ‪2‬‬
‫ﻧﺴﺘﻌﻤﻞ ﻣﻔﻜﻮك ﺛﻨﺎﺋﻲ اﳊﺪ و ﻧﺘﺬﻛّﺮ ّ‬ ‫ﺍﳊﻞّ‪.‬‬
‫ﻣﻦ اﳊﺪود إذن ‪:‬‬
‫‪−‬‬ ‫‪−‬‬ ‫‪−‬‬
‫‪ƕ‬‬

‫‪n‬‬ ‫‪−‬‬ ‫‪n‬‬ ‫‪−‬‬


‫= ‪S‬‬ ‫𝜔‪z+‬‬ ‫=‬ ‫𝜔‬ ‫‪z‬‬ ‫=‬ ‫‪𝜔 z‬‬
‫‪VII‬‬

‫=‬ ‫=‬ ‫=‬


‫‪ℓ‬‬ ‫=‬ ‫=‬
‫‪ℓ‬‬
‫‪−‬‬ ‫‪−‬‬ ‫‪−‬‬
‫‪n‬‬ ‫‪−‬‬ ‫‪n‬‬ ‫‪−‬‬ ‫‪n‬‬ ‫‪−‬‬
‫=‬ ‫‪𝜔 z‬‬ ‫=‬ ‫‪z‬‬ ‫𝜔‬ ‫=‬ ‫‪z‬‬ ‫𝜔‬
‫‪ℓ‬‬ ‫‪ℓ‬‬ ‫‪ℓ‬‬
‫‪ũŏ‬‬

‫=‬ ‫=‬ ‫=‬ ‫=‬ ‫=‬ ‫=‬

‫‪−‬‬
‫)اﳌﺘﻐﲑ ﻓﻴﻪ ﻫﻮ ‪ ،(k‬ﻧﻤ ّﻴﺰ ﺣﺎﻟﺘﲔ ‪ 𝜔 = 1 :‬و ‪. 𝜔 ≠ 1‬‬
‫ّ‬ ‫𝜔‬ ‫ﳊﺴﺎب اﳌﺠﻤﻮع‬
‫=‬

‫«ﻣﻬﻢ ﺟﺪ ًا ﻷﻧﻪ إذا ﱂ ﻳﻜﻦ اﻷﻣﺮ ﻛﺬﻟﻚ‪ ،‬ﻓﻼ ﻳﻤﻜﻦ ﺗﺒﺪﻳﻞ رﻣﻮز اﻟﺘﺠﻤﻴﻊ إﻻّ ﺑﴩوط )ﺗ ّ‬
‫ُﺴﻤﻰ اﳌﺠﺎﻣﻴﻊ ﻏﲑ اﳌﻨﺘﻬﻴﺔ‬ ‫ّ‬ ‫‪2‬اﻟﴩط »ﻋﺪ ٌد ُﻣﻨﺘ ٍَﻪ ﻣﻦ اﳊﺪود‬
‫ﻋﺪد ﻏﲑ ٍ‬
‫ﻣﻨﺘﻪ ﻣﻦ اﳊﺪود ﺑـِ »اﳌﺘﺴﻠﺴﻼت«— ‪ Series‬ﺑﺎﻹﻧﺠﻠﻴﺰﻳﺔ ؛ ‪ Séries‬ﺑﺎﻟﻔﺮﻧﺴﻴﺔ(‪.‬‬ ‫أي اﻟﺘﻲ ﲢﺘﻮي ﻋﲆ ٍ‬

‫𝟵𝟭𝟯‬

‫‪http ://tinyurl.com/Malki1718‬‬ ‫‪0‬‬


‫‪.‬‬
‫إذا ﻛﺎن ‪ 𝜔 = 1‬أي إذا ﻛﺎن ‪ ℓ = 0‬أو ‪ّ ℓ = n‬‬
‫ﻓﺈن ‪:‬‬ ‫•‬
‫‪−‬‬ ‫‪−‬‬ ‫‪−‬‬

‫‪Ŕž Ŧ Ľ‬‬
‫𝜔‬ ‫=‬ ‫= ‪1‬‬ ‫‪1=n‬‬
‫=‬ ‫=‬ ‫=‬

‫‪œǃ‬‬
‫و إذا ﻛﺎن ‪ 𝜔 ≠ 1‬أي إذا ﻛﺎن ‪ ℓ ≠ 0‬و ‪ّ ℓ ≠ n‬‬
‫ﻓﺈن ‪:‬‬ ‫•‬
‫‪−‬‬
‫𝜔 ‪1−‬‬ ‫) 𝜔( ‪1 −‬‬
‫𝜔‬ ‫=‬ ‫=‬ ‫‪=0‬‬
‫=‬
‫𝜔‪1−‬‬ ‫𝜔‪1−‬‬

‫‪Ŀ‬‬
‫ّ‬
‫ﻷن ‪. 𝜔 = 1‬‬
‫إذن‪ ،‬ﰲ اﳌﺠﻤﻮع ‪ S‬ﻻ ﻳﺒﻘﻰ إﻻّ اﳊﺪّ ان ‪ ℓ = 0‬و ‪ ℓ = n‬أي ‪:‬‬
‫‪n‬‬ ‫‪−‬‬ ‫‪n‬‬ ‫‪−‬‬
‫= ‪S‬‬ ‫‪z‬‬ ‫‪×n+‬‬ ‫‪z‬‬ ‫)‪× n = n (z + 1‬‬
‫‪0‬‬ ‫‪n‬‬
‫■‬

‫!‬ ‫‪.‬‬
‫𝟕𝟐𝟐‬ ‫‪.‬‬
‫‪−‬‬ ‫‪−‬‬
‫‪q‬‬ ‫‪+‬‬
‫= ‪ . S‬أﺣﺴﺐ ‪. S‬‬ ‫𝜔‬ ‫‪𝜔 = e‬و‬ ‫ﻟﻴﻜﻦ }‪ n ∈ ℕ ⧵ {0, 1‬و ‪ . z ∈ ℂ‬ﻧﻀﻊ‬
‫=‬ ‫=‬ ‫‪p‬‬

‫أن اﳉﻤﻊ ﻋﻤﻠﻴﺔ ﺗﺒﺪﻳﻠﻴﺔ إذن ﻳﻤﻜﻦ ﺗﺒﺪﻳﻞ رﻣﻮز اﳉﻤﻊ )∑( ﻷﻧﻨﺎ ﻧﺠﻤﻊ ﻋﺪد ًا ﻣﻨﺘﻬﻴ ًﺎ ‪3‬‬
‫ﻧﺴﺘﻌﻤﻞ ﻣﻔﻜﻮك ﺛﻨﺎﺋﻲ اﳊﺪ و ﻧﺘﺬﻛّﺮ ّ‬ ‫ﺍﳊﻞّ‪.‬‬
‫ﻣﻦ اﳊﺪود إذن ‪:‬‬
‫‪−‬‬ ‫‪−‬‬ ‫‪−‬‬ ‫‪−‬‬
‫‪q‬‬ ‫‪+‬‬ ‫‪q‬‬ ‫‪+‬‬ ‫‪q‬‬
‫= ‪S‬‬ ‫𝜔‬ ‫=‬ ‫𝜔‬ ‫=‬ ‫⎛‬ ‫𝜔⎞ 𝜔‬
‫‪p‬‬ ‫‪p‬‬ ‫‪p‬‬
‫=‬ ‫=‬ ‫=‬ ‫=‬ ‫=‬ ‫⎝‬ ‫=‬ ‫⎠‬
‫‪−‬‬ ‫‪−‬‬
‫=‬ ‫= 𝜔 ) 𝜔 ‪(1 +‬‬ ‫) 𝜔 ) 𝜔 ‪((1 +‬‬
‫‪ņőƱŀ Ţ‬‬
‫=‬ ‫=‬

‫ﳊﺴﺎب اﳌﺠﻤﻮع اﻷﺧﲑ‪ ،‬ﻧﺒﺤﺚ ّأوﻻً إن ﻛﺎن ‪ . 𝜔 (1 + 𝜔 ) = 1‬ﻟﺪﻳﻨﺎ ‪:‬‬


‫‪−1 ± √5‬‬
‫= 𝜔 ⟺ ‪𝜔 (1 + 𝜔 ) = 1 ⟺ 𝜔 + 𝜔 − 1 = 0‬‬
‫‪2‬‬
‫‪| −1 ± √5 | √6‬‬
‫| و ‪ ||𝜔 || = 1‬إذن ‪ 𝜔 (1 + 𝜔 ) ≠ 1‬و ﺑﺎﻟﺘﺎﱄ ﻓﺎﳌﺠﻤﻮع ‪ S‬ﻫﻮ ﳎﻤﻮع ‪ n‬ﺣﺪ ًا اﻷوﱃ ﳌﺘﺘﺎﻟﻴﺔ‬ ‫=|‬ ‫ﻟﻜﻦ ‪≠ 1‬‬
‫||‬ ‫‪2‬‬ ‫||‬ ‫‪2‬‬
‫اﻷول ‪ 1‬و أﺳﺎﺳﻬﺎ ‪ ) 𝜔 (1 + 𝜔 ) ≠ 1‬ﻣﻊ ‪ (𝜔 = 1‬إذن ‪:‬‬ ‫ﻫﻨﺪﺳﻴﺔ ﺣﺪّ ﻫﺎ ّ‬
‫)) 𝜔 ‪1 − (𝜔 (1 +‬‬ ‫) 𝜔 ‪1 − 𝜔 (1 +‬‬ ‫) 𝜔 ‪1 − (1 +‬‬
‫‪ƕ‬‬

‫= ‪S‬‬ ‫=‬ ‫=‬


‫‪VII‬‬

‫) 𝜔 ‪1 − 𝜔 (1 +‬‬ ‫) 𝜔 ‪1 − 𝜔 (1 +‬‬ ‫𝜔‪1−𝜔 −‬‬


‫■‬
‫‪ũŏ‬‬

‫‪.‬‬
‫𝟖𝟐𝟐‬ ‫‪.‬‬
‫ﺣﻞ ﰲ ‪ ℂ‬اﳌﻌﺎدﻟﺔ اﻟﺘﺎﻟﻴﺔ ) ‪ z‬ﻫﻮ اﳌﺠﻬﻮل‪ 𝜃 ،‬ﻋﺪد ﺣﻘﻴﻘﻲ و ‪ n‬ﻋﺪد ﻃﺒﻴﻌﻲ ( ‪:‬‬
‫‪z‬‬ ‫‪− 2z cos n𝜃 + 1 = 0‬‬

‫«ﻣﻬﻢ ﺟﺪ ًا ﻷﻧﻪ إذا ﱂ ﻳﻜﻦ اﻷﻣﺮ ﻛﺬﻟﻚ‪ ،‬ﻓﻼ ﻳﻤﻜﻦ ﺗﺒﺪﻳﻞ رﻣﻮز اﻟﺘﺠﻤﻴﻊ إﻻّ ﺑﴩوط )ﺗ ّ‬
‫ُﺴﻤﻰ اﳌﺠﺎﻣﻴﻊ ﻏﲑ اﳌﻨﺘﻬﻴﺔ‬ ‫ّ‬ ‫‪3‬اﻟﴩط »ﻋﺪ ٌد ُﻣﻨﺘ ٍَﻪ ﻣﻦ اﳊﺪود‬
‫ﻋﺪد ﻏﲑ ٍ‬
‫ﻣﻨﺘﻪ ﻣﻦ اﳊﺪود ﺑـِ »اﳌﺘﺴﻠﺴﻼت«— ‪ Series‬ﺑﺎﻹﻧﺠﻠﻴﺰﻳﺔ ؛ ‪ Séries‬ﺑﺎﻟﻔﺮﻧﺴﻴﺔ‪(.‬‬ ‫أي اﻟﺘﻲ ﲢﺘﻮي ﻋﲆ ٍ‬

‫𝟬𝟮𝟯‬

‫‪http ://tinyurl.com/Malki1718‬‬ ‫‪0‬‬


‫‪ .VII‬ﲤﺎرﻳﻦ ﻟﻠﺘﻌﻤﻖ‬
‫‪.‬‬
‫ﺍﳊﻞّ‪ .‬اﻟﻄﺮﻳﻘﺔ اﻷوﱃ ‪ :‬ﻧﻀﻊ ‪ Z = z‬و ﺗﺼﺒﺢ اﳌﻌﺎدﻟﺔ ‪. Z − 2Z cos n𝜃 + 1 = 0 :‬‬

‫‪Ŕž Ŧ Ľ‬‬
‫اﳌﻤ ﱢﻴﺰ اﳌﺨﺘﴫ ﳍﺬه اﳌﻌﺎدﻟﺔ ﻫﻮ ‪:‬‬
‫)𝜃‪Δ = (− cos n𝜃) − 1 = − sin n𝜃 = (ı sin n‬‬

‫‪œǃ‬‬
‫‪. Z = cos n𝜃 + ı sin n𝜃 = e‬‬ ‫‪ Z = cos n𝜃 − ı sin n𝜃 = e−‬و‬ ‫إذن ﳍﺎ ﺣﻼّن ﳘﺎ ‪:‬‬
‫ﻟﺘﻜﻦ ‪ ، 𝜔 = e /‬ﻣﻊ ‪ ، 0 ≤ k ≤ n − 1‬اﳉﺬور اﻟﻨﻮﻧﻴﺔ ﻟﻠﻮﺣﺪة ‪.‬‬
‫‪ e−‬أي اﻷﻋﺪاد ‪ z = 𝜔 e−‬ﻣﻊ‬ ‫‪ z = e−‬ﻫﻲ اﳉﺬور اﻟﻨﻮﻧﻴﺔ ﻟﻠﻌﺪد‬ ‫ﺣﻠﻮل اﳌﻌﺎدﻟﺔ ‪ z = Z‬أي اﳌﻌﺎدﻟﺔ‬ ‫•‬

‫‪Ŀ‬‬
‫‪.0 ≤ k ≤ n − 1‬‬
‫‪ e‬أي اﻷﻋﺪاد ‪ z = 𝜔 e‬ﻣﻊ ‪.0 ≤ k ≤ n − 1‬‬ ‫‪ z = Z = e‬ﻫﻲ اﳉﺬور اﻟﻨﻮﻧﻴﺔ ﻟﻠﻌﺪد‬ ‫ﺣﻠﻮل اﳌﻌﺎدﻟﺔ‬ ‫•‬

‫‪ exp −ı𝜃 + ı‬ﻣﻊ ‪ ، 0 ≤ k ≤ n − 1‬و اﻷﻋﺪاد‬ ‫ﰲ اﻷﺧﲑ‪ ،‬ﻟﻠﻤﻌﺎدﻟﺔ اﳌﻘﱰﺣﺔ ‪ 2n‬ﺣﻼ و ﻫﻲ اﻷﻋﺪاد ‪ 𝜔 e−‬أي‬
‫‪ exp ı𝜃 + ı‬ﻣﻊ ‪. 0 ≤ k ≤ n − 1‬‬ ‫‪ 𝜔 e‬أي‬
‫اﻟﻄﺮﻳﻘﺔ اﻟﺜﺎﻧﻴﺔ ‪ :‬ﻧﻼﺣﻆ ّ‬
‫أن ‪:‬‬

‫‪z‬‬ ‫‪− 2z cos n𝜃 + 1 =z‬‬ ‫‪− e‬‬ ‫‪+ e−‬‬ ‫‪z + e e−‬‬ ‫‪= z −e‬‬ ‫‪z − e−‬‬
‫‪z‬‬ ‫‪− 2z cos n𝜃 + 1 =0 ⟺ z − e‬‬ ‫‪=0‬‬ ‫أو‬ ‫‪z − e−‬‬ ‫‪=0‬‬ ‫ﻣﻨﻪ ‪:‬‬

‫■‬ ‫ﺛﻢ ﻧﺘﻤﻢ ّ‬


‫اﳊﻞ ﻛﲈ ﰲ اﻟﻄﺮﻳﻘﺔ اﻷوﱃ‪.‬‬ ‫ّ‬

‫‪.‬‬
‫𝟗𝟐𝟐‬ ‫‪.‬‬
‫ﺣﻞ ﰲ ‪ ℂ‬اﳌﻌﺎدﻟﺔ اﻟﺘﺎﻟﻴﺔ ) ‪ z‬ﻫﻮ اﳌﺠﻬﻮل‪ a ،‬ﻋﺪد ﺣﻘﻴﻘﻲ و ‪ n ≥ 1‬ﻋﺪد ﻃﺒﻴﻌﻲ ﻏﲑ ﻣﻌﺪوم ( ‪:‬‬
‫‪1 − ız‬‬ ‫‪1 + ıa‬‬
‫=‬
‫‪1 + ız‬‬ ‫‪1 − ıa‬‬

‫𝜋 𝜋‬
‫ﺍﳊﻞّ‪ .‬ﺑﺪاﻳ ًﺔ‪ ،‬ﻧُﺬﻛﱢﺮ ّ‬
‫أن اﻟﺪاﻟﺔ ‪َ tan‬ﺗ َﻘﺎﺑﻞ ﻣﻦ‬
‫‪ņőƱŀ Ţ‬‬
‫‪ − ,‬ﻧﺤﻮ ‪ ℝ‬؛ إذن ﻟﻜﻞ ﻋﺪد ﺣﻘﻴﻘﻲ ‪ a‬ﻳﻮﺟﺪ ﻋﺪد ﺣﻘﻴﻘﻲ )وﺣﻴﺪ(‬
‫‪2 2‬‬
‫𝜋 𝜋‬
‫‪ 𝜃 ∈ − ,‬ﺑﺤﻴﺚ 𝜃 ‪ a = tan‬إذن ‪:‬‬
‫‪2 2‬‬
‫‪sin‬‬
‫‪1 + ıa 1 + ı tan 𝜃 1 + ı cos‬‬ ‫𝜃 ‪cos 𝜃 + ı sin‬‬ ‫‪e‬‬
‫=‬ ‫=‬ ‫‪sin‬‬
‫=‬ ‫=‬ ‫‪=e‬‬
‫‪1 − ıa 1 − ı tan 𝜃 1 − ı cos‬‬ ‫‪cos 𝜃 − ı sin 𝜃 e−‬‬

‫‪1 − ız‬‬ ‫‪/‬‬


‫‪.‬‬ ‫‪=e‬‬ ‫‪= e‬‬ ‫و ﺑﺎﻟﺘﺎﱄ ﻓﺎﳌﻌﺎدﻟﺔ ﺗﺼﺒﺢ ‪:‬‬
‫‪1 + ız‬‬
‫‪ 𝜔 e‬ﻣﻊ‬ ‫‪/‬‬ ‫‪ e‬ﻫﻲ اﻷﻋﺪاد‬ ‫𝜔 اﳉﺬور اﻟﻨﻮﻧﻴﺔ ﻟﻠﻮﺣﺪة‪ّ ،‬‬
‫ﻓﺈن اﳉﺬور اﻟﻨﻮﻧﻴﺔ ﻟﻠﻌﺪد‬ ‫إذا ﻛﺎﻧﺖ 𝜔 ‪، ⋯ ، 𝜔 ،‬‬
‫‪−‬‬
‫‪1 − ız‬‬ ‫‪/‬‬
‫‪ƕ‬‬

‫ﻣﻊ ‪0 ≤ k ≤ n − 1‬‬ ‫‪=𝜔 e‬‬ ‫‪ . 0 ≤ k ≤ n − 1‬و ﺑﺎﻟﺘﺎﱄ ﺣﻠﻮل اﳌﻌﺎدﻟﺔ اﳌﻘﱰﺣﺔ ﻫﻲ اﻷﻋﺪاد ‪ z‬ﺑﺤﻴﺚ ‪:‬‬
‫‪1 + ız‬‬
‫‪VII‬‬

‫‪ ız 1 + 𝜔 e‬ﻣﻊ ‪ . 0 ≤ k ≤ n − 1‬ﻟﻜﻦ ‪:‬‬ ‫‪/‬‬ ‫‪=1−𝜔 e‬‬ ‫‪/‬‬ ‫أي‬

‫‪/‬‬ ‫𝜋‪2ı𝜃 2ık‬‬ ‫𝜋‪ı𝜃 ık‬‬ ‫𝜋‪ı𝜃 ık‬‬ ‫𝜋‪ı𝜃 ık‬‬


‫‪1+𝜔 e‬‬ ‫‪= 1 + exp‬‬ ‫‪+‬‬ ‫‪= exp‬‬ ‫‪+‬‬ ‫‪exp‬‬ ‫‪+‬‬ ‫‪+ exp − −‬‬
‫‪ũŏ‬‬

‫‪n‬‬ ‫‪n‬‬ ‫‪n‬‬ ‫‪n‬‬ ‫‪n‬‬ ‫‪n‬‬ ‫‪n‬‬ ‫‪n‬‬


‫𝜋‪ı𝜃 ık‬‬ ‫𝜋‪𝜃 + k‬‬
‫‪= 2 exp‬‬ ‫‪+‬‬ ‫‪cos‬‬
‫‪n‬‬ ‫‪n‬‬ ‫‪n‬‬
‫‪/‬‬ ‫𝜋‪2ı𝜃 2ık‬‬ ‫𝜋‪ı𝜃 ık‬‬ ‫𝜋‪ı𝜃 ık‬‬ ‫𝜋‪ı𝜃 ık‬‬
‫‪1−𝜔 e‬‬ ‫‪= 1 − exp‬‬ ‫‪+‬‬ ‫‪= − exp‬‬ ‫‪+‬‬ ‫‪exp‬‬ ‫‪−‬‬ ‫‪− exp − −‬‬
‫‪n‬‬ ‫‪n‬‬ ‫‪n‬‬ ‫‪n‬‬ ‫‪n‬‬ ‫‪n‬‬ ‫‪n‬‬ ‫‪n‬‬
‫𝜋‪ı𝜃 ık‬‬ ‫𝜋‪𝜃 + k‬‬
‫‪= −2ı exp‬‬ ‫‪+‬‬ ‫‪sin‬‬
‫‪n‬‬ ‫‪n‬‬ ‫‪n‬‬

‫𝟭𝟮𝟯‬

‫‪http ://tinyurl.com/Malki1718‬‬ ‫‪0‬‬


‫‪.‬‬
‫إذن ‪:‬‬
‫𝜋‪ı𝜃 ık‬‬ ‫𝜋‪𝜃 + k‬‬ ‫𝜋‪ı𝜃 ık‬‬ ‫𝜋‪𝜃 + k‬‬

‫‪Ŕž Ŧ Ľ‬‬
‫‪ız‬‬ ‫‪1+𝜔 e‬‬ ‫‪/‬‬ ‫‪=1−𝜔 e‬‬ ‫‪/‬‬ ‫‪⟺ 2ız exp‬‬ ‫‪+‬‬ ‫‪cos‬‬ ‫‪= −2ı exp‬‬ ‫‪+‬‬ ‫‪sin‬‬
‫‪n‬‬ ‫‪n‬‬ ‫‪n‬‬ ‫‪n‬‬ ‫‪n‬‬ ‫‪n‬‬
‫𝜋‪𝜃 + k‬‬ ‫𝜋‪𝜃 + k‬‬
‫‪⟺ z cos‬‬ ‫‪= − sin‬‬

‫‪œǃ‬‬
‫‪n‬‬ ‫‪n‬‬
‫ﻓﺈن 𝜋 < 𝜃 ≤ 𝜃 ≤ ‪ − 𝜋 < 0‬؛ و إذا ﻛﺎن ‪ّ 𝜃 < 0‬‬
‫ﻓﺈن‬ ‫ﻓﺈن 𝜋 < 𝜃 < 𝜋 ‪) −‬إذا ﻛﺎن ‪ّ 𝜃 ≥ 0‬‬ ‫أن 𝜋 < 𝜃 < 𝜋 ‪ّ −‬‬ ‫ﺑﲈ ّ‬
‫‪2‬‬ ‫‪n‬‬ ‫‪2‬‬ ‫‪2‬‬ ‫‪n‬‬ ‫‪2‬‬ ‫‪2‬‬ ‫‪2‬‬
‫𝜋‬ ‫𝜃‬ ‫𝜋‪𝜃 + k‬‬ ‫𝜃‬ ‫𝜋‬ ‫𝜃‬ ‫𝜋‬
‫≤ < ‪.−‬‬ ‫< 𝜃 < ‪ (− < 𝜃 < < 0‬ﻣﻨﻪ 𝜋 ‪< +‬‬

‫‪Ŀ‬‬
‫‪2 n‬‬ ‫‪n‬‬ ‫‪n‬‬ ‫‪2‬‬ ‫‪n‬‬ ‫‪2‬‬
‫𝜋‪n‬‬ ‫𝜋 𝜋‪𝜃 + k‬‬ ‫𝜋‪𝜃 + k‬‬
‫= 𝜃 ‪ .‬ﻟﻜﻦ ‪:‬‬ ‫أي ﻋﻨﺪﻣﺎ 𝜋‪− k‬‬ ‫‪ cos‬ﻫﻲ ﻋﻨﺪﻣﺎ =‬ ‫إذن اﳊﺎﻟﺔ اﻟﻮﺣﻴﺪة اﻟﺘﻲ ﻳﻨﻌﺪم ﻓﻴﻬﺎ‬
‫‪2‬‬ ‫‪n‬‬ ‫‪2‬‬ ‫‪n‬‬
‫𝜋‬ ‫𝜋‬ ‫𝜋‪𝜋 n‬‬ ‫𝜋‬
‫<𝜃< ‪−‬‬ ‫< ‪⟹ −‬‬ ‫< 𝜋‪− k‬‬
‫‪2‬‬ ‫‪2‬‬ ‫‪2‬‬ ‫‪2‬‬ ‫‪2‬‬
‫‪⟹ −1 < n − 2k < 1‬‬
‫‪⟹ n − 2k = 0‬‬ ‫ّ‬
‫)ﻷن ‪ n − 2k‬ﻋﺪد ﻃﺒﻴﻌﻲ(‬
‫‪⟹ n = 2k‬‬
‫‪n‬‬
‫= ‪ k‬ﻣﻨﻪ ‪ 𝜃 = 0‬أي ‪ . a = 0‬ﰲ اﻷﺧﲑ ‪:‬‬ ‫أي ‪ n‬ﻋﺪد زوﺟﻲ و‬
‫‪2‬‬
‫‪n‬‬ ‫𝜋‪𝜃 + k‬‬ ‫𝜋‪𝜃 + k‬‬
‫≠ ‪ k‬ﻣﻨﻪ ‪:‬‬ ‫‪ z cos‬ﻣﻊ‬ ‫‪= − sin‬‬ ‫إذا ﻛﺎن ‪ a = 0‬و ‪ n‬زوﺟﻴ ًﺎ ّ‬
‫ﻓﺈن‬ ‫•‬
‫‪2‬‬ ‫‪n‬‬ ‫‪n‬‬
‫‪+‬‬
‫‪sin‬‬ ‫𝜋‪𝜃 + k‬‬
‫‪ z = −‬و ﻫﻲ ﺣﻠﻮل اﳌﻌﺎدﻟﺔ اﳌﻘﱰﺣﺔ و ﻋﺪدﻫﺎ ‪. n − 1‬‬ ‫‪+‬‬
‫‪= − tan‬‬
‫‪cos‬‬ ‫‪n‬‬
‫‪+‬‬
‫‪sin‬‬ ‫𝜋‪𝜃 + k‬‬
‫‪ z = −‬ﻣﻊ ‪ 0 ≤ k ≤ n − 1‬و ﻫﻲ ﺣﻠﻮل اﳌﻌﺎدﻟﺔ‬ ‫‪+‬‬
‫‪= − tan‬‬ ‫إذا ﻛﺎن ‪ a ≠ 0‬أو ‪ n‬ﻓﺮدﻳ ًﺎ ّ‬
‫ﻓﺈن ‪:‬‬ ‫•‬
‫‪cos‬‬ ‫‪n‬‬
‫اﳌﻘﱰﺣﺔ و ﻋﺪدﻫﺎ ‪. n‬‬
‫■‬
‫‪ņőƱŀ Ţ‬‬
‫‪.z = a‬‬ ‫‪ a‬ﻋﺪد ﻣﺮﻛﺐ ﺣﻴﺚ ‪ n ، |a| = 1‬ﻋﺪد ﻃﺒﻴﻌﻲ ‪ .‬ﻟﺘﻜﻦ ‪ z ، ⋯ ، z ، z‬ﺣﻠﻮل اﳌﻌﺎدﻟﺔ‬
‫!‬ ‫‪.‬‬
‫𝟎𝟑𝟐‬ ‫‪.‬‬

‫أﺛﺒﺖ ّ‬
‫أن ﺻﻮر اﻷﻋﺪاد ) ‪ (1 + z ) ، ⋯ ، (1 + z ) ، (1 + z‬ﺗﻘﻊ ﻋﲆ اﺳﺘﻘﺎﻣﺔ واﺣﺪة‪.‬‬

‫ﺍﳊﻞّ‪ .‬ﻧﻀﻊ ‪ّ a = e‬‬


‫)ﻷن ‪ . (|a| = 1‬ﺣﻠﻮل اﳌﻌﺎدﻟﺔ ‪ z = a‬ﻫﻲ اﳉﺬور اﻟﻨﻮﻧﻴﺔ ﻟﻠﻌﺪد ‪ e‬أي ﻫﻲ اﻷﻋﺪاد‬
‫𝜋‪𝜃 + 2k‬‬
‫‪ z = exp ı‬ﻣﻊ ‪ 1 ≤ k ≤ n‬ﻣﻨﻪ ‪:‬‬
‫‪n‬‬
‫‪ƕ‬‬

‫𝜋‪𝜃 + 2k‬‬ ‫𝜋‪𝜃 + 2k‬‬ ‫𝜋‪𝜃 + 2k‬‬ ‫𝜋‪𝜃 + 2k‬‬


‫‪1 + z = 1 + exp ı‬‬ ‫‪= exp ı‬‬ ‫‪exp ı‬‬ ‫‪+ exp −ı‬‬
‫‪VII‬‬

‫‪n‬‬ ‫‪2n‬‬ ‫‪2n‬‬ ‫‪2n‬‬


‫𝜋‪𝜃 + 2k‬‬ ‫𝜋‪𝜃 + 2k‬‬
‫‪= 2 exp ı‬‬ ‫‪cos‬‬
‫‪2n‬‬ ‫‪2n‬‬
‫‪ũŏ‬‬

‫𝜋‪𝜃 + 2k‬‬ ‫𝜋‪𝜃 + 2k‬‬


‫‪(1 + z ) = 2 cos‬‬ ‫‪exp‬‬ ‫ﻣﻨﻪ ‪:‬‬
‫‪2n‬‬ ‫‪2‬‬
‫𝜋‪𝜃 + 2k‬‬ ‫𝜃‬ ‫𝜃‬
‫‪ exp‬إذن ‪:‬‬ ‫‪= exp‬‬ ‫𝜋‪+ k‬‬ ‫‪= (−1) exp‬‬ ‫ﻟﻜﻦ‬
‫‪2‬‬ ‫‪2‬‬ ‫‪2‬‬
‫𝜋‪𝜃 + 2k‬‬ ‫𝜃‬
‫‪(1 + z ) = 2 (−1) cos‬‬ ‫‪exp‬‬
‫‪2n‬‬ ‫‪2‬‬

‫𝟮𝟮𝟯‬

‫‪http ://tinyurl.com/Malki1718‬‬ ‫‪0‬‬


‫‪ .VII‬ﲤﺎرﻳﻦ ﻟﻠﺘﻌﻤﻖ‬
‫‪.‬‬
‫𝜃‬
‫) ‪ arg (1 + z‬و ﺑﺎﻟﺘﺎﱄ‪ ،‬إذا ﻛﺎﻧﺖ ‪ M‬ﺻﻮرة ) ‪ (1 + z‬ﰲ اﳌﺴﺘﻮي اﳌﺮﻛﺐ اﳌﻨﺴﻮب إﱃ ﻣﻌﻠﻢ‬ ‫=‬ ‫إذن )𝜋 ‪(mod‬‬
‫‪2‬‬
‫𝜃‬

‫‪Ŕž Ŧ Ľ‬‬
‫ﻣﺘﻌﺎﻣﺪ و ﻣﺘﺠﺎﻧﺲ ⃗ ‪ّ ، O, i,⃗ j‬‬
‫ﻓﺈن اﻟﺸﻌﺎع ‪ OM‬ﻳﺼﻨﻊ زاوﻳﺔ ﻗﻴﺴﻬﺎ ﻣﻊ ﳏﻮر اﻷﻋﺪاد اﳊﻘﻴﻘﻴﺔ )اﳌﺴﺘﻘﻴﻢ اﻟﺬي ﻣﻌﺎدﻟﺘﻪ‬
‫‪2‬‬
‫𝜃‬
‫ﻳﻤﺮ ﺑﺎﳌﺒﺪأ ‪ O‬و ﻳﺼﻨﻊ زاوﻳﺔ ﻣﻊ ﳏﻮر اﻷﻋﺪاد‬
‫‪ (y = 0‬و ﻫﺬا ﻟﻜﻞ }‪ .k ∈ {1, 2, ⋯ , n‬إذن ‪ M‬ﺗﻨﺘﻤﻲ إﱃ اﳌﺴﺘﻘﻴﻢ اﻟﺬي ّ‬

‫‪œǃ‬‬
‫‪2‬‬
‫𝜃‬
‫‪ y = x tan‬إذا ﻛﺎن ‪. (a ≠ −1‬‬ ‫اﳊﻘﻴﻘﻴﺔ )ﻣﻌﺎدﻟﺔ ﻫﺬا اﳌﺴﺘﻘﻴﻢ ﻫﻲ ‪ x = 0‬إذا ﻛﺎن 𝜋 = 𝜃 أي إذا ﻛﺎن ‪ a = −1‬؛ و‬
‫‪2‬‬

‫■‬ ‫إذن ﺻﻮر اﻷﻋﺪاد ) ‪ (1 + z ) ، ⋯ ، (1 + z ) ، (1 + z‬ﺗﻘﻊ ﻋﲆ اﺳﺘﻘﺎﻣﺔ واﺣﺪة‪.‬‬

‫‪Ŀ‬‬
‫ﻧﻀﻊ }‪ . ℤ [ı] = {a + ıb ; a, b ∈ ℤ‬ﺗُﺴﻤﻰ ﻫﺬه اﳌﺠﻤﻮﻋﺔ ﺑﻤﺠﻤﻮﻋﺔ أﻋﺪاد ﭬﻮﺹ )‪(G‬‬
‫!‬ ‫‪.‬‬
‫𝟏𝟑𝟐‬ ‫‪.‬‬

‫اﻟﺼﺤﻴﺤﺔ‪.‬‬
‫‪ •1‬أﺛﺒﺖ أﻧّﻪ إذا ﻛﺎن ]‪ّ 𝛼, 𝛽 ∈ ℤ [ı‬‬
‫ﻓﺈن ]‪ 𝛼 + 𝛽 ∈ ℤ [ı‬و ]‪. 𝛼𝛽 ∈ ℤ [ı‬‬
‫‪ •2‬ﻣﺎ ﻫﻲ ﻋﻨﺎﴏ ]‪ ℤ [ı‬اﻟﺘﻲ ﳍﺎ ﻣﻘﻠﻮب أي اﻟﻌﻨﺎﴏ ]‪ 𝛼 ∈ ℤ [ı‬اﻟﺘﻲ ﻣﻦ أﺟﻠﻬﺎ ﻳﻮﺟﺪ ]‪ 𝛽 ∈ ℤ [ı‬ﺑﺤﻴﺚ ‪ 𝛼𝛽 = 1‬؟‬
‫‪ •3‬ﲢﻘﻖ ﻣﻦ أﻧﻪ ﻣﻬﲈ ﻛﺎن ‪ ، 𝜁 ∈ ℂ‬ﻓﺈﻧﻪ ﻳﻮﺟﺪ ]‪ z ∈ ℤ [ı‬ﺑﺤﻴﺚ ‪. ||𝜁 − z|| < 1‬‬
‫ُﻌﺮف ﻋﲆ ]‪ ℤ [ı‬ﻗﺴﻤﺔ إﻗﻠﻴﺪﻳﺔ أي ﻣﻬﲈ ﻛﺎن ]‪ 𝛼, 𝛽 ∈ ℤ [ı‬ﻓﺈﻧﻪ ﻳﻮﺟﺪ ]‪ q, r ∈ ℤ [ı‬ﺑﺤﻴﺚ ‪:‬‬‫‪ •4‬أﺛﺒﺖ أﻧﻪ ﺑﺈﻣﻜﺎﻧﻨﺎ أن ﻧ ﱢ‬
‫𝛼‬
‫‪ 𝛼 = 𝛽q + r‬ﻣﻊ ||𝛽|| < |‪) . |r‬إرﺷﺎد ‪ :‬ﻳﻤﻜﻦ إﻋﺘﺒﺎر اﻟﻌﺪد (‪.‬‬
‫𝛽‬

‫‪ •1‬ﻟﻴﻜﻦ ]‪ 𝛼 = a + ıb ∈ ℤ [ı‬و ]‪. 𝛽 = c + ıd ∈ ℤ [ı‬‬ ‫ﺍﳊﻞّ‪.‬‬


‫أن ‪ّ a, b, c, d ∈ ℤ‬‬
‫ﻓﺈن ‪ ac − bd ∈ ℤ ، b + d ∈ ℤ ، a + c ∈ ℤ‬و ‪ ad + bc ∈ ℤ‬و ﺑﺎﻟﺘﺎﱄ ‪:‬‬ ‫ﺑﲈ ّ‬

‫]‪𝛼 + 𝛽 = (a + ıb) + (c + ıd) = (a + c) + ı (b + d) ∈ ℤ [ı‬‬


‫‪ņőƱŀ Ţ‬‬
‫]‪𝛼𝛽 = (a + ıb) (c + ıd) = (ac − bd) + ı (ad + bc) ∈ ℤ [ı‬‬

‫‪ •2‬ﻟﻴﻜﻦ ]‪ 𝛼 = a + ıb ∈ ℤ [ı‬و ]‪ 𝛽 = c + ıd ∈ ℤ [ı‬ﺑﺤﻴﺚ ‪ . 𝛼𝛽 = 1‬ﻫﺬا ﻳﺴﺘﻠﺰم ‪ |𝛼| ⋅ ||𝛽|| = 1‬و ﺑﲈ ّ‬


‫أن‬
‫‪ّ |𝛼| ∈ ℕ‬‬
‫ﻓﺈن ‪ |𝛼| = 1‬أي ‪. a + b = 1‬‬
‫ﻓﺈن ‪ a > 1‬ﻣﻨﻪ ‪ a + b > 1‬إذن ﺑﺎﻟﴬورة ‪ |a| ≤ 1‬أي ‪ |a| = 0‬أو ‪ّ |a| = 1‬‬
‫)ﻷن ‪.(|𝛼| ∈ ℕ‬‬ ‫إذا ﻛﺎن ‪ّ |a| > 1‬‬
‫إذا ﻛﺎن ‪ّ |a| = 0‬‬
‫ﻓﺈن ‪ a = 0‬ﻣﻨﻪ ‪ b = 1‬أي ‪ b = ±1‬و ﺑﺎﻟﺘﺎﱄ ‪ 𝛼 = 0 ± ı = ±ı‬؛‬ ‫•‬

‫و إذا ﻛﺎن ‪ّ |a| = 1‬‬


‫ﻓﺈن ‪ a = 1‬ﻣﻨﻪ ‪ b = 0‬أي ‪ b = 0‬و ﺑﺎﻟﺘﺎﱄ ‪. 𝛼 = ±1 + 0ı = ±1‬‬ ‫•‬
‫‪ƕ‬‬
‫‪VII‬‬

‫إذن } ‪. 𝛼 ∈ {−ı, ı, −1, 1,‬‬


‫ﺑﺎﻟﻌﻜﺲ‪ (−1)− = −1 ∈ ℤ [ı] ، ı− = −ı ∈ ℤ [ı] ، (−ı)− = ı ∈ ℤ [ı] ،‬و ]‪. 1− = 1 ∈ ℤ [ı‬‬
‫ﰲ اﻷﺧﲑ‪ ،‬ﻋﻨﺎﴏ ]‪ ℤ [ı‬اﻟﺘﻲ ﳍﺎ ﻣﻘﻠﻮب ﻫﻲ ‪ −1 ، ı ، −ı‬و ‪. 1‬‬
‫‪ũŏ‬‬

‫‪ •3‬ﻟﻴﻜﻦ ‪ 𝜁 = x + ıy ∈ ℂ‬و ‪ .t ∈ ℝ‬ﻧﺮﻣﺰ ﺑـِ )‪ E (t‬ﻟﻠﺠﺰء اﻟﺼﺤﻴﺢ ﻟﻠﻌﺪد اﳊﻘﻴﻘﻲ ‪ t‬أي أﻛﱪ ﻋﺪد ﺻﺤﻴﺢ ﻻ ﻳﺘﺠﺎوز‬
‫⎧‬ ‫‪1‬‬
‫)‪⎪ E (t‬‬ ‫إذا ﻛﺎن ‪ t ≤ E (t) +‬؛‬
‫= ‪.n‬‬ ‫‪2‬‬ ‫‪ . t‬إذن ‪ . E (t) ≤ t < E (t) + 1‬ﻧﻌﺘﱪ اﻟﻌﺪد ‪:‬‬
‫⎨‬
‫‪⎪ E (t) + 1‬‬ ‫‪1‬‬
‫⎩‬ ‫‪ t > E (t) +‬؛‬ ‫إذا ﻛﺎن‬
‫‪2‬‬

‫𝟯𝟮𝟯‬

‫‪http ://tinyurl.com/Malki1718‬‬ ‫‪0‬‬


‫‪.‬‬
‫‪1‬‬
‫أن ‪ n ∈ ℤ‬و ّ‬
‫أن ‪ n‬ﻫﻮ اﻟﻌﺪد اﻟﺼﺤﻴﺢ اﻷﻗﺮب ﻣﻦ ‪ t‬أي اﻟﺬي ُﳛﻘﻖ ≤ || ‪. ||t − n‬‬ ‫ﻣﻦ اﻟﻮاﺿﺢ ّ‬
‫‪2‬‬

‫‪Ŕž Ŧ Ľ‬‬
‫‪1‬‬ ‫‪1‬‬
‫ﻟﺪﻳﻨﺎ إذن ≤ || ‪ ||x − n‬و ≤ || ‪. ||y − n‬‬
‫‪2‬‬ ‫‪2‬‬
‫ﻧﻀﻊ ‪ . z = n + ın‬ﻟﺪﻳﻨﺎ ]‪ z ∈ ℤ [ı‬و‬

‫‪œǃ‬‬
‫‪1 1 1‬‬
‫‪||z − 𝜁|| = (x − n ) + y − n‬‬ ‫‪= + = <1‬‬
‫‪4 4 2‬‬
‫ﻣﻨﻪ ‪. ||z − 𝜁|| < 1‬‬

‫‪Ŀ‬‬
‫إذن‪ ،‬ﻣﻬﲈ ﻛﺎن ‪ ، 𝜁 ∈ ℂ‬ﻳﻮﺟﺪ ]‪ z ∈ ℤ [ı‬ﺑﺤﻴﺚ ‪. ||𝜁 − z|| < 1‬‬
‫𝛼|‬ ‫|‬
‫‪ •4‬ﻟﻴﻜﻦ ]‪ 𝛼, 𝛽 ∈ ℤ [ı‬ﺑﺤﻴﺚ ‪ . 𝛽 ≠ 0‬ﻧﻌﺘﱪ اﻟﻌﺪد ]‪ q ∈ ℤ [ı‬اﻟﺬي ُﳛﻘﻖ ‪ q) | − q| < 1‬ﻣﻮﺟﻮد ﺣﺴﺐ اﻟﺴﺆال‬
‫𝛽 ||‬ ‫||‬
‫اﻟﺴﺎﺑﻖ ﻷﺟﻞ 𝛼 = 𝜁 و ﻟﻜﻦ ﻟﻴﺲ ﺑﺎﻟﴬورة وﺣﻴﺪ( ‪ .‬ﻧﻌﺘﱪ اﻟﻌﺪد ‪ . r = 𝛼 − 𝛽q‬ﺑﲈ ّ‬
‫أن ]‪ 𝛽 ∈ ℤ [ı] ، 𝛼 ∈ ℤ [ı‬و‬
‫𝛽‬
‫𝛼| | ‪| r‬‬ ‫|‬ ‫‪r‬‬ ‫𝛼‬
‫ﻓﺈن ]‪ 𝛼 − 𝛽q ∈ ℤ [ı‬أي ]‪ . r ∈ ℤ [ı‬ﺑﺎﻹﺿﺎﻓﺔ إﱃ ذﻟﻚ ﻟﺪﻳﻨﺎ ‪ = − q :‬ﻣﻨﻪ ‪| | = | − q| < 1‬‬ ‫]‪ّ q ∈ ℤ [ı‬‬
‫𝛽 || || 𝛽 ||‬ ‫||‬ ‫𝛽 𝛽‬
‫أي ||𝛽|| < |‪. |r‬‬
‫ُﻌﺮف ﻋﲆ ]‪ ℤ [ı‬ﻗﺴﻤﺔ إﻗﻠﻴﺪﻳﺔ‪.‬‬
‫إذن‪ ،‬ﺑﺈﻣﻜﺎﻧﻨﺎ أن ﻧ ﱢ‬
‫■‬

‫‪.‬‬
‫𝟐𝟑𝟐‬ ‫‪.‬‬
‫ﻣﺎ ﻫﻲ ﺻﻮرة اﻟﺪاﺋﺮة ذات اﳌﻌﺎدﻟﺔ اﳌﺮﻛﺒﺔ ‪) ||z − 1|| = 1‬ﺑﺎﺳﺘﺜﻨﺎء اﻟﻨﻘﻄﺔ ‪ (z = 2‬ﺑﺎﻟﺘﺤﻮﻳﻞ ‪:‬‬
‫‪z‬‬
‫= ‪ m (z) ⟼ M Z‬؟‬
‫‪2−z‬‬

‫ﺍﳊﻞّ‪ .‬ﺑﲈ ّ‬
‫أن ﻃﻮﻳﻠﺔ ‪ z − 1‬ﺗﺴﺎوي ‪ّ 1‬‬
‫ﻓﺈن ‪ z − 1 = e‬ﻣﻊ 𝜋 ≤ 𝜃 < 𝜋‪ −‬أي ‪ z = 1 + e‬و ‪ z ≠ 2‬ﻳﻌﻨﻲ ‪ 𝜃 ≠ 0‬ﻣﻨﻪ ‪:‬‬
‫‪1+e‬‬ ‫‪1+e‬‬ ‫‪e−‬‬ ‫‪/‬‬ ‫‪+e‬‬ ‫‪/‬‬ ‫‪2 cos‬‬ ‫𝜃‬
‫=‪Z‬‬ ‫=‬ ‫=‬ ‫=‬ ‫‪= ı cotan‬‬
‫‪ņőƱŀ Ţ‬‬
‫‪2− 1+e‬‬ ‫‪1−e‬‬ ‫‪e−‬‬ ‫‪/‬‬ ‫‪−e‬‬ ‫‪/‬‬
‫‪−2ı sin‬‬ ‫‪2‬‬
‫𝜃‬
‫‪ cotan‬ﻳﻤﺴﺢ ‪ ℝ‬إذن ﺻﻮرة اﻟﺪاﺋﺮة ‪||z − 1|| = 1‬‬ ‫ﻋﻨﺪﻣﺎ ﲤﺴﺢ اﻟﻨﻘﻄﺔ )‪ m (z‬اﻟﺪاﺋﺮة ‪ّ ||z − 1|| = 1‬‬
‫ﻓﺈن 𝜃 ﻳﻤﺴﺢ ]𝜋 ‪ ]−𝜋,‬و‬
‫‪2‬‬
‫■‬ ‫ﻫﻲ ﳏﻮر اﻷﻋﺪاد اﻟﺘﺨﻴﻠﻴﺔ ‪. Re (z) = 0‬‬

‫= )‪. f(z‬‬
‫|‪z + |z‬‬
‫ﻟﺘﻜﻦ ‪ f : ℂ ⟶ ℂ‬اﻟﺪاﻟﺔ اﳌﻌﺮﻓﺔ ﺑﺎﻟﻌﺒﺎرة‬
‫‪.‬‬
‫𝟑𝟑𝟐‬ ‫‪.‬‬‫!‬
‫‪2‬‬
‫ﻣﺎ ﻫﻲ ﳎﻤﻮﻋﺔ اﻟﻘﻴﻢ اﻟﺘﻲ ﺗﺄﺧﺬﻫﺎ ﻫﺬه اﻟﺪاﻟﺔ ‪ f‬؟ ﺑﻤﻌﻨﻰ أوﺟﺪ )‪. f (ℂ‬‬
‫‪ƕ‬‬

‫‪VII‬‬

‫ﺍﳊﻞّ‪ .‬ﻟﻴﻜﻦ ‪ z‬ﻋﺪد ًا ﻣﺮﻛﺒﺎ ‪.‬‬


‫‪ũŏ‬‬

‫‪z + |z| z − z‬‬


‫= )‪ f(z‬ﻣﻨﻪ )‪. 0 ∈ f (ℂ‬‬ ‫=‬ ‫إذا ﻛﺎن ‪ّ z ∈ ℝ−‬‬
‫ﻓﺈن ‪= 0‬‬ ‫•‬
‫‪2‬‬ ‫‪2‬‬
‫إذا ﻛﺎن ‪ّ z ∉ ℝ−‬‬
‫ﻓﺈن ‪ z = re‬ﻣﻊ ‪ r > 0‬و [𝜋 ‪ . 𝜃 ∈ ]−𝜋,‬ﻣﻨﻪ ‪:‬‬ ‫•‬

‫‪z + |z| re + r‬‬ ‫‪1+e‬‬ ‫𝜃‬


‫= )‪f(z‬‬ ‫=‬ ‫‪=r‬‬ ‫‪= re / cos‬‬
‫‪2‬‬ ‫‪2‬‬ ‫‪2‬‬ ‫‪2‬‬
‫𝜃‬ ‫𝜃‬
‫‪ ||f(z)|| = r cos‬و = ))‪. arg (f(z‬‬ ‫أن ‪ّ cos 𝜃 ≥ 0‬‬
‫ﻓﺈن‬ ‫و ﺑﲈ ّ‬
‫‪2‬‬ ‫‪2‬‬ ‫‪2‬‬

‫𝟰𝟮𝟯‬

‫‪http ://tinyurl.com/Malki1718‬‬ ‫‪0‬‬


‫‪ .VII‬ﲤﺎرﻳﻦ ﻟﻠﺘﻌﻤﻖ‬
‫‪.‬‬
‫إذن ‪. f(z) ∈ Z ∈ ℂ ||| Re (Z) > 0‬‬

‫‪Ŕž Ŧ Ľ‬‬
‫|‬ ‫ﻣﻨﻪ ﻧﺴﺘﺨﻠﺺ ّ‬
‫⊂ )‪. f (ℂ‬‬ ‫}‪Z ∈ ℂ | Re (Z) > 0 ∪ {0‬‬
‫|‬
‫أن‬
‫و ﺑﺎﻟﻌﻜﺲ‪ ،‬ﻟﻴﻜﻦ ‪ Z ∈ ℂ‬ﺑﺤﻴﺚ ‪. Re (Z) > 0‬‬

‫‪œǃ‬‬
‫𝜋 𝜋‬
‫‪ّ 𝛼∈ − ,‬‬
‫)ﻷن ‪. (Re (Z) > 0‬‬ ‫ﺑﺈﻣﻜﺎﻧﻨﺎ أن ﻧﻜﺘﺐ ‪ Z = Re :‬ﻣﻊ ‪ R > 0‬و‬
‫‪2 2‬‬
‫‪R‬‬
‫= ‪ z‬ﻟﺪﻳﻨﺎ ‪:‬‬ ‫‪e‬‬ ‫ﻷﺟﻞ‬
‫𝛼 ‪cos‬‬

‫‪Ŀ‬‬
‫‪R‬‬ ‫‪R‬‬ ‫𝛼‪2‬‬ ‫‪/‬‬
‫‪f(z) = f‬‬ ‫‪e‬‬ ‫=‬ ‫‪× cos‬‬ ‫‪e‬‬ ‫‪= Re‬‬ ‫‪=Z‬‬
‫𝛼 ‪cos‬‬ ‫𝛼 ‪cos‬‬ ‫‪2‬‬
‫إذن ﻛﻞ ﻋﺪد ﻣﺮﻛﺐ ‪ Z‬ﺑﺤﻴﺚ ‪ Re (Z) > 0‬ﻫﻮ ﺻﻮرة ﻋﺪد ﻣﺮﻛﺐ ‪ z‬ﺑﺎﻟﺪاﻟﺔ ‪ f‬أي ّ‬
‫أن ‪:‬‬
‫)‪. Z ∈ ℂ ||| Re (Z) > 0 ⊂ f (ℂ‬‬
‫|‬
‫ﻟﻜﻦ )‪ 0 ∈ f (ℂ‬ﻣﻨﻪ )‪Z ∈ ℂ | Re (Z) > 0 ∪ {0} ⊂ f (ℂ‬‬
‫|‬
‫|‬
‫ﰲ اﻷﺧﲑ ‪f (ℂ) = {0} ∪ Z ∈ ℂ | Re (Z) > 0 :‬‬
‫|‬
‫■‬

‫‪ D = z ∈ ℂ ||| |z| < 1‬و ‪ f : ℂ ⧵ {−ı} ⟶ ℂ‬اﻟﺪاﻟﺔ اﳌﻌﺮﻓﺔ‬ ‫‪ P = z ∈ ℂ ||| Im (z) > 0‬؛‬ ‫ﻟﻴﻜﻦ‬
‫!‬ ‫‪.‬‬
‫𝟒𝟑𝟐‬ ‫‪.‬‬

‫‪z−ı‬‬
‫= )‪. f (z‬‬ ‫ﺑﺎﻟﻌﺒﺎرة‬
‫‪z+ı‬‬
‫‪ •1‬أﺛﺒﺖ أن ﺻﻮرة أي ﻋﻨﴫ ﻣﻦ ‪ P‬ﺑﺎﻟﺪاﻟﺔ ‪ f‬ﻫﻲ ﻋﻨﴫ ﻣﻦ ‪. D‬‬
‫‪ •2‬أﺛﺒﺖ أﻧﻪ ﻟﻜﻞ ﻋﻨﴫ ﻣﻦ ‪ D‬ﺳﺎﺑﻘﺔ وﺣﻴﺪة ﺑﺎﻟﺪاﻟﺔ ‪ f‬ﰲ اﳌﺠﻤﻮﻋﺔ ‪. P‬‬
‫‪ņőƱŀ Ţ‬‬
‫أن ‪ّ z ∈ P‬‬
‫ﻓﺈن ‪ . y > 0‬ﻟﺪﻳﻨﺎ ‪:‬‬ ‫‪ •1‬ﻟﻴﻜﻦ ‪ . z ∈ P‬ﻧﻀﻊ )‪ x = Re (z‬و )‪ . y = Im (z‬ﺑﲈ ّ‬ ‫ﺍﳊﻞّ‪.‬‬
‫|‪|z − ı‬‬ ‫||‪||x + (y − 1) ı‬‬ ‫)‪x + (y − 1‬‬
‫= ||)‪||f (z‬‬ ‫=‬ ‫=‬
‫|‪|z + ı‬‬ ‫||‪||x + (y + 1) ı‬‬ ‫)‪x + (y + 1‬‬

‫ﻟﻜﻦ ‪ y > 0‬ﻣﻨﻪ )‪ x + (y − 1) < x + (y + 1‬أي ‪ ||f (z)|| < 1‬و ﻫﺬا ﻳﻌﻨﻲ ّ‬
‫أن ‪. f (z) ∈ D‬‬
‫ﻟﻴﻜﻦ ‪ . Z ∈ D‬ﻟﺪﻳﻨﺎ ‪:‬‬ ‫•‬ ‫‪2‬‬
‫‪z−ı‬‬ ‫‪1+Z‬‬
‫‪ƕ‬‬

‫=‪Z‬‬ ‫‪⟺ zZ + ıZ = z − ı ⟺ z (1 − Z) = ı (1 + Z) ⟺ z = ı‬‬


‫‪VII‬‬

‫‪z+ı‬‬ ‫‪1−Z‬‬
‫ﻣﻊ‬

‫‪1+Z‬‬ ‫)‪(1 + Z) (1 − Z‬‬ ‫‪1 + Z − Z − ZZ‬‬


‫‪ũŏ‬‬

‫‪ı‬‬ ‫‪=ı‬‬ ‫‪=ı‬‬


‫‪1−Z‬‬ ‫||‪||1 − Z‬‬ ‫||‪||1 − Z‬‬
‫‪−ı Z + Z‬‬ ‫||‪1 − ||Z‬‬ ‫))‪−ı (2ı Im (Z‬‬ ‫||‪1 − ||Z‬‬
‫=‬ ‫‪+ı‬‬ ‫=‬ ‫‪+ı‬‬
‫||‪||1 − Z‬‬ ‫||‪||1 − Z‬‬ ‫||‪||1 − Z‬‬ ‫||‪||1 − Z‬‬
‫)‪2 Im (Z‬‬ ‫||‪1 − ||Z‬‬
‫=‬ ‫‪+ı‬‬
‫||‪||1 − Z‬‬ ‫||‪||1 − Z‬‬

‫𝟱𝟮𝟯‬

‫‪http ://tinyurl.com/Malki1718‬‬ ‫‪0‬‬


‫‪.‬‬
‫‪1+Z‬‬
‫‪.z = ı‬‬ ‫‪ Im ı 1 + Z‬و ﻫﺬا ﻳﻌﻨﻲ ّ‬
‫أن ‪∈ P‬‬ ‫أن ‪ّ Z ∈ D‬‬
‫ﻓﺈن ‪ ||Z|| < 1‬ﻣﻨﻪ ‪ 1 − ||Z|| > 1‬أي ‪> 0‬‬ ‫ﺑﲈ ّ‬
‫‪1−Z‬‬ ‫‪1−Z‬‬

‫‪Ŕž Ŧ Ľ‬‬
‫اﻟﻜﺘﺎﺑﺔ اﻟﺴﺎﺑﻘﺔ ﻟﻠﻌﺪد ‪ z‬ﺑﺪﻻﻟﺔ ‪ Z‬ﺗُﺜﺒﺖ أﻧﻪ وﺣﻴﺪ أي ‪:‬‬

‫‪œǃ‬‬
‫‪∀Z ∈ D , ∃! z ∈ P : f (z) = Z .‬‬
‫■‬ ‫اﳋﻼﺻﺔ ‪ :‬ﳑّﺎ ﺳﺒﻖ‪ ،‬ﻧﺴﺘﻨﺘﺞ ّ‬
‫أن ‪ f‬ﺗﻄﺒﻴﻖ ﺗﻘﺎﺑﲇ ﻣﻦ ‪ P‬ﻧﺤﻮ ‪. D‬‬

‫‪Ŀ‬‬
‫‪.‬‬
‫𝟓𝟑𝟐‬ ‫‪.‬‬
‫ﻟﻴﻜﻦ ∗‪ Z ∈ ℂ‬ﻋﺪد ًا ﻣﺮﻛﺒﺎ ﻏﲑ ﻣﻌﺪوم ‪ .‬ﺣﻞ اﳌﻌﺎدﻟﺔ ‪ e = Z‬ذات اﳌﺠﻬﻮل ‪. z ∈ ℂ‬‬

‫ﺍﳊﻞّ‪ .‬ﻧﻀﻊ ||‪ 𝜃 = arg (Z) (mod 2𝜋) ، 𝜌 = ||Z‬و ‪ z = a + ıb‬ﻣﻊ ‪ . a, b ∈ ℝ‬ﻟﺪﻳﻨﺎ ‪:‬‬

‫‪e =Z ⟺ e‬‬ ‫‪+‬‬ ‫‪= 𝜌e‬‬


‫‪⟺ e e‬‬ ‫‪= 𝜌e‬‬
‫𝜌= ‪⟺ e‬‬ ‫‪ b = 𝜃 + 2k𝜋 , k ∈ ℤ‬و‬
‫‪ b = 𝜃 + 2k𝜋 , k ∈ ℤ‬و 𝜌 ‪⟺ a = ln‬‬

‫إذن اﳊﻠﻮل ﻫﻲ اﻷﻋﺪاد ‪ z = ln 𝜌 + ı𝜃 + 2ık𝜋 :‬ﻣﻊ ‪. k ∈ ℤ‬‬


‫‪ (𝜌e = 𝜌e +‬و‬ ‫ﻣﻼﺣﻈﺔ ‪ ⧏ : 52‬ﻋﲆ ﻋﻜﺲ اﻟﺪاﻟﺔ اﻷﺳﻴﺔ اﳊﻘﻴﻘﻴﺔ‪ّ ،‬‬
‫ﻓﺈن اﻟﺪاﻟﺔ اﻷﺳﻴﺔ اﳌﺮﻛﺒﺔ دورﻳﺔ ) ّ‬
‫ﻷن‬
‫ﺑﺎﻟﺘﺎﱄ ﻟﻜﻞ ﻋﺪد ﻣﺮﻛﺐ ‪ Z‬ﻳﻮﺟﺪ ﻋﺪد ﻻﳖﺎﺋﻲ ﻣﻦ اﻷﻋﺪاد اﳌﺮﻛﺒﺔ ‪ z‬ﺑﺤﻴﺚ ‪ e = Z‬و ﻫﺬه اﻷﻋﺪاد ﺗُﻌﻄﻰ ﺑﺎﻟﻌﺒﺎرة‬
‫𝜋‪ z = ln 𝜌 + ı𝜃 + 2ık‬ﻣﻊ ‪. k ∈ ℤ‬‬
‫ﻧﻘﻮل ﻋﻦ داﻟﺘﻬﺎ اﻟﻌﻜﺴﻴﺔ )داﻟﺔ اﻟﻠﻮﻏﺎرﻳﺘﻢ( ّأﳖﺎ داﻟﺔ ﻣﺘﻌﺪدة اﻟﻘﻴﻤﺔ ‪) 4‬أي ّأﳖﺎ ﺗ ّ‬
‫ُﻌﲔ ﻋﺪد ًا ﻻ ﳖﺎﺋﻴﺎ ﻣﻦ اﻟﺼﻮر اﳌﺨﺘﻠﻔﺔ ﻟﻠﻌﺪد‬
‫⧐‬
‫اﳌﺮﻛﺐ اﻟﻮاﺣﺪ ‪ (z‬و ﻋﻦ ﻣﻨﺤﻨﺎﻫﺎ أﻧﻪ ﻣﺘﻌﺪد اﻟﻔﺮوع ‪.‬‬
‫■‬
‫‪ņőƱŀ Ţ‬‬
‫= ‪. th z‬‬
‫‪sh z‬‬ ‫‪1‬‬ ‫‪−‬‬ ‫‪1‬‬ ‫‪−‬‬
‫ﻟﻴﻜﻦ ‪ . z ∈ ℂ‬ﻧﻀﻊ ) ‪ sh z = (e − e ) ، ch z = (e + e‬و‬
‫!‬‫‪.‬‬
‫𝟔𝟑𝟐‬ ‫‪.‬‬
‫‪ch z‬‬ ‫‪2‬‬ ‫‪2‬‬
‫ﻧﺴﻤﻰ ﻫﺬه اﻟﺪوال ‪ :‬اﻟﺪوال اﻟﺰاﺋﺪﻳﺔ )‪ = ch‬ﺟﻴﺐ اﻟﺘﲈم اﻟﺰاﺋﺪي‪ = sh ،‬اﳉﻴﺐ اﻟﺰاﺋﺪي‪ = th ،‬اﻟﻈﻞ اﻟﺰاﺋﺪي(‪.‬‬

‫‪. (e ) = e‬‬ ‫‪ •1‬ﻟﻴﻜﻦ ‪ . z ∈ ℂ‬أﺛﺒﺖ ّ‬


‫أن ‪:‬‬
‫ﻣﻌﺮﻓ ًﺎ؟‬
‫‪ •2‬ﻣﺎ ﻫﻲ اﻷﻋﺪاد اﳌﺮﻛﺒﺔ ‪ z‬اﻟﺘﻲ ﻳﻜﻮن ﻣﻦ أﺟﻠﻬﺎ ‪ّ th z‬‬
‫‪ƕ‬‬

‫‪ •3‬ﺣﻞ ﰲ ‪ ℂ‬اﳌﻌﺎدﻟﺔ ‪. th z = 0 :‬‬


‫‪VII‬‬

‫𝜋‬
‫< ||)‪⎧ ||Im (z‬‬
‫‪.‬‬ ‫‪2‬‬ ‫‪ •4‬ﺣﻞ ﰲ ‪ ℂ‬اﳉﻤﻠﺔ ‪:‬‬
‫⎨‬ ‫‪||th z|| < 1‬‬
‫‪ũŏ‬‬

‫⎩‬
‫𝜋‬
‫< ||)‪ Δ = z ∈ ℂ : ||Im (z‬إﱃ }‪. U = {z ∈ ℂ : |z| < 1‬‬ ‫أن اﻟﺪاﻟﺔ ‪ th‬ﺗُﺸﻜّﻞ ﺗﻄﺒﻴﻘ ًﺎ ﺗﻘﺎﺑﻠﻴﺎ ﻣﻦ‬
‫‪ •5‬أﺛﺒﺖ ّ‬
‫‪4‬‬

‫‪ Multiple-valued4‬ﺑﺎﻹﻧﺠﻠﻴﺰﻳﺔ ؛ ‪ Multivoque‬ﺑﺎﻟﻔﺮﻧﺴﻴﺔ‪.‬‬

‫𝟲𝟮𝟯‬

‫‪http ://tinyurl.com/Malki1718‬‬ ‫‪0‬‬


‫ ﲤﺎرﻳﻦ ﻟﻠﺘﻌﻤﻖ‬.VII
.
: ‫ ﻟﺪﻳﻨﺎ‬. x, y ∈ ℝ ‫ ﻣﻊ‬z = x + ıy ‫ ﻧﻀﻊ‬. z ∈ ℂ ‫• ﻟﻴﻜﻦ‬1 .ّ‫ﺍﳊﻞ‬
+ −
(e ) = e = e (cos y + ı sin y) = e ⋅ cos y + ı sin y = e (cos y − ı sin y) = e

Ŕž Ŧ Ľ
=e

: ‫ ﻟﻜﻦ‬. ch z ≠ 0 ‫ﻌﺮف إذا و ﻓﻘﻂ إذا ﻛﺎن‬ ّ ‫ﻣﻌﺮﻓﺎن‬


ّ ‫ ُﻣ‬th z ‫ﻓﺈن‬ ّ ‫ ﺑﲈ‬. z ∈ ℂ ‫• ﻟﻴﻜﻦ‬2
ّ sh z ‫ و‬ch z ‫أن‬

œǃ
ch z = 0 ⟺ e + e− = 0 ⟺ e = e− ⟺ e = −1
⟺ e = e ⟺ 2z = ı𝜋 + 2ık𝜋 , k∈ℤ
𝜋
⟺ z = ı + ık𝜋 , k ∈ ℤ

Ŀ
2
𝜋
. z ∈ ℂ ⧵ ı + ık𝜋 , k ∈ ℤ ‫ﻌﺮف إذا و ﻓﻘﻂ إذا ﻛﺎن‬
ّ ‫ ُﻣ‬th z : ‫إذن‬
2
𝜋
: ‫ ﻟﺪﻳﻨﺎ‬. z ∈ ℂ ⧵ ı + ık𝜋 , k ∈ ℤ ‫• ﻟﻴﻜﻦ‬3
2
th z = 0 ⟺ sh z = 0 ⟺ e = e− ⟺ e =1 ⟺ e =e
⟺ 2z = 0 + 2ık𝜋 , k ∈ ℤ ⟺ z = ık𝜋 , k ∈ ℤ
𝜋
ّ
: ‫ ﻫﻲ‬th z = 0 ‫ﻓﺈن ﳎﻤﻮﻋﺔ ﺣﻠﻮل اﳌﻌﺎدﻟﺔ‬ ık𝜋 , k ∈ ℤ ⋂ ı + ık𝜋 , k ∈ ℤ = ∅ ّ ‫و ﺑﲈ‬
‫أن‬
2
. ık𝜋 , k ∈ ℤ
𝜋
: ‫ ﻟﺪﻳﻨﺎ‬. x, y ∈ ℝ ‫ ﻣﻊ‬z = x + ıy ‫ ﻧﻀﻊ‬. z ∈ ℂ ⧵ ı + ık𝜋 , k ∈ ℤ ‫• ﻟﻴﻜﻦ‬4

2

sh z e − e− e (e − e− ) e − 1
th z = = + − = = =
ch z e + e− e (e + e− ) e + 1
: ‫ﻣﻨﻪ‬
| e − e− | | e − e− |
||th z|| < 1 ⟺ | |<1 ⟺ | | <1
|| e + e− || −
|| e + e ||
ņőƱŀ Ţ
⟺ ||e − e− || < ||e + e− || ⟺ (e − e− ) (e − e− ) < (e + e− ) (e + e− )
⟺ (e − e− ) e − e− < (e + e− ) e + e−
⟺ −e − − e− − <e − + e− − ⟺ 2 e + e− >0
⟺ cos (2y) > 0

: ‫و ﺑﺎﻟﺘﺎﱄ‬
𝜋 𝜋
⎧ ||Im (z)|| < ⎧ ||y|| < 𝜋 𝜋
2 ⟺ 2 ⟺ ||y|| < ⟺ ||Im (z)|| < ⟺ z∈Δ
⎨ |th z| < 1 ⎨ cos (2y) > 0 4 4
⎩ | | ⎩
ƕ
VII

.‫اﳌﻌﺮﻓﺔ ﰲ اﻟﺴﺆال اﳌﻮاﱄ‬


ّ ‫ ﻫﻲ اﳌﺠﻮﻋﺔ‬Δ ‫ﺣﻴﺚ‬
ّ ‫ ُﻣ‬th z ،‫ ﺣﺴﺐ اﻟﺴﺆال اﻟﺜﺎﲏ‬. z ∈ Δ ‫• ﻟﻴﻜﻦ‬5
: ‫|| و ﺑﺎﻟﺘﺎﱄ‬th z|| < 1 ،‫ﻌﺮف و ﺣﺴﺐ اﻟﺴﺆال اﻟﺮاﺑﻊ‬
. z ∈ Δ ⟹ th z ∈ U
ũŏ

. U ‫ ﻧﺤﻮ اﳌﺠﻤﻮﻋﺔ‬Δ ‫ ﺗُﺸﻜّﻞ ﺗﻄﺒﻴﻘ ًﺎ ﻣﻦ اﳌﺠﻤﻮﻋﺔ‬th ‫أن اﻟﺪاﻟﺔ‬


ّ ‫ﻫﺬا ﻳﻌﻨﻲ‬
: ‫ ﻟﺪﻳﻨﺎ‬. z ∈ Δ ‫( و ﻟﻴﻜﻦ‬Z ≠ −1 ‫ و‬Z ≠ 1 ‫|| و ﺑﺎﻟﺘﺎﱄ‬Z|| < 1 ‫ )أي‬Z ∈ U ‫ﻟﻴﻜﻦ‬
e − e− e −1 1+Z
th z = Z ⟺ −
=Z ⟺ =Z ⟺ e =
e +e e +1 1−Z

𝟯𝟮𝟳

http ://tinyurl.com/Malki1718 0
‫‪.‬‬
‫‪1+Z‬‬ ‫‪1+Z‬‬
‫ﻣﻊ ‪ 𝜌 ∈ ℝ∗+‬و ]𝜋 ‪ 𝜃 ∈ ]−𝜋,‬و ﺑﺎﻟﺘﺎﱄ ‪:‬‬ ‫‪= 𝜌e‬‬ ‫إذن ﻳﻤﻜﻦ أن ﻧﻜﺘﺐ‬ ‫أن ‪ّ Z ≠ −1‬‬
‫ﻓﺈن ‪≠ 0‬‬ ‫ﺑﲈ ّ‬
‫‪1−Z‬‬ ‫‪1−Z‬‬

‫‪Ŕž Ŧ Ľ‬‬
‫‪1+Z‬‬
‫‪e‬‬ ‫=‬ ‫‪⟺ e‬‬ ‫‪= 𝜌e‬‬ ‫‪⟺ e‬‬ ‫‪⋅e‬‬ ‫‪= 𝜌e‬‬
‫‪1−Z‬‬

‫‪œǃ‬‬
‫‪⟺ e‬‬ ‫𝜌=‬ ‫و‬ ‫‪2y = 𝜃 + 2k𝜋 , k ∈ ℤ‬‬
‫‪1‬‬ ‫𝜃‬
‫𝜌 ‪⟺ x = ln‬‬ ‫و‬ ‫‪y = + k𝜋 , k ∈ ℤ‬‬
‫‪2‬‬ ‫‪2‬‬

‫‪Ŀ‬‬
‫‪r+1‬‬ ‫‪1+Z‬‬ ‫∗‬
‫= ‪ Z‬و ﺑﲈ‬ ‫ﻣﻨﻪ ‪∈ ℝ‬‬ ‫ﻓﺈن ‪= 𝜌e = −𝜌 ∈ ℝ−‬‬ ‫ﻫﻞ ﻳﻤﻜﻦ أن ﻳﻜﻮن 𝜋 = 𝜃 ؟ إذا ﻛﺎن اﻷﻣﺮ ﻛﺬﻟﻚ ّ‬
‫‪r−1‬‬ ‫‪1−Z‬‬
‫𝜃‬ ‫𝜋 𝜋‬ ‫‪1+Z‬‬ ‫∗‬
‫‪∈ − ,‬‬ ‫و ﻫﺬا ﺗﻨﺎﻗﺾ ! إذن 𝜋 ≠ 𝜃 أي [𝜋 ‪ 𝜃 ∈ ]−𝜋,‬ﻣﻨﻪ‬ ‫أن ‪ ||Z|| < 1‬أي [‪ّ Z ∈ ]−1, 1‬‬
‫ﻓﺈن ‪∈ ℝ+‬‬ ‫ّ‬
‫‪2‬‬ ‫‪2 2‬‬ ‫‪1−Z‬‬
‫و ﺑﺎﻟﺘﺎﱄ ‪:‬‬
‫⎧‬ ‫‪1‬‬
‫‪th z = Z‬‬‫𝜌 ‪⎪ x = ln‬‬
‫⟺‬ ‫‪2‬‬
‫‪z∈Δ‬‬ ‫𝜃 = ‪⎪y‬‬
‫⎨‬
‫⎩‬ ‫‪2‬‬
‫‪1‬‬ ‫‪|1 + Z| ı‬‬ ‫‪1+Z‬‬
‫| ‪ z = ln‬ﻣﻊ‬ ‫‪| + Arg‬‬ ‫و ﻫﺬا ﻳﻌﻨﻲ ّ‬
‫أن ﻛﻞ ﻋﻨﴫ ‪ Z‬ﻣﻦ ‪ U‬ﻳﻘﺒﻞ ﺳﺎﺑﻘﺔ وﺣﻴﺪة ‪ z‬ﰲ ‪) Δ‬و ﻫﻲ‬
‫‪2‬‬ ‫‪|| 1 − Z || 2‬‬ ‫‪1−Z‬‬
‫‪1+Z‬‬ ‫‪1+Z‬‬
‫اﻟﺘﻲ ﺗﻨﺘﻤﻲ إﱃ اﳌﺠﺎل [𝜋 ‪. ( ]−𝜋,‬‬ ‫‪ Arg‬ﻳﻤ ّﺜﻞ ﻋﻤﺪة اﻟﻌﺪد‬
‫‪1−Z‬‬ ‫‪1−Z‬‬
‫ﰲ اﻷﺧﲑ‪ ،‬اﻟﺪاﻟﺔ ‪ th‬ﺗُﺸﻜّﻞ ﺗﻄﺒﻴﻘ ًﺎ ﺗﻘﺎﺑﻠﻴﺎ ﻣﻦ ‪ Δ‬ﻧﺤﻮ ‪. U‬‬
‫■‬

‫‪.‬‬
‫𝟕𝟑𝟐‬ ‫‪.‬‬
‫‪ a‬و ‪ b‬ﻋﺪادن ﺣﻘﻴﻘﻴﺎن ‪.‬‬
‫‪n‬‬ ‫‪n‬‬
‫‪. Z = S + ıT‬‬ ‫= ‪ . T‬ﻧﻌﺘﱪ اﻟﻌﺪد اﳌﺮﻛﺐ ‪:‬‬ ‫)‪sin (a + kb‬‬ ‫=‪ S‬و‬ ‫)‪cos (a + kb‬‬ ‫ﻧﻀﻊ‬
‫‪ņőƱŀ Ţ‬‬
‫=‬
‫‪k‬‬ ‫=‬
‫‪k‬‬

‫‪n‬‬
‫‪.Z = e‬‬ ‫‪e‬‬ ‫‪ •1‬أﺛﺒﺖ ّ‬
‫أن ‪:‬‬
‫=‬
‫‪k‬‬

‫‪ •2‬إﺳﺘﻨﺘﺞ ﻛﻼ ﻣﻦ ‪ S‬و ‪. T‬‬

‫‪ •1‬ﻟﺪﻳﻨﺎ ‪:‬‬ ‫ﺍﳊﻞّ‪.‬‬


‫‪ƕ‬‬

‫‪VII‬‬

‫‪n‬‬ ‫‪n‬‬ ‫‪n‬‬ ‫‪n‬‬


‫=‪Z‬‬ ‫‪cos (a + kb) + ı‬‬ ‫= )‪sin (a + kb‬‬ ‫‪cos (a + kb) + ı‬‬ ‫)‪sin (a + kb‬‬
‫=‬
‫‪k‬‬ ‫=‬
‫‪k‬‬ ‫=‬
‫‪k‬‬ ‫‪k‬‬
‫‪ũŏ‬‬

‫‪n‬‬ ‫‪n‬‬ ‫‪+‬‬ ‫‪n‬‬


‫=‬ ‫= ))‪(cos (a + kb) + ı sin (a + kb‬‬ ‫‪e‬‬ ‫‪=e‬‬ ‫‪e‬‬
‫=‬
‫‪k‬‬ ‫=‬
‫‪k‬‬ ‫=‬
‫‪k‬‬

‫𝟴𝟮𝟯‬

‫‪http ://tinyurl.com/Malki1718‬‬ ‫‪0‬‬


‫ ﲤﺎرﻳﻦ ﻟﻠﺘﻌﻤﻖ‬.VII
.
: ‫• ﻧُﻄﺒﻖ ﻣﻔﻜﻮك ﺛﻨﺎﺋﻲ اﳊﺪ‬2

Ŕž Ŧ Ľ
n / /
Z=e e =e 1+e =e e e + e− /

=
k

œǃ
/ b + b
=e e 2 cos =2 e cos
2 2
b nb nb
= 2 cos cos a + + ı sin a +
2 2 2

Ŀ
b nb
S = Re (Z) = 2 cos cos a + : ‫ﻣﻨﻪ‬
2 2
b nb
T = Im (Z) = 2 cos sin a + ‫و‬
2 2

.
𝟐𝟑𝟖 .
.S = cos (2k − 1) 𝜃 : ‫ ﻧﻀﻊ‬.‫ ﻋﺪد ﻃﺒﻴﻌﻲ‬n ‫𝜃 ﻋﺪد ﺣﻘﻴﻘﻲ و‬
=

. m ∈ ℤ ‫ = 𝜃 ﻣﻊ‬m𝜋 ‫ إذا ﻛﺎن‬S ‫• أﺣﺴﺐ‬1

ّ ‫• ﻧﻔﺮض‬2
. 𝜃 ≠ 0 (mod 𝜋) : ‫أن‬
ّ k ‫)ا( أﺛﺒﺖ أﻧﻪ ﻣﻬﲈ ﻛﺎن اﻟﻌﺪد اﻟﺼﺤﻴﺢ‬
: ‫ﻓﺈن‬
2 sin 𝜃 cos (2k − 1) 𝜃 = sin 2k𝜃 − sin 2 (k − 1) 𝜃

. 2S sin 𝜃 = sin 2n𝜃 : ‫أن‬ ّ ‫)ب( أﺛﺒﺖ‬


. 𝜃 ‫ و‬n ‫ ﺑﺪﻻﻟﺔ‬S ‫)ج( إﺳﺘﻨﺘﺞ ﻗﻴﻤﺔ‬


ņőƱŀ Ţ
330 ‫ ﺻﻔﺤﺔ‬239 ‫أﻧﻈﺮ اﻟﺘﻤﺮﻳﻦ‬ .𝔖 = cos k𝜃 : ‫ أﺣﺴﺐ اﳌﺠﻤﻮع‬،‫ﺑﺎﺗﺒﺎع ﻧﻔﺲ اﻟﻄﺮﻳﻘﺔ‬ •3
=

ّ 𝜃 = m𝜋 ‫• إذا ﻛﺎن‬1
: ‫ )ﻳﻤﻜﻦ إﺛﺒﺎت ذﻟﻚ ﺑﺎﻟﱰاﺟﻊ( ﻣﻨﻪ‬cos ((2k − 1) m𝜋) = (−1) ‫ﻓﺈن‬ .ّ‫ﺍﳊﻞ‬

S= cos ((2k − 1) m𝜋) = (−1) = n (−1)


= =
ƕ
VII

ّ ‫ ﻋﺪد ًا ﺻﺤﻴﺤ ًﺎ‬k ‫إذا ﻛﺎن‬


: ‫ﻓﺈن‬ (‫)ا‬ •2
2 sin 𝜃 cos (2k − 1) 𝜃 = sin (𝜃 + (2k − 1) 𝜃) + sin (𝜃 − (2k − 1) 𝜃) = sin 2k𝜃 − sin 2 (k − 1) 𝜃
: ‫)ب( ﻟﺪﻳﻨﺎ‬
ũŏ

2S sin 𝜃 = 2 sin 𝜃 cos (2k − 1) 𝜃 = 2 sin 𝜃 cos 2 (k − 1) 𝜃 = (sin 2k𝜃 − sin (2k − 1) 𝜃)
= = =
: 0 gray 

sin
=gray −
2𝜃gray 0𝜃 +
sin (sin sin
4𝜃gray
− 
2𝜃) gray
+ 6𝜃
(sin sin
gray
− 
4𝜃) +⋯
(gray ( ( ( ( (
gray
+ (sin(((
2(n −(
1)𝜃 2(
− sin( (n(−(2)(𝜃) + (sin 2n𝜃gray 2(
− sin( (n(−(1)(𝜃)
= sin 2n𝜃

𝟯𝟮𝟵

http ://tinyurl.com/Malki1718 0
.



⎪n (−1) ‫ ؛‬m ∈ ℤ ‫ = 𝜃 ﻣﻊ‬m𝜋 ‫إذا ﻛﺎن‬
.S = cos (2k − 1) 𝜃 = : ‫)ج( ﺣﺴﺐ ﻣﺎ ﺳﺒﻖ‬

Ŕž Ŧ Ľ


= ⎪ sin 2n𝜃

‫ ≠ 𝜃 ؛‬0 (mod 𝜋) ‫إذا ﻛﺎن‬
⎩ 2 sin 𝜃
⧏ : 53 ‫ﻣﻼﺣﻈﺔ‬

œǃ
: ‫ ﺑﻄﺮﻳﻘﺔ ﻣﺒﺎﴍة‬S ‫ ﻓﻴﻤﻜﻦ إﳚﺎد‬، 𝜃 ≠ 0 (mod 𝜋) ‫إذا ﻛﺎن‬

S= cos (2k − 1) 𝜃 = Re e − = Re e− e = Re e− e
= = = =

Ŀ
e −1 e e − e− 2ı sin n𝜃
= Re e− ⋅ e = Re = Re e
e −1 e − e− 2ı sin 𝜃
sin n𝜃 cos n𝜃 sin 2n𝜃 sin 2n𝜃
= = =
sin 𝜃 sin 𝜃 2 sin 𝜃

.𝔖 = ّ 𝜃 = 0 (mod 𝜋) ‫إذا ﻛﺎن‬


1 = n + 1 : ‫ ﻣﻨﻪ‬cos k𝜃 = 1 ‫ﻓﺈن‬ (‫)ا‬ • 3
=
1
ّ ‫ ﻓﺒﻤﻼﺣﻈﺔ‬، 𝜃 ≠ 0 (mod 𝜋) ‫)ب( إذا ﻛﺎن‬
: ‫ ﻳﻜﻮن‬cos k𝜃 = (1 + cos 2k𝜃) ‫أن‬
2
1 1 1 1 n+1 1
𝔖= + cos 2k𝜃 = + cos 2k𝜃 = + cos 2k𝜃
=
2 2 =
2 2 =
2 2 =

: ‫ﻟﻜﻦ‬
2 sin 𝜃 cos 2k𝜃 = sin (𝜃 + 2k𝜃) + sin (𝜃 − 2k𝜃) = sin (2k + 1) 𝜃 − sin (2k − 1) 𝜃

2 sin 𝜃 cos 2k𝜃 = 2 sin 𝜃 cos 2k𝜃 = (sin (2k + 1) 𝜃 − sin (2k − 1) 𝜃) : ‫إذن‬
= = =
sin (−𝜃)) +
𝜃 3𝜃sin 
𝜃) (( −( ((
(sin
gray (sin gray
ņőƱŀ Ţ
= − gray − + ⋯ + (sin (2n + 1) 𝜃gray (2n
− sin( 1)𝜃)
= sin (2n + 1) 𝜃 + sin 𝜃
sin (2n + 1) 𝜃 + sin 𝜃 sin (2n + 1) 𝜃 1
cos 2k𝜃 = = + : ‫ﻣﻨﻪ‬
=
2 sin 𝜃 2 sin 𝜃 2
n+1 1 sin (2n + 1) 𝜃 1 2n + 3 sin (2n + 1) 𝜃
𝔖= + + = + : ‫ﻣﻨﻪ‬
2 2 2 sin 𝜃 2 4 4 sin 𝜃


ƕ

VII

.
𝟐𝟑𝟗 .
.S = sin kx ‫و‬C = cos kx : ‫ ﻧﻀﻊ‬.‫ ﻋﺪد ﻃﺒﻴﻌﻲ‬n ‫ ﻋﺪد ﺣﻘﻴﻘﻲ و‬x
= =
ũŏ

ّ ‫• أﺛﺒﺖ‬1
. C + S = n + 1 ‫أن‬
.( x ∉ 𝜋ℤ ‫ و‬x ∈ 𝜋ℤ ‫ )ﻣ ﱢﻴﺰ اﳊﺎﻟﺘﲔ‬C − S ‫• أﺣﺴﺐ‬2

329 ‫ ﺻﻔﺤﺔ‬238 ‫أﻧﻈﺮ اﻟﺘﻤﺮﻳﻦ‬  . S ‫ و‬C ‫• إﺳﺘﻨﺘﺞ ﻛﻼ ﻣﻦ‬3

𝟯𝟯𝟬

http ://tinyurl.com/Malki1718 0
‫‪ .VII‬ﲤﺎرﻳﻦ ﻟﻠﺘﻌﻤﻖ‬
‫‪.‬‬
‫‪ •1‬ﻟﺪﻳﻨﺎ )ﻧُﺬﻛﱢﺮ ﺑﺎﳌﺘﻄﺎﺑﻘﺔ ‪: (cos t + sin t = 1‬‬ ‫ﺍﳊﻞّ‪.‬‬

‫‪Ŕž Ŧ Ľ‬‬
‫= ‪C +S‬‬ ‫‪cos kx +‬‬ ‫= ‪sin kx‬‬ ‫= ‪cos kx + sin kx‬‬ ‫‪1=n+1‬‬
‫=‬ ‫=‬ ‫=‬ ‫=‬

‫‪œǃ‬‬
‫‪ •2‬ﻟﺪﻳﻨﺎ )ﻧُﺬﻛﱢﺮ ﺑﺎﳌﺘﻄﺎﺑﻘﺔ ‪: (cos t − sin t = cos 2t‬‬
‫= ‪C −T‬‬ ‫‪cos kx −‬‬ ‫= ‪sin kx‬‬ ‫= ‪cos kx − sin kx‬‬ ‫‪cos 2kx‬‬
‫=‬ ‫=‬ ‫=‬ ‫=‬

‫‪Ŀ‬‬
‫إذا ﻛﺎن ‪ x ∈ 𝜋ℤ‬أي إذا ﻛﺎن )𝜋 ‪ّ x = 0 (mod‬‬
‫ﻓﺈن ‪ sin kx = 0‬ﻣﻨﻪ ‪ S = 0‬إذن = ‪C −S = n+1−0‬‬ ‫•‬

‫= ‪. (C − T‬‬ ‫‪) n + 1‬أو ‪ cos 2kx = 1 :‬ﻣﻨﻪ ‪cos 2kx = n + 1‬‬


‫=‬

‫إذا ﻛﺎن ‪ x ∉ 𝜋ℤ‬أي إذا ﻛﺎن )𝜋 ‪ x ≠ 0 (mod‬ﻓﺤﺴﺐ اﻟﺘﻤﺮﻳﻦ ‪ 238‬ﺻﻔﺤﺔ ‪ّ 329‬‬


‫ﻓﺈن ‪:‬‬ ‫•‬
‫‪sin (2n + 1) x 1‬‬
‫= ‪C −S‬‬ ‫= ‪cos 2kx‬‬ ‫‪+‬‬
‫=‬
‫‪2 sin x‬‬ ‫‪2‬‬

‫‪sin(2n + 1)x 1‬‬


‫= ‪ . C − S‬ﺑﺠﻤﻊ ﻫﺎﺗﲔ اﳌﻌﺎدﻟﺘﲔ ﻃﺮﻓ ًﺎ إﱃ ﻃﺮف ﻧﺠﺪ‬ ‫ﻟﺪﻳﻨﺎ ‪ C + S = n + 1‬و ‪+‬‬ ‫•‬‫‪3‬‬
‫‪2 sin x‬‬ ‫‪2‬‬
‫‪2n + 3 sin (2n + 1) x‬‬ ‫‪sin (2n + 1) x 1‬‬
‫= ‪ C‬؛ و ﺑﻄﺮﺣﻬﲈ ﻃﺮﻓ ًﺎ إﱃ ﻃﺮف ﻧﺠﺪ‬ ‫‪+‬‬ ‫‪ 2C = n + 1 +‬أي‬ ‫‪+‬‬
‫‪4‬‬ ‫‪4 sin x‬‬ ‫‪2 sin x‬‬ ‫‪2‬‬
‫‪2n + 1 sin (2n + 1) x‬‬
‫= ‪.S = n+1−C‬‬ ‫‪−‬‬
‫‪4‬‬ ‫‪4 sin x‬‬
‫■‬

‫‪.‬‬
‫𝟎𝟒𝟐‬ ‫‪.‬‬
‫= ‪.T‬‬ ‫𝜃‪cos 𝜃 sin k‬‬ ‫=‪ S‬و‬ ‫𝜃 ﻋﺪد ﺣﻘﻴﻘﻲ و ‪ n‬ﻋﺪد ﻃﺒﻴﻌﻲ‪ .‬ﻧﻀﻊ ‪cos 𝜃 cos k𝜃 :‬‬
‫=‬ ‫=‬
‫‪. Z = S + ıT‬‬ ‫ﻧﻌﺘﱪ اﻟﻌﺪد اﳌﺮﻛﺐ ‪:‬‬
‫‪ņőƱŀ Ţ‬‬
‫ﻣﻊ 𝜃 ‪. u = e cos‬‬ ‫=‪Z‬‬ ‫‪u‬‬ ‫‪ •1‬أﺛﺒﺖ ّ‬
‫أن‬
‫=‬

‫‪1 e‬‬
‫= ‪.u‬‬ ‫‪+‬‬ ‫‪ •2‬أﺛﺒﺖ ّ‬
‫أن‬
‫‪2‬‬ ‫‪2‬‬
‫‪ •3‬ﻣﺘﻰ ﻳﻜﻮن ‪ u = 1‬؟ أﺣﺴﺐ ‪ S‬و ‪ T‬ﰲ ﻫﺬه اﳊﺎﻟﺔ‪.‬‬
‫𝜃 ‪cos‬‬
‫‪. Z = −ı‬‬ ‫‪e‬‬ ‫‪cos 𝜃 − 1‬‬ ‫‪ •4‬ﻧﻔﺮض ‪ . u ≠ 1‬أﺛﺒﺖ ّ‬
‫أن ‪:‬‬
‫𝜃 ‪sin‬‬
‫‪ƕ‬‬
‫‪VII‬‬

‫‪ •5‬إﺳﺘﻨﺘﺞ ﻛﻼ ﻣﻦ ‪ S‬و ‪. T‬‬

‫𝜃‪sin k‬‬ ‫𝜃‪cos k‬‬


‫= 𝔗‪.‬‬ ‫= 𝔖و‬ ‫‪ •6‬ﺑﺎﺗﺒﺎع ﻧﻔﺲ اﻟﻄﺮﻳﻘﺔ‪ ،‬أﺣﺴﺐ اﳌﺠﻤﻮﻋﲔ‬
‫𝜃 ‪cos‬‬ ‫𝜃 ‪cos‬‬
‫‪ũŏ‬‬

‫=‬ ‫=‬

‫𝟭𝟯𝟯‬

‫‪http ://tinyurl.com/Malki1718‬‬ ‫‪0‬‬


.
: ‫• ﻟﺪﻳﻨﺎ‬1 .ّ‫ﺍﳊﻞ‬

Ŕž Ŧ Ľ
Z= cos 𝜃 cos k𝜃 + ı cos 𝜃 sin k𝜃 = cos 𝜃 cos k𝜃 + ı cos 𝜃 sin k𝜃
= = =

œǃ
= cos 𝜃 (cos k𝜃 + ı sin k𝜃) = cos 𝜃e = e cos 𝜃
= = =

: ‫• ﻟﺪﻳﻨﺎ‬2

Ŀ
1 + cos 2𝜃 sin 2𝜃
u = e cos 𝜃 = (cos 𝜃 + ı sin 𝜃) cos 𝜃 = cos 𝜃 + ı cos 𝜃 sin 𝜃 = +ı
2 2
1 1 1 1
= + (cos 2𝜃 + ı sin 2𝜃) = + e
2 2 2 2
‫ ﻧﺴﺘﻌﻤﻞ ﺻﻴﻐﺔ ﺃﻭﻟﺮ‬: ‫ﻃﺮﻳﻘﺔ أﺧﺮى‬
e + e− e + +e − e +e e +1
u = e cos 𝜃 = e = = =
2 2 2 2

: ‫• ﻟﺪﻳﻨﺎ‬3
1 1
u=1 ⟺ + e =1 ⟺ e = 1 ⟺ 2𝜃 = 0 (mod 2𝜋) ⟺ 𝜃 = 0 (mod 𝜋)
2 2

. T = Im (Z) = 0 ‫ و‬S = Re (Z) = n ‫ إذن‬Z = ّ u = 1 ‫ إذا ﻛﺎن‬: ‫ﻣﻨﻪ‬


1 = n ‫ﻓﺈن‬
=

ّ 𝜃 ≠ 0 (mod 𝜋) ‫ أي إذا ﻛﺎن‬u ≠ 1 ‫• إذا ﻛﺎن‬4


: ‫ﻓﺈن‬
u −1 u + e
Z= u =u⋅ = (u − 1) = e cos 𝜃 −1
=
u−1 u−1 + e −1
e +1 e e + e−
= e cos 𝜃 − 1 = e cos 𝜃 −1
e −1 e e − e−
ņőƱŀ Ţ
2 cos 𝜃 cos 𝜃
= e cos 𝜃 − 1 = −ı e cos 𝜃 − 1
2ı sin 𝜃 sin 𝜃
: ‫• ﻟﺪﻳﻨﺎ‬5
cos 𝜃 cos 𝜃
Z = −ı e cos 𝜃 − 1 = −ı (−1 + cos 𝜃 cos n𝜃 + ı cos 𝜃 sin n𝜃)
sin 𝜃 sin 𝜃
cos 𝜃 cos 𝜃
= cos 𝜃 sin n𝜃 + ı cos 𝜃 (1 − cos n𝜃)
sin 𝜃 sin 𝜃
cos 𝜃 cos + 𝜃 sin n𝜃
S = Re (Z) = cos 𝜃 sin n𝜃 = : ‫ﻣﻨﻪ‬
sin 𝜃 sin 𝜃
ƕ

VII

cos 𝜃 (1 − cos n𝜃) cos + 𝜃


T = Im (Z) = cos 𝜃 (1 − cos n𝜃) = ‫و‬
sin 𝜃 sin 𝜃
𝜋
. 𝜃 ≠ (mod 𝜋) ‫ ﳚﺐ أن ﻳﻜﻮن‬،‫ﻣﻌﺮﻓﺘﺎن‬ ّ ‫ﺣﺘﻰ ﺗﻜﻮن اﻟﻌﺒﺎرﺗﲔ‬ • 6
ũŏ

2
: ‫ ﻣﻨﻪ‬ℨ = 𝔖 + ı𝔗 ‫ﻧﻀﻊ‬
cos k𝜃 + ı sin k𝜃 e
ℨ= = = u
=
cos 𝜃 =
cos 𝜃 =

e cos 𝜃 + ı sin 𝜃
: ‫ ﻟﺪﻳﻨﺎ‬. u =
= = 1 + ı tan 𝜃 ‫ﻣﻊ‬
cos 𝜃 cos 𝜃
u=1 ⟺ e = cos 𝜃 ⟺ sin 𝜃 = 0 ⟺ 𝜃 = 0 (mod 𝜋)

𝟯𝟯𝟮

http ://tinyurl.com/Malki1718 0
‫ ﲤﺎرﻳﻦ ﻟﻠﺘﻌﻤﻖ‬.VII
.
‫ = 𝔖 و‬Re (ℨ) = n + 1 ‫ ﻣﻨﻪ‬ℨ = ّ 𝜃 = 0 (mod 𝜋) ‫ أي إذا ﻛﺎن‬u = 1 ‫إذا ﻛﺎن‬
1 = n + 1 ‫ﻓﺈن‬ •
=

Ŕž Ŧ Ľ
. 𝔗 = Im (ℨ) = 0
ّ 𝜃 ≠ 0 (mod 𝜋) ‫ أي إذا ﻛﺎن‬u ≠ 1 ‫إذا ﻛﺎن‬
: ‫ﻓﺈن‬ •

œǃ
+ +
u −1 1 e
ℨ= u = = −1
=
u−1 (1 + ı tan 𝜃) − 1 cos 𝜃
e + cos 𝜃 cos(n + 1)𝜃 − cos + 𝜃 + ı sin (n + 1) 𝜃

Ŀ
cos 𝜃
= −ı − 1 = −ı
cos + 𝜃
sin 𝜃 sin 𝜃 cos + 𝜃
sin (n + 1) 𝜃 cos + 𝜃 − cos (n + 1) 𝜃
= +ı
sin 𝜃 cos 𝜃 sin 𝜃 cos 𝜃
sin (n + 1) 𝜃
𝔖 = Re (ℨ) = : ‫ﻣﻨﻪ‬
sin 𝜃 cos 𝜃
cos + 𝜃 − cos (n + 1) 𝜃
𝔗 = Im (ℨ) = ‫و‬
sin 𝜃 cos 𝜃

.
𝟐𝟒𝟏 .
. S= e : ‫ ﻧﻀﻊ‬.‫ ﻋﺪد ﻃﺒﻴﻌﻲ‬n ‫𝜃 ﻋﺪد ﺣﻘﻴﻘﻲ و‬
=−

ّ ‫• أﺛﺒﺖ‬1
. S ∈ ℝ : ‫أن‬
. 𝜃 = 0 (mod 2𝜋) ‫ إذا ﻛﺎن‬S ‫• أﺣﺴﺐ‬2
e + −1
ّ ‫ إﺳﺘﻨﺘﺞ ﻋﺒﺎرة ُﻣ‬. S = e−
. 𝜃 ‫ و‬n ‫ ﺑﺪﻻﻟﺔ‬S ِ‫ﺒﺴﻄﺔ ﻟـ‬
e −1
ّ ‫ أﺛﺒﺖ‬. 𝜃 ≠ 0 (mod 2𝜋) ‫• ﻧﻔﺮض‬3
‫أن‬
ņőƱŀ Ţ
: ‫• ﻟﺪﻳﻨﺎ‬1 .ّ‫ﺍﳊﻞ‬

S= e = e = e− = e =S
=− =− =− =

. S ∈ ℝ ‫إذن‬

‫ ﻧﺴﺘﻌﻤﻞ ﺻﻴﻐﺔ ﺃﻭﻟﺮ‬: ‫ﻃﺮﻳﻘﺔ أﺧﺮى‬


ƕ


VII

S= e = e +e + e
=− =− =

= e− +1+ e (k = −ℓ ‫)ﺑﺎﻟﺘﻌﻮﻳﺾ‬
ũŏ

= =

=1+ e− +e (k ‫أﺻﻢ إذن ﻳﻤﻜﻦ ﺗﺴﻤﻴﺘﻪ‬


ّ ℓ ‫)اﳌﺘﻐﲑ‬
ّ
=

=1+2 cos k𝜃 ∈ ℝ
=

𝟯𝟯𝟯

http ://tinyurl.com/Malki1718 0
.
.S = 1 = 2n + 1 : ‫ ﻣﻨﻪ‬e ّ 𝜃 = 0 (mod 2𝜋) ‫إذا ﻛﺎن‬
= 1 ‫ﻓﺈن‬ • 2
=−

Ŕž Ŧ Ľ
ّ 𝜃 ≠ 0 (mod 2𝜋) ‫• إذا ﻛﺎن‬3
: ‫ﻓﺈن‬

œǃ
= +
S= e = e− e + = e− e + = e− e
=− =− =− =
+ +
e +
−1 e 2ı sin 𝜃
e− e− = e−

Ŀ
= e =
=
e −1 e 2ı sin
sin n + 𝜃
=
sin

: ‫ و ﺑﺎﻟﺘﺎﱄ‬S = 1 + 2 ّ ‫اﻷول‬
cos k𝜃 ‫أن‬ ّ ‫ﻳﻤﻜﻦ‬
ّ ‫ وﺟﺪﻧﺎ ﰲ اﻟﺴﺆال‬: ‫ﺣﻞ اﻟﺘﻤﺮﻳﻦ ﺑﻄﺮﻳﻘﺔ أﺧﺮى‬ ⧏ : 54 ‫ﻣﻼﺣﻈﺔ‬
=

ّ 𝜃 = 0 (mod 2𝜋) ‫إذا ﻛﺎن‬


‫ ؛‬S = 1 + 2n cos k𝜃 = 1 ‫ﻓﺈن‬ •

ّ 𝜃 ≠ 0 (mod 2𝜋) ‫و إذا ﻛﺎن‬


: ‫ﻓﺈن‬ •

S=1+2 cos k𝜃 = 1 + 2 Re e = 1 + 2 Re e
= = =

e / e / − e− /
e −1
= 1 + 2 Re e ⋅ = 1 + 2 Re e ⋅ / /
e − e e − e− /

2ıe / sin sin


+
= 1 + 2 Re e ⋅ = 1 + 2 Re e
2ıe / sin sin
sin (n + 1)𝜃
=1+2 cos
sin 2
ņőƱŀ Ţ
: 5 ‫ﻟﻜﻦ‬
n𝜃 (n + 1)𝜃 n𝜃 (n + 1)𝜃 n𝜃 (n + 1)𝜃
2 sin cos = sin + + sin −
2 2 2 2 2 2
(2n + 1)𝜃 𝜃 1 𝜃
= sin + sin − = sin n + 𝜃 − sin
2 2 2 2
: ‫ﻣﻨﻪ‬
sin n + 𝜃 − sin sin n + 𝜃 sin
ƕ

S=1+ =1+ −
VII

sin sin sin


sin n + 𝜃 sin n + 𝜃
=1+ −1=
sin sin
ũŏ

.‫و ﻫﻲ ﻧﻔﺲ اﻟﻨﺘﻴﺠﺔ‬


. sin cos = sin + + − ّ ‫ﻧُﺬﻛّﺮ‬5


: ‫أن‬

𝟯𝟯𝟰

http ://tinyurl.com/Malki1718 0
‫ ﲤﺎرﻳﻦ ﻟﻠﺘﻌﻤﻖ‬.VII
.

.
𝟐𝟒𝟐 .

Ŕž Ŧ Ľ
: ‫ أﺣﺴﺐ‬. n ≥ 1 ‫ ﻋﺪد ﻃﺒﻴﻌﻲ ﺑﺤﻴﺚ‬n ‫ 𝜌 أﻋﺪاد ﺣﻘﻴﻘﻴﺔ و‬، 𝛽 ، 𝛼
− −

œǃ
B= 𝜌 sin (𝛼 + k𝛽) • A= 𝜌 cos (𝛼 + k𝛽) •
= =

Ŀ
ّ 𝛽 = 0 (mod 2𝜋) ‫ = 𝜌 و‬1 ‫𝜌 أي إذا ﻛﺎن‬e = 1 ‫إذا ﻛﺎن‬
: ‫ﻓﺈن‬ • .ّ‫ﺍﳊﻞ‬

A= cos 𝛼 = n cos 𝛼
=

B= sin 𝛼 = n sin 𝛼
=

: ‫ ﻳﻜﻮن‬C = A + ıB ‫ ≠ 𝛽 ﻓﺒﻮﺿﻊ‬0 (mod 2𝜋) ‫ ≠ 𝜌 أو‬1 ‫𝜌 أي إذا ﻛﺎن‬e ≠ 1 ‫إذا ﻛﺎن‬ •

− −
C= 𝜌 (cos (𝛼 + k𝛽) + ı sin (𝛼 + k𝛽)) = 𝜌 e +

= =

1−𝜌 e
=e 𝜌e =e
=
1 − 𝜌e
1−𝜌 e (1 − 𝜌 cos 𝛽 + ı𝜌 sin 𝛽)
=e
(1 − 𝜌 cos 𝛽) + 𝜌 sin 𝛽
1−𝜌 e 1 − 𝜌e−
=e
1 − 2𝜌 cos 𝛽 + 𝜌 cos 𝛽 + 𝜌 sin 𝛽
1 − 𝜌e− − 𝜌 e +𝜌 + e −
ņőƱŀ Ţ
=e
1 + 𝜌 − 2𝜌 cos 𝛽
e − 𝜌e − −𝜌 e + +𝜌 + e + −
=
1 + 𝜌 − 2𝜌 cos 𝛽

: ‫و ﺑﺄﺧﺬ اﳉﺰء اﳊﻘﻴﻘﻲ و اﳉﺰء اﻟﺘﺨﻴﲇ ﻳﻨﺘﺞ‬


cos 𝛼 − 𝜌 cos (𝛼 − 𝛽) − 𝜌 cos (𝛼 + n𝛽) + 𝜌 + cos (𝛼 + (n − 1)𝛽)
A=
1 + 𝜌 − 2𝜌 cos 𝛽
‫و‬
ƕ

sin 𝛼 − 𝜌 sin (𝛼 − 𝛽) − 𝜌 sin (𝛼 + n𝛽) + 𝜌 + sin (𝛼 + (n − 1)𝛽)


VII

B=
1 + 𝜌 − 2𝜌 cos 𝛽

ũŏ

.
𝟐𝟒𝟑 .
.B = 𝜔 +𝜔 +𝜔 ‫ و‬A=𝜔+𝜔 +𝜔 ‫ أﺣﺴﺐ اﻟﻌﺪدﻳﻦ‬. 𝜔 = e / ‫ﻟﻴﻜﻦ‬

. (33 ‫ ﺻﻔﺤﺔ‬14 ‫)أﻧﻈﺮ اﳌﱪﻫﻨﺔ‬ 𝜔 = 0 ‫ = 𝜔 ﻣﻨﻪ‬e / =e ّ ‫𝜔 ﺟﺬر ﺳﺎﺑﻊ ﻟﻠﻮﺣﺪة‬


= 1 ‫ﻷن‬ .ّ‫ﺍﳊﻞ‬
=

𝟯𝟯𝟱

http ://tinyurl.com/Malki1718 0
‫‪.‬‬
‫ﻟﺪﻳﻨﺎ ﻣﻦ ﺟﻬﺔ ‪. A + B = 𝜔 + 𝜔 + 𝜔 + 𝜔 + 𝜔 + 𝜔 = −1 :‬‬ ‫•‬

‫‪Ŕž Ŧ Ľ‬‬
‫و ﻣﻦ ﺟﻬﺔ أﺧﺮى ‪:‬‬ ‫•‬

‫𝜔‪A⋅B= 𝜔+𝜔 +‬‬ ‫𝜔‪𝜔 +𝜔 +‬‬

‫‪œǃ‬‬
‫𝜔‪=𝜔 +𝜔 +𝜔 +𝜔 +𝜔 +𝜔 +𝜔 +𝜔 +‬‬

‫𝜔 ﻣﻨﻪ ‪:‬‬ ‫𝜔= 𝜔⋅ 𝜔=‬ ‫𝜔= 𝜔⋅ 𝜔= 𝜔 و‬ ‫ﻟﻜﻦ ‪، 𝜔 = 𝜔 ⋅ 𝜔 = 𝜔 ، 𝜔 = 1 :‬‬

‫‪Ŀ‬‬
‫𝜔‪A⋅B=𝜔 +𝜔 +1+𝜔 +1+𝜔+1+𝜔 +‬‬
‫𝜔‪=2+ 1+𝜔+𝜔 +𝜔 +𝜔 +𝜔 +‬‬ ‫‪=2+0=2‬‬

‫إذن ‪ A‬و ‪ B‬ﳘﺎ ﺣ ﱠ‬


‫ﻼ اﳌﻌﺎدﻟﺔ ‪ Z − (−1)Z + 2 = 0‬أي ‪. Z + Z + 2 = 0‬‬
‫‪ Δ = (1) − 4 × 1 × 2 = −7 = ı√7‬ﻣﻨﻪ اﳊﻼّن ‪:‬‬ ‫ﳑ ّﻴﺰ ﻫﺬه اﻷﺧﲑة ﻫﻮ‬
‫‪−1 + ı√7‬‬ ‫‪−1 − ı√7‬‬
‫= ‪.Z = Z‬‬ ‫و‬ ‫= ‪Z‬‬
‫‪2‬‬ ‫‪2‬‬
‫ﻟﻜﻦ ‪:‬‬
‫𝜋‪2‬‬ ‫𝜋‪4‬‬ ‫𝜋‪8‬‬
‫‪Im (A) = sin‬‬ ‫‪+ sin‬‬ ‫‪+ sin‬‬ ‫‪≥0‬‬
‫‪7‬‬ ‫‪7‬‬ ‫‪7‬‬
‫𝜋‪4‬‬ ‫𝜋‪8‬‬ ‫𝜋‪2‬‬
‫‪. sin‬‬ ‫‪+ sin‬‬ ‫< ‪ (0‬و ‪≥ 0‬‬ ‫ﻷن ‪ّ sin 2𝜋 ≥ 0‬‬
‫)ﻷن 𝜋 <‬ ‫ﻣﻮﺟﺐ ّ‬
‫‪7‬‬ ‫‪7‬‬ ‫‪7‬‬ ‫‪7‬‬
‫𝜋‪4‬‬
‫= ‪ x‬ﻣﻨﻪ ‪ sin x + sin (2x) = sin x (1 + 2 cos x) ≥ 0‬و ﻫﻮ ﻋﺪد ﻣﻮﺟﺐ ﻟﻜﻮن ‪sin x ≥ 0‬‬ ‫ﻹﺛﺒﺎت ﻫﺬا اﻷﺧﲑ‪ ،‬ﻧﻀﻊ‬
‫‪7‬‬
‫𝜋‪2‬‬ ‫𝜋‪2‬‬
‫‪. ( 0,‬‬ ‫)ﻷن < ‪ 0 < x‬و داﻟﺔ ﺟﻴﺐ اﻟﺘﲈم ‪ cos‬ﻣﺘﻨﺎﻗﺼﺔ ﻋﲆ‬ ‫)ﻷن 𝜋 < ‪ (0 < x‬و ‪ّ 1 + 2 cos x ≥ 0‬‬
‫ّ‬
‫‪3‬‬ ‫‪3‬‬
‫‪−1 − ı√7‬‬ ‫‪−1 + ı√7‬‬
‫= ‪.B = Z‬‬ ‫= ‪ A=Z‬و‬ ‫ﰲ اﻷﺧﲑ ‪ Im (A) ≥ 0 :‬إذن‬
‫‪2‬‬ ‫‪2‬‬
‫‪1‬‬ ‫𝜔‬ ‫‪1‬‬
‫= 𝜔 و ﺑﺎﻟﺘﺎﱄ ‪:‬‬ ‫=‬ ‫ﻓﺈن = 𝜔 ﻣﻨﻪ ‪= 𝜔 −‬‬ ‫ﻣﻼﺣﻈﺔ ‪ ⧏ : 55‬ﺑﲈ ّ‬
‫أن ‪ّ |𝜔| = 1‬‬
‫𝜔‬ ‫𝜔‬ ‫𝜔‬
‫‪ņőƱŀ Ţ‬‬
‫‪B=𝜔 +𝜔 +𝜔 =𝜔 − +𝜔 − +𝜔 − =𝜔 +𝜔 +𝜔=A‬‬

‫⧐‬
‫■‬

‫‪) cos‬ﺑﺎﺳﺘﻌﲈل اﳉﺬور(‪.‬‬


‫𝜋‬
‫‪ cos‬و‬
‫𝜋‬
‫ﻫﺪﻓﻨﺎ ﰲ ﻫﺬا اﻟﺘﻤﺮﻳﻦ ﻫﻮ ﺣﺴﺎب ﻛﻼ ﻣﻦ‬
‫‪$‬‬ ‫‪.‬‬
‫𝟒𝟒𝟐‬ ‫‪.‬‬
‫‪17‬‬ ‫‪5‬‬
‫‪ƕ‬‬

‫اﳉﺰء اﻷول‬
‫‪VII‬‬

‫‪ •1‬أﻛﺘﺐ‪ ،‬ﻋﲆ اﻟﺸﻜﻞ اﳌﺜﻠﺜﻲ‪ ،‬ﺣﻠﻮل اﳌﻌﺎدﻟﺔ ‪:‬‬


‫‪z −1=0‬‬ ‫)𝟔(‬
‫‪ũŏ‬‬

‫‪ •2‬ﻟﻴﻜﻦ ‪ Q‬ﻛﺜﲑ اﳊﺪود اﳌﻌﺮف ﺑﺎﻟﻌﺒﺎرة ‪. z − 1 = (z − 1) Q (z) :‬‬


‫‪1‬‬
‫ﺛﻢ اﺳﺘﻨﺘﺞ ﻋﺒﺎرة ﺟﺬور ﻛﺜﲑ اﳊﺪود ‪. Q‬‬
‫‪ Q (z) = z‬ﺣﻴﺚ ‪ّ ، t = z +‬‬ ‫‪t +t−1‬‬ ‫أﺛﺒﺖ ّ‬
‫أن‬
‫‪z‬‬
‫𝜋‬ ‫𝜋‪4‬‬ ‫𝜋‪2‬‬ ‫𝜋‬ ‫𝜋‪4‬‬ ‫𝜋‪2‬‬
‫‪ sin‬و ‪. sin‬‬ ‫‪، sin ، cos ، cos ، cos‬‬ ‫‪ •3‬إﺳﺘﻨﺘﺞ ﻗﻴﻤﺔ ﻛﻼ ﻣﻦ ‪:‬‬
‫‪5‬‬ ‫‪5‬‬ ‫‪5‬‬ ‫‪5‬‬ ‫‪5‬‬ ‫‪5‬‬

‫𝟲𝟯𝟯‬

‫‪http ://tinyurl.com/Malki1718‬‬ ‫‪0‬‬


‫‪ .VII‬ﲤﺎرﻳﻦ ﻟﻠﺘﻌﻤﻖ‬
‫‪.‬‬

‫أﻧﻈﺮ اﻟﺘﻤﺮﻳﻦ �� ﺻﻔﺤﺔ ��‬ ‫‬

‫‪Ŕž Ŧ Ľ‬‬
‫اﳉﺰء اﻟﺜﺎﲏ‬

‫‪œǃ‬‬
‫𝜋‬
‫= 𝜃‪.‬‬ ‫ﻧﻀﻊ ﻓﻴﻤﺎ ﻳﻠﻲ‬
‫‪17‬‬
‫ﳚﺐ ﺇﳚﺎﺩ ﺍﻟﻘ‪‬ﻴ‪‬ﻢ ﺍﻟﺪﻗﻴﻘﺔ )ﺍﳌﻀﺒﻮﻃﺔ( ﻟﻠﺤﺴﺎﺑﺎﺕ ﺍﳌﻄﻠﻮﺑﺔ ﻭ ﻻ ﻳ‪‬ﺴﻤﺢ ﺑﺈﻋﻄﺎء ﻗ‪‬ﻴ‪‬ﻢ ﺗﻘﺮﻳﺒﻴﺔ ﳍﺎ ﺑﺎﺳﺘﻌﻤﺎﻝ ﺍﻵﻟﺔ ﺍﳊﺎﺳﺒﺔ‪.‬‬

‫‪Ŀ‬‬
‫‪ •1‬أﺛﺒﺖ أﻧّﻪ ‪:‬‬
‫‪nh‬‬ ‫‪h‬‬
‫‪−‬‬ ‫‪sin‬‬ ‫)‪cos a + (n − 1‬‬
‫∗‬ ‫‪2‬‬ ‫‪2‬‬
‫‪∀a ∈ ℝ , ∀h ∈]0, 2𝜋[ , ∀n ∈ ℕ :‬‬ ‫= )‪cos (a + kh‬‬
‫‪h‬‬
‫=‬ ‫‪sin‬‬
‫‪2‬‬
‫𝜃‪x = cos 3𝜃 + cos 5𝜃 + cos 7𝜃 + cos 11‬‬
‫‪.‬‬ ‫‪ •2‬ﻧﻀﻊ ‪:‬‬
‫𝜃‪x = cos 𝜃 + cos 9𝜃 + cos 13𝜃 + cos 15‬‬
‫𝜋‬
‫= 𝜃(‪.‬‬ ‫ﺗﻨﺲ ّ‬
‫أن‬ ‫أن ‪ ) x > 0 :‬ﻻ َ‬‫)ا( أﺛﺒﺖ ّ‬
‫‪17‬‬
‫‪1‬‬
‫أن ‪. x + x = :‬‬ ‫)ب( أﺛﺒﺖ ّ‬
‫‪2‬‬
‫)ج( ﻧﺒﺤﺚ ﻓﻴﲈ ﻳﲇ ﻋﻦ ﻗﻴﻤﺔ ‪: x x‬‬
‫)‪ (i‬أﻧﴩ اﻟﻌﺒﺎرة ‪ x x‬ﺛﻢ أﻛﺘﺐ ﻋﲆ ﺷﻜﻠﻬﺎ اﳋﻄﻲ اﳉﺪاءات ا ُﳌﺘ ﱠ‬
‫َﺤﺼﻞ ﻋﻠﻴﻬﺎ‪.‬‬
‫)‪ (ii‬إﺳﺘﻨﺘﺞ ّ‬
‫أن ‪. x x = −2 (x + x ) = −1 :‬‬
‫)د( إﺳﺘﻨﺘﺞ ﻗﻴﻤﺔ ﻛ ﱠ‬
‫ﻼ ﻣﻦ ‪ x‬و ‪. x‬‬
‫‪y = cos 3𝜃 + cos 5𝜃 ,‬‬ ‫𝜃‪y = cos 7𝜃 + cos 11‬‬
‫‪ •3‬ﻧﻀﻊ ‪:‬‬
‫‪y = cos 𝜃 + cos 13𝜃 ,‬‬ ‫𝜃‪y = cos 9𝜃 + cos 15‬‬
‫‪ņőƱŀ Ţ‬‬
‫)ا( ﺑﺎﺗﺒﺎع اﻟﻄﺮﻳﻘﺔ اﻟﺴﺎﺑﻘﺔ‪ ،‬أﺣﺴﺐ ﻗﻴﻤﺔ ‪ y y‬و ‪. y y‬‬
‫)ب( إﺳﺘﻨﺘﺞ ﻗﻴﻤﺔ ﻛﻼ ﻣﻦ ‪ y ، y ، y‬و ‪. y‬‬
‫𝜋‬ ‫ِ‬
‫‪) cos‬ﺑﺎﺳﺘﻌﲈل اﳉﺬور(‪.‬‬
‫‪17‬‬
‫أﻋﻂ ﻗﻴﻤﺔ‬ ‫•‬ ‫‪4‬‬
‫أﻧﻈﺮ اﻟﺘﻤﺮﻳﻦ ‪ 245‬ﺻﻔﺤﺔ ‪342‬‬ ‫‬
‫ﺍﳊﻞّ‪.‬‬
‫‪ƕ‬‬
‫‪VII‬‬

‫اﳉﺰء اﻷول‬

‫‪ z = e‬ﻣﻊ ‪ 0 ≤ k ≤ 4‬أي اﻷﻋﺪاد‬ ‫‪/‬‬ ‫‪ •1‬ﺟﺬور اﳌﻌﺎدﻟﺔ )𝟔( ﻫﻲ اﳉﺬور اﳋﺎﻣﺴﺔ ﻟﻠﻮﺣﺪة أي اﻷﻋﺪاد‬
‫‪ũŏ‬‬

‫‪ z = 1 = cos 0 + ı sin 0‬و ‪:‬‬


‫𝜋‪2‬‬ ‫𝜋‪2‬‬ ‫𝜋‪4‬‬ ‫𝜋‪4‬‬ ‫𝜋‪6‬‬ ‫𝜋‪6‬‬ ‫𝜋‪8‬‬ ‫𝜋‪8‬‬
‫‪z = cos‬‬ ‫‪+ ı sin‬‬ ‫‪; z = cos‬‬ ‫‪+ ı sin‬‬ ‫‪; z = cos‬‬ ‫‪+ ı sin‬‬ ‫‪; z = cos‬‬ ‫‪+ ı sin‬‬
‫‪5‬‬ ‫‪5‬‬ ‫‪5‬‬ ‫‪5‬‬ ‫‪5‬‬ ‫‪5‬‬ ‫‪5‬‬ ‫‪5‬‬

‫ﻼ ‪ :‬ﺑﲈ ّ‬
‫أن‬ ‫ﻓﺈن ﺟﺬوره ﻣﱰاﻓﻘﺔ ﻣﺜﻨﻰ ﻣﺜﻨﻰ ‪ :‬ﻧﺘﺤﻘﻖ ﺑﺴﻬﻮﻟﺔ ّ‬
‫أن ‪ z = z‬و ‪ ) z = z‬ﻣﺜ ً‬ ‫أن ﻣﻌﺎﻣﻼت ‪ Q‬ﺣﻘﻴﻘﻴﺔ ّ‬
‫و ﺑﲈ ّ‬
‫‪1‬‬
‫ﻓﺈن ‪. ( z = = e− / = e − / = z − :‬‬ ‫‪ّ ||z || = 1‬‬
‫‪z‬‬

‫𝟳𝟯𝟯‬

‫‪http ://tinyurl.com/Malki1718‬‬ ‫‪0‬‬


‫‪.‬‬
‫أن ‪ z = 0‬ﻟﻴﺲ ﺟﺬر ًا ﻟـِ ‪ّ Q‬‬
‫ﻓﺈن ‪:‬‬ ‫‪ •2‬ﺑﻘﺴﻤﺔ ‪ z − 1‬ﻋﲆ ‪ z − 1‬ﻧﺠﺪ ‪ . Q (z) = 1 + z + z + z + z‬و ﺑﲈ ّ‬

‫‪Ŕž Ŧ Ľ‬‬
‫‪1‬‬ ‫‪1‬‬ ‫‪1‬‬ ‫‪1‬‬
‫‪Q (z) = z‬‬ ‫‪+ +1+z+z‬‬ ‫‪=z‬‬ ‫‪z +‬‬ ‫‪+1+z+‬‬
‫‪z‬‬ ‫‪z‬‬ ‫‪z‬‬ ‫‪z‬‬

‫‪œǃ‬‬
‫‪1‬‬
‫‪=z‬‬ ‫‪z+‬‬ ‫‪−2+1+t‬‬ ‫‪=z‬‬ ‫‪t +t−1‬‬
‫‪z‬‬

‫ﻣﻨﻪ ‪:‬‬

‫‪Ŀ‬‬
‫‪−1 ± √5‬‬
‫= ‪Q (z) = 0 ⟺ t + t − 1 = 0 ⟺ t‬‬
‫‪2‬‬
‫‪ •3‬ﻟﺪﻳﻨﺎ ‪:‬‬
‫‪−1 − √5‬‬
‫=‪ّ t‬‬
‫ﻓﺈن ‪:‬‬ ‫• إذا ﻛﺎن‬
‫‪2‬‬
‫‪1 −1 − √5‬‬ ‫‪−1 − √5 − ı 10 − 2√5‬‬ ‫‪−1 − √5 + ı 10 − 2√5‬‬
‫= ‪z+‬‬ ‫=‪⟺ z‬‬ ‫أو‬ ‫=‪z‬‬
‫‪z‬‬ ‫‪2‬‬ ‫‪4‬‬ ‫‪4‬‬
‫‪−1 + √5‬‬
‫=‪ّ t‬‬
‫ﻓﺈن ‪:‬‬ ‫• و إذا ﻛﺎن‬
‫‪2‬‬
‫‪1 −1 + √5‬‬ ‫‪−1 + √5 − ı 10 + 2√5‬‬ ‫‪−1 + √5 + ı 10 + 2√5‬‬
‫= ‪z+‬‬ ‫=‪⟺ z‬‬ ‫أو‬ ‫=‪z‬‬
‫‪z‬‬ ‫‪2‬‬ ‫‪4‬‬ ‫‪4‬‬
‫إذن ﺟﺬور )‪ Q (z‬ﻫﻲ اﻷﻋﺪاد ‪:‬‬

‫‪−1 − √5 − ı 10 − 2√5‬‬ ‫‪−1 − √5 + ı 10 − 2√5‬‬


‫;‬ ‫;‬
‫‪4‬‬ ‫‪4‬‬
‫‪−1 + √5 − ı 10 + 2√5‬‬ ‫‪−1 + √5 + ı 10 + 2√5‬‬
‫;‬
‫‪4‬‬ ‫‪4‬‬
‫𝜋‪2‬‬
‫‪ cos‬و‬ ‫أن 𝜋 < 𝜋‪ّ 0 < 2𝜋 < 𝜋 < 4‬‬
‫ﻓﺈن ‪> 0‬‬ ‫ﻟﻜﻦ داﻟﺔ ﺟﻴﺐ اﻟﺘﲈم ‪ cos‬ﻣﺘﻨﺎﻗﺼﺔ ﲤﺎﻣ ًﺎ ﻋﲆ اﳌﺠﺎل ]𝜋 ‪ [0,‬و ﺑﲈ ّ‬
‫‪5‬‬ ‫‪5‬‬ ‫‪2‬‬ ‫‪5‬‬
‫‪ņőƱŀ Ţ‬‬
‫‪4𝜋 −1 − √5‬‬ ‫‪2𝜋 −1 + √5‬‬ ‫𝜋‪4‬‬
‫‪cos‬‬ ‫=‬ ‫= ‪ cos‬و‬ ‫‪ cos < 0‬إذن‬
‫‪5‬‬ ‫‪4‬‬ ‫‪5‬‬ ‫‪4‬‬ ‫‪5‬‬
‫𝜋 𝜋‪𝜋 2‬‬
‫<‪ّ 0‬‬
‫ﻓﺈن ‪:‬‬ ‫<‬ ‫أن <‬‫ﺑﺎﳌﺜﻞ‪ ،‬داﻟﺔ اﳉﻴﺐ ‪ sin‬ﻣﺘﺰاﻳﺪة ﲤﺎﻣ ًﺎ ﻋﲆ 𝜋 ‪ 0,‬و ﺑﲈ ّ‬
‫‪5‬‬ ‫‪5‬‬ ‫‪2‬‬ ‫‪2‬‬
‫𝜋‪4‬‬ ‫𝜋‬ ‫𝜋‬ ‫𝜋‪2‬‬
‫‪sin‬‬ ‫‪= sin 𝜋 −‬‬ ‫‪= sin < sin‬‬
‫‪5‬‬ ‫‪5‬‬ ‫‪5‬‬ ‫‪5‬‬

‫𝜋‬ ‫𝜋‪4‬‬ ‫‪10 − 2√5‬‬ ‫𝜋‪4‬‬ ‫‪10 − 2√5‬‬ ‫𝜋‪2‬‬ ‫‪10 + 2√5‬‬
‫‪. sin‬‬ ‫‪= sin‬‬ ‫=‬ ‫‪ sin‬ﻣﻨﻪ‬ ‫=‬ ‫‪ sin‬و‬ ‫=‬ ‫ﻣﻨﻪ‬
‫‪5‬‬ ‫‪5‬‬ ‫‪4‬‬ ‫‪5‬‬ ‫‪4‬‬ ‫‪5‬‬ ‫‪4‬‬
‫‪ƕ‬‬

‫‪VII‬‬

‫ﰲ اﻷﺧﲑ ‪:‬‬

‫‪−1 + √5‬‬ ‫‪10 + 2√5‬‬ ‫‪−1 − √5‬‬ ‫‪10 − 2√5‬‬


‫= ‪z‬‬ ‫‪+ı‬‬ ‫;‬ ‫= ‪z‬‬ ‫‪+ı‬‬
‫‪4‬‬ ‫‪4‬‬ ‫‪4‬‬ ‫‪4‬‬
‫‪ũŏ‬‬

‫‪−1 − √5‬‬ ‫‪10 − 2√5‬‬ ‫‪−1 + √5‬‬ ‫‪10 + 2√5‬‬


‫= ‪z =z‬‬ ‫‪−ı‬‬ ‫;‬ ‫= ‪z =z‬‬ ‫‪−ı‬‬
‫‪4‬‬ ‫‪4‬‬ ‫‪4‬‬ ‫‪4‬‬

‫𝜋‬ ‫‪−1 − √5 1 + √5‬‬ ‫𝜋‪4‬‬ ‫𝜋‬ ‫𝜋‬


‫‪. cos‬‬ ‫‪=−‬‬ ‫=‬ ‫‪ cos‬إذن‬ ‫‪= cos 𝜋 −‬‬ ‫‪= − cos‬‬ ‫ﺑﺎﻹﺿﺎﻓﺔ إﱃ ذﻟﻚ ‪:‬‬
‫‪5‬‬ ‫‪4‬‬ ‫‪4‬‬ ‫‪5‬‬ ‫‪5‬‬ ‫‪5‬‬

‫اﳉﺰء اﻟﺜﺎﲏ‬

‫𝟴𝟯𝟯‬

‫‪http ://tinyurl.com/Malki1718‬‬ ‫‪0‬‬


‫ ﲤﺎرﻳﻦ ﻟﻠﺘﻌﻤﻖ‬.VII
.
: ‫ ﻟﺪﻳﻨﺎ‬. n ∈ ℕ∗ ‫ و‬h ∈ ]0, 2𝜋[ ، a ∈ ℝ ‫• ﻟﻴﻜﻦ‬1
− − −

Ŕž Ŧ Ľ
cos (a + kh) = Re e + = Re e +

= = =

ّ h ∈ ]0, 2𝜋[ ‫أن‬


: ‫ ﻣﻨﻪ‬e ≠ 1 ‫ﻓﺈن‬ ّ ‫و ﺑﲈ‬

œǃ
− −
+
e −1 e −1
e =e e =e ⋅ =e ⋅
= =
e e −1

Ŀ
e e − e− e × 2ı sin
=e ⋅ =e ⋅
e e − e− 2ı sin

+
sin sin nh nh
=e = cos a + + ı sin a +
sin sin 2 2

.‫ﺑﺄﺧﺬ اﳉﺰء اﳊﻘﻴﻘﻲ ﻟﻠﻌﺒﺎرة اﻷﺧﲑة ﻧﺤﺼﻞ ﻋﲆ اﳌﻄﻠﻮب‬


: ‫• ﻟﺪﻳﻨﺎ‬2
3𝜃 + 11𝜃 3𝜃 − 11𝜃 5𝜃 + 7𝜃 5𝜃 − 7𝜃
x = (cos 3𝜃 + cos 11𝜃) + (cos 5𝜃 + cos 7𝜃) = 2 cos cos + 2 cos cos
2 2 2 2
= 2 cos 7𝜃 cos 4𝜃 + 2 cos 6𝜃 cos 𝜃
𝜋
. x > 0 ‫ إذن‬k ∈ {1, 4, 6, 7} ‫ ﻷﺟﻞ‬cos k𝜃 > 0 ‫ ﻣﻨﻪ‬0 < 𝜃 < 4𝜃 < 6𝜃 < 7𝜃 < ّ 𝜃 = 𝜋 ‫أن‬
‫ﻓﺈن‬ ّ ‫ﺑﲈ‬
2 17
ّ ‫ ﺑﲈ‬. n = 8 ‫ و‬h = 2𝜃 ، a = 𝜃 ‫• ﻧﻀﻊ‬3
: ‫ ﻓﺤﺴﺐ اﻟﺴﺆال اﻷول‬h ∈ ]0, 2𝜋[ ‫أن‬
×
sin cos 𝜃 + 7 × sin 8𝜃 cos 8𝜃
x +x = cos (𝜃 + 2k𝜃) = =
= sin sin 𝜃
sin 16𝜃 sin (𝜋 − 𝜃) sin 𝜃 1
= = = =
sin 𝜃 2 sin 𝜃 2 sin 𝜃 2
ņőƱŀ Ţ
: ‫ ﻣﻨﻪ‬2 cos k𝜃 = z + z− ‫ و‬z = z− ، |z| = 1 ‫ ﻓﻴﻜﻮن‬z = e ‫)ا( ﻧﻀﻊ‬ 4

2x ⋅ 2x = z + z− + z + z− + z + z− + z + z− z + z− + z + z− + z + z− +z + z−

: ‫ إذن‬z− = −z − ‫ ﻣﻨﻪ‬z =e = −1 ‫ﻟﻜﻦ‬

2x ⋅ 2x = z + z− + z + z− + z + z− + z− + z z + z− + z− + z + z− + z + z− + z
= z− + z− + z− + 2z− + z− + 2z− + 2z + 3z + 2z + 4z + 3z + 3z + 4z + 4z− + 3z−
+3z− + 3z− + 4z− + 2z− + 3z− + 2z− + 2z + z + 2z + z + z + 4z
ƕ
VII

=4 z +4 z− = 4 z −4 z − =4 z −4 −z
= = = = = =
ũŏ

=4 z +4 z =4 z = 4 (−1) = −4
= = =

−4
x x = = −1 : ‫ﻣﻨﻪ‬
4

.( z = −1 ‫ﻣﻨﻪ‬ ّ
z = 0 ‫)ﻷن‬
= =

𝟯𝟯𝟵

http ://tinyurl.com/Malki1718 0
.
1
ّ x x = −1 ‫ و‬x + x = 1 ‫أن‬
. X − X − 1 = 0 ‫ ﻫﻲ ﺣﻠﻮل اﳌﻌﺎدﻟﺔ‬x ‫ و‬x ‫ﻓﺈن‬ ّ ‫)ب( ﺑﲈ‬
2 2

Ŕž Ŧ Ľ
1 − √17 1 + √17 1 ± √17
.x = ‫و‬x = ّ x > 0 ‫أن‬
: ‫ﻓﺈن‬ ّ ‫؛ و ﺑﲈ‬ ‫ﺣﻠﻮل ﻫﺬه اﻷﺧﲑة ﻫﻲ‬
4 4 4
5

œǃ
(‫)ا‬ •

2y = z + z− + z + z− : ‫ﻟﺪﻳﻨﺎ‬
2y = z + z− +z + z− ‫و‬

Ŀ
4y y = z + z− + z + z− z + z− + z + z− : ‫ﻣﻨﻪ‬
= z + z− + z + z− + z + z− + z + z−
+z + z− +z + z− +z + z− +z + z−
= 2 cos 2𝜃 + cos 4𝜃 + cos 6𝜃 + cos 8𝜃 + cos 10𝜃 + cos 12𝜃
+ cos 14𝜃 + cos 16𝜃

=2 cos 2k𝜃 = 2 −1 + cos (0 + k × 2𝜃)
= =
×
sin cos 0 + 8 × sin 9𝜃 cos 8𝜃
= 2 −1 + = 2 −1 +
sin sin 𝜃
sin (𝜋 − 8𝜃) cos 8𝜃 sin 8𝜃 cos 8𝜃
= 2 −1 + = 2 −1 +
sin 𝜃 sin 𝜃
sin 16𝜃 sin (𝜋 − 𝜃)
= 2 −1 + = 2 −1 +
sin 𝜃 2 sin 𝜃
sin 𝜃 1
= 2 −1 + = 2 −1 + = −1
2 sin 𝜃 2
1
y y =− : ‫إذن‬
4
4y y = z + z− + z + z− z + z− + z + z− : ‫ﺑﺎﳌﺜﻞ‬
ņőƱŀ Ţ
= z + z− + z + z− + z + z− + z + z−
+z + z− +z + z− +z + z− +z + z−
= z + z− + z + z− + z + z− + z + z−
+z + z− +z + z− +z + z− + z + z− (z = z − ّ
‫)ﻷن‬

=2 cos 2k𝜃 = −1 (‫)ﺣﺴﺐ ﻣﺎ ﺳﺒﻖ‬


=
1
y y =− : ‫إذن‬
ƕ

4
VII

1 1
X −x X− = ‫ ﻫﻲ ﺣﻠﻮل اﳌﻌﺎدﻟﺔ‬y ّ ‫ و وﺟﺪﻧﺎ‬y +y = x ‫أن‬
‫ و‬y ‫ إذن‬y y = − ‫أن‬ ّ ‫ﻧﻼﺣﻆ‬ (‫)ب‬
ũŏ


4 4
x ± x +1
.
‫ أي ﺗﺴﺎوي‬0
2
𝜋
0 < 3𝜃 < 5𝜃 < 7𝜃 < < 11𝜃 < 𝜋 ‫[ و‬0, 𝜋] ‫ ﻣﺘﻨﺎﻗﺼﺔ ﲤﺎﻣ ًﺎ ﻋﲆ اﳌﺠﺎل‬cos ‫ﻟﻜﻦ داﻟﺔ ﺟﻴﺐ اﻟﺘﲈم‬
2

𝟯𝟰𝟬

http ://tinyurl.com/Malki1718 0
‫‪ .VII‬ﲤﺎرﻳﻦ ﻟﻠﺘﻌﻤﻖ‬
‫‪.‬‬
‫‪x −‬‬ ‫‪x +1‬‬ ‫‪x +‬‬ ‫‪x +1‬‬
‫= ‪ . y‬ﻟﻜﻦ ‪:‬‬ ‫= ‪y‬و‬ ‫ﻣﻨﻪ ‪ y > y‬إذن‬
‫‪2‬‬ ‫‪2‬‬

‫‪Ŕž Ŧ Ľ‬‬
‫‪1 + √17‬‬ ‫‪1 + 17 + 2√17 + 16 34 + 2√17‬‬
‫=‪x +1‬‬ ‫=‪+1‬‬ ‫=‬

‫‪œǃ‬‬
‫‪4‬‬ ‫‪16‬‬ ‫‪16‬‬

‫‪1‬‬ ‫‪1 + √17‬‬ ‫‪34 + 2√17‬‬ ‫‪1 + √17 +‬‬ ‫‪34 + 2√17‬‬
‫= ‪y‬‬ ‫‪+‬‬ ‫=‬ ‫إذن ‪:‬‬
‫‪2‬‬ ‫‪4‬‬ ‫‪4‬‬ ‫‪8‬‬

‫‪Ŀ‬‬
‫‪1‬‬ ‫‪1 + √17‬‬ ‫‪34 + 2√17‬‬ ‫‪1 + √17 −‬‬ ‫‪34 + 2√17‬‬
‫= ‪y‬‬ ‫‪−‬‬ ‫=‬ ‫و‬
‫‪2‬‬ ‫‪4‬‬ ‫‪4‬‬ ‫‪8‬‬

‫‪1‬‬ ‫‪1‬‬
‫‪X −x X−‬‬ ‫‪ y‬ﻫﻲ ﺣﻠﻮل اﳌﻌﺎدﻟﺔ =‬ ‫أن ‪ y +y = x‬و وﺟﺪﻧﺎ ّ‬
‫أن ‪ y y = −‬إذن ‪ y‬و‬ ‫ﻧﻼﺣﻆ ّ‬ ‫•‬
‫‪4‬‬ ‫‪4‬‬
‫‪x ±‬‬ ‫‪x +1‬‬
‫‪.‬‬
‫‪ 0‬أي ﺗﺴﺎوي‬
‫‪2‬‬
‫𝜋‬
‫ﻟﻜﻦ ‪ cos‬ﻣﺘﻨﺎﻗﺼﺔ ﲤﺎﻣ ًﺎ ﻋﲆ ]𝜋 ‪ [0,‬و ‪ 0 < 𝜃 < < 9𝜃 < 13𝜃 < 15𝜃 < pi‬ﻣﻨﻪ ‪ y > y‬إذن‬
‫‪2‬‬
‫‪x − x +1‬‬ ‫‪x + x +1‬‬
‫= ‪ . y‬ﻟﻜﻦ ‪:‬‬ ‫= ‪y‬و‬
‫‪2‬‬ ‫‪2‬‬

‫‪1 − √17‬‬ ‫‪1 + 17 − 2√17 + 16 34 − 2√17‬‬


‫=‪x +1‬‬ ‫=‪+1‬‬ ‫=‬
‫‪4‬‬ ‫‪16‬‬ ‫‪16‬‬

‫‪1‬‬ ‫‪1 − √17‬‬ ‫‪34 − 2√17‬‬ ‫‪1 − √17 +‬‬ ‫‪34 − 2√17‬‬
‫= ‪y‬‬ ‫‪+‬‬ ‫=‬ ‫إذن ‪:‬‬
‫‪2‬‬ ‫‪4‬‬ ‫‪4‬‬ ‫‪8‬‬

‫‪1‬‬ ‫‪1 − √17‬‬ ‫‪34 − 2√17‬‬ ‫‪1 − √17 −‬‬ ‫‪34 − 2√17‬‬
‫= ‪y‬‬ ‫‪−‬‬ ‫=‬ ‫و‬
‫‪2‬‬ ‫‪4‬‬ ‫‪4‬‬ ‫‪8‬‬
‫‪ņőƱŀ Ţ‬‬
‫‪ •6‬ﻧﺘﺒﻊ ﻧﻔﺲ اﻟﻄﺮﻳﻘﺔ و ﻧﻀﻊ 𝜃 ‪ t = cos‬و 𝜃‪ . t = cos 13‬ﻣﻦ اﻟﻮاﺿﺢ ّ‬
‫أن ‪ t > t‬و ‪ . t + t = y‬ﻣﻦ ﺟﻬﺔ‬
‫أﺧﺮى‪ ،‬ﻟﺪﻳﻨﺎ ‪:‬‬
‫‪4t t = z + z−‬‬ ‫‪z‬‬ ‫‪+ z−‬‬
‫‪=z‬‬ ‫‪+ z−‬‬ ‫‪+z‬‬ ‫‪+ z−‬‬ ‫)𝜃‪= 2 cos 12𝜃 + 2 cos 14𝜃 = 2 cos (𝜋 − 5𝜃) + 2 cos (𝜋 − 3‬‬
‫‪= −2 cos 5𝜃 − 2 cos 3𝜃 = −2y‬‬
‫‪ƕ‬‬

‫‪−2y‬‬ ‫‪1‬‬
‫‪VII‬‬

‫= ‪t t‬‬ ‫‪=− y‬‬ ‫ﻣﻨﻪ ‪:‬‬


‫‪4‬‬ ‫‪2‬‬

‫‪y ±‬‬ ‫‪y + 2y‬‬ ‫‪1‬‬


‫‪ũŏ‬‬

‫‪ .‬ﻟﻜﻦ ‪:‬‬ ‫إذن ‪ t‬و ‪ t‬ﻫﻲ ﺣﻠﻮل ‪ X − y X − y = 0‬أي ﺗﺴﺎوي‬


‫‪2‬‬ ‫‪2‬‬

‫‪1 − √17 +‬‬ ‫‪34 − 2√17‬‬ ‫‪1 + √17 +‬‬ ‫‪34 + 2√17‬‬
‫= ‪y + 2y‬‬ ‫‪−2‬‬
‫‪8‬‬ ‫‪8‬‬

‫‪26 − 2√17 − −1 + √17‬‬ ‫‪34 − 2√17‬‬ ‫‪8 1 + √17 +‬‬ ‫‪34 + 2√17‬‬
‫=‬ ‫‪+‬‬
‫‪32‬‬ ‫‪32‬‬

‫𝟭𝟰𝟯‬

‫‪http ://tinyurl.com/Malki1718‬‬ ‫‪0‬‬


‫‪.‬‬
‫‪34 + 6√17 + 8‬‬ ‫‪34 + 2√17 + 1 − √17‬‬ ‫‪34 − 2√17‬‬
‫=‬
‫‪32‬‬

‫‪Ŕž Ŧ Ľ‬‬
‫‪34 + 6√17 + 8 34 + 2√17 + 1 − √17‬‬ ‫‪34 − 2√17‬‬
‫= ‪y + 2y‬‬ ‫ﻣﻨﻪ ‪:‬‬

‫‪œǃ‬‬
‫‪32‬‬
‫‪68 + 12√17 + 16 34 + 2√17 + 2 1 − √17‬‬ ‫‪34 − 2√17‬‬
‫=‬
‫‪8‬‬

‫‪Ŀ‬‬
‫‪y ±‬‬ ‫‪y + 2y‬‬ ‫‪1‬‬ ‫‪1 − √17 +‬‬ ‫‪34 − 2√17‬‬
‫=‬ ‫ﻣﻨﻪ ‪:‬‬
‫‪2‬‬ ‫‪2‬‬ ‫‪8‬‬

‫‪68 + 12√17 + 16‬‬ ‫‪34 + 2√17 + 2 1 − √17‬‬ ‫⎞ ‪34 − 2√17‬‬


‫‪±‬‬ ‫⎟‬
‫‪8‬‬ ‫⎟‬
‫⎠‬

‫𝜋‬
‫‪ّ t = cos 𝜃 = cos‬‬
‫ﻓﺈن ‪:‬‬ ‫و ﺑﲈ ّ‬
‫أن ‪ t > t‬و‬
‫‪17‬‬

‫𝜋‬ ‫‪1‬‬
‫‪cos‬‬ ‫=‬ ‫‪1 − √17 +‬‬ ‫‪34 − 2√17 +‬‬ ‫‪68 + 12√17 + 16‬‬ ‫‪34 + 2√17 + 2 1 − √17‬‬ ‫‪34 − 2√17‬‬
‫‪17 16‬‬

‫𝜋‬ ‫𝜋‬ ‫𝜋‬


‫‪ sin‬؛ و ﺑﲈ ّ‬
‫أن 𝜃‪t = cos 13‬‬ ‫‪=+‬‬ ‫‪1 − cos‬‬ ‫‪ sin‬ﻣﻨﻪ‬ ‫أن 𝜋 < 𝜋 < ‪ّ 0‬‬
‫ﻓﺈن ‪> 0‬‬ ‫ﺑﲈ ّ‬ ‫ﻣﻼﺣﻈﺔ ‪⧏ : 56‬‬
‫‪17‬‬ ‫‪17‬‬ ‫‪17‬‬ ‫‪17‬‬
‫ّ‬
‫ﻓﺈن ‪:‬‬
‫𝜋‪13‬‬ ‫‪1‬‬
‫‪cos‬‬ ‫=‬ ‫‪1 − √17 +‬‬ ‫‪34 − 2√17 −‬‬ ‫‪68 + 12√17 + 16‬‬ ‫‪34 + 2√17 + 2 1 − √17‬‬ ‫‪34 − 2√17‬‬
‫‪17‬‬ ‫‪16‬‬

‫ﻳﻤﻜﻦ اﺗﺒﺎع ﻧﻔﺲ اﻟﻄﺮﻳﻘﺔ ﻹﳚﺎد 𝜃‪ cos 3‬و 𝜃‪ : cos 5‬ﻧﻀﻊ 𝜃‪ t = cos 3‬و 𝜃‪ t = cos 5‬ﻓﻴﻜﻮن ‪ t + t = y‬و‬
‫‪ņőƱŀ Ţ‬‬
‫ﺛﻢ ﻧﺘﻤﻢ اﳊﺴﺎﺑﺎت ﻛﲈ ﺳﺒﻖ‪ .‬ﻧﻔﺲ اﻟﴚء ﺑﺎﻟﻨﺴﺒﺔ‬ ‫‪ّ 4t t = 2 cos 8𝜃 + 2 cos 2𝜃 = −2 cos 9𝜃 − 2 cos 15𝜃 = −2y‬‬
‫إﱃ 𝜃‪ cos 7‬و 𝜃‪ ، cos 11‬و ﺑﺎﻟﻨﺴﺒﺔ إﱃ 𝜃‪ cos 9‬و 𝜃‪. cos 15‬‬
‫ﻳﻤﻜﻦ أﻳﻀ ًﺎ اﺳﺘﺨﺪام اﳌﺘﻄﺎﺑﻘﺎت اﳌﺜﻠﺜﻴﺔ‪ ،‬ﻣﺜ ً‬
‫ﻼ ‪... cos 15𝜃 = cos (𝜋 − 2𝜃) = − cos 2𝜃 = 1 − 2 cos 𝜃 :‬إﻟﺦ‪.‬‬
‫⧐‬
‫■‬

‫ﻫﺪﻓﻨﺎ ﰲ ﻫﺬه اﳌﺴﺄﻟﺔ ﻫﻮ إﺛﺒﺎت ﺻﻴﻐﺔ ﭬﻮﺹ )‪ 1796 — (G‬م ‪:‬‬


‫‪$‬‬ ‫‪.‬‬
‫𝟓𝟒𝟐‬ ‫‪.‬‬
‫‪ƕ‬‬

‫‪VII‬‬

‫𝜋‪2‬‬ ‫‪1‬‬
‫‪cos‬‬ ‫=‬ ‫‪−1 + √17 +‬‬ ‫‪34 − 2√17 +‬‬ ‫‪68 + 12√17 + 2 −1 + √17‬‬ ‫‪34 − 2√17 − 16 34 + 2√17‬‬
‫‪17 16‬‬
‫𝜋‪2‬‬
‫‪ũŏ‬‬

‫= 𝜃؛ ‪z = e‬و‪:‬‬ ‫ﻧﻀﻊ‬
‫‪17‬‬
‫‪a =z+z +z +z +z +z‬‬ ‫‪+z‬‬ ‫‪+z‬‬ ‫‪c =z+z‬‬
‫‪a =z +z +z +z +z‬‬ ‫‪+z‬‬ ‫‪+z‬‬ ‫‪+z‬‬ ‫‪c =z +z‬‬
‫‪b =z+z +z‬‬ ‫‪+z‬‬ ‫‪b =z +z +z‬‬ ‫‪+z‬‬
‫‪b =z +z +z +z‬‬ ‫‪b =z +z +z‬‬ ‫‪+z‬‬

‫𝟮𝟰𝟯‬

‫‪http ://tinyurl.com/Malki1718‬‬ ‫‪0‬‬


‫‪ .VII‬ﲤﺎرﻳﻦ ﻟﻠﺘﻌﻤﻖ‬
‫‪.‬‬

‫)د( أوﺟﺪ ﻋﺒﺎرة ‪ b‬و ‪ b‬ﺑﺪﻻﻟﺔ ‪. a‬‬ ‫أن ‪. a + a = −1 :‬‬ ‫‪) •1‬ا( أﺛﺒﺖ ّ‬

‫‪Ŕž Ŧ Ľ‬‬
‫ﻋﱪ ﻋﻦ ‪ b‬و ‪ b‬ﺑﺪﻻﻟﺔ‬
‫)ﻫ( ﺑﺎﺗﺒﺎع ﻧﻔﺲ اﻟﻄﺮﻳﻘﺔ‪ ،‬ﱢ‬ ‫أن ‪. a < 0‬‬ ‫)ب( أﺛﺒﺖ ّ‬
‫‪.a‬‬ ‫أن ‪. a a = −4 :‬‬ ‫)ج( أﺛﺒﺖ ّ‬

‫‪œǃ‬‬
‫)ا( أﺛﺒﺖ ّ‬
‫أن ‪. c + c = b :‬‬ ‫•‬ ‫‪3‬‬ ‫ﻧُﺬﻛﱢﺮ ﺑﺎﳌﺘﻄﺎﺑﻘﺔ اﳌﺜﻠﺜﻴﺔ ‪:‬‬
‫‪1‬‬
‫= ‪cos x cos y‬‬ ‫))‪(cos (x + y) + cos (x − y‬‬
‫)ب( أﺛﺒﺖ ّ‬
‫أن ‪. c < c :‬‬ ‫‪2‬‬
‫)د( أوﺟﺪ اﻟﻌﺒﺎرة اﳉﱪﻳﺔ ﻟﻜﻞ ﻣﻦ ‪ a‬و ‪. a‬‬

‫‪Ŀ‬‬
‫)ج( أﺛﺒﺖ ّ‬
‫أن ‪. c c = b :‬‬
‫أن ‪. b + b = a :‬‬‫‪) •2‬ا( أﺛﺒﺖ ّ‬
‫)د( أوﺟﺪ ﻋﺒﺎرة ‪ c‬و ‪ c‬ﺑﺪﻻﻟﺔ ‪ b‬و ‪. b‬‬
‫أن ‪. b < b :‬‬‫)ب( أﺛﺒﺖ ّ‬
‫‪ •4‬إﺳﺘﻨﺘﺞ ﻗﻴﻤﺔ 𝜃 ‪) cos‬أي ﺻﻴﻐﺔ ﭬﻮﺹ( ‪.‬‬ ‫)ج( أﺛﺒﺖ ّ‬
‫أن ‪. b b = −1 :‬‬
‫أﻧﻈﺮ اﻟﺘﻤﺮﻳﻦ ‪ 244‬ﺻﻔﺤﺔ ‪336‬‬ ‫‬
‫‪.z‬‬ ‫‪−‬‬ ‫‪=z‬‬ ‫‪⋅ z− = z−‬‬ ‫أن )𝜃‪ z + z = z + z− = 2 cos (k‬و‬
‫‪z‬و‪z ≠ 1‬؛و ّ‬ ‫ﺑﺪاﻳ ًﺔ‪ ،‬ﻧﻼﺣﻆ ّ‬
‫أن ‪= 1‬‬ ‫ﺍﳊﻞّ‪.‬‬
‫)ا( ﻟﺪﻳﻨﺎ ‪:‬‬ ‫‪1‬‬
‫•‬
‫‪z‬‬ ‫‪−1‬‬
‫= ‪a +a‬‬ ‫‪z = −1 +‬‬ ‫‪z = −1 +‬‬ ‫‪= −1 + 0 = −1‬‬
‫=‬ ‫=‬
‫‪z−1‬‬

‫𝜋‪2‬‬
‫=𝜃 ّ‬
‫ﻓﺈن ‪:‬‬ ‫)ب( ﺑﲈ ّ‬
‫أن‬
‫‪17‬‬
‫𝜋‬
‫< 𝜃‪0 < 𝜃 < 2𝜃 < 3𝜃 < 4‬‬ ‫𝜋 < 𝜃‪< 5𝜃 < 6𝜃 < 7𝜃 < 8‬‬
‫‪2‬‬
‫أن داﻟﺔ ﺟﻴﺐ اﻟﺘﲈم ‪ cos‬ﻣﺘﻨﺎﻗﺼﺔ ﲤﺎﻣ ًﺎ ﻋﲆ اﳌﺠﺎل ]𝜋 ‪ّ [0,‬‬
‫ﻓﺈن ‪ cos (k𝜃) > 0‬ﻷﺟﻞ ‪ 1 ≤ k ≤ 4‬و‬ ‫و ﺑﲈ ّ‬
‫‪ cos (k𝜃) < 0‬ﻷﺟﻞ ‪ . 5 ≤ k ≤ 8‬ﻟﻜﻦ ‪:‬‬
‫‪ņőƱŀ Ţ‬‬
‫‪a =z+z +z +z +z +z‬‬ ‫‪+z‬‬ ‫‪+z‬‬ ‫‪=z+z‬‬ ‫‪−‬‬ ‫‪+z +z‬‬ ‫‪−‬‬ ‫‪+z +z‬‬ ‫‪−‬‬ ‫‪+z +z‬‬ ‫‪−‬‬

‫𝜃‪= z + z− + z + z− + z + z− + z + z− = 2 cos 𝜃 + 2 cos 2𝜃 + 2 cos 4𝜃 + 2 cos 8‬‬


‫‪a =z +z +z +z +z‬‬ ‫‪+z‬‬ ‫‪+z‬‬ ‫‪+z‬‬ ‫‪=z +z‬‬ ‫‪−‬‬ ‫‪+z +z‬‬ ‫‪−‬‬ ‫‪+z +z‬‬ ‫‪−‬‬ ‫‪+z +z‬‬ ‫‪−‬‬

‫𝜃‪= z + z− + z + z− + z + z− + z + z− = 2 cos 3𝜃 + 2 cos 5𝜃 + 2 cos 6𝜃 + 2 cos 7‬‬


‫𝜃‪3𝜃 + 6‬‬ ‫𝜃‪3𝜃 − 6‬‬ ‫𝜃‪9‬‬ ‫𝜃‪3‬‬
‫‪= 2 × 2 cos‬‬ ‫‪cos‬‬ ‫‪+ 2 cos 5𝜃 + 2 cos 7𝜃 = 4 cos‬‬ ‫‪cos‬‬ ‫𝜃‪+ 2 cos 5𝜃 + 2 cos 7‬‬
‫‪2‬‬ ‫‪2‬‬ ‫‪2‬‬ ‫‪2‬‬

‫𝜃‪9‬‬ ‫𝜃‪3‬‬ ‫𝜃‪9‬‬ ‫𝜃‪3‬‬


‫ﻓﺈن ‪ cos > 0‬و ‪ cos < 0‬إذن ﻓﺎﳉﺪاء ‪ cos cos‬ﺳﺎﻟﺐ ﲤﺎﻣ ًﺎ‬‫أن 𝜋 < 𝜃‪ّ 0 < 3𝜃 < 𝜋 < 9‬‬ ‫ﺑﲈ ّ‬
‫‪ƕ‬‬

‫‪2‬‬ ‫‪2‬‬ ‫‪2‬‬ ‫‪2‬‬ ‫‪2‬‬ ‫‪2‬‬ ‫‪2‬‬


‫‪VII‬‬

‫أن ‪ cos 5𝜃 < 0‬و ‪ّ cos 7𝜃 < 0‬‬


‫ﻓﺈن ‪. a < 0‬‬ ‫و ﺑﲈ ّ‬
‫)ج( ﻟﺪﻳﻨﺎ ‪:‬‬
‫‪ũŏ‬‬

‫‪a a = z+z +z +z +z +z‬‬ ‫‪+z‬‬ ‫‪+z‬‬ ‫‪z +z +z +z +z‬‬ ‫‪+z‬‬ ‫‪+z‬‬ ‫‪+z‬‬
‫‪= z + z + z + 3z + 2z + 2z + z‬‬ ‫‪+ 3z‬‬ ‫‪+ 3z‬‬ ‫‪+ 3z‬‬ ‫‪+ 4z‬‬ ‫‪+ 4z‬‬ ‫‪+ 4z‬‬ ‫‪+ 4z‬‬
‫‪+4z‬‬ ‫‪+ 4z‬‬ ‫‪+ 3z‬‬ ‫‪+ 3z‬‬ ‫‪+ 3z‬‬ ‫‪+z‬‬ ‫‪+ 2z‬‬ ‫‪+ 2z‬‬ ‫‪+ 3z‬‬ ‫‪+z‬‬ ‫‪+z‬‬ ‫‪+z‬‬

‫𝟯𝟰𝟯‬

‫‪http ://tinyurl.com/Malki1718‬‬ ‫‪0‬‬


.
: ‫ إذن‬z ّ z =z
= 1 ‫ﻷن‬ ّ 17 ‫ ﻋﲆ‬k ‫ ﺑﺎﻗﻲ ﻗﺴﻤﺔ‬r ‫ﻟﻜﻦ إذا ﻛﺎن‬
‫ﻓﺈن‬

Ŕž Ŧ Ľ
a a = z + z + z + 3z + 2z + 2z + z + 3z + 3z + 3z + 4z + 4z + 4z + 4z
+4z + 4z + 3z + 3z + 3z + z + 2z + 2z + 3z +z +z +z

œǃ
=4 z+z +z +z +z +z +z +z +z +z +z +z +z +z +z +z
= 4 (a + a ) = 4 (−1) = −4

ّ a a = −4 ‫ و‬a + a = −1 ‫أن‬
‫ أي ﺗﺴﺎوي‬X + X − 4 = 0 ‫ ﻫﻲ ﺣﻠﻮل اﳌﻌﺎدﻟﺔ‬a ‫ و‬a ‫ﻓﺈن‬ ّ ‫)د( ﺑﲈ‬

Ŀ
−1 − √17 −1 + √17
.a = ‫و‬a = ‫ﻓﺈن‬ ّ ‫ و ﺑﲈ‬−1 ± √17
ّ a < 0 ‫أن‬
2 2 2
: ‫)ا( ﻟﺪﻳﻨﺎ‬ • 2
b +b =z+z +z +z +z +z +z +z =a

: ‫)ب( ﻟﺪﻳﻨﺎ‬
b =z+z +z +z = z + z− + z + z− = 2 cos 𝜃 + 2 cos 4𝜃
b =z +z +z +z = z + z− + z + z− = 2 cos 2𝜃 + 2 cos 8𝜃

ّ [0, 𝜋] ‫ ﻣﺘﻨﺎﻗﺼﺔ ﲤﺎﻣ ًﺎ ﻋﲆ‬cos ‫أن داﻟﺔ ﺟﻴﺐ اﻟﺘﲈم‬


.b < b ‫ إذن‬cos 8𝜃 < cos 4𝜃 ‫ و‬cos 2𝜃 < cos 𝜃 ‫ﻓﺈن‬ ّ ‫و ﺑﲈ‬
: ‫)ج( ﻟﺪﻳﻨﺎ‬

b b = z+z +z +z z +z +z +z
= z +z +z +z +z +z +z +z +z +z +z +z +z +z +z +z
=z +z +z +z +z +z +z +z +z+z +z +z +z +z +z +z
= a + a = −1

ّ b b = −1 ‫ و‬b + b = a ‫أن‬
‫ أي ﺗﺴﺎوي‬X − a X − 1 = 0 ‫ ﻫﻲ ﺣﻠﻮل اﳌﻌﺎدﻟﺔ‬b ‫ و‬b ‫ﻓﺈن‬ ّ ‫)د( ﺑﲈ‬
ņőƱŀ Ţ
a − a +4 a + a +4 a ± a +4
.b = ‫و‬b = ّ b < b ‫أن‬
‫ﻓﺈن‬ ّ ‫و ﺑﲈ‬
2 2 2
: ‫)ﻫ( ﻟﺪﻳﻨﺎ‬
b +b =z +z +z +z +z +z +z +z =a
b =z +z +z +z =z + z− +z + z− = 2 cos 3𝜃 + 2 cos 5𝜃
b =z +z +z +z = z + z− + z + z− = 2 cos 6𝜃 + 2 cos 7𝜃
b b = z +z +z +z z +z +z +z : ‫ﻣﻨﻪ‬
ƕ

=z +z +z +z +z +z +z +z +z
VII

+z +z +z +z +z +z +z
=z +z +z +z +z +z +z +z
ũŏ

+z + z + z + z + z + z + z + z
= a + a = −1

a ± a +4
‫ ﻣﺘﻨﺎﻗﺼﺔ ﲤﺎﻣ ًﺎ‬cos ‫ ﻟﻜﻦ‬. ‫ أي ﺗﺴﺎوي‬X − a X − 1 = 0 ‫ ﻫﻲ ﺣﻠﻮل اﳌﻌﺎدﻟﺔ‬b ‫ و‬b ‫إذن‬
2
a − a +4 a + a +4
.b = ‫و‬b = ‫ ﻣﻨﻪ‬b < b ‫[ إذن‬0, 𝜋] ‫ﻋﲆ‬
2 2

𝟯𝟰𝟰

http ://tinyurl.com/Malki1718 0
‫‪ .VII‬ﲤﺎرﻳﻦ ﻟﻠﺘﻌﻤﻖ‬
‫‪.‬‬
‫)ا( ﻟﺪﻳﻨﺎ ‪:‬‬ ‫‪3‬‬
‫•‬
‫‪c +c =z+z‬‬ ‫‪+z +z‬‬ ‫‪=b‬‬

‫‪Ŕž Ŧ Ľ‬‬
‫)ب( ﻟﺪﻳﻨﺎ ‪:‬‬

‫‪œǃ‬‬
‫‪c =z+z‬‬ ‫𝜃 ‪= z + z− = 2 cos‬‬
‫‪c =z +z‬‬ ‫𝜃‪= z + z− = 2 cos 4‬‬

‫‪Ŀ‬‬
‫أن ‪ cos‬ﻣﺘﻨﺎﻗﺼﺔ ﲤﺎﻣ ًﺎ ﻋﲆ ]𝜋 ‪ّ [0,‬‬
‫ﻓﺈن ‪. c < c‬‬ ‫و ﺑﲈ ّ‬
‫)ج( ﻟﺪﻳﻨﺎ ‪:‬‬

‫‪c c = z+z‬‬ ‫‪z +z‬‬ ‫‪=z +z‬‬ ‫‪+z‬‬ ‫‪+z‬‬


‫‪=z +z‬‬ ‫‪+z +z‬‬ ‫‪=b‬‬

‫‪b ±‬‬ ‫‪b − 4b‬‬


‫‪ .‬ﻟﻜﻦ ‪ c < c‬إذن‬ ‫)د( ‪ c‬و ‪ c‬ﻫﻲ ﺣﻠﻮل اﳌﻌﺎدﻟﺔ ‪ X − b X + b = 0‬أي ﺗﺴﺎوي‬
‫‪2‬‬
‫‪b −‬‬ ‫‪b − 4b‬‬ ‫‪b +‬‬ ‫‪b − 4b‬‬
‫= ‪.c‬‬ ‫= ‪c‬و‬
‫‪2‬‬ ‫‪2‬‬

‫)ﻷن ‪ّ a‬‬
‫ﺣﻞ ﻟﻠﻤﻌﺎدﻟﺔ‬ ‫‪ . cos 𝜃 = 1 c = 1 b +‬و ﺑﲈ ّ‬
‫أن ‪ّ a = 4 − a‬‬ ‫‪b − 4b‬‬ ‫‪ •4‬ﺣﺴﺐ ﻣﺎ ﺳﺒﻖ ‪:‬‬
‫‪2‬‬ ‫‪2‬‬
‫‪ّ (X = 4 − X‬‬
‫ﻓﺈن ‪:‬‬

‫‪a +‬‬ ‫‪a +4‬‬ ‫‪1‬‬ ‫‪1‬‬ ‫‪−1 + √17‬‬


‫= ‪b‬‬ ‫=‬ ‫‪a +‬‬ ‫‪4−a‬‬ ‫=‬ ‫‪⎛−1 + √17 +‬‬ ‫‪4−‬‬ ‫⎞‪+ 4‬‬
‫‪2‬‬ ‫‪2‬‬ ‫‪4‬‬ ‫‪2‬‬
‫⎝‬ ‫⎠‬
‫‪1‬‬
‫=‬ ‫‪−1 + √17 +‬‬ ‫‪34 − 2√17‬‬
‫‪4‬‬
‫‪ņőƱŀ Ţ‬‬
‫‪a +‬‬ ‫‪a +4‬‬ ‫‪1‬‬
‫= ‪b‬‬ ‫=‬ ‫‪−1 − √17 +‬‬ ‫‪34 + 2√17‬‬
‫‪2‬‬ ‫‪4‬‬
‫‪1‬‬
‫= ‪b‬‬ ‫‪62 + 2 −1 + √17‬‬ ‫‪34 − 2√17 − 4√17‬‬
‫‪16‬‬

‫ﰲ اﻷﺧﲑ ‪:‬‬
‫𝜋‪2‬‬ ‫‪1‬‬
‫‪cos‬‬ ‫=‬ ‫‪−1 + √17 +‬‬ ‫‪34 − 2√17 +‬‬ ‫‪68 + 12√17 + 2 −1 + √17‬‬ ‫‪34 − 2√17 − 16‬‬ ‫‪34 + 2√17‬‬
‫‪17 16‬‬
‫‪ƕ‬‬
‫‪VII‬‬

‫𝜋‪2‬‬
‫‪ cos‬ﺑﺎﻟﻌﻤﻠﻴﺎت اﳊﺴﺎﺑﻴﺔ ‪ ÷ ، × ، − ، +‬و اﳉﺬور و ﻫﺬا ﻳﻌﻨﻲ أﻧﻪ ﺑﺈﻣﻜﺎﻧﻨﺎ‬ ‫ﲤﻜﻨّﺎ ﰲ ﻫﺬه اﳌﺴﺄﻟﺔ ﻣﻦ اﻟﺘﻌﺒﲑ ﻋﻦ‬
‫‪17‬‬
‫𝜋‪2‬‬
‫‪ũŏ‬‬

‫ﻣﺪرﺟﺔ و ﻣﺪور‬
‫‪ ( cos‬ﺑﺎﺳﺘﺨﺪام ﻣﺴﻄﺮة ﻏﲑ ّ‬ ‫ﻳﺴﺎوي‬
‫»إﻧﺸﺎء«ﻫﺬا اﻟﻌﺪد )أي إﻧﺸﺎء ﻗﻄﻌﺔ ﻣﺴﺘﻘﻴﻤﺔ ﻃﻮﳍﺎ ‪.‬‬
‫‪17‬‬
‫)ﻓﺮﺟﺎر( و ﺑﺎﻟﺘﺎﱄ ﺑﺈﻣﻜﺎﻧﻨﺎ إﻧﺸﺎء اﳌﻀ ّﻠﻊ اﳌﻨﺘﻈﻢ ذي ‪ 17‬ﺿﻠﻌ ًﺎ ﺑﺎﺳﺘﺨﺪام ﻣﺴﻄﺮة ﻏﲑ ّ‬
‫ﻣﺪرﺟﺔ و ﻣﺪور )ﻓﺮﺟﺎر(‬
‫اﻟﺼﱪ و اﳌﺜﺎﺑﺮة ‪. j ...‬‬
‫و ‪ ...‬اﻟﻜﺜﲑ ﻣﻦ ّ‬

‫𝟱𝟰𝟯‬

‫‪http ://tinyurl.com/Malki1718‬‬ ‫‪0‬‬


‫‪.‬‬

‫‪Ŕž Ŧ Ľ‬‬
‫‪œǃ‬‬
‫‪Ŀ‬‬ ‫ﺷﻜﻞ ‪ ، G :12.VII‬ﰲ ﻛﺘﺎﺑﻪ ‪ : Recherches arithmétiques‬ﻛﺎن ﻓﺨﻮر ًا ﺑﻌﻤﻠﻪ‬

‫ﻣﺪرﺟﺔ و ﻣﺪور ؟ اﳉﻮاب ‪ :‬ﻻ و اﻟﻌﺎﱂ ﭬﻮﺹ‬


‫أي ﻣﻀﻠﻊ ﻣﻨﺘﻈﻢ ﺑﺎﺳﺘﺨﺪام ﻣﺴﻄﺮة ﻏﲑ ّ‬
‫ﻟﺪرﺟﺔ أﻧﻪ ﻃﻠﺐ أن ﺗُﻜﺘﺐ ﻫﺬه اﻟﺼﻴﻐﺔ ﻋﲆ ﻗﱪه !‬

‫ﻗﺪ ﻧﺘﺴﺎءل ﻋﻦ إﻣﻜﺎﻧﻴﺔ إﻧﺸﺎء ّ‬


‫ﺗﻮﺻﻞ )ﻋﺎم ‪ 1801‬م( ﻹﳚﺎد اﻟﴩط اﻟﻼزم و اﻟﻜﺎﰲ اﻟﺬي ﳚﺐ أن ُﳛﻘﻘﻪ اﻟﻌﺪد اﻟﻄﺒﻴﻌﻲ ‪ n‬ﺣﺘﻰ ﻳﻜﻮن‬

‫‪ k ∈ ℕ‬و ‪ f ، ⋯ ، f ، f‬أﻋﺪاد ّأوﻟﻴﺔ )ﳐﺘﻠﻔﺔ ﻣﺜﻨﻰ ﻣﺜﻨﻰ( ﻣﻦ اﻟﺸﻜﻞ ‪ F = 2 + 1‬ﺣﻴﺚ ‪. m ∈ ℕ‬‬


‫أن اﻷﻋﺪاد ‪، F = 5 ، F = 3‬‬
‫ِ‬
‫أن اﻷﻋﺪاد ‪ F‬أوﻟﻴﺔ ﻟﻜﻞ ﻗ َﻴﻢ ‪ m‬؛ ﻟﻜﻦ ﰲ ﻋﺎم‬
‫ﻫﻮ ّأول ﻣﻦ ّ‬
‫ﺑﺎﻹﻣﻜﺎن إﻧﺸﺎء اﳌﻀ ّﻠﻊ اﳌﻨﺘﻈﻢ ذي ‪ n‬ﺿﻠﻌ ًﺎ ‪ :‬ﳚﺐ أن ﻳﻜﻮن ‪ n‬ﻣﻦ اﻟﺸﻜﻞ ‪ 2‬أو ‪ 2 × f × f × ⋯ × f‬ﺣﻴﺚ‬

‫اﻷﻋﺪاد ‪ F‬ﺗُﺪﻋﻰ أﻋﺪاد ﻓﻴﺮﻣﺎ )‪ (F‬ﻷﻧﻪ أول ﻣﻦ ﺑﺮﻫﻦ ﻋﺎم ‪1640‬م ّ‬


‫‪ F = 257 ، F = 17‬و ‪ّ F = 65537‬أوﻟﻴﺔ و اﻋﺘﻘﺪ ﺑﻌﺪﻫﺎ ّ‬
‫أن ‪ F‬ﻟﻴﺲ‬ ‫أن ‪ F = 4294967297 = 641 × 6700417‬ﻟﻴﺲ ّأوﱄ‪ .‬و ُﻳﻌﺘﻘﺪ ّ‬ ‫‪1732‬م أﺛﺒﺖ اﻟﻌﺎﱂ أوﻟﺮ )‪ّ (E‬‬
‫ﻳﺘﻮﺻﻞ أﺣﺪ ‪−‬ﳊﺪ اﻵن‪ −‬إﱃ إﺛﺒﺎت ﺻﺤﺔ أو ﺧﻄﺄ ﻫﺬا اﻹﻋﺘﻘﺎد !‬ ‫ّأوﱄ ﻟﻜﻞ ‪ m ≥ 5‬ﻟﻜﻦ ﱂ ّ‬
‫ﻫﺬه اﻷﻋﺪاد ﺗﻜﱪ ﺑﴪﻋﺔ ﻛﺒﲑة و ﳊﺪ اﻵن‪ ،‬اﻷﻋﺪاد اﻟﺘﻲ ﻧﻌﻠﻢ أﳖﺎ ﻟﻴﺴﺖ أوﻟﻴﺔ ﻫﻲ ‪ F‬ﻣﻊ ‪ ، 5 ≤ m ≤ 19‬ﻟﻜﻦ‬
‫‪ņőƱŀ Ţ‬‬
‫‪.‬‬
‫ﻻ ﻧﻌﺮف اﻟﺘﺤﻠﻴﻞ اﻟﻜﲇ ﳍﺎ إﱃ ﺟﺪاء ﻋﻮاﻣﻞ أوﻟﻴﺔ إﻻّ ﻟـِ ‪ F ، F ، F ، F ، F‬و ‪ . F‬و ﻻ ﻧﻌﺮف ّ‬
‫أي ﻋﺎﻣﻞ ّأوﱄ‬
‫ﺗﻢ ﲢﻠﻴﻞ ‪ F‬ﻋﺎم ‪1971‬م و ‪ F‬ﻋﺎم ‪1982‬م ‪.‬‬ ‫ﻟﻠﻌﺪد ‪ ! F‬و ﻗﺪ ّ‬
‫اﻟﻘﻀﻴﺔ اﻟﺘﻲ ﲥﻤﻨﺎ ﻫﻨﺎ ﻫﻲ إﻧﺸﺎء اﳌﻀ ّﻠﻊ اﳌﻨﺘﻈﻢ ‪ P‬ذي ‪ n‬ﺿﻠﻌ ًﺎ ‪:‬‬
‫ﺗﻮﺻﻞ إﱃ‬
‫اﻟﻄﺮﻳﻘﺔ ﻣﻌﺮوﻓﺔ ﻷﺟﻞ ‪) P‬أﻧﻈﺮ اﻟﺘﻤﺮﻳﻦ ‪ 214‬ﺻﻔﺤﺔ ‪ (305‬و ﺗﻮﺟﺪ ﻋﺪة ﻃﺮق ﻹﻧﺸﺎء ‪ P‬و أول ﻣﻦ ّ‬
‫ذﻟﻚ ﻫﻮ اﻟﻌﺎﱂ ‪ H‬و ذﻟﻚ ﻋﺎم ‪1803‬م‪ .‬ﺑﺎﻟﻨﺴﺒﺔ إﱃ ‪ ، P‬ﲤﻜﻦ اﻟﻌﺎﱂ ‪ R‬ﻣﻦ إﳚﺎد ﻃﺮﻳﻘﺔ ﻹﻧﺸﺎءه‬
‫‪ P‬ﺑﻌﺪ ‪ 20‬ﺳﻨﺔ ﻣﻦ اﻟﺒﺤﺚ !‬ ‫أن أﻣﺮﻳﻜﻴ ًﺎ وﺟﺪ ﻃﺮﻳﻘﺔ ﻹﻧﺸﺎء‬
‫ﻋﺎم ‪1832‬م و ُﻳﻘﺎل ّ‬
‫𝜋‬ ‫𝜋‬ ‫𝜋‬
‫ﻷن اﳌﺤﺎوﻟﺔ ﺳﺘﺒﻮء ﺑﺎﻟﻔﺸﻞ‬‫‪ّ ... cos‬‬ ‫إذن ﻻ داﻋﻲ ﻣﻦ ﳏﺎوﻟﺔ إﳚﺎد ﺻﻴﻎ ﳑﺎﺛﻠﺔ ﻷﻋﺪاد ﻣﺜﻞ ‪ cos ، cos‬أو‬
‫‪131‬‬ ‫‪41‬‬ ‫‪13‬‬
‫𝜋‬
‫‪ƕ‬‬

‫‪cos‬‬ ‫ﺗﻮﺻﻞ إﻟﻴﻬﺎ اﻟﻌﺎﱂ ﭬﻮﺹ ‪ ،‬ﻣﻦ اﳌﻤﻜﻦ إﳚﺎد ﻣﺜﻞ ﻫﺬه اﻟﺼﻴﻐﺔ ﻷﺟﻞ‬
‫ﻻ ﳏﺎﻟﺔ ! ﻟﻜﻦ‪ ،‬و ﺣﺴﺐ اﻟﻨﺘﻴﺠﺔ اﻟﺘﻲ ّ‬
‫‪VII‬‬

‫‪170‬‬
‫𝜋‬
‫ﻣﺘﻄﻮع ؟ ‪j‬‬
‫ﱢ‬ ‫‪ cos‬أو ‪ ...‬ﻫﻞ ﻣﻦ‬
‫‪257‬‬
‫أو‬
‫‪ũŏ‬‬

‫اﻟﺘﻌﻤﻖ ﰲ اﳌﻮﺿﻮع‪ ،‬ﻓﲈ ﻋﻠﻴﻪ إﻻّ ﻣﻌﺎﻳﻨﺔ ﻛﺘﺐ ‪ :‬اﳊﻘﻮل )اﻷﺟﺴﺎم(‬ ‫ّ‬ ‫ﻣﻼﺣﻈﺔ ‪ :‬إذا أراد اﻟﻘﺎرئ اﻟﻜﺮﻳﻢ ﻣﻌﺮﻓﺔ ﺗﻔﺎﺻﻴﻞ أﻛﺜﺮ و‬
‫و ﻧﻈﺮﻳﺔ ﭬﺎﻟﻮا )‪ Fields and G Theory‬ﺑﺎﻹﻧﺠﻠﻴﺰﻳﺔ ؛ ‪ Corps et théorie de G‬ﺑﺎﻟﻔﺮﻧﺴﻴﺔ( ﻣﺜ ً‬
‫ﻼ ﻛﺘﺎب ‪:‬‬
‫ﻧﻈﺮﻳﺔ ﺟﺎﻟﻮا ‪ ،‬ﺗﺄﻟﻴﻒ ﺇﻳﺎﻥ ﺳﺘﻴﻮﺍﺭﺕ ‪ ،‬ﺗﺮﲨﺔ ﻣﻌﺮﻭﻑ ﻋﺒﺪﺍﻟﺮﲪﻦ ﲰﺤﺎﻥ و ﻓﻮﺯﻱ ﺃﲪﺪ ﺍﻟﺬﻛﲑ ‪ ،‬اﻟﻨﴩ اﻟﻌﻠﻤﻲ و اﳌﻄﺎﺑﻊ ‪ -‬ﺟﺎﻣﻌﺔ اﳌﻠﻚ‬
‫■‬ ‫ﻣﺴﺘﻮى ٍ‬
‫ﻋﺎل )ﻋﲆ اﻷﻗﻞ ﻟﻴﺴﺎﻧﺲ( ‪.‬‬ ‫ً‬ ‫ﺳﻌﻮد؛ ﻟﻜﻦ ذﻟﻚ ﻳﺘﻄ ّﻠﺐ‬

‫𝟲𝟰𝟯‬

‫‪http ://tinyurl.com/Malki1718‬‬ ‫‪0‬‬


‫‪ .VII‬ﲤﺎرﻳﻦ ﻟﻠﺘﻌﻤﻖ‬
‫‪.‬‬

‫‪4‬‬

‫‪Ŕž Ŧ Ľ‬‬
‫‪C‬‬

‫‪œǃ‬‬
‫‪2‬‬

‫‪0‬‬ ‫‪2‬‬ ‫‪4‬‬ ‫‪6‬‬ ‫‪8‬‬ ‫‪10‬‬

‫‪Ŀ‬‬
‫‪D‬‬
‫‪2−ı‬‬ ‫‪6−ı‬‬

‫‪−2‬‬

‫‪−4‬‬

‫‪−6‬‬

‫ﺷﻜﻞ ‪13.VII‬‬

‫‪.‬‬
‫𝟔𝟒𝟐‬ ‫‪.‬‬

‫‪. ∀z ∈ ℂ : ||z − (2 − ı)|| ≤ 1 ⟹ 3 ≤ ||z − 6 + ı|| ≤ 5‬‬ ‫‪ •1‬أﺛﺒﺖ أﻧّﻪ ‪:‬‬


‫‪ •2‬ﻣﺎ ﻫﻮ اﻟﺘﻔﺴﲑ اﳍﻨﺪﳼ ﳍﺬه اﻟﻨﺘﻴﺠﺔ ؟‬
‫‪ņőƱŀ Ţ‬‬
‫ﺍﳊﻞّ‪ .‬ﻧ َُﺬﻛﱢﺮ ﺑﺎﳌﺘﺒﺎﻳﻨﺔ اﳌﺜﻠﺜﻴﺔ و اﳌﺘﺎﺑﻴﻨﺔ اﳌﺜﻠﺜﻴﺔ اﳌﻌﻜﻮﺳﺔ ‪ :‬إذا ﻛﺎن ‪ z‬و ‪ z‬ﻋﺪدﻳﻦ ﻣﺮﻛﺒﲔ ّ‬
‫ﻓﺈن ‪ ||z + z || ≤ |z| + ||z || :‬و‬
‫|| ‪. ||z − z || ≥ |z| − ||z‬‬
‫‪ •1‬ﻟﻴﻜﻦ ‪ z ∈ ℂ‬ﺑﺤﻴﺚ ‪ . ||z − (2 − ı)|| ≤ 1 :‬ﻟﺪﻳﻨﺎ ‪:‬‬
‫‪||z − 6 + ı|| = ||(z − (2 − ı)) + (−4)|| ≤ ||z − (2 − ı)|| + ||−4|| ≤ 1 + 4 ≤ 5‬‬
‫و ﻣﻦ ﺟﻬﺔ أﺧﺮى ﻟﺪﻳﻨﺎ ‪:‬‬
‫‪||z − 6 + ı|| = ||(z − (2 − ı)) − 4|| = ||4 − (z − (2 − ı))|| ≥ ||4|| − ||z − (2 − ı)|| ≥ 4 − 1 ≥ 3‬‬
‫‪. ∀z ∈ ℂ : ||z − (2 − ı)|| ≤ 1 ⟹ 3 ≤ ||z − 6 + ı|| ≤ 5‬‬ ‫إذن ‪:‬‬
‫‪ƕ‬‬
‫‪VII‬‬

‫‪ •2‬اﻟﺘﻔﺴﲑ اﳍﻨﺪﳼ ‪ :‬ﳎﻤﻮﻋﺔ اﻟﻨﻘﻂ ذات اﻟﻼﺣﻘﺔ ‪ z‬ﺑﺤﻴﺚ ‪ ||z − (2 − ı)|| ≤ 1‬ﻫﻲ اﻟﻘﺮص ا ُﳌﻐﻠﻖ 𝒟 اﻟﺬي ﻣﺮﻛﺰه‬
‫اﻟﻨﻘﻄﺔ ذات اﻟﻼﺣﻘﺔ ‪ 2 − ı‬و ﻧﺼﻒ ﻗﻄﺮه ‪ 1‬؛ و ﳎﻤﻮﻋﺔ اﻟﻨﻘﻂ ذات اﻟﻼﺣﻘﺔ ‪ z‬ﺑﺤﻴﺚ ‪ 3 ≤ ||z − 6 + ı|| ≤ 5‬ﻫﻲ‬
‫ِ‬
‫اﳊﻠﻘﺔ ا ُﳌﻐﻠﻘﺔ 𝒞 اﻟﺘﻲ ﻣﺮﻛﺰﻫﺎ اﻟﻨﻘﻄﺔ ذات اﻟﻼﺣﻘﺔ ‪ 6 − ı‬و ﻧﺼ َﻔﺎ َ‬
‫‪ũŏ‬‬

‫ﻗﻄﺮ ْﳞﺎ ‪ 3‬و ‪. 5‬‬


‫ﻓﺈن ‪ M‬ﺗﻨﺘﻤﻲ إﱃ 𝒞 أي ّ‬
‫أن اﻟﻘﺮص‬ ‫ﻧﺘﻴﺠﺔ اﻟﺴﺆال اﻟﺴﺎﺑﻖ ﺗﻌﻨﻲ أﻧﻪ إذا ﻛﺎﻧﺖ اﻟﻨﻘﻄﺔ ‪ M‬ذات اﻟﻼﺣﻘﺔ ‪ z‬ﺗﻨﺘﻤﻲ إﱃ 𝒟 ّ‬
‫ا ُﳌﻐﻠﻖ 𝒟 ُﳏﺘ ََﻮى ﰲ اﳊﻠﻘﺔ ا ُﳌﻐﻠﻘﺔ 𝒞 )ﺷﻜﻞ ‪.(13.VII‬‬
‫■‬

‫𝟳𝟰𝟯‬

‫‪http ://tinyurl.com/Malki1718‬‬ ‫‪0‬‬


‫‪.‬‬

‫‪.‬‬
‫𝟕𝟒𝟐‬ ‫‪.‬‬

‫‪Ŕž Ŧ Ľ‬‬
‫||‪. |z| ≤ |z| + ||z − 1‬‬ ‫أﺛﺒﺖ أﻧّﻪ ﻷﺟﻞ ﻛﻞ ﻋﺪد ﻣﺮﻛﺐ ‪ّ z ∈ ℂ‬‬
‫ﻓﺈن ‪:‬‬

‫‪œǃ‬‬
‫ﺍﳊﻞّ‪ .‬ﻟﻴﻜﻦ ‪ z ∈ ℂ‬ﻋﺪد ًا ﻣﺮﻛﺒﺎ‪ .‬ﻟﺪﻳﻨﺎ )اﳌﺘﺒﺎﻳﻨﺔ اﳌﺜﻠﺜﻴﺔ( ‪:‬‬
‫|| ‪|z| = ||z − z + z || ≤ ||z − z || + ||z || = |z| ⋅ ||z − 1|| + ||z‬‬
‫و ﻧﻤ ﱢﻴﺰ ﺣﺎﻟﺘﲔ ‪:‬‬

‫‪Ŀ‬‬
‫إذا ﻛﺎن ‪ّ |z| ≤ 1‬‬
‫ﻓﺈن ||‪ |z| ⋅ ||z − 1|| ≤ ||z − 1‬ﻣﻨﻪ ||‪ |z| ≤ |z| + ||z − 1‬؛‬ ‫•‬

‫أن ‪ّ ||z − 1|| ≥ 0‬‬


‫ﻓﺈن ||‪. |z| ≤ |z| ≤ |z| + ||z − 1‬‬ ‫ﻓﺈن |‪ |z| ≤ |z‬و ﺑﲈ ّ‬
‫• و إذا ﻛﺎن ‪ّ |z| ≥ 1‬‬
‫■‬ ‫||‪. |z| ≤ |z| + ||z − 1‬‬ ‫ﻛﻞ اﳊﺎﻻت ‪:‬‬ ‫إذن ﰲ ّ‬

‫أﺛﺒﺖ اﳌﺘﻄﺎﺑﻘﺘﲔ اﻟﺘﺎﻟﻴﺘﲔ ) ‪ z‬و ‪ z‬ﻋﺪدان ﻣﺮﻛﺒﺎن( ‪:‬‬


‫‬ ‫‪.‬‬
‫𝟖𝟒𝟐‬ ‫‪.‬‬

‫|| ‪||z + z || = ||z || + 2 Re (z z ) + ||z‬‬


‫|| ‪||z − z || = ||z || − 2 Re (z z ) + ||z‬‬

‫اﺳﺘﻨﺘﺞ ّ‬
‫أن ‪. ||z + z || + ||z − z || = 2 ||z || + 2 ||z || :‬‬
‫أﻧﻈﺮ اﻟﺘﻤﺮﻳﻦ ‪ 250‬ﺻﻔﺤﺔ ‪349‬‬ ‫‬
‫ﺍﳊﻞّ‪ .‬ﻟﺪﻳﻨﺎ ‪:‬‬
‫) ‪||z + z || = (z + z ) (z + z ) = (z + z ) (z + z‬‬

‫|| ‪= z z + z z + z z + z z = ||z || + z z + z z + ||z‬‬


‫‪ņőƱŀ Ţ‬‬
‫|| ‪= ||z || + 2 Re (z z ) + ||z‬‬

‫ﺑﺎﻟﺘﻌﻮﻳﺾ ﻋﻦ ‪ z‬ﺑـِ ‪ −z‬ﰲ ﻫﺬه اﳌﺘﻄﺎﺑﻘﺔ ﻳﻨﺘﺞ ‪:‬‬

‫|| ‪||z − z || = ||z + (−z )|| = ||z || + 2 Re (z ⋅ −z ) + ||−z‬‬

‫|| ‪= ||z || + 2 Re (−z ⋅ z ) + ||z || = ||z || − 2 Re (z z ) + ||z‬‬

‫و ﺑﺎﳉﻤﻊ ﻃﺮﻓ ًﺎ إﱃ ﻃﺮف ﻧﺠﺪ ‪:‬‬


‫‪ƕ‬‬

‫|| ‪||z + z || + ||z − z || = ||z || + 2 Re (z z ) + ||z || + ||z || − 2 Re (z z ) + ||z‬‬


‫‪VII‬‬

‫|| ‪= 2 ||z || + 2 ||z‬‬


‫■‬
‫‪ũŏ‬‬

‫‪.‬‬
‫𝟗𝟒𝟐‬ ‫‪.‬‬
‫اﳌﺴﺎواة ﰲ اﳌﺘﺒﺎﻳﻨﺔ اﳌﺜﻠﺜﻴﺔ‬
‫ﻟﻴﻜﻦ ∗‪ z ∈ ℂ‬و ‪ . z ∈ ℂ‬أﺛﺒﺖ ّ‬
‫أن ‪:‬‬
‫‪||z + z || = |z| + ||z || ⟺ ∃𝜆 ∈ ℝ+ , z = 𝜆z.‬‬

‫𝟴𝟰𝟯‬

‫‪http ://tinyurl.com/Malki1718‬‬ ‫‪0‬‬


‫‪ .VII‬ﲤﺎرﻳﻦ ﻟﻠﺘﻌﻤﻖ‬
‫‪.‬‬
‫إذا ﻛﺎن ‪ z = 𝜆z‬ﻣﻊ ‪ّ 𝜆 ∈ ℝ+‬‬
‫ﻓﺈن ‪:‬‬ ‫•‬ ‫ﺍﳊﻞّ‪.‬‬

‫‪Ŕž Ŧ Ľ‬‬
‫|‪||z + z || = ||z + 𝜆z|| = ||1 + 𝜆|| ⋅ |z‬‬
‫|‪= (1 + 𝜆) |z‬‬ ‫ّ‬
‫ﻷن ‪ 𝜆 ≥ 0‬ﻣﻨﻪ ‪1 + 𝜆 > 0‬‬

‫‪œǃ‬‬
‫||‪= |z| + 𝜆 |z| = |z| + ||𝜆z‬‬ ‫ّ‬
‫ﻷن ‪𝜆 ≥ 0‬‬
‫|| ‪= |z| + ||z‬‬

‫‪Ŀ‬‬
‫إذا ﻛﺎن || ‪ ||z + z || = |z| + ||z‬ﻓﺈﻧﻪ ﺑﻘﺴﻤﺔ اﻟﻄﺮﻓﲔ ﻋﲆ اﻟﻌﺪد اﳊﻘﻴﻘﻲ )ﻏﲑ اﳌﻌﺪوم( |‪ |z‬ﻳﻨﺘﺞ ‪:‬‬ ‫•‬

‫‪z‬‬
‫= ‪.u‬‬ ‫|‪ ||1 + u|| = 1 + |u‬ﻣﻊ ‪∈ ℂ‬‬
‫‪z‬‬
‫ﻧﻀﻊ ‪ u = a + ıb‬ﻣﻊ ‪ . a, b ∈ ℝ‬ﻟﺪﻳﻨﺎ ‪:‬‬

‫‪||1 + u|| = (a + 1) + b = 1 + a + b + 2a‬‬

‫= )|‪(1 + |u‬‬ ‫‪1+‬‬ ‫‪a +b‬‬ ‫‪=1+a +b +2 a +b‬‬ ‫و‬

‫= ‪ a‬ﻣﻨﻪ ‪ a ≥ 0‬و ﺑﱰﺑﻴﻊ اﻟﻄﺮﻓﲔ ﻳﻨﺘﺞ ‪ a = a + b‬أي‬ ‫‪a +b‬‬ ‫ﻣﻦ اﳌﺴﺎواة |‪ ||1 + u|| = 1 + |u‬ﻧﺴﺘﻨﺘﺞ ّ‬
‫أن‬
‫‪z‬‬ ‫‪+‬‬
‫‪.‬‬ ‫‪ b = 0‬إذن ‪= a ∈ ℝ‬‬
‫‪z‬‬
‫■‬

‫ﻣﺘﻄﺎﺑﻘﺔ ﻣﺘﻮازي اﻷﺿﻼع‬


‫‬ ‫‪.‬‬
‫𝟎𝟓𝟐‬ ‫‪.‬‬

‫|| ‪. ||Z + Z || + ||Z − Z || = 2 ||Z|| + 2 ||Z‬‬ ‫ﻟﻜﻞ ﻋﺪدﻳﻦ ﻣﺮﻛﺒﲔ ‪ Z‬و ‪ ، Z‬ﻳﻜﻮن ‪:‬‬ ‫أﺛﺒﺖ أﻧﻪ ّ‬
‫)إرﺷﺎد ‪ :‬إﺳﺘﻌﻤﻞ اﻟﻌﻼﻗﺔ ‪(||Z|| = ZZ :‬‬
‫‬
‫‪ņőƱŀ Ţ‬‬
‫أﻧﻈﺮ اﻟﺘﻤﺮﻳﻦ ‪ 248‬ﺻﻔﺤﺔ ‪348‬‬ ‫ﻣﺎ ﻫﻮ اﻟﺘﻔﺴﲑ اﳍﻨﺪﳼ ﳍﺬه اﻟﻨﺘﻴﺠﺔ ؟‬

‫ﺍﳊﻞّ‪ .‬ﻟﺪﻳﻨﺎ ‪:‬‬

‫‪||Z + Z || + ||Z − Z || = Z + Z‬‬ ‫‪Z+Z‬‬ ‫‪+ Z−Z‬‬ ‫‪Z−Z‬‬

‫‪= ZZ + ZZ + Z Z + Z Z + ZZ − ZZ − Z Z + Z Z‬‬
‫|| ‪= 2 ||Z|| + 2 ||Z‬‬
‫‪ƕ‬‬
‫‪VII‬‬

‫اﻟﺘﻔﺴﲑ اﳍﻨﺪﳼ ‪ :‬ﻟﻴﻜﻦ ‪ ABCD‬ﻣﺘﻮازي أﺿﻼع‪ ،‬و ﻟﺘﻜﻦ ‪ Z‬ﻻﺣﻘﺔ ‪ AB‬و ‪ Z‬ﻻﺣﻘﺔ ‪ .AD‬اﳌﺘﻄﺎﺑﻘﺔ اﻟﺴﺎﺑﻘﺔ ﺗُﻜﺘﺐ ‪:‬‬
‫‪AC + BD = 2AB + 2AD = AB + BC + CD + DA‬‬
‫أي ‪ :‬ﰲ ﻣﺘﻮﺍﺯﻱ ﺃﺿﻼﻉ‪ ،‬ﳎﻤﻮﻉ ﻣ‪‬ﺮﺑﻌ‪‬ﻲ‪ ‬ﺍﻟﻘﻄﺮﻳﻦ ﻳﺴﺎﻭﻱ ﳎﻤﻮﻉ ﻣﺮﺑﻌﺎﺕ ﺍﻷﺿﻼﻉ )ﺷﻜﻞ ‪.(14.VII‬‬
‫‪ũŏ‬‬

‫ﻣﻼﺣﻈﺔ ‪ ⧏ : 57‬إذا ﻛﺎن ‪ ABCD‬ﻣﺴﺘﻄﻴ ً‬


‫ﻼ ﻓﻤﻦ ﺟﻬﺔ‪ ،‬اﳌﺜﻠﺖ ‪ ABC‬ﻗﺎﺋﻢ اﻟﺰاوﻳﺔ ﻋﻨﺪ اﻟﺮأس ‪ ، B‬و ﻣﻦ ﺟﻬﺔ أﺧﺮى‬
‫ﻧﺺ ﻧﻈﺮﻳﺔ ﻓﻴﺜﺎﻏﻮﺭﺙ‪.‬‬
‫‪ AC = BD‬و اﳌﺴﺎواة اﻟﺴﺎﺑﻘﺔ ﺗُﺼﺒﺢ ‪ 2AC = 2AB +2BC :‬أي ‪ AC = AB +BC‬و ﻫﻮ ّ‬
‫⧐‬
‫■‬

‫𝟵𝟰𝟯‬

‫‪http ://tinyurl.com/Malki1718‬‬ ‫‪0‬‬


‫‪.‬‬
‫‪D‬‬

‫‪Ŕž Ŧ Ľ‬‬
‫|‪|Z‬‬
‫| ‪|Z ′‬‬
‫‪C‬‬

‫‪œǃ‬‬
‫| ‪|Z + Z‬‬
‫‪′‬‬

‫‪A‬‬

‫|‪Z ′‬‬
‫| ‪|Z ′‬‬

‫‪Ŀ‬‬ ‫‪|Z −‬‬


‫|‪|Z‬‬

‫‪B‬‬
‫ﺷﻜﻞ ‪14.VII‬‬

‫‪.‬‬
‫𝟏𝟓𝟐‬ ‫‪.‬‬

‫‪ •1‬أﺛﺒﺖ أﻧّﻪ ﻷﺟﻞ ﻛﻞ ‪. |u| + |v| ≤ |u + v| + |u − v| : (u, v) ∈ ℂ‬‬


‫ﻗﺪّ م ﺗﻔﺴﲑ ًا ﻫﻨﺪﺳﻴﺎ ﳍﺬه اﳌﺘﺒﺎﻳﻨﺔ‪.‬‬

‫‪ •2‬إﺳﺘﻨﺘﺞ أﻧﻪ ﻷﺟﻞ ﻛﻞ ‪: (z , z , z , z ) ∈ ℂ‬‬

‫|| ‪||z || + ||z || + ||z || + ||z || ≤ ||z + z || + ||z + z || + ||z + z || + ||z + z || + ||z + z || + ||z + z‬‬

‫ﺍﳊﻞّ‪ .‬اﳊﻞ ﻳﺮﺗﻜﺰ ﻋﲆ اﳌﺘﺒﺎﻳﻨﺔ اﳌﺜﻠﺜﻴﺔ ‪:‬‬


‫‪ņőƱŀ Ţ‬‬
‫‪ •1‬اﻟﻄﺮﻳﻘﺔ اﻷوﱃ ‪ :‬ﻟﺪﻳﻨﺎ ‪:‬‬

‫|‪2 |u| = ||2u|| = ||(u + v) + (u − v)|| ≤ |u + v| + |u − v‬‬


‫|‪2 |u| = ||2v|| = ||(u + v) − (u − v)|| ≤ |u + v| + |u − v‬‬

‫|‪. |u| + |v| ≤ |u + v| + |u − v‬‬ ‫ﺑﺎﳉﻤﻊ ﻃﺮﻓ ًﺎ إﱃ ﻃﺮف ّ‬


‫ﺛﻢ اﻟﻘﺴﻤﺔ ﻋﲆ ‪ 2‬ﻳﻨﺘﺞ ‪:‬‬
‫اﻟﻄﺮﻳﻘﺔ اﻟﺜﺎﻧﻴﺔ ‪ :‬ﺑﺎﳊﺴﺎب‬
‫اﳌﺘﺒﺎﻳﻨﺔ ﳏﻘﻘﺔ إذا ﻛﺎن ‪ u = 0‬و ﰲ ﻫﺬه اﳊﺎﻟﺔ ﺗﻜﻮن ﻫﻨﺎك ﻣﺴﺎواة إذا و ﻓﻘﻂ إذا ﻛﺎن ‪. v = 0‬‬
‫‪ƕ‬‬

‫‪VII‬‬

‫‪v‬‬
‫= ‪.w‬‬ ‫ﻃﺮﰲ اﳌﺘﺒﺎﻳﻨﺔ ﻋﲆ |‪ |u‬ﻳﻨﺘﺞ ||‪ 1 + |w| ≤ ||1 + w|| + ||1 − w‬ﻣﻊ‬ ‫ﻧﻔﺮض إذن ّ‬
‫أن ‪ . u ≠ 0‬ﺑﻘﺴﻤﺔ َ ْ‬
‫‪u‬‬
‫ﻧﻀﻊ ‪ w = x + ıy‬ﻣﻨﻪ‬
‫‪ũŏ‬‬

‫= )|‪(1 + |w‬‬ ‫‪1+‬‬ ‫‪x +y‬‬

‫‪=1+2‬‬ ‫‪x +y +x +y‬‬

‫‪≤2+2 x +y‬‬ ‫= ‪(b‬‬ ‫‪x +y‬‬ ‫ّ‬


‫)ﻷن ‪ 2ab ≤ a + b‬ﻣﻊ ‪ a = 1‬و‬

‫𝟬𝟱𝟯‬

‫‪http ://tinyurl.com/Malki1718‬‬ ‫‪0‬‬


‫‪ .VII‬ﲤﺎرﻳﻦ ﻟﻠﺘﻌﻤﻖ‬
‫‪.‬‬
‫‪|u‬‬
‫‪−‬‬
‫|‪v‬‬ ‫|‪v‬‬
‫‪|u +‬‬

‫‪Ŕž Ŧ Ľ‬‬
‫|‪|v‬‬

‫‪œǃ‬‬
‫|‪|u‬‬

‫ﺷﻜﻞ ‪15.VII‬‬

‫‪Ŀ‬‬
‫ﻣﻦ ﺟﻬﺔ أﺧﺮى ﻟﺪﻳﻨﺎ ‪:‬‬

‫‪||1 + w|| + ||1 − w|| = (1 + x) + y‬‬ ‫‪+ (1 − x) + y‬‬


‫‪= 1 + 2x + x + y + 1 − 2x + x + y = 2 + 2 x + y‬‬

‫و ﺑﺎﻟﺘﺎﱄ ‪:‬‬

‫||‪(1 + |w|) ≤ ||1 + w|| + ||1 − w|| ≤ ||1 + w|| + ||1 − w‬‬

‫ّ‬
‫)ﻷن )‪ a + b ≤ (a + b‬ﻣﻊ ||‪ a = ||1 + w‬و ||‪(b = ||1 − w‬‬
‫ﺑﺄﺧﺬ اﳉﺬر اﻟﱰﺑﻴﻌﻲ )اﳊﻘﻴﻘﻲ( ﻟﻠﻄﺮﻓﲔ‪ ،‬ﻧﺼﻞ إﱃ اﻟﻨﺘﻴﺠﺔ اﳌﻄﻠﻮﺑﺔ‪.‬‬
‫اﻟﺘﻔﺴﲑ اﳍﻨﺪﳼ ‪ :‬ﰲ ﻣﺘﻮﺍﺯﻱ ﺃﺿﻼﻉ‪ ،‬ﳎﻤﻮﻉ ﻃﻮﻟَﻲ‪ ‬ﺃﻱ‪ ‬ﺿﻠﻌﲔ ﻣﺘﺘﺎﻟﻴﲔ ﻳﻜﻮﻥ ﺃﺻﻐﺮ ﻣﻦ ﳎﻤﻮﻉ ﻃﻮﻟَﻲ‪ ‬ﺍﻟﻘﻄﺮﻳﻦ )ﺷﻜﻞ ‪.(15.VII‬‬

‫‪z‬‬ ‫‪ •2‬ﻧُﻄﺒﻖ ﻧﺘﻴﺠﺔ اﻟﺴﺆال اﻟﺴﺎﺑﻖ ﻋﲆ ‪ z‬و ‪ z‬ﻓﻴﻜﻮن ‪ ||z || + ||z || ≤ ||z + z || + ||z − z || :‬؛ و ﺑﺘﻄﺒﻴﻘﻬﺎ ﻋﲆ ‪ z‬و‬
‫ﻳﻜﻮن ‪ . ||z || + ||z || ≤ ||z + z || + ||z − z || :‬ﻧﺠﻤﻊ ﻫﺎﺗﲔ اﳌﱰاﺟﺤﺘﲔ ﻃﺮﻓ ًﺎ إﱃ ﻃﺮف ‪:‬‬
‫|| ‪||z || + ||z || + ||z || + ||z || ≤ ||z + z || + ||z − z || + ||z + z || + ||z − z‬‬

‫ﻣﺮة أﺧﺮى ﻋﲆ ‪ z − z‬و ‪ z − z‬ﻓﻴﻨﺘﺞ ‪:‬‬


‫ﻧُﻄﺒﻖ اﻟﻨﺘﻴﺠﺔ ّ‬
‫‪ņőƱŀ Ţ‬‬
‫|| ‪||z − z || + ||z − z || ≤ ||z − z + z − z || + ||z − z − z + z‬‬
‫||) ‪= ||(z + z ) − (z + z )|| + ||(z + z ) − (z + z‬‬
‫|| ‪≤ ||z + z || + ||z + z || + ||z + z || + ||z + z‬‬

‫ﰲ اﻷﺧﲑ ‪:‬‬
‫|| ‪||z || + ||z || + ||z || + ||z || ≤ ||z + z || + ||z + z || + ||z + z || + ||z + z || + ||z + z || + ||z + z‬‬
‫‪ƕ‬‬

‫■‬
‫‪VII‬‬

‫‬ ‫‪.‬‬
‫𝟐𝟓𝟐‬ ‫‪.‬‬
‫‪ũŏ‬‬

‫ﻟﺘﻜﻦ 𝕌 ﳎﻤﻮﻋﺔ اﻷﻋﺪاد اﳌﺮﻛﺒﺔ اﻟﺘﻲ ﻃﻮﻳﻠﺘﻬﺎ ‪. 1‬‬


‫‪z+1‬‬
‫‪.‬‬ ‫ﻟﻴﻜﻦ }‪ . z ∈ 𝕌 ⧵ {1‬أﺛﺒﺖ ّ‬
‫أن ‪∈ ıℝ :‬‬
‫‪z−1‬‬

‫ﺍﳊﻞّ‪ .‬ﻧﻀﻊ‬
‫‪z+1‬‬
‫أن ‪ Z‬ﲣﻴﲇ ﴏف‪ ،‬ﻳﻜﻔﻲ أن ﻧﱪﻫﻦ ّ‬
‫أن ‪. Z = −Z :‬‬ ‫= ‪ . Z‬ﻹﺛﺒﺎت ّ‬
‫‪z−1‬‬

‫𝟭𝟱𝟯‬

‫‪http ://tinyurl.com/Malki1718‬‬ ‫‪0‬‬


‫‪.‬‬
‫ﻓﺈن ‪ّ z = 1‬‬
‫)ﻷن ‪ (z ⋅ z = |z| = 1‬ﻣﻨﻪ ‪:‬‬ ‫أن 𝕌 ∈ ‪ّ z‬‬
‫ﻟﻜﻦ‪ ،‬ﺑﲈ ّ‬
‫‪z‬‬

‫‪Ŕž Ŧ Ľ‬‬
‫‪z+1‬‬ ‫‪z+1‬‬ ‫‪+1‬‬ ‫‪1+z‬‬
‫=‪Z‬‬ ‫=‬ ‫=‬ ‫=‬ ‫‪= −Z‬‬
‫‪z−1‬‬ ‫‪z−1‬‬ ‫‪−1 1−z‬‬

‫‪œǃ‬‬
‫■‬ ‫و ﻫﺬا ﻳﻌﻨﻲ ّ‬
‫أن ‪ Z‬ﲣﻴﲇ ﴏف أي ‪. Z ∈ ıℝ‬‬

‫‪.‬‬
‫𝟑𝟓𝟐‬ ‫‪.‬‬

‫‪Ŀ‬‬
‫‪z‬‬ ‫∗‬ ‫∗‬
‫‪.‬‬ ‫‪=0‬‬ ‫ﻟﻴﻜﻦ ‪ . z , z , ⋯ , z ∈ ℂ ، n ∈ ℕ‬ﻧﻔﺮض ّ‬
‫أن‬
‫|| ‪||z‬‬
‫=‬

‫‪z‬‬
‫‪. ∀z ∈ ℂ ,‬‬ ‫= || ‪||z‬‬ ‫)‪(z − z‬‬ ‫‪ •1‬أﺛﺒﺖ أﻧﻪ ‪:‬‬
‫|| ‪||z‬‬
‫=‬ ‫=‬

‫‪. ∀z ∈ ℂ ,‬‬ ‫≤ || ‪||z‬‬ ‫||‪||z − z‬‬ ‫‪ •2‬إﺳﺘﻨﺘﺞ أﻧّﻪ ‪:‬‬


‫=‬ ‫=‬

‫‪ •1‬ﻧﻨﻄﻠﻖ ﻣﻦ اﻟﻄﺮف اﻷﻛﺜﺮ ﺗﻌﻘﻴﺪ ًا أي اﻟﻄﺮف اﻟﺜﺎﲏ ‪:‬‬ ‫ﺍﳊﻞّ‪.‬‬

‫‪0‬‬
‫|| ‪||z‬‬ ‫⎛‬ ‫‬
‫⎞‬
‫‪z‬‬ ‫‪z z‬‬ ‫‪zz‬‬ ‫‪z‬‬ ‫‪z‬‬
‫)‪(z − z‬‬ ‫=‬ ‫‪−‬‬ ‫=‬ ‫‪−z‬‬ ‫=‬ ‫⎜‪z‬‬
‫‪||z || − gray‬‬
‫⎜‬
‫⎟‬
‫|| ‪||z‬‬ ‫|| ‪||z‬‬ ‫|| ‪||z‬‬ ‫|| ‪||z‬‬ ‫|| ‪||z‬‬ ‫⎟ || ‪||z‬‬
‫=‬ ‫=‬ ‫=‬ ‫=‬ ‫=‬ ‫=‬ ‫=‬
‫⎝‬ ‫⎠‬
‫=‬ ‫= ‪||z || − z × 0‬‬ ‫|| ‪||z‬‬
‫=‬ ‫=‬

‫|‬ ‫|‬
‫‪z‬‬ ‫‪z‬‬ ‫‪z‬‬
‫‪ņőƱŀ Ţ‬‬
‫|‬ ‫|‬ ‫ّ‬ ‫‪||z || ∈ ℝ+‬‬ ‫‪ •2‬ﺑﲈ ّ‬
‫ﻣﻨﻪ )ﺑﺘﻄﺒﻴﻖ‬ ‫|‬ ‫)‪(z − z‬‬
‫‪||z‬‬ ‫| ||‬
‫=‬ ‫)‪(z − z‬‬
‫|| ‪||z‬‬
‫ﻓﺈن‬ ‫)‪(z − z‬‬
‫|| ‪||z‬‬
‫=‬ ‫أن‬
‫||‬ ‫=‬ ‫||‬ ‫=‬ ‫=‬ ‫=‬
‫اﳌﻌﻤﻤﺔ( ‪:‬‬
‫اﳌﺘﺒﺎﻳﻨﺔ اﳌﺜﻠﺜﻴﺔ ّ‬
‫|‬ ‫|‬ ‫|‬ ‫|‬
‫‪z‬‬ ‫‪z‬‬ ‫‪z‬‬ ‫| ‪|z‬‬
‫= || ‪||z‬‬ ‫)‪(z − z‬‬ ‫=‬
‫|‬
‫)‪(z − z‬‬
‫|‬
‫≤‬ ‫)‪|(z − z‬‬ ‫=|‬ ‫= | | ||‪||z − z‬‬ ‫||‪||z − z‬‬
‫| || ‪||z‬‬ ‫‪||z‬‬ ‫| ||‬ ‫||‬ ‫‪||z‬‬ ‫|| ||‬ ‫|| ‪||z‬‬
‫=‬ ‫=‬ ‫||‬ ‫=‬ ‫||‬ ‫=‬ ‫=‬ ‫=‬

‫و ﻫﻮ اﳌﻄﻠﻮب‪.‬‬
‫■‬
‫‪ƕ‬‬

‫‪VII‬‬

‫‪.‬‬
‫𝟒𝟓𝟐‬ ‫‪.‬‬
‫ﻟﻴﻜﻦ ∗‪ n ∈ ℕ‬ﻋﺪد ﻃﺒﻴﻌﻲ ﻏﲑ ﻣﻌﺪوم‪ .‬ﻧﻀﻊ }‪. D = {z ∈ ℂ : |z| ≤ 1‬‬
‫‪ũŏ‬‬

‫ﻟﻴﻜﻦ ‪ . a , a , ⋯ , a , b , b , ⋯ , b ∈ D‬أﺛﺒﺖ ّ‬
‫أن ‪:‬‬
‫|‬ ‫|‬
‫|‬ ‫|‬ ‫|| ‪||a − b‬‬
‫|‬ ‫‪a −‬‬ ‫≤| ‪b‬‬
‫||‬ ‫=‬ ‫=‬ ‫||‬ ‫=‬

‫ﺍﳊﻞّ‪ .‬ﻧﺴﺘﻌﻤﻞ اﻟﱪﻫﺎن ﺑﺎﻟﱰاﺟﻊ ﻋﲆ اﻟﻌﺪد اﻟﻄﺒﻴﻌﻲ ﻏﲑ اﳌﻌﺪوم ‪. n‬‬

‫𝟮𝟱𝟯‬

‫‪http ://tinyurl.com/Malki1718‬‬ ‫‪0‬‬


‫‪ .VII‬ﲤﺎرﻳﻦ ﻟﻠﺘﻌﻤﻖ‬
‫‪.‬‬
‫ﻣﻦ اﻟﻮاﺿﺢ ّ‬
‫أن اﳋﺎﺻﻴﺔ ﺻﺤﻴﺤﺔ ﻷﺟﻞ ‪) n = 1‬ﺗُﻜﺘﺐ ‪. (||a − b || ≤ ||a − b || :‬‬ ‫•‬

‫‪Ŕž Ŧ Ľ‬‬
‫ﻷﺟﻞ ‪ ، n = 2‬ﳚﺐ أن ﻧُﱪﻫﻦ ّ‬
‫أن ‪ . ||a a − b b || ≤ ||a − b || + ||a − b || :‬ﻟﻜﻦ ‪:‬‬ ‫•‬

‫|| ‪||a a − b b || = ||a (a − b ) + (a − b ) b‬‬

‫‪œǃ‬‬
‫|| ‪≤ ||a (a − b )|| + ||(a − b ) b || = ||a || ||a − b || + ||a − b || ||b‬‬
‫|| ‪≤ ||a − b || + ||a − b‬‬

‫إذن ﻓﺎﳋﺎﺻﻴﺔ ﺻﺤﻴﺤﺔ ﻷﺟﻞ ‪. n = 2‬‬ ‫ّ‬


‫)ﻷن ‪ ||a || ≤ 1‬و ‪(||b || ≤ 1‬‬

‫‪Ŀ‬‬
‫أن اﳋﺎﺻﻴﺔ ﺻﺤﻴﺤﺔ ﻷﺟﻞ ∗‪ n ∈ ℕ‬و ﻧﱪﻫﻦ أﳖﺎ ﺗﺒﻘﻰ ﺻﺤﻴﺤﺔ ﻷﺟﻞ ‪. n + 1‬‬
‫ﻧﻔﺮض ّ‬ ‫•‬

‫= ‪ . B‬ﻟﺪﻳﻨﺎ ‪:‬‬ ‫‪b‬‬ ‫= ‪A‬و‬ ‫‪a‬‬ ‫‪ . a , a , ⋯ , a , a‬ﻧﻀﻊ‬ ‫‪+‬‬ ‫‪,b ,b ,⋯,b ,b‬‬ ‫‪+‬‬ ‫ﻟﻴﻜﻦ ‪∈ D‬‬
‫=‬ ‫=‬
‫|‬ ‫‪+‬‬ ‫‪+‬‬ ‫|‬
‫|‬ ‫|‬
‫|‬ ‫‪a −‬‬ ‫‪b | = ||A a + − B b‬‬ ‫‪+‬‬
‫‪|| = ||A‬‬ ‫‪a‬‬ ‫‪+‬‬ ‫‪−b‬‬ ‫‪+‬‬ ‫‪+ (A − B ) b‬‬ ‫‪+‬‬
‫||‬
‫||‬ ‫=‬ ‫=‬ ‫||‬
‫‪≤ ||A || ⋅ ||a + − b‬‬ ‫‪+‬‬
‫‪|| + ||A − B || ⋅ ||b‬‬ ‫‪+‬‬
‫||‬

‫‪ (b‬و ‪ّ ||a || ≤ 1‬‬


‫ﻟﻜﻞ }‪ k ∈ {1, 2, ⋯ , n‬ﻣﻨﻪ ‪ ||A || ≤ 1‬و ﺣﺴﺐ ﻓﺮﺿﻴﺔ اﻟﱰاﺟﻊ‬ ‫‪+‬‬ ‫‪ّ ||b‬‬
‫)ﻷن ‪∈ D‬‬ ‫‪+‬‬
‫ﻟﻜﻦ ‪|| ≤ 1‬‬

‫≤ || ‪ ||A − B‬إذن ‪:‬‬ ‫ّ‬


‫ﻓﺈن ‪||a − b || :‬‬
‫=‬
‫|‬ ‫‪+‬‬ ‫‪+‬‬ ‫|‬ ‫‪+‬‬
‫|‬ ‫|‬
‫|‬ ‫‪a −‬‬ ‫‪b | ≤ ||a‬‬ ‫‪+‬‬ ‫‪−b‬‬ ‫‪+‬‬
‫‪|| +‬‬ ‫= || ‪||a − b‬‬ ‫|| ‪||a − b‬‬
‫||‬ ‫=‬ ‫=‬ ‫||‬ ‫=‬ ‫=‬

‫و ﻫﺬا ﻳﻌﻨﻲ ّ‬
‫أن اﳋﺎﺻﻴﺔ ﺗﺒﻘﻰ ﺻﺤﻴﺤﺔ ﻋﻨﺪ اﻟﺮﺗﺒﺔ ‪. n + 1‬‬
‫■‬ ‫ﻟﻜﻞ ﻋﺪد ﻃﺒﻴﻌﻲ ﻏﲑ ﻣﻌﺪوم ∗‪. n ∈ ℕ‬‬
‫ﺣﺴﺐ ﻣﺒﺪأ اﻟﱪﻫﺎن ﺑﺎﻟﱰاﺟﻊ‪ ،‬ﻓﺎﳋﺎﺻﻴﺔ ﺻﺤﻴﺤﺔ ّ‬
‫‪ņőƱŀ Ţ‬‬
‫‪$‬‬ ‫‪.‬‬
‫𝟓𝟓𝟐‬ ‫‪.‬‬
‫‪ •1‬ﻟﻴﻜﻦ ‪ z‬و ‪ z‬ﻋﺪدﻳﻦ ﻣﺮﻛﺒﲔ ﻏﲑ ﻣﻌﺪوﻣﲔ ﻃﻮﻳﻠﺘﻬﲈ أﺻﻐﺮ ﻣﻦ أو ﺗﺴﺎوي ‪. 1‬‬
‫𝜋 𝜋‬
‫‪ . arg (z) − arg z ∈ − ,‬أﺛﺒﺖ ّ‬
‫أن ‪. ||z − z || ≤ 1 :‬‬ ‫ﻧﻔﺮض ّ‬
‫أن )𝜋‪(mod 2‬‬
‫‪3 3‬‬

‫‪ •2‬إذا ُأﻋﻄِﻴﺖ ﺳﺘﺔ أﻋﺪاد ﻣﺮﻛﺒﺔ ﻏﲑ ﻣﻌﺪوﻣﺔ ﻃﻮﻳﻠﺘﻬﺎ أﺻﻐﺮ ﻣﻦ أو ﺗﺴﺎوي ‪ 1‬ﻓﺄﺛﺒﺖ أﻧﻪ ﻳﻮﺟﺪ ﻋﲆ اﻷﻗﻞ إﺛﻨﲔ ﻣﻨﻬﲈ‬
‫ﳛﻘﻘﺎن ﻓﺮﺿﻴﺔ اﻟﺴﺆال ‪. •1‬‬
‫‪ •3‬ﻟﺘﻜﻦ ‪ z ، ⋯ ، z ، z‬أﻋﺪاد ًا ﻣﺮﻛﺒﺔ ﻃﻮﻳﻠﺘﻬﺎ أﺻﻐﺮ ﻣﻦ أو ﺗﺴﺎوي ‪ . 1‬ﺑﺎﺳﺘﺨﺪام ﻧﺘﻴﺠﺔ اﻟﺴﺆال ‪ ، •2‬أﺛﺒﺖ أﻧﻪ إذا‬
‫ﻛﺎن ‪ n ≥ 3‬ﻓﺈﻧﻪ ﻳﻮﺟﺪ ‪ i ≠ j‬و }‪ 𝜀 ∈ {−1, 1‬ﺑﺤﻴﺚ ‪. ||z + 𝜀z || ≤ 1‬‬
‫‪ƕ‬‬
‫‪VII‬‬

‫∗‬
‫أﺛﺒﺖ أﻧﻪ ﻷﺟﻞ ﻛﻞ ﻋﺪد ﻃﺒﻴﻌﻲ ‪ n ∈ ℕ‬و ﻟﻜﻞ أﻋﺪاد ﻣﺮﻛﺒﺔ ‪ z ، ⋯ ، z ، z‬ﻃﻮﻳﻠﺘﻬﺎ أﺻﻐﺮ ﻣﻦ أو ﺗﺴﺎوي ‪1‬‬ ‫‪4‬‬
‫•‬
‫|‬ ‫|‬
‫ﻓﺈﻧﻪ ﺗﻮﺟﺪ أﻋﺪاد ﺻﺤﻴﺤﺔ }‪ 𝜀 , 𝜀 , ⋯ , 𝜀 ∈ {−1, 1‬ﺑﺤﻴﺚ ‪. | 𝜀 z || ≤ √2 :‬‬
‫|‬
‫‪ũŏ‬‬

‫= ||‬ ‫||‬
‫|‬ ‫|‬
‫ﻫﻞ ﻳﻤﻜﻦ ﲢﺴﲔ ﻫﺬا اﻟﺘﺤﺪﻳﺪ ؟ ﺑﻤﻌﻨﻰ ﻫﻞ ﻳﻤﻜﻦ إﳚﺎد ﻋﺪد ‪ 0 ≤ 𝜆 < √2‬ﺑﺤﻴﺚ 𝜆 ≤ || ‪. || 𝜀 z‬‬
‫= ||‬ ‫||‬

‫‪ •5‬ﻣﺎ ﻫﻮ اﻟﺘﺤﺪﻳﺪ اﻷﻣﺜﻞ إذا ﻛﺎﻧﺖ اﻷﻋﺪاد ‪ z‬ﺣﻘﻴﻘﻴ ًﺔ ؟‬

‫𝟯𝟱𝟯‬

‫‪http ://tinyurl.com/Malki1718‬‬ ‫‪0‬‬


‫‪.‬‬
‫‪ •1‬ﻧﻌﻠﻢ ّ‬
‫أن ‪ ||z − z || = |z| + ||z || − 2 |z| ⋅ ||z || cos 𝜃 :‬ﺣﻴﺚ ‪. 𝜃 = arg (z) − arg z‬‬ ‫ﺍﳊﻞّ‪.‬‬

‫‪Ŕž Ŧ Ľ‬‬
‫‪1‬‬
‫إذا ﻛﺎن 𝜋 ‪ّ 𝜃 ∈ − 𝜋 ,‬‬
‫ﻓﺈن ≥ 𝜃 ‪ cos‬ﻣﻨﻪ || ‪. ||z − z || ≤ |z| + ||z || − |z| ⋅ ||z‬‬
‫‪2‬‬ ‫‪3 3‬‬
‫أن || ‪ |z| ≥ ||z‬ﻣﻨﻪ || ‪ |z| ⋅ ||z || ≥ ||z‬و ﺑﺎﻟﺘﺎﱄ ّ‬
‫ﺑﺘﺒﺪﻳﻞ دور ﻛﻞ ﻣﻦ ‪ z‬و ‪ z‬إذا ﻟﺰم اﻷﻣﺮ‪ ،‬ﻳﻤﻜﻦ أن ﻧﻔﺮض ّ‬

‫‪œǃ‬‬
‫ﻓﺈن ‪:‬‬
‫‪||z − z || ≤ |z| ≤ 1‬‬

‫‪ •2‬ﻟﺘﻜﻦ ‪ ، 1 ≤ i ≤ 6 ، z‬ﺳﺘﺔ أﻋﺪاد ﻣﺮﻛﺒﺔ ﻃﻮﻳﻠﺘﻬﺎ أﺻﻐﺮ ﻣﻦ أو ﺗﺴﺎوي ‪ . 1‬ﻧﻀﻊ )𝜋‪ 𝜃 = arg (z ) (mod 2‬ﻣﻊ‬

‫‪Ŀ‬‬
‫[𝜋‪. 𝜃 ∈ [0, 2‬‬
‫أن 𝜋‪. 0 ≤ 𝜃 ≤ 𝜃 ≤ ⋯ ≤ 𝜃 ≤ 2‬‬ ‫ﺑﺘﺒﺪﻳﻞ دور اﻷﻋﺪاد ‪ z‬إذا ﻟﺰم اﻷﻣﺮ‪ ،‬ﻳﻤﻜﻦ أن ﻧﻔﺮض ّ‬

‫‪+‬‬ ‫إذا ُو ِﺟﺪ }‪ i ∈ {1, 2, 3, 4, 5‬ﺑﺤﻴﺚ 𝜋 ≤ 𝜃 ‪ّ 𝜃 −‬‬


‫ﻓﺈن اﻟﻌﺪدﻳﻦ ‪ z‬و ‪ z‬ﳛﻘﻘﺎن اﳌﻄﻠﻮب ‪.‬‬ ‫‪+‬‬
‫‪3‬‬
‫𝜋‬ ‫𝜋‬ ‫𝜋‬ ‫𝜋‬ ‫𝜋‬
‫> 𝜃 ‪ 𝜃 −‬ﻣﻨﻪ‬ ‫> 𝜃‪𝜃 −‬و‬ ‫> 𝜃‪𝜃 −‬و‬ ‫> 𝜃‪𝜃 −‬و‬ ‫> 𝜃‪𝜃 −‬و‬ ‫ﰲ اﳊﺎﻟﺔ اﳌﻐﺎﻳﺮة‪ ،‬ﻳﻜﻮن‬
‫‪3‬‬ ‫‪3‬‬ ‫‪3‬‬ ‫‪3‬‬ ‫‪3‬‬
‫𝜋 𝜋 𝜋 𝜋 𝜋‬
‫> ) 𝜃 ‪(𝜃 − 𝜃 ) + (𝜃 − 𝜃 ) + (𝜃 − 𝜃 ) + (𝜃 − 𝜃 ) + (𝜃 −‬‬ ‫‪+ + + +‬‬
‫‪3 3 3 3 3‬‬
‫𝜋‪5‬‬
‫> 𝜃‪𝜃 −‬‬ ‫أي‬
‫‪3‬‬
‫𝜋‪5‬‬
‫‪𝜃 >𝜃 +‬‬ ‫أي‬
‫‪3‬‬
‫ﻣﻨﻪ‬
‫𝜋‪𝜋 5‬‬ ‫𝜋‬
‫‪𝜃 ≤ 2𝜋 ≤ 2𝜋 + 𝜃 ≤ +‬‬ ‫‪+𝜃 ≤𝜃 +‬‬
‫‪3‬‬ ‫‪3‬‬ ‫‪3‬‬
‫أي ‪ z‬و ‪ z‬ﳛﻘﻘﺎن اﳌﻄﻠﻮب‪.‬‬
‫ﻣﻼﺣﻈﺔ ‪ ⧏ : 58‬ﻳﻤﻜﻦ إﺛﺒﺎت ﻫﺬه اﻟﻨﺘﻴﺠﺔ ﻫﻨﺪﺳﻴ ًﺎ و ذﻟﻚ ﺑﻤﻼﺣﻈﺔ ّ‬
‫أن اﻟﻘﻄﺎﻋﺎت اﻟﺪاﺋﺮﻳﺔ )و ﻋﺪدﻫﺎ ﺳﺘﺔ(‬
‫𝜋‬
‫اﳌﺘﻤﺮﻛﺰة ﻋﲆ اﻷﻋﺪاد ‪ z‬و ﻗﻴﺎﺳﻬﺎ ﻻ ﻳﻤﻜﻦ أن ﺗﻜﻮن ﻣﻨﻔﺼﻠﺔ ﻣﺜﻨﻰ ﻣﺜﻨﻰ ﻷﻧﻪ ﰲ ﻫﺬه اﳊﺎﻟﺔ ﺳﺘﺸﻜﱢﻞ زاوﻳﺔ إﲨﺎﻟﻴﺔ‬
‫‪3‬‬
‫𝜋‬
‫×‪.6‬‬ ‫ﻗﻴﺎﺳﻬﺎ أﻛﱪ ﲤﺎﻣ ًﺎ ﻣﻦ 𝜋‪= 2‬‬
‫‪3‬‬
‫‪ņőƱŀ Ţ‬‬
‫⧐‬

‫‪ •3‬إذا ﻛﺎن أﺣﺪ اﻷﻋﺪاد ‪ z‬ﻣﻌﺪوﻣ ًﺎ ﻓﺎﳋﺎﺻﻴﺔ ﳏﻘﻘﺔ‪.‬‬


‫أن ‪ 2n ≥ 6‬ﻓﺤﺴﺐ‬ ‫ﰲ اﳊﺎﻟﺔ اﳌﻐﺎﻳﺮة‪ ،‬ﺑﺎﻋﺘﺒﺎر اﻷﻋﺪاد ‪ z‬و ‪ −z‬ﻣﻊ ‪ 1 ≤ i ≤ n‬ﻧﺤﺼﻞ ﻋﲆ ‪ 2n‬ﻋﺪد ًا ﻣﺮﻛﺒﺎ‪ ،‬و ﺑﲈ ّ‬
‫اﻟﺴﺆال ‪ ،•2‬ﻳﻮﺟﺪ ﻋﲆ اﻷﻗﻞ ﻋﺪدان ﻣﻦ ﺑﲔ ﻫﺬه اﻷﻋﺪاد ﺑﺤﻴﺚ ﺗﻜﻮن اﻟﻘﻴﻤﺔ اﳌﻄﻠﻘﺔ ﻟﻠﻔﺮق ﺑﲔ ﻋﻤﺪﺗ ْﻴﻬﲈ ﻻ ﺗﺘﺠﺎوز‬
‫𝜋‬
‫‪ .‬ﻫﺬان اﻟﻌﺪدان ﻻ ﻳﻤﻜﻦ أن ﻳﻜﻮﻧﺎ ﻣﺘﻌﺎﻛﺴﲔ و ﺑﺎﻟﺘﺎﱄ ﻓﻬﲈ ﻣﻦ اﻟﺸﻜﻞ ‪ 𝜀 z‬و ‪ 𝜀 z‬ﻣﻊ ‪ i ≠ j‬و }‪ 𝜀 ∈ {−1, 1‬و‬
‫‪3‬‬
‫}‪. 𝜀 ∈ {−1, 1‬‬
‫ﻧﻀﻊ 𝜀 𝜀‪ z = 𝜀 z ، 𝜀 = −‬و ‪ z = 𝜀 z‬ﻓﻴﻜﻮن ‪ ||z + 𝜀z || = ||𝜀 z − 𝜀 z || = ||z − z || ≤ 1‬ﺣﺴﺐ اﻟﺴﺆال‬
‫‪. •1‬‬
‫‪ƕ‬‬

‫‪VII‬‬

‫‪ •4‬ﻧﱪﻫﻦ ﻫﺬه اﳋﺎﺻﻴﺔ ﺑﺎﻟﱰاﺟﻊ ﻋﲆ اﻟﻌﺪد اﻟﻄﺒﻴﻌﻲ ﻏﲑ اﳌﻌﺪوم ‪.n ≥ 1‬‬

‫أي ﻋﺪد ‪ z‬ﻃﻮﻳﻠﺘﻪ أﺻﻐﺮ ﻣﻦ أو ﺗﺴﺎوي ‪ُ 1‬ﳛﻘﻖ ‪. |z| ≤ 1 < √2‬‬ ‫اﳋﺎﺻﻴﺔ ﺑﺪﳞﻴﺔ ﻷﺟﻞ ‪ّ n = 1‬‬
‫ﻷن ّ‬ ‫•‬
‫‪ũŏ‬‬

‫ﻷﺟﻞ ‪ n = 2‬ﻟﺪﻳﻨﺎ ‪:‬‬ ‫•‬


‫|| ‪||z + z || + ||z − z || = 2 ||z || + ||z‬‬ ‫‪≤2 1 +1‬‬ ‫‪=4‬‬

‫|| ‪ ||z − z‬ﻳﻜﻮن أﺻﻐﺮ ﻣﻦ أو ﻳﺴﺎوي ‪2‬‬ ‫و ﺑﺎﻟﺘﺎﱄ ّ‬


‫ﻓﺈن )ﻋﲆ اﻷﻗﻞ( أﺣﺪ اﻟﻌﺪدﻳﻦ || ‪ ||z + z‬و‬
‫ﻣﻨﻪ ‪ ||z + z || ≤ √2‬أو ‪ ||z − z || ≤ √2‬إذن اﳋﺎﺻﻴﺔ ﺻﺤﻴﺤﺔ ﻷﺟﻞ ‪. n = 2‬‬

‫𝟰𝟱𝟯‬

‫‪http ://tinyurl.com/Malki1718‬‬ ‫‪0‬‬


‫‪ .VII‬ﲤﺎرﻳﻦ ﻟﻠﺘﻌﻤﻖ‬
‫‪.‬‬
‫ﻧﻔﺮض ‪ n ≥ 3‬و اﳋﺎﺻﻴﺔ ﺻﺤﻴﺤﺔ ﻋﻨﺪ اﻟﺮﺗﺒﺔ ‪. n − 1‬‬ ‫•‬

‫ﺣﺴﺐ اﻟﺴﺆال ‪ ،•3‬ﻳﻮﺟﺪ ‪ i ≠ j‬و }‪ 𝜀 ∈ {−1, 1‬ﺑﺤﻴﺚ ‪ . ||z + 𝜀z || ≤ 1‬ﺑﺘﺒﺪﻳﻞ ﺗﺮﺗﻴﺐ اﻷﻋﺪاد ‪ z‬إذا ﻟﺰم‬

‫‪Ŕž Ŧ Ľ‬‬
‫أن ﻫﺬﻳﻦ اﻟﻌﺪدﻳﻦ ﳘﺎ ‪ z −‬و ‪. z‬‬‫اﻷﻣﺮ‪ ،‬ﻳﻤﻜﻦ أن ﻧﻔﺮض ّ‬
‫ﺑﻮﺿﻊ ‪ Z = z − + 𝜀z‬ﻳﻜﻮن ‪ ||Z|| ≤ 1‬و ﺑﺘﻄﺒﻴﻖ ﻓﺮﺿﻴﺔ اﻟﱰاﺟﻊ ﻋﲆ اﻷﻋﺪاد ‪ z − ، ⋯ ، z ، z‬و ‪) Z‬و‬

‫‪œǃ‬‬
‫|‬ ‫‪−‬‬ ‫|‬
‫|| ‪.‬‬ ‫|‬
‫ﻋﺪدﻫﺎ ﻫﻮ ‪ (n − 1‬ﻧﺠﺪ أﻧﻪ ﻳﻮﺟﺪ }‪ 𝜀 , 𝜀 , ⋯ , 𝜀 − , 𝛼 ∈ {−1, 1‬ﺑﺤﻴﺚ ‪𝜀 z + 𝛼Z| ≤ √2‬‬
‫||‬
‫=‬ ‫||‬
‫|‬ ‫|‬

‫‪Ŀ‬‬
‫ﻧﻀﻊ 𝛼 = ‪ 𝜀 −‬و 𝛼𝜀 = 𝜀 ﻓﻴﻜﻮن }‪ 𝜀 − , 𝜀 ∈ {−1, 1‬و ‪ || 𝜀 z || ≤ √2‬و ﺑﺎﻟﺘﺎﱄ ﻓﺎﳋﺎﺻﻴﺔ ﺗﺒﻘﻰ‬
‫= ||‬ ‫||‬
‫ﺻﺤﻴﺤﺔ ﻋﻨﺪ اﻟﺮﺗﺒﺔ ‪. n‬‬
‫ﺣﺴﺐ ﻣﺒﺪأ اﻟﱪﻫﺎن ﺑﺎﻟﱰاﺟﻊ‪ ،‬اﳋﺎﺻﻴﺔ ﺻﺤﻴﺤﺔ ﻟﻜﻞ ﻋﺪد ﻃﺒﻴﻌﻲ ﻏﲑ ﻣﻌﺪوم‪.‬‬
‫ﻻ ﻳﻤﻜﻦ ﲢﺴﲔ ﻫﺬا اﻟﺘﺤﺪﻳﺪ ﻷﻧﻪ ﻷﺟﻞ ‪ n = 2‬و ‪ z = e ، 𝜃 ∈ ℝ‬و ‪ z = e +‬ﻳﻜﻮن‬
‫‪ ||z || = ||z || = 1 ≤ 1‬و ‪:‬‬
‫‪||z − z || = ||e‬‬ ‫‪/‬‬
‫‪|| = ||e || ⋅ ||1 + ı|| = 1 × √2 = 2‬‬
‫‪1+e‬‬
‫أي ‪. ||z − z || = √2‬‬
‫ﻣﻼﺣﻈﺔ ‪ ⧏ : 59‬ﻗﺪ ﻧﺘﺴﺎءل ﻛﻴﻒ ﺗﻢ اﺧﺘﻴﺎر اﻟﻌﺪدﻳﻦ ‪ z‬و ‪ z‬؟ اﳉﻮاب ﺗﻘﺪﻣﻪ ﻟﻨﺎ ﻧﻈﺮﻳﺔ ﻓﻴﺜﺎﻏﻮﺭﺙ و‬
‫اﻟﺘﻲ ﻫﻲ ﺣﺎﻟﺔ ﺧﺎﺻﺔ ﻣﻦ ﻧﻈﺮﻳﺔ ﺍﻟﻜﺎﺷﻲ و ﻫﺬه اﻷﺧﲑة ّ‬
‫ﺗﻨﺺ ﻋﲆ ّ‬
‫أن )ﺑﺎﺳﺘﻌﲈل اﻷﻋﺪاد اﳌﺮﻛﺒﺔ( ‪:‬‬
‫𝜃 ‪ ||z − z || = ||z || + ||z || − 2 ||z || ⋅ ||z || cos‬ﻣﻊ ) ‪. 𝜃 = arg (z ) − arg (z‬‬
‫𝜋‬
‫)ﻷن ‪ arg (z ) = arg (z ) +‬ﻳﻌﻨﻲ ‪(z = ız‬‬‫إذا ﻛﺎن ‪ cos 𝜃 = 0‬أي إذا ﻛﺎن اﻟﻌﺪدان ‪ z‬و ‪ z‬ﻣﺘﻌﺎﻣﺪﻳﻦ ّ‬
‫‪2‬‬
‫ﻓﺈن || ‪ ||z − z || = ||z || + ||z‬و ﻫﻲ ﻧﻈﺮﻳﺔ ﻓﻴﺜﺎﻏﻮﺭﺙ‪.‬‬
‫ّ‬
‫أن ‪) −2 ||z || ⋅ ||z || cos 𝜃 ≤ 0‬إذا ﻛﺎن 𝜋 ≤ 𝜃 ≤ 𝜋 ‪ (−‬أي || ‪ّ ||z − z || ≤ ||z || + ||z‬‬
‫ﻓﺈن‬ ‫ﺑﻤﻼﺣﻈﺔ ّ‬
‫‪2‬‬ ‫‪2‬‬
‫|‬ ‫|‬
‫اﳊﺼﻮل ﻋﲆ أﻛﱪ ﻗﻴﻤﺔ ﻟﻠﻤﻘﺪار | ‪ |z − z‬ﻳﻜﻮن ﻋﻨﺪﻣﺎ ‪ cos 𝜃 = 0‬أي ﻋﻨﺪﻣﺎ ﻳﻜﻮن ‪ z‬و ‪ z‬ﻣﺘﻌﺎﻣﺪﻳﻦ‪⧐ .‬‬
‫‪ •5‬إذا ﻛﺎن ‪ x‬و ‪ y‬ﻋﺪدﻳﻦ ﺣﻘﻴﻘﻴﲔ ﺑﺤﻴﺚ ‪ |x| ≤ 1‬و ‪ ||y|| ≤ 1‬أي ﺑﺤﻴﺚ ]‪ّ x, y ∈ [−1, 1‬‬
‫ﻓﺈن ﻋﲆ اﻷﻗﻞ أﺣﺪ اﻟﻌﺪدﻳﻦ‬
‫‪ņőƱŀ Ţ‬‬
‫‪ x + y‬أو ‪ x − y‬ﻳﻨﺘﻤﻲ إﱃ اﳌﺠﺎل ]‪. [−1, 1‬‬
‫|‬ ‫|‬
‫|| ‪.‬‬ ‫|‬
‫إﻧﻄﻼﻗ ًﺎ ﻣﻦ ﻫﻨﺎ‪ ،‬ﻧﱪﻫﻦ ﺑﺎﻟﱰاﺟﻊ أﻧﻪ ﺗﻮﺟﺪ أﻋﺪاد ﺣﻘﻴﻘﻴﺔ }‪ 𝜀 , 𝜀 , ⋯ , 𝜀 ∈ {−1, 1‬ﺑﺤﻴﺚ ‪𝜀 z | ≤ 1‬‬
‫||‬ ‫=‬ ‫||‬
‫■‬

‫‪1‬‬
‫‪$‬‬ ‫‪.‬‬
‫𝟔𝟓𝟐‬ ‫‪.‬‬
‫= ‪ S‬و إﳚﺎد ﳖﺎﻳﺘﻬﺎ ‪. S‬‬ ‫ﻫﺪﻓﻨﺎ ﰲ ﻫﺬه اﳌﺴﺄﻟﺔ ﻫﻮ إﺛﺒﺎت ﺗﻘﺎرب اﳌﺘﺘﺎﻟﻴﺔ اﻟﺘﻲ ﺣﺪﻫﺎ اﻟﻌﺎم‬
‫‪ƕ‬‬

‫=‬ ‫‪k‬‬
‫‪VII‬‬

‫∞‪+‬‬
‫‪1‬‬ ‫‪1‬‬
‫= ‪.S‬‬ ‫‪ S = lim‬و ﻧﻜﺘﺐ إﺧﺘﺼﺎر ًا ‪:‬‬
‫∞‪→+‬‬
‫ﻟﺪﻳﻨﺎ إذن‬
‫=‬ ‫‪k‬‬ ‫=‬ ‫‪k‬‬
‫ﰲ اﻷﺟﺰاء ‪ 2 ، 1‬و ‪ 3‬ﻧﺴﺘﻌﺮض ﻃﺮﻗ ًﺎ ﳐﺘﻠﻔﺔ ﻹﳚﺎد ﻫﺬه اﻟﻨﻬﺎﻳﺔ و ﻫﺬه اﻷﺟﺰاء ﻣﺴﺘﻘﻠﺔ ﻓﻴﲈ ﺑﻴﻨﻬﺎ‪.‬‬
‫‪ũŏ‬‬

‫أﺳﺌﻠﺔ ﲤﻬﻴﺪﻳﺔ‬

‫) ‪. (S‬‬ ‫≥‬
‫‪ •1‬أدرس رﺗﺎﺑﺔ )ﻣﺘﺰاﻳﺪة‪ ،‬ﻣﺘﻨﺎﻗﺼﺔ‪ (... ،‬اﳌﺘﺘﺎﻟﻴﺔ‬
‫‪1‬‬ ‫‪1‬‬ ‫‪1‬‬ ‫‪1‬‬ ‫‪1‬‬
‫‪.‬‬ ‫=‬ ‫‪−‬‬ ‫و ّ‬
‫أن‬ ‫≤‬ ‫‪ •2‬أﺛﺒﺖ أﻧﻪ ‪ّ ∀n ≥ 2‬‬
‫ﻓﺈن ‪:‬‬
‫‪n (n − 1) n − 1 n‬‬ ‫‪n‬‬ ‫)‪n (n − 1‬‬

‫𝟱𝟱𝟯‬

‫‪http ://tinyurl.com/Malki1718‬‬ ‫‪0‬‬


‫‪.‬‬

‫‪1‬‬ ‫‪1‬‬
‫‪.T = 1−‬‬ ‫= ‪ . T‬أﺛﺒﺖ ّ‬
‫أن ‪:‬‬ ‫) ‪ (T‬اﳌﻌﺮﻓﺔ ﺑﺤﺪﻫﺎ اﻟﻌﺎم ‪:‬‬ ‫‪ •3‬ﻧﻌﺘﱪ اﳌﺘﺘﺎﻟﻴﺔ‬

‫‪Ŕž Ŧ Ľ‬‬
‫‪n‬‬ ‫)‪n (n − 1‬‬ ‫≥‬
‫=‬

‫) ‪ (S‬ﻣﺘﻘﺎرﺑﺔ‪.‬‬ ‫‪ •4‬إﺳﺘﻨﺘﺞ ّ‬
‫أن اﳌﺘﺘﺎﻟﻴﺔ‬

‫‪œǃ‬‬
‫≥‬

‫‪ •5‬ﱢ‬
‫ﻋﱪ ‪ ،‬ﺑﺪﻻﻟﺔ ‪ ، S‬ﻋﻦ ﻗﻴﻤﺔ اﻟﻨﻬﺎﻳﺘﲔ ‪:‬‬
‫∞‪+‬‬ ‫∞‪+‬‬
‫‪1‬‬ ‫‪1‬‬
‫=‪V‬‬ ‫‪,‬‬ ‫=‪U‬‬
‫)‪(2k + 1‬‬

‫‪Ŀ‬‬
‫=‬ ‫=‬ ‫)‪(2k‬‬

‫اﳉﺰء اﻷول ‪ :‬إﺳﺘﻌﲈل ﻛﺜﲑات ﺣﺪود‬

‫= )‪ P (X‬و ‪ . a ≠ 0‬ﻧﻘﺒﻞ ﺑﺪون ﺑﺮﻫﺎن‬ ‫‪a X‬‬ ‫ﻟﻴﻜﻦ )‪ P (X‬ﻛﺜﲑ ﺣﺪود ﺑﻤﻌﺎﻣﻼت ﻣﺮﻛﺒﺔ ﻣﻦ اﻟﺪرﺟﺔ ‪ n ≥ 1‬ﺣﻴﺚ‬
‫=‬
‫أن )‪ P (X‬ﻳﻘﺒﻞ ‪ n‬ﺟﺬر ًا ﻣﺮﻛﺒ ًﺎ 𝛼 ‪ 𝛼 ، ⋯ ، 𝛼 ،‬و ّ‬
‫أن ﳎﻤﻮع ﻫﺬه اﳉﺬور ُﻳﻌﻄﻰ ﺑﺎﻟﻌﺒﺎرة ‪:‬‬ ‫ّ‬
‫‪a −‬‬
‫‪𝜎 =𝛼 +𝛼 +⋯+𝛼 =−‬‬
‫‪a‬‬
‫)ا( ﻟﻴﻜﻦ ‪ p ∈ ℕ‬و ‪ . 𝜑 ∈ ℝ‬أﺛﺒﺖ اﳌﺴﺎواة ‪:‬‬ ‫•‬ ‫‪1‬‬
‫‪2p + 1‬‬ ‫‪−‬‬ ‫‪+‬‬
‫= )𝜑)‪sin ((2p + 1‬‬ ‫)‪(−1‬‬ ‫)𝜑 ‪(cos‬‬ ‫)𝜑 ‪(sin‬‬
‫=‬
‫‪2k + 1‬‬

‫)ب( إﺳﺘﻨﺘﺞ أﻧﻪ ﻟﻜﻞ ﻋﺪد ﻃﺒﻴﻌﻲ ‪ p ∈ ℕ‬و ﻟﻜﻞ ﻋﺪد ﺣﻘﻴﻘﻲ )𝜋 ‪ 𝜑 ≠ 0 (mod‬ﻳﻜﻮن ‪:‬‬
‫‪+‬‬ ‫‪2p + 1‬‬ ‫‪−‬‬
‫)𝜑 ‪sin ((2p + 1)𝜑) = (sin‬‬ ‫)‪(−1‬‬ ‫)𝜑 ‪(cotan‬‬
‫=‬
‫‪2k + 1‬‬

‫‪2p + 1‬‬ ‫‪−‬‬


‫= )‪. P (X‬‬ ‫)‪(−1‬‬ ‫‪X‬‬ ‫‪ •2‬ﻟﻴﻜﻦ ‪ p ∈ ℕ‬و ‪ P‬ﻛﺜﲑ اﳊﺪود اﳌﻌﺮف ﺑﺎﻟﻌﺒﺎرة ‪:‬‬
‫=‬
‫‪2k + 1‬‬

‫𝜋‪k‬‬
‫𝛾 ‪ P‬ﻟﻜﻞ }‪. k ∈ {1, 2, ⋯ , p‬‬ ‫‪ . 𝛾 = cotan‬أﺣﺴﺐ‬ ‫)ا( ﻟﻜﻞ }‪ k ∈ {1, 2, ⋯ , p‬ﻧﻀﻊ‬
‫‪ņőƱŀ Ţ‬‬
‫‪2p + 1‬‬
‫𝜋‬ ‫𝜋‪k‬‬
‫‪ . 0,‬إﺳﺘﻨﺘﺞ ّ‬
‫أن ﻛﺜﲑ‬ ‫ﻳﻨﺘﻤﻲ إﱃ اﳌﺠﺎل‬ ‫)ب( ﲢﻘﻖ‪ ،‬ﻷﺟﻞ }‪ ، k ∈ {1, 2, ⋯ , p‬ﻣﻦ ّ‬
‫أن اﻟﻌﺪد اﳊﻘﻴﻘﻲ‬
‫‪2‬‬ ‫‪2p + 1‬‬
‫اﳊﺪود ‪ P‬ﻳﻘﺒﻞ ‪ p‬ﺟﺬر ًا ﺑﺴﻴﻄ ًﺎ ُﻳﻄﻠﺐ ﺗﻌﻴﻴﻨﻬﺎ‪.‬‬
‫)ج( إﺳﺘﻨﺘﺞ اﳌﺴﺎوﺗﲔ ‪:‬‬
‫𝜋‪k‬‬ ‫)‪p (2p − 1‬‬ ‫‪1‬‬ ‫)‪2p (p + 1‬‬
‫‪cotan‬‬ ‫=‬ ‫‪,‬‬ ‫=‬
‫=‬
‫‪2p + 1‬‬ ‫‪3‬‬ ‫=‬ ‫‪sin‬‬ ‫‪3‬‬
‫‪+‬‬

‫)ا( أﺛﺒﺖ أﻧﻪ ﻟﻜﻞ ﻋﺪد ﺣﻘﻴﻘﻲ 𝜋 ‪ّ 𝜑 ∈ 0,‬‬


‫ﻓﺈن ‪. 0 < sin 𝜑 < 𝜑 < tan 𝜑 :‬‬ ‫‪3‬‬
‫‪ƕ‬‬

‫•‬
‫‪2‬‬
‫‪VII‬‬

‫)‪p (2p − 1) (2p + 1‬‬ ‫)‪2p (p + 1‬‬


‫‪.‬‬ ‫<‬ ‫< ‪S‬‬ ‫إﺳﺘﻨﺘﺞ أﻧﻪ ﻟﻜﻞ ‪ّ p ≥ 1‬‬
‫ﻓﺈن ‪:‬‬ ‫)ب(‬
‫‪3‬‬ ‫𝜋‬ ‫‪3‬‬
‫إﺳﺘﻨﺘﺞ ﻗﻴﻤﺔ ‪. S‬‬ ‫)ج(‬
‫‪ũŏ‬‬

‫اﳉﺰء اﻟﺜﺎﲏ ‪ :‬إﺳﺘﻌﲈل ﺗﻜﺎﻣﻼت ﻓﺎﻟﻴﺲ )‪(Intégrales de W‬‬


‫ﻟﻜﻞ ‪ n ∈ ℕ‬ﻧﻀﻊ ‪:‬‬
‫‪/‬‬ ‫‪/‬‬
‫) !‪4 (n‬‬
‫= ‪I‬‬ ‫‪cos‬‬ ‫‪t dt‬‬ ‫‪,‬‬ ‫= ‪J‬‬ ‫‪t cos‬‬ ‫‪t dt‬‬ ‫‪,‬‬ ‫= ‪K‬‬ ‫‪J‬‬
‫! )‪(2n‬‬

‫𝟲𝟱𝟯‬

‫‪http ://tinyurl.com/Malki1718‬‬ ‫‪0‬‬


‫‪ .VII‬ﲤﺎرﻳﻦ ﻟﻠﺘﻌﻤﻖ‬
‫‪.‬‬

‫‪ •1‬أﺣﺴﺐ اﻟﺘﻜﺎﻣﻠﲔ ‪ I‬و ‪. I‬‬

‫‪Ŕž Ŧ Ľ‬‬
‫‪2n + 1‬‬
‫‪.I‬‬ ‫‪+‬‬ ‫=‬ ‫‪I‬‬ ‫)ا( أﺛﺒﺖ‪ ،‬ﺑﺎﺳﺘﻌﲈل اﻟﺘﻜﺎﻣﻞ ﺑﺎﻟﺘﺠﺰﺋﺔ‪ ،‬أﻧﻪ ﻟﻜﻞ ‪ّ n ∈ ℕ‬‬
‫ﻓﺈن ‪:‬‬ ‫‪2‬‬
‫•‬
‫‪2n + 2‬‬

‫‪œǃ‬‬
‫! )‪(2n‬‬ ‫𝜋‬
‫= ‪.I‬‬ ‫⋅‬ ‫)ب( إﺳﺘﻨﺘﺞ أﻧﻪ ﻟﻜﻞ ‪ّ n ∈ ℕ‬‬
‫ﻓﺈن ‪:‬‬
‫) !‪4 (n‬‬ ‫‪2‬‬

‫‪ •3‬ﻟﻴﻜﻦ ‪. n ≥ 1‬‬

‫‪Ŀ‬‬
‫‪. I = n (2n − 1) J‬‬ ‫أﺛﺒﺖ ّ‬
‫أن ‪− 2n J :‬‬
‫‪−‬‬
‫)ا(‬
‫𝜋‬
‫‪.K‬‬ ‫‪−‬‬ ‫= ‪−K‬‬ ‫إﺳﺘﻨﺘﺞ ّ‬
‫أن ‪:‬‬ ‫)ب(‬
‫‪4n‬‬
‫𝜋‬
‫)ج( إﺳﺘﻨﺘﺞ ّ‬
‫أن ‪. S = J − K :‬‬
‫‪4‬‬
‫𝜋‬
‫≤ ‪.x‬‬ ‫‪sin x‬‬ ‫)ا( أﺛﺒﺖ أﻧﻪ ﻟﻜﻞ ]‪ّ x ∈ [0, 𝜋/2‬‬
‫ﻓﺈن ‪:‬‬ ‫‪4‬‬
‫•‬
‫‪2‬‬
‫)ب( إﺳﺘﻨﺘﺞ أﻧﻪ ﻟﻜﻞ ﻋﺪد ﻃﺒﻴﻌﻲ ‪ّ n‬‬
‫ﻓﺈن ‪:‬‬
‫‪𝜋 I +‬‬ ‫𝜋‬
‫≤ ‪0≤J‬‬ ‫‪,‬‬ ‫≤ ‪0≤K‬‬
‫)‪4 (2n + 1‬‬ ‫)‪16 (n + 1‬‬

‫)ج( أوﺟﺪ‪ُ ،‬ﳎﺪّ د ًا‪ ،‬ﻗﻴﻤﺔ ‪. S‬‬

‫اﳉﺰء اﻟﺜﺎﻟﺚ ‪ :‬إﺳﺘﻌﲈل ﻧﻮاة ﺩﻳﺮﻳﻜﻠﻲ )‪(Noyau de D‬‬

‫ﻟﻴﻜﻦ ‪ n ≥ 1‬ﻋﺪد ًا ﻃﺒﻴﻌﻴﺎ ﻏﲑ ﻣﻌﺪوم و ‪ D‬ﻧﻮاة ﺩﻳﺮﻳﻜﻠﻲ اﳌﻌﺮﻓﺔ ﺑﺎﻟﻌﺒﺎرة ‪:‬‬


‫‪1‬‬
‫= )‪∀x ∈ ℝ , D (x‬‬ ‫‪+‬‬ ‫)‪cos (kx‬‬
‫‪2‬‬ ‫=‬

‫‪1 sin n +‬‬ ‫‪x‬‬


‫‪ņőƱŀ Ţ‬‬
‫= )‪. D (x‬‬ ‫⋅‬ ‫‪ •1‬أﺛﺒﺖ أﻧﻪ ﻟﻜﻞ ‪ n ≥ 1‬و ﻟﻜﻞ ﻋﺪد ﺣﻘﻴﻘﻲ )𝜋‪ x ≠ 0 (mod 2‬ﻓﺈن ‪:‬‬
‫‪2‬‬ ‫‪sin‬‬

‫= ‪.L‬‬ ‫‪ L‬اﳌﻘﺪار اﳌﻌﺮف ﺑـِ ‪xD (x) dx :‬‬ ‫‪ •2‬ﻟﻴﻜﻦ ‪ n ≥ 1‬و‬

‫ﻟﻜﻞ ‪. k ≥ 1‬‬ ‫أﺣﺴﺐ اﻟﺘﻜﺎﻣﻞ ‪x cos (kx) dx‬‬ ‫)ا(‬


‫‪ƕ‬‬

‫𝜋‬ ‫)‪1 − (−1‬‬


‫)ب( إﺳﺘﻨﺘﺞ ّ‬
‫‪VII‬‬

‫= ‪.L‬‬ ‫‪−‬‬ ‫أن ‪:‬‬


‫‪4‬‬ ‫=‬ ‫‪k‬‬

‫ﻋﱪ ﻋﻦ ﳖﺎﻳﺘﻬﺎ ﺑﺪﻻﻟﺔ اﻟﻌﺪد ‪ V‬ا ُﳌ ّ‬


‫ﻌﺮف ﰲ اﻷﺳﺌﻠﺔ اﻟﺘﻤﻬﻴﺪﻳﺔ‪.‬‬ ‫)ج( إﺳﺘﻨﺘﺞ ّ‬
‫أن اﳌﺘﺘﺎﻟﻴﺔ ) ‪ (L‬ﻣﺘﻘﺎرﺑﺔ و ﱢ‬
‫‪ũŏ‬‬

‫⎧‬ ‫‪2‬‬ ‫إذا ﻛﺎن ‪ x = 0‬؛‬


‫↦ ‪.f: x‬‬ ‫‪x‬‬ ‫‪ •3‬ﻟﺘﻜﻦ ‪ f‬اﻟﺪاﻟﺔ اﳌﻌﺮﻓﺔ ﻋﲆ اﳌﺠﺎل ]𝜋 ‪ [0,‬ﺑـِ ‪:‬‬
‫⎨‬ ‫إذا ﻛﺎن ‪ x ≠ 0‬؛‬
‫)‪⎩ sin (x/2‬‬
‫أﺛﺒﺖ ّ‬
‫أن اﻟﺪاﻟﺔ ‪ f‬ﻣﺴﺘﻤﺮة و ﻗﺎﺑﻠﺔ ﻟﻺﺷﺘﻘﺎق ﻋﲆ ]𝜋 ‪. [0,‬‬
‫أن ‪ f‬ﻗﺎﺑﻠﺔ ﻟﻺﺷﺘﻘﺎق ﻋﻨﺪ ‪ x = 0‬ﻣﻊ ‪ f (0) = 0‬و ّ‬
‫أن اﳌﺸﺘﻘﺔ ‪ f‬ﻣﺴﺘﻤﺮة ﻋﲆ ]𝜋 ‪. [0,‬‬ ‫ﻧﻘﺒﻞ ﺑﺪون ﺑﺮﻫﺎن ّ‬

‫𝟳𝟱𝟯‬

‫‪http ://tinyurl.com/Malki1718‬‬ ‫‪0‬‬


‫‪.‬‬
‫|‬ ‫|‬
‫|‬ ‫|‬

‫‪Ŕž Ŧ Ľ‬‬
‫|‪.‬‬ ‫≤ |‪h(x) dx‬‬ ‫ﻟﺘﻜﻦ ‪ h‬داﻟﺔ ﻣﺴﺘﻤﺮة ﻋﲆ اﳌﺠﺎل ]‪ . [a, b‬أﺛﺒﺖ ّ‬
‫أن ‪||h(x)|| dx :‬‬ ‫)ا(‬ ‫•‬ ‫‪4‬‬
‫|‬ ‫|‬
‫||‬ ‫||‬

‫‪œǃ‬‬
‫)ب( ﻟﺘﻜﻦ 𝜙 داﻟﺔ ﻣﺴﺘﻤﺮة و ﻗﺎﺑﻠﺔ ﻟﻺﺷﺘﻘﺎق ﻋﲆ ]𝜋 ‪ [0,‬ﺑﺤﻴﺚ داﻟﺘﻬﺎ اﳌﺸﺘﻘﺔ 𝜙 ﻣﺴﺘﻤﺮة ﻋﲆ ]𝜋 ‪ . [0,‬أﺛﺒﺖ ّ‬
‫أن ‪:‬‬

‫⎛ ‪lim‬‬ ‫‪𝜙 (x) sin (𝜆x)⎞ = 0‬‬


‫∞‪→+‬‬
‫⎝‬ ‫⎠‬

‫‪Ŀ‬‬
‫‪. lim‬‬ ‫)ا( أﺛﺒﺖ ّ‬
‫أن ‪L = 0 :‬‬ ‫•‬ ‫‪5‬‬
‫∞‪→+‬‬

‫)ب( أوﺟﺪ‪ُ ،‬ﳎﺪّ د ًا‪ ،‬ﻗﻴﻤﺔ ‪. S‬‬

‫ﺍﳊﻞّ‪.‬‬
‫أﺳﺌﻠﺔ ﲤﻬﻴﺪﻳﺔ‬

‫‪ •1‬ﻟﺪﻳﻨﺎ ‪:‬‬
‫‪+‬‬
‫‪1‬‬ ‫‪1‬‬ ‫‪1‬‬
‫‪S‬‬ ‫‪+‬‬ ‫= ‪−S‬‬ ‫‪−‬‬ ‫=‬ ‫‪>0‬‬
‫=‬ ‫‪k‬‬ ‫=‬ ‫‪k‬‬ ‫)‪(n + 1‬‬

‫) ‪ (S‬ﻣﺘﺰاﻳﺪة ﲤﺎﻣ ًﺎ‪.‬‬ ‫≥‬


‫إذن ﻓﺎﳌﺘﺘﺎﻟﻴﺔ‬
‫‪1‬‬ ‫‪1‬‬ ‫‪1‬‬ ‫‪1‬‬
‫‪ .‬ﻣﻦ ﺟﻬﺔ أﺧﺮى ‪:‬‬ ‫≤‬ ‫أي‬ ‫≤‬ ‫إذا ﻛﺎن ‪ّ n ≥ 2‬‬
‫ﻓﺈن ‪ n ≥ n − n‬ﻣﻨﻪ‬ ‫•‬ ‫‪2‬‬
‫‪n‬‬ ‫)‪n (n − 1‬‬ ‫‪n‬‬ ‫‪n −n‬‬
‫‪1‬‬ ‫‪1‬‬ ‫)‪n − (n − 1‬‬ ‫‪1‬‬
‫= ‪−‬‬ ‫=‬
‫‪n−1 n‬‬ ‫)‪n (n − 1‬‬ ‫)‪n (n − 1‬‬

‫‪ •3‬ﻟﺪﻳﻨﺎ ‪:‬‬
‫‪ņőƱŀ Ţ‬‬
‫‪1‬‬ ‫‪1‬‬ ‫‪1‬‬
‫= ‪T‬‬ ‫=‬ ‫‪−‬‬
‫=‬
‫)‪n (n − 1‬‬ ‫=‬
‫‪n−1 n‬‬
‫‪1‬‬ ‫‪1 1‬‬ ‫‪1‬‬ ‫‪1‬‬ ‫‪1‬‬ ‫‪1‬‬
‫‪= 1−‬‬ ‫‪+‬‬ ‫‪−‬‬ ‫‪+⋯+‬‬ ‫‪−‬‬ ‫‪+‬‬ ‫‪−‬‬
‫‪2‬‬ ‫‪2 3‬‬ ‫‪n−2 n−1‬‬ ‫‪n−1 n‬‬
‫‪1‬‬
‫‪=1−‬‬
‫‪n‬‬
‫‪1‬‬ ‫‪1‬‬
‫) ‪ (T‬ﻣﺘﺰاﻳﺪة‬ ‫‪ T‬أي ّ‬
‫أن اﳌﺘﺘﺎﻟﻴﺔ‬ ‫= ‪−T‬‬ ‫‪ lim T = lim 1 −‬ﻟﻜﻦ ‪> 0‬‬ ‫ﻣﻨﻪ ‪= 1‬‬
‫‪ƕ‬‬

‫≥‬ ‫‪+‬‬
‫)‪n (n + 1‬‬ ‫∞‪→+‬‬ ‫∞‪→+‬‬ ‫‪n‬‬
‫‪VII‬‬

‫ﲤﺎﻣ ًﺎ و ﺑﺎﻟﺘﺎﱄ ّ‬
‫ﻓﺈن ‪ ∀n ≥ 2 : T ≤ lim T :‬أي ‪. ∀n ≥ 2 : T ≤ 1‬‬ ‫∞‪→+‬‬

‫‪ •4‬ﻟﺪﻳﻨﺎ ‪:‬‬
‫‪ũŏ‬‬

‫‪1‬‬ ‫‪1‬‬
‫‪S =1+‬‬ ‫‪≤1+‬‬ ‫‪≤1+1≤2‬‬
‫=‬ ‫‪k‬‬ ‫=‬
‫)‪k (k − 1‬‬

‫) ‪ (S‬ﳏﺪودة ﻣﻦ اﻷﻋﲆ‪.‬‬ ‫≥‬


‫أي ّ‬
‫أن اﳌﺘﺘﺎﻟﻴﺔ‬
‫) ‪ (S‬ﻣﺘﺰاﻳﺪة ﲤﺎﻣ ًﺎ و ﳏﺪودة ﻣﻦ اﻷﻋﲆ ﻓﻬﻲ إذن ﻣﺘﻘﺎرﺑﺔ‪.‬‬ ‫≥‬
‫اﳌﺘﺘﺎﻟﻴﺔ‬

‫𝟴𝟱𝟯‬

‫‪http ://tinyurl.com/Malki1718‬‬ ‫‪0‬‬


‫‪ .VII‬ﲤﺎرﻳﻦ ﻟﻠﺘﻌﻤﻖ‬
‫‪.‬‬
‫‪1‬‬ ‫‪1‬‬
‫= ‪.V‬‬ ‫= ‪U‬و‬ ‫‪ •5‬ﻧﻀﻊ‬
‫=‬ ‫)‪(2k + 1‬‬ ‫=‬ ‫)‪(2k‬‬

‫‪Ŕž Ŧ Ľ‬‬
‫) ‪ (V‬ﻣﺘﺰاﻳﺪة ﲤﺎﻣ ًﺎ و ﺑﺎﻟﺘﺎﱄ ﻓﻜﻼﳘﺎ‬ ‫≥‬
‫أن ‪ U = 1 S‬و ‪ V ≤ S ≤ 2‬و ّ‬
‫أن اﳌﺘﺘﺎﻟﻴﺔ‬ ‫ﻣﻦ اﻟﺴﻬﻞ ﻣﻼﺣﻈﺔ ّ‬
‫‪4‬‬
‫∞‪+‬‬ ‫∞‪+‬‬

‫‪œǃ‬‬
‫‪1‬‬ ‫‪1‬‬
‫= ‪.V‬‬ ‫= ‪U‬و‬ ‫ﻣﺘﻘﺎرﺑﺔ إذن ﺑﺎﻹﻣﻜﺎن اﻟﻜﻼم ﻋﻦ اﻟﻨﻬﺎﻳﺘﲔ‬
‫=‬ ‫)‪(2k + 1‬‬ ‫=‬ ‫)‪(2k‬‬
‫‪1‬‬ ‫‪3‬‬
‫ﻟﻜﻦ ‪ S = U + V‬ﻣﻨﻪ ‪ V = S − U = S − S = S‬و ﺑﺄﺧﺬ اﻟﻨﻬﺎﻳﺔ ﻋﻨﺪﻣﺎ ∞‪ n ⟶ +‬ﻳﻨﺘﺞ ‪:‬‬
‫‪4‬‬ ‫‪4‬‬

‫‪Ŀ‬‬
‫‪3‬‬ ‫‪1‬‬
‫‪U = S‬و‪.V = S‬‬
‫‪4‬‬ ‫‪4‬‬
‫اﳉﺰء اﻷول ‪ :‬إﺳﺘﻌﲈل ﻛﺜﲑات ﺣﺪود‬

‫)ا( ﻟﺪﻳﻨﺎ ﺣﺴﺐ دﺳﺘﻮر ﻣﻮﺍﻓﺮ ‪ . cos ((2p + 1)𝜑) + ı sin ((2p + 1)𝜑) = (cos 𝜑 + ı sin 𝜑) + :‬ﻧﻨﴩ‬ ‫‪1‬‬
‫•‬

‫ﻫﺬا اﻷﺧﲑ ﺑﺎﺳﺘﻌﲈل دﺳﺘﻮر ﺛﻨﺎﺋﻲ اﳊﺪ و ﻧﺄﺧﺬ ﺟﺰ َأه اﻟﺘﺨﻴﲇ ‪:‬‬
‫))𝜑)‪sin ((2p + 1)𝜑) = Im (cos ((2p + 1)𝜑) + ı sin ((2p + 1‬‬
‫‪+‬‬
‫)𝜑 ‪= Im (cos 𝜑 + ı sin‬‬
‫‪+‬‬
‫‪2p + 1‬‬ ‫‪+ −‬‬
‫‪= Im‬‬ ‫)𝜑 ‪(cos‬‬ ‫)𝜑 ‪(ı sin‬‬
‫=‬
‫‪m‬‬

‫اﳉﺰء اﻟﺘﺨﻴﲇ ﻳﻨﺘﺞ ﻋﻦ اﳊﺪود اﻟﺘﻲ ﻳﻜﻮن ﻓﻴﻬﺎ ‪ m‬ﻓﺮدﻳ ًﺎ و ﰲ ﻫﺬه اﳊﺎﻟﺔ‪ ،‬إذا ﻛﺎن ‪ّ m = 2k + 1‬‬
‫ﻓﺈن ‪:‬‬
‫‪ ı = ı‬و ﺑﺎﻟﺘﺎﱄ ‪:‬‬ ‫‪+‬‬ ‫‪= ı‬‬ ‫‪⋅ ı = (−1) ı‬‬

‫‪2p + 1‬‬ ‫‪+ −‬‬ ‫‪+‬‬ ‫‪+‬‬


‫= )𝜑)‪sin ((2p + 1‬‬ ‫)‪(−1‬‬ ‫)𝜑 ‪(cos‬‬ ‫)𝜑 ‪(sin‬‬
‫=‬
‫‪2k + 1‬‬

‫‪2p + 1‬‬ ‫‪−‬‬ ‫‪+‬‬


‫= )𝜑)‪sin ((2p + 1‬‬ ‫)‪(−1‬‬ ‫‪cos‬‬ ‫‪(𝜑) sin‬‬ ‫)𝜑(‬ ‫أي ‪:‬‬
‫‪ņőƱŀ Ţ‬‬
‫=‬
‫‪2k + 1‬‬

‫)ب( إذا ﻛﺎن )𝜋 ‪ّ 𝜑 ≠ 0 (mod‬‬


‫ﻓﺈن ‪ sin 𝜑 ≠ 0‬ﻣﻨﻪ ‪:‬‬
‫‪2p + 1‬‬ ‫‪−‬‬ ‫‪+‬‬
‫= )𝜑)‪sin ((2p + 1‬‬ ‫)‪(−1‬‬ ‫‪cos‬‬ ‫‪(𝜑) sin‬‬ ‫)𝜑(‬
‫=‬
‫‪2k + 1‬‬

‫‪+‬‬ ‫‪2p + 1‬‬ ‫‪−‬‬ ‫‪+ −‬‬ ‫‪+‬‬


‫‪= sin‬‬ ‫)𝜑(‬ ‫)‪(−1‬‬ ‫‪cos‬‬ ‫‪(𝜑) sin‬‬ ‫)𝜑(‬
‫=‬
‫‪2k + 1‬‬
‫‪ƕ‬‬

‫‪+‬‬ ‫‪2p + 1‬‬ ‫‪−‬‬ ‫‪−‬‬


‫‪= sin‬‬ ‫)𝜑(‬ ‫)‪(−1‬‬ ‫‪cos‬‬ ‫‪(𝜑) sin‬‬ ‫)𝜑(‬
‫‪VII‬‬

‫=‬
‫‪2k + 1‬‬

‫‪+‬‬ ‫‪2p + 1‬‬ ‫‪−‬‬ ‫‪1‬‬


‫‪= sin‬‬ ‫)𝜑(‬ ‫)‪(−1‬‬ ‫‪cos‬‬ ‫× )𝜑(‬ ‫‪−‬‬
‫‪2k + 1‬‬ ‫‪sin‬‬ ‫)𝜑(‬
‫‪ũŏ‬‬

‫=‬

‫‪+‬‬ ‫‪2p + 1‬‬ ‫‪−‬‬


‫)𝜑 ‪sin ((2p + 1)𝜑) = (sin‬‬ ‫)‪(−1‬‬ ‫)𝜑 ‪(cotan‬‬ ‫أي ‪:‬‬
‫=‬
‫‪2k + 1‬‬

‫‪2p + 1‬‬ ‫‪−‬‬ ‫)𝜑)‪sin ((2p + 1‬‬


‫)‪(−1‬‬ ‫)𝜑 ‪(cotan‬‬ ‫=‬ ‫‪+‬‬
‫أو‬
‫=‬
‫‪2k + 1‬‬ ‫)𝜑 ‪(sin‬‬

‫𝟵𝟱𝟯‬

‫‪http ://tinyurl.com/Malki1718‬‬ ‫‪0‬‬


‫‪.‬‬
‫𝜋‪k‬‬
‫ﻣﻨﻪ ‪:‬‬ ‫أن }‪ّ k ∈ {1, 2, ⋯ , p‬‬
‫ﻓﺈن )𝜋 ‪≠ 0 (mod‬‬ ‫ﺑﲈ ّ‬ ‫)ا(‬ ‫•‬ ‫‪2‬‬
‫‪2p + 1‬‬

‫‪Ŕž Ŧ Ľ‬‬
‫‪−‬‬
‫‪2p + 1‬‬ ‫𝜋‪k‬‬
‫𝛾 ‪P‬‬ ‫=‬ ‫)‪(−1‬‬ ‫‪cotan‬‬
‫‪2k + 1‬‬ ‫‪2p + 1‬‬

‫‪œǃ‬‬
‫=‬
‫‪−‬‬
‫‪2p + 1‬‬ ‫𝜋‪k‬‬
‫=‬ ‫)‪(−1‬‬ ‫‪cotan‬‬
‫=‬
‫‪2k + 1‬‬ ‫‪2p + 1‬‬

‫‪Ŀ‬‬
‫‪+‬‬ ‫𝜋‪k‬‬
‫‪ sin‬و ﺑﺎﻟﺘﺎﱄ ﻳﻤﻜﻦ ﺗﻄﺒﻴﻖ ﻧﺘﻴﺠﺔ اﻟﺴﺆال اﻟﺴﺎﺑﻖ ﻓﻴﻨﺘﺞ ‪:‬‬ ‫ﻟﻜﻦ ‪≠ 0‬‬
‫‪2p + 1‬‬

‫‪2p + 1‬‬ ‫𝜋‪k‬‬


‫‪−‬‬ ‫)‪sin (2p + 1‬‬ ‫‪+‬‬ ‫)𝜋‪sin (k‬‬
‫𝛾 ‪P‬‬ ‫=‬ ‫)‪(−1‬‬ ‫‪cotan‬‬ ‫=‬ ‫=‬ ‫‪=0‬‬
‫=‬
‫‪2k + 1‬‬ ‫‪2p + 1‬‬ ‫‪sin‬‬
‫‪+‬‬
‫‪sin‬‬
‫‪+‬‬
‫‪+‬‬ ‫‪+‬‬

‫𝜋‬ ‫𝜋‪k‬‬ ‫𝜋‪p‬‬ ‫𝜋‬


‫< ‪ 0‬أي أﻧﻪ ﻟﻜﻞ }‪ّ ، k ∈ {1, 2, ⋯ , p‬‬
‫ﻓﺈن‬ ‫≤‬ ‫≤‬ ‫ﻓﺈن <‬‫إذا ﻛﺎن ‪ّ 1 ≤ k ≤ p‬‬ ‫)ب(‬
‫‪2p + 1‬‬ ‫‪2p + 1‬‬ ‫‪2p + 1‬‬ ‫‪2‬‬
‫𝜋‬ ‫𝜋‪k‬‬
‫ﻳﻨﺘﻤﻲ إﱃ اﳌﺠﺎل ‪. 0,‬‬ ‫اﻟﻌﺪد اﳊﻘﻴﻘﻲ‬
‫‪2‬‬ ‫‪2p + 1‬‬
‫𝜋‬
‫اﻟﺪاﻟﺔ ‪ x ⟼ cotan x‬ﻣﺴﺘﻤﺮة و ﻗﺎﺑﻠﺔ ﻟﻺﺷﺘﻘﺎق ﻋﲆ اﳌﺠﺎل ‪ 0,‬و داﻟﺘﻬﺎ اﳌﺸﺘﻘﺔ ﻫﻲ اﻟﺪاﻟﺔ‬
‫‪2‬‬
‫)‪ . x ⟼ −2 cotan(x) 1 + cotan (x‬ﻫﺬه اﳌﺸﺘﻘﺔ ﺳﺎﻟﺒﺔ ﲤﺎﻣ ًﺎ ﻋﲆ اﳌﺠﺎل 𝜋 ‪ 0,‬و ﺑﺎﻟﺘﺎﱄ ﻓﺈنّ‬
‫‪2‬‬
‫‪ x ⟼ cotan x‬ﻣﺘﻨﺎﻗﺼﺔ ﲤﺎﻣ ًﺎ ﻋﲆ ﻫﺬا اﳌﺠﺎل‪.‬‬
‫𝜋‬
‫ﰲ اﻷﺧﲑ‪ ،‬اﻟﺪاﻟﺔ ‪ x ⟼ cotan x‬ﻣﺴﺘﻤﺮة و ﻣﺘﻨﺎﻗﺼﺔ ﲤﺎﻣ ًﺎ ﻋﲆ ‪ 0,‬ﻓﻬﻲ إذن ﺗﻘﺎﺑﻠﻴﺔ ﻋﲆ ﻫﺬا اﳌﺠﺎل‪ ،‬و‬
‫‪2‬‬
‫𝜋‪k‬‬
‫‪ 𝛾 = cotan‬ﲠﺬه اﻟﺪاﻟﺔ ﻫﻲ أﻳﻀ ًﺎ ﳐﺘﻠﻔﺔ‬ ‫ﻮرﻫﺎ‬
‫َ‬ ‫أن اﻷﻋﺪاد 𝜋‪ k‬ﳐﺘﻠﻔﺔ ﻣﺜﻨﻰ ﻣﺜﻨﻰ ّ‬
‫ﻓﺈن ُﺻ‬ ‫ﺑﲈ ّ‬
‫‪2p + 1‬‬ ‫‪2p + 1‬‬
‫ﻣﺜﻨﻰ ﻣﺜﻨﻰ‪.‬‬
‫ﲢﺼﻠﻨﺎ ﻋﲆ ﲨﻴﻊ ﺟﺬوره و ﻫﻲ‬
‫أن درﺟﺔ ﻫﺬا اﻷﺧﲑ ﻫﻲ ‪ p‬ﻓﻘﺪ ّ‬ ‫ﲢﺼﻠﻨﺎ إذن ﻋﲆ ‪ p‬ﺟﺬر ًا ﻟﻜﺜﲑ اﳊﺪود ‪ .P‬و ﺑﲈ ّ‬
‫ّ‬
‫‪ņőƱŀ Ţ‬‬
‫ﻛﻠﻬﺎ ﺟﺬور ﺑﺴﻴﻄﺔ )ﻷﳖﺎ ﳐﺘﻠﻔﺔ ﻣﺜﻨﻰ ﻣﺜﻨﻰ(‪.‬‬
‫ﻛﺜﲑ اﳊﺪود ‪ P‬ﻳﻘﺒﻞ ‪ p‬ﺟﺬر ًا ﺑﺴﻴﻄ ًﺎ و ﻫﻲ اﻷﻋﺪاد 𝛾 ﻷﺟﻞ }‪k ∈ {1, 2, ⋯ , p‬‬

‫‪2p + 1‬‬
‫)‪ a = (−1‬و‬ ‫‪= 2p + 1‬‬ ‫اﳌﻌﺮﻓﺔ ﻟﻜﺜﲑ اﳊﺪود ‪ّ P‬‬
‫ﻓﺈن درﺟﺘﻪ ﻫﻲ ‪ p‬و‬ ‫)ج( ﺣﺴﺐ اﻟﻌﺒﺎرة ﱢ‬
‫‪2×0+1‬‬
‫‪2p + 1‬‬ ‫)‪(2p + 1) (2p) (2p − 1‬‬
‫‪.a‬‬ ‫‪−‬‬ ‫)‪= (−1‬‬ ‫‪=−‬‬
‫‪2×1+1‬‬ ‫‪3×2×1‬‬
‫‪ a‬ﺗُﱰﺟﻢ ﻛﲈ ﻳﲇ ‪:‬‬ ‫‪−‬‬ ‫اﻟﻌﺒﺎرة اﻟﺘﻲ ﺗﻌﻄﻲ ﳎﻤﻮع اﳉﺬور ﺑﺪﻻﻟﺔ اﳌﻌﺎﻣﻠﲔ ‪ a‬و‬
‫‪+‬‬ ‫‪−‬‬
‫‪a −‬‬ ‫‪−‬‬ ‫)‪p (2p − 1‬‬
‫‪ƕ‬‬

‫أي ‪:‬‬ ‫‪𝛾 =𝜎 =−‬‬ ‫‪=−‬‬ ‫=‬


‫‪a‬‬ ‫‪2p + 1‬‬ ‫‪3‬‬
‫‪VII‬‬

‫=‬

‫𝜋‪k‬‬ ‫)‪p (2p − 1‬‬


‫‪cotan‬‬ ‫=‬
‫=‬
‫‪2p + 1‬‬ ‫‪3‬‬
‫‪ũŏ‬‬

‫𝟬𝟲𝟯‬

‫‪http ://tinyurl.com/Malki1718‬‬ ‫‪0‬‬


‫‪ .VII‬ﲤﺎرﻳﻦ ﻟﻠﺘﻌﻤﻖ‬
‫‪.‬‬
‫إذا ﻛﺎن ‪ 𝜑 ∈ ℝ‬ﻣﻊ )𝜋 ‪ّ 𝜑 ≠ 0 (mod‬‬
‫ﻓﺈن ‪:‬‬

‫‪Ŕž Ŧ Ľ‬‬
‫𝜑 ‪sin 𝜑 + cos‬‬ ‫‪1‬‬
‫= 𝜑 ‪1 + cotan‬‬ ‫=‬
‫𝜑 ‪sin‬‬ ‫𝜑 ‪sin‬‬

‫‪œǃ‬‬
‫‪1‬‬ ‫𝜋‪k‬‬
‫=‬ ‫‪1 + cotan‬‬ ‫ﻣﻨﻪ‬
‫=‬ ‫‪sin‬‬ ‫=‬
‫‪2p + 1‬‬
‫‪+‬‬

‫𝜋‪k‬‬
‫=‬ ‫‪1+‬‬ ‫‪cotan‬‬

‫‪Ŀ‬‬
‫=‬ ‫=‬
‫‪2p + 1‬‬
‫)‪p (2p − 1) p (2p − 1 + 3‬‬
‫‪=p+‬‬ ‫=‬
‫‪3‬‬ ‫‪3‬‬
‫)‪p (2p + 2) 2p (p + 1‬‬
‫=‬ ‫=‬
‫‪3‬‬ ‫‪3‬‬
‫‪1‬‬ ‫)‪2p (p + 1‬‬
‫=‬ ‫أي‬
‫=‬ ‫‪sin‬‬ ‫‪3‬‬
‫‪+‬‬

‫𝜋‬
‫‪. 0,‬‬ ‫)ا( ﻧﻌﺘﱪ اﻟﺪاﻟﺘﲔ ‪ f(x) = x − sin x‬و ‪ g(x) = tan x − x‬اﳌﻌﺮﻓﺘﲔ ﻋﲆ اﳌﺠﺎل‬ ‫•‬‫‪3‬‬
‫‪2‬‬
‫ﻫﺎﺗﺎن اﻟﺪاﻟﺘﺎن ﻣﻌﺮﻓﺘﺎن و ﻗﺎﺑﻠﺘﺎن ﻟﻺﺷﺘﻘﺎق ﻋﲆ ﻫﺬا اﳌﺠﺎل‪ .‬ﺑﺎﻹﺿﺎﻓﺔ إﱃ ذﻟﻚ‪ ،‬ﻟﺪﻳﻨﺎ ‪ f (x) = 1 − cos x‬و‬
‫‪. g (x) = 1 + tan x − 1 = tan x‬‬
‫𝜋‬
‫ﻟﻜﻞ ‪ x ∈ 0,‬ﻟﺪﻳﻨﺎ ‪ f (x) > 0‬و ‪ g (x) > 0‬و ﺑﺎﻟﺘﺎﱄ ﻓﺎﻟﺪاﻟﺘﺎن ‪ f‬و ‪ g‬ﻣﺘﺰاﻳﺪﺗﺎن ﲤﺎﻣ ًﺎ ﻋﲆ ﻫﺬا اﳌﺠﺎل‪.‬‬
‫‪2‬‬
‫𝜋‬
‫ﻣﻨﻪ ﻟﻜﻞ ‪ّ x ∈ 0,‬‬
‫ﻓﺈن ‪ f(x) > f(0) = 0‬أي ‪ x − sin x > 0‬أي ‪. x > sin x > 0‬‬
‫‪2‬‬
‫𝜋‬
‫ﺑﺎﳌِﺜﻞ‪ ،‬ﻟﻜﻞ ‪ x ∈ 0,‬ﻟﺪﻳﻨﺎ ‪ g(x) > g(0) = 0‬أي ‪ tan x − x > 0‬أي ‪. tan x > x‬‬
‫‪2‬‬
‫𝜋‬
‫‪∀x ∈ 0,‬‬ ‫‪: 0 < sin x < x < tan x‬‬ ‫ﰲ اﻷﺧﲑ ‪:‬‬
‫‪2‬‬
‫𝜋‪k‬‬ ‫𝜋‬
‫‪ņőƱŀ Ţ‬‬
‫= ‪ x‬ﻟﻨﺠﺪ ‪:‬‬ ‫‪∈ 0,‬‬ ‫)ب( ﻟﻜﻞ }‪ k ∈ {1, 2, ⋯ , p‬ﻧﺴﺘﻌﻤﻞ اﳌﺘﺒﺎﻳﻨﺎت اﻟﺴﺎﺑﻘﺔ ﻣﻊ‬
‫‪2p + 1‬‬ ‫‪2‬‬

‫𝜋‪k‬‬ ‫𝜋‪k‬‬ ‫𝜋‪k‬‬


‫‪0 < sin‬‬ ‫<‬ ‫‪< tan‬‬
‫‪2p + 1‬‬ ‫‪2p + 1‬‬ ‫‪2p + 1‬‬
‫𝜋‪k‬‬ ‫𝜋‪k‬‬ ‫𝜋‪k‬‬
‫‪0 < sin‬‬ ‫<‬ ‫‪< tan‬‬ ‫ﺑﱰﺑﻴﻊ اﻷﻃﺮاف )ﻣﻮﺟﺒﺔ ﲨﻴﻌﻬﺎ(‬
‫‪2p + 1‬‬ ‫‪2p + 1‬‬ ‫‪2p + 1‬‬
‫‪1‬‬ ‫‪1‬‬ ‫‪1‬‬
‫<‪0‬‬ ‫<‬ ‫<‬ ‫ﺑﺄﺧﺬ اﳌﻘﻠﻮب‬
‫‪tan‬‬ ‫‪+‬‬ ‫‪sin‬‬ ‫‪+‬‬
‫‪ƕ‬‬

‫‪+‬‬
‫‪VII‬‬

‫𝜋‪k‬‬ ‫)‪(2p + 1‬‬ ‫‪1‬‬


‫‪0 < cotan‬‬ ‫<‬ ‫<‬ ‫ﻣﻨﻪ‬
‫‪2p + 1‬‬ ‫)𝜋‪(k‬‬ ‫‪sin‬‬ ‫‪+‬‬
‫‪ũŏ‬‬

‫𝜋‪k‬‬ ‫)‪(2p + 1‬‬ ‫‪1‬‬


‫<‪0‬‬ ‫‪cotan‬‬ ‫<‬ ‫<‬ ‫ﻣﻨﻪ‬
‫=‬
‫‪2p + 1‬‬ ‫=‬ ‫)𝜋‪(k‬‬ ‫=‬ ‫‪sin‬‬ ‫‪+‬‬

‫)‪p (2p − 1) (2p + 1‬‬ ‫‪1‬‬ ‫)‪2p (p + 1‬‬


‫<‪0‬‬ ‫<‬ ‫<‬ ‫ﻣﻨﻪ‬
‫‪3‬‬ ‫𝜋‬ ‫=‬ ‫‪k‬‬ ‫‪3‬‬

‫)‪p (2p − 1) (2p + 1‬‬ ‫)‪2p (p + 1‬‬


‫‪∀p ≥ 1 :‬‬ ‫<‬ ‫< ‪S‬‬ ‫أي ‪:‬‬
‫‪3‬‬ ‫𝜋‬ ‫‪3‬‬

‫𝟭𝟲𝟯‬

‫‪http ://tinyurl.com/Malki1718‬‬ ‫‪0‬‬


.
(2p + 1)
: ‫ﻧﺠﺪ‬ ‫)ج( ﺑﻘﺴﻤﺔ أﻃﺮاف اﳌﺘﺒﺎﻳﻨﺎت اﻟﺴﺎﺑﻘﺔ ﻋﲆ‬
𝜋

Ŕž Ŧ Ľ
p (2p − 1) 𝜋 2p (p + 1) 𝜋
<S <
3 (2p + 1) 3 (2p + 1)

œǃ
p (2p − 1) 𝜋 2𝜋 p − 𝜋 p 2𝜋 p 𝜋
lim = lim = lim = ‫ﻟﻜﻦ‬
→+∞ 3 (2p + 1) →+∞ 12p + 12p + 3 →+∞ 12p 6

2p (p + 1) 𝜋 2𝜋 p − 2𝜋 2𝜋 p 𝜋

Ŀ
lim = lim = lim = ‫و‬
→+∞ 3 (2p + 1) →+∞ 12p + 12p + 3 →+∞ 12p 6

+∞
1 𝜋 𝜋
.S= = ‫أي‬ lim S = ّ ‫و ﺣﺴﺐ ﻧﻈﺮﻳﺔ اﳊﴫ‬
‫ﻓﺈن‬
= k 6 →+∞ 6

(W) ‫ إﺳﺘﻌﲈل ﺗﻜﺎﻣﻼت ﻓﺎﻟﻴﺲ‬: ‫اﳉﺰء اﻟﺜﺎﲏ‬

: ‫• ﻟﺪﻳﻨﺎ‬1
/ /
/ 𝜋
I = cos t dt = dt = [t] =
2
/ / /
1 + cos (2t) t sin (2t) 𝜋
I = cos t dt = dt = + = ‫و‬
2 2 4 4

u (t) = − (2n + 1) sin t ⋅ cos t ‫ ﻣﻨﻪ‬v (t) = cos t ‫ و‬u(t) = cos + t ‫ ﻧﻀﻊ‬،‫ﺑﺎﻟﺘﻜﺎﻣﻞ ﺑﺎﻟﺘﺠﺰﺋﺔ‬ (‫)ا‬ • 2
: ‫[ ﻣﻨﻪ‬0, 𝜋/2] ‫ ﲢﻘﻘﺎن ﴍوط ﻧﻈﺮﻳﺔ اﻟﺘﻜﺎﻣﻞ ﺑﺎﻟﺘﺠﺰﺋﺔ ﻋﲆ اﳌﺠﺎل‬v ‫ و‬u ‫ اﻟﺪاﻟﺘﺎن‬. v(t) = sin t ‫و‬
ņőƱŀ Ţ
/
+ /
I + = sin t ⋅ cos t − − (2n + 1) sin t ⋅ cos t × sin t dt

= 0 + (2n + 1) sin t ⋅ cos t dt

= (2n + 1) 1 − cos t cos t dt


ƕ

VII

/ /

= (2n + 1) ⎛ cos t dt − cos + t dt⎞ = (2n + 1) I − I +


⎝ ⎠
ũŏ

2n + 1
. I + = I ‫أي‬ (2n + 2) I + = (2n + 1) I ‫ﻣﻨﻪ‬
2n + 2

𝟯𝟲𝟮

http ://tinyurl.com/Malki1718 0
‫‪ .VII‬ﲤﺎرﻳﻦ ﻟﻠﺘﻌﻤﻖ‬
‫‪.‬‬
‫)ب( ﻟﺪﻳﻨﺎ ‪:‬‬
‫‪2n − 1‬‬ ‫‪2n − 3‬‬

‫‪Ŕž Ŧ Ľ‬‬
‫= ‪I‬‬ ‫‪I‬‬ ‫‪−‬‬ ‫‪,‬‬ ‫‪I‬‬ ‫‪−‬‬ ‫=‬ ‫‪I‬‬ ‫‪−‬‬
‫‪2n‬‬ ‫‪2n − 2‬‬
‫‪2n − 5‬‬ ‫‪2n − 7‬‬
‫‪I‬‬ ‫=‬ ‫‪I‬‬ ‫‪,‬‬ ‫‪I‬‬ ‫=‬ ‫‪I‬‬

‫‪œǃ‬‬
‫‪−‬‬ ‫‪−‬‬ ‫‪−‬‬ ‫‪−‬‬
‫‪2n − 4‬‬ ‫‪2n − 6‬‬
‫⋮‬ ‫‪,‬‬ ‫⋮‬
‫‪9‬‬ ‫‪5‬‬
‫‪I = I‬‬ ‫‪,‬‬ ‫‪I = I‬‬
‫‪8‬‬ ‫‪6‬‬

‫‪Ŀ‬‬
‫‪3‬‬ ‫‪1‬‬
‫‪I = I‬‬ ‫‪,‬‬ ‫‪I = I‬‬
‫‪4‬‬ ‫‪2‬‬

‫‪2n − 1‬‬ ‫‪2n − 1 2n − 3‬‬


‫= ‪I‬‬ ‫= ‪I −‬‬ ‫×‬ ‫‪I‬‬ ‫⋯=‬ ‫ﻣﻨﻪ‬
‫‪2n‬‬ ‫‪2n‬‬ ‫‪2n − 2 −‬‬
‫‪2n − 1 2n − 3‬‬ ‫‪3‬‬ ‫‪2n − 1 2n − 3‬‬ ‫‪3 1‬‬
‫=⋯=‬ ‫×‬ ‫= ‪×⋯× I‬‬ ‫×‬ ‫‪×⋯× × I‬‬
‫‪2n‬‬ ‫‪2n − 2‬‬ ‫‪4‬‬ ‫‪2n‬‬ ‫‪2n − 2‬‬ ‫‪4 2‬‬
‫‪(2n − 1) (2n − 3) ⋯ 3 × 1‬‬ ‫‪(2n) (2n − 2) ⋯ 4 × 2‬‬
‫=‬ ‫× ‪I‬‬
‫‪(2n) (2n − 2) ⋯ 4 × 2‬‬ ‫‪(2n) (2n − 2) ⋯ 4 × 2‬‬
‫‪(2n) (2n − 1) (2n − 2) (2n − 3) ⋯ 4 × 3 × 2 × 1‬‬
‫=‬ ‫‪I‬‬
‫)‪2n ⋅ 2 (n − 1) ⋅ 2 (n − 2) ⋯ (2 × 2) (2 × 1‬‬
‫! )‪(2n‬‬ ‫! )‪(2n‬‬ ‫! )‪(2n‬‬ ‫𝜋‬
‫=‬ ‫= ‪I‬‬ ‫= ‪I‬‬ ‫⋅‬
‫‪2 n (n − 1) (n − 2) ⋯ 2 × 1‬‬ ‫) !‪2 (n‬‬ ‫‪4 (n! ) 2‬‬

‫و ﻳﻤﻜﻦ اﻟﺘﺤﻘﻖ ﻣﻦ ﺻﺤﺔ ﻫﺬه اﻷﺧﲑة ﺑﺎﻟﱰاﺟﻊ ‪:‬‬


‫! )‪(0‬‬‫𝜋‬ ‫𝜋‬
‫ﻣﻨﻪ اﳋﺎﺻﻴﺔ ﺻﺤﻴﺤﺔ ﻋﻨﺪ اﻟﺮﺗﺒﺔ ‪. n = 0‬‬ ‫= ‪I‬و‬ ‫ﻷﺟﻞ ‪ n = 0‬ﻟﺪﻳﻨﺎ‬ ‫•‬
‫‪4 (0! ) 2‬‬ ‫‪2‬‬
‫! )‪(2n‬‬ ‫𝜋‬
‫= ‪ I‬ﻓﺈن ‪:‬‬ ‫⋅‬ ‫أن اﳋﺎﺻﻴﺔ ﺻﺤﻴﺤﺔ ﻋﻨﺪ اﻟﺮﺗﺒﺔ ‪ n‬أي إذا ﻓﺮﺻﻨﺎ ّ‬
‫أن‬ ‫إذا ﻓﺮﺻﻨﺎ ّ‬ ‫•‬
‫) !‪4 (n‬‬ ‫‪2‬‬
‫‪ņőƱŀ Ţ‬‬
‫! )‪2n + 1 (2n‬‬ ‫)‪𝜋 (2n + 2) (2n + 1‬‬ ‫! )‪(2n‬‬ ‫𝜋‬
‫‪I‬‬ ‫‪+‬‬ ‫=‬ ‫= ⋅‬ ‫⋅‬ ‫⋅‬
‫‪2n + 2 4 (n! ) 2‬‬ ‫)‪(2n + 2‬‬ ‫‪4 (n! ) 2‬‬
‫! )‪(2n + 2‬‬ ‫𝜋‬ ‫! )‪(2n + 2‬‬ ‫𝜋‬
‫=‬ ‫‪⋅ = +‬‬ ‫⋅‬
‫‪4 × 4 ((n + 1) × n! ) 2 4‬‬ ‫‪((n + 1) ! ) 2‬‬

‫أي ّ‬
‫أن اﳋﺎﺻﻴﺔ ﺗﺒﻘﻰ ﺻﺤﻴﺤﺔ ﻋﻨﺪ اﻟﺮﺗﺒﺔ ‪. n + 1‬‬

‫! )‪(2n‬‬ ‫𝜋‬
‫= ‪∀n ∈ ℕ : I‬‬ ‫⋅‬ ‫و ﺣﺴﺐ ﻣﺒﺪأ اﻟﱪﻫﺎن ﺑﺎﻟﱰاﺟﻊ‪ ،‬ﻓﺎﳋﺎﺻﻴﺔ ﺻﺤﻴﺤﺔ ﻟﻜﻞ ﻋﺪد ﻃﺒﻴﻌﻲ أي ‪:‬‬
‫) !‪4 (n‬‬ ‫‪2‬‬
‫‪ƕ‬‬
‫‪VII‬‬

‫‪u (t) = −2n sin t ⋅ cos‬‬ ‫‪−‬‬ ‫‪ u(t) = cos‬و ‪ v (t) = 1‬ﻣﻨﻪ ‪t‬‬ ‫ﻧُﺠﺮي ﰲ ‪ I‬ﺗﻜﺎﻣ ً‬
‫ﻼ ﺑﺎﻟﺘﺠﺰﺋﺔ ﺑﻮﺿﻊ ‪t‬‬ ‫)ا(‬ ‫‪3‬‬
‫•‬
‫و ‪. v(t) = t‬‬
‫‪ũŏ‬‬

‫اﻟﺪاﻟﺘﺎن ‪ u‬و ‪ v‬ﲢﻘﻘﺎن ﴍوط ﻧﻈﺮﻳﺔ اﻟﺘﻜﺎﻣﻞ ﺑﺎﻟﺘﺠﺰﺋﺔ ﻋﲆ اﳌﺠﺎل ]‪ [0, 𝜋/2‬ﻣﻨﻪ ‪:‬‬
‫‪/‬‬ ‫‪/‬‬
‫‪/‬‬ ‫‪−‬‬ ‫‪−‬‬
‫‪I = t cos‬‬ ‫‪t‬‬ ‫‪−‬‬ ‫‪−2n sin t ⋅ cos‬‬ ‫‪t dt = 2n‬‬ ‫‪sin t ⋅ cos‬‬ ‫‪t dt‬‬

‫‪t‬‬
‫= )‪v(t‬‬ ‫‪ u(t) = sin t ⋅ cos‬و ‪ v (t) = t‬ﻣﻨﻪ‬ ‫ﻧُﺠﺮي ﻣﺮة أﺧﺮى ﺗﻜﺎﻣ ً‬
‫ﻼ ﺑﺎﻟﺘﺠﺰﺋﺔ ﺑﻮﺿﻊ ‪t‬‬
‫‪2‬‬
‫‪. u (t) = cos‬‬ ‫‪t − (2n − 1) sin t ⋅ cos‬‬ ‫‪−‬‬ ‫و‪t‬‬

‫𝟯𝟲𝟯‬

‫‪http ://tinyurl.com/Malki1718‬‬ ‫‪0‬‬


.
: ‫[ ﻣﻨﻪ‬0, 𝜋/2] ‫ ﲢﻘﻘﺎن ﴍوط ﻧﻈﺮﻳﺔ اﻟﺘﻜﺎﻣﻞ ﺑﺎﻟﺘﺠﺰﺋﺔ ﻋﲆ اﳌﺠﺎل‬v ‫ و‬u ‫اﻟﺪاﻟﺘﺎن‬

Ŕž Ŧ Ľ
/ /
t t −
I = 2n cos t − 2n cos t − (2n − 1) sin t ⋅ cos t dt
2 2

œǃ
/
t −
= 0 − 2n cos t − (2n − 1) 1 − cos t cos t dt
2

Ŀ
/ / /

= −n t cos t dt + n (2n − 1) t cos − t dt − n (2n − 1) t cos t dt

= −nJ + n (2n − 1) J −
− n (2n − 1) J
I = n (2n − 1) J −
− 2n J : ‫أي‬

4 (n! )
: (‫•ب‬2) ‫و ﻧﺴﺘﻌﻤﻞ ﻧﺘﻴﺠﺔ اﻟﺴﺆال‬ ‫ﻃﺮﰲ اﳌﺴﺎواة اﻟﺴﺎﺑﻘﺔ ﺑﺎﳌﻘﺪار‬
ْ ‫)ب( ﻧﴬب‬
(2n) !

4 (n! ) 4 (n! )
I = n (2n − 1) J −
− 2n J
(2n) ! (2n) !

𝜋 4 ((n − 1) ! ) 4 (n! )
= 2n J −
− J : ‫أي‬
2 (2n − 2) ! (2n) !

4 (n! ) 4n 4 ((n − 1) ! )
⋅ n (2n − 1) = ⋅ ⋅ n (2n − 1) ّ
: ‫ﻷن‬
(2n) ! (2n) (2n − 1) (2n − 2) !

4 ((n − 1) ! )
= 2n ⋅
(2n − 2) !

𝜋 1 4 ((n − 1) ! ) 4 (n! )
× = J − J : 2n ‫و ﺑﻘﺴﻤﺔ اﻟﻄﺮﻓﲔ ﻋﲆ‬
ņőƱŀ Ţ
2 2n (2n − 2) ! − (2n) !
𝜋
K − −K = : ‫أي‬
4n

(‫)ج‬

𝜋
K − −K = : ‫ﻟﺪﻳﻨﺎ‬
4k
ƕ

𝜋
(K −K )= : ‫ﻣﻨﻪ‬
VII


= = 4k
𝜋 1
K −K = : ‫أي‬
4 k
ũŏ

𝜋 1
=J −K : ‫ ﻣﻨﻪ‬K = J ‫ﻟﻜﻦ‬
4 = k

𝜋
. [0, 𝜋/2] ‫ اﳌﻌﺮﻓﺔ ﻋﲆ اﳌﺠﺎل‬f(x) = sin x − x ‫ﻧﻌﺘﱪ اﻟﺪاﻟﺔ‬ (‫)ا‬ • 4
2

𝟯𝟲𝟰

http ://tinyurl.com/Malki1718 0
‫‪ .VII‬ﲤﺎرﻳﻦ ﻟﻠﺘﻌﻤﻖ‬
‫‪.‬‬
‫اﻟﺪاﻟﺔ ‪ f‬ﻣﺴﺘﻤﺮة و ﻗﺎﺑﻠﺔ ﻟﻺﺷﺘﻘﺎق ﻋﲆ ﻫﺬا اﳌﺠﺎل و ‪ . f (x) = 𝜋 cos x − 1 = 𝜋 cos x − 2‬و ﺑﲈ ّ‬
‫أن‬
‫‪2‬‬ ‫‪2‬‬ ‫𝜋‬

‫‪Ŕž Ŧ Ľ‬‬
‫‪2‬‬ ‫‪2‬‬
‫= 𝛼 ‪) cos‬اﻟﻌﺪد 𝛼 وﺣﻴﺪ ّ‬
‫ﻷن اﻟﺪاﻟﺔ ‪ cos‬ﻣﺴﺘﻤﺮة و ﻣﺘﻨﺎﻗﺼﺔ‬ ‫< ‪ 0‬ﻓﺈﻧﻪ ﻳﻮﺟﺪ [‪ 𝛼 ∈ ]0, 𝜋/2‬ﺑﺤﻴﺚ‬ ‫‪<1‬‬
‫𝜋‬ ‫𝜋‬
‫ﲤﺎﻣ ًﺎ ﻋﲆ اﳌﺠﺎل ]‪ [0, 𝜋/2‬أي ﺗﻘﺎﺑﻠﻴﺔ ﻋﲆ ﻫﺬا اﳌﺠﺎل( ‪.‬‬

‫‪œǃ‬‬
‫𝜋‬
‫أي ‪:‬‬ ‫‪𝛼,‬‬ ‫اﻟﺪاﻟﺔ ‪ f‬ﻣﺘﺰاﻳﺪة ﲤﺎﻣ ًﺎ ﻋﲆ اﳌﺠﺎل ]𝛼 ‪ [0,‬و ﻣﺘﻨﺎﻗﺼﺔ ﲤﺎﻣ ًﺎ ﻋﲆ اﳌﺠﺎل‬
‫‪2‬‬
‫ﻟﻜﻞ ]𝛼 ‪ x ∈ [0,‬ﻟﺪﻳﻨﺎ ‪0 = f(0) ≤ f(x) ≤ f(𝛼) :‬‬
‫𝜋‬
‫و ﻟﻜﻞ 𝜋 ‪ّ x ∈ 𝛼,‬‬
‫ﻓﺈن ‪. 0 = f( ) ≤ f(x) ≤ f(𝛼) :‬‬

‫‪Ŀ‬‬
‫‪2‬‬ ‫‪2‬‬
‫𝜋‬ ‫𝜋‬ ‫𝜋‬
‫‪. ∀x ∈ 0,‬‬ ‫‪, x ≤ sin x‬‬ ‫ﰲ اﻷﺧﲑ‪ ،‬ﻟﻜﻞ ‪ّ x ∈ 0,‬‬
‫ﻓﺈن ‪ f(x) ≥ 0‬أي‬
‫‪2‬‬ ‫‪2‬‬ ‫‪2‬‬
‫‪) cos‬ﻣﻮﺟﺐ( ﻳﻨﺘﺞ ‪:‬‬ ‫ﺛﻢ ﺑﴬب اﻟﻄﺮﻓﲔ ﺑﺎﳌﻘﺪار ‪t‬‬
‫ﻃﺮﰲ اﳌﺘﺒﺎﻳﻨﺔ اﻟﺴﺎﺑﻘﺔ )ﻛﻞ اﳊﺪود ﻣﻮﺟﺒﺔ ( ّ‬
‫ْ‬ ‫ﺑﱰﺑﻴﻊ‬ ‫)ب(‬
‫𝜋‬ ‫𝜋‬
‫‪∀t ∈ 0,‬‬ ‫‪, 0 ≤ t cos‬‬ ‫≤‪t‬‬ ‫‪sin t ⋅ cos‬‬ ‫‪t‬‬
‫‪2‬‬ ‫‪4‬‬
‫ﺑﺎﻟﺘﻜﺎﻣﻞ ﺑﲔ ‪ 0‬و 𝜋 )ﻧﻼﺣﻆ ّ‬
‫أن ﺣﺪود اﻟﺘﻜﺎﻣﻞ ﰲ اﻹﲡﺎه اﳌﻨﺎﺳﺐ( ﻧﺠﺪ ‪:‬‬
‫‪2‬‬
‫‪/‬‬ ‫‪/‬‬ ‫‪/‬‬
‫𝜋‬
‫≤ ‪0 dt‬‬ ‫‪t cos‬‬ ‫≤ ‪t dt‬‬ ‫‪sin t ⋅ cos‬‬ ‫‪t dt‬‬
‫‪4‬‬
‫‪/‬‬
‫𝜋‬
‫≤ ‪0≤J‬‬ ‫‪sin t ⋅ cos‬‬ ‫‪t dt‬‬ ‫أي ‪:‬‬
‫‪4‬‬

‫‪v (t) = sin t ⋅ cos‬‬ ‫ﻹﳚﺎد اﻟﺘﻜﺎﻣﻞ اﻷﺧﲑ‪ ،‬ﻧﺴﺘﻌﻤﻞ اﻟﺘﻜﺎﻣﻞ ﺑﺎﻟﺘﺠﺰﺋﺔ و ذﻟﻚ ﺑﺄﺧﺬ ‪ u(t) = sin t‬و ‪t‬‬
‫‪1‬‬
‫‪. v(t) = −‬‬ ‫ﻣﻨﻪ ‪ u (t) = cos t‬و ‪cos + t‬‬
‫‪2n + 1‬‬
‫اﻟﺪاﻟﺘﺎن ‪ u‬و ‪ v‬ﲢﻘﻘﺎن ﴍوط ﻧﻈﺮﻳﺔ اﻟﺘﻜﺎﻣﻞ ﺑﺎﻟﺘﺠﺰﺋﺔ ﻋﲆ اﳌﺠﺎل ]‪ [0, 𝜋/2‬ﻣﻨﻪ ‪:‬‬
‫‪ņőƱŀ Ţ‬‬
‫‪/‬‬ ‫‪/‬‬
‫‪1‬‬ ‫‪+‬‬ ‫‪1‬‬
‫‪sin t ⋅ cos‬‬ ‫‪t dt = 0 +‬‬ ‫‪cos‬‬ ‫= ‪t dt‬‬ ‫‪I‬‬ ‫‪+‬‬
‫‪2n + 1‬‬ ‫‪2n + 1‬‬

‫𝜋‬
‫≤ ‪.0 ≤ J‬‬ ‫‪I‬‬ ‫‪+‬‬ ‫ﻣﻨﻪ ‪:‬‬
‫)‪4 (2n + 1‬‬
‫) !‪4 (n‬‬
‫)ﻣﻮﺟﺐ ﲤﺎﻣ ًﺎ( و ﻧُﻄ ﱢﺒﻖ ﻧﺘﻴﺠﺔ اﻟﺴﺆال )‪•2‬ب( ﻟﻨﺠﺪ ‪:‬‬ ‫اﻵن‪ ،‬ﻧﴬب أﻃﺮاف ﻫﺬه اﳌﺘﺒﺎﻳﻨﺎت ﺑﺎﳌﻘﺪار‬
‫! )‪(2n‬‬
‫‪ƕ‬‬

‫) !‪4 (n‬‬ ‫𝜋‬ ‫! )‪(2n + 2‬‬ ‫𝜋‬


‫‪VII‬‬

‫≤ ‪0≤K‬‬ ‫⋅‬ ‫⋅‬ ‫‪+‬‬


‫⋅‬
‫! )‪(2n‬‬ ‫‪4 (2n + 1) 4‬‬ ‫) ! )‪((n + 1‬‬ ‫‪2‬‬

‫𝜋‬
‫≤ ‪0≤K‬‬ ‫أي ‪:‬‬
‫‪ũŏ‬‬

‫)‪16 (n + 1‬‬
‫𝜋‬
‫‪ lim‬ﻓﺤﺴﺐ ﻧﻈﺮﻳﺔ اﳊﴫ ﻓﺈن ‪. lim K = 0‬‬ ‫ﺑﲈ ّ‬
‫أن ‪= 0‬‬ ‫)ج(‬
‫∞‪→+‬‬ ‫∞‪→+‬‬ ‫)‪16 (n + 1‬‬
‫ﺑﺘﻄﺒﻴﻖ ﻧﺘﻴﺠﺔ اﻟﺴﺆال )‪•3‬ج( ﻧﺠﺪ ‪:‬‬
‫‪4‬‬ ‫‪4‬‬
‫‪S = lim S = lim‬‬ ‫‪J −K‬‬ ‫=‬ ‫‪J‬‬
‫∞‪→+‬‬ ‫𝜋 ∞‪→+‬‬ ‫𝜋‬

‫𝟱𝟲𝟯‬

‫‪http ://tinyurl.com/Malki1718‬‬ ‫‪0‬‬


.
/ /
t 𝜋
J = t dt = = ‫ﻟﻜﻦ‬
3 24

Ŕž Ŧ Ľ
𝜋 4 4 𝜋

œǃ
. S= ‫أي‬ S= J = ⋅ ‫ﻣﻨﻪ‬
6 𝜋 𝜋 24

(Noyau de D) ‫ إﺳﺘﻌﲈل ﻧﻮاة ﺩﻳﺮﻳﻜﻠﻲ‬: ‫اﳉﺰء اﻟﺜﺎﻟﺚ‬

Ŀ
ّ x ≠ 0 (mod 2𝜋) ‫ و إذا ﻛﺎن‬cos (kx) = Re e
: ‫ ﻣﻨﻪ‬e ≠ 1 ‫ﻓﺈن‬ ّ ‫• ﻧﻌﻠﻢ‬1
‫أن‬

cos (kx) = Re e
= =
1−e
= Re e ⋅ (e ≠ 1 ‫ و أﺳﺎﺳﻬﺎ‬e ‫)ﳎﻤﻮع ﻣﺘﺘﺎﻟﻴﺔ ﻫﻨﺪﺳﻴﺔ ﺣﺪﻫﺎ اﻷول‬
1−e
e / ⋅ −2ı sin
= Re e ⋅ /
(126 ‫ ﺻﻔﺤﺔ‬59 ‫)أﻧﻈﺮ اﻟﺘﻤﺮﻳﻦ‬
e ⋅ −2ı sin

+ /
sin
= Re e ⋅
sin
(n + 1) x sin
= cos
2 sin
1 (n + 1)x sin
D (x) = + cos : ‫ﻣﻨﻪ‬
2 2 sin
+
sin + 2 cos sin
=
2 sin
ņőƱŀ Ţ
(n + 1) x nx
‫ ﰲ ﻫﺬه اﳌﺘﻄﺎﺑﻘﺔ ﻳﻨﺘﺞ‬b = ‫و‬a = ‫ ؛ ﺑﺄﺧﺬ‬2 sin (a) cos (b) = sin (a + b) − sin (a − b) ‫ﻟﻜﻦ‬
2 2
+ +
: ‫ ﻣﻨﻪ‬2 cos sin = sin x − sin

1 sin n + x
∀x ≠ 0 (mod 2𝜋) , D (x) = ⋅
2 sin

1
. v(x) = sin (kx) ‫ و‬u (x) = 1 ‫ ﻣﻨﻪ‬v (x) = cos (kx) ‫ و‬u(x) = x ‫ﻧﺴﺘﻌﻤﻞ اﻟﺘﻜﺎﻣﻞ ﺑﺎﻟﺘﺠﺰﺋﺔ ﺑﻮﺿﻊ‬ (‫)ا‬ • 2
k
: ‫[ ﻣﻨﻪ‬0, 𝜋] ‫ ﲢﻘﻘﺎن ﴍوط ﻧﻈﺮﻳﺔ اﻟﺘﻜﺎﻣﻞ ﺑﺎﻟﺘﺠﺰﺋﺔ ﻋﲆ اﳌﺠﺎل‬v ‫ و‬u ‫اﻟﺪاﻟﺘﺎن‬
ƕ

VII

1 1
x cos (kx) dx = x sin (kx) − sin (kx) dx
k k
ũŏ

1 1
=0− − cos (kx)
k k
1 1
=− − (cos (k𝜋) − cos (0))
k k
1 (−1) − 1
= (−1) − 1 = ّ
(cos (k𝜋) = (−1) ‫)ﻷن‬
k k

𝟯𝟲𝟲

http ://tinyurl.com/Malki1718 0
‫‪ .VII‬ﲤﺎرﻳﻦ ﻟﻠﺘﻌﻤﻖ‬
‫‪.‬‬
‫)ب( ﻟﺪﻳﻨﺎ ‪:‬‬

‫‪Ŕž Ŧ Ľ‬‬
‫‪1‬‬
‫= ‪L‬‬ ‫= ‪xD (x) dx‬‬ ‫‪x‬‬ ‫‪+‬‬ ‫‪cos (kx) dx‬‬
‫‪2‬‬ ‫=‬

‫‪œǃ‬‬
‫‪x‬‬
‫=‬ ‫‪+‬‬ ‫‪x cos (kx) dx‬‬
‫‪2‬‬ ‫=‬

‫‪Ŀ‬‬
‫‪x‬‬
‫=‬ ‫‪dx +‬‬ ‫‪cos (kx) dx‬‬
‫‪2‬‬ ‫=‬

‫‪x‬‬
‫=‬ ‫‪+‬‬ ‫⎛‬ ‫⎞‪cos (kx) dx‬‬
‫‪4‬‬ ‫=‬ ‫⎝‬ ‫⎠‬
‫𝜋‬ ‫‪(−1) − 1‬‬
‫=‬ ‫‪+‬‬ ‫)ﺣﺴﺐ اﻟﺴﺆال اﻟﺴﺎﺑﻖ(‬
‫‪4‬‬ ‫=‬ ‫‪k‬‬

‫‪(−1) − 1‬‬ ‫‪2‬‬


‫ﻣﻨﻪ‬ ‫‪=−‬‬ ‫ﻓﺈن ‪ (−1) − 1 = 0‬و إذا ﻛﺎن ﻓﺮدﻳ ًﺎ ّ‬
‫ﻓﺈن‬ ‫)ج( إذا ﻛﺎن ‪ k‬زوﺟﻴ ًﺎ ّ‬
‫‪k‬‬ ‫‪k‬‬ ‫‪k‬‬
‫‪−‬‬
‫‪(−1) − 1‬‬ ‫‪−2‬‬
‫ﺣﻴﺚ )‪ E (x‬ﻫﻮ اﳉﺰء اﻟﺼﺤﻴﺢ ﻟﻠﻌﺪد ‪ . x‬و ﺑﺎﻟﺘﺎﱄ ّ‬
‫ﻓﺈن ‪:‬‬ ‫=‬
‫=‬ ‫‪k‬‬ ‫=‬ ‫)‪(2p + 1‬‬
‫‪−‬‬
‫∞‪+‬‬
‫𝜋‬ ‫‪1‬‬ ‫𝜋‬ ‫‪1‬‬
‫⎛ ‪lim L = lim‬‬ ‫‪−2‬‬ ‫=⎞‬ ‫‪−2‬‬
‫∞‪→+‬‬ ‫∞‪→+‬‬ ‫‪4‬‬ ‫=‬ ‫⎠ )‪(2p + 1‬‬ ‫‪4‬‬ ‫=‬ ‫)‪(2p + 1‬‬
‫⎝‬
‫𝜋‬
‫‪. lim‬‬ ‫= ‪L‬‬ ‫‪− 2V‬‬ ‫أي‬
‫∞‪→+‬‬ ‫‪4‬‬

‫‪ •3‬اﻟﺪاﻟﺔ ‪ f‬ﻣﺴﺘﻤﺮة و ﻗﺎﺑﻠﺔ ﻟﻺﺷﺘﻘﺎق ﻋﲆ اﳌﺠﺎل ]𝜋 ‪ ]0,‬و ﺑﲈ أﳖﺎ ﻗﺎﺑﻠﺔ ﻟﻺﺷﺘﻘﺎق ﻋﻨﺪ ‪ x = 0‬ﻓﻬﻲ ﻣﺴﺘﻤﺮة ﻋﻨﺪ ﻫﺬه‬
‫‪ņőƱŀ Ţ‬‬
‫اﻟﻨﻘﻄﺔ‪ .‬ﻟﻜﻦ ﺑﺎﻹﻣﻜﺎن إﺛﺒﺎت ذﻟﻚ ﻛﲈ ﻳﲇ ‪:‬‬
‫‪x‬‬ ‫‪= /‬‬ ‫‪t‬‬
‫‪lim f(x) = lim‬‬ ‫‪= lim‬‬ ‫⋅‪2‬‬ ‫=‬ ‫‪2 lim‬‬ ‫)‪= 2 × 1 = 2 = f(0‬‬
‫→‬ ‫→‬ ‫‪sin‬‬ ‫→‬ ‫‪sin‬‬ ‫→‬ ‫‪sin t‬‬

‫و ﻫﺬا ﻳﻌﻨﻲ أﳖﺎ ﻣﺴﺘﻤﺮة ﻋﻨﺪ ‪. x = 0‬‬


‫إذن اﻟﺪاﻟﺔ ‪ f‬ﻣﺴﺘﻤﺮة و ﻗﺎﺑﻠﺔ ﻟﻺﺷﺘﻘﺎق ﻋﲆ اﳌﺠﺎل اﳌﻐﻠﻖ ]𝜋 ‪ [0,‬و داﻟﺘﻬﺎ اﳌﺸﺘﻘﺔ ‪ f‬ﻣﺴﺘﻤﺮة ﻋﲆ ﻫﺬا اﳌﺠﺎل‪.‬‬

‫أن ‪ّ a ≤ b‬‬
‫ﻓﺈن ‪:‬‬ ‫)ا( ﻟﻜﻞ ]‪ x ∈ [a, b‬ﻟﺪﻳﻨﺎ ‪ h(x) ≤ ||h(x)|| :‬و ||)‪ −h(x) ≤ ||h(x‬و ﺑﲈ ّ‬ ‫‪4‬‬
‫•‬
‫‪ƕ‬‬
‫‪VII‬‬

‫‪−‬‬ ‫≤ ‪h(x) dx‬‬ ‫‪||h(x)|| dx‬‬ ‫و‬ ‫≤ ‪h(x) dx‬‬ ‫‪||h(x)|| dx‬‬

‫|‬ ‫|‬
‫‪ũŏ‬‬

‫|‬ ‫|‬
‫ّ‬
‫)ﻷن )‪ |t| = max (−t, t‬ﻟﻜﻞ ‪. (t ∈ ℝ‬‬ ‫|‬ ‫≤ |‪h(x) dx‬‬ ‫‪||h(x)|| dx‬‬ ‫ﻣﻨﻪ‬
‫|‬ ‫|‬
‫||‬ ‫||‬
‫)ب( ﻟﻴﻜﻦ ‪ . 𝜆 > 0‬ﺑﺎﻟﺘﻜﺎﻣﻞ ﺑﺎﻟﺘﺠﺰﺋﺔ‪ ،‬ﻧﻀﻊ )‪ u(x) = 𝜙 (x‬و )‪ v (x) = sin (𝜆x‬ﻣﻨﻪ )‪ u (x) = 𝜙 (x‬و‬
‫)‪cos (𝜆x‬‬
‫‪. v(x) = −‬‬
‫𝜆‬
‫اﻟﺪاﻟﺘﺎن ‪ u‬و ‪ v‬ﲢﻘﻘﺎن ﴍوط ﻧﻈﺮﻳﺔ اﻟﺘﻜﺎﻣﻞ ﺑﺎﻟﺘﺠﺰﺋﺔ ﻋﲆ اﳌﺠﺎل ]𝜋 ‪ [0,‬ﻣﻨﻪ ‪:‬‬

‫𝟳𝟲𝟯‬

‫‪http ://tinyurl.com/Malki1718‬‬ ‫‪0‬‬


‫‪.‬‬

‫‪Ŕž Ŧ Ľ‬‬
‫)𝜋𝜆( ‪𝜙 (0) 𝜙 (𝜋) cos‬‬ ‫)‪cos (𝜆x‬‬
‫= ‪𝜙 (x) sin (𝜆x) dx‬‬ ‫‪−‬‬ ‫‪+‬‬ ‫)‪𝜙 (x‬‬ ‫‪dx‬‬
‫𝜆‬ ‫𝜆‬ ‫𝜆‬

‫‪œǃ‬‬
‫|‬ ‫| |‬ ‫|‬
‫|‬ ‫)𝜋𝜆( ‪| | 𝜙 (0) 𝜙 (𝜋) cos‬‬ ‫| )‪cos (𝜆x‬‬
‫|‬ ‫| = |‪𝜙 (x) sin (𝜆x) dx‬‬ ‫‪−‬‬ ‫‪+‬‬ ‫)‪𝜙 (x‬‬ ‫|‪dx‬‬ ‫ﻣﻨﻪ ‪:‬‬
‫|‬ ‫𝜆 | |‬ ‫𝜆‬ ‫𝜆‬ ‫|‬
‫||‬ ‫|| ||‬ ‫||‬

‫‪Ŀ‬‬
‫|‬ ‫|‬
‫| | )𝜋𝜆( ‪| 𝜙 (0) | | 𝜙 (𝜋) cos‬‬ ‫| )‪cos (𝜆x‬‬
‫|≤‬ ‫|‪|+‬‬ ‫|‪|+‬‬ ‫)‪𝜙 (x‬‬ ‫|‪dx‬‬ ‫ّ‬
‫)ﻷن ||𝛽|| ‪(||𝛼 + 𝛽|| ≤ |𝛼| +‬‬
‫|| || 𝜆 ||‬ ‫𝜆‬ ‫| ||‬ ‫𝜆‬ ‫|‬
‫||‬ ‫||‬
‫||)𝜋𝜆( ‪| 𝜙 (0) | ||𝜙 (𝜋)|| ⋅ ||cos‬‬ ‫|‬ ‫| )‪cos (𝜆x‬‬
‫|≤‬ ‫‪|+‬‬ ‫‪+‬‬ ‫)‪|𝜙 (x‬‬ ‫‪| dx‬‬ ‫)ﺣﺴﺐ اﻟﺴﺆال اﻟﺴﺎﺑﻖ(‬
‫|| 𝜆 ||‬ ‫𝜆‬ ‫||‬ ‫𝜆‬ ‫||‬

‫||)𝜋( 𝜙|| | )‪| 𝜙 (0‬‬ ‫|‬ ‫|‬


‫|≤‬ ‫‪|+‬‬ ‫‪+‬‬ ‫‪||𝜙 (x)|| ⋅ | cos (𝜆x) | dx‬‬ ‫ّ‬
‫)ﻷن ‪(|cos t| ≤ 1‬‬
‫|| 𝜆 ||‬ ‫𝜆‬ ‫||‬ ‫𝜆‬ ‫||‬

‫||)𝜋( 𝜙|| | )‪| 𝜙 (0‬‬


‫|≤‬ ‫‪|+‬‬ ‫‪+‬‬ ‫‪||𝜙 (x)|| 1 dx‬‬
‫|| 𝜆 ||‬ ‫𝜆‬ ‫𝜆‬

‫⎡‪1‬‬ ‫⎤‪||𝜙 (x)|| dx‬‬


‫≤‬ ‫‪||𝜙 (0)|| + ||𝜙 (𝜋)|| +‬‬
‫𝜆‬
‫⎣‬ ‫⎦‬

‫⎡‪1‬‬ ‫‪||𝜙 (x)|| dx⎤ = 0‬‬


‫‪0 ≤ lim‬‬ ‫‪𝜙 (x) sin (𝜆x) dx ≤ lim‬‬ ‫‪||𝜙 (0)|| + ||𝜙 (𝜋)|| +‬‬ ‫ﻣﻨﻪ ‪:‬‬
‫∞‪→+‬‬ ‫𝜆 ∞‪→+‬‬
‫⎣‬ ‫⎦‬
‫‪ņőƱŀ Ţ‬‬
‫‪lim‬‬ ‫‪𝜙 (x) sin (𝜆x) dx = 0‬‬ ‫أي ‪:‬‬
‫∞‪→+‬‬

‫ﻣﻼﺣﻈﺔ ‪ ⧏ : 60‬ﻫﺬه اﻟﻨﺘﻴﺠﺔ ﺗﺒﻘﻰ ﺻﺤﻴﺤﺔ إذا ﻓﺮﺿﻨﺎ ﻓﻘﻂ ّ‬


‫أن اﻟﺪاﻟﺔ 𝜙 ﻣﺴﺘﻤﺮة ﻋﲆ اﳌﺠﺎل ]𝜋 ‪ ، [0,‬ﻟﻜﻦ‬
‫⧐‬
‫إﺛﺒﺎﲥﺎ ﺧﺎرج ﻋﻦ ﻧﻄﺎق ﺑﺮﻧﺎﻣﺞ اﻟﺜﺎﻧﻮﻳﺔ‪.‬‬

‫)ا( ﻟﻴﻜﻦ ‪ . n ≥ 1‬ﻟﺪﻳﻨﺎ ‪:‬‬ ‫•‬ ‫‪5‬‬


‫‪1‬‬ ‫‪sin‬‬ ‫‪n+‬‬ ‫‪x‬‬ ‫‪1‬‬
‫‪ƕ‬‬

‫= ‪L‬‬ ‫= ‪xD (x) dx‬‬ ‫⋅‪x‬‬ ‫= ‪dx‬‬ ‫‪𝜙 (x) sin (𝜆x) dx‬‬
‫‪VII‬‬

‫‪2‬‬ ‫‪sin‬‬ ‫‪2‬‬

‫‪1‬‬ ‫‪x‬‬
‫= )‪ 𝜙 (x‬و ‪. 𝜆 = n +‬‬ ‫ﻣﻊ‬
‫‪2‬‬ ‫‪sin‬‬
‫‪ũŏ‬‬

‫⟶ 𝜆 ﻣﻨﻪ ‪:‬‬ ‫ﺣﺴﺐ اﻟﺴﺆال )‪ ، (•3‬ﻫﺬه اﻟﺪاﻟﺔ 𝜙 ُﲢ ﱢﻘﻖ ﴍوط اﻟﺴﺆال )‪•4‬ب( و ∞‪+‬‬
‫∞‪→+‬‬

‫‪1‬‬
‫= ‪lim L‬‬ ‫⎛ ‪lim‬‬ ‫‪𝜙 (x) sin (𝜆x) dx⎞ = 0‬‬
‫∞‪→+‬‬ ‫∞‪2 →+‬‬
‫⎝‬ ‫⎠‬
‫𝜋‬ ‫𝜋‬
‫= ‪.V‬‬ ‫= ‪ 0‬ﻣﻨﻪ‬ ‫ﺣﺴﺐ اﻟﺴﺆال )‪•2‬ج( ﻓﺈن ‪− 2V‬‬ ‫)ب(‬
‫‪8‬‬ ‫‪4‬‬

‫𝟴𝟲𝟯‬

‫‪http ://tinyurl.com/Malki1718‬‬ ‫‪0‬‬


‫ ﲤﺎرﻳﻦ ﻟﻠﺘﻌﻤﻖ‬.VII
.
𝜋 4 4 𝜋 3
. S= ‫ أي‬S = V = ⋅ ‫ ﻣﻨﻪ‬V = S ‫ﻟﻜﻦ‬
6 3 3 8 4

Ŕž Ŧ Ľ

œǃ
Ŀ
ņőƱŀ Ţ
ƕ
VII

ũŏ

𝟯𝟲𝟵

http ://tinyurl.com/Malki1718 0
.

Ŕž Ŧ Ľ
œǃ
Ŀ
ņőƱŀ Ţ
ƕ

VII
ũŏ

http ://tinyurl.com/Malki1718 0
.

Ŕž Ŧ Ľ
œǃ
Ŀ ‫ﻣﻼﺣﻖ‬
ņőƱŀ Ţ
ƕ
VII

ũŏ

𝟯𝟳𝟭

http ://tinyurl.com/Malki1718 0
.

Ŕž Ŧ Ľ
œǃ
Ŀ
ņőƱŀ Ţ
ƕ

VII
ũŏ

http ://tinyurl.com/Malki1718 0
‫‪.‬‬

‫‪Ŕž Ŧ Ľ‬‬
‫ا‬

‫‪œǃ‬‬
‫‪otttttttttttttttttttttttttttttttttttttttttttp‬‬
‫‪s‬‬ ‫‪u‬‬

‫ﻣﻠﺤﻖ ا‬
‫‪s‬‬ ‫ﻛﺜﻴﺮات اﻟﺤﺪود‬
‫‪u‬‬

‫‪Ŀ‬‬
‫‪s‬‬ ‫‪u‬‬
‫‪qvvvvvvvvvvvvvvvvvvvvvvvvvvvvvvvvvvvvvvvvvvvr‬‬
‫‪.‬‬

‫ﰲ ﻣﺎ ﻳﲇ‪ 𝕂 ،‬ﻳﺮﻣﺰ إﱃ إﺣﺪى اﳌﺠﻤﻮﻋﺘﲔ ‪ ℝ‬أو ‪ . ℂ‬أﻏﻠﺐ اﻟﻨﺘﺎﺋﺞ اﳌﺬﻛﻮرة ﻫﻨﺎ ﺗﺒﻘﻰ ﺻﺤﻴﺤﺔ ﰲ اﳊﺎﻟﺔ اﻟﻌﺎﻣﺔ )ﻋﻨﺪﻣﺎ ﻳﻜﻮن‬
‫𝕂 ﺣﻠﻘﺔ ﺗﺒﺪﻳﻠﻴﺔ واﺣﺪﻳﺔ ﻛﻴﻔﻴﺔ أو ﺟﺴﻢ – ﺣﻘﻞ – ﻛﻴﻔﻲ( ﻟﻜﻦ ﻻ ﻧﻮﱄ ذﻟﻚ اﻫﺘﲈﻣ ًﺎ ﰲ دراﺳﺘﻨﺎ ﻫﺬه‪.‬‬

‫ﳎﻤﻮﻋﺔ ﻛﺜﲑﺍﺕ ﺍﳊﺪﻭﺩ ﲟﻌﺎﻣﻼﺕ ﰲ 𝕂‬ ‫ﺍ‪1.‬‬


‫ﰲ اﻷﻗﺴﺎم اﻟﺜﺎﻧﻮﻳﺔ‪ ،‬ﻳﺘﻄﺮق اﻟﻄﺎﻟﺐ إﱃ ﻣﻔﻬﻮم ﻛﺜﲑ ﺍﳊﺪﻭﺩ أو ﺩﺍﻟﺔ ﻛﺜﲑ ﺍﳊﺪﻭﺩ و ﰲ اﳊﺎﻟﺘﲔ ﻳﻜﻮن اﳌﻘﺼﻮد ﻫﻮ اﻟﺪاﻟﺔ اﻟﻌﺪدﻳﺔ‬
‫‪ x ↦ a x + a − x − + ⋯ + a x + a‬ﺣﻴﺚ ‪ n ∈ ℕ‬و 𝕂 ∈ ‪ ، a , a , ⋯ , a‬و اﳌﻌﺮﻓﺔ ﻋﲆ 𝕂 ‪ .‬و إذا ﻛﺎن‬
‫ﻓﺈن )‪ P (sin‬ﻳﺮﻣﺰ إﱃ اﻟﺪاﻟﺔ ‪ . x ↦ sin x + 2 sin x − sin x‬ﻧﻜﺘﺐ )‪ P (sin‬ﻟﻜﻦ ﻻ ﻧﻨﺴﻰ ّ‬
‫أن‬ ‫‪ّ P (X) = X + 2X − X‬‬
‫ﻣﻌﺮﻓﺔ ﻋﲆ 𝕂 و ‪ sin‬داﻟﺔ و ﻟﻴﺴﺖ ﻋﺪد ًا !‬ ‫ﳎﺮد ﻛﺘﺎﺑﺔ ّ‬
‫ﻷن ‪ّ P‬‬ ‫ﻫﺬا ّ‬
‫= ‪ ،M‬و ﻳﺘﻌﻠﻢ ﻛﻴﻒ ُﳚﺮي ﻋﻠﻴﻬﺎ‬ ‫‪−‬‬ ‫ﰲ اﳌﺴﺘﻘﺒﻞ‪ ،‬ﻋﻨﺪﻣﺎ ﻳﺘﻄﺮق اﻟﻄﺎﻟﺐ إﱃ ﺍﳌﺼﻔﻮﻓﺎﺕ ‪ ،‬و ﻫﻲ ﺟﺪاول أﻋﺪاد ﻣﺜﻞ‬
‫اﻟﻌﻤﻠﻴﺎت اﳊﺴﺎﺑﻴﺔ )ﲨﻊ‪ ،‬ﴐب‪ ... ،‬إﻟﺦ( ﺗﻜﻮن اﻟﻌﺒﺎرة )‪ P (M‬أي ‪ M + 2M − M‬ﺑﺎﻟﻨﺴﺒﺔ إﻟﻴﻪ ﻫﻲ اﳌﺼﻔﻮﻓﺔ اﻟﺘﻲ‬
‫‪ņőƱŀ Ţ‬‬
‫ﺛﻢ اﳉﻤﻊ‪.‬‬ ‫ﺗﻨﺘﺞ ﻋﻦ ‪ M‬ﺑﺘﻄﺒﻴﻖ اﳉﺪاء ) ‪ 2M ، M‬و ‪ّ (−M‬‬
‫و ﺑﺼﻔﺔ ﻋﺎﻣﺔ‪ ،‬ﻣﺎ إن ﻧﺠﺪ ﰲ اﻟﺮﻳﺎﺿﻴﺎت ﻣﻔﻬﻮﻣ ًﺎ ﻳﻤﻜﻦ إﺟﺮاء اﻟﻌﻤﻠﻴﺎت اﳊﺴﺎﺑﻴﺔ ﻋﻠﻴﻪ‪ ،‬إﻻّ و ﻛﺎن ﺑﻤﻘﺪورﻧﺎ إﻧﺸﺎء ﻛﺜﲑات‬
‫أن ﺍﻷﻫﻢ ﰲ ﻛﺜﲑﺍﺕ ﺍﳊﺪﻭﺩ ﻫﻲ ﺍﳌﻌﺎﻣﻼﺕ ﻭ ﻣﻮﺿﻊ )ﺭﺗﺒﺔ( ﻫﺬﻩ ﺍﳌﻌﺎﻣﻼﺕ ﻭ ﻻ ﻏﲑ !‬‫ﺣﺪود ﺣﻮل ﻫﺬا اﳌﻔﻬﻮم‪ ،‬ﻫﺬا ﻳﻌﻨﻲ ّ‬

‫ﺗﻌﺮﻳﻒ ‪: 17‬‬

‫ﻧﺴﻤﻲ ﻛﺜﲑ ﺣﺪود ﺑﻤﻌﺎﻣﻼت ﰲ 𝕂 ﻛﻞ ﻋﺒﺎرة ﻣﻦ اﻟﺸﻜﻞ ‪:‬‬ ‫•‬


‫‪ƕ‬‬

‫= )‪P (X‬‬ ‫‪a X =a +a X+a X +⋅+a X‬‬


‫=‬

‫ﺣﻴﺚ اﳌﻌﺎﻣﻼت ‪ a ، ⋯ ، a ، a‬ﺗﻨﺘﻤﻲ إﱃ 𝕂 و ‪ X‬ﻳﺴﻤﻰ اﳌﺠﻬﻮل‪.‬‬


‫‪ũŏ‬‬

‫اﻟﻌﺪد اﻟﻄﺒﻴﻌﻲ ‪ n‬ﻳﺴﻤﻰ درﺟﺔ ﻛﺜﲑ اﳊﺪود ‪ P‬و ﻧﻜﺘﺐ ‪ n = d∘ P‬و اﳌﻌﺎﻣﻞ ‪ُ a‬ﻳﺴﻤﻰ ﻣﻌﺎﻣﻞ اﳊﺪ اﳌﻬﻴﻤﻦ )أو‬ ‫•‬

‫إﺧﺘﺼﺎر ًا اﳌﻌﺎﻣﻞ اﳌﻬﻴﻤﻦ(‪ ،‬و إذا ﻛﺎن ‪ a = 1‬ﻧﻘﻮل ّ‬


‫أن ‪ P‬واﺣﺪي‪.‬‬

‫إذا ﻛﺎﻧﺖ ﻛﻞ اﳌﻌﺎﻣﻼت ﻣﻌﺪوﻣﺔ ﻣﺎ ﻋﺪا ﻣﻌﺎﻣﻞ اﳊﺪ اﻷول أي إذا ﻛﺎن 𝕂 ∈ ‪ P (X) = a‬ﻧﻘﻮل أن ﻛﺜﲑ اﳊﺪود ‪P‬‬ ‫•‬
‫ﺛﺎﺑﺖ‪.‬‬

‫𝟯𝟳𝟯‬

‫‪http ://tinyurl.com/Malki1718‬‬ ‫‪0‬‬


‫ا‪ .1.‬ﶍﻮﻋﺔ ﻛﺜﲑات اﳊﺪود ﲟﻌﺎﻣﻼت ﰲ 𝕂‬
‫‪.‬‬

‫إذا ﻛﺎﻧﺖ ﻛﻞ اﳌﻌﺎﻣﻼت ﻣﻌﺪوﻣﺔ أي إذا ﻛﺎن ‪ P (X) = 0‬ﻧﻘﻮل ّ‬


‫أن ﻛﺜﲑ اﳊﺪود ‪ P‬ﻣﻌﺪوم‪.‬‬ ‫•‬

‫‪Ŕž Ŧ Ľ‬‬
‫إﺻﻄﻼﺣ ًﺎ‪ ،‬درﺟﺔ ﻛﺜﲑ اﳊﺪود اﳌﻌﺪوم ﻫﻲ ∞‪ −‬أي ∞‪. d∘ 0 = −‬‬

‫ا‬
‫‪œǃ‬‬
‫ﻧﺮﻣﺰ ﺑــِ ]‪ 𝕂[X‬إﱃ ﳎﻤﻮﻋﺔ ﻛﺜﲑات اﳊﺪود ﺑﻤﻌﺎﻣﻼت ﰲ اﳌﺠﻤﻮﻋﺔ 𝕂 ‪.‬‬ ‫•‬

‫ﻣﺜﺎﻝ ‪ : 26‬ﻟﻴﻜﻦ ]‪ P (X) = 2X − 28X + √2X − X − 12 ∈ ℝ[X‬ﻛﺜﲑ ﺣﺪود ﺑﻤﻌﺎﻣﻼت ﺣﻘﻴﻘﻴﺔ‪.‬‬

‫‪Ŀ‬‬
‫درﺟﺘﻪ ﻫﻲ ‪ n = d∘ P = 15‬و اﳌﻌﺎﻣﻞ اﳌﻬﻴﻤﻦ ﻫﻮ ‪. a = 2‬‬
‫‪ u‬ﻳﻤﻜﻦ إﻋﻄﺎء ﺗﻌﺮﻳﻒ آﺧﺮ و ﻫﻮ اﻵﰐ ‪ :‬ﻧﺴﻤﻲ ﻛﺜﲑ ﺣﺪود ﺑﻤﻌﺎﻣﻼت ﰲ 𝕂 ﻛﻞ ﻣﺘﺘﺎﻟﻴﺔ )ﻣﺘﺘﺎﺑﻌﺔ( أﻋﺪاد ﻣﻦ 𝕂 ﻣﻌﺪوﻣﺔ‬
‫اﺑﺘﺪاءً ﻣﻦ رﺗﺒﺔ ﻣﻌﻴﻨﺔ )و ﻧﻘﻮل أﳖﺎ ﺗﻘﺮﻳﺒ ًﺎ ﻣﻌﺪوﻣﺔ( ‪.‬‬
‫أن ﻫﺬﻳﻦ اﻟﺘﻌﺮﻳﻔﲔ ﻣﺘﻜﺎﻓﺌﺎن‪.‬‬ ‫ﺳﻨﺮى ﻓﻴﲈ ﺑﻌﺪ ّ‬
‫ﻧﻀﻊ )‪ n = max (s, t‬و ﻧﻜﺘﺐ = )‪P (X‬‬ ‫= )‪. Q (X‬‬ ‫‪b X‬‬ ‫= )‪ P (X‬و‬ ‫‪a X‬‬ ‫ﻟﻴﻜﻦ ]‪ P, Q ∈ 𝕂[X‬ﻣﻊ‬
‫=‬ ‫=‬

‫= )‪ Q (X‬ﺑﺤﻴﺚ ‪ a = 0‬إذا ﻛﺎن ‪ k > s‬و ‪ b = 0‬إذا ﻛﺎن ‪. k > t‬‬ ‫‪b X‬‬ ‫و‬ ‫‪a X‬‬
‫=‬ ‫=‬
‫ُﻌﺮف اﻟﻌﻤﻠﻴﺎت اﻟﺘﺎﻟﻴﺔ ﻋﲆ ﻛﺜﲑات اﳊﺪود ‪ :‬اﳉﻤﻊ‪ ،‬اﳉﺪاء و اﻟﴬب ﺑﻌﺪد ﻛﲈ ﻳﲇ ‪:‬‬
‫ﻟﻴﻜﻦ 𝕂 ∈ 𝜆 ‪ .‬ﻧ ﱢ‬

‫= )‪(P + Q) (X‬‬ ‫‪(a + b ) X‬‬


‫=‬

‫= )‪(𝜆P) (X‬‬ ‫‪𝜆a X‬‬


‫=‬

‫= )‪(P ⋅ Q) (X‬‬ ‫‪c X‬‬ ‫= ‪ c‬ﻣﻊ ‪ 0 ≤ k ≤ 2n‬؛‬ ‫‪a ⋅b‬‬ ‫‪−‬‬ ‫ﺣﻴﺚ‬
‫=‬ ‫=‬

‫و ﻟﺪﻳﻨﺎ ‪ d∘ (P ⋅ Q) = d∘ P + d∘ Q ، d∘ (P + Q) ≤ max d∘ P, d∘ Q :‬و ‪ d∘ (𝜆P) = d∘ P‬ﺣﻴﺚ ‪. 𝜆 ≠ 0‬‬


‫‪ņőƱŀ Ţ‬‬
‫‪ Q = 0‬أو ‪(PQ = 0) ⟹ P = 0‬‬ ‫ﻣﻼﺣﻈﺔ ‪ ⧏ : 61‬إذا ﻛﺎن ]‪ّ P, Q ∈ 𝕂[X‬‬
‫ﻓﺈن ‪:‬‬
‫⧐‬
‫‪ u‬اﻵن‪ ،‬ﻳﻤﻜﻦ إﺛﺒﺎت ﺗﻜﺎﻓﺆ اﻟﺘﻌﺮﻳﻔﲔ اﻟﺴﺎﺑﻘﲔ ‪ :‬ﰲ ]‪ ، 𝕂[X‬ﻧﻀﻊ )⋯ ‪ X = (0, 1, 0, 0,‬ﻛﺜﲑ اﳊﺪود اﳌﻌﺮف ﺑﺎﳌﺘﺘﺎﻟﻴﺔ‬
‫‪ a = 1 ، a = 0‬و ‪ a = 0‬ﻷﺟﻞ ‪. n ≥ 2‬‬
‫ﻧﻀﻊ‪ ،‬إﺻﻄﻼﺣ ًﺎ ‪ .X = 1 = (1, 0, 0, … ) ،‬ﻟﺪﻳﻨﺎ )⋯ ‪ X = X = (0, 1, 0, 0,‬و ﺑﺎﺳﺘﻌﲈل ﻋﻤﻠﻴﺔ اﻟﴬب اﳌﻌﺮﻓﺔ أﻋﻼه‪،‬‬
‫ﻧﺜﺒﺖ ﺑﺴﻬﻮﻟﺔ )ﺑﺎﻟﱰاﺟﻊ( ّ‬
‫أن ‪:‬‬
‫)⋯ ‪. X = (0, 0, 1, 0, 0, 0,‬‬
‫‪ƕ‬‬

‫•‬

‫)⋯ ‪. X = (0, 0, 0, 1, 0, 0,‬‬ ‫•‬

‫⋮‬ ‫•‬
‫‪ũŏ‬‬

‫)⋯ ‪ X = (0, 0, ⋯ , 1, 0, 0,‬ﺣﻴﺚ رﺗﺒﺔ اﳌﻌﺎﻣﻞ ‪ 1‬ﻫﻲ ‪ k + 1‬و ﻛﻞ اﳌﻌﺎﻣﻼت اﻷﺧﺮى ﻣﻌﺪوﻣﺔ‪.‬‬ ‫•‬

‫ﻣﻦ ﻫﺬا اﳌﻨﻄﻠﻖ‪ ،‬ﻟﺪﻳﻨﺎ ‪:‬‬


‫)⋯ ‪(a , a , a , ⋯ , a , 0, 0, ⋯) = a (1, 0, ⋯) + a (0, 1, 0, ⋯) + ⋯ + a (0, 0, ⋯ , 0, 1, 0,‬‬
‫⋯‪=a ⋅1+a ⋅X+a ⋅X +⋯+a ⋅X +0+0+‬‬
‫‪=a +a X+a X +⋯+a X‬‬

‫𝟰𝟳𝟯‬

‫‪http ://tinyurl.com/Malki1718‬‬ ‫‪0‬‬


‫ا‪ .‬ﻛﺜﲑات اﳊﺪود‬
‫‪.‬‬
‫أن اﻟﺘﻌﺮﻳﻔﲔ اﻟﺴﺎﺑﻘﲔ ﻟﻜﺜﲑات اﳊﺪود ﻣﺘﻜﺎﻓﺌﺎن‪.‬‬‫و ﻫﺬا ُﻳﺜﺒﺖ ّ‬
‫‪ X u‬ﻟﻴﺲ ﻋﺪد ًا !‬

‫‪Ŕž Ŧ Ľ‬‬
‫ا‬

‫ﻣﺒﺮﻫﻨﺔ ‪: 30‬‬

‫‪œǃ‬‬
‫= )‪ P (X‬و = )‪Q (X‬‬ ‫‪a X‬‬ ‫ﻳﺘﺴﺎوى ﻛﺜﲑا ﺣﺪود إذا ﺗﺴﺎوت ﻣﻌﺎﻣﻼﲥﲈ؛ ﺑﺘﻌﺒﲑ آﺧﺮ ‪ ،‬إذا ﻛﺎن‬
‫=‬

‫ﻛﺜﲑ ْي ﺣﺪود ﺑﻤﻌﺎﻣﻼت ﰲ 𝕂 ّ‬


‫ﻓﺈن ‪ P = Q‬إذا و ﻓﻘﻂ إذا ﻛﺎن ‪ m = n‬و ‪ a = b‬ﻷﺟﻞ ‪. 0 ≤ k ≤ n‬‬ ‫َ‬ ‫‪b X‬‬
‫=‬

‫‪Ŀ‬‬
‫ﺍﻟﻘﺴﻤﺔ ﰲ ]‪𝕂[X‬‬ ‫ﺍ‪2.‬‬

‫ﻋﻼﻗﺔ ﺍﻟﻘﺴﻤﺔ‬ ‫ﺍ‪1.2.‬‬

‫أن ‪ A‬ﻗﺎﺳﻢ ﻟـِ ‪ ، B‬أو ّ‬


‫أن ‪ B‬ﻣﻀﺎﻋﻒ ﻟـِ ‪ A‬إذا ُوﺟﺪ ]‪P ∈ 𝕂[X‬‬ ‫ﺗﻌﺮﻳﻒ ‪ : 18‬ﻟﻴﻜﻦ ]‪ . A, B ∈ 𝕂[X‬ﻧﻘﻮل ّ‬
‫أن ‪ A‬ﻳﻘﺴﻢ ‪ ، B‬أو ّ‬
‫ﺑﺤﻴﺚ ‪ B = AP‬و ﻧﻜﺘﺐ ‪. A ∣ B :‬‬

‫‪ u‬ﻫﻨﺎك ﺗﺸﺎﺑﻪ ﻛﺒﲑ ﺑﲔ دراﺳﺔ ﺧﻮاص اﻷﻋﺪاد اﻟﺼﺤﻴﺤﺔ )اﳊﺴﺎب( و ﻛﺜﲑات اﳊﺪود‪ ،‬و أﻏﻠﺐ اﻟﻨﺘﺎﺋﺞ اﻟﺘﻲ ﲣﺺ ﻛﺜﲑات‬
‫اﳊﺪود ﻳﻤﻜﻦ إﺛﺒﺎﲥﺎ ﺑﻨﻔﺲ اﻟﻄﺮﻳﻘﺔ اﳌ ﱠﺘ َﺒﻌﺔ ﰲ إﺛﺒﺎت اﻟﻨﺘﻴﺠﺔ اﳌﻜﺎﻓﺌﺔ ﰲ ﻋﻠﻢ اﳊﺴﺎب‪.‬‬

‫ﻟﻴﻜﻦ ]‪ . A, B, C, D ∈ 𝕂[X‬ﻟﺪﻳﻨﺎ ‪:‬‬ ‫ﻣﺒﺮﻫﻨﺔ ‪: 31‬‬

‫‪ •1‬ﻋﻼﻗﺔ اﻟﻘﺴﻤﺔ )∣( إﻧﻌﻜﺎﺳﻴﺔ )‪ (∀P , P ∣ P‬؛ ﻣﺘﻌﺪﻳﺔ )أي إذا ﻛﺎن ‪ A ∣ B‬و ‪ّ B ∣ C‬‬
‫ﻓﺈن ‪ . (A ∣ C‬ﺑﺎﻹﺿﺎﻓﺔ إﱃ ذﻟﻚ ‪:‬‬
‫∗‬
‫‪A∣B‬‬ ‫و‬ ‫‪B ∣ A ⟺ ∃𝜆 ∈ 𝕂 : A = 𝜆B‬‬
‫‪ņőƱŀ Ţ‬‬
‫ﻓﺈن )‪ّ D ∣ (𝜆A + 𝜇B‬‬
‫ﻟﻜﻞ 𝕂 ∈ 𝜇 ‪. 𝜆,‬‬ ‫‪ •2‬إذا ﻛﺎن ‪ D ∣ A‬و ‪ّ D ∣ B‬‬
‫ﻓﺈن ‪ّ A ∣ B‬‬
‫ﻟﻜﻞ ‪. k ∈ ℕ‬‬ ‫ﻓﺈن ‪ . AC ∣ BD‬و ﻛﺤﺎﻟﺔ ﺧﺎﺻﺔ‪ ،‬إذا ﻛﺎن ‪ّ A ∣ B‬‬
‫‪ •3‬إذا ﻛﺎن ‪ A ∣ B‬و ‪ّ C ∣ D‬‬
‫‪. A ∣ B ⟺ AD ∣ BD‬‬ ‫‪ •4‬إذا ﻛﺎن ‪ّ D ≠ 0‬‬
‫ﻓﺈن ‪:‬‬

‫ﺍﻟﻘﺴﻤﺔ ﺍﻹﻗﻠﻴﺪﻳﺔ‬ ‫ﺍ‪2.2.‬‬


‫‪ƕ‬‬

‫ﻛﲈ ﻫﻮ اﳊﺎل ﺑﺎﻟﻨﺴﺒﺔ ﻟﻸﻋﺪاد‪ ،‬ﻳﻤﻜﻦ ﺗﻌﺮﻳﻒ اﻟﻘﺴﻤﺔ اﻹﻗﻠﻴﺪﻳﺔ ﰲ ]‪ 𝕂[X‬ﻛﲈ ﻳﲇ ‪:‬‬
‫‪ũŏ‬‬

‫ﻛﺜﲑا ﺣﺪود ]‪ Q, R ∈ 𝕂[X‬وﺣﻴﺪان ﺑﺤﻴﺚ‬


‫ﻛﺜﲑ ْي ﺣﺪود ﺣﻴﺚ ‪ B ≠ 0‬ﻓﺈﻧﻪ ﻳﻮﺟﺪ َ‬‫ﻣﺒﺮﻫﻨﺔ ‪ : 32‬إذا ﻛﺎن ]‪َ A, B ∈ 𝕂[X‬‬
‫ﻳﻜﻮن ‪ A = BQ + R :‬ﻣﻊ ‪ R = 0‬أو ‪. d R < d∘ B‬‬
‫∘‬

‫ﰲ ﻫﺬه اﳊﺎﻟﺔ‪ُ A ،‬ﻳﺴﻤﻰ اﳌﻘﺴﻮم‪ B ،‬اﻟﻘﺎﺳﻢ‪ Q ،‬ﺣﺎﺻﻞ اﻟﻘﺴﻤﺔ و ‪ R‬ﺑﺎﻗﻲ اﻟﻘﺴﻤﺔ‪.‬‬

‫ﻟﻨﺠﺪ ﺣﺎﺻﻞ و ﺑﺎﻗﻲ اﻟﻘﺴﻤﺔ اﻹﻗﻠﻴﺪﻳﺔ ﻟـِ ‪ A (X) = 3X − 12X + X − 1‬ﻋﲆ ‪. B (X) = X + 1‬‬ ‫ﻣﺜﺎﻝ ‪: 27‬‬

‫𝟱𝟳𝟯‬

‫‪http ://tinyurl.com/Malki1718‬‬ ‫‪0‬‬


‫ا‪ .3.‬اﻟﻘﻮاﰟ و اﳌﻀﺎﻋﻔﺎت اﳌﺸﱰﻛﺔ‬
‫‪.‬‬
‫ﺑﺪاﻳ ًﺔ‪ ،‬ﻧﻘﺴﻢ ‪ 3X‬ﻋﲆ ‪ X‬ﻓﻴﻜﻮن اﳊﺎﺻﻞ ﻫﻮ ‪ ،3X‬ﺑﻌﺪ ذﻟﻚ ﻧﴬب ‪ 3X‬ﰲ ‪ X + 1‬ﻓﻴﻜﻮن اﻟﻨﺎﺗﺞ ‪ّ ،3X + 3X‬‬
‫ﺛﻢ ﻧﻄﺮح‬
‫‪. 3X − 12X + X − 1 − 3X + 3X‬‬ ‫ﻫﺬا اﻟﻨﺎﺗﺞ ﻣﻦ )‪ A (X‬ﻓﻨﺠﺪ )‪= −3X − 12X + X − 1 = A (X‬‬

‫‪Ŕž Ŧ Ľ‬‬
‫‪−3X X + 1 = −3X − 3X‬‬ ‫‪،‬‬ ‫‪−3X ÷ X = −3X‬‬ ‫ُﻜﺮر ﻧﻔﺲ اﻟﻌﻤﻠﻴﺎت ﻣﻊ )‪ A (X‬و )‪: B (X‬‬
‫ﻧ ﱢ‬

‫ا‬
‫‪œǃ‬‬
‫و ‪. −3X − 12X + X − 1 − −3X − 3X = −12X + 4X − 1‬‬
‫‪−12 X + 1 = −12X − 12‬‬ ‫‪،‬‬ ‫‪−12X ÷ X = −12‬‬ ‫ﻣﺮة أﺧﺮى ‪:‬‬
‫ُﻜﺮر اﻟﻌﻤﻠﻴﺎت ّ‬
‫ﻧ ﱢ‬
‫و ‪. −12X + 4X − 1 − −12X − 12 = 4X + 11‬‬

‫‪Ŀ‬‬
‫أن ‪ّ ، d∘ (4X + 11) < d∘ X + 1‬‬
‫ﻓﺈن ﻋﻤﻠﻴﺔ اﻟﻘﺴﻤﺔ اﻹﻗﻠﻴﺪﻳﺔ ﺗﺘﻮ ّﻗﻒ ﻫﻨﺎ ‪.‬‬ ‫ﺑﲈ ّ‬
‫ﰲ اﻷﺧﲑ‪ ،‬ﺣﺎﺻﻞ ﻗﺴﻤﺔ )‪ A (X‬ﻋﲆ )‪ B (X‬ﻫﻮ ‪ Q (X) = 3X − 3X − 12‬و اﻟﺒﺎﻗﻲ ﻫﻮ ‪ R (X) = 4X + 11‬و ﻧﻜﺘﺐ ‪:‬‬
‫)‪3X − 12X + X − 1 = 3X − 3X − 12 ⋅ X + 1 + (4X + 11‬‬

‫ﻧ ﱢ‬
‫ُﻠﺨﺺ اﻟﻌﻤﻠﻴﺎت اﻟﺴﺎﺑﻘﺔ ﻛﲈ ﻳﲇ ‪:‬‬

‫‪3X‬‬ ‫‪−12X +X −1 X‬‬ ‫‪+1‬‬

‫‪3X +3X‬‬ ‫‪3X −3X −12‬‬

‫‪−3X −12X +X −1‬‬

‫‪−3X‬‬ ‫‪−3X‬‬

‫‪−12X +4X −1‬‬

‫‪−12X‬‬ ‫‪−12‬‬

‫‪4X‬‬ ‫‪+11‬‬
‫‪ņőƱŀ Ţ‬‬
‫ﺍﻟﻘﻮﺍﺳﻢ ﻭ ﺍﳌﻀﺎﻋﻔﺎﺕ ﺍﳌﺸﺘﺮﻛﺔ‬ ‫ﺍ‪3.‬‬

‫ﻛﺜﲑ ْي ﺣﺪود‪.‬‬
‫ﻟﻴﻜﻦ ]‪َ A, B ∈ 𝕂[X‬‬ ‫ﺗﻌﺮﻳﻒ ‪: 19‬‬

‫ﻧُﺴﻤﻲ ﻗﺎﺳ ًﲈ ﻣﺸﱰﻛ ًﺎ ‪ 1‬ﻟـِ ‪ A‬و ‪ B‬ﻛﻞ ﻛﺜﲑ ﺣﺪود ]‪ D ∈ 𝕂[X‬ﻳﻘﺴﻢ ‪ A‬و ‪ B‬ﰲ آن واﺣﺪ‪.‬‬ ‫•‬

‫ﻧُﺴﻤﻲ ﻣﻀﺎﻋﻔ ًﺎ ﻣﺸﱰﻛ ًﺎ ‪ 2‬ﻟـِ ‪ A‬و ‪ B‬ﻛﻞ ﻛﺜﲑ ﺣﺪود ]‪ M ∈ 𝕂[X‬ﻣﻀﺎﻋﻒ ﻟـِ ‪ A‬و ‪ B‬ﰲ آن واﺣﺪ‪.‬‬ ‫•‬
‫‪ƕ‬‬

‫ﺍﻟﻘﺎﺳﻢ ﺍﳌﺸﺘﺮﻙ ﺍﻷﻛﱪ‬ ‫ﺍ‪1.3.‬‬


‫‪ũŏ‬‬

‫ﻛﺜﲑ ْي ﺣﺪود‪ .‬اﻟﻘﺎﺳﻢ اﳌﺸﱰك اﻷﻛﱪ ﻟـِ ‪ A‬و ‪ B‬ﻫﻮ ﻛﻞ ﻛﺜﲑ ﺣﺪود ]‪ُ D ∈ 𝕂[X‬ﳛﻘﻖ‬
‫ﻟﻴﻜﻦ ]‪َ A, B ∈ 𝕂[X‬‬ ‫ﺗﻌﺮﻳﻒ ‪: 20‬‬
‫‪:‬‬
‫‪ D‬ﻗﺎﺳﻢ ﻣﺸﱰك ﻟـِ ‪ A‬و ‪ B‬أي ‪ D ∣ A :‬و ‪ D ∣ B‬؛‬ ‫•‬

‫𝟲𝟳𝟯‬

‫‪http ://tinyurl.com/Malki1718‬‬ ‫‪0‬‬


‫ا‪ .‬ﻛﺜﲑات اﳊﺪود‬
‫‪.‬‬

‫‪ D‬ﻣﻀﺎﻋﻒ ّ‬
‫ﻟﻜﻞ ﻗﺎﺳﻢ ﻣﺸﱰك ﻟـِ ‪ A‬و ‪ B‬أي ‪:‬‬ ‫•‬

‫‪Ŕž Ŧ Ľ‬‬
‫‪∀P ∈ 𝕂[X] ,‬‬ ‫‪(P ∣ A‬‬ ‫‪ P ∣ B) ⟹ P ∣ D‬و‬
‫ا‬

‫ﻧﻜﺘﺐ ‪ 3 D = pgcd (A, B) :‬أو )‪. 4 D = gcd (A, B‬‬

‫‪œǃ‬‬
‫ي ﺣﺪود ﻟﻴﺲ وﺣﻴﺪ ًا‪ .‬ﻟﻜﻦ إذا ﻛﺎن ‪ A ≠ 0‬و ‪ B ≠ 0‬ﻓﺈﻧﻪ ﻳﻮﺟﺪ ‪pgcd‬‬
‫ﻟﻜﺜﲑ ْ‬
‫َ‬ ‫اﻟﻘﺎﺳﻢ اﳌﺸﱰك اﻷﻛﱪ‬ ‫ﻣﻼﺣﻈﺔ ‪⧏ : 62‬‬
‫وﺣﻴﺪ ﺑﺤﻴﺚ ﻣﻌﺎﻣﻠﻪ اﳌﻬﻴﻤﻦ ﻳﺴﺎوي ‪) 1‬أي واﺣﺪي( ‪.‬‬

‫‪Ŀ‬‬
‫ﻓﻴﲈ ﻳﲇ‪ ،‬ﻋﻨﺪﻣﺎ ﻧﻜﺘﺐ ‪ pgcd‬دون أي ﲢﺪﻳﺪ ﻓﺈﻧﻨﺎ ﻧﻘﺼﺪ ﻫﺬا ﺍﻟﻘﺎﺳﻢ ﺍﳌﺸﱰﻙ ﺍﻷﻛﱪ ﺍﻟﻮﺍﺣﺪﻱ‬

‫ﻛﻞ ﻛﺜﲑ ﺣﺪود ﻣﻦ اﻟﺸﻜﻞ )‪ ، 𝜆 pgcd (A, B‬ﺣﻴﺚ ∗𝕂 ∈ 𝜆 ‪ ،‬ﻫﻮ أﻳﻀ ًﺎ ﻗﺎﺳﻢ ﻣﺸﱰك أﻛﱪ ﻟـِ ‪ A‬و ‪. B‬‬
‫‪. pgcd (0, 0) = 0‬‬ ‫أﺧﲑ ًا‪ ،‬ﻧﻀﻊ ‪:‬‬
‫⧐‬
‫‪2‬‬
‫ﻛﻞ ﻣﻦ ‪ 2X ، X‬و ‪ − √ X‬ﻗﺎﺳﻢ ﻣﺸﱰك أﻛﱪ ﻟـِ ‪ X‬و ‪. X‬‬ ‫ﻣﺜﺎﻝ ‪: 28‬‬
‫‪3‬‬

‫ﻣﺒﺮﻫﻨﺔ ‪) : 33‬ﻧﻈﺮﻳﺔ ﺑﻴﺰﻭﺕ –‪ –B‬ﺍﻟﺠﺰء ﺍﻷﻭﻝ(‬


‫ﻛﺜﲑ ْي ﺣﺪود‪ .‬ﻳﻮﺟﺪ ﻛﺜﲑا ﺣﺪود ]‪ U, V ∈ 𝕂[X‬ﺑﺤﻴﺚ ‪. pgcd (A, B) = AU + BV :‬‬
‫ﻟﻴﻜﻦ ]‪َ A, B ∈ 𝕂[X‬‬

‫‪ U u‬و ‪ V‬ﻟﻴﺴﺎ وﺣﻴﺪ ْﻳﻦ !‬

‫إذا ﻛﺎن ]‪ A, B ∈ 𝕂[X‬ﻣﻊ ‪ B ≠ 0‬و ﻛﺎن ‪ R‬ﺑﺎﻗﻲ اﻟﻘﺴﻤﺔ اﻹﻗﻠﻴﺪﻳﺔ ﻟـِ ‪ A‬ﻋﲆ ‪ّ B‬‬
‫ﻓﺈن ‪:‬‬ ‫ﻣﺒﺮﻫﻨﺔ ‪: 34‬‬
‫)‪pgcd (A, B) = pgcd (B, R‬‬

‫ﺍﻟﻄﺮﻳﻘﺔ ﺍﻟﻌﻤﻠﻴﺔ ﻹﳚﺎﺩ ﺍﻟﻘﺎﺳﻢ ﺍﳌﺸﺘﺮﻙ ﺍﻷﻛﱪ ‪ :‬ﺧﻮﺍﺭﺯﻣﻴﺔ ﺇﻗﻠﻴﺪﺱ‬


‫‪ņőƱŀ Ţ‬‬
‫ﻟﻴﻜﻦ ]‪ . A, B ∈ 𝕂[X‬ﺳﺘﻤﻜﻨﻨﺎ ﺧﻮارزﻣﻴﺔ إﻗﻠﻴﺪس ‪ 5‬ﻣﻦ إﳚﺎد )‪ pgcd (A, B‬و ﺑﻄﺮﻳﻘﺔ ﴎﻳﻌﺔ‪.‬‬
‫أن ‪ d∘ B ≤ d∘ A‬و ّ‬
‫أن‬ ‫أن )‪ pgcd (A, B) = pgcd (B, A‬ﻓﺒﺈﻣﻜﺎﻧﻨﺎ أن ﻧﻔﺮض ّ‬
‫أن ‪ ، pgcd (0, 0) = 0‬و ﺑﲈ ّ‬
‫ﺑﺪاﻳ ًﺔ‪ ،‬ﻧُﺬﻛﱢﺮ ّ‬
‫‪.A ≠ 0‬‬
‫ُﻌﺮف ﻛﺜﲑات اﳊﺪود ‪ ⋯ ، R ، R ، R‬ﻛﲈ ﻳﲇ ‪:‬‬ ‫ﻧ ﱢ‬
‫‪ •1‬ﻧﻀﻊ ‪ R = A‬و ‪ R = B‬؛‬

‫‪ ، R‬و ﰲ ﻫﺬه‬ ‫‪ R‬ﻳﺴﺎوي ﺑﺎﻗﻲ اﻟﻘﺴﻤﺔ اﻹﻗﻠﻴﺪﻳﺔ ﻟـِ ‪ R‬ﻋﲆ‬ ‫‪ّ R‬‬
‫ﻓﺈن‬ ‫إذا ﻛﺎن ‪≠ 0‬‬ ‫‪ •2‬ﻟﻴﻜﻦ ‪ k‬ﻋﺪد ًا ﻃﺒﻴﻌﻴﺎ‪.‬‬
‫‪ƕ‬‬

‫‪+‬‬ ‫‪+‬‬ ‫‪+‬‬

‫‪. d∘ R‬‬ ‫‪+‬‬


‫∘‬
‫‪<dR‬‬ ‫‪+‬‬ ‫اﳊﺎﻟﺔ ﻟﺪﻳﻨﺎ ‪:‬‬
‫إذن ⋯ > ‪ . d∘ R ≥ d∘ R > d∘ R > d∘ R‬و ﺑﲈ أﻧﻪ ﻻ ﻳﻮﺟﺪ إﻻّ ﻋﺪد ٍ‬
‫ﻣﻨﺘﻪ ﻣﻦ اﻷﻋﺪاد اﻟﻄﺒﻴﻌﻴﺔ ﺑﲔ ‪ 0‬و ‪ d∘ R‬ﻓﺈﻧﻪ ﻳﻮﺟﺪ‬
‫ﻓﺈن ‪ R −‬ﻫﻮ آﺧﺮ ﺑﺎﻗﻲ ﻏﲑ ﻣﻌﺪوم ﰲ اﳌﺘﺘﺎﻟﻴﺔ ‪ ⋯ ، R ، R ، R‬؛ و ﺣﺴﺐ‬ ‫ﻋﺪد ∗‪ N ∈ ℕ‬ﺑﺤﻴﺚ ‪ . R = 0 :‬و ﺑﺎﻟﺘﺎﱄ ّ‬
‫‪ũŏ‬‬

‫اﳌﱪﻫﻨﺔ ‪ّ 34‬‬
‫ﻓﺈن ‪:‬‬
‫‪pgcd (R , R ) = pgcd (R , R ) = ⋯ = pgcd (R‬‬ ‫‪−‬‬ ‫‪, R ) = pgcd (R‬‬ ‫‪−‬‬ ‫‪, 0) = R‬‬ ‫‪−‬‬

‫ﺣﻴﺚ ‪ R −‬ﻫﻮ اﻟﺒﺎﻗﻲ ‪ R −‬و اﻟﺬي ﻗﺴﻤﻨﺎه ﻋﲆ ﻣﻌﺎﻣﻠﻪ اﳌﻬﻴﻤﻦ )ﻟﻴﺼﺒﺢ واﺣﺪﻳ ًﺎ(‪.‬‬
‫إذن )‪ pgcd (A, B‬ﻫﻮ آﺧﺮ ﺑﺎﻗﻲ ﻏﲑ ﻣﻌﺪوم ‪) R −‬ﺑﻌﺪ إرﺟﺎﻋﻪ واﺣﺪﻳ ًﺎ( ﰲ اﳌﺘﺘﺎﻟﻴﺔ ‪⋯ ، R ، R ، R‬‬

‫‪ E’s Algorithm5‬ﺑﺎﻹﻧﺠﻠﻴﺰﻳﺔ ؛ ‪ Algorithme d’E‬ﺑﺎﻟﻔﺮﻧﺴﻴﺔ ‪.‬‬

‫𝟳𝟳𝟯‬

‫‪http ://tinyurl.com/Malki1718‬‬ ‫‪0‬‬


‫ا‪ .3.‬اﻟﻘﻮاﰟ و اﳌﻀﺎﻋﻔﺎت اﳌﺸﱰﻛﺔ‬
‫‪.‬‬
‫ﻟﻨﺒﺤﺚ ﻋﻦ اﻟﻘﺎﺳﻢ اﳌﺸﱰك اﻷﻛﱪ ﻟـِ ‪ A = X + 7X + 17X + 17X + 6‬و ‪. B = X + X − 4X − 4‬‬ ‫ﻣﺜﺎﻝ ‪: 29‬‬

‫أن ‪ّ d∘ B ≤ d∘ A‬‬
‫ﻓﺈن ‪ R = A‬و ‪ R = B‬؛ و ﺑﺎﻟﻘﺴﻤﺔ اﻹﻗﻠﻴﺪﻳﺔ ‪:‬‬ ‫ﺑﲈ ّ‬

‫‪Ŕž Ŧ Ľ‬‬
‫‪ A = (X + 6) B + 15X + 45X + 30‬أي ‪ R = 15X + 45X + 30 = 15 X + 3X + 2‬و ﻟﺘﺒﺴﻴﻂ اﳊﺴﺎب‪،‬‬

‫ا‬
‫‪œǃ‬‬
‫ﻧﻮاﺻﻞ اﻟﻌﻤﻠﻴﺔ ﻣﻊ ‪ . R = X + 3X + 2‬ﻟﺪﻳﻨﺎ ‪ B = (X − 2) X + 3X + 2 + 0 :‬أي ‪ . R = 0‬إذن آﺧﺮ ﺑﺎﻗﻲ ﻏﲑ‬
‫ﻣﻌﺪوم ﻫﻮ ‪ R‬و ﺑﺎﻟﻘﺴﻤﺔ ﻋﲆ ﻣﻌﺎﻣﻠﻪ اﳌﻬﻴﻤﻦ ‪ ، a = 15‬ﻧﺄﺧﺬ ‪ R = X + 3X + 2‬أي ‪:‬‬
‫‪pgcd (A, B) = R = X + 3X + 2‬‬

‫‪Ŀ‬‬
‫و ﻹﳚﺎد ‪ U‬و ‪) V‬ﻣﻌﺎﻣﻼت ﺑﻴﺰﻭﺕ( ‪:‬‬
‫‪15 X + 3X + 2 = A − (X + 6) B‬‬
‫‪1‬‬ ‫‪1‬‬
‫= ‪pgcd (A, B) = X + 3X + 2‬‬ ‫‪A+‬‬ ‫‪(−X − 6) B‬‬ ‫ﻣﻨﻪ ‪:‬‬
‫‪15‬‬ ‫‪15‬‬
‫‪1‬‬ ‫‪1‬‬
‫‪.V = −‬‬ ‫= ‪ U‬و )‪(X + 6‬‬ ‫أي‬
‫‪15‬‬ ‫‪15‬‬

‫ﻣﺒﺮﻫﻨﺔ ‪ : 35‬إذا ﻛﺎن ]‪ A, B ∈ 𝕂[X‬و ﻛﺎن ]‪ P ∈ 𝕂[X‬واﺣﺪﻳ ًﺎ ّ‬


‫ﻓﺈن ‪:‬‬
‫)‪pgcd (AP, BP) = P × pgcd (A, B‬‬

‫‪ u‬ﰲ اﳊﺴﺎب )ﻧﻈﺮﻳﺔ اﻷﻋﺪاد( ‪ ،‬ﻟﺪﻳﻨﺎ )‪ pgcd (ka, kb) = ||k|| ⋅ pgcd (a, b‬و إذا ﻛﺎن ‪ّ k ≥ 0‬‬
‫ﻓﺈن ‪:‬‬
‫)‪ . pgcd (ka, kb) = k ⋅ pgcd (a, b‬ﺑﺎﻟﻨﺴﺒﺔ ﻟﻜﺜﲑات اﳊﺪود‪ ،‬اﻟﴩط »‪ P‬واﺣﺪي«ﻳﻘﻮم ﻣﻘﺎم اﻟﴩط »‪.«k ≥ 0‬‬

‫ﻛﺜﲑﺍﺕ ﺍﳊﺪﻭﺩ ﺍﻷﻭﻟﻴﺔ ﻓﻴﻤﺎ ﺑﻴﻨﻬﺎ‬ ‫ﺍ‪2.3.‬‬

‫ﺗﻌﺮﻳﻒ ‪ : 21‬ﻟﻴﻜﻦ ]‪ . A, B ∈ 𝕂[X‬ﻧﻘﻮل ّ‬


‫أن ‪ A‬و ‪ّ B‬أوﻟﻴﺎن ﻓﻴﲈ ﺑﻴﻨﻬﲈ إذا ﻛﺎﻧﺖ اﻟﻘﻮاﺳﻢ اﳌﺸﱰﻛﺔ ﳍﲈ ﻫﻲ ﻛﺜﲑات اﳊﺪود‬
‫اﻟﺜﺎﺑﺘﺔ ﻓﻘﻂ‪ ،‬أو ﺑﺘﻌﺒﲑ آﺧﺮ‪ ،‬إذا ﻛﺎن اﻟﻘﺎﺳﻢ اﳌﺸﱰك اﻷﻛﱪ )اﻟﻮاﺣﺪي( ﳍﲈ ﻫﻮ ‪. 1‬‬
‫‪ņőƱŀ Ţ‬‬
‫ﻣﺒﺮﻫﻨﺔ ‪) : 36‬ﻧﻈﺮﻳﺔ ﺑﻴﺰﻭﺕ –‪ –B‬ﺍﻟﺠﺰء ﺍﻟﺜﺎﻧﻲ(‬
‫ﻟﻴﻜﻦ ]‪ . A, B ∈ 𝕂[X‬اﳋﺎﺻﻴﺘﺎن اﻟﺘﺎﻟﻴﺘﺎن ﻣﺘﻜﺎﻓﺌﺘﺎن ‪:‬‬
‫‪ A •1‬و ‪ّ B‬أوﻟﻴﺎن ﻓﻴﲈ ﺑﻴﻨﻬﲈ‪.‬‬
‫‪ •2‬ﻳﻮﺟﺪ َ‬
‫ﻛﺜﲑا ﺣﺪود ]‪ U, V ∈ 𝕂[X‬ﺑﺤﻴﺚ ‪. AU + BV = 1 :‬‬
‫‪ƕ‬‬

‫ﻣﺒﺮﻫﻨﺔ ‪) : 37‬ﻧﻈﺮﻳﺔ ﭬﻮﺹ ‪(–G‬‬


‫ﻟﻴﻜﻦ ]‪ . A, B, C ∈ 𝕂[X‬إذا ﻛﺎن ‪ A ∣ BC‬و ﻛﺎن ‪ A‬و ‪ّ B‬أوﻟﻴﲔ ﻓﻴﲈ ﺑﻴﻨﻬﲈ ّ‬
‫ﻓﺈن ‪. A ∣ C‬‬
‫‪ũŏ‬‬

‫ﻟﻴﻜﻦ ]‪ . A, B, P ∈ 𝕂[X‬إذا ﻛﺎن ‪ A ∣ P‬و ‪ B ∣ P‬؛ و ﻛﺎن ‪ A‬و ‪ّ B‬أوﻟﻴﲔ ﻓﻴﲈ ﺑﻴﻨﻬﲈ ّ‬
‫ﻓﺈن ‪. AB ∣ P‬‬ ‫ﻻﺯﻣﺔ ‪: 2‬‬

‫ﻣﻬﻢ ‪ .‬ﺑﺪوﻧﻪ‪ ،‬ﺗﻜﻮن ﻫﺬه اﻟﻼزﻣﺔ ﺧﺎﻃﺌﺔ ﻛﲈ ﻳﺒﻴﻨﻪ اﳌﺜﺎل ‪ :‬إذا ﻛﺎن ‪A = B = P = X‬‬ ‫‪ u‬اﻟﴩط »‪ A‬و ‪ّ B‬أوﻟﻴﺎن ﻓﻴﲈ ﺑﻴﻨﻬﲈ «ﺟﺪّ‬
‫ﻓﺈن ‪ A ∣ P‬و ‪ B ∣ P‬ﻟﻜﻦ ‪! AB ∤ P‬‬‫ّ‬

‫𝟴𝟳𝟯‬

‫‪http ://tinyurl.com/Malki1718‬‬ ‫‪0‬‬


‫ا‪ .‬ﻛﺜﲑات اﳊﺪود‬
‫‪.‬‬
‫ﺍﳌﻀﺎﻋﻒ ﺍﳌﺸﺘﺮﻙ ﺍﻷﺻﻐﺮ‬ ‫ﺍ‪3.3.‬‬

‫‪Ŕž Ŧ Ľ‬‬
‫ﺗﻌﺮﻳﻒ ‪ : 22‬ﻟﻴﻜﻦ ]‪َ A, B ∈ 𝕂[X‬‬
‫ﻛﺜﲑ ْي ﺣﺪود‪.‬‬
‫ا‬

‫اﳌﻀﺎﻋﻒ اﳌﺸﱰك اﻷﻛﱪ ﻟِـ ‪ A‬و ‪ B‬ﻫﻮ ﻛﻞ ﻛﺜﲑ ﺣﺪود ]‪ُ M ∈ 𝕂[X‬ﳛﻘﻖ ‪:‬‬

‫‪œǃ‬‬
‫• ‪ M‬ﻣﻀﺎﻋﻒ ﻣﺸﱰك ﻟِـ ‪ A‬و ‪ B‬أي ‪ A ∣ M :‬و ‪ B ∣ M‬؛‬
‫ﻟﻜﻞ ﻣﻀﺎﻋﻒ ﻣﺸﱰك ﻟِـ ‪ A‬و ‪ B‬أي ‪:‬‬
‫‪ M‬ﻗﺎﺳﻢ ّ‬ ‫•‬

‫‪Ŀ‬‬
‫‪∀N ∈ 𝕂[X] ,‬‬ ‫‪(A ∣ N‬‬ ‫‪ B ∣ N) ⟹ M ∣ N‬و‬
‫ﻧﻜﺘﺐ ‪ 6 M = ppcm (A, B) :‬أو )‪. 7 M = lcm (A, B‬‬

‫ﻟﻜﺜﲑ ْي ﺣﺪود ﻟﻴﺲ وﺣﻴﺪ ًا‪ .‬ﻟﻜﻦ إذا ﻛﺎن ‪ A ≠ 0‬و ‪ B ≠ 0‬ﻓﺈﻧﻪ ﻳﻮﺟﺪ‬
‫َ‬ ‫ﻣﻼﺣﻈﺔ ‪ ⧏ : 63‬اﳌﻀﺎﻋﻒ اﳌﺸﱰك اﻷﺻﻐﺮ‬
‫‪ ppcm‬وﺣﻴﺪ ﺑﺤﻴﺚ ﻣﻌﺎﻣﻠﻪ اﳌﻬﻴﻤﻦ ﻳﺴﺎوي ‪) 1‬أي واﺣﺪي( ‪.‬‬

‫ﻓﻴﲈ ﻳﲇ‪ ،‬ﻋﻨﺪﻣﺎ ﻧﻜﺘﺐ ‪ ppcm‬دون أي ﲢﺪﻳﺪ ﻓﺈﻧﻨﺎ ﻧﻘﺼﺪ ﻫﺬا ﺍﳌﻀﺎﻋﻒ ﺍﳌﺸﱰﻙ ﺍﻷﺻﻐﺮ ﺍﻟﻮﺍﺣﺪﻱ‬

‫ﻛﻞ ﻛﺜﲑ ﺣﺪود ﻣﻦ اﻟﺸﻜﻞ )‪ ، 𝜆 ppcm (A, B‬ﺣﻴﺚ ∗𝕂 ∈ 𝜆 ‪ ،‬ﻫﻮ أﻳﻀ ًﺎ ﻣﻀﺎﻋﻒ ﻣﺸﱰك أﺻﻐﺮ ﻟـِ ‪ A‬و ‪. B‬‬
‫‪. ∃𝜆 ∈ 𝕂∗ : pgcd (A, B) × ppcm (A, B) = 𝜆AB‬‬ ‫أﺧﲑ ًا ‪:‬‬
‫⧐‬
‫‪2‬‬
‫ﻣﺜﺎﻝ ‪ : 30‬ﻛﻞ ﻣﻦ ‪ 2X ، X‬و ‪ − √ X‬ﻣﻀﺎﻋﻒ ﻣﺸﱰك أﺻﻐﺮ ﻟـِ ‪ X‬و ‪. X‬‬
‫‪3‬‬
‫ِ‬ ‫ّ‬ ‫إذا ﻛﺎن ‪ A = 0‬أو ‪ّ B = 0‬‬
‫ﻓﺈن ‪ ppcm (A, B) = 0‬ﻷن ‪ 0‬ﻫﻮ اﳌﻀﺎﻋﻒ اﻟﻮﺣﻴﺪ ﻟـ ‪. 0‬‬

‫ﻣﺒﺮﻫﻨﺔ ‪ : 38‬إذا ﻛﺎن ]‪ A, B ∈ 𝕂[X‬و ﻛﺎن ]‪ P ∈ 𝕂[X‬واﺣﺪﻳ ًﺎ ّ‬


‫ﻓﺈن ‪:‬‬
‫)‪ppcm (AP, BP) = P × ppcm (A, B‬‬
‫‪ņőƱŀ Ţ‬‬
‫ﻛﺜﲑﺍﺕ ﺍﳊﺪﻭﺩ ﻏﲑ ﺍﻟﻘﺎﺑﻠﺔ ﻟﻠﺘﺤﻠﻴﻞ‬ ‫ﺍ‪4.‬‬

‫ﺗﻌﺮﻳﻒ ‪ : 23‬ﻟﻴﻜﻦ ]‪ . P ∈ 𝕂[X‬ﻧﻘﻮل ّ‬


‫أن ‪ P‬ﻏﲑ ﻗﺎﺑﻞ ﻟﻠﺘﺤﻠﻴﻞ ﰲ ]‪ 𝕂[X‬إذا ﻛﺎن ‪ P‬ﻟﻴﺲ ﺛﺎﺑﺘ ًﺎ و إذا ﻛﺎﻧﺖ ﻗﻮاﺳﻤﻪ ﻫﻲ‬
‫ﻛﺜﲑات اﳊﺪود اﻟﺜﺎﺑﺘﺔ ∗𝕂 ∈ 𝜆 و ‪ 𝜇P‬ﺣﻴﺚ ∗𝕂 ∈ 𝜇 ‪.‬‬
‫‪ƕ‬‬

‫‪ u‬ﻛﺜﲑات اﳊﺪود اﻟﺜﺎﺑﺘﺔ ﻻ ﺗﺪﺧﻞ ﰲ ﻫﺬا اﻟﺘﻌﺮﻳﻒ !‬


‫‪ u‬ﻛﺜﲑات اﳊﺪود ﻏﲑ اﻟﻘﺎﺑﻠﺔ ﻟﻠﺘﺤﻠﻴﻞ ﻫﻲ ﻣﺎ ﻳﻜﺎﻓﺊ اﻷﻋﺪاد اﻷوﻟﻴﺔ ﰲ اﳊﺴﺎب )ﻧﻈﺮﻳﺔ اﻷﻋﺪاد( أو اﳉﺰﻳﺌﺎت ﰲ ﻋﻠﻢ‬
‫اﻟﻔﻴﺰﻳﺎء‪ ،‬و ﻫﻲ ﻣﺎ ُﻳﻤﻜﱢﻨﻨﺎ ﻣﻦ ﺗﻔﻜﻴﻚ ﻛﻞ ﻛﺜﲑ ﺣﺪود إﱃ أﺟﺰاء ﺻﻐﲑة ﻏﲑ ﻗﺎﺑﻠﺔ ﻟﻠﺘﻔﻜﻴﻚ ﺑﺪورﻫﺎ‪.‬‬
‫‪ũŏ‬‬

‫ﻣﻼﺣﻈﺔ ‪ ⧏ : 64‬ﳚﺐ ﲢﺪﻳﺪ اﳌﺠﻤﻮﻋﺔ اﻟﺘﻲ ﻧﺪرس ﻓﻴﻬﺎ ﻗﺎﺑﻠﻴﺔ اﻟﺘﺤﻠﻴﻞ إﱃ ﺟﺪاء ﻋﻮاﻣﻞ أوﻟﻴﺔ ﻓﻤﺜﻼً‪ ،‬ﻛﺜﲑ اﳊﺪود‬
‫‪ X + 1‬ﻏﲑ ﻗﺎﺑﻞ ﻟﻠﺘﺤﻠﻴﻞ ﻋﲆ ]‪ ℝ[X‬ﻟﻜﻨﻪ ﻳﻘﺒﻞ اﻟﺘﺤﻠﻴﻞ )‪ X + 1 = (X − ı) (X + ı‬ﰲ ]‪. ℂ[X‬‬
‫أﻧﻈﺮ اﻟﺘﻤﺮﻳﻦ ‪ 137‬ﺻﻔﺤﺔ ‪206‬‬ ‫‬
‫⧐‬

‫𝟵𝟳𝟯‬

‫‪http ://tinyurl.com/Malki1718‬‬ ‫‪0‬‬


‫ا‪ .5.‬ادلوال ﻛﺜﲑات اﳊﺪود و اﳉﺬور‬
‫‪.‬‬

‫ﻣﺒﺮﻫﻨﺔ ‪ : 39‬ﻟﻴﻜﻦ ]‪ P ∈ 𝕂[X‬ﻛﺜﲑ ﺣﺪود ﻏﲑ ﺛﺎﺑﺖ‪ .‬ﻳﻮﺟﺪ ﻋﺪد ﻃﺒﻴﻌﻲ وﺣﻴﺪ ∗‪ r ∈ ℕ‬؛ ﻛﺜﲑات ﺣﺪود ﻏﲑ ﻗﺎﺑﻠﺔ‬

‫‪Ŕž Ŧ Ľ‬‬
‫ﻟﻠﺘﺤﻠﻴﻞ ﻋﲆ ]‪ 𝕂[X‬و واﺣﺪﻳﺔ ‪ P ، ⋯ ، P ، P‬؛ أﻋﺪاد ﻃﺒﻴﻌﻴﺔ ﻏﲑ ﻣﻌﺪوﻣﺔ 𝛼 ‪ 𝛼 ، ⋯ ، 𝛼 ،‬و ﻋﺪد )ﻣﻌﺎﻣﻞ( وﺣﻴﺪ‬
‫∗𝕂 ∈ 𝜆 ﺑﺤﻴﺚ ‪:‬‬

‫ا‬
‫‪P = 𝜆P‬‬ ‫‪P‬‬ ‫‪⋯P‬‬

‫‪œǃ‬‬
‫ﻣﺒﺮﻫﻨﺔ ‪: 40‬‬

‫‪Ŀ‬‬
‫أس‪.‬‬
‫ﻟﻜﺜﲑ ْي ﺣﺪود ﻳﻜﻔﻲ أﺧﺬ اﻟﻌﻮاﻣﻞ اﳌﺸﱰﻛﺔ ﰲ ﲢﻠﻴﻠﻬﲈ إﱄ ﺟﺪاء ﻋﻮاﻣﻞ و ﺑﺄﺻﻐﺮ ّ‬
‫َ‬ ‫ﻹﳚﺎد اﻟﻘﺎﺳﻢ اﳌﺸﱰك اﻷﻛﱪ‬ ‫•‬

‫ﻟﻜﺜﲑ ْي ﺣﺪود ﻳﻜﻔﻲ أﺧﺬ اﻟﻌﻮاﻣﻞ اﳌﺸﱰﻛﺔ و ﻏﲑ اﳌﺸﱰﻛﺔ ﰲ ﲢﻠﻴﻠﻬﲈ إﱄ ﺟﺪاء‬


‫َ‬ ‫ﻹﳚﺎد اﳌﻀﺎﻋﻒ اﳌﺸﱰك اﻷﺻﻐﺮ‬ ‫•‬

‫أس‪.‬‬
‫ﻋﻮاﻣﻞ و ﺑﺄﻛﱪ ّ‬

‫)‪ّ B = X (X − 1) (X + 2) (X − 3‬‬
‫ﻓﺈن ‪:‬‬ ‫‪X +4‬‬ ‫إذا ﻛﺎن )‪ A = 2X (X + 1) (X − 3‬و‬ ‫ﻣﺜﺎﻝ ‪: 31‬‬

‫)‪pgcd (A, B) = X (X − 3‬‬


‫)‪ppcm (A, B) = X (X − 1) (X + 1) (X + 2) (X − 3‬‬ ‫‪X +4‬‬

‫ﺍﻟﺪﻭﺍﻝ ﻛﺜﲑﺍﺕ ﺍﳊﺪﻭﺩ ﻭ ﺍﳉﺬﻭﺭ‬ ‫ﺍ‪5.‬‬

‫ﺍﻟﺪﻭﺍﻝ ﻛﺜﲑﺍﺕ ﺍﳊﺪﻭﺩ‬ ‫ﺍ‪1.5.‬‬

‫ﺗﻌﺮﻳﻒ ‪: 24‬‬
‫‪ņőƱŀ Ţ‬‬
‫= )‪ P (X‬و 𝕂 ∈ 𝜆 ‪ .‬ﻧُﺴﻤﻲ ﻗﻴﻤﺔ ‪ P‬ﻋﻨﺪ 𝜆 ‪ ،‬و ﻧﻜﺘﺐ )𝜆( ‪ ، P‬اﳌﻘﺪار‬ ‫‪a X‬‬ ‫ﻟﻴﻜﻦ ]‪ P ∈ 𝕂[X‬ﺣﻴﺚ‬ ‫•‬
‫=‬

‫= )𝜆( ‪. P‬‬ ‫𝜆 ‪a‬‬


‫=‬

‫⎧‬ ‫⟶𝕂‬ ‫𝕂‬


‫⎪‬
‫= )‪ P (X‬و ﻏﺎﻟﺒ ًﺎ ﻣﺎ ﻧﺮﻣﺰ‬ ‫‪a X‬‬ ‫داﻟﺔ ﻛﺜﲑ اﳊﺪود اﳌﺮﻓﻘﺔ ﺑﻜﺜﲑ اﳊﺪود‬ ‫ﺗﺴﻤﻰ اﻟﺪاﻟﺔ‬ ‫•‬
‫=‬
‫⎨‬
‫⟼‪⎪ x‬‬ ‫‪a x‬‬
‫⎩‬ ‫=‬
‫‪ƕ‬‬

‫إﻟﻴﻬﺎ ﺑﻨﻔﺲ اﻟﺮﻣﺰ ‪. P‬‬

‫ﺟﺬﻭﺭ ﻛﺜﲑﺍﺕ ﺍﳊﺪﻭﺩ‬ ‫ﺍ‪2.5.‬‬


‫‪ũŏ‬‬

‫ﻟﻴﻜﻦ 𝕂 ∈ 𝜆 و ]‪ . P ∈ 𝕂[X‬ﺑﺎﻗﻲ اﻟﻘﺴﻤﺔ اﻹﻗﻠﻴﺪﻳﺔ ﻟـِ ‪ P‬ﻋﲆ )𝜆 ‪ (X −‬ﻫﻮ )𝜆( ‪. P‬‬ ‫ﻣﺒﺮﻫﻨﺔ ‪: 41‬‬

‫𝟬𝟴𝟯‬

‫‪http ://tinyurl.com/Malki1718‬‬ ‫‪0‬‬


‫ا‪ .‬ﻛﺜﲑات اﳊﺪود‬
‫‪.‬‬
‫ﻟﻴﻜﻦ ]‪ P ∈ 𝕂[X‬و 𝕂 ∈ 𝜆 ‪.‬‬ ‫ﺗﻌﺮﻳﻒ ‪: 25‬‬

‫‪Ŕž Ŧ Ľ‬‬
‫ﻟﺪﻳﻨﺎ ‪:‬‬ ‫•‬
‫ا‬

‫‪ P ⟺ P (𝜆) = 0‬ﻳﻘﺒﻞ اﻟﻘﺴﻤﺔ ﻋﲆ )𝜆 ‪ (X −‬ﰲ ]‪𝕂[X‬‬

‫‪œǃ‬‬
‫و ﻧﻘﻮل ﰲ ﻫﺬه اﳊﺎﻟﺔ ّ‬
‫أن 𝜆 ﺟﺬر ﻟﻜﺜﲑ اﳊﺪود ‪ P‬ﰲ ]‪. 𝕂[X‬‬
‫اﻟﻌﺪد 𝜆 ﺟﺬر ﻣﻦ اﻟﺮﺗﺒﺔ ‪ k‬ﻟﻜﺜﲑ اﳊﺪود ‪ P‬إذا ﻛﺎن ‪ k‬ﻫﻮ أﻛﱪ ﻋﺪد ﻃﺒﻴﻌﻲ ﺑﺤﻴﺚ ﻳﻜﻮن )𝛼 ‪ (X −‬ﻗﺎﺳ ًﲈ ﻟﻜﺜﲑ اﳊﺪود‬ ‫•‬

‫‪Ŀ‬‬
‫)‪. P (X‬‬
‫أن 𝜆 ﺟﺬر ﺑﺴﻴﻂ و إذا ﻛﺎن ‪ k = 2‬ﻧﻘﻮل ّ‬
‫أن 𝜆 ﺟﺬر ﻣﻀﺎ َﻋﻒ‪.‬‬ ‫إذا ﻛﺎن ‪ k = 1‬ﻧﻘﻮل ّ‬

‫ﻼ ‪ X + 1‬ﻻ ﻳﻘﺒﻞ ﺟﺬور ﰲ ‪ ℝ‬ﻟﻜﻦ ﻟﻪ ﺟﺬران ﰲ‬ ‫‪ u‬ﲢﺪﻳﺪ اﳌﺠﻤﻮﻋﺔ 𝕂 اﻟﺘﻲ ﻧﺒﺤﺚ ﻓﻴﻬﺎ ﻋﻦ اﳉﺬور ﴐوري و ﻣﻬﻢ ؛ ﻓﻤﺜ ً‬
‫‪ ℂ‬و ﳘﺎ ‪ ı‬و ‪. −ı‬‬
‫ﻣﺜﺎﻝ ‪ : 32‬ﻟﻜﺜﲑ اﳊﺪود )‪ P = X (X − 1) (X + 2‬ﺛﻼﺛﺔ ﺟﺬور ﳐﺘﻠﻔﺔ و ﻫﻲ ‪) 1 :‬ﺟﺬر ﻣﻦ اﻟﺮﺗﺒﺔ ‪) 0 ، (4‬ﺟﺬر‬
‫ﻷن ‪. 4 + 2 + 1 = 7‬‬‫ﻣﻀﺎ َﻋﻒ( و ‪) −2‬ﺟﺬر ﺑﺴﻴﻂ(‪ ،‬ﻟﻜﻦ ﻋﺪد ﺟﺬوره اﻹﲨﺎﱄ ﻫﻮ ‪ّ 7‬‬

‫ﻟﻴﻜﻦ ‪ . n ∈ ℕ‬ﻛﻞ ﻛﺜﲑ ﺣﺪود ﻏﲑ ﻣﻌﺪوم درﺟﺘﻪ أﺻﻐﺮ ﻣﻦ أو ﺗﺴﺎوي ‪ n‬ﻳﻘﺒﻞ ‪ n‬ﺟﺬر ًا ﻋﲆ اﻷﻛﺜﺮ‪.‬‬ ‫•‬ ‫ﻣﺒﺮﻫﻨﺔ ‪: 42‬‬

‫ﻟﻴﻜﻦ ‪ . n ∈ ℕ‬ﻳﻮﺟﺪ ﻛﺜﲑ ﺣﺪود وﺣﻴﺪ درﺟﺘﻪ أﺻﻐﺮ ﻣﻦ أو ﺗﺴﺎوي ‪ n‬و ﻳﻘﺒﻞ ‪ n + 1‬ﺟﺬر ًا )ﻋﲆ اﻷﻗﻞ( و ﻫﻮ ﻛﺜﲑ‬ ‫•‬

‫اﳊﺪود اﳌﻌﺪوم‪.‬‬

‫وﻛﺤﺎﻟﺔ ﺧﺎﺻﺔ‪ ،‬ﻛﺜﲑ اﳊﺪود اﳌﻌﺪوم ﻫﻮ اﻟﻮﺣﻴﺪ اﻟﺬي ﻟﻪ ﻋﺪد ﻻﳖﺎﺋﻲ ﻣﻦ اﳉﺬور‪.‬‬ ‫•‬

‫‪ u‬إذا ﻛﺎن ‪ P‬ﻛﺜﲑ ﺣﺪود درﺟﺘﻪ ‪ n‬ﻓﻬﺬا ﻻ ﻳﻌﻨﻲ ﺑﺎﻟﴬورة أﻧﻪ ﻳﻘﺒﻞ ‪ n‬ﺟﺬر ًا‪ .‬ذﻟﻚ ﺻﺤﻴﺢ إذا ﻛﺎن ‪ 𝕂 = ℂ‬ﻟﻜﻨﻪ ﺧﺎﻃﺊ‬
‫أي ﺟﺬر ﺣﻘﻴﻘﻲ‪.‬‬‫ﺑﺎﻟﻨﺴﺒﺔ إﱃ ‪ : 𝕂 = ℝ‬ﻓﻜﺜﲑ اﳊﺪود ‪ X + 1‬ذي اﻟﺪرﺟﺔ ‪ 2‬ﻻ ﻳﻘﺒﻞ ّ‬
‫‪ņőƱŀ Ţ‬‬
‫ﺇﺷﺘﻘﺎﻕ ﻛﺜﲑﺍﺕ ﺍﳊﺪﻭﺩ‬ ‫ﺍ‪3.5.‬‬

‫= )‪ P (X‬ﻛﺜﲑ ﺣﺪود ﺑﻤﻌﺎﻣﻼت ﰲ 𝕂 درﺟﺘﻪ ‪. n‬‬ ‫‪a X‬‬ ‫ﻟﻴﻜﻦ‬ ‫ﺗﻌﺮﻳﻒ ‪: 26‬‬
‫=‬
‫‪−‬‬
‫= )‪ ، P (X‬و ﻫﻮ ﻛﺜﲑ ﺣﺪود ﻣﻦ‬ ‫‪ka X‬‬ ‫اﳌﻌﺮف ﺑﺎﻟﻌﺒﺎرة ‪:‬‬
‫ﻧﺴﻤﻲ ﻣﺸﺘﻖ ‪ P‬ﻛﺜﲑ اﳊﺪود اﻟﺬي ﻧﺮﻣﺰ إﻟﻴﻪ ﺑــِ ‪ P‬و ﱠ‬
‫=‬
‫اﻟﺪرﺟﺔ ‪. n − 1‬‬
‫‪ƕ‬‬

‫‪−‬‬
‫= )‪ P (X‬؛‬ ‫‪k (k − 1) a X‬‬ ‫ﺑﺎﳌﺜﻞ‪ ،‬اﳌﺸﺘﻖ اﻟﺜﺎﲏ ﻫﻮ ﻛﺜﲑ اﳊﺪود اﻟﺬي ﻧﺮﻣﺰ إﻟﻴﻪ ﺑﺎﻟﺮﻣﺰ ‪ P‬و اﻟﺬي ﻳﺴﺎوي ‪:‬‬
‫=‬
‫درﺟﺘﻪ ﻫﻲ ‪. n − 2‬‬
‫اﳌﻌﺮف ﺑــِ ‪:‬‬
‫‪P‬و ﱠ‬ ‫و ﺑﺼﻔﺔ ﻋﺎﻣﺔ‪ ،‬ﻧﺴﻤﻲ اﳌﺸﺘﻖ ﻣﻦ اﻟﺮﺗﺒﺔ ‪ r‬ﻛﺜﲑ اﳊﺪود اﻟﺬي ﻧﺮﻣﺰ إﻟﻴﻪ ﺑﺎﻟﺮﻣﺰ‬
‫‪ũŏ‬‬

‫‪−‬‬ ‫‪−‬‬
‫‪P‬‬ ‫‪(X) = P‬‬ ‫= )‪(X‬‬ ‫‪k (k − 1) (k − 2) ⋯ (k − r + 1) X‬‬
‫=‬

‫‪.P‬‬ ‫اﳌﺸﺘﻖ ﻣﻦ اﻟﺪرﺟﺔ ‪ 0‬ﻫﻮ ‪= P :‬‬ ‫‪u‬‬

‫𝟭𝟴𝟯‬

‫‪http ://tinyurl.com/Malki1718‬‬ ‫‪0‬‬


‫ا‪ .5.‬ادلوال ﻛﺜﲑات اﳊﺪود و اﳉﺬور‬
‫‪.‬‬
‫‪P‬و‬ ‫‪ P‬؛ ‪ P = 24X − 10X‬؛ ‪ P = 48X − 10‬؛‪= 48‬‬ ‫‪=P‬‬ ‫إذا ﻛﺎن ‪ّ P = 8X − 5X + 1‬‬
‫ﻓﺈن ‪:‬‬ ‫ﻣﺜﺎﻝ ‪: 33‬‬
‫‪.P‬‬ ‫‪=0‬‬

‫‪Ŕž Ŧ Ľ‬‬
‫ﻣﺒﺮﻫﻨﺔ ‪: 43‬‬

‫ا‬
‫ﻟﻴﻜﻦ ‪ n ∈ ℕ‬و ]‪ . P, Q ∈ 𝕂[X‬ﻟﺪﻳﻨﺎ ‪:‬‬

‫‪œǃ‬‬
‫∘‬ ‫∘‬ ‫∘‬
‫‪.P‬‬ ‫‪ d P‬إذا ﻛﺎن ‪ n ≤ d P‬و إﻻّ ّ‬
‫ﻓﺈن ‪= 0 :‬‬ ‫‪=d P−n‬‬ ‫•‬ ‫‪1‬‬
‫)‪. (P + Q‬‬ ‫‪=P‬‬ ‫‪+Q‬‬ ‫•‬ ‫‪2‬‬

‫‪Ŀ‬‬
‫‪ (PQ) = P Q + PQ •3‬و ﺑﺼﻔﺔ ﻋﺎﻣﺔ‪ ،‬ﻟﺪﻳﻨﺎ ﺻﻴﻐﺔ ﻟﻴﺒﻨﻴﺘﺰ )‪: (L‬‬
‫‪n‬‬ ‫‪−‬‬
‫)‪(PQ‬‬ ‫=‬ ‫‪P‬‬ ‫‪Q‬‬
‫=‬
‫‪k‬‬

‫‪. (P ∘ Q) = Q × P ∘ Q‬‬ ‫•‬ ‫‪4‬‬

‫‪ u‬ﺻﻴﻐﺔ ﻟﻴﺒﻨﻴﺘﺰ ﺗُﺸﺒﻪ إﱃ ﺣﺪّ ﻣﺎ ﺻﻴﻐﺔ ﺛﻨﺎﺋﻲ اﳊﺪ و ﻳﻤﻜﻦ إﺛﺒﺎﲥﺎ ﺑﻨﻔﺲ اﻟﻄﺮﻳﻘﺔ‪.‬‬

‫)ﺻﻴﻐﺔ ﺗﺎﻳﻠﻮﺭ – ‪ (T‬إذا ﻛﺎن ]‪ P ∈ 𝕂 [X‬ﻛﺜﲑ ﺣﺪود درﺟﺘﻪ ‪ n‬و ﻛﺎن 𝕂 ∈ ‪ّ a‬‬
‫ﻓﺈن ‪:‬‬ ‫ﻣﺒﺮﻫﻨﺔ ‪: 44‬‬
‫‪P‬‬ ‫)‪(a‬‬ ‫‪P‬‬ ‫)‪(a‬‬
‫= )‪P (X‬‬ ‫)‪(X − a‬‬ ‫و‬ ‫= )‪P (X + a‬‬ ‫‪X‬‬
‫=‬
‫!‪k‬‬ ‫=‬
‫!‪k‬‬
‫‪) P‬اﳌﺸﺘﻖ ﻣﻦ اﻟﺪرﺟﺔ ‪ (0‬و !‪ k‬ﻫﻮ ِ‬
‫ﻋﺎﻣ ِﲇ اﻟﻌﺪد ‪ k‬أي ‪. k! = k (k − 1) (k − 2) ⋯ 3 ⋅ 2 ⋅ 1‬‬ ‫ﺣﻴﺚ ‪= P‬‬

‫ﻟﻴﻜﻦ ]‪ 𝜆 ∈ 𝕂 ، P ∈ 𝕂[X‬و ‪ . k ∈ ℕ‬ﻟﺪﻳﻨﺎ ‪:‬‬ ‫ﻣﺒﺮﻫﻨﺔ ‪: 45‬‬

‫ِ‬
‫)𝜆 ‪ 𝜆 ⟺ (X −‬ﺟﺬر ﻣﻦ اﻟﺮﺗﺒﺔ ‪ k‬ﻟـ ‪P‬‬
‫‪+‬‬
‫‪ (X − 𝜆) ∣ P‬ﻟﻜﻦ ‪∤ P‬‬
‫‪ņőƱŀ Ţ‬‬
‫ﻳﻮﺟﺪ ]‪ Q ∈ 𝕂[X‬ﺑﺤﻴﺚ ‪ Q (𝜆) ≠ 0‬و ‪⟺ P = (X − 𝜆) Q‬‬
‫‪−‬‬
‫‪⟺ P‬‬ ‫‪ P (𝜆) = P (𝜆) = ⋯ = P‬ﻟﻜﻦ ‪(𝜆) ≠ 0‬‬ ‫‪(𝜆) = 0‬‬

‫ِ‬
‫ﻣﺜﺎﻝ ‪ : 34‬اﻟﻌﺪد ‪ 1‬ﺟﺬر ﻣﻀﺎﻋﻒ ﻟـ ‪ّ P (X) = X + 3X − 3x − 7X + 6‬‬
‫ﻷن ‪P (1) = 1 + 3 − 3 − 7 + 6 = 0 :‬‬
‫؛ ‪ P (X) = 4X + 9X − 6X − 7‬ﻣﻨﻪ ‪ P (1) = 4 + 9 − 6 − 7 = 0‬و ‪ P (X) = 12X + 18X − 6‬ﻣﻨﻪ‬
‫‪. P (1) = 12 + 18 − 6 = 24 ≠ 0‬‬

‫ﺍﻟﻌﻼﻗﺎﺕ ﺑﲔ ﺍﳌﻌﺎﻣﻼﺕ ﻭ ﺍﳉﺬﻭﺭ‬ ‫ﺍ‪4.5.‬‬


‫‪ƕ‬‬

‫ﺗﻌﺮﻳﻒ ‪ : 27‬ﻟﻴﻜﻦ ‪ P‬ﻛﺜﲑ ﺣﺪود ﻣﻦ اﻟﺪرﺟﺔ ‪ . n‬ﻧﻘﻮل ّ‬


‫أن ‪ُ P‬ﻣﻔﻜّﻚ ﺗﻔﻜﻴﻜ ًﺎ ﺗﺎﻣ ًﺎ ﻋﲆ 𝕂 إذا ﻛﺎن ‪ P‬ﻳﻘﺒﻞ ‪ n‬ﺟﺬر ًا ﰲ 𝕂 ‪.‬‬
‫‪ũŏ‬‬

‫ﰲ ﻫﺬه اﳊﺎﻟﺔ‪ ،‬إذا ﻛﺎن ‪ P‬ﻳﻘﺒﻞ ‪ r‬ﺟﺬر ًا 𝜆 ‪ُ 𝜆 ، ⋯ ، 𝜆 ،‬رﺗﺒﻬﺎ 𝛼 ‪ 𝛼 ، ⋯ ، 𝛼 ،‬و إذا ﻛﺎن ‪ A‬ﻣﻌﺎﻣﻠﻪ اﳌﻬﻴﻤﻦ ّ‬
‫ﻓﺈن ‪:‬‬
‫=‪n‬‬ ‫𝛼‬ ‫و‬ ‫‪P=A‬‬ ‫) 𝜆 ‪(X −‬‬
‫=‬ ‫=‬

‫ﻼ )‪ُ X + 1 = (X − ı) (X + ı‬ﻣﻔﻜّﻚ ﺗﻔﻜﻴﻜ ًﺎ ﺗﺎﻣ ًﺎ‬


‫‪ u‬ﳚﺐ ﲢﺪﻳﺪ اﳌﺠﻤﻮﻋﺔ اﻟﺘﻲ ﻳﻘﺒﻞ ﻓﻴﻬﺎ ﻛﺜﲑ ﺣﺪود ﺗﻔﻜﻴﻜ ًﺎ ﺗﺎﻣ ًﺎ‪ ،‬ﻓﻤﺜ ً‬
‫ﻋﲆ ‪ ℂ‬ﻟﻜﻨﻪ ﻏﲑ ﻗﺎﺑﻞ ﻟﻠﺘﺤﻠﻴﻞ ﻋﲆ ‪. ℝ‬‬

‫𝟮𝟴𝟯‬

‫‪http ://tinyurl.com/Malki1718‬‬ ‫‪0‬‬


‫ا‪ .‬ﻛﺜﲑات اﳊﺪود‬
‫‪.‬‬
‫ﻟﻜﻞ ∗‪ ، n ∈ ℕ‬ﻛﺜﲑ اﳊﺪود ‪ X − 1‬ﻳﻘﺒﻞ ﺗﻔﻜﻴﻜ ًﺎ ﺗﺎﻣ ًﺎ ﻋﲆ ‪ ℂ‬و ﻟﺪﻳﻨﺎ ‪:‬‬ ‫ﻣﺜﺎﻝ ‪: 35‬‬
‫‪−‬‬ ‫‪−‬‬ ‫‪−‬‬

‫‪Ŕž Ŧ Ľ‬‬
‫= ‪X −1‬‬ ‫‪/‬‬ ‫‪/‬‬
‫= )𝜔 ‪(X −‬‬ ‫‪X−e‬‬ ‫و‬ ‫= ‪X‬‬ ‫= )𝜔 ‪(X −‬‬ ‫‪X−e‬‬
‫𝕌‬ ‫=‬ ‫=‬ ‫⧵ 𝕌‬ ‫=‬

‫‬
‫ا‬

‫‪œǃ‬‬
‫أﻧﻈﺮ اﻟﺘﻤﺮﻳﻦ ‪ 219‬ﺻﻔﺤﺔ ‪312‬‬

‫= ‪ P‬ﻣﻦ اﻟﺪرﺟﺔ ‪ n‬و ﻟﺘﻜﻦ 𝜆 ‪ 𝜆 ، ⋯ ، 𝜆 ،‬ﺟﺬوره )ﳐﺘﻠﻔﺔ أو ﻣﻜﺮرة( ‪.‬‬ ‫ﻣﺒﺮﻫﻨﺔ ‪ : 46‬ﻟﻴﻜﻦ ]‪a X ∈ 𝕂[X‬‬

‫‪Ŀ‬‬
‫=‬
‫= 𝜎 أي ‪:‬‬ ‫𝜆 𝜆‬ ‫𝜆⋯‬ ‫ﻟﻜﻞ }‪ k ∈ {1, 2, ⋯ , n‬ﻧﻀﻊ‬
‫≤‬ ‫<‬ ‫<⋯<‬ ‫≤‬

‫𝜆‪𝜎 =𝜆 +𝜆 +⋯+‬‬
‫𝜆‪𝜎 =𝜆 𝜆 +𝜆 𝜆 +⋯+𝜆 𝜆 +𝜆 𝜆 +⋯+‬‬ ‫‪−‬‬ ‫𝜆‬
‫⋮‬

‫𝜎‬ ‫‪−‬‬ ‫=‬ ‫𝜆⋯ 𝜆⋯ 𝜆 𝜆‬


‫=‬
‫𝜆⋯ 𝜆 𝜆 = 𝜎‬

‫ﺣﻴﺚ 𝜆 ﻳﻌﻨﻲ أﻧﻨﺎ ﻧﺤﺬف اﳊﺪ 𝜆 ‪ .‬ﻟﺪﻳﻨﺎ ‪:‬‬


‫‪a −‬‬
‫‪∀k ∈ {1, 2, ⋯ , n} :‬‬ ‫)‪𝜎 = (−1‬‬
‫‪a‬‬

‫ﻣﻼﺣﻈﺔ ‪ ⧏ : 65‬ﻫﺬه اﳌﱪﻫﻨﺔ ﻻ ﺗﺴﻤﺢ ﻟﻨﺎ ﺑﺎﻟﺘﻌﺒﲑ ﻋﻦ ﺟﺬور ﻛﺜﲑ اﳊﺪود ‪ P‬ﺑﺪﻻﻟﺔ ﻣﻌﺎﻣﻼﺗﻪ ﻟﻜﻨﻬﺎ ﺗﺴﻤﺢ ﻟﻨﺎ ﺑﺎﻟﺘﻌﺒﲑ‬
‫ﺑﺪﻻﻟﺔ اﳌﻌﺎﻣﻼت ﻋﻦ دوال 𝜎 ‪ 𝜎 ، ⋯ ، 𝜎 ،‬ﺗُﺪﻋﻰ اﻟﺪوال اﻟﺘﻨﺎﻇﺮﻳﺔ اﻷﺳﺎﺳﻴﺔ )أو اﻟﻌﻨﴫﻳﺔ( ﰲ اﳌﺘﻐﲑات 𝜆 ‪،⋯ ، 𝜆 ،‬‬
‫𝜆‪.‬‬
‫= 𝜎 )ﺣﺎﺻﻞ ﴐب اﳉﺬور( ‪.‬‬ ‫𝜆‬ ‫و‬ ‫= 𝜎 )ﳎﻤﻮع اﳉﺬور(‬ ‫𝜆‬ ‫ﻟﺪﻳﻨﺎ ‪:‬‬
‫‪ņőƱŀ Ţ‬‬
‫⧐‬ ‫=‬ ‫=‬

‫ﻣﺜﺎﻝ ‪ : 36‬إذا ﻛﺎﻧﺖ درﺟﺔ ﻛﺜﲑ اﳊﺪود ﻫﻲ ‪ n = 4‬و ﻛﺎن ‪ّ P = a + a X + a X + a X + a X‬‬


‫ﻓﺈن )ﻧُﺪﻛّﺮ ّ‬
‫أن‬
‫‪ّ a ≠0‬‬
‫ﻷن درﺟﺔ ‪ P‬ﻫﻲ ‪: (4‬‬
‫‪a −‬‬ ‫‪a‬‬
‫)‪𝜎 = 𝜆 + 𝜆 + 𝜆 + 𝜆 = (−1‬‬ ‫‪=−‬‬
‫‪a‬‬ ‫‪a‬‬
‫‪a −‬‬ ‫‪a‬‬
‫)‪𝜎 = 𝜆 𝜆 + 𝜆 𝜆 + 𝜆 𝜆 + 𝜆 𝜆 + 𝜆 𝜆 + 𝜆 𝜆 = (−1‬‬ ‫=‬
‫‪a‬‬ ‫‪a‬‬
‫‪a −‬‬ ‫‪a‬‬
‫‪ƕ‬‬

‫)‪𝜎 = 𝜆 𝜆 𝜆 + 𝜆 𝜆 𝜆 + 𝜆 𝜆 𝜆 + 𝜆 𝜆 𝜆 = (−1‬‬ ‫‪=−‬‬


‫‪a‬‬ ‫‪a‬‬
‫‪a −‬‬ ‫‪a‬‬
‫)‪𝜎 = 𝜆 𝜆 𝜆 𝜆 = (−1‬‬ ‫=‬
‫‪a‬‬ ‫‪a‬‬
‫‪ũŏ‬‬

‫‪b‬‬
‫ﰲ اﳊﺎﻟﺔ اﳋﺎﺻﺔ ‪ n = 2‬و ‪) P = c + bX + aX‬ﺣﻴﺚ ‪ّ (a ≠ 0‬‬
‫ﻓﺈن ﳎﻤﻮع اﳉﺬرﻳﻦ ﻫﻮ ‪ 𝜆 + 𝜆 = −‬و‬ ‫ﻣﺜﺎﻝ ‪: 37‬‬
‫‪a‬‬
‫‪c‬‬
‫ﺣﺎﺻﻞ ﴐﲠﲈ ﻫﻮ = 𝜆 𝜆 ‪.‬‬
‫‪a‬‬
‫ﻣﺜﺎﻝ ‪ : 38‬ﻟﻴﻜﻦ ‪ . P = X − 1‬ﻟﺪﻳﻨﺎ ‪ a = 1 ، a = −1‬و ‪ a = 0‬ﻷﺟﻞ }‪. k ∈ {2, 3, ⋯ , n − 1‬‬

‫𝟯𝟴𝟯‬

‫‪http ://tinyurl.com/Malki1718‬‬ ‫‪0‬‬


‫ا‪ .5.‬ادلوال ﻛﺜﲑات اﳊﺪود و اﳉﺬور‬
‫‪.‬‬
‫‪) 𝜆 = e‬ﻷﺟﻞ }‪ّ (k ∈ {1, 2, ⋯ , n‬‬
‫ﻓﺈن ‪:‬‬ ‫‪/‬‬ ‫ﺑﲈ ّ‬
‫أن ﺟﺬور ‪ P‬ﻫﻲ‬

‫‪Ŕž Ŧ Ľ‬‬
‫‪−‬‬
‫‪a −‬‬ ‫‪0‬‬
‫= 𝜎=𝜔‬ ‫‪e‬‬ ‫)‪= (−1‬‬ ‫‪=− =0‬‬
‫𝕌‬ ‫=‬
‫‪a‬‬ ‫‪1‬‬

‫‪œǃ‬‬
‫‪−‬‬
‫‪a −‬‬ ‫‪−1‬‬ ‫‪+‬‬
‫= 𝜎=𝜔‬ ‫‪e‬‬ ‫)‪= (−1‬‬ ‫× )‪= (−1‬‬ ‫)‪= (−1‬‬ ‫و‬
‫𝕌‬ ‫=‬
‫‪a‬‬ ‫‪1‬‬

‫ﻣﻼﺣﻈﺔ ‪ ⧏ : 66‬ﻃﺮﻳﻘﺔ اﳌﺜﺎل اﻵﰐ ﻫﻲ ﻃﺮﻳﻘﺔ ﻛﻼﺳﻴﻜﻴﺔ‪ ،‬ﺗﺴﺘﺨﺪم ﻛﺜﲑ ًا ﰲ ﺣﻞ ُﲨﻞ ﻣﻌﺎدﻻت ﺑﻌﺪة ﳎﺎﻫﻴﻞ ﻣﺮﺗﺒﻄﺔ‬

‫‪Ŀ‬‬
‫ﻓﻴﲈ ﺑﻴﻨﻬﺎ ﺑﻌﻼﻗﺎت ﺗﻨﺎﻇﺮﻳﺔ ‪.‬‬
‫⧐‬
‫⎧‬ ‫‪x+y+z=1‬‬
‫⎪‬
‫⎪‬
‫⎪‬
‫‪x + y + z = 21‬‬ ‫)⋆(‬
‫∗‪ ، (x, y, z) ∈ ℂ‬ﻧﻀﻊ ‪:‬‬ ‫ذات اﳌﺠﻬﻮل‬ ‫ﻣﺜﺎﻝ ‪ : 39‬ﳊﻞ اﳉﻤﻠﺔ‬
‫⎨‬
‫⎪‬ ‫‪1 1 1‬‬
‫⎪‬
‫⎪‬ ‫‪+ + =1‬‬
‫‪⎩ x y z‬‬
‫‪. 𝜎 = xyz‬‬ ‫و‬ ‫)‪𝜎 = xy + yz + zx ، 𝜎 = x + y + z ، P = (X − x) (X − y) (X − z‬‬
‫ﺣﺴﺐ ﻣﺎ ﺳﺒﻖ ّ‬
‫ﻓﺈن 𝜎 ‪ . P = X − 𝜎 X + 𝜎 X −‬ﳊﻞ اﳉﻤﻠﺔ )⋆( ﻧﺘّﺒﻊ اﳋﻄﻮات اﻟﺘﺎﻟﻴﺔ ‪:‬‬
‫‪1‬‬ ‫‪1 1‬‬
‫‪ +‬ﺑﺪﻻﻟﺔ اﻟﺪوال اﻟﺘﻨﺎﻇﺮﻳﺔ اﻟﻌﻨﴫﻳﺔ 𝜎 ‪ 𝜎 ،‬و 𝜎 ‪.‬‬
‫‪x‬‬
‫‪+‬‬
‫‪y z‬‬ ‫‪ •1‬ﺑﺪاﻳ ًﺔ‪ ،‬ﻧ ﱢ‬
‫ُﻌﱪ ﻋﻦ ‪ x + y + z ، x + y + z‬و‬

‫ﺣﻞ ﻟﻠﺠﻤﻠﺔ )⋆( إذا و ﻓﻘﻂ إذا ﻛﺎﻧﺖ ﻟـِ 𝜎 ‪ 𝜎 ،‬و 𝜎 ِﻗ َﻴﻢ ﳏﺪدة ؛ أو ﺑﺘﻌﺒﲑ آﺧﺮ‪ ،‬إذا و‬
‫أن )‪ّ (x, y, z‬‬
‫‪ •2‬ﺑﻌﺪﻫﺎ‪ ،‬ﻧﱪﻫﻦ ّ‬
‫ﻓﻘﻂ إذا ﻛﺎن ‪ P‬ﻳﺴﺎوي ﻛﺜﲑ ﺣﺪود ‪ُ P‬ﻋﺮﻓﺖ ﻣﻌﺎﻣﻼﺗﻪ‪.‬‬
‫ﻓﺈن )‪ّ (x, y, z‬‬
‫ﺣﻞ ﻟﻠﺠﻤﻠﺔ )⋆( إذا و ﻓﻘﻂ إذا ﻛﺎن‬ ‫أن ‪ y ، x‬و ‪ z‬ﻫﻲ ﺟﺬور ‪ّ P‬‬
‫‪ •3‬ﰲ اﻷﺧﲑ‪ ،‬ﻧﺒﺤﺚ ﻋﻦ ﺟﺬور ‪ . P‬و ﺑﲈ ّ‬
‫‪ y ، x‬و ‪ z‬ﺟﺬور ‪) P‬و ﻫﻲ ِﻗ َﻴﻢ ﻣﻌﺮوﻓﺔ( ‪.‬‬
‫ﻟﺪﻳﻨﺎ ‪:‬‬
‫𝜎=‪x+y+z‬‬
‫‪ņőƱŀ Ţ‬‬
‫𝜎‪x + y + z = (x + y + z) − 2 (xy + yz + zx) = 𝜎 − 2‬‬
‫𝜎 ‪1 1 1 xy + yz + zx‬‬
‫= ‪+ +‬‬ ‫=‬
‫‪x y z‬‬ ‫‪xyz‬‬ ‫𝜎‬
‫إذن ‪:‬‬

‫⎧‬
‫⎪‬ ‫‪𝜎 =1‬‬
‫⎪‬
‫⎪‬ ‫‪⎪𝜎 = 1‬‬
‫⎧‬
‫⟺ )⋆(‬ ‫𝜎‬ ‫‪−‬‬ ‫𝜎‪2‬‬ ‫=‬ ‫‪21‬‬ ‫⟺‬ ‫‪𝜎 = −10 ⟺ P = X − X − 10X + 10‬‬
‫⎨‬
‫⎪‬
‫⎪‬ ‫𝜎‬ ‫⎨‬
‫⎪‬
‫⎪‬
‫‪ƕ‬‬

‫‪=1‬‬ ‫‪⎩ 𝜎 = −10‬‬


‫⎩‬ ‫𝜎‬
‫‪ y ، x‬و ‪ z‬ﻫﻲ ﺟﺬور ﻛﺜﲑ اﳊﺪود ‪⟺ P = X − X − 10X + 10‬‬

‫ﻟﻜﻦ ‪:‬‬
‫‪ũŏ‬‬

‫‪P = X (X − 1) − 10 (X − 1) = (X − 1) X − 10 = (X − 1) X − √10‬‬ ‫‪X + √10‬‬

‫ﻛﻞ ﺗﺒﺪﻳﻼت ‪ 8‬اﳌﺠﻤﻮﻋﺔ ‪ 1, √10, −√10‬و ﻋﺪدﻫﺎ ﻫﻮ‬


‫أن اﳊﻠﻮل ﻫﻲ ّ‬
‫إذن ‪ {x, y, z} = 1, √10, −√10‬أي ّ‬
‫‪ . 3! = 6‬ﳎﻤﻮﻋﺔ اﳊﻠﻮل ﻫﻲ إذن ‪:‬‬
‫‪1, √10, −√10 , 1, −√10, √10 , √10, 1, −√10 , √10, −√10, 1 , −√10, 1, √10 , −√10, √10, 1‬‬
‫‪ Permutations8‬ﺑﺎﻹﻧﺠﻠﻴﺰﻳﺔ و اﻟﻔﺮﻧﺴﻴﺔ‪.‬‬

‫𝟰𝟴𝟯‬

‫‪http ://tinyurl.com/Malki1718‬‬ ‫‪0‬‬


‫ا‪ .‬ﻛﺜﲑات اﳊﺪود‬
‫‪.‬‬
‫ﺍﻟﻨﻈﺮﻳﺔ ﺍﻷﺳﺎﺳﻴﺔ ﰲ ﺍﳉﱪ‬ ‫ﺍ‪5.5.‬‬

‫‪Ŕž Ŧ Ľ‬‬
‫اﳌﱪﻫﻨﺔ اﻵﺗﻴﺔ ﲢﻤﻞ إﺳﻢ اﻟﻨﻈﺮﻳﺔ اﻷﺳﺎﺳﻴﺔ ﰲ اﳉﱪ أو ﻧﻈﺮﻳﺔ ﺩﺍﻟﻮﻣﺒﻴﺮ–ﭬﻮﺹ ‪. 9‬‬

‫‪œǃ‬‬
‫ﻣﺒﺮﻫﻨﺔ ‪ : 47‬ﻛﻞ ﻛﺜﲑ ﺣﺪود ﻏﲑ ﺛﺎﺑﺖ ﻣﻦ ]‪ ℂ[X‬ﻳﻘﺒﻞ ﻋﲆ اﻷﻗﻞ ﺟﺬر ًا ﰲ ‪ ℂ‬؛ أو ﺑﺘﻌﺒﲑ آﺧﺮ‪ ،‬ﻛﻞ ﻛﺜﲑ ﺣﺪود ﻣﻦ ]‪ ℂ[X‬ﻣﻦ‬
‫اﻟﺪرﺟﺔ ‪ n‬ﻳﻘﺒﻞ ‪ n‬ﺟﺬر ًا ﰲ ‪) ℂ‬ﻫﺬه اﳉﺬور ﻳﻤﻜﻦ أن ﺗﻜﻮن ﳐﺘﻠﻔﺔ ﻣﺜﻨﻰ ﻣﺜﻨﻰ أو أن ﻳﻜﻮن اﻟﺒﻌﺾ ﻣﻨﻬﺎ – أو ﻛﻠﻬﺎ – ّ‬
‫ﻣﻜﺮر ًا(‪.‬‬

‫‪Ŀ‬‬
‫أي ﺟﺬر ﰲ ‪. ℝ‬‬ ‫‪ u‬ﻫﺬه اﻟﻨﺘﻴﺠﺔ ﻻ ﺗﺼﻠﺢ ﺑﺎﻟﻨﺴﺒﺔ ﻟـِ ]‪ ، ℝ[X‬ﻓﻤﺜ ً‬
‫ﻼ ﻛﺜﲑ اﳊﺪود ‪ X + 1‬ﻻ ﻳﻘﺒﻞ ّ‬

‫)ﻛﺜﻴﺮﺍﺕ ﺣﺪﻭﺩ ]‪ ℂ[X‬ﻏﻴﺮ ﺍﻟﻘﺎﺑﻠﺔ ﻟﻠﺘﺤﻠﻴﻞ(‬ ‫ﻣﺒﺮﻫﻨﺔ ‪: 48‬‬

‫‪ •1‬ﻛﺜﲑات ﺣﺪود ]‪ ℂ[X‬ﻏﲑ اﻟﻘﺎﺑﻠﺔ ﻟﻠﺘﺤﻠﻴﻞ ﻫﻲ ﻛﺜﲑات اﳊﺪود ﻣﻦ اﻟﺪرﺟﺔ ‪. 1‬‬

‫‪ •2‬ﻛﻞ ﻛﺜﲑ ﺣﺪود ﻣﻦ ]‪ ℂ[X‬ﻳﻘﺒﻞ ﺗﻔﻜﻴﻜ ًﺎ ﺗﺎﻣ ًﺎ ﻋﲆ ‪. ℂ‬‬

‫)ﻣﺮﺍﻓﻖ ﻛﺜﻴﺮ ﺣﺪﻭﺩ(‬ ‫ﻻﺯﻣﺔ ‪: 3‬‬

‫= ‪.P‬‬ ‫ﻟﻴﻜﻦ ]‪a X ∈ ℂ[X‬‬


‫=‬

‫= ‪.P‬‬ ‫ﻧﺴﻤﻲ ﻣﺮاﻓﻖ ‪ ، P‬و ﻧﻜﺘﺐ ‪ ، P‬ﻛﺜﲑ اﳊﺪود ]‪a X ∈ ℂ[X‬‬ ‫•‬
‫=‬

‫ﻟﻴﻜﻦ ‪ . 𝜆 ∈ ℂ‬ﻳﻜﻮن 𝜆 ﺟﺬر ًا ﻟـِ ‪ P‬ﻣﻦ اﻟﺮﺗﺒﺔ ‪ r‬إذا و ﻓﻘﻂ إذا ﻛﺎن 𝜆 ﺟﺬر ًا ﻟـِ ‪ P‬و ﻣﻦ اﻟﺮﺗﺒﺔ ‪. r‬‬ ‫•‬

‫ﻓﺈن ‪ P = P‬ﻣﻨﻪ ‪ 𝜆 :‬ﺟﺬر ﻟـِ ‪ P‬ﻣﻦ اﻟﺮﺗﺒﺔ ‪ r‬إذا و ﻓﻘﻂ إذا ﻛﺎن 𝜆 ﺟﺬر ًا ﻟـﻪ و ﻣﻦ ﻧﻔﺲ‬
‫و ﻛﺤﺎﻟﺔ ﺧﺎﺻﺔ‪ ،‬إذا ﻛﺎن ]‪ّ P ∈ ℝ[X‬‬
‫‪ņőƱŀ Ţ‬‬
‫اﻟﺮﺗﺒﺔ ‪. r‬‬

‫)ﻛﺜﻴﺮﺍﺕ ﺣﺪﻭﺩ ]‪ ℝ[X‬ﻏﻴﺮ ﺍﻟﻘﺎﺑﻠﺔ ﻟﻠﺘﺤﻠﻴﻞ(‬ ‫ﻣﺒﺮﻫﻨﺔ ‪: 49‬‬

‫‪ •1‬ﻛﺜﲑات ﺣﺪود ]‪ ℝ[X‬ﻏﲑ اﻟﻘﺎﺑﻠﺔ ﻟﻠﺘﺤﻠﻴﻞ ﻫﻲ ﻛﺜﲑات اﳊﺪود ﻣﻦ اﻟﺪرﺟﺔ ‪ 1‬و ﻛﺜﲑات اﳊﺪود ﻣﻦ اﻟﺪرﺟﺔ ‪ 2‬و اﻟﺘﻲ‬
‫ﳑﻴﺰﻫﺎ ﺳﺎﻟﺐ ﲤﺎﻣ ًﺎ )أي ﻟﻴﺲ ﳍﺎ ﺟﺬور ﺣﻘﻴﻘﻴﺔ(‪.‬‬
‫‪ƕ‬‬

‫‪ A‬ﺣﻴﺚ ‪:‬‬ ‫) 𝜆 ‪(X −‬‬ ‫×‬ ‫‪X +b X+c‬‬ ‫‪ •2‬ﻛﻞ ﻛﺜﲑ ﺣﺪود ]‪ P ∈ ℝ[X‬ﻳﻘﺒﻞ ﲢﻠﻴ ً‬
‫ﻼ ﻣﻦ اﻟﺸﻜﻞ‬
‫=‬ ‫=‬

‫‪ A‬ﻫﻮ اﳌﻌﺎﻣﻞ اﳌﻬﻴﻤﻦ ﰲ ‪ P‬؛‬ ‫•‬


‫‪ũŏ‬‬

‫𝜆 ‪ 𝜆 ، ⋯ ، 𝜆 ،‬ﻫﻲ اﳉﺬور اﳊﻘﻴﻘﻴﺔ )ﳐﺘﻠﻔﺔ ﻣﺜﻨﻰ ﻣﺜﻨﻰ( ﻟـِ ‪ P‬و اﻟﺘﻲ ُرﺗﺒﻬﺎ ﻫﻲ 𝛼 ‪ 𝛼 ، ⋯ ، 𝛼 ،‬ﻋﲆ‬ ‫•‬

‫اﻟﱰﺗﻴﺐ ؛‬
‫ﻛﺜﲑات اﳊﺪود ‪ X + b X + c‬ﳐﺘﻠﻔﺔ ﻣﺜﻨﻰ ﻣﺜﻨﻰ و ﻏﲑ ﻗﺎﺑﻠﺔ ﻟﻠﺘﺤﻠﻴﻞ ﻋﲆ ]‪ ℝ[X‬ﻟﻜﻞ }‪. j ∈ {1, 2, ⋯ , s‬‬ ‫•‬

‫‪ ’A–G Theorem9‬ﺑﺎﻹﻧﺠﻠﻴﺰﻳﺔ ؛ ‪ Théorème de ’A–G‬ﺑﺎﻟﻔﺮﻧﺴﻴﺔ‪.‬‬

‫𝟱𝟴𝟯‬

‫‪http ://tinyurl.com/Malki1718‬‬ ‫‪0‬‬


‫ا‪ .5.‬ادلوال ﻛﺜﲑات اﳊﺪود و اﳉﺬور‬
‫‪.‬‬
‫ﲢﻠﻴﻞ ‪ X + 16‬ﻋﲆ ]‪ ℂ[X‬ﻫﻮ ‪:‬‬ ‫ﻣﺜﺎﻝ ‪: 40‬‬
‫‪X + 16 = X −‬‬ ‫‪2e−‬‬ ‫‪/‬‬ ‫‪X − 2e‬‬ ‫‪/‬‬ ‫‪X−‬‬ ‫‪2e−‬‬ ‫‪/‬‬ ‫‪X − 2e‬‬ ‫‪/‬‬

‫‪Ŕž Ŧ Ľ‬‬
‫و ﻧﺤﺼﻞ ﻋﻠﻴﻪ ﺑﺈﳚﺎد اﳉﺬور اﻟﺮاﺑﻌﺔ ﻟﻠﻌﺪد ‪ −16‬ﰲ ‪. ℂ‬‬
‫أ ّﻣﺎ ﰲ ]‪ ℝ[X‬ﻓﻨﺤﺼﻞ ﻋﻠﻴﻪ ﺑﺄﺧﺬ ﻛﻞ ﺣﺪّ ﻳﻦ ﻣﱰاﻓﻘﲔ ﰲ اﻟﺘﺤﻠﻴﻞ اﻟﺴﺎﺑﻖ ‪:‬‬

‫‪œǃ‬‬
‫𝜋‬
‫‪X − 2e−‬‬ ‫‪/‬‬ ‫‪X − 2e‬‬ ‫‪/‬‬ ‫‪= X − 2 e−‬‬ ‫‪/‬‬ ‫‪+e‬‬ ‫‪/‬‬ ‫‪X + 4 = X − 4X cos‬‬ ‫‪+4‬‬
‫‪4‬‬
‫‪= X − 2√2X + 4‬‬

‫‪Ŀ‬‬
‫𝜋‪3‬‬
‫‪X − 2e−‬‬ ‫‪/‬‬ ‫‪X − 2e‬‬ ‫‪/‬‬ ‫‪= X − 2 e−‬‬ ‫‪/‬‬ ‫‪+e‬‬ ‫‪/‬‬ ‫‪X + 4 = X − 4X cos‬‬ ‫‪+4‬‬ ‫و‬
‫‪4‬‬
‫‪= X + 2√2X + 4‬‬

‫‪ X + 16 = X − 2√2X + 4‬و ﻫﻮ ﲢﻠﻴﻞ ﻋﲆ ]‪ ℝ[X‬؛‬ ‫‪X + 2√2X + 4‬‬ ‫ﻣﻨﻪ‬


‫أو ﺑﻄﺮﻳﻘﺔ إﻛﲈل اﳌﺮﺑﻊ ‪:‬‬

‫‪X + 16 = X‬‬ ‫‪+ 4 + 2 × 4X − 2 × 4X = X + 4‬‬ ‫‪− 2√2X‬‬

‫‪= X + 4 − 2√2X‬‬ ‫‪X + 4 + 2√2X‬‬


‫‪ņőƱŀ Ţ‬‬
‫‪ũŏ‬‬ ‫‪ƕ‬‬

‫𝟲𝟴𝟯‬

‫‪http ://tinyurl.com/Malki1718‬‬ ‫‪0‬‬


‫‪.‬‬

‫‪Ŕž Ŧ Ľ‬‬
‫‪œǃ‬‬
‫‪otttttttttttttttttttttttttttttttttttttttttttp‬‬

‫ﻣﻠﺤﻖ ب‬
‫‪s‬‬ ‫‪u‬‬
‫‪s‬‬ ‫اﻟﻤﻌﺎدﻻت ﻣﻦ اﻟﺪرﺟﺔ اﻟﺜﺎﻟﺜﺔ و اﻟﺮاﺑﻌﺔ‬
‫‪u‬‬

‫‪Ŀ‬‬
‫‪s‬‬ ‫‪u‬‬
‫‪qvvvvvvvvvvvvvvvvvvvvvvvvvvvvvvvvvvvvvvvvvvvr‬‬
‫‪.‬‬

‫ﺍﳌﻌﺎﺩﻻﺕ ﺍﻟﺘﻜﻌﻴﺒﻴﺔ‬ ‫ﺏ‪1.‬‬

‫ﻣﻘﺪﻣﺔ ﺗﺎﺭﳜﻴﺔ‬ ‫ﺏ‪1.1.‬‬

‫ﺣﻞ اﳌﻌﺎدﻻت اﻟﺘﻜﻌﻴﺒﻴﺔ و ﻇﻬﺮت ُﻃﺮق ﻫﻨﺪﺳﻴﺔ )ﻧﻘﻂ ﺗﻘﺎﻃﻊ ﻗﻄﻮع ﳐﺮوﻃﻴﺔ( ﳊﻠﻬﺎ‪ ،‬ﺧﺎﺻﺔ‬ ‫إﻫﺘﻢ اﻟﻌﻠﲈء ﻣﻨﺬ ِ‬
‫اﻟﻘﺪم ﺑﻤﺴﺄﻟﺔ ّ‬ ‫ّ‬
‫‪1‬‬
‫ﻋﲆ أﻳﺪي ﻋﻠﲈء ﻣﺴﻠﻤﲔ أﻣﺜﺎل اﳌﻬﻨﻲ‪ ،‬إﺑﻦ اﳍﻴﺜﻢ‪ ،‬اﻟﺒﲑوﲏ‪ ،‬ﻋﻤﺮ ﺧﻴﺎم‪ ،‬ﴍف اﻟﺪﻳﻦ اﻟﻄﻮﳼ‪ ،‬اﻟﻜﺎﳾ و ﻏﲑﻫﻢ ؛ و ﻗﺪ ﻛﺘﺐ‬
‫اﳋﻴﺎم أﻧﻪ ﺣﺎول إﳚﺎد ﻃﺮﻳﻘﺔ ﺟﱪﻳﺔ ﻟﻜﻨﻪ أﺧﻔﻖ و أﻧﻪ ﻳﺄﻣﻞ أن ﻳﺘﻤﻜﻦ أﺣﺪ اﻟﻌﻠﲈء ﻣﻦ ﺑﻌﺪه ﻣﻦ إﳚﺎدﻫﺎ‪ ،‬و ﻫﻮ ﻣﺎ ﲢﻘﻖ ﻓﻌﻼً‪.‬‬
‫ﻓﻘﺪ ﲤﻜﻦ ﻋﻠﲈء إﻳﻄﺎﻟﻴﻮن ﰲ ﻋﴫ اﻟﻨﻬﻀﺔ اﻷورﺑﻴﺔ ﻣﻦ إﳚﺎد ﺻﻴﻎ ﻋﺎﻣﺔ ﳊﻠﻮل اﳌﻌﺎدﻻت ﻣﻦ اﻟﺪرﺟﺔ اﻟﺜﺎﻟﺜﺔ‪ ،‬و ﺑﻌﺪﻫﺎ ﺑﻘﻠﻴﻞ‬
‫رﻣﻮزا ﲣﻴﻠﻴﺔ ﻛﲈ أﴍﻧﺎ إﻟﻴﻪ ﰲ ﻣﺪﺧﻞ اﻟﺒﺎب اﻷول ﻣﻦ ﻫﺬا اﻟﻜﺘﺎب‪ ،‬و‬
‫ً‬ ‫ﺻﻴﻎ ﳊﻠﻮل اﳌﻌﺎدﻻت ﻣﻦ اﻟﺪرﺟﺔ اﻟﺮاﺑﻌﺔ ﻣﺴﺘﻌﻤﻠﲔ‬
‫ﻛﺎن ذﻟﻚ أﻛﱪ ﺗﻘﺪّ م ﰲ اﻟﺮﻳﺎﺿﻴﺎت ﺑﻌﺪ اﻟﻌﺼﻮر اﻟﻮﺳﻄﻰ‪.‬‬
‫و ﻋﲆ ﺧﻼف اﳌﻌﺎدﻻت اﻟﱰﺑﻴﻌﻴﺔ اﻟﺘﻲ ﺗﻄﻠﺒﺖ إدﺧﺎل رﻣﻮز ﲣﻴﻠﻴﺔ ﳊﻠﻬﺎ ﺑﺼﻔﺔ ﻛﺎﻣﻠﺔ ﺑﺈﻧﺸﺎء ﳎﻤﻮﻋﺔ اﻷﻋﺪاد اﳌﺮﻛﺒﺔ‪ ،‬ﻓﺎﳌﻌﺎدﻻت‬
‫‪ņőƱŀ Ţ‬‬
‫ﻣﻦ اﻟﺪرﺟﺔ اﻟﺜﺎﻟﺜﺔ و اﻟﺮاﺑﻌﺔ ﺗﻘﺒﻞ ﻛﻞ ﺣﻠﻮﳍﺎ ﰲ ‪ ،ℂ‬ﺑﻞ ﻛﻞ ﻛﺜﲑ ﺣﺪود و ﻣﻬﲈ ﻛﺎﻧﺖ درﺟﺘﻪ ﻳﻘﺒﻞ ﻛﻞ ﺟﺬوره ﰲ ‪ ℂ‬ﻛﲈ ﺗﻨﺺ‬
‫ﻋﻠﻴﻪ اﻟﻨﻈﺮﻳﺔ اﻷﺳﺎﺳﻴﺔ ﰲ اﳉﱪ‪.‬‬

‫ﺍﻟﻄﺮﻳﻘﺔ ﺍﻷﻭﱃ ‪ :‬ﻃﺮﻳﻘﺔ ﺩﺍﻝ ﻓﲑﻭ ﴿‪﴾D F‬‬ ‫ﺏ‪2.1.‬‬

‫ﻟﺘﻜﻦ ‪ a x + a x + a x + a = 0‬ﻣﻌﺎدﻟﺔ ﻣﻦ اﻟﺪرﺟﺔ اﻟﺜﺎﻟﺜﺔ ﺣﻴﺚ ‪ a , a , a , a ∈ ℝ‬و ‪. a ≠ 0‬‬


‫ﺑﻘﺴﻤﺔ َ ْ‬
‫ﻃﺮﰲ اﳌﻌﺎدﻟﺔ ﻋﲆ ‪ a‬ﻫﺬه اﻷﺧﲑة ﺗُﺼﺒﺢ ﻣﻦ اﻟﺸﻜﻞ ‪. x + ax + bx + c = 0‬‬
‫ﺑﺪاﻳ ًﺔ ﻧﺘﺨﻠﺺ ﻣﻦ اﳊﺪ ‪ ax‬و ذﻟﻚ ﺑﺈﺟﺮاء »إﻧﺴﺤﺎب«ﻋﲆ اﻟﺸﻜﻞ ‪ x = y + h‬ﺣﻴﺚ ‪ h‬ﻋﺪد ﳚﺐ ﺗﻌﻴﻴﻨﻪ ﺣﺘﻰ ﻳﻨﻌﺪم ﻣﻌﺎﻣﻞ‬
‫‪ƕ‬‬

‫اﳊﺪ ‪ . y‬ﻟﺪﻳﻨﺎ ‪:‬‬


‫‪x + ax + bx + c = 0 ⟺ (y + h) + a (y + h) + b (y + h) + c = 0‬‬
‫‪ũŏ‬‬

‫‪⟺ y + 3hy + 3h y + h + a y + 2hy + h‬‬ ‫‪+ by + bh + c = 0‬‬

‫‪⟺ y + (3h + a) y + 3h + 2ah + b y + h + ah + bh + c = 0‬‬

‫‪+‬‬
‫=‬ ‫= ذات اﳌﺠﻬﻮل ﻛﲈ ﻳﲇ ‪ :‬ﺑﺪاﻳﺔ‪ ،‬ﻳﺄﺧﺬ‬ ‫أي‬ ‫‪+‬‬ ‫=‬ ‫‪1‬ﳍﺬا اﻟﻌﺎﱂ ﻃﺮﻳﻘﺔ ﻋﺪدﻳﺔ راﺋﻌﺔ ﻹﳚﺎد ﻗﻴﻤﺔ ﺗﻘﺮﻳﺒﻴﺔ ﳊﻞ اﳌﻌﺎدﻟﺔ‬
‫‪+‬‬ ‫‪+‬‬ ‫‪+‬‬
‫‪ ،‬إﻟﺦ‪.‬‬ ‫=‬ ‫‪−‬‬
‫ﺛﻢ‬
‫= ‪ّ ، ... ،‬‬ ‫ﺛﻢ‬
‫ّ‬ ‫=‬ ‫ﺛﻢ‬
‫ﻛﻘﻴﻤﺔ ﺗﻘﺮﻳﺒﻴﺔ ﻟﻠﺤﻞ‪ّ ،‬‬
‫ﻳﻤﻜﻦ أن ﻧﺜﺒﺖ ّ‬
‫أن ﻫﺬه اﻟﻄﺮﻳﻘﺔ ﺗﺘﻘﺎرب ﺑﴪﻋﺔ ﻧﺤﻮ اﳊﻞ‪.‬‬

‫𝟳𝟴𝟯‬

‫‪http ://tinyurl.com/Malki1718‬‬ ‫‪0‬‬


‫ب‪ .1.‬اﳌﻌﺎدﻻت اﻟﺘﻜﻌﻴﺒﻴﺔ‬
‫‪.‬‬
‫‪x3 − 3x − 1‬‬ ‫‪x3 − 3x − 2‬‬ ‫‪x3 − 2x + 3‬‬
‫‪4‬‬ ‫‪4‬‬ ‫‪4‬‬

‫‪Ŕž Ŧ Ľ‬‬
‫‪3‬‬ ‫‪3‬‬ ‫‪3‬‬

‫‪œǃ‬‬
‫‪2‬‬ ‫‪2‬‬ ‫‪2‬‬

‫‪1‬‬ ‫‪1‬‬ ‫‪1‬‬

‫‪Ŀ‬‬
‫‪−3‬‬ ‫‪−2‬‬ ‫‪−1‬‬ ‫‪1‬‬ ‫‪2‬‬ ‫‪−3‬‬ ‫‪−2‬‬ ‫‪−1‬‬ ‫‪1‬‬ ‫‪2‬‬ ‫‪−3‬‬ ‫‪−2‬‬ ‫‪−1‬‬ ‫‪1‬‬ ‫‪2‬‬
‫‪−1‬‬ ‫‪−1‬‬ ‫‪−1‬‬

‫‪−2‬‬ ‫‪−2‬‬ ‫‪−2‬‬

‫‪−3‬‬ ‫‪−3‬‬ ‫‪−3‬‬

‫‪−4‬‬ ‫‪−4‬‬ ‫‪−4‬‬

‫‪−5‬‬ ‫‪−5‬‬ ‫‪−5‬‬

‫ﺷﻜﻞ ب‪ :3.‬ﺛﻼﺛﺔ ﺟﺬور‬ ‫ﺷﻜﻞ ب‪ :2.‬ﺟﺬران ﺣﻘﻴﻘﻴﺎن‬ ‫ﺷﻜﻞ ب‪ :1.‬ﺟﺬر ﺣﻘﻴﻘﻲ واﺣﺪ‬

‫‪a‬‬
‫إذن ﳚﺐ أن ﻳﻜﻮن ‪ 3h + a = 0‬أي ‪. h = −‬‬
‫‪3‬‬
‫ﺑﻌﺪ ﻫﺬا‪ ،‬ﺗُﺼﺒﺢ اﳌﻌﺎدﻟﺔ ﻣﻦ اﻟﺸﻜﻞ ‪. X + pX + q = 0 :‬‬
‫‪ lim‬و ﺑﲈ ّ‬
‫أن اﻟﺪاﻟﺔ ‪X ⟼ X + pX + q‬‬ ‫‪X + pX + q‬‬ ‫‪ lim‬و ∞‪= +‬‬ ‫‪X + pX + q‬‬ ‫ﺑﲈ ّ‬
‫أن ∞‪= −‬‬
‫∞‪→+‬‬ ‫∞‪→−‬‬
‫ﻼ ﺣﻘﻴﻘﻴﺎ ﻋﲆ اﻷﻗﻞ‪.‬‬ ‫ﻣﺴﺘﻤﺮة )ﻣﺘﺼﻠﺔ( ﻋﲆ ‪ّ ℝ‬‬
‫ﻓﺈن اﳌﻌﺎدﻟﺔ ‪ X + pX + q = 0‬ﺗﻘﺒﻞ ﺣ ً‬
‫ﻼ ﺣﻘﻴﻘﻴﺎ واﺣﺪا‪ ،‬ﺣ ّﻠﲔ ﺣﻘﻴﻘﻴﲔ‬‫ﻓﺈن ﻫﺬه اﳌﻌﺎدﻟﺔ ﻳﻤﻜﻦ أن ﺗﻘﺒﻞ ﺣ ً‬‫ﰲ اﳊﻘﻴﻘﺔ‪ ،‬و ﻛﲈ ﺗﺒ ﱢﻴﻨﻪ اﻷﺷﻜﺎل ب‪ ، 1.‬ب‪ 2.‬و ب‪ّ 3.‬‬
‫)أﺣﺪﳘﺎ ﻣﻀﺎﻋﻒ( أو ﺛﻼﺛﺔ ﺣﻠﻮل ﺣﻘﻴﻘﻴﺔ‪.‬‬
‫ﳊﻞ اﳌﻌﺎدﻟﺔ ﻧﻜﺘﺐ ‪ x‬ﻋﲆ ﺷﻜﻞ ﳎﻤﻮع ﻋﺪدﻳﻦ ‪ x = u + v :‬و اﳌﻌﺎدﻟﺔ اﻟﺘﻜﻌﻴﺒﻴﺔ ﺗُﺼﺒﺢ ‪:‬‬‫اﻵن‪ّ ،‬‬
‫‪(u + v) + p (u + v) + q = 0‬‬
‫‪ņőƱŀ Ţ‬‬
‫و ﻳﻤﻜﻦ إﻋﺎدة ﻛﺘﺎﺑﺘﻬﺎ ﻋﲆ اﻟﺸﻜﻞ ‪:‬‬
‫‪u +v‬‬ ‫)‪+ 3uv (u + v) = −q − p (u + v‬‬
‫ﺑﻄﺒﻴﻌﺔ اﳊﺎل‪ ،‬ﻳﻮﺟﺪ ﻋﺪد ﻻﳖﺎﺋﻲ ﻣﻦ اﻷزواج ‪ (u, v) ∈ ℝ‬و اﻟﺘﻲ ﲢﻘﻖ ﻫﺬه اﳌﻌﺎدﻟﺔ‪ ،‬ﻟﻜﻦ إذا أﺿﻔﻨﺎ اﻟﴩﻃﲔ ‪:‬‬
‫‪ u + v = −q‬و ‪3uv = −p‬‬
‫= ‪ (v‬و ﻫﻲ ﻣﻌﺎدﻟﺔ ﺗﺮﺑﻴﻌﻴﺔ ﰲ اﳌﺠﻬﻮل ‪.t = u‬‬
‫‪−p‬‬
‫أن ‪ u + −p = −q‬أي ‪ّ u + qu − p = 0‬‬
‫)ﻷن‬ ‫ﻓﺈﻧﻨﺎ ﻧﺠﺪ ّ‬
‫‪3u‬‬ ‫‪27‬‬ ‫‪3u‬‬
‫‪−q ± q + p‬‬ ‫‪p‬‬
‫= ‪ t‬ﻣﻨﻪ ‪:‬‬ ‫‪ t + qt −‬ﺣﻼن ﰲ ‪ ℂ‬و ﳘﺎ‬ ‫ﻟﻠﻤﻌﺎدﻟﺔ ‪= 0‬‬
‫‪2‬‬ ‫‪27‬‬
‫‪ƕ‬‬

‫‪−q − q + p‬‬ ‫‪−q + q + p‬‬ ‫‪−q − q + p‬‬


‫‪ v = −q − u = −q −‬؛‬ ‫=‬ ‫= ‪ u‬و ﺑﺎﻟﺘﺎﱄ‬
‫‪2‬‬ ‫‪2‬‬ ‫‪2‬‬
‫‪−q + q + p‬‬ ‫‪−q − q + p‬‬ ‫‪−q + q + p‬‬
‫‪.v = −q − u = −q −‬‬ ‫=‬ ‫= ‪ u‬و ﺑﺎﻟﺘﺎﱄ‬ ‫أو‬
‫‪2‬‬ ‫‪2‬‬ ‫‪2‬‬
‫‪ũŏ‬‬

‫‪−q −‬‬ ‫‪q +‬‬ ‫‪p‬‬


‫= ‪ّ u‬‬
‫ﻓﺈن ‪:‬‬ ‫إذا ﻛﺎن‬ ‫•‬
‫‪2‬‬

‫𝟴𝟴𝟯‬

‫‪http ://tinyurl.com/Malki1718‬‬ ‫‪0‬‬


‫ب‪ .‬اﳌﻌﺎدﻻت ﻣﻦ ادلرﺟﺔ اﻟﺜﺎﻟﺜﺔ و اﻟﺮاﺑﻌﺔ‬
‫‪.‬‬
‫‪−q +‬‬ ‫‪q +‬‬ ‫‪p‬‬
‫=‪u‬‬ ‫𝛼=‬
‫‪2‬‬

‫‪Ŕž Ŧ Ľ‬‬
‫‪j = e‬و‪.j = 1‬‬ ‫‪/‬‬ ‫ﺣﻴﺚ‬ ‫‪−q +‬‬ ‫‪q +‬‬ ‫‪p‬‬
‫‪u=j‬‬ ‫𝛼‪= j‬‬ ‫أو‬
‫‪2‬‬

‫‪œǃ‬‬
‫‪−q +‬‬ ‫‪q +‬‬ ‫‪p‬‬
‫‪u=j‬‬ ‫𝛼 ‪=j‬‬ ‫أو‬
‫‪2‬‬

‫‪Ŀ‬‬
‫‪−q +‬‬ ‫‪q +‬‬ ‫‪p‬‬ ‫‪−q +‬‬ ‫‪q +‬‬ ‫‪p‬‬
‫=‪ّ v‬‬
‫)ﻷن ‪.(3uv = −p‬‬ ‫=‪ّ u‬‬
‫ﻓﺈن 𝛽 =‬ ‫➛ إذا ﻛﺎن‬
‫‪2‬‬ ‫‪2‬‬
‫‪−q +‬‬ ‫‪q +‬‬ ‫‪p‬‬ ‫‪−q +‬‬ ‫‪q +‬‬ ‫‪p‬‬
‫‪ّ v=j‬‬
‫)ﻷن ‪ 3uv = −p‬أي‬ ‫‪ّ u=j‬‬
‫ﻓﺈن 𝛽 ‪= j‬‬ ‫➛ إذا ﻛﺎن‬
‫‪2‬‬ ‫‪2‬‬
‫أن اﳉﺪاء ‪ 3uv‬ﳚﺐ أن ﻳﻜﻮن ﺣﻘﻴﻘﻴﺎ و ﻳﺴﺎوي ‪ −p‬و ّ‬
‫ﻷن ‪.( j × j = 1‬‬ ‫ّ‬
‫‪−q +‬‬ ‫‪q +‬‬ ‫‪p‬‬ ‫‪−q +‬‬ ‫‪q +‬‬ ‫‪p‬‬
‫‪ّ v=j‬‬
‫)ﻷن ‪ 3uv = −p‬أي‬ ‫‪ّ u=j‬‬
‫ﻓﺈن 𝛽‪= j‬‬ ‫➛ إذا ﻛﺎن‬
‫‪2‬‬ ‫‪2‬‬
‫أن اﳉﺪاء ‪ 3uv‬ﳚﺐ أن ﻳﻜﻮن ﺣﻘﻴﻘﻴﺎ و ﻳﺴﺎوي ‪ −p‬و ّ‬
‫ﻷن ‪.( j × j = 1‬‬ ‫ّ‬

‫‪−q +‬‬ ‫‪q +‬‬ ‫‪p‬‬


‫= ‪ّ u‬‬
‫ﻓﺈن ‪:‬‬ ‫إذا ﻛﺎن‬ ‫•‬
‫‪2‬‬
‫‪−q +‬‬ ‫‪q +‬‬ ‫‪p‬‬
‫𝛼 = ‪v‬؛‬ ‫ﻣﻨﻪ‬ ‫=‪u‬‬ ‫𝛽=‬
‫‪2‬‬
‫‪−q +‬‬ ‫‪q +‬‬ ‫‪p‬‬
‫ﻣﻨﻪ 𝛼 ‪ v = j‬؛‬ ‫‪u=j‬‬ ‫𝛽‪= j‬‬ ‫أو‬
‫‪2‬‬
‫‪−q +‬‬ ‫‪q +‬‬ ‫‪p‬‬
‫𝛼‪. v = j‬‬ ‫‪ u = j‬ﻣﻨﻪ‬ ‫𝛽 ‪=j‬‬ ‫أو‬
‫‪2‬‬
‫‪ņőƱŀ Ţ‬‬
‫أن ‪ّ x = u + v‬‬
‫ﻓﺈن 𝛽 ‪ x = 𝛼 +‬أو 𝛽 ‪ x = j𝛼 + j‬أو 𝛽‪ x = j 𝛼 + j‬أو ﺑﺘﻌﺒﲑ آﺧﺮ ‪:‬‬ ‫و ﺑﲈ ّ‬

‫‪−q −‬‬ ‫‪q + 4p /27‬‬ ‫‪−q +‬‬ ‫‪q + 4p /27‬‬


‫=‪x‬‬ ‫‪+‬‬
‫‪2‬‬ ‫‪2‬‬

‫‪−1 + ı√3‬‬ ‫‪−q −‬‬ ‫‪q + 4p /27‬‬ ‫‪−1 − ı√3‬‬ ‫‪−q +‬‬ ‫‪q + 4p /27‬‬
‫=‪x‬‬ ‫‪+‬‬ ‫أو‬
‫‪2‬‬ ‫‪2‬‬ ‫‪2‬‬ ‫‪2‬‬

‫‪−q −‬‬ ‫‪q + 4p /27‬‬ ‫‪−q +‬‬ ‫‪q + 4p /27‬‬


‫‪ƕ‬‬

‫‪−1 − ı√3‬‬ ‫‪−1 + ı√3‬‬


‫=‪x‬‬ ‫‪+‬‬ ‫أو‬
‫‪2‬‬ ‫‪2‬‬ ‫‪2‬‬ ‫‪2‬‬

‫ﻋﻨﺪ اﻟﻨﻈﺮ إﱃ ﻫﺬه اﻟﺼﻴﻎ‪ ،‬ﻳﺒﺪو ّ‬


‫أن اﳊﻞ اﻷول ﺣﻘﻴﻘﻲ و اﳊﻠﲔ اﻵﺧﺮﻳﻦ ﻣﺮﻛﺒﲔ ﻟﻜﻦ اﳌﻌﺎدﻟﺔ ‪) x − 3x − 1‬ﺷﻜﻞ ب‪(3.‬‬
‫‪ũŏ‬‬

‫ﺗﻘﺒﻞ ﺛﻼﺛﺔ ﺣﻠﻮل ﺣﻘﻴﻘﻴﺔ؟! ﻛﻴﻒ ذﻟﻚ ؟‬


‫‪4‬‬
‫اﳉﻮاب ﻧﺠﺪه ﻋﻨﺪ اﻟﺘﻤﻌﻦ ﰲ اﻟﻌﺒﺎرة ‪ . Δ = q + p :‬ﻛﲈ ﻫﻮ اﳊﺎل ﺑﺎﻟﻨﺴﺒﺔ ﻟﻠﻤﻌﺎدﻻت اﻟﱰﺑﻴﻌﻴﺔ‪ ،‬ﻫﺬا اﻟﻌﺪد ُﻳﺴﻤﻰ‬
‫‪27‬‬
‫ﳑ ﱢﻴﺰ اﳌﻌﺎدﻟﺔ و ﻫﻮ ﻋﺪد ﺣﻘﻴﻘﻲ ﻳﻤﻜﻦ أن ﻳﻜﻮن ﺳﺎﻟﺒ ًﺎ و ﺑﺎﻟﺘﺎﱄ ﻓﺎﻟﻌﺪد ‪ Δ‬ﻳﻤﻜﻦ أن ﻳﻜﻮن ﻣﺮﻛﺒ ًﺎ و ﻛﺬﻟﻚ اﳊﺎل ﺑﺎﻟﻨﺴﺒﺔ ﻟﻠﺠﺬور‬
‫أن ‪: (j + j = −1 :‬‬ ‫أدق‪ ،‬ﻟﺪﻳﻨﺎ )ﻧُﺬﻛﱢﺮ ّ‬
‫اﻟﺘﻜﻌﻴﺒﻴﺔ‪ .‬ﺑﺘﻌﺒﲑ ّ‬

‫𝟵𝟴𝟯‬

‫‪http ://tinyurl.com/Malki1718‬‬ ‫‪0‬‬


‫ب‪ .1.‬اﳌﻌﺎدﻻت اﻟﺘﻜﻌﻴﺒﻴﺔ‬
‫‪.‬‬

‫اﳊﻞ اﻟﺜﺎﻟﺚ‬ ‫اﳊﻞ اﻟﺜﺎﲏ‬ ‫اﳊﻞ اﻷول‬

‫‪Ŕž Ŧ Ľ‬‬
‫ﻣﺮﻛﺐ‬ ‫ﻣﺮﻛﺐ‬ ‫ﺣﻘﻴﻘﻲ‬ ‫‪Δ>0‬‬

‫‪œǃ‬‬
‫ﺣﻘﻴﻘﻴﺎن و ﻣﺘﺴﺎوﻳﺎن‬ ‫ﺣﻘﻴﻘﻲ‬ ‫‪Δ=0‬‬

‫ﺣﻘﻴﻘﻲ‬ ‫ﺣﻘﻴﻘﻲ‬ ‫ﺣﻘﻴﻘﻲ‬ ‫‪Δ<0‬‬

‫‪Ŀ‬‬
‫أن ﻣﻌﺎﻣﻼت اﳌﻌﺎدﻟﺔ ﺣﻘﻴﻘﻴﺔ‪ّ ،‬‬
‫ﻓﺈن اﳊﻠﲔ اﳌﺮﻛﺒﲔ )ﻋﻨﺪﻣﺎ ‪ (Δ > 0‬ﻣﱰاﻓﻘﺎن‪.‬‬ ‫ﺑﲈ ّ‬

‫ﻣﺎ ﻣﺪﻯ ﺃﳘﻴﺔ ﻫﺬﻩ ﺍﻟﺼﻴﻎ ؟‬


‫إذا ﻛﺎﻧﺖ ﻫﺬه اﻟﺼﻴﻎ ﻣﻬﻤﺔ و ﻣﺴﺎﻋﺪة‪ِ ،‬‬
‫ﻓﻠ َﻢ ﻻ ﺗُﺴﺘﺨﺪم ﻛﺜﲑ ًا ﻛﲈ ﻫﻮ اﳊﺎل ﺑﺎﻟﻨﺴﺒﺔ إﱃ اﳌﻌﺎدﻻت اﻟﱰﺑﻴﻌﻴﺔ ؟‬
‫ﻟﻺﺟﺎﺑﺔ ﻋﲆ ﻫﺬا اﻟﺴﺆال‪ ،‬ﻧﺪرس ﺑﻌﺾ اﻷﻣﺜﻠﺔ ‪:‬‬
‫ﻣﺜﺎﻝ ‪ : 41‬ﻧﻌﺘﱪ اﳌﻌﺎدﻟﺔ ‪:‬‬
‫‪x + 6x − 2 = 0‬‬
‫أﺣﺪ اﳊﻠﻮل‪ ،‬ﺣﺴﺐ اﻟﺼﻴﻎ اﻟﺴﺎﺑﻘﺔ ﻫﻮ ‪:‬‬

‫‪2 − √4 + 32‬‬ ‫‪2 + √4 + 32‬‬


‫=‪x‬‬ ‫‪+‬‬ ‫‪= √−2 + √4 = √4 − √2‬‬
‫‪2‬‬ ‫‪2‬‬
‫أن ﻫﺬا اﻟﻌﺪد )ﻓﺮق ﺟﺬرﻳﻦ ﺗﻜﻌﻴﺒﻴﲔ( ّ‬
‫ﺣﻞ ﻟﺘﻠﻚ اﳌﻌﺎدﻟﺔ اﻟﺒﺴﻴﻄﺔ !‬ ‫و ﻫﻲ ﺑﺪون ﺷﻚ ﻧﺘﻴﺠﺔ ُﻣﺮﺿﻴﺔ و ﻛﺎن ﻣﻦ اﳌﺴﺘﺤﻴﻞ ﻇ ّﻦ ّ‬
‫ﻣﺜﺎﻝ ‪ : 42‬ﻧﻌﺘﱪ اﳌﻌﺎدﻟﺔ ‪:‬‬
‫‪x + 3x − 4 = 0‬‬
‫أﺣﺪ اﳊﻠﻮل‪ ،‬ﺣﺴﺐ اﻟﺼﻴﻎ اﻟﺴﺎﺑﻘﺔ ﻫﻮ ‪:‬‬
‫‪ņőƱŀ Ţ‬‬
‫‪4 − √16 + 4‬‬ ‫‪2 + √16 + 4‬‬
‫=‪x‬‬ ‫‪+‬‬ ‫=‬ ‫‪2 − √5 +‬‬ ‫‪2 + √5‬‬
‫‪2‬‬ ‫‪2‬‬

‫و ﻫﻲ ﻧﺘﻴﺠﺔ ﻻ ﺑﺄس ﲠﺎ أﻳﻀ ًﺎ‪.‬‬


‫أن ‪ 2 − √5 + 2 + √5 = 1‬و ﺑﺎﻟﻔﻌﻞ‪ ،‬ﻋﻨﺪ‬ ‫اﳊﻞ‪ ،‬ﺳﻮف ﻧﻼﺣﻆ ّ‬ ‫ﻟﻜﻦ إذا اﺳﺘﻌﻤﻠﻨﺎ آﻟﺔ ﺣﺎﺳﺒﺔ ﻹﳚﺎد ﻗﻴﻤﺔ ﺗﻘﺮﻳﺒﻴﺔ ﳍﺬا ّ‬
‫أن ‪ Δ = 20 > 0‬ﻓﺤﺴﺐ ﻣﺎ ﺳﺒﻖ‪ x = 1 ،‬ﻫﻮ اﳊﻞ اﳊﻘﻴﻘﻲ اﻟﻮﺣﻴﺪ‬ ‫أن ‪ّ x = 1‬‬
‫ﺣﻞ ﻟﻠﻤﻌﺎدﻟﺔ اﻟﺴﺎﺑﻘﺔ و ﺑﲈ ّ‬ ‫اﻟﺘﻌﻮﻳﺾ ﻧﺠﺪ ّ‬
‫ﻟﻠﻤﻌﺎدﻟﺔ إذن ‪. 2 − √5 + 2 + √5 = 1‬‬
‫ﻫﺬا ُﻳﱪز أﺣﺪ ﻧﻘﺎﺋﺺ ﻫﺬه اﻟﺼﻴﻎ ‪ :‬ﺑﺎﻟﻨﺴﺒﺔ ﻟﻠﻤﻌﺎدﻻت اﻟﱰﺑﻴﻌﻴﺔ‪ ،‬ﺻﻴﻐﺔ اﳊﻠﻮل ُﲣﱪﻧﺎ داﺋ ًﲈ ﻋﻦ ﻃﺒﻴﻌﺘﻬﺎ )ﺣﻠﻮل ﻧﺎﻃﻘﺔ أو‬
‫‪ƕ‬‬

‫ﺻﲈء(؛ ﻟﻜﻦ ﰲ ﻣﺜﺎﻟﻨﺎ‪ ،‬اﳊﻞ ﻧﺎﻃﻖ )ﺑﻞ ﻃﺒﻴﻌﻲ( ﻟﻜﻦ اﻟﺼﻴﻐﺔ اﻟﻌﺎﻣﺔ ﻓﺸﻠﺖ ﰲ اﻟﻜﺸﻒ ﻋﻦ ذﻟﻚ )ﻻ ﻳﻤﻜﻦ اﻟﺘﻨﺒﺆ ّﺑﺄن اﻟﻌﺪد‬ ‫ّ‬
‫‪ 2 − √5 + 2 + √5‬ﻃﺒﻴﻌﻲ إﻻّ ﺑﻌﺪ اﳊﺴﺎب( !‬
‫ﻟﻨﺴﺘﻤﺮ ﰲ اﺧﺘﺒﺎرﻧﺎ ﳌﺪى ﻧﺠﺎﻋﺔ ﻫﺬه اﻟﺼﻴﻎ‪ ،‬و ﻟﻨﻌﺘﱪ ﻣﻌﺎدﻟﺔ ﺣﻠﻮﳍﺎ ﻣﻌﺮوﻓﺔ ﻣﺴﺒﻘ ًﺎ ﺑﺤﻴﺚ ﻳﻜﻮن ﳎﻤﻮع ﻫﺬه‬ ‫ّ‬ ‫ﻣﺜﺎﻝ ‪: 43‬‬
‫‪ũŏ‬‬

‫اﳊﻠﻮل ﻣﻌﺪوﻣ ًﺎ )ﳎﻤﻮع اﳊﻠﻮل ﻳﺴﺎوي ﻣﻌﺎﻛﺲ ﻣﻌﺎﻣﻞ ‪. (x‬‬


‫ﻼ ‪ x = 2 ، x = 1‬و ‪ . x = −3‬اﳌﻌﺎدﻟﺔ ﻫﻲ إذن ‪:‬‬ ‫ﻣﺜ ً‬

‫‪(x − 1) (x − 2) (x + 3) = x − 7x + 6 = 0‬‬
‫و ﺣﺴﺐ اﻟﺼﻴﻎ اﻟﺴﺎﺑﻘﺔ‪ ،‬إﺣﺪى اﳊﻠﻮل ﻫﻲ ‪:‬‬

‫𝟬𝟵𝟯‬

‫‪http ://tinyurl.com/Malki1718‬‬ ‫‪0‬‬


‫ب‪ .‬اﳌﻌﺎدﻻت ﻣﻦ ادلرﺟﺔ اﻟﺜﺎﻟﺜﺔ و اﻟﺮاﺑﻌﺔ‬
‫‪.‬‬

‫‪Ŕž Ŧ Ľ‬‬
‫‪−6 −‬‬ ‫)‪36 − (1372/27‬‬ ‫‪−6 +‬‬ ‫)‪36 − (1372/27‬‬
‫=‪x‬‬ ‫‪+‬‬
‫‪2‬‬ ‫‪2‬‬

‫‪œǃ‬‬
‫‪100‬‬ ‫‪100‬‬
‫=‬ ‫‪−3 −‬‬ ‫‪−‬‬ ‫‪+‬‬ ‫‪−3 +‬‬ ‫‪−‬‬
‫‪27‬‬ ‫‪27‬‬

‫‪10√3‬‬ ‫‪10√3‬‬
‫=‬ ‫‪−3 −‬‬ ‫‪ı+‬‬ ‫‪−3 +‬‬ ‫‪ı‬‬

‫‪Ŀ‬‬
‫‪9‬‬ ‫‪9‬‬

‫و ﻫﻲ ﻧﺘﻴﺠﺔ ﺑﻌﻴﺪة ّ‬
‫ﻛﻞ اﻟﺒﻌﺪ ﻋﻦ اﳊﻠﻮل ‪ 2 ، −3‬و ‪! 1‬‬

‫ﰲ اﻷﺧﲑ‪ ،‬إذا ﻛﺎﻧﺖ ﺻﻴﻎ اﳊﻠﻮل ﺑﺴﻴﻄﺔ و ﺳﻬﻠﺔ ﻟﻠﺤﻔﻆ‪ ،‬ﻓﺈﳖﺎ ﻏﲑ ﻋﻤﻠﻴﺔ )ﻻ ﺗﺴﺎﻋﺪﻧﺎ ﻛﺜﲑ ًا ﰲ اﳊﺎﻻت اﻟﺘﻄﺒﻴﻘﻴﺔ( ‪ :‬ﰲ‬
‫ﱪر ﻋﺪم اﺳﺘﻌﲈﳍﺎ إﻻّ ﰲ ﺣﺎﻻت ﻗﻠﻴﻠﺔ ؛ ﻟﻜﻦ ذﻟﻚ ﻻ ﻳﻨﻔﻲ‬ ‫ُﻌﱪ ﻋﻦ اﳊﻠﻮل ﰲ أﺑﺴﻂ ﺻﻴﻐﺔ ! ﻫﺬا ﻣﺎ ُﻳ ﱢ‬ ‫ﻏﺎﻟﺐ اﻷﺣﻴﺎن ﻻ ﺗ ﱢ‬
‫أﳘﻴﺘﻬﺎ اﻟﻨﻈﺮﻳﺔ ﻓﻘﺪ ﺷﻜّﻞ ّ‬
‫ﺣﻞ اﳌﻌﺎدﻻت اﻟﺘﻜﻌﻴﺒﻴﺔ ﻗﻔﺰ ًة ﻛﺒﲑة ﰲ ﺗﺎرﻳﺦ اﻟﺮﻳﺎﺿﻴﺎت !‬

‫ﺣﻞ ﰲ ‪ ℂ‬اﳌﻌﺎدﻻت اﻟﺘﺎﻟﻴﺔ ‪:‬‬ ‫ﺗﻄﺒﻴﻖ ‪: 7‬‬

‫‪27‬‬ ‫‪27‬‬ ‫‪x + 3x + 2 = 0‬‬ ‫•‬‫‪1‬‬


‫‪x −‬‬ ‫‪x+‬‬ ‫‪=0‬‬ ‫•‬ ‫‪3‬‬
‫‪4‬‬ ‫‪4‬‬
‫‪3‬‬ ‫‪1‬‬
‫‪x − 15x − 4‬‬ ‫•‬ ‫‪4‬‬ ‫‪x − x+ =0‬‬ ‫•‬‫‪2‬‬
‫‪4‬‬ ‫‪4‬‬

‫ﺍﳊﻞّ‪.‬‬
‫‪4‬‬ ‫‪4‬‬
‫‪ Δ = q +‬ﻣﻨﻪ ‪:‬‬ ‫‪p =2 +‬‬ ‫ﻟﺪﻳﻨﺎ ‪× 3 = 8 :‬‬ ‫‪1‬‬
‫•‬
‫‪ņőƱŀ Ţ‬‬
‫‪27‬‬ ‫‪27‬‬

‫‪−2 − √8‬‬ ‫‪−2 + √8‬‬


‫=‪x‬‬ ‫‪+‬‬ ‫‪=−‬‬ ‫‪√2 + 1 +‬‬ ‫= ‪√2 − 1‬‬ ‫‪√2 − 1 −‬‬ ‫‪√2 + 1‬‬
‫‪2‬‬ ‫‪2‬‬

‫‪−2 − √8‬‬ ‫‪−2 + √8‬‬


‫‪x=j‬‬ ‫‪+j‬‬ ‫‪=j‬‬ ‫‪√2 − 1 − j‬‬ ‫‪√2 + 1‬‬
‫‪2‬‬ ‫‪2‬‬

‫‪−2 − √8‬‬ ‫‪−2 + √8‬‬


‫‪x=j‬‬ ‫‪+j‬‬ ‫‪=j‬‬ ‫‪√2 − 1 − j‬‬ ‫‪√2 + 1‬‬
‫‪2‬‬ ‫‪2‬‬
‫‪ƕ‬‬

‫‪4‬‬ ‫‪1‬‬ ‫‪4‬‬ ‫‪3‬‬


‫‪ Δ = q +‬ﻣﻨﻪ ‪:‬‬ ‫= ‪p‬‬ ‫‪+‬‬ ‫‪× −‬‬ ‫ﻟﺪﻳﻨﺎ ‪= 0 :‬‬ ‫•‬‫‪2‬‬
‫‪27‬‬ ‫‪4‬‬ ‫‪27‬‬ ‫‪4‬‬
‫‪ũŏ‬‬

‫‪− − √0‬‬ ‫‪− + √0‬‬ ‫‪1 1‬‬


‫=‪x‬‬ ‫‪+‬‬ ‫‪= − − = −1‬‬
‫‪2‬‬ ‫‪2‬‬ ‫‪2 2‬‬
‫‪− − √0‬‬ ‫‪− + √0‬‬ ‫‪1‬‬ ‫‪1‬‬ ‫‪1‬‬ ‫‪1‬‬ ‫‪1‬‬
‫‪x=j‬‬ ‫‪+j‬‬ ‫‪=− j− j =− j+j‬‬ ‫= )‪= − × (−1‬‬
‫‪2‬‬ ‫‪2‬‬ ‫‪2‬‬ ‫‪2‬‬ ‫‪2‬‬ ‫‪2‬‬ ‫‪2‬‬
‫‪− − √0‬‬ ‫‪− + √0‬‬ ‫‪1‬‬ ‫‪1‬‬ ‫‪1‬‬ ‫‪1‬‬ ‫‪1‬‬
‫‪x=j‬‬ ‫‪+j‬‬ ‫= )‪= − j − j = − j + j = − × (−1‬‬
‫‪2‬‬ ‫‪2‬‬ ‫‪2‬‬ ‫‪2‬‬ ‫‪2‬‬ ‫‪2‬‬ ‫‪2‬‬

‫𝟭𝟵𝟯‬

‫‪http ://tinyurl.com/Malki1718‬‬ ‫‪0‬‬


‫ب‪ .1.‬اﳌﻌﺎدﻻت اﻟﺘﻜﻌﻴﺒﻴﺔ‬
‫‪.‬‬
‫‪4‬‬ ‫‪27‬‬ ‫‪4‬‬ ‫‪27‬‬
‫‪ Δ = q +‬ﻣﻨﻪ ‪:‬‬ ‫= ‪p‬‬ ‫‪+‬‬ ‫‪× −‬‬ ‫ﻟﺪﻳﻨﺎ ‪= 0 :‬‬ ‫•‬ ‫‪3‬‬
‫‪27‬‬ ‫‪4‬‬ ‫‪27‬‬ ‫‪4‬‬

‫‪Ŕž Ŧ Ľ‬‬
‫‪−‬‬ ‫‪− √0‬‬ ‫‪−‬‬ ‫‪+ √0‬‬ ‫‪3 3‬‬
‫=‪x‬‬ ‫‪+‬‬ ‫‪= − − = −3‬‬

‫‪œǃ‬‬
‫‪2‬‬ ‫‪2‬‬ ‫‪2 2‬‬
‫‪−‬‬ ‫‪− √0‬‬ ‫‪−‬‬ ‫‪+ √0‬‬ ‫‪3‬‬ ‫‪3‬‬ ‫‪3‬‬ ‫‪3‬‬ ‫‪3‬‬
‫‪x=j‬‬ ‫‪+j‬‬ ‫‪=− j− j =− j+j‬‬ ‫= )‪= − × (−1‬‬
‫‪2‬‬ ‫‪2‬‬ ‫‪2‬‬ ‫‪2‬‬ ‫‪2‬‬ ‫‪2‬‬ ‫‪2‬‬

‫‪Ŀ‬‬
‫‪−‬‬ ‫‪− √0‬‬ ‫‪−‬‬ ‫‪+ √0‬‬ ‫‪3‬‬ ‫‪3‬‬ ‫‪3‬‬ ‫‪3‬‬ ‫‪3‬‬
‫‪x=j‬‬ ‫‪+j‬‬ ‫= )‪= − j − j = − j + j = − × (−1‬‬
‫‪2‬‬ ‫‪2‬‬ ‫‪2‬‬ ‫‪2‬‬ ‫‪2‬‬ ‫‪2‬‬ ‫‪2‬‬

‫‪4‬‬ ‫‪4‬‬
‫‪ . Δ = q +‬ﻧﺄﺧﺬ ‪ √Δ = 22ı‬ﻣﻨﻪ ‪:‬‬ ‫‪p = (−4) +‬‬ ‫)‪× (−15) = −484 = (22ı‬‬ ‫‪ •4‬ﻟﺪﻳﻨﺎ ‪:‬‬
‫‪27‬‬ ‫‪27‬‬

‫‪4 − 22ı‬‬ ‫‪4 + 22ı‬‬


‫=‪x‬‬ ‫‪+‬‬ ‫‪= √2 − 11ı + √2 + 11ı‬‬
‫‪2‬‬ ‫‪2‬‬
‫‪4 − 22ı‬‬ ‫‪4 + 22ı‬‬
‫‪x=j‬‬ ‫‪+j‬‬ ‫‪= j √2 − 11ı + j √2 + 11ı‬‬
‫‪2‬‬ ‫‪2‬‬
‫‪4 − 22ı‬‬ ‫‪4 + 22ı‬‬
‫‪x=j‬‬ ‫‪+j‬‬ ‫‪= j √2 − 11ı + j √2 + 11ı‬‬
‫‪2‬‬ ‫‪2‬‬

‫‪ z = (a + ıb) = a − 3ab + ı 3a b − b‬و ﺑﺎﻟﺘﺎﱄ ‪:‬‬ ‫ﻧﻀﻊ ‪ z = a + ıb‬ﺣﻴﺚ ‪ z = 2 + 11ı‬ﻣﻨﻪ‬


‫‪a − 3ab = 2‬‬
‫‪ .‬ﻧﻼﺣﻆ ّ‬
‫أن اﻟﺰوج )‪ُ (a, b) = (2, 1‬ﳛﻘﻖ ﻫﺬه اﳉﻤﻠﺔ‪ ،‬إذن ﻧﺄﺧﺬ ‪. z = 2 + ı‬‬
‫‪3a b − b = 11‬‬

‫‪ 2 − 11ı = 2 + 11ı = (2 + ı) = 2 + ı‬إذن ‪ 2 − ı‬ﺟﺬر ﺗﻜﻌﻴﺒﻲ ﻟﻠﻌﺪد‬ ‫)‪= (2 − ı‬‬ ‫ﻣﻦ ﺟﻬﺔ أﺧﺮى‪ ،‬ﻟﺪﻳﻨﺎ ‪:‬‬
‫‪ . 2 − 11ı‬ﻣﻨﻪ ‪:‬‬
‫‪x=2−ı+2+ı=4‬‬
‫‪ņőƱŀ Ţ‬‬
‫‪x = j (2 − ı) + j (2 + ı) = 2 j + j‬‬ ‫‪+ ı −j + j‬‬ ‫‪= −2 − √3‬‬

‫‪x = j (2 − ı) + j (2 + ı) = 2 j + j + ı −j + j = −2 + √3‬‬

‫■‬

‫ﺗﻄﺒﻴﻖ ‪: 8‬‬
‫أوﺟﺪ اﻟﴩط اﻟﻼزم و اﻟﻜﺎﰲ اﻟﺬي ﳚﺐ أن ُﳛﻘﻘﻪ اﻟﻌﺪدان ‪ p ≠ 0‬و ‪ q ≠ 0‬ﺣﺘﻰ ﻳﻜﻮن ﻟﻠﻤﻌﺎدﻟﺔ ‪ x + px + q = 0‬ﻋﲆ‬
‫‪ƕ‬‬

‫اﻷﻗﻞ ﺣﻠﲔ ﻃﻮﻳﻠﺘﻬﲈ ‪ . 1‬إﺳﺘﻨﺘﺞ ﺣﻠﻮل اﳌﻌﺎدﻟﺘﲔ ‪:‬‬


‫‪3 1‬‬ ‫‪x − 2x + √3 = 0‬‬ ‫•‬ ‫‪1‬‬
‫‪x +‬‬ ‫‪+ =0‬‬ ‫•‬‫‪2‬‬
‫‪4 2‬‬
‫‪ũŏ‬‬

‫أن اﻟﻌﺪدﻳﻦ ‪ −1‬و ‪ 1‬ﻻ ﻳﻤﻜﻦ أن ﻳﻜﻮﻧﺎ ﺣﻠﲔ ﻣﻌ ًﺎ )ﰲ آن واﺣﺪ( ﻟﻠﻤﻌﺎدﻟﺔ ﻷﻧﻪ ﰲ ﻫﺬه اﳊﺎﻟﺔ ﻳﻜﻮن‬ ‫ﺍﳊﻞّ‪ .‬ﺑﺪاﻳ ًﺔ‪ ،‬ﻧﻼﺣﻆ ّ‬
‫‪ 1 + p + q = 0 = −1 − p + q‬ﻣﻨﻪ ‪ q = 0‬و ﻫﺬا ُﻳﻨﺎﻗﺾ اﻟﻔﺮض ‪. q ≠ 0‬‬
‫ﻓﺈن 𝛼 ّ‬
‫ﺣﻞ ﳍﺎ أﻳﻀ ًﺎ و ﺑﺎﻟﺘﺎﱄ‪ ،‬إذا ﻛﺎن ﻟﻠﻤﻌﺎدﻟﺔ ﺣﻼن ﻃﻮﻳﻠﺘﻬﲈ‬ ‫ﻼ ﻟﻠﻤﻌﺎدﻟﺔ‪ّ ،‬‬ ‫و ﺑﲈ ّ‬
‫أن ﻣﻌﺎﻣﻼت اﳌﻌﺎدﻟﺔ ﺣﻘﻴﻘﻴﺔ‪ ،‬ﻓﺈﻧﻪ إذا ﻛﺎن 𝛼 ﺣ ً‬
‫ﻓﺈن ﻫﺬﻳﻦ اﳊﻠﲔ ﻣﱰاﻓﻘﺎن و ﻧﻜﺘﺒﻬﲈ ﻋﲆ اﻟﺸﻜﻞ ‪ e‬و ‪ e−‬ﺣﻴﺚ ‪ ، 𝜃 ∈ ℝ‬و ﰲ ﻫﺬه اﳊﺎﻟﺔ اﳌﻌﺎدﻟﺔ ﺗﻘﺒﻞ ﺣ ً‬
‫ﻼ ﺛﺎﻟﺜ ًﺎ ‪. x‬‬ ‫‪ّ 1‬‬
‫ﻣﻨﻪ ‪:‬‬

‫𝟮𝟵𝟯‬

‫‪http ://tinyurl.com/Malki1718‬‬ ‫‪0‬‬


‫ب‪ .‬اﳌﻌﺎدﻻت ﻣﻦ ادلرﺟﺔ اﻟﺜﺎﻟﺜﺔ و اﻟﺮاﺑﻌﺔ‬
‫‪.‬‬
‫إذا ﻛﺎن )𝜋‪ّ 𝜃 = 0 (mod 2‬‬
‫ﻓﺈن ‪ 1‬ﺣﻞ ﻣﻀﺎﻋﻒ ﻟﻠﻤﻌﺎدﻟﺔ‪.‬‬ ‫•‬

‫‪Ŕž Ŧ Ľ‬‬
‫إذا ﻛﺎن )𝜋‪ّ 𝜃 = 𝜋 (mod 2‬‬
‫ﻓﺈن ‪ −1‬ﺣﻞ ﻣﻀﺎﻋﻒ ﻟﻠﻤﻌﺎدﻟﺔ‪.‬‬ ‫•‬

‫ﻓﺈن ‪ e‬و ‪ e−‬ﻣﺮﻛﺒﺎن و ﻣﱰاﻓﻘﺎن ‪ ،‬و ﻟﻠﻤﻌﺎدﻟﺔ ﺣﻞ ﺛﺎﻟﺚ ‪ x‬و ﻫﻮ ﺣﻘﻴﻘﻲ‪ .‬ﻟﺪﻳﻨﺎ ‪:‬‬
‫إذا ﻛﺎن )𝜋 ‪ّ 𝜃 ≠ 0 (mod‬‬ ‫•‬

‫‪œǃ‬‬
‫‪x + e + e−‬‬ ‫‪=0‬‬
‫⎧‬
‫⎪‬ ‫‪⎪ x + 2 cos 𝜃 = 0‬‬
‫⎧‬
‫‪x e + x e− + e e−‬‬ ‫⟺ ‪=p‬‬ ‫‪2x cos 𝜃 + 1 = p‬‬
‫⎨‬
‫⎪‬ ‫⎨‬
‫⎪‬
‫‪x e e−‬‬ ‫‪x = −q‬‬

‫‪Ŀ‬‬
‫⎩‬ ‫‪= −q‬‬ ‫⎩‬

‫‪⎪ 2 cos 𝜃 = −x = q‬‬


‫⎧‬
‫⟺‬ ‫‪1−x =p‬‬
‫⎨‬
‫⎪‬
‫⎩‬ ‫‪x = −q‬‬
‫‪q‬‬
‫⎧‬
‫⎪‬
‫⎪‬ ‫= 𝜃 ‪cos‬‬
‫⎪‬ ‫‪2‬‬
‫⟺‬ ‫‪x‬‬ ‫=‬ ‫‪−q‬‬
‫⎨‬
‫⎪‬
‫⎪‬
‫‪⎪1 − q = p‬‬ ‫ﻣﻊ ]‪ q ∈ [−2, 2‬؛‬
‫⎩‬

‫‪e−‬‬ ‫و ﺑﺎﻟﺘﺎﱄ إذا ﻛﺎن ‪ 1 − q = p‬و ]‪ّ q ∈ [−2, 2‬‬


‫ﻓﺈن اﳌﻌﺎدﻟﺔ ﺗﻘﺒﻞ ﺣ ً‬
‫ﻼ ﺣﻘﻴﻘﻴﺎ ‪ x = −q‬و ﺣﻠﲔ ﻣﺮﻛﺒﲔ ﻣﱰاﻓﻘﲔ ‪ e‬و‬
‫‪q‬‬
‫ﻣﻊ = 𝜃 ‪. cos‬‬
‫‪2‬‬
‫𝜋‬ ‫‪3‬‬
‫= 𝜃 ﻓﻴﻜﻮن ﻟﻠﻤﻌﺎدﻟﺔ اﳌﻌﻄﺎة‬ ‫‪ •1‬ﻟﺪﻳﻨﺎ ‪ p = −2‬و ‪ q = √3‬ﻣﻨﻪ ‪ 1 − q = 1 − 3 = −2‬و √ = 𝜃 ‪ . cos‬ﻧﺄﺧﺬ‬
‫‪6‬‬ ‫‪2‬‬
‫اﳊﻞ اﳊﻘﻴﻘﻲ ‪ x = −q = −√3‬و اﳊﻠﲔ اﳌﺮﻛﺒﲔ اﳌﱰاﻓﻘﲔ ‪ e /‬و ‪. e− /‬‬
‫‪1‬‬ ‫‪15‬‬ ‫‪1‬‬ ‫‪3‬‬ ‫‪1‬‬ ‫‪3‬‬
‫= 𝜃 ‪ sin 𝜃 = 1 − cos‬ﻣﻨﻪ‬ ‫= 𝜃 ‪ cos‬ﻣﻨﻪ‬
‫ﻟﺪﻳﻨﺎ = ‪ p‬و = ‪ q‬ﻣﻨﻪ = ‪ 1 − q = 1 −‬و‬ ‫•‬‫‪2‬‬
‫‪4‬‬ ‫‪16‬‬ ‫‪4‬‬ ‫‪4‬‬ ‫‪2‬‬ ‫‪4‬‬
‫‪1 √15‬‬ ‫‪1‬‬ ‫‪√15‬‬
‫و‬ ‫‪−‬‬ ‫‪ sin 𝜃 = ±‬و ﺑﺎﻟﺘﺎﱄ ﻓﺎﳌﻌﺎدﻟﺔ اﳌﻌﻄﺎة ﺗﻘﺒﻞ اﳊﻞ اﳊﻘﻴﻘﻲ ‪ x = −‬و اﳊﻠﲔ اﳌﺮﻛﺒﲔ اﳌﱰاﻓﻘﲔ ‪ı‬‬
‫‪4‬‬ ‫‪4‬‬ ‫‪2‬‬ ‫‪4‬‬
‫‪ņőƱŀ Ţ‬‬
‫‪1 √15‬‬
‫‪. +‬‬ ‫‪ı‬‬
‫‪4‬‬ ‫‪4‬‬
‫■‬

‫ﺍﻟﻄﺮﻳﻘﺔ ﺍﻟﺜﺎﻧﻴﺔ ‪ :‬ﻃﺮﻳﻘﺔ ﻻﭬﺮﺍﻧﺞ ﴿‪﴾L‬‬ ‫ﺏ‪3.1.‬‬


‫‪a‬‬
‫ُﺤﻮل اﳌﻌﺎدﻟﺔ ‪ x + ax + bx + c = 0‬إﱃ اﻟﺸﻜﻞ ‪. X + pX + q = 0‬‬
‫‪y = x−‬ﻧ ﱢ‬ ‫ﻛﲈ ﰲ اﻟﻔﻘﺮة اﻟﺴﺎﺑﻘﺔ‪ ،‬ﺑﺎﻟﺘﻌﻮﻳﺾ‬
‫‪3‬‬
‫ﻟﻴﻜﻦ ﻛﺜﲑ اﳊﺪود ‪ P(X) = X + pX + q‬و ﻟﺘﻜﻦ 𝛼 ‪ 𝛽 ،‬و 𝛾 ﺟﺬوره اﻟﺜﻼﺛﺔ ﰲ اﳌﺠﻤﻮﻋﺔ ‪. ℂ‬‬
‫‪ƕ‬‬

‫ﻟﺪﻳﻨﺎ ‪ P (X) = (X − 𝛼) (X − 𝛽) (X − 𝛾) :‬ﺑﻌﺪ اﻟﻨﴩ و اﻟﱰﺗﻴﺐ ﻧﺠﺪ ‪:‬‬

‫𝛾𝛽𝛼 ‪P (X) = X + (𝛼 + 𝛽 + 𝛾) X + (𝛼𝛽 + 𝛽𝛾 + 𝛾𝛼) X −‬‬


‫‪. 𝛼𝛽𝛾 = −q‬‬ ‫و‬ ‫‪𝛼𝛽 + 𝛽𝛾 + 𝛾𝛼 = p‬‬ ‫‪،‬‬ ‫‪𝛼+𝛽+𝛾=0‬‬ ‫و ﺑﺎﳌﻘﺎرﻧﺔ ﻣﻊ اﻟﻌﺒﺎرة اﻷﺻﻠﻴﺔ ﻳﻨﺘﺞ ‪:‬‬
‫‪ũŏ‬‬

‫ﻧﻀﻊ )𝛼 ‪ . 𝛿 = (𝛼 − 𝛽) (𝛽 − 𝛾) (𝛾 −‬ﻟﺪﻳﻨﺎ ‪:‬‬


‫‪p‬‬ ‫‪q‬‬
‫‪𝛿 = −4 (𝛼𝛽 + 𝛽𝛾 + 𝛾𝛼) − 27 (𝛼𝛽𝛾) = −4p − 27q = −108‬‬ ‫‪+‬‬
‫‪3‬‬ ‫‪2‬‬
‫ﻧﻌﺘﱪ اﻷﻋﺪاد ‪:‬‬

‫𝟯𝟵𝟯‬

‫‪http ://tinyurl.com/Malki1718‬‬ ‫‪0‬‬


‫ب‪ .1.‬اﳌﻌﺎدﻻت اﻟﺘﻜﻌﻴﺒﻴﺔ‬
‫‪.‬‬

‫‪Ŕž Ŧ Ľ‬‬
‫𝛾 ‪3𝜉 = 𝛼 + 𝛽 +‬‬
‫𝛾 ‪3𝜉 = 𝛼 + j𝛽 + j‬‬ ‫)𝟏(‬

‫‪œǃ‬‬
‫𝛾‪3𝜉 = 𝛼 + j 𝛽 + j‬‬

‫ﺣﺴﺐ ﻣﺎ ﺳﺒﻖ‪ 𝜉 = 0 ،‬و ‪:‬‬

‫‪Ŀ‬‬
‫‪q √3‬‬ ‫‪q‬‬ ‫‪p‬‬ ‫‪q‬‬
‫‪𝜉 =−‬‬ ‫‪+‬‬ ‫‪ı√𝛿 = − +‬‬ ‫‪+‬‬
‫‪2‬‬ ‫‪18‬‬ ‫‪2‬‬ ‫‪3‬‬ ‫‪2‬‬

‫‪q √3‬‬ ‫‪q‬‬ ‫‪p‬‬ ‫‪q‬‬


‫‪𝜉 =−‬‬ ‫‪−‬‬ ‫‪ı√𝛿 = − −‬‬ ‫‪+‬‬
‫‪2‬‬ ‫‪18‬‬ ‫‪2‬‬ ‫‪3‬‬ ‫‪2‬‬

‫ﺣﻴﺚ إﻋﺘﻤﺪﻧﺎ ﻋﲆ اﳌﺴﺎواة ‪:‬‬

‫‪9𝜉 𝜉 = 𝛼 + 𝛽 + 𝛾 + j + j‬‬ ‫)𝛼𝛾 ‪(𝛼𝛽 + 𝛽𝛾 +‬‬

‫‪= (𝛼 + 𝛽 + 𝛾) − 3 (𝛼𝛽 + 𝛽𝛾 + 𝛾𝛼) = 0 − 3p = −3p‬‬

‫ﻹﳚﺎد ﻗﻴﻤﺔ 𝜉 ‪ .‬ﻣﻨﻪ ‪:‬‬

‫‪q‬‬ ‫‪p‬‬ ‫‪q‬‬


‫= 𝜉‬ ‫‪−‬‬ ‫‪+‬‬ ‫‪+‬‬
‫‪2‬‬ ‫‪3‬‬ ‫‪2‬‬

‫‪q‬‬ ‫‪p‬‬ ‫‪q‬‬


‫= 𝜉‬ ‫‪−‬‬ ‫‪−‬‬ ‫‪+‬‬
‫‪2‬‬ ‫‪3‬‬ ‫‪2‬‬
‫‪p‬‬
‫ﺣﻴﺚ ﻧﺨﺘﺎر اﳉﺬرﻳﻦ اﻟﺘﻜﻌﻴﺒﻴﲔ ﺑﻤﺮاﻋﺎة اﻟﴩط ‪. 𝜉 𝜉 = −‬‬
‫‪3‬‬
‫‪ņőƱŀ Ţ‬‬
‫ُﳏﺪﱢ د اﳉﻤﻠﺔ )𝟏( ﻫﻮ ‪:‬‬
‫‪|1 1‬‬ ‫|‪1‬‬
‫|‬ ‫|‬
‫‪|1 j‬‬ ‫‪j | = 3 j − j = −3√3ı ≠ 0‬‬
‫|‬ ‫|‬
‫‪||1 j‬‬ ‫|| ‪j‬‬

‫و ﺑﲈ أﻧﻪ ﻏﲑ ﻣﻌﺪوم ّ‬
‫ﻓﺈن ﻫﺬه اﳉﻤﻠﺔ ﺗﻘﺒﻞ ﺣ ً‬
‫ﻼ وﺣﻴﺪا و ﻫﻮ ‪:‬‬

‫𝜉‪𝛼=𝜉 +‬‬
‫‪ƕ‬‬

‫𝜉‪𝛽 = j 𝜉 + j‬‬
‫𝜉 ‪𝛾 = j𝜉 + j‬‬

‫اﻟﺼ َﻴﻎ اﻟﺘﻲ وﺟﺪﻧﺎﻫﺎ ﺑﻄﺮﻳﻘﺔ ﺩﺍﻝ ﻓﻴﺮﻭ‪.‬‬


‫ﺗﻢ اﻟﺘﻌﺒﲑ أﻋﻼه ﻋﻦ 𝜉 و 𝜉 ﺑﺪﻻﻟﺔ ‪ p‬و ‪ q‬و ﻫﻲ ﻧﻔﺲ ﱢ‬
‫ﺣﻴﺚ ّ‬
‫‪ũŏ‬‬

‫ﺍﻟﻄﺮﻳﻘﺔ ﺍﻟﺜﺎﻟﺜﺔ ‪ :‬ﻃﺮﻳﻘﺔ ﻓﻴﻴﺖ ﴿‪﴾V‬‬ ‫ﺏ‪4.1.‬‬

‫ﰲ ﻫﺬه اﻟﻔﻘﺮة‪ ،‬ﻧﺴﺘﻌﻤﻞ اﳌﺘﻄﺎﺑﻘﺔ اﳌﺜﻠﺜﻴﺔ ‪ cos(3t) = 4 cos t − 3 cos t‬و اﻟﺘﻲ ﺑﺮﻫﻨّﺎﻫﺎ ﰲ اﻟﻔﻘﺮة ‪ •1‬ﺻﻔﺤﺔ ‪. 48‬‬
‫ﻟﻴﻜﻦ ‪ a‬ﻋﺪد ًا ﺣﻘﻴﻘﻴﺎ ﺣﻴﺚ ‪. |a| < 1‬‬

‫𝟰𝟵𝟯‬

‫‪http ://tinyurl.com/Malki1718‬‬ ‫‪0‬‬


‫ب‪ .‬اﳌﻌﺎدﻻت ﻣﻦ ادلرﺟﺔ اﻟﺜﺎﻟﺜﺔ و اﻟﺮاﺑﻌﺔ‬
‫‪.‬‬
‫ﺑﺪاﻳ ًﺔ‪ ،‬ﻧﱪﻫﻦ ّ‬
‫أن اﳌﻌﺎدﻟﺔ ‪ cos 3𝛼 = a‬ﺗُﻜﺎﻓﺊ اﳌﻌﺎدﻟﺔ ‪ x + px + q = 0‬ﻣﻊ ‪: 4p + 27q < 0‬‬ ‫•‬

‫‪Ŕž Ŧ Ľ‬‬
‫‪cos 3𝛼 = a ⟺ 4 cos 𝛼 − 3 cos 𝛼 = a‬‬
‫‪3‬‬ ‫‪a‬‬
‫‪⟺ cos 𝛼 − cos 𝛼 − = 0‬‬

‫‪œǃ‬‬
‫‪4‬‬ ‫‪4‬‬
‫‪⟺ x + px + q = 0‬‬

‫‪a‬‬ ‫‪3‬‬
‫ﺣﻴﺚ 𝛼 ‪ p = − ، x = cos‬و ‪ . q = −‬ﻟﺪﻳﻨﺎ ‪:‬‬
‫‪4‬‬ ‫‪4‬‬

‫‪Ŀ‬‬
‫‪3‬‬ ‫‪a‬‬ ‫‪27‬‬
‫‪4p + 27q = 4 −‬‬ ‫‪+ 27 −‬‬ ‫=‬ ‫‪a −1‬‬
‫‪4‬‬ ‫‪4‬‬ ‫‪16‬‬
‫أن ‪ّ |a| < 1‬‬
‫ﻓﺈن ‪. 4p + 27q < 0‬‬ ‫و ﺑﲈ ّ‬
‫اﻵن‪ ،‬ﻧﱪﻫﻦ أﻧﻪ إذا ﻛﺎن ∗‪ p, q ∈ ℝ‬ﺑﺤﻴﺚ ‪ 4p + 27q < 0‬ﻓﺈﻧﻪ ﻳﻮﺟﺪ ‪ 𝜌 ∈ ℝ∗+‬و ‪ 𝜃 ∈ ℝ‬ﺑﺤﻴﺚ ﺣﻠﻮل اﳌﻌﺎدﻟﺔ‬ ‫•‬
‫𝜋‪4‬‬ ‫𝜋‪2‬‬
‫‪: 𝜌 cos 𝜃 +‬‬ ‫‪ 𝜌 cos 𝜃 +‬و‬ ‫‪ x + px + q = 0‬ﻫﻲ 𝜃 ‪، 𝜌 cos‬‬
‫‪3‬‬ ‫‪3‬‬
‫ﻧﻌﺘﱪ اﳌﻌﺎدﻟﺔ ‪:‬‬
‫‪x + px + q = 0‬‬ ‫)𝟐(‬
‫‪1‬‬
‫= 𝜃 ‪cos‬‬ ‫ﻼ ﻟﻠﻤﻌﺎدﻟﺔ )𝟐( ‪ ،‬و ﺑﲈ ّ‬
‫أن )𝜃 ‪(cos 3𝜃 + 3 cos‬‬ ‫أن ‪ّ q ≠ 0‬‬
‫ﻓﺈن ‪ x = 0‬ﻟﻴﺲ ﺣ ً‬ ‫ﻧﻀﻊ 𝜃 ‪ . x = 𝜌 cos‬ﺑﲈ ّ‬
‫‪4‬‬
‫ّ‬
‫ﻓﺈن ‪:‬‬
‫‪x + px + q = 0 ⟺ 𝜌 cos 𝜃 + p𝜌 cos 𝜃 + q = 0‬‬
‫𝜃 ‪cos 3𝜃 + 3 cos‬‬
‫𝜌 ⟺‬ ‫‪+ p𝜌 cos 𝜃 + q = 0‬‬
‫‪4‬‬
‫𝜃‪cos 3‬‬‫𝜌‪3‬‬
‫𝜌 ⟺‬ ‫‪+‬‬ ‫‪+ p𝜌 cos 𝜃 + q = 0‬‬
‫‪4‬‬ ‫‪4‬‬
‫‪4p‬‬ ‫‪4q‬‬
‫‪⟺ cos 3𝜃 + 3 +‬‬ ‫‪cos 𝜃 +‬‬ ‫‪=0‬‬
‫𝜌‬ ‫𝜌‬
‫‪ņőƱŀ Ţ‬‬
‫ﻟﻜﻦ ‪ 4p + 27q < 0‬إذن ‪. p < 0‬‬
‫‪2‬‬ ‫‪2‬‬ ‫‪4p‬‬
‫= 𝜌 ﻓﻴﻜﻮن‬ ‫√‬ ‫‪ . 𝜌 = ±‬ﻧﺨﺘﺎر اﻟﻘﻴﻤﺔ ‪−p‬‬ ‫√‬ ‫‪ 3 +‬أي ‪−p‬‬ ‫ﻧﺨﺘﺎر 𝜌 ﺑﺤﻴﺚ ‪= 0‬‬
‫‪√3‬‬ ‫‪√3‬‬ ‫𝜌‬
‫‪4q‬‬ ‫‪4q × 3√3‬‬ ‫‪3√3q‬‬
‫‪=−‬‬ ‫‪=−‬‬
‫𝜌‬ ‫‪8p√−p‬‬ ‫‪2p√−p‬‬
‫ﻣﻨﻪ ‪:‬‬
‫⎧‬ ‫‪3√3q‬‬
‫⎪‬ ‫= 𝜃‪cos 3‬‬
‫‪ƕ‬‬

‫⎪‬
‫⎪‬
‫⎪‬
‫⎪‬ ‫‪2p√−p‬‬
‫‪x + px + q = 0‬‬
‫⟺‬ ‫‪p‬‬
‫𝜃 ‪x = 𝜌 cos‬‬ ‫⎨‬
‫⎪‬ ‫‪𝜌=2 −‬‬
‫⎪‬
‫⎪‬ ‫‪3‬‬
‫⎪‬
‫⎪‬
‫𝜃 ‪x = 𝜌 cos‬‬
‫‪ũŏ‬‬

‫⎩‬

‫𝟱𝟵𝟯‬

‫‪http ://tinyurl.com/Malki1718‬‬ ‫‪0‬‬


‫ب‪ .2.‬اﳌﻌﺎدﻻت ﻣﻦ ادلرﺟﺔ اﻟﺮاﺑﻌﺔ‬
‫‪.‬‬
‫|‬ ‫|‬
‫أن ‪ّ | 3√3q | < 1‬‬
‫ﻷن ‪:‬‬ ‫ﻧﻼﺣﻆ ّ‬
‫|| ‪|| 2p√−p‬‬

‫‪Ŕž Ŧ Ľ‬‬
‫‪27q‬‬
‫≤ ‪4p + 27q < 0 ⟹ 0‬‬ ‫‪<1‬‬ ‫ّ‬
‫)ﻷن ‪(p < 0‬‬
‫‪−4p‬‬

‫‪œǃ‬‬
‫‪27q‬‬
‫≤‪⟹ 0‬‬ ‫‪<1‬‬
‫‪−4p‬‬
‫|‬ ‫|‬

‫‪Ŀ‬‬
‫|‬ ‫| ‪3√3q‬‬
‫| ⟹‬ ‫‪<1‬‬
‫||‬ ‫|| ‪2p√−p‬‬
‫|‬

‫‪3√3q‬‬
‫= 𝜃‪. cos 3‬‬ ‫ﳊﻞ اﳌﻌﺎدﻟﺔ )𝟐( ‪ّ ،‬‬
‫ﻧﺤﻞ ‪= cos 𝜑 :‬‬ ‫ّ‬
‫‪2p√−p‬‬
‫𝜑‬ ‫𝜋‪2‬‬
‫‪cos 3𝜃 = cos 𝜑 ⟺ 3𝜃 = ±𝜑 (mod 2𝜋) ⟺ 𝜃 = ±‬‬ ‫‪mod‬‬
‫‪3‬‬ ‫‪3‬‬
‫أن اﻟﺪاﻟﺔ ‪ cos‬زوﺟﻴﺔ و 𝜋‪—2‬دورﻳﺔ ّ‬
‫ﻓﺈن ﺣﻠﻮل اﳌﻌﺎدﻟﺔ )𝟐( ﻫﻲ ‪:‬‬ ‫ﺑﲈ ّ‬
‫‪−p‬‬ ‫𝜋‪𝜑 4‬‬ ‫‪−p‬‬ ‫𝜋‪𝜑 2‬‬ ‫‪−p‬‬ ‫𝜑‬
‫‪.2‬‬ ‫‪cos‬‬ ‫‪+‬‬ ‫‪ 2‬و‬ ‫‪cos‬‬ ‫‪+‬‬ ‫‪، 2‬‬ ‫‪cos‬‬
‫‪3‬‬ ‫‪3‬‬ ‫‪3‬‬ ‫‪3‬‬ ‫‪3‬‬ ‫‪3‬‬ ‫‪3‬‬ ‫‪3‬‬

‫‪8‬‬
‫‪. x − 4x +‬‬ ‫ﺗﻄﺒﻴﻖ ‪ : 9‬ﺣﻞ ﰲ ‪ ℂ‬اﳌﻌﺎدﻟﺔ ‪= 0 :‬‬
‫‪3‬‬

‫ﺍﳊﻞّ‪.‬‬
‫‪8‬‬
‫‪ ، 4p + 27q = 4 (−4) + 27‬إذن ﺑﺈﻣﻜﺎﻧﻨﺎ إﺳﺘﺨﺪام اﻟﻄﺮﻳﻘﺔ اﳌﺜﻠﺜﻴﺔ‪.‬‬ ‫ﻟﺪﻳﻨﺎ ‪= −64 < 0 :‬‬
‫‪3‬‬
‫‪−p‬‬ ‫‪4‬‬ ‫‪4‬‬
‫‪ . 𝜌 = 2‬و ﻧﺒﺤﺚ ﻋﻦ 𝜃 ﺑﺤﻴﺚ ‪:‬‬ ‫‪=2‬‬ ‫=‬ ‫ﻧﻀﻊ‬
‫‪3‬‬ ‫‪3 √3‬‬
‫‪3√3q‬‬ ‫‪8√3‬‬ ‫‪√3‬‬ ‫𝜋‪5‬‬
‫‪ņőƱŀ Ţ‬‬
‫= 𝜃‪cos 3‬‬ ‫‪=−‬‬ ‫‪=−‬‬ ‫‪= cos‬‬
‫‪2p√−p‬‬ ‫‪8×2‬‬ ‫‪2‬‬ ‫‪6‬‬
‫𝜋‪5‬‬ ‫𝜋‪2‬‬
‫‪.𝜃 = ±‬‬‫‪mod‬‬ ‫ﻧﺠﺪ ‪:‬‬
‫‪18‬‬ ‫‪3‬‬
‫‪4‬‬ ‫𝜋‪17‬‬ ‫‪4‬‬ ‫‪7𝜋 4‬‬ ‫𝜋‪5‬‬ ‫‪8‬‬
‫■‬ ‫‪.‬‬ ‫‪cos‬‬ ‫و‬ ‫‪cos‬‬ ‫‪،‬‬ ‫‪cos‬‬ ‫إذن ﺣﻠﻮل اﳌﻌﺎدﻟﺔ ‪ x − 4x + = 0‬ﻫﻲ ‪:‬‬
‫‪√3‬‬ ‫‪18‬‬ ‫‪√3‬‬ ‫‪18‬‬ ‫‪√3‬‬ ‫‪18‬‬ ‫‪3‬‬

‫ﺍﳌﻌﺎﺩﻻﺕ ﻣﻦ ﺍﻟﺪﺭﺟﺔ ﺍﻟﺮﺍﺑﻌﺔ‬ ‫ﺏ‪2.‬‬


‫‪ƕ‬‬

‫ّأول ﻣﻦ ﲤﻜﻦ ﻣﻦ إﳚﺎد ﺻﻴﻐﺔ ﻋﺎﻣﺔ ﳊﻠﻮل اﳌﻌﺎدﻻت ﻣﻦ اﻟﺪرﺟﺔ اﻟﺮاﺑﻌﺔ ﻫﻮ اﻟﻌﺎﱂ اﻹﻳﻄﺎﱄ ﻓﻴﺮﺍﺭﻱ )‪.(F‬‬
‫ﻧﺴﺘﻌﺮض ﻓﻴﲈ ﻳﲇ ﻃﺮﻳﻘﺔ ﺩﻳﻜﺎﺭﺕ‪ ،‬ﻃﺮﻳﻘﺔ ﻓﻴﺮﺍﺭﻱ و ﺑﻌﺪﻫﺎ ﻃﺮﻳﻘﺔ ﻻﭬﺮﺍﻧﺞ ‪.‬‬

‫ﺍﻟﻄﺮﻳﻘﺔ ﺍﻷﻭﱃ ‪ :‬ﻃﺮﻳﻘﺔ ﺩﻳﻜﺎﺭﺕ ﴿‪﴾D‬‬ ‫ﺏ‪1.2.‬‬


‫‪ũŏ‬‬

‫ﻧﻌﺘﱪ اﳌﻌﺎدﻟﺔ ‪ a x + a x + a x + a x + a = 0‬ﺣﻴﺚ ‪ a , a , a , a , a ∈ ℝ‬و ‪. a ≠ 0‬‬


‫ﺑﻘﺴﻤﺔ اﻟﻄﺮﻓﲔ ﻋﲆ ‪ a‬ﻧﻜﺘﺐ ﻫﺬه اﻷﺧﲑة ﻋﲆ اﻟﺸﻜﻞ ‪ x + ax + bx + cx + d = 0‬و ﺑﺎﻟﺘﻌﻮﻳﺾ ‪، x = y + h‬‬
‫‪a‬‬
‫ﺣﻴﺚ ‪ ، h = −‬ﻧﻜﺘﺒﻬﺎ ﻋﲆ اﻟﺸﻜﻞ ‪. X + pX + qX + r = 0‬‬
‫‪4‬‬
‫إذا ﻛﺎن ‪ّ q = 0‬‬
‫ﻓﺈن اﳌﻌﺎدﻟﺔ ﻣﻀﺎﻋﻔﺔ اﻟﱰﺑﻴﻊ و ﻳﺘﻢ ﺣﻠﻬﺎ ﺑﻮﺿﻊ ‪. t = X‬‬

‫𝟲𝟵𝟯‬

‫‪http ://tinyurl.com/Malki1718‬‬ ‫‪0‬‬


‫ب‪ .‬اﳌﻌﺎدﻻت ﻣﻦ ادلرﺟﺔ اﻟﺜﺎﻟﺜﺔ و اﻟﺮاﺑﻌﺔ‬
‫‪.‬‬
‫أن ‪ . q ≠ 0‬ﻟﻴﻜﻦ ‪. P (X) = X + pX + qX + r‬‬ ‫ﻧﻔﺮض إذن ّ‬
‫ﻧﺒﺤﺚ ﻋﻦ ﺛﻼﺛﺔ أﻋﺪاد ﺣﻘﻴﻘﻴﺔ ‪ v ، u‬و ‪ w‬ﺑﺤﻴﺚ ﻳﻜﻮن ‪. P (X) = X + uX + v X − uX + w‬‬

‫‪Ŕž Ŧ Ľ‬‬
‫أن ‪ u ≠ 0‬ﺑﺎﻟﴬورة )و إﻻّ ﺳﻴﻜﻮن ‪ P (X) = X + (v + w)X + vw‬ﻣﻨﻪ ‪ q = 0‬و ﻫﺬا ﻣﺮﻓﻮض( ‪ .‬ﻟﺪﻳﻨﺎ ‪:‬‬
‫ﺑﺪاﻳ ًﺔ‪ ،‬ﻧﻼﺣﻆ ّ‬

‫‪œǃ‬‬
‫‪P (X) = X + uX + v‬‬ ‫‪X − uX + w‬‬ ‫‪⟺ P (X) = X + v + w − u‬‬ ‫‪X + (uw − uv) X + vw‬‬

‫‪⟺ X + pX + qX + r = X + v + w − u‬‬ ‫‪X + (uw − uv) X + vw‬‬

‫‪⎪v + w − u = p‬‬
‫⎧‬

‫‪Ŀ‬‬
‫⟺‬ ‫‪uw − uv = q‬‬
‫⎨‬
‫⎪‬
‫⎩‬ ‫‪vw = r‬‬

‫‪1‬‬ ‫‪q‬‬ ‫‪q‬‬


‫= ‪w‬و‬ ‫‪p+u +‬‬ ‫= ‪ w − v‬ﻣﻨﻪ‬ ‫ﻣﻦ اﳌﻌﺎدﻟﺔ اﻷوﱃ ﻧﺴﺘﺨﺮج ‪ w + v = p + u‬و ﻣﻦ اﳌﻌﺎدﻟﺔ اﻟﺜﺎﻧﻴﺔ‬
‫‪2‬‬ ‫‪u‬‬ ‫‪u‬‬
‫‪1‬‬ ‫‪q‬‬
‫= ‪ v‬و ﺑﺎﻟﺘﻌﻮﻳﺾ ﰲ اﳌﻌﺎدﻟﺔ اﻟﺜﺎﻟﺜﺔ ﻧﺠﺪ ‪:‬‬
‫‪p+u −‬‬
‫‪2‬‬ ‫‪u‬‬
‫‪1‬‬ ‫‪q‬‬ ‫‪1‬‬ ‫‪q‬‬ ‫‪1‬‬
‫= ‪r = vw‬‬ ‫‪p+u −‬‬ ‫⋅‬ ‫‪p+u +‬‬ ‫=‬ ‫‪u + 2pu + p u − q‬‬
‫‪2‬‬ ‫‪u‬‬ ‫‪2‬‬ ‫‪u‬‬ ‫‪4u‬‬
‫أي‬
‫‪u + 2pu + (p − 4r)u − q = 0‬‬
‫و ﺑﻮﺿﻊ ‪ t = u‬ﺗُﺼﺒﺢ ﻫﺬه اﳌﻌﺎدﻟﺔ ‪:‬‬
‫‪t + 2pt + (p − 4r)t − q = 0‬‬
‫و ﻫﻲ ﻣﻌﺎدﻟﺔ ﺗﻜﻌﻴﺒﻴﺔ ﰲ اﳌﺠﻬﻮل ‪ . t‬ﻟﻴﻜﻦ ‪ t‬أﺣﺪ ﺣﻠﻮﳍﺎ ‪ .‬ﻧﺄﺧﺬ ‪ u = √t‬ﻣﻨﻪ ‪:‬‬
‫‪1‬‬
‫‪q‬‬ ‫‪1‬‬ ‫‪q‬‬
‫=‪w‬‬ ‫=‪ v‬و‬
‫‪p+t +‬‬ ‫‪p+t −‬‬
‫‪2‬‬
‫‪√t‬‬ ‫‪2‬‬ ‫‪√t‬‬
‫ﺑﻘﻲ اﻵن أن ﻧﺤﻞ اﳌﻌﺎدﻟﺘﲔ ‪ X + uX + v = 0‬و ‪ X − uX + w = 0‬و ﺗﻜﻮن ﺣﻠﻮﳍﲈ ﻫﻲ ﺣﻠﻮل اﳌﻌﺎدﻟﺔ ﻣﻦ اﻟﺪرﺟﺔ‬
‫اﻟﺮاﺑﻌﺔ ‪. X + pX + qX + r = 0‬‬
‫‪ņőƱŀ Ţ‬‬
‫ﺣﻞ ﰲ اﳌﺠﻤﻮﻋﺔ ‪ ℂ‬اﳌﻌﺎدﻟﺔ اﻟﺘﺎﻟﻴﺔ ‪:‬‬ ‫ﺗﻄﺒﻴﻖ ‪: 10‬‬
‫‪x − 4x + 8x + 20x − 65 = 0‬‬ ‫)𝟑(‬

‫ﺍﳊﻞّ‪.‬‬
‫‪a‬‬ ‫‪−4‬‬
‫‪ . h = − = −‬ﺑﺎﻟﺘﻌﻮﻳﺾ ‪ x = y + 1‬ﻳﻜﻮن ‪:‬‬ ‫ﻟﺪﻳﻨﺎ ‪= 1‬‬
‫‪4‬‬ ‫‪4‬‬

‫‪(𝟑) ⟺ (y + 1) − 4(y + 1) + 8(y + 1) + 20(y + 1) − 65 = 0‬‬


‫‪⟺ y + 2y + 28y − 40 = 0‬‬ ‫)𝟒(‬
‫‪ƕ‬‬

‫ﻧﺒﺤﺚ ﻋﻦ أﻋﺪاد ﺣﻘﻴﻘﻴﺔ ‪ v ، u ≠ 0‬و ‪ w‬ﺑﺤﻴﺚ ‪. y + 2y + 28y − 40 = y + uy + v y − uy + w :‬‬


‫ﻫﺬه اﻷﻋﺪاد ُﲢﻘﻖ ‪:‬‬

‫⎧‬ ‫‪1‬‬ ‫‪28‬‬


‫‪ũŏ‬‬

‫⎪‬
‫⎪‬ ‫=‪v‬‬ ‫‪2+u −‬‬
‫⎪‬
‫‪⎪v + w − u = p‬‬
‫⎧‬ ‫‪⎪v + w − u = 2‬‬
‫⎧‬ ‫⎪‬
‫⎪‬ ‫‪2‬‬ ‫‪u‬‬
‫⟺ ‪uw − uv = q‬‬ ‫⟺ ‪uw − uv = 28‬‬ ‫‪1‬‬ ‫‪28‬‬
‫⎨‬
‫⎪‬ ‫⎨‬
‫⎪‬ ‫⎨‬
‫⎪‬ ‫=‪w‬‬ ‫‪2+u +‬‬
‫⎩‬ ‫‪vw = r‬‬ ‫⎩‬ ‫‪vw = −40‬‬ ‫⎪‬
‫⎪‬ ‫‪2‬‬ ‫‪u‬‬
‫⎪‬
‫⎪‬
‫‪⎩ vw = −40‬‬

‫و ﺑﺎﻟﺘﺎﱄ ّ‬
‫ﻓﺈن ‪ u‬ﺣﻞ ﻟﻠﻤﻌﺎدﻟﺔ ‪ u + 4u + 164u − 784 = 0 :‬و ﺑﻮﺿﻊ ‪ t = u‬ﻳﻜﻮن ‪ t‬ﺣ ً‬
‫ﻼ ﻟﻠﻤﻌﺎدﻟﺔ ‪:‬‬

‫𝟳𝟵𝟯‬

‫‪http ://tinyurl.com/Malki1718‬‬ ‫‪0‬‬


‫ب‪ .2.‬اﳌﻌﺎدﻻت ﻣﻦ ادلرﺟﺔ اﻟﺮاﺑﻌﺔ‬
‫‪.‬‬
‫أن ‪ّ t = 4‬‬
‫ﺣﻞ ﳍﺎ‪ ،‬ﻣﻨﻪ‬ ‫‪ ، t + 4t + 164t − 784 = 0‬و ﻫﻲ ﻣﻌﺎدﻟﺔ ﺗﻜﻌﻴﺒﻴﺔ‪ .‬ﳊﻠﻬﺎ‪ ،‬ﻧﺘّﺒﻊ أﺣﺪ اﻟﻄﺮق اﻟﺴﺎﺑﻘﺔ‪ .‬ﻧﺠﺪ ّ‬
‫‪ v = −4 ، u = √t = 2‬و ‪. w = 10‬‬

‫‪Ŕž Ŧ Ľ‬‬
‫ّ‬
‫ﺑﻘﻲ ﻟﻨﺎ أن ﻧﺤﻞ اﳌﻌﺎدﻟﺘﲔ ‪ y +2y−4 = 0‬و ‪ y −2y+10 = 0‬؛ ﻧﺠﺪ أن اﳊﻠﻮل ﻫﻲ ‪ y = −1− √5‬و ‪y = −1+ √5‬‬
‫ﻟﻠﻤﻌﺎدﻟﺔ اﻷوﱃ ؛ ‪ y = 1 − 3ı‬و ‪ y = 1 + 3ı‬ﻟﻠﻤﻌﺎدﻟﺔ اﻟﺜﺎﻧﻴﺔ‪.‬‬

‫‪œǃ‬‬
‫ﻓﺈن ﺣﻠﻮل اﳌﻌﺎدﻟﺔ )𝟑( ﻫﻲ ‪x = y + 1 = 2 − 3ı ، x = y + 1 = √5 ، x = y + 1 = −√5 :‬‬ ‫أن ‪ّ x = y + 1‬‬
‫و ﺑﲈ ّ‬
‫■‬ ‫و ‪. x = y + 1 = 2 + 3ı‬‬

‫ﺍﻟﻄﺮﻳﻘﺔ ﺍﻟﺜﺎﻧﻴﺔ ‪ :‬ﻃﺮﻳﻘﺔ ﻓﲑﺍﺭﻱ ﴿‪﴾F‬‬ ‫ﺏ‪2.2.‬‬

‫‪Ŀ‬‬
‫ﻧﻌﺘﱪ اﳌﻌﺎدﻟﺔ ‪. x + 2px + qx + 2rx + s = 0 :‬‬
‫اﳍﺪف ﻫﻮ اﻟﺘﺨﻠﺺ ﻣﻦ اﳊﺪ ‪ 2px‬و ذﻟﻚ ﺑﺈﲤﺎم اﳌﺮﺑﻊ ﻛﲈ ﻳﲇ ‪ :‬ﻟﻴﻜﻦ ‪ u‬ﻋﺪد ًا ﺣﻘﻴﻘﻴﺎ‪.‬‬
‫أن ‪ّ x + px + u = x + 2px + 2u + p x + u :‬‬
‫ﻓﺈن ‪:‬‬ ‫ﺑﲈ ّ‬
‫‪x + px + u‬‬ ‫‪= p − q + 2u x + 2 (pu − r) x + u − s‬‬

‫ﻧﺨﺘﺎر ‪ u‬ﺑﺤﻴﺚ ﻳﻜﻮن ‪ p − q + 2u x + 2 (pu − r) x + u − s‬ﻣﺮﺑﻌ ًﺎ ﺗﺎﻣ ًﺎ و ﻫﺬا ﻳﺘﺤﻘﻖ ﻋﻨﺪﻣﺎ ﻳﻜﻮن ‪Δ = 0‬‬
‫ﺣﻴﺚ ‪ Δ‬ﻫﻮ ﳑ ﱢﻴﺰ اﳌﻌﺎدﻟﺔ ‪ ، p − q + 2u x + 2 (pu − r) x + u − s = 0 :‬و ﰲ ﻫﺬه اﳊﺎﻟﺔ ﻳﻜﻮن‬
‫‪pu − r‬‬
‫ﻼ ﻣﻀﺎﻋﻔ ًﺎ ﳍﺎ‪ .‬ﻟﺪﻳﻨﺎ ‪:‬‬
‫‪x = −‬ﺣ ً‬
‫‪2 p − q + 2u‬‬

‫‪Δ = (2 (pu − r)) − 4 p − q + 2u‬‬ ‫‪u −s‬‬


‫‪= −8u + 4qu + 8 (s − pr) u + 4p s − 4qs + 4r‬‬

‫إذن ‪:‬‬
‫‪Δ = 0 ⟺ −2u + qu + 2 (s − pr) u + p s − qs + r = 0‬‬
‫و ﻫﻲ ﻣﻌﺎدﻟﺔ ﺗﻜﻌﻴﺒﻴﺔ ﰲ اﳌﺠﻬﻮل ‪ . u‬ﻟﻴﻜﻦ ‪ u‬أﺣﺪ ﺣﻠﻮﳍﺎ‪ .‬ﻟﺪﻳﻨﺎ ‪:‬‬

‫‪pu − r‬‬
‫‪p − q + 2u‬‬ ‫‪x + 2 (pu − r) x + u − s = x −‬‬ ‫‪−‬‬
‫‪ņőƱŀ Ţ‬‬
‫‪2 p − q + 2u‬‬

‫‪pu − r‬‬
‫=‬ ‫‪x+‬‬
‫‪2 p − q + 2u‬‬
‫ﻣﻨﻪ ‪:‬‬
‫‪x + px + u‬‬ ‫‪= p − q + 2u‬‬ ‫‪x + 2 (pu − r) x + u − s‬‬

‫‪pu − r‬‬
‫⟺‬ ‫‪x + px + u‬‬ ‫=‬ ‫‪x+‬‬
‫‪2 p − q + 2u‬‬
‫‪ƕ‬‬

‫‪pu − r‬‬
‫‪⟺ x + px + u = − x +‬‬
‫‪2 p − q + 2u‬‬
‫‪pu − r‬‬
‫‪ũŏ‬‬

‫= ‪x + px + u‬‬ ‫‪x+‬‬ ‫أو‬


‫‪2 p − q + 2u‬‬
‫‪pu − r‬‬
‫‪⟺ x + (p + 1) x + u +‬‬ ‫‪=0‬‬
‫‪2 p − q + 2u‬‬
‫‪pu − r‬‬
‫‪x + (p − 1) x + u −‬‬ ‫‪=0‬‬ ‫أو‬
‫‪2 p − q + 2u‬‬

‫و ﺗﻜﻮن ﺣﻠﻮل ﻫﺎﺗﲔ اﳌﻌﺎدﻟﺘﲔ ﻫﻲ ﺣﻠﻮل اﳌﻌﺎدﻟﺔ اﻷﺻﻠﻴﺔ‪.‬‬

‫𝟴𝟵𝟯‬

‫‪http ://tinyurl.com/Malki1718‬‬ ‫‪0‬‬


‫ب‪ .‬اﳌﻌﺎدﻻت ﻣﻦ ادلرﺟﺔ اﻟﺜﺎﻟﺜﺔ و اﻟﺮاﺑﻌﺔ‬
‫‪.‬‬
‫ﺣﻞ ﰲ ‪ ℂ‬اﳌﻌﺎدﻟﺔ ‪:‬‬ ‫ﺗﻄﺒﻴﻖ ‪: 11‬‬

‫‪Ŕž Ŧ Ľ‬‬
‫‪x + 2x + x − 14x − 56 = 0‬‬ ‫)𝟓(‬

‫‪œǃ‬‬
‫ﺍﳊﻞّ‪ .‬ﻟﺪﻳﻨﺎ ‪ r = −7 ، q = 1 ، p = 1 :‬و ‪. s = −56‬‬
‫ﻧﺒﺤﺚ ﻋﻦ ‪ u‬ﺑﺤﻴﺚ ‪ x + px + u = p − q + 2u x + 2 (pu − r) x + u − s :‬أي ﺑﺤﻴﺚ ‪:‬‬
‫‪ . x + x + u = 2ux + 2 (u + 7) x + u + 56‬ﻟﺪﻳﻨﺎ ‪ Δ = 4 (u + 7) − 4 (2u) u + 56 :‬ﻣﻨﻪ ‪:‬‬

‫‪Ŀ‬‬
‫‪Δ = 0 ⟺ −8u + 4u − 392u + 196 = 0 ⟺ −2u + u − 98u + 49 = 0‬‬
‫‪1‬‬
‫ﻧﺠﺪ ّ‬
‫أن = ‪ u‬ﺣﻞ ﳍﺬه اﳌﻌﺎدﻟﺔ ﻣﻨﻪ ‪:‬‬
‫‪2‬‬
‫‪15‬‬ ‫‪225‬‬ ‫‪15‬‬
‫‪2u x + 2 (u + 7) x + u + 56 = x + x +‬‬ ‫=‬ ‫‪x+‬‬
‫‪2‬‬ ‫‪4‬‬ ‫‪2‬‬
‫و ﺑﺎﻟﺘﺎﱄ ‪:‬‬

‫‪1‬‬ ‫‪15‬‬
‫‪x + px + u‬‬ ‫⟺ ‪= 2u x + 2 (u + 7) x + u + 56‬‬ ‫‪x +x+‬‬ ‫‪= x+‬‬
‫‪2‬‬ ‫‪2‬‬
‫‪1‬‬ ‫‪15‬‬ ‫‪1‬‬ ‫‪15‬‬
‫‪⟺ x +x+‬‬ ‫‪=− x+‬‬ ‫أو‬ ‫‪x +x+ =x+‬‬
‫‪2‬‬ ‫‪2‬‬ ‫‪2‬‬ ‫‪2‬‬
‫‪⟺ x + 2x + 8 = 0‬‬ ‫أو‬ ‫‪x −7=0‬‬
‫‪⟺ x = −1 − ı√7‬‬ ‫أو‬ ‫‪x = −1 + ı√7‬‬ ‫أو‬ ‫‪x = −√ 7‬‬ ‫أو‬ ‫‪x = √7‬‬

‫■‬ ‫إذن اﳊﻠﻮل ﻫﻲ ‪ x = −√7 ، x = −1 + ı√7 ، x = −1 − ı√7 :‬و ‪. x = √7‬‬

‫ﺍﻟﻄﺮﻳﻘﺔ ﺍﻟﺜﺎﻧﻴﺔ ‪ :‬ﻃﺮﻳﻘﺔ ﻻﭬﺮﺍﻧﺞ ﴿‪﴾L‬‬ ‫ﺏ‪3.2.‬‬

‫ﻧﻌﺘﱪ اﳌﻌﺎدﻟﺔ ‪ x + px + qx + r = 0‬ﺣﻴﺚ ‪ . p, q, r ∈ ℝ‬ﻟﺘﻜﻦ ‪ z ، z ، z‬و ‪ z‬ﺣﻠﻮﳍﺎ ﰲ ‪ . ℂ‬ﻟﺪﻳﻨﺎ ‪:‬‬


‫‪ņőƱŀ Ţ‬‬
‫) ‪x + px + qx + r = (x − z ) (x − z ) (x − z ) (x − z‬‬
‫ﺑﻌﺪ اﻟﻨﴩ و اﻟﱰﺗﻴﺐ ﻧﺴﺘﻨﺘﺞ ّ‬
‫أن ‪:‬‬
‫⎧‬ ‫‪z +z +z +z‬‬ ‫‪=0‬‬
‫⎪‬
‫⎪‬
‫‪⎪z z + z z + z z + z z + z z + z z‬‬ ‫‪=p‬‬
‫⎨‬
‫⎪‬ ‫‪z z z +z z z +z z z +z z z‬‬ ‫‪= −q‬‬
‫⎪‬
‫⎪‬
‫⎩‬ ‫‪z z z z‬‬ ‫‪=r‬‬
‫اﻵن‪ ،‬ﻧﻌﺘﱪ اﻷﻋﺪاد ‪:‬‬
‫‪ƕ‬‬

‫) ‪⎪ 𝜁 = (z + z ) (z + z‬‬
‫⎧‬
‫) ‪𝜁 = (z + z ) (z + z‬‬
‫⎨‬
‫⎪‬
‫) ‪⎩ 𝜁 = (z + z ) (z + z‬‬
‫‪ũŏ‬‬

‫ﻧﺘﺄﻛﺪ ﺑﺎﳊﺴﺎب ﻣﻦ ّ‬
‫أن ‪:‬‬
‫⎧‬
‫⎪‬ ‫‪𝜁 + 𝜁 + 𝜁 = 2 (z z + z z + z z + z z + z z + z z ) = 2p‬‬
‫⎪‬
‫‪𝜁 𝜁 + 𝜁 𝜁 + 𝜁 𝜁 = (z z + z z + z z + z z + z z + z z ) − 4z z z z = p − 4r‬‬
‫⎨‬
‫⎪‬
‫⎪‬
‫⎩‬ ‫‪𝜁 𝜁 𝜁 = − (z z z + z z z + z z z + z z z ) = −q‬‬
‫و ﺑﺎﻟﺘﺎﱄ ّ‬
‫ﻓﺈن 𝜁 ‪ 𝜁 ،‬و 𝜁 ﻫﻲ ﺟﺬور ﻛﺜﲑ اﳊﺪود ‪:‬‬
‫‪P (X) = X − 2pX + p − 4r X + q‬‬

‫𝟵𝟵𝟯‬

‫‪http ://tinyurl.com/Malki1718‬‬ ‫‪0‬‬


‫ب‪ .3.‬و اﳌﻌﺎدﻻت ﻣﻦ ادلرﺟﺔ اﳋﺎﻣﺴﺔ ؟‬
‫‪.‬‬
‫و ﻫﻮ ﻛﺜﲑ ﺣﺪود ﻣﻦ اﻟﺪرﺟﺔ اﻟﺜﺎﻟﺜﺔ و ﺑﺎﻟﺘﺎﱄ ﺑﺈﻣﻜﺎﻧﻨﺎ إﳚﺎد ﻋﺒﺎرة ﺟﺬوره 𝜁 ‪ 𝜁 ،‬و 𝜁 ﺑﺪﻻﻟﺔ ‪ q ، p‬و ‪. r‬‬
‫ﻧﺘﺄﻛﺪ ﺑﺴﻬﻮﻟﺔ ﻣﻦ ّ‬
‫أن ‪:‬‬

‫‪Ŕž Ŧ Ľ‬‬
‫⎧‬
‫⎪‬ ‫= ‪z +z‬‬ ‫𝜁‪−‬‬
‫⎪‬
‫⎪‬
‫⎪‬
‫⎪‬
‫⎪‬
‫⎪‬

‫‪œǃ‬‬
‫⎪‬
‫⎪‬ ‫= ‪z +z‬‬ ‫𝜁‪−‬‬
‫⎪‬
‫⎪‬
‫⎪‬
‫⎪‬
‫⎪‬
‫= ‪⎪z + z‬‬ ‫𝜁‪−‬‬
‫⎨‬
‫⎪‬ ‫‪z +z =−‬‬ ‫𝜁‪−‬‬
‫⎪‬
‫⎪‬

‫‪Ŀ‬‬
‫⎪‬
‫⎪‬
‫⎪‬
‫⎪‬
‫⎪‬
‫⎪‬ ‫‪z +z =−‬‬ ‫𝜁‪−‬‬
‫⎪‬
‫⎪‬
‫⎪‬
‫⎪‬
‫⎪‬
‫‪⎪z + z = −‬‬
‫⎩‬ ‫𝜁‪−‬‬

‫‪ .‬ﻣﻨﻪ ‪:‬‬ ‫𝜁‪−‬‬ ‫𝜁‪−‬‬ ‫ﺣﻴﺚ ﻳﺘﻢ إﺧﺘﻴﺎر اﳉﺬور اﻟﱰﺑﻴﻌﻴﺔ ﺑﻤﺮاﻋﺎة اﻟﴩط ‪−𝜁 = −q‬‬
‫⎧‬ ‫‪1‬‬
‫⎪‬
‫⎪‬ ‫‪z‬‬ ‫=‬ ‫𝜁‪−𝜁 + −𝜁 + −‬‬
‫⎪‬
‫⎪‬ ‫‪2‬‬
‫⎪‬
‫⎪‬
‫⎪‬
‫⎪‬ ‫‪1‬‬
‫‪⎪z‬‬ ‫=‬ ‫𝜁‪−𝜁 − −𝜁 − −‬‬
‫‪2‬‬
‫⎨‬
‫⎪‬ ‫‪1‬‬
‫⎪‬
‫⎪‬ ‫‪z‬‬ ‫=‬ ‫𝜁‪− −𝜁 + −𝜁 − −‬‬
‫⎪‬
‫⎪‬ ‫‪2‬‬
‫⎪‬
‫⎪‬
‫⎪‬
‫‪⎪z‬‬ ‫‪1‬‬
‫⎩‬ ‫=‬ ‫𝜁‪− −𝜁 − −𝜁 + −‬‬
‫‪2‬‬
‫و ﻫﻲ ﺣﻠﻮل اﳌﻌﺎدﻟﺔ اﻷﺻﻠﻴﺔ‪.‬‬

‫ﺣﻞ ﰲ ‪ ℂ‬اﳌﻌﺎدﻟﺔ ‪:‬‬ ‫ﺗﻄﺒﻴﻖ ‪: 12‬‬


‫‪x − 90x − 320x − 231 = 0‬‬ ‫)𝟔(‬

‫ﺍﳊﻞّ‪ .‬ﻟﺪﻳﻨﺎ ‪ q = −320 ، p = −90 :‬و ‪ r = −231‬ﻣﻨﻪ ‪:‬‬


‫‪ņőƱŀ Ţ‬‬
‫‪P (X) = X − 2pX + p − 4r X + q = X + 180X + 9024X + 102400‬‬
‫ﺑﺎﻟﺘﻌﻮﻳﺾ ‪ X = y − 60‬ﻧﺤﺼﻞ ﻋﲆ ‪. P(y) = y − 1776y − 7040 :‬‬
‫ّ‬
‫و ﻫﻮ ﻛﺜﲑ ﺣﺪود ﻣﻦ اﻟﺪرﺟﺔ اﻟﺜﺎﻟﺜﺔ ﺟﺬوره ﻫﻲ ‪ y = 44 ، y = −4‬و ‪ y = −40‬و ﺑﺎﻟﺘﺎﱄ ﻓﺈن ﺟﺬور ﻛﺜﲑ اﳊﺪود اﻷﺻﲇ‬
‫أن )ﻧﺮاﻋﻲ اﻟﴩط ‪: ( −X −X −X = −q = 320‬‬ ‫ﻫﻲ ‪ X = −16 ، X = −64‬و ‪ . X = −100‬ﻣﻨﻪ ﻧﺴﺘﻨﺘﺞ ّ‬

‫⎧‬ ‫‪1‬‬
‫⎪‬
‫⎪‬ ‫= ‪z‬‬ ‫‪√64 + √16 + √100 = 11‬‬
‫⎪‬
‫⎪‬ ‫‪2‬‬
‫⎪‬
‫⎪‬
‫⎪‬
‫⎪‬ ‫‪1‬‬
‫= ‪⎪z‬‬ ‫‪√64 − √16 − √100 = −3‬‬
‫‪2‬‬
‫‪ƕ‬‬

‫⎨‬
‫⎪‬ ‫‪1‬‬
‫= ‪⎪z‬‬
‫⎪‬ ‫‪−√64 + √16 − √100 = −7‬‬
‫⎪‬
‫⎪‬ ‫‪2‬‬
‫⎪‬
‫⎪‬
‫⎪‬
‫‪⎪ z = 1 −√64 − √16 + √100 = −1‬‬
‫⎩‬ ‫‪2‬‬
‫‪ũŏ‬‬

‫■‬ ‫و ﻫﻲ ﺣﻠﻮل اﳌﻌﺎدﻟﺔ )𝟔( ‪.‬‬

‫ﻭ ﺍﳌﻌﺎﺩﻻﺕ ﻣﻦ ﺍﻟﺪﺭﺟﺔ ﺍﳋﺎﻣﺴﺔ ؟‬ ‫ﺏ‪3.‬‬


‫اﻵن‪ ،‬ﺑﻌﺪ أن ﲤﻜﻨّﺎ ﻣﻦ ﺣﻞ ﻛﻞ اﳌﻌﺎدﻻت اﻟﺘﻲ درﺟﺘﻬﺎ أﺻﻐﺮ ﻣﻦ أو ﺗﺴﺎوي ‪ ، 4‬و ﺑﻄﺮق ﻋﺪّ ة ﻣﻦ ﺑﻴﻨﻬﺎ ﻃﺮﻳﻘﺔ ﻻﭬﺮﺍﻧﺞ اﻟﺘﻲ‬
‫ﻋﲈ إذا ﻛﺎﻧﺖ ﻫﺬه اﻟﻄﺮﻳﻘﺔ ﺗﺼﻠﺢ ﻟﻠﻤﻌﺎدﻻت اﻟﺘﻲ درﺟﺘﻬﺎ ﺗﻔﻮق ‪ ، 5‬أو ﻋﻦ‬ ‫ﺗﺼﻠﺢ ﻟﻜﻞ ﻫﺬه اﻷﻧﻮاع ﻣﻦ اﳌﻌﺎدﻻت‪ ،‬ﻧﺘﺴﺎءل ّ‬
‫𝟬𝟬𝟰‬

‫‪http ://tinyurl.com/Malki1718‬‬ ‫‪0‬‬


‫ب‪ .‬اﳌﻌﺎدﻻت ﻣﻦ ادلرﺟﺔ اﻟﺜﺎﻟﺜﺔ و اﻟﺮاﺑﻌﺔ‬
‫‪.‬‬
‫وﺟﻮد ﻃﺮق أﺧﺮى ﳊﻞ اﳌﻌﺎدﻻت ﻣﻦ اﻟﺪرﺟﺔ اﳋﺎﻣﺴﺔ ﻓﲈ ﻓﻮق ؟‬
‫ﻟﻸﺳﻒ‪ ،‬ﻻ ﺗﻮﺟﺪ أي ﻃﺮﻳﻘﺔ ﳊﻞ اﳌﻌﺎدﻻت اﻟﺘﻲ درﺟﺘﻬﺎ ﺗﻔﻮق ‪ ، 5‬ﻓﻘﺪ أﺛﺒﺖ اﻟﻌﺎﱂ اﻟﻨﺮوﳚﻲ ﺁﺑﻞ )‪ (A‬أﻧﻪ ﻣﻦ اﳌﺴﺘﺤﻴﻞ‬

‫‪Ŕž Ŧ Ľ‬‬
‫إﳚﺎد ﺻﻴﻎ ﻋﺎﻣﺔ ﳊﻠﻮل اﳌﻌﺎدﻻت ﻣﻦ اﻟﺪرﺟﺔ اﳋﺎﻣﺴﺔ ﺑﺪﻻﻟﺔ ﻣﻌﺎﻣﻼﲥﺎ و اﻟﻌﻤﻠﻴﺎت اﳊﺴﺎﺑﻴﺔ )‪ (÷ ، × ، − ، +‬و اﳉﺬور )√‬
‫‪ ... ، √ ، √ ،‬إﻟﺦ( ‪.‬‬

‫‪œǃ‬‬
‫ٍ‬
‫ﻋﻤﻢ اﻟﻌﺎﱂ اﻟﻔﺮﻧﴘ ﭬﺎﻟﻮﺍ )‪ (G‬ﻫﺬه اﻟﻨﺘﻴﺠﺔ و ذﻟﻚ ﺑﺈﳚﺎد ﴍط ﻻز ٍم و ﻛﺎف ﻋﲆ ﻛﺜﲑ ﺣﺪود‬ ‫و ﺑﻌﺪ ﺑﻀﻌﺔ ﺳﻨﻮات ﻓﻘﻂ‪ّ ،‬‬
‫ﺣﺘﻰ ﻳﻜﻮن ﺑﺎﻹﻣﻜﺎن اﻟﺘﻌﺒﲑ ﻋﻦ ﺟﺬوره ﺑﺪﻻﻟﺔ ﻣﻌﺎﻣﻼﺗﻪ و اﻟﻌﻤﻠﻴﺎت اﳊﺴﺎﺑﻴﺔ )‪ (÷ ، × ، − ، +‬و اﳉﺬور )√ ‪، √ ، √ ،‬‬
‫‪ ...‬إﻟﺦ( ‪ ،‬واﺿﻌ ًﺎ ﺑﺬﻟﻚ ُأﺳﺲ ﻓﺮع ﻣﻬﻢ ﻣﻦ ﻓﺮوع اﻟﺮﻳﺎﺿﻴﺎت اﳊﺪﻳﺜﺔ و اﻟﺬي ُﻳﻌﺮف ﺑــِ ﻧﻈﺮﻳﺔ ﭬﺎﻟﻮا )و اﻟﺘﻲ ﻏﺎﻟﺒ ًﺎ ﻣﺎ ﺗ ﱠ‬
‫ُﺪرس‬

‫‪Ŀ‬‬
‫ﰲ ﺳﻨﻮات اﳌﺎﺳﱰ — أي ﺑﻌﺪ اﻟﻠﻴﺴﺎﻧﺲ(‪.‬‬
‫ﻣﺜﻼً‪ ،‬ﻧﱪﻫﻦ ﺑﺎﺳﺘﻌﲈل ﻧﻈﺮﻳﺔ ﭬﺎﻟﻮﺍ أﻧﻪ ﻻ ﻳﻤﻜﻦ اﻟﺘﻌﺒﲑ ﻋﻦ ﺟﺬور ﻛﺜﲑ اﳊﺪود ‪ P (X) = X − 10X + 5‬ﺑﺪﻻﻟﺔ ﻣﻌﺎﻣﻼﺗﻪ و‬
‫اﻟﻌﻤﻠﻴﺎت اﳊﺴﺎﺑﻴﺔ اﻷﺳﺎﺳﻴﺔ‪.‬‬
‫ﻧﻠﻔﺖ اﻹﻧﺘﺒﺎه ﻫﻨﺎ إﱃ إﻣﻜﺎﻧﻴﺔ ﲢﻮﻳﻞ اﳌﻌﺎدﻟﺔ ‪ x + ax + bx + cx + dx + e = 0‬إﱃ اﻟﺸﻜﻞ ‪X − X − A = 0‬‬
‫أي ﺑﻮﺳﻴﻂ واﺣﺪ ﻓﻘﻂ )‪ (A‬و أول ﻣﻦ ﻗﺎم ﺑﺬﻟﻚ ﻫﻮ اﻟﻌﺎﱂ ﺑﺮﻳﻨﭫ )‪ (B‬و ﻫﺬا ﻣﺎ ﻓﺘﺢ اﳌﺠﺎل ﻹﳚﺎد ﻃﺮﻕ ﲢﻠﻴﻠﻴﺔ ﳊﻞ‬
‫اﳌﻌﺎدﻻت ﻣﻦ اﻟﺪرﺟﺔ اﳋﺎﻣﺴﺔ ﺑﺎﺳﺘﻌﲈل دوال ﻣﺘﺴﺎﻣﻴﺔ ‪ 2‬و ﻫﻲ اﻟﺪوال اﻟﻨﺎﻗﺼﻴﺔ ‪) 3‬ﻋﲆ ﻏﺮار ﻃﺮﻳﻘﺔ ﻓﻴﻴﺖ ﳊﻞ اﳌﻌﺎدﻻت‬
‫اﻟﺘﻜﻌﻴﺒﻴﺔ ﺑﺎﺳﺘﻌﲈل اﻟﺪوال اﳌﺜﻠﺜﻴﺔ ‪ — 4‬و ﻫﻲ دوال ﻣﺘﺴﺎﻣﻴﺔ( ‪ :‬ﻫﺬا ﻳﻌﻜﺲ ﻣﺎ آﻟﺖ إﻟﻴﻪ اﻟﺮﻳﺎﺿﻴﺎت اﳊﺪﻳﺜﺔ ﻣﻦ ﺗﻘﺎرب ﺑﲔ‬
‫اﳉﱪ‪ ،‬اﻟﺘﺤﻠﻴﻞ و اﳍﻨﺪﺳﺔ ‪...‬‬
‫‪ņőƱŀ Ţ‬‬
‫‪ũŏ‬‬ ‫‪ƕ‬‬

‫‪ Transcendental Functions2‬ﺑﺎﻹﻧﺠﻠﻴﺰﻳﺔ ؛ ‪ Fonctions transcendentes‬ﺑﺎﻟﻔﺮﻧﺴﻴﺔ‪.‬‬


‫‪ Elliptic Functions3‬ﺑﺎﻹﻧﺠﻠﻴﺰﻳﺔ ؛ ‪ Fonctions elliptiques‬ﺑﺎﻟﻔﺮﻧﺴﻴﺔ‪.‬‬
‫‪ Trigonometric (or Circular) Functions4‬ﺑﺎﻹﻧﺠﻠﻴﺰﻳﺔ ؛ )‪ Fonctions trigonométriques (ou circulaires‬ﺑﺎﻟﻔﺮﻧﺴﻴﺔ‪.‬‬

‫𝟭𝟬𝟰‬

‫‪http ://tinyurl.com/Malki1718‬‬ ‫‪0‬‬


.

Ŕž Ŧ Ľ
œǃ
Ŀ
ņőƱŀ Ţ
ũŏ ƕ

http ://tinyurl.com/Malki1718 0
‫‪.‬‬

‫‪Ŕž Ŧ Ľ‬‬
‫‪œǃ‬‬
‫‪otttttttttttttttttttttttttttttttttttttttttttp‬‬

‫ﻣﻠﺤﻖ ج‬
‫‪s‬‬ ‫‪u‬‬
‫‪s‬‬ ‫اﻟﺤﺮوف اﻟﻴﻮﻧﺎﻧﻴﺔ و اﻟﻼﺗﻨﻴﺔ‬
‫‪u‬‬

‫‪Ŀ‬‬
‫‪s‬‬ ‫‪u‬‬
‫‪qvvvvvvvvvvvvvvvvvvvvvvvvvvvvvvvvvvvvvvvvvvvr‬‬
‫‪.‬‬

‫ﻛﺜﲑ ًا ﻣﺎ ﺗُﺴﺘﻌﻤﻞ اﳊﺮوف اﻟﻴﻮﻧﺎﻧﻴﺔ ‪ 1‬ﰲ اﻟﺮﻳﺎﺿﻴﺎت‪ ،‬و ﻧﻈﺮ ًا ﻷﳘﻴﺘﻬﺎ‪ ،‬ﻧُﺪرج ﻓﻴﲈ ﻳﲇ ﻛﺘﺎﺑﺔ ﻫﺬه اﳊﺮوف ﺑﺎﻟﺼﻴﻐﺘﲔ اﻟﻜﺒﲑة ‪ 2‬و‬
‫اﻟﺼﻐﲑة ‪ ، 3‬ﺗﺴﻤﻴﺘﻬﺎ ‪) 4‬أي ﻧﻄﻘﻬﺎ – ﺑﺎﻟﻔﺮﻧﺴﻴﺔ( و اﳊﺮوف اﻟﻼﺗﻨﻴﺔ اﳌﻜﺎﻓﺌﺔ ﳍﺎ‪.‬‬
‫ﺗﻠﻴﻬﺎ اﳊﺮوف اﻟﻼﺗﻨﻴﺔ ﺑﻤﺨﺘﻠﻒ ﻛﺘﺎﺑﺎﲥﺎ اﻷﻛﺜﺮ إﺳﺘﻌﲈﻻً ﰲ اﻟﺮﻳﺎﺿﻴﺎت‪.‬‬

‫ﺍﻷﺭﻗﺎﻡ‬ ‫ﺝ‪1.‬‬

‫الأرقام العربية الأرقام الهندية الأرقام الفارسية الأرقام الرومانية‬


‫‪۰‬‬ ‫‪٠‬‬ ‫‪0‬‬
‫‪I‬‬ ‫‪۱‬‬ ‫‪١‬‬ ‫‪1‬‬
‫‪ņőƱŀ Ţ‬‬
‫‪II‬‬ ‫‪۲‬‬ ‫‪٢‬‬ ‫‪2‬‬
‫‪III‬‬ ‫‪۳‬‬ ‫‪٣‬‬ ‫‪3‬‬
‫‪IV‬‬ ‫‪۴‬‬ ‫‪٤‬‬ ‫‪4‬‬
‫‪V‬‬ ‫‪۵‬‬ ‫‪٥‬‬ ‫‪5‬‬
‫‪VI‬‬ ‫‪۶‬‬ ‫‪٦‬‬ ‫‪6‬‬
‫‪ƕ‬‬

‫‪VII‬‬ ‫‪۷‬‬ ‫‪٧‬‬ ‫‪7‬‬


‫‪VIII‬‬ ‫‪۸‬‬ ‫‪٨‬‬ ‫‪8‬‬
‫‪IX‬‬ ‫‪۹‬‬ ‫‪٩‬‬ ‫‪9‬‬
‫‪ũŏ‬‬

‫‪ Greek Letters1‬ﺑﺎﻹﻧﺠﻠﻴﺰﻳﺔ ؛ ‪ Lettres grecques‬ﺑﺎﻟﻔﺮﻧﺴﻴﺔ‬


‫‪ Uppercase2‬ﺑﺎﻹﻧﺠﻠﻴﺰﻳﺔ ؛ ‪ Majuscules‬ﺑﺎﻟﻔﺮﻧﺴﻴﺔ‬
‫‪ Lowercase3‬ﺑﺎﻹﻧﺠﻠﻴﺰﻳﺔ ؛ ‪ Minuscules‬ﺑﺎﻟﻔﺮﻧﺴﻴﺔ‬
‫‪Appellation4‬‬

‫𝟯𝟬𝟰‬

‫‪http ://tinyurl.com/Malki1718‬‬ ‫‪0‬‬


‫ اﳊﺮوف اﻟﻴﻮانﻧﻴﺔ‬.2.‫ج‬
.
‫ﺍﳊﺮﻭﻑ ﺍﻟﻴﻮﻧﺎﻧﻴﺔ‬ 2.‫ﺝ‬

Ŕž Ŧ Ľ
Majuscule Minuscule Appellation Correspondance latine

œǃ
A 𝛼 alpha a

B 𝛽,ϐ bêta b

Ŀ
Γ 𝛾 gamma g

Δ 𝛿 delta d

E 𝜖,𝜀 epsilon e (bref)

Z 𝜁 dzêta dz

H 𝜂 êta e (long)

Θ 𝜃,𝜗 thêta t (aspiré)

I 𝜄 iota i

K 𝜅,𝜘 kappa k

Λ 𝜆 lambda l

M 𝜇 mu m

N 𝜈 nu n

Ξ 𝜉 ksi ks
ņőƱŀ Ţ
O o omicron o (bref, fermé)

Π 𝜋,𝜛 pi p

P 𝜌,𝜚 rhô r

Σ 𝜎,𝜍 sigma s

T 𝜏 tau t

Υ 𝜐 upsilon u
ƕ

Φ 𝜙,𝜑 phi p (aspiré)

X 𝜒 khi ( ou chi) k (aspiré)


ũŏ

Ψ 𝜓 psi ps

Ω 𝜔 omega o (long, ouvert)

𝟰𝟬𝟰

http ://tinyurl.com/Malki1718 0
‫ اﳊﺮوف اﻟﻴﻮانﻧﻴﺔ و اﻟﻼﺗﻨﻴﺔ‬.‫ج‬
.
‫ﺍﳊﺮﻭﻑ ﺍﻟﻼﺗﻨﻴﺔ‬ 3.‫ﺝ‬

Ŕž Ŧ Ľ
a, A a, A a, A a, A a, A 𝒶, 𝒜 𝕒, 𝔸 𝔞, 𝔄

œǃ
b, B b, B b, B b, B b, B 𝒷, ℬ 𝕓, 𝔹 𝔟, 𝔅
c, C c, C c, C c, C c, C 𝒸, 𝒞 𝕔, ℂ 𝔠, ℭ
d, D d, D d, D

Ŀ
d, D d, D 𝒹, 𝒟 𝕕, 𝔻 𝔡, 𝔇
e, E e, E e, E e, E e, E ℯ, ℰ 𝕖, 𝔼 𝔢, 𝔈
f, F f, F f, F f, F f, F 𝒻, ℱ 𝕗, 𝔽 𝔣, 𝔉
g, G g, G g, G g, G g, G ℊ, 𝒢 𝕘, 𝔾 𝔤, 𝔊
h, H h, H h, H h, H h, H 𝒽, ℋ 𝕙, ℍ 𝔥, ℌ
i, I i, I i, I i, I i, I 𝒾, ℐ 𝕚, 𝕀 𝔦, ℑ
j, J j, J j, J j, J j, J 𝒿, 𝒥 𝕛, 𝕁 𝔧, 𝔍
k, K k ,K k,K k , K k, K 𝓀, 𝒦 𝕜, 𝕂 𝔨, 𝔎
l, L l, L l, L l, L l, L 𝓁, ℒ 𝕝, 𝕃 𝔩, 𝔏
m, M m, M m, M m, M m, M 𝓂, ℳ 𝕞, 𝕄 𝔪, 𝔐
n, N n, N n, N n, N n, N 𝓃, 𝒩 𝕟, ℕ 𝔫, 𝔑
o, O o, O o, O o, O o, O ℴ, 𝒪 𝕠, 𝕆 𝔬, 𝔒
p, P p, P p, P
ņőƱŀ Ţ
p, P p, P 𝓅, 𝒫 𝕡, ℙ 𝔭, 𝔓
q, Q q, Q q, Q q, Q q, Q 𝓆, 𝒬 𝕢, ℚ 𝔮, 𝔔
r, R r, R r, R r, R r, R 𝓇, ℛ 𝕣, ℝ 𝔯, ℜ
s, S s, S s, S s, S s, S 𝓈, 𝒮 𝕤, 𝕊 𝔰, 𝔖
t, T t, T t, T t, T t, T 𝓉, 𝒯 𝕥, 𝕋 𝔱, 𝔗
u, U u, U u, U 𝓊, 𝒰 𝕦, 𝕌 𝔲, 𝔘
ƕ

u, U u, U
v, V v, V v, V v, V v, V 𝓋, 𝒱 𝕧, 𝕍 𝔳, 𝔙
w, W w, W w, W w, W w, W 𝓌, 𝒲 𝕨, 𝕎 𝔴, 𝔚
ũŏ

x, X x, X x, X x, X x, X 𝓍, 𝒳 𝕩, 𝕏 𝔵, 𝔛
y, Y y, Y y, Y y, Y y, Y 𝓎, 𝒴 𝕪, 𝕐 𝔶, 𝔜
z, Z z, Z z, Z z, Z z, Z 𝓏, 𝒵 𝕫, ℤ 𝔷, ℨ

𝟰𝟬𝟱

http ://tinyurl.com/Malki1718 0
.

Ŕž Ŧ Ľ
œǃ
Ŀ
ņőƱŀ Ţ
ũŏ ƕ

http ://tinyurl.com/Malki1718 0
‫‪.‬‬

‫‪Ŕž Ŧ Ľ‬‬
‫‪œǃ‬‬
‫‪otttttttttttttttttttttttttttttttttttttttttttp‬‬

‫ﻣﻠﺤﻖ د‬
‫‪s‬‬ ‫‪u‬‬
‫‪s‬‬ ‫‪u‬‬ ‫ﻣﻠﺨﺺ اﻟﺪرس‬

‫‪Ŀ‬‬
‫‪s‬‬ ‫‪u‬‬
‫‪qvvvvvvvvvvvvvvvvvvvvvvvvvvvvvvvvvvvvvvvvvvvr‬‬
‫‪.‬‬

‫ﺍﻟﺸﻜﻞ ﺍﳉﱪﻱ ﻟﻸﻋﺪﺍﺩ ﺍﳌﺮﻛﺒﺔ‬ ‫ﺩ‪1.‬‬


‫اﻟﺸﻜﻞ اﳉﱪي )أو اﻟﻜﺘﺎﺑﺔ اﳉﱪﻳﺔ( ﻟﻌﺪد ﻣﺮﻛﺐ ﻫﻮ ‪a + ıb‬‬
‫ﺣﻴﺚ ‪ a‬و ‪ b‬ﻋﺪدان ﺣﻘﻴﻘﻴﺎن‪.‬‬
‫‪ z ⟺ z = z‬ﺣﻘﻴﻘﻲ‬
‫إذا ﻛﺎن ‪ z = a + ıb‬ﺣﻴﺚ ‪ّ ، a, b ∈ ℝ‬‬
‫ﻓﺈن ‪ a‬ﻫﻮ اﳉﺰء‬
‫‪ z ⟺ z = −z‬ﲣﻴﲇ ِﴏف‬ ‫اﳊﻘﻴﻘﻲ ﻟﻠﻌﺪد ‪ z‬و ‪ b‬ﻫﻮ ﺟﺰؤه اﻟﺘﺨﻴﲇ و ﻧﻜﺘﺐ ‪:‬‬
‫اﳌﺮاﻓﻖ ﻳﺘﻼءم )ﻳﺘﲈﺷﻰ( ﻣﻊ ّ‬ ‫)‪. b = Im (z‬‬ ‫و‬ ‫)‪a = Re (z‬‬
‫ﻛﻞ اﻟﻌﻤﻠﻴﺎت‪ ،‬ﺑﻤﻌﻨﻰ إذا ﻛﺎن ‪z‬‬
‫اﳉﺰء اﳊﻘﻴﻘﻲ و اﳉﺰء اﻟﺘﺨﻴﲇ ﻟﻌﺪد ﻣﺮﻛﺐ ﳘﺎ ﻋﺪدان‬
‫ﺻﺤﻴﺤﺎ ّ‬
‫ﻓﺈن ‪:‬‬ ‫ً‬ ‫و ‪ z‬ﻋﺪدﻳﻦ ﻣﺮﻛﺒﲔ و ﻛﺎن ‪ n‬ﻋﺪد ًا‬ ‫ﺣﻘﻴﻘﻴﺎن‪.‬‬
‫; ‪z+z =z+z ; z×z =z×z‬‬ ‫إذا ﻛﺎن اﳉﺰء اﻟﺘﺨﻴﲇ ﻟﻌﺪد ﻣﺮﻛﺐ ﻣﻌﺪو ًﻣﺎ‪ ،‬ﻧﻘﻮل أﻧﻪ ﺣﻘﻴﻘﻲ‪.‬‬
‫‪1‬‬ ‫‪1‬‬ ‫‪z‬‬ ‫‪z‬‬
‫إذا ﻛﺎن اﳉﺰء اﳊﻘﻴﻘﻲ ﻟﻌﺪد ﻣﺮﻛﺐ ﻣﻌﺪو ًﻣﺎ‪ ،‬ﻧﻘﻮل أﻧﻪ ﲣﻴﲇ‬
‫=‬ ‫;‬ ‫‪z =z‬‬ ‫;‬ ‫=‬ ‫ِﴏف‪.‬‬
‫‪z‬‬ ‫‪z‬‬ ‫‪z‬‬ ‫‪z‬‬
‫‪ņőƱŀ Ţ‬‬
‫ﻳﺘﺴﺎوى ﻋﺪدان ﻣﺮﻛﺒﺎن إذا ﺗﺴﺎوى ﺟﺰءاﳘﺎ اﳊﻘﻴﻘﻴﺎن و‬
‫ﻣﺜﺎل ‪ :‬إذا ﻛﺎن ‪ x ∈ ℝ‬و ‪ّ z ∈ ℂ‬‬
‫ﻓﺈن ‪:‬‬ ‫ﺗﺴﺎوى ﺟﺰءاﳘﺎ اﻟﺘﺨﻴﻠﻴﺎن أي ‪ّ ∀a, b, a , b ∈ ℝ‬‬
‫ﻓﺈن ‪:‬‬
‫‪ b = b‬و ‪a + ıb = a + ıb ⟺ a = a‬‬
‫‪−1 + ı + z‬‬ ‫‪−‬‬
‫)‪+ e (1 + ıx‬‬ ‫‪√2 + 2ı‬‬
‫)‪(1 − 2ıxz‬‬ ‫و ﺑﺎﻟﺘﺎﱄ ‪:‬‬
‫‪−1 − ı + z‬‬ ‫‪∀a, b ∈ ℝ : a + ıb = 0 ⟺ a = b = 0‬‬
‫‪−‬‬
‫=‬ ‫‪+ e−‬‬ ‫)‪(1 − ıx‬‬ ‫‪√2 − 2ı‬‬
‫)‪(1 + 2ıxz‬‬ ‫‪ u‬اﻟﴩط ‪ a, b, a , b ∈ ℝ‬ﻣﻬﻢ ﺟﺪا!‬
‫إذا ﻛﺎن ‪ z = a + ıb‬ﻋﺪد ًا ﻣﺮﻛﺒﺎ ﻣﻊ ‪ّ a, b ∈ ℝ‬‬
‫ﻓﺈن ﻃﻮﻳﻠﺘﻪ‬ ‫إذا ﻛﺎﻧﺖ ‪ b ، a ، a‬و ‪ b‬أﻋﺪاد ًا ﺣﻘﻴﻘﻴﺔ ّ‬
‫ﻓﺈن ‪:‬‬
‫= |‪. |z‬‬
‫ﻫﻲ اﻟﻌﺪد اﳊﻘﻴﻘﻲ اﳌﻮﺟﺐ ‪a + b‬‬ ‫‪(a + ıb) + a + ıb = a + a + ı b + b‬‬
‫‪ƕ‬‬

‫‪1‬‬ ‫‪z‬‬ ‫‪(a + ıb) × a + ıb = aa − bb + ı ab + ba‬‬


‫= ‪.‬‬ ‫ﻟﺪﻳﻨﺎ ‪ zz = |z| = a + b :‬و‬
‫|‪z |z‬‬
‫ﻟﻜﻞ ﻋﺪدﻳﻦ ﻣﺮﻛﺒﲔ ‪ z‬و ‪ z‬و ﻟﻜﻞ ﻋﺪد ﺻﺤﻴﺢ ‪ n‬ﻟﺪﻳﻨﺎ ‪:‬‬ ‫و إذا ﻛﺎن ‪) a + b ≠ 0‬أي )‪ّ ((a, b) ≠ (0, 0‬‬
‫ﻓﺈن ‪:‬‬
‫‪1‬‬ ‫‪a‬‬ ‫‪b‬‬
‫‪ũŏ‬‬

‫=‬ ‫‪−‬‬ ‫‪ı‬‬


‫|‪|1‬‬ ‫‪1‬‬ ‫‪a + ıb a + b‬‬ ‫‪a +b‬‬
‫|| ‪||z × z || = |z| × ||z‬‬ ‫;‬ ‫=| |‬
‫|| ‪|| z‬‬ ‫|‪|z‬‬ ‫و ﺑﺎﻟﺘﺎﱄ ‪:‬‬
‫|‪||z || = |z‬‬ ‫|‪|z‬‬ ‫|‪|z‬‬ ‫‪a + ıb‬‬ ‫‪aa + bb‬‬ ‫‪ab − ba‬‬
‫;‬ ‫= || ‪|| z‬‬ ‫=‬ ‫‪+ı‬‬
‫|| ‪||z‬‬ ‫‪a + ıb‬‬ ‫‪a +b‬‬ ‫‪a +b‬‬
‫ُﻳﺴﻤﻰ اﻟﻌﺪد اﳌﺮﻛﺐ ‪ z = a − ıb‬ﺑﻤﺮاﻓﻖ اﻟﻌﺪد ‪ z = a + ıb‬ﻟﻜﻦ ﺑﺎﻟﻨﺴﺒﺔ ﻟﻠﺠﻤﻊ ﻟﺪﻳﻨﺎ ﻓﻘﻂ )اﳌﺘﺒﺎﻳﻨﺔ اﳌﺜﻠﺜﻴﺔ( ‪:‬‬
‫|| ‪∀z, z ∈ ℂ : ||z + z || ≤ |z| + ||z‬‬ ‫ﺣﻴﺚ ‪ . a, b ∈ ℝ‬ﻟﺪﻳﻨﺎ ‪:‬‬

‫𝟳𝟬𝟰‬

‫‪http ://tinyurl.com/Malki1718‬‬ ‫‪0‬‬


‫د‪ .2.‬اﻟﺘﻔﺴﲑ اﻟﻬﻨﺪﳼ ﻟﻸﻋﺪاد اﳌﺮﻛﺒﺔ‬
‫‪.‬‬
‫اﳌﻌﺎدﻟﺔ )‪ (E‬ﺗﻘﺒﻞ ﺣ ّﻠﲔ ﻣﺮﻛﺒﲔ ‪:‬‬
‫𝛿 ‪−b −‬‬ ‫𝛿 ‪−b +‬‬
‫= ‪z‬‬ ‫= ‪ z‬و‬ ‫ﻟﻴﻜﻦ ‪ a, b, c ∈ ℝ‬ﺑﺤﻴﺚ ‪ . a ≠ 0‬ﻧﻀﻊ ‪Δ = b − 4ac‬‬

‫‪Ŕž Ŧ Ľ‬‬
‫‪2a‬‬ ‫‪2a‬‬
‫ﻟﺪﻳﻨﺎ ‪:‬‬ ‫و ﻟﻴﻜﻦ 𝛿 ﻋﺪد ًا ﻣﺮﻛﺒﺎ ﺑﺤﻴﺚ ‪. 𝛿 = Δ‬‬
‫‪b‬‬ ‫‪c‬‬

‫‪œǃ‬‬
‫‪z +z =−‬‬ ‫= ‪ z z‬و‬ ‫ﻟﺘﻜﻦ )‪ (E‬اﳌﻌﺎدﻟﺔ ‪ az + bz + c‬ذات اﳌﺠﻬﻮل اﳌﺮﻛﺐ ‪. z‬‬
‫‪a‬‬ ‫‪a‬‬

‫ﺍﻟﺘﻔﺴﲑ ﺍﳍﻨﺪﺳﻲ ﻟﻸﻋﺪﺍﺩ ﺍﳌﺮﻛﺒﺔ‬ ‫ﺩ‪2.‬‬

‫‪Ŀ‬‬
‫ﻻﺣﻘﺔ ﻧﻘﻄﺔ‪ ،‬ﻻﺣﻘﺔ ﺷﻌﺎع — اﻟﺼﻮرة اﻟﻨﻘﻄﻴﺔ‪ ،‬اﻟﺼﻮرة اﻟﺸﻌﺎﻋﻴﺔ‬
‫ﻓﺈن ﻻﺣﻘﺔ ‪ M‬ﻫﻲ اﻟﻌﺪد اﳌﺮﻛﺐ ‪. z = x + ıy‬‬ ‫إذا ﻛﺎﻧﺖ ‪ M‬اﻟﻨﻘﻄﺔ ذات اﻹﺣﺪاﺛﻴﲔ )‪ّ (x, y‬‬
‫)‪M (z‬‬ ‫‪x‬‬
‫‪y‬‬
‫ّ‬
‫ﻓﺈن ﻻﺣﻘﺔ ⃗‪ u‬ﻫﻲ اﻟﻌﺪد اﳌﺮﻛﺐ‪. zu⃗ = x + ıy‬‬ ‫إذا ﻛﺎن ⃗‪ u‬اﻟﺸﻌﺎع ذا اﳌﺮﻛﺒﺘﲔ‬
‫‪y‬‬
‫إذا ﻛﺎن ‪ z = x + ıy‬ﻋﺪد ًا ﻣﺮﻛﺒﺎ ﻣﻊ ‪ّ x, y ∈ ℝ‬‬
‫ﻓﺈن ‪:‬‬
‫‪O‬‬ ‫‪x‬‬
‫)‪ M (x, y‬ﻫﻲ ﺻﻮرة ‪) z‬اﻟﺼﻮرة اﻟﻨﻘﻄﻴﺔ( ؛‬ ‫•‬
‫→‬
‫‪−‬‬
‫‪u‬‬
‫‪y‬‬
‫‪x‬‬
‫⃗‪ u‬ﻫﻮ ﺻﻮرة ‪) z‬اﻟﺼﻮرة اﻟﺸﻌﺎﻋﻴﺔ( ‪.‬‬ ‫•‬
‫‪x‬‬ ‫‪y‬‬

‫ﻣﻌﺎﻛﺲ‪ ،‬ﻣﺮاﻓﻖ ﻋﺪد ﻣﺮﻛﺐ‬

‫)‪M4 (−z‬‬ ‫)‪M1 (z‬‬

‫اﻟﻨﻘﻄﺔ ‪ M‬ذات اﻟﻼﺣﻘﺔ ‪ z‬ﻫﻲ ﻧﻈﲑة اﻟﻨﻘﻄﺔ )‪ M (z‬ﺑﺎﻟﻨﺴﺒﺔ إﱃ اﳌﺴﺘﻘﻴﻢ )‪. (Ox‬‬
‫اﻟﻨﻘﻄﺔ ‪ M‬ذات اﻟﻼﺣﻘﺔ ‪ −z‬ﻫﻲ ﻧﻈﲑة اﻟﻨﻘﻄﺔ )‪ M (z‬ﺑﺎﻟﻨﺴﺒﺔ إﱃ اﳌﺒﺪأ ‪. O‬‬
‫اﻟﻨﻘﻄﺔ ‪ M‬ذات اﻟﻼﺣﻘﺔ ‪ −z‬ﻫﻲ ﻧﻈﲑة اﻟﻨﻘﻄﺔ )‪ M (z‬ﺑﺎﻟﻨﺴﺒﺔ إﱃ اﳌﺴﺘﻘﻴﻢ )‪. (Oy‬‬
‫)‪M3 (−z‬‬ ‫)‪M2 (z‬‬
‫‪ņőƱŀ Ţ‬‬
‫اﻟﻌﻤﻠﻴﺎت ﰲ ‪ℂ‬‬
‫‪ũŏ‬‬ ‫‪ƕ‬‬

‫𝟴𝟬𝟰‬

‫‪http ://tinyurl.com/Malki1718‬‬ ‫‪0‬‬


‫د‪ .‬ﻣﻠﺨﺺ ادلرس‬
‫‪.‬‬ ‫اﻟﻄﺮح‬ ‫اﳉﻤﻊ‬

‫) ‪B (zB‬‬

‫‪Ŕž Ŧ Ľ‬‬
‫‪z + z′‬‬
‫ﻓﺮق ﻋﺪدﻳﻦ ﻣﺮﻛﺒﲔ ﻫﻮ‬ ‫ﻋﺪدﻳﻦ‬ ‫ﳎﻤﻮع‬
‫) ‪A (zA‬‬ ‫ﻻﺣﻘﺔ ﺷﻌﺎع‪.‬‬ ‫‪z′‬‬ ‫‪−′‬‬
‫→‬ ‫ﻣﺮﻛﺒﲔ ﻫﻮ ﻻﺣﻘﺔ‬

‫‪œǃ‬‬
‫‪u‬‬
‫‪−u +‬‬
‫→‬ ‫ﳎﻤﻮع ﺷﻌﺎﻋﲔ‪.‬‬

‫→‬
‫‪z‬‬ ‫‪=z −z‬‬

‫‪−‬‬
‫‪u′‬‬
‫‪zB − zA‬‬ ‫‪z‬‬
‫‪z‬‬ ‫‪⃗+‬‬ ‫‪′‬‬ ‫‪=z ⃗ +z‬‬ ‫‪′‬‬
‫‪−−→B‬‬ ‫‪−u‬‬
‫→‬
‫‪A‬‬

‫‪Ŀ‬‬
‫‪O‬‬ ‫‪O‬‬

‫اﻟﴬب ﰲ ﻋﺪد ﻣﺮﻛﺐ ﻏﲑ ﻣﻌﺪوم‬ ‫اﻟﴬب ﰲ ﻋﺪد ﻣﺮﻛﺐ ﻃﻮﻳﻠﺘﻪ ‪1‬‬

‫‪M ′ zeıθ‬‬
‫‬
‫‪M ′ z · reıθ‬‬
‫‬
‫‪re‬‬ ‫ﴐب ‪ z‬ﺑﺎﻟﻌﺪد‬ ‫‪e‬‬ ‫ﴐب ‪ z‬ﺑﺎﻟﻌﺪد‬
‫اﻟﺬي ﻃﻮﻳﻠﺘﻪ ‪ 1‬ﻳﻜﺎﻓﺊ‬ ‫اﻟﺬي ﻃﻮﻳﻠﺘﻪ ‪ 1‬ﻳﻜﺎﻓﺊ‬
‫‪M‬‬

‫‪θ‬‬
‫‪rO‬‬

‫إﺟﺮاء دوران زاوﻳﺘﻪ 𝜃‬ ‫إﺟﺮاء دوران زاوﻳﺘﻪ 𝜃‬


‫=‬

‫ﻟﻠﻨﻘﻄﺔ )‪ M (z‬ﺣﻮل‬ ‫ﻟﻠﻨﻘﻄﺔ )‪ M (z‬ﺣﻮل‬


‫‪′‬‬
‫‪OM‬‬

‫ٍ‬
‫ﲢﺎك‬ ‫اﳌﺒﺪأ ‪ O‬و ﺑﻌﺪه‬ ‫)‪M (z‬‬ ‫اﳌﺒﺪأ ‪. O‬‬
‫‪θ‬‬
‫)‪M (z‬‬ ‫ﻧﺴﺒﺘﻪ ‪. r‬‬
‫‪O‬‬ ‫‪O‬‬

‫ﻣﻨﺘﺼﻒ ﻗﻄﻌﺔ ﻣﺴﺘﻘﻴﻤﺔ‬


‫‪z +z‬‬
‫= ‪.z‬‬ ‫إذا ﻛﺎﻧﺖ ‪ A‬و ‪ B‬ﻧﻘﻄﺘﲔ ﻣﻦ اﳌﺴﺘﻮي و ﻛﺎﻧﺖ ‪ I‬ﻣﻨﺘﺼﻒ اﻟﻘﻄﻌﺔ ]‪ّ [AB‬‬
‫ﻓﺈن ‪:‬‬
‫‪2‬‬
‫ﻣﺮﻛﺰ اﳌﺴﺎﻓﺎت اﳌﺘﻨﺎﺳﺒﺔ‬ ‫اﻟﴬب ﰲ ﻋﺪد ﺣﻘﻴﻘﻲ‬
‫إذا ﻛﺎﻧﺖ ) ‪ A (z ) ، ⋯ ، A (z ) ، A (z‬ﻧﻘ ًﻄﺎ‬
‫‪λz‬‬
‫ﻣﻦ اﳌﺴﺘﻮي و ﻛﺎﻧﺖ ‪ a ، ⋯ ، a ، a‬أﻋﺪاد ًا‬
‫‪a + a + ⋯ + a‬‬ ‫ﺣﻘﻴﻘﻴﺔ ﺑﺤﻴﺚ ‪≠ 0‬‬ ‫→‬
‫‪−‬‬
‫إذا ﻛﺎن 𝜆 ﻋﺪد ًا ﺣﻘﻴﻘﻴﺎ‬
‫‪λ‬‬ ‫‪u‬‬
‫ﻓﺈن ﻻﺣﻘﺔ ) ‪ ،G (z‬ﻣﺮﻛﺰ اﳌﺴﺎﻓﺎت اﳌﺘﻨﺎﺳﺒﺔ ﻟﻠﺠﻤﻠﺔ‬ ‫ّ‬ ‫ّ‬
‫ﻓﺈن ‪:‬‬
‫‪ņőƱŀ Ţ‬‬
‫}) ‪ {A (a ) , A (a ) , ⋯ , A (a‬ﻫﻲ ‪:‬‬ ‫‪z‬‬
‫‪z‬‬ ‫⃗‬ ‫=‬ ‫‪𝜆z‬‬ ‫⃗‬
‫‪a z +a z +⋯+a z‬‬ ‫‪−u‬‬
‫→‬
‫= ‪z‬‬
‫‪a +a +⋯+a‬‬

‫ﻃﻮﻳﻠﺔ ﻋﺪد ﻣﺮﻛﺐ‬

‫‪ّ z=z‬‬
‫ﻓﺈن ‪:‬‬ ‫إذا ﻛﺎﻧﺖ ‪ M‬اﻟﻨﻘﻄﺔ ذات اﻹﺣﺪاﺛﻴﲔ )‪ (x, y‬و ﻛﺎن ‪= x + ıy‬‬ ‫•‬
‫)‪M (z‬‬
‫‪y‬‬ ‫‖ = ‪|z| = OM‬‬ ‫‖‪‖OM‬‬ ‫‪‖= x +y‬‬
‫| ‪|z‬‬
‫‪ƕ‬‬

‫=‬
‫‪OM‬‬ ‫‪ z = x + ıy ، B x , y‬و ‪ّ z = x + ıy‬‬
‫ﻓﺈن ‪:‬‬ ‫‪،A x ,y‬‬ ‫إذا ﻛﺎن‬ ‫•‬
‫‪O‬‬ ‫‪x‬‬
‫‪B‬‬ ‫‖ = ‪AB‬‬
‫‖‪‖AB‬‬
‫= || ‪‖ = ||z − z‬‬ ‫‪(x − x ) + y − y‬‬
‫|‬
‫‪z‬‬ ‫‪A‬‬
‫‪−‬‬
‫‪ũŏ‬‬

‫‪|z B‬‬ ‫إذا ﻛﺎﻧﺖ ‪ C ، B ، A‬و ‪ D‬أرﺑﻊ ﻧﻘﻂ ﻟﻮاﺣﻘﻬﺎ ‪ c ، b ، a‬و ‪ d‬ﻋﲆ اﻟﱰﺗﻴﺐ ّ‬
‫ﻓﺈن ‪:‬‬ ‫•‬
‫‪A‬‬ ‫=‬
‫‪AB‬‬ ‫‪| d − c | CD‬‬
‫‪|| b − a || = AB‬‬

‫‪z −z‬‬
‫ﺣﻘﻴﻘﻴﺎ ‪.‬‬ ‫اﻹﺳﺘﻘﺎﻣﻴﺔ ‪ :‬ﺗﻜﻮن اﻟﻨﻘﻂ ‪ B ، A‬و ‪ C‬ﻋﲆ اﺳﺘﻘﺎﻣﺔ واﺣﺪة إذا و ﻓﻘﻂ إذا ﻛﺎن اﻟﻌﺪد‬
‫‪z −z‬‬
‫‪z −z‬‬
‫ﲣﻴﻠﻴﺎ ِﴏﻓﺎ‪.‬‬ ‫اﻟﺘﻌﺎﻣﺪ ‪ :‬اﻟﺸﻌﺎﻋﺎن ‪ AB‬و ‪ AC‬ﻣﺘﻌﺎﻣﺪان إذا و ﻓﻘﻂ إذا ﻛﺎن اﻟﻌﺪد‬
‫‪z −z‬‬

‫𝟵𝟬𝟰‬

‫‪http ://tinyurl.com/Malki1718‬‬ ‫‪0‬‬


‫د‪ .3.‬اﻟﺸﲁ اﳌﺜﻠﱻ )اﻷﳼ( ﻟﻸﻋﺪاد اﳌﺮﻛﺒﺔ‬
‫‪.‬‬
‫‪.z −z = e‬‬ ‫‪/‬‬
‫اﳌﺜﻠﺚ اﳌﺘﻘﺎﻳﺲ اﻷﺿﻼع ‪ :‬اﳌﺜﻠﺚ ‪ ABC‬ﻣﺘﻘﺎﻳﺲ اﻷﺿﻼع و ﻣﺒﺎﴍ إذا و ﻓﻘﻂ إذا ﻛﺎن ) ‪(z − z‬‬
‫‪.z −z‬‬ ‫اﳌﺜﻠﺚ اﻟﻘﺎﺋﻢ و اﳌﺘﺴﺎوي اﻟﺴﺎﻗﲔ ‪ :‬اﳌﺜﻠﺚ ‪ ABC‬ﻗﺎﺋﻢ ﰲ ‪ A‬و ﻣﺘﺴﺎوي اﻟﺴﺎﻗﲔ إذا و ﻓﻘﻂ إذا ﻛﺎن ) ‪= ±ı (z − z‬‬

‫‪Ŕž Ŧ Ľ‬‬
‫ﺍﻟﺸﻜﻞ ﺍﳌﺜﻠﺜﻲ )ﺍﻷﺳﻲ( ﻟﻸﻋﺪﺍﺩ ﺍﳌﺮﻛﺒﺔ‬ ‫ﺩ‪3.‬‬

‫‪œǃ‬‬
‫ﻋﻤﺪة ﻋﺪد ﻣﺮﻛﺐ ﻏﲑ ﻣﻌﺪوم‬

‫‪Ŀ‬‬
‫اﳌﺴﺘﻮي ﻣﻨﺴﻮب إﱃ ﻣﻌﻠﻢ ﻣﺘﻌﺎﻣﺪ‪ ،‬ﻣﺘﺠﺎﻧﺲ و ﻣﺒﺎﴍ ⃗‪. O, u,⃗ v‬‬ ‫•‬

‫‪ M‬ﻧﻘﻄﺔ ﻣﻦ اﳌﺴﺘﻮي ﲣﺘﻠﻒ ﻋﻦ ‪ O‬ﻻﺣﻘﺘﻬﺎ ‪. z ≠ 0‬‬


‫)‪M (z‬‬
‫ﻧُﺴﻤﻲ ﻋﻤﺪ ًة ﻟﻠﻌﺪد اﳌﺮﻛﺐ ‪ّ z‬‬
‫ﻛﻞ ﻗﻴﺲ )ﺑﺎﻟﺮادﻳﺎن( ﻟﻠﺰاوﻳﺔ اﳌﻮﺟﻬﺔ ‪ u,⃗ OM‬و ﻧﻜﺘﺐ ‪:‬‬
‫‪−‬‬
‫→‬
‫‪v‬‬ ‫)‪arg (z‬‬
‫)𝜋‪arg (z) = u,⃗ OM (mod 2‬‬
‫‪O‬‬ ‫‪−‬‬
‫→‬
‫‪u‬‬
‫ﻓﺈن ّ‬
‫ﻛﻞ ﻋﺪد ﻣﻦ اﻟﺸﻜﻞ 𝜋‪ ، 𝜃 + 2k‬ﺣﻴﺚ ‪ ، k ∈ ℤ‬ﻫﻮ أﻳﻀ ًﺎ ﻋﻤﺪة ﻟﻠﻌﺪد ‪. z‬‬ ‫إذا ﻛﺎﻧﺖ 𝜃 ﻋﻤﺪ ًة ﻟﻠﻌﺪد ‪ّ z‬‬ ‫•‬

‫إذا ﻛﺎﻧﺖ ‪ C ، B ، A‬و ‪ D‬أرﺑﻊ ﻧﻘﻂ ﻟﻮاﺣﻘﻬﺎ ‪ c ، b ، a‬و ‪ d‬ﻋﲆ اﻟﱰﺗﻴﺐ ﺑﺤﻴﺚ ‪ B ≠ A‬و ‪D ≠ C‬‬ ‫•‬

‫)أي ‪ b ≠ a‬و ‪ّ (d ≠ c‬‬


‫ﻓﺈن ‪:‬‬
‫‪d−c‬‬
‫‪arg‬‬ ‫)𝜋‪= AB, CD (mod 2‬‬
‫‪b−a‬‬
‫‪z‬‬
‫‪ .‬ﻃﻮﻳﻠﺔ اﻟﻌﺪد‬ ‫ﻛﻞ ﻋﻤﺪة ﻟﻠﻌﺪد ‪ z‬ﻫﻮ أﻳﻀ ًﺎ ﻋﻤﺪة ﻟﻠﻌﺪد‬‫ﻓﺈن ّ‬
‫ﲢﺪﻳﺪ ﻋﻤﺪة ﻟﻌﺪد ﻣﺮﻛﺐ ‪ :‬إذا ﻛﺎن ‪ z‬ﻋﺪد ًا ﻣﺮﻛﺒﺎ ﻏﲑ ﻣﻌﺪوم ّ‬
‫|‪|z‬‬
‫‪z‬‬ ‫‪z‬‬
‫ﺗﺴﺎوي ‪ 1‬و ﺑﺎﻟﺘﺎﱄ ﻳﻮﺟﺪ ‪ 𝜃 ∈ ℝ‬ﺑﺤﻴﺚ ‪ = cos 𝜃 + ı sin 𝜃 :‬؛ و ﺑﺎﻟﺘﺎﱄ ﻓﺎﻟﻌﺪد اﳊﻘﻴﻘﻲ 𝜃 ﻋﻤﺪة ﻟﻠﻌﺪد اﳌﺮﻛﺐ ‪. z‬‬
‫|‪|z‬‬ ‫|‪|z‬‬
‫‪√3 1‬‬ ‫𝜋‪5‬‬ ‫𝜋‪5‬‬ ‫𝜋‪5‬‬
‫‪arg −√3 + ı = arg −‬‬ ‫‪+ ı‬‬ ‫‪= arg cos‬‬ ‫‪+ ı sin‬‬ ‫=‬ ‫)𝜋‪(mod 2‬‬ ‫ﻣﺜﺎل ‪:‬‬
‫‪2‬‬ ‫‪2‬‬ ‫‪6‬‬ ‫‪6‬‬ ‫‪6‬‬
‫‪ņőƱŀ Ţ‬‬
‫اﻟﻜﺘﺎﺑﺔ ‪ : e‬ﻟﻜﻞ ﻋﺪد ﺣﻘﻴﻘﻲ 𝜃 ﻧﻀﻊ ‪ . e = cos 𝜃 + ı sin 𝜃 :‬ﻟﺪﻳﻨﺎ ‪:‬‬
‫ﻟﻜﻞ ﻋﺪدﻳﻦ ﻣﺮﻛﺒﲔ ‪ z‬و ‪ z‬و ﻟﻜﻞ ﻋﺪد ﺻﺤﻴﺢ ‪ّ n‬‬
‫ﻓﺈن ‪:‬‬ ‫ﻟﻜﻞ ﻋﺪدﻳﻦ ﺣﻘﻴﻘﻴﲔ 𝜃 و 𝜃 و ﻟﻜﻞ ﻋﺪد ﺻﺤﻴﺢ ‪ّ n‬‬
‫ﻓﺈن ‪:‬‬
‫‪. ||e || = 1‬‬ ‫•‬
‫‪. Im e‬‬ ‫‪ Re e‬و 𝜃 ‪= sin‬‬ ‫𝜃 ‪= cos‬‬ ‫•‬
‫‪′‬‬ ‫‪′‬‬
‫)𝜋‪. arg zz = arg (z) + arg z (mod 2‬‬ ‫•‬ ‫‪.e ×e = e +‬‬ ‫•‬
‫‪. arg‬‬ ‫)𝜋‪= arg (z) = − arg (z) (mod 2‬‬ ‫•‬ ‫‪.‬‬ ‫‪= e− = e‬‬ ‫•‬
‫‪′‬‬
‫)𝜋‪. arg ′ = arg (z) − arg (z) (mod 2‬‬ ‫•‬ ‫‪. ′ =e −‬‬ ‫•‬

‫)𝜋‪. arg (z ) = n ⋅ arg (z) (mod 2‬‬ ‫•‬ ‫‪. e‬‬ ‫‪=e‬‬
‫‪ƕ‬‬

‫•‬

‫اﻟﺸﻜﻞ اﻷﳼ ﻟﻸﻋﺪاد اﳌﺮﻛﺒﺔ‬


‫ﻛﻞ ﻋﺪد ﻣﺮﻛﺐ ﻏﲑ ﻣﻌﺪوم ‪ُ z‬ﻳﻜﺘﺐ ﻋﲆ اﻟﺸﻜﻞ ‪ z = 𝜌e‬ﺣﻴﺚ 𝜌 ﻋﺪد ﺣﻘﻴﻘﻲ ﻣﻮﺟﺐ ﲤﺎﻣ ًﺎ و 𝜃 ﻋﺪد ﺣﻘﻴﻘﻲ‪.‬‬
‫‪ũŏ‬‬

‫ﻫﺬه اﻟﻜﺘﺎﺑﺔ ﺗُﺴﻤﻰ اﻟﺸﻜﻞ اﻷﳼ ﻟﻠﻌﺪد ‪ . z‬اﻟﻌﺪد 𝜃 ﻟﻴﺲ وﺣﻴﺪ ًا ﻟﻜﻦ ‪:‬‬
‫ﻟﻜﻞ ﻋﺪدﻳﻦ ﺣﻘﻴﻘﻴﲔ ﻣﻮﺟﺒﲔ ﲤﺎﻣ ًﺎ 𝜌 و 𝜌 و ﻟﻜﻞ ﻋﺪدﻳﻦ ﺣﻘﻴﻘﻴﲔ 𝜃 و 𝜃 ّ‬
‫ﻓﺈن ‪:‬‬
‫‪′‬‬
‫‪𝜌e = 𝜌 e‬‬ ‫‪ 𝜃 = 𝜃 + 2k𝜋 , k ∈ ℤ‬و 𝜌 = 𝜌 ⟺‬

‫𝟬𝟭𝟰‬

‫‪http ://tinyurl.com/Malki1718‬‬ ‫‪0‬‬


‫د‪ .‬ﻣﻠﺨﺺ ادلرس‬
‫‪.‬‬
‫ﺑﻌﺾ اﻟﻘﻴﻢ اﳋﺎﺻﺔ‬

‫‪Ŕž Ŧ Ľ‬‬
‫‪/‬‬ ‫‪/‬‬ ‫‪1 √3‬‬
‫‪. √2e‬‬ ‫‪= 1+ı،e‬‬ ‫‪=j=− +‬‬ ‫‪ı ، e−‬‬ ‫‪/‬‬ ‫‪= −ı ، e‬‬ ‫‪= −1 ، e‬‬ ‫‪/‬‬ ‫‪= ı،e = 1‬‬
‫‪2‬‬ ‫‪2‬‬
‫ﺻﻴﻎ ﺃﻭﻟﺮ‬

‫‪œǃ‬‬
‫‪1‬‬
‫) ‪ ∀x ∈ ℝ , cos x = (e + e−‬أي ‪. e + e− = 2 cos x‬‬
‫‪2‬‬
‫‪1‬‬
‫) ‪ ∀x ∈ ℝ , sin x = (e − e−‬أي ‪. e − e = 2ı sin x‬‬
‫‪−‬‬
‫‪2ı‬‬
‫ﺻﻴﻐﺔ ﻣﻮﺍﻓﺮ‬

‫‪Ŀ‬‬
‫‪ ∀x ∈ ℝ , ∀n ∈ ℤ , (e ) = e‬أي ‪. (cos x + ı sin x) = cos nx + ı sin nx‬‬
‫ﻧﺘﻴﺠﺔ‬
‫إذا ﻛﺎن ‪ z = e‬ﻣﻊ ‪ّ x ∈ ℝ‬‬
‫ﻓﺈن ‪:‬‬
‫‪1‬‬ ‫‪1‬‬ ‫‪1‬‬ ‫‪1‬‬ ‫‪1‬‬ ‫‪1‬‬ ‫‪1‬‬ ‫‪1‬‬
‫= ‪. sin nx‬‬ ‫‪z −‬‬ ‫= ‪، cos nx‬‬ ‫‪z +‬‬ ‫= ‪، sin x‬‬ ‫‪z−‬‬ ‫= ‪، cos x‬‬ ‫‪z+‬‬
‫‪2ı‬‬ ‫‪z‬‬ ‫‪2‬‬ ‫‪z‬‬ ‫‪2ı‬‬ ‫‪z‬‬ ‫‪2‬‬ ‫‪z‬‬

‫‪ u‬ﻟﻴﻜﻦ اﻟﻌﺪد اﳌﺮﻛﺐ ‪ z = re‬ﻣﻊ ‪ r‬و ‪ t‬ﻋﺪدان ﺣﻘﻴﻘﻴﺎن‪ ،‬و ‪ . r ≠ 0‬ﻟﺪﻳﻨﺎ ‪:‬‬

‫إذا ﻛﺎن ‪ّ r > 0‬‬


‫ﻓﺈن ‪ |z| = r‬و )𝜋‪ arg (z) = t (mod 2‬و اﻟﺸﻜﻞ اﻷﳼ ﻟﻠﻌﺪد ‪ z‬ﻫﻮ ‪. re‬‬ ‫•‬

‫‪. −re‬‬ ‫‪+‬‬ ‫إذا ﻛﺎن ‪ّ r < 0‬‬


‫ﻓﺈن ‪ |z| = −r‬و )𝜋‪ arg (z) = t + 𝜋 (mod 2‬و اﻟﺸﻜﻞ اﻷﳼ ﻟﻠﻌﺪد ‪ z‬ﻫﻮ‬ ‫•‬

‫‪M reıθ‬‬
‫‬
‫‪ M‬ﻧﻘﻄﺔ ﻣﻦ اﳌﺴﺘﻮي ﲣﺘﻠﻒ ﻋﻦ ‪ O‬ﻻﺣﻘﺘﻬﺎ ‪. z ≠ 0‬‬ ‫•‬

‫‪OM‬‬ ‫ﳉﻤﻞ اﻟﺘﺎﻟﻴﺔ ﻣﺘﻜﺎﻓﺌﺔ ‪:‬‬


‫ا ُ‬
‫=‪r‬‬
‫→‪ −−‬‬ ‫‪ z = re •1‬ﺣﻴﺚ ‪ r‬ﻋﺪد ﺣﻘﻴﻘﻲ ﻣﻮﺟﺐ ﲤﺎ ًﻣﺎ و 𝜃 ﻋﺪد ﺣﻘﻴﻘﻲ‪.‬‬
‫‪−‬‬
‫→‬
‫‪v‬‬ ‫→ =‪θ‬‬
‫‪−‬‬
‫‪u , OM‬‬
‫‪ r = |z| •2‬و )𝜋‪. 𝜃 = arg (z) (mod 2‬‬
‫‪−‬‬
‫→‬
‫‪ņőƱŀ Ţ‬‬
‫‪O‬‬ ‫‪u‬‬ ‫‪ [r, 𝜃] •3‬زوج إﺣﺪاﺛﻴﺎت ﻗﻄﺒﻴﺔ ﻟﻠﻨﻘﻄﺔ ‪. M‬‬
‫‪z‬‬ ‫‪z‬‬ ‫‪ r = OM •4‬و )𝜋‪. 𝜃 = u,⃗ OM (mod 2‬‬
‫‪= = eıθ‬‬
‫|‪|z‬‬ ‫‪r‬‬
‫اﳉﺬور اﻟﻨﻮﻧﻴﺔ‬
‫إذا ﻛﺎن ‪ a‬ﻋﺪد ًا ﻣﺮﻛﺒﺎ ﻏﲑ ﻣﻌﺪوم ّ‬
‫ﻓﺈن ‪:‬‬
‫𝜋‪arg (a) + 2k‬‬ ‫𝜋‪arg (a) + 2k‬‬
‫= ‪z =a ⟺ z=z‬‬ ‫‪|a| cos‬‬ ‫‪+ ı sin‬‬ ‫}‪, k ∈ {0, 1, 2, ⋯ n − 1‬‬
‫‪n‬‬ ‫‪n‬‬
‫ﺣﺎﻟﺔ ﺧﺎﺻﺔ ‪ :‬اﳉﺬور اﻟﻨﻮﻧﻴﺔ ﻟﻠﻮﺣﺪة ﻫﻲ ﺣﻠﻮل اﳌﻌﺎدﻟﺔ ‪ z = 1‬و ﻫﻲ اﻷﻋﺪاد ‪:‬‬
‫‪ƕ‬‬

‫‪−‬‬
‫𝜋‪2k‬‬ ‫𝜋‪2k‬‬ ‫‪/‬‬
‫⟵ ‪𝜔 =0‬‬ ‫‪ 𝜔 = cos‬ﻣﻊ }‪. k ∈ {0, 1, 2, ⋯ n − 1‬‬ ‫‪+ ı sin‬‬ ‫‪=e‬‬
‫=‬
‫‪n‬‬ ‫‪n‬‬
‫ﺣﺎﻟﺔ اﳉﺬور اﻟﱰﺑﻴﻌﻴﺔ ‪ :‬ﻹﳚﺎد اﳉﺬرﻳﻦ اﻟﱰﺑﻴﻌﻴﲔ ﻟﻠﻌﺪد ‪ ، z = a + ıb‬ﻧﺒﺤﺚ ﻋﻦ ﻋﺪدﻳﻦ ﺣﻘﻴﻘﻴﲔ𝛼 و 𝛽 ﺑﺤﻴﺚ‬
‫‪ũŏ‬‬

‫‪.𝛼 +𝛽 = a +b‬‬ ‫‪ (𝛼 + ı𝛽) = a + ıb‬أي ﺑﺤﻴﺚ ‪ 2𝛼𝛽 = b ، 𝛼 − 𝛽 = a :‬و‬

‫ﲢﻮﻳﻼﺕ ﺍﳌﺴﺘﻮﻱ ﺍﳌﺮﻛﺐ‬ ‫ﺩ‪4.‬‬


‫اﻹﻧﺴﺤﺎب‬

‫𝟭𝟭𝟰‬

‫‪http ://tinyurl.com/Malki1718‬‬ ‫‪0‬‬


‫د‪ .4.‬ﲢﻮﯾﻼت اﳌﺴـﺘﻮي اﳌﺮﻛﺐ‬
‫‪.‬‬
‫ﻟﻴﻜﻦ ⃗ ‪ t‬اﻹﻧﺴﺤﺎب اﻟﺬي ﺷﻌﺎﻋﻪ ⃗‪ . u‬ﻟﻜﻞ ﻧﻘﻄﺘﲔ ‪ M‬و ‪ M‬ﻣﻦ اﳌﺴﺘﻮي ‪. M = t ⃗ (M) ⟺ MM = u⃗ :‬‬
‫إذا ﻛﺎﻧﺖ ‪ a‬ﻻﺣﻘﺔ اﻟﺸﻌﺎع ⃗‪ّ u‬‬
‫ﻓﺈن اﻟﻌﺒﺎرة اﳌﺮﻛﺒﺔ ﻟﻺﻧﺴﺤﺎب اﻟﺬي ﺷﻌﺎﻋﻪ ⃗‪ u‬ﻫﻲ ‪. z = z + a :‬‬

‫‪Ŕž Ŧ Ľ‬‬
‫ﺑﺎﻹﻧﺴﺤﺎب ⃗ ‪ t‬ﻫﻲ اﳌﺴﺘﻘﻴﻢ اﻟﺬي ﻳﻮازي )‪(D‬‬ ‫⃗ ‪ t‬ﺗَﺴﺎوي ﻗﻴﺎس ﺑﻤﻌﻨﻰ ﳛﻔﻆ اﳌﺴﺎﻓﺎت‪.‬‬ ‫•‬

‫‪œǃ‬‬
‫و ﻳﻤﺮ ﺑﺎﻟﻨﻘﻄﺔ )‪. t ⃗ (A‬‬ ‫⃗ ‪ t‬ﳛﻔﻆ اﻟﺰواﻳﺎ اﳌﻮﺟﻬﺔ‪.‬‬ ‫•‬
‫➛ ﺻﻮرة اﻟﻘﻄﻌﺔ اﳌﺴﺘﻘﻴﻤﺔ ]‪ [AB‬ﺑﺎﻹﻧﺴﺤﺎب ⃗ ‪t‬‬ ‫⃗ ‪ t‬ﳛﻔﻆ اﻹﺳﺘﻘﺎﻣﻴﺔ‪.‬‬ ‫•‬
‫ﻫﻲ اﻟﻘﻄﻌﺔ اﳌﺴﺘﻘﻴﻤﺔ )‪. t ⃗ (A) t ⃗ (B‬‬ ‫⃗ ‪ t‬ﳛﻔﻆ اﻟﺘﻮازي و اﻟﺘﻌﺎﻣﺪ‪.‬‬ ‫•‬

‫➛ ﺻﻮرة اﻟﺪاﺋﺮة اﻟﺘﻲ ﻣﺮﻛﺰﻫﺎ ‪ I‬و ﻧﺼﻒ ﻗﻄﺮﻫﺎ‬ ‫⃗ ‪ t‬ﳛﻔﻆ اﳌﺴﺎﺣﺎت‪.‬‬ ‫•‬

‫‪Ŀ‬‬
‫‪ R‬ﺑﺎﻹﻧﺴﺤﺎب ⃗ ‪ t‬ﻫﻲ اﻟﺪاﺋﺮة اﻟﺘﻲ ﻣﺮﻛﺰﻫﺎ‬ ‫ُﺻﻮر ﺑﻌﺾ اﻷﺷﻜﺎل ‪:‬‬ ‫•‬

‫)‪ t ⃗ (I‬و ﳍﺎ ﻧﻔﺲ ﻧﺼﻒ اﻟﻘﻄﺮ‪.‬‬ ‫ً‬


‫➛ ﻟﻴﻜﻦ )‪ (D‬ﻣﺴﺘﻘﻴﲈ ﻳﻤﺮ ﺑﻨﻘﻄﺔ ‪ . A‬ﺻﻮرة )‪(D‬‬

‫اﻟﺘﺤﺎﻛﻲ‬
‫ﻟﻴﻜﻦ ‪ h‬اﻟﺘﺤﺎﻛﻲ اﻟﺬي ﻣﺮﻛﺰه ‪ Ω‬و ﻧﺴﺒﺘﻪ ‪ . k‬ﻟﻜﻞ ﻧﻘﻄﺘﲔ ‪ M‬و ‪ M‬ﻣﻦ اﳌﺴﺘﻮي ‪.M = h (M) ⟺ ΩM = kΩM :‬‬
‫إذا ﻛﺎﻧﺖ 𝜔 ﻻﺣﻘﺔ ‪ Ω‬و ‪ k‬ﻋﺪد ًا ﺣﻘﻴﻘﻴﺎ ّ‬
‫ﻓﺈن اﻟﻌﺒﺎرة اﳌﺮﻛﺒﺔ ﻟﻠﺘﺤﺎﻛﻲ اﻟﺬي ﻣﺮﻛﺰه ‪ Ω‬و ﻧﺴﺒﺘﻪ ‪ k‬ﻫﻲ ‪. z = 𝜔 + k (z − 𝜔) :‬‬
‫ﺑﺎﻟﺘﺤﺎﻛﻲ ‪ h‬ﻫﻲ اﳌﺴﺘﻘﻴﻢ اﻟﺬي ﻳﻮازي )‪ (D‬و‬ ‫‪ Ω) h (Ω) = Ω‬ﻫﻲ اﻟﻨﻘﻄﺔ اﻟﺼﺎﻣﺪة اﻟﻮﺣﻴﺪة( ‪.‬‬ ‫•‬
‫ﻳﻤﺮ ﺑﺎﻟﻨﻘﻄﺔ )‪. h (A‬‬ ‫‪ h‬ﻳﴬب اﳌﺴﺎﻓﺎت ﰲ ||‪ ||k‬و ﻳﴬب اﳌﺴﺎﺣﺎت ﰲ ‪.k‬‬ ‫•‬
‫➛ ﺻﻮرة اﻟﻘﻄﻌﺔ اﳌﺴﺘﻘﻴﻤﺔ ]‪ [AB‬ﺑﺎﻟﺘﺤﺎﻛﻲ ‪ h‬ﻫﻲ‬ ‫‪ h‬ﳛﻔﻆ اﻟﺰواﻳﺎ اﳌﻮﺟﻬﺔ‪.‬‬ ‫•‬
‫اﻟﻘﻄﻌﺔ اﳌﺴﺘﻘﻴﻤﺔ ])‪. [h (A) h (B‬‬ ‫‪ h‬ﳛﻔﻆ اﻹﺳﺘﻘﺎﻣﻴﺔ‪.‬‬ ‫•‬
‫➛ ﺻﻮرة اﻟﺪاﺋﺮة اﻟﺘﻲ ﻣﺮﻛﺰﻫﺎ ‪ I‬و ﻧﺼﻒ ﻗﻄﺮﻫﺎ‬ ‫‪ h‬ﳛﻔﻆ اﻟﺘﻮازي و اﻟﺘﻌﺎﻣﺪ‪.‬‬ ‫•‬
‫‪ R‬ﺑﺎﻟﺘﺤﺎﻛﻲ ‪ h‬ﻫﻲ اﻟﺪاﺋﺮة اﻟﺘﻲ ﻣﺮﻛﺰﻫﺎ )‪h (I‬‬ ‫ُﺻﻮر ﺑﻌﺾ اﻷﺷﻜﺎل ‪:‬‬ ‫•‬
‫و ﻧﺼﻒ ﻗﻄﺮﻫﺎ ‪.||k|| R‬‬ ‫➛ ﻟﻴﻜﻦ )‪ (D‬ﻣﺴﺘﻘﻴ ًﲈ ﻳﻤﺮ ﺑﻨﻘﻄﺔ ‪ . A‬ﺻﻮرة )‪(D‬‬

‫اﻟﺪوران‬
‫ﻟﻴﻜﻦ ‪ r‬اﻟﺪوران اﻟﺬي ﻣﺮﻛﺰه ‪ Ω‬و زاوﻳﺘﻪ 𝜃 ‪ .‬ﻟﻜﻞ ﻧﻘﻄﺘﲔ ‪ M‬و ‪ M‬ﻣﻦ اﳌﺴﺘﻮي ‪:‬‬
‫‪M = r (M) ⟺ ΩM = ΩM‬‬ ‫و‬ ‫)𝜋‪ΩM, ΩM = 𝜃 (mod 2‬‬
‫إذا ﻛﺎﻧﺖ 𝜔 ﻻﺣﻘﺔ ‪ Ω‬و 𝜃 ﻋﺪد ًا ﺣﻘﻴﻘﻴﺎ ّ‬
‫ﻓﺈن اﻟﻌﺒﺎرة اﳌﺮﻛﺒﺔ ﻟﻠﺪوران اﻟﺬي ﻣﺮﻛﺰه ‪ Ω‬و زاوﻳﺘﻪ 𝜃 ﻫﻲ ‪.z = 𝜔 + e (z − 𝜔) :‬‬
‫‪ņőƱŀ Ţ‬‬
‫اﳌﺴﺘﻘﻴﻢ )‪ (AB‬ﺑﺎﻟﺪوران ‪ r‬ﻫﻲ اﳌﺴﺘﻘﻴﻢ‬ ‫‪ Ω) r (Ω) = Ω‬ﻫﻲ اﻟﻨﻘﻄﺔ اﻟﺼﺎﻣﺪة اﻟﻮﺣﻴﺪة( ‪.‬‬ ‫•‬
‫))‪. (r (A) r (B‬‬ ‫‪ r‬ﺗَﺴﺎوي ﻗﻴﺎس ﺑﻤﻌﻨﻰ ﳛﻔﻆ اﳌﺴﺎﻓﺎت‪.‬‬ ‫•‬
‫➛ ﺻﻮرة اﻟﻘﻄﻌﺔ اﳌﺴﺘﻘﻴﻤﺔ ]‪ [AB‬ﺑﺎﻟﺪوران ‪ r‬ﻫﻲ‬ ‫‪ r‬ﳛﻔﻆ اﻟﺰواﻳﺎ اﳌﻮﺟﻬﺔ‪.‬‬ ‫•‬
‫اﻟﻘﻄﻌﺔ اﳌﺴﺘﻘﻴﻤﺔ ])‪. [r (A) r (B‬‬ ‫‪ r‬ﳛﻔﻆ اﻹﺳﺘﻘﺎﻣﻴﺔ و اﳌﺴﺎﺣﺎت ‪.‬‬ ‫•‬
‫➛ ﺻﻮرة اﻟﺪاﺋﺮة اﻟﺘﻲ ﻣﺮﻛﺰﻫﺎ ‪ I‬و ﻧﺼﻒ ﻗﻄﺮﻫﺎ‬ ‫‪ h‬ﳛﻔﻆ اﻟﺘﻮازي و اﻟﺘﻌﺎﻣﺪ‪.‬‬ ‫•‬
‫‪ R‬ﺑﺎﻟﺪوران ‪ r‬ﻫﻲ اﻟﺪاﺋﺮة اﻟﺘﻲ ﻣﺮﻛﺰﻫﺎ )‪ r (I‬و‬ ‫ُﺻﻮر ﺑﻌﺾ اﻷﺷﻜﺎل ‪:‬‬ ‫•‬
‫ﻧﺼﻒ ﻗﻄﺮﻫﺎ ‪.‬‬ ‫➛ ﻟﺘﻜﻦ ‪ A‬و ‪ B‬ﻧﻘﻄﺘﲔ ﻣﻦ اﳌﺴﺘﻮي‪ .‬ﺻﻮرة‬
‫‪ƕ‬‬

‫اﻟﺘﺸﺎﺑﻪ اﳌﺴﺘﻮي اﳌﺒﺎﴍ‬


‫اﻟﺘﺸﺎﺑﻪ اﳌﺴﺘﻮي اﳌﺒﺎﴍ ﻫﻮ ﻛﻞ ﲢﻮﻳﻞ ﻧﻘﻄﻲ ﻟﻠﻤﺴﺘﻮي اﳌﺮﻛﺐ ﻋﺒﺎرﺗﻪ ﻣﻦ اﻟﺸﻜﻞ ‪ z = az + b :‬ﺣﻴﺚ ‪ a‬و ‪ b‬ﻋﺪدان ﻣﺮﻛﺒﺎن‬
‫و ‪ . a ≠ 0‬ﻧﺴﺒﺔ ﻫﺬا اﻟﺘﺸﺎﺑﻪ ﻫﻲ |‪. |a‬‬
‫ﺗﺮﻛﻴﺐ ﺗﺸﺎﲠﲔ ‪ :‬ﺗﺮﻛﻴﺐ ﺗﺸﺎﺑﻪ ﻧﺴﺒﺘﻪ ‪ k‬ﻣﻊ ﺗﺸﺎﺑﻪ ﻧﺴﺒﺘﻪ ‪ k‬ﻫﻮ ﺗﺸﺎﺑﻪ ﻧﺴﺒﺘﻪ ‪. kk‬‬
‫‪ũŏ‬‬

‫‪1‬‬
‫ﻛﻞ ﺗﺸﺎﺑﻪ ﻧﺴﺒﺘﻪ ‪ k > 0‬ﻫﻮ ﺗﻄﺒﻴﻖ ﺗﻘﺎﺑﲇ ﻟﻠﻤﺴﺘﻮي ﰲ ﻧﻔﺴﻪ و ﺗﻄﺒﻴﻘﻪ اﻟﻌﻜﴘ ﻫﻮ ﺗﺸﺎﺑﻪ ﻧﺴﺒﺘﻪ ‪.‬‬
‫‪k‬‬
‫‪ B ≠ B‬ﻓﺈﻧﻪ ﻳﻮﺟﺪ ﺗﺸﺎﺑﻪ ﻣﺒﺎﴍ وﺣﻴﺪ ‪f‬‬ ‫ﲢﺪﻳﺪ ﺗﺸﺎﺑﻪ ‪ :‬إذا ﻛﺎﻧﺖ ‪ B ، A ، A‬و ‪ B‬أرﺑﻊ ﻧﻘﻂ ﻣﻦ اﳌﺴﺘﻮي ﺑﺤﻴﺚ ‪ A ≠ A‬و‬
‫‪AB‬‬
‫و زاوﻳﺘﻪ ﻫﻲ ‪. AB, A B‬‬ ‫ﺑﺤﻴﺚ ‪ f (A) = A‬و ‪ . f (B) = B‬ﻧﺴﺒﺔ ﻫﺬا اﻟﺘﺸﺎﺑﻪ ﻫﻲ‬
‫‪AB‬‬
‫ُﺻﻮر ﺑﻌﺾ اﻷﺷﻜﺎل اﳍﻨﺪﺳﻴﺔ ﺑﺘﺸﺎﺑﻪ ‪ :‬ﻛﻞ ﺗﺸﺎﺑﻪ ﻧﺴﺒﺘﻪ ‪ k > 0‬ﳛﻔﻆ ﻣﺮﻛﺰ اﳌﺴﺎﻓﺎت اﳌﺘﻨﺎﺳﺒﺔ و ﻳﴬب اﳌﺴﺎﺣﺎت ﰲ ‪. k‬‬

‫𝟮𝟭𝟰‬

‫‪http ://tinyurl.com/Malki1718‬‬ ‫‪0‬‬


‫د‪ .‬ﻣﻠﺨﺺ ادلرس‬
‫‪.‬‬
‫ﺻﻮرة ﻣﺴﺘﻘﻴﻢ ﺑﺘﺸﺎﺑﻪ ﻧﺴﺒﺘﻪ ‪ k > 0‬ﻫﻮ ﻣﺴﺘﻘﻴﻢ‪ ،‬ﺻﻮرة ﻗﻄﻌﺔ ﻣﺴﺘﻘﻴﻤﺔ ﻫﻲ ﻗﻄﻌﺔ ﻣﺴﺘﻘﻴﻤﺔ‪ ،‬ﺻﻮرة ﻣﺜﻠﺚ ﻫﻮ ﻣﺜﻠﺚ ﻣﺸﺎﺑﻪ ﻟﻪ‬
‫و ﺻﻮرة داﺋﺮة ﻧﺼﻒ ﻗﻄﺮﻫﺎ ‪ R‬ﻫﻲ داﺋﺮة ﻧﺼﻒ ﻗﻄﺮﻫﺎ ‪. kR‬‬

‫‪Ŕž Ŧ Ľ‬‬
‫‪œǃ‬‬
‫‪ a‬و ‪ b‬ﻋﺪدان ﻣﺮﻛﺒﺎن ﺣﻴﺚ ‪. a ≠ 0‬‬
‫‪ f‬اﻟﺘﺤﻮﻳﻞ اﻟﻨﻘﻄﻲ اﻟﺬي ﻋﺒﺎرﺗﻪ اﳌﺮﻛﺒﺔ ﻫﻲ ‪. z = az + b‬‬
‫إذا ﻛﺎن ‪ّ a ≠ 1‬‬
‫ﻓﺈن ‪ f‬ﻫﻮ اﻟﺘﺸﺎﺑﻪ اﳌﺴﺘﻮي اﳌﺒﺎﴍ اﻟﺬي ‪:‬‬ ‫•‬
‫‪b‬‬
‫= 𝜔 )𝜔 ﻫﻲ ﺣﻞ اﳌﻌﺎدﻟﺔ ‪، (z = az + b‬‬

‫‪Ŀ‬‬
‫➛ ﻣﺮﻛﺰه اﻟﻨﻘﻄﺔ ‪ Ω‬ذات اﻟﻼﺣﻘﺔ‬
‫‪1−a‬‬
‫➛ ﻧﺴﺒﺘﻪ |‪، k = |a‬‬
‫➛ و زاوﻳﺘﻪ )𝜋‪. 𝜃 = arg (a) (mod 2‬‬
‫ﻓﺈن ‪ f‬ﻫﻮ اﻹﻧﺴﺤﺎب اﻟﺬي ﻻﺣﻘﺔ ﺷﻌﺎﻋﻪ ﻫﻲ ‪. b‬‬ ‫إذا ﻛﺎن ‪ّ a = 1‬‬ ‫•‬

‫إذا ﻛﺎن ‪ a‬ﻋﺪد ًا ﺣﻘﻴﻘﻴﺎ ﻣﻊ ‪ّ a ≠ 1‬‬


‫ﻓﺈن ‪ f‬ﻫﻮ اﻟﺘﺤﺎﻛﻲ اﻟﺬي ‪:‬‬ ‫•‬
‫‪b‬‬
‫= 𝜔 )𝜔 ﺣﻞ اﳌﻌﺎدﻟﺔ ‪، (z = az + b‬‬ ‫➛ ﻣﺮﻛﺰه اﻟﻨﻘﻄﺔ ‪ Ω‬ذات اﻟﻼﺣﻘﺔ‬
‫‪1−a‬‬
‫➛ و ﻧﺴﺒﺘﻪ ‪. a‬‬
‫إذا ﻛﺎن ‪ |a| = 1‬ﻣﻊ ‪ّ a ≠ 1‬‬
‫ﻓﺈن ‪ f‬ﻫﻮ اﻟﺪوران اﻟﺬي ‪:‬‬ ‫•‬
‫‪b‬‬
‫= 𝜔 )𝜔 ﺣﻞ اﳌﻌﺎدﻟﺔ ‪، (z = az + b‬‬ ‫➛ ﻣﺮﻛﺰه اﻟﻨﻘﻄﺔ ‪ Ω‬ذات اﻟﻼﺣﻘﺔ‬
‫‪1−a‬‬
‫➛ و زاوﻳﺘﻪ )𝜋‪. 𝜃 = arg (a) (mod 2‬‬
‫‪ņőƱŀ Ţ‬‬
‫‪ũŏ‬‬ ‫‪ƕ‬‬

‫𝟯𝟭𝟰‬

‫‪http ://tinyurl.com/Malki1718‬‬ ‫‪0‬‬


‫د‪ .4.‬ﲢﻮﯾﻼت اﳌﺴـﺘﻮي اﳌﺮﻛﺐ‬
‫‪.‬‬
‫ﺍﳍﻨﺪﺳﺔ ﰲ ﺍﳌﺴﺘﻮﻱ ﺍﳌﺮﻛﺐ‬

‫‪Ŕž Ŧ Ľ‬‬
‫اﳌﺴﺘﻮي ﻣﻨﺴﻮب إﱃ ﻣﻌﻠﻢ ﻣﺘﻌﺎﻣﺪ و ﻣﺘﺠﺎﻧﺲ ⃗‪. O, u,⃗ v‬‬
‫اﳌﺴﺎﻓﺔ ﰲ اﳌﺴﺘﻮي اﳌﺮﻛﺐ‬

‫‪œǃ‬‬
‫إذا ﻛﺎﻧﺖ )‪ A (z ) ، M (z‬و ) ‪ B (z‬ﻧﻘﻄ ًﺎ ﻣﻦ اﳌﺴﺘﻮي اﳌﺮﻛﺐ ّ‬
‫ﻓﺈن ‪:‬‬
‫|‪ OM = |z‬و || ‪. AB = ||z − z‬‬

‫‪Ŀ‬‬
‫|‬ ‫|‬
‫ﻣﻌﺎدﻟﺔ اﻟﺪاﺋﺮة اﻟﺘﻲ ﻣﺮﻛﺰﻫﺎ ‪ A‬و ﻧﺼﻒ ﻗﻄﺮﻫﺎ ‪ r > 0‬ﻫﻲ ‪. |z − z | = r :‬‬
‫ﻣﻌﺎدﻟﺔ ﳏﻮر اﻟﻘﻄﻌﺔ اﳌﺴﺘﻘﻴﻤﺔ ]‪ [AB‬ﻫﻲ ‪. ||z − z || = ||z − z || :‬‬

‫اﻟﺰواﻳﺎ ﰲ اﳌﺴﺘﻮي اﳌﺮﻛﺐ‬

‫ﻟﺘﻜﻦ )‪ C (z ) ، B (z ) ، A (z ) ، M (z‬و ) ‪ D (z‬ﻧﻘﻄ ًﺎ ﻣﻦ اﳌﺴﺘﻮي اﳌﺮﻛﺐ ﺑﺤﻴﺚ ‪ z ≠ 0‬و اﻟﻨﻘﻂ ‪ C ، B ، A‬و ‪D‬‬
‫ﳐﺘﻠﻔﺔ ﻣﺜﻨﻰ ﻣﺜﻨﻰ و ﲣﺘﻠﻒ ﻋﻦ ‪ . O‬ﻟﺪﻳﻨﺎ ‪:‬‬
‫)𝜋‪u,⃗ AB = arg (z − z ) (mod 2‬‬ ‫)𝜋‪u,⃗ OM = arg (z) (mod 2‬‬
‫‪−‬‬
‫‪AB, AC = arg‬‬ ‫‪−‬‬ ‫)𝜋‪(mod 2‬‬ ‫‪OA, OB = arg‬‬ ‫)𝜋‪(mod 2‬‬
‫‪−‬‬
‫‪AB, CD = arg‬‬ ‫‪−‬‬ ‫)𝜋‪(mod 2‬‬

‫اﻟﻜﺘﺎﺑﺔ اﳌﺮﻛﺒﺔ ﻟﺒﻌﺾ اﻟﺘﺤﻮﻳﻼت اﻟﻨﻘﻄﻴﺔ‬

‫‪.z = z+b‬‬ ‫ﺻﻮرة اﻟﻨﻘﻄﺔ )‪ M (z‬ﺑﺎﻹﻧﺴﺤﺎب اﻟﺬي ﺷﻌﺎﻋﻪ )‪ U⃗ (b‬ﻫﻲ اﻟﻨﻘﻄﺔ ‪ M z‬ﺑﺤﻴﺚ ‪:‬‬
‫) ‪. z − z = − (z − z‬‬ ‫ﺻﻮرة )‪ M (z‬ﺑﺎﻟﺘﻨﺎﻇﺮ اﳌﺮﻛﺰي ﺣﻮل اﻟﻨﻘﻄﺔ ) ‪ A (z‬ﻫﻲ ‪ M z‬ﺑﺤﻴﺚ ‪:‬‬
‫) ‪. z − z = k (z − z‬‬ ‫ﺻﻮرة )‪ M (z‬ﺑﺎﻟﺘﺤﺎﻛﻲ اﻟﺬي ﻣﺮﻛﺰه ) ‪ A (z‬و ﻧﺴﺒﺘﻪ ∗‪ k ∈ ℝ‬ﻫﻲ ‪ M z‬ﺑﺤﻴﺚ ‪:‬‬
‫‪ņőƱŀ Ţ‬‬
‫) ‪. z − z = e (z − z‬‬ ‫ﺻﻮرة )‪ M (z‬ﺑﺎﻟﺪوران اﻟﺬي ﻣﺮﻛﺰه ) ‪ A (z‬و زاوﻳﺘﻪ ‪ 𝜃 ∈ ℝ‬ﻫﻲ ‪ M z‬ﺑﺤﻴﺚ ‪:‬‬
‫ﻣﻌﺮﻓﺔ ﲢﻮﻳﻞ ﻧﻘﻄﻲ ﻣﻦ ﻛﺘﺎﺑﺘﻪ اﳌﺮﻛﺒﺔ‬

‫ﻓﺈن ‪ M‬ﻫﻲ ﺻﻮرة ‪ M‬ﺑِـ ‪:‬‬


‫ّ‬ ‫إذا ﻛﺎﻧﺖ ‪ M z‬ﺻﻮرة ‪ M z‬ﺑﺤﻴﺚ ‪:‬‬
‫اﻹﻧﺴﺤﺎب اﻟﺬي ﺷﻌﺎﻋﻪ )‪. U⃗ (b‬‬ ‫‪ z = z + b‬ﻣﻊ ‪b ∈ ℂ‬‬

‫‪.A‬‬ ‫اﻟﺘﻨﺎﻇﺮ اﳌﺮﻛﺰي ﺣﻮل اﻟﻨﻘﻄﺔ‬ ‫‪ z = −z + b‬ﻣﻊ ‪b ∈ ℂ‬‬


‫‪ƕ‬‬

‫∗‬
‫‪ A‬و ﻧﺴﺒﺘﻪ ‪. a‬‬ ‫‪−‬‬ ‫اﻟﺘﺤﺎﻛﻲ اﻟﺬي ﻣﺮﻛﺰه‬ ‫‪ z = az + b‬ﻣﻊ }‪ a ∈ ℝ ⧵ {1‬و ‪b ∈ ℂ‬‬
‫∗‬
‫‪ A‬و زاوﻳﺘﻪ )‪. arg (a‬‬ ‫‪−‬‬ ‫اﻟﺪوران اﻟﺬي ﻣﺮﻛﺰه‬ ‫‪ z = az + b‬ﻣﻊ }‪ |a| = 1 ، a ∈ ℂ ⧵ {1‬و ‪b ∈ ℂ‬‬
‫‪ũŏ‬‬

‫𝟰𝟭𝟰‬

‫‪http ://tinyurl.com/Malki1718‬‬ ‫‪0‬‬


.

Ŕž Ŧ Ľ
œǃ
otttttttttttttttttttttttttttttttttttttttttttp

‫ﻣﻠﺤﻖ ﻫ‬
s u
s u ‫ﺣﺴﺎب اﻟﻤﺜﻠﺜﺎت‬

Ŀ
s u
qvvvvvvvvvvvvvvvvvvvvvvvvvvvvvvvvvvvvvvvvvvvr
.

‫ ﺟﻴﺐ اﻟﺘﲈم و داﻟﺔ اﻟﻈﻞ‬،‫ﺗﻌﺮﻳﻒ داﻟﺔ اﳉﻴﺐ‬

1
. i,⃗ OM ‫ ﻗﻴﺲ ﺑﺎﻟﺮادﻳﺎن ﻟﻠﺰاوﻳﺔ‬x ‫ ﻧﻘﻄﺔ ﻣﻦ اﻟﺪاﺋﺮة اﳌﺜﻠﺜﻴﺔ و‬M •

sin (x)
M (x) . M ‫ ﻫﻮ ﺗﺮﺗﻴﺒﺔ اﻟﻨﻘﻄﺔ‬sin (x) ‫ و‬M ‫ ﻫﻮ ﻓﺎﺻﻠﺔ اﻟﻨﻘﻄﺔ‬cos (x)
tan (x) ّ k ∈ ℤ ‫ 𝜋 ﻣﻊ‬+ k𝜋 ‫ ﻣﻦ اﻟﺸﻜﻞ‬x ‫إذا ﱂ ﻳﻜﻦ‬
: ‫ﻓﺈن‬ •
2
sin (x)
O cos (x) 1 tan (x) =
cos (x)
. cos (x) + sin (x) = 1 : ‫ ﻟﺪﻳﻨﺎ‬x ‫ﻟﻜﻞ ﻋﺪد ﺣﻘﻴﻘﻲ‬ •
1
. ّ k ∈ ℤ ‫ 𝜋 ﻣﻊ‬+ k𝜋 ‫ ﻣﻦ اﻟﺸﻜﻞ‬x ‫و إذا ﱂ ﻳﻜﻦ‬
= 1 + tan (x) : ‫ﻓﺈن‬
cos x 2

‫ﺧﻮاص ﻣﺒﺎﴍة‬
ņőƱŀ Ţ
1 1
cos (−x) = cos (x) cos (x + 2𝜋) = cos (x)
x x
−x 1 sin (−x) = − sin (x) sin (x + 2𝜋) = sin (x)
1
tan (−x) = − tan (x) tan (x + 2𝜋) = tan (x)

1 1
π−x +π
ƕ

cos (𝜋 − x) = − cos (x) x cos (x + 𝜋) = − cos (x)


x x
1 sin (𝜋 − x) = sin (x) 1 sin (x + 𝜋) = − sin (x)
tan (𝜋 − x) = − tan (x) tan (x + 𝜋) = tan (x)
ũŏ

π
1 1 2 +
π

𝜋 x 𝜋
2

cos − x = sin (x) cos + x = − sin (x)


2 2
x

x 𝜋 x 𝜋
1 sin − x = cos (x) 1 sin + x = cos (x)
2 2

𝟰𝟭𝟱

http ://tinyurl.com/Malki1718 0
.
.‫ دور ﳍﺎ‬2𝜋 ‫ زوﺟﻴﺔ و دورﻳﺔ ﺣﻴﺚ‬، ℝ ‫ ﻣﻌﺮﻓﺔ ﻋﲆ‬x ↦ cos x ‫اﻟﺪاﻟﺔ‬ •
.‫ دور ﳍﺎ‬2𝜋 ‫ ﻓﺮدﻳﺔ و دورﻳﺔ ﺣﻴﺚ‬، ℝ ‫ ﻣﻌﺮﻓﺔ ﻋﲆ‬x ↦ sin x ‫اﻟﺪاﻟﺔ‬ •

Ŕž Ŧ Ľ
𝜋 |
.‫ ﻓﺮدﻳﺔ و دورﻳﺔ ﺣﻴﺚ 𝜋 دور ﳍﺎ‬، ℝ ⧵ + k𝜋 | k ∈ ℤ ‫ ﻣﻌﺮﻓﺔ ﻋﲆ‬x ↦ tan x ‫اﻟﺪاﻟﺔ‬
|

2

œǃ
‫اﳌﺘﻄﺎﺑﻘﺎت اﳌﺜﻠﺜﻴﺔ‬

cos 2a = cos a − sin a cos (a + b) = cos a cos b − sin a sin b

Ŀ
= 2 cos a − 1 cos (a − b) = cos a cos b + sin a sin b
= 1 − 2 sin a sin (a + b) = sin a cos b + cos a sin b
sin 2a = 2 cos a sin a sin (a − b) = sin a cos b − cos a sin b
tan a + tan b
2 tan a tan (a + b) =
tan 2a = 1 − tan a tan b
1 − tan a tan a − tan b
tan (a − b) =
1 + tan a tan b
1 + cos 2a sin a cos b = [sin (a + b) + sin (a − b)]
cos a =
2 cos a sin b = [sin (a + b) − sin (a − b)]
1 − cos 2a
sin a = cos a cos b = [cos (a + b) + cos (a − b)]
2
1 sin a sin b = [cos (a + b) − cos (a − b)]
sin a cos a = sin 2a
2 a+b a−b
sin a + sin b = 2 sin cos
2 2
a+b a−b
a sin a − sin b = 2 cos sin
1 + cos a = 2 cos 2 2
2 a+b a−b
a cos a + cos b = 2 cos cos
1 − cos a = 2 sin 2 2
2 a+b a−b
cos a − cos b = −2 sin sin
2 2
sin 2a 1 − cos 2a
ņőƱŀ Ţ
tan a = =
1 + cos 2a sin 2a

‫اﳌﻌﺎدﻻت اﳌﺜﻠﺜﻴﺔ‬
. k ∈ ℤ ‫ ﻣﻊ‬b = ±a + 2k𝜋 ‫ إذا و ﻓﻘﻂ إذا ﻛﺎن‬cos a = cos b •

. k ∈ ℤ ‫ ﻣﻊ‬b = 𝜋 − a + 2k 𝜋 ‫ أو‬k ∈ ℤ ‫ ﻣﻊ‬b = a + 2k𝜋 ‫ إذا و ﻓﻘﻂ إذا ﻛﺎن‬sin a = cos b •

. k ∈ ℤ ‫ ﻣﻊ‬b = a + k𝜋 ‫ إذا و ﻓﻘﻂ إذا ﻛﺎن‬tan a = tan b •


ƕ

‫اﻟﻌﻼﻗﺎت اﳌﱰﻳﺔ ﰲ اﳌﺜﻠﺚ‬


ũŏ

𝟰𝟭𝟲

http ://tinyurl.com/Malki1718 0
‫ ﺣﺴﺎب اﳌﺜﻠﺜﺎت‬.‫ﻫ‬
.
ّ ABC ‫إذا ﻛﺎﻧﺖ 𝒜 ﻣﺴﺎﺣﺔ اﳌﺜﻠﺚ‬
: ‫ﻓﺈن‬
C

Ŕž Ŧ Ľ
𝒜 = ab sin C = ac sin B = bc sin A
b a
a sin B sin C b sin A sin C sin A sin B sin C
= = = =
a b c

œǃ
2 sin A 2 sin B A c B
a = b + c − 2bc cos A
c sin A sin B
= C
2 sin C b = a + c − 2ac cos B
a
= s (s − a) (s − b) (s − c) c = a + b − 2ab cos C

Ŀ
b

. s = (a + b + c) ‫ﺣﻴﺚ‬ A c B

.(H) ‫اﻟﺼﻴﻐﺔ اﻷﺧﲑة ﺗُﺪﻋﻰ ﺻﻴﻐﺔ ﻫﻴﺮﻭﻥ‬


ņőƱŀ Ţ
ũŏ ƕ

𝟰𝟭𝟳

http ://tinyurl.com/Malki1718 0
.

Ŕž Ŧ Ľ
œǃ
Ŀ
ņőƱŀ Ţ
ũŏ ƕ

http ://tinyurl.com/Malki1718 0
‫ﻫ‪ .‬ﺣﺴﺎب اﳌﺜﻠﺜﺎت‬
‫‪.‬‬
‫ﻗ‪‬ﻴﻢ ﺟﻴﻮﺏ ﻭ ﺟﻴﻮﺏ ﲤﺎﻡ ﺑﻌﺾ ﺍﻟﺰﻭﺍﻳﺎ )ﻏﲑ ﺍﻟﺸﻬﲑﺓ(‬

‫‪Ŕž Ŧ Ľ‬‬
‫اﳌﺮﺟﻊ‬ ‫𝜽 ‪sin‬‬ ‫𝜽 ‪cos‬‬ ‫𝜽‬

‫‪œǃ‬‬
‫‪10 − 2√5‬‬ ‫‪1 + √5‬‬ ‫𝜋‬
‫ﲤﺮﻳﻦ ‪ 214‬ﺻﻔﺤﺔ ‪ 305‬؛ ﲤﺮﻳﻦ ‪ 244‬ﺻﻔﺤﺔ ‪336‬‬
‫‪4‬‬ ‫‪4‬‬ ‫‪5‬‬
‫‪10 + 2√5‬‬ ‫‪−1 + √5‬‬ ‫𝜋‪2‬‬
‫ﲤﺮﻳﻦ ‪ 214‬ﺻﻔﺤﺔ ‪ 305‬؛ ﲤﺮﻳﻦ ‪ 244‬ﺻﻔﺤﺔ ‪ 336‬؛ ﲤﺮﻳﻦ ‪ 213‬ﺻﻔﺤﺔ‬
‫‪4‬‬ ‫‪4‬‬ ‫‪5‬‬
‫‪303‬‬

‫‪Ŀ‬‬
‫‪10 − 2√5‬‬ ‫‪1 + √5‬‬ ‫𝜋‪4‬‬
‫ﲤﺮﻳﻦ ‪ 214‬ﺻﻔﺤﺔ ‪ 305‬؛ ﲤﺮﻳﻦ ‪ 244‬ﺻﻔﺤﺔ ‪ 336‬؛ ﲤﺮﻳﻦ ‪ 213‬ﺻﻔﺤﺔ‬ ‫‪−‬‬
‫‪4‬‬ ‫‪4‬‬ ‫‪5‬‬
‫‪303‬‬
‫‪2 − √2‬‬ ‫‪2 + √2‬‬ ‫𝜋‬
‫ﲤﺮﻳﻦ ‪ 69‬ﺻﻔﺤﺔ ‪ 136‬؛ ﲤﺮﻳﻦ ‪ 46‬ﺻﻔﺤﺔ ‪ 116‬؛ ﲤﺮﻳﻦ ‪ 152‬ﺻﻔﺤﺔ‬
‫‪2‬‬ ‫‪2‬‬ ‫‪8‬‬
‫‪ 223‬؛ ﲤﺮﻳﻦ ‪ 126‬ﺻﻔﺤﺔ ‪194‬‬
‫‪2 + √2‬‬ ‫‪2 − √2‬‬ ‫𝜋‪13‬‬
‫ﲤﺮﻳﻦ ‪ 153‬ﺻﻔﺤﺔ ‪223‬‬ ‫‪−‬‬
‫‪2‬‬ ‫‪2‬‬ ‫‪8‬‬
‫‪10 + 2√5‬‬ ‫𝜋‬
‫ﲤﺮﻳﻦ ‪ 127‬ﺻﻔﺤﺔ ‪195‬‬
‫‪4‬‬ ‫‪10‬‬
‫‪√3 − 1‬‬ ‫‪√ +1‬‬
‫‪3‬‬ ‫𝜋‬
‫ﲤﺮﻳﻦ ‪ 86‬ﺻﻔﺤﺔ ‪ 146‬؛ ﲤﺮﻳﻦ ‪ 70‬ﺻﻔﺤﺔ ‪ 137‬؛ ﲤﺮﻳﻦ ‪ 71‬ﺻﻔﺤﺔ ‪138‬‬
‫‪2‬‬ ‫‪2‬‬ ‫‪12‬‬
‫‪√3 + 1‬‬ ‫‪√3 − 1‬‬ ‫𝜋‪5‬‬
‫ﲤﺮﻳﻦ ‪ 86‬ﺻﻔﺤﺔ ‪ 146‬؛ ﲤﺮﻳﻦ ‪ 148‬ﺻﻔﺤﺔ ‪ 218‬؛ ﲤﺮﻳﻦ ‪ 70‬ﺻﻔﺤﺔ‬
‫‪2‬‬ ‫‪2‬‬ ‫‪12‬‬
‫‪137‬‬
‫‪√3 + 1‬‬ ‫‪√3 − 1‬‬ ‫𝜋‪7‬‬
‫ﲤﺮﻳﻦ ‪ 86‬ﺻﻔﺤﺔ ‪146‬‬ ‫‪−‬‬
‫‪2‬‬ ‫‪2‬‬ ‫‪12‬‬
‫‪√3 − 1‬‬ ‫‪√3 + 1‬‬ ‫𝜋‪11‬‬
‫ﲤﺮﻳﻦ ‪ 86‬ﺻﻔﺤﺔ ‪ 146‬؛ ﲤﺮﻳﻦ ‪ 148‬ﺻﻔﺤﺔ ‪218‬‬ ‫‪−‬‬
‫‪2‬‬ ‫‪2‬‬ ‫‪12‬‬
‫‪√2 + √3‬‬ ‫‪√2 − √3‬‬ ‫𝜋‪17‬‬
‫ﲤﺮﻳﻦ ‪ 72‬ﺻﻔﺤﺔ ‪138‬‬ ‫‪−‬‬
‫‪4‬‬ ‫‪4‬‬ ‫‪12‬‬
‫𝜋‬
‫‪ņőƱŀ Ţ‬‬
‫ﲤﺮﻳﻦ ‪ 244‬ﺻﻔﺤﺔ ‪336‬‬ ‫أﻧﻈﺮ أدﻧﺎه‬
‫‪17‬‬
‫𝜋‪2‬‬
‫ﲤﺮﻳﻦ ‪ 245‬ﺻﻔﺤﺔ ‪342‬‬ ‫أﻧﻈﺮ أدﻧﺎه‬
‫‪17‬‬
‫𝜋‪13‬‬
‫ﻣﻼﺣﻈﺔ ‪ 56‬ﺻﻔﺤﺔ ‪342‬‬ ‫أﻧﻈﺮ أدﻧﺎه‬
‫‪17‬‬

‫𝜋‬ ‫‪1‬‬
‫‪cos‬‬ ‫=‬ ‫‪1 − √17 +‬‬ ‫‪34 − 2√17 +‬‬ ‫‪68 + 12√17 + 16‬‬ ‫‪34 + 2√17 + 2 1 − √17‬‬ ‫‪34 − 2√17‬‬
‫‪17 16‬‬
‫‪ƕ‬‬

‫𝜋‪2‬‬ ‫‪1‬‬
‫‪cos‬‬ ‫=‬ ‫‪−1 + √17 +‬‬ ‫‪34 − 2√17 +‬‬ ‫‪68 + 12√17 − 16‬‬ ‫‪34 + 2√17 + 2 −1 + √17‬‬ ‫‪34 − 2√17‬‬
‫‪17 16‬‬
‫‪ũŏ‬‬

‫𝜋‪13‬‬ ‫‪1‬‬
‫‪cos‬‬ ‫=‬ ‫‪1 − √17 +‬‬ ‫‪34 − 2√17 −‬‬ ‫‪68 + 12√17 + 16‬‬ ‫‪34 + 2√17 + 2 1 − √17‬‬ ‫‪34 − 2√17‬‬
‫‪17‬‬ ‫‪16‬‬

‫𝟵𝟭𝟰‬

‫‪http ://tinyurl.com/Malki1718‬‬ ‫‪0‬‬


.

Ŕž Ŧ Ľ
œǃ
Ŀ
ņőƱŀ Ţ
ũŏ ƕ

http ://tinyurl.com/Malki1718 0
‫ﻫ‪ .‬ﺣﺴﺎب اﳌﺜﻠﺜﺎت‬
‫‪.‬‬
‫ﺗﻨﻈﻴﻢ اﻟﻮﻗﺖ ﻟﻸﺳﺒﻮع‬
‫اﻟﺴﺒﺖ‬ ‫اﳉﻤﻌﺔ‬ ‫اﳋﻤﻴﺲ‬ ‫اﻷرﺑﻌﺎء‬ ‫اﻟﺜﻼﺛﺎء‬ ‫اﻹﺛﻨﲔ‬ ‫اﻷﺣﺪ‬

‫‪Ŕž Ŧ Ľ‬‬
‫‪06:00‬‬
‫‪06:30‬‬

‫‪œǃ‬‬
‫‪07:00‬‬
‫‪07:30‬‬
‫‪08:00‬‬

‫‪Ŀ‬‬
‫‪08:30‬‬
‫‪09:00‬‬
‫‪09:30‬‬
‫‪10:00‬‬
‫‪10:30‬‬
‫‪11:00‬‬
‫‪11:30‬‬
‫‪12:00‬‬
‫‪12:30‬‬
‫‪13:00‬‬
‫‪13:30‬‬
‫‪14:00‬‬
‫‪14:30‬‬
‫‪15:00‬‬
‫‪15:30‬‬
‫‪16:00‬‬
‫‪ņőƱŀ Ţ‬‬
‫‪16:30‬‬
‫‪17:00‬‬
‫‪17:30‬‬
‫‪18:00‬‬
‫‪18:30‬‬
‫‪19:00‬‬
‫‪19:30‬‬
‫‪ƕ‬‬

‫‪20:00‬‬
‫‪20:30‬‬
‫‪21:00‬‬
‫‪ũŏ‬‬

‫‪21:30‬‬
‫‪22:00‬‬
‫‪22:30‬‬
‫‪23:00‬‬
‫‪23:30‬‬

‫𝟭𝟮𝟰‬

‫‪http ://tinyurl.com/Malki1718‬‬ ‫‪0‬‬


‫‪.‬‬
‫ﺗﻨﻈﻴﻢ اﻟﻮﻗﺖ ﻟﻠﺸﻬﺮ‬
‫اﻟﺴﺒﺖ‬ ‫اﳉﻤﻌﺔ‬ ‫اﳋﻤﻴﺲ‬ ‫اﻷرﺑﻌﺎء‬ ‫اﻟﺜﻼﺛﺎء‬ ‫اﻹﺛﻨﲔ‬ ‫اﻷﺣﺪ‬

‫‪Ŕž Ŧ Ľ‬‬
‫‪œǃ‬‬
‫‪Ŀ‬‬
‫‪ņőƱŀ Ţ‬‬
‫‪ũŏ‬‬ ‫‪ƕ‬‬

‫𝟮𝟮𝟰‬

‫‪http ://tinyurl.com/Malki1718‬‬ ‫‪0‬‬


‫‪.‬‬
‫‪.‬‬

‫هذا الكتاب م ُوجه لطلبة الأقسام النهائية )بكالور يا( بالدرجة الأولى‬

‫‪Ŕž Ŧ Ľ‬‬
‫لـكنه سيكون سندا ً لطلبة السنتين الأوليين الجامعيتين و المقبلين على‬

‫‪œǃ‬‬
‫مسابقات توظيف أساتذة التعليم المتوسط و الثانوي و مرجعا ًللمعلمين‬
‫مجز ّأ إلى الأقسام التالية ‪:‬‬ ‫و هو ُ‬

‫‪Ŀ‬‬
‫‪Ō‬‬ ‫‪Ō‬‬
‫‪ŗ ijęĵĬ‬‬
‫‪ijĂģċĬħú‬‬
‫‪ŋ‬‬ ‫‪Ĉúĉęĺĸú‬‬ ‫‪Ćī‬‬‫• ‪ŋ‬‬
‫‪Ő Ő‬‬ ‫‪Ő‬‬

‫‪.‬‬
‫‪ăģċī‬‬
‫‪ŋ‬‬ ‫‪ĈĉĚħ ķ‬‬ ‫• ‪ō ĂĨĂĬħú ĩĤĎħú‬‬
‫‪Ō‬‬ ‫‪ŗ Āþ‬‬ ‫• ‪Ŏ ĞŌ ĂĂĖāŎ‬‬
‫‪ijĂģċĬħú‬‬
‫‪ŋ‬‬ ‫‪Ĉúĉęĺĸú‬‬ ‫‪ōŋ‬‬
‫‪Ŏ‬‬ ‫‪Ŋ Ō‬‬ ‫‪Ō Ŋ Ŏ‬‬ ‫‪Ŏ‬‬ ‫‪Ő Ŏ‬‬
‫‪ăģċĬħú‬‬
‫‪ŋ‬‬ ‫‪Ķĵ‬‬
‫‪ō ĂĎĬħú ķ‬‬
‫‪ō ĝ ijĂĖ‬‬ ‫‪ō ĞĂħú ĀýĹāĵĆ‬‬ ‫• ‪ō Ăħú ĩĂō ĂĬā‬‬
‫كل قسم مرفق بأمثلة و تطبيقات لتسهيل استيعابه‪.‬‬
‫ً‬
‫حلا ّ مفصّ لا ً ‪:‬‬ ‫مخصص لتمارين محلولة‬ ‫الجزء الأكبر من الكتاب ُ‬
‫تطبيقات مباشرة للدرس ‪ ،‬تمارين بكالور يا و أخرى للتعمق‪.‬‬
‫إرتأينا أن ندرج في الجزء الأخير ملحقا ً يُذك ّر بالخواص الأساسية‬
‫‪Ō‬‬ ‫‪Ŋ‬‬ ‫‪Ŏ‬‬ ‫‪Ŏ ĂĂģ‬‬ ‫‪Ő‬‬
‫‪ijąĊĉħú‬‬
‫‪ŋ‬‬ ‫‪įī ĀýĸĈþĚĬħ‬‬ ‫‪ ĈĴĉĆħú Āúċ‬و آخر فيه دراسة وافية لـ‬ ‫لـ ‪ō‬‬
‫‪Ō‬‬ ‫‪Ō Ő Ő‬‬
‫‪ijĚāúċħú‬‬
‫‪ŋ‬‬ ‫‪Ĵ‬‬ ‫‪ij‬‬ ‫‪ĂħþĂħú‬‬
‫‪ņőƱŀ Ţ‬‬
‫‪ũŏ‬‬ ‫‪ƕ‬‬

‫‪http ://tinyurl.com/Malki1718‬‬ ‫‪0‬‬

You might also like